You are on page 1of 1120

CHAPTER

Calculus Preparation

' 1.1 Introduction


Over the years, I have l earned that the .nos:t f requent reason for students fHi l i r1g to achieve
optimum results incalculus courses is inadcqlw teprepararion. Smdcncsnwy undersrand calculus
(.;Oncepts completely. Lhey may have every formula and every rule memorit~ed _perfectly~ they
may e ven have a c lear idea of the proca.lun::s required to solve prob lems~ but because they
lack skills in a_lgc.bra, geometry, and tr igonorn etry, they cannot put their krlowledge to work.
h i s unfortunate that they never get to dernon~wne their calcu lus knowl edge becc.lll~ of poor

mathematical preparation . To obta in the-be!\1 possib le grades in calculus, it is essential to have

excellentalgebmic skills and a good gmspofthe elementS of analytic geometry and trigonometry.
In this chapter we ghc yoo the op)>Ortunity to test your skills and knowledge in these three an:as.
and, should it be necessary, the meart~ by which 10 make improvements. Titere are also ~tions
lhat may con1ain material unfamil iar to many readers. Topics in these sections are esse.ntialto
some oFthc applications of calculus; your instructor will indicate whether they are required For
your course.

l lte calculus course at your institution may, or may not. spend time Qn the review sectio1s
in this cb1lpter. Jf it docs not, your instructor may ad,'isc you to review ccr1ajn sections or1 your
own. DO l'f! You could regret ignoring review material in this chapter in order 10 get to caJculus
i n Chapter 2 more q uickly. Each review sectio n opens w i th a diagnostic test to d~ tcrmine your

knowledge of the material in the section. Give yourselr the suggested time to take the test. no
longer. You must not only be able to solve the t est questions. but you m ust also be able to do so
reasonably quickJy. Do NOT use a calcubnor unl e~s spe;cific;:ally instructed to do so. Answers

are provided m the end of the section along with marks for each question. Assign yourself
pan ial marks For partially correct answers, but try to he objective in doing so. It is difficult
to be specific as lO what constitutes an acceptable score on the diag.nol)(ic tests. Ce1tainly a
score o f less than 50% indiC>Jtes thai detaile<l Sludy of the section is required. A score of more
thar'l

soc&, but not much JTt()re, would also suggest the need

fo r careful review. ~fart.: ~ in the

80~ 1 00%. range indicate a

good working knowledge of material in the section, but it could be


beneficial to gi"c the. section a quick rcading. Jl'aying Jtrlicular aucntion tO ,,arts corresponding

10 incorrcclly answered lCSI queslions. ln addition, not only is il helpful to refresh your memory
on concc:pu leamed some lime ;lgo. it is also wi.se 10 become familiar with lhe terminology,
nolation, and conventions set forth in these sections. To improve your skills o n, and knowledge
of, dlC material in a rc,icw sectinn. read the di~cussi ons and examples thoroughly, tr) as m;lny
o f 1he exercises as yuu c.:an. unU then n:tukc the diagnost ic test. If you arc conscientious in your
work, we are confident that you will do much b-etter the second ti me. Yo u w ill be well rewarded
for talc.iog the time to do this; your calculus sr.udies wiJI be so much easier. ln fact. the 1ime you
spend on calculus prepa.rarioo now wiiJ more than compensate for ex 1ra time that you would
s pend on solving calculus proble ms late r. Be lieve me; I have Laught t.housands o f s tudenL.s j us t
like you.

Ch(lpter I C.kuhts Pr<parJJion

11.2 Solving Polynomial Equations


Here is the diagnostic test for this section; give yourself 40 minutes 10 do it. When you hove
marked your test using the answers at the end or the section, decide whether a brief reading or
a 1horough treatment is needed for material in this section.

DIAGNOSTIC TEST FOR SECTION 1 .2

Find all real solutions for each or lllc following polynomial equations.

9. x' - 3x 1 - 4 = 0

x' - x' + J.r 1 - .r - 4 = 0

10.

Give multiplicities of any repealed roo1s.

12. 24.rl

l. 9x+5=0

2. x 2 - 4x - 5 = 0

3. 6.r 2 + 1x - 20 = 0

~ . .r 2 + 3.r

+ 10 =

5. x 2 + 6x - 4 = 0

6. 36x 2

IOS.r + 81 = 0

7.

.x 3 - 8 = 0

8. .x' - 4x2

+ 2.r1 -

27.r + 10

=0

lnques1ions 13 :md 14 you arc ghtenthe 7cmsoflhe polynomial. Write

cad1 polynou,iuJ in factored ronn.

13. Zcrt of x 3

+ S.x - 2 = 0

+ 6x 2 -

x - 30 an:; x

= - S, - 3. 2.

14. Zcmsof2x 2 + 4x - IOare.r = - I

./6.

A real polynomial in x of degree 11 , where 11 ::: 0 is an imeger, is an expres:;ion of the


form
( 1.1)

a,, a,_ h . . . , a 0 are real constants with a11 =I 0 . (If a, were equal to 2ero, the
polynomi;tl would not be of degree 11 .) When P,. (x) is set equal to zero, the resulting equation

where

( 1. ~ )

is called a polynomial equation of degree 11. Examples are 3xl- 2.r + 5 = 0 and 2x 2 + 5.r9 = 0. In this section, we are concemed with the number of sohnions of a polynomial equation,
whether solutions are re-JI or complex, and techniques for finding the real solutions. (Complex
solutions are dealt ""'h in Appendix C.) Value.< of x that satisfy equation 1.2 are called root<
or solutions of the equation. They are also called zeros of the polynomial P,.(x).
When 11 = I, equation 1.2 is called a linear equation (or equation of degree 1).

a,x

+ a0 =

0.

( i.J)

Its only sohnion is x = - a0 j a 1 For example, the solution of 2x + 5 = 0 i< x = -5/2.


Quadratic equations (equations of degree 2) are obtained when 11 = 2. h is cu<tomary in
this cnse to de-n ote coefficients as fo llows

ax 2 + bx + c = 0.
Quaclnnic equations can sometimes be soh'cd by

factoring~

(1 .4)

and can always be solved with the

quadratic formula. II is preferable to initial I)' aucmp110 factor. For thcexam)>lc 6x2 - Sx -4 =
0, we write (3x - 4)(2x + I) = 0 . from which Ihe two sohnions are x = ~/3 and x = - I/2.
Likewise, for .r 2 - 6x + 9 = 0, we fuctor in the fom>(x - 3) 2 = 0, and the only solution is
.t = 3. When the quadratic docs not factor easily, we usc the quadratic fomlUia
X =

- b

JJJ1 -

4ac

( 1.5)

2LJ

Application of this formula 10 the equation 2x 2 + 4x - 5 = 0 gives

- 4

j42-

4(2}(-5)

--~2-:-;(2
::-:)--'-.:...:....-

-4 ./56
4

-2
2

Jl4

1.1 Solviog Polynomial EqllltiQns

Quadratic fom1ula 1.5 indicates that quadratic equation 1.4 has two distinct real solutions when
the discriminant b2 - 4ac > 0, two real solutions that are equal when b2 - 4ac 0, and no
real solutions when b2 - 4a c < 0. (Solutions arc complex numbers in the lilst case.)
The next simplest polynomial equation (after the li near and quadrdtic) is the cubic equation,

ax 3 + bx 2

+ ex + d =

0,

( 1.6)

and after that the quartic.


( 1.7)

There are procedures that give exact rooL~ for both of the~e equations, but they are of so little
practical use in this day of the electronic calculator and pe.rsonal comlluter, we omit their
discussions. For such equations, it is often suffficicnt touse a numerical procedure to approximate
roots to some degree of accuracy (Sections 1.1 1 and 4.1 ). On the other hand, when an exact
solution of a polynomial equation can be found, it can be removed from the equation. yielding a
simpler equation to solve for the remaining roots. 1l1c process l>y which thi s is done is a result
of the factor theorem.
THEO R E M 1 . 1 (Factor The o rem)

x -a is a factor of Pn(x) if and only if P.(a) = 0.


The factor theorem is very useful in solving polynomialequations. It ~oes not find solutions.
however. What it does is simplify the problem each time a solution is found. To illustrate,
consider the quanic equation

( t.!!aJ
A moment's reflection indicates that x = I satisfies the equation. The factor theorem then
implies that x - 1 is a factor of thequanic. The remai ning cubic factor can be obcained by long
divi&ion. symhetic division. or mental long division. The resuh is

P(x) = x 1 + 2x;

+ x2 -

2x -

= (x - !)(x 3 + 3x 2

+ 4x + 2).

What th.is means is that equation 1.8a can be replaced by


P (x)

(x - l )(x 1

.J..

3x 2

+ 4x + 2) =

0.

( l.libJ

To find f\tnhcr solutions of quanic cquatioro 1.8a. we need only examine the cubic x' + 3x 2 +
4x + 2 in I .&b for il( zero~. Once we notice tltat a zero is x = - I. we may factor x + I from
the cubic and replace t.8b with
P(x)

= (x - t) (x

+ l) (.x1 + 2.x + 2) = 0.

The remaining two o;otutions arc complex numller!:.


soluliOI\S x = I and two complex solutions.

( I.&J

Thus quartic equation 1.8a has two real

Ot\Ce a ~o!ution a of a polynomial equation ha~ been fount!. x- a canbe factored from the
polynomial. When a is a fraction, it is rcoommcnded thai this procedure be modified slightly.
To

illus trat ~

consider the equation

2x 3 - x 2 - 9x

+9

= 0.

A solution of this equation is x = 3/2 (we show how we found this solution shortly). When
we factor x - 3/2 from the cubic, the result is
(x -

3/2)(2x 2 + 2x

6) = 0,

Cbnpter I Calculus Preparmion


but the work. invo.lves fractions. Tiley cao be avoided by factoring 2(x - 3/ 2) = 2x - 3 from
the cubic instead. Calculations are simpler, aod the resu lt is

(2..r - 3)(.r

+X -

= 0.

3)

The remaining two solutions can be obtained with the quadratic formula

- ]

x=

- I.Ji3

j J2 - 4(1)(- 3)

What we are suggesting is that when a rational number r = p /q is a solution of a polynomial


equatiou (with integer coefficienL~), it is simpler to factor q(x- pjq) = q.r - p from the
polynomial (and in so doing Ottly integer arithmetic is involved).
All real polynomials can be factored into real linear factOrs and irreducible rea l quadratic
factors. An irreducible real quadratic factor is one that has complex zeros, such as x 2 + 1
and 2x 2 x 6; they are characterized by negative discriminants. Finding the linear factors
of a polynomial goes hand-in-hand with finding real zeros of the polynomial. We have shown
the factorization or three polynomials below.

+ +

x1 + 2fl

+ x2 - 2x .r3 - 3x 2 + 3x -

x&

+ 7x 7 -

86xs - 95x'

+ 363x 3 + 486x2 -

= (x - l)(x

= (x -

540x - 648 = (x

+ l}(x 2 + 2x + 2j ,

(1.9a)
(1.9ol

1/ ,

+ 3)~(x - 2) 3(x + 1).

( 1.9c)

Each linenr f>tCtor in these. polynomials leads to >t re>ll zero t>f the poly1tomial. For polynomial
1.9a, the real zeros are 1, and it also has two complex zeros; for polynomial I .9b, the zeros
are 1, I, l; and for 1.9c, the zeros are - 3, - 3, - 3, - 3,2, 2,2, - I. In the case of 1.9band
1.9c, there are repeliti ons. We SllY that _r = I is 11 zero of multiplicity 3 for the polynomial
in I .9b: the multil>licity corresponds to the number of times 1he factor x - I appears in 1he
factorization. Each or the zeros in 1.9a is of multiplicity I. In 1.9c, x = -3 has multiplicity 4,
the zero x = 2 has multiplicity 3, and x = -I has multiplicity 1. These examples suggest that
the number of zeros of a polynomial, taking mul!iplicities into account, is equal to the degree
of the polynomial. This is confirmed;,, the following theorem.
THEOREM 1 . 2

Every polynomial of degree 11 2:: I has exactly n zeros (counting multiplicities).

l EXAMPLE

1 .1

Find all roots and their multiplicities for the polynomial equation
x3

+x2

16x

+ 20 = 0.

SOI.lmOK With a linleexperimemation. we Nnd that X = 2 i~ a sohnion Of the equal ion. II


follows that x - 2 can be factored !rom the cubic polynomial, Md the equation can be wriuen
in the rorm
1

0 = (.( - 2)(x

+ 3x

- 10) = (x - 2)(x

+ 5)(x

- 2) = (x - 2) (x

+ 5).

Thus. x = 2 is a solution with mulriplicity 2. and x = - 5 is a solurion with muhiplicity I.

The following theorem provides a quic.k way to locate all rational roots of a polynomial equation
when ilS coefficients are rational numbers.

1.2

Svlvina; Polynomj:ll

EquatiQf~

THEOREM 1 .3 ( Rational Root Theo rem)


Suppose that r = pfq is a 111tional root (in lowest terms) of a polynomial equation
a.x + + a 1.t + a0 = 0 with integer cocfficic.nts. and ao ~ 0. Then, p divide<> ao
and q divides lln .

This is a powerful resull. II narrows the field of po,;,;ible ratio nal solutions or polynomial
cquatioos with in1egcr coefficients to a finjte set \Vc illusrr.ue wilh two examples.

I EXAMPLE

1.2

.........
Find all real solutions of the cubic equation Sx 3

+x2 + x -

4 = 0.

SOLUTION Since divisors of - 4 arc ::!: I, 2 , and ::!:4, and those of 5 nrc ::!: I and ::!:5, the
only poosible rational solutions are

1. 2. 4, 1/5. 2/ 5. 4/5.
Trial and error lcalls lO thcfactthat x
5x - 4 from the cubic,

= 4/ 5 satisfies the equation. We now factor 5(x -4/5) "'

= (5X - 4)(x 2 + X + I).

The remaining lwo sotucions are complex numbers.

I EXAMPLE

1.3

Find all real solutions of the quartic equation x - x 3 - 3x 2

+ Sx -

2 "' 0.

SOLUTION Sinoc diviso-:; of - 2 are I and 2. and those of I arc I . Lhc only possible
rAtional solutions are lhe integers I, 2. Since x = J is a solution. we faclor x- I from
the polynomial,
(x -

l)(x 3

3.r

+ 2) "' 0.

OllCC agai.n, the o11ly possible rational zeros of the cubic are 1. 2 . We find thnt x "' I is a
zero. and when x - I is factored from the cubic.

= (x -

l)(x - l)(x 2 + x - 2)

= (x -

The only roots of the equation are therefore

x=

1) 2(x

+ 2)(.r -

I)

I (rnu lliplicity 3) and

(x -

x=

1l(x

+ 2).

-2.

The list of possible rational roots. as predicted by the rntional root theorem. is shortest when
any common factors in coefficients of the equation have been removed. For example, 3x 3 +
2xl - 2 "' 0 and 9x 3 + 6x 2 - 6 "' 0 have the same ,;olutions. The rational root theorem yields
I, 2, 1/3, and 2/3 as possible solutions of the first equation, and I, 2, 3, 6,
1/3. 2/3. I / 9. and 2/9 as possible solutions of the second equation. Clearly then it is
ad\'anlageous to remove the common facLOr 3 from the second equarion.
Given the zeros of a polynomial, it is easy to write the polynomial in facLorcd form. For
example, if the zeros of x 3 + 3x 2 - 6x - 8 are known to be x "' - 4, - I, 2,the factored form
of the polynom ial is (x + 4)(x + l)(x - 2) . Zeros of 2x 2 + 9x - 5 are x "' - 5. 1/2; its
factored fonn is 2(x- l /2)(x + 5) or (2x- l )(x + 5). Fioally. the zeros o f 2x 2 + 2x - 4 are
-~ = -2, I ; its factored form is 2(.r- l)(x + 2) . Watch for the number preceding the product
of fac tors. It can always be determined by examining the coeOlciem of the highest power of x.

(h:.!)1er1Qd.:lllasP~:""'~-----------------------------------.

'

In F.-xct~sei

(i

\ nomi .\1 eQuatiOn.


d all n.:<Jl iolutOI\S o f the pO}

t-??.'" I

lrtdUIC.lc nwltlphc-1\ICS WlCii

!(

they are gre:J.ter tNl\ Oil!:

1. ) x - 1 ::-oO

9. l..t'
II.

+ 5.1

x' -

3X' ~ 3.< - I

13. xl _ 2.'

+ 5x -

+ 19.\' -

~. tlx' + llx - 5 = 0

26. ' ' - t5x''

10.

- 10 ~ 0

24. 6.~:\ + ...2

. .,,..... .~s + 5..t'' -

=0

tO = 0

..,.:;.x 4
13. ......

29. ' '

x' - sx + 9 = o

12. Sx' + 12..1" 1 + 6.1: + I

20 = 0

9x - 45

+ SS.t

"' 27. 4.~" + 4x)+ 1 7.,.~

3. .' - 36x + 81 =0

+ Sx + 16 = o

~~J + 5x l == 0

2. t 4.f + 5 ~ 0

6. -4x' ;- tO.< + 9 = o
1. ' '

l..l. 3x"' +

=O

3o.

120x 3

'+274r- t20=0
- 225'
.

+ ,6., + ~ ~ o

+ \<1-)x-, -

+ 9x" + 47.<'

,, + t6x'1 -

Stx' - 12\16

* 16. .t~ + 7J:3 + 9xl - 2t.x- 36 = 0

, tS. ~x + 9.< '- 6.<

8.< - t5

=0

=0

- 5. 1

x = -2. 3. 10

33. xl - tl.\'+ 4x+60.

lS .
1

+ 18.<- 200 = 0

x=(3 -r. ..f6sl / 4

H . l 4x' + 22K' - 27x


17. . - 16 = 0

.. + ,.0
-

. \ nnd it~ zeros Write the


tn E~cfc;iscs 3t-J.6 you arc gi\'cn a po\yncum(t
:..

f10l)'nom i!l\ 1n factoteil fom\.

32. 2t 2 -3.t - 7.

.c)+ l lxl + .l8.r + 64 = 0

.~

.l'."'"

-1- t2Sx '

31. 2.< ' + Sx - tO,


... 15.

=o

.\'4 _

+ 5.

2:(3 + 2x _ 1.

36. t fu:'' -Rx'+ l .

.t

x = - 5/3, 114 t jl

= - t. x = 1 (mu\iplid ty 3)

x =:!:l/ 2cachormuhit>lici<y2

37. Find a polynonlialthul has


l.c~s .x. = 4/ 5. ; C:.\<.:b
with an ulti('\ici.ty 1 am\ x = 4 w 11.h mull!()hcuy 3. l s at untque.

?''\y

20. 12.t1 ~ 19x

+5 = 0

'/.>,

.l&. Prove the fo \lowillg com\lary to 1hc 1"'.\\ionat roo\. theocc1n, cal\~
the '"i1llCCf l'OOI thcOt'C:Ill.. : lf r is U rutionn\ 'LCl'O of !.\ po\ynOI\lla)
,_,l,(x) x" + a,. -1 ..\'11 - 1 + .. . + ao with integer cocmcicms, when::
a0 ;j; 0, then r is :m intcg.e.r thu.t divides ao.

I. -5/ q

(I m.r\)

8. ' with m u\t1plichy 2, 2 (3 m arks)

l . -1. 5 (2 m.ll'l<<)

?. :!:2 (3 maO.s)

3. -5{2, 4{3 (2 1l1<l1'kS)

10. :!: I , (3 n>atl<_<)

4. No reul sohlli.ons (2 m:uks)

II. 2/3 13 nmks)

S. - 3

Ji3 (2 mwb)

6. 3/2 wiLh mulliplicily 2 (3 mlltks)

7. 2 (2 m:lrks)

11. - 5/4, I /2, l/3 (411Ull'ks)


13. (X - l )(X

+ 3)(x + 5) (2 m ar~s)

J4. 2(x + I - ,/6)(x + I + ,/6) (3 marks)

11.3 Pla ne Analytic Geometry and Straight Lines


H ere is Lhe d iagnosLic te,sL for Lhis seCLion ; give yourself 30 m inutes Lo do i L. Wheo you have

marked your test using Lhe answers at Lhe end of the section, decide whelher a brief reading or
a thorough treaLmcm is needed for matelial in this sectjon.

DIAGNOST I C TEST FOR SECTION 1. 3

I. Find the distanc<: between the points (- I, 3) and (2, - 6) and the
coordinnLcs of the midpoi11t of the line scsmcnt that joins them.

2. Find Ihe slope of thdine l.t + IO,v

= 14.

6. Fiod the eqU3tion orthc line through the p<>int (I, 0) p:1r.1llelto the
line 2< - 4y = S.

7. Fiodtheequutionoflhclincthroug)lthcpoint (4. - 2) perpendicular


to the line 3x + 2y = 9.

3. Find the .<and )' intc<:pl$ of thcliD< Sx - 2y = II.

4. find. in gcncrJI ronn. the equation or the line through tile point

(3, -5) with <lope -2.

8. Fiod the point of intersection of the lines 2.t - 3y


S,1 = 6.

= I 0 and x +

9. Find the di."lance from the point (I. 2) to the line .l.r - 4y

5. Detcnninc whttltcr li)C line$ b - 3y

4 and Sx - 8y

13 an.:

par:llcl, f)CI'pcndicular, or neither.

10. Draw the three lines x + ly

= 4, x = 2. and x = 3)' -

=5.

5 on the

same set of axes.

rmnn
A sound knowledge of analytic geometry, a union of algebra and geometry, is essemial
10 lhe sludy of calculus. Oo the one hand, it provides a way 10 describe geometric objects
algebr;eically: on the other hand, il pem1its a geomelric \'isuali7...ation of algebra ic ~aate mems.
Our approach to calculus is visual: w e draw picrurcs at C\'Cry possible opponunity, by hand,
b)' ~rdph ing calculator. and by computer. We draw picJUres to imroduce ide-A.<. 10 illustrAte
concepts~ to n:inforcc pri nciples~ and to solve problems. We want you to see and fed ca lculus
in all i1s a.pects. Analytic geometry is the basis for many of tl~e-<e pictures. In 1his section. we
re'iew I h e fundamcnlals of plane annlytie geometry, paying partieulnr auention to straight lines.
In Section 1.4 we review circles. pardbolas. ellipse>. and hypertx>las.
PoiniS in the plane arc identified by an ordcrc<l pair (.t . y) of rc<>l numbers called their
Carleslan coordlnale..< (Figure 1.1). The x -coordinate or a poi nt P is its perpendicular distant'<'
from the y a.~is. and the y~coordinatc is its perpendicular d is.ta ncc from the x-a.'tis.
The axes divide the plane inlo four parts which, beginning at the upper right and proceedi ng
eountcrdockwisc. arc callod. respectively. lhe first. second. third. and fourth quadrants. The
axes themselves are n()( considered part of any quadrant. Points in the firs t quadrant have .t- and

mue.,. 01 a !'JOint
)'

- - ............:nr.yl
1
y

I::IUJII Ij
eool'lliMtc:li in the

Signc of
qWt.lnlnu

(QUt

Second

First

quodrunt
( - . +)

quadront
(+. +)

Third

Foun h

C.}tt:ldrant

quadmnt

y--coonlinatc-.s that arc both positive: poinLS in the :second quadrant have a ncgativcx-coordi natc

.v

( '

and a positive y-coordinalc: and so on (Figure 1.2).


If P and Q arc any I\VO poims with coordinates (x,, Yo) and (x2. )'2) . rtlotX::Cii,ely
(Figure 1.3), thcn by the Pythagorean relation forJhc righlnnsJcd lriansJc PQS,
I PQ112

= IIPS11 2 + I Q SIJl.

(+, - )

..

where IIPQII dcnoles ohe lenglh of tbc line segmenl joining P and Q,and UP SUand IIQSII
denooe the 1eng1hs of1he related line segments . Rut '
r t ~ Unglh of tht
lir.c ses,n'ICI'I t j(linin,; tv.'U :ttbitr.uy
l)l)int~

IIPSII

= lxo - Xt l

II QSII = I>~-

anti

ytl.

and 1herefore*
)'

II P Qll 2 =

lxz - xtl 2 + lyz - ytl 2

II P Q II = J (x 2

.r1) 2

+ (}'2- y 1) 2.

oo
( 1. 10)

The ve.rticallines ar<)Und ~t 1 - x 11and ly1 - y11deooce a bsolute '-a lues. n le)' operate on whatc\cr is bet'-'o'O
tl1em to produce a oonne.gati\'e-re.sull according to lXI = X if X 2: 0 and IX I = -x i f X < 0 . For example-.
141 = 4 and I - 31 = 3. We discuss aboolute values from n foOOiiOO;tl poiM of view in Seerion I.S.

:i: JX always denotes the pMitil'~ square J\)1)1 of x. so th.a.t J4 =

2.

Mjlclii;IA&*

PC-3. 4)

l.cngda cf line joinin;


h'O .,peoeilic p()inh

IOJI,.Jii!Wij.

Midpoinl of line scgmc11t

>

"'-

Q(2. - I)

Equation I . JO expresses the length of a line segment joining points


(x, y 0) and (x1 , y2 ) . For example, in Figure 1.4

P and Q in te.rms of their

coordinaoc~

QPQII

= )(2 + 3)2 + (- 1 -

4) 2

= s..fi.

Suppose that R(.t, y) in Figure 1.5 is <he midpoint of the line segment joining P (x 0, y 0)
and Q(.r2. )'2) . Since triangles PTR and RUQ arecongrueno. iofollows ohm 0RU II = II PT I,
and for P and Q as shown in Figure 1.5, ohis condioion becomes x 2 - .t
x - x 0 Consequenoly,

x=

(I

ll al

Similarly. equality of II RTJI and II QU II gives

y= Yo+ Yl
2

tl I lbl

rc~ u lcs a~sume.d chat x 1 > x 1 and y2 > y1 buc the ~mne forrnul a~ are Vlllid
for any x 0 x 2 y 1 and y1 whal,oe,er. Coordina1esofthe midpoim of a liooe ,egmeno. oherefore.
:ore averages of coordinaoes of ohc ends or !he li ne segment.
An equation such as .v = x 1 speci fies o rchuion:,hip bel ween nuf'nbcrs represented by the
feller x and those repre.;ented by the leuer y : y must alway' be the square of x . i\lgebmieally.
we speak of pairs of ,alues x and y that satisfy thisequation. It is customary to write these pairs
in 1he lom1 (.r. y) . a few simple ones being (0, 0). ( I, 1). (- I, 1}. (2, 4). and ( - 2. 4). If we
imerpreo each pair of solutions (x. y) as coordinates of a point. we lind that all such points
lie on the curve in Figure 1.6. Th i~ curve, then, is a geometric visualizatio11 of solution pair:i
ofth~<(JUlllion y = x 2 Every pnir of values x and .v 1ha1 sa1isfies y = x 1 is represenoed by
a point on the curve. Conversely, the coordinllles (.1, y) of ewry point on the curve provide a
pair of number~ that sntisfie~ )' = x2 . Algebraic ~olutions. then. are repre.,1Wented geometrically
as points on a curve; J>OinlS on ohe cun1c provide algebraic soluoions.
If x and y !U'e 1he coordina1cs of a poim in the plane. men the poim is on the curve in
Figure I.6 if and only if x and y smisfy the equa1ion y = x 2 . In olher words, this equation
completely charac1erizes all poin1s on the curve. We 1herefore call y = .r 2 the equation nf

Derivation ofrhc~c

35
30
25

20
15

the curve. Thus the equation of a curve is an equation that Lhe coordinates of every point on
the curve satisfy and at the same Lime is an equation that no point off Lhe curve satisfies. In
the remainder of this section we discuss straight lines and their equations. In Section 1.4, we

show ohat parabolas. circles. ellipses. and h)perbolas can be recognized by Lhe disoinctive fonns
of their equal ions. Once again oote the algebraic-geometric interplay; the form of an equatioo
dictates the shape of the curve. and, conversely, the shape of a curve delennines the form of ils
equation.

The Straight Line


The slope o f 1he line 1hrough I he poinL~ (.r., y 1) and (x2 , y2) in Figure 1.7a is defined as the
quotiem

m =

.l'2- Y1

( 1.12)

The difference Y2 - y 1 is called !he rise becduse it represe111s the vertical distance between the
points. and x2 - X1 the run. the horizonlal d istance between the points. It is easy to show using
similar triangles that m is independem of the two points chosen on the line; that is. no matter
whjch two pojots we choose. on the line to eva_luate m , the resu.lt is a.lways the same-. The tOur
uumbe.rs x,. x2. )'1, and )'2 vary, but the ratio ()'2 - y ,)f(x2 - x,) remains unchanged. A
ho rizontal line (Figure I. 7b) has slope zero (si nce y1 - y 1 = 0). whereas Ute slope o f a \'ertical
li ne is undefi ned (since x 2 - x 1 = 0). In Figure 1.7c, li ne'" w hich leans 10 1he rig.h1, has
positive slope, and Une 12 , which leans 10 the left, has ncga1ive slope.

Mjtflll;lmut"

Slope of a

line throu,gh two poin1s

MijiCIII;IW'U.if' Slopes of
hori:zontal :lnd '~rtico'tl lines

10jiCUI;Iji(j'fl#J"

Lines with

II

MiiiCJII;I#UtJI Equ:nion of
line thn)Ugh t1.I.'O given po1n1.;

y
(x. )')

(3, 5)

In Figure 1.8 we have shown the line through lhe points ( I, 2) and (3, 5) . lis slope is
(5 - 2)/(3 - 1) = 3/2. To 6nd 1be equation fotthis li ne, we let (x, y) be the COO(dina1es of
any 01he r poinl o n the line. Since the s lope of Lhe li ne mu>l also be given by (y - 2)/(x - I),
it follows !hat
y -2
3

x - 1
3

2'

and when this equa1ion is simplified.


(I, 2)

2y = 3x
2 J

IIOilffi[J]\IQ I
tine lhrou~
dope

Equ;uion of a
'i'en point with i:)ven

I.

If (,{ . y) arc the coordinates o f ny point not on this li nc, !hen they do not satisfy this equation
because1he slope (y - 2)/(x - I) joining (x, y) 10 ( I , 2) is not equal to 3/2. Thus, 2y = 3.r + I
is the equation for the straight line thro ugh ( I , 2) and (3, 5}.
We can use this procedure to find the equation for the<traigh1 line through any poinl (.t1, y 1}
with any slope m (Figure 1.9). lf (.r. y) a rc !he coordinates of aoy other poi nt on 1hc line, then
!he s lope of !he line is (y - y 1 )/(x - x 1); therefore,

Ill

or

y - y 1 = m (x - ,r 1) .

(1.1 3)

(x. y)

'Ibis is called the polnl slope fo rmula for the equal ion of a stmighl line; it uses the slope m of
1he line and a po int (x 1 , y 1) on the line 10 detennine 1he equmion of the line.
X

S lopem

Other formulas for Lhe equalion of a line are available when different characteristics of the
line are given. '!bey are listed below. The point-slope fonnula is included for cornple1eness.

10

CNp~er

I c..Jculus Ptt~ioo

Form of Equation
)'- )'1 = m(x - x1)

Name of Equation
Point-slope formula

Characteristics
Determining the Line
m is lhe slope;
(x., y 1) is~ poinr on lhe hne

x -x1
Y- Y1
- x2 -x,
Y2 - Yt
y = mx +b

Two-poim formula

nvo poi ms (x1, y1) and

Slope y-intercept fom1ula

m is the slope;
b is rhc y-intercepl

y = m(x- a)

Slope x -inrercept fom1ula

m is the slope:

(x 2 y2) on 1he li11e

a is the x-intercept
X

)'

-+-=I
a
b

Two-imcrcepl fomJUia

a and b arc x - and y-imcrccpts

Point-slope fomJUhl 1.13 encompasses all of these. The chaructcns1ics in rhe right column
determine the &lope of a line and a poim on it; therefore, with minimal ca lculutions. the pointslope formulu C<lll be u.cd in al l situations. For cxnntplc, if we know that the slope of a line
is m am! the y-intcrccpt i~ b. then a poim on the li ne is (0, b). Hence, equation 1. 13 gives
y - b = m (x - 0), or y = llt.t + b, the slope )'-intercept formula.
There is one other equation dlat is worth mentioning. Vertical lines, which do not have
slopes. Cdt\0101 be represented in form 1.13; lhC) hai'C fonn x = k. for some COtlSiaOI k .
Howel'er. all lines can. for varioiiS 1'31ues of the constants A. 8, and C, be represented in the
fom1
I I. I -1 I
Ax- By+ C = 0.
This i< often culled the general equation or a lint.

I EXAMPLE 1.4

Find 1he IJCtteral equmion of the li ne 1hrough 1he poitll< {- 1. I) and (2. 3) .
SOi l 1'10\1 Since the slope of the line is (3- 1)/(2 + I) = 2/3. we can use the poinl
( - 1, I) lind the siOJlC 2/3 in poim-siOJlC formula 1.1:110 !!,ivc
2

)' - 1 = -(.t

1)

2.\ - 3y

01'

+ 5 = o.

The same re~ult i< obtained iflhc poim (2. 3) i< used in place of (-1. 1). Therwo-point formula
could ai>o be u>al.

...-...

I EXAMPLE 1 .5
Ftnd equmion.< forthc lu>es i n Figure I I 0.

Gnph.< tlf

Y = W- 5< od < = -1

Since the slope of I 1 is the poinl (0. 5).

SO I.L fi ON

5 / 4 and

y- 5 = - -(x - 0}
~

The equmion of 12
I,

2
I 2 J ~\_5 6

>"

is clearly x =

- I.

its )'intercept is 5, we US<! formula


or

4y = 20-

.;.~ .

1.13 with

We do not rcconmtend the usc of the >lOp<: y-intcrcept fonnula )' "' mx + b for finding the
C{luation of a line: the point->IO)lC form ula )' - )'t
III(X - Xo) is more versatile. The Slope
y-intcrccpl formula is. however. tt-;crul in linding the ~lupc or n line with given equation. For
instance. to find the ,Jotx: of the line 2x + 3y "' 9. we ex pres.~ it in slope )'-intercept fonn
)' = -2x/3- 3. It follows thatthe coefficie,nt of x. nantcly. -2/3. isthc slope of the line.
To find the point of imersection of two straight lines - say. 3y = 2.< + 5 anti 4y + x "'
14 (Figure 1.11) - we find the point whose <:oordinatcs (.<, y) satisfy bolh cqu11ions. If we
solve each equation for x and equate the expressions, we obtain (3y - 5)/2 "' 14 - 4y,
which imntcdimely yields y "' 3. Either of che original equations then gives x "' 2. and the
pOint of intCNC(.1ion of the lines is (2. 3). N01e once again II~ algebraic-geometric intcrpluy.
Geometrically. (2. 3) is the point of intcrscccion of t\\0 scroight lines. Algelmtieally, the two
munbcrs constitute the rohnion of the equations 3y = 2x + 5 and 4y -r x "' 14.
Of panicular imponrutce to dte study of lines are the concepts of p;mtllelism " "d perpendicularity.

DE FINIT ION 1 . 1

1\,o distinct lines are said tO be parallel if they have no point of intellicction.

I EXAMPLE

1.6

Verify thm the lines with equations 2.< - )' = 4 and 4x - 2y

= 7 arc parallel.

SOTlmON If we solve each of these equations for)' and equate the resulting expressions,
we obwin

2.< - 4

= 2x -

-,
2

an obviotl~ i mpo~~ihility. Co n ~eQL1e111 ly, the li ne10 do not irliC~~

Geometrically. the following is clear.


THEOREM 1 . 4
1\vo dist inct lines nrc purullel if and only it" OlCy 00\'C the me slope.

For example, if we write t11c equations of the lines in Example 1.6 in the fonn y"' 2.{- 4
and y "' 2x - 7/2, coefficientS of the x -terrns identify the slopes of the lines. Since each has
slope 2, the lines :1re parallel.
D EF INITION 1 .:z

Two lines are said to be perpendicula r if they .intersect at right angles.


The following theorem gives a test for perpendicularity of straight lines in tcmtS of slopes.
THEOREM 1.5
Two lines with nonzero slopes m 1 and m 2 are perpendicular ir and only if
lil t =

or

1Y111Y1 2

= -1 .

( 1.15)

I EXAMPLE

1 .7

Fi nd the equation o f the s mtight line that passes through the point (2. 4) and is perpe.ndil'ular
10 1he line 3x + y = 5.
SOLUTION Since !he slope o f <he given line y = - 3x + 5 is - 3, the required line has slope
l /3 . Using poim-slope formula I. 13. we fi nd !that the equation of the required line is
I

y - 4 = - (x - 2)
3

or

3)' = - 10.

X -

There is a useful formula for Hnding the (shortest) dis<ancc, d , from a point (x 1 , y 1} loa line
wilh e qua1ion A.t + By + C = 0 (Figure I. L2). It is

d=

+ By 1 + Cl .
J A''+ Bl

1.4 x 1

( 1. 16 )

Fo r example. !he distance fromlhe poi111 ( - 2. -1 ) to the line 3x

+ 2( - 1)
J32 + 22

13(- 2)

II?Jilffii

Shorteist distance from u poinl tu a line

- 21

.ljiCjii;Uinrg

+ 2y =

2 in Figure 1.13 is

10

v'G'

Distance frum ( - :!, - I) to 3x + 2y = 2

(-2. -I)

3.\' + 2y = 2

EXER C I S E S 1 .3

In Exercises 1-4 find the dis.tancc bctw~cn che points.

16. Thro11ghthepoin< (-1 , -2) andcrossing<he y-axisal4

I. (I' 3), (3. 4)


3. (- 1., -2), (-3, -&)

17. Crossing th.e x- and

5 .~.

2. (- 2. 1), (4, - 2)
4. (3, 2), ( -4, -I)

Find the midpoint of the tine segmenl joining the poinl!: in

y-axc~ !ll

I and - 3, rt.-:spe<::tively

_18. Throug.h the origin :md the mldpoint of the line segment j oiniog

(3, 4) \lOd (- 7. 8)

Exca:clscs l-4.

In Exercises 9- 18 draw the line described and tind its equation.

Jn &crd scs 19-26 Uctcnninc whether the lines arc perpendicular. parallel, or neil her.

9. Through 1hc points {I, 2) and ( - 3. 4)


10. Through !he poin (3, - 6) and (5, - 6)

19. y

= -x

+ 4, y = .t + 6

+ 3)' =

+ 0)' =

II. Through the pOint (-2, -3) with~lotx:3

211.

12. Through lhepoinl (1 . -3/2) with slope -1 /2


13. The .r -a;ds

21 .r = 3J+4.y=.r/3-2

14. 11)c x -axis

22. 2t

JS. 11wug,b the poinl {4, 3) and crossing the .r-axis at - 2

23. y

.r

+ 3 )' =

4 , 2.r

l. 3.r - 2y = 5

= 3.r + 2, y = -.r/2 + I

24. x - y = 5. 2x + 3y = 4
25.

.f

= 0, )' =

.t

~ 49. Cons ider a mct:al bat of length

L0 at tcnlpcr-J.ture T0 . If the bat is


its l~ngth changes. The Ull0Ullt is described by the
cocRick:m of lii"'C.u.r expansion t:t . It is the change in k:nglh pc.r unit

26. x+y+2=0.3x-y=4

h~ntcd or cooled,

lcrtglh pel' degree Celsius.

ln fxcrcises 27- 3 2 find the point oflntcrstct.ion of the li.ncs.

27.

X+)' =

0. X

28. X = [ , y

=2

2y

48. Repeal Exercise ~7 for the rel~iou.:~hi p between tc:mperatui"CS in


<k:gr"~ Fahrenheit. Tp. ami in J..elvin. TK .

(a) S~~Jthat the length L ofahcb:rra.t tcmpcrnturc Tis given

=- 3

by the fonnula /.., = Lo[l

)'+

10-r c.

5(). A lltt:diwt Of a ttianglc is a line Sf...'gJllCnl drawn (rom a \'CnC.AlO the

31. X/2+ .V/ 3 = I. 2.< - y / 4 = IS

midpoi "' of the OWOSite side. Find equations fort he 11tree medians or
1he 1riangle wilh venice< ( I, 1), (3, 5) , and (0, 4). Show thai all three
medians intersect in a point called the Ct!ntroid ofthe triangle.

32. 14x - 2y- 5. Jx + lOy- 12


In E><aci>c>3;}-38

fin<! the (shonc><) disumcc from dtc point to the line.

33. (3, 4 ) to X+ y

35. (S. I) tO X- y

=4

34. (I. - 3) t<>X + 2y

36 . (3, 1) tO

Ill

Excn;is~

* 51.

If (X t. y,} . ( x, . y,). (X;. .\) ). and (x, . Y ) an: \'<!Rices o r any


quaclrila1eral. show 1hat1he line segments joining lhe: midpoinl.s or 3d~
jocem sides form a parJilelog.ranl.

=3

=- x

* Sl.

Find tiiC C<}llation o r the pcrp;:nuicul<tr bise<1or of the line ..:grnem


j oining ( - 1. 2) and (3. - 4). (The perpendicular bisector is !he line
1ha1 em~ 1he line s:eg,menl in half and is perpendicular to it)

38. (4. - 2) to l5x - 2y + 3 = 0

37. (6. - 2) lOX : - I

.,.. 53. f ind toordiuatt) of the t;"'Oint that is


points ( I. 2). (-1. 4). and (-3. 1).

39-46 find till; equation of the lir111; de~ribc:d..

+ 39. Parollet to x + 2y 15, and t.hrough the point of intersection of


2x - y = 5ondx + y= 4

lenglh P R
length RQ
where,., and r 2

.. 42. Perpendicular to the line thr<>ush (- 3, 4) and (I , - 2), ond

through tbc poiot (- .' - 2)


C~Glng 1hrough 1he tim qu:.dranl to fonn an isoscele!::
wilh arc:. 8 square un_its

.a: 45. Has slope 2. and Ihal pan in the second quadrant has length 3

* 46.

P.. trdiltl to lhc J.' aXiS. lx:lov.- Lhc point of intcrM:CliOO Of the lines
:r = y :md x -r- .\' = .1, and fonns with 1hcsc lines a triangle with :uc:;~
9 square w1ilS

* SCS.
"'*

I~ a

and

(d) A t what temperature are Tf' and Tc numerically equal"?

y=

R arc

' Y: + ' 2YI

' + Fz

line JX~,rallclto iu,cl f?

57. Prove that in any triangle the sumo( th e squ.uc.s of the lengths or
the mcdiar~ i~ cc.1ual to thn.:c-foonh~ or the Mint of the squares of the
length.;; or Ihe sides. (A mPtlitm C\( ~~ crianglc i11. a line segment drJ.wn

from one vertex IO the midpoinl of lhc opposite side.)


P be any point inside an cquilatcrnl tri3nglc ( figure below).
ShowLetclutthc
su.rn of the distances of P from the
sides is alw:1ys
~

th~c

cqt~a..l to tl1e hcig.htlr o f

the triangle.

(a) Wh:u is the fonnul:a by which we: convcn 1cmpcr.uurc T,.. in

degrees Fahrcnhcitlo tcmpcf'3ttarc Tc in degrees Celsius?


(b) Wh:lt 1t::thc rom,ul!l by which wecocwcrt t.cmper.:atute Tc i n
degrees Ccl~iuslo tcmpcro.~un: Tr in dc-g.rcc:s Fo:hrcnhcit?
(c) Cttn we intcrpn::11hc fo nnulas in ptJfiS (n) and (b) as cqu a~
tjons fOr su-aigiH l ines in a Tr Tc -J)fanc? Draw bOI.h lines.

't

,,

pcilht.: irucgc~. then the c:oordintstQ of

trian~le

44. Throtlgh ( 3. S) and crossing through the first quad ranl 10 form a

1 47.

<21\:

x=

lrianglc with art{!. 30 square units

(rom lhc 1lvce

55. C.encr.ili1c the result of equations l.l la and b to l)fO\C tltat if a


poinl R di,; dcs t he length P Q '"'thai

4 1. Parnllello the line through (I . 2) and ( - 3, 0 ) . and through the


poim (5, 6}

* 4.!.

atuidi~tanl

+ 54. Pro\-c 'lbcoJ:CI.'n I.S.

40. Ptrpendicular 10 X - y
4 , al\d through the poim of intersection
of2t + 3y = 3 ond x - y = 4
~

- To)). Draw its grJj>h.

(b) If steel railrood rails 10 '" long are I:Jid with their adjace llt ends 3 mm apart at a tempcrdture of 20 ~ c . at
what tempernture wiU their ends be in contact? For steel.
& = 1.11x

29. Jx + 4y = 6. x - 6y = 3

30 .\ = 2.< + 6. X -

+ cx(T

"

ANSWERS TO DIAGNOSTIC TEST FOR SECTION 1 .3

10. (5 marks)

l. 3Ji0. ( l /2, -3/ 2) (2 mru'ks)

2. -3/ lO ( I mark)

x=Z

3. ll/ 5, - 11/2 (2 marks)


4.

2x + y - I = 0 (2 marks)

x d y -5

5. Neither (2 mu-ks)

6. x - 2y = I (3 murks)
7. 2x - 3y = 14 (3 marks)
8. (68/ 13. 2/ 13) (3 madts)

-5

9. 2 (2 marks)

I 1.4 Conic Sections


Here i.s th.e diagnostic IC.'\ I for this sec1ion. Gi\e yoursel f 60 minute.~ 10 do iL

DIAGNOSTIC TEST FOR SECTION 1.4

In <1ucst..hm.'i 1-10 identify the curve a.'i a straight line. parabola. c-i rcJc.
elli p~. hyperbola, or none of these.

J,2 - 2y + 3

I . 3x - y = 4

2.

3. '' =.r' + 3

4. 3x' +

5.
7.

9.

.r, - 2xl = .t
x' + y' - 2y = 16
X l + 2y 1 + X ;;:; 2 )'

6.

X =

.v' = 4
X~ +4y1 + j =

+ y l + y :;;;: 0

11 . Which, if any, of the points ( I, 0), (0. 1), ancl (2. - 1) are on the
circle 3x' + 3)'2 + 2y 6x + 5'!

+ + 2x- 4y = 25.
+ 4.r + 6yz + ~Jy = 36. What

_12. Find Lhe c;.cntn:.and riiOius ofthl;circ!c


13. Find (he centre o f lhe elliJ)~ 2.r
arc its axes of syl'luhc-t.ry?

x1

IS. Find equations fOr the asymptme.s of the hyperbola 4x2 - 2y 2

y2

4.r - 10y .
16 . Fin<.l thc poiruCs) of i ntei'S(::(Iion of the ~u1'ves

8. 3x+y'=3

10. x l - x

14. Find equation-; for thc asymptoLC.'i of the hyperbola x1 - 3y 2 = 4.

.rl + lyl

=9 .

17. Find lhe hight!:l poinl onrhe parabola

I S. Find the poi111~ where 1he p:tr:"lbol:l .t

+ 2y =

5 ancJ

y = -x 2 + 6..r + 4 .

= y2

5y -

6 c~~ Ihe

x axis and they -axis.


1.9. Find lhe equ:ujon o f

:1 pa~bo l :.~

that passes through the points

(- l , I), (0. 3), und ( I, l) .


20. Find 1.he equ~ni on ofrhe ellipse with centre al the M igin lh al pas:~
throug h !he poinoes ( - 4, 0) ond (3, 4) .

Parabolas. c ircles, e llipses. a nd hyperbolas arise in a mu ltimde of applications. We do


not give a complete de't'clopment of these cur..tcs together with their many properties here; we
show only how 1he rorm of the eqmuion ror each conic section relate...;; to its. shape. Detailed
d iscussions of conic sectio ns in te rms of foc us. and dirccttix arc ghcn in Sections 9.5 and 9.6.

The Parabola
When the y-coord inalc of a point (x. y) on a c urve is rclalcd to itS x-coordinatc by an equatio n

of the fomt

y = ax 2

+ bx + c ,

( 1.1 7)

where a. b. a nd care constantS (with a # O).the curve is called a p a r abola. The simplest of a iJ
parabolas is y = x 2 (Figure 1.6). For every po i.n t (x, y) 1.0 the right of the y -ax.is on th is curve,
there is a point equidi.sLanLto the left of the y -<llxis w hich has the same y coord inaLe; that is , the
point (- x , y ) is also on the curve. Puuing it another way, that part of the parabola to the left of

lA

Conic Se<tiOfl!>

15

the y-ax_
is is the..image iothc y -axis {thought of a!) a mirror) of that part to the right of they-axis.
Such a curve is said to be symmetric about they-axis. It happens whenever the equation of a
curve is unchnngcd when each x therein is repI aced b)' - x . In other words, we have the test: A
Cllrt.'e is SJIIU11etric abmtr I he y-tLr;s i{ifs equmiun remains unchanged when x i.'i teplaced by - x .
The parabola y = \1 - x 2 is shown in !Figure 1.14. It is symmetric about the y-axis;
replacing x by - x leaves the equation unchanged. The parabola is said to ope11 dowmvarrl.
whereas the parabola y = x 2 in Figure 1.6 opens upwnrtf. The sign of the coefficient of .x 2
d ictates which way a parabola opens (positive :for upward and negative for downward).
The parabo la y = 2x 2 + 4x - 6 is plotted in Figure 1.15. It is not synunctric about the
y-axis; its equation changes when .t is replaced by - x. The parabola appe.ars to be symmetric
about the line x = - I with lowest point ( - I . - 8) . To prove that this is indc~d the case, we
rewrite lhe eqmuion as

10

5
I 2

y = 2(x 2

+ 2x) -

6 = 2(x

+ 1)2 -

8.

This form clearly indicates that the smallest v<tlue for y is -8 , and it occurs when x = - J.
It also indicates that the parabola is synunctric about x = - I. To see this we replace x with

lj!CJI)jl

UU.f

Parabola

opening \lpw"ard

y~ 21: 2 + 4x -6

- I a, where il > 0 is any number whatsoever. The resu lt is.\' = 2(- 1 a + t/ - 8


= 2a 2 - 8. Th.is shows that points oo tl1c parabola with x -c001:dinates smaller t.hao -I and
larger 1han - I hy the san1e. amount a have Lhesame. y-coonlinau:. mlme.ly 2n 2 - 8.
The techniq ue used above to rewrite y = 2x 2 + 4x- 6 as 2(x + 1} 2 - 8 is called comp/etillg tlte square. Tf we apply the S<mle technique to tlle ge.neral parabola y = tiX 1 + bx +c.
wcobcai.n

y =

a(x + ;x) + c (x + 2~
2

= "

r (c- ::).
+

( 1.18)

This form for the eq uation of the parabola shows the following:
1. When a > 0. the parabola opens upward and has a minimum at the point

.!!.. . ,c _ bz).
( - 2a
4a
2. When a < 0, tlle parabola o pens downw, ml and has a maximum at the point

!!.. . ,c _ b2).
( - .2a
4tt
3. The parabola is symmetric about the line .r = -b/(2J:t) ( Figure 1.16); that is, for every
point P ou one side of the li uc. there is a point Q ou the other side that is the mi rror image
of P in the line.
M:ilflii;IWMifM Symmetry

4. The parabola crosses the x -axis when

o=

b
x=- 2o

)' = {/

(x+ :aY+ (c- ~~) .

To solve this equation for x. we flrsl write


p
X

f_ !!_, c \ 2o

bl)

4o

and then take square I'OOIS of each side.

b
X+ -=
~
- 2 - -c .
2

2a

4a

16

Cblpl(r l Calcu.lns Pn!panuion

UjtC)IJ ;li:

inrersec's

5ln I Parabola ahat

.t -a~ is

llj[c\il;lj

in lWO p0in1s

Pambola that

l'oirdbola that

I'Otl;he-s x -a'OS at one point

<IOS r>01

inters! x-a<is

J
"-.

-b - ~11'
> - 4ac

- b + \ b2 -4ac

2a

2tr

Finally, then,

= _.!?_

jb

= -b Jb2- <Ute.
( 1.19)
2a
a
2a
This is quadratic formula 1.5 that we encountered in Section 1.2. but now in a geometric
setting. It determines points where the parabola )' = ax 2 + bx + c crosses the x-axis.
When b2 -4ac > 0, the parabola crosses the x -axis twice (Figure 1.17a): when b2 -4ac =
0, it touches the .t-axis at one point (Figure Ll7b); and when b 2 - 4ac < O.the parabola
x

_ :_

4a 2

has no points in common with the .r-axis (Figure 1.17c).


When x and y in equation ! .17 arc intcf'hangcd, tlte resulting parabola,

x = a/

+ by + c,

( 1.20)

opens to the right or left rather than up or down. Figures 1.18 and 1.19 show the parabolas
x = y2 + l and x = -y 2 + 4y - 4, respecri,eJy. The parabola x = y2 + I is symmccric
about the x-axis; any point (x. y) on the parnbola is synunetric with the corresponding point
(x. - y). Tn general. a curve is symmelric flboul ;he xaris if i1s equmio11 remains unchanged
when y is rep/treed by - y.
I@UjiJ;I:JMTJ

Pat<lhola ope,ning ro r~ right

Parnbola ope12ing to lbe left

FIGURE I.

'I

3
2

-I

-2
-3

tO

15 20

"

3
2

- 10 - 8 -6

-4

/
-4

-2

I EXAMPLE 1.8
Find equations for the parabolas in Figures 1.20 and 1.21.

SOI.L'TI01\ n,c fact that the parabola in Figure 1.20 is symmetric about the .t-axis means
that b in cqu,uion 1.20 must vanish: that is. its equation must be of the fonn x = ayl + c. Since
the points (2, 0) and (0, 3) are on the parabola, their coordinates must satisfy the equation of
the parabo la,
2 = a(0) 2 + c,
0 = a(3) 2 +c.

MOjiUII:IWWIM

PataOOl~

throu,g.h illllY three points

'

--

(4, 6)

-3
These imply dun
2
X= -2y / 9 +

c=

2 arld

fl

= -c/9

-2/9. Thus. the: cqmuton of the pantbolt' is

2.

The pantbola i 11 Figure J.2 1 has no special atuibUles tha[ we can utilize (~uch as the position
of the lile oJ symmetry in Figure 1.20). We therefore u::;c the facts that it.s equation must be
of the. to m> y = u.r1 + bx + c . a11d the three pui11tS ( - 1, 3}. (0. 2}. and (4. 6) are on the
l~'lmholn. Substitution o r th~c coordiMtes into the equation g ives

a(- 1}2 + b(- 1} + c,

The se<.'Orttl equation yield~

= a<oP + b(O} + c,

= a(4)2 + b(4) +c.

c = 2. antl whcnthi~ i!i: substituted into the other two \.'(tuations.


(/, - b = I.

These can be solved for


y = 2.<2 /5 - 3xf 5 + 2.

tl

= 2/ 5 and b

1611

+ 4b

- 3/ S: therefore. the required equ<~tion is

= 4.

Suppose we had not g iven you the pictures in thisexatnple, bul o nly asked fQf the equa tion
of' u pambohtl>as.iny through the pOints. There would have been an additionul purnbola through
the points ( - I. 3). (0. 2) . an<l (4, 6), one ope.ning to the right: its equat ion is .t = (2y 2 -

13y + 18)/3. Only a parabola opening to the left can be found through the points (0, 3).
(0. -3), and (2, 0 }.

I EXAMP LE

1.9
When a s hell is tired from the anillcl) 1 gun in Figure 1.22, it follt>ws a parab<.>lic t>ath

)' = -

4.905
1
v cos1

ex +.r tan@.
where v is. the muzzle velocity of the shell and e is the angle at which the s he11 is fired.
the range R of the shell and the maxi mum height auained by the shell.

)'

1----------- R-----------1

Find

18

Cha.ptt.:r I

Col<11lus Pre~tiOn

SOLUTION We can find R by setting )' = 0 and solving for x:

- 4.905

0 = (

IJ2 cos2 8

+ tan8) x.

One solution is .\' = 0. corrc.:sponding to the firing position of the shell. l11c other ~o lu tion gives
the range of the shell.

.\' =

v1 sin 9 cos9
4.905

Maximum height of Ote shell is attained when x = R/2. in which case

2
2
- -1.905 [ 11 sinO cos8]
)' = ....,...,..:,
+ tan 8 [11 sinO cosO ]
vl cos20

2(4.905)

2(4.905)

v2 sin2 0

19.62

ln many of the examples and exercises of the book. we osk you to drow ondlor plot cut\'CS.
To t>lvt a curve. you are to use a grat>hing calculator or computer. To dmw a cur. e. you are to do
so without these devices. Sometimes. as we shall see. a drawing is more infonnnti,e thttn a plo,.
EXE R C I SES 1.4A

In f..:.Xcrciscs 1- 12 dmw the parJbola. Usc a cnkulator or <."'O'lputcr to


plo1 lhc p;~ rabo l a as a check.

= -.\'1 + 4,\' -

l. y = lx' - 1

2. )'

3. )' = x' - 2.r + 1

4. 3x

= 4y2 -

16.

17.

)'

s. .i = ''' + 2)'
1
6. 2.1' = - + 3x.,. 4

7. x+ .l 1 = 1
In E>en:i""" 18- 23 11nd all points or inten.ection for the curves. In

8. 2y 2 + .r = 3y + 5
9. y = 4x'
10.

<."":'tell ca(.C: drn\v or plot lhc:: cur\'e~.

+ 5x + 10

18.

= lOy'

I I. y

= -.r' + 6.r -

12 r

= - (4 + y)'

y = 1-x' .1

19. y
20. )'

=x+ 1

+ 2x = 0 .1' = I + x'
= 2.t - .T~ - 6, 25 + .T = 5y

11. X:)'()'- I ) . 2,1' : 2X +I

13. Find .Y~ alld ,\' -imer.:ep1s for the parabolas (a)
and (b) x = 4y 1 - 8y
4.

= x~- 2t- 5

= )'1 + ly - 3
2J. y = 6x~ - 2. y = x2 + .r + 1
24. For whal angle 0 is 1he range oflbe artillery shell in Exantple 1.9
12.

= - y1 +

I. X

largest~

In Ex.cwises I..J-1.7 ktcmUnc the oqwtion forcacb par.lbola shO\"'""

15.

14.

(2. 3)

-t

25. lbecablc of the suspc.nsion bridge in the follcrwiog figure bangs in


the shape of a parabola. 111c towers an:: 200 111 apart and ex lend 50 m
abo,e the roodway. l.flhe c:~ble is 10m :.lbo\.'e the roadway at its Jowcs:l
point, find the length of the supponing rods 30m (rom the towers.

Tower
Cable

50

10
200

Roadway

*' 26.

Find I)OillLS of intcrbCtlion (QI' Lhc

Sx = .''

l~trnbolas J

(U) Usc lh c f'ac iS lhul n,:~ist;mc;'('$ OI ICUlJ)Cntlurcs o~ c. IOO~ c.

(X - 2) >Utd

und 70Cf'C arc, I'C$pcctivcty, IO.(X)() 0 . 13.946 Q, aad


24.112 Q todctcnninc Rn.a. and b.
(b) Plot a gro:.~ph of 1he func.tion (In the intc:r\ttl 0 S T 5 700.

4.
21. Fi.nd tile h<:ihl oflhc p:u<.ibolic nrch in the figure below.
-

(c) A l wh~11 tCnlJ)Cr.tturc i~ lhC rcsislance 20 0 ?

:=.-;-+
--:'

.;

t:

I ! ..
.. i...........
~

'----- , - - 5 - - L _ j

30. Tht: l)atabolu y ~ x z - 1 in Lhc following figure n.-pe-cscni.S the


lxlSC of a w:.tll pcrpcntlicuku lo the X)'IJI:Ine (X and )' rn~;:::ssutcd in
me1res). A rope is auac-hcd to a stake :.t posi1ion (3t 4) , pul led tight,
wr01ppcd atouncl thm pan ofthc ba~ o f the wall containing the vertex
tJf the p!lrabo1a. and tied 10 the vertex. A t \\ll~~ pcinl th.:~: il meet the
wall?

... :2:8. Dclcmlinc the C.."t(u:uion of a parJbola l>flypc 1. 17 pa'5sing throu,(lll


ll1C poon" ( I. 2), ( -3, 10) , W\d (.', 4) ,

i "'

29. Rcsis1aoc-c R in ohms in t1 pla1imun rc.liiswnce lhcrmomctcr i:s


rdmc.d to lcmpcl'auu.: T in dcgr<cs Celsius by t.h<.: <..qu:aLion

4)

II = /10 (1 + aT + bT').

The Circle
\Vhen the x- and y -coordinAtes of point~ on a curve are rehued by a 11 equatiol of the form
(I 7 1)

where h. k , and r > 0 :.lreconslant~, the curve is cillled a circle. lt Utkes but a quick recollection
of disumce t'onnula 1. 10 10 c onvince ourselves tha t this definition of a circle <.~oirt<.:ic.lcs with our
intuitive idea of a circle. Lfwe write equation 1.2 1 in ahe fom1
c~ntre

(11.k)

;'lttd

I"Uditu r

./(x - h)!+ (y - k)' = r,


the left side is the distance frorn the poiH (x, y) to the poim (h, k) . Eqluuion 1.21 therefore
describe.< al l poinJS (.r, y) at a fixed diSJance r from (h, k), a circle centred at (h , k) with
mdiu.; r (figure 1.23). For cXllnll>le. the mdius of the circle in Figure 1.24 is ~-qual to 2. and its
equation is lherefore
1
(x + 1) 2 + (y - 2) = 4 .

When cqumion 1.21 is expanded, \\e h.we

x2

Circle wifh

tttre ( - 1, 2) and

mdi~JS

x1 +

2/r x

+ h2 + y 2 -

l- 2h,f -

2k,\

+ k2 = r2

2ky + 11 2 + k1

or

r1 = 0.

This shows that the equation of a circle may be given in another fonn, namely,

x 2 + y2 + f x

+ gy + e =

0,

( 1.22)

( I. 2)

f. and g are constants. Given this equation. the.centre.and theadius can be identified
by reversing lheexpansion and compleling lhesquares of x 2 + fx and y 2 + gy . for instance,
if x 2 + y 2 + 2x - 3y - 5 = 0, then

where. e.
-I

(x + 1)2 + (y - 3/2) 2 - 5 - I - 9/4

(x + 1)2 + (y - 3/2) 2 - 33/4.

"rlle centre of the circle is therefore (- I, 3/2) and ilS radius is ./33/2.

20

Chav,c:r I

Cak:uhlS Pr~p: raliun

When the centre of a circle is the origin (0 . 0) . equation 1.21 simplifies 10


( 1.231

Be carc.ful to usc equation 1.2 1~ not equation 1.2 3, when the centre ofthc circ.1c is not the origin.
It is a common error to use equatioll 1.23 .

I EXAMPLE 1.10
Figure 1.2 5 shows an arc of a circle. Find the eq uation for the circle.
Equahon ol

SOLUTION From the symmetry of the figure. we see that the cemre of the circle is o n the
)'axis. hs equation must be of the form

n cirde p;.1ssing throus b :l points

3
2
Bec~use

(4. 0) is a poim on the circle. these coordinates must satisfy its equation; that is.

or

-2
Simi larly, since (0, - I) is on the circle,

( - 1 - k)l = ,.2

or

If we subtract lhesetwocquations, we o btai" 2k - IS= 0, from which we sec that k = 15/2.


Consequently, r 2 = 16 + k 2 = 16 + 225/4 = 289/4, and the eq uation of the circle is

x2

I EXAMPLE

+ (y

15/ 2)2 = 289/4 .

1.11

The straight lines it1 Figure 1.26a represent a welded frame where A 0 B is a right angle. The
circle rep1esents a wheel rotating on a pin through 0 perpendicular to the plane of ~tc frame. If
the minimum cle.ar.mce between circle and side. A B must be 10 em, find the maximum radius
of the whc:cl. l ocate the point on the circle closest to A B.

M4Uii1.14CWJ.M

Wheel loi)inning

(I ll

wdlk:tl (r.uue

Milclii.I"J\IEI!lli

R~uricnl ali on o(

whoel on (r.unc

fi

~01 l no:-. Suppose "~ roca1t and Hip the figure and ~s1ablish 1he coordinale S)<l~m in
Figure 1.26b. Le11he moin>um r:>dius of 1he circle be r and lei P(a. b) be 1he poinl on l he
circle closes! 10 A 8. l n1ui1hcly. 1hc shones1 di>Wncc II P QII between wheel und A 8 occur>
when line 0 P Q i< perpendicnlann A 8 . Since lhe<lope nfl he line A R is - 1/2. i1< equalion is

y - I

I
- - (x - 0)
2

+ 2y -

,r

2 ~ 0.

According 10 formula 1. 16. the distanee from lhe origin to line A B is

IOQI

= I<O) + 2(0) - 21 = 2...


J1 1 + 2'

../5

For lhe length of PQ 10 be 1/10 m. n follows 1ha1 r + 1/ 10 = 2/../5. and lhe radius of 1he
wheel is r = (2/ ../5 - I /10) m. To locale P . we note l hnllhc slope of 0 P is 2 (Ihe ncgu1ivc
of 1he recipr0<.-.1 of lite >h!pc <~fA 8 ). h follows thai bfa = 2. Funhermore. becaus~ P b 011
.
' we musl hn1~ a.. + b' = r , . \" hen we subSIIIule
.
Ibe ctrcle
(whose equauon IS x-, + )'"., ~ r),
b = 211 into tlli> cqu.stion. \\C <>blain

EXERCISES

In l:..terci~ 1- IOdr.lw lhe cm:le. U<e 9 C9!cul91or or compnetlo pial

l.

13.

4.

+ ,-: a: 50
2

x' + 2x + .' = 15
x1 + y 1 - 4y + 1 = 0

+ y'- 2y + I 2r 1 + 2r' + 6.r = 25

S r - 2.r

4x

+ y' -

2y =

-3

'- .../

(3. 2)

(S, I)

(2. -2)

17. A l:ldder of length L rcsh vertically against a wall. rr the ao"'er


end of the ladder is mo\ed along IC\cl g.roond a"'-ay rrom the "all while
the top of lhe ladder renwns 111 conloct \\'ilh lhe walL find an cqu:uion
f the cut\'e followoo b y OIC midpoinz of lhe ladder.

12.

16.

)
-/ L
3

In l:):CI'Cl"S I J- 16 find an cquahon tc.r Lhe card e.

------ \3..

(-5. 6)

+ 6x + 3y + 20 = 0

I I.

12. 7)

9. x 1 +y'-l.r-4y+5=0

Ill. ' + >'

= ../5'

14.

k:\
_h_/....

15.

7. 3.t 1 +Jy 1 + 4.r-ly=6


8. .r1

../5

2r

6.

and

)'

2. (X ,. SJ' ,. (y - 2)' "' 6

.1.

1 .4 8

\he circle: as a C'hcck..

.x4

a=

18. Find lhe equation of a cirde thai passes throogh lhe point< (3, 4)
and ( I. - IO).and has itsccnli'C(a) on thcline 2x + 3y+ 16 - 0 and
(b) on the line .t + 7y + 19 = 0.

ln ~-<.crciscs 19- 22 tlnd points of inlcrsec..1i<ln for the curves.

+ 2.v + y 2 = 4. ~ = 3x + 2
x 2 + )'2 - 4)' + I = 0. 2x + ." =

19. ,v2
20.

r.tdi t.~s (Kc Cxcrc1sc 24) and, ( b)

laking lhc. cqui:lion of the. circle in


form 1.21 anrl requiring A 1 B 1 :and C to be on lht circle.

* 28.

Prove thlltthc three ahitudet of the tri:tng.1c in Exercise 21 intersect


in a pGnH c:etlled th..: orthcx:tlltte of the triangle.

+ 11.

+ )'2 = 9. ,r = 3x 2 +4.
(x + 3) 1 + .v' = 25. y' = 16(x + I)

* 23.

Show thut C\'cry c;quulion of the form 1.22 represent" u circle, 01

**

"' 2 1. x 2

llOiOI, or no1hing a1all.


<f!

24, PI'OV lh!U the pct~ndk:ul ar bi~cctor oru clwrd


1:ms10es through the ccnuc of tl1e circle.

I(AII + Bk +C) r,f A' + B'l


.JAl+ fF

or a Cii'Clc always

"" 25. Two Iights nrc I00 n apart. one nt the OC'igin, <uld the OlhCI' ul J)oint
(100. 0) in the .x,rplunc. T he light at the origin is 10 tirncG ns hright
as the othct' light. Find. and Uraw. ull points in the xy-plone m which
the umOtu\1 of light recd\'Cd (rom both sources i ~ the sum..:. A)'}sunlc
th,\l the QIUOUOt Of Ji&hl fCCC.ivCd ill a l)()int is directly prOJWlrlion:ll to
the bi gh t 'K:~S o f the souu:e 211d iu,erscly proporLiMil to the square or
the di'\Ulncc rrom th-e source.
' 26. Two loudspc.al.:crs arc 20 1h ''Piltt. One is :H the origin, fuld LJ1e
other is at the point (0. 20) in the x )'-plane. '111c speaker Ul lhc origin
i ~ only 1<Kf 1u loud w; 1~ other. Fi nd. and dn\w, nil points in the Ji,Y
plane ~ll whi<:h the :unount or sound n:ccivcd fro m boch ~pc;_tkCI'$ i~ the
.sam~. Ab.SUIHC th;al th<: umount of $Ound -cc.civcd at a point is directly
proponionW to the lou:JtlCSS of tile apcakcr und ira\'Crscl) pl'upotionul
to the t;quarc of the disuncc: rronl the s peaker.

27. 'fhc t'i m un<'inltt for a uilngtc is that dtdc which pas:s.es through
all three of its \'c nices. Find the cin:umcirck for the 1nungk: with
vcnicos A(l. 1). 8 (-3. :1) . and C(2. 4) by (a) Onding ilsccntre and

29. Show thai i ( a l i1\C Ax + By + C


0 rul1 a circle (,l ' - 11) 2
2
2
+ (r- k) r do no1 inlcn>cct. then lht s.hon.cst d i!)(:tnu between
lhCnl is the Slll8J!Ct' or the tWO numbers

**

30. n \e int'itd t"' of a Lriunslc is tJun circle \\hich lies interior to the
trian~lc bur tmache~ t)Ji thrc:c sides. The ccmrc of the inc.irclc is called
the h1umrr. Show that ihe incentre (..\", )')of the lriang5e with vertices
(0. 0). (2, 0), und (0, I) must satisfy thejuu<ions

l.tl

lx + 2v - 21
= 11'1
'
' =
J5'

Solve lhc.-.e equntions for the inccnlre. and explain why thcre ate four
points that satis fy these cquuti\)DS.

**

31. Luud!~ix:ukcrs ul points (x t . ,vt ) :md (.t~ .\'2) emil sounds with
intcnshies ' and !, , rcspcetivcly. n1c arnount of sound n.x:civcd ut u
pOitH (.r, y ) from eith~ r speaker is: directly t>rop<"N'ti()l\3l to the i n t cn~ity
of the sotJnd ul the speaker ond in"crscly proportionnl lo the square of
the <.listan<:c from the point to the source. Show that all points in 1hc
.ty planc :,u whkh th..: amount or smmd r<..'Ccivcd l'l'om both sou.rcc:s
i s equnl lie on ~l circle with centre (>n 1hc line thrCiugh (xh y 1) :md

(x,, )'J) .

The Ellipse
The SCI of poims whose coordinates (x, y) satisfy an cquatiorl of the form
.. l

y2

-; +,., =1.
a-

( 1.24 )

"

where a and bare positive constants, is said 10 constitute an ell ipse . Since 1his equation is
so similar to equation 1.23. and is exactly I he sanlC when c1 = b = ,.. it is not llllr'C4:t.Sonablc
to expect that the shap~ of this curve might be similar to 3 c irdc, cipccially when values of a
and bare close 111ge1her. Tl1is is ind~ 1he case. as Figure 1.27a and b illustrate. The. e llipse
is symmc1ric abom the X and y-a.xcs, and 1his is cork~istenl with <he fact thlll cquu1ion I.24
remains und'I4:111Sed wher'l x ar'ld )' are replaced hy - x 4Htd - y . The e ll ip~e is eklllgated ln the
x -direction whe11 a > b (Figure 1.27a), and when b > a it is elongated in the y dircction
(Figure I.27b). The pOinl of inlcrsecliOn ()[ !he lines of symmetry of an ellip'e is called the
centre of the ellipse. For equation 1.24 the centre is the origin since the .r -and y-axes are the

lines of'symmeuy.
'1'

.\'

-o

a x

-b
-b

1.4 Conk Seclion.-

23

I EXAMPLE 1.12
Find the equation o r the ellipse th:\t has its tentre at the origin, the x- a ndy-axes as axes o r
symmelr)', and passes through the points (4, 1) a nd ( - 2 , 3).
SOLUTION If we substitute coordinates of the points imo equation 1.24 (since they a re both
on tbe elli pse).

42

12

a + fP ( - 2)2

16

===}

32

-al- + -bl =

===}

+ 11'

= 1,

+ b2 =

al

I.

When the first equation is multiplied by - 9 and added to the second, the result is

140

- -

a'

= - 8.

1l1us. a 2 = 35/2. and when this is substituted imo the first equat ion.

32

35

The equation required is therefore 2x 2/35

' = -.
35
3

- + -2 = I

b'

+ 3// 35 =

I, or 2x 2 + 3yl = 35.

---When equation 1.24 is changed to


(x - 11) 2
.:_...,..c... -'-

a2

'

k)z

()' -

b1

- I
-

( 1.25)

"

where h and k are conslams. the curve is still an elli pse; its shtpe remains. the s.ame. Just as
a change fro m equatio n 1.23 to 1.2 1 for a c ircle mo,es the centre of the circ le from (0. 0) to
(/J , k) , equation 1.25 moves thecentreoftheellipse to (h , k) . Li nesx =handy= k are
the new lines of symmetry (Figure 1.28}, and a and b arc the distances be tween the centre and
where the ellipse crosses the line.s of symmetry.
M4Ujil.tojlJEliM

EtliJ)* with ttntre (/r.k)

)'

-1-t-~~-~-~- y =k
(II + 11, k)

x = i1 (h.k - bl
X

I EXAMPLE

1 .13

.._...
Find the centre of the ellipse 16x2
SOI, UTION

+ 25y2 -

160x

+ 50y =

I 175. and d raw the cllip.sc.

When we conple te squares oo the x - and y-tenus,

16(x - 5) 2

+ 2S(y + 1)2

(x - 5)
.;,_..,...,...:....
+ (y + 1)
2

= 1600

or

The cenue of the ellipse is (5 , - I); it cuts the lines y


8 units from the centre, respectively (F'igwe 1.29}.

100

=-

I and x

64

= 1.

= 5 at d istances of I 0 and

24

Ch:lJll<r I Calculus Prtporation

IJ(rlli;l

SlM

t\n ellipse gr.t.phcd from i1 ~ C<JUation

(15. - L)

(- 5. - 1)

(5, - 9)

EXERCISES 1 .4C

In Exercises 1-% drawthe ellipse. Use a calculator or co111putcr to pi()(

* 10. Find the width of the elliptic arch in the figure below.

the ellipse as a check.

I.

:t.

/ 25 + )' 1 / 36 = I

2. 7.t 2 + 3)'' = t6
3. 9x 2 + 289y 1

4.

= 2601
In ExerciSts 11- 16 find all points of intcrsoction for the curves. In
c.ach case dmw the curves.

:u 2 + 6y 1 = 21

S. x' + 16/

=2

x 2 + 4y2 = 4,y = x
12. 16x' +9y' = 144,)' =x + 3

II.

6. x1 +2.t+4y 1 - 16y + 13= 0


1. 9.\ 2

+ y' - tSx -

H. .t 1 +

J .t

6,1'

~ IJ. 9.t 2 - 18x + 4y2 = 21, 2y = .f3x + ./3


l4. 9x2 - 18x .,. 4y2 = 27, 2y = - .f3x + 5./3
-. IS. x 2 + 4y 2 - 8y = 0, y = x !

= 26

+ 2y 1 + 16y + 32 = 0

16. x 2 + 4y2 = 4. y = x' - 4


9. Find the equation ofrnellil>~c:thatpa$~ 1hmugh lhc point,:: ( - 2, 4} ** 17. Show 1hat e \oery ~..,int on lhe ladder in Exercise 17 of Stetion I.AR
and (3. t).
foii0\1/S an cllipM:.

The Hyperbola
Changing one sign in the equation of an ellipse le<~d> to a cuf'e with totally diiTerent character
istics. The set of poini.'S whose coordinate.\ (.x . y) satisfy an equation of the form

{12 -

yl
b1 =

)'2

.(1

X"'

b2 - a l = I ,

or

( 1.26al
( 1.26b)

where l l and b arc positi vc constants. is called a hJperbola. Hyperbola I .26a cro.sscs the x -axis
at x = a. but does not cross the yaxis. Since the equation remains unchanged when x is
replaced by - x and y is replaced by - y. the hyperbola is symmetric about both the x axis
anu they -axis. It follows that i.f we dmw thai pan o.f 1he h)perbola in 1he first quad ran~ we
can obtain its seco nd, third, and fourth quadrant points by retlec.tion . By taking positive square
roots of

we utm:tin
Ono
quoocr of tbt hypertula
, :fa: y:/h1 ; I

from which

'

.
~

h)pctl'\tht

lWI)J;

/)

\~~-
. II

t:ntu'l:

its ..ymn-=tl)

y=!!.x "/.

- a1

Titi\ cqutttion descnbcs 1~ tol' Ntl( of the h) JJ<tbola. BccauL-..c a j,. a fixed constallll we can ~y
1
dat ror large "alues of \". values of a 2 are in:s.ig.niftea~u conlparcd to \"8IU(';$ of x and v111~
of y arc appro<imltCI) equal tc> b .</ a This means dtlll for laiJie values of A , the hypetboln i>
very close to !he li11e y
b <f a. We ha;-.: shown these facts in Figure I..lOa. The complete
hypcrbolu. obtnined by rcne(.'ttng Figure 1.30a in th~ ue~. i.!oo )hown in Figure l.JOb. 'fh c liiX"
y = b.<I a that the hypct"bola appro.1chcs for r.uge 1><><ihvc thKI ncg3tive values of .< nrc C;tlled
a symptotes of the hyperbola.
Hyperbola 1.26b Is shc>wn in Figure 1.31.

.!

".

= ~ .Jx

Ill

~ -

v--.-
.
a

'

. .

.<

." >-a r

Jl

I EXAMPLE

1 .14

Find thecquariort o f a hypcrbolo lhal cur~ lhe yaxi s ,;u y = :i:S tHu.J

h ilS

l in~s y =

.xf J3 tt;;;

n~y mpcvtcs.

SOl U liON Since the hyperbola crosses the ,:VUI> at y


I U>b " i1h b = 5,

,.z

= 5. \\C wriiC its equation i n form

.\"2

- - -= I
25
a l

\Vhcr' we ~ohe t.hjs equ.-rion for )" in terms of .:r. the result j,

Since asympOLes or this hypcrbolu itte 5xfa. it follows thnl 5f a


The equation of the hypetbola is !herefore

y'
x'
- - - =1.
25
15

1{../3. or a

sJ3.

26

Ch.1pw:r I Cn!:ulu:s Pn:(Y.lr:uion

Whe n

and y in equations 1.26 are replaced by x

- h

(x - IJ)l

(y - k)l

Ill

bl

(y- k)l

(x - h) 2

b!

(12

andy- k. the result ing equmion~

(1.27a)

and

(U7bl

sti ll describe hyperbolas. They are shown in Figures 1.32 and 1.33. T hese are ~l<' hyperbolas
of Figures 1.30 and 1.31 ' hi fled so that the asymp<otcs inten;cct at the point (h. k). Equations
of the 31>)1mptotes ~re y = k b(x - h )/a. and the lilies x = I! and y = k arc now axes of

symmetry.

MUUIIIj

Milciii;IWHCFM

Hypelbola witb teltre at. (h , t )

Hype.bola w1th ceu-re 111 (h, k)

/(,.*>-..

...

../...

~-.

I EXAMP LE

1.15

Find asymptotes for the hyperbola .r 2 - y 2

'

+ 4x + lOy= 5.

SOLUTION Tf we complete square.< on x- and )'-lem", we obrain

mm

Ahyp..-

ool.- gn.tphed ftom iu. eq;.atioo


(.r

'. /

+ 2) 2

()' -

5) 2 = - 16

(y -

The

axe~;

of symmetry

or the hyperbola arc X =

5)2

16

(x

+ 2)z
16

= I.

-2 ''nd )' = 5. intersecting eil the point

( -2, 5). When we sol\'e for y in terms o f x, the result is

<
-2.5!:: :..

y =

(-2,'1)

'
.
,v = 5 +(X+ 2)

or

y= 5 - (.n 2)

Therefore, the asymp1o1cs are y

..-..

s Jcx + 2)2 + 16.

5 (x

+ 2). The hyi>Ctbola is shown in Figure 1.34 .

Hyperbolas are but one o r many curves that have asymptores. For insrance. in Figure 1.35.
rhe curve y = (x 3 - 3x 2
l}/(.r 2
I) is asymptoric to rhe line y = x - 3 . The curve
y = 2e-x'/ lOO is asymptotic to the Xaxis (Figure 1.36). Limits (d iscussed in Chapter 2)

provide a unifying structure for all types of asymptotes.

I J Co.ic 500n,

27

flOUR~ 1 . ~

_j_
- 20

I E XAMPLE

o.s
10

- 10

"-----20 -.<

1. 16

To the righ1 of 1he righ1 hranch of Ihe hyperbola rZ - 4yz = 5 in Figure 1.37 i.a <wmp. A
p1peline is 1o origina1e from poin1 ( 15. 10) 10 mee1 wi1h pipeline running nonh from poin1
(- I , -100) along the line .t - I . The pipeline muSI meelthe north-south line as for down
lhc:linex =-I aspoosible. Gi,cnthauhcpipdinefrom (JS.IO)shouklheslnlighl,delerminc
where it should mcc1 the northsouth pipeline.

MJt.III.IME

Bot li1tc W ug whkh to build a pipeline


)

/'1 (1~ .

10)

s
Sw;orup

- 10

10

P1(- 1,))

\ OJ lllO'\: The pipelme from P1(15, 10) hould mee111>e loner= - I al P:(- l.y) <o
that li ne P1 P2 just IOU<hcs the right half of the hyperbola. If we lc1 m b<: the slope of P1 Pl,
lhen U\ ing point-~lope fllmlul~ I . n. I he e(JU<llion of li ne PI P, ;,

y - 10 = m {.t - 15}.
'Jb find I he required po<ouon of P2 line P1 P2 mu>1 imersec1 1he h) perbola in exaaly one 1>01n1
(most lines inlcrscct in 1wo points. or OOiatalJ). Points of im=ioo are found by soh ing
)' - 10 - m(x - IS)

one!

Substituting from the fi rst equation into the second gives


.1

4[m(x - IS)+ 101 2

5.

28

Chapcer I C:li(.1JIUS Prep;lrtUion

u we expand the second 1em> o n Ihe left,


.Y 2 - 4fm 2(x - 15) 2 + 20m(.r - 15} + 100]

5.

This can be rearranged imo the form

( I - 4m 2)x 2 +(120m 2 - 80m )x

+ (- 900m 2 + 1200m - 405) = 0.

Given a value. form. solutions ror x oflhis quadratic equation (jre x -coorc.linates of points of intersection of 1hc line with slope m and the hyperbola. We want o nly ooc solutio n. Conscqucndy,
the discrim.i nant mus1be equal co uro.

(1 20m 2

80m) 1

4(1 - 4m1 )(- 900m 2

I 200m

- 405} = 0.

Whentenns are expanded. this eq uation reduces to

0 = 176m 2 - 240m

+ 81 =

(4m - 3}(44m - 27),

and therefore.solutio ns are m = 3/4 and m = 27/ 44. Thus, lines through (15, 10) with slopes
3/4 and 27/44 to uch 1hehy1>erb<lla monly one l)int. The line wilhsmaller slope 21/44 touche.'
the left branch of the hyperbola. The line wi1h larger slope 3/ 4 1oucbcs the right branch: this is
lhe line we wan1. TIS equa1ion is)' - 10 = (3/4)(.r - 15) , and it cu1s lhe line x = - I when
y ;; 10 + (3/ 4)( - I - 15)
- 2. Thus, the pipclio>c should meet the noo1h-soulh l>ipelinc a1
!he point (- 1, -2) .

We shall 6nd a much easier solutio n 10 this problem when we have scudied some calculus,
bur 1he solution above shows thai it can be done without e-alculusj albeit no1easily.

EXERCISES 1 .40

ln Exercises l- lO draw Ibe bypc-rbolu. Usc u cukulalor or computer to


plol Uoc toypcot>ola iiS check.
I. yl _ xz = I

2. x2 _ y2 = I

3. x2 -y'j l6 = 1

4. 25y2 - 4x 2

5. y 2

10(2

+ x')

7. <2 - 6x - 4y 2
8. 9x

9. 4y' 10. x '

In Exercises 12- 18 find all poinlS or imersec1ion for 1he curves. In


e:lch t:lse dr:tw the curves.

100

12. x1

6. 3x' - 4y ' = 25

24y = II

sx' + 8y -

= 91

lOx= 21

16y2 + 64y

= 79

2y!

IJ. 9)' 1

16y' - 18x- 64y

+ 2x-

-+ ll. Fi.nd l.hc cquution of u hypcrbolu tbuLpasses th.rough abc point


(I, 2) and has w;ynoptote< .v = 4x.

= I. x = 2J

4.x 1

= 36. J =X

*
*
*

14. 9y - 4.<' = 36. X = Jy


15. 3.r 2 - y 2 = 3. 2x + )' = I
16. x1 - 2x - y1 = 0. x = y 2
17. x1 - 2x- y 2 = 0 . x = - y2
18. 9(x - 1) 2 - 4(y - 1) 2 36. 21x

= 5(y- 1) 2

1.5 FuncliUllS and Their G l'tl~

29

ANSWERS TO DIAGNOSTIC TEST FOR SECT ION 1 .4

I. Straight line (I mark)

12. (-1, 2), J3(i(3maru)

2. Parabola ( I mark)

13. (-1 . - 3/4),x

3. None of these ( I mark)

14. y =

4. Ellipse (I murk)

=-

l.y

= -3/ 4 (4marks)

=xtv'3 (2 marks)

5. Hyperbola (I mark)

IS. 5/2 ./l(x - 1/2) (3 marks)

6. Noneofthesc( l mark)

16. (4. -3). (19/3. -2/3) (4 m:tfts)

7. Circle (I nmrk)

17. (3. 13) (3 marks)

8.

Parabola ( I mrt)

18. (- 6, 0), (0, 6), (0. - I) (3 marks)

9. Ellipse (I rnark)

19. ,1' = 3- 2.\ 2 (3 marks)

10. Circle (I mark)

20. 16.< 2 + 7y 2

11. (0, I) t3 mark.>)

2S<i (4 rnarks)

1t.s Functions and Their Graphs


Here is the diagnostic test for this section. Give yourself 60 minutes to do iL

DIAGNOSTIC TEST FOR SECTION 1.6

ln questions I and 2 (i nd the l:argcst possible dom:1in f(\r Ihe function.


I.

2. f(.c) = (.,. + 3)/(.,2 - 2x- 4)

f (x) = .,[4::::Xl

16. [(x) =

18. Tile graph of a function f(x) is siKJWn 10 the left bciO\'-',


graph of the function [( - X) on the axes to the right.

-J4-"- xi

4. / (x)

(1.1)

= x' - 2.r
7. /(.t) = .t'- 2X

S. f(.r)

6. f(x)

=x' + 5x' - x

8. /(X)= (X+ l)/(.r 2 - 3)

9. Find tlrc even and ood pans uf tlrc rw-.:tion [(x)


10. Doc:, the CQua.tion .1.

= yl -

= 3+ 2l.rl
(2, I)

In q uestions 5-8 dctcm\inc whether the runclio n is C\'Cn, odd. or neither


even nor odd.
2

Dr.~w

In qucstions 3 and 4 lind the rnnge of the function.


3. f(x) =

17. f(x) =.r 3 - 4.r

l.r 3 - 81

.r

A'

19. lfthesrphof [(x) iSihatinqucsrion IS,dr:>wasrphof f(x+2)


on the len sc1 of axes below.

= x j(x + 1).

)'

2,\' + I define .\' as a funcaion of .t?

I I. Whal i.s a rt!tional l'unction~

In que.stion~ 12-17 dr.tw Ihe gmph of the function.


X

12. f(x) = 8 - l.rl

13. f (.r) = ,/9- x'

)4. f(x) = - 2./X

15. j(x) =

- ll -

x 2 1- (.r' - I )

20. lfthcgr.lphof [(x) is that in question 18,<11-,wagroph of [(2x)


OJ' the right SC( oC axes abo\'C.

Most quantities that we encounter in everyday life are dependeru on many, many other
quantities. For example. thi nk of what might be affecting room tempe rature as you read this
seotence - thermostat seniog; outside temperature; wind conditioos; insulat i.on of the walls.
ceiling, and Hoot1i; and perhaps other factors that you can think of. Functional notation allows
interdependences of such quantities to be represented in a very simple way.

30

Chnpctr I

C;.J~u) U$

Prepornti-on

When one quantity depends on a second q uantity. we say that the first quanti ly is a function
of the second. For example. the volume V o f a sphere depends on its rndius r ; in particular,
V = 411' r 3 f 3. We say that V is a function of r. When an object is dropped, the distance d
(metres) that it falls in time I (seconds) is given by the formula d = 4.9051 2. We say that tl is
a function of 1 . M athematically, we have the following detini1ion.

DEFINITION 1 .3

A quantity y is said to be a function of a q" antity x if there exists a rule by which we can
associate exactly one value of y with each value of x. The rule that associmcs the value
of y with each value of x is called the function.
If we denote Ihe rule or runction in this definition by the lcucr
assigns to x is denoted by f (x), and we write

J . Ihen Ihe value Lh:ll J

= /(.r) .

( 1.28J

In our first ex.ample above, we write V = f(r) = 41l'r3f3, and the liu>cliOil f is the
operation of cubing a number and !hen multiplying the tc>uh by 411'/3 . Ford
/(1)
4.9051 2 , I he function .f is the operation of !'quaring a number and multiplyi ng the resull by

4.905.
We e<lll .~ in equation 1.28 1he lndepend enl varia hi e because values of x are subslituled
into the function~ and y the de:pendent Vttriable because its "nlucs depend on the assigned
valUe: Of X . 111c domain of a functi(lll is I he Se t of all SI)CCific<J (real) valueS for the independent
variable. It is an essential part of a function and should alwltys be specified. Whenever the
domain of a functjon is noLmcntioncc.l, we assume thnt it consi.sl.S of all possible va.luc,s tOr

which /(.r) is a real number.


As the independent varittble x lakes on values in the dom11in. " sci of V1tlucs of the dependent
variable is oblained . This set is called the mnge of the function. For the function V = f(r )
= 4;r 13 {3, which represents the volume of a sphere, Lhe largest possible domain is r > 0. and
the corrc:;po11ding range is V > 0. Note thai mathcmmically the function j(r) is define<! for
negacive as well as positive v~tlues of,. , and r = 0~ it is because of our imerpretouion of r as
the mdiu of a pherc that we rcLritt r > 0. 11lc funclion d = .((1) = 4.9051 2 fcprc:$Cnl'
the distance fallen in time 1 by an object IhaL is dropped al timc 1 = 0 . It' it is dropped from a
height of20 m. then it is clear 1hat the range o f this function is 0 :; tl :; 20. The domain that
give.< rise to this mnge is 0 :; I :S j20/4.'~J5.

I EXAMPLE

1.17

Find the largcSLilO<'ISiblc domai n for I he funct ion

.f(x) =
SOLUTION

8 + 2x -

xl

x + l

We begin by factoring the quadratic in the numeri1tor

f (X) =

J<2 + .r)(4 .t

+I

.r)

and note that .f (.r) is defined whenever (2 + .<)(4 - .t )/ (.t + I) ;:: 0 . To derem1ine when this
is Lrue. we examine. in tabulcar form. signs of the individual factor:-;. The (irst lint! in TabJc 1.1
indicates Lhat x + 2isposil.iveforx > - 2 ,andis negaliveforx < - 2 . Thesecond and lhird
lines show similar results for 4 - x and x + J. The last line of the table t'OUlllS 1he number or
negaLive signs in the lines above it for the intervals x < - 2, -2 < x < - I, -I < x < 4 ,
and x > 4 to anive at !he sign of (2 + x )(4 - x ) f (x + I) . When we note 1hat division points

1.5 Pnt":tions and 'l'lle 1r Gr~

31

-2 and .t = 4 are. accepL.able. but x = - 1 is not (it gives di visi on by 0). the. largest
domain of the fuoctioo consists of all values of x in the intervaJs x =:: - 2 and -I < x ~ 4.

-3 -2 -1 0

::~: ::~::!:::~::::: ::::::::::::I::~:::

(2 + .<)(4 -

I EXAMPLE

x)/(_;'++1~ :::~::rJ:::::::L:::::r:~:::

1 . 18

Does the equation x - 4 - y 2 = 0 define y as a function of x for x 2: 4?


For any x > 4. the equation has two solutions for y:

SOLUTION

y= Jx - 4.
Since the equation docs not define exactly one value of y for each va.luc of x, it docs not define
y as a function of x .
If we add 10 Jhe equation in Example 1.18 an addiJional restriction such as
defined as a function of .\' , namc.ly,

y
Note that .r - 4 -

Jx -

0 , then

y is

4.

y 2 = 0 does, however, define x as a function o f y:

X= /
In this c.ase,

y 2:

is the independent variable and

+4.
x is th e dependell t variable. In other words,

whenever an equation (such as x - 4 - .r 2 = 0) is to be regarded as defining a funcaion. it musl


be made clear which variable is to be considered as independent and which as dependent For
example. the equa1ion x - 4 - )' = 0 t!efi nes y a.s a function of x. ant! x as a function of y .
ln the study of calculus and its applications we are i11terested io 1be beha"iour of functions;
thai is, fo r cenain values: or the inc.lependent variable. what can we say about the depe.mlenl
variable? The. simplest and most reveaJing method for displaying characteristic.s of a fu nction
is a graph. To obtain the graph of a function f (x) we use a plane coordinacized with Canesian
coordinalcs x and y (in short, the Cartesian xy-plane). The graph of tbe function /(.r)
is defined to be lhe curve wi th equa1ion

y = .f(.t: ) , where x is lim ited to the dnmain o f

the function. For example, Jhc graph of the function /(x) = x 3


Figure 1.38.
Mitijll;iiJIUi!M

C.ropb of a fuoetion

-40

27x

+I

is shown in

tttl~ .:un~ l,; II. p k r.vl.:.l olrtC'OlnlC'tYk tepr~oll.llloMI ('>(the 1\lll~I\Wl f{;o:) - .~: -"!1.~: + I .
ThC' ..ahoi' o f th~ fu nc tion IO)r;. g.h~n .\' i,; vi11uall y displayed as th~ y c:u,xdiouot t>t' th.at p;>ir,t<)l\
thc> ..vrq) with .o:-.;<.~un,Jin;.t~:- ('(I~,,~ I I<> t}t(. ;:.jv, .o:. T hi.s ;:r,,ph ek<~rl y i.l l u~r.t.ll:~ """'PI=-~'lk~ <.tho:
r....,, ,lio l) tltlll Ill~) 1101bl. 1>or..:ad il) o.Jb\ivu~ from the ~tl:,;_c.lm:tk Udinitiuu j C\) = .\' - 2.1.\ + l.
Fvt i to~(t'lto()l.'.

J. I~QC ll~(t\' \';.li!}C;S (Of J,;> fhl: IM)~C'Sl .... lht Of j (:r) IS {( - ;) .'1" .lhe.:un.:~Jlc:;t va lue- uf f(.d is { (3)

~~j fQ(

POOtti\1 V'olli.IS Q'f

= -53 .

2. 1\$ x iJ!<"f'Cd,$("$. ''tlhcs or f(xJ ; ,,~,~:.lsc rot x


dec~.:t :\"e f<r - .3 ..-: ..f ~ 3 .

l. f( .t) i." t.<o;lt..t...tl to 'l~tll fu~ th r~'<! v.:tluc." cof .J:


th~tro U, ;tod ur.e:. li u le llt.rger tl'l:m S .

'

- :.; aoo x > :;. and vatU('-S ot' f(.Y.')

lite a litt.lc: lc,;,; lhlm

-s. '''"" 11 lillh. bry_cr

( ;r;tf'h lll!': (;;th:ul:ttoM'lt ;ud t:f>mpu t~r" h;o"e N',(:mnl'. i mli .'(~n$;1hle t o tll..; f<w m<,:lt':'nl l:ienti .;t~~
t h i~ i<: fhe Olt"lity of th'1('. eloctro-.n k device-: tO g r.1ph fu nc i c:on~~o ~o:o q\1\ckl y. We
h;J,..;tc n lu po1inf u u l. hnwevc:r. lh:tl nt:o.o.:hi nc.!:'t:rl:;;ril l c(~ )!,ono.ph_..:rwry llllli:oli mo.:~ ho..: u\\Sic.;tding: c.one

o ne TCMr.>n fnr

mus t be t.akc-n not to mt~kc- rash assumptions bt:tscd on muc-hinc output. \Vc i.Jlus-trotc witb some
~),,mpfl".$

hl'!low In nrlflition, 1;11'\r::m.o;l'\ c:ok ui:Uol"ll. <'tnrl c.omr,mtl".rll. h;w to;

1-ittit ~

l'('..fl"en rr.<;r">l n tinn,

OJ ~p<:<.1. (.0( ~ mw.."hi no.: g~:-nct ;.tcd


,~;r:lph. M:uh ~mliiiCn l ;u'laly:>.ts m.rty he requi~d tt) C)llflnn t) r de ny w hat tS heinA ~ ~~p;~Cll.t.e<l hy

i( III<IY n vt alway$ be JNrl'cclly <:kat II(Jw lOJ imo.:t-pt'1:t o.:vc ty

the g;r.;tph. T hi.; i.; \'t:ry f m po)l1m11' We u "e m;~chi ne-gen er.lte.;l gt7~phs. e:<ten$he1)'> l:wl.tt w e :. re
a lways pr<:J)!Lred to ..-:orroborate what we .see. or don' t see. with rigorous n l<tthem atica\ ana\ysi~.
Rii:Co'lil that wh~rl w~ ;'l~l( )'(Ill f(t 11/m:. t,r.lp h , " -.o: into?.uil f<tr yt"'u 10 u<:;t'J :\ f',NIJthi"1. ~~;lkut"'t .:'r (It'
<.:omputc:r. When we ask you to draw a graph. we expect you to d o so without th\:se devK:c$.
W h a l (uJicn~~ ;u ~ J>II /Ut! l!x ;uupl~.. <,tl' c u nlpul t'.-1: t".~n~nc(t'cl t. aph~ llmt

m t!

tui::,.1t'lailiue,.

I EXAMPLE 1 . 19
A \'c r y powerful sot\warc packag e once ,gavctltc g raph f'orthc fu nctio n [(x) =."\:+sin ( ln x)

on chc intenul 0 :5 x 5

~4

in Figure 1.:19. (lt docsn' in newer \'CrSlons or the J>at.\.:.nge..) How

do you reeJ aboul wll<ll you see?

Mijtdil IAWfM

lneuro"CCI p p.!J prndncc:d hy Cllm tptrtcr proeram

.r
20
1.5
10

.5

10

1.5

20

SOLUTION The graph appears to be the sLr.ti ghL line y = x, implying that therefore
sin (2n x) is always zero. n.is is ridiculous; it is a lluke. resulting from the c hoice of sam
pliog po ints t.ake o by the software package in plou ing the graph. Choosing the plot ioterval to
be 0::; x ::; 25 gave Figure 1.40. This is stillotisleadiog. The plot in Figure 1.41 gives a true
indication of the nature o f the functio n. Example.'.i like this are rare, but they c-an occur. Use
your commo n sense.

DEJ!l n =rJ11

Another incorrect
gmph pt:Wuccd b>'

t1 COinpUiet

prugrruu

Conea gruph ptodutcd

by o ..:onputer program

20 '1.)
15
10

5
5

15

10

20

25 X

2Q

15

10

,T

We cncoumc.red absolute values in Sc~t ion 1.3. 1l1c absolute value funclion. denoted by lxl.
is de lined as
-x X< 0,
( 1.29)
lxl {
X'.
X?! 0.

h defines the size or nwgniuule o f its argument without regard for sign. Tlte gr:1>h ofthe function
is composed of two straight lines v.,ith slopes I meeting at right angles at the origin. 1l1i& does
oot appear to be the case in the computcr-gC<leJ"atcd plot of Figure 1.42. Wh)'?
,IOilclll.l!t

Graph u(

rC;r,

lA

2
1.5

0.5

_,

-2

l EXAMPLE

1 .20

2 ,\'

The most up-to-date version of tile sofTware package that produced tile graph in Figure 1.39

yields Figure 1.43a when asl:ed to plot the function f (x) = x I/J on the imenal -8 :;: x :;: 8.
It is also accompanied by waming messages 10 ohe effecothat values of f(x) are not real for
negative values of x . Define input fOr the: computer SQ that a proper graph is genc_nttcd.
l'il tlllfi:l

graph of

fFEW

W)l'ii m

lncmrcct

Corrl

j;J~ I)h o ( .t 1 1~

x1 ' ~

)'

)'

)~~

1.5

0.5

-s
0.5
2

8"

-6 - 4

-!o.s

sx

SOL t.:TrON To fuHy explain what the c;omputc:r is tloing, \Vt: lee\.! complex numbers from
Appendix C . To be brief. every nonzero rcu l number has thi'CC cube. roots. one of which i$
real and rwo of which are complex. TilC computer is prog.rarrm1ed w yield the real cube root

of positive real numbers (8 111 = 2, for ins;~;ance). Unle~~s direacd otherwise, howccr~ the
computer. und maybe your calculator. produces a complex number when asked for the cube
root of a negative real number such as -8. lt d~ not yield - 2. which is also a cube root
or -8. unless specifically wid to do so. We can instrucrthe computer to do thi.s by 'tX.Iefining
j'(x ) = .r t/3 as

j'(x )

= ~ -lxlt/3,
~r' /'J,

x < 0.
.r ~ 0.

A plot of this funct ion is shown in Figure I .43b.


Quite ofte.l the a-gunlCm of the absolute value funclion is a function of x mther than x
itself. Such is the case in rhe following example.

I EXAM PLE

1.21

A computcr-gener:ucd gmph o f the full<tion f (,t) = l.r' +51 is shown ill Figure 1.44. It
x = -2 and .x = - I , but we cannot be
J>O<itive of this gmphically. Ill addition. we ~a n not be sure whether the gmph is rounded or

ltppears l O touch 1hc x-a;(iS somewhere between

whether il COillCS 10 a sharp point at this same value of .x. Perfonu :some elcmenttlry graphing

10 answer 1hcse questions.


M#fU:Iihl4@

t2
tO
8

6
4

-2

-t

SOLLIION To unw~lC curve .v = 1.~ 3 + 51. we begin with )' = x 3 in Figure 1.45a. Next
we draw the cunc y = .r 3 + 5 by adding 5 to every o rdi na1e in Figure I .4Sa. The result j1l
Figure 1.45b is the curve ill Figure 1.45a shifted upward 5 units. It crosses the .~ axis at -s t/3.
The hl.' t ste p is to take the alxolute ' alue of every ordinme on the curve

I1.l!: .J t:Rlt
Buildi11g the graph of ~\'1 +51
)'

y
)' =XJ

-51/3

y = x 3 + 5. This

changes no oolinate that is already I>OSitivc. but cht~ngcs the sig,.1 of any urdinmc dun is negative.
The resuh and fimll tlr"dwing issh0\\11 in Figure 1.45c. Clearly then. the graph couches the x ~axis

at .r

..-.

- 5 1/3. and dtcrc is a sharp point on the graph a1 (

- so. 0).

We ntighl get the impression from Figures 1.42 and 1.45c that absolute values lead to

CUI'\'Cs

wi th sharp poin(s. 1l1is is not always Lhc case. When absohnc values nrc placed around the

function .r3 in Figure IA6a. the .-.:suiting function l.r 3 1 is shown in Figure IA6b. There is no
shrp point t the origin.
li(IUII;I

I }'

ld'
,Y : X\

y. "''"'

0,5

0.5
-0.5
-I

0.5

t X

_,

- 0.5

0.5

l .<

- 0.5

-0.5

-tl

- t

1 I!i!1Ji nt-iAM

liil?illl3

4X

-5
- 10

The absolute value of the fu nction 2.r3 - l l.r 2 + 16.< - 7 in Figure l..l7a leads to the
gntph in Figure I A7b for 12x 3 - ll x 2 + 16x - 71. It has a sharp poin~;ttthe position between
.\ = 3 and x = ~ where it 1ouches ahe .t-axis. but not at :c = I .

I EXAMPLE

1.22

mr

Plot a grnph of the function f( x ) = 2..14="X2 (usin:; tt calcubnor or computer). Docs the
gnoph look familia( / ConHnn what it suggescs.

tW!

.n

The top

ot "'' cllip;e

SOLUTION Thedomain orche fun( tion is - 2 !: x !: ' }sgrapjtin Fi gure 1.4S appcars to bc
the top half of an ellipse. ' lb confirnllhis. we square y
2 4 - x . obtaining y2 = 4(4 - x 2 ) .
2
2
or 4x
_v = 16 . This is indeed the equation of un ellipse. Only !he top half of the ellipse is
defi ned by y = 2J4 - x 2 since the square roo1 requires y to be nonnegative.

.1'

36

Chaplt'!r I

I EXAMPLE

C:d"-'tll~ Prep:lr:UtOII

........

1.23

Repeat Example 1.22 with .f (x ) =

J4 -

x 2.

SOI.U"nON The graph io Figure 1.49a on ce again suggests the top half of ao ellipse. Squaring
y =
leads to x 2 + y 2 = 4 . a circle. not an e llipse. The reason the graph does not
appear to be sem ic ircular is that the computer has chosen diffe re-n t s caJes for the ..t- and y -axe.s.
If we instruct the computer to usc equa l scales. the graph in Figure 1.49b docs indeed look like
(l scnliclrclc.

...14="?

FIGURE 1 . 49a

~micirclc

MiiilfiiiIWI-J:W

1hat does not look lil.:c a stnlicirclc

A ;cmicird c th:ll doc.s look li1:e a semicircle

y
y = ~4- .t'

-2

1.5

1.5

o.s

0.5

-I

I EXAMPLE

y=

-2

i 4- x2

- I

1.24

Plot a grph of the function f(x)

= x + J3x 2 -

4 . .r :=:::

2f.J3. Docs it appear to have a n

asymptote'? Cootfimt this mathematically.


SOLUTION The plot in Figure. I.SOa appe.ars to be fai rly s traight, sugges ting an asymptote.
To confirm this. we note: that for large .t. the 4 i:; insignificant compared to 3x 2 . and therc.fore

.f(.t )

""' X+ ,J3;'i = X+ .f3x

= ( I+ ./3) x

Thus, the line y


( I + v'J)x is a11 asymptote for the graph. lt is shown along wi1h the graph
of the function i.n Figure I .SOb.

lliJiiJII;JHf1fW

Gr.lph lht"lt sugsem an

~y mplote

lil[tJII;IWiftoiM

G~pb

sho\l.'i_ng 01symptC)Tt

81

61

y =x+ ~3.,2- 4

~~'~
2

I EXAMPLE

, .25

When n I>:JII is dropped from the tOJl of a building 20 "' high at time 1 = 0. the distnncc d (in
metre.s} that it falls i1l tin-.e t is giYel by the function tl
J(t) = 4.9051 1 . Draw itS graph.

m!] !IiJ.=1
Di st\.,nt.-e
Ji.tllcn b)' ot bill I druppnl (urn A

t:eig.h1 l)f 20 m

of the pambola d = 4.905t2 in Figure 1.51. The remainder of d>c parabola has no physical
signili camx: in the <.:OiliCXI ol' l hi~ problem.

20

10

SOL UT IO:'\ In this exnmplc inJcpcndent and dependent Vll!"iablcs arc denoted by lcllcr> t and
d. which suggest their physical meaning- ti ntc and di:\tance - rarher than the generic labels
x and y. With the axes labelled correspo11di ngly as the /axis and d -axis, we dra" rhat pan

J4.:S
I

Even and Odd Functions

= x 1 +I in Figure 1.52 is Lhcgr.:tphol"thc fun<.:rion x 2 + I; it issymmerric about


they-axis. TIIC curve in Figure 1. 53 is the graph of the functioll (x - 2x1 + 5)/(x 2 + 6); iris

The parabola y

also symmetric about they-axis. These are examples of a special class of functions identifie<.l
in lhe foJJowing c.Jctlnilion.

DEF INIT I O N 1.4

A fUilCtion f(x) i<said to be an e''en function if for each x in its domain

/ (- x) = /(x);

<UDal

it is said to be an odd fun<lion if for each x in irs domain


/( -.r)

= - f(x ).

( 1.301>)

The fiN of these implies that the equation y = .{(x ) for the graph of an even function is
unchanged when x is replaced by -x: therefore. the graph of an even function is symmetric
aboul the yax is. As a result. f(x) x 2 +I and f(x)
(x' - 2x 2 + 5)/(x 2 + 6) are even

function ~.

F.quo1ion 1.30b implies that if (x . y) is any point <Jn the gmph or an odd function. so 1<10 is
the point (-.r, -y) . TI>is is illustratcdbythcgraph of thc <>ddfunction f (x ) = 2xS -3x)+ x

M:IC'flii;IAMlij

An

t \ tm

M:IICJII;IiiU 1?1

futk:tion

jY

y= ..r:! + J

3.5 '
3

2.5

2'
1.5

-2

-I

An (\~n functioo

-2

-1

"

38

CMpcer 1 C:.k'ulus Prep:lr<lttU)

in Figure 1.54. Another WD)' 10 describe the graph of an odd function is to note that either half
(x < 0 or x > 0) is the result of two re.flcctioos of the other half, first in they-axis and then
in the x-axis. Alternative ly, either half of the fllpb is a result of rotating the other half by 1'C
radians (one-half a re-v olution) a round the orig in.

-0.5
-0.75

J EXAMPLE

1.26
Which of Lhe following fLtnctions are even, odd~ or neither even nor odd'?

M)lcUI;JMMOifF M Half the


Sf'..lpb of Ill) evel'l ftu.1CiiOO

(a) f(x )

(b) f(x)

= x5 - x

(c}f(x)=x 2 +x

Draw a graph of each function.

SOLL'TIOl\
(a} Since

j(- x) = Jl - xl = _,lj;j =

/(.r),

thjs function is c,en. Jts graph, the curve y = ,IJXT, is symmetric abouc the yaxis.
x > 0, this equation becomes y = .Ji, the half'- p,trabola in Figure 1.SSa.

When

The complete gr-dph or the function is s hown in Figure 1.55b.


(b) Since
M::llflll tjnffi!W

gA)pb from

'fhe full

f(-x) = (-x ) 5

S)'OlJ)lC\t)'

(-x) = -x 5 + x = -(x 5

this function is odd. Its

gr~ph

x) = -f(x) .

is shown in Figure 1.56. The right hal f of the graph

in Figure 1.56 was drawn by analyzing signs of f (x) in the factored form f(x ) =

x (x2 + 1)(x - I )(x

+ I). The 1efl half is the result of a half revolution of the right

half abou t the origin_

(c) Tile function / (.<) = x 2 + x is n either even nor odd. Its graph is a parabola that
opens upward, c rossing the x -axis at x = 0 and .t = - I (Figure 1.57).
MOiiUJII.O'l&lfJ

An u..td fuuctiuu .t ; - .\

FIGURE 1.57

A function that is neither even nor odd


)'

\ Ve h.w~ seen C\tn functtonti, otld funcuon'-. and funct10M that are ne1ther even nor Odd Can
functions be both even and odd? Such a function would ha'e to s.>~isfy both 1.30a and 1.30b.
and hence

f(x)

= - j(x ).

B tllthi Implies th,u /(t) = 0, and this thcref<>re is the Ollly even~ ood fUJICtion.
It is oflen useful to divide u function into w hat are called it even a nd odd pan.~. For example.
if f (x) i defined lhr all real x. we may write that
.f(x )

= [

f(x )

+/ (-:1')] + [ .f(x) ~

f (- .r) ] .

( 1.31)

llis stntightforw,ud tosl>ow th<ll l.f(x) +.f( - x)]/2 is tlnc,e n lu nc tion.und 1/(x)- f (-.t}l/2
i< an odd fUJICUOn. In other worlls. we have ' "iucn j(x) AA the um o l a n even 1\uiCti on ruld
>1n odd fun.:tion; th~) ore called the CVCJI ld odd p rt.< of /(t). Jrwc dcntMc them by

= f (x) +2 /hr)

j,(.t)

a nd

/(.r ) = f ,(x)

I EXAMPLE

1. 27

_ / (.t) - /( -x)

/.o ( X )

(1.32a

+ /o(l') .

.,....
Write the fu nction' (u)

f (x )

= x3 -

2x 1

+ x + 5 and (b) x(.v) =

even and ()tid pMIS.


SOLU riO' Sol ution
(a) Cle.trl), thcevcJHUJdodd pans of /(.t ) are J,(.r)
rcSJlC'Ctively. Thus.
/(A )

+ I + -.x +
= -2I [X
-x - 2 -x -

I]

= .tx

1
2

... - 2

in tcnn o f their

= -2.f1 +Sand / 0 (.t ) = .r ' +x.

= (-2x1 + S) + (x 1

t ).

~re

(bJ The even and odd parts of 1/(X)

g,(x)

.r +

+ 2
-

.
4

flo (.X)

=-I [X+
- -I - -.r- 2 =
2 .r - 2
-x +I]

3x

x2 -

Titus.

x2 + 2
[((X) = --:-x2 - 4

3x

+x 2--~

Polynomials and Rational Functions


1\"o of the nlOSI un1>ortall( classes of fu ncliOM arc polynomials ond ra tional functions. A
pol) nomialol degree 11 i a functton of lhe fonn

f (A ) = o.x" - o.-tx- + .. + a2.x, + o,.r ..-. aa.

<1.33

where n is a nonnegative i nteger. and ao. a 1 , 0 11 arc real numbers (a" :F 0). They are
dell ned for all "llucs of x. Graphs of linear and quadra tic ll<>l)'nomial> are stright lines and
par-.Jbolas. The c ubic ll<>lynomial .x 3 + 12x 2 - 40x+6andthequanic x 4 10x 3 + 6x 2 - 64x+5
are ploued in Figuros 1.58 and 1.59. Figure 1.58 has a low point j ust tO the right of x = J and
a high point jujo;t to the left of x = -9. We could approxi1nutc these J>Oints more and more
closely by 'educing the plot interval. C alcul us provides an analytic way to determi ne them.

.(

40

Chapcer J Calculus Ptep.1r:>.tion

l@t!JII;t=t '* !i

A cubic polynomial

FIGURE 1 . 59

A quarric polynomial

)'

600
400
200

-5

- 10

-200
-400

A r-ationalrunction R(x) is defined as the quotient of two polynomials P(.r) and Q(.r):

R(x ) -

P(x)
Q(x)

Rational functions are undefined m points where Q(x)


j(x) =

x3

+ 3x 2 .f 2 -

= 0. For example, the rmional funcJion


12x

+J

2.f - 3

is undefinedat x =-I and x = 3. Thecomputer-generatedgraph onthe irucrval -2:;; x:;; 4


in Figure 1.60a indicates I hat functi on values become very large posiJively and negati,ely near
x - I and x 3. We investigate this behaviour in detail in Chaplcr 2. The graph may or
may not be accompanied by messages indicati ng that division by 0 is encoulltered at x = -l
and x = 3, depending on the plot interval specified to the plotting device. A plOt on ihe larger
interval - I 0 :;; x :;; 10 in Figure I.60b suggeslS that the graph has an asymp1o1e. We shall see
why in Chapter 2.

itCJII;JjtlliJlH

Unbounded bchll\'iour Qf

il

rational function

-2

Saau,:c!loted n~yrnpiOlc of a

f'Jtiooal function
y

tOO

40

50

20

- 10

-t

.f

-5

- 50

L
5

10

- 20

- tOO

E'"Cf)' function j(x) can be represented

pictoriaJJ~ by

i<s gr.1ph. the curve with equation

y :=: f(.t} . But whut about the reverse situation? Docs every <.."Urvc in the ,\'y wplnne teprcscnl
n funion j'(x)? The curves in Figure 1.61, which both extend between x =a and .r =b.
illusl rate that the answer is no. The curve i n Figure J.61a represenLS a function. whereas !he

curve in Figure 1.6lb does no1. because for values of X betweeo a and c the re a re two possible
values of y . In other words, a curve represents afimctiouf(x) ifevery vertict~ltine that intersects
the curve does so ill e.mcrly one poi1U.

1.5

ljlclll

FIGURE t.Gt
o(

Bm

F11ndi<tn( ood Their Gr..p~

41

O.uve du1 is oot the

a rurK.1 ion

Tran sl~ ted

cir.:lts
y

x2+y2 = r2
r

Jl<- c)'+ y/2 = r 2

.<

Translation of Curves
Jn Scc1ion 1.4 we saw how graphs of c onic soc1ions can be shined in 1he xy-plane. and how
1hese shifts, or lranslalions as !hey are called, are reflec1ed in lhe equmions o f1he curves. This
principle applies to aU cur\'CS. nOt just conic sections. When every .t in the equation of u cur'-c
is replaced by x - c. where c is a conStam. 1he c urve is shifled c u nils 10 lhe right When every
x is replo~d by x + c, lhe curve is shined c unilS 10 lhe left For exornple, when x in lhe
cqualion xl y 2 ~ r 1 is replaced by .r - c . lhc a:ntrc oflhc resulting circle is shiflcd from 1hc
origin 10 1he pOim (c. 0) (Figure 1.62). When each x in 1he parabola y = x 2 + x is replaced

y = {r + d + (x + c)
= (X+c)(x + c+ I)

1y

.Y

\ )

-< - I - - -<

=.t 2 +X

byx + c.

=x(x+ I I

= (x + c)2 + (x +c) = (x + c)(x + c +

1},

c units 10 1he lcf1(Figure 1.63).


Vertical shi fls resuh when y is repi:'K:ed by y c in !he equa1ion of a curve. For ins1ance,

I he parabola is shifted

-1

--c--

the curve .t 1

(y
If "' I is lhe hyperbola x 2 - )'1
I unit (Figure L64b).

ljCcliJ.IWM.(I 1

The hypctWlot

.t:-) ! -

= I in Figure 1.6!la shilied downward

141111.14

y
. y

=-x- I

\{ _

y = A- 1

)j

H . -I)

I EXAMPLE

( 1. - 1)

1.28

Describe lhc rclalionship between lhc curves


0 is o cons1an1. Dr-aw bo(h c urves.

lxl

+ lyl =

I and

lxl - l.r - al = I. where

a>

SOI.l'TTO'I Since lxl + y - al = I ca.n be obwined from I.<I + IYI = 1 by replacing y by


y - a, Ihe firs1 cuf\e is the second shifted a units upward. The cur-.-c 1-<1+ lr l = I is easily
drdwn without an clccuonic device. Since 1his equation remains unchanged when x is rcplacctl
by - x and y is replaced by - y, 1he cune l.rl + lyl ~ I is symmc1ric about bolh lhc .r-axis
and they-axis. This means thm we can concenuate our efforts on drawing the graph in the first
quadrant. where the equatioo reduces 10 x + y = 1. The segment of thjs straight I ine in the
(irst quadrant is shown in Figure L65a. To ob1ain lxl + IYI = 1 (Figure L65b). we renect
lhis curve in 1he axes. Finally, lx I + IY - a I = I may be oblained by shifling lx I + l_y I = I
upwrud a uni1s (Figure I .65c).

M:Jitiil;tWM!

l1l+ lyl

One.qnana of

1- l + lyl =

MjlrliiIMM$i'M

=I

J.rl+b-a =

by uanslatioo

(1\)rn ))'ftlnttttry

.Y

a+1 > lxl+ly-ol=l

~y = l
(-1, a)

.<

<

(1, a)

a- 1

EXERCISES 1 .5

In
I.

Ex;cn;i~s

f(x)

3. f(x)

* 5.

/ (x)

7. f (x)

I
2. f(x) = - -

= Jq- x '
=
=

25. Vcrif)' th!ll when un odd fu.nction j(x) ls defined al :c


\"<IUC IIIUSI be f (0) = 0.

l -8 find the lu.rgcsl possible domain of lhc function.

,'(- 2

4. /(x)

xJxl+ -l
I

6. f(x)

xJ4 - x'

= .:1- 9 + 6.< -

x'

* 8.

(a) n,c producJ. of two C\C.n functions or l\\>0 odd funclions is


an er.en funetioo.

~x

(b) Tile J.ll'Otlue1 or ~n C\'CI1 41111.1 an OI.W function i::. &l oWJ func
1io.n.

9-

f(x) = Jx 3

x'

In E~cn:ius 9- 18 de-termine algebraically whctherthc function is even,


odd. or neither even nor odd. Plot each function 10 confinn yow con
elusion geometrically.

9. / (.<)

= 1 + x'

II . f(x )

IJ. f(x)

12.r2

+ 2.c

2.r

x- l

= -x +l

xl-<1

15. j (x ) = - -

3+ .<'

17. f(x)

x1 + 1
1- 2x'

= x(x + x)

A'+ J?+l

. + 3.<'

x -2
= - --

x 1 + 3x1 - 2x

20. j(.r)

lxl

22. f(x) = - -

2x
23. f(x) = - 3 + Sx

24. j'(x) =

= J5 4x

= -J5- x 1

30. f(x)=~

35. /(-<) =

--FX

37. j(x} -

x'''

38. / (x) - 3x 111

3 2. J(x)

33. /(.r) = l.r I + 2x

16. f(x) =x'+fi

f(x) =

28. f(x)

= - J5 4x'
34. f(x) = lxl- 2x
Jo. f (.<) = x'''

31. f(x)

18.

= J;- x'

29. /(x)= - ~

* 39.

[(.r} = 3l.rl'1'

40. f(x)

= x ..fX7T

41. f(x} = x~

42. f (x) = - xJ4 - 9x 1

* 43.

[(.()

44. f(x)

45.

f(x} = x J4 - llx '

= .t3 j 4 - x

= x1 JAi- 4

46. f(x) = lx' - x - 121

x+~

,,

21. f(x) =

27. f(x)

ll. f(x) = x 'l ;

In Excrcisos 19-24 ftnd the even and odd JXU1.S or 1bl: fwx<ion. if' lhey
cxbL
19. j(x)

In Exercises 27- 56 first dr.lw, then plot,:.\ gr.lph of the function*

10. /(x) = x' -x

14. /(x)

0. i&S

26. Pnwc coc:h .ofthdollowing:

-~

=\

47. /(.<) = ,/2x

.r 1

.r 2 +3

49. f(x)

= J 4x'- 2x

..;x::l

Sl. j(x)

=X+ 2 + ,/X

= J2.c - 4x'
f(x) = J1x -x'- 4

-18. /(x)

50.

1.5

52. /(.<) =

53.

J9 -

4.< 2 +

., 73.

x2 -

(y

+ 5) 1 =

7 4.

+ l)' - (x + 1)2

75 . y - (x

= J(x' - 4)
f(x ) = J2 - v'f+x

* 77 . .v = 13- l.r + 211- I 78.


* 79. X = 4 - 1,\'1
* 80. (.r - I)' = (X - 1) 211 - (x - 1)2]
* 81. An clcctrotic signal is de lined by

S6. j (x)

= .j(x'

(x2 - I)

I)'

57. What condition cnsun::s tlmL a <.:ur"'C in the xy-p lant: rcpn:~c;.nL:s a
function x = f (y) ?

s(t) =

A curve is tran.-:ohuc:d vcrtiC!lJly \\bc.n coch y in i1s cq..ution is replaced


by y + c: it is shined horizo-ntally when each x is replat'ed by .r + t.
If each x in lhc cquntion or a c-urve is replaced by ex. where c > 1 is

x-dinx:tio.n. When c:.ch y is replaced by cy. the cur\'c is compressed


or Sln:lchcd in the )' -diret.:tion. Olu..'>trC~h: this by dr.:1wing ~nd plouing
the p:tir of cune s in Excn::.iscs 58-63 .

6 1. ..\'l - y l

+ 1.11 =I . lxl + l.v/21=I

63. x 2

> 3,

(a) Find un <!l:ebr.Jic 1\:l)rt.::St:lllatio n fol' S(J).


(b) Draw gJaph" of the timeshil'ted signals S(l
s(t - 3).
(c) What arc alg<:bmic reprcsc.ruo.tions

1/2) and

f()l' l.h.c grapbs i.tl past

(b)?

(y + 5) 2 =I, x'- ()'/2+5)2 = l

\\'hen each .r is replaced by

-x

i n the Cl)llation n f a cun 'i!. the curve

is rdlocr:cd in lhe )'AXis. When ea.eb )' is replaced by - y , the curve is


t-enected in the xaxis. IIIU!5tnu.e this by drawing and plOtting the pair
or curves in Exercises 64--67.

= x 3 - 3x 2 ,

65. (x - l f

)'

= -xl - 3x 2

+ y 2 =4.

(x

+ 2) 2 + .r 2 =4

- I
<~-

RJ. A cl"tet'r)' 01'<:hard ha~ 255 ttcc.<; . each of which produces an ih'C1agc:.
of 2.S baskets or cherri e~ . For c:l<:h nd dit i on~~ ll rcc: pl:tnJed, the yield per
lrtc dc(..Tcascs by ooclwcUt h of a basket. Jf x represents t.hc nu111bct
of ex<cd trees (beyond 2jjJ arnJ Y the tocal yield. 011d Y as a fu nction
of x, and draw riLii gruph. How 1nany more trees should be planted for
lt\a..'t_iotum yidd?

66. x = y 2 - 2y . J. = y 2 +2y
67. X

~ f $3.
1

62. lxl

64. y

[ (3 - 1)/2.

< 0,

The graph n f an electronic s.ignal s(t) is shown in the figure ))elow,


where 1 is LinlC ..

"2 + 16y2 = 1

1, l ' l - 4,\'2

0::5 1 ::5 1.

* S2.

59. y = x , Sy = x
60. xz + y2 = 1,

/,

where t i$ Lime. Ora\\' graphs of the tlme-shil'tcd signals s(/ - 2) and


S(l + J) .

= x' . .v = .x1 /9 = (.x /3) 2

58. y

lxl + IJ + 21lxl - I.YI = I

0,

o.

a constanl, the CW"\'C is compressed b}' a racta.r of c in the x -direction.


When 0 < c < I . the c .ut'\'C is SU'<:tcbcU by a (aciOI' of 1/c in tJ'c

76.

43

x = - J M + 9(y + 5)'

/(x)= x'+lxl-2

" 54. /(.<)


55.

Jx' -

Rnnctions !lll " 'l'helrGr~pbS

* 84.

= ..jY, X = N

A rectangle w ith sides pamllel to the axes i~ inscribed inside the

ellipse bzx 2 + a 1 y 2 = a 2 b 2 (figure below). Find a formula for the


area A of chc roctanglc in terms of x . Draw a graph of Lhis func:tion.

6S. The floor ftmction is deli ned by f(x)


that docs not exceed x .

= [x J = gremeSl integer

(a) Drow a gntph of lh<> floor function.


(b) Lf firs:tclass poswge 1s 5 1 cents for e:a<:h 50 g. or fraclion
lhcroof. up (0 and iucl\ldiog 500 ~draw a gmpb of this COSt
funclion.
(c) Express lhe cost funcljon in part fb} in tcnns of lhc floor
fum;tion.
In E..'<.ercises 69-80 draw lhe curve. Indicate whether the curve defi nes
y

as a (unctiOn or X .

*- 69. y =

x 2 - x,

71. (X - 2) 2 + y 2 = 4

*
*

70. J = (X

72. y=

+ I) 2 - 2(x + I)

j4-(x - 2)'

* 85.

A man 2 m taU walks along the edge of a slraighl ro~d thal is 5 m


wide. On thc olhcrcdgeoftJtc roadstandsastrccl lighl iOm ltigh. Find
a fum.'tional relations hip for the length of the man's shadow in lcrrns of
his distan<.'C from Otc point on his side of Otc road din::ctJy across from
the light Plol a gmph of this f\mclion.

When two substances. A and 8. arc brou~ht t('licthcr. a chemical


reaction 1akes place to form a new ~ub~a~~. C. It requires 2 L or A
for each li1re ()( B 10 produce 3 L of C . n\C~ rate H at which A :HY:I B
reoct H> fonn C is proporti on~l rothe product of Lhe amounts of A o.nd
8 present nt thm insto.t1L If the origin"l "mounts of A unci 8
20 L
and 40 L.. respectively. and if X represents the unu.xmt
pre sen( in
tbc reaction at any gh,'CO time. find a romlula for R as u function of x.

"* 86.

or c

,. 90. A l>ox mea~uring I m on each side is anched to a r(lf)C w; shown in


1he hgure beiO\v. T ile rope passe..; over a pulley I 0 m fmm 1he ground
and a truck l)ulls on the other end in a horizontaJ din."Ciion al on~ !I line
I rn nbovc the gn>und. W.-: denote positions o flhe Lruek and lhe t>onnm
of th<:: boJO by.\' and y, r<::spcctivcly. Fi1.td )'as u function of x if lb-c
U\lck .sturt.s uc !)Osition .\' = 5 1n and slops wbcn 1bc top of the box

'"'C

tou~hcl)

abc pulley. Assume that the lcngll1 of rope between truck and
box i..\ 2.5 Iii. Draw a graph of the function.

Dnt"'' a gmph of this function. and dctcnninc when the-reaction tdlC is


highesl.

* 87.

In c l<IS'i<"al ph).sics and engineerint. the 1na.'\S m of an object is


constanL independem or how f:1s:1 i1 ~ moving. In ~:pecial rela1ivi1y.
h.owe\-er. m is g,i\en b) the formula

.L
0.!.
10 y I

mo
m = --;=~~""';c

,/1 - (v' {c')

-:-!~

wbcrc mo is the ma..<softhcobj<Xt when it i.s not moving. vis the speed
of the rnass. and c is a con&alll (the speed of light). Draw a graph of
this fu nction. and dr.lw any conclusions thai you feel are suggested.

~L---- X ------+
~ --~------~~~

* 88.

A square plate 4 m on each side is slowly submerged in a large


tank of waler. One diagonal is kept venica l and lowered at a rate
of 0.5 n:J's. entering the water ru time t = 0. If A is tbc area of the

submerged portion of the sutf'acc (one side only) at time I wuil complete
submi:rsion occurs. J1nd A as a functit)ll of 1 and dmw its graph.

* 89.

&.'Cause of torbtruclion.no I'J<:I&singis pcm&ittcd on a I0-km.strctt:h


or highway. If cars tm\'el at 1; kmfh along this stretch. a sale distant-e
be1ween them must be maintained. and this distance increases as v
incre~s.

In particular, the highway traffic commission has dctcnnincd

lll<:H fo rspc:cdsO\'Ct 50 km/h., thcdistatlCC iO lllCIICS 1Jc(WCCJ1 catS ShOUld

be at least

3v 1
d = -

In Exercises 91-94 drawn groph of the fu1tction when; l.t J is the floor
fun..:.tion of Excroisc 68.
91. /(.x)

= [ 2.\'j

92. f(x)

= .x + LxJ

93. f(x)

=.rLxJ

?4. f(.r)

= [x + [xJJ

95. b l/(x) + g(.r)j

= [ f(x) j + [g(x)J ?

96. U :t thc ro:~:tll nudcar rcnctor is built in the sh:t.pe of a right circular
cylinder of radi.\IS r and bcight h, tbcn nculrondilT\tsion tbcory requires
r and h to sat.i.sfy an equation of the form

500

Ir itis s:uppo$ed lh:.u ~'t:ryone m:lintains the sate dinance :lnd the same
constunt speed v lhrough the stretch~ find the number q of co.rs leo.,.ing
the "bcttk:mxk"' per hour as a fum.:Lion of speed v. Draw a grn r~h
or this function for $0
v
100. and ~ctenni nc the SJ)(Cd that

wtlerc

1nax i mj?.e~ q .

o and b arc posili,e constants.

Draw a grat>h or lhe runcliOfl

= f(IJ) dc:rined by this equmion ror h;:: 2b.

ANSWERS TO DIAGNOSTIC TEST FOR SECTION 1 . 5

II. T he quoticnl of two oolynClmiall\ ( I matk)

I . - 2 .S X .S 2 ( I mark)
2. All reals excep1 x

./5 (l mriJ

J, All rcol S - 4 (2 marl<>)

12. (2 mrui<s)

4 . All reals ;:: 3 (2 maries)

5. E'c:n (I mark)
6. Odd ( I mark)
7. Ncid>er (I mark)
11. Neither (I mark)
9. -x'{ (I - x '). x /( 1 - x ' ) (3marks)
10. No ( I mark)

_g

1.6 ln,trse Ft.tncti03$

13. (2 owh)

45

17. (3 n10Ji<s)

_r---...._
-2

-3

'

14. (2 rnarks)

18.
)'

(2 " ""'''

H<ol r parabola

19. (2 marks)

15. (3 marts)

(,.r"~

>

'

(-3. I)

-2

16. (3 marts)

.v

'

20. (2 Ularl<s)
)'

(-Ill,\
2

'

1, 1)

'

1.6 Inverse Functions


Thi~ material is likely to be new for many students. h does llOitl\ei'Cfore have a diag.nosaic t C.')t
~:~ssodatcd with it.

If you have previously s tudied inverse functions. it 'vould s till be advisable to

at leasl read the section to ensure th;Hyou me fanliliar with the ' 'ocabulary. notation. and theory.
When we speak of a function y = f (.<), there is a unique y associated with each x: thai
is. g iven a value o f ,, in the domain of f(x}. the function associates o ne - and o nly one \'aluc of)' in the range. However. i1 may h~11ppcn. and qu ite often docs. thai 1.1 \"'i11ue of y in
the range of t he func tion may be associated w ith more than one value of x. For example., each
y > 0 in the range of the function y = .f(x) = x 2 (Figure 1.66) is associated with two values
of X, n;llllefy, X = .jJ.
Some func tion.~ have the propcny thtll each value of y in the r.mge arises from only one X
in 1he don1ain. For insc.ance. given any value y in the range of the function y = f (x) = x 3
in Figure 1.67. there is a unjque x such that y = x 3 namely. x = y 113 . Such a function
is said to be one-to-one. Formally. we say that a funct ion .f(x) is one-to-one iJ for any two
distinct values x 1 and Xz in the domain off (x), it follows that f(x 1) : f(x 2) . To repeat,
a one-tcrone function f(x) has the propeny that given any .Yin its range, there is one- and

46

Cl101ptcr I Q;kul\11 Prcp::u'3t"->n

only one - x in its domain for which y = f(x). We can therefore define a function that maps
values io the range of .f(x) onto values in the domain, n. function that maps y onto x if xis
mapped by /(x ) omo y. We cnllthis function the inverse fmtctioo of f(x} . il reverses the
action of /(x). Fort.he function f(x) x 3 in Figure 1.67. the inverse function is the function
that takes cube roots of real numbers.

lijtijiiIAiiJfl,

A function that i~ nl)t ooMo-one

I:UfliiIWMQ

A function !.h:at is one-~~

X=-{)-

y=f(x ) =xl

Unless there is a good reason to do otherwise, it is our custom to denote the indcpendem
variable of a function by the leuer .r and the dependent '"ariable by y. For the imerse function
of a function y = f(x ) , there is a natural tendency to denote the independent variable by y and
the dependent variable by x, , ;nee the inverse function maps the range of f(x) omo iL~ domain.
\Vhen discussing general properties of imcrsc functions, as we do in this section. it is usually
better to maintain our usual practice of using x as independent variable and y as dependem
,ariable e,en for i11Yerse functions. This may 110t be advisable in appl_ications when letters for
independent and del>eltdent variables represenQphysie<tl or geometric quantities.
When a functionj(x) has an inverse func tion, we adopt the notation/- ' (x) to represent
the inverse function. For example, when f (x) = ' J , the inverse function is 1- t(x) = x 'n
Be careful with this notation. Do not interpret the "-I'' as a power and write f - 1 (x) as
1/ I (x) . This is not correct. n tc nollltion ~- rcprc:;cnts. function, just as tall represents the
tangent function and .j represents the positi\'C square root function.
The inverse function f - ' (x) of a function / (X) ''undoes" what f(x) "does": it reverses
the effect of f(x). For example, the function j (x) x 2, x ::: 0 in Figure 1.68 is one-to one:
it squares nonnegative numbers. Its invel'$e is f - t (x) = ./X. the positive . quare 1'001 function
(Figure 1.69). Squaring u positive number x and then taking the positive square root of the
result re~urns the original x . Similarly, the inverse of g(x) = x 2 , x ~ 0 (Figure 1.70), is the
ncgati,esquareroot function g-t(.r) = -,JX (Figure 1.7 1).

MiflfJII;I"''iHJ
)'

)'

)'=[(x)=x2

Tbe U1\~ func tion .ji

MijlijiiJM

The function x~ . :r :S 0

The fact that 1- 1(x) undoes


the domain of l(x).

f (x) can be sUited algebraically us follows.


rl(f<x>)

X.

For each x in
(I 35)

This is the defining relation for an inverse function. We understand that the domain of /- 1(x)
is the same as the mngc of I (x). so that 1- 1(x ) operates on all outputs of I (x) .

I EXAMPLE

1.29

+ 1)/(x - 2P
To find the algcbmie definition of 1-' (x) . we solve)' = (x +

What is the inverse function of f(x) = (x

SOLL'TTOK
x in tenns of y (you will see why in a moment). First we cross-multiply.
x -

I )/(x - 2) for

y(x - 2);

then group tcm1s in x ,


x(y - 1)

and finally. di,idc by y - I.

'-

2)'

2y

+ I;

y- 1

What ha,e we accomplished by solving for x in tenns of y? If we take an x in the domain or


l(x). then f (x) produces y = (x + I ) /(.r - 2). Now this some x and )' satisfy the equation
x (2y + 1)/(y - I) because this equation is simply a rearrangement of y = (x + 1)/(x-2).
Con.<ec1uemly. if we substitute y into the right side or the equation,

x=

2y +

y- t

we obtain the original x. This equation must therefore define the invc.rse function of f(x ). In
other words. if we den()(C che independenc variable h)' .~. the inverse ru.u.:cion is:

Example 1.29 has illustnlled that to find the inverse of a function y = l <x ). the equation should
be solved for x in rerms of y. and then variables should be renamed. I f rhe equarion does not
have a unique solution for x in terms of y, then l(x) does not ha,e an inverse function. Such
is the case for the function g (x ) = x 2 i.n Figure 1.66. Solving>' = g(x) = x 1 for x gives
two solu1 ions x = .JY.
So far our discussion of inverse functions has been algebr.tic. The geomeu-y or inverse

functions is most re1ealing. You may have noticed a relationship between the graphs of f(x)

Mlta!liiiJili I

and / - 1(x) in Figures 1.68 and 1.69, and of g(x) and g - 1(x) in Figures 1.70 and 1.7 1. Tile
(jl'llJ}h M

.t 1 a11d its mir in'li.lge in ~

"

''

/// ~

.\

MirTur imlib'C of
y = .~: 2 in y : x

.r-

J(r 1(x l) "'x.

14C..Iil.liilll[l!il]

All inc~reusilS

function

(1..16)

Gcomclrically, a funccion /(x) has an irnverse i fche rellc<:cion of ils grnph y = /(x ) in
1he line y = x represtniS a func1ion. Buc we know 1ha1 a curve represtnls a funclion i f every
vertical line [hat intersects i[ does so at exactly one point, Funhcnnore, a venietlll ine intersects
1hc rcHcc!cd curve a1 cxaclly one pnilll only i.. it< hori?Oillal rcftec1i0n inccr.<;ects y = f (.r) a1

c:<actly one point. These two facts Cllnblc us 10 state that a funcrion f (.'\') N1s an inverse function
if - and only if - every horizo11ta lline that inter.secL'\ it does so at e.'<~lCIIy o ne point. This is
a geometric interpretatiOfl of a function being oncptoune. Sec,. for C.'<amplc. the functiQns in

y
lnneasin~

runcticn

'

E1!J Il

inverse function is lhc mirror image of the fun-ction in the line y = x; that is. graphs of i1wcrsc
pairs nrc symmetric about the line y = .x. These two examples are not mere coincic.Jcncc.
graplt." 01' inverse. functiotL... are always mirror im~es of e.ach other in the line y = X,
This suggests that a very simple way to graphically detemline d u: inverse Qf a function
f (x) is to take its mirror image in the line)' = x . Note. too! that if lhe mirror image docs not
represent a function. then no inven;e func tion for .f(x) e.'tisls. For e-xample. the mirror image of
y =.r 2, -00 <.l <: oo.in theliney =X i~shownin Figure 1.72anddoesnotrcpresenta
funccion. We conclude as before 1hu11his funclion docs 1>01have un inverse.
If y "' f - 1(x) is che mirror image of y = f (x) in che line y "' ,v, !hen y = f(x ) i<~hc
mirror image of y "' f - 1(x ). T his means lh<ll /(x) is 1he inverse of f - 1(x ), and 1ha1 f(x)
undoes whal
(x) does:

A d ecre:~si ng

fUnction

Figures 1.68 and 1.70 . Horitonlallines 1ha1 inlersecr rhe curves do so a1 exactly one point The
function in Figure 1.66, which has no inverse. is intersected in two poinL'\ by every horizontal
line y = C > 0.
\Vhen the graph of a fun-ction always moves upward and to the right on an interval I (Figure
1.73), the function is said to be increasing on the interval. V\'eshall discuss itl<.Teasing functions
in Scction 4.2. What is clear is that such a function passes the horizonial line test (and is therefore
onc-roonc). Likewise, a fiuiCiion whose gn1pm moves downward and co 1hc rigllc on an incerval
T (Figure 1.74) is said ro be decreasing on l; ic is one-ro-one. A funclion cha1 is eilher increasing
on al''l interval I or decreasing on I is ~1id co be a strictly monotonic function 011 / . What we
have shown is 1hc following re~;ull.

Oecrcm;ing
function

TH EORE M 1 .8

A function that is srrialy monotonieonml interval has an in\'crsc function on that irHcrvtll .

<

It is important to realize that bei ng Sltictly monotonic is a sullicicnt condition for cxi:;Lc.ncc
of an inverse function: lhat is. if a funclion is strictly monotonic. Ihen it has an inverse. It is not.
however. 11 necessary condition. 1ltc function in Figure 1.75a is not strictly lllOnotonic on the
interval -I =:::: x ::: I, but it does have the inverse function in Figure 1.15b.

/
/

.:, /

~ 1

.:, /

--~1
/

I EXAMP L E

1. 3 0

= .r1 -

2x ba,e an in~? Does it ha\t ' ithcr< when ib domain


is rcstritlcc.J to the intef\al r ? 1. and to the intcn-al \ ~ 1?

Docs the funcli t / (X)

SOLUTIO' The gn\ph of f(.r } in Fi&urc 1.76 indic:a~es thul f (X) de,.,;: nut have an in,eO<e.
\Vhen resuic-tc:cJ to the: intcr,al .\ 2:. I, the functior1 is onc.-to-onc. a.nd duc:tt ha.Yc an inverse. To
find il. we SOI\'C y = x l - 2x for .r in terms of." TI'IC quudofltic formuln applied to

x2 -

.r

2x - )' = 0

J4 + 4 )'
2

VY
'V+'i'.
T I

Since x mu<t be grcmer thlln or equal to I. we must choose .r = I


in"eroc of f(,t ) = .t 2 - 2.t r > I is

r'(.r) =

+ ./Y+T.

Hence. the

JX="i'.

I -

SimiiMI). when rc<tricted tO the imerval .r :! I. {(.r) has the inverse


/ -t{.r)

../.t -

I -

I.

This e~amplc illu>t<'IIICS that when a function is not str.ctly 1110tl0tonic on an ioncl'\al / , the
interval can usuully be subdivided into subintervals on which the function is sniccly monotonic,
anti on each ~uc h ~ubinccrval the f unction has an inverse.

EXERCISES 1 8

In E~ercu~s 1-14 tletenntr.c gr:aptucalt) "hcthc..Yil'k: functtOn hJs :an


in,coc. Find ~h inver~ function.

1. /(t) "' 2.\

.1. /() =

+'

+x

7. /(>)=~.

/(X)

= 2.1 +

4. /(.r)

= Ji+T
=

S
2x + ~
X

6. /(x)=Jx'+2

5. /(X)= 1/X

s.

2. /(.r)

'"I

9.

f(:r)

=x +lxl

10. j'(.T) = ~
II. f(.r) = x + 2.1.'
12. j(x)
13.

=r

+ 2. .t ~ 0
2.r + 4, x > I

r<x> =G~~)'
X

14. /(X)= -. -

-' +x-

ln Exca:i-tc.s 15- 20 show that the fune;tion <kx:s not ha"x: :m inverse.
Subc.tivide its donwin or d efinition into Sttb inlef\ Uis. o n which the rune
tiOt'l hns at'l ln, crsc, and find dlC in, crs.c funct iOJl Ot'l each subinterval.
~ IS. /(x) = .r'

17. /(.<) =
19. j (x )

16. j (.t )

+2

_,,4

= "1
-'+'-4

20. f (x ) = - 2x +4

_. 21 . Give: J n cxmnple of a lunction th:u is defined I'm all x. btll docs


nOI Iuwc tan i!wcrsc ()t\ any intctva1 whatrocvcr.

n . Give an eurnrlle n (

u f unction

i nverse on Lhe intcr.'a l 0 ~

thnl is dcfin(}d ror all

X.

h&

r= -

24. Show th:H the demund ftmction

ttn

.\' ;5 l. but docs not have on in\'Cf'SC on a.ny


x = f (r ) = 4a 3 - 3m'

other interval.
~

+ c, a, b.
x +b
a
and c positive OOtlStan l~; (h ) r = -,-- + c. a . b. ~nd c po siLi\'C
X +b
Gonslwtts. DrJ."' the dcmanc.l r~,~_nctiOI\ .r = f 1{1') and given flUtetion
r = / (.<) ill CllCI\C..<C.

(righ l ilg urc). Fintl demand functions if {a)

= 1/.<

18. / (.t) = _, . + 4'

x' + 2.x + 3

funcliOn X ~-I (1'). h l lhis runc1ion X depends o n r, indi(; ~ti ng that


ir the ,,rice o r the obj ect is set al I' ' then the m:ukcl will tkmanrJ .~ o r
li.Cm per week. T his fu nction is therefore called the t/(!m(tiUl fimrrion

23. if a m anufacturing finn sells .r obj'-'CU of u cc.rtain commodity 1:x:r


week, it sdh them nl a pric.c of r 1>e1' object, and r depends on x,
r
f (.x). In ~t:onomic theory this fu nction is usua1Jy consWcred
decreasing. as sh0\\ 0 in the lei\ fi gure below. hence it has m'l in\'erSC

h!IS

an inverse f unction r

+ r '.

0 < r < 2a

= f- 1(x ) . Whal is thcdOnl.i.1ill or / - 1(.1') ?

Or.tw it:s g raph.


r

* 25.

Find lhc im'CISC lunction for

f (x )

1 1.7

= ( I +''x)'

- I < X $ 0.

Trigonometry Review
Here i~ rhe d iagnostic tesl for this section; give yourself 60 mi nutes
only in problems that s peci fy their use.

w c.lo it.

Use a cal culator

DIAGNOSTIC TEST FOR SECTION 1. 7

In ull quc~t i o ns, :.1 nglc:o; urc in r..ulian lllc-~ur\: unld$ ind i(.'tltcd othct wisc.

In tlUCStions 5- 8 draw a gmph of the func1ion.

t. Express the. angles 13511 01nd

5. f ( x ) = 3sin2.r

-2 70 ~

in r.tdian measure.

2. E.x pr.:s.Jihc i1nglcs 2tr / 3 and -9tr/4 in degree measure.

3. Evuluatc the folltl\\'int; qu.antitic.-;:


( u)

sin (tr /3)

(b) COt(- ;r/ 4)


(c) cos (5;r {3)
(d) e>e (- 3n / 4).
:&. Flnd ~he unspecified Wl.Q.lcs und Lhc length of the tJinJ side of &he
tri11nglc below. You will 11<:cd a <:ulcuhllur for this quest ion.

2.~

3.5

6. f (x) = 2co<(x -,T/3)

+ rr / 2)
f (x) = 4 cos (2.r + rr/2)

7. f (x) = tan {x

S.

ln ques.tiOftS 9--ll fittd all solutions to the etlti<Hioo.

9. siox = J3/ 2
10. cos .r =- 1/2
II. tan 2x

=1

12. Express the hmctjon f (x ) = 2sin 2x - 3 cos 2x in the form


A sin (2.r + </J) where A is positive and </J is ;m angle in the in tcrV"J.I
0 < 4> < 27f'. You may use a calculator.

1.7

MiJt111;14fiM

Deliniliun

of a adian

TrigMOLMiry Re\'iew

51

Many physical systems exhibit a o osciiJatory nature: vibmtions of a plucked guitar string.
motion of a pendulum, alte rnating electric cunems, fluctuations in room temper.tture as the
thermostat continually engages and d isengages the furnace on a cold d ay, and the rise and
fa ll of tides and waves. Magnitudes of these. oscillations are best descl'ibed by the sine and
cosine functions of trigonometry. rn addition. r.ues a1 whkh these oscillations occ ur c an be
represent.ed by derivatives of these functions from c--.llculus. ln this section. we briefl y review
the trigonometric functions and their propenies, placing special emphasis on those aspects that
are most useful in calcul us.
In tl'igonometry. angles are.usua lly measured in degrees. radians. or mils; in c alcul us. angles
are a lways measured in radia os. In preparation. the n. fortheca.lculus oftrig:onometric fuoct ions.
we work c.omple te ly in radian measure. By definition, a radian is that angle .subtended at the
centre of a c ircle of mdius r by a n arc of equal length r (figure I. 77). When an a rc has length s
(Figure I .78), the number of units (if radius in th is arc is sIr ; this is called the mdian measure
of the angle subtended at the cemre of the circle

8=

MiJICJII;I ~ Relationship
be1ween arc leng1h and angle a1 cen1re
tlf Cii"C.Ie

( 1.37)

In pan icular, if s contains n ullits of mdius (s = n r ). the n s represents one half the circumfe re nce of the comp lete c ircle. and (} = ;r rf r = ;r radians. ln degree measure. this angle is
180 ; hence we can state that rr radjans is equivalent to 180 . This sta tement e nables us to
conven angles expressed in degrees to radian measure. and vice. vel'sa. If the degree measure of
a n angle. is f/J. then it is 7f l/> I 180 ra.dians; conversely. if an angle. meaSliJ'es 8 radians. then its
degree measure is 1808 In . for example, the mdia n measure fo r = 45 is 45n I 180 = nI4
radians.
Eleme.ntary trigonometry is c.oncem ed with rela tionships among ang les and lengths and, in

particular. angle.s and sides of triangles. Tit is naturally restricts angles to the mnge 0 ::; & ::; 1f.

M::iilclil;l#
posili()l) of :m angle

Sl;~ndard

for the purposes o f calculus. however, we lleed tOtalk about allgle IJ. whe re IJ is ally real llUmber
- positive, negative. or zero. To do th is we fi rst define what we mean by t.he standard p osition
of an angle. If & > 0, we draw a line segment 0 P through the origin 0 of the x y -pla ne
(figure 1.79) in such a way that the positive x -axis must rotate counterclockwise through an
angle. (J to coincide w ith 0 P. In other words. we now 1-egard an ang le as rota tion. rotation
of the positive x axis to some terminal position. If 0 < 8 < n /2. then 0 P lies ill the tirst
quadr m; and if 11:12 < IJ < 11:, the n 0 P is in the second quadrant. But if 8 > 11:, we have a
geome.u ic representa tion of 8 also. For example, angles 7n14 and 9n /4 are shown in Figure
1.80. When & < 0. we regard & as a clockwise rotation (f igure 1.8 1).

)'
~

Pc>sili\'e no-

Ncg;lti,e

angle as clockwise mentions

gles as counterclockwise rotations


)'

.\'

p
X

0
X

3/f

For any angle 8, the ang les 8 + 2n1f, where n is an integer, have the same terminal position
or 0 P as 8. They are diffe re nt angles, however, beca use the positive x -axis must encircle the
orig in one or more times before reaching the tenninal position in the case of & + 2nn.

Oc6nlliUQ:)
,., 1~\Mtutlwtnc:' ,.,)1,7~

With anl;;b

n 1$
. easy tn ~lefit'le th~ ~i ~ ui
. reprr:-!<1lted M I'UtitiOOS.
. (

~ an an~k: IH MandauJ posit tQn ( Fiurc 1 821 __ ,, (


:onomc:u~e UOCliOI\5. h

.. .., \ ,:.) are lhc: NOnbnaa_es, of P, \.\C: define

ll<ill8 -

P(x, >)

!
r

csc6 r
y

coso = ,.

tan8 ~

=0=

cot8 = :..,
y

When!\~T .l~ flh~ are defined. lind where T

=:

~ ~ postJve an ch

,.

!,

"I'

(1 .381

J ri + Y. l is alwaY'(~

t~IC~ tngonomet~ae func ttons 111 Lhc vureous qu;admnb nn: as shqwn in 1''1~ur

Slil.lhv.f

f1 ~
df
~
un"""' pQ-.111\e, mce
. e 1r1>t. an . oun 11qu~rJr.uu~. :md y i ... pt..l'iiilivc i11 the llrM and ~on\1, i ~ns nf

1.83.

Funhc~no~

~nee r :fi 0, ~l 0 t\nd cosO ate defined f0-1' all 0, whereas tan 0 a1ld l\.tc9 are not d n d ~ '
0. an~ .c:sc 0 and tot () do noc t:'<i$t whe-n )' 0
e 11( or

.\ =

Ddinlltons 1-18 indic.a.te thc1t

lriJU~K~fnt:trk furtaio$ ht fuur


pl.ldr'MI~

l
!J.in 8"

c~~;c8 = )

Smt Olhl '-\I!C.:rutt All tnJ!OilOt'I\Cb'ioc


p0!1hhc, 1111
othn!'o. ~r;tf.lvt:

""'~'tiOn t rusiti \~

In uddition, the h-e ttuu

r2

IUI'l9

.\ 2 + y1 leads to the iUentity

-.n2 9 + cu.2 9

C."1~1 f'IO'dh't. JlOS!II\-f; .1ll


:sll othcn. 11cgat i\~ vtht fiCJI.ilflw

1.

I 1.-l()a l

which. in wm. implies that

+ tan 'O
1
l + cot 0

= >~:<;> 0.
=

( 1 ,.((1~

c<c2 0.

(l

.10.: l

Table 1.2 contains value:..ufth~ trig.tmometric functlot'l~ (m the rmN commun1y u11ed ang1e....
You should comrnltat least Vjlluc:s for sine. col)ilk:, Htld ta.n.gcnt m tl'll:mo ty .

'ill '

CoMnc llw

>'

(I

(!

~.. I

~~

~'(

1f'

/6

0
1/2

73/2

1/JJ

undetinc._J
2

1f'

/4

1/12

I /J2

.../2

11'/3
11'/2

../3/2

l/ 2

../3

2/ ../3

undeliilCil

II

()

..

(','

..

UI..ICflned

2/../3
../2

../1

2
undct!ned

1(../3

ldc nlity I.-lOa is imply a rc.~uncmem of the f>ythagmcan relation (r 1 = ,.t 2 + y1). which allow~
us 10 express the hypotci1U<;C or" righiUng lcu ui,mgle in term~ ol' the Olhcr tWO \ides. I t' lhc
uiangle is not righHtn1ed (Figure l 84). il;. not possible to <Xf"<"' one ide. c. in ICtn" <f 1hc
<~her two sides. a and b alunc, hul i1 is possible to e.pres~ <' in 1erm< of a ond band lh< oo&lc
0 htiwccnlhcm. The ~oordinmc< ol P and Q in Figure J .84 ore (b cos 9, bsin 9) nnd (tr, 0);

lherefurc, by i.lislunce form ula 1.10,


I'

,., = I PQ 11 = (bcosfl -t1) 1 + (bsin9)2


= t?(oos2 8 + ..in1 8) + 111 - labco.8.

(I ..II

This result, c.allcd lhe cosine law. generalizes the Pythagorean relation to triangles that arc not
2
ightangled. 11 reduces to the l'ythagol'l:an oclali on c' = a
b1 when II = 1C /2.

I)Cclll;JAh-

Jf A. B. and C are the ang.les in1he triangle. of Figure 1.85. and ll. b . and c are the lengths
of the o pposite sides, then b)' drawing alii tudes of the triangle we can show that

Sine lllw

sin A

si n B

sin C

(I A 2)

This res ult, known a~ the s ine law. is also useful. in many problems. The sine and cosine laws ca.n
be used to find angles a nd lengt hs of sides of triangles as illustrated in the following examples.

I EXAM PLE

1 .31
Find length a and angles fJ and <f> in the triangle of Figure 1.86.

SOI.l"TION

We can calculate a with the cosine law,

angles and sidC's of a triang.Je

a' = 22 - 32

2(2)(3)cos(J.2).

This gives a= 2.9. The sine law can now be used to find 8,
sin 8

sin 1.2

2.9

sin O =

3 sin 1.2

2.9

The iol\'erse sine buuon on a t'id culator gives 0 = 1.3. (We will c.leal with inverse trigonometric
functions in dctoi l in Section 1.8.) Finall y. <f> = 11 - 1.2- 1.30 = 0.64 .

..-...

I EXAM P LE

1 .32
OiStall<'es aero.-;.~ water are often much grenter than they appear from land. We would like to
t-alrul ate the distance from the straight shorel i11e to the island in Fignre 1.87a. What we could
<.lo is take two points B and C on the shore some distance apan. l km ~ay, and rnea~ure the
angles ABC and AC B as shown i n Figure 1.87b. Use this infonmllion to calcu l<>te how f<ll
the islillld is from shore.

I@IUII;I#J
Oisum"'t
..

f'i1;l'll

m,m :hx-C' 10

IOjtclii;IWI
u1

EJW

IJCciii.IWI.fJW

idmd

C:::::> Island

~
0.

l.l

Shoreline

Shoreline

I)

SOLUTION Werequire theleng thof AD in Figure 1.87c. Angle BAC = 11-0. 8-1 I =
1.242. We usc the sine law to find b.
sin l. l

sin 1.242

b=

si n l.l

sin 1.242

0 .942.

We can now use triangle A DC to calculate that the length of AD is b sin 0.8
0.75 km .

..-...

(0.942}(0 .8)

A large number of identities ure S4:tlisfietl by the trigonometric functions. They tan all be derived
fnJm the foliO\\ ing compoundungle fommlas ror .sine:, aJld cosi ncs:

Pruu( ot'
con1X'Itll1d n&!le foonul.l

+ B)

= sin A cos R +en,, A si n B .

O A3al

sin(A - B) = si nA cosB - rosA sin B .

OA3bl

cos(A + B) = cos A cos 8 - sin A sin TJ .

( IA3c)

cos(A - B)= cos A cos 8 + sin A sin B.

(I A 3d)

sin (A

Let us prove one of dlCSC. say idemity 1.43d, where A and 8 arc the angles in Figure 1.88.
If P and Q lie on the circle x 2 - y2 = r 2 then their coordinates are (r cos A . r si n A}
und (r cos B , r sin B). respectively. A<X.'ord ing to fon nula 1. 10, the length of P Q is given
by

II PQ11 2 = (rcosA- rcos B}2 + (rsi nA - r si n 8 )2


= r 2(cos1 A + sin1 A)

+ r 1 (cos2 8 + sin1 B)

- 2r 1 (cos A em 8 - sin A sin 8}

= 212

21 2 (cosA cos 8 +sin A sin B) .

But. according to cosine law 1.41.

IIPQ12 = II OP11 2 + IIOQII 2 - 2IIOPII DOQII cos(A - B)

= r2

+ r2 -

= 2r 2 -

2r2 cos(A - B)

2r 2 cos (A - 8 ).

ComJ>arison of tl>esc two expressions 10< UPQll2 immediately implies identity 1.4J d.
By expressing tan(;\ + B) as sin (A + 8 )/ cos (;\ + 8) and using identities J.43a and
1.43c. we find a cumpoumJ-angle runnula fur the cangerll runclion,
uur A + tan 8
tan Jl tan B

( + B)
tanA

tan (A - B)

B
= Itan+ Atan- Atan
tan R

I -

and similarly,
( I.+I b)

By seuing A = B in 1.43a,c and 1.44a, we obt<rin the double-angle formulas.


sin2A

= 2 sin A co." A ,

co~2A

;;;;;: cos2 1\ - ~in2 A,

( I A6a)

= 2cos2 A- I,

( 1.46b)

l - 2sin 2 A,

(1.46c)

=
tan2A

2tan A
I - tan 2 A

( 1.~ ~ )

( 1.47)

1.7 Trigt,llOillelry Re\'iew

I EXAMPLE

55

1.33

Use a trigonometric identity to tind the cosine of the angle I} /2 if it is known that the cosine of
I} is 0.3.
SOLUTION
obtain

If we set A = I) /2 in double-a ngle formula J.46b, a nd solve for cos2 (8 /2) . we

cos li = 2cos2 (8/ 2} - I

cos (8 /2) =

l +cosli

I+ 0.3

1.3

Hence,

(13

cos (&/ 2) = y

= 0.81.

Without fun her information about li or f) /2, we cannot decide which sign to choose. for
instance, li = 1.266 is an angle whose cosine is 0.3. The cosine o f half this angle is 0.8 1. On
the other hand, 8 = 5.0 is alw an angle whose c.osine is 0.3, bul the cosine of hal f this angle
is - 0.81.

..-..

When pairs of compound-angle formulas 1.43 a re added or subtmcted, the product


formulas result. for example. subtract ing J.43c from 1.43d gives
sin A sin 8

= r- cos ( A+

8) + cos(A - 8 } 1.

( 1.4Sa)

'11Je other product ro,m ulas are


I
sin A cos 8 = - [s in (A + 8) + sin ( A - 8)],
2

cos A cos 8 =

By selling X
fo r mulas.

2rcos (A

+ 8 ) + cos(A - 8 ) (.

(1.48b)

( 1.4Sc)

= A + 8 and Y = A - 8 in 1.48, we obtain the sum a nd dilfer enee

( 1.49a)

( 1.49h)

cos

X+ cosY =

2cos

Y) cos (X -2 Y) ,
( X+
2

( 1.49c)

( 1.49d)

The following example is Lypicnl of problems in C<Jiculus when LrigonomcLric funclions <lrc
involved.

I EXAMPLE

1.34

Wrice chc expression cos 0 in terms or cos 20 and cos -18 .


If we repl ace A by 0 in double-a11glo formula l.46b,

SOLlJTION

coo 211 = 2 cos1 i9 - I.


II follows chac

I+ cos20
2

Consequencl y.
4

cos 0

= (cos18)., = -I (I + cos 28)> = -4I ( I + 2 cos 219 + cos., 20).


~

Buc if we now replace 0 by 211 in the idencicy cos' 0 = (I

cos1 28 =

+ cos 21)) /2. we obcaicl

+ cos41i
2

Thus,

) =
<-os' e = 4I ( 1 + 2cos 21i + I + cos48
2

By regarding arguments of lhc trigonometric runc tions tu angle:... we ha\'C bccll stressing gc
ometric properties ofchese funcoions. In pnrticulnr. i demi tics 1.43-1.49 have been based on
defi nicion.~ of' chc trigonometric funccions ns functions of angles. W luu i~ importune ;tlx.lut a
function - be ii a crigonomeioic funciiOtl or any other kind of function - is that there is a
number assuciatOO with C'.dt:h V'"l ue of the indepcni.Jen( variable. How we arrive at lhis number
is irrclcvllJU. As fior as propen ics of 1hc funccion are concerned. only it< values are token into

aC("OUUt.
..
'ThuS. when we (j iS<.:USS propenies Of (I trigonometric function, what iS important is not
that its argument can be regarded as an angle or that its va luc.o; c an be <.ld l ned a~ ratios of sides
ot' ~l triangle. but 1ha1we know its values. I ndeed. it is sometimes unwise to regard argument$
of trigonometric functions as angles. Consi<ler, fur example, the motion of n mass m suspended

from " spring wich spring coot<!a lll k (Figure 1.89). I f,, = 0 is che position at which the mass
mi!J.;.,
fll\!1\l

unJI

u( m~1:-.s

Displac:c

in vitmting. ~)~tom

.
l_

would hang nu.>tion le~~. then when venical oscilltUiOtlS arc ini timed. the positiOtl of
ftonction of time 1 is al ways of the form

as a

where A and B are consmms. Clcao1y, there arc no angl es " 'socimed with 1hc motion of
m , and it is therefo re unnatural to auem(>t to inteq>ret the <trgument (./f7i1i.)t as a phy>ical
angle.

x=O

Hef'K.'Cfonh. ' ve consider 1hc rrig()nometric func1ions ns those of a real variable. 1f it is


<."'il\'Cnicnt to regurU the. argunl CIH ns o.n angle. then we du so. but onl y if it i~ convenient. To
cmphasi r.c l his, we replace 8 wi th our usual gca~erlc label for the independent variable of a

funccion, namely, x . \ith lhis ch:1nge, the lrigonometrlc fu nctions are sinx. cos .t . hm .r,
esc.r. sec x, and coc x. Their graphs are shown in Figures

1.\10.

These graphs illustrate that shl X. cscx. tan x . and cot .\' are odd functions. and cosx and
scc.r arc even. Trigonometric funct ions nre per iodic. A funcrion f (x ) is said to be r>eriodie if
there exi~tS ~I number T :O:UCh that for all .\" in itS domai n Of definitiOn,
f(x

+ T)

= f (x).

( 1.50)

The smallest such posi tive number T is called the per iod off (x) . Clearly, then, sin x, cos x,
cscx , and sec x are peJiodic with period 2ir , whereas tan x and cot x have period 1r.

MOhtlil IMHJ,fW

W)Uiil I 4@ (.1

Su'le ftuw.:dt)ll

tM:lu

Uxar~nl r\lth~

,.

y c,."f;CJ(

1=SX

-n

\
"

:1
\:..-:-;.
;.!!,

: )ll

) l: - -n '
.

:'

"

,
'

v :

'

'''

\Vhcn 1wo pt::riodic runctiQn$ ore added together, the rcS\Ihing function mfl.)', or may not,
lx p!:riodi~;. ~Vr :Xliln'lplc:, lhe fu nction f ( .t) = 3s in '2.r + 3,/3 cos 2., is the addition of two
Tt periodic fulctioM 3 s in 2.\" and 3../3 (OS 2.\" . h tlh~Q hm~ 1>eriOO ;r. Fm'lction j (X) = '2 :oin lx
has period 2 JT / 3; functiOI'I g(x) = - '3 cos l.\" h.,'1...;; petiod 1r. n,eir sum 2 sin 3x - '3 CQS1.t
in Figure 1.91 has period 2rr : it is the smallest iotC"-1C\1 i 1\ which both fu fiCtions 1'\)roducc:
thcmsc:lvcs. The func tion .f' (.\) = s in x +2 s in ..fix in Figure 1.92 is not pcriodic cvcnthoug,h
s ilL\' and 2 sin ..fix ;u c b01h periodic.

3l\

_, 0

2M

~s

M
10

,(\(\~(\(\

~v\

V ~ x

-3

-2

-.1

Seldom is lhe Hrgumenl of n trigonOmetriC funtlion just


tiu~ecion of x.

I EXAMPL E 1 .35

Draw 1he graph of 1he func1ion f(x) = cos (x SOLUTION

(Figure 1.93}.

1r/

The graph of this function is lhal o f

in appliculions: il is U<Ually

1\

3}.
COSX

tmnslaled tr/3 unilS 10 the right

FIQUR& 1

v= ros(x - :</3)

3
-I

I E XAMPLE

1 .36

When a rna"" vibmll!.\ on the en.J of a spring as in Figure 1.9-1. and there i no friClion with the
su1face.. or air rc~h.Utncc. the po.siLion of then~ (in centimetre)) rchlli\'c to il;) po~il.ion when
the spring i~ unwctchcd take; the form
x(l)

= 3sin 21 + 3 J3 coo 21.

Numbers would be .Jiffercnt, but .<(1} would be a cumbinmion of u sine function and a oosine
function with the >ante argunlCnt. We have chosen >implc number> her<: .u that unnecessarily
compliedted calculation do not obscure the significdnce of the di>cuion. Plot a gr.;ph or x(l)
for 1 > 0. and then dm" a graph. What advantages are derived from the drawing as opposed to
the plot?
Mjld 11 1* lj

Mjl III:IWI

Ctw'I1JIIlcr plot i'or dlq)~t


of m.tn in vihrari11g $)"'fiU:m

Spnng

Mass

-2

x =O

-4
~ -

SOU 7TION A plot of the function is shown in Figure 1.95.


To drnw a graph of x(l). we first write it in the form x(l) = A <in (21 +) . where A and
are coasumta. When we equate this to thcghen expression fur x(l). and expand sin (21 + )
with compmondo<~ngle formula 1.43a, we obtain
3sin 21 + 3.fi cos21
This equation will he

A sin(21 +)

A[sin 21cosql +co~2r sin ql].

moe for alii i f we choose A and c/J to smisfy

3J3 =A sin ,

3 = A cosql.

To solve these for A and 1{1, we squnre each equation n11tl add the resuhs.

27
This imp lies t hat A

= 6.

+9 =

A 2 sin 2

If we choose A

3.J3 =
These equmions are satistied by r/1 =
be ex pres.sed in the fl-.rl'll

+ A 2 co.-l ,P =

= 6 (A = -6 works equally well). then

6 sin~.
tr /

A 2

3 = 6cosr/l .

3 (!here a re other angles also), and therefore .r(l) can

= 6sin (21 + tr/ 3) = 6sin [2(1 + tr/6)).

.r(1)

The timction is most easily graphed by shifting the graph of /(t) = sin 21 in Figure 1.\l&l tO
the left by ;r / 6 u niu; and changing the scale on 1hc .< ax is. The result is shown in Figure 1.96b.
With x(l ) cxprc.sed in the fonu 6sin (21 + 11/3), it is clcanhat thcfunction has period 11
nnd that oscillatio ns lake place between x = 6. These are facts that we could surmise from
Mdclll;l4n H

.n.l"r;

ll~nd{lr'tVIl '"~*'or di~J'I.K'1l1 C'tll

ur ma>S in \'ibtttlini;

x =1\in 2J

"2

1(

x = o.<in 12(1 + :r/ 6)]

3
2

>)~em

2.T Sn

!!
J

- I

the plot in Figure 1.95, but evidence WOtlld llOl be concl usive. \Vere we 10 l)eet.l values of I
at which the function has value 3. sny. it is defini tely ad-.unageous lo have x(t ) in the fonn
6 in (21 + 11 /3). We can visualize possibilitics "' t -coordinates or 11oint&of i ntcr..cction of the
curve in Figure J .96b wlth the horizontal li ne x = 3. There is an infinity of values. and LO find
t hem ulgebmi cally, we mu51 solve J = 6 si n (21 + :rr: /3), or sin (21 + ;r /3) = 1/2. We can
th ink o f 21 ;r/ 3 as an angle whosesim: is 1/2. one possibilit)' being ;r/ 6. But there arc many
other angles with a sine equal lo 1/2. Bcc11usc the si11e !\me lion is 2:rr: -periodic. each of the
angles ;r/6 + 2mr. where 11 is an integer. also has sine equal to 1/2. Funhem10re. t he sine of
Sn / 6 is also equal to 1/2, and when multiplies of 2;r arc added, each ofthcang lcs 5;r / 6 + 211n
ha,; s ine eqwllto 112. In other words, all angles that have asineequal to I/2 are

(;
" + 2mr
where

11

5Jr

6 + 2nn:,

and

is an integer. In ocher words. we can

~e c

::.6 + 2 1111

;r

21+-=

5:rr:
6

+ 211Jr.

Consequently.

21 = [ :

~+

_ !:._ + 1111
211Jr

2 + 21171

12

1r

- + fllr.
4

60

Cbopler I Calculus Prel>.tr.~~iun


Because we are only inrerested in posilive values of I, we must choose 11 ::: I when combined
wilh -n/ 12, and 11 ;:: 0 when combi ned with n/4 . 'llte smallest posilive value of I is Jr/ 4;
1he second smallcs1 is lin I 12.
You begin 10 apprecia1e the elegance of this solution and the simplicity of the result when
you compare 1he magniiUde of the problem were you 10 auempi iO find, using a calculator or
com puler. all sollllions of 3 sin 21 3J3 cos 21 = 3. Think about i1.

The curve in Figure 1.96b is an example of a general sine funclion,


f(x) = A sin (wx

+if>),

11.51)

where A and w are posilive corJSrams and if> is also cons1an1. The graph of lhe general sine
function is shown in Figure 1.97. The number A, which represents half the range of1he function,
is called the amplitude of the oscillm ions. The period is 2;rfw, and - if>/w is called the phase
shift.
M#jlclil;flli(f' I Gcncml ;;ine funcli(').ll

1- 2Jr
- 1
w
Example 1.36 is an illusrration of lhe following \'ery useful resull. A function of 1he fom1
f (x) = B >in w..v + C cos wx c.an always be C:\pressed in form l.5 1 for a general sine function.
The amplitude is g iven by the formula
( 1.5~)
Function B sin wx + C cos wx can "lso be exprced in I he form ,t cos (w,r
once again given by the formula in equation 1.52.

+ if!), and .4 is

I EXAMPLE 1 .37
L~

The em.f device in the LC -circui t of Figure !.98 produces a constant voltage of II volts. lf
the switch is clol),C(.) at lime I = 0. anU 1hcrc is no initial t.:hk!rge on the t:Jpaciror. che charge
th=aftcr is given by

ScMti1<Uic

for l..C -<.-i:uil

Q(t) =

~ (1

-co s

lz:c) ,

:::. 0.

Dmw a gmph of this funct ion.


SOLUTION We begin by drawing a gmph of cQ<S (t/./LC). It is a standard cosine functio n
(Figure l.99a). The gra ph of Q(t ) in Figure l.99b is then obtai.ned by
with period 2rr
tuming Figure 1.99a upside down, shifling it upward J unit.., and changing the scale on the
Q-axis.

.JLC

. , 111.148 J

'd'4"
Q

!VTC

_,
f l is wu11h\\ hilc J>Oinlina o ut here, a-. we afe sure you rc_.li7..e, ' hat el tronic dcvicu cannot
p lot cut'\es containing u t~pcd l iecJ J):;ll'lltnetei'S. This is anocher rc.IM)Il wh) \\C must d C:\'t1op our
gn.1phh1g ~kil l ~ ~uld not rely 101.tl1y Oil graJ>hing calculaU:'IN a~nd tlomputer,.

.-.
I eXAMPL E

1 .38

Find all Wutiofl.) for each of 1he fol"'"in~ ~u..taion..~~


(11)

<in

= ./2I

<bl e<l012< =
(c) Jan (J.r

v'l
-T

+ I) .. -

.fj

SOilrrlON
(a) One MJiuciun or the cqmtliou hill X = 1//2 is 1f / 4. Thi" is noc the u nly solution,
however: chcrc ar-c many angles wi th o.t sine equal co tJ-Ji Sul~.:.: )lin:c i~ 21r
potiodie. 1he angles 211n + nf 4, fo r 11 any integer, all have ine equal 10 1/..fi.
Berau'c , ;., (n - X)
>inx, i1 follows lhm >in (311'/4) = 1/./2. lln<lthe..,forc
3n / 4 i<anll!hersolution. Wh~n multiples of 2n orr addedtolhJ> on:;lc. 2111r + 31f/4

.trenl -..o $Oiucion~. Thu~ the complete~~ of ~lution~ ~


21flt -

s..plotied
tlrpr\")t.t.lfiiJf'l

of SoiJIIHJ4)11~ (O

"

2nn

- .
4

3Jr

+ -.
4

where It is llJllillC'!h-Y. f1gUrt J. l00 SU~'OolS ttut thi~ llCl ur numbcn CWl be rt"pn:
sen1cd mo.., compactly as an initial ro lalion of Jr/2. plus or minus n / 4 . .md possible
llllllliJ>IC! of 21f : Chill is.

ltiJOICNJlCffk ftillibJn

tr
n
x =-:1:
-+211/f
= ( 411 + I )
2
4
2

If
n -.

(b) Onewlution or111c cqu111ion C'O> 2A = - .j'jf2 ror 2x;. Sn / 6. Ducthcrc nrc others.
Since th(; ~ine fu ncti on i~ C\'t:fl~ all solutions are g.ivcn by
X

2.\ =

SIT

6 + 2/IJT

("he~

n is an inccgcr).

Consequently.
511'

x = :1: -

12

+ mr.

62

Calculu~ Prepn'Otion

Chapter I

(c) One solution oftheequation tan (3x + I) = - ./3 tor 3x +I is 3x +I = 2Tr/3.


Since the tao gem function i~ 11' -pcriodic, all solutions cao be expressed in the fonn
3x

2;r
I = 3

+ 111r

Consequently.

(311
-

+ 2)JT

(where 11 is an imeger).

(311 + 2)11'
X - - -- -

EXERCISES 1 .7

In Exercises 1- 10 express the angle in radians.


I. 30''

2. 60'

3. 135"

4. - 90''

5. -300

6. 765

1.

lhc smaller building 10 lhe bouom of 1hc 1aller building is 3/ 5 radians.


llow tall are the buildings?
24. If the angle of elevation of the sun is 0.80 radian, and the length
of U.: shadow of a Oagpole is 20 metreS. how high is the pole'!

For c-Jch of the lriangles in Elercises 2.i-28 use the cosi.nc law and/or
Ihe sine law 10 rtnd Ihe lengths of all tbree sides and lhe measures of all

8. - 128''

9. 321"

the interior angles.

10. -2 1J "

,-"

~ 26.
2

In Exercises 11- 20 express the angle in degrees.


II.

rr/3

rr/3

12. - 5rr /4

13. 3:r / 2

14. Srr

IS. -5rr / 6

.16. l

17. - 3

18. 2.5

19. - 3.6

20. 11

il

17.

28.

~
4

21. Wh,il angJc is subtcndcU al the ccntr~; ol' a circle or radius 4 by an


ore of 1cnglh () 2 , (b) 7. and (c) 3.2?

29. Use compound~ ;msle formulas 1.43 to pro\'~ idcntitie:s 1.44.

22. 1'1\e an,lcorcle>ation from a tmnsit lo the lopofa building is t.30


r,uhans (!ell figu,c below). If the <ransil is 2 111<..'UCS abo"" lhc groun~.

J l. Use compound-angle fonnulas 1.4J to prove pl'llduct formulas


1.48.

and the di~tance rrom the building to U1c lxan~i t ls 30 metres. how high

J2. Show that lhe sum and difrerence fonnula-: 1.49 can be obtained
from 1.48.

JO. Verify doublcangl-c fonnules l .45, 1.46, und 1.47.

~the building'?

c
0

CJ
CJ
CJ

.....

c
c

----~:~i.-,
-

.30m -

.,

..

'.

In ExerciS<S 33-54 drw u grph of the function.

..

J3. f(x ) =3 sinx

o ............ w .. up.r(r..:.
2m

35. J(x)

"(.(_;) . . . .

.l7. /(X)= 3sin (X+ 'T /4)

0
Cl

39. f(x)
41. f(x)

lOOm

= 3sin (2.r + IT/ 4)


= 2 sin (.r / 2 -rr)

43. f (x) = sec2x

~ 23. 1\\'0 btlildiogs arc I 00 melte.s aJ)al'l (righl tiglil'cabovc). 11le angle
of elevation from the tO)> of tJte S1naller building to Lhe tOJ) of the taller
building is I I / I 0 radians. 11le angle or depression fro1n the tO)> of

= 3sin lx

45. f (x) =csc (.r -rr/3)

47. f (x) = 1an2 x

34. f(x) = sin 2.r

36. J(.r) =<in (.r

+ ;r / 4)

.18. f(.r) = sin (2.r + :r /4)


40. f(.r)

= 4co<(.rf.>)

42. f(x )

= h-os(rr/ 2- J.r)

44. f(x) = Lan3x


%. f(.t)=COI (.t +JT/ 4)

48. f(x)

J 1-cos' x

49. J(x) - ./1 + twll x


51 .

f (X) =4 + 2ton t

SJ. J(;t) '"


-t

50. J(x) - 5- 2ox


52. f(x )

7 1. Amplitl.lde modulaJion i:t the prou~u of muJtiplying a lowfrequency sisnal by a higllfroqucncy inusoid (as occum:d in Exercise
70). II os en, cechruquc u<td co brooadc:l<t AM radio pSnll<. The AM
<ranal is a produce of 111<! form t(/ ) = 1 (/ )CO< (2rrj 1). where lhc
lrcquency f is mooh hi:;hcr chan ny frequency on V(l). The cosrnc
term is the c;tlrri~r liXIIUI and U(l ) b the 't<Oicc or lll u.)K; sig_naJ to be
tr.UlSilliltcd.
C) Ploc a graplll>f .t (t ) """" f - 700 II < and v(l)- S +

tan lxl

54. /(X)= Je>c(.r/ 2)

-I COl 2xl

55. \Vl\cn 0 j ll\ din j~ rcltl&t>Cd rrom height h af)()\'C the ground with
speed v :u angle V with the horizontal (figure below), I he hotizonHI
distance R that it travel~ <gi"cn by the formula
R

2 cos(.W1fr} .
(b) \Vhat a!'\! minimum aml mu1-imum amplitud~ of the mcxJ-

"' coo:O
- ( sin9 +
= --

ulillc:d sigrwl?

where g

In Excrci.scs 72-77 Cl.pt'CS!.). eat.h rufK:cion a_.; a gcncnd .sine function.


idcotirying it IPiitude. l><r;oo, nd 1~,..., >l>iA_ O.w l!f"'ll> or ca-b

9.81 i lhc acceleration due toP"' ity.

function.

72. /(XI= 3sin:tt + 3coo3.r

7J. j(x)
74.

JC.ri = - Zsinx + 2..fScoS1

75. /(.t)

and angle. but a1 height

(c) WilJ1 wh~l ~d mu.st tJlrO'\\'CT A rck~ the j~'c:li n i r he


i.s h> ~~<."hie'c: the 5a.mc R as (} li'-Q'Io\'c:r 8 . U$.Sutnin.J tluH 0
:r/4 nd lr = ~ n>?

In t-\CfCt<c< ~hAd all (()lutions of tb! tqu.arton. All <Oluton<


imoh~ the qandard antic:<: an T:.b"= 1.2. For cqualtOM. "'''h <OiuiJOft(
lnvoiVInJ ~r :angf.. 'Ae reqUlre the IR\"t'f'Se ln000me1tlC fUnctiOnt
01 Sect 10<1 I 8

./3/2

57.

~.

co,.t=-1/l

59.

60.

~i n x

56. sin.t-

=cosx
=-

62. C0<2t

64. 2iin3.r

C<dX;
C<)l .t

63.

1311 3.t

= ./3

66. sin 2:c - sin x

2- 0

""nt

f~quencK'f..
not~$ rcsu..l Suppose. for example. th:11the (tgn~~
produd by til<! onstrvm<nts :tre ot 180 Hz and 220 Hz. soy>< (360:r 1)
:>nd <00 ( 44();r I) .

Cot Show llulch<!o:olllboned <ignaiC3Jl be t>l'f\'<d on 111<! fnrm


t (l) = 2<c><(40!rr)coc(400;rr) .
(b) l'lot. on the same a.cs. graphs of 2cos (40rrr) and x(l)
~ I

mndu l~tcd

( X) -

- ,' '-n..:...r_
:-

8 1. ,.., 2

J + 005X

82. I +lallX = tan ( x


1 - &.lDx

+"
-)

3.1. Show I hat a lunciiO<I

f (.r) =

A CO< w.t

+ 8 <in w.t

can 3lway<

0.8. The amplicude or CO< (400Jrt) i<

= / A 1 + B' >Ill (Wf + cj>).

is defined by the equation'

sin ell

I- 0
69. sinx + cosx I
70. When C\\'0 mu~~eal ms:trumcnts pb y noaes With nc:~.rly dental

for 0

19. ~i.n4x; 8cc.n.3 .c ~nx - 4~A ~inA


3 tanx- tan) x
80. l4ll Jx ~
I- 3tan1 .t

-./2

"' 67. sin x - sinx

JC)- >rn1 2x - coc.1 2.t

7tt CCl"3.t= 4 cosJ.r-JCOIOX

.here~

611. 3cot' .t

lj.:;

/ (X)

= - ..rs

6S. sec4x =

17.

he ~n t t<:n in lhc 10nn

61. 2cos' < = I

1/./i

2./Jco<SA

In ExCI\:iscs 18-82 verify the ii.lcntily.

(b) Wh:u is I( for thrower 8 . who. bd ng!:lightly l;tllcr, rclco~c'=


~poed

= -2 .,n SA -

76. /(.<) - sin.> cox

(a) What b R ror thrower A. who rckdSC) the j .a ~o c lin Wllh


.)P\.'Cd v- 20 m/.)atangJc O = .T /4 and hci~ht 2m'?

the Javelin wilh the same


2. 1 m1

= 2si.,.,_t - 2coo.4.r

= - 17iF=ffi
JA' + IJ'

and

"""' - r7T=:=~
../A1 + 8 1

~. I n Excteise 83 can we rcpiMCC the 1wocqua1mn~ defining tP with


Ill<! <rngle <qll31inn L1ll ~ = A/ R r

Ill Exercises &~ fic:d all solutions of

~X<

equation on tbe ioten-.1

2.

8S. a.ln 4x - cos 2x


1!6. cos . + cos 3.r 0
87. sin 2x + cos lt = sin .tx

88. sin x

+ cosx = J3 sin.r ro~.t

by 2cos (401l"t ) . Yo u wo uld hear the signal

co~ (400rr /) fade in and out as its amplitude ri"lc~ and falls.
n lis J>hcnomcnon is cal led beating ol'toncs in music. Mu
sician'> usc it to tune two instnunems to chc s~unc J>ilch. (Sec
alliO Excl'cisc 7 1.)

89. Verify thm if A,

+ tan 8 + t:111 C = t:m A tan 8 tan C.


Expand tan (A + 8 + C) in tctm ~ of tan A , tan 8 , and tan C .
tan A

Hint:

8 . and C Me the a ngle~ of l- Cl'ionglc. then

ANSWERS TO DIAGNOSTIC TEST FOR SECTI ON 1.7

1.

3></~ .

-3><n (2nwts)

7. 12 marl<s)

2. 120. - OS' (2 marks)


3. ()

,/3f2 (b) -

1 (c) 1/2 (d)

- ,fi (4 mali<s)

4. 0.74 r<~c.l i ans. 1.8 radit~n8. 2. 1 (6 miirkS)

5. (2nwb)

8. (3

lll3ric:\)

7
6. (3 nwks)

+ 2n.rr. 2;rf3+ 2mr , nan imcgrr (3marks)


2.rr /3 + 2nJr. - llr / 3 + 'lttrr, rt an integ-er (3 mark$)
rr/8 + lllf _ 11 an ir'llegc.r (3 tnltl'b)

9 . .rr.J3
1().
11.

12. JEsin (l.<t + 5.3) (3 orlfks)

l t.s The Inverse Trigonometric Functions


Thi~

ma1arial is likely new for ma'l}' smdems. T1 do.es nO( 1herefore h<lve a diagnostic te..t\l

et~wc itt l.ed with

it.

FtuK:ti<>ns thm invo lve o fini te number of additi ort~. subtnlCtions. multiplictllions, divisions.
and roots arc called >llgebr aic !'unctions. More specilically. a function y = f(x) is said to be
algebraic if. for all .f in it~ domain. it sati sfie.~ an c<juation of the fom1
P0(x )y"

+ P t(x)y"- 1 + + P._ ,(x )y + P,,(x)

= 0.

( 1.51)

where P0 (,<) ..... P. (x ) are polynomials in x. and 11 isa positive imeger. A polynomial P(x)
is i!Self algebmic si nee it satisfies y - P(.r) = 0 : that is. it satisfies 1.53 " ith 11 = I ,
Po(x) = I. and P,(x ) = - P(x). Ratiurwl functions P (x)/ Q(x) are also algebmic [11 = I ,
P0 (x) = Q(x), a nd P 1(x) = - P (x )]. The func tio n j(x) = x 113 is algebraic since it
S<ltisfics y } - .< 0 . The equation y 3 + y
.r dclincs y as" function o f x (a grph of the
curve would illustnllc that it satisfies the vcn[cal line test). We cannot fi nd Ole function in the
form y = f (x ) . but accord ing to equation 1.53, the function so defined is algebraic.
A 1\uK:tion that is not "lgcbr" ic is called a transcendental fun ction . The trigonometric functions and the exponenlitl l <1n0 IOg<lrilhm funclions (lObe reviewed in Section 1.9) are

transcendental. In thi s section, we consider the inverse trigonometric functions~ they arc also

transcendenta l.
rn Section 1.6 we discus..~tl inverse functions, whnt it means for one function to be the
inverse of another. We learned that, algebraically. a function has an inverse only if it is oneto-one. or geometrically. if its graph passes the horizontal ljne Lest. Since the trigonometric
fuoc1 ions do nol satisfy lheseconditioos. 1hey do not have inverses. But we also learoed that it
is usually possible to restric.Lthe domain of a function that is not one-to-one in suc.h a way that
an inverse function can be defined. We do this for the trigonometric functions in this section.

I.S The I n\'er:;t Trigormme1ric fut..:lioll'>

65

The function f(x) = sin x. defined for a ll real x . docs not have. an inverse: it is not oneir.s graph falls the horizontal line test. By restricting the domain of sin .\:. however. the
fuHctioH can be made one-to-one. and this can be done ill many ways. ln particular. that part of
sin.\ on the intcrvaJ - :Jr / 2 ~ x ~ 1t / 2 is one-to-one, and therefOre ha.-s an in\'crse function.
This function, denoted by
to-one~

y = Sin- x,

(1.5-'l

and called the inverse sine function, is: shown in Figure 1. 101. The range of the function is
tr

- - < Sin-
2 -

tr
< - .
- 2

.t

( 1.55)

The values that fall in this range arc called pri ncipal \aJues of the inverse sine function. l11cy
have resulted from our rescriction of che domain of sin x to this same intcrvaJ.
An equivalent way to dc.ri,c the. inve rse sine function is as follows. 'Jbc reflection of the
graph of y = sin x io the Jj ne y = x does not repl'e,se-ot a fuo-c.tion. but by restrkth.tg the
range of values of rbe reM cered curve, we can produce a single-valued ftll>etion (Figure I . I 02).
OnL-e again this c;.e.n be done in m;my w ays. ami when we do so hy restricting the y -values to
Lhc interval [ -;r/ 2 , IT / 2J. the resulting function is call ::!<.I the:. invc:r~c si ne func.:tion Sin- x .
Note very carefully that Sin - x is not the inverse funct ion of /(x) = s in x. because the s ine
functioo has no inverse. Jt is, however, the inverse of the sine function restric ted to the domain
- ;;r /2 ~ x ~ 7r f2 . Tt is perhaps then a mis-n(.)mer to call the function Sin - 1 x the inverse sine
function. but this has become the accepted termi no logy.
\Ve now know what the itwerse s ine function looks like graphically. but what does it mean
to say that y = Sio - x? Certainly, g iven any value of x, we can push a few buuoos ooan

electronic calculator and find Sin - x for that

x.

To usc inverse trigonomclric functions in

pntctice, we musr have a fee ling for whnt they dn. Th oblain this insight. we nQfe I hat if (x, y)

is u point on the curve y = Sin -

I X.

then (y, x) is a point on the sine curve: that is.

y = Sin - .x only ir x = sin y.


M:ilfiii;J:jiFCi!J

lrn-e.w: of l'il ;r found b)' restricting the

..Jomain of l}M; fwxaion

( 1.56)

Miitijii;I31'1J Ci!ia lnve~ of Sill X fc)Und b)' reflecting il~


graph in li ne; J = x a.}lj rcstria ing d~e range; of the rctlcc:tiOJ

~'") ry -= Sin- x
y =sinx

- .rr/ 2 - I

n/2

.r

- 2"

-l
- rr../2

, /

In the laner equation y is an angle and x is the sine of that angle. llws. when we see y =
Si.n- 1 x ~ we may read ;y equals inverse s ine x," but we should think :y is an auglc whose s ine
is x: For instance, i f .r = 1/2, then y = Sin- 1 (1 /2) means 'y is an angle whose s ine is
1/2." C learly, the angle whose s ine is 1/2 is tr/6, and we write .Y = Sin - ( 1/ 2) = tr/ 6.
Re membe r that we must c hoose the principa l value tr /6; ohere a re many angles th at have a
si ne equal to 1/2. but the ioverse sine functiOl\ dernaods that we choose that angle in the r.mg:e

- tr/2 :S y :S tr /2, and slate it with a number in this inteJYal. For example, 2tr tr /6 may
represent the same angle with the positive x-axis as tr / 6, butt he number 2tr + tr f6 does not

v
I EXAMPLE

lie bemo:n - 1r /2 and 1r /2. In summary. tile fuox:tion sin x regards x ~an angle and assigns
10 .t the >inc of lhc angle: the io1cr..e oino function Sin- x rq;.~rth ,, ~the >inc of an an~le
and assigns 10 x the angle with lhat sine.
Another n<X~iOilthat i~commonlv used for the imcrsc .sine function is arcsin x. Oncreasoo
the notation arcsin.\ is prcfernblc I~ Sin- .t is the possible misioucrptetntion of Sin- X.
Sometimes students regard the - ., as a power and write Sin- t x llS I/ sin x. This is not
corTect, a fact chat you were w:1med abnm in Section 1.6. Sin- is the nnme of a function.
just a~ sin is the name of the sine lionction. and J is the notation for the positive square root
function. The capitul Sin Sin - I should also warn )'OU that this i) the inverse sine function.

1. 39
Simplif~

each of the foliO\\ ing c<prcssioos:

(a) Sin- (-./3/2)


(d) Sin- 1 (3/S) Sin- (~/5)

(b)
(e)

Sin- ( I)
sin [Sin- ( ./3/2)]

(cl Sin- (3)

SOl l.'llO!\
(a) Sin- (- ../3/2) asks for the angle "hose sine is equal to -../3/2. Clear!),

Sin- (-../3{2)
[!

To;.

!lll1C IU 01 lhc. Milltmcnl ~ -.

S;n

(3/S)

= -n/3.

<u> s;,,- ( 1) = ~r/2.


(c) Sin- (3) is no1 defined since the domain of Sin - .t i< - I ~ x ~ I .
(d) If t/1 = Sin - (3/5). lhen t/1 is illustrated in the triangle in Fi~ure 1.103. Since lhe
Lhird side mu<l have lenglh 4. it follows 1ha1 Sin- (~/5} = rr/2- t/1. and

(el <in( in

<J3m] = <in(rr/3) = J3/2.

I EXAMPLE 1 .40
Find the vnlucs of x for which the following are vaijd:
(a)

sin (Sin- x)

= x

(b)

Sin - 1 (~in X)

SOLU'I 101\ These two cqunoions express lhc fco lhntthc inc function nnd the inverse sine
runcUon are invc:r~c~. provided that we are careful about domuin.):
(a) noc equation >ill (Sin- x) =X is \'lllid for -I <X < I. Given an X in this
ontcrv:ll, Sin- x rinds lh3t angle (in lhc principol """" r~ngc) "hlch has x as iiS
>inc. Then >in (Sin -I .t) takc>lhc >inc ofthi> an~lc. Naturally. it return> thcori~al
number c.
(b) The funCiion Sin- (sinx) is defined for all x. bul oroly on the domain - Jr/2 ~
X~ n f2 IS II equal to X.

Our analysis of the i nverse sine fune1 ion is oow complete. We could give a similar d iscussion

fo r each

or the other live trigonometric fu nctions . Instead. we give un ubbrcviatcd version fo r

the inv~rse cosine function and tabulate results for the remaining four functions.

T he reHection of the grnph of the function /(x ) = cosx is shown in Figure 1. 104. and it
does not represem a function. If we restrict the y-values on the reflected cune co

0 .:5 )' .:5

( 1.57)

1f'

then we tlo obtai n a function called !he hlvt rse cosine runclion. denOted by

y = Cos- 1 x.

( 1.58)

TI1e vctlucs in 1.57 are l11c principal \1alucs <!)f tht: invt:rsc cosine functio n. Note again that
Cos-t X is 110 1 !he i rwerse function of COSX , but of f (x) = COSX, 0 ,:5 X ,:5 7r,

M:llrlii;IW8ht

ln\ti"\C

<.v~ine

fl..mclion lrt"'fTT relledion of cos1ne: ll.faph in line: y

=x

\ Vhen we write )1 = Cos-J x. we read this as y equals: inverse cosine x but it means
that " y is an angle whose cosin e js x '" for if y = Cos- 1 x ~ then ..t =cosy.
The remaining four inverse trigonometrk func tions, along with those of imerse s-ine and
cosine, arc shown in Figure I. 105. Bluck tmd blue curve~ represent reflections in the line .)' = .r
of the Lrigono meLric functions tan x. cot x . esc .r, and sec x . Blue cunes re1>resent principal values of the inverse lrisonomcttic funclions Ttlll _ , x. Col - I X , Csc- x. and sec- x.
Principal values o f the six i nverse trigono metric functinns are listed i n Table 1.3.

( 1!1 iill
Sin

Bl&f]

IJ

Crupb of

Ccx

,t

Gnacm or

Li1!l!J ill i&l!l13

t!Iil48loJ.1
1

Tan ' x
)'

)'

y=Tan-lx

"
y = Cos- x

................... ................
l1

.:(_!!.
. )_/
_~
2

;..----

y= Sin-

- I

Gr-...ph of

FIGURiiii t . tOiid
C111 1 X

M4l3111i

FIGURii 1.10tie

Ci<e

Sec

iil!13a

Gr..tph of

)'

y =Sc~-J x

-3; !~-~~~~:~---

:::::::::::::::::: fc::::::: ::::::-=:?:~: t::::::::::::::::


I

~~--- ; ----~~

......

- I

4-404444.4040+

: :: : ~~~- ::~::::::

"'\-rr

-- ~ - - 4 - 4 0 0 4 4 4

.4 0

440

~
Inverse I ngollnmclnc h 111c1ton

P ru1CI J\ill \ 'ni11C.<:.


1f
1f
--< .y -<2

Si n- 1 x

1f

- -2

Tan- 1 x

1f

< )' <2

Co:;- .r

o::;y =::rr

co,- x

O< y <lf

Csc-l X

- ;r

Sec- x

- 1f ~)'

1f

1f

1f

1f

2'

<)" -< - - 0 < y <


- 2
2

< - -. o::; y <-

It would be reasonable 10 ask why principal values of Sec_ , x were not chosen as 0 ::; )' <
TT/ 2. rr/ 2 < y ::; ;r . Had they been chosen in this way, they would have been very similar
to those of Cos- x aod Cot- x . Likewise. wby are the principal val ues of esc - x not
- 1r/ 2 ~ y < 0 , 0 < y ~ n /2? The. answer is that lhey could have been selected in this
way. otnd some authors do indeed mtlke lhis ch oi ce~ it is si mply a mauer

or preference.

F~ch

choice docs, however. c reate corresponding changes in later work. Specifically. in thccxcrci~cs
of Section 3.10, if principal values of Sec- .~ and
x arc selected in this altcrnati\'ll way,
then derivat i\es o f these func tions are modified corres:pondingly.
In the remainder o f thjs secLion we sol \'e problems that make use of lrwe1se trigooome tric

esc-'

functioos.

I EXAMPLE 1.41
Find a ll solutions of the following equations :
(a) sin x = 0 .4

(b) 3 cos 2x

-0.21

(c) S tan (3x- I)= 4

(a) One: sol ution of this eq~tation is

.< = arcsin(OA) = 0.412 radians (to lh~ dectml pi"'-'C..').


A '-CeOfld solution i, 1r - O.-l12. and u1:ten we 41dd multipl::.oo: of ltr, "C obtain thr:
complccc set of SOltcion<

r.

Sitpfi&N

2mr

~t.ltk.... vf ~(M~ '-l

+ 0AI2,

2mr

+ (n

- OAI ~).

l"f&Ot\llfl"'"..: cqq-.,.

"here 11 is an inceger Folio.,. in& lhe lead of Example l.lti, Figure 1.106 SUUC>b

tiUit thi set of numbers can be represented more compocll) LIS an onllilll l\ltation of
;r /2. plus or minus n / 2 - 0.412 = 1.159. and I'Q&'iblc ooollipl'"' of 2n : lhll is.

n 1.159 + 2n;r

~o+l)
= (-- 1f
2

1.1 59.

Often in problem like Lhi<, x = = in(0.4) 0.~12 i the only oluli"n gtvon.
We can see the reasoning behind this <Ot>eh.I.'Sion. There is only one prinoipall'aluoof
the inver~rtC sine fmx.:tion. anc.l therefore oncsolutio.1to thee<auation. Dlu thecqu;uion
10in .\' = 0.4 SU)~ 11\lthing abv1.1t the inverse sine fUilC:ti()l\, We ha\.'C inuoduc:Ct.J it
sirnpl y ali a convenietK"C. \Vhcn we obtain the solution 0.41 2. we must ad: wt1ethct

there arc other solutions to the original equation. For the cquution ~in ,x 0.4, there
are o ther solutions. In some problems. the princi pal value i the unly ucrer>tablc
~oluti on. Rf:ntcmbcr. then~ if \VC intnxhu:c an inver~~ tligonurncuic runction intu ~
problem along wiUt its COITcsponding principal \<3lues. wt nt\ISl nsk ''-ht ther thete
are other possibilities be~ides the principal \alucs.

(h) Si11ce cos 2x = - 0.07. one solutinn of this equation I'm 2x is


2x

= Cos- ( - 0.07) = 1.6409 radians

(to fou r d:in'k11 plncc).

Rut there arc uth.:"'. Si11ce the cosine function is cwn. all sulution, lU't llti\'en by

2x =

I.M09 -

2n;r

(\~ohere 1J

is an intcg"~r)

Consequently.

x=
(c)

:1.6409 + 2nn
= m r 0.820 (to three dedm.il pl.oc.:s).
2

s1nce tan (3.~ 3.t - 1

1)

= O.S. one w lutiun of this equation for 3.t -

= Tan - (0.8) = 0.6747 radians

I is

(to four deconll~ plt..:e>l.

Since the wngent function is 11 pcrio<lk, all solutions can be t((>rcss<d in the fonn
Jx - I = 0.6747 + mr

{where 11

i~ un integer)

Conequently.

x=

1 + 0.6747
3

+ nn

= 0.558 +

nn

(to three decimull>lace>).

70

Cbap1.cr I Cakulus P1-ep:u-:uie>n

I EXAMPLE 1.42
Find all solutions of the equmion
cos2 x

+ 3cosx

= 2.

SOLUTION The quadratic fonnula appl ied t<> the equation


(cosx)2

=0

3(cosx) - 2

yields

cosx

-3 .JI'i
2

Since cosx can olly take on values i n che intel'val - I ::: cos.x < I , the pos:sibility that

cos x = (-3 - ./f'i) /2 must be rejected, leaving


cosx
From the inverse cosine solution x
solutions

= - 3 +2 .JT'i

= Cos- ' [{Ji7- 3}/2) I = 0.')75 radians. we obwin all


2111T

0.975,

where n is an integer.

I EXAMPLE 1 .43
Find al l tjmcs when the l11ilSS in Example 1.36 is 2.5 centimetres to the right of its equiJ ibrium

position.

= 6sin[2(t + IT/6)) , it is

SOLUTION Since the position of the moss is given by x(l)


2.5 centimetres to the right of its equilibrium position when
2 .5 = 6 sin [2(1

One solution of this equation for 2(1


of the equation are given by
2

(r + i) =

+ n/6)) .

+ n /6) is Sin- 1 (2 .5/ 6) = Sin_ , (5/ 12} . /\ll solutions


~ [ ~ - Sin - 1 (5/ 12)] + 2mr

where n is an integer. \Ve can no'"" solve this equation for I:

IT

= IT4

I+ -

[n4 - 2~Sin- 1 (5/12)]

+fill'

=~
[~ - ~Sin- 1 (5/12)] + I!IT
L2
4
2

= ('

fll: I)

l'l'

0.571

Since 1 must be positive, acceptable :solutiols are

t=

l(

l2n+ l)

rr+0.57 1,
12
1211 + I ) rr - 0.57 1,
(
12

where 11

:::_

where 11 > 0.

I EXAMPLE 1.44
Find aU solutions of Lhe equation
~a n (cosx)

DI!I!Jajii(Bl!iA S imp liftl

= -

./3
= 1/ ./3. a nd one solution of this equotiot is

SOI.UTIO" rf we set y = cosx . 1hen tan y


y Tan - ' ( 1/ ./3) JC/6 . But there are many other solutions for .l'. namely

c.qu;~lion

Jr

y =

6 + ""

w here u is an i nreger. But )'


CO.'U: . atd cos.t mu~l mkc on values in the iuei'Val - I S
cosx 5 I. There is only one possibility for n. namely 11 = 0 : hence.

1.02

rr

oosx = -.
6

X
1.()2

From the sol ution

x=

Cos -

(rr/6)

= 1.02 and Figure 1.107. we obtai ll

x = 1.02 + 2mr.
where 11. is a n integer. as the compJele set of soluli(lns .

..-..

I EXAMPL E

1 .45

---

Find cotlS!aniS A > 0 and 0 < </> < 1r so lha< lhe func tion /(x) = 3 coswx - 4 sin wx ,
where aJ > 0 is a constant. can be expressed in dtc form A sin (C!>X + </>) for all real x .

SOLUTION If we expand A sin (wx


equate it to f (x ). we have

A Isin wx cos <1>


Th.is equation will be valid for all

+ 1/J) by means of compound-ang le fom1ula 1.43a, and

+ cos wx sin <1> J =

3 cos wx

x if we can lind values of A

A cos = - 4

and

A sin</J

and

- 4 sin wx.
<1>

so that

= 3.

\Vhen we squflre ~nd add Lhe~eeq u.alio11s, 1he re...;,uh is

A2 cos2
Consequently, A

+ 1l~ sin~<P

= A 2 = (-4}~

+ (3) 2

= 25 .

= 5 , anJ
cos</> =

and

sin<l> =

3
s

The o nly angle in the rdnge 0 < rJ> < 11; satisfyi ng these equations is rJ>
arccos( - 4/5)
2.50 radians. Notice that a rcsin(3/5) docs not give this angle. T hus. /(x) c an be expressed in
the form

f(x ) = 5 sin(wx

+ 2.50) .

Perpendicularity and parallel i~m deal with lines Lh:u make~~ righLangle ~t their point of intersection or that make no angle since parallel lines do not intersect. Lines that intersect usually do
so at angles othenhan JC/2 radians. In order to detemline the angle at which two lines intersect.
we 6rst define the inclination of a line.
DEFINITION 1 . 6

The incllnatlon o f a llne I is the angle of rotation (0 :::;


x -direction to the line.

<1>

< 1r) from the positive

72

Oupter I C11ctliUS l'ropOr:ltion

Line y = x - 3 in Figure 1. 108 has inclination TC/ 4 radians, and line )' = -x
inclination 3Tr / 4 radians.

+ 4 has

lndinarions of lines " 'ilh slopes :!: I

FIGURE 1. 1

y=-.< + 4

y =x+3

When the slope m of a line is po.~iti\'C, as: shown in Figure l.l 09a, it is clear that </> mUSI
be in the interval 0 < </> < TC/2, and m and if> are rehued by tan </> = m . When tbe slope of I
is negative as in Figure 1.109b. we use idemil}' l.-14b to write

m = -tan(JT -) =

E' W
tion

o(

q;rn.

line v.irh

Inclina-

(ICKiti\'e ~It~

...

tun 1r

tan

I+ tanJTtan<P

= tan 1,6.

.Qiil moJii1'lU'II lndinarioo of li11e \\-i th neg_arn-r '>lope

Thu..'i, whenever the slope m of a line is defined, the inclination is related to m by the cquution
tan

= m.

(I

5\ll

In some sense 1his cqua1ion is uue C\'Cn when nr is n01 defined. This occurs for venical lines.

which have no slope. For a ven ical line, </> = JT / 2 nnd tan 4> is undefined. Thus, equation
1.59 is ahu valid for \'C::Ilical lines from th:;. puint or \'iew that ndtlu:r side of the c..-qu.ation is
defined. Notice !hat it is 1101 correct for us to write 4> = Tan- m, since the principal ,alues
of the imer:se tangent function ( -rr/ 2 < Tan_ , m < 1r / 2) do not coincide with the specified
values for inclination (0 ~ </> < JT ).

I EXAMPLE 1.46
What are the inclinations of the lines 2x - 3y

= 4 and 2x + 3y = 4?

SOLUTION f l'om y = 2x /3- 4/3. the slope of the lirst line is 2/3. The inclination of this
line is</> = Tan- (2/3) = 0 .588 l'adians. From y = - 2.</3 + 4/3, the slope of the second
line is - 2/3 . The inclination of this line is</> = 1t + Tan- 1 ( - 2/3} = 2.55 radians.

J.S The hm:r5( TrigoooQn)Ctric fu(ICli(luS

73

When 1wo Jines I 1 and 12 wi1h nonzero slopes m 1 and m 2 in1e.rsec1 (Figure 1.11 0), 1he angle 9
(0 < (:} < n ) be-tween the lines is given by the equation

or

w::na-

By applying the 1angent funcrion to both sides of rhjs equation ::~nd using identiry J.44b. we can
express 8 in tenus of 1n 1 and m 2.
Ansle

ran8=tan( 1 -cf>z)=

ron 1

ron 4>2

+tanq,,,an <i>l

This cqua1ion dcrcnnincs I} when <1> 1 > <1>2. When 1 <


lan 0

I,

m1

m2

lllillll

4>2, !he cqualion is replaced by

mz, - m 1

In both cases we may write


X

1an8 =

m,- m21
+ m 1m2

'

for the t1cute angle between the lines. It follows then lhat the acute angle between two lines

with s lopes m 1 and m z is

8 = Tan- 1

1m- m,- I.
1

I EXAMPLE

( 1.6Q)

+ m1m2

1 .47

Find Ihe angle between 1he lines 2.r - 3 y = 4 and x

+ 4y =

6.

SOLUTION Since slopes of these lines arc 2/3 and -I /4. it follows Ihal

8 = Tan- 1

2/3 _ (- 1/4) I
= 0.833 radians.
1I + (2/3)( - t/4)

EXERCISES 1.8

2J. 4sin2 x- 2cos2 x = 1

ln Exercises 1- 16 evaluate the. ~pression (if iLhas a value).

ii
ii

i:

il

I. Tan- 1 (- 1/3)

li 25.

2. Sin- 1 (1/4)

4.

5. Col _, (I)

6. Cos- ' (3/ 2)

7. Sin- ' (;r/2)

8. T,m- ' (- 1)

9. sin (Ton- 1

10. "" (su,- ~)

II. Sin

(lan (1 / 6)]

iii

12. Tan

13. sec [cos- 1 ( l/2))

14.

15. sio [su, - 1 (lJ/2J)

16.

21. J(x)

17. sinx = 1/3


19. cos2x = 1/3

1!1 21 . 2sin(l - x)

= 1.4

il
!il
!il

29. f(.t) = Sin

su,- [sin (3lT/4)]


su,- [cos (sec-' (- /2)))

30. j(.t ) =Sin-'

;r

(x - 3)

X+ Cs.;- 1 x

lfl EKcrd.scs 31-36 find the iodination of the line.

3t.

t~. 1anx = - 1.1


20. C(ll 4x +I= - 1.2
22. 31an3x + 2

= 2 + (esc- ' .t)'

28. f(.t) = /Tan- .t + /s,c-

(<in (1/6)]

=-

1.2

li

X - )"+

I= 0

33. 3x - 2y = I

35.

X=

=1

26. sin'x - 3<in.r - 5

In F.xercises 2?-~)0 draw a goph of 1he func1ion.

In Exercises 17-26 find all solutions of the cquaLion.

i
!!!

cc><'x - 3cou + I = 0

c..,-(- 2/ J3l

3. Sec -' (J3)

J3)

24. 4sin 2 x + 2cos2 x

il
i!

32.

.t

+ 2y = 3

34. y-3x= 4

36. y

=2

=0

fn E.'tcrci.~ 37-44 dclcnninc wf'lthcr lhc 'nu .uc p.:rpcndic.:ular, pur


atld. c r neither. In ahc lnt c:.u~ de:mninc 1hc an&k beLween th\: li~

.17. )'"' -x + ~.) -x+ 6


J8. X+ 3)' = 4, 2X + 6y 7
.19. X 3)' + 4, )' .t/3- 2
~o. lx + Jy
1. 3x - 2)'
5
41 . )' J. + 2. .)'- -.t/2 + I

l
i

42. X - )'
~J.

lincsohuhen:<ullins<o&"'li<.\(1) /(t) + g(t)~h(t). E>pres<


,\(1) in dlC form A :.in (WI ~ ..) for pprcpriOic valu.:< or A > 0 >lld
-JT < J < !f

t. 6-.a~

A ~r:an L of leni1h R with slider C h roUUJng C!O<J..WtSC :about 0


us 6fl0Wn belcM. Tile )lidcr mo'~ in a ~lotlcJ k,.-\'\.'1' hin,cd ut .i\ al a

disl<lllC:C /, from 0. l'md .ln$lc 0 fuo>Ction or ~I<"'.

= 5. 2X .o. J)' = 4

Slid<r

x :O,y =S

44 ..f

+ )' + 2 -

0. :\t - ! - ~

45. If .P i< 1hc antic fonn.:d b) A B and ;\ 0 in 1he fog..re llclo-,, find
1/> u func1i~n I)( 0 .

l.

I
6..'. Th:: :tnglc <lf clcv.uiun o( th.c t<>s> eM ato....,tr from A 1~<. t/1. al\d
the W1jilc from IJ " distsnr:c d from A is 0 (fi~urc below). Find a
forn1la for 8 in terms ol tJ.

Jl

-1

'

,,I

In E:Acn:iM::o: 46-5 I hnd all solutions of" the CGOOllOn.

"'' )inxtan 1 x - 3 +tan1 x - 3~in.t=O


47. sin.t + cos .\ - I
43. sec (sin.t)

=- J2

so.

= -.fi

il

! (l:m .r)

Sl. cos- (Htn (.r2 + 4)1

49. <'OII(Sin

= lrr -

'x) =1/2

* 66.

Sl. Drwtoupl\'>ufO>efollowin~rur.ctions: Ca)f(x )


tb) /(,t) = Sin - (<inx)

=sin(Sin .<):

SJ. IXaw&IUf~"''fthcfoiiOW>ngfur.ctions: (a)f(x) =~<(Cos 1x):


tb) f(x) co, 1 (""'.r) .

i! t

54. litpreos 1hc function /(x) - 4sin 2t + cos 2t in 1he form


R sin (2..1' + !/>). whcRO R > 0 and 0 < t; < :r.

!!

~S. l;tprtIIIC runc1ion /(.<) = -2sin3x + 4cos3x in the fom1


R cos(3x I ). v.~'tC(c R > OandO < < rr .

il
ilt
il

~6. Rcpcl Excox:i,;c 54 for f(x)

57. Rcpcoll:lxerci..e SS f"" [(.)

= - 4!-in:\t + 5eos3..

Ri.!JX:~U Excrci.;c 58 but ex pre~~ ,\'(1) in lhc forrn A co~ (WI

+ ,P).

Two electric ;ignal> /(1) = 2;in (wt + 4) and g(t )


(au+ I) an.: (<..'d into the ~me line <\Q that the rc~ulting signal
i<X(l) = /(t) + g(t). llxprcs. .r(t) in the form A <'OS (wt + ~) fot
appropri:uc ''alucs of A > 0 and -it < 9 < ir .
3~in

6 1. Repeal Exerci: 60 b<u <.<press x(l) in the fO<m Jl sin (wt

+ 1/>).

62. Thn:e electric signals [(I)


$><(wt + 3rr/2), g(l )
4 co< (wt + rr/3). and 11(1) 2 sin (wt + tr/4) arcrcd in1o 1hc sante

=0(1) =Ooeoswt + wl.


~:<in wl,

1 ?: 0.

where w
l9.ii'Tl. pro' idcd 11~u an)' rcbiswncc dt.~e to the air is
ncsiIO<l. Show lh:lt O(t) ... be C>(lfeSS<d '" the form

6(1) =

is.r(l) ~ j'(t) + 8 (1). Expross X(l) in thcfonn ;\ sin (M + <1) fot


appr01>rilue vaiU(Sol' A > 0 and - rr < 4J < :r .

i
i

-loin 2.t + 4 cos 2..

58. Twn electric signls /(1) = 4cos(wl + 21r/3) and g(l ) =


3 sin (uu + tr / 3) t1rc fed into tile s:.unc hnc so lh:ll the rC1!ulting 1ign~d

iii+ 59.
i 60.

A pend11.tum~'On~istsofu n~5Stn sus~ndcd fronlu slrin of lcr1gth


l. tfigur llclow). At tin>e 1 = 0. the nms is 1XJ1!C'd throu2}> o >onll
tUJj;lc 0.,- 10 the righ1 """" 0., > 0. >nd to the !eft when Oo < 0
- nnd gi"cn 1n inilhtl spc:td ''o to the risht. liS subscq\.'<:nl n&ubr
displ.nc.:cmenl-.. t.ru jh\:n by

ej

.,;

+ --;--;sin
(wl + ~).
w-t-

67. WhJi1 dwnt<''- I' any. mull be nu.L:.

11'1

t-:-utcisc 66 if lt< tntltal

ipcc..t V(l u: to dll.! ten nnhcr thu:n ttlc n_Qhtl


~H.. A mU>s m is suspended fmm 11; ltol)n n& wllh cur~aru /.. Uijure
hi:low). 1r Ill time I = 0, lhC 11lli~ I~ J.i\'Cn an inilia! c.lisp!ocemcnl )h
and Ill upw"'rd ::ifX.c l t.\:1 iL.; sub~cqucnl dt~pi~CR)C:flli ~ giwn hy

.,

.' .Y(l)- \'~a"b.il + -..., "nw1.


-

"htrr

01 -

7J . Show lhul

1 !:' 0.

("~ I .t

./1iTii. pnwukd m. any rc.ow~"C dut 10 the a.:ir i-co

r.::allal. S""-' tl>.lt y(l J <"" t.: <>p""""J 1t1 the fonn

I.

~ oon (WI + ) .
Jo T w
~

V(l) ;

\\heu. 4> -1in

74. Show thai

S in

(W)'o/t\)).

15.

; { -cos-' Ji"'=":iT. - :s .t < 0.


Cos- 1 .fi'Xl.
O:>.r~ l.

PIU\\! lhoil

.r

- 1.

> 0.

< 0.

(;9, Wh..u chung;c.~. ;r :my. ntu~t be m.1do u\ lx<:n.:ig.t 68 tf the i nitul


tiiW"e1l ltfl ill downw:trd rather' th;a~\ UI)WIII\ 1?
76. PfO\'C dlal

70. An inducl.l.ll<X L. rQ,httuV..'\: R, and capxiuncc. C arc <.'(!Ooo


necttd tn ~YICS Wllh

_Cnct;at()r J'f\)Joci !1g ;11"1 ~lllatof'y \'OlbSC

1: - l:,cosw1, 1 ?: 0 (fi~Lro be'""') II I . C. R. 0 > 0 , '-I


(II > 0 ue all (()n).U;11s~ the sk!JJ)nt.tc <.WT'Cat I nl the circuit i~
I

_I)'

wC

[Rcoswt + (wr- .,c);nwr].

77. Verify lhal

~'ii)I'CSf

Sec

I in tile fonn

= {ron- J\T=I.
- 'T

II'"' lx

+ T"" Jii"'==.
1

X~

1,

$ - 1

.(

!S - 1.

f.),

"beoc A > 0 lftd - ;rfl ~ ~ ~ :r/2.

7J. Vcnfy lhll

j Cot- .;:;-r=-1,

1 -:~ + c.,- ~-

c
R
i

71. It;, ln<>Wn !hot Soo<wl = A ro< (WI - tr/6) + Sro< (WI + ~)
for 1111. v.hc:rc w i( a fhcd C"M"iitlnt. find. c\acdy, all pck-(ibte valuei
f<>< A > 0 and.

79. Vcriry that if 0

:5 .x < I. then

I.

11.9 Exponential and Logarithm Review


Here is the diagnostic test for this section. Give yourself 30 minutes to do it.

DIAGNOSTIC TEST FOR SECTI ON 1.9

1. Evalumc the following quantities: (a) log2 32 (b) log, ( 1/27)


(c) 10,u-,111 u (d) <"l ..

In <)ueslions 5-8 draw 3 graph of lhe function.


5. j'(x)

= ul'

6. f(x)

= log 10 (x - I)

7. / (x)

= 4-

8. / (X)

= I"'+ ,.-

In questions 2~ find all solutions of the equation.

2. log,. 3x = 2
4. ln.t - ln(.t- I)= I

9. I( )'

= 4et.-t , fi nd X in 1crms o ( J .

In clcmcmary algebra we lcamcd the basic rules for products and quotients of powers:

a 11 cl' = ,,h+<-;

(1.6 1a)

(I~
_
_ (lb-c ,

a"-

(1.61b)

( 1.6 1c )

where

a>

0. and band care real consta ms. These rules are used 10 develop the exponential

function

.f(x )

= a' .

( 1.62)

that is) t l raised to the exponent x for variable x .


We CQtu.::entmlc on the case when a > I. The meaning of ax when ~t is an integer is clear,
and wheo .x = 1/ n , where n > 0 is an integer. a ... is the ,e~~ root of tl . \Vhen x is a
posilivc nuional number 11/ 111 ( 11 and m posilive imegcrs). 1.6 1c implies that

a,,,,

or
that is. tln/ m is the m th root of t1 to the integer J>owcr 11, or a 11/m is the integer power 11 of t he
mlh root of a . \Vhcn x is a ncgmi\'e rational, we wrim a x
Ifa-x . \\.'here - x is posili\rc .
These resu hs lead 10 lhe points and lhc g.oph of y = a' in Figw'C 1.1 I I.

M414ii;I4MIIM

1'he c-,;poncntiul f11Jlc1ion

)'

- 3 -2 - J

There is a d illlcully with the definition of ax and its graph in Figure 1. 11J. We are not
prepared to resolve the problem now, but we would be remiss in not pointi ng it out. How do

we define ax when .-\" is an irrational number~ For in.stan<.:c. what is Lhc ''aluc of a ./i or a"?
If a' is uodefioed whenever .r is irrational, tbe graph u1Figure !. Ill is ro.isleadiflg. Altbough
we havej oined the poinls with thesmoolhest possible curve, there is actually an infinile n1.1 mber

of values of x (the irrationnl numbers) at which there is, as yet, no dot on the curve. We
require UmiTS from Chapter 2 to deal with this problem. and we therefore se1 it aside umil

Se<:tion 2.4.

I EXAMPLE

1.48

Plm graphs of the exponen1ial func1ions 2T am.l y: on I he same axes.

SOLL'TION Graphs of these functions arc shown in Figure 1.1 12. Notice that both curves
pass through the point (0, 1). More generally, a 0 = I for any fl. When x > 0, the graph of
Y is big her than that of 2x. whereas the oppo~ite is true for x < 0.
FIGURE 1.11

Two

e~ponenli al

ftmttior1s

y = 3'

I EXAMPLE 1.49
Draw graphs of the functions x 4 (a power function) ,.,..1 2' (an exponential function) on the
same axes.

SOLUTION Graphs of these functiOM are shown in Figure 1. 113. but no auempl has been
made to usc a scale on eilhcrthe X or y-axis. Notice here that for x > 16, 21 > x. This is
always rhe situation for power and exponential functions. G iven any exponential function tl.x

(a > l ). and any power function x" (11 > I). there always exists a value of x . say X. sucb
that when x > X , \ve have. ax > .t 11 rn shon~ exponential functions grow more rapidly than
power functions fur large value. of X .

16

78

Chnp1cr I Cak:uhl\ PttJW"alion

In t.enns of the exponential function, rules 1.61 take the fornt

a:"'' a... -: =
0

x,

a ...:

tr'+.r!,

( 1.63<1)

= a'-.:r-: .

( 1.631!)

(ax'}'' =

( I.63c)

tr'X:.

The cxponentiul functiou i~ onc- toOilt; it~ graph passes the horizontaJ line lest 1\ s a result,
it has an inverse; we call it the logarithm function to base ll. de.noted by 1Qg6 ~,. As em inverse.
the logarithm fllllCtion reverses the action of the exponential function. Por il'lstance, working
wit h base 10 . since I(}' = 1000. it follows thai log 10 1000 = 3: si nce 10- 2 = 0 .01. we
have log 10 0.0I = -2. We eM work with logarithms 10 any base. but il is customary 10 use
bases greater than I . \ Vilh base. 3. say. we can write thut logJ 81
4 since 3.- = 81. and
that log, (I / 9) = - 2 since 1
1/ 9 . TIICSC examples dcnl<.ln>Jr!IIC lh~t lhe logarithm of a
number is a power. To say lhal y is the logarithm o f .r 10 base a is 10 S!\)' that x
a>'. In
gcncrul. 1hc logarithm of .x 1<1 bflse o, is 1he power to which il must b( miud in order to prQIIuce
x. Algebraically, we write this as

r =

y = log.

o nly if

,\ ' = (I

)'

'

(1 .6-l)

When the fi rSl of lhcsc is subsl illlled into the SC<.'OIId, the I'CSlllJ is
a'"'&.~ =

x.

( 1.65a)

= .r.

(I 6Sh)

\Vhen the second is ~ubstinned into the fi~t.

or sim~:c y is arbitn try. we may replace it with

.x.

log. a'

the ti ~1 i$ true only for x > 0. The~ equ<Ui<ms


simply c;-.;prcss 1he fact thai exponential and logarithm functions arc inverses uf each other~ d {l
one, then the other, and you are back where you started (see equations 1.35 and 1.36).
A graph o f the logarithm fuo1<.1ion [(x ) = log,, x can be obtained by rcil(l(.1 ing lhc graph
of y = a' in ~1c line y
x <Figure 1.11 4). h passes through the poi nt ( I, 0) for My a. In
other words, log, I = 0 . As values o f x gel closer and closer to 0 , their logarithms become
very large negative numbe rs. We cannot take the logarithms of0 or negative numbers.

Tilt second of the.-.e i:-; valid for all

~ . btll

FIGURE 1 . 114

U.~th nl

(ul1(1ion

)'

1.9 Expooenti:.l :tnd Log:Mithm Reviev.

79

.
I I 63 for lhe exponenlial fullcliOil are lhe followillg rules for logaCorresi>Oildong 10 ru es
rithms:

log (XJXl )
0

lo~

= log, X 1 + log" X2,

(X) = log.x1- lo&,Xz,


.o

( 1.66a)
( 1.66b)

X2

(1.66c)

log0 (xi" ) ==

X2 Do&, X t

To prove 1.66a, say, we set Z = log, (x JX2) ' in '"hich case

(using 1.65a)
(using 1.63a)

log, XJ

log0 x 2 =

= 1o~ 0

(XJX2)

We leave proofs of 1.66b and 1.66c 10 the exercises.

I EXAMP LE

1.60

Simpl ify the following expres.sions:

SOLlill ON
(a)

ldenlit)' 1.65a implies that

(b) Since - 4 intervenes belween the logarithm and exponential operalion.s. we cannot
use L.65a immediately. The - 4 can be rclocmcd, however, with l .66c:
10- ~10'( = 1011:-aao<x

> =

x'

{ I' t' X> 0) .

(c) ldenlity 1.65b gives

(d) Since log 2 8

= 3 and log3 ( l/27) =


log2 8

-3 ,

+ log3(1/27)

= 3 - 3 = 0.

80

Ch3:1Jicr I C:..li:ultt;: Prcpal":l!i()n

I EXAMPLE 1 .51
Solve the t'<>llowing equations:
(a) logsx = - 3 (b) 10g 10 x

+ log 1 ~(.r + I )

= 0 (c) 10' - 12 + 10- c

SOLUTION
(a) By n~rans of equation 1.64,

.r =
(b) Since log 10 .r

s-l =

125

+ logt0 (.r + I) = log10(.r(x + 1)]. we can write


0 = log 10 (x(.r + 1)1.

If we now take expc"WlCntials to hase I0~


or

x(x

+ 1) .

This quadratic equation has solutions

x =

-I .f i"TI

- I ../5

Since x must be pcr.-;i tive (the original equation demands this). the only solurion i.s
(../5 - 1)/2.
(c) If we multiply the equation by I0' , the result is
X =

= tif' -

12(10')

+I=

(10') 2

12(10')

+ l.

Btu this is a quadnltic equBtion in JOx. so thai

10' =

12 j l-44 - 4
2

= 6 ./35.

Finally. we have

I EXAMPLE

1.52

Tile cnr hears by detecting pressure variations of impinging sound waves. 'T'he loudness of
the sound is related 10 the inums.iay of 1he sound wcwe, which is measured in wans per square
metre (energy cransmiucd by the sound wa,c ~r uni t time per unit area). The lowest intensity
detectable by the car is nomtnlly taken as /0 = JO- Il W/m2 m a frequency of 1000 Hz; it is
called the twdible .<mulll th reshald. By oom;parison. the imcnsity of sound from a jet engine
i~ nbout 101 Wim2; it is I0 16 1imcs 1ha1 of the audible sound threshold. Bccnu.<c the r,utgc of
intensitie.s towhich thccaris sensitive is so large. d(;alingdirectly with intensities iscumtx-rsome.
Logarilhms provide a way to reduce this enormous mnge to a manageable size. As a number
increases by a factor of 10. its logari thm increases by I. Fore.xample, lhe differellce betweet the
logarithms of 10 and 100 is 2- I
I. If a number increases by a factor of IO" . its logarithm
increases by 14. 111is mngc is dccmc.d to be a little too compact: it is expanded by a factor of
10 in the following definition. The loudncs.s of a sound is said to beL decibels if

L = 101og 10

CJ

where I is the intensity o f Lhe sound and /o is the intensity of w und at Lhe audible threshold.
Use th is definition to answer the following questions.

(a) What i~ the loudnes.s o f sound at the audible sound Llweshold?

(b) Express the intensity I of a sowt~ i ntcrms of / 0 and its decibel reading L .
(c) If decibel readings fora voice. a car. and a jet e ng ine are 70, 100, and 160, respectively.
what are lhe corre-sponding intensities of the sound waves relati"e to l o?
(d) If the pnin threshold for sound has a n intensity 10" rimes /Q. what is its decibel
rcilding?
(c) If the i ntensity / 1 of one sound i< 10 time.~ the intensity / 1 of a sec011d s<>und, h()\v
do their decibel readings compare'?

SOLt:110N

(a) The decibel reading for the audible sound threshold is L


10 log 10 Uu/ fo) = 0.
(b) When we mkc both side as CXJXtnenrs of powers of 10. and use propcnies 1.~ and
1.65a. we <>blain

(!._)'
"
fu

If we rake IO'" roots o f both sides. we have


or
(c) Since the decibel level of the norm>1l voice is i O, its intensity is

I = /0 JOmtro = 107 / 0 .
Similar1y, the intensities of;~ car ;.Hl.d a jel arc 10 10 and 10 16 tirnes fo.
(d) For'"' intensity of I 0 11 !0 , the decibel reading is
L = 10 log 10 (10 1' ') = Hl( 14)
(c) I f

= 140.

L 1 and L2 are the decibel readin gs for sounds with intensities 11 and
L 1 = 101og 10 (/ ,/fu)

/ 1 theo>

Lz = I0 log,o(/z/ /o).

\Vhcn we subtrdCt these readings, we obcain


L1

L 2 = 10log 10 ( l ,f lo)- 10log10 (/z/ fo)

IO(Iog 10 (1,(/o) - log 10 (/z/ lo)J

lo)
-/= 10log10 C'
lz/ lo

= 10log10

(using 1.66bl

CJ

/2)
---;.;
= 10 log10 ('U
= 10 log10 10
= 10.

(since J, = IOlz)

Thus. when the intensity of one sound is I 0 times that of another. their decibel readings
differ by 10.

11 is somcumes ncccsl><!ry 1ochange from one base oflogarithnt> 10 another. lr\\e take logarithms
to base b on boch ~ide, of identity 1.65a, we obtain immedia1cly

11.61

(I+ -)"

2.000000

.l

2.370370

2.4SS320

10

2.S93 7~2

100

2.704814

100)

2.716924

10000

2.718146

10000)

2.7 18 255

1000000

2.7 18282

This equation defines Iogb a ""the conversion ftLotor from logarithms 1o base a to logariLiuns
to base b.
Before 1he discovery of calculus. 1he lxl<e of logarithm~ wa1 invariahly chosen 10 be 10.
Such lo~arilhnt> are called commun logarirhms: the) corrc>pond 10 1he exponential fw1c1ion
Jo-t. AnO(hcr busc for exponential$ and logarithms. however, thut i.s much nlOrc: corwcnicnt in
most applicntiOtiS is n number, denoted by the lcuer ~. und defi1Cd in u vuricty of ways. One
way is to con~ider the numbers

( 1.68)

for e\crincre.soing value of 11 . The numbcr.s in Table lA an: .rcadny inc~ing but getting
closer togclhcr. The) sugg.,.tlhat for larger and larger >aluc. of 11 , the function (I + l/ 11)" is
indeed getting ciO>Cr to wmc number - to 12 decimal places lhL< number ;.,
('"" 2.718281828459.

This number I? is irrntional, with a nontcnninating. nonrcpcating decimal expansion. \Vhy

this number is so convenient as a bn..~ for logarithms and exponcnlials is shown in Section 3.1 1.
For now. Ict us rewri te some of the more imponont fom>ulos of this section with a SCI equnlto
1!. The CJ<ponomiol function to base e is~. and equations 1.63 in terms of<'' read

.... ,.,= r ": .

,...,..,

(e"')'' =

t l ~~a l

(",( - ~ .

( J.69b)

("'1l! .

J.69e.

Logarithms to base t' arc usually given the notation In x rJihcr than log. x . nnd are called
naturallogariJJuus:
( 1.70)

In x = log,.x.
In terms of In x. rules 1.66 ore

r, + lnxz.

ln (x 1x2l = ln ..

(Xt) =

In -

In X t

In .t } .

(1.7 1a)
(1.71 b)

.lz: ~

( J.71c)

ldcntitic.' 1.65 bc<onoc


X= <'JArt ,

= In (e").

X >

0,
( 1.72b)

Graphs ore' and In x in Figures 1.115 and 1.116 have the same shape as ~hose or ax in
Figure 1. 111 mld log(tx in Figure 1.114. On ly lhe ".steepness" oflhccurvc.s is affected by a
change of ba,.,.

1.9

lijiijii;IWHIL.

Grapl1 of

e..

lijiclii;IWMIIW

EXJ)Ooential and Lvgruidun Re\'iew

83

Ornph of tn x

)'

-====:::::::::._- - - - l - - - - -x
Neither of the functioos e~: nor In x is eveo or odd. but functioos derived from them may
be even or odd. This is illustrated i n the followi ng example.

I EXAMPLE

.........

1.53

ls the function f(x)

SOLUTION

e -ax

1
,

where a

> 0 is a constant., even or odd? Draw its graph.

Since

.f(-x)

= e-l- x)' = e- ' =

.f(x),

the function e-x' is e1en. Graphs for a = I, 2, and 3 are ploued in Figures 1. 117:H:. The
value of a controls the spread of the curve. For any value of a. the graph passes through the
point (0, I ) and is asymptotic to the x -axis ( Figure 1.1 17d). This curve is very imponant in
statistics . ll is c.alled the bell CUI11t: or normal distribution.
FIGURE 1.117a

FIGURE 1.117b
)'

0.8
0.6

0.4
0.2

-2

-I

-2

FIGURE 1.117c

Graph of ,-:b~

- I

FIGURE 1.117d

-2

- I

You wiJI be makj ng graphs of fu nctions at every turn in lhis book. We rcmjnd you that when
we ask you to plot a grapll. you a re to use a gmphlog calculator or computer. When we ask you
10 dr.tw a graph, you arc to do so \\'ithou 1hcse devices. In the preceding exan1ple, we ploucd
graphs of f(x) = e -a"' for a = I, 2 , and 3, and then drew

y = e-""'

by hand.

I EXAMPLE 1.54
Draw a graph of the funct ion

(x)

In (x

+ a) , where a

> 0 is a constant.

SOI.LTION By trans lating the graph of f(x ) = ln.x in Figure 1 11 8a to the lcrt by a unit,,
we obtain the graph o f .f(x) = In (x +a ) in Figure 1.1 18b.

FIGURE 1 11811

Gmph of In .r

FIGURE 1 118b

Graph of ln .r 1r.1nsltued n 1.mits to the ldt

) = lnx
In u

-ll,

I EXAMPLE

1 -fl

.._..

1. 6 5

The emf device in the R C -circui t of Figure 1 . 119 produces a conStant voltage of V volts. If
the. switch is closed a l Lime I = 0, and there. is no initial c harge on the capac itor, the c harge.
thereafter is given by

Q(t) = CV [I -

e-1 /CRCl],

I :::_ 0.

Draw a gn:~pl1 or Ihis runction.


IOilclll.l;;j IHIQ
for RC~ ircu i l

Schcmaltc

SOLUTION A graph of e'f( IIC) (Figure 1.120a) has the same shape as tha t in Figure 1.115;
the constant R C affects only the s teepness of the curve. The graph of e - I /(IIC ) in Figure 1.1 20b
is that of Figure 1.120a reHcctcd in the vertical a.<is. The graph of Q(r) in Figure 1.120c is
rhen obta ined hy turning Figure 1.1 20b up~ id.e down. s hifti ng il upward I uni t, changing the
scale o n the \'ert.i cal axis by a factor C V, and rctai nio1g o nly that part of the curve f :::_ 0 . It is
asymptotic to the line Q = C V .

R
ljldii;IWMEJ.O.

Craph of
)'

t~ r t ttc

FIGURE 1.120b

)'

y=

e""~I (RC,

-----I

1.9

M;tlrlii;I#MfiQ

f.'<pontnri:ll :.nd Log:withm Reviev:

85

<.:haflC on c apadtol'

cv
Q; CV [l - e-<I (RC!]

EXERCISES 1 .9

In Exercises 1- 13 find :lll V'J.Iucs of x s :ttisfying the cqU:;tt(on.


I. log, 0 (2

2.
3.

di\'idc tbc N people into groups of x people, pooJ their blood, :md test
for the virus . If tbc blood is disca..~free for a group, the: in<lividuals
need not be tcsH:d separately. If the blood lest is po~itive. c.tch o( the
X people in the group is tes.led sepamtely. 1l is shown in ~obabi lily
th.c ory th~l when the group s i-ze is x, lhcn on the avcr.1gc, lh~ e."tpcctcd
Iota! number of tests needed to test the N people oomplctcly is

+ x ) =-I

10', = 5
log10 (x 1 + 2.( + I) =

4. In(,<'+ 2.t + 10)

= I

s. los-' ~ too
6. to-' = 100
7. log"' (x- 3) + log., x = l
8. log 11, (3- x) + log., x = 1

Plot this function for N

9. log"' lx(x - 3)1 ~ I


10. 21og10 x + log 10 (x - I)= 2

+ 29. Showthutf(x) =( I

* j(),

+ II. los,.< + l~(x + 2)=2


l2. log., (.t{.t + 2)) = 2

[;;

31 .

(a)

16. f(.r) =

18.
* 20.
*

f(x)

log. lx ' -

= log10 (4x)

/(<) = In (I - x')

22. j(x)

= IO'+'

II

15. f(x ) = ln(coox)

17.

19.

* 2 1.

= a'''"" 'Tll
/(x ) = ln( l -x)
j(x) = ln(x -l)

23. /(.t )

24. j(x) = e''

* lS.

+ 26. /(.<) = ~-.r C()S-(

27. /(.<)

i *

of timber ::tfiCr l )'Cal'\\,

33. lf the cfl'ec.ivc height o r the carths ntnospherc (ir. mctn:s) i:. the
so ltllion of the C(tuation

10-6

- l~.r:=:t

28. A large nu mber N of people arc to have their blood tested to


determine whether tJtey have been infected by a virus. One way is to
test alJ N people individualJy. resuJtin g in N tests. An aJternathc is to

2 1og,, .<'1

yean;:?

e-' sin .<

= x 1c'.

+ 32. A new car eoslS $20 C):)(). In o.ny year it dcprecintcs to 75~ or its
val u~,; atth~o: bc2 innin, of that ycal'. What b the value of the car aflcr 1

= _,_,

/(.< ) =

+c- '' ' ) is an odd function.

(h) f<low long docs h take fC'Ifcimbcrofchis type to double?

/(.<)

. - '"')/(!

lntltc early pcri<.1<l ofrcfor~uu ion. tiM: pcrccmaee i ncf!.::s~


of timber C3ch year is almost con4;t3nt. If the origin:.l
am<>um Ao of ::a. <.~rtai n timber incrc(kSCs 3.5% the first yc:tr
and 3.5% each year thcrc.a.llcr. lind an c~prc~iot\ for the
Ul!1t)UIH

In fu.crdscs 14-27 tlraw a gmph of the (unction. As a i:hcck. usc 1.t


calculator orcOmJXIICr 10 plot a graph
14. f(x) = .-'

I00 to determine the grou.p size that mini

lht:rc a difTcrencc l)ciWl.'Cil lhC Jnlph.s Of Ihe functiOtl<o f(;c)

log.(.<') unci g(x)

13. log 10 [1o1J 1 o ('~) +4)= -1

mites)' .

= (I -

2.08

10- 6 y)'6

find y .

34. Show dlaL if y is tJ\C logaJ'ilh ln


logarilhm of x to base 1/" .

or X

LO base " . tJ\cn - y is tJ\C

35. Prove 1.66b and 1.66c.

* 36.

Is idcnth y 1.66a valid for all x 1 and

x 2 '?

86

Chapter 1 C:1il.:ulw, PJcp;uation

{a) Tile lime constanL r lOr 1he c.irctJit is the len~th or time ror
the currenl to become iofe. Find r.

37. Tile ma~nilude of an earthquake is n"'leaSured in much the san-.e


W'd)' as noise le\'el. An eanhquake or minimal s ize is ~aten as having
value 0 on Lhe Richter scale. A ny 01.her earlf'k}uake of imensity I is
s.aid 10 have magniiUdc R on tile RiChter gcale if

R =

rb) Show t.hat if i is t.he current at any l ime 1. then t.he current
at time 1 + r is i { e.

Jos,. (f).
In Exercises 4 1-44 futd all values of

where / 0 is lhe inte.nsity o f the. minim:U e.arthquake being, used llS ref

crence.

.. J8.

3o2'"

42.

zx+ 4x :::: ax

+ 43. 3-H~

(a) If P f.loll a1'S is invested at it;Ocompoun(k:J n times per


year, show thai the accumulcued \laluc afler 1 yeai'S is

* 44.

(I + - . )"'
1

A = P

10011

+ 3a- 2:r = IO

* 4l.

(a) Express Lhc inlensity of an eanhqu:tke in tenns of / 0 and


its reading on lhe Ricb1cr scale.
(b) What arc rcudings on the Richter scale of can.hquak.cs tlml
have iotcnsit.ics 1.20 x lcf and 6.20 x 10" tintcs lo?

x sati:;fying the equal.ion.

= 7x- t

IOl:.v 2

= log2.\ 8

45. Show that /(.r) = In (x + Jx 2 + 1) is on odd function. Plot


il~.; grJph to confirm this geomctricaHy.

(b) How long docs it take U) double an imc.suncnt if intcreil is

46. Repair costs on ahe car in Exercise 32 are e~ti ma led at $50 the
tirst year, increasin by 20% each year thereafter. Sel up a functiol\
C(t) th3.1 reprcscnL'> the tl\'Crag.c ye:l.rly cost of r~pairS !lssociatcd with
owning 1bc cur for I years.

8% c.ompounded semiannual ly'!

(c) Calculate tllc maxlmum possible ' 'al uc of A if i is fixed but


the number of Limes tl:tat intc:cc.st is compounded is un.lim
il.ed: that is. calcu1al.e what happens to A as n geLs larger
and larger and large.r. T his method or calculating interest

is called contimumsly tcnrpounded in1errs1.

47. i\ straightwirc oonduclOl has length 2L and circulurcrossscclioo


of 1-adiu.s R. If the wi re carries curre.m i > 0 .. tllen 1he ma:nitude or
the vector potential m a dist:mce r from the centre of 1he wire is

(d) What is the accumulated Vitlue of a $ 1000 imesunent after

10 years: at

compounded cootinuously'/ Com(XIrc this


to tbe accumulated value if intcrcs.1is calculated only once.
6<.'~:.

c.ach year.
;.f;

lli [In (1+ -4Rl')

...!..

39. rr, in the <.:ii'Cllil shown below. Vo is the voltage iJCI'OSS the capacitor
at l ime 1 = 0 when the ~w i t ch i!': closed, the \'Qitage thereafl.cr is
V

4rr

= Vl)e -1/(RC).

/(r)

Jtoi

In
4tr

4L')

1+ -

r2

r']

1+ -

R2

'

> R.

where.: 11'> is ::t posit..i\'C constant

(a) Draw a graph of this function.


(b) Find lhc m liu>r >

ca) The l ime conswnt r ror lhe drcuil is the lenglh (lflimc fOr
the voltage on the capac-i tor lO bccotn.c V0 f e. Find t .

t + r is Vfe.

~ 40. If. i.n tln:-ciu.:uil~hown below. io i:s thecum:nl i_n the imlucwrat~i~ne.
t = Ow hen the switch iscl~"'"<"l, the current the.reafter is i = i 0e - Rrf L.

= /(0).

In Exercises 41:1-50 solve the given equation for J in tenns of x .

(b) SilO\"\' th at if V is lhe vohage at any time r . then t.he vollagc


Ill time

R for whid> /(r)

* 48. Y =
49.

r=

* so.

)'=

*
R

e'h - e-2.r

2
~'

+ e- r
2

r- e-v
e- + e- Y

A N SW ERS TO DI AGN OSTIC TE ST F OR SECTION 1. 9


I. (a) 5 (b) - 3 (<) 2x (d) 16 H marks)

7. (2. nmrk.s)

l . e2/3 (2 marks)

3. (ln4 - 1.)/2 <2 n~rl:)


4. / (t- I) (3 marl:s)
.\

5. <2 muks)
)

8. (3 nwks)

2
)

I:

'

'
9. (1 / 2)11

11.10

+ !n ()'/4) 1 (3 murks)

Hyperbolic Functions
TI'lis lll!lletiHI is like1y new for most students. II doc.s no1 therefore hme a diagno:~t ic t e.'\t
ussocimcd wi1h i1.
Certain (.'Ombinat ions of the exponential function occur so often in physical applications
thm they arc given special m1mes. Specificall y. half the ditt'erence of e' and e- A is defined
as the hype~bolic sine function and half their sum is the hyp erlx11ic cosin e fUJJction. TIese
runctions are denoted as follows:
sinhx =

co:-.h

( I 73)

According to equation 1.32. they are the odd and even pans of e'.
'The names of these hyperbolic functiOflS and thei r nomtions bear a striking resemblance
to those for the trig(')I'IQJnetric fm\Ctions. nnd there are reaSOI\ii for this. First, the hypertxll ic

functions sinh .< and wsh .r nrc related lo the curve x 1 - )'2 = I (sec Fi~ure 1.1 23). called Ihe
unir hyperbola, i n much thes.ame way as thetrig:onornel ric f uoct ions si n x aod cos x ar e related
to 1he uni1 circle x 2 + y 2 = 1. We will poim out one of 1hese similarities in Example 1.56.
Se<.'.<.>nd, for eac.h identity satisfied by the trigonometric fu nctions, Lhe re is a correspo nding
identity satisfied by the hyperbolic functions- not the same identity. but one ve ry similar. For

exam ple. using equaLions 1.73, we have

( cosh x)

(si nh x)

t!''

+2 e- '

)2 (e' -2e-x)2

= 1.
Thus the hyperbolic si ne and cosi ne funct ions satisfy the identity
2
2
cosh x - sinh x

= I.

( 1.74)

w hich i s remj niscent of the idenLity cos2 x


si n2 x = 1 for th e trigono metric functions.
Just as f our other Lrigonom eLric functions are deHned in Lerm s of si n x and cos x. four
correspondi ng hyperbo lic functions are dellned as follows:

e"' - e-x

sinh x

= cosh x = e-" + e- x '

tanh x

= coshx = e'

sech x

cothx

+e

csch x

.\' '

cosh x

e"' + e-x
e- x '

= sinhx = e-" = sinhx = ex -

( 1.75)

2
e -.~:

"lbese del\nitions and 1.74 i mmediately i mply thm


I -

tanh 2 x

= sech 2 x,

( I. 76a)

= csch 2x .

(1.76b)

coth x analogous Lo 1

FIGURE 1.121a

G mph uf tosh x

+ 1an2 x = sec2 x

FIGURE 1.121b

and 1

+ coL2 x = csc2 x . respectively.

Graph of sinh X

FIGURE 1.121c

}'

)'

y = sinh x
1

y = cosh .t:

FIGURE 1.1

1d

Gra1)h of c::oth x

)'

-'

Graph of s.eeh x

~
)'

~-~ - 1

FIGURE 1.121e

Graph of tanh x

-------------y: tanh x

__-/

-------------- - 1

Graplo of cs<hx

)'

'

~hx
-'

Crnphs of the si~ hyperbolic functions are shown in Figun:$ 1.121. The functions coshx
and Stch x are c\cn: the olhcr four are odd.
M ost trigonometric identities can be derived from the compound-ang le fonnuhl.S for
sin (A :!: B) and cos (A :!: B) . It is easy 10 'crify similar formulll.S for the hyperbolic functions:
sinh (A :!: B)
cosh (A

8)

cosh A sinh 8 .
= ccsb A cc>h 8 si11h A >inh 8.

sinh A cosh 8

( L77b)

f'Or example, equations 1.73 give

.!. [<'A- R _

.. - (A- 81)

= cosh (A - 8).

With lhcsc formul.u. v.c can dcri'c h)pcrbolic identities analogous to trigonometric ideotities
1.-H-1.49:
tanh(A

8) =

tanh A :I: tanh B


I

(1 .77< )

tanh A tanh 8

sinh 2A

2 sinh A cosh A,

(1 .77111

cosh 2A

cosh 1 A + <inh 'A

t I 77el

=2 co>h l A-l

( 1.711'1

= I+ 2sinh 2 A.

(1.7igl

2t'inh A
tanh2A = l + tanh l A'
l
sinh A sinh B = -c~ (A

sinh Acosh

B =

coshAcosh 8

,inh A

I
- cosh ( A - B),

+ 8) + -2I sinh (A -

I
I
-c~(A + B )+ - cosh(r1 -

B),

8 ).

B) (" - B)
A+ 8) (A- 8)
(
(A+8) (A - B)
t'"h

2cosh

sinh

+ cosh 8 =

2cosh

( 1.77il

= 2 sinlt ( A+

+ sinh 8

sinh A - sinh 8

cosh A

==

- silth (A
2

+ B) -

t L77hl

cosh

( 1.77j)
( L7ikl

(I 7711

( L77n )

cosh A

- cosh 8 = 2sinh

(A+ B) ("' - B)
2

sinh

( 1.77o )

In Example 1.56, we illustrate a geometric paralle l between the trigonometric sine and cosine
fuoctioos and the hyperbolic sioc and cosine functions.

I EXAMPLE

1. 56

Show that:
(a) every point (x, y) on the unit circle x 2 + y 2 = I cnn be expressed in the form
A. = cost. y = sin/ for some real number 1 in the interval 0 ~ J < 2"i:
(b) every point (x, y) on the right half of the unit hyperbola x 2 - y 2 =
expressed in the form .t = cosh t, y = sinh I for some real number 1.

I can be

SOLUTIO!\'
(a) If I is the angle iJl Figure 1.122, then clearly the coordinates of P arex =cos 1 Al\d
.:v sin 1. As angle 1 ranges from 0 to 2n. P ttaces the circle exactly once.
(b) A sketch of the unit hyperbola x 2 - y 2 = I is shown in Figure 1.123. If x = cosh 1

und y - sinh I are coordinates of a point P in the plane. where t is some real nu111ber,
then idemity 1.7-1 implies that .t 2 - y 2 =cosh 21 - sinh 21 = I . In ()(her words. P
is on the u nit hyperbola. Furthermore, since x = cosh t is always posjtive, P must
be on the right hulf of the hyperbola. FinaUy, because the range of ,1 = cosh I is
x ;::: I in Figure 1.12 1a. and the runge o f y = sinh 1 is - oo < y < oo in Figure
1.12 1b. it follows rhar every poilll on rhe right h:llf of the hyperhola can be obtained
from some ,nluc o f 1. Note that I is IIQtthc angle formed by the positive ,1-nxis and
~te line joining the origin to (x. y).
Rt:l iHi on~hip ~ween

mm.L

unit circle: and

hypetbc~ic. titnCfiOn(

trigonometric funetioM

P(.r. y) ,.

t< 0

EXERCISES 1.10

il

In Ex.crcl$CS 1- 10 evaluate the expression ( if it h:.s ~ value).

~ 2 . si nh (rr { 2)

I. 3 cosh I

~ 3. tanh Jr

sin 3

i ' 4.

Sin

(<ech 10)

1!1 5. Cos- ( 2 csch I)

!!!

6. coth (sinh 5)

lil

il

8. scch [scc(rr /3) l

7. Jtnlsinh( -3)1

JO. sinh[Cot- 1 (-3:r/ IO)j

11. Verify dtc results in identities t.77c-o.


~

12. Vcn ic.al vibralions Clf the beam in t.he lig urc l)clow involve lhc
function

f(x)

= Aooskx + Bsink.t + Ccoshkx + Dsinhkx

w here A, 8. C. and D arc c..-onslants such that C = - A.


and A and 8 must satisfy the equations

D = - 8.

Sech

A(cosk L - ooshk L)
A(cosk L

+ coshk L ) +

B(sinkL- sinhkL)
B(sinkL

= In (x +

Cosh- 1 x

ln (r

.. _1

the '-=()(ldition

=1anh kL.

tanh

**

x = - ln

./x' + I);

+ ./ x' -

(I+X)
1- x

--,

t);
lxl <

I.

14. When a 11 object of mass m falls from rest at times

= 0 uoder the

innuence o r gr.Wity and :in air re!:istancc proportional to the square of


vdoc-ity~ its veloc-ity 11 as 3 function of Lime is defined by the cqu:tli.on

'I

Sinh

0.

Eliminate A and 8 bctwc.cn th<:sc equations lO show that k l'll ust satisfy
oankL

.x.

(C) Show thai

=0.

+ sinhkL) =

fp ( ,Jii18TlJ -

v)

{!1

l v;;;g In ,Jii18TlJ + II = - -;;;'

13. Each hyperbolic functi_on h3-5 associated with it an inverse hyper

where fJ > 0 i' a constan1 ~~nd g > 0 is the acceleralion due 10 gr:wlty.
(a) Show that when thi s equation i ~> solved for tJ in terms of 1 ,

bolic- fWlction.

the .result is
(a) Drowtbclnvcrsc functionsSinh- 1 x , Tanh- 1 x. Cotband Csch- 1 .\ for sinh x. tanbx . colhx. and cschx .

x.

Vfiiii
T tanh ( V{7ii
-;;;') .

v(l) =

{b) Whv do cosh :r: :10d scch :r oot have inverse func::ti o~ts? lt is

cus;omary to associate functions Cosh- 1 x and Soch- 1 x


wilh co~h x and scclu by rcstricling their domain;:, to
nonncg:Hive numbers. IJr.lw gr.~ ph~ of Cosh 1 X and

(b) Determine

Lht limit or V fur large

I.

called ll1C limiting

\'t.>/(Jt('iTy.

11.11 Approximating Solutions to Equations


This material is likely 11ew for most studellls. It does not therefore have a diagnostic test
associated with it.

In writing this book we have assumed llu\1 you have access 10 a graphing calculator and/or
a computer with a mtllhem~ll ical software package. E\'ery software package has one or more
equa tion-solving commands; most graph ing c.alculators have an cquationsolvin g routine . In
other words. you have a device <hat solves equations either exactly or approximately. In !his

sec lion we discuss some uscful concepts related to accuracies of approxi mations. Suppose. for
example, thai we are 10 ltnd all solu<jons of the equalion

2x - 4 = cos (x 2 - 1x

+ 10).

(l.78a)

We can express lhe equation in the equ ivaiem form

.f(x) = 2.t - 4 - cos (x 2

1x + 10) = 0.

(1.78b)

where soluLions are now vis uali:tcd as x inte rcepts of Lhe graph or the function f (x) . The graph

in Pigure l.l24 indicates 1hm 1he only solutiom of 1he equation is be.tween x = 2 and x = 3.

COROLLARY 1 .7. 1 {The Zero Inte r m e d iate V a lue Th e orem)

ff j (a) f (b) < 0 for a function /(.t) that is continuous on a .::: x ::; b,then there exisL'
at lellSt one number c bet1vee" tl and b for which f(c) = 0.

The conditioll f(a)j(b) < 0 rec1uircs that uno of f(a) and .f(b) be poo;itive nnd the
nthcr he llegativc. (We have showll / (tr) < 0 nnd .f(b} > 0 in Figure 1.125b.) The c ho ice o f
k = 0 in Theorem I .7 give.< this coroll ary.
\Vithout continuity of f(.t). we ca nnot be lWre. in geleral , whelher there arc ~l utions to
the equation j (x) = 0 whell J(a)f(b) < 0. The funct ion in Figure 1.126a has what is
call ed a discuminuiry at.t = d, and there are no solutions of f(x) = 0. The func tion j(x)
in Figure 1.126b aloo has a d iscontinuit)' at.< = d , but there. are hvo solutions o f .f (x) = 0

belween t1 and b.
There are cwo common way!'> co discu~"S the accuntt.'Y of an approx imution to the solution of
an equation. and simple us d1e zero intcmlediatc value theorem is. it handles both simations.
M41flil;lj
!;unctions,

110t

"'Ulll
SltJ~I ymK lbc cund1U~m~ of lhe ~ in!iCnncdiate YUille theorem

II

--"

--"

Ap)Jroximations Rounded to a Specified Number of


D ecimal Places
\Ve say that x is an approximation to the root. a of an equation

f(x) = 0,

( J.7Y)

correctly rounded to k decimal place.~. if .r h<l< k decimal places, and 01 rt>und s to the >1<\me k
dccim!liJ>Iaccs. For example. the al>proximation x = 2.323 762 2n rounded to four deci mal
r>h>ccs is x = 2.3238. How Clln we verify dllll the r001 or equation 1.78b rounds to these smne
fo ur decimal places? We evaluate /(.t) a t x = 2.323 75 and .r = 2.323 85 ,

/(2.323 75)

= - 0.000047,

(2.323 85)

= 0.000 33.

& cause one of these ,alues is positive. and the othc.r is negative, the zero intcm1c.diarc value
theorem implies that the sohuion ofequation 1.78bmust lie between 2.323 75 and 2.323 85. But
every number between 2.323 75 and 2.323 85 rounds to 2.3238.
other worcl,, x = 2.3238
is a solution of ruation I .78b corn:<.tly rounded to four decimal plllCcs.
In general, we t>ln say that .r is an Hpprox imulio u to u ro<.ll of the cquati<>n f (.r) = 0,
correctly rounded to k dtcimal ploccs, if x has exactly k digits t>llcr the decimal, untl

y,,

Jo-')
- f ( +2
( 10-k)
2
X -

< 0.

( 1.80)

Maximum Possible Error


We are oncn asked 10 fouo an approxima1ion1o 1he solulion of'"' equalion >uch as 1.78b. and
be sure lhal 1he error i~ les~ lhan some given "llue e, say e = 0.0001 ore = 0.000 000 I.
The smHIIcr tl1c vulue oft. the. more accurate must be the approximation. To illustrate, suppose
an am>roximalion 10 1he solution or 1.78b is required wi1h error less 1han < = 0.00001. We
could verify lh;ll x
2.323 762 277 has error less Ihan e
0.000 OJ , bul Ihere is litUe point
in carryi11g on approximation wilh nine declm.al places when an CITOr of 0.000 OJ is concerned
with the fifth dccinlal place. We suspect that if we round the approximation to r.ve decimal
places. the result x = 2.323 76 has error less than 0.000 0 I. To verify this, we e'aluate

/{2.323 75) = -0.000047

/{2.323 77) = 0.000029.

anti

The fdct that ~tcsc values have opposite signs guarantees thm the root is between 2.323 75
;md 2.323 77. Since the difference between these numbers is 0.000 02 and our approx imation
2.323 76 is halfway between them, itfollows th'tt thocrrorin 2.323 76 must be less than 0.00001.
fn gcne.ral. we can say that x is an approximation to a root of the tquallon.f(x) = 0, with
error less than ~ . if
( I.RI )
j(x - E)/(x +e) < 0.

There is the potenti.al to use calculators or compmcrs unwisely here. Avoid operating
culcul111ors and computers at or near their limit.s. For example, suppose tbat the solution .< =
2.323 762 277 of equation 1.78b cons1itu1es the full dispi:Jy of a calculmor. h would be unwise

to uucmpl to verify that this solution has error less than 10-9 using the same calculator. To do

so wou ld require /{2.323 762 276) and /(2.323 762 278). Tilese values are very. very close
to ~cro. How could we be cenain of their J)Qsitivity and n~g~tivity when we are asking tho
~lculator w perrunn very sensitive calculations with nuntlxrs at the limits of its capabilities'?

I EXAMPLE

1.57
FincJ an approximation to the s rnal lc.~t roOL of the equa1ion

corn;:.c;tly rounded to six decimal places.

I ..5

- 1.5

.f

SOLUTION The graph off (x) = 2x3e-x + 5x 2 - I i.n Figure 1.127 shows illfee solut ions.
Our com puler gives x = -0.766 05 I 059 as an approx imation 1.0 the srnallesl root. To verify
thal x
-0.766 051 is an approximation, correctly rounded to six decimal places, \VC c.akulate

1.11 ApprqWuatiug SoiUiiOfb to Equ~tiou.)

j'(-0.766051 5) = - 8.1

10-7 ,

9S

f( -0.7660505) = 1.0 x 10- 6

The fact that these values have opposite signs confirms the six-Ocl:imaJ-placc accuracy of
X = - 0.766051.

..-..

l EXAM P LE 1 .68
Find an approximation to the largest root of the equation
x+6sin x = O
with error less than 10-s .
Ujilcliljijl#j LWY .

Gmphic11 solutiotb O( A

+ 6 ~~~ :r = 0

-'jV

- 10
-20

SOI.CTIO\' n te graph of f(x) = x + 6~inx in Figure 1.128 shows live solutions. Our
computer yields x = 5.225 963 530 as an approximation to the largest root. For an approximation with error les s than 10-s. we cake x = 5 .225 963 53. For verification, we evaluate
/(5 .225\16352) = - 4.1

10- S

and

/(5.22596354) = 3.8 x 10- 3 .

When we have nn npproximntion to the solution of an equation correct!)' rounded to k d:imnl


places, we can say that we have an appro,imation with error no greater than Io-t /2 (c<>mrorc
equations 1.80 and 1.81 ).
Knowing an approximation with maxinmm possible error. howe\>er. does nm guarantee a
predictable number of correctl) rounded decimal places. Let us illustrnte. Suppose that we
have used our calculator or computer to appro.ximate the root of an equation f (x) = 0 and the
res ult is 3. 1 I5 00. Suppose funher that we know that the error is less than 10- 5 . Can we give
an approximation correctly rounded to two decimal places? No! The fact that the error is less
than Io-s allows us to say that the solution satisfies

3. 115 00 - 10- 5 <

< 3.115 00 + 10- 5

3. 11 4 99 <X< 3.1 1501.

Since left and right sides of the latter inequality round to 3.11 and 3.12, we do nOI kltow !he
approximation CQITectly rounded to two dec.imaJplaces. However, we do know an approximation

correctly rounded to three and four decimal places, nantely, 3.1 15 and 3. 1150.

As a :;econd example. suppo:;c we know 1ha1 an approxima1ion 10 <he roo<of

f (x)

= 0 is

3.435 with e rror less than 10-J. How many decimal places can we guarantee'? \ Yc ca n say that

1he solttlion sa<isfics

3.435 - 0.00 I <

< 3.435 + 0.00 I

3.434 <

< 3.436.

Since the numbers in the right inequality do n04 buth round to the same two Uccirnal places. we
can guiln\ntcc only one correctly ro unded decimal. namely 3.4.
io efnphasizc thi"\ point ag~1i n. we cannot m:akcgcncrali7.Jtlions to thectl'cct th:u ta maximum
possible C1Torof I o-J. .say, guarunl.ccs any nuntlber of deci mal places. Jn t\'ery specific exampl e.
we will be able 10 determine how many decimal ph c~ are pussiblc. bm general S (~Ue rn ents
covering a11 situation.s are not possible.

I EXAMPLE 1 .5 9
Find points of iHeJ-sect.ion of the curves
l'

= x3

3x2

+ 2x + 5,

y = 6 - 5x2

3.r'.

Give c.:oordin-.1tes correctly rounded to four decimal places.

awn::"VV

1 #UJII;Ij

ll)ftrseclion pol.tUS of two

m.Th

lmersectcCJl pou:u. tJ(

Cl.ll'\'e$ W1lh btuu c luJice of .\:-range

Y
100
-6

-4

-2

SOLUTION Graphs of <he curves in Figure I. I 29a indica<e thai whatever points of inter:>ection
1htrt are. they are in the imerval -2 ~ x ~ 2. Plots on this intervJ.l in Figure 1.129b indicate
two points of intersectiOil. To 1i lld x -coordinates of the points of intersec tion. we ~olve

3x4

+ x' + 2x1 + 2x -

I = 0.

Solutions with four decimal places are - 0.8924 and 0.3422. To confirm <hal all four decimal
places are correc1. we calcula<e the followi ng ,alue~ of f(x ) = 3x' + x> + 2x 1 + 2x- I:
/ ( - 0.89245) = 3.1
/ (0.34215)

J0- 4 ,

= - 4.0 X

10-1 ,

f (- 0.892 35) = -4.6


[ (0.34225)

= 2.1

10- 4 ,

10- 5.

ln tc~ <ion poims on the curve corresponding "' x = -0.8924 and x = 0.3-122 can be found
by substituti ng <he.<e values imo <he cqua<ions for the curves. If we sub,tiw<c x = - 0.8924
inlo y = x' - 3x2 + 2x + 5, we oiJiain y = 0. 115 379. whereas in y = 6 - 5x2 - 3x'. we
get y = 0 . ll5 459. The.s e numbers do no t ag re.e to four decimal places. What lhis poinls out is

thaLthe resuiLof calculati.ons with numbers accurate Lo four decimal places is uoJikely Lo yield
numbers accurate 10 f our decimaJ places; accuracy wi II be lost . How much depends on the nature
and Lhe number of c alculatjons. A similar situation arises with x = 0 .3422 ; the equa tion o f one

curve gives)' = 5.373 17 and lhe Olher gives y = 5. 373 36. They do no1agree 10 four decimal

1. 11 Appro~ i m:uins Sohuion!> 10 &1umion.:;

97

places. Wlult we should hwe done is carry more decimal places in intermediate ealculacions.
For instance. we could catTy six decimal 1>lac-cs with x
-0.8924 10 and x
0.342 245.
Verilicat ion that x
-0.8924 and x
0.3422 are correct tO four decimal place~ is the same.
But using .v = -0.892410 and .r = 0.3422451ead<to corre<ponding y-values that agree to
four decimal places no matter which equation y = x 3 - 3x 2 2x 5 or)' = 6- 5x 2 - 3x 4
is used. namely y
0.1153 and)' 5.3732 .

I EXAMPLE

1 .60

..-...

+ +

At time 1
0. a 5 Q rc~istor. a 2 H inductor. and a 0.01 F capaci tor are connected wi th a
gcncrmor producing an altcmtl.tins voltage of I 0 sin St. where t ~ 0 is time in seconds (Figure
1.1 30). Titc current i in the ci t-cuit thercafter is i (I)
/(1) + g(t), where

{(I) = - e

-~1 [ -18 COS


5

(5511) + - - (5-./311)]
12J3t sut
.

--

165

--

and

g (l) = Sco:s5t

Miilciii.J;:j

lili

+ 5sin 5t

Schemalic fo LCRcircuh

Hl

10 sin 51

are called the transien t and steady-s late pans of the current. respective!). Plot graphs off (I).
g(l}, and i(l ), and explain why the names for .{(1} and g(t) are appropriate. Determine the
smallest ti me-(corrcc.tly rounded to three decimal places) at which Lhc magnitude of the currc:nL
in lhe circuit is I A.

SOLUTION Plots of /(f) and g (l) are shown in Figures 1.13Ja and b; their sum is plotted in
Figure 1. 131c. T he exponelllial factor r 5'1" c~uses values of / (1) to approach zero within a
few se<:onds. In otl1cr words, f (r) is significam only forsmall I ; it dies off q uickly, and therefore
tile adjective lrtlll.tlntl is appropriate. The function g(t) is periodic with amplitude 2/ .J5. It
remains for all time, and o nce .f(I) becomes insignificant. current i (t) essentially becomes
g(r). In other wnrds, the current eventually !Illes down to g(t) , and hence the terminology
S!<atly srarc for g(l) is appropriate.
lii:lldiiIWWFIDI

1'ransient pan of cunent

ljldil;j'WJ

mr;w

Steady-slate pan ot current

0.75
0.5
0.25

+-~~~-+~~~~-+~~

3
- I

4 I

-Q.25

-o.s
-o.75

mm

Atldition of tr.m,icnl and )te.tdy-~c p.u-b of currcnl

1.148f1N

-I

-2

Figure 1.13 1c makes il clearthaL Lhere are nine Limes al which the magnitude of Lhe currem
is I A. fo ur cirncs when the curre m is positive a nd five times when it is ncgali ve. The smalle.sl
is near I ~ 0. 15 s. It satisties the equation i (t ) = - I , or, writing the equation in our standard
form 1.79, we hove

() =

h(l)

= -e - S/' [18
- cos
5

(S.f:ilt)
12..J:il . (5.f:ilt)]
+ --4
165
4
- --

+ -cv~51 + - sin 51 +

s 111

---

Whe.n solving equations o f this complexity for one o f several roo<s, e<~lculators aod computers

usually require the equaLion and a reasonable npprox imation 10 the root the closer Lhe beuer.
Our compULer murned !he root /
0.146 206 when given/
0.15 as inilial approximaLion.
To verify that I = 0.146 is correcLiy rounded to three decimal places. we c.lleulatc that

h(0.1455) = - 0.012

lr(0. 1465) = 0.0052.

and

EXERCI SES 1.11

Verify the a..x:uracy or each root. Make a plot iJt order to dctcmtinc the

In Excrciscs 1- 16 usc a calculator or comJ>u1cr to find approximations


to all roo~ of thl! Ctju.ution ucxumtc to ~i;\ ..Jccirnul pluc~. Ycril'y IJ~c
accuracy of each
of roob.

i
i
i
i

I'UI.)I.. Ma~e

+ 3.\' + I 0
x' + .T - 3 0

I . .rl

3.

5. x 1 7. .r '

S.rl -

+ 1r2 -

x
7

+ 4 c: 0

=0

a s>fot in 01\k:r to dc(crminc the nuntbcl'

il
i!
i!
i!

4- 0
3
4. x -.r l + ,( - 22 0
6. .rj +x - 1 = 0
2.

.\'l - .( -

8.

x+1
.,
- - = x- + I
x- 2

i.

9. ' - I O~inx = 0

i ...

10. sctX= - -

i
ii .
ii

I I. (X+ 1) 2 = sin4X

12. {X+ 1) 2

13. x + 41nx = 0

;r . 14. x ln.r

15.

p\

+ (J

~ =lOr

~g

1 + x"

=5sin4x

IIUIUbcf ul" l't}()C..).

i
i

.r' + l r ' + 6-t = 0.


fi 19. -_r- =.T ' + 2. o-
.r +l

I
ii.
!*

IM. ,r'' -

iii*

=6

10

'

= .r 1 - 4.r. 10-3
10-3
(.c + 1)1 = 5sin4.r.
cos2 x = x 2 - 1. lo-

20. (.r + 1)1


21.

22.

i x 2-l. x + (lnx)' = 0.
24. e,l

+ e" = 4 .

10- }
10.

In Exe rcises 25- 28 find all poinl$ol'inlcrsccrion forlhCClltvc~; a(.'Curate


to four dec;imal pla..."Cs.

16. ....~ - 4., 1' = 0

In Exercises 17- 24 use a c-alculaloror computer to fi nd approximations


to all roots of Ote equation with error no greater lJtaJt that specified.

10- l

17. x' -s.r-1 = 0.

li
li

25_ y= x' ,

y=x +5

26. y = (x+l ) 2,

y =x' -4x

Sumnl[ll')'

i ~ 32.. Pla11ck's lmv for the energy densi ty of bladb<Jdy radi~tion al

i 27. y ~ x - 20. y ~ x 3 - 2.< 2


X
1
li 28. )' -- x+-1' .Y = x + 2

il.t:

I ()(KJ>K slates thl ~~~

E =

29. When the beanl in Llle tlgurc below vibrates vertic-ally, there ore
certain frequencies of \'ibntlion. called natural frcquciH:ics. They "rc
solmions of the cqlation
wnx

99

e<"-l = en -

1'

whc.:1t: k > 0 is a t:i.m~1tuH anr.J c = 0.000 143 86. 'fhis (unction is


shown in the figrurc below. Ttle vahc of A. at whieb E is a maximum

= e" + e-'

must ~uisfy the cqua1ion

divid<:cl hy 20n. Find the 1wo smallcsl frequencies com:cl to four

dcd mal places.

(51. -

c)e'' '

- 51. ~ 0.

1---10 m-----l
Find lhi:s \'<tluc l:'f A CQtTcct to :s..:vt.:n l.lt:cinl<:ll places.
M axinm.n E

II+ 30.

A stone o r muss I 00 g is thrown venic:tlly upwa rd with spccd


20 nVs. Ai r exerts a resistive foroe on the stone J>roportional to its

speed. and has magnillK!c 0. 1 N when lhc spocd of lhc .stone is lO nVs.
It can be shown that tbc heigh! y tllXl\'C 1bc
by the stone is given by

- YS. It

+ 118 1 (I

p~jcction

point

~tta i ncd

- ,..- r/ IO) m,

where 1 is time (m<:asurcd ln seconds \vitb I = 0 ul ti1C i.nslunt of

projection).
<a) The l ime taken ror I he stone 10 remrn 10 i1:;:; projoclinn poin1
c::~n he oblaincd by setting)' = 0 otr.d so1\'ing lhc cquatiun
f04 1 . Do so (COITccL io lwo decimal ph..'ccs).
( b) When air tcsistancc

tS

.r = 201 -

ncglcctcd.thc fotllluJa foe.\' is

4.9051 1 Ill.

Wh<Jt is 1hc elaJJSCd time in lhis case from the i nstt~nt lhc
slone is projected unlil il etums to the projcc.' lion point?
il+ 3 1. A uniform hydro c~tble P = 80 m long with mast> per urtit length
p 0.5 kg/m is hung rroJO IW() SllppOr1S ~t Ihe ~{lnlC level /.. = 70 nl
upart (fjgure below). T he tension T in the cable ut i t~ lowc!)t point must
sati:;fy lhc CC.lU:ttion

= ePr:I../C2.f)

pg P

e-p~l,J(2TJ,

where g

=9.81. If we sc1z = pg{(2T), 1hen z mus1 sa1isfy


2Pz

= e L: -

, - t.z.

Solve thiscqmltion lOr and hence find T corrcc.l tOo ne dcdu-Ktl place.

* 33.

Let /(.'() 1>e a continuous function wilh clotntlin ttnd range bolh
cquaJ to the inte.Tval la, b) . Show that there is :.tt l east one value of :x
in a ~ .x ~ b (or which .f(.r )
x.

34. Use the 'l-Ct'O inu:rrnedi:ne value J hc~em 10 prove thai 3 1 any given
time Ihere is~ JXI.ir of poinls directly opposite cuch olhcr on Ihe <:qu:uor
or lhc <:~uth lh411,. h~vc c:<ac.tly the 84111\C LCillJ)(:I'".UUIC. Him: Take lhe
equator 10 he the circle .x 2 + y 2 = f!. Le1 f(.x) be the 1ernr':lt~r.tture
on 1hc upper Sc 1nicirdc und g(.r) be tht: tcmpcn'tlurc on the lower
f(x) - g(- x) .
semicircle. Con;oo;idcrthe runction F(.:c )

"' 35. A m:lralhooer runs the 26-odd miles frt"lfn poim .4 1.0 point 8
suu:ting at 7:00 u.m. Stuu.rduy m<>rning. Stw:tin:g at 7:00 u.m. Sundny
morning she runs lhc course uguin, bt11 this lime from point 8 10 J>Oinl
A. Prove Lhul H~: is a poinc on the t:oursc Ltuu she p<l:-tSi..x.l at ex:~ctl )'
the sa.nc time em both days.

36.

(a) Usc 1hc zero imcrmediarc ' 'nluc Llteotclll to pro,c thot whc1\
lltc d t)main of a continuous funcLion is an interval. so abo
is its range. Hinr: Use Ihe idea Ihat a setS of points on lhe
y-ax.is constitutes an intc.r,,al if for at\}' two poi1\IS c and d
inS ~ the poinlS c < y < d arc all inS.
(b) IFthe domain is an open interval, is the rant<: an open inlcrv~tl'?

SUMMARY

l.n this chapter we have reviewed basic concepts from algebra, anal ytic geometry, and uigonomelry, and illlroduced mate rial thai is essential 10 mally of 1he applica1ions of calculus. To tilld
real solutions of polynomial equatioos with inte.ger coefticient.s. we use the r-o:l t ional root theorem
to narrow the field of possibilities and Lhe factor theorem to remove roots rrorn the equation as
they are found.

100

Char.<tr I

Cal<.mhl~

Prep:1r.wio11

Analyricgeomerry is a combinalion of geometry and algebra. Algebraic equations are used


1o de.<eribe geome1ric curves and curves are Ihe geomerric represenm1ion of equalions. The form
of an eqllalion dic1a1es 1he shape of 1he curve and. conversely, 1he shape of a curve influences
iL~ equation . To illustrate this fact. we discussed strdight lines, circles, parabola-;.. ellipse.~. and
hyperbolas. The most common forms for equations of these cur\'eS are as follows:

y - y 1 = m(x - x 1) PoinHiope
X - Xt
Y- )'1
Two-poim
X 2 - x,
)'2 - Yl
Slope y-imercep1
y = mx + h
Slope x -inlercepl
y = m (.r -a)

Straight line

~+r = l
{J

A.r

P..uabola

Tw<rintercept

+ By+

\' = a.r 2

+ bx + c

Vcrlical axis of symmetry

{ x = ay1 + by+ c

Circle

""~

x 2 + y1

a2

y2
bl

+ (y

= 0

k}' = ' 1

(x - h ) 2

az

Horizorual axis ot" symmetry

+ .fx + /0' + e

{ (.r - IJ )l
x2

General

(y - k ) 1

bZ

Hyperbol<t

Basic to a.ll mathematics is the concept of a function. a rule that assigns to each number x
in a domain. a unique number y in 1he range. A function is simply another way of ~aying "a

quan1i1y y depends on x ." The notation y = f(x) for a function immcdialely suggesrs thai
a futiCiion can be represe111ed geornelrically by a curve - 1he curve wi1h equation )' = f (x)
- and we call lhis curve the graph of the funCiion. PolynomiAl< are func1ions of 1he form
a11 .t" + + a 1x + a 0 , where n is a nonncgarivc integer nnd coefficiems au, .. . ao arc
1

conMants. Rational runction~ are quOlieniS of polynomials. Sollle run<:tions are even. some are
odd, most are nei1her even nor odd, and only f(.r) = 0 is both even and odd.
Some of rhe most important functjons i.n mathematics have inverses. in particular, the

exponemial and logarilhmic funclionS,Ihe trigonometric functions, and the h)pertJolic functions.
A function f - 1(x) is the inverse of f (x) if f - 1 (f(x )) = x for each x in the domain of
f(x ). The in\'crse runclion 1 - '(.r) reverses the aclion of l(x). The graph of 1 - '(.r) is lhc
mirror image of the gruph of ./(:~:) in the line y = x. Increasing functions ha'-c inverses! us
do decreasing functions. The domain or a function that <.loes not have an inverse can usually be
subdivided into subdomaios on which the function does have inverses.
Trigooomeuic fuoct ions play a prom.ineo1role io many areas of applied mathematics. Par
ticula.rly important are descriptions of oscillaLOJ)' systems by the sine and cosine functions. The
sine, cosecam, tangem~ and cotangent are odd functions, whilecosine and secam are even. All are

periodic: >inc. ccc<:am. c~inc. and ~cont ha-c period 2tr. and tangent ar\d cocungcnt hove period 1t . Trig011omttric funcuonssausfy mony identities. Recogni7j ns when the~ identities con
be u.cd 10 adant~c to>implif) "'~"'"''"ion" or write them in ahemati-c fom~>o i a hu~e a..,-cc.
The: trigonometric functions do not hnvc in\'Crses. butlhcirdon13inscttn be restricted so 45 to
cr<"ate in\c~ function<: the<edomains tum om to be the principal 'llluesofthc a<eiated in~r;e
function. 1lle in\'Cr)C trigonometric function.:, reverse the role) of trigonometric func1i on~.. A
trigonometric function such :\s the sine function a.ssoci3les: a v:tlutcalled s.in .t with a n.~l nun1ber
(angle) x. The <OIT"C:l>por\ding iowcr.c: functior1. Sin _ , .1. regards .r as the sine of an angle. and
yields the angle in the princip.>l value range with sine equal to x .
F.xronenrial :tnd logarithmic function< are also imponant in applications; they are intr<e<
of each Other. These functions arc not periodic. nor arc they Cl"cn or odd. The exponential
runcrion n"' ro1ses a to power x . As its imtrs.c, l.h2 logarithm funetion to&, x does 1he rever.;e.
h dc.tcrminc. the power that a mu>1 be rdi>cd to produce .r.
Hyperbolic function$ ore speciol combinotiOI\S of exponential functlol\$ that arise sufficiently oflctl in at>plicalion' lO warrant 'J')e(ial tCOfl'\itltnuion. 11ley Ntlisfy i<kmi1ies \Cty simi lar
to the trigonom<:ttic functions. Each h) pcrllie function has an associated inverse h) pert>olic
funcrion.

In approximating ~lu 1ion~ 10 equation.~. it i~ alway~ llCC~'-'ll)' 10 indkatc the accurctcy of


the approximation. This can be d011e by correctly rounding the approximation to a specified
number of decimal place>. or determinin an agJproxinratiort " ith maximum error. ln bolh cases.
the zero intemtcdiote olue theorem is instrument41 in -erifying the :K"CUr.>C)"
KEY TERMS

In reviewing this chapter, you should be able to define or discuss the following key terms:
Pol)nomial

ROOtS
Linear equarion
Quadratic fomtula
Irreducible real quadratic factor
Carte."ian coordinates
Leng1h of a line seg.ntent
Midpoint of a line segment
E(luation of a curve

Rise
Point-slope fonnuln
I"Mallel lines
l"o1mbola
Circle
Hyperbola
Function
Dependent "ariablc
Range
Absolute: vnluc function
Odd functi011
Rational func1i011
One-to-one
Strictly moOOooic function
Stano.lard po;;iti011 of an angle
Sine law
Double-angle fonnulas
Sum and difference fonnulas
l'eriod
Amplitude
Inclination of a line

Polynomial equarion
Solutions (zeros)
Quadratic equation
Discriminant
Multiplicity
Quadrnts
Absolute "lllues
Coordinates
Slope or a line
Run
Gcneml equation of a line
Perpendicular lines
SymmNric-about the x -axis and y-axis
EllilJSe
A symptotos

Independent ''ilriable
Domain
Graph uf a function
Even function
E'~n and odd pans of a function
Translation
lmerse function
Radian
CO"i ne law
Co mpound-ang le fomlUias
Producl formula.<
Periodic
General sine function
Phao;c shift
Algebmic function

(......,, r ........ ,.....,... ,.,._

- ---- - - -- - ---

HI! VIII&:W

&X A c rzes

-- - -- --

J l , 'h :l 4 ~\ ~ , . - ))~ ' \l )

.u. 1.,2_,\- ) ,\ I

Jl. 0 .-l\.

)'

T2x ... ,

,\4, ( - ! .

S).

X 4 - \y

6. (2. I ),
if'l bc:rci~c" 7-10 t 1r1d dte f.'I.IU,dlo" fur t l1c lh't decil)(XI

1. ~nl&eltn the IN.' I -

h <& and l oro~tfh d'(' r-.:M (l, ~}

3. f\.'tpen~lcul:u tu ~ he line jlkunt ( - l. 1) 1\l lhc 1lgin and thnlUJ,I'


lhe uodpomt of~ hne ~ntnL iott~ina ( 1. ) ) 1n1 ( -1. ')

.n. ., a.

-" , ,., - \ l

~ Pclpendic:\l.b,r k the 111'10 X 4\ - II .u'ld thu.'14lp:h lhC 5)1)n\l ol


t~liC~ti(,n of 1.hi" lhtc and x = .) ) ~ +I)
'

10.

JotnlnJihe

!'>('IIIII ~ of llll Crtieelil)R oflt~ C:\11'\"CI'i \' a ,\ I r.11d 5.t

6 - yl

II. /(<)

11. f

.f):T+3

,. ,

....

/()

IS. j(x) ~ (xl

16-

f (.<) = " '

IK.

f ( l)

x+J.r -+-2

>

<-) .. .f'i!"='3

IJ. /(<)- -.- --

17. / (<)

19. /( )=

JJii

8) 1i>

+ <L<

lf + I
~+2

' - x'+!., z+.I

51. )' w l \n \)t - 1

7.9""!~.J'!x1 1 4x S

20. {( .<) =

.r .
1

+ 5.\.

=.-,.

In E~tr'(tioe' 2t - 32 ukn11r)' dw:uii'V ;,~' 111 <o;~,r.iilhl hnc.. p;.at"Jbi.i c.uck.

11 M.. y J.,ml 'l:


' ~7. y,. <ooh(lx - I)

cthpsc. hyf)CI I)()Ia. \if none of thcbC.

21. ... -r2r=4

lJ. ' .. .. '+3


25. y2 - x 1 = x
27 . .r 2 - l.t'

+ y 2 s: 16

29. x' +2/ + y = ll

22. .f - ).l -ly+J

u. x 1 -ly1 =.a
26.

x' + y' + 5 = 0

+ 2)' =

59 . t

60. ' =4y+2.

4.

= 3x- 2

28 . .r+y 3

30. '-+Y'+y=O

2x+3y =5

Re\ iev. C~en:iM-.5

103

In E).en:il>C!!. 61-6-l give un cumplc ur a fwx.'lion )' =/(X) with the


indicated properties.
Spring(k~

61 . TI-.e rrutgc of the fwlC'tion consbtsofonc: number oc1ly.

16)

,. 62. The _.Jgcbro.tic formula defining the function ca.nnot be cuended


beyon~

-I

M l

:5! x :5! 2 .

.., 63. The domui11 Qf the function co ns i slt~ of all rcals except .t = :I.
64. The donwin of the function is x

In E.xcfc:i!lts 6:>- 68 ~how that d\C function docs not ha"e on n,ese.
Sulxlividc its dclmuin of dcllnition into subnu crvals on which the fune
tion has an inverse, and find the inverse runction 011Ci.1Ch subintcr\':tl.

* 65.

[(x)=x'-4x+ 3

* 61.

/(.<) =

66. /(.<)

x' + 2

* 68.

x' + 3

'0

0 and the run~--c is _\1 2: 1.

j'(x)=

.x' -

.\' C

i t~ g), A lighthouse is 6 ~m ofrshorc and o cabin on Lhe stnl.ight :d10rclinc


is 9 k1n frunlhc IH>illtOillhc~I'On: rteun:sl thc lightllClUSC(figun: below).

8x 1

iff;

69. Expo'l:s. f(.x )

cos2. - sin 2x in ohc rorm f(x) A sin (2x + f/J). Usc thi s to dr;;~w u grtlph of the function und finU
the scoond sm.-llcst ~itivc vul1.1c of x for which f(x) = 0.

70. Is lltt function [(x)


IS its f>ciod t

= 2 sin 2x -

Shorcli-.c _ _ .

71.

1---~---1
Show lhat iru man rows ut 3 k1n/h ~nd walks ~~ 5 knv'h, und he bcac hc~
Lhc bolll at distuncc x from d1c ne<V poin1 on the shore. then the total
lfti\IC) time rt'OIU liJ]hthOUSC LOCObin iS

cos',, + S cosx - 6 = 0
4sin 2.<

=I
I

~. 7.1. coc 2 (.< +I) = 3

il u .

C 1bin

3cos 3.\' periodic'? If so. what

In EAcrcil<:~ 71-80 li nd all solutitMlS of the cqu.ation.

i . n.

- -1

T.-.n- 1 (3x + 2);;::; S- 2rr

= /(.<) =

J.< 1 + 36
3

9 -.<

+ -5- .

0 :0

.X

5 9.

Plot a gr.Jph or this runetion.

75. cos 2x = ~in .T

i
i

76. In (inx) +In (I+ ,in.<)= In 3 -In 2

il

78. 3 sin (e'..2) = 2

1~. '"" (x etlbh 2)

77. 3 Sin-

(t'+Z)

In Elcrciscs 84-SS find :111 so1u1ions or the equation .:otT'CCtly rounded


10 lhrcc docin\.al place$.

= 1/4

!ij* 80. Silth X = 4


.>l

81 . Onsw gr<tphs of the following functions:


(a) f (X ) = ltm (Tan 1 .r)
(b) j(x ) = Tan 1 (oanx)

1!4. x l - 2.< 2 + 4.< - S = 0

SS. x 2

I ;;;;;;;

sin x

In Exercises 86-87 tind all sol1.1tions of the cquaio.n with error Jess
Lhiul

10-".

j .,. 82. When the lllilSS in illC fi~urc below is pulled 5 em lO the right of
the position ( x = 0) ic would occupy were the spring unstrctchcd. and
given speed 2 m/s to the right. its positionthcreancr is giva1 by
I

ct>s4t

2sin4t.

Find when the n-wss 1>asses through x

0 for the lirsl ti mc.

x(t)

20

"' 88. PI'Ove that the diugonuls o( u lilombl:o; intersccl at righl angles.
(A rhonbus is a parallelogmm with ttll sides of Ctlual lcngth.)

CHAPTER

Limits and Continuity

With a solid foundation of fundamental,; in Chapter I. you arc well-prepared 10 study calculus.
We hope that you have been conscientious in your review. The better your algebraic sk.ills and
the more familiar you are with analytic geome!Ty and trigonometry. the easier calculus will be.

Tn Chapter I, we placed tremendous emphasis on graphing. This was by de-s ign. The
most d illicult part of the solution 10 many problems is frcqucrnly the initial step. Once sta11cd,
the solution often unfolds smoothly and easil), but that first step sometimes seems impoosible.
One of the best way!> to stan a problem is with a diagram. A picture, no rnatter how rough. i1)
invaluable in giving you a "feeling'' for what is going on. 1t displays the-known facts surroundilS
the problem: it pcnnits you to "sec" what the problem really is and how it relat.cs 10 the known
fa~.:ts: and it unen suggests that all -irnportunt first step. We encourage you to dC\'Ciop the habit
of maki11g diagrams at evecy opportunity- n.otj usl co solve problems. buc to understand what
calc ulus is all about. We want you to see lmdjeel calculus in all its aspects.
We introduce each remaining chapter of nhe book with an Appl ication Preview. a problem
from one of the engineering disciplines, the solution of which requires material to be inrroduccd
in thm chapter. The solution of the problem is identified as the Application Review Rev isited at
the appropriate place in the chapcer. Here is the Applicadon Preview for this chapter.

Application P review

The figure on the left below shows a complicated electrical network containing c.tpacitors,
inducwrs. resistors, and a source of electric vohage E. Electrical engineers. are imerested i n the
induced current i11 various parts of the network whcu the source is tur11cd on and oft' very quickly.

E
4

- - ,(1)
6;(1)

E.,(I)

6 ,!1)

a+l

Funct ion 1(I) in the figure on the right represents I V of potential being mmcd on at time
t = a, <lnd turned ofl' again one second lat.er. n,is is not a very short pc.riod of time, Graph
2(1) in the same figure represenl.'i 2 V cumed on for one-half of a second: 3(1) is 3 V for
one-1hird or a second; and E~(t) is 4 V for oncCiuancr or a second. Were we to concinue this
process indefinitely, the source wou_ld apply cvcr-i ncrcasing volt.ages over ever-decreasing timt
intervals. but the product of the voltage and ~~e Jengll>of the time imerval i~ a lways unity. The
uhimatc result of this process is what is called an insrantancous appliCiuion of I V 10 the circuit.
THE PROBLE).1
How do we re presemthe result of th is process as a mathematical.fimction,
and how do we operate wilh this functioo in equ.cnions'! (For the answer. see Dirac-della functions
in Section 2.5 on page 14 1.)

104

'l, I

U mili

105

0111 you see 1hc problem'/ As a gc1s c loser >111d closer 10 zero. the graph of is zero
C\'crywhere. exec pi nt 1 = a. where it bcCOI'IlCS "infi nite.'' In what sense is this ~l f unction'! Get
rhe jeeliflg 1hatthis cann(H be a function as w,e now understand f unctions.

1he con(.'ept o f a limit is crucial tO calculus. for the two basic opcra1ions in calculus are
ditlCrenth-llion and intcgnuion. each of whicl1 is defl ,le:d in cen'I1S of a li111it. For this reason
you must have a clctlr undersltulding o f lim its t'r01n 1he beginning. In Section~ 2.1 - 2.4 we give
an il1tui 1ive discu!ision of limits of functio ns; in Section 2.6 we show how thc.se i dC-~.'\3 car1 be

fonlu\lizcd mathematically.

12.1 Limits
The functions in F igure 2. 1 all have value 3 c:u

.r

= I . bl1t behaviours of the functions close w

~" = I are totally d iffe rent. As x gets closer and c IQscr 10 1 in Figure 2. 1a, func.:c ion ' :&luc!\ get
closer and closer 10 3. In Figure 2.1b, function , aJucs ge1closer an~ closer 10 3 if x apprvaches
l lhrough number.; small er lhm> I. but Ihey gel closer and clo.o;er 10 2 if x approaches I through
ll u1nbcr~ larger thau I. Functiot1 values 8 J>I~lch 2 as x gets closer and closl."!r 10 I in r:igure

2. 1c wheLhcr x aJ)proachcs I through numbers smaller than I or larger than I . In chi~ section
we emphasize the distirx:tion between the vatuc of a functionaL a point. and the values of the
fu nction a.s we approach the point. \Vc <.~m see the disci net ion graphically; we 1\0\V want to

expre.s i1algebraically.

'

l!IJ

The \'<lluc: of a funC'tion ut

riD
x = J nco.l ool equ.al its, liruil a.) "

MijU-ili.J#Hf#ll
('PJ)r\!'~bes,

)'

=x1 -

4x + 5 at x = 2 is .f(2) = I . A complclc.l)'
difl'ercn1considcnuion is comained in dte qucslion,"What number do values of .f(x ) = x 2 4X + 5 gel elMer and closer 10 as x geL<clo.<er and closer 10 2?" Table 2. 1shows lh>ll tl.f x gms
d usc:r mul clmcr tu 2. ,alues u/ x 2 - 4x + 5 gel clu.w:r wu/ close r to I .
The value of the func tion

1.9
1.99
1.999
1.9999

.f(x)

= x2 -

(x )

4x + 5

1.01
1.000 I
1.000001
1.00000001

2 .1
2 .01
2.0()1

2.0001

106

Chap1er 2

Lirni1s nnd Con1inuity

I I~I?JII;I#IOO

Limit

t)f

x! - 4x

+ 5 i~

I as x appro3C'hes 2

y xl - 4x + 5

--- -- - ----

Likewise, the gntph of the fi111ction (Figure 2.2) clearly shows that values off (x) approach
I as x approaches 2. This smtement is not precise enough for our purposes. For insmnce. the
grdph also indicates that as x gets closer and closer to 2. \<tlues of f(x) get closer and closer to
0. They do not get ve.ry close toO. but nonetheless. values of f(x) do get closer and closer to 0
as x gets closer and closer to 2. In fact, we can make this statemem for any number less than I.
To distinguish I from all numbers less than I, we say that x 2 - 4.t + 5 ca11 be made arbitmrily
close to I by clwosbrg x ,u(ficiemly close to 2. We can make values of x 2 - 4x + 5 withio 0.1
of I by choosing values of.< sufficiently close to 2; we can make values of x2 - 4x 5 within
0.0 I of I by choosing values of x even closer to 2 ; we ''"'' make values of x 2 - 4x + 5 within
0.001 of I bv choosine values of x vet eve.n closer to 2, and so on. For numbers less than l,
we ctuutol d~ this. Fo~ instance, it i; not true that x 2 - 4x + 5 cart be made arbitrarily close
to 0. The closest the fwtction gets to 0 is I unit when x = 2. In calculus we say that the limit
of x 1 - 4x + 5 as x approaches 2 is I to represent the more lengthy statemem "x2 - 4x + 5
can be made arbitrarily close 10 I by choosing x sufficiently close 10 2." In addilion, we have a
nOiation to repre.~em both swtements:

lim (x 2

x_,.z

4x

+ 5)

= J.

(2.1)

This notation is read "the limit of (the function) x 2 - 4x + 5 as x approaches 2 is equal to 1,"
and this stands for the statcmc.nt "x2 - 4x + 5 cao1 be made arbitrarily close to I by choosing
x sullicienlly close to 2."
We emphasize that the limit in 2.1 is not conccmcd with the \aluc of x 2 - 4x + 5 at x = 2.
It is concented with the number that x 1 - 4x + 5 apJ)fOaches as x approaches 2. The.~e numbers
arc nO{ alwJys the s:tmc. For example, the limit as x approaches I of the function in Figure 2. lc
is 2, whereas the value of the function at.\' = I is 3 .
Generally, we say that a function f(x) bas limit Las x approaches a, and write
lim f(x) = L

x ...:.a

(2.2)

if f(x) can be made arbitrari ly close to L by choosing x sufficiently close to a . Sometimes


it is more convenient to write f (x) --+ L as x -> a to mean that .f(x) approaches L as x
approaches a. This is especially so in the middle of a paragraph, as opposed 10 a displayed
equation such as 2.2.
The value ofthe function f (x) at x = a is irrelevant to the limit off (x) as x approaches
a. In Figure 2.3a. they are the same; the value of the fuoction at x = a is L. 1he same as the
limit as x - a . In Figure 2.3b, the value of the fuoct ion .f(a) at x = a is di.ffereot from the
limit L as x ~ a. Finally, in Figure 2.3c, Lhe function has no value al x = a, but Lhe limit as
x-+ a is L .

MiUijll;i##I:W

MOU311.1#Wr:w
l~gll~$

'-( .:. ......, ....~ at " ~..., .... ...,.

h.) IIIUsttUie l h<lt t It \ '<l UC v

fll<l)'

,
. . .
be d itfel'enl lt c.'lnl tU lnnJt

US X -

ll
)'

)'

ft J

L ....... .

L - -- --- --- -- ~

I. .....

(J

(I

"

I EXAMP L. E 2. 1
Evaluate. lim (x 2 + 2x
s- 1

sOLUTION

+ 5) .

As .r gers closer and closer to I , values of .r 2 + 2x

+ 5 gctnrbitrarily dose to

8; therefore, we \vricc
lim (x 2

+ 2x + 5)

= 8.

This is conubora1ed by 1hc g raph o f the funcJion in Figure 2.4.

MifUJII;I#JI'J

Limit of

x: + 2x + 5 is 8 u

X ~l(lptalt(.'hcs

)'

-I

To calculate the limit of a func tion f (x) as x approaches a, we evaluate .f (x) at values o f x
that get c loser and closer to a. For limits of complica ted functions such as

it would be tedious to evaluate x 2 (3 - x)j(x 3 + x) at many values o f x approaching 5. The


following theorem provides a much e asier me thod.

THEOREM 2 . 1
If limx- /(A)
(i)
(ii)

(iii)

= F aml lim,_

lim [/(:r) + g(x)[ =


-Q

lim (/(.T) - g(.r)]

-
,_.,

lim [c/(:r))

= c F.

lim (/(:r)g(.t)]

=G . thcn

g(.o:)

F + G.

t2Jal

= F - G.

12 1hl

when cis a constant.

12 kl

= FG.
provided that G

(v)

12 3~1

0.

\ Vhat this theotcm says is that a limi t such as lim,_, [x'(3 - x )f(:r'
oown into >~nail er problem> and n:a:.scmblcd later. For itl>tancc. >ince
lim :r 2

- s

= 25.

lim (3 - .r)
-S

= - 2.

lim x3
-S

= 125.

+ :r)) can be broken


lim .t
-S

= 5.

we ntR)' write

. xl(3- .t )
25( - 2)
-so
hnt
=
= -- =
x-s .TJ + x
125 + 5
130

5
13

Although the resultS of Theorem 2.1 fl'l.a)' seem evident~ to prove t.hem nlathcmatk.ally is

not

a simple 1a.~k. In fact. because: we have nOt )'et given a precise definition for l imiL<. a proor
is impossible at this time. \Vhcn we g,i\'c definitions for limits in Section 2.6, it will then be

possible to pro,e the theorem (see Exercises 31 - 35 in Section 2.6).

I EXAMPLE 2 .2
X ~2

E''llluate lim - -.
- - zx 2 + 9
SOLUTIO~
2.1 give,;

Since limx- - l (x + 2) = 0 and lim_,__2 (x 2

x+2
0
= 13 =
- - 2xl + 9
lim - --

+ 9) =

o.

I EXAMPLE 2 .3
x 2( 1 - x 3)
Evaluate lim

x- -1 2.t 2 +x +I

SOLIJ il0:-1 Using Theorem2.1. we can \\file


.

hm

.t 2 (1

- x 3)

~-t2.fl TX+ I

(I )(2)

2-(-1)+

Be sun: that you understand how we. obtained the cxJ)rcssion

(1)(2)
2+(-1)+1

= 1.

13, pan ('')ofTheorem

2.1

Umh:;;

109

In panicular. w<=ditlllot>et X = -I iou 2 (1 - x 1)/(2.r2 +. + 1). l ndccd. thi>i,n(Kpcroniucd


because to cvoluatc a limit as x approaches - I , we are not to set x = - I ~ we nrc to let ~t
get clo~er and c loser to - I. Whao we did do i' take limil< of .r 2 , 1- .<). zxl. and .r as .r
!Opproaches - I . omd lhcn u se n lcorc.m 2.1 .

The rollowing example ilhowates \\float cao1 happe.1 if we suhsoiuue x


evaluation of Lim,_.j(x).

= a into f (x) in the

I EXAMPLE 2 .4
9

.(2 -

E\aluatc lim - - x- 3 .\" -

~oumo:-:

Recause lim,_ 3 (x - 3)

0 , we cannot use TI1eo>rem 2.1. Nor c;m we set

x = 3 in (.r 2 - 9) / (.r - 3) because it is inherent in the limiting procedure lhat we do not

r>ut .r
3. Besides. if we did. we would obtain the meaningless expression 0/ 0 . Figure 2.5a
contain> a typical graph of the function (x2 - 9)/(x - 3} on the interval -3 ~ x ~ 6 . using
a calcut:nor or com puler. It s hows no anomaly in the behaviour of the funcrion at .t
3. The
grdph may. howe, cr. be accOill(Miliccl by a message indicaling thatlhe function is undefined a1
x
3. as indeed it is. But the fact that the function is undefined a t x
3 does not concern
us here; we are interested in value> of the function near x = 3. not a t x = 3. and the graph
indicates that as .t -+ 3, values of the function approac-h 6. To verify lhis algebraic-.!IJy, we
fac tor x 2 - 9 imo (.< - 3}(x + 3) and divide 001 a factor of x - 3 from numerator and

dc.nominator.
x2 - 9
li m - - -

..~-3 .l" -

lim

x-3

(x - 3)(x
X -

+ 3)

lim (x

x-3

+ 3) =

6.

D ividing by the factor x - 3 would not be permissible if x - 3 were equal to 0. that is. if x
were equal to 3. But once again this caonot hapJ>Cn, because in the lim.iting operation we le.t x
get c loser and c loser to 3. but do not set x = :~.
~

IIO'llPh

u.oo.

13

CQmputcr

or <~'- Vl/(> -

'"""" '"'ph ol <x' - 9)/<x -

3)

3)

x - 9
=-.r - 3

6 ...
6

Note in 1his example that ahhoug.h the lirni1 is 6. there is no value of x for which the function
(x 2 - 9)/(x - 3) is c,er equal to 6. The graph of the fu nction is a straight line with the poiou
at x = 3 rcmO\'cd. We have shown this with an open circle in Figul'c 2.Sb.

110

Ch.:~pw- '2

I EXAMPLE

Limits :.od C<>t.ninuity


2.5

- 2..r

E valuate lim

+ 3.r2

.x-..o 4x - x 2

SOUmON The function is undefined m .< = O. but


3x - 2
- 2x + 3x 2
- 2
_x _,_
(- 2_ +-'---3-:.r")
lim - - - ' -, - = hm
= lim
7
=
=

.. - - o

4..\' -

x~o

x2

x(4 - x )

x- o 4 - x

Figure 2.6a shows a computer-generated gr"ph of the function on the interval -2 ::; x ::; 2 .
\Ve httve redrdwn the gr.-_ph in Fig ure 2.6b \~rthh a hole at .r

= 0.

W:UdiiI#IX* Compurer

Mifliriii;JWiiiiW

~~>""Ph of ( - 2x +

gmph of ( - l.t

Jx')/ (4.t - x'l

-2

Redruwn

+ 3x 'J/ (4x - x ' )

-t

2X

I EXAMPLE 2.6
Evaluate lim

JI+X - 1
-'---=--

SOLUTION

Since the. limit of the denominator as x approaches() is 0~ \Ve cannot immediately

.r~o

,fi

use Theorem 2. 1. The f uncrion i s undefi ned fo r x 5 0; i rs graph on the interval 0.001 5

I in Figu.rc 2.7 suggests, a lthough not co ncl usive ly. that the limit is 0. To verify tlt is we
n tiona lize the nu me r-. uor, Lh<H i s, rid the numerator o f the square root by multiply ing numerator
a nd de nominator by JT+X + l :

li m

x- 0

.Jl+X r.:

"1/ X

. (.J\+X
- IJT+X
-.fi+X + 1
)
.JX
+

= li m

--o

-o

= lim -::=-:--,==--.,.
jX (.Jl+X + t)

JX

lim -,===~
x~o ..Jl+X +

= 0.

l 1

J...i mil~

111

y
0.3

0.2

ye

,'i"'+:( - I

,r;

0. 1

0.6

0.2

O.S

The following is a second e.~urnpleor 1his 1ypc.

I EXAMPLE 2 .7

(Jx_+_3_- _:..:...:
2}( JX=i
+ 3)
Evaluate lim .:_:.
_ ___c---'.c- 1

X -

SOI .l '110N On~-c again we canl1<lt usc Theorem 2. 1; ihc dcrwminuto< Ita. limi10 liS .1' -+ I.
The graph of the func1ion tbr 1.001 ~ ,r ~ 2 in Figure 2.8 may suggest a lim.i~ bul cer1ainly
1hc evidence is far from conclusive. Following 1hc letld of' Example 2.6. we raJiOI>ali7.c the lcrn>
Jx + j - 2 in 1hc numera1or by muhiplying numerator and denominmor by .Jx 3 2.

+ +

. (;_;_
Jx
- _;o...:..;
2) ( ./X"'=~
hm
_+
_3_
_ _ _+
_3)

,_ 1

.l - 1

(JXTI -

2)(.JX=i + 3)(JXTI + 2)
(x - i)(,JXTI + 2)

= lim

,_1
(.r

= lin

4)( ./X"'=~+ 3)

(.r-I)(Jx+3 + 2)

<- I

= lim

+3-

+..::.3
..:..J.;.x=-=-'..::.

- Jx + 3 + 2

=
M:IIUII;I#W:L

.,

0.8 .

Saucsacd tlmi1 o( ( J,t

+3-

------

M
0.4 .

.r

~)(,;;-=-i + 3)/(.f- I) as .t t;Pflf\)JChts I

(\~- 2)(..rc=l .. 3)

0.2 1
o L---~--~~--~--~~---x
I~
I~
I~
I~

I EXAM PLE

2.8
sin 2.t

E'aluale lim - . - .
.r-o sm x
SO LUTIO'-; Onc:c ugail we cannn1 immediately use 1bcortm 2. 1 since the limit of the de
nominator is 7..Cro. Uul using the doublcanglc formula ~i n l.t"
2 sin x eosx. we fi11d that

!)inlx

lim - - = 1o m
sin.\
. . -o

2:;-in.tcosx

~n.t

-O

= lim (2cosx) = 2.
t'-O

You may fcclthm we nrc ovcrempl1llsizing limitS in which bolh the numerator and dcnomintnor
nrc llpproochins 1.cro (Exnmplcs 2.4-2.8). We stress this l)'pc of lin1i1 because when we usc the
c.Jdinit ion of~' dnlt'tUl~t in the nexc chapter. we alway~ cncoumcr this situation.

I EXAM P LE 2 .9
Do the functions

/(x)

;md

sinG)

h:wc limits as x !lpproachc> 0?


SOl lrl 10 ' Tilc.')C linul$ ore more diOicuh 10 find. To get afcdm~ for the behaviour of the
function :;in ( 1/.\') n~ar x = 0. we piOl its graph oo the interval -0.0 1 ;: x ;: 0.0 1 (f-igure
2.9,t): it is a \\FJ<hoot. The grnph fur - 0. 1 ;: .< ;: 0. 1 in Figure 2.9b i' more in,tructi,oe,
h shows that the fun" tion o>cillatc:.< back and l'u nh bct'"""'n I more wld more rupidly ~
npr>roo<:llC$ 0. ~~ n resuh. the limit of sin( l/x) does noc cxi<t a< x approaches 0.
IJCoriii.U

The run~ lion g (x) = x 1 sin ( 1/.t) has exactly lhc sanlC number of oscillalions as f (.<)
= i" ( 1/x). butt he tr;<;illt"ions becomesm;~llcr ond s m<tllcr 3S x '~Jtlrot>ehcs O (Fi~ure 2.10a).
In other words. limx- o g (x ) = 0.
liJ[4il;l #4lo)

)'

)'

0.001

- 0.001

'

y c ,~.2 sin ( 1/x)

0.004
'' ,

0.01

O. t

, .... .., r1' , .

o.or

-O.t

.'

-o.oo4 y= -.r \

21

We can confinn 1he geometric conclusion 1ha1 lim, _ 0 g(.r)


following 1heorem.

THEOREM 2 . 2

Limits

113

= 0 in 1his example using 1he

(Squeeze o r Sendw ich Theorem)

Suppose 1ha1func1ions f(x) . g(x ), and l1(x ) sa1isfy 1hc following two propcnies:
I. / (.r)

.S g(x) .S h (.r) for all x in some open imerval comaining x = a :

2. lim f(x)
x-a

Then lim g(x)


x-a

1lhtSir.'ltion
of the :lo((Ucc:.l.c theorem
)'

~X)

= L = lim h (.r).
1'-n

= L also.

Wilh Figure 2.11we can su 1his resul1 geome1rically. The graph of g(x) is always benveen
1hose of f (x ) and h(x) in an open interv-al around x = a (oondilion I ). Since graphs of f(x )
and h (x ) come 1oge1her a1 x = a (condition 2), so also must the graph of g(x ) .
For g(.r) = x1 >in (I/ .<) in Example 2.9. we lmow that -I .S s in (1/.t) .S I for all x . If
we multiply all terms by x 2 , we obtain

, , (I)
X

- .r- ;:: x- sln

y = f(x)

,:=:

.x 1

x Th.is shows thallhe.graph of x 2 sin (1 / x) is belween 1hosc of - x 2 and x 1 (Figure 2.10b). Since
lim, - o ( -.r 2) = lim, - ox 2 = 0, 1he squeeze theorem gives lim, - ox 1 sin (1 / x) = 0.

I EXAMPLE 2.10

.........
Use the squee-t.e theorem to evaluate lim .r cos
,.~ 0

SOLUTIO~

(~).
if it e xiSIS.
X

The function cos (3/ x ). like sin ( J/.t ), osci.llates violenlly as x approaches zero.

8111 we know <hat -I .S cos (3/ .t) .S l, and muhiplica<ion by lx l gives

Since - lx I is always less Ihan or equal 1o zero and lx l is always grca1er 1han or equal to zero,
we c.an write that
- ,.r l .S xcos

G)

.S lx l.

Since limx-o (- I.<I) = limx- o l.r l = 0. thesquee7.ethoorem requi res that lim, -ox cos (3 / x) =
0 also.

..-..

Here is a good question for you. Why in 1hc last example did we multiply all pans of the
inequality - I S cos (3/ x) S I by l.rl rather than .t?

One-Sided Limits
When we write L = limx-+<> f(x) , we mean that .f(x ) gelS arbitrmily close 10 L as x gets
closer and closer to a. But how is x to approach a'? Does x approach a through numbers larger
1han a . or does i1approach a through numbers smaller than a'! Or does x j ump back and fonh
between numbers larger than a and numbers smaller chan a. gro:1duaUy getting closer and closer
to a .) \Ve have not previously mentioned "mode.. of approach simply because iLwould have

made no difference to our discussion . l.n each of the preceding examples, all possible modes of

114

Ol:.p1er z UmiN and Cunlinuiry

PI>roat:h lead to the same limit. In panicular. Table 2.1 anti Figure 2.2 illustrdte that I is the
limit of f(x) = x ' - 4.r + 5 as x approaches 2 whether x approaches 2 through numbers
larger tha n 2 o r lhrough numbers smalle r than 2.

Approaching a number a either through number.> larger than a or through numbers smaller
than a are two modes o f ttppro::tch I hat will be very impon am~ therefore~ \Ve g ive lhem spec ial
notatjons:

lim l(x)

lim l(x)

:r- o-

x.-a+

indicates that x approaches a


through numbers smaller than a

indicates that x approaches a


lhrough nurnbcrs larger than G

(often called a left-hand limit


since x approaches a along
the x -axis fro m the left of a )

(often called a right-hand limit


s ince x approaches a along
the x -axis from the. right of ll)

Example 2.6 shoul~ in tUct, be de.s ignatcd a right-ha nd limit,

JJ+X - I
..r-o+
fi
Jim

0~

since the presence of .ji in !he denominalor demands that x be positive. Similarly, the lintit
of Example 2.7 should only be right-handed.
Do not interpret the - and + in a - and a+ as approachi ng a through negative and positive
numbers. This is the case only when a = 0. For instance. when a = 5. 5 is approached through
positive numbers whether it is approached from the left or fro m the right.
A natural question to ask is: What should we conclude if for a function 1 (x)

'I

lim f (x)
." r-(1..

lim f<x)/
.r-..(1-

Our entire discussion has suggested (and indeed it can be proved; sec Exercise 20 in Section
2.6) that if a ft1nction has a limit as .x approaches a. then it has o nly one such l imit; that is, the

liJltit must be the same for every possible method of approach. Consequently, if we arrive at two
d iffereot results depending on the mode of app roach, then we conc lude that lhe functio n does
not htwe a Iimit. This sjlUation is i1Jus1nucd in Figure 2.1 b and again in the following exam ple .

I EX AMPLE

.........

2 . 11

. lxl.
1 poss1.b le, hm
E \laluate, r
,~,

Mjlc;jii;I+BFJ 1 lllusu-ntion
or no hmi1 a'\ x <"l ptm)aches Q

...... o

SOLlJflON If x < 0, then

lxl = - x, and
.

lxl

hm -

y
)l =

.-.-o- x

txt
-

'

If x > 0. then

Bim
x- o-

- x
.t

= -1 .

lx I = x. and
Jim

x ~ o

lxl
x

lim
= I.
.r- tP .:r

The function has a righ t~h and limit and a lcfL-hand limit at

x = 0, but because they are not

the same, lim.,-o (lx lfx) does not ex ist. The graph of I (x) =
illustrates the situatjon.

lx 1/ x in Figure 2.1 2 clearly

I E XAMPLE

2.1 2
11\e \\'d,1\l \V Q( lUl obje~;t depends UU it&cJi.:MUDiX ( / from the l'Cillre 0 1' tl\C earth. lr d is I~
tha~ the radius R of the earth. then W i,., dirccll) pt'OpQitionalto d : and if d is gteatc-r than Of
equal U> R. then W tS 1nverM:I)' propon tonl\1 to d 2 If the wciJ.ht of the o b;oct on th4.: eanh's
surface is W0 , find o formula fM \V us a futlttion ford and dr.'w it~ grGJ>h.

SOi l'I IC)N


\\httc k and

When d < R, we mo.l) write that \V c lccl : und Y.hcn d ~ R , W t Jd2


arc cons.tanu;: or proponiooalit): that is,

IV

O :S d< R

ktli.

IV =

R :!:. d.

cP'

Since the weight ()f the object i~ lV0 on the s urface of the cnnh wheo d = R. il follows that
W0 l/ R 2 fr'ont \\hich t.
W0 R 1 h now re1n~in., to fi!ll.l /.. . If "e physic<-IIY m.Qved the
objecl fmm below the surf- ofthe.an h to lheourf~~Ce. il would <IQ\\ I> grun \\tight. I" weight
would approach \\' 0 that on the surfocc of the eanh. ln other \\ nrds. the 1inut of lV UJ r __,. R
must be W0 ; 1htll is.,

Wo =

hm
,_,

=k R

kd

IVo
R

k =

Thu.:,.

A grph is sho"n on Figure 2. 13.

E X E R C I S ES 2 1

.\ l - fu I+ 1\.t- 6

19. lo m ::._....;:::._:....:..;.::,_..__;;
-

_. >-s

2.

I. l i nl - -

-7,x l 2

._,

.l. 1om

'

s.
7.

.... ~ J.r + 2

"' +25
~

2.1

lun - -

. ..... ) t l -

\ <4+5.ri

tun

:'-'.\'" - ,(J
{2

"'

x' -

llm

-s-

,_,

13. li Ill

25
\ - 5

A'-Ax+4
.t - 2

X' - 6X~ + IIX - 6


IS. lim
x:: - 3x + 2
- o
17.

.. _,tim

A' -

.\ ' + 8

1irn - -

' --t.r+S

...,

f2='X
.. v~

1 L Inn

x'+J.r

.. '"' 3x 2 - lr
4
b
6. lim
~ -~ :\1 + 2

lJn.t

l4.

2.1. lion -.-

-t ;.;an ..\

11m
I

.,,~

i;inx
Ui.ft ;l;

2x + .i
8. lim ' I
x - 3

,_,.

10.

' -~- lim2


' :: X
II.

.h+2

x1

+s

12.t

10. bno
- .\l - 2:t-+ \

lim
,_,,

,_,

12. lao\

,.

.t

sinl.r

n. ,-1nntt

,, - 2.r - 3
1-x

29.

'-6x'+ ll.r-8
-l ,,. - 4.\ + 4

UU1

lS. un

jT.:..'_-.:.\_-_-1!...:...
' +~,\

\unI'

-.t

).

,_,

16. lim

6x' + l lx - 6
18. tim
x- 3x 2 -3x+ 2

llx-6

J l . hm
, ~,

lx'

r;,

.JO.

14. lin'\

x'-6x~+

x- 2

.. -,..{X- v2

251

x1-2S

x' -

31. ,_,
tim ,x..

.r' - 3x + 2
_.I-

6x 1 + I IX- 6

.r ::- 3x + 2

---

33. tim

34. lim

,_o

1-

251
u

1-J

JXi+i
..

l.r

--1 ,FX -

37.

+ 38.

../1_
+_
.T -_..fi=X
lim ..:....
.:...__ _

.'(+2

35. lim

,fi

1- 0

../x + 3 - J '

lim

.--1
Jx + 2
lion~
l-0
X + 4-2
1

JT+"X- ..!~=X
39. Inn "'===-~==
- 0 ../2 + .< - .j2 - X

i "'

40.

* 4l.

..;x::FT -

56. E\'aiUIItC lim

-o

1 -e-l f.,

liRl

SS. A t the present time it is impossible for us tocalculalc algcl>r:tic.1Uy


sin .T - .T
lim
. \Vbat can we do?

-I

> 0 is a oons tanl and calculate lhc

h1 E.:<crciscli 42-49 aS-')u.mc lhat a


i r it exists.

xJ

(aJ One ~ussestioo might be 10 usc a calculator or computer to


c'1lluaac th<: function (sin x - x )f x J for various '\1llucs of
X that approach 0. Try 1his '~ith x = 10-. n = I . ... . 7.
and make any conctusion thai )'OU feel is justified How
would you ftcl aboul lhc usc oft.bc: calcularor if you knew
l halthe ' '"d lueofthc limil were - 1/ 6?

Jx +3-2
..:....----,.X

What does ohe


l + c- 11'
plotsuggest rorrighthandand lcft hand limits or f(x) asx "''PfO""hes
O? Whill is f(O)'!

J3x+ 4-../2x+ ~
,1- l

S7. PlOt a graph of the runcoion f(x)

"- o

J'2X+T

11111 -:.;;:=~-=-:;;;:,;,=

../x + h - JI

limi t~

(b)
x~

- a!
42. lilll - - .

- 0.1 5 X 5 0. 1. - 0.0 1 5 .l 5 0.01. - 0.001 5 X 5


0.001. :u1d -0.000 1 S x 5 0.0001. What happens?

,\ ' - a

Y-tl

.1 3

A I"'IOhtr s ugg~tion n\i,Sht be 10plo1 the function on smaller


nnd srMlkTintcl"'o'als ;)n)UJ\d x = 0. Do lhis on the imcr,als

-a'

43. lun - - -

x -a

.x- .e

.r+a

im ....,.,--- - 44. ,. l__


"
+
x- ll
.r~

ll .~-

. J.._;;,_
I -..;:.fii
..;,.
a

46. l1nt -

,_lol
.

.t - il

_.. 59. Assume the cxiMcncc of lim

sin lax
+ 45. lirn - - -

,_.u

T-d

6(). tr /{.T) is an C\-cn function and lim. - f

47.

../x'+a'-J2x' + a'

Whal d11e;;. it suggest for1he li mil oft he func1ion


be cooJinncd in Scc:tion 3 .~.)

:1 ~

x-,

all x

OC'

63.-6~ .

find. ir

= l., find. ;f

Rcp<aoF.xercises 60-62 ror an odd runclion / (.<).

F.""'"'

T- Il

62. If /(X) is un c,cn funclion und lim, _ u /(X)


PO>>ible. lion,_ ...,, /(.t ).

= L.

60. If lim,_0 f(x ) =


do you conclude oboutlim,_ 0 f(x)
,;n(l / x)? /lim: Sec EJ<Ompk 2.9.

52. lirn .t" cos (~)


,'(

53. Ooc-s tbc ftoc.N" fu n<:tion LxJ o r E<crcisc 68 in Scc:tio n 1.5 have a
lirnit, a rig.ht-h:llld I imit, or :a leO-hand limit :l5 .r ;approaches i nteger
values?

54. Prove

(."r) = I... find. if pos-

61 . If f(.x) is an evc.n runcoion an~ lim, ' /(x)


i)OSSible~ limA--Q f(x) .

0 7 (fhis will

l1t Exercises S J and 52 usc the squcca thcoccm to disc:tbS the limits.

. If lim ,_n /(.t )

:libk:, lirur~- /(.t) .

ja+ .t-jli-X

-lim
J.x sin(.!.)

.f - II

L . find lim, _ . g(x) .

Sl.nll X

+ 48. Inn
+ 49. lam /~F:'=.--r.:"f":"':
-
.<
-II JJ.r' + 4 - ../2-r' + 4
!I+ 50. Plo<agrphor /(.1) = ( 1/ xhin,\ unobc intCf\al - ;r 5 x 5 , .

St.

f(x)- g(x)

67. If a. b. c. and tl

;m: con~anl~

riml dte following li111it.s. if lhcy

CXISI:

d;sptovc the following slotcmcnt: 1r /(.<) < g(.T) fO<

- a. then .r-a
lim/(.<) < lin1 .~(x) .
,_lit

55. If n is a positive integer. cvaluaJ.c lim

(x +h)" - x"

-o
c;ilhcr the binominl lhcorcm or lhc rcsuh that

.1/illt: Usc

a+ ce
, _ ob + tlt

11

"'

(<) lim -'----~


1l~

12.2 Infinite Limits


Fuoc1 ions do 001 always have limits. Sometimes 1his is d ue 10 1he fact that right~ hand aod left
hand limits are not idemic:al; sometimes it is a result o f eaTatic osc illatio ns. Exam ples o f both of
these siouaoions were discussed in Secoion 2. 1 (sec Examples 2.9-2. 11). Nonexisoence of a limio

2.'2

lt~fini:e L.imil$

117

may also be due 10 excessiely large ,a lues of lhe func1ion. For ins1ance, consider I he funclion
/(x) = l/(x - 2)', which is not dcfiocd a1 x = 2. Docs it ha,c a Um.it as x approaches 2
Table 2.2 indica1es 1ha1 as x approaches 2 , values of 1/(.t - 2) 1 become very large: in facl.
values of lhc function can be made arbitrarily large by choo.,ing .t sullicicntly dose to 2. Thus.
1he func1ion does not have a limi1 as x approach..os 2. We express lhis symbolically in lhe fom1
lim

-> (x

2)'

= oo.

The symbol 00 represents what mathematicians call infinity. Infinity is nola number: it i.s
simply a symbol that we find convenient 10 represent various ideas. In the equation above il
stales 1ha1thc limit docs not exist. and indicates thatlhc ncason i1 does n01 cxisl is Ihat \'alues of
lhe funcrion become arbilrarily large as .;r approaches 2.

The graph of f(x) in Figure 2.1 4 further illusua1es thb poin1. We say 01a1 01e line x
is a vertical asymptote for the gmph.

1.9

1/(x - 2)'
101

2.1

1.99

101

2.01

1.999

106

2.001

1.9999

101

2.0001

1.~

10

10

Unbound:<!

bdm\'iour ot 1/ (..l' - I) near A

=I

.~

\~

SOLL'TION In this example we consider ri~lu- and lefl-hand limits as x approaches I. We


find that

.r-1

;I

;2

2.00001

I
E.valua1e lim - -. ifi1exhas.
A- 1 ,\'- I

y= - -

.\

I EXAMPLE 2.13
(!l!Lt;'

Unb~Ju;oc,b.~ bcbd\iuur uf 1/ (A - 2)1 nut " - 2

lif-J, J!#f'W

=2

lim

.t-1 ' X -

= 00

and

Jim
..t-t

:c - I

= -o:>

1he Iauer meaning that ns .r approaches I from 1hc left, 1he func1ion takes on arbitmrily ''large"
negcuhe values. Ei1her one of these expressions i~ suOiciem 10 concluc.k: that the function
I/ (x - I) docs not have a lintit as ,r approaches I. The funetion is shown in Figur~ 2.15; the
1ine x = J is a venical asy-mp101e .

..-...

I EXAMPLE 2.14
Discuss left- and righ1-hand limits of the func1ion

f(x)

as x -> I and x -> -2.

x2
x2

+ x-

MUclli.I#WI-S'

Ul'lbOUillted beh;wiouJ' .,r (X~ - ~)/(X ~+-" - 2) near-" = -2 cu'lll

v:

x= I

x 2 -9
2
x +x- 2

SOLliTTON The function is undcfiucd at x = I and x = -2. (\Vhy 0 ) Its graph is shown
io Figure 2.16. To coofirm algebraicaUy what we see geom.etrically, ll is advantageous to factor
numerator and denominator of f(x) as much as possible,

j (x ) =
Con.side.rnowlhe(right-hand) limitas .r

x+J-> 4,

X-

(x
(x

+ 3)(x + 2)(x -

3)
L)

1+ . Each ofthe four factorshasa limitas .r ~ 1+:

X+

3 -> -2,

2 - 3,

X-

I -> 0.

\Vere we m combine lhem according to Theorem 2. 1. we would wrice

4( - 2)
:J.(O)
IJ1 sp ite of the fact that this is not correct - [part (v) of the theorem does not allow a 0 in the
denominator - this expression lets us sec wh at is happening to f (x) as x --> I+ . It sta tes
that as x --+ r t , the numerator of .f(x) approaches -8 and the denominator approaches 0.
But this n1e-<1ns that .f(x ) must be taking on larger a nd larger values as x -> I+. Are these

values positive or negative? The numerator is clearly negative. bul lhc sign of the denominator
depends on whether the 0 is approached throUJgh positive or negative numbers. Recalling 1hat
0 arose from the fact that x - I ...., 0 as x -+ I+. we can be more specific: x - I must
approach 0 through positive numbers since x > I for x ...., I+. We indicate this by writing
X -

I -+ o+as X-+ I + . The fraction displayed above is therefore replacetl by

4( - 2)
3(0+)'

and it is clearly negative. In other words,

.
xl - 9
. (x + 3)(x - 3)
= hm
= - oo,
x-.1 x2 + x - 2
x- t (x + 2)(.t - 1)
hm

as indeed Figure 2 .1 6 indicates. S imilarly, as x ~ 1-

the fraction

4( - 2)

3(0-)'
which shows limit> of the four factors off (x) , indicates that
.
(x
hm

_, .... ,- (x

+ 3)(x
+ 2)(x

- 3}
- I)

00.

2.3

1\t yrc
I'
" \" In the follow irlg. the fractio ns on the ng
u rcsu1ts a.',
lim

(x

+ 3)(x -

+ 2)(.< (x + 3)(x -

., __,, (x

Jim

, __,

(x

3)
I)

l,.u\'lhs m )nt\nhy

118

- 2 + ondx -)> - 2- :
(1)(-5)
(0+)(-3)

oo.

(1)(-S)

3)
= - oo.
2)(x - I)

co-><-3)

fn the la~t1imit of this example. we did r1ot w rite


.
(.<
hm
,,_, (x

+ 3)(x

- 3)

+ 2)(x

- I)

( 1}(- 5)

= (0- )(-3) = oo.

To include the fntclion ( 1) ( - S) as pnn uf a mathematical equution is rlot a cceptable: di\'iSior1


(0- )(- 3)
d
1
b t lonot
by 0 tc; iml:x.t~iblc. Phacc
the l'rac1ion to the rrg.hl of 1he I umt to tll ul n s C\'H uat\Qil. u c..

include it in the cqurHiCMl.

EXERCISES 2.2

* 21.

In E.~CC('ci:scs 1-24 cvaiU#lC the limit. if il c:<l5t$.


I.

lim - -

.~ - l: t X -

.\ - !

4.

J. lim - -

'

~l

:r- 2

5. lim

'

7. li m
,._, t

9. llm

8.

I J'

2.r + 3

15.

1f/ l '

12.

lim sec (x -

"/~)

lim 1an.x

x 1 -lx +I
17. Jim
' l x' - 3.r' + 3.o - I
2x

19. lim - - ===

- I- JXI+I

14.

lin1

, _ if,~

l.r-a\

Z6. lun

"fl,t l -!(u+a

6X 1 + 1:C- 5

2.r - I
Jl -

,_,.

\ -J'T/4

In Excrcis~:s 1:.}-2$ \t5nnne ~hat a > 0 is tt c:onst.ant ;;uld caku1t'H: the


limit. if il e.xillL~.

2)'

28.

10. linl
.l'-- l.r-2- 4.\'

II . lim c~.r

13.

' l (.r -

---- 1}2

.r ~lx2 - 2.t+ l

lim <,JI.rl

lim

lim

f _, ()

- 1

, -.: (X - 2) 1

5.
(X

.f

6. liol

{.r- 2)'

r - 1ni X- I

u .

2. I im

-. ll. lim

lim In (4:c)

.- - o ~

+4

scc(x- JT /~)

i: .tt

(<t) Cato.l~le .t l,

.
\4-x\
-,<'- Sx+ 16

In .r, ~nd x 11n x fQI x

10

1
'.

tJ

1 .. . 10 . und uK thi~ rnftmt\l!tioll to decide Or\ a vnl\te

fo tthe limi1.

tan.\"

20. lml _,...:......,...._.:_

, -.o

thilllim,_.),. .x1 0 :md 1im_._ 0 li\X


- oc. h dc~ds on which
t(:ml tS 111orc domin:mt in the pmd\l(.;l, .x1 o r b\ .~.

litn C'CM. ,\"

'

n\

2?. ll is notclear whtlhcn hclinlit lim x : lnxc:<i!>lSdu ctothc t'ac'

, _ !t+

Itt. ' lim


:r/!-1
.. 18. tinl Ji+X
x'

linl ~l l(l 'l

x ..... -(J

(b) PI<H graphs of .r 2 ln .r ncor X


lion in pnrt (a).

i * 30.

0 to COftlinn ynur cukulu

Rcpcut. Excreille 29forthclimit lim .~ 10 c 11 . butpiek yourown

v-.ot
values of.\ m which lO e,uluate x 10 um'

e''' .

12.3 Limits at Infinit)'


In many applications we a re concerned with 1he behaviour of a function as its independe nt
var iable takes on very large values. positively or negatively. For instance, consider tlnd ing:. if
possible, a number Lhatthe function f(x} = (2x 2 + 3)j(x 2 + 4) geLS closer and c loser Lo as x
becomes larger and larger and larger. The graph off (x) on lhe inLerval 0 :;: x :;: I00 in Figure

120

Ch.'D't' 2 l imits :.nd Con,inuil)'

2.17 suggests thai function values are approachi ng 2 for large x . To confinnthis algebraically,
we divide the numerator and denominator of .f(x) by x 2 ,

f(x)

+3 =
=
x2 + 4
2x 1

.J.. -

For very large .r, the terms 3/xl and 4/x~ are very close 10 zero, and therefore f(.t ) is
approxima1ely equnl 10 2. Indeed, f(x) can be made arbitrmily close 10 2 by choosing x
sufficienlly lal';e. In calculus we express this fac1 by g,ying thm the limit of (2x 1 +3)/(x2 + 4)
as x approaches infinity is 2. and we wri1c

Once again we stress that oo is not a number. The nouuion .x ~ oo simply means ''as ;c gelS
larger and larger and larger." We say that the line y = 2 is a horiwntal asymptote for the
graph of the funcrion f(x) = (2x 2 + 3)/(.<2 + 4) .
\Vc can aloo fiud lil'nilS of funclions as x takes ontubitrarily large oc.gacive numbers, denO(ed
b) x -. - oo.

PIOUftE 2.17

Lintil of (2x1 + 3)/(x~

+ 4)

for lm-e,c x

2.f2 + 3

1.4

.~~--
,i'l +4

1.2 .

20

40

100 .

80

0.8

We can also find limits of functions as x take.< on arbin-arily "large" negative numbers.
denoted by x -+ - oo.

I EXAMPLE

2 . 16

E\ah13(C lim

5x- 3x + 5

x--oo .tJ - 2x

SOI.lmO'\

+ 5 . i ril ex ist.S.

Divbion or nurnel"ator and denominator b)' x' leads to


J

lim

x.,..-oo

5- -x3
2

5
+ -x4
5

l - -x1 + -x4

= 5.

2.3 Limits:u lnriaJt)'

MjliJII;I#MI..

limits: ~>f (5.\' 4

-2

3.~ +S)J(.'t~- 2x!

+ 5)

( targ~

positiw ud ncsa:ivc

121

If the limit in Example 2.15 is 1aken as .r - oo. the same "">Uh i~ obtained. The graph of
this function is shown in Figure 2.18. The line y = 5 is a horizontal asymp101c.
In gcncn1l. we say that a line y = L i> a horiwmal asymptote for the gn11)h of a function
f(.r) if either, or b<llh, of the following situations exis1:
lim J(x) = L

x--x;

lim /(.r) = L.

or

x-~

In Example 2. 15. both of th~ condition~ are satisfied for L

= 5.

I EXAMPLE 2.16
Evaluate the following limits. ifthcy exist:
(a)

(b)

2.r1 - 14

lim
, _ - oc 3.rl

2xt - 14
(C)

+ Sx

lint

x - c.x;.. 3x.l

+ 5x

sou.mor.;
(a) To obtain this limit we divide numen1tor and denominator of the fraction by .r 3 :

4
2 -.)

= 2.

1+-+x .r3
(bl Once again we divide numerator an.d denominator by .r ':
2

lim
, __
.,., 3.r3 + 5.< =

14

. -;-\.3
lam
.

x--"'0

3+ -xz

Since the numerator approaches 0 and the denominator approaches 3. we conclude


that
2.<2 - 14
lim

0.
- - Oo

3.r' + 5x

\Ve could also ha,e oblained chis l i mit by di viding numerator and denominator by .r~

instead of .r 3.
14

2- -

xl

3x

+ -X

Now Lhe numer.ttor approaches 2, but since the denominator becomes very large, the
frac tion once again approaches zero.

(c) l);~ision by .r 3 in this c-a~ gi ves


14

lim

2x 4 -

.t-~

3x-'

14

+ Sx

2x - =

x3

lim

.r-oo

3+xl

Since the 11umeratOI' bec()lll e.~ ;u'bitr.nily bu-gc us x l!J)J>roaches J. it follow s that
lim

2.x.l -

.t- - 3A3

14

+ 5x

oo und the denominmor

= 00 .

I EXAMPLE 2 . 17
D rti\V v. graph llf the function

f(x)
Doc~

-sin .<.
X

x :!;

1r.

it have a hori7.ontal nsymptotc1

\ Vc druw the graph in Figure 2.19 by making the osdllations of sin x become
maller Clnd smaller as .t gets li.ll"gcr and larger. The g,raph indicates that the positive x-axis is a
huritontal a~yn1 ptotc . This is confinned by the fact that
SOLUTION

li m - S'in ..t=O.
x-oo x
\Vccoukl hnvc rea~OilCd thi~out a~ fo llowN. The functio n sin x docs not haven limit as x ..,. 00.
x ~ 00 i1.5 values oscillate back and fo rth between I. S illcC the-l)e values are multiplied
by I j.x. w hich i s gcUing smnllc and sm~lle r, the J>rOI.IUCt ( 1/x) si n x must be getting closer
and closer lO 0. " 'c could also use the squeeze theorem to arrive at the same limit.
Notice that the graph actually crosses the asy mplote an i11fi11ile number of times..
l:~.1tas

UI! I I I i I illf'iiT'
I

y=-Snl.r
X

l;r

Figures 2. 17, 2.1 8, >Hid 2. 19 i ndica1e 1ha1 1he g raph o f a function y = .f(x ) can a pproach
an asymp101c y
L i n 1hrcc ways: fro m above (Figure 2 .18). from below (Figure 2. 17).
nn<.l oscillating abl>UI lhe asymp101e, g m<.lually geu.i ng closer and clo.er 10 i1 (Fig ure 2.19). A
computcr-gcncmtcd grnph m 11y or mny not "lwnys make it clear which siluation prc\'ails.

I EXAMPLE 2.18

.........
Plot a graph of the func tion f (x) = (3x - 6) f (x 2 + 5) . Indicate any h01i <on1al asymptotes
and determine how 1he graph approaches 1hese asymplotes.

2 3 Umi1s ;1( lnlinity

MjtUIII#WIJM

123

Horitonlal. u.-,.ymptocc of (3x - 6)/(x ~ + 5)


j )'

- 100

1.00

50

-0.5
-{).7

3.(- 6

y~ -,-

-I

X"'+5

- 1.25
- 1.5
SOl UTION We begi n by plouing u graph <>f l hc func1ion on th~ domain- 100 ~ x ~ 100
(Figure 2.20). It suggcsiS t hlll y = 0 is a hOI'i zo111al ll~)'lllJXOIC. nnd 1hm the graph appro<~chcs
y = 0 from above when x ~ 00 Hntl from below when x -+ - oo. We can confim1 thi~
a.lgebraic.ally in vari<HtR\Yays. To veriry 1hc siu&<uion as x ~ oo , we cnlcul:ue

6
J - -

3x -6

lim - - - = lim _ _,x_ = 0.


-ex..
+ 5 fi-oc
x + -S
.xl

This contlnns 1hat y = 0 is indeed a horizol!ltal asymplote as .x _,. oo. To decide hov.rthe
asymptote is approached, we note 1hat 10.. large positive x , both 3 - 6/.x and x + 5/x arc
1-.osltive, a11d therefore. /(x) must approach 0 through positive mambers. \Ve i1 1d i ~h! thi:o~. hy
writing

3.< - 6
+5 =

li m
x-oo,xl

o+.

We also could have reasoned M follows. T ne g,raph of 1hc fu11etion eros~ the asymptote y
when

3.x - 6
,x2 + 5

=0

0,

and the o nly solution or this cqumion is x


2. Combine this with the g mph in Figure 2.20,
and we conclude 1ha1 f (x) > 0 lo.- all x > 2., and hence 1hc t~raph "' ""approach y = 0 from

above as x ~ oo.
Similnr reasoning shows that lhe graph approaches)' = 0 from below as x

- oo.

Tn the following example, 1he function is lll()re complicated. Jt discusses a11 alternative way
for determining how g.mphs of rational functions with horitontal asymptote...,;; npprooch lhc.sc
asymplo tes.

I EXAMPLE 2 . 19

Find vcnical and hOt17-0ntal asympt01es for 1hc graph o f the functi Oil J(x )
(x 2
x - 6). Determine how the graph approaches horizontal asymptotes.

(.12 - 16)/

SOLUTION The plot on the interval - 10 :;: x :;: 10 i n Figure 2.21a suggests vertical
asympto tes al.x = 2 and .x = - 3. T hese are confinned with th e followi ng limits:
.

(x

hm
_,.... z+ (.x

+ 4)(x + 3)(.x -

4)
2)

= - 00,

6( -2)

(5)(0 +)

lim
... - 2

(x

+ 4)(x- 4)

(.t

+ 3}(.t -

+ 4}(x x- - J+ (x + 3)(x (x + 4}(x lim


- - J- (x + 3)(x (x

lim

2)
4)

lim

X - 00

U l!Iili
(A ' -

I! IEifi 4

16)/ {x' +X

'

X-

16

+ .\" -

2)
4)

(0+)(-5)

(l)(-7)

= -oo.

2)

(1)(-7)

(0 )( -5)

l is a horiwntalusymptotc as x --> oo. To

16
x2
Ii m - - -;1....::.--::6,....

.t _.OO

Su&,-eMcd borizotllal

<bymptot~ or (.t ' - 16)/(x:

,.

I.

I +--.
.!
x-

MJI.III

\ 'Cnical asymptotc5 of

6)

{ 5)(0 )

= 00 .

The plot in Figure 2.21 b suggests that y


verify this. we calculate

x2 -

6(-2)

= 00.

+A - 6) a, x

o..

y
1..0 1

15

x 1 - 16
10

5
10 X

-5

-10

-5
-10

- l5 !

FIGURE 2.21

20

SuggesaOt.l horiZ<Iotal O.S)'mptiXe: of (x! - l6)/(x' +-:r - 6) ns x -

60

80

10{)

- oc

0.8

0.6
0.4

0.2

- 30

- 25

- 20

- 15

- 10

-5

To show whether the graph aprroaches this horizontal asymptote from abo' e or relow. we t<c
long d ivision to express f(x) in the form

f(x) =

I0} / (x 2 + x - 6) is positive for large x, it fo llows Lhal f (x} is less than I for
large x , and the gmph approaches y = I from below as x --> oo.
Because (x

x 2 - 16
X+ 10
= 1.
2
x2+ x - 6
x +x - 6

2J

lin\it$ at hlftni l)'

125

The plot in Figure 2 .21 c indicates that y = I is also a horizonwl asy111ptote as x --> - 00.
S ince (x + 10)/(x 2 + x - (j) is negative for "large" negative x . it follows that f (x) is greater
than I for such x. and the graph approaches y = I from above.

Previt1UN gruph:-. in this se<,.'tion have had


example has two.

I EXAMPLE 2.20

al

most one horit.ontal asymptote. The fo llowing

..._..
2

J 2x + 4 1'f tt ex1sts.

x-oo x + 5

E vaI uate I .un

Wh
s ~r -+ -oo.
at ts t hc 1tmtC

SOLL'TION When we d ivide numerator and dcnontinator by x , and take the x inside the
square rooc as. .r-.
'
I

J2.t 1 + 4
lim
.l-OC
x+S

lint
x-oo

- hx2 + 4
X
I
- (x
.t

+ 5)

lim
x-oc

~=
2

I+-

Ii m
x-oo

I+-

./2.

l n evaluating the limit as x ~ - oo>we must be extnl careful :

.J2x 1 + 4
. . - -oo .r + 5
lim

lim
= s--ao.

I+ .r-

It is not correct in this case to take .1 inside the square root as

since for negative x , the expression on the left is negllli\ C wl<lthut o n the right is JX!Sitive. In
this ca<e. we should replace x by -

J2x' + 4
.r

JX2. and write


_
-

J2x + 4

_ _

-# -

f2x> + 4
.t l

Hcn<.'C.

- /2 + 4
X

+5

lim

_, _ _OQ

xl

I+ -

- ../2.

The graph o f this fu nction (Figure 2 .22) confi rms these limits; there are Lwo horizontal

totes, y =

./2 and y = - ../2.

asym p~

126

Cbapkr 2 L1n1it! arid Cuotinit)'

I - ./l.l: + 4/ (X + S) iii1Stnllli1J

FIG URE 2 .22

t'"' """'"''til ") "'""'"'

)'

y=

10 (

x +S

st....
--*---29--~- -~~

20

10

1- JO!
j

rs
Graphs of Junctions have horizontal as) mptotcs if ctther. or both. or the con<litions limx-oc
f(x ) = L hold. What this means is that for large positi\e ornegdtivc vulues ofx , the function is
approximately equal to L . The larger the value or x, the bcner the approximation. Hyperbolas
also have asymptotes. but they are not us ually horizontal. For cxam1>lc. asymptotes of the
hyperbola x 2 - y 1/ 9 = I in Figure 2.23 are y = 3.r. These are often called oblique (or
slanted) llsymptolc< .

__
>' ,

PIOUfU! 2 .

' . -J.r

FIOURE 2 -

IS

....'-

-10
- IS

30

. .............

..,

,,
'

',

Graphs of function" can alw ha>e oblique: :ll>} rn)llo:>. In Fis urc 1.35. y = x - J i> an
oblique a>)nJJ)IO(C for the graph of j(x ) = (,r 3 - J.1 2 + I )/(.1 2 + 1). We can confinn rhi>
algebr&icall) if "e di1ide x 2 - I into x 1 - 3x 2 + 1. 11le r~ult i;
/(.1)

x l - 3x 2 - I

'

.r- .,.. I

4 - .r

= .. - J + ,

.r- +

A x -. co. the tcmr (4 - x )/(x 1 + I) -+ 0. and therefore /(.1)-. x- J . For large


values of x. the function f(.r) can be approximat<>d more an~ more closely by x - 3. This
reprcscmation for f (,1) can be ~mo-d to show that the ~CJ>iction of the usymptotc in Figure 1.35
s incorTect. For large positi\'e x . the rem (4 - x)f(.r 1
I) < 0 , so that .f(x) < x - 3 and

the graph ; hould approach the asymptote from below, not above.
Rmionnl function~ P(x)/ Q(x) always have obliqueasymJ>totes when the degree of polynomial P(x) is exactly one more than the d egree of polynomial Q(x). A second example is
the function /(x) = (2x3 + 4x 2 - x + 1)/(2- x- x 2 ) in Figw-c 2.24. It has an oblique

2.3

asymp~ote

= - 2f -

limits a Jntiniry

127

2 identified from
x+S
2- x- x l

= -2x -2 + ---~

II also has two \ Cnical asymptotes, one at x -2 and the other ut x = I.


Functions other than rational functions can also have oblique asymptotes. The graph of
{ (X) = X + 2 .l. 5e-X in Figure 2.25 is asymptolic IO )' =X+ 2. The SlraightllCSS of the
graph of /(.r) = J (2xJ + 4.r2- x + 1)/ (.rl + x- 2) for large positive x in Figure 2.26
suggc~tl> an US) mptOIC. It is .fi(x- 1/2), but to ; how this algcbrnically is dillicult.
Ot-Aiqooe
:b~ nproet:

of

+ 2 + St

'

a;~~uoe

OI>J.que
ut a rooc I'IM<tl\111

)'

25

20
15
10

r.\ -+-2+5r"
5

10 .r

10

15

lim

- cosx

20

EXERCISES 2 .3
In

J!>crc;,,.,. 1-38 C\-.Juaoc lhc lumt, tf ol o.t>IS.

2I. lim J.-!);)

' +-I
I . lint ~ -1'\)lx - I
x' + 1

J. lim

7.

lim
, __ "'

hm

-""'

lr' + .r +

b '+ I

,_...., 2.r +I

hm

10.

11.

,_., <x'- > '>


.r

17. lim ....,.~=


( ... "'0 Jx + 5

'

lim 1 ~
, __
,., v4+x'

* 25.

J-..-"" 1x

x3 - 2.r'

lim

+ 4..\ l

27.

x' -lt' +x+

lim

,. ___ ....,

sin 1 A

;rl -

+I

24. lim - , _"' :t

x2

2ti. lim - t:m .r


, ........\) .r

lim lan X
v--oo

lim (~ -

.r)

-oo

29. ,_,.,
lim (, ltl + I -

r)

J.t- I

lion r.;=:=,=,
,_,.,
Js + 4r'

Jl. lim

........ ~

. . . . .\" + -

IS. lim

_._I'Q

t -4x'

.c---v J.c l

8.

Jt- 2x"
.,

13. ,_.,.
Iillo

19.

6.

x+ 3x

U.

sin .J.x
23. l i m - -

., 3 + 2.r - x

9. bm -'----'--

.. ll .

lim - -

,_.., 3+ 2x

J ~ta 4 .\

lx

cosx

1->

4. lim

+
2 + x - x'

... I'\. 1
-\

S.

2.

lim

...

./I - 2r

.....,xt~

33.

lim

,--~

-"" (x+ ~)
.\'

18.

lim

X+ 4
.J3:r1 + 2

34.

Xi- 4

'--CQ v J - x

1<

. (.Jr-;;-37. lull
.r + \ ' .t.-oc

Jx

20. lim r.~o;=;


.-oo ;;2 + 4x'

JH.

lim (Jx' + .r -x)

..,.... _ 00

li m ( /2,r 2 +I -

, _ _ "IC

lim

J~r;

+1

- " lr+J

* J6. ,_..,
lim (U o+ .r- :If)

x'

r.;--::

30.

J4_.l + 7
32. hm -":--:--:, .... "' 2x+J

14.

16. Inn

2K. ,_.,
lim (Jx'+ ~ - .r)

x)

ln Excr<;i$C$ 39-42 ussumc thai a > 0 is a coostunt and ~ukull!tc lbc


litnil. if it existS.

... 40. r~
lim -;:=::;~:::;:::=;:"'
oo Jaxl + 3x + 1

39.
41. lim
A ,. .

(lx 2+ax-x)

42.

Jax' + 7

lim
_. ....,.

X-

3a

45. f(x ) =

47. f (x )

.lx- 1

46.

J5 +2r '
I - 4.t 3

= 3 + 2.t _ .f,

+ 48. f(x )

2- x

/ (X)= - -

3 ,. 4.t

44. { (X)

2.< +3

+ 2x- I
= 3x'
,
1 - J.r + x

Jax 1 + bx + c

.~ - -c

tlx + e

.._.. 50. What conditions on tbccOO$tanlS a. b. c. d.

c. and f

" ill ensure

lhoo
,I

* 43.

Jsx + 1

-,------,-

49. What i~ the ,aluc of lirn -'--,-'---'-- .where a > 0. and

b. c. d, and e are con.stams?


In Exc(ciscs 43-48 idc.ntify ull horiOnlal, \'Crttcal, and obliqucasympIOCCS for the &mph of the function. l>eaermine whether the gmph approaches horizontal and oblique $}'tnplotes: from above or beloi.Y.

I (x) =

x+3
=2t- 5

l im
(/
H,.;l

ax'+ b.r + c-

/ d.r'

+ ex+ f)

exists? What is the ' '41uc of the limit in this ca.sc?

12.4 Continuity
We have noticed thai sometimes the limit of a function f (x) as x --. a is the same as the value
f(a) of I he function a1 x
a. This propeny is de.<cribed in 1he following definioion. This is
the situation at x = I in Figure 2. 1a, but not the case in Figure 2. lb or 2. Ic.

DE FI NITI ON :z . 1
A function 1 (.t) is said 10 be continuous :ll x

= a if i1 saaisfies three condilions:

J. f(x) is dcfoncd a1 x
<t:
2. lim,_0 /(.t) cists:
3. The value of 1he function in I ond its limit in 2 are the same.
All three conditions c"n be combined by writinglhe single equation

'<-

lim /(x)

= /(a).

!2.4al

Equi\'ale m to this. and some1imcs more usetUI, is the equation

lim f(a +h)

A-o

/(a).

!2.4b (

(Sec Exercise 46.)

"llle graph in Pigure 2.27 illustrates a function that is discontinuous (i.e., not continuous) at
x = a. b. c. d. aod e. For ins1ance. a1 x = a. condio i.ons 1 and 2 are satisfied but coodition 3 is
violated ; at x = b. condit ion 2 is satistjed but conditions 1 a nd 3 a re not. Figure 2.27 suggests
that disc.ontjnuiticsor a runction arechar.tctcrized geomctricaJiy by separ.ttions in its graph. This
is indeed uue, and it is often a ve ry infonna tive way to illustrate the nature of a discontinuity.

:!..1

Vuioas lypc.s

M@llii.IWiffi

o ( discuttilluitit:)

Conti nuity

129

foJ u fwx:cion

.:.--r_r. ) l:A
.

/ :.

...

...'
.

...

\Vhen afunc tion is defined on a c lo.<ed interval tr :::; x :::; b , Definition 2.1 must be rephrased
in termsof righl- a nd le ll-ha ntlli mit> for continu ity at x = a and x = b. Specifically, we say
that .f (x) is continuous from the righ I a1x = a if lim<- n' .f (x ) = .f(a). a nd thm .f (x) is
ctmtinuous front the left al x = b if limx- b f(x ) = .f(b).
A fu nction is said 10 be continuous on an i nterval if ir is cominuous ~t each point o f that
interval. ln the event that the interval is closed. a ~ .\. ~ b . cont inuit}' at x = a and x = b
is interpreted a"> continuity from the dglu and left, respective.ly. Geomctric.ally, a funct ion is
continuous o n an interval if a perlcil can trace its graph completely wi thout bei ng lifted fro m the

page.

I EXAMPLE 2.21
Draw g raphs of the fo llowing func tio ns. indicati ng any discontinuities::
(al

f (x ) =

.r 2 -

16

:....__~~

x- 4

/(.<} = (x -

(b)

~) '

Mj1Cjii;J#FJ:I#1

(c)

f (,t )

Cl.)mputer pluts (_lo

1-< 2 - 251
x 2 - 25

Il l)!

l>how h()/~$

x2 - 16
x- 4

y= - -

8
y=

-4

-4

-2

SOLU'IIO'I

(a) Since f(x } is undefined a t x = 4. the function is discontinuous there. Fo r x


f (x ) =

(x

or

+ 4) (x
.r - 4

- 4)

= .v

4.

+ 4.

Consequently, the g raph


/(x} is a Straight line with a ho le at X = 4 (Figure
2 .28a) . The computer-generated graph io Figure 2 .28b does oo1 d is play the d iscontinuity at x = 4 ; depending: on the plot i.n te r\lal. the graph may, or may oot.
be accompanied by an error message abo ut dh ision by 0 at x = 4 . Note that
lim_..,.. l(x 2 - 16)/ (x - 4) 1 exist> a nd is equal to 8.

130

Ctmp:cr 2 Lithts !11td C01n.inuity

rl'.jll.l#*fM A functiun m;~y


be unboun<k:d ncar a discontinuity

(b) Since 1/(x - 4) is undefined at x = 4. the function is discontinuous there. T he

limits

:~

.
I
lim --~
,_,, (x- 4) 2

oo,

y=

(x - 4)2

I.

lim -:---.-:- = 00' lim -,-----.,. = 0,

x-

(x - 4)2

x-oo (x - 4) 2

liOl
- - oc

(x - 4)2

= 0

are displayed in Figure 2.29.

= lx 2 - 25 l/(x2 - 25) is undefined at x = 5, and is therefore


discominuous at these val\tes. When -5 < x < 5.

(c) The function f(x )

f<x)
and when

25 - x2
x2 - 25

= -I.

lxl > 5,
/(.1) =

x 2 - 25
2

-25

= I.

T he graph is shown in f igure 2.30a; Figure 2.30b is a COOlputer ver:.ion. Neither of

tile following limits exists:


or

hm

lx 2

251

x- - s x 2 - 25

although right- and left-hand limits exist as x -+ 5 and x -+ -5.

FIGUR 2 . 30a

fut~.c rion

MjUJII;I#FIHtl

Comp11tet pl01s do ~ slww t~l a funcrion


may be undefined

1.0

1.0

0.5

- 10

\\.ilh diffttent Jeft.hiuxl :uMi

ri~th tmd limit~

-5

Lt2 -251

0.5

y--_y l-25
5

lx' - 251

y - - x2-25

10 .\'

- 10

-5

10

- 0.5

0>- --+e+- - --()

\Vc now mention some gencrol properties of conti nuous funcrions. According 10 the foiJowing
lhc'Orem. when continuous functions arc added, >Ubtracted. multiplied. and di,ided. the result
is a continuous function.

THEOREM 2.3
If functions f(x) and g(x) are continuous a t x = a. then so a lso are the functions
f(x) g(x), f(x)g(x) , and f(x)jg(x) [provided that g(a) i 0 in the case of
division].

2A Continuity
This is easily established wirh Theorem 2.1. For insmnce, ro verify that .f(x)
continuous. w<; note that since limx- f(x)
/(ll) and limx- g(x) = g(a),

lim [f(x )

.:r-n

+ g(x)] =

lim j(x)

.r-"

= f(a)

+ :r-a
lim g(x)

+ g(a)

131

+ g(x ) is

(by Theorem 2. 1)

(by continuity off (,t) ''"d g(x}].

COROLLARY 2 .3 .1

If functions f(x) and g(x) are continuous on an interval, then so are the functions
f(x) g(x ), f(x)g(x), and f(x )jg(x ) (provided rhat g(x) never vanishes in the
imerva.l iu the case of division].
11 is an immediate consequence of this corollary that polynomials are continuous for all real
numbe<'. Rational functions P(x)/ Q(.t), w here P and Q arc polynomials, arc continuous
on inte rvals in which Q(x) f 0. The uigonomctric functions sin x and cos x arc continuollS
for all x; sec x and mn x have discontinuities: at x = (21l + I)Jr /2, where ll is an imeger; and
esc x and cot x have discontinuities at x = n:n: (sec Figures 1.90). The inverse trigonometric
functions are continuous wher-ever the.y are dcJincd.

From our graphs of exponemial and logarithm functions in Section 1.9, we might also be
led to conclude that these functions arc cominuous. Bur recall that there is a prol>lcm defining
exponential funcLions for irrational e.xponents. \Ve know what 10' means when x is a rational

number. say. .< = 11/ m. h means them"' root of I0" when 11 is an integer. or it means the 11'"
power of them"' root JO''"'. But whaL does 10-!2 mean? We define 10"1 as lirn, _,niO'.
where x approaches -J'i. b\11 docs so only nhrougll rational numbers. For instance. from the
infinite decimal expansion for ./2 = 1.414 2 13 562 .. . we could create the rollowingscqucncc
of rational numbers that approach -J'i ever more closely as we include more digits after the
decimal:
t .4. 1.41 , 1.414. 1.4142, 1.4 1421, 1.414213, 1.4142135, .. . .
We define I0J2 to be that number approached by 1he following sequence of numbers:

Notice that each term in this sequence is 10 miscd to a nuional number. It can be shown
that these numbers do indeed approach a limit. and that this limit is independent of the panicular
sequence of rational< u<ed to approach ./2. With this V'dlue for 10./2. the function 10' is
continuous at x = ./2. Jn this way, cxponcmio\1 functions a ... can be mflde continuous for
all rcals. According 10 Theorem 2.3. the hyperbolic functions ore all continuous except for the
hypc:rbolk cotangent and cosecant which arc discontin uou~ when their argumcms arc zero. The

following theorem implies that logarithm functions log. x. which are in,erses or exponential
functions, are continuous for x > 0.
THEOR EM 2 .4

'Vhcn a functjon f (x) is continuous on an irltcrval I , and has an invcc:sc function on I.


then f-' (.r) is continuous on the r-nge of f (x) .
Discontinuities are often characteriz.ed according to "size.'' A discontinuity of a function

j(x) at x = a is said to be a temo,able discontinuity iJlim_,... f (x) exists, and either f(a )
is undetined or f (a ) is detined and not equal 1o the limit. The former is iiJ ustmted at x = b in
Figure 2.27 and the Iauer at x = a in the same figure. A removable discontinuity can be removed
from a functjon by detining or redetining the function at the discontinujty as its lirnjting value

132

Ch:::ptc1 2 UtnJti. :lt.J Continuity

as the discominuity is approached. For exanJ>Ic in Figure 2.28>1 of Example 2.21. we would
define the value o f /(X) at the rcmovablcdiS<.-ontinuity x = 4 as .[(4) = lim_, _ , /(x) = 8.
TI1is new runction, which differs from / (x) only at x = 4, is now t.'Onlinuous for all x.
A func tion f(.r) is said to have a (Onite) jump discontinuity 111 x =a ifright hand m1d
left-hand limits c.<ist as .t a pproaches a . but these limitS are d ifferent,
lim f(x) = finite. nonzero mnn l>e1'.

lim f(.r) -

.f~ (f +

_,....,.q

Such is the c-..tse at the discontinuities in Figures 2.12 and 2.30a. Jump discontinuities are
nOt remo\able; the !'unction cannot be defined or redefinetJ unly althe discominuity to create a
continuous function.
A function j (x) is said to have an infinit e d.isconti nu it y a t x =a if lim , ....~, j(x) =
oo o r limx-~ f (x ) = oo or bod1. Examples c'm be found in F igures 2 . 14. 2.15, and
2. 16.
Not a ll disconti nuities t>ln be dassifled according to size. 11c func tion f (x ) = sin ( 1/x)
in Figure 2.9 is UisCOI''Ilinuous at x = 0 due to violent o...:;cilhuions. This discontinuity is not
removable. nor i~ it i:l jurn1>discontinuity or an infinite disco1Hiuuity.

I EXAMPLE 2 .22
1l1c floor function (al~o called the greatest integer functio n) f (x)
Section l .5 has n1any applie~:nions. Charact(.'Tize its discontinuities.
,.ll

~>m

1lle floor

of Exercise 68 in

SOl UTIO'I The gmph oft he func tion in Fig.ure 2.3 1 indicates thlll it hns jump discorllinuitics
at i nteger values of .x.

~4b
~----------------------------------------------

)
-J -2 - 1

= LxJ

.r

Besides atltli ng. ::.ubtrmting. multiplying, and di\'iding functions, function..o.: cnn also be cum
/>USed, o r s ubst.ituted one into another. for i ns~<ucc. when f(x) = .Jx' + x and g (x) = e' .
the ftulttion obtained b)' replacing coch x in / (.r) by g(x) is called chc composition of f and

1/.
o-Q

f (g(x)) = /e2x + e'.

-3
Composition is of1en denoted by

= f(g (.r)).

([ c g )(x )

Were we re<Juired to find the limit o r this fu nction <IS


nonchalantly that
lim

-t

Jf?-r + e'

x --+

I, say, we would pn.bably wri te

= Je' + e.

Eft'ectivdy, we ha,e interchanged the openuion or Ulking square roots and that of taki ng limits:
th:u is, without lhinking we t\4l\e wri tten
lim

.1- 1

/e2x + e' =

/ tin
\'-

(e1' + e') = /e1 +e.

This is correct PCCording to the following theorem. becnusethe:squnre root functioll iscominuous.

THEOREM 2.5

If lim,..,.. g (x )

L, and
li.m

.~-+fl

.f (y) is a functio n that is continuous at y =


J (g(x)) = f

( tim

,\'~ 1)

g(x)) = j ( L).

L, then
(2.5)

1\n inunedhue con$C.qucnc.:c of this result is chat the composition or a coruinuous functiOI)
with a continuous function yiekls a continuous function. We S{3tc this as a corollary.

COROLLARY 2.6 . 1

If g(x) is continuous at x
conti nuous at x

tt . and / ()') is continuous m g(a). then f (g(x )) is

(1.

I EXAMPLE 2 .23
2 2
E,aluale lim sin (x - ).
<- 2
2.r- 3
.SOLlJfiON Since the sine runctiun is concinuous for tll l values ot' i ts nrgumcnt. we may tnke
the limit operation inside the function and write

x2 -

2) = (

lim sin (
<-2
2x - 3

sin

x2 -

2)

lim - - -

x- l

2..r - 3

= :">in 2.

\Ve have used compute r plot"l in thi.s section to illustrate various kinds of limits. We wanted you
lirni1s graphic.ally and get a ' 'isua ljce/iug fo r them. In 1he next t\ VO exam pi ~ we i llustnlt.e
how Jimjts can be used to drnw graphs of nuional functions in the absence of technoiD'~Y
tOS(!e

I EXAMPLE 2 .24
2

Use limits to drawn g raph of the n Ltiunal fur\Ction /(x)

= .rX l -x+l2
.
4 X- 5
T

SOLUTION
= (x - l)(.c + 5). thefunction i~unucfincdat..r = I nnu
x = -5. In order 10 discover the nuture of the grnph ncnr these d t~con tinuil ics.. we c1lculutc
lcfl- and righl-hand Iimit~ for each value of .t.

Rccllusc .r 2 + 4x - 5

x2

+ 12
= -oo
x- -s (x - l)(x + 5)
.c 2 - ..r + 12
= oo
(.t - l)(x + 5)

+ 12

I im

~--~--~,.= 00

(.r -

lim
.l-1

(,t -

l)(.r

+ 5)

l )(x

+ 5)

= -00

x2

L-6~~o+>J

~--...,.,,....:..---:c-

(0~~(6)].

\Ve use long division and limits as x -) oo to find horizontal asympt01es,

..r 2

li Ill
s- -ro (.r -

X+ 12
+ 5)

I)(.r

.,-'-r---..,. ,. ~.,.....,..1 ..,2,.


-oo (.r - I )(.r + 5)
2

lim

= x-lorn. - eo ( I+ ,t

17 - 5..r )
+ 4x- 5

17 - Sx
= ,_,.,
lim (1 + ,
x + 4x -

= 1+.
= ,-.

These limits are shown i n Figure 2.32a. To finish lhegraph c.ts in Figure 2.32h, we jnin the po1rts
smoothly, 11cld the yinlcrcepC at -12/5, a nd note that the gmph has no .r-intercelll<. When we
ha\'C learned how to take derivatives in Chapter 3. we will be able to l ocate the precise posi tions
of the high poi11t of the graph lx::lwcc.cl lhe vcnical asymptotes a11d the low point to the right o f
x = I. Notice thtu the guph eros..~.;;: the horizo.ual a~yruptore OtlCe. \Ve carllocare this 1)().~i t icu1
by sol ving
x 2 - X+ 12
..r 2

nte solution is x = 17/ 5 .

+ 4x

- 5

= 1.

IQEI!1TI EFf!"W
V i!S

{it~til( II) ob'JW !he IIJr.ll)<h ()( /(t l

M jlslii I WW FJ ,W
.r! - .\ + l l
X~+ , . _ j
4

J!

_, :

fI

>

I:

:1

......i..... \1;\................
..-......./

.~........ t..! ~- ~

: - 12/5

if ..

I EXAMPL E

'

2 .2 6

Use limit.s 10 dnl\v M i rnph of the rntiunal (ull<:Lion

X4 -

(X) = x J

3.\'J

+ 2 -"2 + ,\' ..1.. 2

SOLLi'llO" ' 10 discover J)Oincs of tli ~c~)nti m ic y of the !f~'r>h we ftH:Ior lhc deno ninatOr. nxl
for timil (!O I CII I Afio L~ we alrro l'tteror the nu m~mtat
.r 3 ( .r -

.f( x ) = (.r

1"he funt.1ion i ~ undefi ned oc x

limz
, __

3)

+ 2)(.<2 +

I)

= -2. nnd we therefore cakubue


x'(x-

3)

(X+ 2)(x' + I)

= - oo

[ <- 8)(- 5)]


(0-)(5)

nnd

Jim
.t~-1

x 3(.T - 3)
(.r +

2) (.r 2

+ I)

=00

[ (-8)(-S)]

(0+)(5)

Since the dc-drce of the mmcrocor is 011e mote lfwn that of 1hc denominator. we have a slanted

asymp1o1e 111<11 c.m be oblained by long divisirnl,

/(.f)

X -

9x 2 + 3x + 10
5 + -:;-:-;:--:;-:-~
.r 3 + 2x 2 + x + 2

llcx~wse (9.rl + J.r + JO)/(.r 3 + 2x 2 .r .,. 2) is posili ve f<.- Jurgc x. the gmph "l'l""""hcs
the slan1cd a.<ymplotc y = x - 5 from above as .r -+ oo. It nppronches the ;JSymptote from
below as x -+- - oo . These facts are shown in Figure 2.33a along wiLh a y~im.ercept of zero.
which is a lso an x -iotercepl. and an additionaJ .X inte rcept of 3. We join these pans of the graph
smoolhly as shown in Figure 2.33b. We have tlauened the gr.tph at the od gin because of the
.r 3-fac1or in the numerator (just as 1he curve y = .r 3 is Oat at lhe origin) .

2A

Ii1 Ill '"EJE =:M


Using limits to dr.rw

Fl

Continuity

135

URE 2 .33b

x l -3,.,~

~r..lph

->f j(x) = ....,--":--:-.""---,--,


;xl + 2.r + "' + 1

..

.~..;

y= - 5

The following l(tble indicates when ratiOn<:ll functions: have horizontal or obi ique asympc:otes.

MIJ:I!I#IU
Hmitontalomd O blique A'ynpwte:-. fur Ratl on,ll

Fun..:l l t'll~

I r degree P

< degree Q,

graph has horh~o1tal asymptote )' = 0.

rr degree p

= degree Q.

.
graph has horozontal a.<yrnptote )'

If degree P

If degree P

> degree Q

degree Q + I.

+ I.

')

/' (

Q.\

wcfficicm of hishest power of P (x)


= cocmcient
of highest power of Q (x)

graph has oblique asymptote.


graph has neither hori7..onra1 nor oblique asympco1e.

E X ER CIS ES 2 4

l!
li

12. {(.r) =sin ( 1/x)

13. /(.r) =

14. /(x)=~

+2

!i

15. f(x) =

12
= -=-----,--...,.
x + 2x + 2

16. /(X) =

li

18. j(x) =

In E:<ercises 1-30 plot a gruph of th~ runc:tion indi<."tuing any <.liscon


Chu$;ify each di~ i nui1y a~ a n!J>vabl~ discofuinuity. a
jump di~:contintri ly. nr an infinite di10conlinuily. if poss-ible.
1 irwi1 ie~.

i
i
Iii

I. f(.<)

I
X+"i

3. f(x) = !x' - 51
12

5. j(x) = -,.,...,..-::ex + 2.<


7. f(.r)

3 + 2x - x 2
x+ l

= '--'---

9. f(x )

lil

10. f(.r}

= to -,,.,
.v +4

i1
iii
i1

x'_ -_ 2._r '_+'--:5._


r _- _tc.
O
X -2

= tanx

2. f<x>

4.

12

f(x) = - 2x

'

6. j (x)

,.J

8. f(x) =

+ ,.:~ - ,,.

,\'2 - ,\"

ll. [(.<)

"""2.x

x in (l/ .t)
X + 12

20. j(.r} =

li

22. .((x) =

- 5 :s ,\:

x' +2x
x1

x>- 27
Jx -31
3x +2
3x

x2 -

6.r

6x -7

,\'2- 2.\' +4

x-1

-3 5 X 5 6

x 2 -.t-

il

'
.r
17. /(.<) = - ,-

t9. f(.r)

=.x' - 3x + 2

il

21. f(x)

3x +2
= x'1 +x+4

il

23. f(x) =

.X -

x'+ 4x - 5

x .l - 4x

136

Chapter 2 l.imilS and Cocnjnuil)'

zx; - 2
= ,
2

Ill

24. f(x )

26 J(.<) = l3.r

+ 5.<

+ II

25. j (x) =

if 27.

x+5

.:r

28. J(.<) =

Js + x

30. J(.r ) =

!S{-3
.r + 2

i -t;

+3.r 2

.I"'')= 5=1
X- I

45. lllu.striJtc g.r.c1phically t.hat lhc function j(x)

46. Verify that t he condition in equa1ion 2.4b is equiv:tlent 10 that in

33. f(x)

dcronw by

.;s=x

- I.

>J<~Lt

2x- I
32. J(.r) = - 3 -4.(

x-2
x+3
x+2

34. f(.r ) = 2.r'

' ' - 4.t - 5

x' +x + 2

* 36. f (x ) =

~;...:;...:...=o

x'- 6x +9
2

x'

f (x ) = x' - I

* 4 1.

/(X) = -,---,.,x 3 -&I

40.

x -x +2x- 8
f (.r) = .:.,.----:.:....,...:,...::::...----=..
.r.>- 3x 2 + 3.r - I

+6

* 42.

/ (.r)

continuous at

* 44.

x in (1/.t ).
0,

X -

positive numbcN after the first

(a) Dmw a~raphofthefunction j(x)


is the function discontinuouS"?

= Lx + l/2j. Where

(b ) Prove thm / (x) rountl'i positive numbers to the ncarc.st

integer.

* 5L

Wh~t

function rounds positive numbers to:

(a) the n earest le nth ;

(b) the nearest hundredth:


(c) 10-n, where It is o positive i.ntcg~r'?

o/o 0

x = 0?

rr x = /(z)

* 50.

++ 52. D c::tcnuinc

f(x)

LIO.rJ/ 10. Where

_.. 49. What function ut utcatcs ncgatj\'C nuntl'<::rs aJtc1 two th"CimaJs?
..

x 3 - 64
.r 2 - S.r +6

43. Ts the function

+ I ) - sgn

decimal.

1- X
1

> 0.

(a) Draw a graph of UlC functi<)ll f (x ) is the runction discont.in uou..~1


(b) Prove thtn f (x ) tnmcatc:s

38. /(.r)= 3x - 2x + 5
3

* 39.
*

~.---=.:.._---:

=0

1-x
+ 5..<- 3

3r- 14.r- $

Dr11w its grupb, indicating aoy di.:)Continuitics.

48.

x' -3x -4

< 0

(b) Draw a graph of the function f(x )


sgn(x
(x - 1) . lndicating any discontinu..ilics.

x - x - 2
* 37. f (x) = "-:--"-___.c.
3x + 1

x' - 5x

= { 0.

I.

= .-'-C...:....:~

35. j(x) =

{a) "lbe siguumfimcziclu (or !tign fimtiion), dc.notcd by sgn, is

.f(.t) = - -

31. f(x) = - -

= x-1 sin.r Ita.$ a

equation 2.4a.

ln Exercises 31-42 usc limits to llraw a graph of the function. Usc a


plot Lo dltck the accuracy or your graph.

=0 .

removable discominuily ;U x

47.

~~
h

~
II)

point~ of continuity,

.( )
X

= { 0.'

x
x

if any, for the f unc.:tion

is a rational n umlx.'f.
is an imttiona1 number.

53. Uctenninc points of continuity, if any, for the function

represems Lhe displitcem enl func1ion o f a panicle

mov-ing along the x -axis, can f {t) ha\e djscominuities'! Explain witb

gruphs.

x 1 x is a mtionn1 number,

f (x) = {
0,

.X IS

ao ltJOI:i.OJ.lal nu01bc.r.

12.5 Heaviside and Dirac-DeJta Functions


One of the simplest, but at the same time most useful, runttions in engi neering and physics is
the Ht:aviside wlit step function. It is defined as follows with the graph in Figure 2.34:

0,

h(x) "' {
I,

x<O
.t

>

o.

(2.6)

We have drawn small circles at x = 0 t.o ind icate that. the function does not have a value
there; it has ajumpdiscootinuity. Closely relaLe<lto this fu nctioo is that io Figure 2.35~ the jump
from value() to value I takes place at x = a rJlher than at x = 0 . Since this simply shift..s Lhe
graph in Figure 2.34 by a uniiS 10 lhe right, i1is cus1omary to denote lhis function by h (x - a).

'l.S IIC.I\'iside lind l>irnc-DdU FU1"-'1iOfH

Algebraically. we have

lz(.r -

)l

a =

0,

I,

< a
> (l ,

137

(2.7)

For example. consider Lhe Hca,iside function lo (.r - 4). To evaluate this function 01 x = 6.
say, we have two choices. Because 6 > 4, equmion 2.7 implies that the value of the function
at .r = 6 is 1. Allenlatively, we e M substitute x = 6 into h(x - 4} 10 get it (6 - 4) = lz(2),
and this is cqualto I by equation 2.6.
liijlc:III;I#ICf"

llu,i:iKit u.nil

~c p

function h (.t - a)

y
.I = h(.<)

y =/i(X- (1)

The product of h(,t -tz) with any other fu nction f (x) results in a function / (.r)h(.r - a}
wtm.e valuc.s are equal to those of f (x) for ,, > a, but whose values ure 0 for x < a. For
example, if f(.r ) = .r 2 + 2 (Figure 2.36n), a g raph of f (x )lz(x + I) = (x 1 + 2)/z(x + 1) is
as shown in Figure 2.36b.
Think of lz(x- a) a.> a switch that tums values of a function .f(x ) on for x > a. For
the function f(x) in Figure 2.37a. j (x)lz (x- cz) is shown i n Figure 2.37b. Values are 0 for
x <a and those of f (x ) for x > a .
Tho Pl'JOOO)O A' + 2

FIGURE 2.36a

)'

-2 -1.5 - t

-o.s

o.5

Mijl?iil.lr

iJ

Al)'

l'unccion ,{(.v)

t.5

2X

-1

-o.s

Mijlclii.J#;

0.5

0
i:J

l-'\ln~tion

1.5
/(X)

nt~.llli pli ed

.I'

b) l1(x -a)

)'

y=f(x)

y=f(x)l(x-a)

-J

- I
-I

- 0.5

-t

-2

-2

-3

-3

0.5

].5

2.5

The futlCtion h(,, - t' ) - It (.t -b) for b > '' is shO\"'' i1\ Figure 2.:\K. Algchmicully, we
hove

0. .r < a
I. a < x < b

la(x - 11)- h(A - I>) =

\ 0.

> b.
When muhit>lyi~lg the fu.ction [(,<) in Figure 2 .37a . II yields the functi on J (.<)lll (x - n )
- h(x - b )l Wllh gmph Oil Figure 2 ..W. ThiRk or Ia(.<- tl)- h(x- b) .. an Qn-<:>IT wilch
ltlums 'uluc' ul' other fuottiOilS on fur .t > a and un u~a in for x > b .

4'

,.

Ill. I

0.,..0 of /f.>) mUif'l"'l loy b(x-a)-h(A - b )


)'

y =/()( /(.<-

3
2

- I
(/

a)- h(,. - b) I

0.5

-I

..

2.5

-2

-3
-4

:u lll.lkt
.,.

______

,.

Heavisillc: func1ion~ pm\idc o cOn\tnienl repre:~n111i00 for functiOn) that have difTcrcnt
definitions on different inte"al>. Such fune1ion< an: uid to be pie\<M defined. For tn$\ancc.
the function in Fagure 2.40 is defined '" follows:

),\' + 3
f(x)=

{ ~-x
2

.r < 0
0< .<<2
\ > 2.

11 mn be l\.1ll'esented algcbraicall) inlhc form

j(x)

+ 3)1 1 - il (x)l + (4 - x 2)1il(x) - il (x - 2) 1 + 2il (x


= 3x + 3 + ( -3x - 3 + 4 - x 2 )il (x) + (x 2 - 4 + 2)/t (x = 3x + 3 + (I - 3.r - .r 2)1!(x) + (x 2 - l)lt(x - 2}.

(3x

- 2)

2)

2.S

Bc.wiside 21KI Dir~Oc i U\ F~ul-.:li 4Jil S

139

There are many physical examples in "hich Hcavisitle function~ 11rc very useful. Consider
a mass m mtached to a spring as shown in Figure 2.41. Motion is initiated at time I = 0 by
pulling the mass away from its equilibrium position and releasing it. During the subsequem
motion. various ron.-cs could act on the ma,s. iolcluding the spri ng, friction with the surfoce on
which it slides, air resis~,ncc, and others. Suppose that among the 01hcrs. a force to the left
with magnitude 5 N is applied for 3 s beginning at time I = I s. This Foree can be represented
algebraically
F (t ) = -S[h(t - I) - h(t - 4)).
The generator in Figure 2.42, were it openuionul from ti me I = 0, would produce !Ill
oscillating volmge A sin wt , where A and w are positive con"
Tf the generator is indeed
turned on at time 1 = 0. by cl osing the switdt and ofT again ancr 10 s by opening the switch,
the voltage applied to the circuit can be exprc,~scd in the fonn

a"''

E(r)

= A sin wt [h.(l) - h(r -

10)).

The beam in Figure 2.43 is made more rigid by aua~hi ng a second beam over the middle
half. This extra ~uppon creme~ >ldditionlll loading on the original beam. If the ~Uf)p<lr! has m:l$S
per unit length 111 . then its weight per unit length is -9.81 m , and the extra loading per unit
length can be cxpres:sed algebraically as
F(x)

-9.81m(h(x - l./4) - l1(x - 3/./4)).

FIOURE 2.41

FIGURE 2.42

Sctlcmatic t'l)r LCR cireuil

A sin il

f----- x
x =O
I.
MjiUII;i=

Sc-hematic for lo.Klod

bc~l nt

y
L

U 2
fEX In.l

support

Beam

'

Throughout the text. we wil l be hired ~Lt\ Consulting engineers to u'cklc projects. Jnfonnation
will be given to us that may n01 always be sutliciem tO finish our work. We may have to make
justilinble assumptions. Here is Project 1.

The natiooal space association is buiJdiog a two~st.age rocket wherein the rocket burns
fuel during the first sLage, jettisons the thrusters, bunts fuel in the second stage, achieves
orbit, and turns the remaining engines orr. T he initjaJ mass of Lhe rocket is Mo kilograms~

...... "'~.,...,,~- ""-the~ ..........iani~e~ ..--.~

l'uel. ThO\osters burot fuel at :t<ons,.ntrat.eof ' ktlog'"""Jt<r ""<<lnd for r ""'"""" 1\IIQ'
tile ' co.,d,, thothm<tors ure lottned. and
...,;.;.,, eng,...,
1Jo:n 1\lcl
1
"' a <ons.ant cto of ' > kiloKron" P<r ""'"'"d r..- the ''"" 1 "'"'""'- At th" ...., the
fO<:kot U<i>ievo$ Orbit Und <ngill ro shUI, down. r.,. ...........
ftiU\'oben, t>f <<leul.._,
1
.......
"
r.,.
tlw:
.....
<Ill t en...
tho
on lld ar..,.
"'ht Oor
;, to find one

.,,J

~-iatioo,""<~tin "~ e r...,.,.,.


~~

or the"""" ..
~"'""
We '1"'-1.1) nt.lo>c that U..rc IM\11 be dilftn:"' f""""' IO< the -llftlt<

roc~.,

r,~ ,....,."""~
lif~otl

'>t I l>Ttc 1';


durina tho: '""'"' ""lles. H ..-;,;,s., rune,,..,,. e ttlc;.l
defined fUilQion, in a .-j"1< fOI'll1ula 1'1te 11\a"-< ()(the t\>ol.et duriO\l
1

p;..,,.;'<

is

~~n ~HJ.'- l \il."~tlf

J\1\or t seeonds the thruste" are i<llisoned, llnd the "'"'lni,tg cngu"'>lgn;.., anJ burn
fuel at u con.tmtt rate ol r 1 ktl<>gmnos per seton or
"' 1.
the rockcl duting tht~ tin~e intcn.~l is
1
'
.
d he --M
The
Mo- M , - '!(1- IJ),

Fi.naJI)~ \\u~:n I e -.

rocket i~

..._

M(t)

~"''' """"'.. orbit

and

'

<' < , , -t-60r, .

~;..,, .,. "'"' ~. tho -of . .

lht ...

(M,l- r,t ) h(r ) - [ M , - rllt

+ (rI -

r,)tlf(r -It)

_ It _ 6011)/1(1 - It - (~)II).

+ r,(r
'
nllnuit) at I = ' '
1
. F'g:urc
2.4-t,d"cllnh
JUmp
"
an
1l htl' t Ut
,.., ..t\h!\~
I - IJ -t 6()rl \loh.cn

. h f this fun<ll<"'
lOvable
'
_
11 "''
A gtHpl
llrll jettisoned
.ultlrcmov
re: the di>atntinuity
r, 601,. h)when
It
fT We CQUld
engines
arc
- If, - ('l()t!r:.
llf(lt + 601,) (I

~~ter.tUn~~A~

~huwn

M(t)
0

dcfi11m~

25

Application Preview
ReYisited

ln the Application Pn:vicw

We iouro~uce<l

Heavi~ide and

the problem

DiDcDtll:. Fun~.:tiO!t(

141

or de.cribio)g an iU>lantancously

applied \"'hage 10 an c:1ccU'ical network. The :same siluation would arise were we to consider
lhc malheonatical representation of striking the mnss in Figure 2.41 with a hnnuncr. We would
also find the same situation if a poimIMd were to replace the extra support on the beam in
Figure 2.43, or be in addition io the exu'>l suppon.

To find a malhematic.al representation for the instantaneously applied ''o1tage. the forte
due to an impinging hammer. and a poinr-lond on a bearn, we begin with the function (b a)- 0lh(x - o) - h(x - b)] in Figure 2.4Sa. Whao is imponant oo nooice here is ohallhe area
of the rectangle fomted by drawing vertical lines no x
a and x b is I. If b is replaced by
a + l so that< is the width of the nonzero portion of the graph, then the area of the rectangle is
also I ifthe height of the function is l/< (Figure 2.45b). The function desc.ribing Ibis graph is

- fh (x - a) - h(x - a - <)].

This is often called a unit pulse function.


MiUdiiI:FW-ift

Unit pulit t'unclioo (b- a ) 1(h(v - n1- h(.r - b) l I 0) ...-lll;lj FIPI[!W

<

b- o

(I

.<

Unit pulse fun~lion ~fh(x -o)- h(x- (1- )]

----.r

(/

Suppose we make< in Figure 2.45b smaller and smaller, so 1hm a e --> a and the heigh1
of the hori-zontal line mo\-e..l\ upward ( in order that the area under lhe curve will always be 1).
A few smaller values of arc shown in Figure 2.4-6. If we take the Umit as E
what is called the Dirac-delta funcllon. It is deroOicd by
o(x - a)

1
= .-oe
lion - (ll(x -a) -

h(x - {/ -

)'

3
<

<

< a+ -<
3
2

a+ -

(I+(

'

~ )).

.-..

0, we obtain

(2 9)

142

Chlpler 2

Umits 300 Cunlin\111)'

#- a. and somehow its value at x = a is "cqualto infinity.'' Clearly,


as we undersrand func1ions. since func1ions cannot ha\e infini1e val ues. 1t

Its value is 1.ero for every x


lhis i" not a function

is called .~cllt'f'ttli:cd fimctioll. Generalized functions hn"c opcnnional properties that make
them very useful in handling point sources in engineering and physics. We will introduce these
propenies at appropriate ploces iJlthe text.
Point sources arise in 1110Sl areas of engineering and physics. For instance. suppose we
wtre to suike the left

~ide of the

vibrating ma.!;s in Figure 2.41 with a hammer at time t

10

This would be cquivalc:m to applying a force over a ,.ef) short time intervaL If it were dcc.mcd
that striking the mass is equivalent n0110 a 10 N force for Is. nor a 20 N force for 112 s. nor a
40 N force for 1/4 s, but to the limit of this sequence of forces, we would say that the mass has
heen stnock with a force of I 0 N at 1 =

to . and expre~s the force in the form

F(t)

= IOcS(I

- 10) .

If we were to take a sharp object, place it at the centre oft he beam in Figure 2.43, and push
downward with a forceof200 N. we would represent litis force as

F(x) =

-200~(x

- L / 2).

It is a point fon:.'e of 200 Nat x = L/ 2.


A unit voltage at time t = t0 for the circuit in Figure 2.42 is &(t - 10 ). It is equivalent to
appl)ing a \'OIIage n01equivalenuo I V for I s. nor 2 V for In nor .J V for lf.J but a \OIIage
equivnlenl to lhe limil or such voltages.
We shall leam how to manipulate Dirdc-<lel!a runclions in these applications calculus
unfolds throughout the text.

EXERCISES 2 .5

In Exercises 1-6

exprc~

H eavi~ic1e funcl i on~.

I. j(x)

the piec-ewise defined function in term'S of


Draw a graph or each runclioo.

runctions or Oirac.dcho functions. as :apprc.pri:uc.

X<0

2 - x. 0 < X <2
2.
X > 2

1. f(x) = [

I,- x '

.t

X <
X >

0
0

).' < - I
."( - 2. - 1 < X < 3
X - 4. 3 < X < 5

0.

S.

f(x)

x.

X >

IO$in4tthatt'>tumedonrortW(Ipcri()t1(

10. A funx of 100 S lhlltlx:git\.\ ~t l imc I


in si1c to 200 N O\'ct the net t .SO seconds

=4

= tO anc.J increases linearly

* 11 . E""'Y tO second< begi nning at 1 = 0 sand coding at 1 = 60s.


11\e mas is ~tucL: with !1: hlmn\er wlth 3 fOt"CC o f 60 N to the ng.ht

In E:(crci.scs 12 - 15 the lo.'\d described is applied to the bc.a m in

X <0

O< .t < l

Fis~

on: 2.43 withotu the extra ~uppon . Represent the force in temu of
lkavbidc: func1jon.s or Diract.lcha function~. as appropriate.

1-x. 1 <x < 2


0,
.r > 2

12-. An c.~tr.t supp<>n of length L/ 2 and mass m O\'c r the ldt half of
the. beam

o.

13. A downwatrd force or F > 0 N (.'OOCCntmtcd at X

!:10 .\ ,

6. j'( ) =

lcfl rorthc lir<t T <ootls af\cr 1 = 0

S. Apcrit>tJicfM-c F(t)
beginning ;at lin"lc 1
I

9. ;\n in.st~nta~ fon:c: of 50 N to dlC ldt at tirnc 1

.< < 0
. . + .. c."'S.t. x > O

r
r

7. A rorce ofF' N to the

4 3. /(x)= l ~inx;

+ 4. .f(x) -

in f.:\crcisc~ 7- 11 lhc force dc;)cribed ;."<:I.:,. on lhc nla)~ on 1hc end of


the spring in Fi,sun:: 2.41 . Rcpre.<C:Cnl lhc force in letnlS of licavi<idc

2 <in (x - rr),
3sin(x- 2rr),

4sin(x-3rr),
0,

X< 0

O<X<Jr
"< x < 2rr
27T <.r<3Jr
3JT<.r< 4Jr
X > 4 7T

14. An 11pward force of

downward force

F1 >

0 N conccntmtcd at
= X1

or F: > 0 N concentrated at X

= L/3
x = x 1 and

IS. A total mass m evenly distributed over the last two-third.; ol' tJte

beam

A MJ.tltcmatical Oeti nilitxt or Limjls

2.6

16. The funclion h(x - a ) - h(x- b) is an ononswilch. Whal


function would be an on-oll'-on S\\.'itch'J

;~o;,

25. Expressthe --.tcpfiwcticm shownbe.low in l.erms of Hecwisidefunc


tions.

l7, HO\.v woukl you represen1a S\Vilch that turns on at tillle t = 0 . otT
at t = L. on again at t = 2. olf again at t = 3, and so on'?
18. Is h (x - a)h(x - b) = h(x - b). when b >a?

= lr(2.r- 1)
*
2
(x) = lr(x - 1)
*
2
2
(x) = x /r(x - 2x- J)

21. f

22.

23. f

24. f(x )

= (5 -

= h(J- 2.<)
/(x) = 5h(4 - x 2)

20. /(x)

3
2

rn Exercises 19- 2.4 dr;1w a graph oflhe funct ion.

19. f(x)

143

x ) lr(2 - x 3 )

12.6 A Mathematical Definition of Limits


Our work on limi1s has been intuitive. bu1 some la1er 1opics require a p1ecise definition of
limj1s. To obtajn sucb a dcfinitjon. we be-gin with our iutuitivc SWlCiltent of a Limit an.d make a
succession o f paraphrases, each of which is one step closer to the definition. \Ve do this becau.se
the definition of a limit is at first sight quite overwhelmi ng. and we wish to :show that it can be
obtained by a fairly straightforward sequence of step<;. We hope lhm this will be a convincing
argu menl that the definition of a limit does indeed describe in mathematical terms our intuitive
concept of a limit. Our intuitive statemc.n t is:

A function f (x ) has limit L as x approaches a iff (x) can be made arbiltarily close to

L by choosing x sufficiently close to a .


Next we par~phrase I (x) can be made arbitrarily close to L ."
A function f (x) has limit L as x approaches a if the differe nce lf(x) - L I can be made
arbitrarily close to zero by choosing ,r sufficiently close to a .
Now we take the impurtant s tep - make "lf(x) - Ll .:an be made arbitrarily di)Se
10

zero'' mathcmaticnl.

A func1ion .f(x) has limit L as x approaches a. cf given any real number > 0, no matter
how small, we can make the difference l.f (x) - L I less than l by c.hoosing x sufficiently
dose to a.
Pcnullimately, we paraphrase by choosi_ng x sufficiently close to a .''

A function

(x) has limit L as .t approaches a if given any > 0, we can make

1/(x) - Ll < ~

by choosing

lx - al sufficiemly close to zero.

Finally, by making ""choosing lx -a I sufficiently close to zero precise, we arrive at the definition
ofa limit.
DEF I NITION 2 . 2
A function .f(x) has limit L as x approaches a. if given any ~ > 0, we can find a 8 > 0
such that
lf(x) - Ll < E
whenever 0 <

lx - al < J.

Not ice that by requiring 0 < lx - al. this detioitioo states explicitly that as far as limits
f (x) at x = a is irrelevant In 1ak ing lim its we consider vaJues of
x c.Joser and doser to a. buL we do not consider the val ue of .f (x) aL x = a . Th is definition
states in precise lerms our imuitive idea of a limit: that f(x) can be made arbiuarily close lO L
are concemed. the vaJue of

144

Chlp(tr 2 Limits and Ca~inuity

by choosing .l uflicientl y close 10 a . Perhaps you will gel a beuer feeling for this definition if
we imerpret it graphically. Figure 2.47a indicates a function thai has lintil L as.t approaches a .
Let us illusmue what must be done to verify algebraically lhatlim._, [(x} = L . We suppose
that we are given a vnlue ~ > 0, which we should envisage as being very SJtulll? although we
a rc neve r told e.x..ctly what it is. We must show thai x can be re,;trieled to 0 < lx - a I < 8
so that lf(x) - Li < l. The Iauer ineqw>l ity is eq uiv:\lem u> - l < f (x} - L < E or
L - l < f(x) < L + f, and this describes a ho1i zmual ha11d of width 2E around the line
y = L (shaded in Figure 2.47b) . Whm Definition 2.2 requires is that we find a n interval of
width 28 a round x
ll. lx - al < 8 is cqu ivalcl\l to a - 8 < x < a + 8, such thm
whenever x is in this imcrval, the val ues of f(x) me all wi1hin the shaded horizontal band
around y = L . Such an interval is shown in F igure 2.47c for th~ give n . Now Definitio n 2.2
requires u~ to verify thtu the 8-in1Cr\1UI can always be 1'ound no 1naner how small f is chosen
to be. This is always possible for the function illustrated in Figure 2.47. and it is clear that the
smaller the given value of c. the smaller 8 will ha\'e 10 be chosen. For i nst.ancc~ for the value of
E in Figure 2.47d. ~ is smaller t han that in Figure 2.47e. In other word>. the value o f S depe11d~
on the value off . Hc.ein lies 1hc dillicuhy in tosing Definit ion 2.2. In ordcrtocnsurc thalli eaJl
be found for a ny gi\'en value of l. we usually de1em1ine precisely how S depends functiona lly
on ~. We illustrate with two example<.

M41CJII.I El(iilj

Gnph-. lo ilhmr.au: lhC nwtbtnlalical dc:finilic.'CI of linit

L, ~----------~~--~
L ................. .

L - ~------~'-~------'

(/

Mji?jii;J

ijtijii.IWHi@

)'

L~ . ~-------------+------~

L-~~------~~~~~--~
tr -"S

t1a~8

I EXAMPLE 2 .26
Use Defin ition 2.2 to prove that lim (2.t
A-3

+ 4)

= 10.

SOLUTION It is true that based on Theorem 2.1. this is quite obvious. but we are required to
use l)etjoition 2.2. We must show that giveo any > 0. we can choose x sufHcieotly close to
3 that

1(2x

+ 4)

- 101 <

To do so, we rewrite the len >ide or the inequality with all x s in the combination x - 3,

1(2.\

+ 4) -

101 = 14 - 61 = 21.1 - 31.

We must now cboo>C .r >U that

2Jx- 31

c.

<

Bu11his will he true if lx - 31 < E/2 In Other word~. if we choose x 10sa1isf) 0 < lx- 31 <

E/2. lhen
!(2x

+ 4) - 101 =

21x- 31 < 2

(i) = f.

The ,eriflcalioo is now complete. We ha\'c ~h01vn thai we can make 2x + 4 as close 10 10 as
we wam (within E) by chOO>ing x ;ufficiemly close 10 3 (\\ithin 8 = i/2).
The following exampl< is more complicated, but a m:magea ble one.

I EXAMPLE

2 .27

+ 5) =

Use Deliniuon 2.2 10 prove that lim (x '

- 2

~()1

9.

UTIOil: We must show that given any E > 0, we can choose x sufficiently close to 2 that
J(.t 2 +5) - 91 < E

or

Once ag.ain we rewrite the left side of the inequality with all ;c.s in the combination x - 2:

lx 2 - 41

= l(x -

2)1

4.\ - 81

= l!x -

2)2

+ 4(.t

2)1.

We must now choose x so th~


l(x - 2) 2

+ -t(x - 2)1 <

f.

Now. all real numbers a and b ><lli>f) the inequality

a + b! .:: Ia! + Jbl.

12. 101

With a replaced by (x - 2)' and b replaced by -t(x - 2). we can s.y that

l(.t -

2)2

+ 4(x -

2)1 5

l.t - 212 + -tlx - 21.

11< a re.<uh. we consider hnding x <O that

lx - 21 2 + 4 x - 21 <
y

If we set z = lx - 21 aod l'Oil!>iderthe pat"dbola Q(:) = : 2 + 4: be able 10 see what to do. The parabOla cro!:<es lhe :-axis when

=Q(:)
s

:~ + 4:

f.

~2

+ 4: -

= 0,

a quadratic with solutions


t

= -4 JJ6 + '* = - 2 J'4+"'.


2

The graph show>thal whenever 0 <

z<
~2

-2 + J4 +( .then

4~- < 0.

in Fi~ure 2.48. \\'e "ill

COOpter 2 L.im.itS tHid Cvol.iouil)'

146

Since z =

lx - 21. we can say that ifO < lx- 21 < )4 + - 2. then


lx - 21 2 + 4lx - 21 < ~-

aud therefore

l(x - 2) 2 + 4(x

- 2)1 < .

Verification is 110w complete:. \Vc ha\'C shown U1a 1 we can make x 2 + 5 us dose to 9 as we want
(within ) by choosing x sufliciently close to 2 (within = ~- 2).

...-..

These examples hal'e indicated the difference between "e\'aluation of' and 'Yeritication of' a
limit. Evaluation is calculational; it comes first. Verilic.1tion is much more difficult: it requires
a clear unde-rstanding of Dcfinirion 2.2. Results in succeeding chaple.rs rely heavily on our

inluiti,e understanding of limits and the ability to calcu late limits. but use of Definition 2.2 i<
kept to a mi11imum.

EXERCISES 2 .6

., 23. lim (x + 5) =

In Exercises 1-9 usc Dcli nilion 2.2 to "'airy the limit.


lim (2x 2. ,._,

,_, (x + 5) = 6

I. lim

+ 3) =

4.

3. ,lim
_o (x'

5. lim (3- x = - I
1)

.r- -1

7.

lim (x' - 3x + 4) = 8

.r--1

l im

- oo

3) = I

* 2.'.

(.r '+4) =

' I

6. lim (x 1 -7x) = - 12

* -J
lim (x' + 3x + 5) = 9
8. ,_,

27.

.r+2

Jim - -

J-f\>X- 1

..

00

=I

lim (5 - x) = oo

2.-1.

..,.

26.

-- = 1
T'---MX- I

28.

- -~

lim (5 - x 2) = - co
Jim

x+2

lim (3 +.r - x') =-co

.r-. -oo

29. Prove~'" ' if lilll,- f(x) = L > 0. ti= ~1c:n: cxisiS an op.:n
irucrvul / containing a in whi<.:h f(x) > 0 cJCc<:pt possibJy ut x:::: a.

30. Docs rhc function

x+ 2

9. lim - - = 4
I

.r.-2 X -

ln Exercises I0- 19 give a mathcmalical delinirion tOr each statement.


lim f(x) = L

10.

11. lim /(x) = L


x- ..

.-:- a
X

14. l i rn f(x) =co

lim f(.r) = -oo


15. -

16.

18.

IJ.

17.

lim f(x) =co

$._N

lim /(x) - co

. 19.

.-:- - ~

where

12. lim f(x.) = L

'~"'

lim

..:~-0)

a!' ;r

a ppmachc~>

lim f(x) =

:c-.o.:-

-oo

lim /(x) - -co

x--oo

a.

21. Jj m

.-.:~ I

) "' = 00

(.r - I -

- I

22.

lim

.H-l

= 1/ (n:r),

n is nn in1cger. ha\'C ~ hrnu 3( x -;. 0?

In Exercises 3 1-35 we use Uefinition 2.2 10 pm\'e Thc-ortm 2.1. In


e:k.'hexercige :u:~u:meth!H Jim,.-. 0 f(.x) = F and lim f ....,.(/ g(.r) = G.

* JL

Show that gi,en any E

., = -oo
(x+2)

> 0. there exist numbers 61 > 0 and

., > 0 <tiCh lh>i


lf(x)- F J < ( / 2

In Exerci'C:~ 21-28 use chc ap(>fOIX'iare defi nition from Exercises 1019 10 ,'erify the limit

.oin (1 / x) . .r 'I 1/(n;r)

= {
I,

f(x.) = L

20. Usc Definition 2.1 to t>rove th~u a (unction f(x) cann01 h:wc two
li mil~

f(.r)

Jg(x) -

Gl

whcnC'.::rO < Jx - aJ <

~1

ond

< </2

Usc ll'lCSC results <!long with 1dcn1ily 2. 10 to pro'-c par1 (i) of Theorem
2. 1.

* 32.

Usc u proor

~ i m i lar

10 that in Elerci(C J I 10 \'etiry (Wl (ii) or

1~orem2 . 1 .

33. Verify part (iii) of Theorem 2.1.

Key Tet'n'IS.

J4.

**

(a) V\!riry th~u

>

0. lt~rc cxi :H numb..:rn 8,

> 0,

82 > 0, and &3 > 0 .o;uch th:tt

1/(x)l < !Fl + I

lg(x)- Gl < l (I FI + I)

(~t) Va-ify thUI when

j(x) _ F
G

l f (.<)g(x) - FGI !: lf (x)llg (x) - Gl +I Gil/(.<) - Fl .


(b) Show that J:_i, cn any f

35.

0 :/; 0 .

ll(x) - Fi
lg(x) l

IFII G- g(x )l.


JG IIg(.<)l

(b) Show that g:i\'<:n My t > 0 . thccc ctiSL number$ 8 1 > 0,

8z >

0 . and 83

whon<:vcrO < lx - al < &, .

IGI

l.~(x)l : > -

wheneverO < lx- a l < 8-, .

1/(x)- f'l < <.IGI

and

Jg(x) - Gl <

> 0 such thai


whcm.:v.:r 0 < ~t - ll l < fl 1 ,

wiiCilcvcrO < lx - a! < B,.

<IGI'
(1FJ + I)

wheneverO < 1-< -

tmd

a! <

~J

(c) N ow IHU\'C palt (v) o f ThcUf<:m 2 . 1.

(c) Usc these results 1..0 l>ll,)\'C part (iv) of'f hcorcl1l 2. l .

S U MMARY

I<

l g(x)

147

In Section 2. 1 we intn:.x.luccd l imilS of functl u n:-;. For the nt()St pa1t our d iscu~~ ion wa~ i n= L ir j (x ) can be m;ade 3rbitrari ly
close to /~ by choosi ng x .~ ufficie ntl y clo:o;e to tl." This idea was then excended to i nclude the

l uitive, beginning w i th the statement "l irn,._ 0 / (x)


fo l lowin~:

lion f( x);

Right-hand limi os:

\'-(1'

lion [ ():);

lefthmd limi1:1:

v-tl

lim .f(x ),
x-c-o

Limits at infi11ity:

lim f (x);
,\'--0()

lim [ (.r ) = oo,

lnfinite limits:

-o

lion f( x ) = -00.

x-11

Keep in rniiH.I t hat the term infinite limit~ is ~mc.whtH of (& misnomer SillCC in both sltuatiOilS

lhc lintit does nut c;~.iso.


A func1ion / (x) is continuous ac a po int x =a if

lim /(x) = [(a ).

\' tr

To be.continuous at .\' =

a, a furn:t ion mtL~t be d e.fined at ,,; =

11 and have a

lim it as

a, and

these num bers m lL~t be [he R:lnle. The funcci on i.~ conti nuous, on an imer\'al if' ir is cominuous

ao each pOint or thao imerval. Geomeorically. this means thm one must be able to trace its
graph completely without lifting pencil from page. Most disconoinuities can be characoerized
as rcmo, nblc. finite. or infinioc.
We illustnued that discontinuous fu,ocoions such as the Heaviside and Oirac-deltcofunctions
model many physical situations.
In Section 2.6 we developed the mathcrna.tical definition of a limit.

KEY TERMS

In reviewing this chapter. you should be t~blc 10 define or discuss lhe fullowing key ocmJs:
LimiL

Leflhand limil
l.ntinity

Squeeze or saodwich cheorem


Righi hand limil
Venical asymptote

l.tmiLS at infiniry
Oblique a.ynpotcs
Continuity from the right

Continuity o n 1m illlei"Vt~l
(Finite) jump diM:orHinuil)'

Horirooual 8S) mpw<<


Continuou.\ function
Continuity from th~ kft
Re movable discontitutity
ln(initc diSCOI'IIillu ily

Compo,ition
Piecewis.e defined function

Hcuvl~itJe unit step function


Unit puiM! t\ulf.:tion

Din:~e:.-dclta

function

AI!Vti!W
EXERCISES

Jn E<trd'C'f: 1- 20

'

I. hn'

5.

7.

..

hm

'

2.r

.t'l+4.r-4

.r+l

'+3x+2

lin1

2x 1 - 5

3X 1 TL\ - 5

lim

x 2 + 5x

~-I\,.

.t 1 -lx

9. lull

II.

,\'7

+ 2.r

.r' + 2x

llftl

..(i- I
.r

' I

x'- 1

li1n

' + I

+5
4. lun '
-v .t - 3
6.

5-

tun

.r'

- --

., ~ + 4x 3

H. 11m

4-3.<+A'2

' ""

3 + Sx3

Ill

l1n1
,_,

X:- 4X

u.

x + 5.r
Cx - 1) 2

- '\ - 2xl

23. j(x)

.r'
=r -2

24. f(x)= - -

l5. j(.r)

= x+lI

26, j(.C) =

.X -

X -

ll. j(x) - - -' - 2

x'- 36
x- 6

X-

2,. J(.r) = l.r +II

28. j(.r)

.t- 1

X+

II

x- 1

x+l
=lx - II

2.X
= -;---:---:
x' - 3A' - ~

.
2xl
.!0. j(x)= ..->-3<-~

+4

.r- 2

.fi -

. l

Jl. /(.r)

liln

, .,

l
--

1 1. j(x)

29. j (X

14. lim

1.1. htll - - -

Ill Exercio:e' 2 1-32 druw .a J rUph of the fu1k.'tion, indK:atin@ l.lO)' d.sU)ntinuiLics. Dctcnninc \\.hcthcr li\(.OilCi nuii.JO arc 1'(11'10\ablc. j"'"P.
o mhnitc,

the lunJl. if it e"in~.

2.

~+S

' I

J.

c,~dJuatc

= c l - 'x'x +I lc

XJ-JA '+JA - I
Jl. /(.r) = ---,; --:--:-:x - 2.< -i l

In E"<erciS 33-34 express the piccc"i5e defined funcdo1l in tcnm or'


ll c.avi~de func1ion~ .

15.

(2- 4x)3

lim

1/2

.r(lt - I)

16.

tun
,_._,

cos5x

.r

JJ. j(.r)

~(

;>
5

17. ..hnl

19. ,_,__
lim

X sin X

J~.r' + 4

2.r + 5

Ill.

J3x' +4
"' l .r + S
lim

20. ,_,.
lun ( JlA'

+ I - JJ-,

34. f{.r) =

O< .t< ~

2.r

.r> ~

J+x', .r<-1
- 1<A < 2
(
.r 1 + 2.
4,

1)

.r < 0

X >

t JS. Few wh:ll value~ C)f .r i~ Che funCii(V'I /(.f)

Draw i~ gr11ph.

= L.t 2J diu:ontinuf~u~'!

CHAPTER

Differentiation

Ap plication Pre,icw

The container in the figure below represenL~ a chemical reactor in which a chemical is either
created or broken down. The chemical enterl> in the form of a solution at one concentmti011and

lcmes the reactor at a ditTcrcnt conccmnnion.

Tli E PROBLE\11
lncomin~

chemical

Exiling

chemical

,..

Chc:!mical reaclion

Given the ftow rates at which solution enters and leaves the reactor, aod
the concentration of chemical entering the reactor. tind the concentrmion at which chemical
I ves the reactor. Can you see that there is not enough information to solve the problem? Some
onnation must be given. or assumed. about the nne at which chemical is fom1ed or broken
own.
Chapters I and 2 have prcpMed the way for calcu lus. The functions and curves in these
chapters yield a wealth of examples for our discussions. and limits from Chapter 2 provide the
tool b)' which calculus is developed. Calculus has two major components, diffe"mimiou and
imegmtiou. Jn this chap(CT we study diffcrcntjation. We learn what a derivtuive is algcbra_ically
and geometrically, and develop some of its properties; we learn how to differen tiate polynomial~ ond rational , trigonometric. ex ponential. logarithm, and hyperbolic functions: and we ~cc
glimpses of the applications that are to follow in Chapter 4.

13.1 The DeriYativc


Very few quantities in real l ife remnin constant; most tlrc in a State of change. For example, room
temperature, the speed of a cur. and the angle of clcvmion of the ~un arc three commonplace
quantities that arc ..:onsUltHiy changing . A few more tec-hnical ones an: current in a transnUs:;;ion
line. barometric pressure. moisture content of the soi l. and stress in venical members of tall
bui ldi ng$ during hurricanes. Rates at w hich (hese qwmrilies change are called deri1nti1 1es. To
study 1he concept o f a (ate of change more thoroughly, we consider two commonplace physic-al

situations.

Displacement and Speed


Calculus plays a c.entr.tl role in djscussions that uace the rnoLio n of particles and objec-t s moving

under the innuence of forces. Wilhout calculus, analysis of a simple system such as a mass on
the end of a spring would be diflicull; planning lift-off. space travel. and re-entry of spacecraft
would be impossible. \Ve i.nLroduce basic ideas coooected wiLh motion here. and w ith each new
development o f c.aJculus, we take the analysis a little fu rthe r.
Suppose a particle is at some position on the Xaxis al time I = 0 , and for 1 > 0 , various
forces act on it resulting in motion that is contlned 10 the x-axis (Figure 3.1).

149

ljUlii;JHM

Di:~plooc-ucm

of particle moving u!one. lhc. x -nxi.s

PanicJc
------~---------+---------------- x

~ -1
The posi1ion of the panicle can be idenLified by its x -coordi nate, and x can always be

regarded as a function of time/. Suppose. for example. that


.t

= f(l) = 1 - 271 2 + 1681

+ 20,

I :0: 0,

where x is measured in metres and I in seconds. This is called the displacement function;
it gives a complete history of the panicle's motion. II tells us not only where the panicle is a1
any given time. it also comaius i.nfonnmio n about the "elocity. spet(J. and acceleration of the

particle. We begin by plotting a graph of the displacement function (Figure 3.2). Ordinates of
this curve represent hori2oma.J displacements of the panil'1e relative 10 x = 0. For ex.amplt.;
the height o f th~ d.isplace.Jncnt graph a t time t = 0 is 20. This means that the part.iclc begins
motioo 20m to the right of d>e origin. At 1 = 4 s, it is 324m to the right of the origin, and at
1 = 14 s, it is 176 rn to the left of the origin. Whene"cr ordinates are positive, the particle is
to the right o r the origi n. \Vhenever ordimues are uegati\'e. the panicle is to the left of x = 0.
Figure 3 .2 indicates that the particle is to the. right of x = 0 for the first te.n seconds and the1l
agMin afl.er 1 ~ 17 s. Becween Lhe~e time.-.;, it is to the left of the origin.

W@ J;J#Q
X

- 100
-200

For 0 < 1 < 4. values of x are gettcng larger. This means that the pa,ticle is moving to
the right. For 4 < 1 < 14, the panicle moves to the left (values of x are getting smaller), and
for r > 14. it moves to the right once again. This means t.hatthe panicle stops and reverses
direction at I = 4 s and at 1 = 14 s.
Everyone has an imuitive idea about speed, when something moves qui ckly and wheo h

mo\'es slowly. Th discuss speed of the above particle quantitatively, we need deri'"dtives. but
even now we can derive qualitative information about how fast the panicle is moving. The more
distance the pw:~nk.le covers in a given time interval. the fa:;ter il is mo,i ng. In Table 3. 1. we

have tabulated the distance that the particle tn>vels each second for the first twcmy se<:onds.
The first two entries indicate that the panicle travels 142 m in the first second. but on ly 94 m
from t = 1 s to I = 2 s. The particle is the.refo re travelJing faster during the first second of its
motion than during: 1he oext second. The remaioder of the Iable suggests that 1he particle slows
down untiI. as we know. it comes to a stop at 1 = 4 s. It theo speeds up until somewhere arouod
I = 9 s when it starts to slow down again, and comes to a stop at 1 = 14 s. It then pic.ks up

speed thereafter.

).I

The: IJeriV.Mht:

151

1:1181

diStance (m)

11me (~ J
d iStance (ml
We can see this graphically as well as in labular form. The steeper the displacement gr~ph.
the larger the speed. the flauer the graph. the smaller the speed. This means that speed is greatest
when the graph rises or falls quickly. At the beginning or the panicle's motion (for small 1 > 0 ).
the gr.tph is relati>ely steep. and therefore the panicle is mo1ing quickly. Steepness decreases
as we head toward 1 = 4 s, and therefore the particle is lowing down. At 1 = 4 s. Ote panicle s
)X.-ed is zero (it reverses direction). "The parliclc then picks up spee<l. moving now to the left
umil steepnes> is greatest somewhere around t = 9 s. It then stans to slow down. coming to a
stop at/ = 14 s. It then moves 10 the right thereafter picking up speed (as steepness increases).
Again . all of this is quite qualitati1e. but when we leam about dcri,atives. we can be much rnore
speeilic and we can add informatio n about the 1elocity and accelemtion or the panicle. We
begin with velocity.
Figure 3.3 is an cnlargemem of that pan of the graph in Fig ure 3.2 fo r the first four seconds .
At time I = 1 s , the particle is 162 m to the right of the origin. ,_\t 1 = 4 s, it is at position
x = 324 m. During these three seconds, the panicle has moved 324- 162 or 162m; its
d isplaccmcm at t
4 s relative to its position at/
I s is 162 rnto the right. When we divide
the d isplacement by time taken to travel it. we obtain 162/ 3 = 54. This is called the average
velocity of the panicle: during Ote tinte imerval and ha. unitS or metres per >econd. During
the time interval 1 .::; t .::; 3, the disl)facemem or the lli!niclc i< 146 m; lherefore. il< average

ve locity in this time interval is 146/2 or 73 m/s. 11te 3\'erage \'elocily during I ::S 1 ::S 2 is
94 nVs. \Vhat we u.re doing is taking average velocities over s honer and shoner time intervals.

nil beginning at I

I :s. I f we continue the process indefinitely. we are in effect taking the


lirnit of avemge l'eloci ties as the length of the time i ntenal Marting at/ = I approaches 0. This
will be called rhe insrnmnneous ''t!locity of the panicle at I = I s. Let us imroduce notation 10
describe the limiting process, and do so at an arbitrary li.me t0 rather than at t = J.

I 141CJII.I4lf)i1 I Enlu;cmtnt I)( :rapll in fi:,urc .).2


(4. 324)

300
250

200
!50

(1. 162)

100
50
2

We let 6.1 repre><:nt a small inter,al uf time. often called an increment or time. The
(unction value /(to) is the position (or displacement) ofthe particle atti01e lo. fUo + ll.t ) is
its position ntti mc 10 + .6 r. and f (t0 + 6.1) - f (10) is the d ifference in 1hcsc displacements.
It represents the dispiaccl'l'lent of the particle at time 10 + l!lt relative to its position at time lo.
IL may or may nol represenl the disLance travelled by the pan icle during the time inlerval /j.f .
(Can you explain why?) The quotient

.f(to + ll.t)- .f(to)


ll.l

is the average. ve_locity of ,he particle during the tio:te jnterva_lbetweeo to and lo +D. I . The limit
of this quotient os 6.1 ._ 0 is called the ins tantaneoll$ velocity of the par1icle at Li me lo: it is
denoted by
. /Cto + D..t) - f(to)
(3.1 )
u(tu) = lun
.
&t.-O
!J.t
lt represe.nts how fas t the particJe is moving at. time /0 ; it is the instantaneous rate.of change of
displacemem wilh respecl 10 lime.. For e.~arnp le, the insramaneous veloci1y a1t = I is

v(l) =

hm

/(I

a,-u

+ D..t )

- /(1)

t:lt

(I + 6.t)>- 27(1 + 6.1) 2 + 168(1 + D..t ) + 20 - (I - 27 + 168 + 20)


.
61 ~0
t:.t
.

Simplification of the numermor gives


v(J ) =

hnt

<>~ ~o

11 7(t:.r) - 24(t:.t)1
D..t

aod when we djvide oumerator and denomiJlator by


v( L) =

ar.

lim (J 17 - 24(6./)
~~-~o

+ (t:./) 3

+ (<l/) 2 ]

= L17.

T1te panicle is travelling 117 m/s at timer = I s.


A similar calculation at I = 5 s leads to an instantaneous velocity v(5) = - 27 m/s. The
negati\'e sign indicates that the pan icle is moving to the left. The positive velocity v( I)

117

means ihai the pnriiclc is moving io the right nt I = I s. We shall have much more io say aboui
velocity in Section 3.6 whe n we relate velocity and speed, and we ;also introduce acceleration.
For now, we sim.pJy wam you to appreciate that i nstamaneous velocity is a nne of change.: it is

the. instantaneous rate of t~hange of displacemenr with respect to time.


Mii#ill;l111
t.:~lchi ng

Cyli.01kt

Rate of Rainfall

rainfall

For our second situation in which to introduce rates of change, consider an open cylindrical
container placed outside u house. The depth D (measured in millimctrc.'i) of water in the
comainer during a raiostorm (Figure 3.4) is a f unction of time I (in hours), say D = f(t). A
grctph of this function might look like that in Figure 3.5. T1tere is no water in the container at
time 1 = 0 whe-n the storm begins. As the ra in falls, the depth of water inc.rea.~s, and fi nally,
when the rttin stops at time I = T. thedc.pth of watc:r remains at a constantlc.vcl D thereafter.
lntLiilively, rain is falling fas-teSl when this curve is S-leepest, somewhe.re around the time indi
cated as i . When tl1ecurve is nat (just after t = 0 and just before 1 = i'l. very liule rain is falling.
We want to be more specific; we want to be able to say exactly how fast the rain is faUiog at any
Mi::UdiiI#IW

i5

Ocpdl of rainw..ucr in cylinder as a fur.ction of tine

given lime r0 . We proceed os we did in deAning velocil)'. If we let 61 be a sma ll incremen1


of time. 1hc n th<: difference .f(lu + 61 ) - /(lo) is 1hc m11nlx:r ofmillimclres o f rain 1ha1 falls
during the time interva l llt after to. 11lC quotient

/(to-r 61) - f(to)


61
wilh unit') of millimeues per hour is ca.lled 1he twemgc rille of rainfall during the time inter\al.
TI\C limi1
.
/ (to+ 61) - / (lo)
Inn

61

A1-0

is called 01e. iiiStanlllllfO<IS rtue of rainfall al lime 10 It is OlC instantaneous rate o f chnge of
dcplh ofwa1cr in the cylinder with rc;pcciiO time, at lime 10 Ifits value were 5. say. 1hcn were
rain to fall at this rate o,cr an e.'\tCndcd periOO of time. the depth of water in the container would
incrca:;c

by 5 mn1 C\'Cry hour.

The r<ltcs of change in thcl)(' two ~ituation~ arc t)'pi<.:al of mtcs of change that occur in a
multitude of applicaliuns - applica1ions from such diverse fie lds as engineering. ph)'Sic.s , eco
nomics, psy<.:hology. aod medici I'M!. to mmc a few. \Ve now retbrmulate them in a nulthcmalical
l'nunework that allows us to introdu-ce them in every area of applied mathen1.acies. Suppose a
hutclion f(x) is defined for all .,. and x = a and x = a + flare two values of x (a is taking
1hc pluce of 10 and h is replaci ng 61 in 1he previous situations). The quotiem

.f<a + h) - f<a)
h
is Ci1!!c<! lhc ~vc.ruge r "!e of ch~l!ge o f .f(,r) wi1h rc;.<~ci iQ
+ h. The lintil of Lhe quotie.nt liS h approaches zero.

x in 1hc in!crvall)c!wccn a !>nd

ll

j (a + II) - .f(a)

h-0

It

hm

is called the instnnta n cous rate of change of .f (x) with rcspccno x at x


lhederivativeof / (x) alx =a.

ll .

It is also called

DEF INITION 3 . 1

The d erivative of a function / (x) wilh respec110 x a1 a poion x =a. den01ed by f'(a).
is defined as

.f'(a) = lim f<a +h) - j (a),


~~-o
IJ

(3.2a)

prO\'ided chat the limil exists.

The following limi1 is equivalem to thai in cqumion 3.2a bul il avoids the iouroduclion of It :

'( )

a = 1nn
.l -

(1

.f(x}-f(a)
X -

(l

(3.2b l

The o pera1ion o f laking the derivative of a func1ion is called differentiation. We say that
be appropria1.e lOuse
Oetlojtion 3.1to calcula1.e derivatives of various functions at various poinlS. Before doing so.
however. we feel that it is imp011am to discuss the geometric interpretation of the derivative. It
will be prevalent in many applications.
we differentiate the function when we tlod its derivative. JL would now

154

Ch.ap1er 3 Differentiation

Tangent Lines and the Geometric Interpretation of


the Derivative
Mj1Hii;I1.W Tangen! line
at one. poinl may intl!'rSc:ct curve. at
anotJJc:r point

Algebraically. the derivatjve of a function is its instantaneous rdte of change. Geometrically.


derivatives are intimately connected to tangent li nes to curves, and most students of calculus
have an i nw itive idea of what it means for a Line to be laogent to a curve. Often. it is the idea

o f wuchiug . A line is tangent to a c urve if it touches the curve at e xactly one point. For curves
such as circles. ellipses. and parabo las. th is notion is adequate, but in ge neral it is unsatisfacto ry.

y
P ( l.

For instance. in Figure 3.6 we have dJ'awn what would look like the. tangent line. to the curve
y = x 3 at the point P(l. 1). But this line intersects the curve again at the point Q(- 2, - 8).
The tangen t line at P does not touch the curve at pnx:isd y one point; it inte rsects the c.urve at

l)j
X

M:llcliiI+Q Tangen! line


to y :::: x 3 at (0 , 0 ) cn:,..<>ses curve

a second point.
You might reply that tangency is a local concept; whethe r the tangent line at (l, I ) in
Figure 3.6 intersects 1he curve at another poiot some d istance from (1. 1) is irrelevant. T rue.
and perha ps we c.o uld remedy the situation by defining the tangent line at (I, J) as the line that
touches the curve a t ( I, I) and does not c ross it the re. Unfonunately, this detinitio n does not
always work either. For instance. what is the tangent line to y = x J a t (0, 0) (Figure 3.7)?
The o nly reasonable li.n e is the x -ax.is. but notice that th e x -axis crosses the curve a t (0, 0). To
the right of (0, 0), the curve is above the langem line, and to the left of (0, 0), it is below the
tangent line. This happens quite often, as we shall see later.

How then are. we to define. the. tangent line to a curve:? The idea that a tangent line touches

Tangen1 l ine

at (0. 0)

a curve is good. bui it needs to be phrased p.-operly. Consider detining what is meant by the
tangent line all he po int P on the c urve y = .f (x) in Figure 3.8. Whe n P is joined to a nother
point Qt on the curve by a straight line I , cenainly i t is not the tangem line to y = .f (x) a t
P . If we join P to a point Q1 on y = f(x ) closeno P than Q, then /1 is not the tange mline
at P either. but it is closer to it than i t . A poim Q3 even closer to P yields a line / 3 that is even
closer to the tangent lioe than 12 . Repeating lh is process over and over agai o leads 1.0 a set of
lines / 1. 12 , (l, ... , which get closer and doser to what we feel is the langent line toy = .f(x)
at P . We the refore de llne the tangent line toy = f(x) at P as the limiting position o f these
lines as points Qt, Q2 , Q 3 , . . . gel arbitrarily close to P .

y=f(x)

The li ne in Figure 3.6 satisties th is detinition; it is the limiting position o f lines joining P
to other points o n the curve w hich approach P . lt i s irrelevant w hether this Li ne intersects lhe
curve again at some di stance from P . Wh at is the t.ang:em li_oe to the curve}' = x 3 at the origio

(0, 0)? According to the above definition, the limit ing position of lines joining (0, 0) to other
points on the c wve is the x -ax is; that is . the x -axis is tangent toy = x 3 at (0, 0) (Figure 3.7).

As mentioned ealjer. the. tangent line actually crosses from one side of the curve to the Olher at
(0, 0) . To the left o f x = 0 , the tangent line is above the curve, whereas to the right of x = 0,
it is be low the curve.
Now that we understand what it means fo r a line to be tangent to a c wve, let us make
the connection with derivatives. Suppose P (a, /(a)) is a po im on the curve y = f(x) in

3. _1

Figure 3.9. If Q(a

Tl~ Oeti~tive

155

+ h, [(a+ h)) is another point on the curve, then the quotient


f(a + h) - f(a)

h
in equation 3.2a is the slope of the line joining P and Q. As h ~ 0. point Q moves along the
cun.: toward P. aod the li.ne joining P aud Q moves tOward the tangent line at P. It follows
that the limit in equation 3.2a, thc derivative f'(a). is the slope of the tangent line to the curve
y = f(x) at x = a (Figure 3.10).
FIGURE 3.8

Lines usc:.d to find tangent line to

F"IOURiii 3.10

3 CUf'\' C

Tanscnt l ine to a curve hilS slope

f'(a)

)'

)' = [ (X)

~(
_..... a.[(a)J

Y =[ (xj

~,

slope= f(a + II) - f (a)

Q (o + h.f(a +h))

"

f '(ll) = slope of

tbe cangent line


X

To summarize. algebmically the derivative j'(a ) or a function .f(x) at x = a is its


instantaneous rate of change: geometrically. il is the slope of the tangent line to the graph of
f(x) atthe point (a . f (a)).
Let us now use equation 3.2a to calculate deri,<ati,es of some simple lirnctions.

I EXAMP LE 3 . 1

..........

~r;:t:t~

Find /'(1) if f(x)

line to par:tbullt

.r

T::mg_tnl
= xl aJ (I. I)

x2

SOLUTION According to equation 3.2a.

J'(l) = lim

f (I +

h-()

(I

= lim

h ) - f(l) = lim (I
h
h-0

+ 2h + h 2 ) -

Jt.-o

= lim

+ h)Z-

2h

h- o

+ h2
h

= lim (2

h- o

+ h)

= 2.

Algebraically, the instantaneous rate of cloange of f(x) = x 2 when x = I is e(lual 10 2.


Geometrically. the slope of the tangent line 10 the curve y = x 1 >It the point (I , I) is 2
(Figure 3.11 ).

........

..........

I EXAMPLE 3.2
Mj!Ciil;i

taf.JI

Fi nd f'( 2) if f(x)

Ta ng~n l

line may hnve slope equal

SOL UTI ON

co 0

Using equation 3.2a,

f ' (2) =

. /(2 + h ) - f(2)
hm
A-o
h

= lim

11....,0

= x-' - 12x .

6h 2

+ h3
h

= lim (61!
11 - o

. [(2 + h) l - 12(2 +h)]- (8- 24)


h no .:..;__ ___;_;_ ___;-.,.-_ _:_:._ _;__~
A- o
h

+ 1! 2 )

= 0.

Thjs result is subst.ant iated in Figure 3.12. where we see thaLLhe laog:el11tine to y = x 3 - 12.x

is horizontal (has zero slope) at x


(2. - t6)

= 2.

156

Ch:.p~tt 3

Oitfemuiaiion

In Examples 3. 1 and 3.2 we requi red the uerivative for llle function at on ly ono ' 'a lue of a.
and therefore set a in equation 3.2a equal to this value. An aJttrnnivc. which is far more
advantageous. especially in an example where the deriva1ivc is rcqui1'Ccl i:H a number of poi nt!>. is
to evaluate /'(a) and lhen set a to its de.t;ired value, or values, later. For inst..1ncc, i11 Example
3. 1 we calculate that

f(a + h ) - f(a)
'( ) hm
. -'--'-----'---"--'~
=
! a = h-0
II

= lim

2ail

h- o

+ 111
h

= lim (2a

h- o

. (a + h)2 - a 2
!om -'-----'----

11

h-o

+ II )

= 2a .

Thi i the derivative oF f(x) = x 2 at any ''alue x = a . For a = I, we obmin f'(l)


= 2( 1) = 2. But it is also easy to calculate /'(a) at other values of a . Por example,
/'(0) = 0. J'(- 1) = -2.a nd J'(4) = 8.
We now carry this idea to its logical conclusion. T he derivative off (x) at a is denoted by
f'(a). But whal is a? It is a speci f-ic value of x at which 10 calculate the derivative. and it ran
l>c any value of X. Why not simply drop references to a. and talk ahout lllc derivative of / (.t)
at values of x? Following thi suggestion, we denote by f'(x) the derivative of the function
((x) at any value of x . With this notation, equation 3.2a is replaced by

:... :........;h')-------"f .:...


<x..:..)
. :<.. f.<x_+
lun r' = h-o
h

<3.3)

. (.t)

for the deriv:nive of' f (x) at x. We call .f'(,r ) the d erivathe function.
W hen a fu notion is represented by the le tter y , as in y = f (x). 11110ther common nOlation

'JOr thc dcnvattve


IS tly. B ccarc f\J I when usmg
. t h'IS nolatton.
.
Do not .mtcrpi'Cl .ll as a quoucnt:
.
dy

tlx

and tlx do not have >-eparate mean ina>: it i one ;ymbol representing an accu mulation of a ll the
OJl"rotions in equotion 3.3. It is not therefore to be read as "dy d i,ided by dx ." Typographically.

it i~easierro prinl dyjtl x ra1her I han d y. and "'e will Lake Ihis libeny whenever it isconvenient

dx

to do so. But remember, dy/d.r for the moment is not a quotient, it is one symbol rcprc.....,nting
the limit operation in equation 3..3. We will t:.hange this in Section 4.12.
Some-tinlCS it is more convcnicm to usc parts of each of these notations and \Vtitc

d
- /(x).
dX
In this fo rm we understand tha t d / dx means to(}iiTercnl iate with re.pect to x whatever follows
it, in this cusc. f(x) . Let us usc these ne.w notations in calculating two more derivatives.

I EX AMPLE

3 .3

..........
Find dyf dx if y = j(x) = (x - 1)/(x

+ 2) .

Usiog cquatioo 3.3,

SOLt;TION

dy
-d -x

I (X+

X- I)

. j (x + It)- f(x)
h-I
= ~~o ::....:_.:_-;h- --'-.o....:. = ~~ h x + lr + 2 - x + 2

I f we bring the tenns in parentheses to a common denomjnator. 1he result is

dy
dx

lim .!_
h~<>

[<-" + h -

l )(x + 2)- (x + h + 2)(x - I)]


(x + h + 2)(x + 2)

3.1

The Deril'!llh..

157

When we simplify the numerator, we find

-d y
dx

= li. m-I [
h-o It

311
(x

+ h + 2)(x + 2)
3

= lim -,---..,,.-....,..,.- ,--,.,h~o (x

I EXAMPLE 3 .4

Find du f dt if u = f(r )

+ h + 2) (x + 2)

3
=--...,.
(X+ 2) 2 .

= 1/t.

SOU liON In terms of variables v and r. e<Juation 3.3 takes the fom1

dv
dr

= lim

f(t + h) - f(t)

ll ~o

= lim 2_

It

. I - (I
hm

+ h)

ho r(r + lt )h

h;,.oh

= li m

(-1- _~)

-l

h-ur(t +h)

r +It

- ~

r-

In Seetion 1.& we proved thm the inclination M a line I, where 0 ::; r/> < 1T . i~ related to
ilS slope m by the equmion tan = m (equation 1.59). When I is the tangent Iine to a cune
y = f(x) at poiou (xo. Yo) (Figure 3.13). its slope is f'(x0 ). Hence. the inclination of the
tangem line to a curve at a pOim (.r0 y0 ) i~ given by the equation

)'

Taogem line
uo <xo- >o)

>

-r---~~--------- x

l EXAMPLE 3 .6

.........
Find the inelinmion oflhe l<~ngcn tlinc to the curve)' = (x - I)/(.r + 2) at the point (4. 1/2).
SOLUTION In Example 3.3, we calculated the slope of the tangent line to this curve at any
poim as
d.r
3
=
dx
(x + 2) 2
According to equation 3.4, the inclination 1/J of the tangem line at (4, 1/2) is given by
tan <f> =

3
(4 + 2) 2

<f>

= 0.083 radian.

In applications, inclependentand dcpcndcm variables of func tions y "' f (x) represent ph)<sical
quantities and have lmits as~ociated with them. Units ror the derivative f'(x) are units of y
divided by units of x. We sec 1his from equlltion 3.3. where uniL' of the quotient (/(X+ h )
- / (:r)J/ h are clearly units of y divided by units of x, and the limi1 of I his quo1ien1"~ h -> 0
does nm aher these units. ForeX.ample, if .r measures l~ngth in metres andy measures Jnass in
kilogram,, , the un its of dy{d:r are kilograms per mclre.

EXE RCI SE S 3. 1

In E.u :rdsc. s 1- 10 usc equation 3.3 10 Jind j'(.t).

=.r + 2

I. j(x)

In Exercises 24-27 fi nd the inc l i n;~tion of the umg\' nt tine to the curve

2. /(x) = .Jx' + 5
4.

5. j(:x ) "' :x' + 4.r- 12

x +4
6. f(.r) = .r -s

'+'
= :..:........:
.r +3

* 9. I (X ) =
~~~ E.r.crc;i~cs

/(x)"' .r '

8. f(x )

3.r - 2
4-X

--

ii
i

+ 2x1

3. j(:x) = J +2x-x'

7. j(x )

UllhC gi\'Cil 1int.

= x 1 (.r + 2)

25. .1' = x 3

llx at (2, -4)

26. !'= l/.< 2 at (2. 1/4)

27. y = 1/(.t

1
10. j(x) = x - x + I

fC

U . .v "' x 2 ( I, I)

+ I)

0 1 (0.

I)

In Exen:iS<s 28-321ind f'(x) .

.r' +x + 1

2S. f(.r ) = x'

I 1- 1-' tind 1hc specified r~uc or ch:mgc.

II . n.e nue of dl.:11lg.C or the ciocumfcrencc C of a circle with respect


to ils radius r

30. / (.r)

12. Tite rate of cJmngc o( the area A or a cirdc with respect to i1s

.12. f(x )

29. f (x)

I
(:r _ ),

3 1. j (x)

= JX+1
=

I
r.:--<

vx - 3

=' 5+1

rldius r

IJ. 11lC role of change of 1hc area A or a SJ)hcrc wi!h

rc~pccl

to its

In E).tf'CiSCl'l 33- 3-S 11nd the !'JI)Cci ficd 18CC or chontc.

mdi~r

14. nlC rah! of d~ngc of 111-c vo!unlC V of i.l ~ptK:t'C with r~JX.'Ct tv i1s
radius r
Answer Exercises 15- 19 by d rawing a gn~.ph o f the func1.ion. Do 1101

<,;;.dculiue 1he rc:qvirc~l dc ri"~'i"c


15. Whal is J'(.r) if J(.r)

= 2.1' -

)t

* 34.

17. Whal is/'(0) if /(.r) "'.r' ?


Ill. What i> f'(l) if j(.x)

= (:r - 1)2'!

19. What is /'(OJ if / (.1')

=.r''3?

:1

drdc with respect lo its

The n:llc of change of the \'Oiume \' or a spher\! with rcspe<."l h'> ils

nrea A

J5. Fmd the unglc ~ween the wnscm lines lo lhc<.,UVC$ )'
x = y 1 at their poim ol imetAXI rQn i Ill he 11rsl qu:adnuu.

.16. Find f'(x ) if /(.t} = (.< (.

4?

16. Whal is f'(:r) if j(x ) "' mx +I>. where m and b are constams'!

33. The ulc of c:htmgc of lhc rudiut r o (

area A

.'(2: und

R ll::lS !l uni fonn charge distribution of p


coulombs per cubic mctr'C. The clcCU'UStulic I)CJle<llial V Ul a liil)tancc
r from the centre or tile sphere is dcfi ned by

1 37. A sphere of r.uJius

O::; r :;: R,
In Exc:rc-iscs

2~23

t1nd the (."(JU:Jii(lfl o ( !he 11ngcn1 line IO lhCcurve al

the given IX'Iinl.

20.

y=x 2

+3al (l. 4)

= j(r) =
r > II.

21. y= 3 -2x -

22. y = l/x 2 (2. 1/4)

23. y = (.1'

+ ll/(.1' + 2) nl (0 . 1/2)

x2

a1 (4.-21)
whc:rc fo i~ a conM!lnl. Draw a grnph o f this ruJltlion. Oc>C$ f (r )
appear to have a derivalive ~lllhc :surl'acc of lhc sphere'? T hal is. doe.s

/'(rn cxisr? Prove your conjcclurc U"ing Dclinition 3.1.

3.2 Rule$ for Differerui:uion

;~ .

E = f(r ) =

o:sr :s R.

pR'

-, .
~o r-

159

46. Let f(x ) be a function with the po-opcny Lhat f(x + z ) =


f (x)f(z ) for all .t ancl z. and be such lhal /(0)
['CO)
I.

R.

I' >

Pnwe .tha.t f'(x) = f(x ) for all x.

13.2 Rules for Differentiation


Sincecalculus plays a key role iJ.t mm.ty branches ofapplied science. we need IOdiffcrentia.t e nwny
t)pesof functions: polynomial. rational. trigonometric. exponemial, and logarithm fUilCiions. to
name a few. To use equatim.t 3.3 each time would be exuemely laborious. Fortunately, however.
we can develop a number of rules for taking derivatives thal eliminate the necessity of using the
definition each time. We state each of these formulas as a theorem.

THEOREM 3 . 1
If f(x) = c, w here c i< a con.tant, l hen .f'(x)

= 0.

PROOF By equation 3.3.

.,

J (x + 11)

j (x) = ltm

ft_.O

- f(x)

I!

c- c

= lim - - = 0.
h-"0

In short. the deriva1ive of a cons1a01 func1ion is zero;


{/
-(c)=
dx

0.

(3.5)

THEOREM 3 . 2

If f (x)

= x , then f'(x) =

I.

PROOF With equation 3.3,

M@lciii;I:U[k::::J Deri:nives
of functions y = l' and y = x

'( )
x

(x + /1) - x
II
. j'(x + II) - f(x)
= 1m> .:..._--...,---'----- = lim --.,----- = lim - = 1.
h ....,.O
h
fi.....,.O
II
h ....,.O IJ

In short,
.

,.

d
- (x) = I.

y =x

(3.6)

d.x

)'=C

Wilh the graphs of 1he functions .f (x) = c and f (x) = x in Figure 3.14. we can see 1he
resuhs of eqwnions 3.5 and 3.6. The 1angem line toy = c always has slope zero. whereas 1he
tangent ljne toy = x aJways has slope equal co 1.

THEOREM 3.3
If .f(x) = x'1 where 11 is a positive integer, Lhen f'(x)

= riX"- 1

I'ROO ~'

Equ<~tiun

3.3 gives

/"(x) = lim f(x +h)- j(x)


A ~o
h

lim (x +h)" - x"


A~o

If we cxpan.J (..r + h )" by mcilns of the binomial theorem (un altemalive proof is given in
Exercise 35), we have

The firs~ ~111d la..11t terms in brackets cancel, (Uld dividing

/ ' (.t)

h into the remaining terms gives

= lim [nx"-1 + n(n- I) xn-l h + ... + hn- 1]

llXn-1 .

~- o

l1l s hon,

(3.7 )

This is called the power rule for differentiation. Although we have proved the power rule
only for 11 a positi"e imeger, it is in fact true ror every real number n. \Vc will ussume lhat
cquttlion3.7 can be used for any realnllmber n . and wi ll prove this more gcneml result in Section
3. 1I. For cxllmplc.

I _

dx

- (x '/3) = - x

and

2/3.

THE OREM 3 .4
If g(x) = cj (x), where c is a constant, and! j'(x) has a derivative, then

g'(x) = cj' (x ).

PROOF

i3 .8a)

By cqmuion 3.3 and Theorem 2.1 .


g ' (.r)

o8 <:::x_+:....:..:,h:>....---!!g..::.(x:.!.)
= h-lun
o
h
. f (x+ll) = c h-0
lim
,,

c-<f....:<_ __:
+_ 1_,.)_-_c~
f....:<-~r)
= hm
h-o
II

f(x }

= cf'(x).

Thus. for)'= f(x) . we may write lhm

d
d)
-d (cy) = c- .
x
dx
THEOREM 3 .5
If p (x) = .f(x) + g(x), where .f(x) and g(,r) have derivatives, then
p'(x)

J'(x)

+ g'(x) .

(3.9a)

P ROOF

Equation 3 .3 g ives
,

p (x) = li m
11-o

_:_+_/..:.')_-__,_P_,_
(...:..
<)

,_JJ_,_
( _,_.

. l.f(x + h) + g (x + lt)l - [f(x) + g (x)]


= li m "'-'---=--'"-'-- -,-"---'::....:.....:....___:::....:....:..:
h-0
h
. [ f(x + II) - f (x)
= ltm
h- 0
h

.f'(x)

+ l1) - g(x)J
+ :g(x
. . :.- -'--"--'--'h

+ g'(x ).

In shon, if we set u = f (x) and u = g(,<) .

- (11
dx

+ v)

du
= -

dx

dv
+ -dx
.

(3.% )

or, in words, the derivacive-of a sum is the sum of the. derivatives.


\ Vc now use these formulas to c~\lcula te derivatives in the following examples.

I EXAMPL E

3 .6
Finu dyfdx if

(a)

y = x '1

(b)

y = 3.x 6

x- 2

SOLUTION

(a) By power rule 3.7,


dy
dx

= 4x l

(b) Equation 3.9 allows us to differentia te each term sepmately; by equatio ns 3.8 and 3.7
it follows that

(c) If we write .Y in the form y = (x /3) - 2x -

dy
dx

I EXAMPL E

3.7

1,

we can proceed !IS in pan (b):

x2

2 = - + -.
= - (1) - 2(-x -)

..._..
If /(x) = 3x"- 2 , evaluatej' ( l) .
SOLUTION Since .f'(x) = 12x>. i1 follows 1hat j'(l) = 12. Geome1rically, 12 is the slope
of the tangent line to the cune y = 3x 4 - 2 .at the point ( I , I ) in Figure 3.15. Algebraically,
the result implies lhat at x = I, _y changes 12 times as fast as x .

FIGURE 3 . 1

Tangent line to y:.:

3x~-

2 at (I. 1)

)'

y = .l.r' - 2
(I. I)

l EXAMPLE 3.8
Find the cqumion of the tangem Iinc to the curve y

=f (x) =x

+ Sx an he J>Oint ( J, 6) .

SOI.lmO:-o: Since /'(x) = 3x 2 + 5. the s lope of the tungcntliole to the curve at ( I, 6) is

J'(l) = 3(1) 2 + 5 =

s.

Using point-slope formula 1.13 for a straight line. we obtain for the equation of the tangent line
1111< 10 a

cur.-. '-'

Xomu!
pe<pelldo,.-ulor to

at (1. 6)

)' - 6 = 8(x - I}

tan&tnlline
)

or

8x- )' = 2.

Tan.cent line

The line through (I. 6) perpendicular to the tllngcnt line in Figure 3. 16 is called the uormal
line to the curve nt (I , 6). Sinre two lines are perpendicular only if their slopes are neg;uive
rcciprocul; (:,ec equation 1. 15), the normal line at (I, 6) ntust h d i'C ~lope - 1/8. The equation
of the normal line to)' = .r 3 + 5x m ( I, 6) is therefore
X

~ -

l EXAMPLE 3 .9

- - (.r - 1)

or

..1

+ 8y =

~9.

Find, accur tc to four decimal places. points on the curve )' =


the slope of the t.tnemline i> - 1.

..1 - 4.r

x 2 + .r at which

~0 1

I I l i ON II i< al11ays wise to have an idea abom hmv many <ohuion< to expect for a
problem nnd Approximate values for them. (We might cnll this "boll -parking'' the nnswer.) In
thi~ problem. n plm of the curve in Figure 3. 17 ; hould provide thi< information. However. we
mu~l be careful in trying to estimme where the slope or the gnaph i~ - I. Because scales an;
difrert:nt on the nxes. a line with slope - l is not inclined m 1r /4 radians w ith respect to the

negative x -axis. The line in the ligure has slope - I for reference. The graph suggeslS three
points at which the 1angcnt line is parallel to this line: one just to the left of x = 0~ o ne just to
the right of x = 0. and one near x = 3.

J.'l Rutes ft l)iffec:nli.t!ic)n.


lif1CJII.iif 2

U "I

Fild.ng poir.Ls on a cun't: wftt.rc

40 y

tan~-cn l linc

ha.;

113

a"cn sklope

Line with dupe :;; -1

.lO

20
10

-2

-I

-10

-20

- 30
To cOilfirm th.is, we set the. slope of the tangcm line lO the curve cquaJ to - 1:
-I

dy

= -

tlx

= .tx 3 -

12.r

- 2x

2(2.t

6x

.r

+ 1) = 0.

Six-digit <oluoions of ohis equation are -0.459261 , 0.350168, and 3.1 09 09. To confim,
- 0.4593, 0.3502, and 3.1091 as soluoions correct oo four deci mal places. we calculme ..:(x)
2..r 3 - 6.t 1 - x
I at the foiJowing values:

g(- 0.459 35) = - 5. 1 )( 10- .

g(-0.45925) 6.5 " w-~.

g (0.350 15) = 8.0 X IQ-5,

g{O 35025) =

g(3. 10905) = -9 )( 10- .

g {3. 109 15) = 1.1 )( 10- 3

1bc zero intemlediate 't'alue theorem conl1rnts the rour.dec imal ~place approximations. Us~
ing ohe six-decimal -place approximaoions, and rounding results to four deci mal places, cor
reponding points on the curve are (-0.4593, - 0.2382). (0.3502. 0.0708), and (3. 1091.
-33.3326) .

......

Increment Notation
In Sc.crion 3. I. we have used the letter h to represent a small clumgc in x when cakulaling
deriviuives. An alternative not:uion that is sometin1es more suggestive was used in the incroduc
wry exnmples or tht\1 section . A srnall change in .r, called nn increment in .t , is often denoted

by D.x. It is pronounced "della x" since D. is the capi tal lcucr delta in the Greek alphabet
'-''hen x is given an increment l:l.r in the functiQn y
j(x). the corrc.sponding change or
increment in y is denOted by D.y . It is equal to

D.y

= /(.r + D..x) - /(x) .

(3. 10)

With lhis notation. equal ion 3.3 lakes the fom1

I
TilC olOiation

'

(x)

/:l.y

= 4'""
- ,
s -o D.x

(J.Il ,,,

dyjdx for 1he derivat ive fits very nicely with increment nOtation.
dy
- =

dx

ll.y
hm ,
tt.x-o f:l.x

( 3. ll b)

the derivative of y with respect to x is the lim it of the change i l1 y divided by the change in
as the change in x approaches zero. \Ve use this noLation in the followi ng example.

164

Chap1c1 l

DiiTeuntllwic.1n

I EXAMPLE 3 . 10
Use equation 3 . 1 I to calculate the dcriv.lli\"eor y = .f(x) = 3x 2 - 2.t. and check your answer
using the diO'ccmiat.ion rules cJiSCliS~ed earlier in this SCClion.
SOLUTION

Since

+ C:.x)

D.y = .f(x

- f(x)

= [3(x

t.x)2

= 31x1

2x t.x

- 2(x

+ D.x)] -

+ (6xf] -

[3x 2 - 2x]

2x - 2t.x - Jx 2

.C.x(6x - 2 + 36x),

.
Ax(6x - 2 + 31l.x)
hm
=
A x-o
Ll.x

+ 2x

equation 3. 11 gives

dy

dx

ll.y

=lim-

A..l-o Ax

lim

t:. ~- o

(6x - 2 + 3.C..r) = 6x - 2.

Power rule 3.7. and rules 3.8 and 3.9 for d ifferentiation ol' 3x 2 - 2x, al~o yield(.- - 2.

Figure 3. 1811 shows parts of 1wo 1rnnsmission lines. ooe ~lntight. and the other in the siUlpc
of a parabola a. it circumnavigates a lak<. Numbers represent lengths in kilometres btu
no a11cmpt has been made to at! here to Hscale. Tile I\VO cransmi:>Si on lines arc to be joined
by a l.hird that should be a.< short as possible. We are to lind its length and where it should
join the existing lines.

Schem:u k

f'IOURE 3 . 18b

(or ft1tclin;; .shl>ttcsl distUI1(.'"e bC'NietO IW() ff'<ll\llnliGt'iOil line,.;

Parabohc transmiss-ion tint.

tOO

40

Slntight ltan~>ntis!lion line

f;wn

SOLUTION We begin by choosing a coordittatc system to facil itate analysis. Since


equations for straighLiincs arc suuightforward in nny coordinate system. we choose axes
to simplify the equatio n of the parabola (figure 3.18b). In lhi$ coordinate system. the
equation of the parabola is y = 30.x1 and the equation of the line is x + 2y + 100 = 0.
Our problem now is to find the shortest distance bet ween the line and the parabola (which
must be a suaight li rle distance} 1W1d ~h e poirHs on tht"sc cutvcs at which the line segmem
sh<'>llld he drawn. l.etthe required poims on the straight line and pamllola have coordinates
P(a, b) and Q(c, d), respectively. This is an important ste p in our analysis. When
unknowns are required, idcntiry Lhem, in this case give names to the coordinates of the
unknown points. In addition, do not designate either one o r the points by coordinates

3.2

Rules fOf' Oift'trcntia.tion

18&

(x.) ). Leuer:. x andy already identify var iable points on thc~trig)Hiinc and parubola:
to identify fixed points with the same letters would lead to confusion. Now the big step.
Qf the infinite number Of (_inc segments join_ing points p 00 the straight lrUfl$mission
line to poini' Q on the parabolic trdnsmission line. \\hich i< <hone<t? Our intuition tell'
us that the shortest line segment is the one that is simultaneously perpendicular to both

RecogniLing this. we could auempt to ballpar~ pu~ition!) of P and


Q by ploUlng the existing transmission Lines with equal scales on the axes (othernice.

uan~mi~iOfl line~.

Mjhllit~

length.> and angles are not what the) .eem). We ha'e dooe this in Figure 3.19 and drawn
a line that <eem< perpendicular to both tran$mi<sion lines. It inteNeCI< the parabola very
close to the origin so that coordinates of Q should both be cl05C to zero. Coordinatu of
P "<Mild ppear to be close to ( - 20. - -10).
Usong slopes we can now set up equations for the unknown coordinotes. Since the
,rope of line ~gmcnt PQ i, (d- b)/(c- a). oklthdt of x .._ 2y + 100 = 0 i' -1/2.
pcrp<!ndoculariry rcquues

d-b

= 2.

c-a

T11c slope of the tangent line to the parabola at any value of .t i; 60x. oo that the , lope
of the tangent line a t Q is 60c. Since the slope of the normal line is -1 /(60c), and this
n1ust be para11el to line segment P Q. we must haYc

d-b

-- = -

c-a

60c

\Vc nuw have two equations in the four unknown coorc.Jirwtcs. 1\vu more C4tnttion:, CHrl
be obtnined by using the fact that P is on the line x + 2y + 100 = 0 and Q is the on the
parHbola y = JOx1

+ 2b + 100 =

\Vhen we equate right sides of lhe first two

'2 =

0,
equation~.; .

we obto1in

c=- -

120

60c:

Thi' now implies thai


I

480
We ;ub>titute the>e 'aluc:.\ for c and d into the first equation.

=>
If \\C double this equation and add it to a - 2b

Sa=-- 100

80

8003
a = - --

3
t60

= -100. we obtain

400

2a- b =

8003
3
- -=
200
160

b= - -

31997
--800

Thu.therequircdpoint on the straight tr.msmission line is ( - 8003/400. -31997/800) ~


( -20. - 40) and the point on the parabolic line is (-I/ l 20. l /480). These agree with
our poedic tions for positions of P and Q . The length or the line segment joining the"'
point.~ is
2

IIPQII =

I
8003) + ( - I + 31997)
- 800( - -l 20 + -400
480

~44.7.

The shoncst transmissjon line joini ng the e.x.isting lines is approximately 44.7 km long.
Since the length of the line segment jo ining P to the y -intcrccpt of t he str.light transmissio n line is j(SOO'J/400)2 + ( 50+ 31997/f!I.X))2 "'22.4. the now tran"nis<ion
line should begin 62.4 km west of the town and be 1>crpcndicular to the existing straight
transmission line.

EXERCISES 3 . 2

signal x(t ) = co.~ ( 1000JU 2 + IOO;rt), whc1c: 1 is time in f.OOOnd;;.

In Exercises l- 20 find/ '(.<).

= 2.< 2 - 3

l. f(x)

2. j(x ) = 3.r '

3. f (x) = 10... 2 - 3x
5. J(.r)

= l {x 2

7. j(X)

= Sx -

9. f(x) = x
I I. f(.r)

Jx 3 + I{X
l

15. J(x) =

2
r.:

vx

2x"' - 3x U3

= (x 2 + 2) 2

19. f (.r) = ''''-

8. j(.<)

2.t

" ''

=- - . + -

x''' + 3

12. f(x ) =

' -

:!.<

ot

30. Al what poinl(s) on Lhc curve )' =


umgcnt line p:~ through chc origin'?

31. At "'hal point(s) on the p.'lfabola y = x 2 docs lhc nornmllir~c. pus


1hrough the pomt (2. 5)'? Can }'OU 5-1.1ggcs~ un oppljcalion of thi1; rcsul l'?

= .rl.2 +.r'1'

16. f(x)

= ;rx"

18. f (x)

= (4.< 6 -

20. j(x)

= (2.1 + 5)1

.t 2 )Jx'

In Exe1-ciscs 21- 24 find equal ions for the iangentand normal li1x:s 10
the <:ul'vc a1 the point indicated. ln each case. draw the cuvc and lines.
2 1. .)' =

29. Tbc general romtula for a <:hirp s.gnal (sec Exc:n;isc 28) is ,, (I) ;
cos (t>t 2 + fl t + >), where ex > 0, {J > 0. and 9 on: consti!nl$.
TilC d erivalivc o f cw 2 + fj t + t/J , divided by 2rr . is the inslanwnoous
frcquaq of 1hc sigmd. If lbc signal begins lcllti_nlC 11 and end!S :n ti nc
12, what is lhc diCfertnct in initial and rirtal fn:q ucncics uf1hc signal?

./X

14. /(.t)

+ 5 . , (2 . 5)

* 3.$. A hill is bes-t described by a paruboht contnining the three poinis in


the figure below, nil mc.asurcm-t:nl..'> in lYIClrCS. A lrunsmiucr 30m high
stunds at the point (- 120.0). Wrnu is thecloocs1 point 10111.: ba~
of' the hill on the positive Xru<is th:u a r('(."Ci"cr cnn detect the signal
unotx1ructed hy the h ill?

UL which Lhc ~lope ot' 1hc IU11COI li ne i:t> 2.

+ 26. Show thm lbc X intercept or the lmgcnt line U1 uny point (xa. )'o)
on Ihe l~lnlhc)ta y

= Xo.

=ax1 hit.ecl~lhal purl of thCX:IXisbclwccn X =0

* '27.

Draw a gr.aph ol" the ll mction x = f (t) = tl - 8t 2. Find tJte


value(s} ol" 1 at which the langent line to this curve is parJ.tle l to tJte line

X=

i *

)'

20

JY+i' Ul (3, 8)

+ 25. Fi11d ti1C Jk.\irliS on the cur"c y ; x~, / 4- 2.t3/3 - 19x1/2 + 22x

and x

22.\' + 20 doc.:s the

3J. Sho\V that the line scgmcnl CUI from dlC UUlg<:11t line lU u poim p
C)l1 the cmvc y = I f.-.: hy 1hc coordin~ t c axes is bisected by P .

= JX + 5 at (4. 7)
)' = Z.r 1 - 3.<1 - 12x at (2. -20)

24. X

x3 + :r 2 -

32. Show thallhc sum of Ihex 81lCI y ~irHCtC<:i> l ,. Clfthc tangent I inc lo
the curve ./X + .fY = ,Ja is ulwiys cquulto a.

22. _v

23.

;S 0.1. Notice how the frequency

4.\'

.<

= A.c; - lOx '+ 3.r

10. /(x) = 5.r ' + -I ,

0.

1 (.<)

as t incf'Cascs.
(b) The frequency in hen ~ of the .;ignal, Ill ar1y gi\'Cil lime, is
dcfinedas thcderhativeofthep/we 1000nt 1 + l OOn/ a
lhat time, divided by 2:n. Find the rrcqucm:il!.i oflhc signal
at 1 = Oand r = 0. 1.
i1lCI"caSCS

6. /(.<) = 2/x '

= 5x - + - -5
4.c-

1 ~, f(,~) =

17. f(x)

10

~-

(a) Plot x(r) for 0 ;S

+ 4.t + 5

6t- 3.

28. A chit]J signal in ;u:ouslics is a signal whose fre<Juency changes

Lincwty from a low value lo a hjgh value. f or c~amplc. consider tJ1c

+ 35. Give nn ahcmutivc dcriwtion o( power nlc 3.7 bused on the idcn ~
l ily

a" - b't = ((I- b)(n"- 1

+ a"- 3 b + + ab-l + b-) .

36. Find all pair~ or point:> on Lhc cur\rt;S ." ; x2 und y ; - .t2

+ 2x

- 2 thnt slwrc n conmlon lll0Cill hnc.

* 37.

Show thai the tangent lines al any two points P and Q on the
parabola y = ax'1 + bx + c ilttcrse(.'t at a poinl lhal is on lhc \'Crtical
line halfway lxlwccn P and Q (provided lhat neither P nor Q is at
the vertex of the parabola).

.
JS. Fnd u fonnultl
to

39. Pnwe Ch:u

((II'

- l.t IN Y.hcn n >

(/.;a

,,

- (ax+ b)" = an(ox


J.,.

when

I il- an i ntcl:,'t"''.

+or-

f.

a and b arc oolSUmcs. m\!J rr > I is an itlle.g<:t'.

-'0. Find lhc lWO poi.lllS Oil lhe\:une )'


a comn10n (angcnt lint..

= x (l + 2.x -

xJ) th :u shar~

13.3 Differ en tia bility and Continuity


Many
E(!1

Em!J1 F'mtt:cion

j (.t) l,,f has oo dcri\~.. ti\'e

~ll

" 0

fu ncti or~

fai1 10 have a <.lcrivacive at isolatcc.l points. For examJ>Ie. consider the function

f(,Y)
f.tf io~Figuro3.20. lt isclearthmfou > 0. J'(x) = t, <ll'odforx < 0, .f'(x) - 1.
At x = 0. however, there is a J)fO blem. If / (.r ) is 10 have~~ derivati ve m .l = 0 . it must be
gi\'CI) by
. Ihi
. j'(O +It) - j'(O)
I un
Inn - .
l- 0
"
h"'"'7(l 11
Our 1his limit does not exi:,l :,i1M.:c

.''

lhl

hm -

=-

and

I'I

ll- O'

hm -

11-o ''

= I.

Consequently, .f(x) = fx f dncs !lOt have a derivative at x = 0.

nte

same conclusion cnn be drawn (ll any point at which the graph of :.l function takes an
ilbfll j)l ch3nge ill directiorl. Such u point is o ften ca lled a C()l 'llt:l'. As u rc~ull~ the rllllCtion in

Figure 3.21 docs not hi\VC a derivative at x ~ 11, b . c. or tl. h i~ also true that a funcciQn ~-mot
have ;1 derivative nt a poim where the function i~ discontinuou~ (!<CC, e.g.. the diSCQtttinuitics in
Figure 2.27). This l'e.~ult is an imnediate consequence of the following theorem.
whc~ 1t

'-'OAtinuous futM:t iOl fls

THEOREM 3. 6

no derindi\l:

If a function has a derivati,e at

)'

:
...

~
.

'''
'

(I

.''
'

''

'

'

PROO}..

x = a. then the fu nction i~ contir1uous en x = a.

~~~

f (.x) =
=" (Dcfi11ition 2.1 in Section 2.4).

To prove lhis theorem we~how tlmt exi~tenceof f ' (tl) impl ie:o:, thatlimx-r1
at~

[(a ) . the condition that dclincs continuity of / (.r)


We show llwlim, _ l/(x) - f(a )] = 0:
(/( }
/'( )
:r - . a ]

. [/(.~ )- / (ll}
= ,~~'!,
.\' _ a (.x -

>] = [1.tm .rc.~x>-- a.r <")]lt\-o


],
tm c- n >

J.:

; r -11

provided both li miL~ on the right exist. Qb,iously, !l'IC second limit has Yaluc zcm, and the first
is definition 3.2b for j"(a) . which we have assumed existS. l l1us,
lim [f(x) - f (a}]
.\ -t- (1

J'(a) 0

= 0.

Tile following result is logically cquivalcm to TI1corcm 3.6: it i~ called the CQIIIItlfiO$itlve
of the theorem. Although e<luivalent to Theorem 3.6, we shall designate it as a corollary.

COROLLARY 3 .6.1

If .f(x) is discxmtinuous at x =a, then .f'(a} does not exist.

168

C~)(tr 3

Uifferc:rwiation

Studenl.S arc ufccn heard to~y that h~wing a c.lcrivcttiv.; is <.."tjuivalcnt to having a tangent line
are slopes of tangent lines): that iii- a runct ion f (.<) has derivati\'C f'(a)
at x = a i f anu only if the gnoph o f /(x) has a tangcm line ;l U' = a . n,is is not q uite true.
For c xonplc. con~idcrthe func tion f(x) = x t/ l (Fig ure 3.22). At (0. 0), the tangent l ine to
the gr:>ph is the )'axis (use the definit ion of 1Cwgem line in Section 3.1 to convince yoursclr of
this). T he derivative or f(x) is J'(x) = (I /3)x-21'; it does not ex ist at x = 0. Tht~ we
have a tangent line, but no dcti,ati vc. 11lc rcetson is that the tangent line is a vcnicallinc. and
\'rtical I ines do not ha\'e slopes. \Vht.u can we ~ly. theu"t 'T\vo things:
(bee~ usc <.lcoivati-

l. lf /'(a) exists, then)'= f (x) has a ta>gcmlinc at

(11,

f (a)) with slope J'(a).

2. If / ' (o ) does not exis~ then y = f(x ) eithe r has a vertical tangent line at
does not tuwe :awngem line when x ~ a .

Mjldil;l #,

=4

T<~nzc:r~t liBe 1~

y = .r 1fl at (C), 0) i-s.

tll

(a, f(a))

or

.\' a.xi-s.

Ttu-.gent line
Ill (0. 0)

We introduced this section by showing that the derivati ve of lxl is I when x > 0 ; it is - I
when x < 0; ~md it does not exist at :r. = 0. The."e can be combined into the simple formula

d
lxl
-lx l = - .
dx

(3 .1~ )

lt is Slraightforward to gcncroliZ cqumion 3.12 a nd obtain the dc riV>ltivc of If (x) I ar any point
at which f (.r) ;f. 0 ;md f'(.r) exi<L, , When j(.r) > 0, we may write

d
d.t

- 1/(x)l

On the other hand, when f (x)

< 0 , we ha\'C

~lf(,r)l

dx

= -dd.r. f(.r ) = f (x).

~ I-.{(x}) =

dx

- .f'(.r).

Both of these result:') arc conlained in the one equalion

_ l.f(x) l j'(x ).
.f(x)

(3.13)

This is the derivative of 1/(x)l at any point a t which f(x) ;f. (). The exce ptional case when
f (x) = 0 is discussed in Exercise 28.

l.J Ditr'em1liabilil}':lnd Contintail)'

169

Right and Left-Hand D erivatives


When a function f (x) is defined on a closed interval IJ !: x !: c, C!(luation 3.3 can be used to
b < .x <c. For inbtancc, it is impos..~iblelo
eva.lua1c
("'(b) = lim /(b + h) - /(b)

11-U
/1
c~tlculatc /'(.\")only :11 points in ahe open in1erval

since j"(x) is no1 defined for .r < band therefore f(b + /1) is not defined for h < 0. When
a function /(X) b doli ned only to the ri&ht o.>f a poin1, we dclinc a rigla-hand dcri\'Oti\"C at the
point; and when f(x} is defined only 10 I he left ol' a point we define its left-hand derivsti,e.

DEFINITION 3.2

n1c right-hand duivalive of /(X) wilJ1 rt"()OCI to X is defined as

'( }

+x=
f
lun

h-OI

/ (.r+ ll) - f(x}


,
h

(3. 14a)

provided dull the limi1 cxisiS. The left-hand derivathe of f(x ) is

f :_(x ) =

lim

,_0

f(x + II) - f(x)

(J.I~h)

h.

if the limit exisL").

The lefthnnd derivative at a point is tlOl confined to the situation whete a funclion is defined
only 10 1hc lefl of the point: nor is the right-hand dcrivmivc rcstrie~ed 10 1hc situa1ion where 1hc
function is defined only to 1he right of the point. We may consider left- and right-hand dcriva1ivcs
at any point. as well as a ''full '"derivative. Ob.,iously, when a function has a deri\'ative ala point
x. iiS right- and left-hand derivative:> hoth cxi>t at x and arc equal t~> J'(.r) . h is possible,
however. for a function to have both a left- and a right-hand derivative m a poi n1 bu1 1101 a
.JorivaliV<. An <Xarnt>l< or lhis is th< absolulo V"diUo funclion f(.~)
I~ I at X
0 (Figure
3.20). h; right-hand derivative 111 x = 0 is equallo I and its lcti-hand dcriv"tivc there is -I.
When a function has a deri,ative m x = a. we say that it is difl"en:ntinbl e at x = a. When
it hus ~~ dcri\'ativc at c\'cry point in some intcr-.al, we :;ay thut it is diffcrcnliuble on thul interval .
In the event that the inlerval is closed. IJ S x S c . we undcrsta~~d thai deriva1ivcs at x = IJ and
x c mean right- and left-hand dcri\ativcs. rcspcc1ivcly.

I EXAM PL E 3. 1 1
Wll!lt is Ihe dcrivalivc of the Hcavisidc funclion h(,t - tt) in1r<.x luccd in Section 2.51
SOl UTION n,c Hl ngenl line to the grdph o f 1hc function (Figure 3. 23) is horizontal '" every
point except x = tl , where the func1ion is undefined. In mhcr words, h' (.r - n) = 0 except at
t1 either.
the disconti nuity x = a. Right- and lcfl-lllu>d dcriva1ivcs do n01 cxisl "' x

MOjiUII;I!IIC\fF\

Tt111gcnt Jine to ) = h(A' - a) is ltcri-zcnu.l e:\ccpt

a x = a

)'

y=h(x-a)

By examining thcdcflnition ofthcdcrivativcof h(x -a) . a very useful rcsuh for apJ>Iications
emerges. According to equation 3.3.
'(
)
Iz x- a

.
h(x
1om

+ {).x -

A-0

If we change variables in this limit by setting f =


,
h (x - a)

. h(x -<-o

ax
- ax ,

-a) - lz (x- a)

lorn

a ) - h(x - a )

. lt(x- a) - h(x - a <- o


e
hm

f)

But a<:<."Ording to equation 2.9. this is the definition of the Dirac-delta function S(x - a). In
other words, we may write rha< h ' (x - a) = ~(x- a) .

I EXAMPL E

3 . 12

Is the function

sin(l / x).
() _ {x
0,
2

'# 0

X=

diiTerentioble at x = 0?
SOLLilOJI: We enc<>unrercd the function .r 2 sin (1 /x) in Example 2.9 of Section 2.1 and
drew irs grnph (exCCJ>I for the point at x = 0) in Figure 2 . IOa. According to equation 3.3, the
dcrivmive of ~(x) nt x = 0 is
g

' (O)

g(O +h) - g(O)

h-o

It

= 1tm

provided that the limit exists. When we substitute from the definition of g(x) for g (h ) and
g(O) . rhi> limit rakes the form
h1 sin( l/ h) - 0
g,(0) = lim _
_:....:.......:...._
= lim h sin
h- 0
,,
h- 0

(.!.)
h

= 0

(See Exercise 5 1 in Section 2.1 ).

Angle Between Intersecting Curves


Tire angle 0 betw""n two cun es that inttrSrtt at a poirn (xo. )'o) (Figure 3.24) i:, deflne:U
as the nnglc between tbe tangent Jjncs to the curves at (.<0 y0 ) . This enn be calculated using
formula 1.60 OOQC slopes of the tangent lines are known .
In the event that 9 = ;r /2 (Figure 3.25). the cur-os ore said to be orthogonal or pcrpcnuicrolar at C(o, Yo) . Should 0 = 0. r.hc cur-oes arc <aid to be tangent at (xo. Yo) (Figure 3.26).

!lm

Angle

bc.l'wcen
)'

t':W'\'~ l11UJ.

imcnc4.'1.

CUI"\'e) 4 1 ( ,tfl, )'rt)

Tangenl

J!'

J:lrl~

CUI'\"C) 81

T'n_gen1

(.GJ, Ja)

)'

lines 10

)
Cro. Yci!

Tangent
line lo
b<~h

curves al

(xo. Yci!
X

3.3

I EXAM PLE

---

3 .13

Find the Mgle between the line .<

+ 2y

Oit'fe.~n tiabi l ily and Co1Ui11Uil)'

= 5 and the curve y = x 3

171

+ 3 1 at their point o f

i mcrsection.

5fl

x+2y=5
5

'

SOLL n oN A quick d iagnun (Figure 3.27) a llows us 10 bull park the location of the point of
ionerscction of the curves. To Nnd il. we solve the cquatiool of the line for )'= (5 - x)/2 11nd
equate it to .l. .l - 3 1,
.r 3 + 3 1 =

5- .r

2x 3 + x

+ 57 = 0.

Accordi llg to the nuio1lal root theorem o f Section 1.2. theonl)' possi ble rational solutiorlS ofrhi~

I
3
19
57
1, 3, 19. 57. -. -.-.-.

2
2
2
2
f'igure 3.27 makes it clc.a r that we should only ll) ' - 3. It is ind~ a solution. and the only one.
The point ofinterscctiO<l of the curve.~ is d>crefore (-3. 4) . The s lope of the line is - 1/2. and
to iind the s lope of the cubic, we calculate thlll tly f d .r
3.r 1. The s l,>pe :11 ( - 3, 4) is 27.

Usi ng fo mlula 1.60. the tlCUte angle between the curves atthdr point of inter:st:c.:ti on is

() =

Tan -

27 _ ( _ 1/2)

1I + 27( - 1/2)

= I . I rmlians.

It looks much larger than this. Why?

E XE R CI S E S 3 . 3

In Excrc~iCS 1 ~13 delcrmine whee her lh~ xtatement i~ trut o.. rat~.

In Exercises 1-6 dctcnnin< wbctltCI' the funclion hu a right-hand


dcrivmive, a lcfthunddcri,sti vc, und a dc.rivmi\Cti.llhej:!i ven vuluc of X .
I. f(x)

= lx - 51 al .r = 5

2. f (x ) =

x'''

.1. f(x)

= lx -

4. f (x )

5. f (x )

= (x 2 -

6. f (x)

10. If a (unction j'(:c) h01s a dcri\':Ui\'C :11 x


tangent line ill the poinl (a. j {u)).

01

~nx a1

h:a~, :1 dcri\'uti,c:

=5

at x

(a . f (a)). Ul<:n il

=a.

ll. If a fun<tion does n<JII"'v<: wngcnllinc al (a. /(a)) ,I hen it docs


no1 h:t\'C n derivative at x = a .
U. If a rWlctioo /(x) docs not have a dcriv-.Ativc at x
a. then it
d6e-1i rw have a1angcn1 line fit (a.f(a)) .

x = 0 (Sec Exercise 47 in Scclioll 2.4.)

1)/(X - I) al X

[x j al .<

tl, chen itsgr:tph h~ :t

11. If a func tion j'(.r) has a tangent line at a poinl

x =0

51 3 at x

= I

ln E."~tcrdscs 14-16 show algcl>rak."ally tholt / 1(0) docs not exist IJww

I !Sec Exercise 68 in Section 1.5.)

a graph or the (unaion.

7. Oi>es it make any diffCI-cncc in Example 3. 11 if we detine h (a )= 0'1

*
*

8. Docs i1 make a ny difference in Exarnplc 3 .1 I i f we define h(a)

=l ? *

9. Does iLmake any diiTe1-ence in Exctcisc 4 if .sgn x does not have a


value at x = 0'!

*
*

J4. /(.<)

= x IJ

15. f(x) = x'll

16. /(.<) =

x'''

29. 1r h nl,_" j (X)


f. {M) th:il ,\' (. ia, a hc\n70nlal uynlploM:
(m thC: !ll'....,tl of the (unction). i~ it llCSSaf} 1h;.r limx !"\) J'(x) 0?

In Ext'fCik"tl 17-18 fi nd lhCallICi (Ot' analcto) bclwccn the C:llf\("-'~ ( lhcir


point (or points) or irtl<nt-"Ciiun

i'

17.

\'=.t:.

x-+ \

18. \'= X : ,

)'&

or

30. The Green's ru.JKiiM for displOJCCO'I(Ill' a IIlli lllrin& \VIIh con~
$Lilli tension f and lcnglh L.. und ends fi!U:J al x - 0 tJt'ICJ \' t~ ,.,
the X-a-'t&S IS

l - r2

In EurciK.) 19 20 dcccnntnc lo'hcdu lh..: <.uf\~ :.rc orthop1.ll.


19 .'( - 2,\

+ I = 0,

20. ) =l -.\ 1 . . .,

')' l

:!2. y a " '

23.

(Q)

.\ :

) ' a ,1 l

+.t

JtC 1 .1r'l~cn1

the rndit.I'Cd

(a)

l.rat (-1.-:t)

Oraw CUf\'C~corrc'Jk'l't'ldtftJ lOC,

2. -1 .0.1.

~-~

J2.

/'().
18. If f

din"crcrUitlblc ul .x

/(.<)

A
, \'

0
0

Ia

,.

difl"cra11i:tblc al. x = 0?
J..&. ls &he flux:& ion

Unw ~"'1>11> or j(x) 111xl

(x) cJo' ..a dtfTcn....,~u~hlc. futXtKu1, doC'~

0,

01

f(x) =

AI potDI> ~h<rc

1...... ( 1/ .<).

.fC.r>= Lx J 1<r1 lhc floor

26. l>lh<: fum!lic>n I IX) = xlxl diiTc:n:nliablc Cll X

11.

the funclioll

JJ. For wh~ \11ltacs o(lhc real numhcr 11 iii the fu nction

.. 24. m rwh:11 vllhk' ufk. rc I he oncpanuncttr rwntltcs 2x - ;.\v - C 1


and .r + .l:y = CJ onlmQtM13llrll.jec1ories !
.

=lXI +-I

I~

.f(xl =

(a) aod (h) ortf1(~Unt l t r~jc4.;w ri c~t7

,,r

L/Z).I>it oymut.rkol>oul

L/2?

Jl . Me f' (o) or><l lint f'(x) the s.lnl<71 11ustoatc woth h(x - n).

(b) Oniwn lew curve~ (n'lm thc,'W\Cp.ttun\.."'ter (tul1tly 2.r- y ==


C 2 Otl the &rnl>h i n 1>1ut (u).
(c) 1'wo f.uniltCI'o O( CUI'Vt) ai'C SJuJ hl he anltf!~Ottlt llrtj}t!f'fO
rlt'.) ' ' every '-"rvc fr'Uin lilt\' (tlll ,il)' 1HICI'$(:CIS C\'(ty C:Ut\'C
fwmchc olllCf fnrnilyorthOJOI11tlly. Arc lhC: (,unil i~ in p:.~

27. Find f'lx) ir .f(x)

.t)h!.t- X)).

!b) O..w a cr('h of G(.t: XI when L/ 2 <: X < I.


(c) Show al_gd)taically th~ G (.\; X) '' C\)(l!inuoth lur 0 <
.t :S 1- . cxc<pl fo r a n:mo,abiC' diiieon&muity ~~ x ~-. X .
(d) Show rtl'l.ll dG/d.x is (,:ooinuou." for nil A. f.. X . :and 1\J_.. u
jumpili=ntinuily liLt= X or - 1/ T

c.

t>mw u gruph 1he r_.ncricln /'(,~) wt.cn


function (Exan11)1C 2.22 in Scx.1ion 2 4).

Drawa~~ofG (x:
A=

I ot (I. I )

2s.

+ X(L

X) i' me llta\'K.idc fIK"Iioa ia S-taon 2..5 Thil'lt or


G{.t: X ) ..u; .1 runa10n ol x l.llbl aliO Jcpca<b oo X ..,here X 4..'\lll hm,:
uy \'alue bctwe!n 0 :md L ll is lhe di.q1lxem:.m :at posilioo x in the:
s.nn.: ira unit rO<cc '" the pos.iti'-e )'dmtil)'1 i1 uppi-<:d to dlC ~olrina
at pos-il.ton X .

1ltc C(JI.t.\lion X I 2J = I wtl.._-re


b u COilStMI. rtprc-Stlll'i a flf11" 1Kfnunrttrfimul)~ of c:wn.v.r. hY coch vaJuc
\tf C 1 the rununc tc r .a dilrcn:m (.'\U've irt &he rum1ly is ob

wined.
2.

lx(L- X)h(X- x)

.. here h (s -

1= , ,

.r'

Lr

Ia Ettrc-isti 21 -22 liYow ,._,. lhc Cllf''f'


poinL

21 . .\' - A-lz:.

G(..: X) = -

1/( \)I ha'c alk.,'I\Ui\C

= 0 Jltnt: l)ru~ SOniC pictures.

x'.

\" a rational nun\bcr

o. xu in"l.tio.ul number

dllfe101tiublc 81 x = 0?

13.-t Product and Quotient Rules


In thts 5Cction we add two more fomJUias to those: of Section 3.2 for calculating cleri\llthc>. 1llc
first is a rule for diffc~miating a function that is tlx product of two other function<.

T H EOREM 3.7

If p(x) = f(x)g(x). where f(x) and g(x) are differentiable. then


p'(x) = f(x)g'(x)

+ f '(x)g(x).

(I

I SJ)

PROOt' By cquacion 3.3.


,

p(x + It} - p(.v }

J) (X) ~ h m

11-0

'

[(.< + h)I/(X + h ) - j(,l')/1(.<)


.
h- 0
IJ
.

IIIli

To organize this quoticllt fu11hcr, we add nnd subtract the quantity f(x + fl)g(,t) in the flmner

, =.I

tUor:

p (.<)

l.f(x

Inn

+ h )I/(X + h)

- [(.v

+ IJ)R(x) ) + lf(x + h)g(.<)

h-0

- f(.r)g(.<)) }

"

. [!< x + J)g(x
1

lnH

ll-o

+ h ) - g(x)
!J

+g

( ) f(x + h ) - f(x) ]
I

= .f(_,)g' (.r ) + g(x).f' (x) .


In Ulking the limit of the firsl tenn, we h;wc usc<.J the fnct that linlh-0 J(x +h)

= /(.t ),

which follows from continuicy off (x) (see Theoren1 3.6 and Exercise 46 i n Section 2.4).
ThLs result i.s called the product rule for differentiation. Ifwe set u
j'(x) ancJ o = _c:(.x ),
then the pro<luct rule may ctlso be expressed in the form

d
du
- (uv} = u d.r
dx

du
v- .
dx

(3. 151>)

For a change. we use i11crement notatio1l to prove the quoUent r u_le. Use of IJ ill place of
l!.x works equally well .

THEOREM 3.8
If p(.v ) = f (x)/ g(x), where j (x) 11nd R(x) are differcnciable. chen
,

P (x

P RCHW

U~ing equation

g(x)f'(-<) - f (x)g'(x)
[g(x)]Z
.

(3.16.11

3.11 yields

'( )

fl .r =

1lffi
~, -o

p(x

= Icm

- p(x)

ll.x

lim _ I_
~-o

+ 6 ,<)

t:..t
f(x

" '-o

[f(.< + Ll.x)
g (.<

+ Ax)

+ 6x).~(x)

{(.<)]
g (x)

- g(x

t.xg(x).~(.<

+ 6x) f(x )

+ A.r)

To simplify chis limit. we odd lmd ~ublrncc .f(x)g(x) inlhe numenuor:

'( )

p x = lim

6.r ...

(j(x

+ t:.x)g(x)- .f(x)g(x))- [g(x + 6x)/(x) 6xg(x)g(x + t:.x)

lim

6.r ...

o g(x)g(x

+ t:.x)

{ g(x) [ /(x

+ t:.x)
t:.x

f (x)g (x)]

- f(x)] _ .f(x) [ g (x

+ 6x) t:.x

g(x)] }

174

Chapler 3

Oitfertnlulion

I
{ ( ) 1.un j (x
1 X x
lg(x))
~-

+ L\x) -

f( x ) - j ( r ) .101 J: (X
1
~x-o

L\x

+ L\x) -

g(x )

t>x

= ~
g <:..:x~)f~'.:.:.
<x.:...
>_
-__:l~<::;
x>:.:::11....:.'(.~t)

lg(x)]2
If we set u

= f(x ) a nd v =

J1 (X) . the11 quot iem rule 3.1 6a can be expre5$Cd in the fom1

du

d11

.!!_ (~) = v-;;;_ - 11 (i;.


dx

(.l16b l

v2

lt mokcs no dilr erence which tc m1 in product rule 3. 15 i~ written first: it dele.~ make a
ditlCre1H.:e in the CII.IOticnt ru le. Do not in1crchange the terms in 3. 16.

I EXAMPLE 3.14
For the tb llowing two fu nctio ns. find J '(x) in sim1>lificd form :

(a) f(x) = (x 2

+ 2)(x 4 + 5x 2 +

I)

(b) .f(x) =

.jX

3x 2

SOL UT ION

(a) With pr\Xl~~~:l r1ol' 3.15,


J'(,t) = (.t 2 + 2).!!_(.t 1 + 5x 2 + I) + (.t 4 + Sx2 + l}dd _(x 1 + 2)

= (x2 +

2)(4x 3

= 6.x s + 28.t '

+ lOx)+ (x'' + 5x 2 + 1) (2x)

+ 22x.

(b) With quotient rule 3. 16 .

'

1
(3x 2 - 2) ( - - ) (.<)

2../X

j.i'(6x )

(3xl - 2)2

I EXAMPLE 3.15
DrJw a gmph o f the fu nction y = .f (x) = (x - 1)/ (x
qualitatively that the graph agrees with your calculation.

+ 2) .

Calculate d yfd x and show

J..l Product :lo<l QuOtient RuJes

JIIIQURE 3.28

176

Rclaliousbip bc.1ween tJ)C;: tleri\'atj\e of a func1ion and ib graph':. .shape

/ I

I .............. .
-2 :

- l l

SOLL'TION Limits were used 10 draw the graph io Figure 3.28. Using quotient rule 3.16, we
find that
dy
(X+ 2)(1) - (X - 1)(1)
3
=
=
(x + 2)2
(x + 2)2
dx
The sketch in Figure 3.28 and dyj dx agree that:
(a) The slope of tbc curve is a.lways pos itive.
(bl The slope becomes larger and larger as x approaches - 2. either from the left or the
right that is.
lim / ' (x) = 00.
x- - 2

(c) The slope approaches 7.ero as x approaches oo: that is.


lim j'(x) = 0.
x.- *oo

Stalemems(a). (b), and (c) in Example 3.15 were arrived at by examining d y/dx = 3/ (x+2) 2 .
They could also have been realized by drawing a gl"dph of the derivative function (Figure 3.29).
Ordinates of this graph are slopes in Figure 3.28. For example. ar x = I, the height oft he CUT\'C
in Figure 3.29 is l/ 3. At x = l in Figure 3.28, the slope of the tangent line is 1/ 3. Notice
how clear slatcmculs (a), (b), and (c) arc from lhc graph of .f'(x) in Figure 3.29. The slope is
always positive, limx- - 1 f ' (x) = oo. and lin't<- oo f'(x ) = 0.
M:Jirlii;I#Ftl'

The derili'Jiive fluK'tion

-2

lo Chapter 4 when we apply dc.rivativcs to gcom<.rric aod physical problems. we occd 10


~nd when i( does nor ex ist. and when il is positive. negati\e,
nnd zero. A graph of the d criv~uive fw1ction is an excellent way to discover and visualize these
properties .
know when the derivative exi~ts

I EXAMPLE 3.16

.........
Plot a graph of the function/(-~) = (x 2 + 4x- l)/(x 3 + 2) on the interval - 10::; x::; 10.
Find, to tlve decimal places. points where the tangemline to the graph is ho1i.ZOntaJ.

M41"111.1 *'*

Plot

h) indi(.'\l :c

where 1hc 11me"C'n1

l i1le

ij horizontu.l

3
x 2 +4x-l
x 3 +2

y=

-5

- 10

10

SOLI"110N A graph is shown in Figure 3.30. Tile graph has a venical asymp101e a1 ,t
- 2113. Obviously. 1hcrc is a poim ncar x = I a1which 1hetangem linc is horizomal. In addi1ion.
because tile graph crosses 1hc .t -axis near x = - 5. andy = 0 is a horizomal asymp1o1e as
x -+ -00, 1here mus1 be at least one poimto 1he lef1 of x = - 5 at wh ich 1hc 1angem line is
horizontal. Whether there is more than one such point is not clear in Figure 3.30. The plot of
,

(x )

(.rl

+ 2)(2-r + 4) (x3

(.r 2

+ 4x -

-x 1 - Sx 3 + 3.r 2 + 4x + 8

1) (3.r 2 )

+ 2) 2

(.rl

+ 2)l

in Figure 3.3 1a crosses the x-axis near .r = I. thus confirming the poim in Figure 3.30 where
the tangcntliolC is horizontal. h docs not make clear, however, the number of points 10 the left of
x
-5 m which the gr.tph of .f'(x) crosses lhe x -axis. n 1e graph of f ' (x) in Figure 3.3 1b
clearly indicates one, and only one, poimto 1h-c lcfl of x = - 5 a1 \\hich j'(x) = 0 . To find
1hc 1wo point< 1hen where /'(x) 0. we sci

-x

&x 3

+ 3x 2 + 4x + & =

0.

Our conllluler gives 1wo (real) solu1ions of 1hi s cqualion, -8.3 16793 and 1.2386560. The
zero imermcdia1c value 1hcoreon of Scc1ion 1. 11 gLmramces 1ha1 -8.316 79 and 1.238 66 are

solutions. correct to five decimal pla(.'eS. when we c.akulate


J ' ( -8.316795) = -3. 1 x
/ ' (1.238655)

2.2

ro- 9

J0-6

.f'<-8.3 16785) = 1.5 x 10-s.

.f1 ( 1.238(i(i5) =

M::llclil, 7

- 1.9

10-s.

"5JJi'W

10
8
6
4

./ (.r) =

- x" - &l+Jx 2+4x +8


l
l
(X + 2)

0.004
0.002

3.4.

Pn:d-.::l:.ndQ\;(lfieuc: R&Jies

177

I EXAMPLE 3.17
Show that when /(.r) is u function. differentiable for all x. and lr(x- a ) is the Heaviside unit
step runction o r Section 2 .5~

- ( f(:c)h(x -a)]
dx
SOI.F TION

= f , (x)h (x

- a) when .<

a.

The procluct rule gives

.!!....lf(x)h(x - a }]

dx

= /'(:c)Jr(x -

a)+ f (x )h' (x -a).

Because h'(x- a ) = 0 at all x. exec pix =a where the derivative docs not exist, the requited
result now follows. If we wish a result that include ,I' =

a. we could use Example 3.l lto write

d
dxl.f(x )il (x- a)] = f ' (.<)il(x - a) + f (.t)8 (x - a ).

EXERCISES 3.4

In Exercises 1-16 find /'(x) in simplirted rorm.


1. j(x)

2. f(x)

J. f(x)

s.

= (.r ' + 2)(.r + 3)


= (2 - .r1)(.r1 + 4.r + 2)
X

= --

r'
-

= ,{X(x + I)
-

= 4.<'' '_ 5

6. /(.r) - 4.<' + I

ii 23.y =5-.r 2

II. f(.<) = X(.X + I)


I - 3.\

12. f(.r)

14.

f (X ) =

ii 2 1. y =.rl .

..rx

= J.r + 2
X+ 5
f (X) = 2x' - I

tl. f (x )
10.

t he ir pc)illl (0 1' p() i ll l ~) Of intC I~Ci i()(l.

.r 2)
ii 22. y=2x+2 . .v=x'f<x -1)

9. f(x) = - 3x + 4

13.

20. Find u rule for the dcri\'ttlivcoflhc 1>roduc1 of three functions .('(.r).
s(.r) . aud h(.r).

In Exercises 2t-23 lind the oqgle (Ot' angles) between the curves "'

xl

2.r- 1

2x 2

4. f(x)

3.< +2

j'(x)- -

7. j'(x)

1.::

.+. 25.

(.x

... 15. f(x) =

x1n
r.:
- vx

CC)II Uilt}(lily I)'CI' W!.:Ck

P(x )

+5.

= 3x -

16. f(x ) =

,(X+ 2x

P(x)
f'(X) = - - .
X

P101 a gr.Jtlh of th1s function.

(c) If u poim (x. P ) on tl>e tOtal profit curve is joined to the


origil. the slop! of Lllis line is lhc a' cragc pro61 p (.r) for
lh3t x. Usc this idc::l IIJ tint..l lhe sales level for h ighesl

(b) Fin<! [' (x) and show that it ~rccs witlt ~lC ptoL

-+ 19. Lf Ihe IOolal cosl of producing x itc..uu of a COilmlOdily is given by


tl>e <<juation
C (x) =ox (>
-+b)
- .
x +c

avcritgc proiiL

whete a, b. aod care constant~ , show LhaLthe narg.inal cost C' (x) is

c)]

2(X)
400

r.:

"1/ T -

c(b 't
(x +c)

hy

Plot a gmph of ths function.


<b) The .avera~c profh per kilogram when x ki!ogmms are sold
is Bi"cn b)'

+ l)l

a[ l +

i.S given

X ~

4
i 17. Find cqua1ions ror the tangent ru1d normal lines 10 the CUI'\'C y =
(x + 3)/(.r 4) 01 ~1e point (I. - ~ {3). Plouhe cu"oeand lines.
ij .,. 18.
(a) Plot a gra1>h of the function f(.r) = .r 1 f(x 1 + X - 2) .
I

x}/ (6 + .r) '"which the

(u) A m anufuciUrer's proli l from the sak: of .x kilogru.ms of a

.r 3 + 3x 2 + 3x +tO
f (x) =

= (5 -

lUJI_gcJU lilC l)llSbCS lhr'ough lhc origin.

I
X )_>2
.Lf + 2;t

)'=3.x /(x-t)

24. Find all poi n!S on the curve y

= x: + 2.< + 3
x - Sx+

y = 2/( 1 +

26.

(a)

(b)

= h(x + l)(.r1 + .<). Draw graph


Docs 1hc derivative exh>l ttl x = - Lif we define f (- I) =

Find f'(x) if [ (.r)


of f(x) and f'(x) .

0?

2 7. Re1>eat Exe.cise 26 if f(x) = lr(x

+ l )(x + 1) 2

13.5 Higher-Order Derivatives


When y = j( x) isafunctionof.r . itsderhativef'(x) is also afunctionofx . Wecantherefore
take the derivati\'e of the derivati,e to Qet what is caJled t_he second derh,at.ive of the function.
f (x) x 3 I/.r. then

This can be repeated over and over ag.tin. For instance. if y


,

dy
(x) = =
dx

= +

3x-- -.
xz

We denote the second derivative of y with respect to x by d 2 y / dx 2 or f" (x ):

d
2

2
d (d\')
- =6x + -

f"(x ) = - v2 = d .r

dx

X3

dx

Similarly.

fll

d 3y
6
= 6 - -4.
dx '
.r

(x) = -

This is called the third derhathe or the deriv.ative of order three.

Clearly~

we can continue the

diiYercnLiation process indefinitely to produce derivatives of any positive integer order wlmsoevcr.

I EXAMPLE 3.18
How numy derivatives does f(.r)

= x811 have at x = 0 ?

SOl tmON Since

f'(x) = -xm .

J"'(x ) =

G) G) G).-N

and /"'(0) is not defined. f(x ) has only a first and a second derivative at x = 0.

I EXAMPLE 3 . 19

.._..
Find a formula for the second deri,ativcof a product. cf2(uv)/dx 2 if u = f(x) and u = g(x).

soamoK Since
(/

- (uv)
tlx

du
dv
= v+ u-.
dx
d .r

then
tfl

--(uv)

dx2

= -d ( vdu- + udu)
dx

d.r

d ,r

tl1u
= v-

+ - - -1- u -

t12u
"' v -2

+ 2dx- d.r
- + 11 - 1.
tl.r

tl.r1

tlx

dvdu
tl.r tlx

du tlv

tl1 v
dx'

ducl v
d x d .r

+ --

tl1 v

Thjs formula is very handy. his worth memoriziog. See Exercise 14 for ex1.ensions to higher
order derivaLives of product..s .

...-..

EXERCISES 3.5
1n
I.

Exerci~:e!t

1- 10 find 1he cl cri vati~'C indic.atetl.

f"(x) if f (x) = x '

2. f"'(x) if /{,<)=

ll!:

IS.

+ Sx'

x' - 3x 1 + 2x +

.. 16. The Greens runcl ion t"or lhe deflection ol'a divingbo;lrd oflength
L as lt bcnd:s under its own wcighLand uny other loodio.g. is

3. J"(2) if f(x ); (x + l)(x l + 3x + 2)

4. /'"(I) if f(x) ; x ' - 3x 2 + 1/x

5. f"(x ) if /(xl

= (x + l)j.,fi

G (.r: Xl

6. J'"(t ) if / (1) = 13 - 1/ 13

f" (u) if / (u)

= xf( Ji + 1)

t 1. Stc-ildy-stall: lcnlpcl'"dtuJ'c T in a (c.gion bowtdcd by two concentric


spheres of radi.l a aod b (where a < b) 01ust S31isfy the cqu:Uit>n

2 dT

+ -r -dr

(a) Verify thcu for any cons1ants c and d , the funcli oo

T - /(r ) =

12. If titAn is genera1ed :.u the cemre of the ~ilhere i 1l Exercise 11 a1 a


const.::u n rate, the cqu111ion satisfied by T (r) is

d (r dT)
=kr
dr

where k

i~

1-

(a) Drow a grnph of G (x ; X) wben X = Lf 2. Is a part of i1

(b) Verify that che tlmml i~ s1raigh1 when x > X hw any X .

+-r

Ta , and T& .

is massless.

(c) Verify 111a1 G (x: X ), t!G f dx , and d 1 G f tlx1 are all continuOtJS for 0 ~ x :5 L , except for a rc.nlO\'i.lblc discontinuity a1 .t
X.

satisfies the cq u:.ttion.


(b) If tempe.r.nurcs on the two ~pheres are m.a:inltlincd at con:stanl vcdue~ Ta (.IJld Th. (;<llt:ulalc c an.. d i..nlcrnlS of a. b .

dr

2f /

slraiglu?

= 0.

where r is the rattiaJ diswnce from the common coo l.r~ of the spheres.

6f/

where X can t:tkc o n o.ny value between 0 and L. h is the dcfte<::tion at


position x i f the only force :.tCting ( lfl the di\ing hCI(InJ i ~ a tH)it foC'cC i o
the positive y-di.rcction ut posjtion X. h ulso ao;sumc:; thuLl.hc board

d 21j tfX 2 if I = X j(2x- 6)

d 2T
-,
dr

6f/

X.r'J
.
x3
(x - X)3/i(x - X) - - + -

I arc const:l.nlS depcndjng tln lhe matcrilll of lhc bo::ud :lnd i L~ crosssection. Thitlk uf G(x ; X) a:; a fu nc tiOtl of x that also del)Cnds on X

= jlij(u + 1)

10. f"(x) if f(x)

wh<::rc h (x - X) is tbc Hcavisiclc function or Section 2.5. and and

7. dl:lyf(/J:(J if y = .r '(l

* H.
* 9.

(d) Vcrifythatd 1G/ tl.r3 isconlinuou$forall.( '# X.andhas


a jump dlscontinuily at x = X of( I ) - 1
... 17. lllC Orccns fum;.tion lOr it bcanl or length
damped horizomally al x
0 and x
L is

L tbat has iLS t.:nili;

2
.l'.;

a oon~lant.

+ - - (-L3 + 3LX'- 2X 3)
6/ L'

(a) Ver ify 1hm

T(r )

kr'

+ -111_2
'- <X 3 - u.x' + l.' x).

-6 + ,.- +d

satisriel' the eqm.uion for ;lily constanls c and d.


(b) [f temperatures on the two spheres arc maintained at con
stant ' 'alucs Ta and Tb. ealculau: c and d in ttnu ~ of a . b.
1~,

Tb 1 andk .

13. Find cons1ants a, b. c. and d in order th.nt t~ func tioo y =


/(.f) = ax 3 + bx 1 + ex + d has its first dcri"at.hc equal to 4 when
x = I and y 0. and itS second derivative equal to 5 when x 2
and)' = 4,

14.

(a) Find o fomJUio for d 3(uv) jdx 3 if u = f (x) and v =


g(x).
(b) On the basis or 1)a11 (a) and Examt)le 3. 19, could you ha.z.
atd a guess at. a fommla for d~(uu)fdx" ( i.e., do you see
the JXUtel'fl e.merg.ing)7 Exercise I 8 asks you to verify tll e
COI'I'ect fot mula.

(See Exerci~ 16 for a de..~criplion of E and f and lhe inlerpretation of


G(x; X) .)

Ia) Draw o graph of G(x; X) when X


L/2. Is itS 1"ngen1
lillC ho.ritootal :II :c = 0 3IId x = L '? Is i.t syou:oct.ri.c
ubout x - L / 2? Docs it ha\'C a removable discontinuity
al X

= L/2?

(b) Verify lhot G (x; X) . dG f dx , and d 2 G j dx 2 arc all conlinuC)uS for 0 :5 X ~ L. except for a rcn10vablc disconti
nui1y tiL .\'
X.

(c) Vcrifythatd 3G/dx 3 iscontinuous foraJix ::/; X ,andhas


a j ump dist."Ontjnuity at X= X of ( E n -l.

180

Cb lp~er 3

Oiffe.renti:ujoo

~ 18. II' 11 is a positil'c in1cgcr. and

11

and v urc functions of x. shi>W by

mathematical induction that

where d0 ufd.x 0

= u, und

G)

ure 1hc binon>iul cocllicicnts

G) =

n!

r! (11 - r)!

d"

- ( u v) -

dx"

" (")r (d'-d.ru)' (d"-'v)


-~
dx- r

(Ir you arc II()[ l'amilinr with sigma ll<llation, sec Section 6. 1. Muthc111Uiical inducrion is diS<:usscd in Appendix A .)

13.6 Velocity, Speed, and Acceleration


Most applications of differentiation are dealt with in Chapter 4, but velocity, speed, and acceleration are so imporh\nt in engineering aJJd the physical sciences that it is impc>rtanr to discuss
them as soon as possible. In Section 3.1, we used the position function
x(t) = t 3

Tit1

+ 1681 + 20,

t ~ 0,

to define velocity as the instantaneous nue of change of displacement with respect to time. (Unless we indicate olhcrwise, velucity means htShlrHancous vclocily, rather 1han average velocily.)
\Ve c an now suy that velocity is the derivative of displacement,

dx
u(f) = - = 31 2
dr

541

+ 16& rn/s.

In terms of the g raph of the displaccmeot function (Figure 3.32), velocity is the slope of the
tangent line. For instance, v(3) = 33 m/s. The particle is movi ng to the right at 33 m/s; the
slope of the tangent line at (3, 308) is 33. (See the triangle on the tangent line and remember
that the axes have d ifferent scales .) After 5 s, v(5) = -Tl nlfs. The particle is moving to
the left at 27 rn/s: the slope of the tangem line is - 27 at (5. 310). By factoring v(t) in the
form
V(f ) = 3(1 - 4 )(1 - 14),

we sec that vclocit)' is zero at 1 = 4 and t = 14. Tltis is consistclll with horizontal tallgcm
lines at (4, 324) and 14, - 176).

llt!lffil*..::Jfi11
500
400

Oic;placemeu ftlnelion l llu~ l rates the \'elociry of objea

300
200
tOO

-tOO

66

The particle is moving to the right whenever its velocity is posithe. to the left when ics
velocity is negative. Graphically, it moves w the right when the slope of the tangem I ine is
positi,e (0 :;: t < 4 and t > 14), and to the left when the slope of the tungent li nc is negative
(4 < I < 14).

Spt'Cd is de tined ro he the magnitude of velociry.

speed = lv(t)l.

(3. 17)

v
4fiil'

(t) for' a patt:id c ru.ovj'k" nlons


tO answer the following

.
3 sht>WS lhc.: S'1'J'h vf the di:<~pla:ccmct'lltO(u,lut:~~:~ . . n~ph
0
F1$UfC' -'" '

1 0 ..::. I <.

.,.....

the ,\' ..ct:Cb,

d !lfirl"' (ltC. (llli C Ul[CI'\'S

.,:.

&

q~cions:
.
he lcfi Of right at r
7 s'!
(a ) ~ che par1idc mowng 1<> t
. .,
~

, dQeS tllC JUII'ticlc :\h.)p lllO'o' lll.\0

(b) How nnmY miles _ Q j $


(C) , .._ d1C \'cJ ()(:IIY i:\1 I -

tf(Ca i CI OV )PUt

..,

110, thun uc t = 3.5 s?


- 3 5 s?

~
. tl . 1 = O.S s " I'!.Ul CI' vr s nudler th.m 0\l l
(d) r~ the spee .at
I co\ Qfthe patticle O\'Cr the inlt:fval.
(c) E~inl~ltc the uvcmgc vc oc Y
. . .-. .....ace~c ._111d ,omallest'?
sav dut the velocity
. 11:1 !">'- .1
......,,.,
(f) At wh~l t ime:; n<Otrld you
th.
s
~d
l1i greatest attu sn\8 11..,.,.,.
1
1
(g) At what tifnc:\ wouh.l you .O:t'IY uu c I

200 -'
ISO

wo

- 50

-100

sournoN
(U) Since ahe- ;sh)fX' of dw fllllgcut li11c m t
the jX'r"ficle is m oving

10

= 7 .s is nc1;ativc. the velocity is ncg.at hc. and

the left.

(b) Thcp.utielc stops moving when its ' 'clocity is uro.und chis occurs when the tangent
line cv the g.f~lph is hol'iwnwl. TI1is h<..'lppens three tirut!.S.
(c) At t 0.5 s. tile vclociLy is negative~ at t
3.5 s. it i~ J>OSitivc. Thm;. it is smo11cr
a1t = 0.5 >.
(d) The wngcnl line a1 I
0.5 s is mucll Sleeper than it ism I
3.5 s. Since ~peed
measures stc'Cpness ( love without rcg;ord for sign). speed i> grctcr ol 0.5 s.
(c) AVerage velocity is the dillcrencc between inilial and final displacements divided by
the length <li' the time iuterval. If we <Stimate .r (0) : 200 and X( 10) = 120.

.
120- 200
average velc>elY ~
= -8 nlls.
10

(f) Since the slope of the graph appeas to be greatest at t = I 0 s. the velocity must be
greatest then. The slope and velocity appear to be smallest at t = 0 s.
(g) Be<.-ause speed is the magnitude of velocity, it is never negative. Speed is zero if
velocity is zero. Since this happens thrl!e times. speed is smallest (and has val ue 0) at
the 1hree times when 1he 1angem ljne is horizomal. Speed is greatest when steepness

o f the graph is greatest. This is ei1her at 1 = 0 s or t = 10 s. It is dif(icult to tell


from the graph, but we favour a Sleeper 1angent line at t = 0 s.

182

Ch.'lp!tt' 3

Oil'fel'tHi:uion

I EXAMPLE 3.21
You are now cold thal lhe displacernent function for lhe gmph in Fi~ure 3.33 is
,

x(t) = t - 20t 3

12~3r

5211 2

399

+ - 4 - - -~- + - 2 .

Answer the following que-'.!lions:

(a) What nrc rho speed a nd velocity all = I s?


(b) Is rhc velocil)' equal IC> 7.ero ar 1 = 2 s'
(c ) What is rhe average velocity of ~1c p<>nicle for

0 :S 1 :S 10?

(d) Ar wharrimc is spetd grearcs r?


(e) To rhrcc decimal places, at what rimes is the speed of rhe panicle equal to 20 m/s?
(irapll o( \'dOCily

,.

I'

200

S2 1t
2

1243
4

\~= 4t ) - 60t + - - - -

100
10

10

- 100
- 200
- 300

-JOO
SOI.lJfiON

The velocity of rhe particle i<


v(t)

tlx
= 41 3 - 601 2
dt

= -

5211

1243

+ -- -m/s.
2
4

The above questions will he answered algebraically, but ir is helpful to visualize responses by
plouing the velocity and speed functions. \Ve have done this in Figure 3.34. Figure 3.34b
refiC<:rs thlll part or the graph in Figure 3.34a below rhc t -axis, in rhe /axis.
(a) Art

= I s. velocity is
v(l) = 4(1) 3 - 60(1 ) 2

The speed is 425/4

521

1243

+ -2 - - 4

425

= - -

mls.

m/~ .

(b) GeometricalI). the velocity is zero a t t = 2 s if the tangent line in Figure 3.33
is horizontal al I = 2 . h looks clo<e. Equi,,.lently, does the graph in Figure
3.34a cross rhc t -axis ar 1 = 2? Again, rhc decision is not c lear. Since v(2)
= 4(2) 1 - 60(2) 2 + (52 1/2)(2) - 1243/4 = 2.25 nl/s, the velocit> is not zero ill

I = 2 s.
(c) Since d isplacemcllts at/

= Us and 1 =

average velocity =

lU s are 399/2 m an<l 117 rn.

117 - 399/2

10

= - 8.25 m/s.

(d) Figure 3.34b iodicates rhar speed is gre atest at / = 0 s or 1 = 10 s. favou ring t = 0 s.
Bec-usc u(O) = -1243/4rnlsand u( IO) = 1117/4 m/s, ir follows !hat speed is
greatest at t = 0 s !see also parr (g) in Example 3.20].

3.6 Vek....:-icy, Speed, nrt~.l AccCicr.llklll

183

(e ) SohHi0 1S can be visualiz('d as times whe n lhc height of the. graph ill Figure 3.34b
is 20. C learly. there arc s.ix such times. Alternatively, solutions are 1imes when lhe
ordinate in Figure 3.34a is 20. the same six times. Solutions or
3

20 = v(l) = 4t - 60t

5211

+-

1243

- -

are 2.306 \15, 4.24043, and 8.45262 . The zero intcrmediarc value theorem cnn be
used 10 confirm three-decimal-place solutions 2.307 s. 4.240 s, and 8.453 s. Threedecimal-place :;olutions of v(l )
-20 are 1.7 18 s, 5.300 s, and 7.982 s.

The velocity o f a particle moving a long lhe x - axis is the derivat ive o f iL'" dlS1Jia<.cmcm func tion
x (l) with respect to/,

dx

= -.

v(l)

dt

(.1. 1M)

The (Instantaneous) ltcctle r atl!on of the parriclc is defined 11s the nile of change of velocity
with respect to cirnc:

(1,(/ )

dv

= -dt

(3. 1'.1)

In actua l fact, x(l), u(l) , and t1 (I) are tho .\:-component> of the displac~ment. vducity. ami
aeoclcration vectors. reSJlCctivcly. In the absence of a complete d iscussion of vectors. we omit
the terms "ecrvr and cvmpcmenr. and s imply Cllll ;r (1), u(l), and tl(l) d isphlccment. ve locity,
;tnd accelerntion. \Vhen distance is measured in metres and time in seconds. ~eiO<.' ily is me.nsured
in metres per second (ntis). Since acceleration is he time derivative of velocity. its urtits 1'11USt be
units of velocity di,ided by units o f time [i.e .. metres per second per second (m/sls)]. Usually.
we shorten this by saying ''metres p.:r second squared'' and write m/~2 .
For example. in Figure 3.2 the accclcrruion of the parLiclc with displacemcm function
- 27t t + 168r + 20 is

x(t) = r 3

a (1)

2= !!._(31
dt

541

168) = 61 - 54m/s 2.

The grnphical imerprer:uion ofacceler:uion requ ires the concepLof"concavity."


in Section ~.4 .

11li~ isdi~cussed

An indusuialist is having a problem wilh the design and manufacture of a cam. A cam
is a machine part that rolates about an axis to cause periodic movement in another parl~
caiJed a rouower. The plate cam in Figure 3.35a rotates about an ax.is through the origin
0 and perpendicular to the plate. The follower moves up and down along Lhe y-axis as
po inl A on its end remains in contact with the cam. Suppose Figure 3.35b represents the

displacement y = f(IJ) of the followeo. from its lowest position (1 em above 0 ). as a


function of angle through which the cam has rotated.

MAl Ill.

Follower
II

~Cam

)'

""

J/4

:Z;r - U1

"

217

(}

The follower rises 2 em during a rotation of 2rr/3 radians. Because velocity of


the follower is proportional to the slope of the graph, velocity inercllSCs in the intc rvnl
0 < 0 < 01 , when the follower rises 3/4 em. and decreases in the interval O, < 0 <
2TC/3 when the follower rises the final 5/4 em. The follower is <tationary in the inrervll
2TC /3 ~ (} ~ ;r. It then retrace-s its path bnck to zero displacement ubovc its min imum
po>ition in a >imilar fa>hion. All curves arc parabolas: the first has a minimum ut (0. 0)
;md the <:econd ha< I m~ximum at (2rr/3, 2). The indu'lrialiM ha< "'" pmhlems for
us. One is to ensure that the fOllower mo\'CS smoo1hly; chat is. its motion is nOt 'Jerky.''
Secondly. he W<lnlS U>IO gh-e him a scale diagram of the cam itself. He hus not ;pecifiw
angle 8 1
SOl l no" We begin be using our knowledge of equations for parabolas. Equations
of the fiN ti\O parabola< mu<r be of the form

o,.

a(J2.

/(O) = { A(8 - 2rr / 3) 2


11> >hown here where a

't'

0 ~ 0 <
2, 8, 58~ 2rr/3,

and A arc constants. For (Ot . 0.75) tobeapoiotoo both parabo)..,..

0.75 =

a8r

0.75 = A(81

'l.o/3)1

+ 2.

Now comes the most difficult consideration of the project. interpretation of the follower
moving smoothly. This means that there can be no sudden changes in its \Ciocity. Since
velucit) i~ related to 'lope of the curve. we ask where ,udcJen chanC> in 'llliiC CIOUIU o~cur.
Slope changes gradually along parabolas. and there is no difficulty tn the t.nlnsiuon from
par.bula to horioomal u-..i&ht line at (0. 0) and (2rr /3. 2) . TI1e only quc>tionablc puim
is where the above two p;~rabolas join. We must ensure Ihat slopes of the two parabolas
mmch a1 the JlOilll (0,. 0. 75). Otherwise. there will be jump in 1he veloci1y there For
lhc left-hand derivative of all2 and U>e right-hand derivative of A (8 - 211' /3) 1 2 to be
equal a1 (111 0.75). we must have

2all,

2A (1! 1

2;r /3).

We now have tltrce cquations in three unknowns. a. A . and 8 1 Lf we solve !he first two
CCJumiQns fol' o a-.d A in 1emt~ of 81. and sub, tinuc imo 1he 1a-q equation,

0.75)
[ 0.75- 2 ]
2 ( 8~ 8, = 2 (ll, - 2Jr /3)' (ll, - 2rr /3) .
'lnesolutionoflhis cquationis 9 1 = Jr/4. Tllisgi,esa = 12/JT2 und A = -36/(~;rl).
The industrialist thm:fore has no choice for ang.le 81. Smooth motiOf'l of the follower
tee1uires 81 = rr / 4. TI'Ie e<1muion of the tli:splacemem curve for the follower ttbove il.!!.
minimum position is therefore

o::::e::::"/4
rr/ 4 :::: 8 :::: 21t/3
2rr/3 < 9 !S Jr .
Now consider the shape o f the cam itself. When we add unity to I (8), we obta in the
distance or the. end A of the follower ab<we 0 . This represents the distance from the
cen1re of 1he cam to ils outer edge :lS a function of angle through which it has rotated,
call it ~?(8) = I + I (II) (Figure 3.36a). To obmin a scale diagram of the cam, we need
rhc equation of this cur\'e. If (,t . )') is a point on the curve. then trigonometry i1ldicatcs
!hat x = g(9)cos8 ond y = g(9) sin9. Unfortunmcly. it is not possible for us to
eli rninate 8 hcrween these equations ond find on equation in .t and y only. On the other
hand. electronic devices have prosmniS to plOt curves given in this form, und we will deal
wilh them ot length in Chapter 9 3S p:tromctricolly defined curves. The resulting plot ond
rcquired cam shape is shown in Figure 3.36b.
S~tk

ror ~h:lpe or

C>m

-2

)'

-l

-2

EXERCISES 3.6

1. n)C figure at right shows the gruch orthe displaccmc;nt fufK.>,ion


.t(t) of :1 parliclc moving along the .tax i ~ eluting the time iniCr\'al

0:5

:56. Ans:\''01' lh followingquc!aions..


(a) ls the particle to the left or righl of the origin allimcs t

(b) Is lhc paJ1ide moving lO the rigb1 or tO 1he left !II limes

andt =4?
1

= l/2and l =3?

(c) How many Limes does Lhe pa11icle change ditC<.'tjoo?

(d) Is d1e velocity greater a1 I = 7/2 or at 1 = 9/2?

(c) Is the speed gr~ncr a1 1 = 7/2 or at 1 = 9/2'?

~here .x i.s mcasUI'\:d in n1e1ms ;md 1 ~ 0 i5 lime in tiC(lOOds.


Octenninc algebraically (a) pcll'ilion \Ciocity. 'PCd. and acceleration
at 1 = 3 <; (b) when th< m,ect ic on<tanconenu<ly at re<t; (c) when
"""'k.T11tion va.ishes: (d) tinlCS when the l)hj(:CI o< lllO"niiO the n&hl
and len: (e) if and when \Ciocuy ,. I mi.: (f) of ond when >t)CO(! is
1 nils. (g) if and when vclocit)' i> 20 nil: an~ Ill) if ll!ld when sp.."Cd i>
20 nvs.

2. You Ill\! gi\"(n that th: displaocmcnt function in Ex.crcisc I i~

.r<t>

14

251 2

321'

6 - IS + - 3-

2511

30 .

Confirm eileh :.n~oowcr '" Exerci,;c I aJgebrJically.


3. Repeal Excrd>C I ror ~te graph below.

18. Repeat Exercise 17 if


x(1) "' 1 3

9t 2 I l SI - 2.

19. Can the position cuoc .t = A (I) or u rcall'iic parliclc moving


along lhc x~ax i.s be rcprc.!lcntcd by :.1 l'unclion X(t ) that h4h a discontinuity! Explain.

+ 20. When o..n objccl 1Jl0\'CS wilh coMtanl *=lcrucion a u!ons the x O.'<is, its position ;ss 3 funct ion or l ime I mu$1 be of lhc (orm

- 2.

where

4. You o.re si\'C'R that the clispl3CC-mcm function in Excn:i.sc 3 is


.r(l )

14t'
JOlt'
- --

4~

4~

1321

+-

+ 2.

4~

Confirm e.aeh un~wcr in fxcrets.e 3 algebr.Ucally.


~ the di~pluccm.cn l f\mction for a panicle nicwing :long
the .r.nxis, the third dcrivotive .r '"(t) iselllledjtrk. Why " thi<notno

5. When .t'(t)

approprtolc1 Find Jcrlc

rot the displacelllcnl function in E:<erci~ 2.

6. Find jerk for the displacement furxtion in E.'<crci.sc 4. CScc E'<crcse


S (or tht <kllnition of -"=d..)
In E<et<:it<s 7 I I find the ' '<locity and >e<:dc:rallOO of an cbjt !hot
ano\'C-S Ill on& the .-t ~nxi 1 Vrrilh the gl\'CO position fuoction. Assume that
A i) in nM;.Uu atwlt i-' in lCC:onds. Dmw g.rapbsof x(r) . v ( l ). Wtd a(1) .
otnd cAarninc 1he ~:r.I,J)h~ from lhc (X)inl of \'iC\\ th211 (lfdinatc' on r1(1)
reptt<enl <l(lj!C< on r(t). and mhnate< on o(t) are lope< on l'(l) .
l>r'Jw n g.ri.iph of !Opec."'d ~ a runchon of t also.

7. X(l)

9.

= 21 4

S,

X(l ) "' 11 + 51

+ 10.

= 41 -

11.

II . ,t(l) = I f / ,

I(I. X(l)

x(l) =

S.

11 - 71

+ 6.

1~ U

= .r(1) = 2I or' -

hi ... r.

band c arc constanu..


(a) Jfthe objec1 is at f)O'i tion' .r1 nnd .r : :u ltmec t 1tnd t:. what
is its avetage veloctty o\~r che 1ime ntervll t 1 .:$" 1 .:$" t: 1
(b) At what tim~; i1l this t.iule i11Ua "~ I i ~ the ini)lauUtncou:; ''C
lucity t:qual to the uvcrugc vc.:IO\:ity? Where i$ the object HI
thb Lime? Is il at the u1idpoim of lhc intc"~ul bclwcc.n Xt
and x 2 is it closer h> .x 1 ur is il clo;..cr to .l,? tA.s.~untc in
thi!'> pan or the: prohlem lhl1l '-1 > 0 and th:u lhc velocity or
the obJOCI i~ ~1 1ivc ~r time t 1 .,

.. 2 1. Find /(8} in the consultmg prOJffi 01 1h1c <t'CIIOR when rotarion


27</ 3 is replaced by IJ,. and diplllt<n><nl< 3/4 and 2 arc rcplac:oo by
y, llt1d y2 respecti..cly. In lhc: process. how thlll y,jy, c 6,/ 9 2, ond
11ut this implies lh>t the point (01 .11) i> oo the line joinittg (0. 0) ond

co,..Il l

22. The: figutt bclm. rCJ!rcscnb a poniun of !he di>piOCCniC!ll f (0) of


a foJiowe.r in a cam mechani~m. ll crwht'l' o.f 1 '-frniehc line ponion
bcrween A and 8 and two parJbohc pon1onc OA and BC thac are
hori.zontal 0.1 0 and C. Gt\'Cn ure the ti;cs y1, )';:, o.nd y,,. and Bntlc
03 . It is required thcu the slope or the cur. c be cootinuoult o.t 0 1 ur.d 01
By taking /(0) in dtc rorm

y(O) = { ::~;+b.

e:

I ~0

A(O - O,)'

e: I

0 .S 0 .S 111

+ .n.

o, < 0 < o,
o,.s o .s o,

show lhat
Rc:~.allhc c.atculuciOC\$ for~ abiDe fi,-e e.'<ttcisc:s for E>.crcisc.s 12 16
We plot lfiii.l"~"'~ ir1 ~.u.l ur lit.twin: thcrn.

i
i
i
i
i

11. A( l )

=- 21' + lt

tJ. .<(1)- l l

+ 161 -

91 1 + 151 + 3.

14. X(l) = I +4/ 1.

I.

e: 0

c
R

I ::: I

17. An object mo\'ing along the x -axis has positjon given by


X(l)

= 1'

0, -

y , - y, + 1))

I!:: 2

I ::: 2

16. X(l ) = (I - 1) 1 J/,

and find o. m. b.lltld A .

I::: I

15. X(l ) = (I - 4)/1 1,

o,

- 911

+ 241 +

I,

(y + y,)O,
,\' - .'': + 2y,

13.7 The Chain Rule and the Extended Power Rule


\Vi1h che d ifferentiation n iles in Sectio ns 3.2 and 3.4, we can differentiate any polynt.M'I"tia.l
3 - 3.rl + 2.r + I. then / ' (x )
whatsoel'cr. f'or instt~ncc. if / (.r)
3xl- 6x + 2. But
2
consider the f>olynornial / (x ) = (2x - 3)s. w hich ha.-; been conveniently factored for us. To
find its dcrivHCi,e, we could expand (2t1 - 3)8 by lhe binomial theorem. ~my. then differentiate,
nnd tinally. si mpl ify. We <.'Quid nlso consickr using the product nate o,cr and over and over
ag~lin. Thus. in spice of rhe fact that the rules of Sections 3.2 and 3.4 permit d itTc:rcntiation of
(2t 2 - 3) 8 , they arc not ~.:on,enient to use. Even more unpleasant would be differentiation
o f the nuional function g (x ) = l /(3x 2 + 8) 12 In this section. we obta in results thlll enable
u:o; to diff'crcnciutc quick ly much wider classes of functions. which include (2.r 2 - 3) 8 und
l/(3x 1 + 8) 12
Suppose thm y is defined as a func tion of" by

= ,{

y = .{(11)

"

u~
. ~

nud u. in turn, is defined us a function of .t by

u = g(x) =

.x +2
These equations imply that y is a function of x: indeed, y is t he composition of J and g,

...fi

.)' = f (g<x)) =

x +2

-..fi
-+1
x+ 2

Aficr some a lgebraic simpl ificalio11 we fhld Ihal

y-

Ji-+x + 2'

wk.l we can therefore calculate

(../X + X +
=

2)(-I
) - .jX (-I
+ 1)
2,/X
2$
(J\- +X + 2)2

This can be reduced to

dy

d,t
BlllrlOtice lhln i f we d i fferentiate 1he orig i nal functions, we obtain

dy

du

du
- =
tlx

(u + 1)( 1) - u( I }
(u
(X+

1) 1

(tt + l)l

2)c~) _ $(1)
=

(x + 2)2

and

2- x

2.fi(x

2) 2

The product of these derivativcg is

tly tlu
-- =
du d.r

2- x

.,...--;::-:--.::........,7-----~

2../X(x

+ 2)l(u + I ) 2
2- x
2

2.fi(x + 2}1 ( .fi + 1)


x+2

2- x

188

Ch:.p1cr 3

Diffcn:nti:lUOn

Conse.que11Ciy, for this example, we can write

dy

dy du

According to the following theorem, the derivative of a compo.~ite function y

f (8 (x)) can

always be calculated by the above formula.

TH EOREM 3 . 9

(Chain Ru le )

If J = f(u) and u = g(x) are differemial:>le functions, then the deri,ative of 1he
composite function y
f(g(x)) is

dy

d y dtl

dx

(\.20al

du d .t

Increment no~>nio n is particularl y useful in proving the chai n rule.

PROOF

B y equatiOl3. 11 , the derivative of the composite functi on is

dy

.f(g(x

.
1\y
.
l1m = hm
~- 1\x
o.,-o

dx

+ t. x)) - f(g(x ))

t.x

Now a change t.x in x produces a change g(x + .C..x) - g (x ) in u . If we denote this change
by t.u , then it in lum produces the change .{(11 + t.u)- j (u ) i n y. We may write

dy
. f(rt
- = hm
dx
~-o

+ 6 11) -

j (u )

t.x

Si nce /(u) is differentiable. its derivative C11: i:,ts and is defined by

. f
' (u) = hm

(u

+ l!.u ) -

~u-o

f(u )

t).u

A n cquivalem way to express the fact that this limit is J'(u) is 10 say th<ll

:__
t!.,-:
u')------"f _,_
( '..:..
')
- u_+
= .f,(u)
l!.u
::..f.:_
<

where~

must satisfy the condition lim&11 -o t: = 0. We may write

f (u
a 1\d

+ ~.

+ l!.u)- .f(u)

= [f'(u )

+ <') l!.u,

if we substitute this into the second expre.ssion for dyf dx above~ we obtai n

dy
- =
dx

[f' (u)

lim

+ <') l!.u
J.:\:r

A X-tQ

But

l!. u

g(,,

lim

~l"-0

+ l!..r)

[f (u)

t:.u }.
+ <']6,,r

du
= - .
~-o
l!.x
dx
Funhennore. since g(x) is differentiable. it is continuous (Theorem 3.6). and this implies that
D..u ~ 0 as lJ.x ~ 0. Consequently. lirn:.\t"...,..O = limA u-to = 0. and these results give
hm -

t:.x

hm
~-O

dy

du

- g(.r )

d y du

d~ = f (u}dx = du dx

This re~ult is called the chain rule for the tlcri\"llti\e of a composite fuoc'tion. It expresses
the deri\'lltive of a composite function as the produce of the derivath'CS of the functions in the
composition. From the point of view of rates of change. the chain mle seems quite reasonable.
Tt state< that if n \'Uriublc is defined in terms of a <econd '~ri,tble, which i< in tum defined in
tem1s of a third variable. then the rdte of change of the first variable with respeCI to the third is
the rate of change of the first with respect to the second multiplied by the rate of c hange of the
second with re>pect to 01e third. For example. if car A navels twit-.: as fast as car B , and car B
tra\ cis three times us fust 11s car C, the n car A mwcls s ix time us fus t u,o, cur C.

It is c~ntinlto understand the d iffere nce between the clcrivnti vcs d yftlx nnd dy/dtt in
c<Juation 3.20;1. The second, dyfdu , is thederiv,llive of y rcgartlccl a< a function of u, the given
fu nction )' = f (11 ). On the other hand, d y / dx is the derh1ltive of the compoite function

/(g(.t ))
The chain rule can also be expressed in terms of the circle nocmion for composite functions.
With /(g(.l}) tlcnoted by(/ o g)(x), equation3.20a ldl.c. the fom1

(/ o g)'(x) = J'(g(x)) g'(t).


or

(/ o g)'(x) = (f' o g)(x) g'(x).

I EXAMPLE 3.22

..........
at/= 4 when y = xl - .rand x = ..j/f(t-'- 1) .

Find dyfdt

SOLLIION

By the chain rule


dy = dy dx = (2x
dt
dx dt

When 1

= 4, we nnd x =

_I)[('- l)(l/2)t-tfl- ./i( l)].


(t

+ J) l

. -dy'
..f.i/(4 , I) = 2/5. We use the notato()rl
dJ

dy/ dt e\aluatetl at I = 4.

tly'
-

d t ,...

.. (2(2/S)- I]

[(4 +

1)( 1/2) (4) - lfl(4 + 1)1

to represent
14

J4] = -

3
.
500

The Extended Power Ru le


When the function f(u) in Theorem 3.9 is a power functron, ohe chain mle gi\-es what we
call the extend ed power rule, ofocn considered the most important diffcrcmiati()rl formula of
calculu

COROLLARY 3 .9 . 1

When u

= fi(X) is differenoiable.
-

dx

tt

JJU

11 _ 1 du

dx

(3.2 1)

190

Ch<lpter 3

OifferecuialiOrl

11 is essentially power rule 3.7 in Sec1ion 3.2 wi1h an exira factor du/dx 10 account for the
fact that what is under the power (u) is not j ust x ; it is a function o f x. In the specia l case that u
is equal to x, equation 3.21 reduces to 3.7. Power rule 3.7 was veri lied only for ' ' a nonnega tive

integer. but \ve have-been using it for any real number n. \Veshall use-its gene.ndization 3.21 for
a ny real number n also in s pite of the fact th ai. in effec4 it has only been verified for nonnegat ive
integers. In Section 3.1 J we provide the justification.

11 is important not to confuse rules 3.7 and 3.21. Rule 3.7 can be used only for x"; if
anything other than x is raised to a power, formula 3.21 should be used. The most common
error in using equa1ion 3.21 is 10 forget the du/dx . Wilh equation 3.21 i1is a simple mauer 10
d iffere ntiate the fu nctions in the Hrst paragr-iiph o f th is section:

.!!_ (2x 2
dx

3) 8 = 8(2x 2

3)7 .!!_(2x 2
dx

3) = 8(2x 2

3) 7 (4x) = 32x(2x 2

3) 7

and

- 12

-72x

= (3x 2 + 8) 1J( 6x) = (3x'+8) 13 .


Notice that in oe ither of these examples do you see the leuer u. ahhough rule 3.21 is s1a1.ed io
tenns of u s. We could have introduced u, in the fi rst example, say, by selling u = 2x 2 - 3,

and proceeded as follows: With" = 2x 2

3,

d
2
8
d 8
d 8 du.
7
- (2x - 3) = - u = - (u ) - = 811 (4x) = 32xu.' = 32x(2x 2

dx

dx

du

dx

3)

But this is unnecessary; with a n understanding of equation 3.21, we should proceed d ireclly
to the-del'ivatives without de-fining an inte-nnediate variable. With a little pnlctice. the writing
should be shonened even more. For example, calculation of1he derivative or (2x 2 - 3)8 should
appear as

bring down rhe power


d
d\

(2,:1 - 3)8

J lower power by one

=8(2r2- J )1(4x) =32t(2,:l- 3)7


~

same expression under power

_j

derivative. of expression under power

I EXAMPLE 3.23
A balloon always remains spherical when i1 is being filled. If the radius or the balloon is
increasing at a rate of2 mm/s when the-ra.dius is 10 em. how fast is the-volume changing at this
instant'!
SOLUTION Because the ra1e or change or the radius r of !he balloon is 2 mm/s, we can say
chat the deriva1ive

dr
dt

0.2cm/s,

when r = 10 em. Since the volume of the balloon is V = (4/J)rr rJ, and r is a function of
time t , we may differentiate-this equation with respect to t.

d\1

df

3JT

d r)
2dr
3' 2 dt
= 41rr dt '

= 10,

Whenr

-dV
= 4rr {I GI) 2(0.2) = SOn.
dt
Thus, the volume is increasing at 80;r crn' ls.

E XE R C I SES 3.7

In Ex<m:i ,;c::; 1-8 u~ !he ch ~in rule In find


I . I' = t 2
..

+ -.
I
II

2. )' = - -.

II + I

J. y

(u

'- 2

5, y = ( v 2
6.

\1

+I

21!.

8. .l' = 111 (l

9. j(x ) = x(.r 1 + J)4


x 2(2x + 1)1

= (x + 2) 2(x 2 + 3)

3X + 5

(2x- I )"

= f(x).

12. / (X)=

' IJ . j(x )

II

= --,

+5

l4

: X;.;:>:)

~
= -'--J.'

= 11 4 (113 - 2u) 2
)' = I + j, + ,fi,

t = --

(~) '

35. y=

x' + I

x2 -

v = --:---:-xl

37. If .1'

=j(11) , 11 =g(.<), ond < ~ IJ (x).show '"" '

x'

d~ = du tis dr'

= S x 10- 6 Cit; Jt a di.$1.;mcc r


meti'CS ft'Oill a s.atiOOUI')' ChO.I'gC Q = 3 X IQ- et C. the masnitudC o r
the force or rcpuJsion or Q on q is

.._ 38. When un

= (2- Sx' )'l>


j(x ) = (x + J/ (Jx + 1)1

cl~c l.rt\Stmic churgc (f

f (x)

19. J (.r)

x'' J

r.

l - .;X

21. /(.r) =G:.~\ Y


23. j(.r)

22.

= <x ' - 2x1)1(x- 2x)1

24. j(.r)=(x + S)'J L+ x'

25. f< x)

26.

F=
20. J(x )=

x JI - x'

= (3 + x)'''
f< x) = x (x + 5) )1 +x'
4

f(x)

~
x'

f -.r

\ 2 + .(

dy ~!!!,

d \'

+5

16. j (x) = .r1(2- 5x'J'"

+ 3 .\'2 + 2

.v = "'I-+-t,...-+""t"'' .

(2x - 1) 2

14. j(x)"' 3x

.If

J6.

v2x+l

J.r - 2.<'

* 34.

1 - "'

.tJA' + l

II

.r, + 5

II =

10. / (X)=

11 . j (x)

s = --

JJ, y

:c + l

11=-).'- x2

In Exercises 9- 36 find d yfdx . where y

_
f( X ) -

v'
=-,
vl - I
11

= (3.< + 2)(.<2 + +x)

+ Jii).

I'

32. )'

x-2
1= - x+ l

''= l -4.

7. ,.

17.

* 3 1.

+ v)(Jii+ 1). v =~- 1---1

= --,

f<x)=xJI+.t~

<- 30 .1' = (2s - s 2 ) 1 ~'.

,;; - 4

1+3
I - 4

u =---

s=.c- -2.t +3

,, + 3

15 ,

27. /(.r)

II = ,/X+ I

+ 1)(11 + 1) .
,

4. r = - - ,
.

I = .t 1

x (x' + 3)
= ~~:-'-'-~
(r - 2)(x + 5) 2
2

dy/d~.

Qq

- - new tons,
4neor 2

= 8.85 ')( 10- 12 If q is 1110\'Cd dirccUy :lWO.I)'


2 mls. how fast is F changing when r = 2 nrr

where

E'l)

rron1

Q at

39. When a mass m of 5 kg is r ll\CifCS fron\ the ocmrc of 1hc C8rth,


the m:agniuJdc ()(the force of anraction or the: earth on m is

GmM

F -2

ncl'11)1lJ.

where M is the mass of the emt h in kilograms and G = 6. 67 x I o- 1


lf m is [alJing a t I 00 km/h when it is 5 km above the surfat-e of lhc
cm1h. how fast is F changing? The mean dcns ity ofthccarth is 5.52 X
103 kglm 3 and its mean radius is 6370 km.

192

Ch3J')ter 3 Oifferenri:uion

In Exct<:isc:s 5~58 show that lhc f:.:u:njlics of cur''CS urc orthogonal

+ 40. Show that if an cquatioo c-an be .sol\'cd for both )' in terms of x

l,r

= f(x )J and x

trujcc~orics

in tenns or~- [x = g(y)l. thcn

(sec Exercise 23 in Section 3.3). Draw both ramolics of

e:ur\'tS.

dy
dx

= dx
dy

* 41.

Prove that the derivatiYe of an odd function is an even func1ion.


und thai lbe dcrhiuhc or an C\"Cn runction is an OlkJ function. Rccull

DcfinitiOJl 1.4 in Section 1.5 .

42. Ocl<:mtinc whe ther the followins very simple: proor or the chain
mle ha< a flaw. If O.u den04es the change in 11 = g(x) resulling from
a ChWlgc O.x in x. &ld O.y i the change in y f (u) n:,;u!ling fnlln
llu , then

dy

O.y
. t>y t>u
= G.t-o
lim
= Ax-o
hm - - "'
1:1x
flu llx

(/x

y~

* 59.

If y

=ax"

= f (u) and u =g(x). use !he chain ruk: 10 show that

/>u)

( lim t>y) ( lim


6.1-o 6u
6.t ..... o 6x

( Iii') ( lim O.u)


lim - -&u-o 6u

~t-o

.. 64). u~c thcrc:sult ofl~xcn:i:sc59 tolinddly/dx 1 UlJ." ~I when

6A

2
ll =)A' -

d.\' du
=
du dx
,.. 6_1. find the l'alC

J/=1.

II =

63. 1r y = f(u) a"d 11

X+ J:r +I

= (X + .r')'

d'y
dx 3

jX
* 46. .\' -- - . s = :1+
-:=. /X;:::x
.< +6
$

In

Excmse~

Find the rate of cl!<tOi<

xf(x + l ).

x+l

I
44. 1 = (II + I) - - ,

45. )' =

* 62.

,,_ - -

+ v,

f(.\ ) :

+ .t6 with r~<."t tO

x3.

In Excteises 43-16 find d1 yjdx 1

43. )' - v

or chang~ Qf )' ;

)
X

41-54 as..t~tnnc that [(u) is~~ dillcrcntiablc func tion of

u . Find th.c: dcri'>ath'C of the given function with rcspcc:t to :r in

or y = f (X) = .fi""=X! \\; lh re.<pect lo

= g(.<) . show thai

= ddx'
-udy
du

dy(d")'
3

tl'yd'udu
du ' dx' tlx

I 3--- I -

tlu'

"' 64. At what point(:~o) on the: hyperbola x 1 line$~ thrvugh the pni" t (2. 3)?

:as

16y1 ;

dx
16 do tangct'.t

65. (icncrnli:rc lhc: result of equ ation 3.13 IO find a fonnul:a for

simplified a fonn as po"iblc. Then SCI f(u) = u' - 2u ond simplify


futlhet,

* 47.
* 49.

/(2x + J)

<IS.

l/(3- 4x)J 2

/(1 - x ')

* 50.

/(.T + 1/X)

' Sl.

f(l(x))

.f(- .\-)
53.
3+2/(.r')

52.

b - 41J(l -

54.

/(.r- f<xl)

:. 1/(.t)l"

i
3x))'
Yi<

(Ot'

n > I

:tn inregct.

66. TN! cum: x'y' = (x + 1)'(4 - x 1 ) is called a con,hoid of


Nirouu:tles. Plot the cun'C- und lind points where its tangent ljnc is
horitontal.

67. Repc:lt

Exerci~

34 in S~ion 3.2 ift~K' hill is the ucof Q circle.

13.8 Implicit Differentiation


We say that )I is defined txt>licitly as a function o r X if Ihe dependence of yon .r is given in

the form

.Y = f(x) .
Examples are y

x2,

(3.22)

= 3x + sin x, a nd y = 1/(x + 1). In each case, the dependent

variable s tands alone on the le n s ide of Lheequation. The di ffere.ntjation rules in Theo rems 3.1
10 3.5.

3.7, and 3.8 are applicable 10 explicitly delined functions.

3.8 lmplicil OiJTercotinc.too

193

An equa1ion in x andy may define y as a funCJion of x even when il is rlOI in form 3.22.
f.qua1ions for which y ha< not been separated ou1 are oflenwriuen in the generic form

F (x. y)

FIOURii 3.37

3y- 2x =

0~

)' i$ t\(1(

0.

t.Ul)

The notation F(x . y) is used 1odenote ar~ expression that depends on 1wo variables x and
y. For example, the volume of a r ight circular cylinder depends on its radiu< r and height h;
in particular, II = JTr 21r. In such a case we write thar V = F (r, h) = JTr2 Jr . Examples of
equations of form 3.23 are y - x 3 0 and y'- 3y - 2x 0. Equmion 3.23 is said to define
y huplidtl~ as a function of .r in ))011\C Uol1lain D. if fur each x in D , then! bone., and only
one. V'dlue of y for which (x. y) salisfies the equation. The equation y - x 3 = 0 de line~ y
implicitly as a function of x for all (real) x . 'll'e explicit definition of the function is y = x 3 .
The equation y 3 - 3y - 2x
0 does not define y as a function of x . For each x in the intervl
- I < .< < I. lbcre are three solurions of the equation for y. For x < -I and x > l, 1he

For yl 11

t'uncoon

of ."r

(-1. I)

equation has only one value of y corresponding to each x. This is rrlO>t easily seen from the plol
of the curve in Figure 3.37. We shall have mOore to say about the equation y' - 3 y - 2x = 0
huer in chis section.
Each of the following equations defines y as a function of x. and docs so implicitly:

+ xl -

(1, - I)

2x

+5=

0;

2y+ .<l + / - 25=0,

y>-1;

x + y5 + x 2 + y = 0.
It is

~asy 10

soh'e the firS! equalion for Ihe

cxp~icit

)' = -x 2

defini1ion of the function,

+ 2.< -

5,

and find its dcrhati,'C,

dy
= -2.(
dx

+ 2.

We can also obtain an explicit defi nition ofrhe function deli ned by the second equation, but not
so simply. If we write
/ + 2y + (x 2 - 25) = 0,
and use quadratic formula I 19, we o btaio

y=
Since_\~

- 2

j 4-

4(x2

25)

= - 1 /26 - x 2 .

is required to be greater than or equal ito - I, we must choose

y =

- 1

+ J26- x 2

This is the explicit defi nition of 1he function, and it is now easy to find rhe derivati\'e ory with
respect to x:
tly

d.r
The third equation is quil.e difTer~nt, for it is i mpo~sib l e to solve this equation for an explitit
dcfini1ion of the fu nc.Jion. Docs this mean !hat it is also impossible to oblllin dy/ dx? The
answer is no! To see this. we. differentjate both sides ofthe.equatjon with l'espect to x. keeping
in mind that y is a function of x. Only the term io y; prescniS aoy dif(iculr.y, bur. iiS derivat ive
can be calculated using extended power rule 3.2 J ~
4 dv

dv
dx

1 + 5y - - + 2x + - - = 0.
dx

194

Ch~pter 3

Oifferentiltiun

We can now solve this equation for dyfdx by grouping the 1\\U terms in dyjdx on one side
of the equation, and transposing the remaining two 1em1s:
(5)'~

dy
+ 1)d:r
- =-I -

2:r.

Oi' is ion by 5/ + I gives the required dcri~-ati\e,

dv'
dx

2r

+I

Sy~

+ ,

This process of differe11tiating an equation that implicitly defines a function is called Implicit
2
differentiation. II could also have been used in each of the first two example-~. If y +x 2.x + 5 = 0 is ditTeremiared with respect ro x. we have

dy
dx
-

..1.

2x - 2

= 0,

from which, as before,


dv
-
= -2.r + 2.
dx

When 2y + x 2 +yl- 25

=0 is differentiated with respect 10 x. tvc lind that


dy

dy
tl.r

2-

+ 2x + 2y- =

(2

dt
+ 21)-
= -2.r.
. d.r

dx

from which

anJ therefore

0,

dy
-x
dx=y+t'

i\hhough this resuh appeJrs different from the pre' ious expression for dy/d:r, wh~1111e re<.-.lfl
thai )' = -I + J26- .r2, w.: find that
-.r

dy
d.r =

-x

-_-:-1-:-+-../
'2:;::;6=-=.~1.+-1 - -:.;;::26::==-=....~
l

These examples illustr<ue tllac if impl icicditreremimion is used 10 obcain a derivacive. tlten
chc result may depend on y as well as .r. Naturally, if we require L~e derivative ac a certain
vftlu~ of-~. the~t they-value useJ is detennincd by the origimtl equation defining y implicitly
a) .t lunci K>Il

ut J".

I EXAMPLE 3 .24
A.w.Jming !hat ) i defined implicitly as a function of .1 by !he equauoo
X3J3

+ .fl J' T

2A

= 12.

fin<idy/dx whcnx =I.


SOl UTION

When we differentiate both sides of the equation with respect lo x. using the

product I'Uie on the ltrst two te nns~ w lind


2 J

3x y

y
d.r
+ 3x 3y 2ddx
- + 2xy + x 2 - + 2 =
dx

0.

3.8

l mr;iici iDitftrentbtion

195

Thus,
(3x 3 y2

dy

+ x 1 )d-

= - (2

dy
dx =

+ 2xy + 3x 2 y 3)

and

+ 2x) + 3.r2y3
3.3yl

+ xl

\Vhcn .x = I is w bstitutcd imo th~ given equa1ion defining _vas a function or x . 1hc resuh is

=l

+ .v -

10

= (y -

2Hl

+ 2y + 5) .

and the only sohotion of this equati on is y


2. We now substitute x
fonnula for tly fdx to calcu~ote the derivaoive ao x = I.

2 + 2(1) (2)

+ 3(1)1 (2) 3
3(1 )3 (2)2 + (1)2

= I und y = 2 iono chc


30

= -

i3'

Tmpl icit dift'erenti at ion can also be used to fi nd second and highc.rordct derivatives of fundio1\S

thut urc defined implicitl y. Culculutions cun be messy. but the principles are the same. For
cxnmplc, when the cqumion x 5 + y 3 + y 2 = I defines y implicitly AS a fwlt'tiool of .r. it is
straiglnlorward to calculate that
dy
- 5x'

dx

= 3y2 + 2y '

To find the second derivative d 2yfdx 1 , we differentiate both sides of this equation with respect
to x . We use the quotienl rule on the righl. and when differe111ia1ing the denominator we once
again keep in mimi lhat y is a function of .r. 1l1e result i~
(3y2

dx 2

dv)

dv + 2-
+ 2y)( -20.II 3) + 5x 1 ( 6y-
dx

dx

(3y2 + 2y)1

We now replace dyf dx by i1< expression in terms of x andy,

+ 5.r 4 (6y + 2) ( .,.....,-2, 5,-x-:


"- )
3y + 2y
(3y1 + 2y)1

-20.r 3( 3/ + 2y)

d,r2

<lJt<.l bring the two terms in

the numerator to a common denominator:

-20.r3(3y 1
2

d y
=
t/,rl

20.rl(3y2

+ 2y) 1 -

25x 8(6y
2
(3y + 2y)
(3)'2 + 2y)2

+ 2)

+ 2S.r 8(ti.v + 2)
(3y 2 + 2yp

2y}2

It would not be a pleasant task 10 proceed to the third deriva1ive of y with re,spect to x .
ln the above examples we seem to have adopted the principle that both sides of ao equatioo
can be differeotiated with respect to the same variable. Thjs is not always true. For example. if
we dilTeremiate both sides of the equation 4x = 2x. we obtain the ludicrous result that 4 = 2.
Obviously, then, this equation cannot be diffeuentiated with respect to x.

Possibly tht reason 111.11 difTerem"-ion fails in the above e<ampk is tlwt the equation
COfllainsonty Qflc van able. nanlel). ;r. Perrn.ps a more n:tbOiliableqvu aton nu&h' be Can every
equatiOtl cattMintnJ 1wo v~nable.~ be differentiated? To at'\\\Cr thi~ quefolkw"l \\t consider the
situation of a ~ip hudng northa.ec from a pier. Let us choo~ eau a~ lhe posiu'c x -direction
"'ld 110nh as tht positive y-direction. both originating fr<>m the pier (Fiilre 3.38). Since the X
~uxl y,c:t'KW'tlinJtes of the ,hip ch..tngc ,tt the $;m'le ra1e. it follow~ 1hat 1hc rme or cho.Ulge of the
y<ootdinate of 1hc ship with rc>pe<.'t to its .x- t;Q()(dinate '"'hen the ~hip is 2 lo.m from the pic::r i~
equal to 1. \Ve c:u1 get ttus rc.sult by noting that the path ,,,. the shill h the hne )' = .v. and the
derivative of this JI\'C~ dy/d.t = 1. Rut consi<.ICJ the fnllowin~ arv urtlCill.
Whe111he ~hill is 2 lim from 1he pier, ,t = y
./i. ~md thercfme

,i
...

Ship~-(\, y)

2/

r iel' / '

: .J2

JJ we di fl'erem iat.~ this equation with respect to .x. ""e find lhiH

2-t
and wilh .t "' ) "'

dy

+ 2y -d .

"' 0

d)

'"

dx

..
)

./2.
dy
"' -1.
tlx
yl = 4. which contain~ twu Vlltinblcs, ~ led tu an
-

D1fn.-rcnuation of the CQ!.I4Uion .rl +


erroneous result.
Thee tw<> c~.tmple< have certt~inl) illuSlratecl that nN 1111 equlltlons can be differentiated.
\Vhal thc1l tliMillj Ui!\hcs ml equtuion thut can be diffcrt!nti:atcd fron1tli1C thltt t"HIU)Ot? Recall
that difterentimion is n ti milin~ t>ro<:c&s. Evaluate whatever is to be diff'ercnti,ued. say j' (.t).
a1 x + h . sub1n.1ct ics 'o lue nt x . divide by h, arld cuke the limit us lr -to 0. If bo1h sides or
an equation are co be differentiated with respect to a v<~riahle, lhen both sides mu~t be equal for
a continuous range ,)r'a lues of 1hat variable. The equntioe' J.\" c=~~ 2x ca''Klt be differentiated
because it is uvc 011ly for .t "' 0. ThecquatiOtl x 2 + )'1 = 4 cano101 be diff'erenthued in the ship
example becuu<e ;,. tltm t.tample it i alid oo>ly when the -.hip i< 2 km from the pier (i.e .. only
when \ = y = ./2). The equation )1 = r CO:lll t:.e differentiated bau:,c it ~~ \<alid at ~
point along lht pmh of the >hip. This i< an e.<tremely important princi1lle. and we. will return 10
it milt\) times. Tu cmphasZt i1orx.~ J.gain. w< ma~ dl}ftrtmialt ffli ~qutu,on "'"It rtspccr to a
"'nriable ouly ifII-, ~qunliV'l u '-'"lllld/or a coruinuous rang~ oj,aluts o{llmr l nrillblt.
F.:oeh or the three equations 111 the lx.11inning <>f this ~'loon wn< gi'"'' a< dcfoning y as a
function of x. and u <toch ddiiiCs the function loc some donoain of \'lllue for .t . Differentiation
of the."' cquati'"" \\a therefore acceptable ac:<:onling 10 the princople stated abo-..
\\'hen \\e ure tokl chat cqu~nion 3 23 tlciines y a:, a funcaion of :t . in1plic it diiTerentiation
lead< to the<.leriv,ui\c <ly {tJ,, . Out how can we tell whcthcrao~equatitln F(x . )'} = 0 defines y
implicitly aHl function of x ? Additionally. givcno that F (x . y) = 0 due ddinc y as a function
of x , how do we know that the C.XJ>ression for d y f tlx <>bUlined by implicit di ft'erentiation is a
valid rcprc,cnllllinn nf the dcri\'lltivc of the function? After all. functions do n()tl\lwnys have
derivmhcs at Dll point< in their doo11ains.
t\ n.I\WCrR co tht(C quc,tion~ arc intimalely related. To M..'C how, !ooUI>IX"N! dun an equation
F(x ' y) = 0 in ,\ nnu )' i >ttti,fic<l b) II point (.ro, )'o). und \\hen the cqu~lion is diJTcrentiar.ed
implicitl) it lcad!lu a quotient for dy f dx.
dy

dx

P (x . y)

Q (x. >)

h can be shown that if Q(xo. Yo) ;! 0, then the equation F (.t. y) = 0 defines y implicitly
as a function of x ror ~orne open interval containing x0 and Lhe derivJtive of this function
at -'o is P(xo, Yo)/ Q (xo, Yo). Proofs are usually given in advanced books on mathcmatic-.11
analysis . When Q(..ro. Yo) "' 0. two possibilities exist. Fir.l, tho o<1uation F (x, y) "' 0 might

3.8

lmpli.:it Oifttrenti:uioo

197

not define )' as a function of .t i11 an interva l around x0 . Second, the equation ol.igbt de6nc
a function. l>tll the function does II()( ha'e a deri,ative at xo. To illustmte. consider firs t the
equation y 3 - 3y- 2x = 0. which we introdoted earlier in thi< section. The curve defi ned
by this equation is shown in Figure 3.37~ and it clearly i llustrates that lhc equation does not
define )'as a function of x. It doe~ define the th.r cc functions of x in Figure 3.39. Suppo.o;e we
d ifferentiate the equation with re<~ct tO x

., tly

dy

dx

dx

3y"- - 3- - 2 = 0.
and ;olvc for d yjdx.

dy

lfx

3(yl - 1)

This der ivative is obviously undefined allhc points (- I. I) and (I. -I) . and these arc precisely
the points that separ~te the original curve into three parts. It is impo.o;siblc to find a portion of
the curve around either of these points thm de fines )' :lS function of .r. At any other point on
the curve. the formula d y j dx = (2/ 3)(y 2 - 1)- 1 is a valid n::pre.<cntatiun fur the derivative
for whichever function of Figure 3.39 contains the point.
I*I:'IJ!:l:ci[!tli;JI:,,.__~-

Oi'oi-skm of a C\U'\e into pans each of 'IA'hich rc:prtSel'llt a funcriou

( - 1, t)

(- 1, I)

mi!Iil
\"tt1ical

~
Graph uitll
an:em line at (0. O)

( 1, - !)

( I, - I )

Consider now the equation

x = y 3 >ho,~n grdllhically in Fig ure 3..!0. Implicit diffc rcnti

ation leads to

dy

-dJ.' = -,.
3y-

C iear!)', y is a function of x for all x. but tlyj dx is undefined at (0, 0) since the tangent line
is venical at this point.

I EXAM PLE 3.25


Curve wilh

no tangent

li~ ~ I

(0. ())

The curve y 2 = .r 2 - .r is called ~ /enmiscate. It is ploued in Figure 3.41. Use implicit


difleremiationto find d y j dx and discuss wlliiL happens when the poim (0. 0) is substituted into
the .result.

SOLUTION Implicit differentiation of y 2 = x 2

x g;,..,,

tfy
dx

2\'- = 2.T - 4,\.>,


from which

dy

dx

-2.r )
= .r(
-l -.
. . . .;,
)'

This result is undefined at (0. 0), and Figure 3.41 indicates why. The equation does not define
y as a function o f x around x = 0.

118

Ch.pac J

I EXAM

l) iA'clellltiuion

PLE 3 . 26

...
Solve Example 1. 16 using derivtUi\'CS.
SO Ll,..rJON 1b the right of the r ight b nulCh or the hypcrtx>l:' x~ - 4y2 = 5 in Fisurc 3 .4 2
is a swamp. We a.re required lo fi nd the point P1 att which a str aight pif>elinc from P 1(15 , 10)
n1eers i.l pipeline a iOJlS the litlC X = - 1 as fa r d own the litlC Ul" PQ:"Siblc. The required point
Pl occurs when Iitle P, P'l is latlgenl to the hyperbola. l,..ct the point of t3lg_e.tl<.::y be Q(tt , h).
l)iffCI'CtHiat io n of .;\'1 - 4y2. = 5 with n:spcct 10 x ti\'eS

d )

2.< - 8yd :r

=0

t/y
tlx

4y

T'hc slope o f lhc UUlgem line 1u Q(a~ b ) is lhcrcfore d / (4h). Since the slope of P 1 Q is
(IJ- 10)/(11 - IS) , io follow, olmo

b- 10

"
4/>

15

(I -

P 0( 15. 10)
.... , -4) 2.

----,0
-5

Cros-5-multiplicn1ion leads lO

a' - 4b' = 15a - 4()/).


Si nce Q(a. b ) is on ohc hyperbola. ioall<! follows lh>~
a2

1
- 41J

5.

\Vhen we subtracl these equations.

0
Substitution of this

= I Sa - 40b - 5

i 1110 tll -

a =

I
J( l

+ Sb) .

4b1. = S gives

-I ( I + 81J)' - 4/Jl = 5

16b

+ 64b 2

36b2

45.

~I11Us.

0 = 2Sl>2

+ 16/)- 44

= 4(iiJ

ll ) (h- 1) ,

1 and b = - I l / 7. The sec.o nd solution gives a point on the left branch


of ohe hyperbo la ao which a oangeno line passes ohrough (15 , 10). "!be soluoion b = I gives
the point (3, I) on the right branch of the hyperbola. The slope of the oangem line at (3, I)
is 3/4. and therefore ios equation is y - 1 = (3/ 4)(x - 3). IL cuos the line x = - 1 when
y =I + (3/4)(-1- 3) = -2. Thus, P 2 has coordinates ( - 1, -2) .

a nd solutions arc b

..-..

3-S lmplieit l)ift'ertntiMion

199

EXERCISES 3.8
rn E~ercises 1- 10 fi nd dyf d x whcre\er .l' is defined as a fund ion of x .

+ )' = 4X3
x' + y 2 + y > = I
A',\ ' + 2.t <lyZ + 2

Cakulale el:lSiiCily for the ruoction defined h)' c.ach of lhC followi ng
equations:

I. .\'"

2.

3.

(o)

4. 2x' - Jxy' + 5x1- 10= 0


5. x+xr' - x'y3 = 3

6. (.T + y )'

8.

X+)'

----:'--'---

3-_v

.10. Fi.nd that point P(a. b) on the lir>t-quudrant pun of the cltipoc
2J. 1 + 3y1 = I 4 at which the tangent line a1 P i~ perpendicular to Ll:x:

= 2x + 5

line joining P and (2. S).

Jx+.r+.1' = 12x 2 + y

400r + 200

X =

Sbow thai thcclasticily of a f~.tncl i on is equal to I if and only if Lhc


tangent line to it.s gruph ~s thiOUih the origin.

31. Sbow tbtll lbc cquution of the tungcnl line 10 lhc hyperbola
b z.A 2 -a 2) ' 1 = 11' b1 a1 1hc point (.to. Yo) is b~ X.(o- l4 2.\'Yo = a 1b 1

9. /1 + xy - xy = 15

(b)

* 29.

= 2x

7. x(x - y) - 4yl

x +-')
j(x) = x ( x+2

32. Pruv<that for uny circle (x - /r) 1 + (y - 1:)1

10. - - - - = 4

= .-'.

tl yfdx
I
(I + (dyf dx)'J'Il = -;:

11. Find cquaaions for the 1ungcm und normal lines h>thccu.r\'C .xy2 +
y' = 2 allhc poinl (I. 1) .

t 33. A solution passes lhroush a conical fUtcr 24 em t:kcp und 16 em

In E.xcn:tscs 12-15 find tl2 y f dx'! where\'er y is defined as a function


of x.

* 12.

14. y'

* 16.

+ y3 + y

+ 2y =

=1

5x

Fmd tlyfcl.\ t .r

13. 2.<' - y'

=a -

1s. < + rJ'

= .<

= I when

.1 ' )'

xy

tbc top into a cylindrical \x;sscl of diameter J2 <:ut. Find u.n


relating the dcJl(h If of solution in the filler am.l dCI>th H of
~lui ion m the C)rlindcr. What is the rate or change of h with rcspoctlo
H1

UCI'OSS

cqwaion

34. If x ob_ie.,ts arc sold at a prioc of t(x) per objcel. the lollll rc.-cnuc
is R(x) = .r r(x). Find lh<: marginal revenue R '(X) if price is dell ned
implicitly by lhe equation

+ .ry' = 2.

In Exeroses 17-20 lind tlyf tl.< anti d'.l f dx' .

17. (.T

19.

.Ty1 -

y)' =X' + _r'

3x1.1

- .T

18.

x'y3 + 2x +4y = 5

= y/1 -

.. 20. .t

+I

where a > 0 is aoonstant. and 0 <

y'

* 35.

x < 4<1 3

The general JP01ynomhil of tlcgrcc u h

2 1. Fi".d d.lfdx when.<= 0 if Jl - xy + 3y = 4 .

* 22.

find dy/ tlx when x

= I if x 1 y 3 + X ) ' =

2.

=2 if y' + (.r -

23. Find dyf dx and tf'yjdx' when x

24. Hnd dy fdx o.nd d 2 y fdx ' when J = I if x '

* 25.

Find poinl( s) on thc(UC\'C XJ


Ihe langcnt line 1s txlual lo zero.

+ 26. J::"a.nd poinl(s) on lhc: cutve x 2

2)y

= I.

+ 2,t y + 3y 2 =

2.

= 16 al which the s1opc of

+ y ll'

- 2 at which Lhc ~nd

+x

deivati\'c: is equal to ~cro.


... 27. If:. then nat nuclear 1'\."'.:ctor i:i bui lt i n the :-.I'IU I~ of a rig,hl cin.:ular

cylinder of mdiu"' r and height h . ahen. accordi11g 10 neutron dinusion


1.hcory. r nnd h must satisfy Lhc !!qut:uion

c~sy + G)'= k = cootstant.

whc.:re a0 a 1 a,. arc cons:1ant~:. Show lh.al two

polynom i al ~

of

degree n.

c--Jn be cq~l for all x if:md only if au=

* .16.

(>) Find / '(0) if y

bo. a 1 = h 1 . . , 0 11 = bJ~

= f(x) is defined impl;citly as a functio<l

ofx by

xl l + 2y = ,.> -

y.

(b) Show thll by squaring 1he equarion in p art (a), we Dbtairl

Finddrj dh.
*- 28. TI1e elaslk ity ol' a fu nction )' = f(x) is de fioed as

Ey
Ex

xdy

= y dx

DiiTerentiatc this equatio n witJ1resr>ect to

Do you have any dil'fitultie..c;? Explain.

Lo fiod /'(0).

Usc imphcil d ill'cr<.'JUistion 10 r.:dll Excrdsocs .5>-58 i n S.:e~

,. .1 7.-40.
;t

t io n 3.7.
4 1,
(:t)

U~

1hc..:un c .( y 2

i nt()licil dill'CICilljuliOO 10 find d,l'/(/X if

i' =

x2- -'x".

(b) Can you ,ollculalc ti)'/(/ X ~u :r :;;~ 0 using t he rcsull of part

.1:' - 4,t" in Clrdc-r to e'<plain

(a)" Or3W ti)C curve y 2 =

tJIIS d i(JICUI()'.

.- 42.

(a)

= l,wl~rco ~ x

Pind,ly/dxit' ,IX+ ,/Y

,:!: l , dctiet<'s

y imf,lki ll)' a" u ftm~do1t of ,\'.

.Jfi7 + Jf;il = I
+ .r 2 or 6y + 15 = 0'?
= uf~. and .( 1 11'! + C-=1' =

( b) l)ruw lhc c urve

.&4. Find d)'/ Jx if

"' 49. Find points Oil


t nnge ~u line i.s Ol.l ll<~ l to I .

<J $

= 2 ut which the !'lope o f Lite

''rt

... S l. Let (,r. y ) be any poi111 on dlccunc x ?/l + ) ' 'l" = (l'l :.. \\hCtC
a > 0 i:; u wniit:ant. l~int.l !l l''o rmuhl fnr t he; lc:nglh of th.ut p.'trt llf t he
l:angcnt line "' (.A'. )') between thc\!OOIinatc Wtl.!< .
1

Find point s whcr"C: Lhc slope of the l!angetll line i $ cquull\l - I .

. . , .. r' = Jtu'>

45. G iwn IJl<ll 1hc eqwltion~

tk:llnc y a!: o1 function of 11, anti v

i.;.a a-ationul oumbct.

_, 50. S ilO\\' that the thmili~s of cur\'CS y 1 -3 x } / (a - .r) und


(,\': .1 y z) : = IJ(J ;.;2 + y l)
o1thoconal trujt-(.torics.

defines 11 impficid) tlS a furl(:tion of.\' .

1ernts o l' X .

+ "' J

11

..- 52. rhc {,.'\U\' C: d clH:ribt.xl h)' the. eqlc&tion .\' 3 + )' 3axy. where
a > 0 is a con!'l:tiH. is callc:d thefolittm of /)~.\cartes ( t1gw-e belo\~1).

4J. Wllal i11 d,l{dx if x 1 + 4.t

., 48. Vclif)' power rule 3.1 in t he case thnt

;;1 fu nctjun (lf X, timl ll)'{dx in

y , .l!ld v.

,. 46. Show that Hll) fundiun defi ned i111pliciLiy by Lhe t:qw.u ion

....
-

.v'

C,

X:::

:t

where Cis n consrnnt, sutisticsthc cqu:atiu1l


tl)'

3x 2 -

dx

+ r'' =

53. Usc implicit Uiffcccmiation to soi\'C 1:-:Xcrcisc 6 7 in Section 3.7.

...... 54.

2xy .

,. 47. Show th.:aL:lny funct ion tk,ined implicil_ly by 1f1e ~1Li<t11

(!!)

When a point (.\'. )') lll0\'1!; in the: xypl:mc S() that the
produc.1 of it,. d ist;anc<:s f1o nllhc poinu (a . 0) and (-a , 0)
2
is atwuys equal to a (.'OnStanl ~hkh we denote by c (C >
ll ). the cwvc thut iL rotlows is called the cwalJ of Ca:~sini.
Verify thauflc cq_uiltiorl fonhC:')Covals is (x-z + ~,z +a 2 ) 2 -::;
'-'4 + 4ll, x2.

(b) Show thaa when c <

..fia. there an: six poi nts on the o ,a1s


111 whkh the 1angcntlinc i" horitonl!ll. bul when c ~ . / ill .
dltro i:U'C only two such poini.S.

where C is t cnn..;t :~nt, ll;;.tisJie.s chc:cq uation

d l'
(.r)' - .r ')d-'X

+ y'

- 0.

t>+ 35. Shuw that the ellipi'IC b ?x 2

J 2x 1 -

c 2y 1

c 1tl'

+ a 1y 2 =

o 1 b? and the hypcrbuhi

int~t orthogonallr if a 2 - 1)1

;;;:; c

+ d 1

I3.9 Derivatives of the Trigonometric Functions


The trigonometric functions. their propcnie~. and the identities that they satisfy \VCrc discussed
in Section 1.7. \Vc emphasi;:c an important convention in calculus. Arguments of trigonometric
funcrions arc alwnys measured in radians, never degrees. \'lith this in mind, we prove 1he
fol lowing theorem.

THEOREM 3.10

lim

e.. o

sine
(I

(.1.24)

Thi.s is strongly suggested by the machine -generated plot of (sin 0) JB in Figure 3.43. There
should be an open circle at the point (0, I ) since the function is undefined at/} = 0.

3.9 l_
k ci\':uives of lhc T:i~on<)flt<:eric FUI..::Iions

201

FIQUFtE 3.43

sin 0

y = -11

10 0

-10

PROOF If (} is a posili\'C ucutc angle as shown in Figure 3.44. then


MOJI#Jil;t#letf)

sin t9

With (} expressed in radians. the area of a sector of a circle is (r 2 9) / 2. Consequently,

lim - - = 1

i1

area of triang le 8 0 P < area of sector 8 0 A < area of triang le 0 B T.

P!ouf thai

()

Tangent line
Y oci rcleat 8

~ IIBPII IIOPII

<

~(I/B

<

~ 1 0811 11BTII.

When we multiply each term in these inequal ities by 2 and express lengths o f1he l i ne segmems
in terms of II.

(sin ll)(c<J<\B) < B < (l}(tan B) .

Divis.ion by siu gives

cosO <
radius 1

e
sin (J

<

cos 9 '

and when each term is inverted, the inequal ity signs are reversed:

I
si nB
- - > - - > cos& .
cos O
()
We now lake limiiS as 8 ._.
Scc1ion 2.1 implies tluu

o+. Since cos e .....

I and I I

cos e ....

I . lhe squeeze lheorem of

sine
= I.
o-o+ 8
Since (sin 8)/ 9 is an even function (Figure 3.43}. i1 follows that
lim

sin(}

lim - - = I.

o-o- B

Since lefl- and righ1-hand limiiS of (sin 9)/9 are bo1b equal to l, the full limit as fJ -+ 0 is
aJSC) I.

sin 8
lim - - = I.
u- o e

With this result \Ve-can now find the.derivative. of the sine function. Derivatives of the other
(ivc trigonometric.: fum:tion.s follow c~i ly.

THEOREM 3.11

- sin .t' = cosx


dx

(.1.25 )

202

Cb!\p1er 3

l)ifferen!i.'\lion

PROOF If we sel j(x) = sin x in equa1ion 3.3 and use 1he 1rigonome1ric idemi1y

.A .B

s 1n

= 2cos

SJJl

(A +
B) sin (A -2 B)
2

we obtain

' (X).

. "-f.:....
11 )'-------"f _,_
(x..!.)
= h-hm
- (x_+"---:
0
lr
. sin (.r +h) - sin.r
= hm --"----'----'--"--.:;.
h

h ~o

I[ (2r 2+ ")

= li m - 2 cos
a-o h

sin (")]
-

. [ cos ( .~ + -")sin(h /2)]


= hm

1. - o

=
=

. cos ( x
hm

h....O

+ ")
2

11/2

.
sin (h/2)
hm

1!2-o

h /2

COS.t.

II is a simple npplicalion of lhe chain rule to prove the following corollary.

COROLLARY 3.11.1

If u

= /(x) is a dirferemiable function.


d
du
sinu = cos u - .
dx
dx
-

026)

I EXAMPLE 3. 27
Find dcriVtltivcs rorthc following funclions:

(a) /(.r) = sin 3x

( b) /(.r)

sin 1 4x

~oumoN

(a) According to formula 3.26 ,

/'(x) =

cos3x~(3x)
d.r

= 3cos3x.

(b) Fo r !his function we must use extended power rule 3.2 1 before cqualion 3.26,

Since Lhe cosine func.tion can be expressed in terms of the sine function, it is str.tighlforward Lo
11nd its derivative.

l>eriv:.niwll> of lhe Tri,gonomel'ic f'll!'K'Iion~

3.9

203

THEOREM 3 . 12

d
-cosx- -sinx

(3.27)

dx

Si11ce cosx can always be expressed in the form cosx

PROOF
thai

d
- cosx = dd
dx
X

sin(~2 -

.t) =

-cos (" - ,,)


2

sin (;r /2- x). it follows

-sin x.

CORO LLAR Y 3.1 2 .1

If u

= j(x)

is 11 ditTcrcntiabte function,

du

-cosu- -smu- .
dx
dx

(3.28)

Derivatives of the 01her trigonometric functions are obtained by expressing them in tem1s
of the sine and cosine fun~tions. For the tangent func tion. we usc thcchnin rule and the quoti ent
rule to calculate
cosu(cosu) - sinu(-siuu) du
d
d ( sinu) du
dx tanu = du ~ dx =
cos2u
dx

du

Consequently.

du

d
-

dx

laOIJ

= sec2 u- .

11.29)

dx

A similar calculation gives

cl

, du

-cot tt = -esc u-.


clx
dx

(3.30)

For the secaJU fu.n ctiou, we obtain

-c/ SCCII
X

= !!._
c/11

(-1)
COSII

du
c/.t

du
(-l )(cos u)-2 ( - sintt) c/x

sin u du
cos u cos u clx

= - - -- -

or
d
du
- sceu = secu 1an u -.
clx
dx

CU I )

Similarly,

- cscu
dx

tfu
-CSC IICOtfl -

dx

(3.3:?)

Notioc the relationship bci\VC:Cn dcri,atives of the cojimctions - cosine is Lhe cofunction
o f sine. cotangcnl of tangent. and cosccan1 of sccam- and corrcsponc.Jing dcrivali,cs for the
func1ion~. F.ach function is replaced hy its cofunction and a negative ~ignis added.

l EXAMPLE 3.28
dyfdx iJ y is detined as a function of x i n each of tile following:
y = sin 2x
(b) y = 4 sec (2x 3 + 5)
2
y = tan 4x
(d) y = 2csc 3x 2 + 5x sin x
2
(e) x tan y + y sin.< = 5
Fi nd

(a)
(c)

204

Chapter 3 Oiffereoti:uio1t

SOLUTION

dy

d
= (cos2x}-dx
(2.r) = 2 cos 2x

(a)

(b)

-d = 4 sec (2x
x

dx
dy

+ 5) tan (2x'' + 5)-d

dx

(2.\"

+ 5)

= 24x 2 sec (2x 3 + 5} tan (2.r 3 + 5)


(c)

dy
d
- = 2 Uln4x - tan4x = 2 tan 4x(4sec2 4x) = S tan 4x sec2 4.r
dx
dx

(d)

dy = -2csc 3x2 cot3x 2


dx
dx

~(3x 2 ) + 5sinx + 5x cosx

= - 12x esc 3.r 2 cot 3x 2


(e)

+ 5 sin x + 5.r cosx

If we differentiate the cqumion (implicitly) with respect to x , we obtain

'
dy
2x U\o y + .r- sec 2 y
dx

dy
+ -dx
sin x + )' cosx = 0.

When we solve this equation for d y I d.1, the result is


dy
=
dx

2.nan y + y cos x
x 2 sec2 y +sinx

I EXAMPLE 3 . 29
In the mechanism of Figure 3.45, crank 0 A rotates at one thousand re,olutions per minute
(rpm). Find an expre.ssion for the velocity of' the piston in terms of R and 8.
lilijii ;J FlmJ

Velocity of

p1s:ton in cr.mk mechanism

SOLUTION If the crank starts with B = 0 ~t time r = 0, then x and y coordinates of the
moving end of the crank are x
R cos 8 and y R sin 8. The position of the piston relative
to 0 is x + L
R C<JS IJ + L . The \'e.locity of the piston is therefore

diJ
u = - Rsin B- .
dt

Since the crank rotates at 1000 rpm.


-- L-

t/8
2rr(IOOO)
IOOrr
- =
=
dr
60
3

radians per second.

Thus.

J00JT
u = - --R sin8 mls,
3
provided R is _in metres.

I EXAMPLE 3 .30
The I kg mass in Figure 3.46 is pul led 10 em to the right of its position when the spring is un
stretched (c~lled I he equilibrlruu pu.tilion), and given velociry 3m/s 10 I he Jcf1. 11~ di~ placemenl
from the cquilibriumJXJSitiouthcrcaftcr is given by

. 20t m
x(l) = -I cos201 - -3 sm
){)

2()

'

r ::: 0.

Find all times when the velocity is zero and alltirnes when the acceleration is zero.

3.9 Deri~r,~rives vfrhe Trigonoltlerric FuncriuiiS

Velocity and acceleration of oscillating mass

FIGURE 3 .46

k=400

I I

000000'

=I

111

.\ =0

Simple h.;wnonic motion of oscillating mass


f1

II

0.15
0.1

0.05
05

1.5

-0.0

5\

-<l.l \
-01

S\

\1

SOU TION The displacement function in Figure 3.47 indicates thm the mass oscillates back
and fonh about the equilibrium position. This is called simple harmonic motion. It is a direct
result of the fact that the mass moves along a fricrionJess surface and 110 account has been taken
of air resistance.
To detemtine times at wh'ch velocity and acceleratjon vanish. it is ad\'lllllageou:; to follow
the lead of Example 1.36 in Section 1.7 and express .r(/) in rhe form
I

-cos 201 - - sin20t


10
20

= A sin (201 + !).

When we c~pand the right side with a compound-angle formula, we obldir.


I
3
- cos 201 - - sin 201 = A(sin 201 cos f/>

10

20

This equmion will be valid for all/


I

lO

+ cos 201 sin)

:=: 0 if we choose A and to satis(v

=A sin~.

3
=A co ,P.
20

Squaring and adding these equations c~minates :


I

'J

-100 +=
400

A =

A2

:r mo
20

If we choo>c to use the positi\e value. then


0

-'-

su1 w

When we let <P


the form

= -

and

..m

.l

cos> = - - .

..m

= 2.55 be the smallest positive solution o f these equations, we express x(l)


x(r)

./13 sin (20r + </1)

20

mo

in

205

Cb:l.puw 3

Z06

DiffercnliAiion

Velocity and acceleration are


V(l )

.Ji3<.'0s (201 + </>) m/s.

a(1)

+ ) mls1 .

= - 20.Ji3sin (201

Tite velocity or the muss is zero when the mass is at the poims fanhest from the equilibrium
position. h occu~ when

0 = ,Ji3 co; (201 + if>)

201

+ <I>

Jr

2 + tltr,

where n is an integer. Thus,

and these are positive for


0

=-

11

</>

IITT

Jr

= 40 +

20 - 20'

2: I. The accelenuion vanishes. when

+ if!)

20.Ji3 sin (201

201 +

mr.

where 11 is an imeger. Thus.

= 20

and this is positi,c for

11 ~

20
I. Notice that accclcmtion is z.cro when the mass passes through

the equilibrium position . This i s to be expected because at x

= O, the spring has no stretch and

therefore exerts no force on the mass.

EXERCISES 3.9

In xercise< 1-30 lind dy f tl.r .


I.

.r = 2 ~i n 3x

1:

3. y = sin1 x

.v = t'O"X -.a sin Sx

25. y= u ~sccu.

U.

4. y utn- J3x

s. y = scc4 lOx
7.

=in' (3- 2.r')

6. y

~in

2x

II.

15.

2~iny

+ 3co~x =I

14.

I,

= Ulll JX

1 ~sin (sinx)

sin1:c

,1' = --..:..:.....:-,-1 +cus'x


1 + lan3 (3x'

.r +I

I J. y= J~in3.x

, Jr 2 +

s. y: .\'cotx1

1 ~.

SC<V.

= tsc(4 - 2x)

x o;i n.\'
10. y - -

.v = sin3 x + ca5.x

= JJ

27. y =

29.
9. y = - cos Sx

,1'

u = xhm(x + 1)

- 4)

* 30. ,\' = --...,.,---.:c 2:sin x.

,\' = ~i n 2xccl"2X
)' = (1 + wn' x )'1'

16 X

COS)' -

,V C<,S X

* 31.

:-;in.r =

.c1 + .v

* 32.

w.ny = x + xy

= .'\
In Ext~rci~s 33-38 cvaluare Ihe limil, if i1cti!:ls.

17 4 s in 2 .x - 3cosJ y = I

I<Jn .~

34.

33. li m - -

19. .r +
*

18. tan (x + y)- y

2J.

~-''.V

= .5

x = y3 esc-\ y

23. y

,r -..()

2
* 20. '-'''' + tan y = 3.\'

22. )' =OOS(I!InX)

= xJ- x'1 t.'OsX + 2.\' sin x + 2c..-osx

*
*

sin 2x
35. lim - x~o

37. lim

(x

,_o

+ 1) 1 s in x
3 .\'J

-
lim

36. lim

JS.

I -

cosx
X

sin (2/x)

,_""sin ( 1/x)

cosx

lim
..
x-.T/ l (x - 1f/2)

* 39.

IJ1-aw grnph<ofthcfunainn<J(x) = l sinxlandg(.r) =sin 1-<1-

Where do theo::c functioM fail to be diOCremiabJe'!

* 40.

Does Um,_og'(x) exist for tt-o: func tion g(x) in Exercise 32 of


Section 3.3?

43. The two idenl~dl cranks m the figure below rot.:uc al con~tant
angular s:pced.~ wa and w1 radian<: per second. dri"iOS the auachmcnt
'"nically. Find 1he angular speed d 8 f dt nfthe slolted bar in t<m>< or
a , Jl, o>o- and w2
.)

41. The angular displaecmc:nt of the pendulum in tbc following figure

IJ

at tim<:! is g iven by

= f(t) = A cos (wr + 4>).

1 2: 0.

- - -3R - - -

wl>ore A.
equation

w. and d> n: <OI>stants. Verify that (I = f(t)

.atisflcs the

+ .W. Find all po:,iti \"C ' ulucs of,,. for which the: dc1 iw.ti'I'C of ahc runc-tion
f (.<) = CO> (X + 1/ A) i, cquaiiO IJ!C\l.
45. An e lliptic cum roaa.tescou.tuecclockwi5e a1600 tpm (fi gure bciO\\').
find D (orm u l~t (or the \"CIOCity o( the follower if the ellipse has mujor
anc.l minor axes oftcn~th.s 2a and lb.

*- 42. \1\' hen the mass m in the naurc below 1110\'C!!. \'Crtically on the end
of the: spring. ils dic:placemen1 ')I nu~ c:atisfy the equalioo

d'v

m ~,

dr

+ k\' = 0.

where k > 0 is tlx:: cons.~um of elas:ticit)' for the (pring. Verify that the
fu ne1ion

46. Find a formula for the following dcri,uti,c. simplified as much as


po~ib!c:..

cl

dxlsmx l .

satisfies this equation for &ty constants A and B.

**

where n

;:: I is an integer.

47. Shou1 that lh.e funclion

f(x)

= {x ' sin (1/x),


o.

xi O
x ; O

has a derivative al x = O.lxlt f ' (x) is no t t.-ontinuous al x = 0.

**

_[
y=Oat
equil ibrium

48. Any cross-scctjon o f the n::flcctor in a car hcadJight is in lhc fo rm


of a pmabola. y 2 = 4x. 0 ~ x ~ 2. with the buJb al lhc point
( I , 0) caUcd the focus of the parabola. It is a principle of oplics that
all Jjghl mys from the bt1lb arc reflected by tJu:: mirror so that the angle
between tbc incident ray and the normal to the mirror is equal to lh e
angle between tbc re flected ray and tbc normal Show that aU rays me
reflected pmaJiel to the x -a:~is.

13.10 Derivatives of the Inverse Trigonometric Functions


Derivatives of invcr~c trigonomeuic fullCi iOilS arc
ation. \Vc tx:gin with the inverse si ne functiOt\.

Oh)$1 e~\s i ly

obtai ned with im,,licit c.J iffcrcnti

THEOREM 3 . 1 3
(3.33)

tlR0011 If we. set y = Sin- x ~ then ~t = sin )' a'ld we can differentiate in1plicitly with
respc.."Ct to x:

tly
tlx

= cos y- .
T hus.
COS)'

To express cosy i_n terms or.x we can proceed in two ways:


(il From the trigonometric identity sin 2 y

+ cos2 y =

I. we obtain

We know li1at y = Sin - x and the princip"l values o f Sin- x are - rr/2 ;S y ;S
1f /2. Therefore, y is an angle in either the lirst or fourth quadrant. It follows that

cosy 2: 0. and we must choose


[3Il1
fit

cqut~~icn

sin )

Triangle fo

(ii) We use the triangle i n Figure 3.48 tO replace the trigonometric idemity . The triangle

is designed to fit the equation x = sin y. The third side is then

,/1

xi. h follows

that

To cn:\urc that cosy is indeed J>OSiti vc, we rcsorl once Hgain to the r~lct thal
Sin - 1 ,r '"'d principal val ue$ arc - rr {2 ;S y ;S rr {2. Fi nal ly. the!\, we have

y =

dy

d,v
It is a straightforwtlrd application o f the chain rule to obtai n the following coro11ary.

COROLLARY 3 .13. 1

If u.(x) is a diffel'entiable func tion. then


d

tlx

Sin - 11

du.
= -;==;;
.J1- u2 tlx

(3.34)

,ext we OOtuin l hC llerivH(i\1e of the i nverse (."()~in e runclion.

T HEOREM 3 . 14

-Cos- x
dx

PRO OJ'

If we set y = Cos - x .then x = cosy. and we can differentiate with respect to .r:

m:J!IJ
tqu:~~1 ion .\ o (..V~ )'

dy
dx

-son y - ,

I =
Iii

- 1
= ._,!
-r;==~
1 xi

Triatl$)t lo

and ~ohe for

dy

-1

dx

sin y

n., triangle in Figure 3.49, obtained fro m X = COS)' , )'ields sin y


y

principal value:s of y
and therefore

Cos-

are 0 ~ )' ,::: 7T . it follows that sin

dy
=
dx

= ~. Since the

.v is i ndeed nonnegative.

- I

COROL LARY 3 .14. 1

Tf u(x) is differentiable, then


- 1
dll
d
-Cos- u J1 - u> dx
dx

Derivatives for the other i nverse trigononnetric functions Clm be derived inn simi lnr wuy.
\Ve l ist them below und i nclude uc.ivntivcs of Sin- u Ulld Cos - 1 u ror COnlJ>ICh:.nc...-.;s:

d
d;<

-Sin- u

dcos-u
.\'
d
- Tan- u
dx

dCot- u X

d
-Sec- u
tlx

du
J1- 11Zdx
-I

du

(U7h>

u! dx '

._/ 1 -

du
I + u2 dx

----- I

(3.37c l

du

-----;
1 + 11 2 dx

(3.37d)

du

(3.37e)

uJu!- 1 dx '

- I

dx

-esc-' tt -

(3.37,1)

uJu

du
-

(3.371)

1 dx

Note that 1he deri.vative of an i nverse cofunction is lhe negative of the deri va1ive of the cor-

respondjng inverse func Lion; that is, derivatives of Cos - 1 u. Cotnegati ves of the deri vati ves of Sin - I u, Tan - u, and sec - l u.

ct,

and Csc - 1 u are Lhe

I EXAMPLE 3 .31

Find dy fdx if)" is deftnoo as a function of x in each of the following:

(a)

(d)

y =

Sco- 1 (x2 )

(b)

Sin- x
Cos- x

(e)

Co~- t (xy)

-=-~
1

= 2x Cot- 1 (3x)

( c)

= [Sin- ' (.r')] 1

+ x1 y + 5 = 0

SOT.IffiON

Cos- 1 X

Scn-

1X

~(Cos - ' x) 2
(e) Differentiation with respect to x gives
- l

/ 1-xl yl

( y+x -dy) + 2xv+x 2 -=0.


dy

dx

dx

Thus,

,,

/l-

x l yl

- 2xy,

from which
y - 2xy j l - x'y2
- x + x 2 j 1 - x'1yl

I EXAMPLE 3 .32
Find the derivative of the fw>ction /(.r) = Sin- 1 x
SOLUTIO'I/

+ Cos- x. What is your conclusion?

The derivative of the function is

f'(x ) ~

J i -x>

-1

Ji-x2

= 0.

Si nce this rcsuh is valid for theemire domain -I ::: x ~ I of f(x ). it follows that the function
f (x) must be a constant,

f(x)
Sinoe /(0) = Sin- (0) Con.seq uently,

Cos- (0) =

a constant.

+ rr/2 =
1l

rr/ 2. it fullow that C

Sin - ' x + Cos- ' x = - .


2

= n/2.

EXERCISES 3.10

In Excn:lsc:; 1-30 y is dc.:fiucd us a function of .t. f'i.nd d y/tl:c in as


simplified 3 fotm as possible.
I

(2_t + 3)

2. y = C<>t- 1

<x' + 2)

I.

y =Cos

Z2. ,. -

Z3.

y =.rc.,.-'G) - ,~

24. )'

esc ' (3x)

J9x'- 1

= ---'-~

)' =esc -' (~- 4x)

3.

25. y = x' S.x

4. ,\' = Ton - o (2 - x ')


5.

~coc-'(~)~
3
xl

.'' = s._...,-o (3 -

x - / x' - I

26. )' = x (Co< - 0 x)'- 2x- 2i1 -x2

2x1)

27. y = (I

es--' (x' + 5)
(x' + 2) Sin I (2.t )

+ 9x') coo - (Jx) + J.r

6. y = '
7.

y=

8. y = Ton -

9.

* 28. Y = (x -

Jx + 2

30. y = Tan (

10. y=COI 'Jx'- 1

(Ton- (.r 2)j'

12. y

= x'S<X- x

13. y

= oan {3Sin - x)

Jlx )
,fi"+?

3l. Evaluate the dcriv:uhc of


[(.t)

14.

= S<X

'x + Coo

.r' - I.

)'=COl-I

15. .1

~ 2)

29. ,. = (2.r 2 - l ) Sin- 1 .r + .r/1 - .<2

.v = Sin- J I - x'

II . y

1h l 4x - x' + 4Sin- '(r

=Csc-

Whal is your CQncllbion?

32.

(( I + x)/( 1 - x)l
( I/ X)

16. y = Sin - 1(1 - .<)/(1

d
- S<X
t1.r

+ x)l

17. y = Tan- (11 2 + 1/ 11 ). " = tan (x2

18. y- 1 Co> - 1.
19. yz t:in.\'

20. Sin

+ y = Tan-

(xy)

J'i"'=X!
1 .\'

= jx + 2y

21. y=/x:-I-Scc- 1 x

{a) Show'""' if ""' principol volucs of f(x) = sec- X ere


chO!Cn as 0 ~ y ~ :r. )' =F JJ / 2. then the dcriv.ui.vc of
1he fwx:.tion is

+ 4)

(b) What is thederh'3tiveof .((X)= Csc-l X if itS principal


\'3luo;ore chosen as - :r /2 $ y $ :r / 2. )' # 0 ?

* 33.

Nndlheanf!lebc!tw~ntheC'ur\'es)"

= Sin- 1 x and y= Cos- x

ru. their point of illh.':rsettion.

* 34.
* 35.

Verify ahe re~ull!> in eqmuions 3.37c- f.


l ftheC1'3nk in E~cn..."ise 64 ofSeaion 1.8 ro1 a1e~ a1300 rpm. find

n pression for d O{dt.

13.11 Derivatives of the Exponential and Logarithm


Functions
Exponential functions obound in opplied mnthemotics, and much of this is due to the fact thot
ahc dcrivati\'C or an exponenLial function alwa)s rcwm~ the exponcmial function. \Vc obaain
derivaLives of exponential and logarithm functio ns in Lhis section, but many of the applic-aLions
that rely on these derivatives are delayed unti l Section 5.5. Exponential and logarithm functions
and their propenies were d iscussed in Sec1 ion 1.9. \Veassume that the reader is famj liar wi1h !his
material. To d iffereotiate logarithm and exponential functions. we must retum to the detloitioo

of Lhc derivative (see equation 3.3 in Section 3. 1). We begin with Lhe natural loga.rilhm function
lnx.

2 12

Ch:.p1er 3

Differenll:lli{MI

THEOREM 3 . 1 6

The derivati,e of the logarithm funcaion In x is


(/

=x

- ln x

dx

(.U SJ

PROOF By equation 3.3,


d
In (X+ lz)- lol.l'
In x = lim __;:;._.;_,.:...._ __;.;.
dx
1o-o
lz

=
=

' [ (x+h)]
x

li m -

In

/t....,.o IJ

(since In b - In c

--

In (b/ c)J

lim~ [ In (1 + ~)]
x

11-o h

~[In (1+ ~)
]
~\'

= .!_lim
x JJ-o h
I

= - lim In

.r

~o

I + ")x{lo
x

S ince the logarithm functio n is continuous, we may interchange the limit and logarith m opera
ti ons (see Theorem 2.5 in Sect ion 2Al ,

d
- l nx
dx
But if we set

lim ( 1 +
~-o

It- O

).r/lo].

v = x/ h. then

~)xflo =

= -I In [ lim ( I + -II

lim ( 1 +

hfr- o

~) flo =
x

lim ( 1 +

1/v-o

.v!.)"=

')"=

lim ( 1 +-

IJ-?O

(!

(sec expression 1.68). and therefore

- l n ~'t=

dx

The chain mle give-s the fo llowing corollary.

COROLLARY 3 .16.1

If 11(x) is a differentiable function, then

d
lnu
dx
-

I du

udx

(.D9)

3.11

Derivmi\'es of thc Ex.1>ooenliol11nd Log::uilhm Aar"M:Iions

213

In the rare ins1ance that we might need to differentiate the logarithm function to base a # e,
namely, log, x, we rewrite the change of base fommla 1.67 wi th a replacing b and e replacing

-".

los,, x = (In x) (log, e) .

Differentiation with respect to x gives the ncx.t corollary.

COROLLARY 3 .15. 2

-dx log,
_,

x = - log. e

(3.40)

The chain rule gives the next result.

COROLLARY 3 .15.3

If u(x) is a diO'ercntiable function of x. then


I du
d
log., u = - - log.e.
dx
udx

<lUl

We now use implicit diiTere.miation to fintl the derivatives of exponential fu nctiurt~.

THEOREM 3. 16

( 142)

PROOF

to x

u.~ing

If we scL y = t!'r , then x = In y, and we can diftCrcntiutc (implicitly) with respect


equation 3.39:
I dy
- y dx '

Thus.

-d y = y
dx

= e'

The expone-ntial fu nction c.x is 1hcrcforc its own derivative. In fact, for ru1y constanl C
\Vhatsoever. the function C ~-' d ifferemi;nes to g i ve i tsel f. I n Chap1er 5 we sh ll see that C ,-' is
the only function thot is its own dcrivntivc. The chain rule gives the dcrlvutivc of e" when u is
a funelion of x.
COROLLARY 3 .16.1
For a differentiable function u(x ) ,
(3 .4 ~)

214

Chaplt-r J

l.>lfftrMiation

We can find the t.lcrivati"c Qf ex poncntial functions a x with base-s other lhcn e by writing

.!!_n = .!!_ [eln <' J) =


d.r

We state

thi~

d.r

as a corollary.

COROLLARY 3.16.2

(3.-Ul

The chain rule now g.ives

COROLLARY 3.16.3
For

a differen1iable function u(x),


d "
dx

-(l

I EXAMPLE 3 .3 3

du
dx

= o"- Jnn.

Find dyf dx if y is defined as a func tion of x in each o f the following:

(d)

y = (In x )/.r

(b)

y = log 10(3x2

(c)

y =

+ 4)

(c)

y = xe- h

J I + e2

SOLL"TIO!\
(a) Usiog equation 3.45 yields

-dy =
dx

- d

2~ - (3x)ln2

dx

(31n2)2' ' .

(b) With equation 3.41 , we have


I
d
,
dy
d
(3x
2
dx = lr+4x

+ 4) log10 e

6x

3.r 2 + 4

logto<'

(c) The pnxluct rule and equation 3.43 give

ely =

dx

2xe-:~.x + .r2e- ' (- 2)

= 2x(l - x)e-2A .

(d) The quotient rule and equation 3.38 yield

dy

- =
clx

(e) With extended power mle 3 .2 1 and 3.43, we obtain

dy
dx

J .ll

I EXAMPLE

l)eliV.IIi\'CSC.lrlhe fu.ll()ncnti:.J

:~~11d

Loprilhm Functiont

216

3 .34

Firld values or.\' for which the fii'SI deriv:u ive of Ihe funct iOl f(.'()

= x 2 1n X

b equal tO 2eft),

and val ues o( x for which the second deri\'ottive is equal to 1..ero.

: :----/- - -1.5
--

-0.2
SOI.UTIO~ A graph of tht: function is shown in Figure 3.50. It suggests that the tangent li ne
is hori wnltd m tl v.:due of x nc-ur 1/2. To find it we sc.."t

0=

J ' (x )

Since the fu nction is undefined tll

2 1n x

2.t ln .r

x=

.r

CD =

.r(2 1n.r

1).

0. we mliSt ha\'C

= 0.

or

ln.r

Thus. x = e= I1.j{i.
The second derivative is equal to zero when
11
1

0=

f "(x )

(2 1nx

The ollly solurion of rhis cqucuio11 i.s .r

+I)+ .<(~)= 21nx + 3.

= e-\fl . T he gcomcu ic signi fi~\llCC of the point on the

graph at which /"(.<) = 0 will be discussed in Section 4.4. Can you see it?

rn Section J.2 we prt)\~ power rule 3.7 only in the cR..;e 1hat n is a nonncgati\'e integer. It is
now easy to pmve it for tllll'eal n . nt lcaq when .t > 0 . To do this we write X 11 i11 the fonn
x" = " 1' ' ('ee e<:juution 1.72a ). Fum ~to las 3.43 ond 3.38 give

.:!.....\"' = ~,. ,.__, = ~" ~~'~ " ~(n ln ..t) = x" (~) =

dx

dx

dx

A diSCUS..'iion or the power rule in the case that

ux"-

0 depe nds on the ,alue of, (see Exer-

cise 49).

I EXAMPLE 3 .36

IJ ai_
r resistance proportional to \'elocity and friction with the surface o n which 1he mass slides

are tnkcoointo account in the nmssspring system ofE.,ample 3.30. the clisplaccnocnt function is
of' the fonn

x(t)
from time t
tirne.

=0

= e - r/l [ _!_cos v0599r- .!.sin


10

v'i599t
]
+ - 1- ,
2
4000

when motion begins until the mass c.omes to rest for Lhe first time. Find this

UCUII;J!iii::&il

Disph.<'cmcnt of mass tuku~g tfrictioo and mr resistance into account

'
0.1

0.4
-0.1

SOLUTTO'\ A plot of x(t ) is shown in Figu re 3.51. bill it Ctlll only be u.<ed until lhe tangcou
l ine is hori?..ontul for the first lime. This is fo:r t ~ 0.1. To fi nd ic more ilccunttely, we $Ct the
velucil) e<1ual to 0.
I - of> [ I
0 :;; - -e - - cos
2
10

.J 1599t
2

I . ..:..J--,15,.-9'11]
_
- - ~m
8
2

Jl599 . ../imt
+ e- o/l [ ----sm
20

J1599
-- - cos -J 159W] .
16

O f t he infinity of possible solutio ns to this equation~ o nly the s mallest positive one is acceptable

here. To find it we simplify the equation to


0= - I e _ 'l2 [ (4+5 Jim
159\l)co;;
80
Th i~ cquutiun irnpli~::,

,J'I599t
2

15\19- S)sin .fi599t ] .


+ (4 .Ji5t)9
2

that

J l5991

t.'lll---

The snmllcst positive angle with tangem cquil1o (4

Jl599 t
= Tan-
2

(4
+
s-

+ 5J I599)/ (5 -

4 / 1599) is

5J I599 )
= 2.2205 r-ddians.
4/1599

Con.sequentJy, the time when the mass s tops moving to the left is
I =

Application Preview
Revisited

=
(2.2205)
v 1599

= 0. 111 s.

The comainer in Figure 3 .52 represems a chemicttl J"eC.lotor in which a chemic.:e:ll is either crec.Hed
o r hroken down. The clte1nical enter'$ in the fo nn of a ~ol ution at o ne concen u~uion and leaves
the r~cto r at a dif'fcrcrH cortccntration. We assume thac now races;,, and out arc chc same.
s uy q {..'l.Jbic metres per second. und thcre fon: the vo lume V of solution in the reactor remain.-;
constant. Let us assume that C; is the concentr.ttion in kilograms per cubic metre of th e chemical
enteri ng the reactor be.gion ing at time l = 0. aod there is i ni tially no che.n1ical i n the reactor.
As the chemica.! is created (or broken down) i n the reacto r. its concentr ation varies lhroughout
the reactor so that concentration depends not only o n time but also on position. This makes

the problem of predicting concentration o f chemical leaving the reactor f:u more diOkull than

we can handle at this time. Suppose we add t l mi xer that is so efficient that conccntnuion of
chemical i~ che same at e\'ery point in the reac tor. Concentration then depends only on lime t.
denoted by C(t) . and this is also the concentration m which chemicul lcavcs the reactor. Our
p1oblem then i' 10 find C(t ). Now. conccnlnnion of the chemical in the reactor is the amount
A(t ) of chemical divided by V. the volume of solution. We find it easier lo work with A(I)
than C(l) .

1--

1-

Thederivati'e d A/ dl is the nue of change or the amoum of chemical in the rcac1or at any
g:ive11 time. This musr be equal ro the r-ate at w hich chemical enters less the rate-at which it

leaves, plus (or ntinus) 1he rate at which it i created (broken down).

dA _
dt -

rate a1 which } _
chernical enters

rate a1 which }
chemica.llcavcs

rate at which chemical }


is created or broken down

The rate at which chemical enters is qC;. The rate at which it leaves is q(A/ V ). For muny
chemical reactions it has been shown experimentally that the rate at which chemical is being
created or broken down at any ghen time is proportional to how much chemical is present at
that time. 1'hus, the rate at which it is creaced or broken down is k.A~ where k is a cons1-an1
(k > 0 for chemicul fonnmion, k < 0 for c hemical breakdown). When we substitute these
expressions into the above cqutu jon. we obtain

dA
dt

qA
= qC; - V

+ kA

= q C; + ( k - Vq) A.

Thi~

is the cqtultion thnl the amou111 of chem ical A (I) must sa1isfy atony given lime: it is known
t QlUltion. A d ifferential equation i~ c\n eqwui011 that cont<1ins one or mo re
derivatives of a runction, and th<.: obje<.'ti\'c is t.o solve the cc;ttmtiun for fu nctions that satisfy the
C(IUation. For thc c<luntio n above~ we must find functions A(/ ) whose firM c.Jcrivtltivcs urcequ:l
to k- q JV times themselves plus acunstant qC; . Differential cquatiuns arc d i:;cu.scd in detail
in Section 5.S and Chnptcr IS.
But we also know lluu Ihe inilial amoull o f chemical in 1he renc1or b zero so lhut the
function A(l) must also :;atisfy the condi1ion A (0) 0. In ulhcr words. A(l) 11111 t >ati>fy
a~ a ditTerenHal

q)

-dA = ( k - - A + qC
dt

"

A(O)

= 0.

"

This is known as an initial~''alue problem . ft consists of the differential equation and an i nitial
comJitio n for the unknown fu1cc ion A (I) . \Ve s hell I llnd ahat 1here are m(lny fun<.t iOilS satisfyi'IS
the differential equatjon, but only one of them also satisfies the initiaJ condition.
Si.nce k - q f V is a constant. equation 3.43 indicates that the derivative of the function
e<k-q{ V)I is k - q I V times itself. Furthermore, if we multiply e<k-q{ V)' by :my constam D
whatsoever, derivmi,es of the functions De<k-q f VJl are k - q I V times themselves. This must

be a pan of A (t). The other pno is simple in fonn but harder to d.iscover. lf we express the
differential equation io the form
dA

dt -

(k- q JV) A

= q C;,

we need u fuoction !hut yields qC1 when substituted into the left side of the equation. The
constant function - qC; J(k - q / V) docs it. Putting th= together. we suspc<:-t that A (I ) must
be of the form
A(t)

Detk-qJ V)t -

qC,
k- q f V

It is straightforward to check that the deri,ative of this function does indeed satisfy the required
differential equation for any constant D . It remain$ only to choose D so that A (I ) satisfies the.
initial condition A(O) = 0. This requires

D=

qC;
k- q f V

Consequently, the amount of chemical exiting the reactor at any given time is

A(t) =

qC;

,p- qf V)r -

k - q/ 1'

.,-..:.
q_C-'-;,..,..,. = q C; V
k - q/V
q - kV

(I -

t?(k- qfV)t] .

The conccntn:uion of chemical c.l(itlllg the reactor is


C(l) =

q ci

q -kV

1 - e<t-q/ Vl<].

The analysis in the above Application Pcview Revisited will help us in our oext, more
challenging consult.ation .

Figure 3.53 is a schematic for a heat tank. Water enters at constant temperature LO"C. the
water is heated. and it then leaves at bigher temperature. The ca.n_k is perfec,ly i_nsuJated so
that no he-at can escape from its s ides. and therefore all heat supplied by the heater raises
the tempenuure of the \V',.Uer. \Vhen the tank is fuJI ~ and this is always the case, the mass
of water is 100 kg. Water eme1s the tank a t a rte of 3/100 kg!s. and leaves at the same
rate. The heater adds energy to the water at the rate of 2000 joules per second (J/s). We
<lre asked to determine whether the temperc:~tu re of the water i n the t:mk j ust kee~ rising

and rising, or whether it somehow levels off after a long time.

SOLUTIO!'\ Tempenlture of the water i1> the tank depends on both time and position
in the tank. To remove spatial dependence (olherwise the problem will be impossible to
solve), we add a mixer assumed so efficient that temperature of the water is the same at

every point in the tank. Temperature then d:epends only on time t . denoted by T (I) , and
this is also the temper.:mare at which water leaves the tan.k. _If we can tjnd a formula for

T(t). we will know what happens to tile system.

Sc:hc:m011til: for he-Jt rant

Coolliquid

Heater

l ,L...-__1----+---

I1

Perfec1lnl\ul:llion

Mixu

Where do we stan'? We wam tempenmre. temperature is due to heat and heat is a


fonn of cncrg~. We ha\'C encrg~ cmcring me lllllL in vatcc altemper.uure 10' . cne<g>
being added by the heater, energy raising the tempermure of the water, and energy leaving
in water at tempcrdture T(l). (For simplicity, we l'i ll inore energy associatcxl with
the mixer. h will ha\'c minimal effeC1 anyway.) Since we con calculate rates for these
energies. and the tank i perfectly in<ulatcd. we must be able to e<lltblish an energy
balance e(IUation for the system. We feel that it should take the f\>nn

ntc first three rutcs arc easily cah.:ulatcd; chc founh b more dillkult. The ;ccond term 011
the left is 2000. For me remaining tenns, WC mU>t introduce tJte 1p<Cifi< lfeat Cp == 4190
Jll:g C of wau:r. It i$1hc cncrg) required to raise the u:mperoture of I kg of water b) I ' C.
(To rnisc 5 kg of water IO' C, s.ay. requires (5)( 10)(4190) == 209 500 J .) Since 3/100 kg
of water at temperature Io>c enter me tank each ""cond, the rate at which cnerg)' enter..
the tan~ due to thb w.atcr is (3/100)(10)(4190) == 1257 Jls. (Thi.s is the cncrg) required
to rnbc 31100 kg orwutcr from o c IO IO"C.) Ira II si milar way, the rule at which crtcrgy
lcae> the cank inche wmer at tcmpenacurc T i (3/IOO)(T)(4 l90) = 1251T /1 0 Jfs.
Thislea\e< only the la.t term on the right <ide of the energy balance equauon. 1lle rate of
change of lempernmrcof the 100 kg of water in the mnk is tiT(dt; that i<,the cemperaiUre
chttngcs dT/ d I degr~s coch second. It follows tlu11 the rate ot which energy is used to
ruise the tcmpc.rature of this Wdtcr is (100)(4190)(tiT/ dl).
Wilen these 11lt"" are subl.titutcxl imo the encr~ balance equacioo. the ~~:>ull i<
12.57T
dT
1257 ... 2000 = - - - 41!1000-.
10
tit

Titis is another c.uunplc of a differential cquation; lind the function T (I) tluu satisfies the
equation. We also need to spectf) lhe inilial temperniUre of the water in the tank We are
told that "hen lhc hcaccr is turned oo. temperature of the water in the tank is the same as
the incomhtl! water, nmely 10 C. Let u~ choose thi< to be time 1 0. The initial-wtlue
problem lor T (I) is then

dT
3T
dt = - 10000

3257

+ 419000'

T(O)

==

10.

F_..qutiou 3 .43 $11ss,en-. ll"~<ll

tc)

*-"-"Oulll f<.w tl\e tetm -Jr / 10<l00 VI\ the: ri;,tu ~i-lk

O( I he cJifrere11tial ~(lua\i\)1), t,- 'N / I\1(\0(> ~hQU\d he itNOl VC<J, ll\1)4, the olcri\'Mi"'~ Qf \he
fun(ti(ln De- ) / IOOOU i5! -3{\Q U(X) t h)l(',<; !l,.....i:tl' fc1r nny (:(li\.>I.U\1\\ 0 Whiii\SC)I,WC.'r, This

mu.o;t be a t>lll C)f T(t) . l'hc otl\11,'1" pa n i$ t;inl\'le in f or m b ut hru'd(:r


expre.....;; the d i iT"n~'ICll\l eqm"i.c)n in the ron1\

tiT

dt +
v..~ n.;:..:d 11 1'111\Ction that y ields

tQ

d \lSCQvct. \(we

'JT
tOOOO

3257/4 19 000 when ' ubstituted il\l<l t~ l(:ft s.ide oft he e-qua.

tiOtl, T he consum t rvncll()n -

32S1
-

lOOOO

.:q 9000

,3

32Sf0

~ -

rJQe~ t. P uttu\1! t\\C:<Se t Qethcr

\251

we ~u~;-pCCl lhat tempcr11mre o l d..C water 111 the ltll\k n~ust be of the lonn

T(l ) -

Dc

-~I tO

:n 570

+ \251 .

IC is s tra.igtl.tl'()rward to chcc.'k thitt the deri,t~ tisc: ol' 1.!\t$ functiot\ does lt\dttll s.ati.${)' the
rcQuir.::d diffcrenth\1 equation fo any coi'IMiuH I), l\ l'ttnni n ~~> t)ll\ ~ to cl~e I) S<'lt\l11.t
T(l) h<!.lli sfi cs the it\hi!il c()l'ldition T (O) = \ 0 . This req~,~irc.-.
1 0~

32570
1257

0 +--

o ~

COelSC(Juemly. u~mJ)Crnturc of the w<\ttr " ' any time


D(l ) = 32 570 _
125'7

- 20 000
12S7 .

i.:o

~c.~-''' '"OOJ.
\ 25'7

n, e g,rtlph of 1' (t) is shown in Pigurc 3 .54. ll bcgin.s at T


asymptote. This L~ so bccau$e

'

= 10 a1'lld has a hor\70t,ta\

32 570
1257

lim 1 (l) = - - "' ~<>.

r-<JO

In other words. lhe lempc-nuurc of,hc wau:r docs no\ r\sc indctinite\y: it le\'Ch of{ at 26 1 C.

Oi!lllil
~0

25
20
1.1
10

s
5000

10000

15000

20000

EXERCI SES 3.11


~

In Exercises J- 30 y is defined as a function of x . Pind <l yf<lx i n as

3. y

s implified a fo rm as possible.

s. )' =

1. )'

= 32-A

2.

y=

In (3.< 2

+ I)

(, 1-~.l

log 10 (2x + I)

4. J =

Xl'2x

6. y= .rlnx

7. )' = t'~ IIU'

8. y

log 10 (3 - 4x)

9. ' - ln(ttnK)

10. y s ln (.'cos l )

II ,.'\ = -'ln(.t+ 1)

12. ) - X! - .XI~~

IJ. :f

14.

,. '

'
In (ln .l )

1! . '

Jl. l(hc.lt is1e ~~.dilJ aconSbmt r.atc in the~w irc o(Excrei ~


J t (p.::rha~ tleeau.se of e 'l.'t1fk curn:nt), 1.hc' d.itrcrutial eq11ation is
"'-pl>d by

> = Jin (t"2')

!!._

Jr

Cu) Verify that tl\c: fun~.: lion

r' - r '
,v ;;...__;_,.

7'(r)

c' + ,., -~
" 19. .v t - In .\'

= In (cos

11), ' '

.rt' + x
,l

Stn .t

i + JJ.

+ y tinx = 0
+ tan.r)

26. .r a ' l:;T+i'"- In('"" + '~


2?. ' -

In(.r ~

- AJJ' Ct + 5("'>

" 29.

(.r

Jll. r 1r

A(l ) -

+ ct

e '")
2.- .,

+ \ ').1

11

"

,\'

.\'

!!...
dr

.t),

(b) Plot A"(l). Whnt is the limit uf .l'(l) f'o r large t'! ht thi$
rcasorw:ble'!

34.

T lfl lhc .n~uJatwn is a functic...., or I' dli&l 11lUM llo~di~ot'y Ihe

c.fitYt.lcnl!.al ~uation

,\ )(1()

"tit' - (20

=- +-

J l. Tl1e flauc below 5how-s ,a rong cylindrical cahlc. CopJlCI wh'c


nu~doo n thoccncn:oflhccHhlesnd insulmionaWet"$ thc Wit~. II t
m:.a~urc~o rildt~l dr...tancc from the cenuc oi'Lilc cubic. then 'lc;d)...."l~lc:
h:mf)et>ih.lr\:

" 2H. " = '


r"

60( 1

\ 'ax+ x 1)

..&

tr tcnii~'1'Utu rc on i nner unll QUII.!I cd~'CS of the inw lation


= b are ec m~ UII"\ ~r,. and 7~. find c .tnd t/
inten n$Ofo . b. T~. 7t. nd k.
(a) TwosubR311CCS .A ncJ B rue. I tohwn1 1 third "'llb.!>l~c C
it~ ~Ius wuy ~~ I 1 of A n:acts wrth I J of R 10 produce
l&ofC. lfiOI:Of A ood 13acf 8 llcbroo,ht togelhcr
ucainl4! t = 0. the nurnbc' of vanK cl C in lhc mi.J.Il.ft $
3 funaion of time 1 is
r = n and I'

1 1ny

In (~A

k 1 t tln r I d

(b)

lJ, ln(..r y) : . l 1,-r


24
., 25.

satisties the diffCfc:ntiul cqultion fur IU1yt.:<'lr\~laJ\ts c 4ind tl .

20. '' ,1!1\tll (lrLA) t:ns(ln .r)j


2J. .\' t"lnt~, II = 1'1).
12. ''

4.

wn.....-c k !'> a COI!'I5tanL

17 .r In ( t' 1<'"')
IK.

(r dT)
=
dr

(rdT) = 0.

{:.) Two subst:mcc~ A tmd 0 rncllO Orn1 1 third substance C


in sudl a way d\Jl I g Of A tt1C" Wtth 2 &or 8 to produce
3 &of C. If 10 sofA ..nd 30 8 e>t fJ arc brouglttogctl>e<
at time I = 0. lhc nutnbcr of &f~nl< Q( C in the milUU"C at
a funaion of time 1 rs

Jr

.< (t ) =

T(r) =<lor+ J

')()(I - ~ "')

1r- "

-;~lt !flellh ediffcn:nt ial cqu;Jlcon for .anyccn~t.a,tt f :md d.

f t)) II IC'Itlf'ICttUutc on i nner and outer eclfJC:.fii


r - 0 und r a: IJ ate COflSUJ flL~ Tu O;Rd
illiC1111< u l a. b . Tu. und

rh.

or the it\~u l a li on
r,. find c and d

tl.\
tit

(.l0-.t)(4S- ,T).

(b) Plot ,t (/) . W~ot is the limit of .T (/) 1'0< larsc r Is this
l'c:l.liOII:!blc?

i.

35.

(a) What is Ihe <li ff4:rcn1inl Clllllllhm ~~l lli'lk:c l by lc nlJ)I..Yalutc


of lhe wak.T icl the tunk nr FiJt.uc J..S~"l if chc ra.lc a1 \\hich
W<~let enters and leaves the c.anl is 100/(t
I) kB)~>?

lns.uJ 3tion

(b) Assuming that tcmpcr.Uui'C o( ti'M: w:.acr in the tank is I0 C


al time t = 0. ,crify that
7 (1) =

41901

+ 4189 + I+ I
+ I)
4 19 .

419(/

1't01 a graph of T(t).

.16.

(a) \Vh.Jt t.$ the differential equ:thon:saaisfied by ttn~OJtutl: of


lho water on lho lank of Figure 3.53 irtbc amount ofenergy
>~~pphed b) tlle beater for lho first 10 '""' ., q
201 lf>.

of ~lution i ~ tCOlO\"ed from the Uanl.. cxh .)CC()n-.l h ~ i!SM~med that


the mh.cr is so c ffK.:ieOllh.Jtdlc c:onccntralJoo or wit i tbc sunc at u.ll
points in tile tan!.. and thi> 1> lh<: concentruon 01 .-hich solutioo leave>
the tank. If 5(r) represents tltc number of l.olognom> of Slltmlh<: tan!.

0:1:600?
(b) Aumina thottemperatureofthe water in the tonk i< IO' C
at lime t = 0. \'Crify t hat

tany given time. then dSjdr is the rate of ehange o( the amount of
salt in the tank. It must be equal to the rate ;u \\1tich salt enters less the
r:uc m which sa h Jcnvcs:

200r
I %2 2<JO
'( )
7 1=---

1257

3771

+ 2 000 000 e_3,1,0 wo .


3771

O<r<600.
dS

di =

Plot a graph of T(r).

i!

37.

(a, \V'h:u i~ the dtffc~nLia l equallon satisfied hy 1e1npcr:uure of


a.hc wa1cr in the; tank of Figure 3.53 i.fl.be tcmpcr-.uw-c of1hc
oncommg water is a functioo of time. T. = I~ '.I > 0~
lb) Auonin! ohat tcn>pcnoturc oflho "ollcrio lho tanl. i> 10 C
Jt tionc 1 = 0. \mfy that
T(r)

= 20000 _ 7~240000 ~- 31, 10 ~.,


1257

12566229

r;uc (II which

$ah 4.:11h.!l'b

lUI\~

I~~~

rate

IU

which }

ICU\'Q Uutl.

The rnre at " hich :.alt enter> the t:lttlos ( 1/ 20)(4) = 1/ 5 tg/s. The
rJtc at which"'" leave< i (S/ 10000)(4) S/2500 t g/s. Thus.

S(O)

JO , -.
9997

100.

Vcnfy that the solution of this doiYcrcntosl o:qooauon and initial condition

Pln4 a graph of T(r).


J8~

is

A ;)prin& wilh con~t.ant S :'Vm is auacbcd to u fh.ocJ -all on one end

;md a I k& mM~~o: m on the Olhcr (figure below). We chon!:c a C('l('lrdin211c

system with po>ithc to the right and wit!Lr 0 m tl~eecnuc ofma.s


of m when the spring is in the unstrctchcd position. At lime 1 0, m
is pulled I nllnlhclellofx Oandgivcna spcedo13ml<tolhcriJ!hl.

Plot iL~ groph. Is; it what you would C}CJ)C(l?

During the wbl)Ctjucnt lllOlion, a fr-i.ctiomtl fon:.e l'quul in nc*U.Hll) IU

"tn

(a) Show that if solutioo in J,xcrti<c ;N o< rr~KWOO al 2 LJs

twi lho ,.~l~ity of the mass ao;ts on m. It


be shown that if X(l)
;, the po<tinn of m .then r(r) mu<tsato<fy the dlllerentoat equation

rudoer than -1 11>. Uoc din1:rcouiul "'lUUiion fO<' 5 (1) i>

d'x
dx
+2- +Sx = 0
dr'
dt

dS
dr

and lho onol031 tondHiM< .r(O) = -I and .r'(O) = 3 \'mfy tluot


.r(r) = ~ '(.ln2r-cosU)s.ti>fitstheo:quationanchnotol<:onJotio1S.

= 5

2S
10000 + ~,

S(O)

= 100.

(b) Vorif)' ohat obe >lutooo i

5 (1} - SOO I

2
10

I(

10'

S000 +I

llfot w guph of S(r). OtiC~ il huvc Ull f:b)'mptocc:?

f----.;
.r 0
i

39. The hgure below shows a tank containing 10000 L of wate-r n

41.

(a) Sho'v thal il' solulion ln Bxerci..e 31:1 is remcl\'ed at 8 Us


rather th:m 4 Us. the dITeteltl i:ll equal ton for S(l) s

whi.:h IJ dic$olvcd tOO kg or saiL


-

\ti'ta'

~ 1./c

1-

41J>-

1-

t-

d5

dr

=S

S(O) - 100.

(b) Vaify that the solutton ''

L--

"""'

8S
10000-41

S(t)

3r
r'
100 + - - - -.
25
t$625

For how long is ll1c i'Ohllion vulid'? Plot a grat)h of S(t ).


Starting 01 lime 1 = 0. a solution oomaioillg I kg of :-a1l few cnch
20 1.. ol' SC)IuliOn is !lddcd to Lhe lank aL 4 Lis. Simultaneously. 4 l

(c) \Vh.at is t.he maximuru amounc nf snll i n the tank!

i ll 41.

(u) ,\ bc:cr \~II e<)fll;tin~ 2000 L or beer, 4~f. ol'which is1tlcohol.


6 r.cr wi dt8~ urconor i$ ndded 10 rhc ' 'a t m 2 I h. und wc lluim:d tni).rt~ it~: rcnlO\'Cd IW the .umc r.uc. Silo~~' 1/'Wt it
A(t) rcptaclll " the num ber ,,(liiJCS of .1lcohol in the \ UI,

,.. 4S. Verify thst the function

)' = /(X) =

;\.A'

+ I)

[ ,

1]
i In ( :r, ~ 1)
1 +

'"'"
=

JI (O)

SO.

(b) Verify llwl thc ~oJulicw~ of thilt diOCrcnrial equ.111ion is


t\(1)

160 -

8(le- l ll(t)),

(or :~ny

Plot agnljlf)Of 11(/).


tc:) Whc:11 io; lhe ~r in the val 5~ nlc:oh,>f?
f.

<'' -

U. 1\vo loo,;. pJIN:flcl rccutn.gul~r Joops lying in1hc ~unc: pl:ult hll\'C
lengths I and L ruKf width.; UJ nod IV. rc-.spoct'ciY (flgme belowJ. If
dtc JOOf~ d ,, 1101 O \Crlap. w,Lf dlC di~lwtCC between t:hc ncar t:ic.k..,_. i.o:: J.
the muw;~J incluc.iilncc bclwccn the loops i$

,,,, [
M ;

s + 1

2,. In .<(I + s :,. )

., 46. r'hc eqt~ioo x =


e r ~lc tinc&> y imr1li.;itly:as tl flmction of
.~ . FirlCI <
ly/d.x ;,, tcnm of x in two ways:
( it)

t()

j'(.t).

.x.

( :1) Show thai when :1 litJuid enter.; the lal'lk in Figure 3.53 <~I
til k'i log.runN IX.T toecOnd, lhc 1nad of liquid in 11~ tnnt j.(

.. 47.

M kik)grllrns. the tcm pcnuurc of liquid e ntering dtc 1nnl

;, To' C. the $pCCi(k; hc-.:n or(hc lit]Uid is c,). otrld the hCJICt
acids. energy at the ra1e of q j oules per $CCOnd, the di ffti'CO
tiul cquntiun go\'trning tcnlpcnttu.rc T (r) of l iquid i n t:lac

L3nk is

1/

dT
dt

;;; mc,1 + Mr,-.


(b) Verify that the sotutic:lll of this tMI'c.rcntinl oqmdion i:s

11

/-IV-I

T(l)

" 4-J. 'fwo paruJlel wil'\:s '-"aa'l'ying CUITc.'nt i (figure below) crc<:~k: 01. rnagJh.:lk fh;.ld. The ilu~ <P cf lt'u.') (ic.:ld thr'Otgh tbc loop of duncns:ron~ II
ami w i s
.,..
fl nlri [ R {r T
~ =--In
,
2;T
r( R + Ut)

/.lo > 0 is a con"t.anr. I)Dw:. gr.rph of 4>, lirbt

as~

func1ion of

= lo' '" 1"' + (r, ~ ..2_,.)


{I c,,m

e ""1")

wb..:n the ltmptmluJC or the liquid in lhc Ulnk i.s


I = 0. and 1ir. To. a"ld q are aJI OOOSIOill~.

tr)J

wft~I'C

Solve first ron1.1l explicit dttilliticn for y

Cb) Oilferemialc:: implicitly \\lilh n.:.,!)(<:t

""here p1, > 0 is a consuuu. S ho" th.tl the: derivnti\'C of Jll as H


ftmcrinn o( s is rtcg:u i ~~t: tbr 1 ;::.. 0.

JIJ-

con,.tunu A and B.

To nt time

48. Find tl: yfd.r 1 if y i'> defined implid1ly us a fun~tion of .x by

h and tlt<:n as t1. function of w.

-ror

.. 49.

-w-f

fl -

(a) The funclion f (.) = .t" is no I defined for .x < 0 if 11 is


i nut ion~rl, chat i !<>, i :ot n01 rJiionotl. Verify power rule 3.7 l'l)r
A' < 0 if n is r.Hionnl and x" is deli ned.
(b) Oi<euos <he dcriva~ive of /(.<)= x ut. = 0.

13.12 Logarithmic Differentiation


Compare 1he foiJowiog 1hree functioos:

.x". a~ . and x~.

T he firsL. i n which the expooem is

constant and the base is variable, is c-alled a power function. The second.. in which the base is
constam and th e exponent is variable. is called an exponential fu nction. We have d ifferenriation

rules for power functiotJS and exponential functions. namely


d .
X
- a'== a Ina,
and
dx

e~ponential

re.specaively. The third function x", which we conside.r only for x > 0, is neither a power
nor an
function; aherefore. we cannot use ei ther of the formu las above to find its

=x". and take tlaturallogarithJUS,

derivative. Instead. we sci y == .f(x)

In )'== xln x .

Implicit diiTerentiation with respect to x now gives

.!. d y

- Ill X

+ .r- -

y dx

In X

+ I.

Consequently,
dy

"j; =

y (Jnx

+ I)

or

..
Ulllecennauon.
This process of taking logari thms and then difrerentiatin_g is caJled fooarithmJ'c """
. .

I EXAMPLE 3.36

Find Ihe derivative of the function j(x) -_

x. ~ . ... when x > 0.

SOLUTIO!'\ If we set )' = x''"x, then


lny

Dilferentiation with respect to

== sin x ln x.

x gi,,cs

!. dy
y dx

-co
'-' Inx+ - si n x
X

'

. GX,
and this can be solved for d)/
I

dy

dx

Log:uithmic
di ~erenuauon
. .
.as
~.
"' the following
examplecan al s~o be used to dilfereOLi ate com I" . .

0"""""""-----------:qp::::::::::::~::~:=---------------------~p::'c"a"te:d"p"r:o:d uc:ts:o rcq:u:oti:e:nts,


EXAMPLE 3.37

Find I he derivative of the fuocuon


.

y = .f(x) = x l (x '

+ 1)'/J

. 3 .

on the iOLerval 0 <

Sin x
vaJues of X,
x < rr Extend the resu1t to other

3. 12

Log:uilhmk Oiffc:ccnt1al.k>IJ

225

SOI.l'110!1: When 0 < x < ;r , we la.ke namrallogarilhms of bolh sides or 1he definition
for y :

lny = 31nx

2
3 1n(x + 1) -

31n(sinx).

Differentiation with respec( to x gives


I dy

2x

3 :r +I

- + -....,--- - cosx .
2

-- =
ydx

sin x

and lhcrcfore

-dy
= I'
dx

[3+
-

4x
J(x'l + I)

3c01 r

\Vhen x is not in the interval 0 < .r < 1f , this derh-ation may not be ,alid. For instance,
when .t < 0. it is nOt acccpuablc to write hLf. and when x is in the interval rr < x < 21r,

Ihe 1erm In (sin x) is n01 dell ned. These dimcllllics are easily overcome b)' first laking absolulc
values,

lxl 3(x 2 + 1)213


IY =
I sinxl 3
Logarilluns now gh-e
2

ln lyl = 31nl.rl + :;-ln(x- + I) - 31n l sinx l.


>

To differentiate this equation~ we usc equation 3.13, which stntc,s that when f(x ) is diffcrcnliable. and f(x) ' 0.

~lf(x)l

dx

= l/(x)l / ' (x).


f(x )

\Vhen this is combined with fommla 3.39, \Y-e obtain

(3A6)

- In 1/(.r)l
dx
Applica1ion of 1his rcsuh 10 1hc equalion in In I.I'I gives
I dy

y dx

3
x

2 2x
3x2 + I

oos x
sin.r

= - + - - - - :)- -.

This equation i s idenLicaJ to thaLobtai ned for d y / dx when 0 < x <


shows that it is va lid even when x is not in the interval 0 < x < 1r.

1r.

buLiLS derivation here

EXERCISES 3.12

In Exerciges 1-24 usc logarithmic diO'erenliation lo fmd j'(x) .

I. f(.x) ~

2. f(x) ~ x 4"'" .

3. f(.r) ~ x'' ,

4. f(x ) ~ (<inx)',

x- . x > 0

19. f(.r )

x >0

X> O

8. f(x ) (:;)"'

x>O

f (x)

= :;:

= (si n.r)~" .

10. f(x) = (lnx)'" .


II. f (x)

+ 12.

* 24_ f(x ) =

with respcclto X.

26. tr ~ compwny scUs


demands

0 <X < 1T

sinx

c.cnuin coom1odily

1:1

x > I

.t'

ttl

price r, the market

r " e - b(r1r

itcllls per week. y.,Jxre r > a (b. and a . b. and cart: posjti\c constants .

+ lx')
J 1+ .r '

f (x) = x' ''

siu l.\ sccSx


( I - 2C<>IX)'

25. If u(x) is positive for :til x, find a fonnuJa (or the derivative of ,u

(.r ' + 3x 4)'(x1 + S) 4

13. f(x) = - ~ 1

* IS.

sin3 3x
23. f(x ) = - -

./i'( l

/(x) =

lan5 2x

x >O

* 9.

~
"'I
x l

x >O

f(x)= (l+;)''
/(.<')

,fi( J - x')

0 < x < rr

cr

* 7.

=(x' + 3)'(x' - 2x)

20. f(x)=

x >0

. s. f<x>-(~ +;r
* 6.

~
18. f(x) = :--:-'---,:ln(.r - I)

17. f(x) ~ ..:1 ln.<

(a) Show that the demand lncrcascs a.s lhc price d<x:reuscs.
(b) Cal<:ulalo lho c!astici1y of demand defined by
+ 14. f (x)

= (x' + 3x)

= x'''t - ' '

16. f(x)

( . '

+ 5) 4

x dr

(;'r

13.1 3 Derivatives of the Hyperbolic Functions


Since 1he hyperbolic ~ine and cosine func1ion.s are defined in tem1s of1he expone ntial ti.mction.

for which we know the derivative. and the remaining bypetbolic funclions are defined in terms
of the hyperbolic sine and cosine, it follows Ihat calculation of the derh 1ali\"e,c;; of Lhe hyperbolic
func1ions should be suaightforward. Indeed, if u (x) is a diffcrcmiablc fu nction of x, I hen

dx

dx

dx

dx

sinh u

du

= coshu - .
du

coshu = sirlhu -

dx

du

cothu

= -<-sch 2u

sech 1 u -

tanhu

d
- sech u
dx

(H7a)

dx

dx

(3A7b)

du

dx

(3A7c)

OA7dl

du

- sec.h u Lanh u -

dx'

d
du
- cschu = - oeseh u cothu - .
dx
dx

(3.47e)

(3.471)

3. 13

I EXAMPLE

Deri\'il.tives of d.e Hypei'I)Oiic Fu,).;:tions

227

3.38

Find

dyfdx

if y is dell ned as a function of x by:

(a) y = = h (3x 2)
(c) y = cos 2x sinh 2x

(b) y = tanh (I - 4x}


(d) y = cosh (Tao - I x 2)

SOLUTION

dy

(a) = - sech (3x ) tanh (3r) - (3r) = -6.~ sech (Jx-) tanh (3x )
dx
dx

dv

(b) - = sech 2 (1 - 4x) - (l - 4x) = - 4sech 2 (1 - 4x)


dx
dx

dy

(c) - - = cos 2x(2cosh 2x) + ( - 2sin 2x) sinh 2x


tx
1
= 2(cos 2x cosh 2x - sin 2x sinh 2x)

dy

(d) _ .
dx

I EXAMPLE 3.39
ljtUII;IH4:W
hanging cable

Shape of

When a uniform cable is hung be1wee11 two tixed suppol'ls (Figure 3.55). the shape ofthe curve
y = f (x) must satisfy the d ifferential equalion

y=

pg

dx 2

I +

(ely)'.
dx

where p is the mass per unit length of the cable, g > 0 is the acceleration d ue 10 gravity, and
H > 0 is the tensioo i o the cable at its lowest poilU. Verify that a solution of the equation is

) + C,
y = f(x) = -H cosh (pg.~
pg
H

where C is a constant.

SOLUTION

In Exercises 42 o f Section 8.4 we derive this solution. For now we simply wish

to veri fy th at the hyperbo l ic cosine is indeed a sol ution. T he first derivative o f the functi on is

dy
. ( pgx)
dx = smh H '
and therefore its second derivative is

~ cosh ( p~~ ).

On the other hand.

pg
H

pg
H

1 +s1nh2

(pgx)
H

pg
H

cosh 2

(pgx
H )

pg

= Hcosh

(pgx
H.)

Thus, the function y = [ H j(pg)] cosh (pgx / H )+C does indeed satisfy the given diiTerential
equation.

Example 3.39 shows that the rnany telephone and hydro wires crisscrossing the coumry hang i n

the form of hyperbolic cosines. Engineers often call this curve a ca/eiUll)'.

I E XAMPLE 3 .40
In stutl~ing ''''" ~uidc"' the clcettical engineer ofien encount<n.lhc differential equation

d!y
- k)

dx2

"here k > 0 is a constant. Verify that y


for any constants A and 8 .

o.

= f (x ) = A ~'06h lfx + 8 inh lkx i> a :.olution

SOLUTION 1nc first tlcrivatiV~: or y = /(.r) L<

dy =

d.r

..Jk A ~inh If x + If 8 co-.h If ~,

and therefore

Thu..

EXERCI SES 3 . 13

In E:~;crci\C.S 1- 10 J is defined as a fun ction


siq>ltllcd 01 f<trm ttS possible.

or'.

Find dyjdx in II!>

I. ' ' ~ 0<:11 (ll: + 3)

\'I'IL'tC

JI

,.=tanh(ln.r)

S.

co~h(\ ~

is a

COI1:'1.3nt.

.\ = /(x) = Aeosh + R -..nh + CcO<hh + D sinhbfer :m) constlnl A 8 . C. :Jnd D .


(bl If the l<fl md <
0) ;, f1N<r<d hctiiOOitlly and the
right cn.l (X = I.) " p!nncd, then { (<) n1USI <>ti <f) the

S<>:tu

~.

(I) Vcrif!t' thai a ~lu lio n ill

l. ,. = x Stnh ( t / 2)
l . y ,.

k>

cor.diticn.~

J<<J>= f'(o>

J ) l.r

= Jn> = I'<LI = o.

Show lh.u these c:~lr~uon.~ imply tha1 C = -A .


D = - 8 . Md A and 8 mu$1 stHi~l'y the equal ions

6. )' + coth ,\ - .,!'~+";


7. ~ :;; u cmh u , u ~c..:+ e-"

A(coskl. -c<>Sh kL) + O(in kt -inhkL)

M. )' l.ln(cc,.h t), 1 cos (tonh.x)

A(cod l. ~ coshk/.)

9. )' = T;m

(c) Elimil\illC A ond R bct"n tl1c._<e "'1\lillicns to show


k. must l:lli!il)t he condilioo

("nh )

10. \' In JLJJ~\ll:

+ O(sinkL

tank L

II . \Uif~ the dlrcn:ntiation f<mnolas 3 47.

12. Tn analy1c \'Cftlcal \lbrations or the beam in the figure below, \\C
mufit w.h,.c 1hc differential equation

d'y

- k
dx"

= 0

'

UJ\h k L.

= 0.

t <inhk/- ) 0.

that

+ IJ. Each hyperbolic run<:lion h~s ;.ssocillU.:d wilh il ~\f'l invers<: hyper
bolic funccton. <Sec E.'<ereise JJ i n Scccton I. JO.) Obtain the following
dc ri vath.~ of these functions:

"

-Tw1h- 1.\'

tl.:c

d
- Coth- 1 X =

d.<
d

Sinh- 1 ;r

= ....,.,..,.

-Cosh- oX
d ;r

= -,."!!"'-

dx

-S~ch- 1 x ::;;

dx

l - x1'

1 - ;r> '

(.\' ( < 1:
I.< I > I ;

-t

- ===

x~~

J .r2- 1

13.14 Rolle's Theorem and the Mean Value Theorems


Certain re._(;.ult ~ in calculus are immediately seen to be imponam. For example. the power. procJ
uct. and quotic::11 1ru h:8 th:u c:l imi nate Ihi! nt:ce:.sity of using equation 3.3 to cak ulutc dcrivati\'cS
arc clearly indispensable. Even the algcbrllic an~ geometric ioncrprctations or the derivative
iL<elf are recognized as useful. Through Vllrious oxnrnples und cxcrci~es oftltis chapter. we hn\C
hi nted at 1he vn.ricty and qLumti ty of applications of the dcrivati\'C. These will be de4111 with
al length in ChHpter 4. Other results in cnlcul us. especially those of n rheoretical nature, urc
regarded as less imponant, or even unimport~llll , clften becaul\e it i~ not obvious how they will
be used. In this :-;e<.:tion \\C consider three very ill'IJX)rtant theorems, without which we woukl
encounter seri ou~ difficuhy in 1ra1ting muny of the topic..'\ in the remainder of this book. The
first theorem is needed to prove the second. and the sccoold leads immediately to the third .

TH EORE M 3. 1 7

(Rolla's Theorem )

Suppose n runction f(x) satislics the following three properties:


I. f(.r) is continuous ror a ~ .r
2. f ' (x ) exislS for a < x < b:
3. f(a) = .f(b) .

b:

Thcn, thcrecxiSis at leastonc point c in the open interval ll < c < bat which f' (c) = 0.

~l!lll

Drn3

For the function in Figure 3.56 thcrc nrc two possible choices for c. Geometrically. Rolle'>
theorem ~eem~ quite evident. Begin with two I>Oints, say P and Q in Figure 3.57, which have
the smnc y -coordinatc. Now try to join these points by a curve thlll ne,er htls u horiz<>ntal
tange1lt line while satisfyiilg the following two conditions:

Roti<'s

tl ~rc: m

)'

f (a) ; j'(b )

/Ct~J ~/fb)
I

Uf!-"'--'rn""
..._.....Yd

Two level

puintt. for Rtlif)e's tl~.n'


)'

P.

.Q
;r

(a) Do not li ft the pencil from the page - continuity of f (,t ) :


(b) TilC curve must htiVe u tangem line tal a1ll points. and this tangent line must nol be
Yertical - J'(;r) exi'ts at all points.
It is im1><>ssiblc: tht.Ycforc. n point c where J'(c ) = 0 must exist.
To verify this theorem directly requires Theorem 4.2 in Section 4.7. Since the latter result
is quoted without proof. it :seen' as reasonable to accepLRolle's theorem on obvious geometric
grounU.s as to base a proof on a theorem which itself is stated without proof. But for those who
would like to see a pi'Oof based on Theorem 4.2 in Section 4. 7. see Exercise I0 in that section.
Like the intermediate value theorem in Sectioo 1.11. Rolle's 1heorem is an existence theorem; it stipulates the ex istcm.-e of c, but docs not provide a way to find it.
Rolle's theorem can be used to prove the following result.

230

Chi11Jit1 3

Uifrerentiation

THEOREM 3. 18 (Cauc hy's G eneral ized M e a n V a lue T heorem)

Suppose funttiun> j(x ) and g(x) satisfy the following three properties:

1. j(x) and g(x) are continuous for ll!: x!: h ;


2. J'(x) and g'(x) exist for a < .< < b:

3. g' (x)

f' 0 fora< x

< b.

Then, there e xists at least o ne point c in the open inte" 'lll a <

[(b) - f(a)

:....:..,.;'----'~

g(b) - g(a )

c < b for which

/' (c)

(3.48)

g'(c)

I'ROO I' First n01e t hat g(b)- g(a) caoul()(cqual zero. Ifit did. then g(a) would cq1~11 g(b),
and Rolle's theorem applied to g (x ) on the interval a
x :S b would imply the existence
o f a point c m which g'(c) = 0. contrary to the given assumption. To prove the theorem. we
cons1ruct a functio n h (x) 10 sat isfy the conditions of Ro lle's theorem. Spccificully. "c consider

:=

h (x)

j(b)- f(a)
)
) [g(x) - g(a)) .
;:(b - ;:(a

= f (x) - ,((a ) -

Since .f'(x) and g(x) arc continuous fo r 11 :::; x S b, so too is h(x ). In addi1ion,

IJ '(.r)

= /'(x)-

f(b) - .f(a} '(,r);


g(b) - g(a) g

therefore. h' (x) ex ists fo r 11 < x < b . F inally. since h (a)


lr(b)
0. we may conc lude
from Roll<'s th<l>r<m lhat lhere ex isis a numbe r c such 1ha1 11 < c < b , and

= it'(c) = f'(c) -

.f(b) - f (a) 11' <c) ;


g(b) - g (cr)

that is.

j"(c)

ll'(c)

j (b) - j(a)

g(b ) - /I(CI)

1llc next theorem slmes an important special case ot'lhis result thai occurs when g(x) = x .

THEOREM 3.19

(Mean Value Theo rem )

Suppose a function j(x) satisties the foiJowing two properties:

j(x ) is conti nuous fol' cr !: x ::; b ;


2. f' (x ) exists fol' a < x < b:
I.

T hen, there cxisLS at leasl one point

c in Lhe open intcrvaJ a < c < b

j '(c)

j(b) - j(a)

b - 11

for which

(.1.49)

3. 14
[i1

Marl v.alue

diC'Or~m

l b,Jib) )

Rolle'$ 111-eurcm and tht Mc.(ul Value The<ou~

231

From a geometric point <>f' view equation 3.49 seems as obvious as Rolle's thcon:m. Figure
3.58 illusrrme<lhm the qu01 ieru If(b) - f (a) J/(b - a) is the slope or the line I joining the
points (a, f (a )) and (b . .f(b)) on the grnph y = f(x} . The mean value theorem states thm
there i~ ru least one point c hetween a and bat which 1he tangent line is parallel to l. In Figure
3.58 there are clearly two such 1>oinrs. Algebraically, equation 3.49 states thm at some point
between a and b. the instantaneous rate of change or the function f (x) is equal to its average
ra(e of change over the intenal a .:::; x ~ b. Similar interpretations of Theorem 3.1 8 are gi\'Cil
in Section 9. 1.

a r

I EXAMPLE 3 .41
Find all va lue.< or c satis fying the mean value thCOI'Cil>for ~>c function f(x}
inter\'al -I :;: x :;: 3.

= . l -

4.< 011 the

Si nce .f(x} is dillerentiHble. wld therefore cominuous, at each point in


we can indeed apply the mean value lheorcm and claim I he cxisleiK"C of at
least one number c in - I < .t <: 3 such that

SOLlJI IO'I
- I ~

x ::; 3)

/'(c)

= } (3) - f <-I};
3 - (-I}

that is.

3?- -1 =

15 - 3
4

3.

Consequently. c = .J'fl'j. Since - ..trlJ <: -I . c = .,J7T'J is the only value of c in the
interval -I <: x <: 3 (~ec Figure 3.59).
Mean \uluc lh(()tem fo1 .\':t - 4X on - I ..:;; X <:: J
)'

(3, t5)

I EXAMPLE 3.42
A traffic plane (Figure 3.60) measures the time that it takes a car to travel between points A and
B as 15 s. and radios this infonnation to a patrol car. What is the maximum speed m which the
police officer can claim that 1he car was travelling between A and 8 '!

SOLUTION The average speed of the car between A and B is 500/ 15 = 100/3 m/s. According Lo 1he mean vaJue 1heorem. the instanlaneous speed of 1he ca.r must aJso have been
100/3 m/s at least once. This is the max imum speed attributable to the car between A and B. lt
may have bee.n travelling much faster at some points, but from the infonnation give.n. no speed

greater than 100/ 3 m/s can be claimed by the ollicer.

232

Chapter J

t>in't.renei.atiun

Tr:rffi: pl:ant nle2'\unng speed of car

,,
1 - - - - - - - 500 m - - - - - - -

EXERCISES 3. 14

in Exercises 15- 18 decide w~lher CAuchy'& gcocntli:ood n-w::an vulue


thc."'R:m can be applic.:d lu th1:. function~ on the: intcn"d. Ir it cannut.

In Exc.n:.ises I- I4 doe:idc " 'hcther the mean value theorem c.an ~ Bp


plied to the fwtction ou thc intc.:n..dl. II it cannot. CAplain why nut. If il
can. find all values of c in the inten'3J thai s.a1isfy equ~uion 3.49.

explain why n()t If it can. find an \o3lues or c i n the inten'al l hal ~lisfy
cqo:uion 3.48.

= x' + .lx,

I. /(x)

-35 x5 l

2. f(x ) = 4 + 3x - 2x 1.

* 3.

f(x) = X+ 5.

1 1

.. 4. f()=

~ IS. f(x )

25 X5 3

+ 18. f(x) = xf(x

*
-35 x 5 2

x' + 2x 1 - ' - 2.

-1 5 x :;; 2

7. f(.r)

g. f(x)

= (x + 2)/(x -

9. f(x)

1),

+ 1)/ (x + 2) .

1(1. j(.<) ~ x'/ (X + .l).


II. J(.<) =sin.<.

ln ( 2x+ 1).

",

14. f (x ) = sccx,

+ 1).

g(x) = .<f (x - I).

-35 x5 - 2

- I

Jf'(x)J 5 M on a 5 x 5 b, ohcn

quadratic function j(x) =: d .v: +~.'(


whotsoc'"' is c ~ (a + b)/2.

2 5 x 54

+8 on an)t intCC\tal a !::: .\' !::: b

21. Usc the mean .....due theorem to show that

- 3 5 x 52

lsina - >inbl ::; 111 -

bl

r,wallrcal a and b. Is the same: incquahty \'alid rorthecO'inc runction?

* 22.

Lei f(.r) ond $(X) be IWO funcliO<IS lh:ll are diffcrenlioble :II e\Ch

poinl or lhc inlervnl a 5 X 5 b. Prove lh>l ir f(a) = g(a) and


f(b) c g(b) . then \ltCI\! cxi.su C in lhc Open inlcrval a < X < b roc
which f'(c)
g'(c).

0 :;;x :;; 1

~how ohat ir

* 20 . Show that the \'"Jiuc c that satisfies the ltlC31l '-aluc theorem for an)'

-2 5 X 5 .l

19.

1/(b)- f(a) J 5 M (b- a).

0 5 x 5 2:r

+ 12. f (.r)
13. /(X)

I 5 X 52

- 15 .< 51

= JxJ, 0 5 x 5 I
f(x) = x> + 2x' - x - 2.

(x

S(X) =X.

16. /(.<) = X+ I. g(.C) = J.cJ31' . - 1 < ,\ 5 I


17. f(x) ~ x 1 + 3x- I , $(X) ~ x> + 5.< + 4, 0 5 X 52

I 5 x :: 3

"' 5. f (x)
6.

= x',

23. Vcriry 1ha1 fO< a c u bic polynomial /(.t)


tlxl -rex'+ gx + h
defined on !ln)' inter\l':ll a ~ x .:5 b. the value~ of r 1hiu :tlidytqu:.tion
3.49 ore cquidisl4nl from x ~ - c/(Jd).

0::; x::; rr

SUMMARY

l.n Lhis chaple r we defined the derivaLive of a function y

=f

(x ) as

(x_ +
.:........,
h >:....-__::_
I _,_
<-..:.
)
d y = f ' (x ) = li m -::..f_,_

dx

~t-o

Summary

233

Algebraically, it is the instantaneous rale of change of )' with respect tO x; geomcrically, it


is the slope of the tangent line to the graph of f (x). To eliminate the necessity of us ins this
definition over and O\'cr again. \VC deri,ed the sum. product~ quOLient, and power n1lcs:

d
(u
dx

du

d
- (u v)

dx

dv

+ v) = -dx + dx
- ,

dv

du

dx

dx

= u- + v-

du
du
v- -u (':) = d.<
dx .

dx

v2

d
du
- {u") = nu"- 1- .
dx
dx
These four simple rules arc fundamental to all calculus.
When a function y = .f(x) is defined innplicitl)' by wmeequation F(x. y ) = 0. we use
implicit diffcrenciatiol'~ to find ics deriv~.ui,e. \Ve differen1ia1e each tenn in the equation wirh
respect to x. and then solve the rcsuhins cquat ion for dy ( dx. We pointed o ut that care must be
taken in d il'feremiati ng equations. An equatio n e<nt be differentiated with respect to a variable
only if it is valid for a continuous range of values of lhat variable.
Tite chain rule defines the derivative of a composite functi<>n y = f(g (x)) as the pt'Oduct
of the derivatives of y
f(u) and 11 g(x):

dy

dx

dy dll

dtidx'

These rules and techniques fonn the basis for the rest of differential calculus. Wlten they
are combined wilh che derivati\'CS of 1he trigonomcaric. inverse lrigonomcaric. e.xponemia1.
logarithm, and hypcmolic functions, weare well prepared to handle thoscopplieations ofcalculus
thai involve differemiation. Derivati ve fornluJias for these trancendemal functions are lisced
below:

du

d
- Sin II= COS I/ - ,
x
dx
tl
du
- tanu = sec2 u - ,

dx
d

dx

secu

tf
tfx

dtl

dx

Ida
=- log, e.
u tlx

du

csc11

dx
, du

= -

CSC"'II-,

=-

cscucotu - .

dx

dll

dx

d
ldtt
lnu = - -.
dx
u dx

d u
.,du
- a =a - Ina..
dx
dx
I

COS/I= -SlllU -

-COlli

= secu tanu -dx .

dx

d.r

d
los. u
dx -

du

- e = e11 - .

dx

du

dx

- 1

drt

-Sin- 1 u =
.
dx
~dx

-Cos-t 11 =
.
d.r
~ tf.r '

-Tan- 1 a = - - - .
dx
l + u'dx

tf

-cot- 1 u

d
- Sec - 11 =
dX

dx

du

-===
It

du

Jii2"== d X

d
du
- sinh u = coshu - .
dx
dx

d
dx
d

- 1 du
= ---.
I + u' d.t
- 1
du
Csc- t u =

u Jii2"== dx

d
du
- coshu = sinhu - .
dx
dx

234

Oup~er

3 Oil'feren1i:t1ioo

t1
, du
- co1h u = - csch u - .
dx
d,r

d
tlu
-1anhu = sech u - .
dx
dx
d

dx

scch 11

d
dll
- cschu = - cschucothu d
- .

du

= - scchu mnhu - ,

ch

tlx

Velocity and accelennion have finally been given fonnal definitions. Velocity i~ the deriv<l
tivc o f displacemen4 and acceleration is the de rivative of velocity, or the second derivative of
displacement:
v(t ) =

dx

fl(l)

dt '

du

tit - dt 2 .

We comple1cd 1hc chap1cr by using Rolle's 1hcorcm 10 prove 1wo mean value !heorcnJS.
When / (x} and g(x) are con1inuous for a :;::_ x :;::_ b and differcn1iablc for a < ,, < b,
Cauchy's gem:.rdlizcd mean value theorem guarantees the c.~istencc of at lc:-asLone: IJ(Ji nt c
be1ween tl and IJ ~uch 1ha1

['(c)
/(b) - /(a)
=
g'(c),
J:(b) - J:(a)
provided also !hal g'(.r)
1hc mean value lhcon:m

0 for 11 < x <b. When g(x) = x. we oblain as a special c>~se,

/'(c)

/(b) - j(a)

"'

b-a

KEY TERMS

To reviewing this chapter. you should be able to ddine ot disc~-. the following key tcm1:s:
Oi;,placcmenl function

Increment
Avcru,gc rate of change
Derivative
Tangen! line
Powe.- rule
Right-hand derivative
Differentiable
Onhogonal curves
Quo1icn1 rule
1l1ird derivative
Speed
Chain ru le

Explicil definition of a func1ion


lmpl icit di rrerentiation
Differen[ial e-quation
Eoergy balance equal i.on
Ex ponenLial function
Rolle's theorem

Mean value theorem

Average vcloci1y
lllSiantaneous velocity
Instantaneous rmc of chungc
DiiTeremialiOn
Deri"rative function
Normal Hne
Left-hand derivative
Angle between intersecting curves
Product mle
Second derivative
Vclocily
Instantaneous accclerntio
Ex1ended power mle
Jmplici1 dcfini1ion of a function
Simple hannonic m01ion
lnitial-\-alue problem
Power function
Logarithmic differeotiation

Chaucys generalized mean value theorem

R EVIEW
EXERCISES

I. r= .r ' + -

J. \';; 2 t - - 2 +
3x
JX

4.

5. .1' = x (x' + 5)4

6.

3.r'
7. Y = - -

s.

.r' - 5

9. r=
II.

x' -r 2x + 2

3)' sin x

= 14

IS. y = ' (:l.r + 2)

17. )'=
1 ~.

)' =

sin !x

CO!:)X
2

21. y = u 2 - 211,

- X

.v =

46.

- = x-y
-x+y

3x - 2

y =-.t +5

14.

4x

x 2 + 5x- 2
)'

/4

)' =

41.

oiS. x =

49. y =

+I .

'' -" + y./l+x =

y3 + 4y

x' - 6x'

)' =

18. )' = S: (lllll 2.1')

51.

)' = x l'

)' = ~ i n- X cut X

= {1 + 2x).)13

(eosx)"~

= ttm.r

+6

+ 12x +4

JX- ./2
52. y

,..

0 < x < n/ 2

53. r= ~),+I
X

55. y

" 57.

22. y=t + cos 21> t=.l. - cos 2.T

;(

x- 2

so.

x+5

.t 2 - 4..\'

16. )' = ,..;2 (I - 4,T)

- - ' --

y' - 2y

+ - =x

'

\ 4- I

x .,fi'=X

44 .

y = (.r ' + 2)'(x3 - 3) 3

20.

.t COSXZ

2 sr
; .J

x JJ -

X
12. -_,.

X)'+ 3)'1 = X+ I
-

)' =

10. y =

.tl - 2x - I

13. x 2y 2

1/ 10

4t. x=/ l +xc.ot,r2

2. r = 3.<"' + 2x + -I

.o:'

+ y )) -

40. Ln (Tan - 1 (x

In EtcrciS('S 1-58 :u .sufnc that y is d<:.Jlacd as a fwlclion of .t and lind


dyj dx in as sin1pli fied a focm as possible.

= e'

lnx

xyt"'') =

58. x ' )' +In (x

+ y} = .t

~ 2

In Exercises 59-62 find equations ror the tangent and normal lines

(O

Lhe cune aa the point il\l.lic.ated.


24. )' =

II CO$! tJ,

11=

25. .r = / 1+../l+x
2xy

27.

- - = .c+2
3.< + 4

2~.

x sin y + 2xy

31. y

33. y =
35.

=4

/ 1.- x'
26.

X =

59. y = x~ + 3x - 2 a1 ( L, 2}

60. .\'

= X~
-I-5 al (0, 1/5)

61. .1' = cos 2x at (JT/ 2. -I)

62. y

= x'2.1'+- 3x5 at (J. -4/3}

r.2).

28. y = (.< 2 + l)ln (x' - Li


30. 5 cos (.r - )') = I

ln Ex<:rciscs 63-65 fi nd d 1 ) / d:t.2 as:suming that)' i_s defined impl.ic:itly

as a function of x .

x' -

y' + 2(x - y) = y3

s;, - l (2 - 3x)

63.

Coo- l X

64. (x - y) 2 = 3x,v

Sin - 1 x

" 65. sin (x

)' = ccosh .l

37. sec- ' (x

+ y) =

36. sinh y
xy

38.

y =

= sin x

x Csc-'(~)
x-

+ }') = x

66. Dn\w Ll gruph of f (x)

* 67.

= s in x 2 Is il periodic'!

Find aJI points o n lhc curve y =


passes 1Ju ough the origin,

x 3 +x 1 at which the tangcnl linc

236

Chap;ter 3 Differentiation

6H. Fmd that point on the curve x = ,v2 - 4 at '~'hich the nonnal line
passes 'throt~.gh the point (-6, 7). \Vhal appUcati.on cmdd be made of
this :resu.]t?
(a) How many fum.::tions with domain - [ < x < [ are defined
69.
immplicitly-by the eqtwtion x 2 + y 2 (b) How m-."Lny C<Jnti.nuolJS tiu.nct~ons with domain - 1 < x < 1
are defined hnplidtly by tbe. eq~ation?

70.

7I ~ In a heated house, the temperature varies as d!Je therr:~~n.ostat conti.ne


m.tiJy e n.gages. and d]seng()ge'S the fum()Ce. Suppose Uilat at ~he ~hermo~
stat,. tlhe ten1pe:ra~ure T in degrees Celsius ove:r a four-~~mu ~lllne interval
0 < t < 4 is ,given by

72-. The c~rve defined by the equation (x 2 + i~) 2 ~ .\' 2 - y 2 and


shown below is caUed a le11miscate. find the four poin~ al whid:~ u~e
tangetU line ~s ~orizontat

l? -

\~'hat is. the nlte of c:hange of dile area


wiU~ 1~spect w ]ts side ]en.gth L ?

f(t)

(a) Draw a graph of

20

A of an equ:iilaleral triangle

+ 3s.]n (4irt- tr/2).

f (t).

(b) How n11any times ]s the fuma.ce on dtuing the

-l

:~

73. Find all points. c ~n the ~nlerv.al 3 <


3.49 w[ilen f(.t) = x 3 + J.x - 2.

7'4. Find an points c in the interval - ~. < x < [ that sa~~sfy eqoalion
3.48 when f(.t)
3x 2 - 2x 4 and g(x)
x 3 ' x -

.t'

four-~mur

period?'
(C) \Vhat ~s. the maximum tiime rate of dii-."Lnge of lemperature'?

= +

CHAPTER

4 Applications of Differentiation

Application P1cvicw

Slider-cn111k.s .such as that showtl below. transform rocary motion to bt-ICk-iuxJ.f(mh mot ion ulong
a <tmightline, and vice versa. Rod A B. of length r. is pinne<lat A, lind rod BC. oFlength/.,
is pinned to rod A 8 at 8 . As A 8 rouucs around A, C is confined to move along a horizontal
line segme11t belwee11 poi1llS D (ulc.l F. , <;ailed thcstnlke of the mechanism. Rotary motion of B
is tralsformed 10 suaiglu-l ine mocio' of C along DE . This crank is said to be offset bccau:5c
the e;c:tcr1sion or line segment
by an nmoum e .

DE tlocs nOIJ'ass through the centre of 1hc cirdc:

A: it is oll"sct

. "-8

/~
'

~
" \.'
/,
C

!t'

.. .....

THE PROBI E\1


4 .28 on page 284.)

Find the offset that maxim ius the stroke. (For the solution. see Example

Our discussions in Ch..'\pter J himed at som e of the ppl ical ions of the derivative; in this
cha pter we deal with them in deta il. In Section 4.1 we s how d1at derivath es provide one of

the mos< powcf ul methods for approximatillg solutions to equations. Sections 4.2-4.7 arc
devolec.l tct the topic o f oplimiz.ation. its theory and tlppl iauions. Sections 4.7-4.1 0. with their
wealth of applied problems. show the power o f calculus in applied nutthcmatics. 111 Section 4 .7
we illustrate the simplicity that calculus brings to sol vi ng a pJ>Iicd llHlxinuHninima problems:
in Section 4.8 we Ue\'Ciot>a deeper understunding for the ulreac.Jy rami liar notions o f velocity

and acceleration; in S~ion 4.9 we use the in~>erpretation of the derivative as a rate of change to
investigate interdependencesofrelaced quantili es i n a wide variety ofapl)lications; and in Secti on

4. 10 we apply derivatives to LC R-circuits. The calculation of many otherwise intmctable limits


becomes relatively stra ightforward with L' H8pital's rule in

S~'tio n

4. 1J. In Section 4 . 12 we

discuss differentials, qualllities essential to the tor>ic of intcgn11ion, which begins in Chapler 5.

14. 1 Newton's Iterative Procedure for Solving Equations


In a lmost every area of applied mathematics. il is

n ece~ sary

10 solve e-quations. \Vhen the

problem is to solve or1e cquatiotl in one mlknown, the cqumion can be expressed in the fom<
f(x) =

n,

(-1 1)

where f(x ) is usuall y a di ffcrcntiablc ti,>ctlo:n of x. It might be a polynomial. a trigo.lOnlCtric


function. an exponential or a logarithm function. or a complicated combi nati on of these. The
equation may ha,e unc solutiun or many solut ions: these solutions arc a lso called mu1s of the

equation or zeros of the function. Few equations can be solved by fonnula. Even when f (x) is
a polynomial, tile only s imple formula is quadratic formula 1.5, which solves the equation for
second~degree

polynomials. Functions other than polynomials rarely have fo rmul as for their

zeros.
237

238

Cb3pler .a

r l:n!l!

Appl~.s

uf Oirfc.rentimion

Volumt or' a
hal(.fiUed hemispherie.al U1nt

Since we can seldom find exact solmions of 4.1. we con~ ioe r approximacing cite solucions.
Sophisticated c:1lculators have romines for approximating solutions to equatiOI\S; computer
software packages have one or more commands for doing chis. Ultfortunmely. with calculmors
and compucers. you re"AIIy have no idea what i s going on inside lhem. You SUI>PIY an equation
and out cornes a number. ln this secti on we develop one of the most po\verful techniques for
approxinuuing solutions to equations. The method is calcuh.ts~bascd and it, or n modification
of il. i~ used by many calculacors and comptHers to solve equal ions. Tf your machine uses this
1eclul.ique you will now undersland what it is doing.
To incroducc the tcchuiquc. coosidcr the problem of linding che dcpch of \vtller that half fi lls
(by volume) che hemispherical lank in Figure 4. 1. II can be shown (see Exercise 2 1 in Seccion
7.4) that whe11 the water is x metres deep, the volume of 'vatcr in the tank is gjvco by the formuJa
15x
"-(
J

x 3) .

Since che vulumc of one-half chc cank is une-founh chat of a sphere of radius 5. it follows
that the tank is h~1 lf full when .\' s:nislics 1he equntion

and chi> oquacion simplifies 10

f(x)

x3

15x 2

+ 125 =

0.

How do we solve this e.qumion when we know intuiciveJy chat che solmion we want issomewhere
around x = 3'? A skecch of j(x) betweell x = 3 and x = 3.5 is shown in Figure 4.2.
The curv:uure has been exaggerated in order 10 more clear ly depict I he following geometric
construction. If we set x 1 = 3. chen x 1 is an approxi.marioJt to the sohniou of the equation
- not a good approximacion. btu an approximacion nonetheles$. Suppru:e we drdw lhe tangent
line to y = f (x ) al (x 1, f (x 1) ). Tf x 2 is the point of intersection of lhis tangent line with
che x ~ax i s. it is clear chat x2 is a bener approximation than .r 1 to the solution of the equation.
If we draw the tangcncline to y = f (x) at (x 2 , j(.r2 ) ), its intersection point x3 with chc
x -axis is an eve.n beuer approximation. Continuation of this process leads to a succession or
numbe.r:s x., x1 x 3 , . .. , each of which is closer lo lhe solution of the equa1ion .f(x) = 0
than the prct.:cding numbers. TI1is procedure for finlli ng bcncr and bc.ucr approximcuions to
the solution of ttl\ equation is caiJed Newto n~s iterative procedure (or the Ncwton- RatJhson
itenlli1e prr.K..'edure). \Ve 83)1 that the nun1bers ..v 1 x 2. x 3. . . . converge 10 the roOtortheequaaion.
MijldiJ,:y"lfl I

Geometric: interpretation of N("\\tun's itcr~tivc procedure

Tangent line at (x 1 j(x1))

3
Tangent line at

(xo. /(xiJ)

(3.5. - J5.9)

\\'hat we need now i:; tln algt:bruic rurnnulu by which to culcula1c the approximations
.r 2. X;. X.o. .... The equmion ur the tllngcnt lio'IC tO y = .f'(X ) at (xt, f (Xo)) is

Y- /(.to} = j'(xo)(x- x 1).


To tind the poi Ill of intersection of this wngent line with the .r-axis. we set y = 0.

-.f'(.l' o) "' f'<x ,)(x- Xt).


and solve for :<.

/(x o)

'J- - - -,
/'(Xo )

Rut Ooe point of intersection of the tangent lio:te at

(.r 1 f(x 1) )

with the x-ax is is the second

approximation .r2: thm is.

/ (xt)
- /'(.t o).

To find x3 we repeatthio procedure with x2 replacing x 0: the re.<ult is

As we repeat this process over ml<.l over ugain. the following formuht for the (n + l)l11 approximation Xn + l in tcnns of the 11 1h upprox ima1iun x. emerges:

~l; 11 + l

XII -

.f(x.,)

(4.2)

- - -.

/ ' (.t,.)

This formula Uetincs ~i.:h appruxinuuiun in Nc..:wcon's itcn:uivc procedure in terms of itl' prcde--

res.<;Or.
Let us use Lhis pro<:c<lurc to PI>mximtc the solution of /(.t) = x l - 15x 2 + 125 "' 0 in
the example >llxwc. The dcoivat ive of f(x ) is f'(x) "' 3x 2 - 30x . and fornoulu 4.2 becomes
Xu+ I

.f11

x;- z15x,; +. 125

:lx,. - 30.\,1

Calcu lation of the next f<.lur approxi mation..;; beginning whh Xt = 3 gives

+ 125
3(3)2 - 30(3)
"' 3.2(19 84;

3~ - 15(3)1

X3

x. =

X2 -

.TJ -

Xs = x. -

xJ-

+ 125

IS.t i
l

3x2

30.t l

xl- l ISxi + 125


3x3

30.t3

x,f - 15x1 + 125


2

3x4

0..
3 '

3-

3.26352:

"'3.263 5182 ;

= 3.2635182.

h"'

Newton's itcrati\'C 1>rocc<lurc


therefore produced 3.263 51 &as an approximate solution to
the cqumion x~ - 15x 2 + 125 "' 0 . In spite of the fact th!ll we have wriucn six clecimals in
this fi nal an.swer, our analysis in no way gua.trantees this degree of accuracy; we have si.mply
judged on the basis of x 4 = x5 Lhat 3.263518 might be accurate to six decimal places. The
zero intermediate value theorem (of Section 1.11) confinns Lhis when we calculate

/(3.263 5175) = 4.8

10- S

and

/(3.2635 185) "' - 1.8 x

w-5 .

240

Cbapcc-r 4

Al)l)liC~Jtjons of Diffcrcntiali'IJn

I EXAMPLE 4 . 1
Use Newtons iterative procedure to find the only positive mot of lhe equatio n

3x

+ 1Sx 3 -

125x - 1500

accurate to five decimal places. (This equation will be

e~lCQuntcred

in Exercise 48 of Sec

tion 4.7.)

400 y

200
3.5

-200

4.5

-400
-600

y = :lx"'+ IS., 3- 125x -1 500

-800
- 1000

SOLUTION T he plot of f(x)

= 3x' +

15x3 - 125x - 1500 in Figure 4.3 shows that the

soluLion is between .x: = 4 and .x = 5. T he approximations prec.lk ted by

New tun~s method are

defined by

/(x,}

Suppose we choose

X, -

XJ

f'(xn) = x,-

3.{;: + ISx/, - 125x, - 1500


12xn3 + 45xn2 - 125

4 as the initial approximalio n; this is clo;scr to the root than x

5.

We find that

x2

= 4.19956,

-"3

= 4.187 268,

x, = 4. 187218 7,

' 5 = 4.1.87 218 7.

Since

f'(4.187215) = - 5.7 )( 10-3

and

/ {4.187225} = 9.7 x

w->,

it foiJows that the root is x = 4. 187 22, accurate to five decimal places .

...-..
Tn the following ex~unp le we again use Newton':\ i terdlive procedure tOappmxi mi:ll ethe ~olution

of an equatioo but specify the required accu.r acy as max.i mum error.

I EXAMPLE 4.2
Use NewLo n's meLhod to find the smallest root of the c.ubic equation x 3 - 3x

errorless than

w-s.

+J=

0 wilh

M:U~IIII#M

Initial appro'timanon for NtWtoll'< herativc: pi'OC'ecfure

3 y
y =.r' -3.r+

2 .\'
-I

SOLUTION '!he graph in Figure 4.4 indicates that the smallest root is just to the right of
= -2 and usc: Newton~ iterative procedure to define appruximatiuns:

.t = -2. \Ve take: Xt

Xn -

f(x.)

- - - =XII-

J'(x.,)

.t~ - 3x. +
.,
3.r,; - 3

The nexr fo ttr approxim:-1tions are

-'2 = - 1.888 89'

x,

= - 1.87945.

x, = - 1.879 385.

,,, = - 1.8793852.

o-s.

For an approximation with error less than 1


we su~~c..st - 1.8i9 39. The zero lntermetliate
value theorem con6nns th.is 'vhen we cakuhne
J(-1.8794{)) = -1. 1 x

w-'

and

J(- 1.87938)

= 4.0 x

10-' .

The value of a technique for approximating solutioos of an equation depends on two fa cto~ .
applicabillty and rate of convergence. Newto n's method scores well in both categorie-s . The
method can be appli ed to any equation 4.1, P"Ovided that f(x) is differentiable in an interval
containing the 1'001. It converges to a solution x = a. provided that j'(a) # 0 and X t is
~ tnitU.I ;ap
J)rudmauoo for Neo.lOil') ileronhe
chosen <ufficient ly close to a (see Exercise 68 in Sectio n I 0. 1). The condition f' (a) # 0 is
proctdutt tmK"I bt: S-1111lcttntly
included :so that the denominator i.n equation 4.2 docs not approach 0 as Xn ~ a. However,
dusc 10 root
f(x,.) usually approaches 0 faster than J'(x.,) so that even when J'(a) = 0. Newton's method
may be successful. It is impQssiblc to indicate: how close x 1 must be to a in order to guarantee
convergence tots . In some examples x 1 can be any number whatsoever, but we are also ~ware
of camplcs where l.r, - al must be less thnn 0.01 (sec Ex(lmple 4.3). Provided that x 1 is
sullicientl)' close to the rom ~ convergence orth:e: npproximmions to the root is rApid . This makes
Newton's method vnluablc from the point of ...icw of the second criterion. rate of convergence.
\Vhen .\' 1 is not sufficiently close to tl , Newtons i terative procedure m;1y not converge, or
may corwcrgc to a solution other than expected. f"Qr in~tance. if we attempt to appro:<im.me
the largest root in Example 4.2, and inndvcnently choose Xt = I, we cannot fi nd the second
approxil'nation: algebraically because 3xf - 3 = 0. aml geometricall y because at X1 = I. the
----+-...:,- -j+"-..:::..J.___.x tangent li1l C is horizontal, and does not imersect the x-a"<is. The fu nction /{x) in Figure 4.5
has zeros ne~:~r x = 1/ 2 anc.J x = I. If we a ttc:mpt lo fincJ lhc snmllc:r zero using an inilial approximation x 1 = 0. we (ind the second approximation X2 to be larger than 1. Further il.e rations
then converge to the ~ero near .~ = I, tlOt the zero near .~ = 1/2. We conclude. the re fore, that
the initial approximation in Newton s iterative procedure is most impot1ant. A poor choice for

x 1 may lead to numbers that either converge to the wrong root or do not converge at all.

I EXAMPLE 4.3
10jldilil3U I Finding
tension in h),.dro cable at its low~
point

A uoiforro hydro cable P = 80 m long w ith mass per unit length p = 0.5 kglm is hung from
two suppons at the same le'-el L = 70 m apart (F igure 4 .6). The tension T in the c"ble at its
lowest point must salisfy tbe equation

1 70 m ~

{[-som-J \

whereg = 9.81 m/s2 . Tf wesetz = pg/(2T) . then z must satisfy

Find an approximation to the solution or this eqmuion for

z that yields T correct to one dedmal

t>lacc.

SOI.U'n ON

When we substitute

P = 80 and L = 70, the equation for z become.s


160z = ~"'= - e- 10'.

The exponential function e iO:. grows very rapidly for

z>

Ql and e -1'0: approaches zero very


quickly. Tt follows that the solution of the eq uation must be quire small. If we plot J(z) =
e 70: - e- 10: - 160: on the interval 0 ::0 z ::0 0.02 (Figure 4.7), we capture the solution. With
z 1 = 0.0 13, Newton's iterative procedure defines funher approxi mation.s by

Zo = 0.013,

ln +J

z, -

e70:. - e-10~ - 160zn

70e7,._

+ 10e

7rr~

- I 60

To get an idea of how accurate z should be i_n order lO give T correct to one decimal place. we
note that for z = 0.0 13, tension is T ~ 189- Thus, we should determine z to around tive or
six figures. henuion of Newtons procedure ghes
Z2 = 0.012 957 3,

~J =

0.0129570,

z,

0.0129570.

With z approximated b)' 0.0 129570. tension is T = 189.3 N. We could verify that this is
accurate to one decimal place by defining g(T) = e~LftlT) - e-n~LfflT) - pf!. P f T and
evaluating g (189.25) and g( 189.35). The first is positive and the second is negative.

0.1

/(:)= , 7oo_ , - 70:_ 160:

0.05
0.005
-().05

O.QI

EXERCISES 4.1

m tinll'S per )Car are given by the formula

In Exercises 1- 16 use ~ew10n's itetati.-e ptoce<lure to lind approxima


lions to all fl)()(S ufUlC eQuation accunnc to six dccintLl places. In each
case. make aplot in on.lcrtooblain an initial <lt)pro:<il1\ation to ca~.:h root
I. x 1 + 3x + I ~ 0

i
i
i

3. x 3 + x - 3 ~ 0

7. ,, + 3.t 1

~- x- IOsinx=O

II. (x

iii
i

13.

x+4 1 nx~o

IS.

ex+ e-x = lOx

i! 2. .:c1 - .'1: - 4 - 0
i! 4. .\'} - .\' 2 + ."( - 22 = 0
i! 6. .t> +.< -I= 0

s. x' - sx' - .< + 4-0


7

+ 1) 1 =

=0

x+l
- - =x1+ I

8.

i!

10. secx

sin4x

ll 12.

!! 14.
m 16.

]
i /(lOOm)
M =P [
(
i ) -'"'.
1- 1 + - IOOm

X-

(a) What :ttc monthly payments if

i '*

.Tlnx = 6
.{2 - 4c- ll'

J2. When the beam in ~>e figure below l'ibcatcs I'CniC<Illy. then: are
cenain fr('(Jucncic:s or \'ibr.nion <:a11ctl ntlttmll frtqurm:its. They arc
solutions or the equation

=0
l...r -

tanxln Ell:erci.scs 17-24 use 'cv.'lon's iterative prcx."\."'<lure to find 3pprox.


imntio ns 10 all rools of the eqomion '"''ith error no greater th:.L1 \Mt
specified.

17. x

Ill

18. x 4

19. _ x_ =x2 + 2.

..

20 . (x + 1) 2 =

;;;

i
i
i
i

21.

5x - I = 0.

x 3 + 2x 1 + 6.t = 0.

decimal places.
~---10 m---1

10-.1

10-'

x'- 4x ,

.t 1 -

'

10-J

10_,

(X+ 1) 2 = hin4X.
I.

10-

x + (ln x) 1 = 0.

10 -)

24. ,J>

divided by 207r . f ind the two small<:$ I frequencies com.-.:1 to fou r

22. cos:. X
23.

+ ,.X = 4, lo-

In txe reis:e~2S-18 find sll point~of i ntersection for the curves :l:CCurJt'
to four dcximal places.

m,.. 2.5.
iii 26.
iii* 27.

y=x 3 ~

y:x' -4.r

y = x - 20.

y=

can be obtained by setting y - 0 ttnd sohiug the equation


!'or 1. Do so {oom:ct to l\\'t>dcd m.aJ places).

x' - 2x 1
2

(h) When air resi~1ance i ~ neglected, the form ula for y is

+2

+ 29. Show algebrail-:ally and geomelric.."a.lly that Ne\o\ton's method never


gives lhc t:Oluti<'n or 1hc equ:uion [{X) = xn = 0. tOr any initi:al

What is the clapsell time in this case from the instant the
is pn)jCCied until i1 reiurns 10 Ihe projec.ti011 point?

~one

approximation whalsocvcr.

* 30.

Sho w algcbrnically land gcomclricUy 1ha1 Ncwlons method al-

ways gives the solution of the equation f(x) = x 1n = 0, for any


initial approximation whatsoever.

i * 31 .

r = 0 ac 1he inscam or

projection).
(o) The time token for the.st(lne tl) return 10 it~ proj..:ction poinl

y: (X+ 1)' ,

33. A stone of ma.ss 100 g i:, lhrown vcnically upward with speed
20 nlls. Air exerts a rcsisti\.e forc:.-c on 1he scone prOt)OrCional 10 its
s1~d. and has magniltUic 0. 1 N when the speal of the stone i< 10m/$
lc can be ~h own IMI lhe height y abo"e lhc ptojeccion point anaintd
by the S*one is given by

where J is time (measured in se<."'nds with

y =x+5

~- = x
"'* 28. '\' = X+-.
I

u-

10- 3

x+

= 100000. i = S. 2nd

(b) What imcres1 tate would yield monthly payments: of SSOO


for P
I 00 (:00 and n = 25 1

2
= --l +x

(x + 1)1 = 5sin4x

n = 25'1

SuppOse you mortgage your house ror P dollars. To repay lhc


loan a1 an inlcrcSI rate of i %, aroor1izcd over n ycms, paymentS (nndc

i*

34. Planck"s /(JIV for lht:: energy dcnsily E


1ooo~ K s1a1cs hu

or blackbody rndi!lliOn !tl

k)._- S

= (>.) = f"'l ' -

I'

w here k > 0

u c:on~t<lrlf and

(i.i) Riu t im~:, Tr. defined as

c = O.O(X) 14386.

1"1lis func tion i s


shown in the fi gure bciO\\, The value of>. at which E is a muim11m
must satist'y the tXJUatic:m
IS

find l his valuet)f

tbc time for t he tr:tl>h to rise from

0 . 11 oO.~:

(iii) Stilling Jim~. Ts. t.lcHrH:d :as the time to reach and t~m !litl
\Ooithin the inlcJ"otl O.t>S ~ j(s) ~ 1.05 .

find the~ lin~ts. Q.XTOO\ to l\VO decimal p laces.


t 2 :-.fiir sin (20t - ;r /2) .

l t(u1'CCI LOSC\'CO decimal plm:cs..

ir

j(t) -

1.25
1.0
0.75
0.5

0.25
;,

.\5. 1h.: ~pr~.:.d. of rCSIXli"ISC of an oscillatory sy$t:clll whose !lnlph y =


I (1) is shown below is ortcn dctcnnincd hy um: ur lhc fo llowing thrC(\
1i1nc t."Un~t:m~:
(i) Vt'lfO rim~. 1d. d<:fi ncd

os the tin-.e rcqum.~ for the &N!.Ph

Ul reach O.S;

0.2
to

0.6

0.4

.36. Sui)IX)o;;c that :t cubic J,.., l)'nonial P (x) hus llwccdi~ i~L rca1-..crt-.&.
Show lhlU \\hen 1'\cwiOt:fs mcd)Q(.I us.c$ an iniliul U4JI"Wciumlioe\ that U)
oqual lo the 1.1\'<.'1'agc 4..'1' twn of the ?ctos.. then the liN-t itentlK>n ;,lwuy~
yicl$ thc third 1.cro.

14.2 Increasing and Decreasing Functions


Many mathematical c:oneepts ha\e the ir origin in intuitive ideas. lr1 this section we analyze
1he iltuitive idea o f one q ua ntity "geuilS 1arsel"'' a1\d a r1other .. getti ng s maller.~ To de$.Cribe
what it rneans for~ quantity 10 be increas-ing (gcuing larger) or decreasing (getting s.mutlcr).
we first .upposc tht the quantity i reprcseoucd by some funcli<>n f (.<) of u vnriable x. noc
l'nath.eu-.Hical det1nition for f (x) to be in<:rcasing or dec..."'l'e(\.;;;i ng i~ as follows.

DE FINITI O N 4. 1
A funclion f (x) is said to be increasing on ful intcrva1 I if for all .rt > x2 in J ,
(-1.3Jl

A function f (x ) ;, snid "' be d w-cn.<lng on f if for all x 1 >

' 1i n f.
(-Ubl

The continuous funt.1ion in Figure 4.8 is increasing on the intervals


ll

and decreasing on the

~ X ~

b,

~ .t ~

d.

e~ x ~

f,

interval~

IJ

_:S X _:S C.

d:Sx:Se .

When a fune1ion has poinL">of di~co ntinu ity, suc h as in Figure 4 .9, the situation is somewhat
more complicated. This function is. increasing on the intervals

e < x < f,

a < x < b,
and decreasing for

b < X < d.

Pay special atte-ntjon to whether endpoints of each inte rval are included.
Figure, 4.8 and 4.9 indicate that the sign of J' (x) determines whctiY.:r a function is incn:as
ing or de<:reasing oo an i11tervnl . The tollowing test describes the situation.

ln1cnuls un \\hich

o di5CQI~i nu

func1ioo is increasing and decreasing

(I

Increasing and Decreasing Test


(i) A function /(.r) is increasing on an interval / if on / ,

/ '(x) 2: 0

!4.4a I

and is equal to zero at only a fini te number of I>Dint~.

(ii) A function f(x) is decreasing on an interval I if o n/.

f' (x) ::; 0


MjldiiJt-WitM

f(:r) is

not incrc.asing on a ~ x ~ b if

l'(x )

iijUIII;I

(-lAbt

and is equal to zero at only a finite number of points.


Figures 4 . 10 and 4 . 11 illustrate why we permit / ' (x) to vanish at only a finite number of points
and do not, therefore. allow it to vanish on an interval. In Figure 4 .1 0, f' (x) is equal to zero on
1he inteJVdl 11 .S: x 5 b . and cenainly. /(:r) is no1 increasing on any intenralrhar coma ins these
points. The function f(x ) in Figure4.11 has /'(0) = 0 = J'( l) , and yet f(x ) is increasing
on the interval a ::; x ::; b.
Conditions 4 .4 arc sufficient to guarantee that a function is increasing or decreasing on an
interval; they are not necessary. For example, the function f(x ) = x- sin x in Figure 4 . 12 ha.<
dcrivatjveequal 10 Oat the infinj ty of \'alucs .:r = 2n n. where n is an integer. yet the fu1lCtion is
increasing on the inten.-al - 00 < x < oo. \Vhm we are saying is that tests more general than
4.4 can be fo rmulated. but we fed that the cxtrd complexity i.s not worth the gain. For a proof
of 4.4,
Exercise 51.

Even if J'(;t.) = 0 at an infinity uf poinLS. /(x)

f<> ;,

inctea)oillg. on ( t ...:: ' .... b c:\ ec~ if


J'(,r) = 0 ar a tinile oumber of

10

poinu

tn.l)'

be in<=Ril~ng

y=x - sinx

)'

- 15

-10

-5

I ()

-5
- 10
-1 5
lmcrval.s on which a func tion i.s increasing a nd decreasing arc separated by po ints whe re
the derivative is equal to ze ro or does not exist. Keep this in m ind in the following example.

I EXAMPLE 4.4
Find intervals on which the followiug functions arc increasing and decreasing:

f(x) = 2~3

(a)

+ 3x2 -

5x

+4

(b)

x2 - 9
f(_t) = .-------:x2 + x - 2

SOLUTION

(a) As mentioned above, the key to finding imcrvals 0 11 which the derivative of a function
is positive and negative are poinl'\ where the derinuive is either zero or nonexistent.
S_incc. polynomja}s have derivatives cvcrywhc.rc. we investigate where. the-dcrivatjvc.

is equal to zero,
0 = f'{_r) = 6.r

+ 6x -

.-

I. -

-6

-3 J39
6

J36 + 120
12

To dccem1ine intervals on which che runction is increasing ttnd d ecrea.~ing we can


proceed in a number of ways. Firstly, if we have a plot of the function (Figure
4 . 13). it is clear that f(x ) is increasing on the imervals x 5 ( -3- J39)/6 and
x::: ( -3 + J39)/6, and docreasiugon (-3- J39)/ 6 S x S ( -3 + ./39)/6.
Secondly, if we do not have a graph of Lhe fuoction. we could visualize that a g raph of
f ' (x) (Figure 4. 14) is a parabola ll>at crosses the x-axis at the poi ms ( - 3J39)/6.
Notice that we said visualize tttis grapb~ \VC have drawn it, but it would be necessary
only to memally visualize it. Since the pardbola opens upward. f' (.r) 2:: 0 on
the. intervals x S ( - 3 - ./39)/6 and x > ( - 3 + J39)/ 6, aod j'(.r) :::; 0 on

( -3- ../39)/6 5

5 ( -3 + ../39)/6.

Gruph to determine wb<:n


i;; incrca"mS and decrcasine,

GURE 4.13
ftanctiC~n

FIGURE 4.14

Gntpb to determine the

f'(x)

20
y = 2x; + Jxl- Sx > 4

15
10

-2

-I

.r

-5
Finally, we could eoostnlct a s ig n tab.le for f' (x) as iotroduced in TabI.e l. l of
Section 1.5 (see figure 4 .1 5 below). h also shows that f' (x) 2:: 0 <>n Jhe illlervals
x S ( - 3- .J:i9)/6 and x ::: (- 3+ ./39)/ 6, and f ' (x ) ~ 0 on (- 3- ./39)/6 S
X 5 (-3 + ~f39)j6 .
ljlclll.l30

(b) The graph of .f(x) in l, igure 4. 16a. indicates discontinuities at x = - 2 and x =


aod the poim between x = -2 and x = 0 where f ' (x) = 0 separates iotervals oo
which t.he fuoct.ion is increasing and decreasiog. Whether t.here are other poilUS far
om on the Xaxis where .f'(x) = 0 is not clear. To find all points where .f'(x) = 0,
we set

(x 1

0= /(.<)-

+\-

Thi< implies that


n.uncl)

m-

Z)(Z.r) - (x 2 - 91(2.<
(x 1 +x 2) 2

l ~.r

+9 =

I)

x 2 + I4X "- 9
(x'

+'

1)l'

0. and !hen: arc two pooniS \\h<te /'(.<) = 0.

-7 :I: 2Ji0. Figure 4. 16b i~ nn C'(U~\.'futcU \CN1on or the gruph of

/(.r) tu thel<ft uf x = -2: it is not aco01puter plo< Tcoll u<tn<c.that f'(x) = 0 at


-7- 2./iO and that the gnlph is asymptotic to the line .1 = I (rronl "bo":).
We cun uow ~uy ch.11 th0 runctiun is increasing on the intcrvuls

x=

<

-CXl

.I

2Jiii.

- 7

2Jiii !$ X

< -2.

-1-

+ 2-Jiii

S ~ < I.

+ 2Jiii.

< x < o:>.

and dccrear.in~ l'or

-7-

< .\

- 1

'

.1()

20

\..

\2

-3

-2

-I

-20
-40

x 1 -9

J ,. ~.

~---~ -------

r-2

r,

-3

-60

EXERCISES 4 2

ln &crc~C I 26 t.k.'1c1mtr1e &nlcrvalson "lltch the ("'ICtion it ncre.t'

ina and ~.ui"'1. /(x)

=2.t

2. f(x)

J. j(.r).l 1 - ll + ~
5. f(x)

= J 11

9. f(x)

.1

- 2.r1 -5.r
4.r'

4.r' - 18.r 2 I I

21. /(.<)

=.u -

2.1. /(.r)

=In (x' + 5)

25.

j'

(x

9
= ...
- --

.r - 3

18.r-9x 2 -2.rl

II. /(<)- '1 +41 3 - 2~< +2


4

13.

2-x

19. f(x)= - x+ l

18.r 2 ~ 48.r + 1

+ 6.1 1 +Ill+ S

10. /(x) -4

12. /(.r) -

= --

:u - 4A 3 + 24x 2 -

+4

,l

l g, /(r) - \""2 -

x>

5r

6. f(.r) = S + 2.r

1 6.r - 2

7. f(.r) c 2.<3

8. j'(.l')

4. j'(x)

=4

17. /(.r)

2.0. /(X)

I I+ I

22. /() - . .-
2-1. {(1)

l(i. /(.t )

~X

hl.l

x3 +2.rl - .x - 2
1 - :r -.r'

27. If the pru:c ()(a ccn:in cur is <~CCI at r. th4:'n the nwrtcl dcnl:Jilds X
cars per ycz.r. "hat:

48x

! <> = '--h 1 -!lr1 +48 +l~

14. /(x)

15. /(J.)

x'- Sx + 2
I

=).

+r

16. j (x) =x+-,


x-

"'here a > 0 is a ronstanL and 0 < r < 2o . Show thai Lhe price
function r
f(x) defined implicilly by thu equation is a decreasing
function.

Figure (a) below conlains the graph of a funclion /(.\' ) und figure (b),
tho graph of f' (.r) . Note that f' (x) ;:: 0 whon f(x) is increasing
and / ' (x) ~ 0 when f(x ) is decreasing. In addilion, the corner in
f (x) a1 x = 2 i~ rcllccccd in lhe di~ontinuity in /'(x ) . In Exercises
28-35 draw similar gruphs for Ihe funclioo and it,; derivative.

Graph off(x)

3
2

40. Show algebraically th:at the equ.:1tion x ll

are consmnt..tsuch lhal ab > 0. hasexaclly one (real) sohtLion.

* 44. Show lhat si n x

/'(x) fO< f(.t )

< x for all x > 0.

Show 1ha1 cosx >


lcchniquc of Exercise 44.

t -

.10. [ (x )

29. f(x) x' - x 2

x- 4

=.r+ 2

31. /(X)= -

lxl
X

, l4. /(x)

= lx' -

a = 0.

Calculate /(0) and

x' / 2 for all x > 0. Hinl: Sc< tho

x'

> X- - .
6

47. Usc Lhc n:Mdt uf Excn:.isc 46 to pmvc: that for .r > 0.

,-

33. f(x) = ix ' - 41

32. f (x ) = -

H in1:

+- 46. Usc the rcwh of Exercise 4-S 10 proYc thu.l for x > 0.
SiJlX

X -

+ x" - 1 -

= .r- sinx.

* ~.
.(

(b)

+ lx

1 0

* 41 . Show thai 1hc equation ax5 + bx 3 + c = 0. \Vhere a. b . and c

43. Repe-at E.xe:rc1sc 42 for the equation x~

Graph of/'(x)

(a)

28. f(x) ~ .< 2

+ 3x U + 4x +

has c:<nclly one (real) solution.

..,. 42. Show Lhat the equation x" +ax - I - 0~ wltelc. a > 0 and n
arc conslants.. hus CXtK.<'tl y one positi \'CrooL

)'

3
2

1<

cosx <

.t z
I -

48. Usc lhe: tecOmitlue of Exercise 44

x4

+ -24 .

10 verify

tho I for x > 0,

41

.t - 2

3x
> I - - .
t + 3x
2

49. If f(x) and g (.t) arc diffcrtr'ltiable a.nd increasing on 31\ Uncr\'nl
I , is f (x)g(x) increasing (>n / 'I
In Excn.:i:scs 36-39 find (a(.;<;Urutc to four decimal places) intervals on
which the fun<:.tion is incrca~ing and dccn.-asing.

+ SO. If positive funcLion~ f (x) !l.nd $(X) :~re ditl'ercnlioble and inc~:as.
ing on un jntc.rvul. I, is f (x )S {.t) incrc:nsing on I ?
51. Verify tcst4.4.

* 52.

iii t 36. [ (x)a .t' + 1x - 6.< + 5


2

-"~t

!I ..
ii

37. f(x )

= 3x1 -

20x'- 24x 1

39. f(x ) ~ tonx - x(,r

+ 2).

+ 48A

**

A number ,'(,) is called ajhw/ point of a function j(.r) if f(.ro) =


h!!.v~ if / ' {x ) < I for :111

J.low m:1ny lixed points can a. function

x?
53. Pnwc that when 0 < a < b < rr / 2.
tanb

-;r / 2 < x < JT/ 2

t.un a

b
> -.
a

14.3 Relative Maxima and Minima


One of the most imponam applicmions of calculus is in 1he field of optimization. the s1udy of
maxima a1'1d minima. In this se.ction we begin discussions of this topic, whic.h continue through
to Section 4.7. Fu nc.J~mcma l to d i:,c.:us:sion.:; on optimiuuion are crilical poims.

DEFINITIO N 4 .2
A crllitlll point of a function is " poin1 in the domain of the functinn at which the first
derivati\'e either is equal to zero or does not ex ist.

SpeciticaiJy. x = cis a critical poim for f (x) if .f'(c) = 0 or .f'(c) does not ex ist. but io
the latter case~ f (c) must cxisL With the interpretation of the derivative as the slope o f a tangent
line, we can s1a1e thai corresponding 10 a c ritical poinl of a function, the graph of lhe function

has a horizontal tangem line. a vertic.al tangem l ine. or nu umgcnt l ine at aU. For example, the
eight pointS a through II on the x -axis in F igure 4. 17 arc all critical. AI a, b. c. and d. the
tnngcnt line is hori?..ontnl: tlt r 8 11d f. the tangem line is vertical; and at g und h . there is no
tangent line.
Often o"erlooked by :;tudcnts, t;x.lt Ycry important in this defioiti011. is tha t" funclion must
be defined at ;.t c ritical point: it mus t hnvc a value. and therefore, there must be a point on the
gmph of the function at n crilital poinl For instatlc-e , if the c.lot in Figure 4. 17 at the tlisconlinui ty
X = h i s rcplacctl by an OJ>Cn cil.'cle, 11teo .r = h is no longer a critital point.

I>

MjH-I II;IXMI.M Critkt~.l


poinl> cU t'lld pc:oint~ \\ ~n (UI'K:IiQn

tlefir't.:1 Oil t i{WC:d inter\'3J


)'

.r

When the domain of II func tion / (.t) is a c losed interval a ::; X ::; b. the end1>0i nt X = n
is said to be c ritical if the right-hand detivative of f (x) at x
(t i~ equal tO ?.ero or does not
exist. l i kewise. x
b is crirical if the lefi.hatld de rivuive vanishes or OOes not exisl there.
This is cons is tent with 1hc definition or d incrcnti:bility in Section '3.'3. The func tion in Figure
4.18 h;ts domai n I :S x :S 3 . Both c ndpoims are critical - .< = I because /~ ( I) does not
exist. and x = J l~cause ,(:._ (3) = 0.

11te fu nction i n Figure 4 . 17 is di sconti nuous at ,\' =h. atd we k now thnl a f unction canno1
have~l deriva1ive a t a po int of discontinuity (Theorem 3.6). 'l'his docs not mean thai every )>Oint

,\"

ot' d iscont inuity of a function is critical. Remember, u function muS[ be defined at a point fo r
that point to be c ritical. Thus. points of d iscontinuity are critical only if the function is defined
at chc poim. In the remainder of this section we consider only critical points ac which a fU11Ction
is conrinuous and which are not end poi nt.s o f ils clom<lin of definilinn.

I EXAMPLE 4 .6
Pind critical points for 1he following func1ions:
(a) f(x)

(b) f(x)

= .r 3 - 7.<1 +
xl

= x' _

l lx

+6

(c) f(x) = x ln x
(d)

/(x) = lx I

SOLUTION
(ll) For critica l poi n1 ~ we firs1 sc>l' e

0 = f'(x) = 3x1
and obtain x = 11/3 and x

14X +II = (3-t - 11)(-t - 1),

= I. Those are the o nly critical points sin<< tht:re arc

no points where f'(x) doo; n()( exist.


(b) For critical points we calcula te

- x(xJ + 2)
(x3 - I )l .

Cleorly, f' (x) = 0 when x = 0 and when .< 3 + 2


0 , which impl.ies that
x = -2 11J The dcri,<ati\'edocs not CJ< ist when x'- I = 0 (i.e., when x = 1).
But f(.r) is not defined at x = I e ither. and therefore x = I is not a criticall>oint.
There are only two critic.a l points: _r = 0 and x = - 2 1/ 3
(c) For critical points we lirst sohe

0 = J'(x) = In x

+ -XX

= lnx

+ I.

The only solution of litis equation is x = If e. Since J'(x) exists for all.r in the
domain off (x ) . namely, .r > 0, the function has no other critical poiniS.
(d) The only critical point of f(x) = lxl is x = 0. Its dcri\'otivc is equal to I when
x > 0. and to - I when x < 0. but does not exist at x = 0 .

At the critical points band f in Figure4.19, the graph of t he function has ' 1tigh" points. They
are de.<cribed in the following defini1ion.

D E FINITION 4 .3

A function f(x) is said to have a relalh'e(or local) maximum f(.ro) at x


exists an open imerval 1 containing .r0 such that for all x in l ,

f(x)

= Xo ifthene

f(x<J).

Since such intervals can be drawn around X = b alld Jt =

r. relati\e maxima occur at

the..<;e pointS. At a relative or local maximum, the grnph of the function is hig.hesr relative to

nearby points.
Critical poi nt~ d and lr in Figure 4.19, where the graph has "low" points, are described in
a similar definition.

t iit!Jl
)'

.<

,,

DEFINITION 4.4

A function .f (x) is said to have a relative (or local) minimum f (xo) at x


exists an open interval I containing xo suc h Lhat for all x in l ,

f(x) 2: .f(xo).

= Xo

if there

(4.6)

d and A= h in Figurc4. J9.


The critical points x =a and x = c. wl1ere j'(x} = 0 and x = e and x = fl, where
j'(x) does not exist, will be discusse.d in Section 4.4, At x = i . the graph takes an abmpt

Rehltivcm ininmthercf'orcoccurat .x =

change in direction. The functjon has a left- and a righc-hatld dcf'i vative, but f'(i) does not
exist. In Section 3.3 we called che correspon:ding point on the curve a comer. Corners can
sometime.~ be reh1tive extrema (x = h yields a corner and a rcJati vc minimum).
Relative maxima and min ima represent high and low poin!S on the graph of a function
rela1ive to poims near them. It is not coincidence in Figure 4.1 9 that the two relati1e maxima
and the t\VOrelative minima occur at crit-ical poinls. According to lhe following rheorem, this is

always the case.

THEOREM 4 . 1

Relative maxima and relative ntinima of a function must occur at critical points of ll1e
function.

lijll;lt

*1'

relati\'e e:urerua
points

Proof lhlll
c-ritk-.al

I'ROOF To "crify this we prove thm at nny point ai which the defivative f '(x) of a funct.ion
f(.r) exists and is not zero, it is impossible f<>r f (x) to ha,c a relative cxucmum, that is. a
relative maximum or a relative minimum. Suppo~e that at some point x = n. the derivmi ve
/'(a) exists and is positive (Figure4.20):

~>.."'Cur at

)'

0 <

f'<a) =slope> 0

'( )
! a =

1101
h-+0

[((1

+ lr)

- [(a)

According to the proof developed in Exercise 29 of Section 2.6. there exi;t$ an open interval
l : b < x < c around x = a in which

;f(a h)

/(a) :

ba a+/1 c

.f.:..
(a_+'---'11)' ----"-/_,_(a...:..)
> 0.
h

.(

This implies thHt when h > 0 (and a + h is i" ! ). [ (aT /1) - j (a ) must also be positive,
and therefore f(a + II) > f(a). But when f1 < 0 (and a+ II is in ! ), [(a + h) - /(a)
must be negative. and therefore [(a+ h) < f(n). There is an interval IJ < x < t1 in which
f(x) < f(a),a nd aninterval n < x < c in which f(x) > f(n) . Thus,.~ = a cannot yield
a rc.Jativc extremum.

A similar proof holds when /'(a} < 0.

AIthough every relative extremum of a function must occur at a criucal point . not all critical
points give relative exm~ma . For continuous functions, there is a simple test to determ ine
whether a critical point givc.s a relative maximum or & relati\'e minimum. To understand this
lest. consider the critical points in Figure 4.19 as you re.ad the statemems.

First-Derhative Test for Relathe Extrema of


Continuous Functions
(i) If f' (x ) (slope of a graph} changes from a positive quamity to a negative quantity
as x increases through a critical poin1. at which f (x) is continuous. the critical point
yields a relative maximum for f (x) .
(ii) U f' (x) chaoges from a oega1ive quan1 i1y to a positivequaotity as x iocreases through
a critical point at which
mum for f(x).

.f(x )

is continuous, the critical point yields a relative mini~

252

0u()ler ~

AppiK:alions of Oitftrentiati<lr'

Various possibilities can occur if f'{x) does not change sign as x increases through a critical
poioL f-or ins~ance, if f'(x) is positive (or negative) on both s ides of a critic;~l point (such a~
is the case for the critical point x = 0 of f(x) = x 3 ], l11en lhe crilical pou11 cannot yield a
relative maximum or minimum. h might also happen that f ' (x) is both positive an<.! ncgati vc
i.n every interval around a cri aical point, no maucr how smallahc imcnal. ll1is is in\c.stigatcd
in Exercises 76-78.

I EXAMPLE 4 . 6
Find relative maxima and relaaive minima for ahe fol lowing functions:

= 2x3 - 9x 2 - 23x + 6
(b) f(x) = 3.0 Jx- sin (3x + 1), 0
(a) f(x)
(c)

J(x) = x;IJ-

(d) f(x)

x 213

= 5

SOLUTIO!\
(a) The graph oflhc func1i on in Figure 4 .21 a indica1es a rei alive maximum near x = - I
and a relati\e minimwnnear x = 4. To locale !hem precisely, we lind critical points

by solving

= j'(x) = 6x1 -

ISx - 23

x=

18 ./324

+ 24 . 23

12

9 /2 19
= ---6

These are the only crilical poi111s. a.< !here are no poi111S a1 which f'(x) is undefined.
Figure 4.21a indica1es that a relali\'e maximum O<:turs at x = (9- M)/6 and
a n:la1hc minimum at x = (9- ../fi9)/6. This can be confirmed nlgcbrdically or
ith the grdph of J' (.t) in Figure 4.21b. h shows that as x increases through (9 M)/6, J'(.t) changes from positive to ncga1ivc; 1hcrcfon:, x = (9 - M)/6
yields a rela1ive maximum of/((? - M)/6) '>< 18.02. Since /'(.t) changes
from nega1ive 10 posi1ive as x increases through (9 + ../219)/6. lhis value of x
yields a relmie minimum or /((9 + ../219)/6) "= -1 02.0.
(b) The gmph in Figure 4.22 does 1101 make it cleat whether 1hc fuoction has n:L11ivc
ex1rema; the tangenl line may become horizontal at or near x = 2 and x = 4. but
we;: canno1be sure:. Criaical points of the runcrion are. gi\'c:n br

0 = J'(x) = 3.01 - 3cos (3.t

3.0 1
cos(3x + I) = .
3

+ I)

Since 3.01 / 3 > I. this equation has no solutions. Hence. /(x) has no criti.:lLI poin~
and there can be no relaahe maxima or minima.
M@IUII.IM fiN

Rcl uti\~

txl!cnlJ

)'

200

}' =!x'-9.( 2 - ~.:+ 6

30

100
8

-4

/'(x)= lit~ -I S.t - 23

20
10
X

-2

10

-200

-2

- 300

-30

4.3 Rebtivt ~kr~inl2 ~ Mirtirna

M:IUIII;IY EEL

ljllii;J::a

~ing

Grllph shov.:ing a

~);uive

253

minimnm .md

a rtlati\e ma~hmlfl'l

.. y

)'

15

0.5
10

-2

y=3.01.r - sin(3.r + I)

-I

(c) The graph of the function in Figure 4.23 makes it clear that a relative minimum occurs

just to the left of x = 1/2. It also suggests that /(0) = 0 is a relati\>e maximum.
Confirmation is provided by c-ritical points, found by first sohing

2
3

0 = /'(.t) = - x 2fl' - - x - 13

5x- 2
-

3xl/3

Clearly. x = 2/ 5 is a critical point. The relati\'Cminimum at this point is /(2/5) :::::


-0.33. In addition. because /'(0) does not exist, bul /(0) = 0, it follows that

.x

= 0 is (ilso a critical point

There are two

reason~.

e.ach

~urticient

b)' itself 10

conclude that /(0) = 0 is a relati,e maximum. First, f(x) is continuous at x = 0,


and f'(x) change.< from a l)t)Siti\'e quantity to a ncgati\'C quaauity a.< x increase.<
through 0. Second, since /(.t) = x l/J (x - I} is negative for all x < I, except

at x

= 0 where /(0) "' 0. there must be a relative maximum at x = 0. ll is also

inte-resting to note that

li.m J'(x) = oo

lim J'(.t) = -00.

311d

.r-o

.r-{1

This means that the graph has a ery sharp point at (0, 0) .
(d) The graph of this function is a horizontal straight line. Every value of x is critic:>l
and at each value of .t. the function, has a relative ma:~tirnum and a rclati\e minimum
of5.

I EXAMPLE 4.7
The equation x 2 y + y 3 = 8 <lefines y implicitly as a function of x . Find all criticalllOints of
the function and determine whether they yield rcl:uive maximo or minima for the fUJwtion.

SOLt.illO'\ When we differentiate the equ:uion implicitly with I'C>I>CCt to x. we obtain


dy

2.1)'

dy

+ .t2 dx
+ 3y2 dx
=0
-

dy
dx

2.ry

The derivative is defined for all .r a1l<l y except when both arc simultanCOIJsly 1.cro. a llOim that
does not satisfy the original equation. ConseqiUcntl): we consider when the dcri\"ativc vanishes.
T'or dtis to happen. either x or .l' muol be zero. n,c original equation llocs nOL I>crmit y to be
7.ero, and when x
0. Ihe only solution of the e<jU3tion is )'
2. Hence, x
0 is the only
critical point of the intplicitl)-<lefincd function. Since the function must be continuous at _.. = 0
(Theorem 3.6), values of yare close to 2 when va.lues of x are close to zero. It follows that the
derivative changes from a positive quantjty to a negaLive quanlity as x increases through zero,

and the function has a relative maximum m x = 2.

I EXAMPLE 4 .8
The Be<tltie-Bridgeman equmion or state ror an ideal ~..,; relates pressure P and volume V
according to

p =
where R

RT
(I _CT)
[v+ B(I _~)]
_~vz (t _ ~)
y2
V3
V
V '

0.082 06 is the universal gas constam. T is absolute temperature of the gas. and

a, A , b , B , and Care constants. For air, a = 0.01931, A = 1. 3012, b = -0.0011,


8 = 0.046 I I , and C = 4.34 x 10'. ForT = 300. plot grnph or Pas function of V on

:s

the imcrval 300


V
rel:uive maximum.

IVlii!lil

:s 400.

Veriry that to one-decimal-place accuracy. V

= 373.5 gives a

Relo.tiw; Dil.'{itmun of 8 .:-altic-DridsemO\n cqllalion -.>f Slate

,p

O.tl4'15
M~9

0.0485

0.04S
0.0475

360

340

400

380

0.0465

SOLUTION The plot or the runction in Figure 4.24 confirms o relative maximum r1car V =
370. To verify that 373.5 is the critical I>Oirtt yieldin~ this maximunL we calculate

P '(V) = RT (- :

+ RT

~:) [ V + 8 (I - ~)]

(-1_CT ) [I+ Bb] _A(-2_ + ~).


v
\f2

V5

Vl

Vl

Since P '(373.45) = 2.29 x 10-s and P '(373 .55)


- 1.19 x 10- ', it follows (by the ?ero
imcrmcdime v-alue cheorem) thm, to one decimal place, the critical poinc is 373.5 .

...-...
EXERCISES 4 .3

In Exc:rdses 14-.4 find all critK:.al poinlS of the function and dctcnninc
algebr.Hcally with the first..(l.erivath'e lest which C..'fitical points gi\'e re i ~
alive maxima and rdati\'C minima. Usc a plot to confirm your findings.

I. f(x) =
2. /(x}

3.

(x}

= 2.r
=

x'

x'- 2< + 6
3

.. 12. f(.<) = -.--

.r + I

+ IS.t + 24.< + I
.r - 4.r1/ 3 + 2.t 2 - 24.r
5

x2 + 1
6. j(x) = - x- 1
.

8. j'(x) = x1 -6x2

.. .13. f(.<) = 3.<' - 16.< 3 + l8x 2 + 2

x+ l
5. j(.T) = x> +
8

4. j(,,} = (.T - 1)

7. f (x)

+ 12x+9

9. f (x ) = x 3/ 3 -x1/ 2 - l .r
JO. f(.,) = x 113
H. f(.<) = sin2 .T

= .v,......-,:
I -X

14. J(x)

=x+-

t 5. J(.r)
2x'- tS.r'
* 16. /(.<) = lxl x
2
17 .((x} = (x- 1) ' 1

( X- l) l

18. .((x} = (X + I )'

+ 6x + 4

* 19.
* 20.
* 2 1.

+ 2) 3(x (x ) = x + 2sin.r

f(x)
f

f (x)

... 22. f(x)

* 23.

f(x)

(x

defined i111plicitl )' b)' the cquaaion 2x 2 - )':; +X.\' = 4.

~)l

2.;t='"Yf defines )' implicitly a.s a. function


of x . Find crilic.ll points of the function ~~ which the tir.u deri\'Ui i\~
\"3ni.shcs by(a) using implici1 differen tiation. and (b) findinglhce~ plici1
delinuion or lhe funclion.

54. The cqu3tion }' =

= x''' + x
25x 1

=r +

* S$.

(.r - 2) 2

8)

j(x )

I + x +x' +x'
* 24. I (X) = -'---:-':--;,.:...-1 +x~

+ 26. f(.r) - '' -

* 2M. f(x) =

f(x) =<in' x oos.r.

+ 32.
., 33.

:s x :s 2:t

[n E>:t r~iscs

0 <

function.
59. A fu nction is discontinuous at a poinl if. and o nJy if. i_t hi1S oo
dcriva:ti\c at the point.

36. f(x) = x ln.<

= x 2 1nx

.!S. f(x )

= u -"

39. f(.r)

= u -'

40. j(.<)

= x ~"

* 41.

= x' - Tan- x

~ 42. f(.r) = Cos- (l.r) - 5x 2

= Lx j

60. A function can ha\'e a relalh-e maximum and :1 relathoe mmimum


at the sam! poim.

* 61.

lfth'\: deri\-ali\c of02 func;tion chang~:~ ::..ign when ~ing tltruUgh


a point. the point mus1 yield a relati~c extrtlllWU for the function.

43. f(.r) = lr(x - a) (Sec equation 2.7.)


44. f(x )

57-64 <klc:rm.im; wbc;t}l(.T abe stcuc:mc;ot is lrUC or raJsc.

58. When ll function i.s defined only on lhc intcnal a ~ x !::: b,lhe
ends x = a and x = b must yield rdati\'C ma.:~im.a or minima for- lhe

x < Jr/2

37. f(x)

f(x)

PointS or discontinuity of a fu nction arecriatcal if. and only if.lhe


function is defined at the point of discontinuity.

0 < .< < tt/ 2

0 < X < 2..T

35. f(x) = '' '

'
+ bevf

(b) Docs the function h.-c u rclati\'C minimum off (0) = 0


and a rolat.i\'( ma.'<imum or /{1) = l '?

(2x - l )(x- 8)
(x
l)(x 4)

+ 34. f (x ) =X~ sin1 X.

* S7.

= 2esc.< - coL<.
f(x) = e.cx + Sscc.r.
I (X) = ll!ll.f

/(.<)

x ' i< defined only lbt 0 :S .r :S I. are x = 0


and .x = I cri1ical points?

(a) If f(x)

:10. f(x ) = <in x +co<.<


+ 3 1.

where a > 0 and b > 0 an: constams. Pro'" that f(x) has CJ<<>etly
two critical points. one the negative of the other.

x'

x'''

.r'
27. f ( x)= - , x' - 1

* 29.

AnN wave solution or the Burgtrs equaTion in Ouid dynamics is

of the fotm

..L
= ( X~
Jxz + 100

25. f(x) ~ ,,>i<-

.l'

:t 62. If :1 function Jus t YO rdo.tive maxima, it must ha\'t 3 rdcnhe nin

imum bctw~n them.


.., 63. It

!Sec Exct~isc 68 in S<xtion 1.5.)

i~

possible for every point in til'.: c.lumain of a funclion

LQ

be

criticaJ.

* 64. On an mterva1 of fin ite length. a nonconslant funct ion can ha\'C
only a finite number of critical points at which its derivative is cquallo

In lli.crtiscs -45- 50 y is (ii;[mt-d implicitly it'S a function of X . f ind all


crilic:al points of the runnioo at \\ hich its der'hath-e h equal to tcro.

2Cro.

anll classify each as yielding a rcla1ivc maximum or minimum.


45. x" + YJ

47.

+ ."s =

46. x 2 + y3

-18.

x y + .ry' = 2
3

/ +xy' = 1

.. so. x y"+y =

49. x'.r + .v' = 32

i .t.

+ y =4

I.

y~O

In Excl(:ili:CS S I-S2 Rnd all c:rilical point" a1 "hich the fiNl dcri\'(tli\'{:
vanishes for any fune1ion, witn y :ts ckpcndcnl variabfc. dcfir.cd im-

plicitly by the cquotion.

51. x1

I * 53.

+ 2xy + 3/ =

52. x' y

+l

= 4x

Find. accuraLe 10 fou1' decimals. aU cl'itical points at w hich the


ti1'.SI derivative vanishes fM any function, wilh y as de.peodenl variable,

65. Vcrifytbalt.hcfunction f(x) - .x 3 +cos.t l:wstwocritic.dlpoinb.


one of which i:, .t
0. Find d.c odK..T c..-rili~al point currc,.'1 to dttc
cJccimal p laccs.

66.

n~e g~

equation or Dicterici relating s~urc P and \'Qiwnc V'

is

where R is the uni\"Cr$:.1.1 g~ COO$lant. T l$ absolute tempcnuu_re. Q.nd


a :> 0 and b > 0 an; con~uutts.
(a) Verif y th at the fun ctjoo
\'ided T < a f (4b R ) .

P (\f) has two Cl'iljcaiJ)OinLIO pro-

(b) Verif y that when T = Tc = a / (4bR ) , there is oneC1'itical


point at which P = aj(4b2e2 ) .

256

* 67.

Ch:apte .& ;-\ pplil.::uiun.s ..-r Differentialion

Ia) The Diric!J/nfimctiOJJ in digital signal processing is

ft(w) =

sin ((vl./2)

L sin (w/2)

where L is a consHml. Pl01 it for L


10 on !he imerval
- 10 .s (I) .S lO. Use the J>lot to aid in ans wering. the
lOUowing quc:slions.
(b) Is f 10 (0) dclincd'1 What docs the "'ph sugscst for the
limit or fL(w) as w .... 07
(c:) ls lz.,(w) C\'t:ll. odd. or neither even nor odd?

* 75.

+ y2)1 =

x ' y describes a b ifotium (figure


below). Find dlC points on the curve farlhcst from the origin.

The equation {x'

(dJ I< / 10(w) periodic) Whtll is it< period?


(e) For what smallesl positive ' ' olucof w does / oo(w ) have iiS
.s.nulllesLpositive relative mtu:imum?

(() What are the zer<>< of f L(lO) 'l

iii ,

68.

(a) Plot agruphofthcfunctionf(x) = c- sinx. x =:,0.

(a) Verify that .t

= 0 is a entical point lbr the function

(b) For whllt \'3lucs of x docs /(x) ha' ~rel ative extrema':'

x sin (1/x).

I (X) = { 0.

In Exercises 69- 72 lind all critical points of the fwtction correct to four
decimal places. l)c!ennir.e whe1hcr each critical point yields :l relllli\le

ii*

6~. [(X) =

i 70. fix) o

intrC<bCS through J.'

+ 6.t + 4X + I
x' - 10x 2 - 4x + 5

(c)

jjj , 71. f(x) = x' - 2oosx

Docs x f(.<)?

= 0.

0 yield a relative maximum

01

minimum for

iii* 77. Repc:ll Exercise 76 for the function

x'- 4

/(.<) = { lx sin( l /.t) l. x;o!O


X ~ 0.
0,

/(X)= (x ' - S.< + 4)'


.+. 73. 1bc di~tancc from lhc

origin Lo any point (x. y) on tlw.: curve


C : .1' = [ (x) isgi,'<nby D = jx'+y'. SilO\' that if P(.'(o. )b)
is a jXIim on C for ,.,bicb D ha$ u rduti,c cxtrcnt\ml, then the line 0 P
is pctpendicular to the ta.n~cnt lint to C 3 1 P. Assume that f <x) Ls

i*

78. Repeal Exercise 76 for the function

f(x)

x2

= {-lx sin (1/X) I.


0,

diffc:t'd"'tiable, nod thlt the curve doc~ n01 p:l$$ tlvoug.h lhe o rigin.

* 74. The equation (.r! + y 2 + x) 2 =

x=O

CbJ Use a plot to show that f'(x) does not change sign as x

.t'

;o! 0

bocausc /'(0) does n01 exist.

maximum or a rclati\'t: minimum.


1

+ y2

describes ;l <'tJrtlioill
(shown in 11\e fOllowing 11gure). Find the rna~imunl )'COOrdina te for
pointS on the cune.

**

X# 0
X= 0.

79. Show lhalthe function f(x ) = x 3 + p:c + q h;a~ three di~inc.1


zeros if and only if 4p1 + 21Q 2 < 0.

14.4 Concavity and Points of Inflection


In the fim three sections of this chapter we concentrated on the first derivative of a function. In
1his scc1ion we tum our ane.ntion 10 lhC.SC(X)Od deriva1ive.
Consid~r the function in Fig un: 4.25.

Tf we drdw tan gem lines to the graph at the fi,e poinL~

c 1, c1 . l'3 , c.1 nnd c5 il is clear that 1he slope is greater al c5 I han it is a1 c.1 g reacer ac c.. 1han
a1 C'J and so on. In fnct, given any two pointS .v1 and .r1 in the imcrval tl < x < b. where

Xl > X~o the slope at

X2

is greater than at Xt.

j'(x2l > /'(x,).


What we ;ue saying is that the function J'(x) is increasing on the interval a < x < b.

Mii)ICJII.J

EJ1"'

liltij'l;l WJ.W

J'(X) iBCrc.:asing ~)n

j'(x ) decreasing on

61#Jij;ICWJM

f'(x) <.:hanges

b<X<.i'

(l <X< b

)'

~
/
~i
'
. . .
C X

(/

(/

'The tlrst derivative: of the fun~Lion f (x) i n Figure 4.26 is decreasing on the. inte.r val b <
c. In Figure 4 .27, we have pieced together the functio ns in Figures 4 .25 and 4.26 lO form
a function llwt has j'(x ) inc reasing on a < x ::; /J and dccrctSi ng on b ::; x < c. We
>ISsume that the left-hand dcriV!Itivc .1:.. (b) in Fi~urc 4.25 anti the right-hand deriYati ve .f~ (b)
in Figure 4.26 are the same, i1l which case /'(b) exists jn Figure 4.27. \Vc give names to
i n1crvals on which the first derivnti ve of a funcl io' is increa~ing and decreasing,. and poi nts that
separate such i ntervals i n the follow ing definitio n.

<

DEFINITION 4.6
The graph of a function .f(x) is said to be c:on cac upward on a n interval f if /'(x ) is
incrca~ing on

I , tlnd concave downward on I if .f' (x) is decreasing on 1. Points on a

groph that sepanu e imervals of opposite concavity are called points of inflection.
11tc graph in f igure 4.28 is C\lllcavc upward o n the intervals

IJ

~ X~ C,

d :0

.T

< e,

e<:r < f,

illld concave dowmwu:d for


{I

;x ~

b.

j :=;.r < g,

X < x :'! II.

The po ints x
e and x
g are not incl uded ; n these intervals t>ecausc f'(x ) is no t defined at
these poi n L~. Points on the curve corresponding to x = b, c, d, and f arc points of' inRection:
those at". = e and g are not. T11ey do not separate intervals or opposite concavity.
The tangent line has a pecul i<tt' propeny at. a poinc o f i nflecl ion. To see it, draw the ran gent
l ine ut the four points oJ in lle,tion in Figure 4.28. The tangent line actually crosse~ from one
side of the curve to the other (somethi ng many readers think n tangent line should not do).
Mjlclli;IJFFII Intervals
on wh.ich gr::aph i5 C\)DlCil>'C upward
and ct)l:<:avc downwnrJ

"

258

Ctoplc.-4 A&
.,Pi..::alioas or Diffcrenti:uion

I?
point of infle.; lion

The cri1icai1X>in1S x =a and x = c in Figure 4 . 19. where /'(x) = 0, and 1he c rili<.:lll
I)Oints .< = e md x = g. where /'(x) doe~ nol cxiSI, do nOiyield relalive ex1rema for .f(x).
Cri1ical point~ of lhese types are illustralcd again i n Figures 4.29 and 4.30. The 1angcn1 line m
X = a i horizonlal in f igure 4.29 [f'(a) = 0). Since the graph is concave downward 10 the
left of x = a and conca"<' upward to the righ1, thc point (a , .f(a)) is a point of inftcction. We
call it a horizontal point of inflection. The point (a , f(a)) in Figure 4.30 is >I> a point or
inflection . As rhe can gem line at this p"int is vcnical, we call i1 a ,erlic-dl point inOeclion.
l11 Seclion 4.2 we stated that a function is increasing on an imenral if its first derivative
is grc3ter th3n o r equal to zero on that interval. and equal to zero at on]y a fini te numhtr nf
points; it is decreasing if its firs t derivative is less than or equal tO zero. Funhcm1ore. points
t hai SCJ)(:m:Uc inter\'als on which the funct.ion is: incn:a."iing and decreasing are points a1 which
t he first derivHi i"e is eithe r equal to 1.ero (and changes sign) o r does not exist. We can u.c these
ideas 10 locale a nalogously poims o f inftection and iniCN lls o n which the grnph of a fUIIClion i~
concave upward and concave downward.

or

IJ

W)IiJ m_ .U:W
poin1 cf il)flecliort

Veniel

y
y

=fix)

(11, ,(((1))

Test for Concavity


a

(i ) 'Tl'le graph of a func1ion f(x) is C{lllcave upward o n an interval I if on / ,

f "(x) ~ 0

(4.7a)

and is cquai Lo l-ro ul only u fi ni te number of points.


(ii )

n,e graph of a funcrion j(x) is conca'e lkJWI)wtml Qn an inlen al /


1

if on/,

.f"(x) ~ 0

(4 7b)

and is equal lo zero m only a finite number of poi1US.


(i ii) Points of inflection occu WIICI'C

j" (x) = 0

(4.7c )

and /'' (,t) changes s ign, o r perhaps a lso where .f"(x ) does not exist.
Consequently, j ust as che firs. I deriv:uive of a fuetction is used to determine it...; relative extrema
and intervals on which il is increasing and decreasing, the second derivalive is used co fi nd
points of innect ion and interv.d s o n which the graph of the funcllon is concave upward and
<.:onc,l\'e downward. l11is paral lelism between the first and second derivatives is reiterated in the
fol lowi ng t>1ble.

First Derivative

Second Derivative

l. A rela1ive extremum occurs at a point


wher<. j'(x) = 0 and f' (..r ) changes sign
as x i ncrease s through the point.

where /"(.<)= 0 and j'"(x) changes sign

2. A rela1ive t '< tremum may

al~

I. A point of infle.c1ion occurs ill a poi1H

as x increases through the point.

occur at
a poim where /'(x) does not exist.

2. A point of inflection may also occur at


a point where /" (x) does not exist

3. j (x) is increasing o n a n interval if

3. f (.r) is conc:we upward on an interva l


if / " (.r) :':; 0; j' (x) is conca"c downw<U'd
on an interwl if f "(.r) :<: 0. j"(x ) may
be equal to zero at o nly a finite number of
poilllS in e ither case.

j'(x) :':; 0; j (x) is decreasing on an

i nter\ al if j '(.x) :<: 0. j'(.x) may be equal


to zero at only a finite number of pointS in
e ither case.

These discussions lead to what is o flen c-alled the second-deri11ative test ror determining
whethe r a c ritical point at which f'(x) = 0 yie lds a rela tive maximum or a relative m in imum.

Second-D erivative Test for Relative Extrema


Suppo:>e f" (x ) is continuou~ on <Hl ()J>trl intel'\';rl CV1lt<1ir,ing a criticcrl p4Jint .\'o of l1 function
j(x) 01 which .f' (xo) = 0. Then:

(i) lf f"(x0 ) > 0. then x = x 0 yields n o"elntive mi11imuno for f (x }.


(ii) lf f ' ' (xo} < 0. then X = .to yid ds a relative maximum fo r f (x).
(iii) 1r / ' ' (.10 ) = 0. tl1e-n no conclusit>n cam be made.
In (i). f'(x o) ;;; 0 implies that x = x 0 is a critical point with a horizontal taogc.nt line;
f "(xo) > 0 im11lics tho! around x = x 0 the cu"c is concove upward. and x = xo therefore
yields a relat ive minimum. Simi larly, f''(x 0 ) < 0 at a critk.al poim in (H) implies that the
curve is concave downward M d therefore h3S a relative rnaximurn.
To show thlll the !<:;;t li1ils in cosc (iii ), C()ft;idcr three functions f (x ) = x 1. f (x ) = - x 1,
and f (X)
x' in Figures 4.3 I, ~.32. and 4.33 ""'pcctivcly. For Cllch. J'(O) / " (0) = 0:
yet1he fus has a rehui ve Jl1inimunl, the sectwtd 1:1 relalive mt1x.inmn11 a nd the third u horizomul
poim of intlectiOtl a1 ,.\' = 0.

a:Dll!J,GEliiJ
mi11i tnum ar

.r =

;\ rel.alive

ll.El!l

tit'J!J"il"1f A rch11he
x- 0

puinl oi iofk..:tiou llil

nu1\imum ll

Hlll'iznnwl
0

.r

.r

The St!COild<.lcrivmivc lest very quickly determines the IHHUrc or 3 Critical point Xo when
/'(x0) = 0 uncl J'' (,<o) :/: 0. For iol,tancc, the second derivative of the func1.ion / (x)
2.r 3 - 9x 2 - 23.x + 6 in Ex:omple 4.6(a) of Section 4.3 is / "(x) = 12x - 18. Since
/" ( (9 - ./2i9)/6) < 0, the critical point x
(9 - ./2i9)/6 yields a relative maximum,
and l>ecau.<e / "((9 + ./2i9)/6) > 0, x = (9 + M )/6 give.< a rebuive minimum.
A more refined test todctcm1inc the nmurcof11crilical poinl x 0 when f'(xo) = f" (Xo) = 0
is discus5ed in Exercise 35.

I EXAMPLE

4 .9

For each of the following fu11<1ions. find all points of inflection and ioucrvals on which the graph
of Lhe function is .. oncavc upward and concave <.IO\\'Ilwant.
(a)

2
f (x) = -xl - 6x~
3

+ 16x + I

( b)

f(x) = x 6/S

+ x tls

SOTUTION
{a) The graph o f the func.tion in Figure 4.3 4 ind icates t hat thc.rc. is o ne p<>int o f inftcction.
To find it. we solve

'
0 = ,( " (x) = -d (2x-12.r

dx

+ 16)

= 4x - 12,

and ob1ain x = 3. Since .f" (x) changes sign as x passes 1hrough 3. x = 3 gives Lhe
point of inRection (3, 13). The graph y = .f(x ) is concave downward fo r x :S 3
Isinc-e .f"(x) :S 01 and cQncave upward for x 2:. 31/"(x) 2:. OJ. S ince .f"(x) is

dellncd for all x, lhere can be no olher poin1s of inflection.

260

ChJptet 4

Applkationsof l)ifferc.ntirniun

ll1!1il;ltt
)'
18 '

(ntm-als on

~hicb

= /(.\') is ccnta\te upward ald downv.aN

y jx>-6xl+ 16x + I

16

12
10

M:Uclil.l=<lf.JOm
Intervals on which y = j(x) ls concao.e upward and downwan:l

ljjlf\IJ.It *'"1M

) ...-6fS + xiiS

uj
1.0 .

o.s
2

-1

0.$

- I

-2

(b) The plol or the fuuc,lion in Figure 4.35a suggeslS a poim of inflCClion between x = 0
and x = 1: doe CUt\'CscentS lobe concave downward for small posi1i vc x and concave
upward for larger x . The graph on the smaller interval in Figure 4.35b does not make
this any clearer, bu1 it suggests that (0, 0) is a poin1 of inflcc1ion. 1b confirm these
suspicions~ we first solve
0

f " (x)

.:!..._ (~xi/S +

dx

~.t-.1/S) =

6x - 4
25x9/S

and fi nd x = 2/3. Since f "(.t) changes sign a< x


gives a poinl of inflcc1ion with /(2/3)"' 1.5.

passe~

lhrough 2/3. x

= 2/3

We also note lhm f'(x) and / "(x) do no1 exisl a1 .t


0, ahhough f(O) d~ (= 0). Since
f" (.t ) changes sign as x passes through 0. (0, 0) must also~ a )>Oint of inflcion. 1bc fact
1hat
lim

.r-n

f 1(x) "'

lim

r-fl

(6- xn
S

+ -SIx-ts)

= lim 6. +I
"' oo
.r-o Sx"15

indicalcs thlll (0. 0) is a Yertical poiIll of inflcc1ion. With the poi nil> of inflection ncn in place.
we can say th:Jt the graph is concave upward for .r < 0 and .\' ~ 2{3. and concave downward
for 0 < x ::; 2/3.

I EXAMPL E 4 . 10

n 1e Van der Waals equa1ioo for an ideal gas rela1es pressure P and specitic volume V by
p "'

RT
a
V- b - V 2 '

4.4

V.,~n

COIOC:wity nt'-d Point$ of l11ft ecc:ion

261

where T is absolute 1emper:uure of the gas. R i s the uni\'ersaJ gas constam, and n and bare
positi\'c cons tants that dcr>cnd o n the gas. S uppo:;sc pressure and volume mcasurcmcms arc Utkc1l
for a specific gas a1 Vllrious 1empermurcs, a nd lhc results arc ploncd as in Figure 4 .36. Each
curve co1Tesponds LO a fixed value o f T . One of the curYc.s has a horizontal J:x>int o f inflection.
II can 1>e used 10 de1em1ine values of a a nd b for 1his gas. Suppose the 1erupernture o f the
gas for this curve is Tc. and
i~ the voiUIIIC, whidl giv~ the horizontal JX,lint of inflection.
Find e xpressions for ll and b in terms or 7~ :md Pc = P{ Vc) , called c ritical temperature and
pressure.

,lc r \V.u.b C"qUUtioo for ''Uli .JU.)

v('

tcmpc:ntur~

dP
SOLUT IO N

RT
(V _ IJ) l

Since d V =

point of infle.c tion occurs for

2a
dl P
V', flnd cJVl

2RT

6a

( V - IJ) l - 1fT' a ho riwntal

= Tc and V = V,. if

v
0=

To fi nd

RT,

2a

(V, - b)2

21?7,
0 = (V, - b )'

+ Vj'

a nnd b, we sol vc eac h e quatio n for a and eq uate rc.s ults:

3(Vc - b) = 2Vr

b=

Vc
3

Subs.tiluti on of lhis. into the first equation gives

a =

We now have a a nd b in terms of 7~ a nd Vc. To replac e Vc w ith Pc, we usc Van dcr Waals
equation to wri1c

'Vhcn we s ubstitute the c:tprcssio ns for tl anc.l

b into this equation we obta in

3RT)
RTc
This then gives a = 9RT
__c ( _
_
c = 27 R T' and b = __
8

SP"

64Pc

8Pc

EXERCISES 4 4

lo Exct':iscs 1- 14 dctcmli nc where lhc gruph or the funclion is concavc


up"~ is conca.\'C downwan.l, and hu.$ J>Oi~ tS ,-.,f inftcclion.

x' - >x' -

I. f(.<) =

2. f(.<)

* 3.

= 3.r' + 4.<'

.\' l +4

* 6.

f(x ) =

~- f(x )

= x' - 4in x.

. :5 b.
+ 32. If /(x) and g (x) arc nvicedi.N'crcnliablcand c.oncavc upward on
an i111crval I. is f(x)g(x ) C{)nc~e upward on I ?

x' - 2sin.r

f(x ) = .t ln.r

=~'''
f(.<) = x'"

II. f(x)

13.

x .:S x 0 , increasing on Xu < x :5: b,

sin x

7 . f(x )

* 9.

31. /(:t') is increa!)ing on a ~ x ~ xf conca\'c tlownwanl on a ~


x < -'0 c.lcc.rca~ing on .ttt ,S x :5 b. ami conca\c upward on X(l <:

+ coSX. lXI < 2tr

.r' -

'28. f(x) is dcca'C::.sing <tn I . f(.r) is conlinuous :iL\'o, but /'(x) is


w~onl inuo ~ al .t'u.

... 30. f (.t) i) concuvc downward on a ~ x <xu. conca,c downward


on Xo < X :::: b. but nt)( conc.u.vc c.Jownward on I .

= -x .- I

/(.<) : X

*'

29. f(x) is incrca.:,lng on a


but not incrc.usiog on I.

= x' + --;
x-

5.

- 24.< + 2

/(x)

4. f(x )

3.< + 5

27. f"(x) =:: Oon I and .f(.<) isnolconcavc upw.-don/ .

* 33.

l.rl < 21f

10. f(x ) = .r 2 1nx

12. f(x )

14.

=..,-:.

j(x ) = x'- e-

If the functions in El:ercise 31 are also increa!l.i ng on I . is

f(x)g( x ) CGJ\Ca\>e upward on /"1


4t

34. Repeal Example 4. 10 for Dieterici's equation in Exercise 66 of


Section 4.3

* 35. The second.derivaLive test fails to classify a critical point .r0 of a


functioo f(x) os yielding n rcl:u hc maximum, a relative

minimum~

hQrizonull pQinl of inRccliQn if f'(x 0 ) ; f''(x0) ; 0. 11>e


foiJowi.o g L~l c.an be used in such ca~::;. SupJXJSC f (x) has dcriv.al,ivq
of ull ordcr:s in an open interval arou nd Xo. and the fi~t u dcriV'dtivc.:,
all vanh-.h at x0 butlhe (11 + 1)., d<:rivath"C at x~,~ is noLtCI'o. If we
denote the II L't deriv:~tivc of f(x) :11 x 0 by /(ft) (x0), these c..-ondilions
3rc
(>r

In Excn:istS 15-22 usc the second-derivative test to detennlnc whether


crit.icnl points when! f ' (x) = 0 yield rd:uivc mu.x.iltU or relath..:.

tu.inima.
15. f(.<) = .<3

3x2

3.< + 5

16. f(.< )= x +-

0 = /'(>ro) = f''(JCo) = = /

'

17. /(.r)

= 3.<' - 16.< 1 + 18.<2 + 2

Then:
(i) II 11 is: even. f(.r) has a horirontal point of infi(Xtion at x0

IX. /(X) : x >l - x 'i'

(ii) II tt i~ t'ICidand

19. .f(.r) = .t ln .t

20. f(.r) = x ' ln.<

* 21.

n.

/(x)

= xe"

f(x)

(iii) If n i~ odd and /(,-tl)(.~'o)

ha~ arcl:uivcrnini

< 0, /(.'()

h:~~ !'I rel:uive max-

imum tU x0

x' e- 1'

A I)TOOf orthis rcsu.h require~ the usc of material from Chuptcr IOand is
thereforcdela)'cd untiltlmt timc (see Exercise 16 in S..x:tion 10.3). NOlc
thai rhe second-deriwuive I C~t i s the speci al C.1!i.e when n = 1. UtC 1t1is
t!S'L todetennine \\rhether tritical pOinl$Of L~ fol!owing. f(tnCti<.lnSyie!(t

Is: l.he graph or a func-1ion concave upward, concave downward,


both, or neither on :.tn interv:ll I if on I its sooood deri \:llive is slw::tys
oquaJ to zcru?

rclati\'c nnx.Ur1a, rch:t.tivc mjnimu, or horiwnw1 points of inRcction:


ta) f(x )
(x'- 1) 3; (b) f(x ) .Y ' ~.

24. Pro\-c that tbc c;utvc y = 2 <:oo x pusses through all points of
inflection of the curve y = .( sin x.
25. Show that C\'Cry cubic polynomial has cxaclly one point of inOcx..'tion on i1s gr.lph.

f (.\') is increasing o n

I a nd discontinuous

fJI Xo.

36. Show !hal 111:: fX'ini>Ofintlcelion of f(x) = (k- .r)/(x2 + k 2).


where k is a const.-1n1) all lie on a s lraight line .
.f. .f.

D1aw the graJ)h or a function f (x) that is defined cve1ywhere on tll e


intetval I :a :S .r :S b. and tl\al i)()Ssesses the J)rope11..ies in Exettiscs
26-3 1. In each case assume tl-.at a < .ro < b.
26.

[ <' t i)(Xo) > 0, j'(I)

Mlum at .t:0 .

* 23.

11
" (-<o).

37. PIO\'C thal if a cubic polynomial has both a ttlati\'c max.in\un\ atttl
a relative minimum. then the poinl of inftct"tion between these cxhcma
iS lb e rnidpOinl Of lht! line St::gmt::.nl jOining thcrn.

**

38. Show 1b a1 the g;raph of Ibe function f(x)


sin(.\' - sin x) hns
an infirU1e Rl.l(nbt::r Of horitonlaJ pOinl$ Of iuOcttiOn.

".$ Ur.1wing. Gr.1pbs with Cak.:lllus

.... 39. Showttwllhccqu:llilln .((x)


{I > 0, b > 0, and 11 ~ 1 arc

= .t" t- 1 - b",, +ab''

posieivc solu1ion' i f and only if h <


.,..., 40.

hub
nb

(.."OUSitllliS,

('1

+ I) l u

= 0, when:

f(;r )

= {;r' sin (~).

0,
wus shown lobe'"''" u1.r

(b) Stlow thai f''(O) doc.'$ not c:<i~t 1/i~tt: See EJ.crci,:e 47 i11
S<ct ion 3.9.
(c) l 'l the point (0. 0) 2t rclatiYc 1111lX111lum. a rclttivc minimum,
a OOrizonL!~I po~ nt o ( i nflection. or none of th(sc?

C:(liCIIy IWO di~.iJK.1

l)f

- ,.

(a) Jn Example 3.12 of Scceion 3 .3. Ihe derh'<.llive o l'lhe ftuX>


tion

_,. ~ 41. Show aualyLico.lly Lhat the equation

2s;n 9
x ;o! 0
X =

263

= 0(1 I cos())

hou, no WI111K>n in the interval 0 ..:. 0 <

1r .

=0 !i.e.. /'(0) =01. II follows

th:n Lhc grJph of the func1 ion h:u a horizonlaltangcnt line


:tt (0. 0 ). Dww the gn1ph :.nd the tungcnt line.

14.5 Drawing Graphs with Calculus


I n Sec1ions ~. 3 and 4.4, we used plols of fullCiions lo help yo u see Ihe geomelric u11eqJre~11ions

of c ritical points, relative extrema. <'oncavity, anti points of inHection. In this section we use

critical points. relative extrema. increasing anlll decrea."ing functions. corn.-avity. and points of
inAection to draw (or sketch) graphs of function-s. \Ve assume no access to graphing calculators

or computers for the firs t four examples.

I EXAMPLE

4 .,,

U<ecalcnlus 1odraw a graph of the func1ion f(x) = (.r - 4)jx 2.


SOI .I TfJO;>;

Wi1h "" x interccpl cqual10 4. and the limi1s

lim /(x) = - ex>.

<-0

lim .f(x) = - ex>,

. . - o

lim f(x) =
.A--oo

o- '

lim /(.r) =

.r-oo

o '

we begin our skclch as shown in Figure 4.37a. Utis infommion would lead us 10 suspcct1ha1
1he graph should be con1pleted as shown in FLgure 4.37b. To verify this '"'e fi nd critical ' ){)i lliS
for j'(.<) :

r, (x) = x~( l ) -

= .

(.t - 4) (2.t)

x4

x (8 - x)

8- x
~

Clemi y, x = 8 is lhe only crilical poinl of lhe func1ion, and since the firs1 derivative changes
from a positjve quantity to a neg~l tive quantily as .\' increases through .l = 8. lhcre is a relative
maxi munt o f /(8) = 1/ 16. Our skc1ch is now as shown in Figure 4.37c.
Figure, 4.37c makes i1 clear lhat lhcre is a poi111 of infteclion 10 1he righl of x = 8, which
we could pinpoint wi1h .f"(x ) :

/"(.r) =

x 3( - l ) - (8 - x)(3x 2)
x6

2(x - 12)

Since .f"( 12) = 0 and .f"(x) changes sign as x passes lhrough 12, ( 12, I/ 18) is lhe poinl of
inftec1ion. l, igure 4.37d comains 1he llnal graph.

264

Claactc .a Appli..:llions vr DIIYerc:nti:lioo


MjHIII;JXIfl
.t

f<xl= - x.

St<'ptS

in dr-,J.,,,:ins

FlOURS. 4.37b

p-4ph of

I,
,
\I
FIGURE 4.37d

FIGURE 4.37c

1116.J--.c:.:;:;:.;::=
8 12
X

I EXAMPLE 4 . 12

..._...
Use calculus to dra\v a graph of the function

(x)

= x 513 -

.r2/J.

SOLv'llOI\ We first note thntthe three points (0, 0), (I, 0), and (-I, - 2) are on the graph.
Next we add the facts that
lim f(x) =
x---w

- oo

lim J(x) = oo,

and

.\-e<t

as >hOwn in Figure 1.38a. We canol()t be sure that the concavity is as indicated. but thb will be
verified shonly. To find critical points or j(x) , we lirst solve

0
1l1e only !)(Jiution

i~

=f

(x)

213

= :/

= 2/5. and because

J.r

/ '(.t)

- t/l

5x - 2

= 3x'l.l .

~.:hangcs

from ncgathc to positive-as .x

increases through 2/S. we have ll re lative minimum of /(2/ 5) = (2/ 5) 513 - (2/ 5) 213 "='
- 0.33. Since j'(,<) docs not exist at x = 0, this is also a critical point, and we calcula te that
lim J'(x) = oo

x~o

and

lim f ' (x) =

:r- or-

- oo.

This infonnation, along with the relative minimwn, is shown in Figure 4.38b. \Ve now join
the:;c parts smoothly to produce the ske tch in Figure 4.38c. To verify that the concavity is as

indicated we calculate

f " (x )

= (~)
(~) x - fl _
3
3

(:)
3

(-~)_..-.on
3

= 2(5x9xJf!+ I).

Since J"(x) ::; 0 fo r x ::; - 1/5. tile gra ph is concave downward on this inte rval. It is concave
upward for - 1/5 :S x < 0 a nd x > 0 s ince f"(x) 2:. 0 on these intervals. A point of
inflection occurs at x = - 1/5 where .f"(x) = 0 and changes sign, but not at x = 0 where
J"(x) docs not exist. The tlnal sketc h is in Figure 4.38d.

4.5 Otawjrtg Gr.cphs wic.h Calcrlm

llm"' I;IMWtH

Step~

in dl'a.,..i n;: VJPh of

265

FIQURE 4,38b
)

,I

I
..

( -1 . -2) .

I
M41clii;IM41.fW

ljitJII;I#+i ftl

<-0.2. -0.41)
H. - 2)

(-1. - 2)

..

I EXAMPLE 4 . 13

Use calculus to draw a graph of the functio11 f (x) = x + sin x.


SOLlfl ION We c:m add ord inates o f the gnphs of y = x and y = sin x in Figure 4.39a.
This creates o>Cillation> a ruund y
They might look lil<e Figure 4.39b. c. or d. To decide

=, .

which, we find critica l poinlS,

0 = f'(x) = I + cosx .

The solut.ions of this equation arc x


(211 + I)Jr, where 11 is un integer. Since .f'(x) docs
riOt change sign as x passes tltrough these points. they cannot yield relative ""'xima or minima.
They must give horizontaJ points of inAection. The C()I'I"CCI gmph is Figure 4.J9c.

MJC..I!LL a]
j'(x) =x + sin x

l,o~Nbili 1 i e.s

fur graph of

IJCc111,1

../,'
p,' '-

)'
)'

y=x

' '

rr

2rr

3rr

266

Ctulple1 4 Almlic:ationl ol' l)im:~ mimHJt


MJiclil.I>6-IJW

Mflttlll.l

'tt"lltll

/
'

.....'

- 2.7

'

..

.-

.l

-~

2rr

if

J;r

2;r

,T

.l

'
'

3it

.t'

'

--.........

I EXAM PL E 4 . 14

Usecalculustodmwagraphofthe functionj(x)

= (x' + 1)/(x - I) .

SOIX TION The function is discominuous at x = I. We calculate that

. x2 + 1
hm - - -

'"-11 .x - I

= 00

.r 2 +

x-1

X-

lo nt - - - = -00.
I

If we tli ,ide x 1 + I by x - I, we write .f(x) in the form


j (x } =
This shows thai y = x

2
r - I

+ I + - -.

I is an oblique asymJXote for the gmph (figure 4.40a). Clearly we


should find critical points by solving
0

= j'(x) =

I - .,...---~
(x - 1)1

(x - 1) 2 - 2

(x - 1) 2

Titis implies that (x - 1)1 = 2. from which x = I ..fi. Because f'(x ) changes from
a positive quantity IQ a negative quanti!)' as x increases through I - ..fi. there is a relat ive
maximum at .< = I - ..fi of .f( L - ./'i) = 2- 2../'i. Simil<~tly. a rchuivc min.imum of
j( I + ./'i) = 2 + 2.../'i <><:curs 111 x = I + ..fi. The final gntph is ~hown in Figure 4.40h
WjUIII;II(li71..
f(.r)

.t

Steps

111

drawing grn:ph ,,(

+t

=-:;:-:)

- I .... ..
. :;;;:.:.

-I

. .... ;. ... .

.. ----~

'

I:

Electronic devices ca.nnot plot curves that conta in unspecified parameLers. For example, we can
l
use a graphing calculator or computer to plot y = e-ox for any given value of a , but we cannOL
plot th e curve wi1 hoot s pecify i og a value for a. What is appropriate is co cooj ecture the shape
2
of y = e -ax from plots wilh various values o r a, and veri fy Lhe conjecture with calculus. Let
us illustrate with Lwo examples.

4.S Draw i n~ Graphs wi1t1C;.~ l~.:ulus

287

I EXAMPLE 4.15
When two s ubs tances A and 8 a re brought together at time 1 = 0 , they react to form substance
C in s uch a way that I g of A reacts with I g of n to form 2 g of C. If initial amounts of A and
B a re Ao and Bo (where Ao > Bo). then the number of grJms of Cat any timet is given by

whcrck >

O i>acon~tant. DrawagraphofC(I).

SOLUTION

Let us get an idea of the s hape. of the graph by plouing the function with values

Bo = 15, :.:tnd k = 0.1 . Atle.r experimenling wi lh various domains.


Figure 4.41 as the moot infomtative.
Ao

20 ,

~- 0 (
w:unn-t*h. ~ph
30

we arrive ac

C (t) = :.;
-2A.::;::lloc:(:.;t;:---':,.''..:c)
Ao - B(tt' u

c
280

20

10

t5

20

The grdph ha:; no relative extrema. no points of inflection. and is asymptotic lo lhc line

C = 30 . \Ve now determi ne wherher these fea tures remain true for a ll vnlues of Ao > Bo and
k . Critjcal points of C(l) are given by

2 Ao Bo(Ao - B0 )ke-"'
( Ao -

Boe

''>'

There arc no solutions and C ' (t ) always cxis.L<t. sothaLC (/ ) ha.'i no rclatiYcmaxima and minima.
For points or inflection, we consider
,.,

0 = C (1) = 2A 0 Bu(Au - Bu)k

[ (A 0 -

2A oBo(A 0 - B0 )k2 1!-k1(80

B0e- tt)2 ( - krtt) (Ao _

e - k' (2)(Ao -

Boe k')'

Boe-*')( Boke-") ]

+ A0 1!-k1)

(Au - Bur")'

Once again there are no sulutions. and C"(t) :tlways exists. He nce the gmph of C(r) has no
points of inflection.
The graph bcgilts <ll (0, 0) willt slope 2Ao Bok/( Ao - Bo). and is asymptotic IIH he line

The graph is s hown in Figure 4 .42.

268

0:13pler 4 Applic3tioos ofOlffetenli:llil)n

I EXAMPLE 4 . 16
When a drug is injected into the bloo<.l at t ime 1 = 0. it is sometimes <~S>un!W in biomedical
cngjnccring lhat the concentration of the drug in a nearby organ is given by n funclion of the
form

/(1) = k(e-' - ,.-&r},


where k . a. and b are positive constants with b >
any relative extrema and poinL< of inllec tion.
ijlciiiil]

JtEi

PhJt (lf /(t) = ~

1 -

a. Ornw a graph of this function ind icating

~ %1

f(r)

0.2

SOLUT ION l b ge t an idea oft he shape of the grnph we J>lot the function with values a = I,
b
2 , and k
I (Figure 4 .43). To detennine whether this s hape remains constant for all
v-lues of a and b (k has no effect on shape o f the grnph), we begin by finding critical points of
chc funct.ion,

Thus.

ae-ot = be-bt
the solution of which is I = (b find points of innection. we use

a)- 1 ln (bf

(b- u)r

= -b .
a

a). To test this for ma.,imum or ntinimum., and

It is nm obvious whclhcr

is positi\c or negative. If we note. howe\ cr. that at the critical point.


substilllle this into f " (t ), we can sec that

f'"[(b- ar

In (bf a)J = k[a1 e"""' - b(ae- ')J

ae- 0 1

= be- h' ~ and

= kae- "' (a -b).

This is clearly negative (since b > a ), and therefore /(1) has a relative maximum at
(b - a)- t ln (b / a ). Forpointsof inflectionweset

I = -

2 In(~) .

b- a

To confirm that this value o f I gives a point o f intlec.Lion we should check that f ''(t) changes
sign as t passes through this value. This is not obvious. \ Vc can, however, conclude that a point
of inflection must occur here if we examine the infonnatjon in Figure 4.44a. The graph begins
at the origin and achieves a relative maximum at t = (b - a) - 1 1n (b/ a) whe re the graph
is concave downward. his also asymptotic to lhe positive f ax is since Lirn1 _,. 00 f(t)

o+.

Hence, there must be a change in c.oncavity to the right of the maximum, and Lhis can only occur
at t = Z(b - a) - 1 In (bf a). The 11nal gmph is shown in Figwc 4.44b.

4.S

Dr"'ing Gtl(III:S wi1h Cllctth!S

269

)'

""-

~----------------~

- 1 In()
b

(b)/

', (b)

I
---In -

~~

/)-{1

-"-Inb - fl
(t

(I

EXERCI SES 4.5

In Exercises I -22 find all rd:uivc mJXitna and mir1i1nu for Lhc fund .ion
and points ofinftcction on its graph. Usc this tnfoml:l tion, and wh~ t cv.e:r
else is approp,iatc, to dmw a gn1ph or the runt.-t i uc~.
.t3

. t 2

= - - -

I. j (;r)

' 2.1. j (x) .r /(x2 + 3)


24. f (x) = x' + lOx' + 6x 1 - 6-lx + 5

- 2.r

.t~ -

25. f (x)

2. J(x) - .r) - 6..- 1 + 12.r + 9

27. j(x) = - 2- .\' -4

28. f!,r) = x" '

4. j {x ) = 2.r 1

!5x 1 + 6.r

29. f(.r) = lx' - 91+ 2

30.

.. J l. j(x) = sin1 .rcos.r,

0 :5. x :! 2tr

= .r - 3.r 1' '

6 ' /(.r)

9 f< X ) -

(.r

+ HI. J (x)

x'+ 16

II . f {.r) = "---''---'-

.r

x +l

19. f(x)

.r
x 2 - 8x+l2

17. j(.t)- (

x2 + 1
16. f (.r) = - ,X

'f:

35. An emf device producins,conSiant volmge V and w i lh COMhlnl inlcrnl!I I\!Si))l.uncc J' ruainlains c:urn.:m i through u ci1\;1.1it with n:~ i sl:.mc:c
R i1t thc Jigurc: below. where

V = i(r

The power (wor ~ per unillimc) 11CCCS$:try to mn intain this currem in r

= i 1r + ;1 R.
lfwcl.lcfinc PR = l 'l R and Pr = ; z, ~lhCPQ'VCI'lli:i
P = i 2(r

(u)

J)aled in

(.r + 2)3(.r - 4))

+ Cos x

22. / (x) = .rSin" 1 x

+~

+ R).

nnd R is t iven by

- I

18. f (.r) - (x - 1)1/J

)'

21. j(x) =Sin"' x

x 2 + ;r + I

+ 12x '''

(2.r- t )(..r - 8)
.':.(X- ..,.1):'-;(.;_<-_--,4,.:.)

ro---c:
v l -x
8
34. /(.c)= ( ' ; ) J x ' + 100
33. j (x)

12. f (x) = ::.__::.__.:.._

20. f( X) = x+2 sin x

9x

f (X) =

- x'1'

in .r +

x'

(.r- t)'

= 2xll2 -

+ 14. J (x) = - 2- .r - 4

.r

15. f (.r)

32. / (.<)

= x2 +4

~. f (x) = x'' 3 <8- x)

+ 2)'

13. J (.r) =x + -

il

x'

16.r 1 + 18.r 1 + 2

> 7. f (.r) ~ (.r- 2J'(x + 2)

x- 5

+4

= x'- 2x' + 2x 26. [ (x) = - -

3. / (.r ) = 3.r4

S. f (.r)

.f.

ln Excn..'iscs23- 3.l tir.d :.til n:luli\'c muxima !lndminim!i for the function.
UK this inrorma.ion, omd wh:u:ver else is appropri:ue, 1.o drd\v <t gra,~;~h
of lhc function.

+ R)

R aad r . respcc1ively,dmw PR and Pr asrunctions

or R.
(b) Druwu grupll of P(R )

~ -,

L~~--_-_:_

_J

= PR(R) + 1', (11) .

270

Chapter 4 ,.,PJ,Iications of Oifferenlimion

36. The rate of pho tosynthesis P in a leaf depends on the intensity of

chemkal reactor reaches a stable steady-state condhion. a function that


is encountered is

light I o n the leaf act.-ording to

Ml

P= - - - R
I+ K

J(x) =

'

where M > R 3Jld K are all positive tonstants. Dmw a graph of this
function.

= (~,) (2- ~)' e- 1",

r:;:

I * 42.

ln the kinetk theory of gases. Maxwell's speed dl:ftribulion law


defines the probability P that a molecule of gas moves with speed u as

0,

0,

r.

(a) Plot P(v) foroxygenat50Kand300 K usingM = 0.0320


and R = 8.3 1. Do tlley have the same shape? 'Ole fact
tllat most or the 300 K curve is higher thaJl the 50 K curve
meM s that oxygen molecules are more likely to move faster

al higher lempemmres than allower Jemperatwes,

where a > I and I < k < 2 ate constants. Draw a graph of Ibis
fuottion identifying its relative e:<rrema.

40. In q uantum mechaJlics. tile probability that an energy level E will


be occupied is

when:
the ltmpcnmtrc of the gas (in kelvin). M. the molar (tJass of
the gas. ami R, the gas conSI!ml, arc aU conSI!tnlS. We shall have mc.wc
to say about this function when we know how to integrate. Por now.
we simply wish to g raph the function.

speed v occurs is

(b) For aJl)' gas at aJl)' temperature. find the value of v Olat gives
tlle relative maximum and Ole values o r v that give pointe;
of inflection.

43. 'Ole fu nction

e<- r>l(tTJ + I'


f (x)

where E1 is the Fermi energy. k is tlle Boltzmann constam. and T is


tetnperatW'e. Draw a graph of tlle function identifying relative extrema
and point.; or inflection.

i*

* 39. T11e Weibul/ distributio, for the probability that a ccrt<ljn wind

P (E:) =

0,

(b) Now show Lhal j(x)


(b + e"f x)- 1 x > 0 has Ole
same shape. Veril'y tllal it has no relativeexbema. but it has
exactly o ne point of iniicctjon.

where a > 0 is a constant. Dmw a graph or the fum:tion identifying


its relative extrema.

X >

(a) lh get an idea of whallhis funclit)l\ looks like. plm a gl'aph


when b
4 aJld a
10 o n the interval 0 < x S 20.
Find itc; poim of inflection.

r ~ 0,

when:: a > 0 is a constant. Onaw a gn:tph of the funcliOn idenlifying


iLS rcltu ive (naximutn !tnd points of inJlcction.
38. 'Ole radial probability density 11mction lOr tile second state or tlle
hydrogen atom is

P(r)

+ e"#

where a and b we positive constants.

37. Tile radial probability density function for the ground state of tlle
hydrogen atom is

P(r)

4.1. ln Ex!Unplc 3.35 in Scx:tion 3.11 we discussedche(okal formaLiOn


in a chemicaJ reactor. Wbttl we ignc)rcd I here was the fact th!tl tcmperatlli'C usually val'ies in dle reacLOI', and th e talc a1 w hich tile c hemical
is fol':tncd or broken down depcods on tempetatm-e. In odlet words.
lhe situation is (nuch ltl(li'C complicated lbtm we prcscn1cd. When a

where f1 > 0 and Jt me constants is called tlte nomwl probability


deusity function. Draw its graph.

44. Oraw a gtaph or the function


2

f(x)

= I+ xI ++x.r3 + x

14.6 Analyzing Graphs with Calculus


l.n Section 4. 5 we drew graphs of functions without the use of calcul ators or computers. I n
this section we take a di fferent approach; we use technology to plot the graph of a function
and then use caJculus to aoaJyze what we see OJ' do not see on the plot. Remembel' thm '"'e
have illustrated on several occasions Lhat elecuonic output c an som etimes be inconclusive and
even misleading (see fo r instance Example 1.19). Elecuo nic devices can never replace sound

mathematical analysis. Whenever there is a question about electronic oulput. we use calculus
to lind the answer.

4.6

Analyzing Graph~ wilh Calculus

271

I EXAMPLE 4. 1 7
Plot a graph o f the function j(x) = (x - 4)/x 2 and then use calculus to pinpoint signitkam
information concerning the graph.

SOLUTION A plot o f the function on the interval - 10 5 x 5 10 in Figure 4.4Sa does not
show a lot. h certainly indicates that the. y -axis is a ve11icaJ asymptote. and this is confirmed
by the fact that limx-. o j(x) = -oo. The plot also sugges ts th at the x -ax is is a horizontal
asymptote, confirmed by
lim

j(x) =

-~~-00

o-,

lim j(x) =

x.... oo

o+.

The fact that the graph approaches y = 0 from above as x ~ 00 means that the graph must
cross the x -ax is at least once. It does so exactly once since f(x) = 0 only when x = 4. To
get a beuer visualization of the graph for x > 0, we now plot it on the interval 1 ~ x ~ 10, at
the same time restricting they-values to - I 5 y 5 I ( Figure 4.45b). 'rllere mus t be a rela tive
maximum to the right of x = 4 (perhaps more than one). To lind it (or them). we solve

= j'(x) = x 1 (1) -

M:UCJIII-W-Lft

j( x) =(X - 4)/ ).' 1 on - 10 :::S X ~ 10

-10

(x - 4)(2x)
x4

MiiiJiiIJfF!!W

=8-

/(X)

= (X -

4)fx 1 on I ~X :::S 10

1.00 )'

-5

10

0.75

-2

0.50

-4

0.25

- 60

-Q.25

- 80

- 0.50

~~

10

-o.75

- 100

- 1.00
Mii[Jii;I:Wii"i4

- 2tr

j(x ) == (x- 4) /x 1 on - 20 :::: x ~ 20

----~~----t-~==~======~x=========================
-Q.2

.8

- .0
The only solu1 ion is x = 8. Th is yields a rei alive maximum. Wi1h lhegraph concave downward
at x = 8 , and concave upward fo r large x (recall that y = 0 is a horiwntal as ymptote ). there
is a point of inilection to the right of X = 8 . To lind it we solve

= ("(x) = x 3( .

l ) - (8 - x}(3x

x6

= 2(x x"

12)

Since .f"( 12) = 0 ~nd f "(x) changC sign as x pas:;cs through 12. there is n p<.>im uf innectiun
at (12, 1/18). Uy restricting .v-vnlucs c\'en more and plotting on the imerval - 20 ,:s x ,:s 20.
we oblain the lliOt in Figure 4 .45c ; it ho"s all of the foregoing inform,ltion .

...-._

I EXAMPLE 4 . 18
Repeat Example 4.1 7 for the function f(x)

x 51l - x 21l .

SOLUTIO N The software package in our computer does not automatically plot a graph of the
function J (x) x 5/3 - x 2/3 for negative values of x. (It interprets x 513 and x 2/l for n~ati\'e
x as complex numbers.) Your calculator or compuc.cr may do lhc same. To rectify this, define
j(x}
- ( - .v) SI> - (- x ) 213 For x < 0. A plot then looks like th3t in Figure ~.46 on the

interval -I :::; x :::; 2.


A relative minimum is indicated just to the leftof.r "' 1/ 2 . To locate it, we solve
0 "' /' (')
,,

= ~,.2/l
- 3.
~r- t/3 = -:
5x
- 2
3.
3-.r""

fl" "'

1be only solution is .r = 2/5, and because f'(x) changes from negative to (JQSiti,-~ a.~ x
increases through 2/ 5. we do indeed have a relative minimnm of /(2/ 5) = (2/5) 5/l (2/ 5) 2/l = -0.33. The deriv;llive j'(x) does nru exi'! ''' x = 0, but j (O) = 0 . We calculate
that
5x- 2
lim J'(x)
lim
,
::x>
and
lim j ' (x) "' -oo.

= .c-o

..... . . o

3x 11

.t ~ o

This means chat there is a \ "Cry sharp poiU m (0, 0) wltci'C the !'unction luL') a relative maximum.
hs tangent line is vcnical at this poillt. 1t is difficult to gee a selSC of the concavily of 1he gritph
for x < 0, it appea~ ~ ~uai ght. To a~~s this we c.onsider
2{5x

I)

9x 4 /3

Since f"(x) changessignasx passes through - 1/ 5. wehavea point ofi nllection ( - 1/ 5, - 0.41).
The graph is concave downwurd for x :::; -1/5 and concave upward for - 1/ 5 ,:S x < 0. It is
also concave upward for 0 < x < oo. so thai (0, 0) is not a poi nt of inflection.

0.4

'Y

0.2
-I

-Qj

Qj

1.5

I EXAMPLE 4 .1 9
Rrpcnt Eumplc .1.17 forthc function f(x)
A plol of /(A)

SOU 110 :0..

=x +

in r.

= x + ;inx is s110"11 in Figure 4..17.

h is not clear whe1hl:r it

has relative eurema. Cri1ical pOiniS will provide the anMcr. We :.ohe 0 = /'(.r) = I +cos .r
for x = (211 + l)rr. where 11 is an integer. Bt:calL'<! f 1(.r) u<JC~ no! ch11ngc sign as x passes
through these poims. !hey canno1yield rclati\c maxi11w or minima. They must give horizontal
poinL~

ofin llection.
r1n1 of /{."C) x + s i~ax

7.5
5

-5

- 10

_/

10

A .S
5

- 7.5

EXERC ISES 4 .6

In 1:-.xcrci.c< 1-20 piOI tr"''h of the funcuon. lJS< dcn\livcs 10


di"""< ony <tnrfw:anr feotures of lhe pl<11.

l. /1)= 1 - (t< +1lr +9

.!. /(.1) = ~,- 16.1' 1 + 18.11 + 2

4.

i.

H. f(.<)

19.

i,

20. J(x)=x+2<uu

f (x) = (..- + 2)1(x -

22. j (x) = x + IOx1 + 6r 1 - 64x + 5

2J. /(x) =

l-1. f(x )

= x'
..- _ 58

25. j(X)

= x-,-x'

i.

27. j(x)- x 2 - IJI+2

x2- 4

29.

= -xX 1-+- II

30. /(x)

2) 3(x 1 2)

j' (x) = (x

x'+ 16
x

=' +X

13. /(.1)

ii

IS. /(.1) =

2x 1

x'- S.t + 12

4) 1

it

II. j(,l ) - -

IH. j'(.l)= (r- 1) 213

21. /(.r ) = x j(.r 2 + 3)

= x"'(S- x)

i
i

t)'
4
(K + 1)

In E"<crrisc!': ll - 32 piN u gr.Jph or I he runcliOr'l. ltknlify all rcbilivc

x - 3x111
7.

17. /(X)= (A' -

e"rema.

f<l=2.' - tsx' +6.r +4

S. f(.r)

il.

14. j(x) = -

il*

16. j'(.<)

.r'-

j'(x)

x- 2.r 3 -

2x

l6. /Cx)

= si"' xcosx,

.1 1 - x'''

2H. /(.I) = (Zx-l )(x-8)


( .t - l)(.r- 4)

0!:; .1'!:; 21!'

=siu + cosx ii.

J l. f(x)

=~
1 -x

274

il *

Ch..'tpter 4 App\ic:aliousof Differenli3tion

32. J(.t) = ( r +8)

/t 2 + LOO

(a) PIOlagraphofthdunctionJ(.t)

(h) llrc average production cost per kilogmm wlr~n x kilograms arc produced is given by c(x) C(.r)j.r. Plot u
graph of this func.rion.

= xs-4x6- 8xs+40.t3.

(b) Find all critical points and classify them as yielding relative

(c)

maxima, relative minima, or horizontal points of inflection.


(a) A company produces x

Show thar the output at which lllC average cost is leasr satisfies the equation

k.ilograrns of a c-omrnodiry per day

mu total cost of

x (x + 100) + 60.
300 x +300
2

C(x) = -

I :S

Do this in two ways: (i) by rrnding critical points for c(x) :

:S 200.

(ii) by noting that !he average cost is tire slope of the line

joining a point (x. C(xi) to the origin; hone.: minimum


uvcrage cost occurs when the tangent line 10 the C(x) gr:rplr
passes through the origin.

I'IOl a graph orthe function. Show th:ll it is always C011Clii'C


Uf)WUrU.

14.7 Absolute Maxima and Minima


Seldom in applications of maxima and minima theory do we hear queStions such as: What are
the relative maxima of this quanti!)', or what arc the relative minima of that quantity? More
likely it is: What is the biggest of these, the smallest of those, the best way to do this, the
cheapest wny to do that? A diffe rent kind of extremum is involved - considering all points in
the domain of the function. what is the largeSt (or smallest) ''aiue of the function'/ We define
this type of extremum as follows.

DEFINITION 4 . 6

The absolute maximum (or globnl nraxirumu) of a function j(x) on an interval I is


j(xo) if x 0 is in I and if for all x in / ,

j(x)

f(xo) ;

(-I.Su)

f (x0 ) is said to be the a bsolute mlnlmum (or global minimum) of f(x) on I if tor all
.rin/ .
f(x) :::: / (xo).

(-i.8b)

Forthe function in Figure 4.48, the :1bsolutc maxi rtlum off (x ) on the interval 11 ~ .r ~ b
is J<c). and the absolute minimum is j(tl) . For the function in Figure 4.49. the absolute
maximum on 11 ::; x ::; b is f (b ). ana the absolute minimum is j(a). For the function in
Figure 4.50. the abwlutc maximum on a ~ x ~ b is f(b), and the absolute minimum is
f(c). Note that "c speak of absolute ma.l'ima m1d minima (absolute extrema) of a function
only on some specified interval: that is, we do not ask for the:: absolute maximum or minimum

of a function f(x) without specifying the interval I.


Each of the functions in Figures 4.48~.50 is continuous on a closed intet\-al a ::; x ::; b .
The following lhcorern asscns that every such function has absolute extrema. For a proof of this
result, Lhc irr tc~cd reader should consult books on advanced analysis.

)'1M..

AbiOiutc:. m:t'(ima and minima of ooolinuous funetioos on closed i.ntcn;ds

:
u

)'~

.. .

cd

)'M
:...
..
... ...

..

THEOREM 4 . 2

A function thac i< continuou< on a closed intel"\'31 mu<t auain an ab<olute maimum and
an ab>olute rmnimum oo lhat intcf'Val.
The condition< of thi' theorem are sufticiem to guarantee existence of absolute extrema:
that is. if a function i> cominuous on a closed ime"al. Uten it muM ha1e absolute e'lrema on
l

ma~i m111n

Abwlule
alld ninirnuu' of a

d1~lll i1111011' i'lllk'fiOfl

(. ..[
(J

~
. ..
. .

that interval. However. the conditions are n01 nccc>sury. H they are not rnct. the function may

or may not ha\'e ab:.olutc c.xu'Crna. For instance, ir the function in Figut"C 4.48 is confiued to
the open intcr\'al a < x < b, it still attains ii ts absolute e:urcnut at .t = c und ..t = d. On
the other hru1d. the function in Figure 4.49 docs not have obsolutc extrema on the open interval
tl < x <b. The function in Figure 4.5 I is not continuous on the closed intern! a !: .r !: d:
it ha.~ no absolute m:lXimum on this inler\'al, but it does hO\'Cbsolutc minimum f(a). This

function is not continuous on d !: .r !: b , but it has absolute maximum f(d) aod absolute
minimum /(b)
Absoluce '"trema of the functions in Figures 4.48-4.50 al" ays occur dlher at a critical
point of the function or at an end of the intCI'\'111 a !: x !: b. This resuh is alwli)S true whether
lhc function is continuous or not, "'hether the intel'\al is closed or not. If function has absolute
extrema, they occur at critical point> or the end~ of the intel\al (if there ore ends).
T H EOREM 4 . 3
If a function hos absolute cxu'Cma on an intCmtl, then tl>ey occur either at critical points
or at the ends of the intcr"al.
A plot of a function on an interval normal ly makes it clear where absolute e.,trcma occur.
provided of IX)UI'SC thalthcy exist. It is then u m.attcr ofc,alunting the function at the appropriate

criticdl pointhl or endpoint(>). When a plOt ohhe furlCiion is um,.ilable (as \\OUid be the case
if 1he function rontdined unspecified parameu:rsl. the follll\vina procedure ~ ieltls its absolute
e'lttrtma.

Finding Absolute Extrema for a Continuous Function


f(x) on a Closed Interval a ~ x ~ b
(i) Find all cntical point< x 1 x 2 ,
(ii) f.VHIIIHIC
[(a ).

. . ., Xn

of /(.t) in a < x < b.


((b).

'' I

(iii) ll>cab,ohuc nra~irmnn of [(~) on .a !: .f !: /1 is the larc>t of the "'""bers in (iil:


the absolute minimum is the smallest of these numb~rs.
Note that it i\ n01 nec.:,..ary 10 classify 1he crili.cal point> or /(A) "'yielding relati\'t maima.
relative otinima, horizontal points of inflection, vertical points ofinHcccion, comcN. or none of
til=. We need only t\aluatr /(.1) at x = a . .I = b anJ at it> critical point>. The larg~ and
smallest of these numbers must be the absolute extrema.
Wllcn lhe fut>elion has discontinuities or tile intcl\al is not of finite length.. Theorem 4.2
can1101 tl<' used. Careful rot><ideration oflhe fmk'lion at discontinuuie< and lim irs as x --> oc
may be rcquin:d

I EXAMP LE 4 . 20

Find the absofule 1naxmmm and minimum of the function


f(x) =

on the intel"\al 0 ~

x !:

6. if they exisL

x3 - 2x 2 + x

x2

+5

+ 20

r=

J .7$

...3- 2.2+ .t' + 20

JS
J.25

3
2.15

2.5

2.25

SOI.lffiO\l Si11ce f (.x) is c::omi llU<)uS on the d osed imer"al 0 ;:;;: x :;: 6. the fulction mu.o;t
hnvc obsohuc cxtrCll'it. T he plot in Figure 4.5:2 makes it clc~1 r thm the absoh.1 tc minimum is at
1he critical J>Oirll to d\C-Icfc of J.' = 3. The absolute n\clxirnu Ill appears to be at x = 6 . but there
i.~

a rclmive muimmn near .r :::;;: 0 tluu shc.~u ld be iewestig.med . For cri tical points we sol\'e

0 = / '(.<) =

(x2 + 5)(3xl - 4x

1) - (x 3 - 2.< 2

(x2

x + l4x' - 60x
(x' + 5) 2

.r

20)(2x)

+ 5) 2

+5

To four dccim,11 pl;>ccs. soh11ions of this equation arc x

0.0850 ao>d x

2.7188. The

absolurc mini mum of(he function i s j(2.7188)


2.262. and bcc.ou.~
. /(O.ClS..'iO)
:u1d J(6) = 4. 146. lhc tlbMJilllc ma ximum is 4 . 146.
Without the gr.1ph i11 Figure 4.52, we: would eva luate

/(0) =

/(O.OSSO) = 4.008,

J,

/(2.7 188) = 2.262.

/(6)

= ~ .008

= 4, 146.

The nbsohne minimurn of the function is 2.26'2 a11d the absolute rnnximurn is 4. J46.

J EXAM PLE 4 .2 1

Find the absolute maximum and mil1imum for the fU ilCii Ol


f(x)

2.x 2 + 3x
x1

+4

on tho imcr,.als (a) 5 :S .r < oo and (b) I :S x < oo. i f they c.t isl.

SOUITIO'I
(a) The graph in Figure 4.53a appears to be asymptotic to the line

relative n1aximum at or near x

y =

= 6. T'he asymptote is COilfi rmecJ by


= I.om
x- oo

2 and have a

3
= 2.

+4

1+ _-, 2

For Cl'ilicaJ points. we solve


0

f ' (x)
.

(x

+ 4)(4x + 3)

- (2x

~2 +4~

+ 3x)(2x) =

- (x - 6)(3x

+ 2)

~ 2+ 4)2

4.7 Abt:0lu1e Ma'(ima :~ nc1 Minima

277

The o nly critical point in 5 :S x < oo is indeed x = 6 . Consequently. the


absolute maximum of the function is .((6) = 9/4. Because /(5) = 65/29 and
lim,_ 00 .f(x) = 2+ . the function doe>: not have an absolute minimum on the interval

5 :S X < 00 .
(b) The graph on the iJUcrvnl I :S x < oo in Figure 4.5Jb indic~tcs that the absolute
maximu.nt is still .((6) = 9/ -l. and the function now has an absolute nili1imum of
.f( l) = I.
llillciii;IWIOi]

/(.t)

(:!x' +~tl/(.<' +41 on l

E1!U C&iWW

< .t < OC

2.5

2.5

/ (X):: (2x~+lt)/(x~+4) on I < .\ < cv

Jj

0.5 '

--------:---:-:---:-:X
tO

t5

20

25

30

\Ve now consider applied maxima-minima problems.

I EXAMPLE 4 . 22
A rectangular field is to be fenced on three sides with 1000 m of fencing (the fourth side being

a straight river's edge). Find the d imensions ()f d1e field in order that the area be as large as
possible.
1413111#1Jili:M F'encing a
rtoo,gul...- ficld

SOLUTIO:-J

Since 1he arc" of the field is to be maximized, we first define a function rcprcscming this mea. The area of a field of width wand length l (Figure 4.54) i~

A = lw.
1l1is reprc.scms the area o f a Reid witharbilrry length I and ao"bitrury width tV. But !here is only
1000 m of fencing available for the three sides: lhercfore. I and w rnus1 sa1isfy the equal ion

2w + I = 1000.
\Vith this cqu~11io n we can express A completely in terms of w (or f):

A(w)

= w( IOOO- 21u) = lOOOw- 2w2

To maximize. the area of the. field, we must therefore maximize the function A ( w). But what
are the values of w under considerdtion? Clearly. w cannot be negative. and in order to satisfy
the restriction 2w + I = 1000. w caou1o1 exc:ecd 500. The physical problem bas now been
modelled mathematically. Find the absolute rrnax imum of the (continuous) function A (w ) on
1he (closed) interval 0 S w :S 500.
The griiph of thjs function in Figure 4.55 clearly indicates that area is max imized by !he
criticaJ poinl at or near x = 250. To find Lhe critjcal point, we solve

0 = A'(w) = 1000 - 4w;

278

Chaplet 4 Applir..';ll;ioM or OilYe-tnliatiOil

the only solution i~

w = 250. The !argest )><)SSil>le area is obta ined when the width of the field
is 250 Ill a nd its length is 500 rn.
Without the graph in Figure4.55, we would evalua te

A(250) = 125000 ,

A(O) = 0 ,

A(SOO) = 0,

and draw the same oonclusion.


m

'!!'H

Arc..1 funtttt"'fl for

100

200

rcc-un ,u l;:~r

llc.lrt

300

I EXAMPLE 4 . 23

Pop catt< that must hold 300 mL are made ill t:he shape of right cireular cyli11ders. Find the
dirnensions uf the can thc1t minimize ils surface a rc.a.

I!I

Frt. I

Diag,'"' of (lOp can

I
1
II

tcJ!JjDOut

Top

Bottom

GG L-__ll
"

SOLUTIO'\ The s urf;~ee :lrea of the can (Figu re 4.56a) consists of a circular top. an identical
bottom, and a rectangular piece fonned into the side o f t he can. The total area of these three
pieces is

A = 2nr 2

27rrh,

where we measurer a nd II in centimetres (see Figure 4.56b). Since Cllch ctll hold s 300 mL of
pop, it follows that
or

300

h = -.

;rr2
This equation can be used to e xpress A completely in te rms o f r:

A(r) = 2nr

27r r

G~) .

Fo r what values of r is A (r ) detioed 0 Themdiusof thecan must be positive and the refore r > 0.
How large can r be? Ineffec.t, it can be as large as desired. The height of the cylinder can always
be chosen sufllciently small when r is large to s atisfy the volume condition 1f r 2h = 300. The

J,7 Atxolme

can may no1 be acs1herically pleasing:

\fax i nt:~

a11d Minima

279

fOt large r and snHI11 It , but no mathematical dif ticuhics

occur in thjs situntion. To sol ve rhe problem we ntust 1herefore n1inimize

A(r)

= 2nr, + -600
.
r

r > 0.

2500
2i)()()

1500

1000

10 r

The grnph of the funct ion in Figure 4.57. together w ith the f:.ctf' that

l im A(r) = 00

,._()1

l im A(r) = 00.

and

r-oo

allow us to bnllpflrk the answers. A rea is rninimized at the crilical point near,. = 3.5 . To find
th is critical point. we sol ve

0:: A'(r) :: Jrrr

600

ur

=r2

r = (1.50/n) 1/J. Thus. minimum surface aret1 occurs when the mdiu.s
of the can is (1 50/rr)' /3 "mand ios height, obtained from the ti'"t tht h = 300/ (rrr2}. is
The only sol ution i s

2(150/ rr)l/3 em.


Wilhom the graph in Figure 4.57. w e wou ld arrive 31 the s.~:une concl usion by noc.ing that

A (r ) is alw:tyS J>ositi,e mt<l


lim A(r)

r-o

= 00,

A((ISO)/rr )'l3) =a finite number,

lim A(r) = 00.


,._0.:

I EXAMPLE 4 .24
l'!J'!Uhi!"WT'
nowing,

ill

\Vdla'

T he rate o f discharge

Q from a circular pipe is

cir<ufllr p~

where K > 0 is a constant, R is the radius of the pipe, and(} is the angle s ubtended at Lhe centre
of the pipe by the weued perimeter (Figure 4 .58). Find the angle for which Q is a maximum

accurale to three decimal places.

SOLUTION We lirs t note th at Q is max imized when the funct ion j(8) = (8 - sin 8) 3/8
under the rad ical is maximized. T he plot of ./(8) in Figure 4.59 indicates that the maximum
occuos for the c ritical point near 8 = 5 mdians . To llnd this value, we set the derivative of .f (8)

equal to Z1'0,

We11cd pcri n~er

0=

8 (3)(8 - sin 8) 2 (1 -

<OS /J)

82

- (/J - sin 8) 3 (I)

in a eirC'ul:u pip<:
j(IJ)

40

20

,,

6 0

This implies that

0 = 38(1- cosll) - (II- sinO)= 20- 31JcosiJ + sin II = g (8).


Newton's itennive procedure for find ing approximate solutions defines the seque11ce

o, =

- 0 - g (().,} _ n _ 28, - 391, cos 811


.
"
g'(O.) - vn
2 + 30., Sill (1, -

5,

ltoratiooq~i vc~ 0 2

11.,... , -

+ sin 01,
2CQ~;Q.,

5.369 55 , 63
5.37849 . a nd O,
5.378 5 1. Since g (5.3785)
1.1 X l o - andg(5.3795) = - I. I X I0- 2 , wccansny lhutt<> tlucc dcciomol placcs.O = 5.379.
The p lot in Figure 4.59 is essential ill th is problem. Without it. we would not have known t hm
f ((J ) hnd only one criticul ])Oint. and a n >lPJiroxinuuion 10 it.

On the basis of these three example:;, we :suggest the following s teps in solving applied maximaminima problems:

l. Sketch a diagram illustrati ng the si wa1ion. i f appropriate.

:!. Identify the quantity th(at is to be maximized or mini mized, c hoose a le uer to represent it,
and fi nd nn expression for this quantity.

J. Jf ncces~ ary. use information in the problem to rewrite the c.~prcssion in 2 as a function of
on I) one V<.lriable.
4 . Dttemninc the domain of the function in 3.

5. Maximize or minimize the function in 3 on the domain in 4,


6. Interpret the maximum or mi nimum valurs in terms o f the original problem.

S tep 2 is c rucial. Do not consider subsidiary infom1ation in the pmble m until the quantity to
be maxi mi1...ed or minimiz-ed is dearly idcmificd.lahellcd. a nd an expression found for it . Only
then should other infonnution be considered. For in>tru>ee. in Exnnoplc 4.22. the I"C~triction
2w + I = I 000 for the length o f fencing available w. r; not introduced until the expression
A lw had been identified. Likewise. in Example 4.23. volume condition 1rr2 h = 300 was
introducedafrersurface area A = 2rr r 2 + 2.rrrll.
\Vc now ttLkc a luuk at lhr.,;~: adUitional cxarnp1c~.

I EXAMPL E 4 .2 6

..

A lighrhouse is 6 km offshore and a cabi.n on the s1raigh1 shoreline is 9 .km from the poilU on the

shore nearest the lighll1ouse. If a man rows at a mte o f 3 kmlh. and walks at a rate of 5 km/h,
where should he beach his boaL i n order to get from the lighthouse to the c-abin as quick ly as

possible? Repeat the problem in the case that the cabin is 4 kon along the shoreline.

4.7 Absotme. M ninu Md M i:n.iroo

lill~lii;ICI.i.M

(mm JighlhOU$e

(O

Futt:):t p<tlh

<:<llJ.fl

~ Li~hthouse
<>

2:81

SOL UTIO'\ Fig ure 4.60 illustnotcs thc pmh foll owed by the n1an when he beaches the bo3t x
kilometres from tl1e poim on land <loscst to the lighthouse. His uavcl time 1 for this complete
journey is Lhesum of his time 11 in the wuler and his time /! on land. Since s1>eec.IS are con!'!.tam.
each of these ti mes may be calculated by divi<ling distance by ~peed : th;tt is. lo = J x 2 - 36/3
and 11 = (9- x )f5. To minimize 1 for some value or x between 0 ,md ~. we minimize the
function
l(.t)

J xl

= ,, + 12 =

+ 36

9- x
-5-.

0 !:f

!:f \1.

The gm1>h in Figure 4.61 indicates that I (x) h"1s one critical poinl and the fun'"1ion has it~
absolute rni nimunl m thi ~ critical point. To find it, we solve

X
= I , (X) = -::-;:::;;=;::::::;;
3.J x2 + 36

Sx = 3J.r 2

or

+ 36.

Thus.

25x 2 = 9(x 2 + 36).


from which wcncccpt only the positivesolutiool x = 9/2. Thebo>ll should therefore be beached
4.5 kn1 fron1 the cabin. Without Figure 4.61. we would cval11atc

9
1(0) = 2- -: = 3.8,

1(4.5) =

./4.51 + 36
3

9 - 4.5
5

= 3.4,

I (9)

../81

+ 36
3

"=' 3.6.

'llme r'un c 1 i ~u1 wh(n cahul 1s

t=

'\/'xl +:\6
3

4- x
5

-1 -

4 ,\

When the cabin is 4 km ulong the ~horeli ne.

) _ -Jxz + 36

I (.< -

4- x
-.
5

+-

~ X ~ 4.

ll1c gJl lph in Figure 4.62 indicates lhal / Ct) docs not have ~my c.:riliculpoints in the interval
0 ~ x ~ 4. This is confirmed by the fact that critical points arc again gi ven by the eq11ntion
X

3 .J".,"'2=:+=:3~6 :-

5 = o.

but the solution x = 9/2 must be rejected since it does not fall in the imerval 0 :$ x ::; 4.
Tr-avel time is therefore minimized by heading directly to shore or dircctly to the cabin. alld
Figure4.62 suggests the cabin . Ve-rification is provided b)'

../36

I(O) = - 3-

14

+5= 5'

1(4) = ../16+36 = 2Ji}

<

14

282

~pter ol

Applkotions of Dill"e reoliJ~Ii()lt

._..

I EXAMPLE 4.26

A rectangular beam is to be c ut from a circular log. Naturally. it is desirable for the beam 10 be
a> strong as possible. From Ollf experience "e know that the beam >hould be deeper than it is
w ide. Fo i.lstancc, plncc a standard 2 11 x 411 x 8' piece of Jumbcr between two supports 2m
apart, Md sit on it. It will probably suppo rt you if you sit on the narrow edge, but not if yo u sit
o n the wide edge. In other words. the 2 x 4 is much stronger when its depth is more than its
width. Experime.ntal evidence in structural e ngineering has shOwn that the srre.ngth of a beam
is proportional to the product o f it.s width and the square o f its depth. Find the dimension of the
strongest beam that ctlll be cut from a circular log of radius R.

Log
/

SOI.UTION
given by

For a beam with wid th

w and depth d as ShoiVn in Figure 4.6 3a. its strength is


S = kwd\

where k > 0 is a constant. lf v.~ join the centre of Lhc log to o ne of the comc.rs of the beam
(Figure 4.63b), then from tbc rightangled triangle

=
Thus,

kw(4R2

w2).

This function must be maximized on d1e interval 0 <


critical point(s) .
.
-

w ::;: 2R . We begin by finding its

w=

2R

.Jf

We now e<>lculate

S(O)

= 0.

5(2!1) = 0.

Con~cqucndy. the beam is strongest whc.n it is 2R f ../3 C<:lllimetrcs wide and J 4R2 - 4R2/3

2./'iRf.../3 centimetres deep.

I EXAMPLE 4 .27
A wall of a building istobebmced by a square beam !Ocm by !Ocm lhal must. pass over a parallel
wall5 m high and 2m (Tom the building. The beam is tosil Rush againstthe building and on the
floor (Figure4.64). The top of Lhe 5-m wall c.an be cut al any angle to accommodate Hush conLact

wilh the beam, but the wall must not be shortened. Find the length of the shortest such be<trn.

SOLUTION

The 1en2th of beam from A to 8 is


I

D = IIEFII + \I ACII + KBGII = xsec8 + Io cot8 + Iotan9.


Sincex- 2

= 5 cot0,

= f(9) = (2 + Scot9)scc9 +

I
9.
-10 (cot9 + tan )..

0 < I} <

rr
2

The graph of this function in Figure 4.65 shows that the absolute mi nimum occurs at the critical

poillt ncar 8 = I. To lind it we sol ve

>

.,

= J'((J) = =9 tan 6(2 + 5cot9 ) + sec8(-5csc-IJ) + lo(-csc e + sec- B)


sioO (
5cos0)
5
= coslfl 2 + sioB
- sin20 cosO +

I (

to

I )

coslO - sio12 0

Multiplication by 10 sin1 0 cos1 0 gi,-es


0

. "e - cos..1 0
= IO sinl 8 ( 2 + 5cosB)
- .- - - 50 cos e +Sillsm 9

= 20sin 3 8 -'- 50cos8 sin1 (J - 50 cosO

+ sitl1 9 -

(I - sin 9)
= 20sin3 0 + 50cos 0 (1 - cos2 0) - SOcosO + 2sin 20 - 1
= 20sin3 9 - 50cos 3 B + 2 sin2 0 - I.

Mjt ..ill;i WI .(M

Mjl~tli.IMM

Beam pas<;ing. uver intentning. ,.,~.!II

Leng1h ci bc:un

R G
F

Building

To solve this equation we usc Newto n,s iterative proce.dure. Approximations nrc defined bv

8n+ l

_
-

8"

+ 2sin2 11,. - 1
60 sm-u,
. ' " coson
" + ISOcos2"un sm8n
. . + 4sm8,
.
cosO, .
20sin3 &,. - 50cos 3 0.

If we use 8, = I as the first approximation, the oext three approximations are 8 2 = 0.9278,

83 = 0.93 14, and 1i4 = 0.93 14. Consequently, the sho1test beam is f (0.93 14) = 9.8 m .

..-...

I EXAMPLE 4.28

In the' Applicafiot f"re\kw. we posffl rhc problent oflinding '"'"offset rluu maximitC'S dk' sttoke
r~diu" of the -.ircle.

;,, rhto ~J-ank shown in r:aure 4.66 ifd1C" otr.,cr rtlu.,t no c cxcttd eM

Appl k11Hon Pr<l<'n

Rei'\itl

'\OI .rTJo"\ Ti) lind a fomtula forth/! kn8th s oftltt: srrok~ofrheofr~ crunl', we first dNw
rhc .TJink (Figu rr -i.il1H) in whill ; CJIUcd ;,... flll/l'rtf,"tul pn.ltlrm. P.-.inr
ooinc fde~ with
and A . H. aud C are c:vll~ttc:.r. 'flu: lt>JJS.tlt ofFD 1.S .j(L
r)l - i'J.

Figure4.67b>howslho<-r.onk in iO>llm~rd<tu/JW$/titm. Thelenglho( FE is /CL


dac l~:ugth vi' du:: ~ lH.1 kc i~

,. ~.....

.i

r )' - ~

C<m~.>t:'ll.lt:lllly.

.----- .JJ

i' ",...,.

----. r

'--..
E

We mu.(t n-u'CimJ7C the funchOrt s(t') noft the iutc:rvat n


funcriM. we .Wn::

_s i'! ~ r . RN- trr1csi JlOU1(( o f rhc

J(l. - rf - ~

4Lr- 0,

an impossibiJicy. Thus. s(c) h.1s no cricical poinrs; ics maximum muse occur ar either e = o or

iOi:

r. \\'c therefore catcul,tc


s(O) -

(L

+ r)

s(r) /( L

- (!. - r) _ 2r,

+ r)l - rl - /(L _ r)l _ ,.z

/U + 2Lr - JU- 2Lr.

II h 1101: tiPf.JttfCIII whidt of the tnu of dtc.'e is la.Jcr. but it C diJ be .\ho\~n alj.d)raicalf} that 2r h
'"'"m"ller (rry ir). Alternlllh-ely. we nnrc rJwr , ;,.,e / (1. r)i - p1 < / ( L + r)l _ pi i t

foliO\'> rJw

'

/(L- r)'- ~~

>

and therefore .<'(e) > 0. The f unction s(e) must thercforc be i ncteasi ng and its maxi mum
,alue must be s(r) when e = r .

AbsoiUie \flllima a~~d \linin\:11

47

285

Conaulting

Fixure -1.68 ~how; s.>il to depth D co,ering a horizomalla)er of roc~. A gco~ic;l


engineer set< up a <OUrce at A to emit a <ignal into the roil. 1be <ignal penetrdle< the
soil to !he rock below. Some of this sigrul is reflected back into the soil, some pcnetmlrs
the rock. and some tru\cl in the surface of the rock. Each part o f the rock actS.., o1n
emitter -.ending the signal back imo the s.>il. <Orne of .-hich reache' the recci,cr at R

This happcru in many wnys nnd the recei'"-er records an accumulation of many 'igru.tls.
Our problem 111> consultants is to usc the signals to dctc:nninc depth D.

Surfoce

Jl (Eminc:r)

Sool

/)

Rock

SOI.l. TIO'I The first question to ask is: Which of the ma55ivc accumulation of interfcrin sig nals recei\'ed at B from \"arious points o n the rock can be u<ed? h seem< that
the o nly signol distinguishable from the rest is the one that arrives ot 8 first. Let us

com.:cntrute on it unO M!C whal we can determine.


We let Vt and u2 represent how fast the signal travel~ in the soil and in the rock. It is
known that 111 :> v1 . that VI is measurable_ but Vl is unknown. N'c;tt question: \Vhut is
the path followed by the signal that atri\'es at 8 first? In Figure 4.69a, we ha\c sho"n a
signal following jXllh ACE B. making acute angles 0 and </> with the s urface at emitter
and recei,er. Thi< cannot be the fastest path_ If is mo\'ed to the right, the <ignal tr3\'l!l<
less distance in the soil where it is slow t~nd further in the rock where it is fast. In other
word>. nalc rfJ 'h<.luld be made la~r. But it ;hould n()( be made laf<!r than 8. cl-.c \\C
would mnkc the s:tmc argument about 8. We conclude that the fastest path must occur
"hen ngle,(l and drecqual "'' ;hown in Figure-1.69b. Tire problem;, riO\ hl<letemlinc
Qlllc 9 for whtch trn,eltimc is smal lest.

Po~thl fur ii#Mh. emd

ltd a1 A aod rei\'ed a1 R


J

I
n
I

Surface of canh

'

Surface of canh

41
(Soil)

(Sorl)
speed= ,.1

(Rock)

(Rock) <peed

'Ore time for the signal to uavcl from A to B in Figure 4.69b is


1

20
= - cscll

v.

S-

20 COlli

+ - - -vz

=''l

The smallest possible ' 'Blue for(} occurs when C and E become coincident. in which case
fl = Tan-' (20/s). Thu>.thedomainoffunction t(8) is Tan -' (20/s) <; (J <; rr/2.
Oiucol points of I (9) ore siven by

dt
d(}

2D

2D

- csc6 c016 .,.. = - Vt


Vl

,
csce

- 2D

----:-."'1'
1 ~ (v1 cos6-

lltll:z Sill

v1).

Thw;. the only critical point is (J = Cos- 1 {vtft1). II is physically clear thot end potniS
of the don..,in Tan- 1 (2 0 /s) ::0. 0 ~ Jt/2 cannot minimize 1(0). TI1e critical point
must do so. {II would be a good exercise for you to pro'e this mmh<matically.) When
oo,() = Vt/l'l it follow> that sin(} = j1 - (vdvl>' in which ca..e minimum time h

r-pv
2

2D 1- ( ::1 )J +s
=-{
v, \
VJ
Vl
Now that we hove a form ula for the time that the fastest signal takes to nmve at
11 1 and I in thi> formula are mea>urdble. and
therefore known. \Vhat are not known are D and v2. If the receiver is moved to 11no1her
locaaion. ho\vc\cr. a new distance and minimum time. say S ami T . cue obtained. They
sausf>

8, \\hat do we do with it? Quantities s.

_T =2D
- /1,., '

(u
-

1)

S
+ -.

u,
l'l
flll the-e IN two equatioo~' could be soled for D and t'l To reduce error due to
CJ<perimcntal measurc,.,.,nts. we might sussest to the geological engineer 11\at he take 11
mea.\uremenb re>ulting in 11 p.tir> of V'.tlue~>. ""> s, and /,. and plot them un a ~ph
of l against s {Figure 4.70). They look oii110SI collin=. as they should. If we set

'

4,7

AtN:ilWt M a:o..i11m and Minima

287

b = (2D/v 0 )Jl- (v 0 /v;)' and 111 = I /U2.thcn

I=

ms

+ b,

the equation of a s trnight line. By measuring the slope and {. intercept of the best-fitting
line to the )Joints. it is (hen possible lo ~o l ve rthe equation:)

m=
for V! and D. A mathelllfltical W<\)' orfinding the best-fitting line (inStead ofjust eyeballing
it) is discuss.cd in Se.c.tion 12. 13.
EXERCI SES 4 . 7

In Exercises 1-6 lind ubM.Jiutc cxu-cma. if lhcy cx.ist. ror thc. l'urx.1ion
on the intcrvol. Oo ~o widlCW.It piOoHing u gratJh Ol' the funcliun.
I. f(x) ;;;;; x ' - x 2 - Sx

X - 4

- 2~x ~J

2. f (x)

= - -.

3. / {.I")

=X+ - , -2 -<X<- 5
X

x+ l

0!: :c

;.

f(A)

10

= x..IX+i,

1 1 11 1 1 1 1 1
13. A closed box is to have length equal to thn::c times its width and
total surface areaof30 m 2 Find the din'lCOSions th:!l pr<x.tucc n'la.xinu,un
volunlC.

O~x ~411'

_. 4, j'(x) =x -2 sin.:r.
4

+4

- I $ X$

+;. + 2
I'

6. j'(x)

= '

12. An agronornist wis hes to f~ nc.'C eight rccu.mguhn plots for e,;pcr
in :nlaticm .~... shown in the figure below. If cnch plo1 mu:-1 conwin
)(XX) m 2 , fi nd the: rninirnum amount or fencing thnt can be used.

Ill

14. A square box v.ith Of>l.:ll w p is to hll\'C" volume or 6000 '-Find


the; di n ll.~llSiO.fiS 0 f lhC bollhal minimia,} the UIHUUnl or nU.!U:ri:.d used.

15. A .:h cny orch~fll h:.1s 2..S5 tree.;. .;;.1.;.h of which 1'11'\1\.h.K.'GS un the
avcrugc 25 basket$ of chcnic~ . Forccll.:h utiJitionallrec plunlcd in lhc
sante area. the yield pet tree dcCI'ca;;cs by onctwdflh of a. btl:~kc:::t. How
many tnorc trCC$ will pn:xltK.."'C u muximwn c..Top?

16. A high u hool foolball field is being de~ig.ncd to U~,:(.'Omnl()(!:al~


tl 400nl track. The uack c'uM along the lcnglh of 1hc fichl Olld has
SCIIl i c.i rd c~ hcyuc'IC.i tlk: cml Ylllll.~ with d i arnc;:h! l~ C((Ui.llto Ihe width Of
the field. lf the end zones RlliSl be I0 n1 deep. find the di n lC~sions of
the Hc::1d lhlll maximize playing l.UC.l l {not including the end zones).

17. When u mu.rtulh<.:tu.ring conpwl)' sell) -" objects I)Cr n'Kllllh. it $..:LS
Lhc pnce ,. of e:.u:h t.Jbjcct m

x <0

l.n Exercises 7- 9 find absolute extrema. if they exist. for tbc runction
on the inten"nl . Uf.C! a plo1 of the graph of the fun<:tion Si=: an aid.

i
-

1. f(x)

.<+ I
=-,
x- 1

X> I

i1

M. /(x)

= -X ,-+ 3. < > 0

(2.< - I)(A - 8)

! 9. f(x) = (x - l )(x - 4)
10. Usc rhcorcm 4.2 co prm-e Theon!m 3. 17 in Section :\.1 4.

11. Onccndofa.unifo nn bcamo(Jcnglh Lis built int03\Y".t.ll. andthc


other end i~ simply supportcc.J (figun: below), If the beam hus constant
m:.-.si per ulitiCil.glh m. its deftcction ;; froulliK: horizonui.lu.t udis:t:~ncc
.t from the builtin end is g.i\cn by

(48/ )y

= - 111g(lx4 - 5Lx'

r(.t)

+ 3L'x').

= 100 - -x'-.
10000

1hc 1om\ CO"'l C of producing the x object(. J)C1" momh is


x2

whcre E ltnd I Hre <.:on!Janls depending on 11'w= moucriul and crt)$&


~clion ul'the bcwn. untlg > 0 is:tCQil:)(ant. 110\'f' t'ur (n.>n thctJuiltin
end doc:s maximum deflection occur'?

C(x) = -

10

+ 2x + 20.

Find the numbca- of <)bjoctii the compuny liohou!d sell per monlh in or&.:r
to rcalite max.imum profit$.

18. 11te base of an isosceles uiangle. which is not one. of the equal
sides. has length b. and its ahilllde has length a. Find the area of the
lagesli'CCLa.ngle: thal can be placed irL~i dc dte triangle if oneofthesides
of the rectangle must lie on dte base of the uianglc-.

.f.

_19. A m:lnufecturcr buiJds cylindrical mclal cans ihat hold 1000 cm 3.


There. is no wa:s.tc invohcd in ctlttio.g mtUcriaJ for the CUf\'Cd surface of
Checun. llowcvcr, each circularCI'K.I pi(:(.'(: is<.:ut from a~u~re pice~ of
n\etal. leaving four wu:~tc pieces. r;irtd the hcig.hl and I"Jdiu.4> of the c.ut~
lhat uses the lc:asl anKlUilCof mcl:ll. including all wuslc nw.ceriaJs.

20. Sides J\ and A or an isoscclc;( triunglc 1\UVC cquullcngth. 'IlK:


bu..'iC BC has length 2a, 'lsdocs the ahitude AD from A to BC. Find
U..:: hcighl of u poinc P on AD at v.hich the ~um o f the dis1ance:. A P,
B P , Wl(( C J> is :1 minimum.

produces p::r yci!!r, ;r is th: numbc;r of employees. and )' is the daily
operating budget. Annu.al operating costs amount to S20 000 per cmplo)'C.X: plu.:-. the operati ng bud,;ct ofS36S y. If 1hc manufachtrer wishes
10 produce I()()() automobiles per )CID" at minimu1n cost. how many
cmplo)'CI.:S slttXIId it hire'!
... 2!). The IIIJ $~ fltl\V n llc or gas through :l nonlc io,:; sivcn by the runcti()ll

Q(p)

2y
[( ,, )
- - t>oAJ
-

+ 21. ' 1\vopoles are driven i11tft theg_round 3 rn aparl. One pole protnu.fe::.
2 Ill above the ground and the othc:rpole: I n1 above the grollnd. A single
p iec-e of rope is auachcd to the tot) of one J)Oic, passed thfough a li.JOP
on the ground, puJJed tmn. and attached to the top of the other pole:.
Wht.Te :.houkJ the loop be placed in order thtu the rope be as shon as

+ ll. In designing po.1gcs fOI' t~ book. a publish~r decides that lhc recum-

23. Find lhc poiniS on 1hc hypcr~ola )" - .v2

..._ 24. find the poir1t on Lhe paraholu y


1-

= 9 clOSCSIIO (4, 0).

= x 2 clo~.;;ato (-2, 5).

25. A light roun:c is to be pi ~Jc.-cd dircclly ubovc the centre of a circular


area vr radius r (figure lx:low}. The illuminution at MY point on the
edfC of the circle is llirottly p ropot1ional to the co~int:

p )''
Po

tl)/y].

_. 30. ,, printer contracts to print 200000 copies ora membership card.


It cons SIO per hour to nut the press. and lhc press pl'oduccs 1000
imprcssi()l\S per hour. The prinlt r it: free to ChooM: the number o l' set
I)'IX'S per i.n1pn!$Sion IO a lim it of 40; each set type cosLS S2. If X set
1)1>CS arc chosenM ca(h i mpn.--s:;iol yicllls ..r <:"l'~ls. How nllUIY ~ ~ t)'IJCS
should be used?

S1 is 2() k.m north of ship Sz. 1r S1 sails sooth


at 6 knVh. ;;uld S1 c.a~ at8 knVb. find when the lwo ships arc closest
log<:lhcr.

,.. 31.. At noon a ship

or the angk:

0 and ill\'Crsel y propot'tiO!l<.d to the square of the tliSU"UkX" d from the


soorcc. r ind the height II above the cidc; at which illuminalion Ofl liK:
edge of tbe table is maximized.

Po

where p ls the discharg.c pressure of the sa~ from the nozyJe. 11\e
di.scha.-gc cross-sectional area of the notzlc A. the stagoat ioll pressure
l'o the ::;_IJ.lgnuti on dcn.<,ity Po , and )' > I arc ~II con~lant..~. TilC
dischiV"gc prcssl~rc for "'!flich Q is a ma~imum is cJHcd the critical
pres..~ure p 11 Find Pe in tCI'tnS Of /)(j u'k.l y .

pos..~blc?

gularprintcd H.."t!ion on each page must ha\'c area 150cm 2. II' the page
nlU&I have 2.5-cm margins on cach~i dc and 3.7S..cm m~rg.i n$ at lop and
bouonl. find the dimensions of the pa.gc of smallest possible area.

y - 1

!fy

>t

32. A 1Hilitary c ourier is lot-3.tcd on n desert 6 km from :1 poinl I'


which is the poi11t on u fong. straiglu road 11C'at(St him. He i..~ 01dcred
to a poi.m Q on the road. lf we assume thai he c~ultmvel al ~l
rete
l4 krnlh on the descn and a nuc of 50 kn~ on the road, iind the
point when: he ~h(ltlld reath 1hc rood in order 10 gel 10 Q in t he lea.st
po~ iblc time when (u) Q i~ 3 km from 1> and (b) Q i5 ll:m (rom P .

10 report

or

Soura:

0
h

d
:t

26. A mons aU l ine: 1\CgnlCnt,; th;11 ~ trtCh frtun l)(.li nt ~ em the poo::i livc >:
:s"i" to poi~s on the positi\'c yax:i~ and )Xlt:S throu ~h the point (2. 5}.
find th:n one thm nt.1kc.; with the ll<)'itive X and y-axc-. the uiunglc
wth kMt posiiblc area.
f

17. A longpicccofmctal 1 111 wide is to be bent in two places to form u


!tf~illwS) so di;.H ibcro~-rsection ht~ l'l i~osc.:des lntl)ct..oitl ( ligurc below,.
Firkithc 3ngk: 0 :!t which the bends should be ronncd in ordtr to (lbLain
nu:<inaun possible 0 (~" along the ~pi ll way if thl! k:ng Lhs o f A B. 8 C,

.. 34. ,\mc>ngall rectangles thm can be ii\SCribed inside the ellipse b1.r1 +

a 2y 2 = a 1b2 and have sides parnllcl

,. 35. Among,ull ra.Ungk~lhal c..m be it~ribctl i~idc the ellipse bl:<l _._
a2 yl a1 b 2 and ha,c: ~de$ 1~n.llcl to the w<cs. find the one with
largest pcrimctcc.

* 36.
,j.

28. An automobile manufaclutel' hac; a Cobb-Douglas production


function q = x 115 yl.fS . where q is tJ1e number of automobiles it

FincJ the tJrc:; ufthc lmgc.St i'Cclang.lc th:u h~ one side on the .t 3xis

aod two verlices on lhc cunc y = ~-x~.

-~

tn the u:<cs. lind Ihe one with

I311!.::St :lrc:t.

and CD are all 113 m.

ll. f ind the arc.a uf Lhe lnrgc ~L n::clangJc LhaLCal\ be insc:.ribc.d in.;,;idc a
circle of r.ldtus r.

37. Find the \'Olumc o f the h11'g cst 1ig.ht cin;uler C)'lindcr lh41l em-. be
inscribed in n sphere of radius r .
J~.

Find the volume o f the lmgcst right cin:"lar cylinUcr

t h~ll

c::1n be

inscnbcd inside a ri~lu cirtular C(lne or l'adtus r u.nd height h.

39. Tho beams are to be cui fro1n Lhe lal'g ct pieces of wood left ovet
whe n tJ\e sttongest beam is cut rmm the log in ExalllJ)Ic 4.26. What
are dte dimensions of the sun ngcst such beam.c:?

*'

4(). Whcfl :tstlolptUeJ pmjecl$ a shQI from height lr above the ground
(figure below), at Sl)(4.' d v. its rttngc R is given by the fo rmula

sin 2 0

where 0 i.;; the angk of ptojc:etiOC1 with the horizonLal. Find the angle
{) that m:u.im i:~.c.-s R. What is the .i!.nglt "hen t' = 13.7 mis and
h ; 2.25 m'!

+ 46. T~ iJJumin<1tior1 ala point i!S imcrscly proponior...a tu tl~~.: s.qu:arc


of the distance from the light source and dii\Xtl)1 proponiot\al to th~
illlensity of the source. If lwo lihl sources of imensitics / l and 12
arc a dlst.u.nec d :ll):lrt, at what pOint on the line &ejtlllttlt joini1\S the
M)Ur<."CS is the sum of thc.ir illumina.tions a minimum - rel;_tti\'l' to <til
other I>Oints on the line segment'?
ot 47. A window is i1l the forrn or~' rocmngle ~ltlll<.~.mtcd by a semicircle
wi th diameter cqt,~.;zl to the width nf the winOOw. II' the nx:t:~ ng,lc is
of clear glass while the: semicircle i~ of coloorcd glass tha ua.nsntits

onJy half as mu.ch light per unit urea as the elcttl:' glass, and if the
101al l)CI'iu-.c:tcr is fixed, find U'IC ptt'~ l)()ttion s of the rc:Cl.itl~ular ~nd
semicircular part, or the window that :u hnitth.:: nl()St lig ht.

i ..

'

*'

4 1. 1\to corridors. one 3m wide and l heOlher 6111 wide. nlCCt m right
angles. Fi11d dlC length of the &or1gcst bc.~=un that can be tmnsponcd
hoizQntally around the corner. lgnt">re tht dinlensic.ns or the beam.

ii +

4:2. RCJ)CQI E),ereise 4 1 raking into account that the beam has :;quare
Ct'\l.SS-scctiora 1/3 m on cath s.idt:.

4J. A bee's cell il> always constructed in the shape: of :! rcguk\r hcxagon::ll C)*lindcr open on one end and :.rlrihcdral :tpe'( :Jithc other (figure
below). (I can be shown thaltbe total area of the nine faces is ghe n by

J 2
= 6.t)' + -x
( J3csc0 2

48. A company wishes to C()ll~trucl a. st()f'1.g tank in the fonn or ;a


rectangular pt r;.slld cpipcd with a sq\I..TC hori zontal CtOSSSOCiiun. 11lc
volume ol' lhc 10111k llli.I:SI be 100 m'.

(a) If mtlf.crial for the :o.i\k~ wvJ top C('~M< S 1.25J m1 (uld lll\1
tcri;.ll ro. the bouum (().:)($ ~.15/nl2 find the dimcns.ons
th~d 1111in i 1ni~c m:.tlcri(ll costs .

(b) Repeat part (u) if lhc 12 edges musl be wclckd ala cosl ()(
57 .50/ m or wekt.
,._

coLO).

Find d~. ;angle th!U minimizes A.

4~.

When Wl unJoOOcd die is thrown, there is 41. prob:abi lity or 1/ 6 that


it will come up "two... lf Lht: die is lootk:d. on the other hand. the
probabil ity U~ll :;1 "two.. wi II <'I)IX"".tr is not 1/ 6. lJut i!; ~o n.:: number p
belwccn i"ero <Jnd one. To fh1c.l p we could I'OU the <.lie a large number
o r times, !'.H)',. ~md count the numtx:r of times -two'' appears.. Stl)' m.
It se<:ms rct1SOOL1ble Lhtu rut eslintate ror p is mJn. M:nhemalicians
define o likdih.Q()cl function. which for the pr<:scm silualiOll tum~ uut
lobe
.({x) =
n!
.c"( L - .c)''-".
m !(n-m)!

,\C
c.a lied

value of .r dun ll"Uxinizes f (x) on the interv:al 0 .:5: "' .::: 1 i:o
the nuul ,tum lrkl'lilrotHI tMimtlll' of p. Sh!)'' thm Ihis csaim.-uc
i.s m fn .

SO. An underground pipeline is to be constructed beween wo cities.


A and 8 (ftgut(: below). An analysis of the suMlructurc indlca:lei thnt
c.:.on!'tructioo cos.as per ldlomcu\! in region" I ()' > 0) and II ()' < 0)
arc ( 1 and c2 C"<l!opccli,d y. Show lhal the tot.al C()ll!'truction cost i~
mininlized w hen .X is c hosen so that c 1 sino. = c 2 sin02.

jY
~

'--i-- .1,

44. Find the nhsolutccxtrcn.. of thc funct;on /(.t) ~ .r/(.r 1 + c) on


lhcintCI'V'dl 0 ~ X ~ c, ifthcycxi:sl- Trcat c > I 3.$tagivcnconstunt...

45.

(u) A rcltt l COII'fi:Jemy buys u new machine fOe" p dollurs. which


j11hcn rcniS to~'\~Horncrs . Lfthc<.'Oinpany keeps the m:tchinc
for 1 )<.'<.:!"$ (before teplocing it). d1e 11veragc rcpl"'c:.-cmerH
<O:.l per year fur lhc 1 ycu~ is p f t . During th~.-sc 1 yc~us.
the <.:-ornpuny must n\l:lkc r~in,ortthc 111~h i nc. the number
n depending on t a$ g.t''CII by n ( t) c lc' f{J. when; 0t > I
and fi > 0 arc c.oJlstants. If r is the average cost per repair,
then 1hc avc::1'3g<: n13intenance cost r><:r year' over the Iift of
the R\<M.: hinc is nr/ I. Tin: tulal yt:arly ex1X:1lsC aN~l~C.:-ial t:d
with the machine: if it is k:cpt rcr 1 ycws is thercfOI'c
/:>
C(t) = I

ur

+ -I .

Fi nd lhe optimum tj.ne al whid t to reploce the machine.


(b) Discuss tJte c-.ases where

a = I and 0 < a < I .

'

.t

51. A subnlil.l'ine i s sailing on the surf8 due easaat a ra1e of .t kil()mctrcs per hour.. II is to pass I km north or a point of land on an island
at midnjgbt Soldiers on the island wishing to escape the enemy plan
to intercept the submarine by rowing a lUbber rafl in a straight-tine
course at a mte of v kilometres per hour (v < s). \Vhal is tJ1e last
instant tJ1at they c-an leave the is land and expct.'t to make c.-ontact with
the submarine'?

2:90

Ch.'\pi:er 4

A p()l iQI i on~ uf m treren1 i:u iun

x 52. A packing comp.any wi shc~o; to IOnn lhc I m by 2m pie<:c of c:lrdboard in figure (a) below into a box a.s shown in figure (b). Cuts arc to
be n1adc uhl solid lines and folds alollg doucd li nes, ~ n d Lwo sides
arc to be taped 1ogcther as shown. If the outer flaps on wp and boL1on1
mus1meet itl thc:: CCIHJ'C buLLhe inner H.ups need nol, find the dimensions
of Lhc box holding lhe nx)St \'()lumc. How far apart will the inner tlaps

- )J

Home 3bn

T
5

_]

be?
2

II !'

-----i"..

:t

l;;"j.....;-r;;..
,

pounds force by
F

(a)

4w2

6.5:rp 2 A 2u4

0.0238 slugs per cubic fooL


(b) F'ind lhc optinlum speed at 30 000 ft when dcnsily of the
ulllwsphcre is !tboul 0.37$ lh(ll alsea level.

>t-

TtJ{Je

'*

(a) Find the OpLinlum speed of Lhc jcl at sea level when! p =

~ I ll
A

II'

= -1 pAv, ( 0.000 182 +

where p is Ihe c.Jcnsity of dlC atmosphere. Optlmum speed occurs when


!he r;llioof thruSI to Speed is ~-1 minimum.

::.;::::~
B

56. A Boeing 727-200 jcl trenspoc1 ha.~ wing pltnfontl area A


1600 fl2 and u oss weigh! w
150 000 lbs of (Qft:C:. A t Cfu isilt;
speed v (in miles per hour), the lhrur.~ F of Ihe engines is g:i\'en in

53. Ttlc cost of fuel 1>cr hour for runni ng a ship varies dircclly as the
cube of lhe speed, and is IJ doll ars 1x:r hour when lhc speed ls b
kilomcC1'Cs per hour. There ruc also fixed costs A dollars per hour.
Fi nd the tn()SI econornic.:al s1x:cd at whi<:h to make a ttip.

57. WhcJl :.t fo.t'CC F ls applied Co the Q ~jcct of Ola.ss m in lhc figure
below. lhn.-x: other forces Hct on m: the force of gr.wity mg <
s > 0)
direct ly downward. a renctionul fort.:c or the SUJ)I>Orling surfac.:c, and n
h(lri'Wf'llal. flictio na.l l'orce opposing F. The least l'orc<: F thaLwill
<.wcrcon\C fricl i_~.m und prO<lucc motion is given by

F=

or

cos/1

tu ng
)
+ tLslnl1

whcl'e J,t. is a ct,.nsumt called ~he cOf!fficicm (~/' stml<:friaion. Find the
angle 9 for whi ch F i s mininlal.

*'

54. A paper drinking cup in the fo rm of a righ1circul ar cone can be


made from a c.irt.:ula.r piece or pupcr by rcmovj ng a '>Cctor and j oi n.ing
t:Uge$ 0 A omd 0 8 . al; ~hown in lhc figul'l;: b..:.l ow. If lhc.: re~cli U$ of II-.;:
cirde is R , whozl choice of () yields a <:UJ) or muximum vol ume?

* SK.

Reperu Exerci$.C 20 if the length of the hose of the u i-tmg,le i;; 2h


inslcad of 2tl (bul Lhc ahiludc: is still 2a).

* 5?.

A tn~eking company wanl~ to dctcrnlinc tbc highway speed to


rct.:onmend to its drivers in order that colltpany c.:Ol)lS arc kcp~ to n
minimum. Ta~en inl') account will be ()Ill) hourly waec 111 (in cl')llars
and lli'S.umcd C()tiS;IUnt) of drivers, and g::ti: cort<;urnplion. T he company
hus duta 1.0 ~upport the hypothcsi5 thai l'or speeds V lx:Lwt,:cn 80 kuv'h

and 100 k111/h.ll>c number of kilometres per litre usc<l by tl>c tnocks ls

n l inear functioo f(v) = l l - bv (wl>cre b > 0). Fincl a formula for


the reconrncnd<:d spcOtl in ICn ns (lf a. b, and w . and p, lhl.! price per
55. Tile mad i nfo1malion sign in the following figurcspecifi.es distance
10 I he ncxl t ily with markings l m high. Bouoms of m!trk.ings !arc 5 UJ
aboYe mad level. If Lhe average lllOLt)riscs eye level is I .5 m above
ground lcvc.l, aL whaL distance from Lhesign do Lhe lcUCI'.S appear tallest'!

litre for gas.

60. Verify formula 1.16 by minimizing dte distance functit)O from a


poinL 10 a line.

4.7 AMulule Ma'(ima:and Minima

quOOrilutcral lhat cao be $0 fo rmed.

"' 6J. A window is in the fonn Qf u TC'-'Iunglc sur01ountcd by u scm.ici.rclc


wilh cl i a n'M.~Ier equal 10 the widlh of the f'OCimtgle. 1lte tec1ang.lc is
or clear glass costing a dollars per unit area , while the scm.iclrdc is
of coloured ghtss C(>Sting b doJlars (>l'r unit area. 1llc coloured glass
tr:m.smits ooJy a (racti.oo p (0 < p < l ) us roucb li,gbt per uo.it 3.(CU
CIS the dear glass. h~ C~ddit ion , &
he curved portion of Lhe window is
sunnoWltc.d by a special fralllc Ill a cost of c dollars per w~i t length.
U the tOLal cost of the window mu.~t nol exceed A dollars. find the
di.tnco.sions of the \vin.dow th:u :ttlt:njt tOC most light.

* 62.

9)1

291

36 clo~s h) ..._,.. 68. The lake in the figure below is basicaUy circular witb radius r. It
h :J ~ a narr<w Slrip (l( beach tt11 round ~tnd beyond the be'~h i~ bu~.
You arc walki.n.g frutn point P to point Q, both of which arc on t.hc
63. The fram..: for a kite is to be malic from six pitXcsofwood as shown
ex1ension of a di amc,er of the lake eac::h distance J fl'o m tile cc.ture of
in the foiJowlJJg figure. T he fou r outside pieces have prcdctennincd
the lake. You can walk twice as l'ast o.n lhe beach as in the bush. Design
Jc:ng.tl\S a and b cb > a). Ycllo be t.:ut oln:: the two c.liHgonaJ pie<.-es.
your travel path t o get fro!ll P to Q as quickly as posl!iblc.
How long should they be in order to make the are-a of ahe kite as large
a~ possible?
Fi1td the poirn 0 11 the ellipse 4x 2

(4. 13../5/6) .

take
Q

(/

{/

"'*

69. A rectangle has widlh w and lengrh L. Fiod dle areaofthe largest
of all rcctw'lglcs Lhat ha\'C sld<:.s passing through the corntI'S of the gi\'Cll
rectangle.

.+

70. There arc u red ~ luke$ securely driven Wt.o the ground in a strujght

* 64.

Suppose that the line Ax + Ry + C = 0 in1e~ccts 1he parabola


y =ax' + bx +c at poiniS P and Q (ligure below). Find !he point R
on 1he pa~ rabolu between P and Q tJla~L ma.xi.m_il:c.t- 1hc om.:a of tri ~ngk

PQR.
y
y = ax2 +bx+t:

line. A blue Slake is 10 be add<XI to the line. and each red stake is ro
be joined to 1he b lue one b) !l s tring. Where-,:;hould the blue stake be
placc:ll i11order that the totallcng-lh of all strings be as S-1nal1 as possibk?

**

71. When blood flows through a vein or artery, it encounters uist~mce


due to friction with the walls of the blood vessel and tbc "'isoosity of
rhe blood itself. Poi.\cui/lc slaw for laminar blood now states that for a
c-ircular \'C$'el, re$iStance R l(l blood flow is pr0p011ional lo the: length
L of the blood vessel and inversely proponional to the founh power of
hs rddius r:

L
R = k- .
r
wbcrc k is a conslanL The figure below shows a blood vcSScl ol' rudius
r 1 from A to 8 and a branching vessel (rom D to C of radi\ISr 2 < r 1.

** 65. nwgent lines to the cur\le y

= ~ make triangles: wilh the


positjve x- and y-:t)(es. Find the-area M 1hc s ma11es t t:uch lri3ngle.

66. A fanner has a squo.rc ploLor u.nirrigored lands metreS by s metres.


His J)I'QfiLis a dol1ars per square rnc.(n;. He wishes to insta11 an irrigation
system thai consisls of a !'Oiating am1 Ihat ph~s at the centre of the
fieJd. The cost ror Lnig:ttion is c dollars per square metre. Protit on
irrigated land is b dolhu-s per S<JUarc metre. \Vhat shotlld be the length
of the rolating arm for ma:~ i mu m profit? Assume th:U b > a + c.
otherwise it makes no sense to inigale.

**

67. Four r(X'IS of lengths I n. 4 m. 2m. imd 3 m 11rc hinged together


as shown below tO form a q uadrihHt::ral. Find tht:: area of tht:: largcSI

2r 1

--.A~--~~~~'~

_j_

Lt
The rcsi.stuncc e ncountered

by lbc blood in flowlng from A to C is

given by

L,
L,
R=k - +k - ,

r:

when:: L 1 and

ri

L?. arc Ihe lengths of AD !md DC, res-pectively.

(a) If

8 is assumed to

the right or

expres~d i n term5 of 8

E, show th;11

a."

k
k
R ; f(O) ; .-(X - Y co< O) + 4 Y cscO.

r1

r1

where X and Y arc the length< or t1 and C . respecth'ely.

(b) Show th:ll f (tJ ) h:ls only one critical pointli in the range
0 < 8 < ;r / 2. and i'i i< detinet! by

r~

.......
r,

(c) Verify that if yiehls a relati'" minimum ror f(IJ).


(d) Show th:\1

- = k-......-X+ -kYFGJr
)s <
,
I -

f(IJ)

r,

rl

....!.

.-, 74. A ro~ \IIlith a ring al one end is p:a~d through t wo fixed rings m
the s:tme le\lel (figure below). 'rhc end or Ihe rope wilhOut lhc ring is
then passed lhrough the ring a t I he other end. and a m:s~~ m i" attached
to u. rr1tle rope 11l0\'CS so a." 10 maximi:tc 1hc dtstance from m 10 1he
line throogh lhe fixed rinas. find anale 8.

Ring

Rins
f(rr/ 2) .

'

(c) Doc< ii prmodc an ab<o!Ule minimum for f(IJ)'I

** 72.

A right circular e<mc has radius Rand hcig_ht H . A nght circular


inscribed inside the cone so that its upper edge is on the cone

C)Iinder is

(figure below). Find the radius or the c-ylinder io O<der that its surracc
area (including top. bottom. and sidc) be as large as pOSSibk:.

15. rr the fencing in Example 4.22 is to form the ace or a circle.. what
is lhc ma.x.imum possible area?

.,.

76. The UPJXYiefccomer of a piece of paper o units wide and b units

long (b > il) is folded to the right edge as shown in the following
fisurc. Calculate length X in ()('dCr that th k ngth y or the fold be 3
minimum.

2/ { -

** 73.

1'hc: trhmglc in the following figure rt:ptelien l~ cros:ssccrions of ~~


,.-ism (the length or the pri<m heing pcrpendicul:lt ro the page). A my
of lis,ht (in the plnne of the page) is incident on the prisn\ nt ongle i
reluLi\'C: to the normal to the prlsm. The light is n;froctcd ru the two
faces uf the prbm und Jca,~ ;at angle of de' iution "' rclativ..: to the
incidcnl direction. SntU 's rm.. relates angle) of in..:ident and rcrnu.:tec.l
hght al each of lhe face~. 11 sto:ttcs lhal
ns:ina =tini

ami

11

~>in fl

sin if',

wherl! n is lhe index of rdroction of the Oll.tcriol or the prism.


( :l)

Find V :.\$an ~xplicit function of i.

(b) Show that the an:le of de,iation tj is a minimum when

; = ,P. Pro\<ethatif\'f,.. isthe minimumangleofde,ialion.


then
n

sin l(v~

+ r )/21

{I

sin (y / 2)

(cJ 'Ibis cqu;nion c.an be w.ctlto Uctcrmine 11 cx.perimcntally.


Angle of incidence i is Vl'tfit::d untiJ tfrm is achieved 11nd
tneasurod . Witb y !also known, u c~m bt:: calculated.

** n.

The li ne y

= mx + c intersects the ellipse b2.t 2 + a 2y 2 = a2b 2

in two points p a nd Q. F'ind l hC point R on


triangle P QR has (naximu(n pOSSible area.

lht!

t::lljpsc:: in order th!ll

4.8

Velucil)' and

At:cderal~l

293

lih<:ct o f metal a metres wide ;md L mcircs long is to l.x:: bent


into n trough o:<> shown in lhc fo llowing figure. F.nd pieces areal~ 10 be
au.ached. If edge A 8 must be the arc or a ci.I"Cic, dctctminc the mdi.u.s

t* 78. A

of the ci rcle i n order that the t.routh hold the bigJ;.<.'Sl possible \'Oiumc.

14.8 Velocity and Acceler ation


Kincmmics is the s wdy of mOlion -

rclmion:.hips nrnong the displncemcnt. vclocit) . r.nd ac

cclcration or a body, pmicularty as they pcnain to forces acting on the body. In this section we
for a (inal d iscuS-!<,ion c,vcything we have learned about displacement. "elocily.
and acceler-oiltion, but do so mainly i11 a one-dimensiollal se-tti11g, and only from a dift.crcntiation
poi111 o f vie w ~ g iven the posit ion of a panicle moving along a StrJight line, find its velocity and
acceleration and use these quantities to describe the moliOJ\ of the particle . l11 Section 5.2 we
reverse these operations: beginning with the acceleration. we find \lt!locity und JXlSilion. This
process is essential to physi cs a nd engine.e ring. where acceleration of a ~lrtklc is dctcrminctl

brin g together

by

the forces act.ing on it.


When the position of a particle movi ng al ong the

J'

-axis is known as a function o f t ime 1.

say, x(t ), its lnstantru>eous velocity is the dcrivmivc of x(t ) with respect to 1,
(/,r

v(l ) = -

dt

(-I 'J )

11>e instantaneous accelcrntion of the panicle is defi ned as the nueof change of velocity with
respect to time:
a (l ) =

ll

dv

t-1. 10)

dt

IJ:
.t

soo
-tOO

(~.

324)

300
200
100

15

- too
(1 1. - 176)

In Section 3.6 we: disc\Jsscd the motion of a panicle moving along the x~axis with po.~i tion
runction

x(l) = t 3 - 2712

+ !681 + 20,

1 ::: 0,

where x is measured io m etres aod t i n seconds. We now add a tjoal touch to 1he d iscussion.
The best way to describe the motion of Lhe pa1ticle is with a graph o r the d is placemcm function

(Figure 4.71 ). With calculus we can show how geome.tric propenies of the graph reflect imponant

feat ure:, aboulthe vdocity and accc:lcrution Qr the pmticlr. The YelQcity and accclcratiQil of the

partide .. re
U(!)

dx

= -

fl(t ) =

dt

.,
Jr - 54t

d"x

-,
= 61 tit

54

= 3(t -

+ 168

= 6(1 -

4)(! - 14) mls,

9) "'' ''

Ignori ng OlC physical inlcrptctations of t/.1'/dl and tl2xfd12 as velocity and acceleration for
the mo1nc.nt. nnd concentrating onl)' on the tUc1 that lhey are the first nnd second derivatives of
the function .1'(1), we immediately li nd thm x(t) has a relative maximum of x (4) = 324 and a
relative minimum of x( l4)
- 176 at the critical poinL< t
4 and 1
14. 1l>e graph ha< a
point of inflection at (9, 74).
Let us now d iscuss whm the gntph tells us about the motion of the particle. Ordinate.s
represent hori1.ontal distnilres of the pan icle from the origi l x = 0. When an ordinate is
positive, the panicle is that d istance to the right or the origin: when an ordinate i!<o negative.
the panicle is thm distance 10 the left or the origin. Fo1 instance, a t time 1 = 0, we c alculate

20. and chcrcforc the particle begins 20m to the right of x


0. At time 1 4. it is 324 n1
to d1e right of x = 0. and at 1 = 14. it is 176 '" to d1C left of the origin.
The slope of the graph represen~~ velocity of the particle. When slope is positi\'e, namely
in the intervals 0 < 1 < 4 and 1 > 14, the particle is mo,ing tO the right along the x-axis;
for 4 < t < 14. velocity is ile"gc\tive, in(.)iCi:iting that the particl e is mo,ing to the left. At times
t = 4 un<.J 1 = 14, the l>article is instantaneously at rest.

T he concavity or the graph renects the sign or the ;;tcccleration. R.w 0 < I < 9. the
gmph is concave downward. it.~ slope is decreasing. Physically, this m~ans that it has u negative
acceleration: that is. its velocity is decreasing. Fort > 9. the gn1ph is concave upward . iL~ s lo pe
is if'lceasil&. Physic:llly. acceletatiol is llO\iti ve; thf'l i ~. velocity i$: ilcrea..\ing. At the point
of inflection (9, 74) . the ucc.elenuion changes s ign. Notice that the acceler-ation is not1'ero at

r = 4 and 1 = 14. In spite of the fi1Ctthat thc veloci t~ is zero at chcse ti n>es. the acceleration
docs not vanish. You ntig,ht ask yourscl f what feature of the graph would rcAccccoincident zc.ros
for velocity and accelenuio.1 ( ~e F:xel'cise 16).
Speed is the 111~1gnitude of velocity. 1t rc1'rcscnts how fast the panicle is moving without

regard for direction. For instance, at time 1 = 0. the velocity and speed arc both 168 on/s.
whereas ut time I
10. velocity i:; -72 nlls and speed is 72 ntfs. Geometric~llly, speed is
rcJ>rescntcd by the slope of ti1C graph without regnrtl for sign.
With the ideas above in mind, let us detail che history ol' the varticlc's motion. 1\t time
1 0. il begins 20m 10 the right of the origin. moving to the right with velocity 168 ntis. Since
the accclcrmion is negative (to the left). the pa1ticle is slowi ng down (both velocity and speed
nrc decreasing). until ut time 1 = 4 s. it comes to un instanwncous stop 324 m to the right of
the origin. Because the acceleration continues to be negative. the particle moves to the left. its
velocity clcc.rcasing, but its speed increasing. At time I = 9 s, when the particle is 74 m to
the righL <)f the 01igi n. thea<.'t-clcrntion crumgcs sign. A t this instant~ the velocity hns attained~

(relut.ive) minimum value, bul speed i:, a (relative) muxim urn. \Vilh uccelemlion to the right for
t > Y s. t he partic le continues to rnove left. but slows down. i ts \'Ciocity increasing until nt time
I = 14 s. when it once again oonlCs to a scop 176m to the left of the origin. Forti me 1 > 14 s.

it moves to the right, pick.ing up speed. and passes through the origin just betbre 1 = 17 s.
Further analysis of the imerdepcndences of d isplacement velocity. a nd acce.leration is
CQtll.ained in the following example.

I EXAMPLE 4.29
The. positjon of a partk le moving al ong the x-axis is given by the functjoo

.1'(1) = 3t

321 3

+ 11412

144t

+ 40,

0 ::;

I ::;

5,

where .t is measured i11 metres and t in seconds. Answcrthc foiiO\\ing quc:,tiOlS concerning

it:> mo1ion:
(a) Wh>U arc its velocity and s peed at 1

= I /2 s7

(b) When is ilS acceleration incrc.asi1g?

(c) Is the velocity increasing or decreas:ing :H

= 2 s?

(d ) Is t he panicle speeding up or slowing down at I = 2 s?


(e) Whnt is the ma<imunl velocity o f the particle in the time interval 0 ::; t ::; 2'?
( f) Whm is the n~:l:<im um distance the particle ever attains fron1the origin?

SOl U 110'1 We use the !!"'Ph of thc<.lis placcmcno function in Figure 4 .72 to suggest anSwC1"$
when possible, and calculus to eonfinn them, when necessary. T he velocity and acceleration
ttre

u(l )

= 1211 -

961 2 + 2281 - 144

= 12(1 -

I) (I - 3){1- 4) m/s,

a(l ) = 12(312 - 161 + 19) nv's1.


MjUIII;IXftJ

Displacement function

10

30
2()

- 10

-20
(a) TI1e velocity is v( I / 2)
(b)

= -1 05(2 m/s. ,

,., speed is 105/2 m/s .

n,e graph does not tell us whe n accclemt ion is increasing. i\lgcbraicully. accclcmtion
is increasing when i1~ derivative is nonncgflti,c. Since da j t/1 = 12(6/ - 16)
24(31 - 8), acceler.otion i.< incre>L<ing ror 8/3 5 I ::; 5.

(c) TI1e grnph appears to be concave downward at t = 2. implying that velocity is


decreasing. Confinnation is provided by the fact that a(2) = -12 nv's 1.

(d) TI1e gmph seems to be beco ming less stCCI>around I


2. indicating tha t the panicle
is slowing down. Since u(2) = 24 m/s. the partic le is moving to !he right. But
according to part (e). its acceleration is to the len ut this ti111e. Wi th \ Ciocity and
accelemtion in opposite directions. the partjcJe is slowing down.
(e) Maximum velocity on 0 :5 1 .$ 2 occ ur~ at the point of innc:ction to the lc:ft uf 1 = 2.
To find it. we set

= 0 (1) = 12(311 - 161

Of the two solutions ( 8

+ 19).

../7)/3. o nly (S- ../7)/3 is in lhe interval 0


Maximu m velocity is therefore v((8- ../7)/3) = 25.35 mJ,.

2.

(f) Maximum disLance from the origin is represented by the point on the graph farthest
from the t -axis. This is at 1 = 5 s, for which x(5) = 45 m.

296

Chlipt< ~

-\f'Piic:ltions of Oitrereoli>lion

I EXAMPLE 4. 3 0
The mass M m Figure 4.73 is pulled .ro meues to the right of its position where the spring is
un>tre.tched, and then it is releascll. If at ti111e 1 = 10. duting the subsequent oscillmions, it is
stru~k with a force or F newtons to the right, its position x(r) mUSt satbf~

d2x
M1

+ b = n(t -

dt

x(O)

to).

= xo.

x'(O) = 0,

where k > 0 is the spring constant. and 6(1 - to) i~ the Dirnc-<leha 1\mction of Section 2.5.
This assumes that air resistance due to motion is r~tli\l igible. as i~ friction with the surface on
which oscillations take place.
(a) Verrfy that
x(t)

{k
= xo cos,( -;;'

' /11

{k

sin , / 7;(1 - to) lr(t -to).


v kM
' M
/'LU

where lr(t -111 ) is the Hcavisidc unit step Iunction. Stllisfics these conditions at every
1 ~ 10

(b)

Show that the velocity of the mass changes by FI M metres per second as a result of
being struck al 1.imc tu .

~~-----

' 000'

\01 lfflOI\

(a) Since the derivathc of 11(1 - to) = 0 for any 1 ~

Thus, for r
111(d.t

dt

we find that for I ~

to,

-I~x.
sin rr,+ .::. cos ,~<, - '> h(t - '>
\ M
VM M
M

x'(l) =

t"(r)
.

10 ,

-~.;os
M

p;

F .Jk
lk
-,in,( -;;(1 - lo) lr(l- /o).
M >ll
' M

-1- M

'# r0 ,

+ kx =

M [ - kxo co>

V/k,
iii -

F .Jk
M )/l

sin \~(/
M

+ ~ (xu c< \~r + _!.._ ;in (k(I

lo) h(t - lu)]

VM

lo} h (I - lu)]

that is. x(t) ..ati,fic the differential cqudtion M A"+ kx


In addition. x(O) = xo. and

F8(r - tu} for I

Jk M

= 0:

x'(O) = [ -

~xo >in Vfk,


+ F cos (k(l Ai M VAi

'M

= 0.

to) h (t - to)]
lrll

'# to.

(b) The change in ' e.locity at I

lim .t'(t )
,_,0
.

lim x'(t )

t - ~a

= fo is

= ,_,(1
lim, [-~MkXo sin , ({;,+ ~ cos .{J;(t .
- lim [- {kxo si"

,_,~

VM

to)]

{kf]

VM

= -l~xo in /J;to+ ~ + \{f;xo.<in /J;to


=

F
M

f EXAMPLE 4 .3 1
Rod A B in the ol'fbet 5li<.ler-cmnk of Figure 4,74 rot ales countei'ChK:kwise with conMant ungular speed w ~1 bout A . End C ofche follower BC is confi11Cd to stmight-linc motion along n
horizo nt~ll line between f) and E . Find expressions for Lhe velodty and acceleratioel of slider

C.

Suppose we let .t be the dislllnCe from F to C . Then .

SOL UTION
.t

= IIFGII + I GCII

= 11.'\H II

+ iiGCii = rcos8 +L eos .

Now angles 8 and 1/J are not independent: they arc related by the offset equation
<'

= IIB(; II - II BH II = Lsin - rsio> 6.

Although these equations have been de"cloped on the hasis of F igure 4.74. which shows$ as

an acute angle. it can be shown that they are valid for any value of 0 \\1latsoever. We could use
1he offsel eCJual ion to express .t completely in 1enns or 0 (see F.xcrcise 24), but it is simpler to
work with both 8 and</> . Diffcrenthuion of the expression for x with respect to time I gives the
velocity of C.

dx
de
d<f>
v = - = -rsin(i- - I. sin</>- =
dt
dt
dt

- (J)r Sill v -

A.

d</> .

v Stll Y' -

dl

Differentiation of the offset equation relates d/dt and w.


d
d8
dif>
0 = Leos </> - - rcosll - = I. cos - - wr cos 8
dt
dt
dt

d</>
dt

wr cos (J
L cos

298

Dupler 4

ApplicatiOI'IS or Ditfertnlimion
Thu s~

cvr cos ())


v= - wr sin 0 - L sin <f> (
Leos
= - wr (

sinO cos+ cos() sin )


-wnin (0
=
cos
cos

+ )

A second diffcrcmiation g,ivcs lh<: accdcration of the slider,

a=

dv
dt

coscos(9 + )

= -wr

(d(J
+ d</>)
-sin (9 +)(-sin/
dt
df
df
cos2 1/>

=
=

~;~ [coscos(9 + ) ( w + 7c:s;) + sin(9 +)sin (w;,:s;) J


- ulr

L cos'

[cos<t>cos(9 +</>)(Leos+ rcos9) + .-sin(9 + cf>)sincos9]

<I>

EXERCISES 4 .8

In Exerci~c~ 1- 10 lind the \'Clocil)' and :tccele!'21io.n of an object that


move.(; ~l long the x-axi!: with lhc given position (unction. In each exercise. di~lSS Ihe motion. including in your discussion a graph of the
funcli<>n .r(t) . Assume thal .xIS me<t"urcd in melrcc: and tin seconds.
I. .t(l) 21

'

+ 5.

2. X(l) = t' - 11
v 3.

I 5.

+ 6.

- 21'

x(l) = 1> -

I ~

91 2

(b) Use Newton's ilcrati'e procedure 10 find .1ten \'tkx:ily is


tero.

I ~ I

I.

+ !51+ 3.

= 3co<41, 1 2: 0
* 1. .r(1) = l / 1. 1 2: 1
o K .r(l) =I+ 4/ 1. I 2:
9 . .<(1) = (1 - 4)/ 1'. 1 2: 2
I 10 .<(1) ~ (1 - 1) 1,/i.
I 2:

I ~0

(c) Uset\cwton's iterati\'eproctdureto find ''hcn an:eleration


i::. ((lualtO I.

I ~2

6. x(1)

t' - 91 2 + 241 + 1.

0 :So

:So 6.

0 :So 1 :: 4.

by rhc panicle.

... 15. Repeal E)r,CfCisc 14 for the posilion function

x(1)

inc rcas.in~.

12. Repeat Exercise II ir

= 2- 151 + 91 '

31' - Ui1 3 + 18t 1 + 2.

interval& the speed i5 incrC11$ing 11nd dcc~ing~ (c) when the \l:locily
i ~ u maxinu.1111 und a minimum. (d) whc.n lbc ~is a U:kl.:\ imum and
a minimwn. (c) lhc nKUimumlliMance rromthc origin ochit\'Cd by the
panicle. and <0 ah:.: maximurn diStance rrom the poin1 x = 5 achiC\'Ctl

mall:imum and minimum \'Ciocity. (e) ma'(imum and minimum speed.


(d) nl!U:unutn and minimu1n :lCtt~rntion. and (e) \vhen accell!r.'Uion i(

x(t)

where :c i5 measured in mt:tres Md t io seconds. Dctcn:ni.nc (a) 0.1


wltal intervals the velocity is incrt.asit and doc:reMin~. (b) on what

whctc x is mcasutcd in metres and 1 :!. 0 is tilflc in socortds.. Determine (a) whcthcl' spcct.l is incn:.asin: or ili."Crcibing a1 t = I s. (b)

14. An object monng along the x -axs has pos.ilion funclion gi\<en by
x(l )

+ ll. An obj~ L n\O\'ing along the :c -o.xis has position given by


x(1) =

0 :5 t :59.

(a) Plou groph of x(1).

+ 21' + !61 -

13. A particle nlO\'i':~ along the .t -axis in such a w3.y th:u it~ prn;-ition
as a function of aime 1 is given by
x(l) = tsin1.

I ~5

x(l ) e 12 +51 + 10,

4. .<(1)

I -t

*
- t 3,

:>

:>

6.

141'

101t 1

1321

-4S - -4S- + -+ 2.
4S

16. Whal feature o n the displaccmcnl graph would indicate a time

whe n the velocity aJld atceler.ujon are simultaneously ?..em?

4. 9

17. Are critical point$ ror thc \'dOCity (unction thes:uuc t t') those i'o r
the .speed function? Explain, using graph.s.

t:

l tf.. Wh:l1 t.S the R\aX:imum ~fX'Ctl of the: pu1,iclc in


the lime intcrvul 0 :S: I :5 2'?

E>..!.UllJ>l~

\Hi u c n

E~li m;.ttc 1hc length oftlle...arok.; frornthc graph in pro-~ (b),


Check lhi' ~g.ainst the formula in Example 4.28.
(d ) A~SULlling thlll A IJ I'Oiatcs with CO!ntc.n t anguhu spc.~d w ,
diiTen....nti;Ue the funetion in part (Ol) co find the veh.,city uf
th~ :-I ide in 1cm15 of I). Vcriry th:.n )'OUt' ~h it'con~ilttcnl
with tJ)C velocity fornUJin in E'<nmple 4.31.

4.29 0 \'tl'

(c) Plot the velocity fune1ion in

in the ronu

It"''

* 20.

When posi tion has a Jclati\e m.-ximum. sa does l hc absolute \'alt.tc


o f the distance fnxtl the origin to the particle.

" 2 1. Whc~ position hus u n:l.1tivc nlinimurn,l)Q docs the absolute \"'.Iuc
ol'thc c.li$ttu)(.'C rrutn the ori&in to lhc p~art id~:.
t.

22. When \'Ciocity has a relative tninimum. s:o d~>es speed.

11

2.3. When \d<H.:ity has a r<:IUih'C maximum. so doc ~ speed.

rcosO

t. 1

l!i. A landing approach is lO bcblltiJX'd gcnc.rally ;es ~'10\v n in the fi~ urc
he low. 111C following condilioos arc itnpo~ed on Ihe."ll'lln ltiCh p:mcrn:

(n) A.h.imdc must be IJ metres when d(';S(."Cnt ~01nme nces .


(b ) Smooth touchdown must OC(.."\lr aL.\'

= 0.

(c) Cllnuant horizont:.J speed U nlcuc,: per St.'Cond must be


mainaained 1hmugh01.11.
(d) t\ t nn tim( must vcrticul oocclcration
cccd :~ bnlC liKed t><:u:ithc consu11t k ,

i' abso lute 'o.tlu~ c,.,~

Pinel wl\cn dcsctnl should conuuence..

II

+ J t,>- (e + r:>in0)2

(b) Pkn this fuJK..'l.ion fot 0 ~ (J .:5 2;r when L


r 2 em. !md e I {'1n .

21r

(c) T h<: gl illc path nm:a ~ u cubic polynomial.

Usc the oiTM:t cquution e = I_. -.in 1/J - f e.:in 0 to shm'' thst
position x of the s lider in l::.l;nntplc 4.3 1 can be ext>rcssed
in the runn
X=

:s () ::,

o ne re\o lulion coch second.


-zero when the position graph in part (b) if: !U it:=: lli~hesl and
h1w~ ~IJOi nl.$? E!>limatc maximum and minimum vcloc:itic:.~
from the g raph.

In E~c:rci~>cs 20- 23 a"-Sumc


u p;~rtidc lllO\'~"" along the .'< -axi1; in
such a w.n y t h~t it..s J>OI';ition. velocity. and tlocelct.:ttion are continuous
runctions on the interval a :5 1 S b . Ui ~u:ss the valid ity of each
statement

(!t)

(d) t'o t 0

r itl purt (b) i f A 8 rotates


Doe:; the 'cklci.ty r~ppc.lr lobe

tlv
dx

24.

~n

u~ing the val ue~ of r . I~ . und


( I = tJ - .

i!

299

(c)

19. In many velocity wld accclcr:.uion problems it i ~ mvrc convcmcnc


lo C)(pc'CSS ac:c~ J en!.lion in tenns ()( tl derivative with n:spc.<:l to posicion,
a.;: OppOSc..--d It) a t.lerivutivc with respect to Lime. Show lh:i.l acceleration
<.-;en be

Kd:.k'\1 RatcS

= 9 em,
X

14.9 Related Rates


Many interesting and prnctical problems iowol,.ing nHcs of change are commonly referred to as
r~lated r ate problems . In the.se proble111s. two or more <lllantitics arc rclaled to each other and
nues cu whkh some of lhem change are known. his required to find rates at which the others
change. Rel:uecJ rnte problems deal a lrn~lSt c:xrdusivcly wi th ntcs of chunge of quantities with

respect to time. To solve these problems we first consider three examples. These will suggest
the general procedures by which all related rate problems cm be anal}'1.ed . 'Ve shall then discus.~
1wo somewhat more contplicatcd problem~.

I EXAM PLE 4 .32


Shtldow uf

man ..... LIIdng. t~way fi<Hn

l ighcpo~t

A man 2 111 tall walks directly uway fmm t1 streetl ight that is 8 m high at the rate of 3/2 m/s.
How fast is the length of his shadow changing,?
SOLL'TION Whe11 x denotes the dista11ce between the man and the lightpost (Figure 4.75).
the fact that he walks directly away from the light at 3/2 m/s means that x is changing m a rate
or 3/2 on/s; that is. dx / dt = 3/2 on/s. If s renl!'esents the length of the mans shadow. then we
are searching for ds/dt. Similar uiangles in Figure 4.75 enable us to relates and x.

x+s
8
=
s
2'

and this equation can be solved for s in tcnns ()f ,;r:


X

s Now.

s ru1d x are each function of tirne I.


s = /(1)

a nd

g(l}.

although we ha,c not caku latctl the exact form of thc-:sc: fun.;tions. Jntlc:ecl, the: essence of the
related rJle problem is w find dsjdr withoul ever knowing /(1) explicitly. To clo thi$ we nme
that since the cquution s = x /3 is \'Oiid nt anr lime 1 when the man is walking away from the
lighL we may dill'crcntiatc with respect to I to obtain

ds

dr

I dx

= -3 d1

This cq umion relates the known rate dx /dl = 3/2 with the unknown rmc ds / dt. It follows
thai

~-; =G) G)= ~

and the man's shadow is therefore getting longer at the rate or I /2 ml~.

Knowing dx /tit i n Ihis example~ we ho.H'e calculaletlt/S/tlt ?tlnd h;wedone ~o wilhout llnding

s expliei!ly as a fu nction of lime I . This is the essence of a related rate problem. Since dx /dl
is a constalll value. i1 is qui1e easy 10 finds as a function or 1. and hence ds/ dl. Indeed. if
we choose time I = 0 when the man stans to walk away from the streetlight. then his distance
from 1hc light at any given time is x = 31/2 01. Combine this with the fa~tthat s = x/3 and
we may write

s =

H~) = ~ m

Wi1h lhis cplicit formula for s. it is clear lhat ds/ dl = 1/2 . What is important to realize
is that lhe solution in 1his paragraph is possible only because the man walks a1 a constam ra1e.
Were his speed not cons1ant, it might be impoosiblc to finds explicitly in terms of 1. The next
example illu&trates this point in that the given rdle is kno\vn only at one instant in time.

I EXAMPLE

4 .3 3

'l!Im. !Il

r...ldcr

~lld1n.~ dO\Io'n a wall

dy

--1
dt

20

A ladder leaning against a house (Figure 4.76) is prevented from moving by n young child.
Suddenly. something di str<~<:ls lhe child and she releases the ladder. n~e ladtk:r begins slipJ>ing
down the wall of the house, picking up speed liS il falls. If lhc top end o f the ladder is moving
at I ml> when the lower en<i is 15 111 from the hou:;c. how fas1 is the fool of I he ladder movinll
away from the house t\l this instant'!

SOLUTION Figure 4.76 indicmes 1hat when y deno1es 1he heiht of the top or 1he ladder
abo-e 1he ground, then dy f dl = - I mls when.< = 15 m (the negative sign because y is
decreasing). We emphasize here 1hm dy/dl = -I u11/y when x = 15. \Vll<u is required is
dx j dl when.( = 15m. Because 1he lrinngle in 1he figure is righl-:mglcd. \\<e may write
x2

_I ' - - - - -----"'--1- - - x - --1


dx="latx=IS

dt

+ .y2

= 202 '

and lhis equmion is valid a1 any 1irne during which 1he ladder is slipping. If we differenliate
with respect 10 timer. using ex1ended power rule 3.21.
d-~

2x dr

d\

+ 2y - '

dr

= 0.

\VJtCII .~

15, we ~l.::u l:ole ltltol

y = .,/400
Jjil /t

'"

225

5../7,

$./1(- 1) -

;1!1 \.f

Chc:~(Qn:' ill litis in ~l;;<illl

0.

Thi$ yickl.; d.,.jdt = ../7jJ. ;1t1d w " '" ' SJoY rhm when the fuo1 vf the ladd~r is ISm (~)m the
w,,ll, lt i~ movin;; lfW"Y fr.;,m the WJt ll ;If ,f'ff~l mill .

I!XAMPL.I!! 4 . 34

...

;\ t.;lllk 111 f.h<" fOrm ora ri~he drculm' (.'C)Ite v.'id'l id dcudeO nlO!tHJ base rnd itH 3 tu ( Fig u re 4 .77) i.s
l' duh! /ilk"(.!\\ flit W.tfCI'fJf n n uc o f 4 000 l../miu. I fuw tit ~l j.., 1hc "urft~~CC o f 1hc \\"tii CT r-i~~;i ns when
tile dcpdt Is 3 m1

SOUJT IO' ' Pi.gurr: 4.11, which illu!lltatc..<? a cro.s....;-sc<:1ion of the t.;mk, in(liclltCS thflll
dCJifh q(w;ucr i11 d te c.alk ~ D. th<" volume V Q( willet is

w h~t'l

the

Ot'tllc cht't"e vuriabJcs V, r. 41ld {)ill tlli,.. cqu:~tkwl. we arcCJOtX.'t-tncd Otlly with V iuld l). ~in..:c
dV/ til i $JJi\C'tl lli){J dD/tll h whm W<" w ;m. 1lli.s suggc,..;l~ !hilt \v tlimil~U O r uJOing $imilar
uilmglcs. Si1K.'e rjf) = :3/6, we haver= D / 2 : cherdbre,

' ('->)' D - -11'


' D '.

= 3- n

12

l}c(','tU\'C tlli11 n::o;ult j,.. wdicJ foJ' ttfiCime I d iiTill,; lhC: fillill proCC:.SS, WC Cllfl <Ji ffert ntiau: With
re10pcct to 1. o nce 11gnin usins C'l:tCIIdcd power nle 3.21 :

J
,tiD
-dV
= -11'
/) - .
dt
4
dt

Si11<:e dV/dt =4m3/min (C(lnvencd from rhe nue of .U)()() Umin since the litre is nol an
AI.'('CJ)tabre unit of

n iC~bU f'C

fur volume), \\'C th1d IJwc whe11 D

4
from whic/1d D / dt

= 3.

= ~rr(3}'dD .
4

dt

= 16/ (9rr) . The su11aco is thcrcliJre rising ot nuc of 16/ (91!") m/min.

"11re.<o rxmtoplc., i/lu<trllto the following gcriCNII pnx-cdure for w iving related nue problems:
J. Sketch" diagnrm illustrating all gi,c n informmion, especially gil"en rates ()( chMge and
dc.')jrc(l r.uc~ ofdumse. Do not dnnv rhe di1cgr~'"' ~- the in~c;uH in CJtestivn; drlw.. it slightly

before. or slightly after.


2. f ind an eq uation valid for all time (in some interval about the instanl in question) that
involves only variables whose l'ates or change aJ'e given Ol'l'equired.
3. Differentiate the equation in step 2 and so lve for the required rate.

302

Clup1er .l s.\pplic:uiun5 of Ditfcrent i ~u ion

I EXAMPL E

Steps 1nn<l3 are USll(ll ly quile st.r-.Jightforward; step2, on the other hand. may ta.'< your ingenuity.
To fi nd the c<tmHion in the appruptiate variables. it may be necessary to imroduceand substitute
for additiona l variubles. Ji'inding these s ubstitut ions tcquircs you to a naly1.c the problem very
clnsdy.
Be carcf ullOt to sutMtitt11c numeria.l datn that re1)resent rhe instant at which the dcriva1 ivc
is required before di fferemia tion has take n place. Numerical data must be s ubs tituted after
diO"eren1huion. For i11.c;;tance, in Example 4 .34 , rudius r o f the surface of the water when D = 3
is 3/ 2. If we ;ubstitutc this into V = n r2 D /3. we obcain a function V = n (3/ 2) 2 D /3 =
3Jr DJ4. which is valid o nly whe n D = 3. It c"nnot there fore be d iiTcrcntiatcd: only equations
that a re valid for a r'dnge o f \~t.llues oft can be d ifferentiated with respect to f .
\Vc now appl y this procedure to l wo rurther examples. Titc fin;t is un ext.e nsion of Exam
pie 4 .32.

4 .35

A '""" 2 111 tall walks a long the edge o f a straig ht ro.1d I 0 m wide. O n the othe r edge of 1he road
stalds a streellight 8 m high . Jf Otc man wal ks aL3/2 nlls, 1\QW fW~t is It is shadow lengthening
whe n he is 10m fro m the point di rectly o pposite the lig ht?
ljC#Jil;l3 "l.il!J

Shado"" of man walldni:. awr fro"' strc;ctli,thl

,,,

tis :?au: 10

SOL UT ION First we draw Figure 4.i8. wherein the m~n~s speed is represenl.ed as the time
n:tc of change of hi s dist:utce .x from the poim on his side of the: road d irectly oppol'ite the light.
What is required is the rate of change ds j di ofthe le ,lgt h of his s h>1dow when x = 10 . To find
a n equation relating x and ,f. we fin;t use simi lar (vertical) trian gl e~ to write

)' ... s

- - = ;;
from which )' = 3s . an cqumion that relates
)'2

= 4.

to )'. rather than s 10

_.,2

x . However, since

+ I()(),

we $Ubstitme to obta in
The derivati,e of this equation with respect to time t gi' es

ds
dx
= 2x - .
dl
di

ISs When

x = 10. we obtain s =

.JJOO + 100/3 = 10,/2/3, a nd a t this instant,


18. (10,/2)
- - -ds = 2(10)

dt

(3)
-

4.9

Rcl:iled Rf11cs

303

Thus,

30

ds

= 60v'2 -

dl

and the man's shadow is therefore lengthening ot the rotc of ./2/4 rn/s.

It is wor1hwhilc

no1in~ i n 1his cxnmplc 1h111 were we 10 solve for

= 10 and s = IOJZ/3. then we would have

d s I d1 before substiiUiing

ds
x dx
dt = 9s dr
Since dxfdt is always equal to 3/2, and

ds
dt

= Jx1 + 100/3 , we can wri1e

(3)

= JJxl + 100 2 = 2Jx2

: 100

a 1\Cncral fonn ula for ds/ dl . The l imit of this nue as x becomes very large is
ds

lim -

<- O<> dr

= lim.

, ....."" 2.Jx2

+ 100

But for very large x, the man essenti ally walks di rec11y <~way f rom the li gh1, and this answer. as
we might cxpccl. i s idcntical1o that in Examp le 4.32.

I EXAMPLE 4 .36
One end of a rope is 1ied 10 a box. The other end is p>ssed over a pulley 5 m above the floor
anJ tied at a level I m above the floor to the back or a truck. I f the rope is taut and I he truck
moves at 1/ 2 m/s, how fast is the box rising when the 1ruck i s 3 m from the plumbline through
the pulley?
lijC.JII;U
tiJChco.l 10 a trucl: p;s.s.scd
llOJ)Iey

5 .0

0\'U 11

Lengt~ofrope=~
-

I ''Y:.?ru:.t":3
y "'
I
II

SOLt:TION In Figure 4.79, we have represen1ed 1hc speed of the 1n1ck as 1ho ra1c of change
of length x, d xjdr = 1/2 mls. Whal isrequired is dyjdr when x = 3. To fi nd an equ"tion
relati ng x and y, we first usc the fllcttho11he le11g1h: of rope bc1woen pulley and rmck is 1hc
h)'()()lenuse or a right-angled triangle with side, of lengths x "nd 4,
z~

= _,.,

+ 16.

For an equation relming y and z~ we note that the length of the rope, call it L , remainscom;tam,
z and 5 - y.

and is equal to the sum o f

L = z + (5 - y) .

1 -x--J
These

l\VO equations

can be combined into

(L - 5 + y) 2 = x 2

+ 16,

and differentiation with respect to Ljmc I now gives

dy
2(L- 5 + y) -d
.

+ y) 2

= 9

dx
2x -d .
.f

When x = 3. we may write that


(L - s

+ 16 =

25 .

We l'Ould solve lhis cquu1ion for y (illi.C1'11\S of L), btll it is rcully tlOI y thut is nc..--c::dcc.L to oblain
dy I dl from the preceding equation. It is L - 5 + y. and this is clearly equal to 5. Thus, when
x = 3. we have

dy
2(5)dl
that is, d y j d l

= 2(3) (I)
- :
2

= 3/10. lllld the box is rising at a rate of 3/10 m/s.

The reader shoult.l cumparc this example with the problem in Exercise 2 . They nu_ty i1ppcar
similar. but are really quite different.

Mechanic~ll engineers h:.we a question conu~ming chc mechanism in Figure 4.80. Rod

0 8, of lengd1l. rotates c'Qunterdockwise in the xy-plane around the origin at w revolutions per second. Rod A B . attached to 0 B. is such that A is COI)fined to s liding
horizontally along the x-nxis. For unresnic1ed motion, the length L of A R is greater
than twice/ . The engineen; wish to know the maximum speed attained by slider A.
I 0!1J!loJ ,,('J,,

SJ'(td of slider in a l-.r.ro-Nr

ntt'\:hani~m

'

A .,-

()

'

\.

,,/

'....... '

s----

SOI.l'TION Slider A moves back and footh along !hex-axis, repealing irs morion for
each revo lution of B. Th determine the maximum speed of A . we need only cons ider
its motion as B moves from (/, 0) to (-I. 0) along !11e upper semicircle. We therefore
take angle 0 in the interval 0 :;: 0 :;: 1r. The cosi ne law applied to triangle 0 A B gives
L 2 = f2 + x 2 - 2/x cosO. If this i s diiTerentimed with respect to lime,

dx
dx
d(J
= 0 + 2x-21 cosO -d + 2/x sin 9 -d .
dl
I
I

When we set dO I dl = 2rr a>, and S<JI\'e for dx fd t.

dx

dl

2/x sin II (2Jra>)

21l<vlx sin II

2/cos8-2x

/cos&-x

= -::-:----::---:-- =

m/s.

Two Lhings arc worlh noticing. Velocity is zero when sin 8 = 0, and Lhis is when 8 = 0
= 1r , w hen B is on the x -axis (as we would expect). T he denomi nator l cos (J - x

and (J

can never vanish since. x/ l is always greater than unjty.


To find maximum speed of the follower. we should determj oe m ioimum velocity since

A is moving left when () ::; (} ::;

;r . S ince this occurs when acceleJation is zero, we set

4,9 Rthno.l Rlllt)

2:rw1 sin 0 dx
lco>O-xdt

dO
1=0 -x dt

2::rwlx cos 0

2:rwlxsio0 (
dO
dx)
- /sinO - - (lcos0-x)2
dt
dt

2:rwlsin0

dx

= (I cosO- x}l(lcos9- x

2:rwlx

+ (I cosO _
=

- x)2

= ~rr ait'x>in

. ,

('l!rwl.r sinO)
I co:sO - x

0cos0

~rr 2 ailx(l - x cosO)

(I r:osO

+ 4tr 2 ft}lx(l(/cosO -

dO

+ Ism- 8] dt

.r)l (cosO(/ cosO - x)

2rcwf sinO cosO

(I cosO

+ x ) dt

- x)l

xcosO)(lco:.O- x)

,,y

mJs-.

We now se1 lhe numera10r equal 10 zero, a1 1he same lime removing 1he 1ac1or

4rr 2w2/x.

=
=
-

+ (1-xcosO)(/co<O -.r)
12 (1- cos2 0) cos 8 + 12 cos8 -lx - lxcos2 8 + x 2 cos9
2
2
2
2
- / COSJ II - /x cos 0 + (2/ + x ) COS 0 - /.r.
12 si n2 8cos8

= I' + x 2 -

This equal ion mus1 be combined wilh L 2

set

2/ x cos 0 10 yield x and 0.


y = cos 6. then we must sol\'e the followi11g nonlinear equatjons for .x and y.

Normal procedure would be 10 solve one of these equa1i ons for

in terms of x.

~ubstitut.e

in 1eoms of

y, or y

into the other equation. and thereby oblain one equation in one

unknown. Unfortunately, none of these possibilities seems appealing. If we set )'


in c::~ch of the equations. we obtain

When we cancel an

from the first equation. then both contain only x 2 '>,

(I - 2/a)x 2

= L' - / 2

When we MJhe each of these for x 2 . and equate results. we obtain

21 2a - I
llal

+ la 2

- a

L2 -

/2

I - 21<1

When we cro~5 multiply,

+ la 2 - a)- t2(1!a 3 + la 2 - a)+ I- 412a + 4/ 3a 2


(1 2 L2 -1 4 )a 3 + (IL 2 + 3/ 3 )a 2 - (L 2 + 3/2)a +I.

0 = L 2 (1 2a 3

If we

= ax

305

306

Chapter J

Applic:nitmS o10ifftrtnti:uion

This cubic equation n1ust be solved for a (once I and L are specified). When this is done,
x2 = ([.2 - /2)/(1 - 2/ll} gives the position or maximum speed, and angle is given
by IJ = Cos- (ax) . For example, if[. = 0.6 m and I = 0.2 m, the equation fo r a
reduces to

0.01 28a 3

+ 0.096a 2 -

0.48a

+ 0.2

= 0.
=

Of the three solutions - L1.0287, 0 .461 987, and 3.0667 of this equation, only a
0.461 987 is acceptable (a cannot be negative and the largest root leads to a negative V'dlue
for x 2 ). With this value of a , we find x = 0.626 529 and IJ = 1.277 lS. Maximum
speed of the slider is therefore

dx = 12rrw(0.2)(0.626529) sin 1.277 151 = 1.3253w.


di
0.2 cos 1.277 15 - 0 .626 529

EXERCISES 4.9

.., I. A CQ11\'Cr1iblc is lraw.:lling iilong ii ~\rd igln big:hwuy atlOO k.n:Vh. A


child in Ihe caraccidcmally releases a helium-filled balloon. which Lhen
rises \'Crtica.lly at 10 nlls. llow fast are the child and baUoo1l separating
4 s aftt t Lhc balloon is J'clcastd?
.+.

JJ tln: water lcvd is rising aL l em/min whl.:n the depth is I nt at 1b1.: dcq J
end, <ll what nlle is wme1 heing pumped into I he pool?

+ 8.. Boytes low tOr u p-erfect gas stntes thul tbc pressure cxc-:-ted by Lhc
gal> on its (.U'Jlaining vessel il> imersely pruponional LO the volume

2. A rope passcs O\'Cr u pulley and one end is auachcd to u can as


,;:how11 in ahe figure below. l f 1hc rope is J>tllcd ven ically downward <U
2 1nfs> how fast is Lhe c:trl moving w hen s
6 m'!

occupied by the gas. If when the \'Oiume is 10 Land the pressure i'
50 NlnlZ, the volutnc is lnc.rea...:;ing a1 1/ 2 U s, find the rote of change
of 1he pr.:ssun:. of th~:. : as.

Tiii

* 3.

A light is on the ground 20m from a building. A man 2 mtall walks

from dlc light ditcctly toward the bui.Lding at 3 m/s. I low l'as.t is the

length of hi.sshadow on the IXJ.iJdingchanging when he is 8 m from the


building'!

4. A funnel in the sbapc of a right circular cone is 15 em across the


top and 30 c.n1 dcq>. A liquid is flowing in at the 111tc of 80 mUs and
flowing out at 15 mUs. At w h ;~t rate is Ihe surface of the liquid rising
when the Jiqui.d fills the funnel to a depth of20 em?

* 5.

A watc.r tank. is in the form of a rigln circular cylinder of dianlClcr


3 m and height 3 m on lop of a right circular cone of diameter 3 m
and height I 1n. lf wutcr is being drJwn from the bouom at the rate of
I Umin. how fast is tbc water level fal\jng when (a) it is l m from the
top of thc tan!.: and ( b) i( is 3.5 m rrorn the top or Lhl! tank?

* 6.

A point P moves aJong lhc cun e y = x 2 + x + 4, where x and


X coordinate decreases at 2 m/s, If the
pcrpcndicuJar fTom P to lhe x -axis intersects this axis at point Q.
how fast is lhe area of the hianglc with ,.e11iccs P . Q. and the origin
changing when the x t'OOrdinatc of P is 2m?

y wc measured in metres. lis

* 7.

Water is being pumped into a swimmjng pool wbkb is 10m wide.


20m long, I m deep at the shaJiow e nd, and 3 m deep at lhc deep end.

i<

9. A wom:1n driving. 100 kmlh aJong a s~raig,ht highw:ty note~ lh!tl the
sbOOow of a cloud is keeping pace with her. \Vhat can she colcludc:
about the speed of 1he cloud?

1<

10. A lishcm.l :m is tro\Jing at o r::~ t e of2 m/s wjth h.is lure lOO m bchi.nd
the boot Md on the s1.1rfuce. Suddenly a fish .strikes and dives vcLticnlly
a1 a r.tte of 3 nlls. rr the fishemlan permits Lhc line 10 mn freely and h
al\\t::ty~ remains ~trnighl, how Elst is 1hc l ine being played oul whe-.n the
reel j:s SO m frotu its position al thc time of the strike?

+ 1L Air expands adiabatically in oooordonce with the law P V 1J:'l =


<.:()flStanl. If <tl a ~i\'c:n tiuu:. tho~;: voluux: V is 100 L uud I he.pu;~u.rc P
i s 40 N/cml. at what rate i:s the pl'e."sure changing when the volume i!'
decreasing ::at I Us?
12. Sand is pourc.d into a righLcircularcylindc.rof radius 1/2 m along
its axis (llgurc below). Once $:.~ n d c.ompletely cmers the bon otn. ~l righl
ci.revlar cone is formed on the top.

lf 0.02 m.) of sand enters the cont..1.incr every minute., how


fas t i.~ the top of tlw WJd pile;. ri:;ing?
(b ) H()w fa.o;t is the sand rising a.long the side of the cylindt.r?
(a)

Sand

4.9 Relaled Ratts

"' 13. A balloon h3S the shape of a riJhl cirtui:Jr cylind:r o f radius r and
length I .-ilh a hcmi<phcrc a1 each end of rJdiUs r . The balloon is
J,eing filled a1 a rJIC of 10 mi.Js in suet\ wy ll"tl incrca!s 1wice as
fMtMr. ~indlhc:r.Ucofclun&COfr v.hC'n r -. 8cmond/ e 20cm.
a

An ovalmccuack has a su-..ghl stretch 100m long and two ~mi


circks.cxhofradiu< 50m(fiaurc be: low) Car I , oo the infield. mo,cs
along Lhc Xa.\iS from 0 10 B. h ocxdcrntcs rronlf'<:S* al O.atta.ins Q
~or 10 rnl>ul C. und m.linl.rrru.lhr>>f'C<'daklng C 8. Cllr2traels
alo~glhequancro'll ADE 8 . llrsal D v.tlcn Car lrstl C. Bclwcxn
D and B. Car 2 nuin11ins lhe,amc nlc of<Nnt.colits x~inatc
., doc. Cu I.
(a) Find a fotmul for the rare of <lunge of rhc ,I"<OO<dirualc
or C.rr 2 llclween D Wld 8 .

1~.

307

10

1
A

20. Ld P (.t. y) be: a point on tbc: fir<1-quadmn1 portion of the hy~r

x'- .''

bola
~ I. Ld R be the foot of the pcrpcn.Jieul>r from P to
lhe x -axis, and Q(.t'. 0) be Ute x -intCJoepl of the nomldl hrK: to lhe
h~pcrbolaat P.

(a) Show Ihat x'

(b) I low fast is 1hc y<OOtdinate of Cur 2 dlOl.ng.ins whc.n ia ig

= lx.

(b) If f' 1110\'tS along tbe hypa1Jola SO lhat iiS X<OOrdiM~ IS


decre!l$ing :11 3 unil ~ per unil lime, how (at"l i' the 0~3 f'f
triangle QP R changing when x = 4?

a1 porn1 E'!
(cJ If the ct~.rscolh<k ttl B. ~hich c-1u sullers the nkht dwn.tgc?

21. A solution p.asses from aconrc.al lillcr24 em dcepdnd 16cm across


Lhc top inlo o. cyf indric.aJ cOJHUiner of diamclc 12 ( IU. When the depth
of <olution in the filler is 12 em. iiS level is lalling a1therate of I cnllmin.

j-IO<l-t---

I low f~ l is the lc"cl of ~h.Hion rising in 1hc C)' Iinde ul thb in~Utnl?

* 22.
C;1r I

100

'--~c:z>-+8
~X
C

Cnr2

;~r
1-so- 25
0

11

IS. A ship is I krnoonh ofupicrandr<lrJclllntN30 l:al .Hm/h. A


second ship i> 3/4 ~m Ca'!t of lhc pier und "lrl' ell in~ east at 7 knllh.
B"" f"" m: the >hip '"'p.uatinE?

16. The cuclc in the fisurc b<low rcprcscnL< o long-plo) ins r<ro<ol
"'h!dr < rotaung clocl "'..e Ill 100/1 rpm. A bug o< w>llong awoy
from t~.c centre of the 1\.-conl dor<'<11y toword pc11111 P on ohe nm or the
!'\'COrd all cm/s. When !hello$ o<ot po<IIIM R. IOcm from 0 . nale
8 ,( rr/ 4 radanc: hnd lhe rate 111 \\hd\ 1~ d1Uruw::e tmm thr hua 10
ohe hed ponu Q "chana& "'hen the.,.._ ,. ot R

A light is at tl1c wp or a pole 25 rn high Wld a ball "dl\lPJlc:d althc


sumc height fron 1 u ptJint 10 .n (nm Lhc ligh1. I low fil~l is the tt.hOOuw
or the ball moving :along 1hc ground I s Inter? 1llc disl:tnce fallen hy
the ball / s.xonas t~lcr it has becll dropped is d = 4.9051' n>etrc..

.,. 23. A point nlO\'C$ ulong lhc patuboh.1 )' ~ ., 2- 3.r (,\' ami )' mc~urcd
in mclre&) in s ut.-h a way 1ha.t i~ .\'~coordinll1C changes a.l the nue of
2 rn/s. Bow raso is ils do stance fro m lhe porn! ( I. 2) chana.na when II
is aL (4, 4)?

24. Repeat E,,c ~isc 23 given thai Ihe purabolu is rcplaced by the curve
+ y)' = 16.t.

(.t

.. 25. The \'olwnc of wood in the trunL. ot u tree h somctunc,.calculalcd


b) considering it as a rrusarum or a righa cu\.-ularconc ~ fiJ:urc below.
(a) Verily Utat the -olumc of the tnml. is

V
ib)

31..~
0

R p

17. Eight skaters fonn a .. Y..hip .. Shov. 1ha:a the "'tnth person on the
"hip lr.t\ds twi 115 fo.st II) chc: fou_n h pc:~.

19. 1Wo pt.'Oplc ( A anci/J i n rhc following figu1-e) wnll along opposire
s ides or ~aro~d 10m wide. A w:alkc: to the ig.ht :u I rnh, nnd B wtalk~
10 1hc lcfl :11 2 1 n/~. A third J:lefMln, C. wnlk.c: nlnng o ~idcwnl k 5 m fn)ln
tbc road in ~ut h a wny thnt 8 i:-.ulwny~on chc li ne j oining A and C.

Fi1ld the speed ol' C.

- trh(R

+ rR + r).

uppn<c tlut at the Jl<e"CCIt tune the rlldn of Ihe l"f' :1M
bouom arc r = I 0 :.nd R = SO em, ;nd the h(a.tM 1<
h =30m. Jflhc IJ'ttC'Of\IIOUCt IOgr'~ ~thai r.allM rj R
and r I h lw>y< rcll\:lin tbe ""'"' .. they orc now. and R
inercucs OJJ o r.atc o{ 1/ 2 c:W,ur. how ful will the \~IUI'I"C
be ch:m.;ing in 2 )'c.o~A?

j:.. 2r

IK. A J>31ticle IUO\'CS counterclockwise around a circle of rad i u~ 5 ten


centred at lhe Ol'igin making 4 I'C\'Oiurionc: each ~C!'t'l nd. How fast is the
J>3Jt tcle nto\ing ~w~ y f i'Oill t.l1c poi1U with coodinnrcs (5, 6) when it is

ai J>Osidon (- 3, 4)?

26.. SOVkl is poun::d into 4' right cin:ul::.r cone of r:Jju.J 2m ~d hciht
3 rn .alon& ib ._,h Cfi&urc bciOt*). 1bc ~ (Offfb l\4QCont' oh.-qU4I
llci!ht h, one''"""! on top of the Oth.
(a) lr 0

oz m 1 or <and tnttr< UtC:COOIJin<te\<r)' OlllltJIC. """'

the- rop of th~ pile rising "'hen it is JUSI bel "llh the
top of the coo1Wn~:r?
(QSt ii

(bJ llow (GStt:S the sa.rtd riiing :llon_g lhc 'iJdc ol lhc conuai~t
UlthU in:,ldnt?

..

.\\

1--4-T

.. 1!.--{-fS;~

"

I
27. l(tolhc nlC.Xbotni~m in figun:~.80wc ~a rod AC (li:urc bctO\\).
'4hcre C hconfinat Lo~tdm.& \enica1ty. find he \'elc-cityor C '" tmn'

or .t . II. andy.

J l . A herruspllerial t11tl ol t:ld.u, J m h.< 1 hght on 11< upper edge


(fi!un: bc:l"" ). A $U>O< f..US '"""'''lly <llot~g the .,,. of symmcuy of
the 14nk. a.nd .ben it j, I m from Lhc buctum o(thc UnL. it h f.a.llin.g at
2 mf<. Ba.. r... ,., .. lh:ldow
1M2 t.hl: <url- or the lllnl:

""'""!

thas Mtlnt?

2X. In lht ligure N-.h)W, a pl::u')r: flies due north 21100 Lmlh Ill Cfii'\U::&nl
oltiludc I km. A c:ur lra,.cls due cast on tl.str..cight hih"'U) 111. 100 t n\lh.
At the rnomcnaahc plane cros...'C:t o,crdtc: high\\uy.thecw i' 2 l nl ~
of the p<ltllt <>n the rnad d ir<etly helnw the plane. How fa<t &r< the plan<
mnd nr iepclt31t.ng I tnin GOa- this'!

3l. If the sicks ~fa triangle hi\C ltnglhc n. h . and C'. ets :tre-a i< gi\cn
by

= , s(s - n)(r- h)(<-<").


,.h<R s = Ia + h + <")/ 2 1< on<half 11< ponm..'t<r
A

II the length of
c:Kh side i.nc:reolStS at a rute or I C'mlmift. ~ fast is t\ chll'lgin~ \~~hen
a- 3 em. b; 4 cn1.a.rxl c- S e,:m?

1
ll:m

33.

North
EAst
+ 29. The infict..l or u b.IM:bdll diiunond is a :,quan:. with tliMwk: boo
''"ecn ha~~ l'leini 21 m {appmximcucly). 1lle hiHr hh( a gmunct hall
to the third b~ man, oocelerJtes quick:l), :lnd :ut~in :1 speed of 6 ''"'
as :;he runs to fir~ b~. lbc third baseman "'".!tC"hcs the bcll.al u point
2 m rro.n ll'K! b.ea on tJ'IC: line bct..,.en )(.'C(Xllj and third ba.,e. anti thro.~
the 1\allro lht 11ro;:t M4i:eman at 3.5 mls:. lrthe tull ith31fwa} 1n fir.l ~
when lhc ~tter li lhree..qu:uter-softhe w'Jy to 6rst ~.hew. ful is~
distance between tbcm changing at this inswu?

JO. In the foil..,.. in' ftgW"< the boy'> fret mal.< I re1\llution !>'< ...-w1ld
an'Wimd a sprodtl of radiu~ R met~. The chain tnn-el~ II'Olind a
~kCI ol

t:ld.ut T m<Ws 00 the bock .,,_l,.1tocb 1ts<lfh3s ..diu<

R me"<$. If stone cnbcdd<d in the tire bccoiiX$ dislodged. hi>" fast


i~ it ara,cllina "hen il k:a.\'e:S dx tire? Assume 1ha1 dk! rear "heel 1~
been pbccd on :l sland so thal the bicydc is stationary.

(a! If 0 is the angle r.,_l by the nllnutc a1111 hour hands or


chxl<, wh:lt is the time .... or change or (I
minute)?

(m ...!ions per

(b) l(lhe lengths ofthe htln~on the cloc:.l arc IOcmand 7.5cm.
fi11tl the nuc ill which 1l1cir lips approach cuch Other at .}:00

( i .e. find t!z/ r!r in the llgure helow)


(c) Repe~ t pan (b) but re(>loc:c umc: 3:00 with 8:0.5.

+ 34. ;\ numcr 1110\~..:,::; countcJclvc.kwisc w'Qund the truck in the.: figure


below at a rate of 4 nlls. A camera at the centre of Ihe track is placed
on a swi\-el so that lt can foiJow the runner. Fi1ld the n.tc a1which the
earners turns when (a) the nu'lner is ;U A and {h) the runner isat B.

~.: . ..

Ctt.mcra

35.

r\ nuan 2 01

tall walks ulong the edge of a $Lraig.hL t'Qi.li.l I0 m wide

(the figure lOthc right). On the otheredge of the road sl:.tnds tl SU'Ccl lighl
8 onlligh. A building runs parallel to the road and I m from it If the
man walks away from the light at 2 111fS 1 how ra..,_l i:, tJtc height of the
shackrw on tbc wuiJ cbunging when he is I0 m from the point on the
road directly opposite the light<)

14.10 LCR-Circuits
Modern electronic equipment contains a vast array of devices; man) find 1heir origin in three

antl!J Li U 4

Cil'l:uit
tonlaining <aplCilor and \'Oil meter

fundamental el ement~- capacitors, resistors. and induc tors.. How these e le ments relate to
one another and bow they affect \'Ohage,s. charges, and currcnrs in clectric circ:ui1s ca11 be fully

C:apaciror

understood with calculus. We begin with capacitors.


A capacitor is a device that stores equal amountS of posith c charge and ncgatiw;:.charge in
such a way that the charges c.am1ot ncutrdli7..c one another. Suppose Q > 0 is t.hc amoum of

positive charge. and therefore - Q is Ihe negali'e charge. Separation of lhese charges creates a
potential difference V a11he tenninals ofthe capacitor. h can be measured by placing a voltmeter
across the terminals as shown in Figure 4.81. TI1e si>e of V depends on how charges are stored
in the capacitor; different configurations lead to different potential differences. When we divide
Q by 1he potential difference V 1ha1 it produce. . we obtain what is callod ~1e capacitance C
of the capacitor.

Volrme1er

ii!VTI
Circu11
ronuuninz capac1tor :and b:HitT)'

.n

~~ -

c =

c-U ll

11 has units of coulombs per voh. called.fa mds (F). The higher the capacilance. lhe more charge
that c-an be stored per volt of potential difference.
If a 9 V banery is connected 10 a 10_,; F capacitor(Figure 4.82)and the switch is closed, the
bauery creates a flow of charge in the circuit until the capacitor is charged with Q = 9( 10 - 6)
coulombs. The rate at which charge Aows is called current, denoted by the letter i. It is
measured in amperes (A): I A is a now of I C of charge per second. For a simple circuil like
chat in Figure 4.82, we can think of i as the r~te of change of charge Q on the capacitor, and
therefore

9V
~

Di.c'ccli.vn

of curreut llQW ret.ued

lO

pvtcncia:l

1-+.1 2)

Q nowing

past any specific point in the circuiL How much charge flows pasl this point per uni1 lime is
represented by i.

Potential

+ - - - dil'fcrencc
: V

A rt:othtor is an electronic c.Jc\icc that rctanls the flow of charge in a circuit. \Vhcn a
potcnl ial difference V is maint.'lincd b2twecn I he terminals A and 8 of the resistor in Figure
A

Current

tlr

\Vhe.n I here is I'K> capaciwr in <t circuit, or even when there is. we can 1hink of charge

difference

dQ

4.83. where the + and - signs indicate that B is at higher potenti al than A. positive ciHu'ge
Oows from 8 10 A . II the rate of now (current) is i amperes. the ratio

R=

(4. 13)

is called the resistance of the re.<istor. h is measured in volt< per ampere. called oilms ( 11).
The b.igher the resistance, the smuller the current generated by a given voh<~ge. or the larger the
voltage required to produce a given currcm. For example. to maimainll cuncnt of2 A through
a 3 Q resistor requires 6 V. but to maintain the same current through a 30 Q resi(.tor require..'i:
6()

..,

~~111

for

Kirchhon'< loop rule

v.

The third funda.me.lta) circuit element is t11e induct.or. Voltage across a capacitor is rel:ued
to charge: Yoltage across a resisto r is related to currem (the rate of change uf charge): voltage
across an inductor is related to the rate of change of current. In Olher words, an inducl.or reacu;
to changes Ln currcnL If V is the voltage across the terminals of an inductor and current i is
changing. then ahe inUuctance of the inductOr is defined as

c
II

L=

(4. 14)

rlifdr

v
I.

It has units of voltS per ampere per second, calle d hmries (H). When induttante is large, a small
rate of change of current produces a large voltage across the terminals of the inductor.
Charge Aows through a resistor or any otbcr electric device when a potential difference is
created between its terminals. A device that creates and maimain~ po1emial difference is an
emf device (emf is short for clcctromotie .force). Examples are baueries, e lectric generators,
sol~r cells. a nd thermopile>;. t\n e mf device is shown in F igure 4.84 in a c ircuit including a

capacitor with capacitnnce C , a re.sis-tor with resistance R , an inductor with inductance. L, and

a ;witc h .5. This is callctl an /, CRcirc uil. When the ; witch S io c losed. poiClllial difference
across the terminals of the emf device causes charge to flow in the circuit creating a current
i . K irchloff's loop rule lor electric circuit> implies that the sum of the potentia l diiTorenoe.
across the co.p.'lcitor, resistor. and i nduc.tor must be equal to output potenlial of lhe ernf device.
U,in ~ e(jltatiUil> 4. 11 , 4. 13. and 4 . 14.

We Obl<li ll

di
Ld!

+ Ri +c=

V.

(1.15al

tr i i replaced by d Q/ d 1, we have

(~ .1 5b)

and if this equ11tion is differentiated with respocLto I . we also have

d2 i
L- 2

dt

di

+ R -rlt + -C =

dV

dr

These are differential equations tbat must be solved for


given as a func1ion of 1.

I EXAMPLE 4.3 7

(4 .15rJ

Q (4.15b) or i (4.15c) when V is

At t ime 1 = 0, a 6 11 resistor, a I H inductor, and a 0.04 F capacitor a re connected with a


gcner"Jtor producing "J voltage of I0 s in 51, where 1 2:: 0 is time in sec.onds, by closing Lhe
switch (Figure 4.85).

~. 10

FIGURE 4.85

LCR-Circnirs

311

in an LC RCiJcuit

Curr~~~~

,----- "~---li~
-----,
s
0.04
p

10 sin5t

60

IH

(a) What differential equation must current in the circuit satisfy for I >
(b) Verify that
5

o>

+ 10sin 4r)
3
3
satisfies the equation in pan (a).
(c) Plot a graph of i (I) and explain the significance of each term.
i (t )

= - sin5r - - e- 3'(5cos4t

SOLUTION
(a} With the particular values for L , C, R , and V in Figure 4.85. equation 4. 15c becomes
d 2i
d t2

di

+ 6 dt + 25i =

50 cos5t.

(b) Since

di

25
= - cos5t
dt
3
I

+ e-

(5cos 41 + 10 s in41 )

- - e - ''(-20sin41 + 40cos41}
3

25
3

eos51

+ ~e- 3'(-25cos4t + 50 sin41)


3

nnd

d 2i
125
= - - si n 51- e - "' (-25cos4/
2
dt
3

+
=

3 e - "(100sin41

125

- - sin51
3

+ 50sin 41)

+ 200 cos 4t)

- e- 3'(275cos4r - 50sin41),
J

we llnd that
d 2i
di
-dtZ + 6 -dt

+ 25i =

125
- 3 sin5t

e-v(275 cos41 - 50sin4/)

+ 50 cos5t + 2e- 3' (- 25cos41 + 50 sin41)

125

sinS/ -

3
= 50cos5t.
Thus, i(t) does indeed satisfy i"

25 3
e- ' (Scos4r
3

+ 6i' + 75i

= 50cos5t.

+ 10sin4t}

lj( Ill, I Cli

CIIJTUI

lf)

w. LCR-cin:'llit

1.5
0.5

{\

(\

-0.5
-I

- I..5

/\

(\

{\

I
6I

(c) TI1e p iOI of i(t) in Figure 4 .86 is conop<""tl uf t\\O fun...1ion (5/3)in5t and
-(1 /31~ ., (5 cos 41 + IOsin 4t). Just after the '"itch 1> closed. lxxh pans con
tribute signi focao1tl) to i (I ). WilhiD a few >erond;. llUWC\cr. the expon<ntial factor
~-ll C8U.<cs the second term of i(t) to become negligible. This is called the trmtsimt
pan or the current; it p<nilS fo1 a ''elY shon time imenal. The term (5/3) sin 5t
remain~ for all lime. and once the transient pan of the current becomes insignificant,
i (/) i>es<entially (5/ 3) sin St. This is called the sTeady Mate p>n of the currenL

I EXAMPLE 4 .38
When an inductor and c:1pacitor ;uc connected to ru1 emf de' icc (Figure 4.87). thc charge on the
capacitor must satisfy equation 4. 15b with R = 0,
ttl Q
Q
L+
- = V.
dt 2

( a)

Vcnf) that of the emf de,oce is a baucry proJucing COO\>hllll \Ohage V. beginning at
time 1 = 0 and I he ~,;pacitor ha no initial cllllll-_'C, llw:n th.: function
Q(1 )

cv[J - cos(~)]

smisfic this equntinn, and the condi1iuns Q(O) = 0 :ond i (0) = 0.


(b) Dr.aw a graph nf Q(t) >Jnd interpret it in terrns of charge on I he capacitor and current
in the circu1t.

'il' Jil ,

I'

SOI.l '110N

Sir~oe

(tl)

diQ C V [
s i1\ ( ) . . , ) ] it rolh:m s that
tt
v l.C
v l.C

1. ~1~ + ~ = l. [~~<o(~)] + c~r [ - cos ( ~)1 =


1

V.

Clcurly. Q(O) = 0 and i(O} = Q "(O) = 0 nlw.


(b) To graph Q(t). we first draw - cos (1/../Lc) in Figure -tS&a. Shifting th iJO cu"e
upward I UJlh anti changing the J;<.-ale Qlllhc vertic;al ;txis g ives the graph of Q(t) i1l
Figure 4.88b.
When t he circuit is c losed :u t = 0 . there is nooh<lrge (Hl the <:.aJJ<\CitOr()r Curren I
in the <:irt::uit. The bo.1UCI)' inuncdiatcl)' beg in~ chnrg.ing. the ca~tei tor. but because

there is linle charge

Oil

the capacitor.

tnl t

' 'o hage across its tcm,ina1s is sma\1. The

renudndcr of che vuh:.ge ( m:.\king Ul> V which is t"'nstal\l fol' aU tilne) ls across the
induc.'tor. 'fh is is co nsistent with d1c ftAct thm <.-unca"ity i~ relutivcl)' h' rgc here nnd
vohagc. across the irlductOt' is J)I'Q(Xltt i ona l w the second derivou i-.c of Q. lind lhc
sccmxl derivati ve iSJ>O:..itiiiC. A.~ ci n1e t' PI>J\\Cht.":S 1r ..JT:E/2 . chnrge on th e capacitor
tlf>proaches C V . what would JlOnually be: its t:ap<!c.:ity if th:: inductor were nc)t a pan
Qf the ci rcuit.. A t time 1 = 1T .J/:C/2 . -.v 1tnge <~CI'C)..')S lhc. <.' UJ)ucitor is. V . tui<.J that
fiCI'OSS th~ i11duetor vani shes. There is n JJOim of'ilftection at which ct1 Q/tlt1 = 0.
Current is now a1 a maximum i1llhe c.:-if!Cuit and the \::apacitor con tinu~s to accumulate
d1w-ge. The -.o ltage crQSS Lhe capacator cxtceds V . and ther-efore that tK.'1USS the
j,,ductOC' is 11egative. TI1is agrees with the ract thm tb::- c urve is conc~wc duwnwsrd
<ild thert!furei/ 2Q/ciP < 0. At t
Tt ../LE. charge h:.u. r~ched a maximum -.:.due
o f 2C V . voltage across the eapaci tQI:' i~ 2 V , :and that aci'Q$$ the inductor ~~ - V .
C harge low begins to How i1l {he rev.erse d ir -tiof'l and current (slOpe) is 11cga\ive.

The C>lJ"'Cilor dichorge.l comple1cly t>l t

= 21!' ../T:C.M<llhe cycle repealS.

Q
2C\'

I EXAMPLE 4.39
1l1c emf device in the RC -circuit of Figtu'C 4.8\l produces a constmll

voltt~ge of

V vohs. lf the
swi1ch is close<! at time I = 0 and then opened again at/ = 10 chal)!e on lh<: Cll~l<:iiOr must
satisfy

dQ

R dt

C = V(l - h(t - to)] ,

t > 0.

where h (l - to) is the Heaviside unit step func1ion ofSec1ion 2.5.

3l4

Chapter 4 Applicmions of Oifferemiation

FlOUR& 4 . 8

0\.1rg.e on

cap~aei1or

m RC w(;ircuit

c
v

R
(a) Verify !hat

Q (t ) = CV[I - e- I<RC)] - CV[l - e-<- lf(RC)]h (t - to)


satisfies this equatio n for oil/

# 111

(b) Draw a graph of Q(t ) . What is Lh.e initial charge on the capacitor?
(c) Are charge on the capacitor and currem in the circuit continuous?

SOLUTIO!\
(a) Since the dcri\"ative o f the Hcavisidc func tion is zero for e very I

1
dQ = CV ( - - )
dt
RC

e- /IIIC) -

1
CV ( -- )
RC

=I to,

e -(l - lo)/(IIC)fl( / -

lo)

Hence,

- V[ I - 1'-(l-lo)/(RC)) h(/ - to)

= V(l - il(t -

t0) ) .

(b) To graph Q(t). we write it in the fom>

O ;:; t < to
r > to

O;:; t < lo
t > t0 .
For 0 S 1 < 10 we first draw e- f(RC) as in Figure 4.90a. tum it upside down, shi ft
it ven ically one uni<, a nd change the scale on the venical axis (Figure 4.90t>). As
1 ..... r , the graph approaches CV[ I - e- f< 11Cl ] .
For I > to, the gmph declines cxponcmially. As I ~ r;j, iL approaches
CV(l- e - s,, /(HCI J. and it is asymptotic 10 1he 1-axis (Figure 4.90c). Combining

Figures 4.90b and c gives the final graph in Figure 4.90d. The capacitor ha.s no initial
charge.

t [3fjtill# IX.I

q ,4f('' ((H

, 8 ,,

0 < t < '

~ 011 CII..IX!Cit(l( (Of

0 !: I < t,

Cl' (I - t -V<ROJ

'o

: h
Q

cv I I

. - ,/!RC>]

'
(c) If we dcline the value of the charge on the capacitor at 111 to be CV[T - c -lo/t Nn J,
it is cominuou~t. Since current in the circuit i~ the slo1le of the cur\e. it b undefined
at/0 and therefure currem is discominuous at time 10. Tt suddenly re~<erlles direction
when the emf de"lce is disconnected.

EXERCISES 4.10

I. lithe ~witch in the RC -cin.,'Uit shuwn tu &he: rig_hl ~ el~d at lime


t ; 0. c4uiuion ~ .15b roc chace;: on lh~ ..:~poteiloc bl:comes

dO

R d;

+C

- V.

(c) Drw

~rpll of the

fun.-tion in pan (b) Yo hen Q0

> 0.

(a) \~nry 1ha1 v.hcn ' ' i!C con'lli31'1t lhc function

Q (/)

= 0,

o/f UO

+ CV

the equahon ror any con<ttnl D .


(b) If Qo is the charge on ahc C"dpactor wben the switch 111
closed. sh0\11that
Qb~fics

0.

.. l. "fhc current; In the RC""CifCUit

fcc:ntialf.'qUation

di

II-+
,,, c

or u,a-t:isc I musl .satisfy th-e r.Jif

(a) Veri f) ttl<Jt when V :: A !linu..l. where .4 ~nd w urccun&fants. lhC runchOft

=-.
til

tiV

= D co< vLC
~ + li >in 17"7'
v LC

Q (r)

If V a Y; 1 Mnwt. where V(ll.nd w Me cotuutnu. verify th:u !lsolutinn

;,
1 ;

= Ae I t ' c1 +

f(t)

Z
"

>On

, In WI

WL - u>!..'

r....,. 11.1\) ("()O.W.nls D 4Rd E

.sruis:fics lht CQUJlion

(wt - ).

A fw

tb) I( Qo i~ the d'l::ltJ.t: on lhe CI(XICitor wM:n the '"il,h ' ""
dosed. f.how 1h:111

'""~

1 3. I( lhc swit..:h m lhc LR d n:uit below 1r. dosed at ti m~


oqu:~litln 4, I S~ for cum:nt in Ihe ci1'Cuit t.ccomcs

di
Ldt

+ Ill

v.

=--u;CR'

Q(t)
1

QQ<OS '-""

vf.C

A .JrC
wL _ _

vLC

we

= 0.

;\ j(A

,;, 17"7' -

wL _ _

<;nwt.

we

> 0.

( a) \bl.fy tb.a.l "hrn \' t!iconstli"L the (rL'lCiion

i(t)-

f)t

I Vf R

,,.rislics thcjuation for anyconbhtnt D.


(b) lJh.ing the ~"act lt.;.tl i(O) = 0, dctcnnim: D. and drawn
&nrh of i(t).

I.

,. ' II' the \'Oitagc sour,~c Jubcllcd V in lhc circuit of ExerciseS i~~: sud
denly $.hcnt'"('itcuited bylh~ ,w.tch l:abc:Ued S. the cttrrern i in 1hecirni1
thcn:a.Rtt 1nus1Slllisfy the cquatlion

d 1i
L-

,,,, + -ct = 0,

wl-..:.l'c / , und C arc 1hc 1tbct of lite induc.tor vndcapaci10r", r~1ivcly.


nnd 1 iK tinlC. Vcrir)' 1hu

R
~IA I:t~fy

the diffcrcoti.al equation

di
Ldt

= /(t) = Acos C :.J + /J sinC~c)

4. Tht cum:nc i in the L H-cin:.uit tn Ih.: cin:ul of E.xercise 3 must

w;.til.li(:l. this CQUJtion for :ill) conlilatll


+

Ri

= \1.

7.

tr V =

(a)

A and 8

wh:a~'G(levc.r.

.4 sin c>r. where A and corn coa.JOtL"'ts. in E<crc.'IM!

I. and there h. no dUI)c on the np;:ciiOr


\'Cnfy lh:l

~~

Ua-.: 1

= 0.

i
~'fc:

= f(t ) =

A,-tiL

+~

,;n (wt -

1/>).

Q(l)

CA

u1NC A
= I + w'
R'C' Sill WI + _;;,_:...__ [t- tf(RCo _
I+ w'R'C'

COSW/1

s;uislic< RQ' + Q IC .. \1.

A 15 ~LnY conuant. and

(b) Show lh Q (l) ... be cxprcJKd in '"" r.,...,

l = ,I R'

+ w'U.

18111/>

wL

tr

Q(t)

S. lrlh.! tv. itch in !he (ollowwl I.C -<ire.:ut,cboood 11 rime t .; 0.


c:qua.hoo 4,15b lor(.mtgc oo d-.c capxihJt bccolnt'S

- v.

, >

o.

wRC' A ~
I + w' R'C'

,,..C>

A/w

-zcos(wt

"').

where

1
JR
'+
'
w'C'

and

I
""' = - wCR '

lndect-rmirurt Fom1~and L'lh)pil:ll's Rul~

J . ll

t.

(:~)

8.

i(t)

If V ;; A $in Wl . wl1cn.:: A anUw an:. constants. in E.:...crcil:!C


3. verify thot

wLA

R' + ~V

(-k </L -

coswt)

/?A

R1 + w1 L1

.
srnwr

317

10. An inductoc L, a r-~i~or R ~and u l.:Up~itur C arc connoclcd wilh


ngcncrJ.tor.producinganoscilla.toty\'Oitage = v.,coswt.fort ~ 0
(figure below). If L , C, R, V0 lll1d w nrc all const11nts, stcdystutc
current i (1) in the ein:uit is given by

satisfies Li' + Ri
v.
(b) Show tha< i (1) can be expressed in the form

i (l) =

wLA e
R1+ W'L'

H</ L

+ ~ Sin (WI
Z

</>)

i (r)

= .j !?' + w'L'

= , - J<ZL>(o cos vr +

R'

Vo
, [ReMWI
(wL - ~.)"

(wL - - - ) sin<l1].
wC

Find the ' '31ueoli w tha1mates1heamplitudcofthecuncnt a n~lxinnnn.


Do this in two w ays:

_.. 9. 1lu: current in lhc RC L~o:i.fl.."Uit below mw.;t satisfy cquation 4. l.X.
Ir v ,; ;: A sin (1)/' where t\ und (}) un.: c.onstanLS, \'Crify that u solution
i$
i(t)

sin VI) +

zsin
A

(WI -

(a) Express /(1) in lhe fomr

</>).

i (r)

= Zv. eo (wr -

tj1),

s
in whic-h case amplitude \1o/Z must be maximized.

R
A ~in <!Jt

(b) Find ~;ritical points ror i(l) as gi'"cn.

where D and arc uny consw.nb wh{ttSoc.,-cr. anl.l

v-

z=

I
R
l,C - 4f.2'

s
1

R2 + ( wL - wC )'

wl.- -

!an tP

= --;:-'w"'C~
R

14.11 Indeterminate Forms and L'Hopital's RuJe


!Xriv01ives are inst<urlaneous rmes of change. <lefi11ed as ljmirs or average nnes or change. As n
result il was necessary 10 discuss limits in Chapter 2 prior 10 the intro<luction of derivalives in
Section 3 . 1. Now that we have derivatives, it may seem quite surprisi ng that they can be used
to evaluate many limits.

The Indeterminate Form 0/0


lf we-let x appl'oach zei'O in numenllOI' and de-nomj nator of the li mit

lim
x-o+

J"f+X r.:
..y x

( 4. 16)

318

Chapter 4

APt>lic~tion s of OiiYerenthuiou

we find that
lim (~- 1) =0

and

lim

x-ot-

,r-\ool

../X= 0.

We say that limjt4. 16 is of the indeteminate form 0/0. Similarly, the limit

x3 - x2 - 8x

+ 12

lim - - - : - -- - -

(4.17)

x l- 4x+ 4

x-2

is of the indeterminate form 0/0 since both numerator and denominator approach zero as x
approaches 2.
In Section 2.1 we evaluated limit 4.16 by rationalizing the numerator (see Example 2.6}.
Factoring numerator and denominator in li mit 4. 17 gives

x 3 - x 2 - &x + 12
lim - ----::-2- - - - -

x-2

+4

- 4-V

. (x - 2)2 (x
hm

-2

+ 3)

(x - 2}2

= lim (x
x-2

+ 3) =

5.

These two examples illustmtc the "trickery" to which we resorted in Clwptcr 2 in order to
evaluate limits. With Cauchy's generalized mean value theorem from Section 3.14. however,
we can prove a result called L'Hopital 's rule, wh ich makes evaluation of many limiiS of the
indeterminate form 0/ 0 quite simple.

THEOREM 4 . 4

(L'H o p it al'a rule )

Suppose functions f (x) and g (x) satisfy the foll owing conditions:
1. j (x) and g(x) arc differentiable in an open i111erval I except possibly at the point

x = a in 1~
2. g' (x) 'I 0 in I except possibly at x =a:
3. lim j(x) = 0 = lim g(x);
x- tl

;r-. (l

. / ' (x)
4. lim - - = L.
x~a g'(x}

Then.

j(x)
.
lim - - = L.

- g(x)

PROOf We define two functions F(x) and G (x) that arc identical to .f(x) and x(x) on I
but have value zero at x =a:

F(x) = { f< x) ,
0,

.1

'# a

C(x) = { g<x). x
0,

X = 0,

#a

{1 ,

Since F'(x) = J'(x ) and G'(x) = g'(x) fur x in I exc-ept possibly at .r = a. F(x) and
G(x) are therefore differentiable on I except possibly at x = a. In addition, differentiability
of a function implies c<lnlinuity ur the function (Theorem 3.6). su thai F (.r) and G(x) must
certainly be cominuoLL-. on I except possibly m ..r = a . But
lim F (x) = Jim j(x) = 0 = F (a),

( ~ (l

\ ...... 0

and the same is true for G(x) ; hence. F(x) aod G(x) are comiouous for all x in /. Consequently, F (x) and G (x) are identical to .f(x ) and g (x) in every respect, except !hat they
have been assigned a value at x = a to guarantee their continuity there. This extra condition

t II

lndetenninrue Formsand L'HQpirors Rule

319

permits u&to apply Cauchy's generalized mean value d1eorem 10 f(.r) and G(.r) on cheintervaJ
between a and x, as long as x is in !. There exists a number c between a and x such that

or since

F (x) - F(a)

F'(c)

G(x)- G(a)

G'(c)'

=G(a) =0,

F(a)

F(x)

F'(c)

G(x)

G'(c)"

-- =

Since x and care points in/ , we can also write that F(x ) - f(x) , F'(c) - f '(c) ,
G(.r) = g(x), G' (c) = g' (c}. and therefore
J(.r)
g(x)

lf we now let

F(x)
G(x)

F'(c)

G'(c)

f '(c)

= g' (c)

x approach a, then c must also approach a since it is always betiVeen a and .r.

Consequemly,
lim /(x) = lim / ' (c) = lim f'(x),
c... a g'(c)
x ... a g'(x)

x... a g(x)

and if

. / ' (x)
L = bm - -,
x- g'(x)
it follows that
. j(x)
Inn--= L .
x-a g(.r )

This theorem is also valid if I. is replaced by oo or - oc. The only dille renee in the proof
is to make the ;ame change in1he last semence.
Theorem <~.4 is also \alid if x -+ a is replaced by eilher a riglu-hand limit x -+ a+ . or
a lcfl-hand limil, x - a-. The only difference in these cases is that interval I is replaced by
open intervals tl < x < b und b < x < a, o'Cspccti\'cly, and the proofs arc almost identical.
In addition. 1he following theorem indicates 1hat x -+ a can be replaced by x -> oo (or
X-

- 00).

THEOREM 4 .6 (L' H o pital's rule)

Suppose ful)(;tions f(x) and g {x) satisfy 1he foUowiog conditions:


L f(x) aod g(x) are different iable for some imerval x > b > 0;
2. g' (x) #- 0 for x > b > 0;
3. lim f(x) = 0 = lim g(x) ;
,\'-00

,\'~0

4. lim f'(x)
x~oo g' (X)

(or oo).

Then,

.
f(x)
r->oo g(x)

1om - - = L

(or oo).

In other \\'Ords. t:HOpitars rule applies to any type of 1irni1 thai yic.lds Lhc indeterminate
fom> 0/0 (be it x - a . x-> a+, x-> a -. x .- o:>, or x -> - oo). A common CITor

/(.t)/~(x) with the quoticm rule and then take


the limit of the resulting deri\'ati\-e. L' Hopital's n ole calls for the limi t of j'(x)/g'(x); f (x)
and f((.t) ore dillc.rcmiated separately.
If we use L'Hopitar s rule on limit 4. 16. we fond that

wben using L'Hopital's nolc is to differentiate

lim

x- o 1

2-fi+X

lim

>- 0

Fx

lim --;~=-

x- o

../i+X - 0

2./X
For lintit 4. 17, we hnve

which is still a limit ofthe indetem1i nate fom1 0/0. Note that this is a conditional equation: that
is, it says that the linut on the Jcfi is cquol to the limit on the right, provided that the limit on the
right exists. If we apply L'H(ipital's rule a second time. to the limit on the right. we obtain
1 3 - x 2 - &x+l2
6.t - 2
lim :;___,::_____,:::__.:__:.: = lim -.,---- = 5 .

.r- 2

.r2 - 4.\'

+4

,r-.2

I EXAMPLE 4 .40
Evaluate the following linUts:
(a)

3+x

lim

x--3

.j'j -

x- 4
- x 2 - 8x +
lim

(C)

t.ln x

lim--

(b)

r- 0

lim

(d )

16

SOI .L IION

.1 3 -

4x 2 + 9x - 36

x2 + 5

(a) Since we have the indererminate fonn 0/ 0, we use C HOpitars rule to write
.
IJOJ

+X

--3 ./3 -

lim ~~ =
1

x--3

lim 2~ =

x--3

2./3.

2H

(b) If we usc L'Hopital's rule. we have

tan.r
lim - -

.r-o .x
(c)

scc2 x

I.

~ lim -- =

x-o

By L' Hopital's rule,


X- 4

lim
- xl -

8x

+ 16

= lim - -- 2t - 8

Since

lim

x-4

2x - 8

=oo

and

= o:>

and

lim
x-2< -

= -oo.

we cooclude 1hat

x- 4

lin1
x-<~+ x2 - 8x

+ 16

liJn

x-4-

x- 4
= - oo.
x- - 8x + 16
?

(ll' Thi ~ li oU( ~.!.> ovl vi ~Itt: iruJ~t~:c. r,_uuut~: (OA1JJ 0/0 ~inw li.r u.. - ~ ( .\? + S) = 21; 1J1ollo
\ve cannot uFe L. HOpi tar~: rule . S i r~ce lim.,_1 (x' - -lxz
9x - Jo6) = 0.

~i~~

x:J - 4.r::

x2

+ 9x +5

36

= 0.

Had we -.:sed L'HOpical's I'Uie iJt part (c.l) of thi.s example. we (\'<tuld have obtained an ineon-ect
answer:
x 3 - 4.~;2 + 9.r - 36
3x1 - S.x + 9
25
lim
li m
.\ - -l
x2 + 5
-~- _.
2x
3
In other words. l"HOpital's ruJ ~ is nol to be us~.d i ndiscriminately~ it must be- u:.-etl only <m Jhe
imhrerminatt~f<nmsfor whic-h iris desigm_
,/.

The Inde lerminate J<'orm oofoc


The limil

I + ,;x=-1

lim

2.\ + 5
is said (0 be of the ind.~t~rruina t t form oo/oo s inu n umerator and de-rto minato r bec.ome
iucrca:singly _large- as ,\ ~ oc.. l 'bcorc.u s 4...1 a nd ..&..5 (tX LHopltal's ruJ: c-ar.\ be adapted to 1hJs
indc.let-miHUc. fonn aJso; hetcc. we. calculate. that
x-<

lim

.- cv

+ ~/X"=I = lun
. 2J. -

- .,.--=-:--,.2x +5

.- ...... ::v

= lim

.~- - c:u

r.:--o = 0.

4v X -

f EXAMPLE 4.41

E valuate the foJiowing limits, if 1hey cxis1:


(a)

r'

fim :...._
). -

lim

(h)

tX.i (,.X

J1x> + Jx
1- x

SOLUTION
(a) Since this limit exhibits the indeterminate form

+2

oofoo. we usc L' HOpitaJs rule to

write
lim

x-oo

x2
(!"-

= lim

.......... !)0

2x

rr

S ince this limit is still of the form oojoo, we use T.'HOpi1al's rule again:
=

2
=
eJ'

lim ,Y_.OO

o.
x"

The same resull would occur for any positive power n on x: lluu is. lim - = 0.
:c-oo eX
What this shows is that exponential func1ions gTow more rapidly for l arge x than
power fu nctions.

(b) By L'Hopitars rule.

+3
J2x + 3x + 2
2J2x2 + 3x + 2
=
= x-lim
_,..,..lim_co
- oo
- 1
1- x
4x

lim

+ 3)
+ 3x + 2

- (4x

x- - oo2J2x 2

This limit is a lso o f the indetcrm.ina te fom>oof oo. Further nppl ications o f L' Hopital's
rule do not lead to a simpler form for the limit. llllls, L'Hopital's rule docs not pro,c
advanUigcous on thb limit. J1 i:.- bcnca- to divide numcrdlor and c.lcnominal.or b)' x:

lim

.J2x2

..--->

+ J.r + 2

lim ---:-~.r_ __

- ../2.

lim

x --:x>

1 -X

x--CIO
X

I EXAMPLE 4.42
Show that L'Ht)pitars rule cannot be used to evaluate

lim

X -COS .\'

.r....:..oo

What is the '"'lue of the limir?

SOtUllO'\ lltc limit is or the indetenninatc romt oo/00. If we apply L'HOpitar s rule we
obtain

li m.

X - COSX

A - 00

I + sinx

lim
A-00

But this limit docs not exist. and therefore L' Hopital's rule has f,ti lcd. But we do not need the
rule .since division of numerator and de nomina lu i' by .r give.s

x - cosx

lim

- co

lim

x--oo

cosx)

t - -A'

= I.

The Indeterminate Form 0 oo


The lim its

lim .r 2 1n.r

x- o

are snid to be of the indeterminate form 0 oo. L'H()pital's rule can again be used if we first
rearrange the limits into one of the forms 0/ 0 0<' oo/oo:
t
=lim - - = 0;
x - QC 2e2t

lim x 2 lnx =

.. ~u t

In X
lim - - = lim x = lim
1
.t-rUI
.(~0' - 2
.\'- 0 1

Note that had we converted 1he se<:ond limi t into the 0/0

(-xl)
2

= 0.

ronn. we would have had

x2

lim .t 2 In x = lim - -

.r- o-t

x-01

lox

x(ln x ) 2

Although this is conect. the l imit on the right is more diJ(lcult to evaluate than 1he original. Ill
other words, we must be j udicious i n c.onvening a limit from the 0 oo indeterminate form Lo

either 0/ 0 or oof oo.

I EXAMPL E

4 .43

Evaluate the following limits if they exist:


(a)

lim (.t -

:A. -:r/2

2) secx

;r/

(b)

SOLU110N
(3)

(b)

lim (x-

.:t"~.'T /2

lim xe 'fx

x- u

1r /2) sc<:.t

X -

= lim

x -:r/l

~ t/>

/2

lim

-l / .t 2

100 ,

.x- .'f"{"J. - Sill.\"

e 'lr ( - l /x 2)

= lim - - = hm
x- o 1/x x- o

T he Indeterminate Forms (fJ,

Tr

COS.\"

= - I

lim e 1' ' = 00

x-u'

oo0 , and oo - oo

Various ocher indelcrminate fom1s arise in the evaluation of limics. and many of these can be
reduced to the 0/ 0 and 00/ 00 forms by imrodudll logarithms. In particular. the limits
lim
.~.~ o --

x~ ,

lim ( +

.r-~

.!.)''
x

li m (sec .<)..-..',

Md

.l -.'T /2

lim (sec .t - tanx )

(4. 18)

x- .'f/2

arc said lodisploy the indetennintnc fom>s cf. 100 oo0. a nd 00 - 00. rc.<pcctivcly. Toe,alunte
1im,, _ 0.. x . . , we ~t

L = lirn x"'

,-u

and take natural logarithms or both sides.


In L "" In ( lim

x- o

.r') .

As the logarithm runction is conti nuous. we may interchange the limit ancJ logarithm operations
(see 11>eorem2.5).

In L

lim (lnx ' )


.l ....,.O I

lim x ln x
,l" -

01

ln.r

lim - 1

......... o

We are now in a position to use L'Hopital's mle:

= x-limO' ( -:r) = 0.

In L "" lim L
x- o - 1

x2
Exponentiation of tx~h sides of In f. "" 0 now giv.:s l. "" e0 = I: that is.
li m

,t..t

=I .

.~. - o '

f"or the second limit in 4 .18 we again set


L "" lim

.\'-oo

(1 + .!.)"'
X

324

Cb:\P4cr 4

Applk:ui(ttlS uf OifYctCOiiativn

and mke nawn:1l logarithms:

By l ' Hopiull's rule. we have

In L =

- -(-I)]
x
X
X+ I

lim
A - 0<.1

x'

lim

- ov 2(x + I)

= oo.

Conscquenlly.

L =
In lhe lhird

li~ni1 <1f 4.1 8.

lim

x- oo

(I+ ~)''
= 00.
X

we sci
L =

(sec x )""' .

lim.
x - :f(l

in \Vhich crt~e

In L

= In [ x - li11/2.
m

(sec.r)""" ]

In (secx)]
[

= .{-1f/Z
lim

secx

cosx

= .r-:r/2
lim

[cos X In (sec .t}J

_ J

St.'CX !<lOX]

lim
_,s-"
ec"-'"'-- - - x- :r /2 [
:sec X tan X

= 0.

Thus.

____lion.,,2- (secx)<"' ' =

e 0 = I.

Finnlly, the lost limit in 4.18 is cvalualcd by rewriting i1 in 1hc 0/0 form,

lim (secx - 1an.r)


.r- :r/1.

lim ( I - sin x )
.t - .1/l

co.sx

lim _-_cO<>:-'-"

,, - ..,12 -sin x

0.

I EXAMPLE 4.44

.r

Plol a graph o f the function (x) = x 2 In X. Find limils of


Where is lhe poinl of inOcc1ion on lhe graph>

.r (x) and .f'(x) as

o+.

)'

0.8
0.6

y = .x2Jnx

0.2

-0.2
SOI.lTfiON The plot in Figure 4 .91 suggcsiS that /(x) and / ' (x ) both III'J)('Oitch 0 as x _,
o+. To confim1 this we usc L'HOpital's rule to c alculate

lim j(,r)
..t-O"

ln.r

lim x 2 1n X =

lim - -

x- o-t 1

), ~o

lim __
2

x-o..

lim ( - 2X 2 ) = 0.c- ot

.v l
and

lim J'(x)
x-0

li m

x~o

2.r lou

x')

+-

.\'

lnx

-:

= 2 lim - 1- = 2 lim --"--1


x-o' .r-.O'
--

2 l im (-.r)
x- O'

= o-.

Rn the point of infle.c tion

bet'-" .t

0=

= 0 M d x = 1/2. we solve

J "(,r )=2 1n x+

-2.r +l =2 1nx +:l .


X

The only solution is x = e- l/2 . Since f ''(x) <'hanges s ign as x passes through e- l/2 , there is
a point o f inflec tion at (e-lfl . -3c- 3/ 2) .

..-..
EXERCISES 4 .1 1

Jn Excrc.iscs 1~ 2 evaluate Lhe lirnit, if it exist".

I.

3.

A'l

lim

+)X

.t-o ,r) + sxz


x)

lim

+ 3x - 2

.--oo .1'- + 5x + I

5. lim

.r 1

.\-.S

7. lim
x~<XI

9. lim

- 10x + 25
x l - 125

Jx' +I
2x +5
sin (2/x)

-"" sin ( 1/X)

II.

lin'l
,.....
,.

13.

.t 2 - I)

,_, X-

2. li n t - 3
2r2 + lt
4. lim
'
Sx' + ~

....

6. lilll

17. lim

- (x - 1)'

8.

lim

'--oo

10.

,,-1)

sinx

cosx

(I

ccs.r) 2

3x'
sio3x

19. Jjtn - x~o tan2x

lim
..
x-.T/2 (X - 1r /2)

.x - sin .r
.r'

.[X- 3
,/9- .t

lirn

15. lirn

( I - 1/..t Jl

21. Utn
x-ll

.JX+1 - .,.I2X""+1
,/3x + 4 - J2x + 4

sin (l/x)

12. lint

lf x'

z-.ro

J 5+ X - .f5=X

14. lim

16. 1_
lim
,,

~-

.r

,'('' - (('

x -a
Ulll .~

lim - -

20. lhn
... ~

.{

(I -

.J2'=X)J/l
x- 1

2-2. htn
,_n

.J:

i * 58.

(l - cos.<)'

3x..

... 23. llnt .r sin


x- :\3

(.!.)

.
(x -2) 10
24. lmt-'--..:..._-,
where k and
radiation.

29. Jim ,,. cot x

* 32.

lim (sin xY

x-o

lilll .\'sin (

T- -n::

x+J

* 36.

35. lim x lx
A- 00

.--.o

-
-

40. li.m
r-

x- o
lilll

* 42.

-X- - -I- )
I
ln .\'

... ~. ( X -

+ .59. The follC>wlng limit arises in the calculalion of lhc electric field
imcnsity fot a half-wa,c amcnn.a: ltm f (0). wh<:t'C
e.~o

. o ( 'in [rr{2(cos8- l)l

f( 0 ) =

SID

cos8 - I

e valuate lhis limit by lirl!ltshow ing that f

lim ( - I2 - -I2-)
x
sin ,,.

Fi nd aU values of a. b. and

ln Ex.cn.:iscs 43- 54 drdw <~ grupb of lhc funclion.

45. f(x)

= xe-'

46. f(x)

lnx
X

* 50.

49. / (.< ) = xel


5 1. J(.<)

= .r', x >

il~ 53. f(x)=c - lnx.

* 54.
~
*

/(x)

,\ - ->0

x'

The maximum nowr.ucofga~ lhruugh a noz.dc is govcmc"tt


.. by Ulc

-<

57. W hcn anclet:ttmtaticficld f isapplicdtoagascousor liquidtx>lar


diclcclric, :;1 oc1 dipole JOOment P per unil \'O}umc is set up, where
I

E
Show ohao Ume-o+ P()

= 0.

1 1)/ (.t

l{e.

(b) Plot the function on lhe imcrwl 0 s x s 200 . Ooes Lhc


graph appear to have a horizonta1 asymptmet Show lhat
2.
lim,_ __, f(x)

0 < x < rr/2

r/'.

- 5.

(a) PIOI ;o grJph off (x) on the intenal 0 ~ x ~ 20 . Did you


g_
ct aoy error messages? Should you have? Show that the
fwlctio n is discontinuous at x :=: I. butlimr-1 /(x) -

= xr'

56. The indttenni nate f<)n)u:


I o-:. :tnd co0 are o flcn ev:du:ncd by
Uuroduci ng logurilbiJlS.. Show lba1 lhc limil tim (x - lox) can be

+ "X~)

2 ) '-'
/(X) = X - ( .+I

h).t

(:cX++ aI> )ufo r any conS13nlS a. b . and c.

e<J' - bx -cos (x
'J
2.rl + 5x l

function

x> O

c\aluatcd by inltoducing cxponcnti.als.

i * 61.

f(x) = -

52. f(.<)

= 2cscx- cotx.

55. Bvaluale lim

'"-

48. f(.<) = x 2 1n.t

47. f (x) = -

c fot which

linl

x'e3 '

= e 11'

:snO

and then using L~HOpiut.l's ruJc .

* 60.
44. /(x) =

f(Y) =

T~o

= u-

sin [rr{2(cos8+1)) }

cos8 + I
(0) can be wriucn in lhc form

:zcos(~ cose)

- X - -I -)
ln.x
xln.x

~ 43. /(~)

1.- oo

tim (lonx- cscx)

lim lft(J..) = 0.

lll>d

(<) Dmwagraphoftbcfonction '(!.) whcnc - 0.000 14386


and k
I.

lim l ln .,l4 1l...

_.., 39. lin (cscx - co1 x )

x- J'

=0

.T-I)"t

* 3~.

A is I he wavelength of Lhc

(b) Show that \l'(X) has one cril;c.! point that m\lst satisfy
the equation (5). - c. )eel~. = Si~. Find lhc critic.al poi.nt
ac.cur.uc to SC\'CR dcdmal places when t
0.00014386.

xPl.,

l im

+- 37. lim .ulx

* 41.

~)
X

34. lim [1 + x)~'

lim ("' + ) '

.c - ~

c are po,:itive conslants and


1-111

30. tim cscx( l -cos.<)


-0

.\~0

* 33.

28.

.+.

.r - "'Q

* 3 1.

kl- s
= e<i> - I '

IV (A)

(a) Shoov that lim (!.)

27. lim x 2 e_4 .,.

"' =

,_, (JX- J2) '"

4 2
)
,.. 25. lim x-o ( x 1
I - c.osx
~

Planck slaw for the energy density t/t ofblackbodyradiati on states

that

**

6 2.

(a) Sketch a graph of the runcticm

f(x)

e-l/X!
{

0,

X -t

=0.

(b) Show that for every positi\"C in.Leger n,


t>-Jjx~

lim - - = 0.

.r-.J

x"

JV')

(c) Prove by (tJtJihc(otJiit:d ind uction 1ha1


(0)
0. where
fV')(O) is lhc u L"~ dc riv:uivc of f(x) cvalwucd m .\'
0.

14. J 2 Differentials
l n Section 3. 1 we pointed o ut that t.he n04ation d y f dx for the derivative or a fu nct ion y = .f(x)
should 1101 be considered a quotient Beginning in Chapter 5. however, it is essential that we beable to do this. and therefore in this :,ection we define 'differentials" dx und d y so that d yfd x
can be rcgnrdcd as a qumient.
When we usc the nuu~t ion

d )'

dx

.
f(x + 6.x) - j (x)
= 1.)"-o
hm
A-x

fQr lhe dcrivmive o f n func lion y

[ (A). we call t:J.x an im::mmem i1l x. It represent~ a

change in the val ue of the ildependem \!1Hiable th ..l"'l some vnlue

.x

10 nnothcr vltlue x

+ Ax.

This change can be f>OSitive or negative depenc.ling on whether we want x


ll.x to be larger
or smaller than x. \Vhen 1he indcpencJent V'driablc ch,mgcs f rom .x 10 x + ~.\'. lhc dependent
vari(tble changes by an amount IJ..v. where

6y

= J(x + 4.x)- J(x}.

t4. 19 )

In other words,~)' is the chlmge-in y re-sulting from the ch~mge ll..x in x . For the function in
Figure 4.92. ~y is positive when t:.x is positive. and l:ly is negative when 6.x is negative.
MiilcliJ;I4W!fJ.

(/y and 6y fur dx > 0

t/y and A )' fm dx < 0

/(.f)

1
;

:
x

IL\'

r1x ~:~

.
..
:

:&e.=;;::ndy
......, .._.,
~ tl1: :
'
x + .C.x .t

.r

.t +O..x =.t +dx

I~

~
For ex:unple, when y =

x2 -

2,;( ,

t:::.y = 1(3.2) 2

n,., function increases by 0.84 when .x

the change 6 y in )' whc,:,n


2(3.2)1 - [31

x i:; chungc:\l fn.)m 3 to 3.2 i~

2(3)] = 0.84.

increases from 3 to 3.2.

For purpOses of intcgnnion. a topic thm beg ins in 0'1aptcr 5 and continues i n C\'CI)' chapter thereafter. an alternative no1~1tion for a n increment in the independent variable x is more
suggcs!i\'C.

DEFINITION 4. 7

An increment D..x in the independent variable xis denoted by

dx = 4.x.
and when written as dx, it is called the diiTerenliaJ of x.

(4.20)

The. differential dx is synonymous wirh the incrcmcm 6,\ : it represent~ a change in x


(jn ftlOSI appHcaio\s a very sma.JJ change). T he di.Uecemial of the dependem val.'iable is not
5)'1lOTlymou~ wi1h 6y.

DEFINITI O N 4 . 8

The diffcrcmial o f y = .f(x), corresponding 10 the diffcremial dx in x. is den01ed by


dy and is defined by

(-1.21)

The d ifference becwee11 D.y <iJld d y is ltl()Sl easi1y see11 in Figures 4.92. \Ve k110w thaLfly

is the exact chaug_e in the fuoction y = f(x) whe.n x is changed by an amount l!!..r or d.,. .
It is 1he difference i 11 the heigh1 of che curve at x and .\' + t.\ .\ = x + dx . Now the slope o f
the ta ngem line 10 1he gntph a1 1he poim (x, f ) is /' (x). Definition 4.8 indicates thai dy can
be illlerpreted as the difference in the height of this taog,e01 line at x and at .r + dx . Lo other
words. dy is the change in y con-esponding to the change dJ. in x if we fol low the tangent line
10 y = f(x) at(.<, y) o'ather th;Hl ohe curve itself.
Figures 4.92 also suggest that wllen d:r: is very small (close to zero), t/y is approxi mately
equal to 6. y: thai is,
when dx ~ 0.
d y "" 6.y
This is illustrated in the following numerical e-..xample.

I EXAMPLE 4 .45

.._..
Find 6.y anddyfor thefunction y = /(x)
SOLUTION

Jx' + l wbenx =2 a.n dtlx

According to equation 4 .19, the ch;uoge in

6.y
Since

/(2.1) - /(2)

J(2 . 1)2 + I -

y as x increases

Jz2 +I

=0.1.

from 2 10 2. Lis

0.08987.

.f'(x) = x f ,Jx2 + I, ohe d iffere ntial or y for x = 2 and dx = 0. 1 is


dy =

(2)(0. 1) =

2
.=--:-;-(0. 1) = 0.08944.
2
v2 + 1

The difl'erence between d y and 6.)' is, therefore, 0.00043, a difference of 43 parts in 8987.

Before the invention o f e.l ectronic calculators, d ifl erentjaJs were used to approximate a function
near poims at which il w"seasily evillwtred. For example. imaginetrying lOevaluace the fu nction
/(.r) = x' ' 3 at x = 126 wilhouta calculator. We could usc d iffere ntials to approximate 126 1/3

as foiJows:

126'' 3 = /(126) = /(125)

= 5+

(5JCJ) = 5

+ 6)' ""
+

/( 125)

3(125)113 =

+ dy
I

5+ -

75

376
75

With the advent of the e lectronic calculator. p roblems of Lhis type are archa.ic. On the other
hand. differentials are indispensable when we examine changes in a function without specifying:
values fo r the independent variable. Very prominent in Lhis context are relative and perce ntage
changes.

.&.l2

Oiffer'C!ltials

329

When a quumity y undergoes a change t.y, then its relath e change is defined as
t.y

(4.221

y
aod iiS percentage change is givc.n by

100 t.y.
y
Sometimes relative and percentage changes are more important than actual changes. To

illusrrme this. consider tlle function V = 4;r r 3 /3. which represents the volume of a sphere. If
the radius of the sphere is increased from 0.10 m to 0. I I m. then the change in the volume of
the sphere is
4
4
!!. V = - rr(O. U ) 3 - - rr(O.lO)' = 4.4Jt x l0--4 m3
3
3
This is riOt a very large quamity. but in relation 10 the original size or the sphere. we have a
relative change of
4.4rr x
t.V
- -:-::::--:-=>
- ::7" = 0. 33
v
4rr(O.l0)' / 3
and a perremage change of

1o-

t.V

100-

33%.

Suppose tho same increase of O.OJ mis applied to a sphere with radius 100 ru. The change
in lhe \'Oiume is

This is quite a large. change in volun'lc, but the. relative change is

av
v =

4.0rr x

1ol

4rr ( IOO)l /3

= 3.o x lo-',

and the J.."Crcentage change is

!!.I'

lOOv = 0.03%.

Although the change 400rr i.o V wbeo r = 100 is much larger than the change 0.000 44rr
when ,. = O. l. the relative and pen.."Cmage changes are much smaller when r = 100. \\'e see.
then. that in cenain cases: il may be relat ive and percentage cha n ge~ th<H are significant rather

than actual chaogcs.


To the example above. when r = 100 rn. !he change dr = 0.01 iscertainly small compared
tor. Therefore, we shouJd be able to use the. differe.ntial d V = V' (r) dr = 4trr2 dr to
apprmimate !!. \1 . With r = 100 and d r = 0.0 I. we have

(To two significant figures, dV is cqua1 to 6 V .) Thus. relative change in V is approx.iJnatdy


equal to d V f V and percentage change IOOdV / V. So, for smilll changes in ~m independent
variable, the differential of the dependent variable may be used in place of its increment in the
calculation of relative and percentage changes; that is, equations 4.22 and 4.23 can be replaced

by
dy

y
We do this in the following example.

and

dy
100- .
)'

330

Cbapte. 4 Applications o.f OifferentimiOil

I EXAMPLE 4.46
When a pendulum swings> the frequency (number of cycles per second) of ilS o scillations is
given by

f =

It(/)

2lT

If

where I is the length of the pendulum and g > 0 is the acceler.J.tion due to gr.wity. If the lenglh
of the pendulum is increased by ~%,calculate the a pproximate percentage change in f .
SOLUTION

The a ppooximate change in

is g iven by

hence the approximate percentage change in f is

dff =

100-

]()()

(-;r.J8dl)
(t 3/Z
2lT ,Jg
f tl

But because I i.n creases by ~%.it follows that 100(d/ I I )

dl) .

= - -I ( 1002
I

1/ 4. and

Therefore, Lhe rrequency changes by- ~ %, the negative sign indicating thal because I increases,
f decreases.
FIGURE 4.93
Difft otn
tials do ooc approximate-changes at
critical points

""'*
""'~----------------------------The d if(erentia_l d y cannot a lways be used as an a pprox imatio n for lhe actuaJ change 6.y in a
funct ion y = f (x) . Sometimes it cannot be used even whe n dx is very close to zero. For
example, if f(x) = 2x 3 + 9x 2 - 24x + 6 , then

dy = J'(x) dx = (6x 2

+ ISx -

If x is changed from I to I .0 I, then the approximate change in


is

dy = (6

+ 18 -

24) dx.

y as predicted by the diffe rentia l

24)(0.0 1) =

o.

In fact. for any dx whatsoever. we lind that dy = 0 . Geometrically speaking. we can see
why. Since f ' ( I) = 0. x = I is a critical point of f (x) (Figure 4.93); therefore-. d y. which
is lhe 1a ngent lioe approxjmation to Lly . wiJI aJways be zero. We cannot use differentia ls to
approximate function changes at critjcal points.
Thedilfe remial forthe function y = f(x) = x 100 is

dy = IOOx 99 dx.
lf x is changed by l %fromx = l tox = I.O I, thendx =0.0 1 and

dy = 100(1) 99 (0.01) = 1.0.


The. actual c hange in y is
D.y = (1.01) 1<X1 - 1lOO = 1.7.
We would hardly regard th is dy as a ve ry good approximation for D.y even though the c hange
in x is only I%.

4. 1!

DiWen:.nliuls

331

1l1e Iauer example raises the question "IHow small, in general, must dx be io order that
dy be:. a reasonable approximation for 6yT' Thi:s is nOt a simple q uestio n to an:>wer. \Ve
will discuss approximations in more detai l in Chapter IU, and then be able to answer a more
important question: How good an a pproxinmtion to Oy is d y? A fter all, it is not much usc to
say that tfy ~ t.y if we cannot say to how many decimal places the approximation is accurate.
Suffice it to say now that usc of the differential dy to approximate l!.y is to be regarded with
some reservation. This is not to say that differentials are useless. We will see in Chapters 5- 7
that difterentials a.re i ndispensable to the topic of iotegration.
We make one last comment before kav:ing this section. H equation 4.2 1 is divided by
dilferential dx, then
dv
-d- = f'(x).
X

Now, the left side of this equation is the differentia.! of y d.ivided by dJe differential of x . The
quotient of differentials dy lind d;; is equal to the derivative j'(x). The entity dy/ dx can
henceforth be regarded e ither as "the derivative of y with respect to x" or as '"dy divided by
dx : whiche\'Cr is appropriaLe for the discussion at hand.

EXERCISES 4 .1 2

sidcrcd :t pcri:Ccl sphere (r-.-djus 6.37 x 106 m), then !his law prcdicls :t
graviwtional attractjon of 9.8 Lm newtons on a mass m on its surface.
Use diffcrcnliab to llctcn:ninc the hc.ight abO\'~! the surface of the earth
a t which lh e grav1taticmal aur;.tction decre-dse..~~; 10 9 . 80m ne\\>1ons .

ln Ex.crciscs 1- 10 find dy i.n tcrros of x aod dx.

I.

.r =

.!. ' '


S.

x'

+ 3x -

= J x'- 2x

.r ;;;; x 113 -

x'/3

2.

x+ l
\'= - -

"

X -

= .sin (x 2 + 2) 6. -" = x 3 / 3- 4x 2
4. y

8.

cosx

x'-

3x2 + 3x + S
.v = ----i'---=-:....:.....,.:....:.
x' - 2x +I

..., 17. Whctl a force F is applied 10 llte objc.\:t of mass m ill dtc figure
below. three otller forces ace on m : the force or grt1vi l)' diec1fy downward, <l reac1ion al force of llle .suppot1ing surface, iltld a hori21.)ntitl

x 1 (x - 2)

1O. J - -x-7.,+-5-x"'

11. The momentum ;\/ and kinetic energy K of a moss m moving


t

with :,peed v arc given by M = nw and K = - m ri . IJ v is l.:hi.mged


2
b)' 1%. whuLare upproximalc pcn:cntagc (.;bang~ in M and K ?
+ fl. The magnitude of the grn vitatio~13l foroc of uU.ractjon bctwc.cn two
pointmus.se.s 111 and AI is given by

frictional force ()I)I)O.Sing n10Lion. TI1c least force that will O\'croomc
friction and prl"141
_uce m~ ion is. given by

F=

GmM
F = - -,- ,
r

where Jt is a constam called Ihe coefficiem of .Haric frir.tio,t. Use


diJfercntials to calculate the approximate pcrccntasc change in F if IJ
is increased by 2% from an a.t1glc of ;r/4 radians.

where G > 0 is a constant and r is the dis-lance between the masses.


u r cb:.mgcs by 2%, by how much docs F change approximately?

*
*

.+.

13. According to Example 1.9. the range or a shell tired from an artillery gun wjtb velocity vat angle t) lsgiveo by R = ( v2 sin 2B)/9.8l.
Usc diiTcrcnLiuls to find the. approxinlMC percentage change in R if 0
is i ncrea...OO hy I% from an angle of iC /3 radians.
14. According to Example 1.9. the maximum hcight tuta.incd by a shell
fired from an anillery gun with vek>city tJ at angle 8 is given by H =
(v 2 sin' 9 )/ 19.62. Usedilferentialsto find the approximate percentage
change in H if 0 is increased by 2% from an angle of J"'l/ 3 radians.

15. Under adiabatK:-cxpansion. a gus obcy:s thc lilw P V1i 3 = a c.:onstunl.


where P is pressure and V is volume. If the pressure iS: in(..-reased by
2%. find the approximate percentage change in Ote volume.
16. 11te magnitude of the gravitational force of attraction belween two
point masses m and M is defined in Exercise 12. II' Lhe earth is con

9.81J'm
.
cosO+ I' sin O

+ 18. 1bc volume of a right ci.tcular cylinder is \1 ; 1r r 2h , '"'hc.tc r is


L11e radius and h is t11e hci~hl. Use liiO"el'ential s LO ~how lhe. following:
(a) Jf h c.:tn be 1.ocasu('cd cxac:dy, but r is subject to 3J1 error of
a %. the CITOr in \f is 2a %.
(b) Ir r Can be me~lsured ex~lc.:lly, buliJ is subject LOan error M

b %, lhc enor in V is b %.
(c) Wltal is the maximum percentage error in V if,. is sul:!.jccL
to an error Clf l~% and h i!' subject 10 an error of bt;r.?

19. Use djn'erentials to show that if y = x" . where IJ is a nonzero


constant. and x is subject to an error of a%. tl1en tlte re.suhjng error in
y is 1w %.

3:32

* 20.

Chap!er 4 ApplicaliOnS of Differenli:uion

Suppose that z = x" )Jf'' ~whe-re

11

and m are nonzero constants.

(a\ If x is subject to an error of a%, but y is subject to no error.


wh;:n is: lhc rcsulling error in -z.'?

* 22.

A J>L'iSill c.an be used to me-asure the iodc.x. or rc(racLioo n of Lbc

material in the prism. Acc{)rding to Exercise 73 in Section 4.7.


given by the fonnula
IJ

(b) try is subjt-ctto an error of b%, but x is ~ubjcct to uo error~

what is the resulting error in ;.")


(c) \Vhat is Lhc maximum. pcrecntagc error in z if x nnd y a.ro
subject to errors of a 'it and b %, respecti\'ety?

11

is

sin[(oJrm + y)/2]
sin (y / 2}

Usc differentials to find 1he approximate (>erccntagc error in n i f the


l!leasurcment of 1/f"' can be out by 1-:b when l.jtrn = n /6 and y = n J3.
.0\.ssume that y is known exaclly.

* 23.

Repeal Exercise 22 if \""' is known precisely but the measurement


of y can be out by I%.

21. Repeat Exercise 20 if z = xtt f .'.'N.

SUMMARY
In this chapter we discussed a number of applications of differentiation, the first of which was

Newton's iterative procedure for approximating the roots of equations. It is perhaps the most
pOpular of all approximation methods. be<:ause of its speed, simplicity, and accuracy.
ln Section 4.3 we deft ned a critical pOint of a function as a pOint in its domain where its first
derivative either van.ishes or does not exist. Geometrically, tltis corresponds to a point where the
graph of the fu nction has a horizontal Langcnt line, a vertical tangent line, or no tangent line at

all. The first derivative test jndi.cates if criti.ca.l points yi.eJd rela1i.ve m.ax_iJna or relative mio.ima.
A function is increasing (or decreasing) on an interval if its graph slopes upward to the right
(respectively, left). and this is characterized by a nonnegati\e (respectively, nonposit ive.) deriva-

tive. his concave upward (or downward) if its slope is increasing (respectively, decreasing),
and consequently ir its second derivative is nonnegative (respectively, nonpOsitive). Points that
separate intewAis of opposite concavity are c.alled points of inflection.

ln Se~ tion 4.7 we illustrated that many applied extrema problems require absolute extrema
rather than reJativc e-xtrema. Absolute extrema of a cont.iouous functioo o.n a closed io.tcrvaJ

must occur at either critical points or the ends of the interval. This fact in1plics that to find the
absolute extrema of a cominuous function f (x) on a closed interV'di ct =s x =s b, we evaluate
J (x) at its critical poinL~ hetween a and b and at a and b . The largest and smallest of these
values are the absolute extrema of f(x } on a ~ x ~ b. Plotting f(x) can also prove valuable.
When an object moves along a straight line, its velocity and acceleration a re. the firs t and

secoud derivatives, respectively. of its displacement with respect to time. In other words, if we
observe straightline motion of an object. and record its position as a function of time. then we
cal calculate the velocity and acceleration of that objecr at ~ny instam.
Changes in a number of interrelated q uOJltjtics usually produce chang~ in the others -

sometimes small. sometimes large. How the rates of change of these variables relate to each
other was the subject of Section 4.9. Related rate problems made us acutely aware of the
importance of diiTerentiatUlg an equation with respect to a variable only if the equation is valid
for a continumLs rnnge of values of that varir:tble.
Potentials across resisto~ and inductors are cx.prc$.~cd in terms of dcrivatiYc..s. Potentials

across capacilors, resistors, and inductors are respecli\ely,

V =

V =i R = RdQ
dr '

V = Ldi
dr

Cauchy ':5 generalized mean value theorem cnablc:xl us u.u.levelop 1....' l'i6pital':s rule in Section
4. t t for evaluation of various indetcmlinate forms such as 0/0, oo f oo, 0 oo, 0, I~, oo0 ,
and oo- 00.

When variables change by small amounts. corresponding changes in related variables can
often be approximated by dHlerentials . Particularly importam in enor analyses are rela tive and

percentage changes.

KEY TERMS

Iu reviewing this chapter, you should be able to define or discuss the following key tenns:
Newton's iterative approach
Relative (or local) mal(imuru

Increasing function
Critica.l poim
Relative (or local) minimum

First-derivative test

Concave upward

Concave downward
Horizontal point or inflection
Second-derivative test
Absolute (or global) minimum
JJlStanlaneous acccle!ralion

Points of inflection
Venical point of inllection
Absolute (or global) max.imwn

Decreasing function

Instantaneous velocity

Speed
Capacitor
Current
Resistance
Emfforce
lndetenninate rorms
Di.ITcrcmial
Percentage chnnge

Related rate problem~

Capacitance
ResistOr
Inductor
Kirchhofl"s loop rule

L' Nopital's rule


Re.Iative change

REVIEW
EXERCISES

I.

(::t) Prove Lhtll Lhe llrt':l Of lhc i.sosceles trio.ngle in lhe figure

below is

5. Of all pairs of positive numbers that muhiply to some given conshtnt


c > 0. find Lhat pair which hal> the smallest ~um.

* 6.

A=
( h) I( the angle f) is increasing at 1/2 rJdian per minure, hul I

remains constant. how tast is lh area of me trianglechanging? What does your an~wer predicl when 8 = 0. ;r/2.
and rr?

Sol 'e Exercise 5 for tbe lurgest sun~

7. Two sides of the triangle in the figure bcJow muintain constant


lengths of 3 em and 4 em, but the length I of (he third side deci'C\lscs at
the t:lte of I em/min. How fas1 is tlltglc 9 ch:lllgi:Jtg when I is4 em'?

(c) When docs A change most r-olpidly and most slowly; that
ts, when is IdA /dtllurgcs[ and S 1nilllcsr~

In Exctciscs 8- J9 C\'aluatc. the limit., if it cx.isL".

8. lirn

Jx 1 + 2x 3

10. lim

x2 -

1<

2. Or-'*'"' a grapb of each of 1hc foUowing functions, lodic-ati.ng all rcl


a (i\'C muxima und minima and points of inflection.

(a) f(x) = 4x3 + x' - 2x

* 3.

+t

(b)

Of uU pairs of posith'C numl>crs Lhat add

[(x) =

4. Solve Exercise 3 for tJte smallest product.

:_,....~;_;_.,.
J,'

10

2.\'

some given constant

c > 0 . find th:n p~~ i r which has the largest product

x'- 2x +4

12.

X-

x 2e-.J,1

litn x .. b1x

r-o+

18. lint

.t.-ro

e+r
-X- I

2x

.r ..... Q

~:i nx -

--

.t - oo

16.

II. li l'll - -

lim
s-. -o.> 2x

14. lim

sin 3x

16

~in3x

-.t-oo 2x

9. lim

x-u 3xJ _ 2xz

13. lim

15 Lim

17. li m -x- o 1an 3.r

19.

ft- .J2

x~2 '

.flr

~o+

sin 2.x

lim

A.--ro

xe;~

20. Usc Newton'~ method lo find :111 crilical points for lbc following
functions accur:.tc to ~ix decimal places:

i 1:

(a)

{(.<) ~

x'

+ 3x 1 -

2x

+5

(b)

f(x ) =

3.<'

+ $x + 1

21. All objc....;t m i.wcs along tl1c .a:-~xis with it)) po~ilion defined a~ a

fun<.1iun of time l by
X =

X(l)

= i'- 344 r ' +

621 2 - &41,

28. A (QOtb;alt team presently sells tid:cts a t prices of $8. $9, ond.
SlO per scot. depending on the positio n o f tbc scat. AI these prices

I ;:: 0.

Plm a gr.tph of this tunc..-tion. ind1catmg 1imc-s \Vhen the velocity and
accelerJiion or the l1hject arc t!(tu:111o ?.cro

,. 21. If the yaph in the figure below represents the position :c{I) of an
object lllO\'ill$ nlong the x -nxis, \vhalcould physically cause the comer
IlL time f 1J?
X

;t O\'Crogcs slllcs of 10 000 at SIO. 20000 at S9, and 30000 at 1>'8.


The team wi.hc$ to ruisc the price of cw:h ticket by the same amount.
but (eels that rOI' every doUur the price is ruisc~ 10~ fewer Lid:cLS of
c.ach type will be sold. What price increase per ticket will maximize
n:''cnuc?
29. Repeat EXL"'"cise 28 gi\'en thallhe te.run lakes in10 3CCO\Inllhc fac1
Ihat profit rrorn concession sales rorcach peBOn at1he game is 50ccnls.

"' JO. If a panicle moves away from the origin along Lhc po~iti vc X axi~
with a constant speed of I 0 mls.. how fa5t is ils di,taOL-c rrolllthc c urve
y = x 1 eh;;m,ging when i1 is at x = J m?

* 31.

Eac.:be,eniJJg a cow inapas:tureretunb tOibbarnalpoint B (Ogurc

below). But it ah, 'a)'S doe.s so by first walking to the river for~ drin~ .
If 1he cow walk~ at 2 kmlh and slops to drink for 2 m in. wl\31 is the
minimum 1ime it lakes foe- the cow 10 get from 1he pa~ure 10 Ihe banr!
What is; J.he minitn~.un t.ime if the ccw. \\'alk.:: twice 3~ f:lSfl

23. Ir an obic:ct nlOY~ along the Xaxis wilh c;on:>Wnt ar.:c:ckr4tioo, b


it pos.~ibte for a &raph of its position function X(t ) tO have a pOint of
innoction?

* 24.

An open box i~ tonned from a square piece or cardboard (I units

long on e-deh side) by cutting out a square at each comer and folding
up the sides. What is the maxi.mum pt>ssible \'Ohunc: for the box?

* 25.

(a) If the an~rJge speed of a car for a lrip is 80 kmlh. muo:t it


ru. some time ha\"e had an inslanlaooous speed of 80 l:m/h?
E."< plain.

.!. km

H2

~
I r--1----;1

Posture

lkm

Ri\'cr

(b) Mu::.t the c.:a.r al some insus.m huvc: h~ a spcctl or83 km/h?
2 6. l)r:lw gmph~ ol' thc l'ollnwing function~. indieatin,:'1.11 ~hui,e .n:.xim~

and minirna and pOints: of in11oc-tion:

(b)

.f.

j(.< )

= x' +sin' x
i'

27. I( aa some instant or tinlt sides a and b of the triangle the


following figure roml '' right angle. and if lhese sides are increasing at
equal r.ues, does il remain a right-angled triangle?

3.2. A fanner t\J.1s 100 ha 10 plant in rom and potatoes.. Undamaged

corn yields p dolbrs per hectare and potatoes q doll an; per hectare. For
each crop. the lcss due to disease and pestS per wti.1 hccta.rc is ditcdl)'
pr()p()ftiooal lO the area planted. tr the rannct plants .... hectares or
com. the loss due to disease and (XSlS is equal to ax per hect.are. The
lotalloss of com is therefore ax 1 hectares. Similarly. 1he tolal loss or
potatoes is by 2 if the area plan1ed in potatoes is y hc.ctates. Find the
areas that shouJd be planted in corn and p0l3IOCS in order 10 miniuti.zc
rnooetcs.ry I~ Su~itule ~a111ple v-d.lu~ ror a. b. JJ. and q to sec
\\hether your re:sults look reasonable.

CHAPTER

The Indefinite Integral and the


Anti derivative

Applicati on P review

Suppose aU vehicles in a single lane of traffic on a higllway have the same speed v. Let I be
the a\er-ge length of the vehicles (figure !><low), and let d i>c the distance between ,ehicle_<
(assumed uniform).

()

()

I( )

!)

The rater at "hich rral~ic Mows is defined 10 be the number of vehicle~ passing a fixed point
A J>eru nit time. This is equal 10 the inverse oft he time taken for one "ehicle to pass A. including
the distance between vehicles. AI speed v. thi.s time is (I + d )fv. so that r = u/(l -'- d).
Naturally a tratllc; engineer would like to move U"dffic along as quic.kly as possible, and this could
be accQmplishc:d by increasing u and decreasing d. But safety is an important con.sidcrdtion,
and d is not, or should not be, independent of u; it should be increased as 11 is increased.
Experimental measurements have suggested that the shortest stopping distance is 52 m when a
,chicle is travelli ng 22 m/s, and 96 m for 31 m/s.
THE PROBLEM

f ind a re<Lwnable dependence for d as a function of v and use it to find

a suggested ~peed 11 for max.i mum flow rm e J' if cars mai utaj n a safe driving d istance. Take

= 4 m for the a,erage length or a ,chicle. (See Example 5.7 on page 346 for the solution.>
In Chapter3 we introduced thederivmive and wa)'S todifferemiate functions deli ned ex plic-

itly and implicit ly. \Ve then d iscussed vnrious ~ pp licat ions of c.alcul us, i ncluding velociry and

acceleration. related rotcs, maximo nnd minima. Newton's iterat ive procedure. and L'HOpital's
rule. In this chapter. we reverse thc difTercnciatio n process. Jnl)tcad of giving you a rune( ion and
asking for its derivative, we g ive you the derh'4-ll ive and ask you to find the function. This process

of backwMds differentiation or antidiffcrcmiation has such djvcrse applications that am_idi ffc.r
entiation i s as impOrtant to <.:alcul us as differen tiat ion. Jn many problems we find oursei \'CS
differentiating at one stage and antidiiferentiatjng at anothe r.
Antidifferentiation is a much more difficult process than differenthttion. All but one of the
rules for differem iat i.on were developed in Chap1.e r 3~ Lhe one remaiojng rule is i n Section 9.1.
I n contrast to 1h is, lhe list of f ormulas and technjques for findiog antiderivatives is eod less. Ill
this chapter we introduce the three simplest bul. mosl. important techniques; in Chapter 8 we
discuss many others.

335

15.1

The Reverse Operation of Differentiation


l o our disc\lssions on veloci ty 11nd accclcra(ion in Section 4,8, we showed that when an ob
j cct rnovc.-; nlong the x-axis with i t ~ position described by lhC! f unction .\'(f), i t ~ vci<)Cil)' :md
acc.elermiot) ate the lif:-.l a11cl stx:ot)d deri\cu.i,ello uf x(t) with respe.ct to titHe t:

11(1)

For example, if x (t)

t '3

= -dx
dl

lllld

+ 3t 2 thc.r'l

v(t) = Jt

+ 61

a(t) = 61

and

+ 6.

\Vhcn cnginccr.s and phy... icist!> .-.tudy tile motion ... of object .... a mo re commo n type of
IJ_robJcm is to dctcn.nioc the J)Os.i tion of ao o bject; that j s, the positjon j~ 111.1i given. What they
mjght know, however, is the acceleration of the object ( perhaps through Nc\\!ton's second law,
which S-tates then accelenuion is propon ional to the resultant force on the object). So the question
we must 110\Y ask is: IJ we k11ow the accelenuiou a(t) of 1111 object as a ful ction of tin_te, cao
we obtain ils \'elocily and posilion by reversing the differeoti::uions'? In the e.:<ample above, if
we know that

dv

a(r)= - =6t+6,
dt
<.'an we filld the fu ll('tion v(t) t.h at diiTt-l'ellliate.s 10 ghe 61 + 6? \Vt- k1lOW that

to

<\l'l'ivt- a11he

tcrmli: 61 otnd 6 after diftCrcmiation. v(f) might have contained the 1c.rms 3/ 2 and 6t. Tn other

wolds. one possible vcloci<y func<ion <hal dillcrcn~imcs 10 give a (I) = 6l + 6 is v(l) = 312
+ 61 . 11 is no<, howeve., the o nly one; v(l) = 3t 1 + 61 + 10 and v(l) = 3t 2 + 61 - 22 also
have derivative 6t + 6. In fact. for any consta nt C whatsoever. the derivative of
t(l) = 31 2

is a(l) = 61

+ 6.

fum::tiou ~ that hllvt:

+ 61 + c

ln Theorem 5.1 we shall show thm Lhis velocity function represents all

6t

+ 6 as their der ivative.

We shc1ll al.so dc111oustrate how to eval ualc the

constaJ.H C.

For <he prcscnc, Icc us sec C = 0 so chac

dx

u(f) = dl = 31

+ 61.

We now ask whac posicion function x(t) differentia<es to give 31 2 + 61. One poosibilicy is
x(f} = t 3 + 3t 2 , and tOr the same reason ns above, so is

x(l) = 1> + 31 2 + D
for any consr~lnl D wluu~oever.
Thus, by reversing the differentiation operation in this cxamp1c we haYc proceeded from

the acceterat.ion a (I) = 61 +6 co possible velocity fuoc<ions v(l) , and then to possible posicion
funccions x(l). This process of antidiffc rentiation has applicatioos far beyond velocicy and
acceler.uion problems, and we shall see. many of Ihem as we progress through this book. We
begin our formal study of antidiffcrcntiatioo with the fo Uowing definition.

DEFINITION 5. 1

A function F (x) is called an anlid e rivalive of .f (x) on an interval I if on I ,

F'(x)

.f(x).

(5. 1)

F'o r example. since


3
-d x ' = 4,\',
dx
\VC say th<il ""' is lm aruidcl'iwtivc uf 4x 3 for all :x. But for an y ('Orlslant C. the f unction
.x"' + C is also an amiderivmivc uf 4.t 3 . The following theorem illdic~ncs thMtlhese are the tJlly
tuHidcriHnives of .lx 3.

TH E OR E M 5.1
Jf F(x ) is cln antidcrivutive of /(.t} on W'l i11tervul / ,then evety untiderivative of j (.:c)
011 I is of th e fOC"m

FCx) +C.

where C is a constant.

PROOF SupJ>OSc that F(x) and G(x) arc two antidcrivatives of .f(x) on f . JF we define
function D(x) = G(.<) - F (x). theo> on I

D' (.x) = G'(x) - F'(x) = f (x)- f(x) = 0.

If .r 1 and .r2 are any two pOints in the intenrall /. then cerminly D'(x)
0 on the intenal
x 1 S x .:S ,\' 2 But di fferentiability of D(.r) o n .r1 .::;: ;r ~ .tz implies continuity of D (x)
thereon nl~ (Theorcn1 3.6). \Vc moy therefore ttpply the mcm1 v.-lue theorem (Theorem 3. 1t.J)
to D(x) on the inrerval .x 1 !: x ::; x 1 ::uld concl ude thot there c>: is l~ u number c belWCCil ,, .,
HllcJ .x2 such that

D'(c) = D(xl) - D(x t) .


X2 -

.X1

But bcc.ause D '(c)


0 . il fo llows [hat D (x 1 )
D(,\'1;) . Since x, and x 1 arc any two pointS
in / , we <:OI''Icludc thnc D (.r) 1nus1 have lhC: ~anlC value m every point in I; lhtu i~. on I .

D (x)

= G (x)

- F(x )

= C,

where C is a constant. Consequently,


G(x) = F (x )

+C.

Becuu:;e of Jhis tll<>Jrc'" if we li nd ooe antioerivative F(x ) of f'(,v) by any means what
soever. Jhen we have founo every antidcrivative of" function j'(x). s ince every antideriv.uivc
can be writlen as F (x) plus a c::onstanl C, T'hus, every antiderivalive of a function /(.t} is o f
the form F (x) + C. where F(x) is any o ne antidcrivative. We call F(x) + C the indefin ite
integr a l of f(x). Titc operation of taking the i ndcfini1e inJegral is clenoo.e d by

J .r(x) d x

F (x) + C.

(5.2l

where the differential dx indicates that integrati on i~ wilh rcSJ)CCI to x. We <:all


iotegnlJld of the i''ldcfinitc inJcgral. RJ<exa~tlplc. we write

and

l x'

~dx =

J (x )

the

--+ C.

2x2

Distinguish between an amiderivative of a function f(x) a nd the indefinite integral of


Both reverse differentiation. An antiderivative of f (x) is a function that d ifferentiates
to j(x); the inde fin ite ituegral of J (x) is a ll funct ions that d iffere ntate to f(x) . it adds a 11
arbitrary coos ta nt to a ny a nciderivative.
The fo llowing theorem is fundamental to t he calculation of antidcrivatives and indefinile
integrals . hs proof is a straightforward exe rcise in differe ntiation and the use of Defi nition 5.1,

.f(x) .

338

Ch.ap1er 5 The Jndetini1e lmeg-al and tJ)e Aruideri\'ative

THEOREM 5 .2
If f (x) and g (x) have antiderivatives on an
(i}

(ii)

l [j(x ) + g(x) ]dx


l kf(x)dx = k

inten-~1

=I

I, then on I :

f(x)dx

(2x 3 - 4x} dx =

I
2

2x 3 dx

x 3 dx - 4

x dx

(5.3a)
(5.3b)

4x. we wl'ite.

- 4x dx

g(x)dx;

k a constant .

f(x)dx,

For example. to evaluate. the indt:li nitt: integral of 2x 3

+I

[by pan (i) ofTheorem 5.2]

I by part (ii) of Theorem 5.2)1

(x')

x ) - 4 '2 + C
= 2 ( '4

=- 2

zx' + c.

Every differeotiation fomJula developed in Chapter 3 can be expressed as an integratioo


formula. In fact, equations 5.3 a re integra l counterparts of equations 3.9 and 3.8. Some of the
differentiation formulas for trigonometric, inverse trigonometric, exponential. logarithm, and
hyperbolic functions are resta ted below as integration formulas.

l sinxdx = - cos x + C,

+ C,

(5.4b)

sec2 x dx = tan x +C.

(5.-lc)

+ C,

(5.4<1)

l cosxdx =

(5.4a)

sinx

l sec xtanxdx = secx

l csc2 xdx =

I
I JJ- x'
I+
1-I

cscxcot x
1

1
-

x2

x J x2

-cotx

+ C,

(5.4e)

dx = - cscx

+ C,

(5.4f)

dx = Sin- ' x + C.

(5.4g)

dx = Tan- 1 x

+C.

(5.4h)

dx = Sec-t X

+ C.

(5.4i)

le"dx =tr' +C,

(5.4j)

S.l The: Rt::\'C:()C ()pcr.llion or Oi.tfcrc:miat;or~

a' dx

J~

339

(S.~k)

= a IO!Ja I! + C,
ln lx l

+ C,

(5.41)

sinhx

+ C,

('i ~m )

dx =

coshx

+ C.

(5.4n)

sech 2 xdx =

tanhx

+ C,

(S.4oJ

dx =

J coshxdx =

J
J
J

sinlu

csch x clx

Jsechx

tanhx clx

+ C,

('i .4pl

= - scdu: +C.

(5.-lqJ

J cschxcothx dx =

- c'OihX

-cschx

+ C.

(5.-lr)

Equation 5.41 follows immediatel y from equation 3.-16 with f{x) "' x.
Pcrhapo; the ntllSt important integration formula i s the counterpart of power rule 3.7. Since

- x

+ t

dx

i t follows that

(11

With property 5.3b. the 11 +


the equation:

(u

l )x ,

l )x" dx = x+ +

C.

I can be removed from the i nregral anti taken tll the other side o f
1

x"dx = - li T

x+l

+ C,

-1 .

where C
C f (n + 1) . As i ndicated. this resul t i s valid provided that 11
5.41takes care of this e<ceptional case.

(.'i 5)

':ft

-1. btl! fonnul a

I EXAMPLE 6 . 1
Eval uate

SOT.UTION Using Theorem 5.2 anti formul a 5.5. we ftntlthm

! (2x + ~)dx "' 2 j xdx + ~~ dx = 2 (,rz)- . !. + C "' x


x2

Where is x 2 - I fx

x!

- x. !. +

+ C , the i ndefinite integral of2x + I / x 2? It is valid 011thc intcrYals x

C.

< 0
and x > 0, bm not for all x because the function is not defined at x = 0. To make this clear,
we should write
2
1

!(

2x

1)

+ -x1 dx

- -; + Co.

=
[

2
X

< 0

> 0

'
-

+ C2,

340

~pier S

The h1definite lnRgr31lnd the Anlideriwui\'e

where the constants C 1 and C2 need not be the same. For brevicy we ofte-n write

= x
!(2x + ~)dx
x

-.!.x + C,

thereby suppressing U1e complete description or the indefinite integral. When che comexc demrultls that we distinguish various intervals on which the indefinite imegral is defined. we shall
be careful to give the extended version.

I EXAMPLE 5 .2
Find a curve that passes through the point (I , 5) and whose 111ngcm line at each poim (x , y)
has slope sxt - 3x 2 + 2.
SOLUTION

If y =

J (x) is the equation of the <.urve, then


dv
- = 5x' - 1~ 2
dx

+ 2.

If we take indefinite integrals of both sides of this e(Iumion with respect to x , we obtain

dy dx=J(5x 4 -3x 2 +2)dx


dx
y = x5

x3

or

+ 2.x + C.

Since (I, 5) is a point on the curve. its coordinates must satisfy the equation of the curve:

5 = 15 - 13 + 2( 1)
Thus C

= 3, and the requi red curve is y = .1 5 -

+c.

x 3 + 2.x

+ 3.

.........
I n taking the indefinite integral of each side of lhc equation

in the example above. we added an arbitrary constant C to the right-hand side. You might
question why we did not add a constant to the left-hand side. Had we done so. the r~~u 11 woul d

ha' e be-en

+D=

x5 -

= x5 -

.c + 2x + (E -D)

If we had then wriuen

.t 3

+ 2x +

E.

.,

and defined C = E - D. we would have obtained exaclly the same result. Hence. nothing is
gained by adding an l\rbitr-.1ry constant to OOth r>ides; a constant on one side is sufficienL
The problem in Example 5.2 was geometric: 1:-:ind the equation of a curve satisfying cenain

propcties. We quickly recast it as the problem of tinding the function y =


the equation

dy
_. = 5x4

dx

3x 2

+ 2,

f (x) that satislies

The Revese Opc:r.:~ ion of Diffcen1i:uiu.n

S. I

341

subject to the additional colldition thut /(I) = 5. Ont-c again this is a di iYc:rcouial equation.
\Vc di1:1cus~ difrcrcmial equati00!1: briefly in Section S.S1Hx.l study them in dcwil i1l Chapter 15.
\Vhcn we solve n differemiaJequmion with l<> subsidiary <Xlnditi ons. say.

dy

= 4x2 + 7x,

dx

we do not get ;, funclion, bm rather a Ot te-panlmeu~r family of functiuns. For this di ffcrcmial
equation. we obtain
4

y= - x

iiltlil;l? ;aM
t\.\1nily of
equation

<;\)!\1liOilS

One1);ltamtk."r

of a diifcrcnliul

7 2
- x + C.

3
2
a one-parameter family of cubic polynomials, C being lhe parameter. Ccometrically. we have
the one-paramerer family of eur,es in Figure 5.1 . Parameter C represents a vertic<il shifr of
one curve relative to an01hcr. Note that if a vcnical line is drawn at any position .t to intersect
these curves. then at the points of intersection. every curve has exactly the same slope. '"uncly
4.< 2 +?x . For examp)l,, the slope of each cubical x = 0 iszcro. If an c.xtro condition is added
to the diffcrclltial equation, such as 10 demn><lthat y be equal to 4 when x = 2. thcll

4 .
7
4 = - (2) 3 + - (2)2

+ c.

or. C = -62/3. l11is co11dition singles out .:me particular furx.:tion from the family. namely
y = 4x 3/3+ 7x 2/2 -62/3. Ge()metri cally. it determines that curve in the fan1ily which posses
through the (l<>int ( 2. 4 ) .
Jn this chuptcr, we discuss three basic ways to find antiderivatives. First. sonlC ancidcrivati,cs arc obvious, nnc.l the better you arc m di fren.~Hiatiou.the ntOc'e obvlous they will be. For
example. you shou ld have no trouble recognizing that
and

-I

.:_ dx = 2x2

x'

+c.

Our second method r-esuhs from the nn~wcr lQ the fulluwing que~tion: How do we check
that a function F (x) is an antidcri\'ative of j(x)'l We differentiate F (x), of course. This
simple fact suggests an approach to s lightly more comple x problem,;, say.

(2.<

We might reason that in order to have 2x

2x

+ 3) 5 dx .

+3

raised to power 5 after dift"crcntiation. we had

3 to poweo 6 before diffcremiation: thm is, a reasonable I)I'Oposal for an antiderivative is


(2.x + 3) 6 Differentiation of this function gi\!es
tl

-(2.t

tlx

+ 3)6

= 6(2.r

+ 3)5 (2) =

12(2.t

+ 3)5 ,

and we S..."C 111.11 (2x + 3)6 is not a correct antiderivative. 1t has produced (2x + 3) 5, as required,
but it has al<o given an 1111dosirable lactor or 12. We theref'orc adjust our original proposal by
multiplying it by l/ 12: 01at is. the correct indefinite integral is

(2x

+ 3)5 dx =

2..(2x
12

+ 3)'; + C .

Th is is whnt we a lii at{jusriltfl consrnm$: we propose an aHidcrivativc thm is within a mu l-

tiplicative constant of being correct Adjusting the 01iginal proposal by the inverse of this
constant gives the. col'rect antiderivative. But remembel'. our initial proposal must be. within a
mttlriplicarh'e con.swn.r. We canoo1adjust x 's. Let us illustrate wi1 h two very sin1ilar problems.

f -:-: - -+:1

-::-:-;

(Sx

2 )5

tfX

arnd

342

Ch.aptea 5 The lndelinite loteg<lJ and tlie Antideri\'ative

+ 2} -'1 Differeotiation gives

For the (irl;t problem we propose as ao antiderivative (Sx

I
]
- 4
-20
-d [
=
(5) =
dx (Sx + 2)4
(Sx + 2)5
(Sx + 2)s

Since we are out by a factor of - 20, we adjust ow original proposal with - 1/20.

.,.----1-~ dx =
- I
+C.
(5x + 2)5
20(5x + 2)4
It might seem as logical to propose (5x 2 + 2) - 4 as an antiderivative for the second problem,

but differentiation yields

d [

dx

(5x2

+ 2)4

-4
-40x
(lOx) =
.
(5x2 + 2)5
(5x2 + 2)l

This time Lhe discrepancy is -40x. We cannot adjust .t' s. The 01iginal proposal must be
abandoned. This indel1nite integral is quite difficult; it will have to wait for the more powerful
techniques of Chapter 8. To emphasize once agaio, do not try to adjust x's, only constams.

I EXAMPLE 5.3
Evaluate the following indetinite integrals:
(a)

cos 3x dx

(c)

3x 2

- 4

dx

SOLUTION
(a) To obtain COS 3x after differentiation, \Ve propose sin 3x as an antiderivative. Since

d
- sin3x = 3cos3x,
dx
it is necessary to adjust with I /3,
I

J cos3xdx = - sio 3x +C.


3

(b) With an initial proposal of e-ZX for an antideri,ative based on the fact that the derivative of an exponential function aJways retUJ'OS the same. exponential. we caJcuJate

.!!._e-2'( = -2e - 2x .
dx
Consequemly, we adjust with -1/2.
1

- -e

-2.\'

+ C.

(c) Since
1
d
2
ln(3x - 4) =
(6x),
2
dx
3x -4

the required indefinite integnll is

---=x - dx
3x 2 - 4

- lnl3x 2

41+C.

We have inserted absolute values as suggested by fonnula 5.41.

The third techoique for ~ndiog antiderivatives is discussed in Section 5.3.

5. I TheReverse Opcl'{lriull uf Oifferellliotiun

343

\ EXt\MPLE 5.4

Find continuous indefinite integrals for the Heaviside unit step function II (-r -(I) introduced
in Section2.5.
SOLUTION Because the function has two values (see Figure 2.35), we subdivide the integration into two parts. For x < a, h(x -a) = 0, and the indefi1tite integral is aconstant. For
x > a, h(x - a) =I, and the indefiniteintegn1Jis x plus a constant. Thus,
h(x- a)dx =

x<a

. ,+ D

..t

.r > a.

For continuityat x = a, we require C = a+ D ==> D C- a. Consequently, continuous


indefinite integrals are

J h(x -tt) dx

'j

Slope= I '

={ ~-a)+C,

< tl
x > a.
X

For future applications, it is conveniemto express these ill the form

h(x - a) dx

( \----- --.../

= (x -

a)h(x -a) + C,

x where, for continuity at x = a, we understand that the value of lhe indefiniteimegral at .r =a


is C. A graph of this function is shown in Figure 5.2. 11 is called a ramp functirm.

(I

EXERCISES S .,

ln Exercises 21- 24 find the curve y = j(x) lhaJ p;l$$C$1hrough lite


given point and whose slope al cuch poinl (x. y) is dclincd by lhc
dcrivmirc indicated.

In l'.~tt<:io;cs 1-10 c...-J!uatc the iNicfinitc integral.

\. J{x) -lx)dx
~. J(l~l - lx + 6x + 6) dx 4. Jsinx d.t

21. dyfd.r = x 2 - 3~

JJC\>~Xd~

S.

1.

6.

J( :~}~.~
J~x>i> - x' dx

22. dyfdx = 2x 3 + 4x,

9.

II.
13.

Jf\ .x',:._

..-1t1 '')d<

J..:.. ,J>

J/itlx

24. dyfd.r = 2 - 4-l

19.

Jc.~
J

2
) dx

(x' +I)' tlx

+S.X'.

(r, 0)

( 1. 1)

Find lhe equation of rhc curve dull h:Js a second <kriva1ivc equ;<l
10 6x 1 and pas.S through rile poirus (0, 1) :lnd (-I. 3).

10.

IL

16.

17.

(0. 5)

* 25.

14 .

'

(2. l)

2J. dyfdx = - 2.r f- .lr1 + 6,

1<1-

11 )

+ 2.

J(;,+ 2~)

f(-~1 +3 1)d<

I llJi ...
J +

- ~X' 1) ,,,

In Excrc.~ 28-67 u"'c tN~/INIIIJt HNf\lflll/1 tn C"\'.tlmtre lllc inddini'c

integral.

28.

+ x 2)2tlx

30.

J
J

32.

(x-1) 1
/

3> , ,,

27. U it possible ro find 2 fun.ctiott f (_r ) th:n ba( a relatj~e mjmm.um


f (2) 3 .00 h.. a <CCOnd den'" ';." "'l'J31 10 - 5.1"7

I) d .1

../X(J.

18. / x' ( l
20.

dr

26. Findu function j'(.r) 111.11 h.u nr<la<ivema.umom /(2) = 3und


has a S.."Cond d:nvativcequallo -5x.

.,fX

tlx

Jx+2dx

29.

J2-xdx

J l.

J~dx
4.t + 3

(2x - 3)' 'tlx

33.

(x

+ 5)' ' dx

(3x + 1) dx

..-... lneftniiC hllcnl anil the ,\ntidtri\':tlil't

Cbap4er 5 "'' "

"

J ~ 4F
J + ti
J :x2)!

35.

3().

(I + 3.t)6

Jx (1+
1

311.

x(.xl

3 1
3x ) dx

J~os'lx dx
4l. J3~n 2x cos 2.t dx
-14. Jcsc 4x dx

40.

39.

(2

43.
-'5.

JXt-' d.t

48.

Je- J dx

49.

j
Je

3x

I - ~.!

3 IIX

J
56. JsinxCI + cou 1 d.t

S4.

5&.

3" dx

Jt" ( I + r

1 1
') d.1

* 64.

Jcosh4.r dx

* 66.

ss.

68.

Jx

I+ .5.r 1

dx

d.r

x sinh Jx 2 dx

j.r

67.

9xl

csch 4x dx

J_!___,
J
l - .r

69. - = J3 - 4x
dx

d l'
' J
).
71. - =.r(lx + 4

70.

J.t = (3.r + S).!fl

72.

dr
x'
d~ = (2 + 3.1.--')2

d.t

d t

d t
I
3
-r
dx -
x2

tl)'

- dx
3.t ~ 2

74. find o funttion y =

d.t

Q\' = sin.t(f ~ cos' x )


73. -

dx

f l.r) thai suosncs the difTcrentiul equation

'2" dx

65.

scc.h 2x1anh 2x Jx

!+
I

In Exercises 68-7J find 3 oocp:nmclcr family of functions satisfying

J-

53.

I
de
x/J.xf-n

dx

eli
- , dx

63.

lhc difl'en:rllial C\ILIUiioo.

4
'

52.

" 6'
. -

sec 12.m.n l2x dx

51.

- -- d.r
1 - 5.t

dx

41.

46.

so.

dx

~s1 xsinxdx

47.

dx

(x

37.

61

dy
tlx = .r1

e'

- - dx
~+ I

and posses through lbc two poin~.>. ( I , I ) and (- I. - 2).

CO!>X

75.

Siut.\'

(I lond diC i~imte iUClJI".tl of rhe "'gnum funclion of bcr


ei<;e 4 7 m Sa;Cioo 2.4.

scc! x

(b)

57.

59.

---~- J.t

- , - d .(
IJII'l'

Pn:l\'c llmr sgnx c;~nnc< h~t un anaidcrivnrivc on any intcrvul containing x = 0.

l s.2 Integrating Velocity and Acceleration


In Section -1.8 we diSLussed relationship., among llOSilion,

>elOCil)'.

end acceleration from lhe

, iewpoim of \k'l'WI\lil-e.. We nO\\ consider thc,c ""'"'ionships ahrouh untideri\'Oti~ tu1 appr0.1l"h pl'O\ iding a far more practical ' 'icwpoinl when a("(),._~ to \Application~.
Nt I\ICIIOil ulong lhc .r-axis. \'elOCity is the dcriv.uivc Of position with respctl 10 rime:
~(I ) = liA f tll. We c.on Y Ulercfore, thou the iudclinjtc ime;;ntl o f velocity rcprCl>CtiiS every
possible JloSilion 1\rnction with this velocity:

-~(/) =

f ll(t ) t/1.

15.61

Similar\) , a< 11CIX'I<'Tlltion i< the dcn\'011\e of velocity, !ht iodclinire imegrnl of aec-eleation
rep"'"'"'' e><f)' pos<ble \'tlocil) function.
v(r)

= J ll( t ) dt.

c5.7

Thus. givc.:n the acct;:len.Hioo of an object moving al ong a slraight line, WI! can ::.ntidi(ft;!rentime
to find its \'el ocily and amidiJrerentiate aguin for i1s posi tion. Since e'-!ch amidifferentiarion
inlroduces un ur1>itrury coost.anl. r~ddilional infonnution must be s pecified in order to evaluate
lh~c constants.

I E XAMPLE

6. 6

I:JirliiJW4 Cw: ilC<:eler.ll


an,g (runa 1~ 1

The car in Figure 5.3 accelerates from rest when the light tums green. Initially, the acceler<u ion
is 10 rn!s2 . but it decreases linearly, reaching zero after 10 s. Find the velocity and position of
the car during dtis time interval.

Let us set up the coordimue sy~tem i n Figure 5.3 and choose time I
0 when
the car pulls a wa)' from the li ~ln. Given this lime convention. the accc1cnuion of the car as a
function of t ime I (~Figu re 5 .4) is

SOLUTION

a(t )
(f

= 10- t,
du

= 10- 1,

dt

,,

and intcgr:lliOtl gives


v(l)

ation function
a

10.

IJ(/) c.Jenotes the velOCity o ( the C;lr during the time interval. then

liltflll;l ~ ew:s lltCCitt-

~ 1 ~

= 101 - '2 + C.

By our tin1e convelltiOl, the velocity o f the car is zero at time


from this we obtain
(O)l
0 = 10(0) - 2 + c.

0~ that is, v(O)

0 , aud

Consequenll y, C = 0 , and
v(t )

(!

lO t - -

Th is is the vdocity of the car duri ng the time interva l 0 ~ t ~ 10 , measured in mettes per
second.
Since the position of the car with rcspcc.t t() itli original positio n at lime I = 0 is denoted

10

b) .<.
dx

v=

=l Ot

"'

Thus.

.t(l)

Since x (0) = 0 , the constant D must alw be t.cro , and the position o f the car ind icated by its
distance in metres from the sto pl ight, fo r 0 :;: 1 :;: 10, is

x(s)

I EXAMPL E 5 .6

= 5t 2 -

,~

- .

A Stone is thrown vcnically upward over the edge of c liff at 25 nvs. When docs it h it the base
of the cliff if the cliff is 100m high?
SOI .LT I ION U:t us measure)' as positive upward. taking y = 100 and 1 = 0 at the point
a nd installl of projection (Figure 5.5). A law of ph~ ;ics states that when an object 11<.'ar the
earth's su1face is acted on by gravity alone. it experieoces ao acce ler-.u ion whose magn itude is
9.8 1 m/s2 lf a denoteS the accele ration of the sto ne a nd v its velocity. the n
{I =

dv
dt

- 9 .8 J

(a is negmivesince it is in the negative y -clircction). To obtnin v we integrate with respect to/:

Stooc~n

upwml O\'(r eda.e of cliff

..

u(1) = - 9.811 + C .

: 1=0
: >' = 100

= 25 , so thm 25 = - 9.8 1(O) +C. Thus, C = 25, and

By ounimeconvcn1ion, u(O)

; -v~25

= -9.811 + 25.

v(l )

\Ve now have the velociLy of the stone ill o~ny given insuu1t. To find the position of the stone , ....e
set

u = dyfdt.
dy
dt

= -9.81 / + 25,

and integrate once again:


y(t) = -4.9051 2
Sincey
and

+ 251 + D.

= IOOwhen l =O, itfollows th<11 100 = - 4.905(0) 2 + 25(0)+D . Hence, D =


y(l ) = -4.9051 2 + 25/

100,

+ 100.

We have found the equmion that tells us exactly where the stone is m any given time. To
determine when ~1c stone strikes the base of the cliff, we set y(t) = 0; ~1a1 is,
0 = -4.9051 1

+ 251 + 100,

a quadr:Hic equation with solutions

-25

)25 2 - 4(-4.905)(100)
- 9.81

25

J2587
9.81

= 7.7 or

- 2.6.

Since the negative roOLmusl be rejected. \VC li nd I hat the $tone ~trikes the ba.sc o f the cli ff after

7.7 s.

,\n important point to note in Examples 5.5 m1d 5.6 is thm the coordinate ; ystcnt and time con' 'ention were i:peei fted immedhHely; chat is, we decided, <:Jnd did so at 1he stan of che problem,
where to place the origin of our coordilote system, which direction to choose us positive, and

when to choose 1 = 0. Only then were we able to specify the CQITCCt sign ror acceleration. Fur

chemtore, we derermined constams from anlidi ffere.uiations using initi al C'Ondilions cxpr~ed
in tern)!) of our coordinate system and li me convention. Throughout the solutions, we were

carel\ol to refer e"erything to our choice of coordinates and time. Remember. then. to specify
clearly the coordintne system aKi time conventioll at the. beginning of a problem.
We should also note that in e-ach of the e<amples we integrated with respec1 10 time only
those ct}uathms thm were validfur tl ran)!c o.f values of lime. It is a common error co integrate
equa1ions that are only valid at one instant. For instance, students re-ason in Example. 5.6 that

lite. initial vd od t)' is 25. u


dy / dl
25. and hence y (l )
251 + C . This is incorrect.
because the equation dy / dl = 25 is valid only at time I = 0, and canll()( be inte~ratcd.

I EXAMPLE 5.7
Find Lhe s peed thal maximi:tes How r.ttc in the App lk aLio n Preview and dr.tw any c.o nclusions

A)l)Jiication Preview
Revisited

that you feel are j usti lled.

Flow rate is, at the momen~ a function of speed v nod distance d between
''Chicles, r = v/(1 + d ). To mW< imize r, we must cxpt'cSs it a> a function of one variable.
As was suggested, u and d arc related, d should increase when u increases. We use the
fact that d should be the stopping d istance. for car< tra\elli ng at speed v to find a functional
relationship between u and d. The stoppiug d iswocc of a vehicle is composed of two parts,
distance dr that ~te ' 'ehicle trdwls during ~te time it takes the driver to get his foot from
the accelera10r to the brake, and do . the distance travelled while the brake is applied. If T
represents the reaction time for the driver to apply the brake, then dr = vT, where v is
the speed of the vehicle before broking. If a is the acceleration of the vehicle during the
braking period. its velocity is V
at + C. If we choose t
0 when the brake is applied.
then V = u at t = 0, and this implies that V = at + v. Dismnce travelled during the
braking period is x = at 2 f2
vt D. If we choose x = 0 at I = 0, then D
0. and
x = at 2/ 2 + ut. The vehicle stops when 0 = V =at+ u
1 = - u/ a,anJ ~terefore
th1 = ll( - v/o) 2 /2 + v(- vjo) = -v~j(2a) . The stopping distance for a vehicle when
reaction time is T and acceleratjon during braking is a is
SOLI.l llO'I

+ +

tf = dt + do =

vl

vT - -

2a

We can C\'lllu31e T and a using tbc fact that d = 52 m when v = 22 ntis, and d
v = 31 ntis. These give

31 2
96=31T--.

22 2
52 = 22T - -

211 "

2fl

the solution of which is a

-6. 138 m/s2 ancl T

r ( v)

v
=-=
I+ d

= 96 m when

= 0.5715 s. The flow function is


v

-:---:::--=.---::-..,.,.--,.
I + vT - vl j(2a)'

h.s graph is shown in Figure 5.6 for l = 4. \ Vc;;. sec thai it has one critical point~ \\lhich is gi\'cn
by

0 = r'(u) =

(1 + vT - ~) (I ) - v(r - ~)
(1

When we set the numerator to


lllJ1tj[A Flow runenun for c:~rc on his:huoay

0.6

~cro,

+ vT -

~r

and multiply by 2a,

= 2a/ + 2avT- v2 - 2twT + 2v 2 = 'UI/ + v2

==>

= J-2n1.

The speed for maximum llow mte is J-2(-6. 138)(4) = 7.0 m/s, which is about 25 kmfh.
Obviously highway speed limits ,ue not set with safety in mind. On the other hand, safe
distances between cars at typical hig)lway speeds of say 100 km/h, OJ' 27.8 m/s, would be
27.8(0.5715) (27.8) 2 /12.276 = 78.8 m. and we k11ow that d rivers do not maiotai.n any such
di.slance.

0.4

0.2
2

" ~db
~~---------------------------------------------------------

EXERC I SES 6 .2

In Eu.n:n-C.) 1-8 "'-eh~cdcfioedaceelcraliooo( l) of nOO,ec:1n1min,


along the .\ ~a.~is durin1 wn lime interval and !i)OCified d~ 1nih;al
c:t.'llditicns .r(O) :U1d u(O) Fond the \'ctOCoty u(l) ond po.,tion t (l) of
the objxl ~s functions of time.

I. a (l) I + 2.
2. u(t ) 6

0 !: I

21.

= 6- 21
a (l) = 1201 o(l ) = 11 + I.

J. u(l)

4.
S.

6. a(l )

0 ;:: 1

121 ' .

S 4:
0

=0, X(O) =0

u(O)

=' 1 ::; 10:


u(O)

+ 16. You arc ~and ins utthc bit<c uf a h.titdins: and wtfih to lhrow a 0011
10 mfriend on the roof :!0 m abo\e rou. Wi1h wh.u minimum speed
llltto,i )'OU throw the bull?

= S. x (O) = 0

v(O) = S. r (O)

0 ::; 1::; 5;

u(O)

=-

0 ~ I ::; 15:

1 2::.0:

8. a(/) 1.;n1.

u(O)

0 ::; 1 ::; 3:

= 12 + St + 4,

7. o(l ) - co<t .

3:

IS. Yoa ore SlarlCtit on a bn~ 2S 0 1 ..OO.c ri~r. If you "ish 10


dn'J) .a g~ on10 a piece of lo~ -.'OOd,. how oon k-tlW'c tlli.: 'WOOd
rcache< the oppropnatc pot hould )W dtop the .co,./

1 17. A n r is travelling at 20 n ~s when lhC brtl ~ IU\! UJIIJiii.:d. Wh:at

=0

= 0. x(O)

c.: onstant dc<.clcrution must chc c.a.r csrwrienee if it is 10 stop before


striking a t.ree thlu is SO rn fro1n tho car ott Ihe ilt.ount the brakes arc
npplicd ... A'isun'IC ths t U'IC c.a tnwcb 1n a !llfil~ht line.

l ..t(O) = I

v(O) ~ -2. x(O) -~

1 18. rhc

1 2::.0:

I'Cr K.."(.'Ond by
v (t )

r(O)

X(l)

= l . .x(O) = 4

(U) W I\Jt b lhe llCCCk1'a tion o f lhc objl Ill I

I'"'"

in niCtrc.' per Jaund p.:r ~-.:onJ for ti nlC I ~ 0

(O) I( lhC punktc SlllltS Ullhc jX)int ..t = I R'kt\ i r~ IV th.,: left
with ~d 3 mk. find us po:sition 15 a ful'l(."CtCfl lllnc '

or

(b) At whlu 11me does the panicle ha''C t.cro \'CIOCrly (if .trty)"
.._ II. I M :tccck.YlliOtl of a pattid c DlO\ing alons the J. aXi( i\ 21\cn in
mctrei per M..X"'O'\d per second h)'
u(l)
~

" "" u(O)


t

20.

Yo~.a

rn'C coiled on as nn expert to leSJify ut auomc helll iJ'~.

TI ~

11. When the hrnL~ d an euttMoob~lc rc ~Ired. they Pfodv a

coru.t...uU docclcr.botl of S ml$z,

(a) What is thcdr,larn.\!. ftcntllt..:l>ttint of'a,)I>IKu&tonofbloakcs.


r'C<)uirt'd to stop a car U'Ovellln~ at tOO ~mlh l

= 61 - t5.

tb) ReP"' pon (a) f01 SO kmlh.


(c) What is the rmio of these tliswuccs?

cu) If the velcxity of Lhe pcu1iclc mt ~ 2 ' i,; 6 rnl$, wh.tt is 11~

(d ) Repeal rmrt.:: (u), (b). and tc) &1\ Cn Ihat II"'C reactio n tin'ICo f
lhc driver to g...- hct f~ l'romaccclcl'ator lo brake iii 3/4 s,
and diswnccs 1uc calculated U1Ung this reaction tiolC in1o

vclocily ul.f - I s?
(b) lft hc part ide i$ 10 mlo thc ri ghtofthc otigin ol linlC I 0,
wt11.u is its position as a runction Qflin.c t?

is the closest the puniclc c~r oomulo the o.igin?

ll. A c:Jt is sauu11al rest a a stoph~ht Wbcn the light tw-ns ~en lit
unte 1 = 0. the: dn\cr immcdi..lldy presses the aL'l..'Cicrutur. nnp.min&
an .octtlcnlion of a (l)
(3 - t / 5) mls' to thee..- r.,.. 10 s

(a) Whc:rt isthc: cor aftorlhe 10 s?


(b) tf the drover applies the brakes at / = 10 s. and the cor
cxpcracntts a constanl deceleration of 2 mls~. where rtnd
when doc'i the car rorne to a stop?
13. f ind how far a !>lane will rnove when landing if in t seconds after to uching lhc gr'Ound. il.s speed in meues per sct'Ond i~ given by

= - 33 and x(O) = 400.

b r-a~cslockcd :1nd wheel.! bl 1d i1 ~. 'r'hc: ~k rd m .uk on Ihi; ro:d mc;~surcd


tJ m Assummg thai the dccckntttOO of the C"dl" w~&s conscanc and could
not ucccd the O<:CCiemtion due tOA'"' ity of a freely falling body <and
thi\ is indeed a rcasoro.blc ~swnplion). "hal CJ!'t you 4U> tlbolt 1hc
~reed ortheC"..tr before. the tr.akc. WCfC JppHcdf Are )OU tcslif) in& ror
the prosecution or lbc defence'/

0 i" l;i nrc in tcc.;onds.

(C) Wh.lt

= 61 - 30. 1 2: 0.

quculunC<llleel'M U1e qJeeJ uf J cJr llutondc W1 <ftlei'<<Y ' '"P wi01

.r-axi~ h;lSaccclcr.iliQrl

= 61 -

u(l)

a (t )

= S/4 s?

(d) Wll.&t i11 I he cl o~(:"Li hcol>j ocl cve1comer, l<J lhc ol'iGil?

I ~ 0.

1~ . Rcpw E=ioc 18 sovcn lhJitht liCX>OI<r.llon of the ob}Xt is

5 ~?

(b) Wlu t ;, the


lion o l'the objcct :.u 1 = 2 1
(c) hI he llhjccl ~peed inn up or s lowinu, down ~H r

10. A p:.1rtlclc mo\lna along the

=11 -611 +W- 20.

,.,crt b)'

O<aw graphs of lhf 1)01>101>3. \1:lo<:.ty. ontl ccelcration fll!lCtions for


tlli rn01ion. Py rt.l onoo"on 10 dx: f<l d.a v(l) n:pr<>cnts dx:
IOCJC of .x(t ) and o (t ) ,. tile slope of t I>(I).

= ~1 2 -W +6

where t i\ lUI'IC irl ~C4. 0nd). IJ the: o bjL"Lt st1u1Jl from pclSIIJOft .\ :a: 1 m
ul tunc 1 0 . ur.wcr c~ 'h uf Lhc following quCbC.to n t~:

where l

o r an oh~ movn& ~klon, t.h!; .r-:n~ as

t(O); O..x(O) = 0

9. 'f hc YtJocil) Of Jnobi~"Cl nlO\iflt~ lhr.:.XUi~ i.s~ivcn in metres

JX-~i l i on

aL:<.."'unl.

1'2. A !~tone is c!.rop-pcd into a well and Lhe wurtd of thl.! li.L(lf'IC striling
the 'A'at cr i$ hc.:ard J. l "i)aicr. I( the ~pt.'Cd or M.'IUnd i$ 340
deep is 1he surface ortbe water in the -.ell'

mf'~

how

2.1. Twonins. one r.noveltng ot tOO Lnllh arwl thcothc:rot60 kmlh. aoe
Ma<k:d I0\\1'lnl ~ach Olh~r- a.Mng o Jll'.llS,hl, levellnlC."k. \\fMn they are
apul't~ C:!lch engineer 10ec~ the tlhcl'i( lnin 1t11d kx.-ks h is wb.x:ls.

2 km

(a) lf lhe deceleration of each ll'ain ha'i mag.ni1ude I /4 m/s 2


dt:t.e n:ninc whcthcl' a eolli,K'Jn <lCttlrS.

180- 181 .

(b) Rer>eat prut (a) given Lhal the docclcmtion is caused by the
wheels being I'C\'Crscd rather than locked.

14. A ~l o ne is thrown directly upw;u-d with an iniljaJ speed nf 10 m/s.


How high will il fi o;c?

(c) Ulustr.tlc gr.:tphicaJiy lhc diiTcrcn<."C between the situations


in p:irt.S (a) ond (b).

$.3 Chongc of V~riable in

* 24.

A SICCI bc:~ring i< dropped from 1he roofof a building. An obsen'er


standing in from of a window I n1high notesahatlhebearingaakes 1/ 8s

10 fall fro1111he 1op 10 01e bouom of1he window. The bearing cominues
to fall. makes a c-omp!cldy ctasticcoJJision wiUt a horit.nntal sidewalk.
and reappears at the l>ouom of 1he wiodow 2 s after passing il on 1flc
way down. Afw.r a cl>mplctcly cla:,tic collision. the bearing willlw.'-e
the same speed m a poinL going up ai it had going down. How tall is

349

* 29. T'vo slones are lhrown \'CrtiC"JIIy upward over the edge of a boiIORlle.ss abyss. the srcond stone to units of lime :after Ihe hr$:1. The tir;:t
Slone has an inihal speed or uO. and lhe second 3n iniual speut of 11;.
(a) Show tbat if the ston~ an; C\Cr tO pass c.a<:-h other during

their motions. lwo conditions musl be saai:sficd:

25. A con.,oaruclion elev:llor without a ceiling is: ascending with COC\Stant


s peed 10 m/s., A girl on lhc clc\'ator thrO\\IS a baH dinxtly upW3rd from
a height of 2 m above the ele,a1or floor just as the elevator floor is

and

2& m abo\oe the growtd. The initial s:pc.."'ed of the ball with respect to the

elevator is '20 mls.

when:

(a) What is the uwxirmnn height attained by the.: baU?


(b)

(b) I low Jo1lg docs it take for the ball to rctwn to the clc\ator

floor?
26. 1\vo SIOOCS are lhrO\VO \'Crtic.ally upwanJ one second apan_ O\er
the edge o fthecl iiT in Example 5.6. The first is thrown at25 nll's. the
second a1 20 m/s. Determine if and When they et'Cf pass each other.

* 27.

fndclinite lrM.tSr.ll

28. Whal speed maximi7cS 1hc flow r;Uc in Exarnplc 5.7 if c:trs :~rc
required to m:oinoamonly a (rae! ion k (0 < k < I) oflhe.:afe di$1ancel

the building?
i<

lhC

ln th elheoryofspecialrclali\+ity.Ncwlon'ssccondl~w(F

v0 > t;lo/2.

8 > 0 is; the accdenlion due to gravity.

Show thatlhc fl~lcond1tion iscqui\-alcnttolhen:quircrnenl


that Slone I mus1bcf:in it~~; downward trajectory before stone
2.

(c) Show that the second condition iscqui\'alenc to the require.


mcnl thnt s1one I mus1 nru p!1.(1; its oriBin:al proj~ttion point
before lhc projection or stone 2.

=ma )

is rcpla<..-ed by

F = m 0 -d [
dl

II

/ 1- ( v'fc1 )

where ,.- is the applied force. m 0 the nla~ of the particle mca~red 31
rcsl. v its speed. :md ~ the. speed of light - a cons1an1. S hO\v al'k:ll if

we set a

= dvjdl. then

F=

moa
( I - (v'Jcl)Jl/1

Explain the difference bel ween this: law and Newlon's second law.

30. Speed bumps arc to be pi a.ccd on a su ai glu srn:-teh of road in onkr 10


cn~urc that tnallic~ddoe~notcAc"d 10 n1/:,, TilCq~tionconccns
their :,pacing. Sup~ \'Chick: speed is reduced to 2.5 uti:, Oil btuups.
\ \:Jlfcb ac:cclenuc uniformly aW""o:iY from a bump at 3 mls 2 and then
llccclcratc unifonnly towdn.J ahc;: ncxl bump at 1 nu's 1. Find the dis1ancc
I'IC!w~cn 11\!mp~

31. h wkes time T 10 dri'~ your car a distance D al()ng a :\l.r.1ight


highway. You do :w by uccclenning uniform!)' fronl rest, auaining
m~U.itnum spccU V. wl1ich you maintuin for some length of time. and
I hen tkx.-c lcnning uniformly 10 a :Mop. I low long do you main rain sp;:.ed
V irthe magnitudcsorareaccdcrarion and lkcelcnuioo arc the same?

15.3 Change of Variable in the Indefinite Integral


Tn Section 5.1 we sugg.Qt.ed awo methOt.ls for e\aluating indefinite integrdl~ - recognirion and

adjus1ing cons1an1s. Jn this section we show how a change of variable can often replace a
complex inrcgration problem with a simpler one.

Consider Ihe indcfini1e imegral

jx J2x

+ ldx.

Whal is annoying aboUIIhis in1egrand is Ihe sum of rwo 1Cm1s 2x + I under a square 1'001. This
can be changed by seuing u = 2x + I. As a result, J2x + I - -Jji, and lhc x in fron1 of the
square roo1 is equal1o (u - 1)/2. 'ow Ihe differemial dx is used 10 indicale imegration in the
problem wilh respect 10 x . Bu1 surely 1here must be aoo1her reason why we have choseo 1be
diJTercmiallo denote Ihis. If we regan! x J2.r + I dx a.~ a prod uti or x J2.r + I and d.r. Ihen
perhaps we should obtain an expression for dx in tel'ms of du. To do this we note that since
u = 2r + l,then
du
= 2.
dx

3$0

Cb.lJ!Ie 5 lb: Jnde(initc: Integral und tlte: Aftlillcri\~IJivc

As derivatives can be regarded as quorients of d_ifferenrials, we can rewrite this equation in the
form

d11

dx = - .
2

If we make all these substitutions into the indefinite integml

j xJzx + ldx,
the result is an integration problem in Ihe \'ariablc u. which isca.sy to evaluate:

f( I) ../ii 2 4II
u-

d11

- 2-

If 11 is now replaced by 2x

(ul/2 -

u 1/Z) du

= -4I(2
- u'll s

2 1' )
-u'
3

+ C.

+ I, the result is

Differenriatlon of this function quickly indicates 1ha1 its derivative is indeed

:c ../2x +

I and.

therefore.

x.J2x + ldx = - (1x


10

1))/l -

- (2x + l)lll + C.
6

In this example. the substitutioo 11


2x + I replaces a complex integration in x with a simple
one in u. Once lhc problem in u is solved. replacement of u's with x's gives the solution lO the
original indefinite integral. Th i~ method is ge;nerdlly applicable and is j ustified in Ihe followi ng
theorem.

THEOREM 6 . 3
Suppose the change of variable 11

= h(x)

wilh

f(x) tfx

x = g(11) replaces the indefinite integral

f(g(ll)) g'(u) d11,

where g(11) is dift'erentiable on some inten-.ll, and /(.t) is continuous on the range of
g(u) . If F(u) is an antidc:ri<ativeof /(g(ll)) g'(u) . thcn

f(x)dx = F (h(x))

I EXAMPLE

5 .8

+ C.

Evaluate the following indetinite integrals:

(a)

!./2x

+ 4dx

(b)

I .lndx
x+ l

(c)

SOI.U1lON
(a) We could adjust co,,
, .
Clllculale
t11niS n this case.

With an initial &uess of (2.r

d
- (2r
d .r

+ -t)6f>

6
5(2x

Thus.

(2x

1\ltcrnativcty. if we set 11

=2 r + 4

.I1>en

(2x

5
4) 1 dx

(b) If we SCI II = x

Julsdu

X
r::--:--:
dx =
vx+t

+ 4) ;s(2).

~(2x
+ 4)6/S , C
12

+ 4) '1S dx =

du == 2dx . and

~ (5 6/S) ,

6"

., c == 12 (2.t + 4)6'' + C.

+ I , d>en du = dx, and

! "-I

-pvii d11 ==

(u ''' -

= 3(A' +

1)3/2 - 2(.r

.,

~~-ll) du = :3 , 3/l -

du =

X
J./X+!

dx "'

2JX+i

J(u1-

= ~(X+
3

2 1/l
11

T 1}1/2 + (,

!\different substitut ion is also I>Ossible. If we se1u

(c) If we set u

+ 4)6/$

d.<

1)(2cfll)

= JX+T. then

and

c; -II) -(

I )J/l - 2(.< + 1)1/2 +C.

= co~x , !Ilen du--sinxdxand


-

J
,
=
J
=

(I _

co~2 x) cos2 x sinx dx

( l - 11") 111 (-drt) -_

j <u' -u') tlu

u5
u3
; - - - +C"' -I cos5 x - -I cos3 x + C

+C

~e

E X ERCI SES 5 .3

In Ex~rciscs 1-30 cvafu:llc lhc indelini1c intcgru.l.


I.

+ 14)

(S.v

3.

5.

J.r ( 3x

dY
-

(3,1'- 12)'' '


2

2.

tlx

4.

+ tO)' tl.r

6.

7. Jsin" xcosx dx

8.

9. J J'i"'='Tt rlt.

10.

I I.

13.

15.

17.

19.

21.

+ 23.

+ ./X dx

J
J

14.

sin X COS .t d.t

.t'
(l _ x' )' d.t

J + ../ii)'''
'
J+fiz'l,. d~
(I

..Iii

(.r - 1)(.\'

.;;

Jv' t J s
J +
J x'
J./2x +

12.

.;;

I 6.

18.

d ll

20.

+ 2) d x

f JI + sin 41cos3

22

4/ cl/

" 25. j 'tan?.tsec .td.f

2,-,/,r

* 29.
d

(5 - 42x)'' '

(.<'

'*

dx

30.

xln ,t

dx

32. F.v:.tlu.ntc:

. (x2

intc~rnl

ln.t
dx

+ l )(ln (.x ' + 1) ]-, dx

sin.l ;r t.os) x d x.

tilt:

33.
~

34.

J,'ll'-4d
y
. 'j x (3x' - 5) tlx
j (x' +x2x++ d x

36.

u c: sin x.

Jj

x' dx .

l+ .x

In E.xc'Ci!O;C.t:; :n -36 u10c lhc suggc,;ted dHdl\ge or ''aiablc to C\'uluatc

35.

2)'il

indefinite intcgr.d.

J 4,'( - ,'(2
,
dx:

J
J
f
J

+ .;;,, _,

$C( If -

.r

v'x - x'
X4

dx;

S<'tll

(5

4x

x )

u' =(I -

+ 4.r) v"i"='?

+ 37. Show thllt the $ubslilution u imcg:ral

d x:

.<)/( t +X)

I / .,.z.

.jxi + .\' + 4 rcplucc; the

.f

+X + 4 (/ .'(

\\ith the in(Cl;ral of u mtional fun<.:tion of n .

38. Show ohutthc M<b>litlllion (x


the imcg.ral

+ 1)11
xi

,\" s.ec x tlx

set ' = (5 + x )/( 1 - .r)

3( 1 - .r') - (5

2/X

= l /X

-;;--:--;:,.,,1;;, dx:

t.:os J.
d.r
(3- 4 inx)'

Jt:~n

- - d.'(

f
f

!8.

(c) Vc1il'y t hat these an~-wcrs arc the san'IC..

J l - c.os x sinx dx

26.

11

" Jl. Consider the

dx

I i~

-21- - d r
e +1

(b) Evatu.."'te it by lll::lkin& the ::,.ub,titution 41 ;;;;;; cos x .

j s\ls 2 +Stl.'i

J ''
f

(U) Evnluutc it by m;.lktn~ the s.t1 ~ ituti1>n

4)'

(.l - 2)'

24.

t<-

... 27.

= ,/4 + 3x

.t1 rcpluc.:~

d.\'

whh the integral of lii'Uticnul runction or u'

ls.4 Deflection of Beams


rcpUTla::1fJ1
beam un\.ler

O~lle<."t i on

When a beam th"t might otherwise be horizontal i s subjected to IO<tds. it bends. By anal yring
imcmal forces and momems. i t cnn be shown that the shape y(x) or a uniform bc>nn with
constant cross-sect ion (Figure 5.7) is governed by the equation

of

l~r.c llrJ:

13catn

..

d4y
dx 4
X

F (x )

= fl '

( 5.8)

where E is a constant called l't:Jung s modulus of elasticity (dependjng on the material of


the beam), and I is also a constant (the moment or inertia or the cross-section or the beam).

8 t.111.. bead
111;, llnl.k1

ow ""'C'I'h t

I''---- m-M__:;'_"_~ O ~
I0

Quantity F (x ) i the load placed u n the bcwn: it i the \crti<al rorce per unoo lcngtl1 in th<
.t -dite\.1ion. placed WI J~i tion X, lllCiuding the \Veight Of the: be:un il$elf. Por t'<.ample. if a
beam has mass 100 k(land length I0 rn (Figure 5.8). then the luud due to its ''<ill Ill is a consun\1
F(,<) = - 9.81( 100/ Hl) = -~8. 1 Nllllutev<r) ll<>int uf dlCbemn.
Suppose a block \vith m~ SO ..,. unifonlt i1l cross-scctiorl. IAI.I length S m i~ ploced on ahe
left hal f nf the beam in Figure 5,8 (see Figure 5 .9). h adJs an addioional lo.>d or 9.8 1( 10)
98.1 N/m o,cr lhc imcn a1 0 < .\: < 5. 11lc totulloud can be reprcstHtcd in h:rms ofHccwi~h.le
un it seep functi on.) us

F (.t )

Rta boldb.,_ un.Jtr h ow ""'eitht ad an


llddii1U.MI lood

F ( r)

Stmrlc

,. 0

= - 196.2 + 98. 1 h (x

<X

< 10.

= )"'(0) = 0.

2. Uuiltin End
If the end X = 0 or the beam i pcnnancntly fi>ed in ,, horitontal
y(x) sntisfies

>(0) = y'(O)

(~

l>~>llioo

= 0.

'Ia\

(Figure S. ll ),
(~.' h

3. Free Support
If the end x = 0 of the be<>m i< not supponc<l ( Figure 5. 12). y (x) satific<

L
nu '!oupport al ,\

l. Simple Suppt>rt
Tiu: cucJ of a bcan1is simply SUJ)vortcd whc:n il cunnot mU\'Cvertic all}' hut il) free to rntate.
Visualitc n hori zontal pin perpcn4.1icular to the \ .f plane p;h,ing, through a hole in the end of
the be,ull at .r = 0 (Fij!ure S. IO). The prn is fiet.l, hut th< end of the beam can r<>~atc oo the
pin. In lhr> case, y (.\ ) must satisfy the lx>undary conditions
y(O)

.. A=O

- 5).

ACCOIIII>Mying equation 5.~ will be four boundary conditions defining the type of Sllpi'Ott
(if nn) ) nt each end ol' th<! beam Three l)pc> of >Upports arc oommon. We di.""" lhem ot 1h<
left cod of the beam. butoh<)' also oecur a t tl1e riJht cnd.

r.

5)).

(Re..:all thntthc funetjOil hCt ) - h(x- 5) turn,; lhc: runctiotl (- 98. 1 in this C~lSt:) in rrwu uf
it on at .r = 0 ;utd oil at x = 5.1 Rocau,c the beam c<tcnd from .r = 0 10 .r = I0. nd
h(x) = I th<re<Jn, "e c,tn write th:ll

10

~m

= - 98.1- 98. l[fl(.r)- h(.< -

=0

y" (O)

> ~'(0)

= 0.

\Vhcn two bouutla') t."onditions ut cuch end ora beam ttccumpany d1ffcn:ntial eqmui un 5.8.
\VC

I EXAMPLE 5.9

have w llal

is

called 11 boundary\'ttlue problt nt .

unifornl bemll

with mass 100 kg 1md length 10m has both ends buill in hori1.,.llltnlly. Fi11d lhe
dcOcctiOil curve for the beam.
J\

sot l -no-.;

The boundary-\'alue problem for deftection y (x ) i<


~8.1

d l

d.;. = - 1 '
y(O) = ) '(0)

0 < .< < 10,

= 0 = y ( IO) = y' ( 10) .

Four inlegrmions of the di fferenth1l equation g ive


y(A)

= -1

- -9S. -Ix
24

+ Ax , + Bx 2 + c .r +

o) .

354

Chapter 5 Tht lndtfi11ite lf'l(ttr.\1 at1d Tht: Antidorivativc-

where A. 8 . C. and D are COtlStliOI>. 11~e lboumhory conditi ons rc<1uirc these constants to
~~l li~f)'

0 = E l y(O) = D.
0 = E / y'(O)

= C.

0 = E l y( I O) =

0 = E / y'(I O) =
These yield A

98.1(1 0) 1
.c..:._:-'-..:.:c..

24

98. 1( 10 ))
6

= 327/4 and 8 = -

+ .4(10) 3 +

/1 ( 10)2

+ C (I O) +D.

+ 3A (10) 1 + 28( 10) + C.

1635/4.and therefore the curve of deflecti on fonhebeam

is
I (

y(x)

= El

327x

327x 3

-""SS + --~--

1635x 2 )
4

M ax imum deflecti on o f the beam should oc'Ur :11 i ts midJ>Oi m. To cunfi rm this. we find criticlll

pnims,

0 == y'(.r) ==

Ef( - 4x'

80 .

+ OO.r 2 -

3V

200x) ::: - - E x(x - IO)(.r- 5),


20d

SoiUiions arex = Oand .t = IO (bc<:ausccach cnd is fixedhorizomally) Mdx = 5. Maximum


deflection i s )'(5)
-40875/(16 / ) . r'Or a t>eltm such that the product /
10~. thc
deflection <ll x = 5 i s y(S) = -2 .55 x IU-J m, that is, 2.55 mm.

T his problem was relativel y str:tightforward due m the fact that the io<td fnnct ion F(.<)

- 98. 1

is continuous. Discontinuous lond funclion.s lc:t\d to more compli<.:allcd ctt.lculations.

I EXAMP LE

5 . 10
The end x = 0 of the beam in Figure 5.9 is ho rizontally built-in. and the right end i s f ree .just
l i ke a di ving board (Figure 5.13). Find t he curve o f deflection.

SOI.V I'ION

I'

s
I

The botmdary-value problem fo r deflections .i s

tO
I

'

d4y
1
= - (- 196.2
1
dx
I
-

y(O) = y'(O) = 0,

+ 98. I h(x -

.
5) ],

0 <

y"(IO) = y'"(IO) = 0.

< 10,

$A

l rttcgration of the differential equation four cirnes Oil the intervals 0 <
gives

l)dlt(.1ion cl lbnl'l

255

x < 5 and 5 < x < 10

I { - 8. 175.r + A.r 3 + 8.<1 + C.r + D . 0 < .< < 5


y(r ) - - F: f -4.0875x ' + l'x 1 + Qx2 + Rx + S . 5 < x < 10.
To ev<.1l mue the eight ctul s tolJl ts. we imJX)SC the four boundary ('onditiuns. and also derl1i111d that
y(.t ) nrld itS first three dcri\'ativcs be continuous ac ~t = 5. This means lhat left- and right-hand
limi1s of E f y (.<). E f y' (x}, E f y" (x ), and E I y"'(x} are 1he same a1 x = 5.

Ef y (O)
0
E / y'(O} = 0
l': l y"( IOJ = 0
f y"'( IO)
0

lim E l y(x )

.t ~ S

+ 25 8 + SC +

lim l':I y' (x ) = lim /)"(.< )


-32.7(5) 3

D = - ~.0875{5)4

+ 75A + lOB + c =

x - 5 ..

-98. 1(5) 2

-1 6.35(5)3

lim E f y"(x) = lim E l y" (x )

+ 30A + 2B

+ 1251' + 25Q + SR + S.

+ 75 P + IOQ +

R,

+ 30P + 2Q,

= -49.05(5)2

lim /y"'(x ) = lim / y"' Cr )

,'( """!'~

= 0.

===}

... ~s 1

.t- s-

- 98. 1{10)

=x-lims-t E f y(x ) =

- 8. 175{5)'1 + 125A

.t-.s

+ 6P( IO) + 2Q
+ 6P = 0.

- 49.05( 10i

= 0.

c = o.

===l===i-

,, _ , ,

- 196.2(5)

+ 6A =

-98.1(5)

+ 6P.

Solutions of these equations arc

B = -3065.625.

A = 245.25,

P = 163.5.

Q=

-2~52.5 .

= 0.

R = - 2043.75.

D = 0,
S

= 2554.6875.

The funCJ ion describing deRec1io ns of 1he beam is, therefore.


( }
)'X

= El

A graph of this runction for E I =

2452.5.x - 2043.75x

+ 2554.6875,

JlP is shown in FigureS. 14.

10
X

-0.02
-0.04
-0.06
- 0.08
-0. 1
-0. 12
-0. 14

o :: x :: s

30 65.625x1.

+ 245.25x3 4
- 4.0815x + J63.5x3 -

{ -8. 175x 1

5 <

X :::;

10.

Fortunately there arc easier ways to solve this problem. One such is to use what arc called
Laplllcc tflllf1}0rms. You will le.arn them in tltlvanced calculus courses. Even at this stage we
C>llt simpl ify calculntio ns considerably. Difficullics aro~e when we imegnucd the diffcrcmia l
equation ,;eparately 0 11 the intervals 0 < x < 5 and 5 < x < 10 . Titis intnxluccd four
additional constants of integration that were evaluated by demanding that y (x ) , y'{.t) . y" (x ).
and y'"(x) be continuous at .< = 5. Suppose we demand from the beginning that y(x ) and itS
first thn:c dcrivati~ be continuous for the length vf th~: beam. What this 111CiUlS i~ that \\C nt."ed
continuous a ntidcrivativcs of lr (x -a) . We did this in Example 5.4. Titc e<>ntinuous indefinite
integral o f II(.< -a) is

ll (x - tt)dx = (.< - a)lr(x - <t) +C.

provided we undersuutd thal the ' 'UILK! is

Cmx =

t5. 10a)

The following indefinite integmls a re

(t.

also continuous:

I
I (,<I (,<-

(x- a) h(x - a)dx

2(x I

a) il(x- <t )dx =

cx - a) 3h(x -

I
-<-<
4

a ) 311(x- a)dx =

(~.l Ob )

a) lt.(x- <t.) + C.

a) + C,

tt}' h(x - a )

+ C.

(S. IO<l)

pru''idcd t~gain lhat each indefinite integral is given \':tluc C nt .:r ;;;; fl. In general.

(x- tt ).h(x- a)dx = -

-(x- a)"+'h(.r- 11 ) + C.

(5 .1 1)

11 + 1

With these, the solution to F.xamplc 5.10 is far easier. Four iottegnttions of

d4v

1
d.;' = /l-196.2+98.1/t(.r
-5)1

using formulas 5.10 g ive


1

)(,r)

I [ 196.2.r
98.1
= -;:-+ -(x
r, 1

24

24

- 5) h(x - 5) + A.r - Bx + C.r + D .

The boundary conditions require that

= E l y(O) = D.

E ! y'(O)

C.

0 = E / y"(IO) = -

1\16.2(10)2
2

0 = E l y"'(JO) = -1%.2(10)
These can be solved for

y(x) = -

98.1(5)2
2

+ 6A(I O) + 28 .

+ 98.1(5) + 6A.

A = 245.25 and 8 = -3065.625. The deflected c ur\'e is

196.2x
24

1
+ 98.
(x
2A

'

- 5) h (x - 5)

+ 245.25x

'

- 3065.625x.

This is equivalent to Lhe funcLion y(x) found in Example 5.1 0, but its derivation and final form
are unmistakably simpler.

In Sc<.:cion 2.5 we s uggested thn11hc Dirttc<.leii:J ti mc rion is used to model point sources in
engineering and ph)'Sics. We use it here to model a point force applied to a beam . T he load due
to a point force o f magnitude F newtons applied vert ically downward at a pohH Xo nn a beam
is represented mathc:m'-!tically by -Fd(.t' - .\"o). Suppv.sc: it is applied to a beam of lenKlh L.
and t hat F is so lar~c tiHH lhc w eig lu of' the bt:arn is neglig i ble b) comparison. In this case. thi,:.
diiYerential equation describing deflections is

d'y
d.t'

F
- - i i (x- .to).
1

If the heam is horizontally fixed C:ll x = 0 ;uld sin1ply supported at x = L . deAecliOil.S mu~'t
alsosatisry y(O)
y' (O)
0 y(L)
y"{L ). According to Examplc 3. 11 in Scction 3.3,
h'(.< - a) = d(x- a). Consequently. we can write that

= =

8(x - tl) dx

(~.12)

l!(x - a) + C.

Using this result, imegration o f the d ift'eremia l eCJUiiLion g i\'CS


I

= f [-FII(x -x0)+C].
T hree additional integra tions using formulas 5.10 yiel d

where we have absorbed a factor o f

6 i nco C.

The OOundary conditions require ti'Ull

0 = /y(O) = D.

0 = E fy'(O) = B.

= f y(L)

o=

F ..:..(L___,.
- -<_::;o>:. . l
6

+CL 3 + AL"' + BL + D

Ely"(L) = - F( L - .to}+ 6CL

+ 2A .

Values of A and C are

FxQ(L - xo) (x 0

2L)

= F (L- xo)(2L + 2Lxo- xJ)


1

4U

12L '

Thus.

_
Y( X ) -

I [

F (.X 6

. ) 31_I ( .X.

.X(I

Fxo( L - xo)(x0

4Ll

X(l

)(.:..
2L
.:...,...+
.:..,..::2..:.
L....:
x0::.....-.....:.:x!!:
J)::_
x
+ _F..:..(L_ - _x..::o:.:
12U

2L)x 2 ]

A beam in a building is horirontally fixed at both ends. The si2c a nd di<Uibution o l


load Ott the be.u11 dttemtine its deftec~ion 81 ' 'ariO<" points: the mort load at o partcullir
pcnm oo the beam. the greater the deOeotKJn at !hot point. We are asLed to det emtin.:
the difTen:nce between dcfla.1JOn< at the midpoint of the beam when a given tolill lo.~d"
~enl) di~ributcd along the beam as opposed to \"\hen it is concentr~v..ed at the rnidpoim
ohheheam.
sc I 11 ' ' Let uHuppose tba11helength of the beam is L and tho! IO(alload is F > 0 .
When thil. load is evenly di.wibtned along the be-.un. differential eqwtion 5.8 become~

d' _v

-ffi

dx' =
~UbJCC t

to the boundary conditions

>(0)

= y'(O) = 0 = y(L) = y' (L).

Four intcgrutiont. or 1he difl"erentinl equation give

~'
y(x) = EI I ( - F~L

+ Ax'+

Bx

+ C.~ + D)

The boundary conditions require

0 = / y(O) = D ,
U = E l v' (O)

= C.

FL3
0 = El y(L) = - 24

+ AL 1 + B L2 + CL + D,

F L2
0 = Ely'(L) = - - 6

+ JA L2 + 2BL +C.

Th=givc A= F /12and 8 = - FL/24.SJJthat

\'(X) = -

EI

Fx
- 24L

FLx )
+ -Fx
- 12
2~

Deflection at the midpoint of the beam is y(L/ 2) = - FL3 / (384E I ). When F is


concentrated at the midpoint of the beam dill'cn:otial equation 5.8 become

Four intcgr.ations ghc

y(x) =

~ 1 [ - : (x -

L /2) h(x - L /2)

+ Ax 3 + B.r2 + Cx + D].

The boundary conditions require

0 = / y (O) = D .
0= / y'(O) =C.

0 = Ely(L) =

0 = Ell(!,.)=-

F(L - L/2) 2
2

+ 3AL 2 + 28L + C.

These give A = F / 12 and B = - F L / 16, so that


y(x) = -

El

F
- -(x - L/2) 31z(x - L/2)
6

FLx
+Fx
-- -] .
12
16

Deflection atlhe midpoint is y(L/2) = -Ff->/( 192 / ). 'The mtio of concentrntcd


deflection to distributed deflection at the midpoi nt is

- FL'/(192/)
= 2.
- FL '/(384/)

EXERCI SES 6 .4

J. Find dcRcctions of a unifoml beam with ruas.s m und length L when


both cuds arc simply supported.

+ II .

tal tlnirorm load of 1000 N (including its weight)- and


a conccmrdJCd load or 1500 ' at its ccnue. Al>SURling

2. Rtpcllt I!x~i'ci~o: I if both cllil$ nrc fixcJ hori2ontally.

3. Repeat Excn:isc I if the lcl"t end x 0 is fixed horizonlaJiy ttnd the


right end is froc.

4. RCI)Cilt Exercise I if the left end is fixed horitontBI!y nnJ the ri,g;ht
end Ls simply ~uppcrtcd.
5. A <xmc.~n1nl1cd feu-ex of F m:wtons is applied crtically tlownward
at 1hc midpoint of a uniform bt:.um of k:nglh L. Both cnc.l::. or the bc;.uh
arc buill in horiLOntally. IfF is .o large that the weight or the lle<un is
negligible in conlparisoo. fond dellectioos of the beant

(a) A si mply~supponcd beam or len:th 4 m is to carry a to-

= trf, u~ Exercise 10 to lind delleclions ofrhe beam.

(b) If safety codes do not pennit dcRceti<>ns to exceed lf360 of


Span.., is the beam acccplable?

12. Find deRe<:tions of the beam in Figure 5.9 if the 5Q kg block is on


Lhe rigtu hal r or the ~--arn. Is 1he deHection at Lhe rigtu end larger or
smaller than in Figure 5.<n

6. Repeat Exercic;c 5 if the left end of the beam is built in horizonrally


and the right end is free. Is the bca.m straight ror x > Lj2't

"' 13. Find deflections or 1he beam in Figure 5.9 if Lhe 50 kg block is
cenl.J'ed on Lhe beam. HO\\' d01..>s the deflection at the right end compate
1..0 l.h:lt i.n Figure 5.9 and Exercise 12'?

7. Repeat Exercise 5 if both ends of the beam are slmpl) supported.

* 8. Repest Ext:rci.se 5 if 1he n\3S~ m of the bc:im is taken into account.


* 9. Rcpc2t Exercise 6 if the 1:1U$S m of the beam is l:~kcn i.nto account
* 10. Rcpc..:ll Exercise 7 irthc mass m of the beam is taken intoaccouJtt.

J4. A conccnt.n.\ tcd for.c.c

F is applied vcr1ic-.a.Hy clO\VJl\llo'W"d utlbc right

end of a uniform beam of length l,.. The lcfl end of the bc;wn is built
in hori.zontaJiy and the right end is free. If Lhc weight of the beam
is negligible in comparison 10 F . find dcficclions of 11:'-l.C bc'..un. I lim:
Place Fat a point ..t'<J to the lef'l or X = L. solve fordctlections. and
take the limit as ..\'o """"" L -.

15.5 An Introduction to Separable Differential Equations


A differen tial equation is an equation that contains derivat ives of some unknown r1mcrion: lhe
equation must be solved for all functions that sulisfy it. Because differential cquatjons arise
in so many engineering problems, both elememary .oU
. ld advanced, we discuss what are called
separable differential equatioos here. A full treatnJent of differential equatioos is taken up in
Chapter 15.
Whe n a d iffere ntia] equation for y as a function of x c.ontains d y Jdx , but no higher o rder
derivatives, it is called a first-o rder differential equation. '!be vast rnajoriry of such differential

cqualions can be cxprc.sscd in the fonn

dv
- = F (x.y);
dx

(5. 13 )

thai is. they c-An be solved fO< dyj dx in 1enns of x and y. Examples are

dy
dx

(I - v) -

3x2

dl'

....:.. + 2x y =

and

dX

4x

Each of these can be sol ..ed for dyf dx.

dy
3x 2
=
dx
I - y

d)

- = 4x - 2xy.
dx

and

Differential equation 5.13 is said to be separable if it Ciln be c. pressed in the fonu

dy

1'>'1 (x )

= N(y)

dx

(5.1 41

that is, d yfd .f is a function of x diided by a function of y . Both of the above examples are
scparnble. The tirst is already in this fom~: the second can be put so,

-dy
= 4x dx

= 2x(2 -

2xy

y)

2t
-~-

2-y
Wl1a1 i~ equi\alcm 10 cqua1ion 5.1 4 i~ to >3Y thai 5.13 is :separable if it can be cxpre~scd in the
fom1
N( y )dy
M (x) d x.
(5.151

\Vhen a ditTcrcn1ial equation is wriucn in this way, it is said to be separated

scparotcd in the

>e11.>e that x- and y-variables appear on op~ite sides ofthe equation. For a >eparated equation
we can write 1herefore that

dy
N(y ) dx

M(x) .

(5.16)

and if we integnne bnlh side~~: wilh respect to x. we have

I
I

dy
N(y) -dx
dx

M (x)dx +C.

c5.1 71

Cancellation of dill'crcntials on the left leach to the solutions

What we

m~an

N(y)dy

by saying that 5.18

=I

represenl~

M(x)dx +C.

(5 . 1~)

solutions for 5. 14 is that any function defined

implicitly by 5.18 is a solution of 5. 14. A word of warning is appropriate here. looking at

equation 5.18 in isolation, it might appearthal we have imegrmed the left side of equa1ion 5. t5
with respect to y and the right side with respect to x. TI1is is not true. We do not differemiate
one side of an equation with respec.L to one variable and the others ide with respect to a different
variable. Why then would we expect to be able to integrate different sides of an equation with

respect to different variables'? What we did was rewrite5.15 in form5.16 and iotegrate both sides
of this equation wi1h respect to x . Caocellations of differentials led to 5.18. Thus, allhough we
did not integrate the lefl side of equation 5. 15 with respect toy and Lhe right side with respect
to x to get equation 5. 18, we can now imerpre.t 5. 18 in this way.

For instance. in the firM c.~~;a mplc above. we write

According 10 equation 5.18. solutions arc defined implicitly by

=
We can find explicit solutions by solvi ng the equation for y in lcmlS of x . Multiplying b y - 2

expresses the equation as a q uadmtic in y :

+ 2(x 3 + C)

= 0.

y = 2 ::1: j 4 - 8(.,> + C ) = I ::1:

JI -

y2

2y

Titerefore

2(xl + C ) .

2
Explicit so lutions of the d ifferential equation are therefore

y(.r)

= 1+

J1- 2(xl +C)

and

y(x )

J1 - 2(xl

C) ,

provided expressions on the right are indeed fu nctions of x. O nce C is dctcmlincd, this will
be tnoe only for cenain \'lllues of x . For e xample. <uppose we require the solution of the
diffcrcmial equation thm satisfies the extra condition y(O) = 3 . The second function y(x ) =
II - 2(.t 3 + C ) cannot satisfy this condition because )' cannot be greater than I . If we

substilutc .r

= 0 and y = 3 into the other function~

3 = I+ J1 - 2C ,
and this requires C

is

=- 3/ 2. Thus.thesolutionofthcdifferemial equation for which y(O) = 3


y(x) = I + J 1- 2(xJ - 3/ 2) = I + J4- 2.r J .

Sinc.c 4 - 2r 3 must be nonnegative for this function to be defined. the solution is valid only on
rhe imerval }l ~ 2 1/3. In fan, because rhederivati\'C of this function is not defined a1 x = 2 1/S.
we should consider only x < 2 1/3.
We now consider fou r problems that gi\'e rise to separable differential equations.

I EXAMPLE 6.11
An ore sample contains. along with various impurities. an amounl Ao of rddioacthe material.
say. uranium. Disintegrations graduaUy reduce t.h_is amount of uranium . Experiments have _led

to the foiJowing law uf radiuacrirc disimc:gration: T11c time rate of change of the amount of
r.ldioacti\e material is: proponional at any ins1ant to the amount of radioatti\'e matt:>rial presenr
at that time. Find the amoum of uranium io the sample as a function of time.

SOLUTION lf we let A (I) be the amount of uranium in the sample at any time I , then the
law o f radioacti\c disintegration states that

dA

dt

= kA,

(5.19)

where k is a constant. Since A is decreasing, dA / dt and hence k must be negmi,e. If we


choose t = 0 wheo1he amouot of uraoium is A 0 , then the differenJjaJ equation must be solved
fo r A (1) subject to the initial cond ition A (0) = Ao . The d ifferential equation is separable:
I

AdA = kdt,

and solutions arc therefore defined implic.itly by

>A =

kdl

ln iAI= ki + C.

We can omit the absolute values since A is always positive, in which case the initial condition
requires In Ao = 0 + C . Thus.
In

A = kl

+ In A o.

To find A (I) explicitly. we take exponentials o n both sides of the equation:

The amount of uranium therefore decreases ~ponentiall y in time. To find k we need to k.now
A at one additional time. For example, if \Ve know that o ne ten~ millionth of 1% of the originaJ
amoum of uranium decays in 6.5 years. then

0.999999999 A0 = A0 em12.
If we solve this fork . we obtain
2
k = - In (0.999 999 '999) - -1.54 x 1013

10 ,

and therefore
A(t) = A oe-1.5-lx to ,_

The law of radiotlctive disintegrarion ha~ an imponam application in the dati ng or once.. Jiving

plants and animals . All living tissue contnins two isotopes ofcnrbon: C " (cnrbon- 14). whicb is

f"ddioactive. and C 11 (carbon- 12), which i~ s table. Jn living tis~ue. the nuio oft he ;.~mount o fC 14
to that of C 12 is l/ I 0 000 for all fragments of tile. tissue. When the tissue dies, however, the. ratio
changes due to the fact that no more carbon is produced, and the original c" present decays
rddioacLively into an elc;ment olher than C 12 . Thus. as the dead tissue; ages. the rdtio of c ~ to
C 12 decreases, aod by measuri og this r-Jtio, it is possible to predict how .long ago the tissue was

alhe. Suppose. for example. the present r&io of C u to C 12 in a sped men is 1/ 100 000; that is.
one te.nth that for a living tissue. Then 90% o.f the originaJamount of C 1.._ in the-specimen has
disintegrated.
If we let A(l ) be the amount of C " in tile specimen at timet, taking 1\ = Ao to be the
amount present in the living specimen at its death ( I = 0), then A = A 0 et' . To determine k , we
usc the fact that the half-life of C" is approximately 5550 years. (The lwlf-li{e of a radioactive
element is the time required for one-half an o~igina.l sample of the material to disintegrate.) For
cart>on- 14, this means that A is equal to Ao/2 when I = 5550; that is,

Whco we divide by Ao and take natural logariithms,


I

k = - In 2 = -0.000 125.
5550
ConsequenLiy, the amount of C 14 in the specimen of dead tissue al any Lime 1 is given by

A -_ A()e - o.ooo 12sr.

If T is lhe pr~nl time, when th:: anlount ofC 1.. is k.nO\\n to be I()!K or its onginal amount,
~nat

chis time
O.IAo

Ac:.<'-<> OOJIUT.

The solution of this eq11ation is

In 10
0.000 125

= tS400,

and we conclude chllt the tissue died alx>ut 18400 ye.tt$ <~go.

I E XAMPLE 6.12

\Vhen a hoc (t'r cold) object is placed in an environment thut hus a different lempcnuure. the

object cool< down cor heat< up). For example. when a hot cup of coffee is pi.teed on a cable
it rool~ down due to colder room temperncure. Suppo<e. forexan11Jie. cJw che cup of rofl'ee is
initially at cemperacure 95-c and the room sta)s a1 constantcempemcurt 20 C. Neww11's law
of cooli11~ scoce. that the race of change of che cemperoture of the coffee i proponional co che
difference bel\\'ef!n cempenuu"' of~ coffee and 1ha1 nf 1he room. Find che ccmpcracure of che
coffee a' a funccion or rin1c.
SOLl'llUI\

If T (I) dcnoc<> temperature of the coffee, then Ne" ton'> ltnv of cooling can be

Stated algebraically as

dT
= k(T - 20),

(5.20)

dt

where k < 0 isH COI1>Cmtt. It b negative becllusc T - 20 > 0 nnd tiT /tit < 0. According
10 equation 5.20, the coffee cools quickly at first lxcause T - 20 1~ hut-;c. bul more and more

slowly as T approoches 20. DitYerencial equation 5.20 for T(t) is separable:


I

:::--= dT = k dt.
T- 20
Consequently, solution~ arc defined implicitly by

T - 20

dT = f kdt

In IT

- 201

= kt +C.

We can drop absolute \&I~ ~inre cemperaturt of the eofYee is ne-er le. than 20c Exponentioting both sides of chis cquauon ghes

= eC,!t => T = 20 + D~ '.


If we choose 1 = 0 when cemperuturc or the coiYcc i> YS C. then 95 =
1

T- 20 = ..''+C
where D
20
D

= e C' .

D .., 7S. Thus. temperature of the coffee is


T(t ) = 20

+ 75et'.

To find k we need tOknow tcmpenuurc of the com~e ac one Other time. rorCX<IIIIpl e. if we knew
thHt tempenuurc dro1>ped I<> 50 C iu S min, then

so = 20 - 7~<11

k =

I
S In (2/ 5)

- - 0. 183.

Coffee tcm~-nuurt i>


T(t)

= 20- 75e-0 111' .

Wecan also cxpr<$$ T(t) in th< fonn


T (t)

= 20 + 7S<'rt/ SII(l/SJ = 20 + 75e'lll/ S)'' ' 1 = 20 + 75 (S2)''5

The plot o r these func tions in Figure 5.15 indicates chat coffee cemperatul'\! never reaches 20"C;
the graph is asymptotic lO Lhc li ne T = 20. However. temperature or the corrcc is within 5 of
20C in about I S mi n. and within 1 in about 24 min.

364

C1'1:al)'er 5 The l1ldtlinit.:: lmegr-al:tnd lhe Antideri":uive

Temperature uf <."'il'cc

100

60

40
20 ------ --------- --- -------- --- -------- -0

10

20

15

25

Admiuedly, Newton's law of cooling is an approximation to what really happens physically. It


assumes tht there is no temperature variatioo\ withi n the cup of coffee, and that room temperature
remains constaoJL Idealistic as this is, it is a g ood starting pos ition for further uiscussions.

[ EXAMPLE 6.13

Figure 5.16 shows a liquid contai ner with n hole iJI its bottom. It is imcresting, an\) p<;rhilp~
surprising, to find that the speed at which liquid exits through the hole is independent of the
shape of the container: it depend~ only on the depth of liquid. We show this here. and then
di~over a technique for finding the depth or liquid in the container as a function or ti me.

Surt':xc orliquid
Thin

l~ytr

of liquid

Hole
J[,ve consider a thinsurface layerof the liquid. then during a small interval of time. the depth
of liquid in the comainer drops by an amoun t equal to the thickness of the layer. Simul!neously.
a volume of liquid equal to that in the layer exits through the hole nnd) in so doing, causes the
y.ravitational potential cncrt:Y of the layer. due to its elevated position relative LO the hole. to
be convened into kinetic energy or the liquid p~tssi ng through the ho le. Suppose we let u be
the speed at which the liquid leaves the con1aincr when the depth of liquid is y . The IXllential

energy o f the h1yer relative to the bottom o f the container is


(mi\SS of lnycr)(g)(y),
where. g = 9.81. The kjnetic energy oran equal amount or liqujd as it leaves the comaine.r is

I
- (mass oflayer)v2 .
2

5.5 An lnlnwhlction to Sepanble D11feren1iall!qu:uions

365

When we equa1e Lhese energies, and cancel1he mass of 1he layer. we ob1ain

J l
-2 v =
In other words, Lhc speed

aL which

sv.
.

liquid ex its through the hok is

v=

/2iY.

(~ .21)

This result is k.J1own as Torricelli's Jaw. It results from an idealized situation in which all
po1emial energy is convened inlo kinelic energy. experience sugges1s that exit speed depends
on o1her factors as well - the size of the hole. for one. Wmer leaves more slowly through a
small hole than through a large one. It is often ;lSsumed that v is somewhal less than ./'fiiY,
am! eqmlLio n 5.21 is replaced by

v = c./iiY.

(5.22)

where 0 < c < l is a constant called 1he discharse coefficient. We call 5.22 the modified
Torricelli law. We usc it to fi nd a diffcrcnnial e<1ua1ion salisficd by deplh of liquid in 1he
container. Suppose the area of lhe surface oflhe liquid is a function of depth y deno1ed by
A(y), and V(l) is the volume of liquid in the coma iner at any t ime 1. Since depth of liquid
changes at rate dy / dl. tbe rote at which the ''<>lume of liquid in the con~ai ncr changes i.s

dV
dy
"' A (1) - .
dl
. dl
Botl1 d V f dt and dy fdt are oegotive since V andy oredecreasiog. On the otber hand, the. rate
at which l iquid exi1s through the hole is 1he product a" or the area a or 1he hole and ex i1speed
v. It follows !hen !ha t
dy
(5.23)
A(y) = - nv = - t1Cv2gy.

dr

Once the shape of the container is spocified. then A (y) is known, and this equal ion becomes a
d ifferen1ial equal ion lilr y (r) . Solve for deplh when the comaine r is a right c ircular cy linder or
radius r and height !J with vcn.ical axis. Dctcrrnjnc how long it takes n full tank Lo empty.

SOLUTION When Lhe colllainer is a right ci rcular cylinder wilh radius r , then A (y) =

IT r 2

in which case dif!'ercmial equation 5.23 bccon1es


2 dy

lfr -

dr

- {IC..,r2gy

d)'

..fi .

.,Jrgac d
I'

JTr 2

a separated differential equation. Solutions are defined implicitly by

f .JY
_I

dy "'

.j'igac dt
nr2

2../Y "' -

.Jiiac
nr2

+ D.

If tltc cylinder b originally full and liquid cxil> 'tarling attimc I "' 0. then 2..//i"' D , and

.j'igac r

-'--..::..,rrr2

+ 2 v"hn

..jgac1 )
,. "' ( vr.
h - -=-.=---...,.
"'
fin r 2

It is now a simple. maucr Lo dctcrm.inc 110\V loog the cyl.indcr takes Lo empty. Sccting y(/) = 0
and solving for t gives

This was a particularly simple example in tha.t the cross~sectional area A (y) of the container
was constant. Containers with variable cross-sections are discussed in the exercises.

I EXAMPLE 5., 4
A umk ori~i n a lly contains 1000 L of water. in which 5 kg ofsah has been dissolved.
(a) If a brine mixture containing 2 kg of salt for each 100 L of solution is poured into the
talk at 10 mUs, find the amounr of StlJt in the mnk as a function of time.
(b) I f m the same time brine is being added, the mixlUrc in the tank i~ being drawn off at
10 1nUs, find che mnount or ~alt in the cank as a fu lction of ti me. Assume that the
mixture is stirred constantly.

SOLUTION
(a) Suppose we let S(l ) reprc:sem the number of grams of salt in the tank m time 1. If
we choose tirne I = 0 at the i nst~:lllt the brine mixture begins entering the original
solution. then S(O) = 5000. Since 10 mL M mixture cotters the tank each second.
and each mill ilitre contains 0.02 g of :;;tit, it follows that 0.2 g of salt emers the tank
each second. Consequently, after 1 seconds, 0. 21 gr-oms of salt has been adde.d to the
rank, and the tOtlll amoum of salt in the solution is. in grams.
S(l) = 5000

+ 0.21.

(b) Once again we let S(l) represent the number of grams of salt in the tank at time t .
Its derivative d S/ d t . the rotc of ch.ange of S(I). is the difference between how fast
sah is being added to the tank in the brine and how fast salt is being removed a:; the
mjxturc is rc.rnovcd.

~; = {rate !;>Ill added 1- rrate salt leaves I


As in part (a), sait is being added at the constant rate of0.2 g/s. The nue at which salt
leaves the tank, on the other hand. is not constant; it depe.nds on the concentration of
suh in the tauk. Since the tank always c(mtains 106 rnL or solution. the ooncentnllion
of salt. in the I'Qiution at time 1 is. in grams per mi llilirre, SI I0 6 where S = S (I) is
the amount of salt in the tank !ll that time. As solution is being drawn off' at the. rate
of 10 mLis. the rate at which salt Jeaves Lhe tank is, in gwm.s t>cr scct.11x.l.

-106 (JO) lOS .


Consequently.

dS
dl

I
S
20000 -S
= --- =
I~

100000

To find S(t) we must solve this ditTerential equation subject to the condition that
S(O)
5000. The differential equation is separable:

20000 -

I
dS = __,.:...._.,. dt.
100000

Solutions are defined implicitly by

- In 120000 - Sl -

100000

+c.

Let us solve for S before evaluaLing the consLanL of imcgraLion. Multiplication by


- I and exponenti ation give

120000 - Sl = e - C - s/ 100000

Be-cause e- c js an unkoowo constant, we simplify matters by setti ng


in \Vhich case

D = e- c,

S(l) = 20000 - D e- lf iOOr:n:J.


MOjiUII.I8Q, Amoont u(
sah ;. lank

The initial condition 5(0) = 5000 requires 5000 = 20000 - D ~ D = 15000,


and therefore the number of grams of salt in the Lank i:.

5(1}

20 000 -

J5000e

I/ IOOIJOO.

J\ graph of this function is shown in Figure 5.17. h is asymptotic to the lineS =

200000

400000

20000. After u very long time. the amount of salt in lhc tank levels o ff so that il';
concentmtion is 20 000/ 1000 000 = 0.02 g/mL. 'T'his, as mighl be expecled. is the

concentr.ttion of the incoming solution.

This project concerns lhc role of cvaponuio:n of wa1er from a hemispherical lank, open on
the top. Al1 vertical cro~ sections of d1c tank arc sern icirclcs with radius r metres. one
of which is shown in Figure 5.18. The Lank i~ o riginally full ofwa1er and 1he problem is
10 determine how long i11akcs for 1hc wa1er 10 eomple1ely evaporate.

....

of water from

:.1

hemu:pheric:al

-r

SOLUTION We must make some kind of assumption aboul 1he ra1e m which wmer
t\apurates. Since t\'aporation lakt:,:) place at the surface uf tlte water. it seems reasonable.
to assume that evaporation is p_roportional to the surface area of the water at any g.iven
time. If V (I) denolcs 1he volume o f waler iJllhe tank a11ime 1. evaponuion is represcnled
by the derivative d V fdt , 1he time rale of c hange o f the volume of water in lhe lank. If
A (I) b the surface area of the water al thnc f. then we have

dV

= kA .
dt
where k < 0 is the constant of proportionality. h is neg:a1i.ve because A is posit i.ve and
dV / ell is negative. Essentially our problcnn is to solve lhis dilferemial equation for V (I)
and determine when V = 0. An immediate d ifficulty is thal the equation comains three
variables. r. A. and V ; one of which must be elimjmued.
Cert.ain ly. t must remain. so either A or V must go. Jt would seem that we need a
functional relationship between V and A. h is easy to see that the area of the surface of
the water m any instant is 1r x 2 where

x is the radius of the swface. What can we say

abom the volume V of a spherical segmenr/ In many refere nces we find the following
fonnula for such a volume,

Tr

"2

V= T(3r- II),
where r is the rudius of the sphere and II is the depth of water. (You may recall that we
used this formula in Section 4.1. We will verify it in Seetion 7.2.) With 1r = r + y. we
can write that

Since the equa1ion of the semicircle is x 2 + )'2 == r1 we may a lso e'<press A in 1erm.~ of

y,
A

Tr (r 2

- yl).

These twocqua tionsdctcmtinc V in tcmtsof A, bllltofindc~a<tlyhowwould be difficult.


Instead. notice that if we substitute hO!h of them into the differential equation. we obtain

zdY) = krr(r z -

"- ( 3r 2dy
- - 3y 3
dl
dl

y 1).

When we simplify this equation, the result is

dy
dt = k.
How simple. We now have a differential equation for y as a functio n of 1 . Integration
kt + C . If we assume tltat evapomtion begins at time 1 = 0 when y = 0. we
gives y
must set C
0, and therefore y = kl. Water hascompletely evaporated when y = - r ,
and this occurs at time 1 = - r/ k. This is the required time. It is known c~plicidy when
k i s known. For ins1ance. if measuremcms indit-a1e 1ha1 the water level in lhe lank: drops
by I% of the mdius in 4 days, then

- -r

roo

4k

k=

400

It follows tltcn that the tank empties in

-r ( -

~)

= 400days.

EXERCISES 5 .5

I. Dm:tcriu in a culture incn:~ at a rate proportion.o.J to the number


pcc...o.cnt. If Lhc original number increases by 25% in 2 h. when will it
dcblc?
2. If dte number or bacteria in a culture; c.louble:.. in 3 h. when will it
lriplc7
3. If one half of a ~:trf'4)1 e 01 radio.w:tive suiJs.tance d3.)'$ in 15 d.ay.s,
how tons dou it t:a..kc for 90% or the a.umplc to decuy?

6. Suppose the unJOUill of a drug injcctc:d lnto the body c.kcrca.,.'<;,s uta
nne propooional to the anlOWu >till present 1r a dose doct.:a>c> by 5~(;
rnlhe fi~l h our, when w'il lrt clccreatc toonc-halfil~ orig_in:d :irnoun11
7. '' sugar cube I em on c:K'h side is dropped into :a cup of coffee.
lf the .SUg.tr dls.solvc$ in :.ouch u way thtU the C'Ubc aiYo'a)S rcrnuin5 a
cube. compare lhc tirtlCS ror the c..-ubc to c..-untplctcly tlhsoi\"C under the
following condihons::

h:>lf-ti[c?

(a) Dissolving occurs a l a rate proportional lO the surl"ace area


of the remaining t ube: and

5. After 4 half-lives or a radioactive substance. what percentage of the


originaJ amount remains?

(b) Dissolving oc.-curs al a rate prop<>l1iona.J lo the amounl of


sugar remaining.

4. If 1.0% of 11 satnplt:: of radioactive mtrlt::riaJ decays in 3 S. whaL is iLS

5.S An ln!t\kluctiQO Lu ScparJblt DHJc::rc:mial Equations

8. Solve Exercise 7 if the sugar is nOl in the lOon ora cube. but ra11-.er
in free f()(m from 1he sugar bowl. Assume 1ha1the sugar conl'isls or 11
spherical par1id es each or radius r Q em.

9. An analys is ol' a sample or fossil re mains shows Lh:u it contains only


1..51 % of the original C 14 in Lhe li\'ing creature. When did the creature
die?

lO. lf a ros.sililL'd creature died I 00 (X)() )'Cars ago. what paccntagc or


IA re mains?

309

* 2{1. The \\a l.er trough in the figure be-low i' 4 m long. Its cross--sec-tion
is an i"'SCeles tri :.mgle with a ha l f- n~tre base and a ha lf-mene allitude.
W:~ter leah out [hrough a OOie of area I cm2 in the bon om wil h speed
in me1n..--s per second gi~,oe o by v
JsD/ 1. where D (in ntctre~) is
the dcptb or wah:r in the !rough, and g > 0 is tbc ::lcccleration due to
gravity. This ls the modified Torricclll law 5.22 with c = l/2. Find
how long a fulltJrougb tokes to empty.

the Original C

ll. 111e amoum or a drug such as penici !lin injected imo the body is
used up a1 a nne proponional 10 the arnoum still pre~m. If a do~
decreases by 51Po in the first hour, when does il decrease to onehalr its
original amount?

12. Glucose is administered i!llnl\'Cnously to the bloodstream at a constant ral.c or R units per UJ.l .it li.n"K:. .As the glucose is uddcd. il is
converted by lhc body iot.o oLber subswm:~ at n .rate pro1xmional to the
amonnl of ghJoosc il1 the blood at thal lime. Show 1 1 ~1 the amount of
glucose in Lhc blood a~ a function of li me 1 is given by

where k is a constant and Co is the amnunt a1 lime t = 0 when


t.he imr:wenous feeding is initiated. Dra'v a groph of this runction for
C0 < R/ k and C 0 > R/ k .

*
*

1.3. Prove tbut iJ a quantity dcc('CUSCSat a rotc proportional to it~ present


amount. and if its pcrccmagc dccrc.asc in some interval of time is i ~.
Lhcn i L~ pcrcc::magc doc.rca.M: in any imcrval of Lime ol'thc same length
isi %.
14. W3tcrat tcmpc:rawrc 90" C is plact.'tl in a mo!ll at c.o nstant tc::mpt."f"d
lure 10C. Ne'"'ton's law of cooling st.al.es that the time rate ofchange of
the temperature T of the water is proportional to the ditfereoce between
T llnd the ternpcrarure of the environment:

dT

dt "'

+ 21. A spring of negligible nu'l.SS and closticiry c-OnStllnl k > 0 is attacbcd loa wu.tl al one c11d und u 1:nass !If at the other (C.gurc bcJow).
TI1c mass is free to slide horizontally along a fricL.i onlcss surface. If
x = 0 is takco as the p!.)S.itiun of M when the ll-Pring is wtstreu..:hed. and
M is set into JnDLion. the dill'erential equation describing the position
x(t) of M is
k

- -x.
M

If motion is initiated by i.mpartingu spocd V(J in the positive .x-dircction


to A1 at position .t = 0. 1nd the \'Chx::ily of M as a functionofpo~i lion.
lfilll: E:~pre..o;.s d 2 X f dt 1 as fC11Jows. and then separate Lhe differential
equation
dv
dvdx
dv

dt

= v- .
d.r

k(T- 20).

where k is a COit.slaOI. ]f the wa1er '-:ools tO 60&C in 40 min, lind T as


a fm1<:1io.n of 1 .

* 15.

d de

A thcrn"KKneter rem.ling 2S'~ C is taken outside where the tempetawill it read


ture is -20"'C. If the reading drops LOO"'C in 4 min,

Whc'

.r =O

* 21.

When a mas-s m falls under 1he influence of graYily alone, it. experiences a.n occeiC<..ration J Zr j d1 1 dcS<:ribcd by

- 19' C?
"' Ui. A boy lives 6 km t'rom school. He decides to walk to school al
a speed ihut is always proporlionu.lto lhc squan: of his distance from
the ~boot. If he is b alfw~y to scl)OOI aft..:r one IJOur, r.m.l his di:aancc
rrom sc.hool ;:n any time. How tons does i1uake him to reach school?
:1:

17. A lank has I00 L of solution conlilining 4 kg or s ug~lr. A mixlure


with 10 g of sugur per lit.re of solution is added at 200 mUmin. At

d 2r
Gm M
m"'-- d/2

where M is the mass of lhc eanh. G is a positive constant, and r is


the distance from m to the centre of the earth (figure below), lf m is
dtoppc:d from a helglu h above Lhc sutfoce of t.he cwth~ find lts ''Ciocity
when il strikes the canh. W11a1 is the nmimum auainabJcspccd of m?

the .sarm; Jjmc. 200 rn.l or wcll:slim:d mi~lu.n.: i:s n.:nto\'cd each minute.
FiOO rhe amoum of sugar in lhe tank as a runcrion of 1i1tle.

18. A tank in IJte form of an inverted right-circular cone of height H


and radjus R is fiJicd with waler. Waler escapes through a hole of
cross-sct'tionaJ area a 3l the vertex. Usc IJtc modjficd Torricclli l:'lw
5.22 to fi nd :'1 formub for the time the tank lakes to emply.
19. A container in the l"onn of an inverted rightcirrular cone ol"radius
4 em and height I0 em is full ol" water. Water evaporates from the
surface at :'1 rale proportional to the :ue a of lhe surfat'C. If lhe water
level drops I tm in Lhe firsl 5 days. how long will it take for the water
to evaporate completely']

r2

-,h

23. P1&::es ofi<:c produced by !.ln!fligcntor ore in llw:: fonnof htdf di~k:s
(figure below wil.h dinlension.s ira ccnlimctrcs). J\ s lhc icc melts. lhc
rdliOori tl" l':.u.liul' to its 1hK:kncs.s rem ;~ ins consl;ml. It' the rate of change
of the volume or lhc picoc i" pM{'K'Irtiomll 10 illi: ~urfacc area, and iu
r:\dius is 1/.! em a ncr 10 .ni' \\her will it complcLcly meh?

whc~ and I ttrc conslllnlS depend ing on the ti'O"~t~scclion and n\ll
tcri:tl ofth~ board. A is the weiglnsupponod by the boy at x = 0. and
g > 0 is the ;u:c.:ch:raLivn due to g_m"i ty.

\
L

1 - I- 1 -I -

* 24.

As a Sl>hcrical mindrop fall~ Lhrough a cloud> its m:as.-; itlete.NM


ula rut.c pr<.lJXX'tional to the produce of i ~,S surfncc :m.~ ~nd its vcloci1y.
Assuming Lha.tlhe raindn.')f) bcgin:s with zero ro~d_i\IS.. filld ils rudius as a
fuocl ion of distance IUU.cn tJV'ough the elolKI.

=
= 0 to lind [(x) :JS func.1ionor .<. E. I. m . ~.

(a) Soh-c this e<~uation along with th..: coutlilion!!i J'(O)

When a mothb;dl is exposed to the a_ir, tl slowly tw~po ratcs. The


mothball remains sphcric:ul nt all times. tmd cvaporution is: propo11ional
to the ~urfaee area or the mothball a1any g_iven time. If ils r:Idius :.fler
one ye;~r is half its original radiu:s. when will it disop1:car COtl1pletc.l) 1

* 25.

.f(L/2)

and A.
(b) pj,ld a conditiorl that peunits tbc rm in~ t i oo of A .

28. U~t: stl!ady-sl31C ICIIll>craturc ;f in a n..'gion bounded by two cc)n


centric s phc.rcsol'radii l m and 2m m u~ts:uh;fy thc d i.m~rcntiOO C(lua tion

d 2T
2dT
+ - - =0,
d r2
r rlr

* 26.

The figure lxlow .sf101., ., an op<:;n cylinder lhut is al\liays kepi full
by lhc tap. A hok is to be drilled in tht side of the cylinder a1a point
where the strcanl ofw11.1cr" ill hit tht: ground as far from the cylinder a.s
POSSible. The higher the hole. the rnon: lime the wu(cr has to rl;!Uch the
ground. ;md thcrdorc the fanhcr from the ~y l in dcr it \\ill ~ach. On the
other hand, Lhc lower the hole, the ercatcr lit~ :oopccd that the water c~its
through the tlolc. Usc the modi lied TorrK."i.:Hi l.aw to lind the o pti_m um
JlQ!'j-ition taking both fa<:IOfS into 3('('0Unl.

,..,.here r is the radial dilotancc fn;,lm Ihe coutn1on <.:entre of lhe spheres.
ll' tcn\J)Cn(urc!<t Onlhc inner Unc.l outer spheres urc maimaincd UL I0 C
a.ud 20"C respccti,'CI)', find the tcmpcrulun: distributicm T(r) between
th:: sphc.rcs.

29. A room With \'Oiutn::: 100 ml initially contains 0.1% car\l(m dio:c:
ide. Beginning at time I ;;;;;:; 0. frt.':'l.hc::r air ctlnta.ining 0.05% t:arbon
dioxide OO\\ S into the room at S u13 /nlln. The v.ciJ-mi:<cd air in the
nK)m fl ow~ out 31 the s.1n1e rate. 11nd the amount of c:utxm dioxide
in the room as a rwlCtion of titnc. What is tttt: limit of lh-c run(tion as
1

Tap

-~-...~ y

_..eo?

30. It is liOil\Ciimes a.""unted !hut lhc dcn<i.ily /} of lhC almosphcrc is


rcla1ed 10 heigh! h abo,c sea level according to t he dfiCrcminl equation
tip

Bole
II

-~

...

.. ..

dh
where ~ > I and k > 0 nrc c on~>~ant".

~~.

(a) Show thai i f

'

* 27.

Three boys of the smuc height cany a board of length L and uni
form lll3SS JX~r unit length m horizontall) ~as shown in the tigurc thul
l'ollcw. s. The \\'c..i ghl of the lx)i,lfd c-.aUM-s il to lx:nc..l. anc.l its shape is
lhc satnc.on ciLhe-r ~i de of the middle boy. Between the fi~ L\VO boY!)-.
fot 0 :5 " :::; L J2. 1he. tJis:plac.-enlCBI of 1tt~ boortl frotn 1hc hotitOI\Ial
y f(.r) "'""'satisfy d~;;dilrcrcntial equation

(E I) d'y = Ax
d .t'!

"'8
- r

dcnsil)' Jt ,;c.-a &eve I, then

, ,= - "(S
) ,,
k - -1
8-

+PQ.

(b) If nil' pressure P a11d density are related by the equation


P = kp8 prove that

where P(l i~ air p re~surC at ~:.a lt!\el.


(c) Show thal thc clrcc thc height ofthc :Hlno sphctc i~

8Po

'I

2 .

Poi~

'

3 1. A tcnain chemical di ~ohc:~ in wulcr 1.11

;~ r~I.C

J2. Two $Ub~Wncc:; A and 8 1\:dellu rvrm a lhir\1 ~u~ncc C in wth


" way Ihat 2 sofA r<3CI with I 8 or 8 l O produce 3 g of
The role
a1 which Cis roronod is propoo1ioowl oo the amouniS of A and 8 sJIJI
p1'<:~cnl in the mixture. Pind the uruourll oJ' C ptcscnt in the mi>.turc
at: :1 funclio-n of time when 1hc ori,ginal amounls of A and R brought
l(liCiher al time 1 = 0 arc 20 g and 30 ~-respective!).

propottion.allo 1 ~

c.

product or the nmour.1 of undissol,cd chenlicaland the diffcrcn4.-c be


1weenconccmnuion in a saturated solution and the existing conccntru
ti011 i- Lhc soluLiot. A satur~t.tctl MJiutiun contains 25 g or ch<:mic.ul iu
100 n>L of solu1ion. 1f 50 a of chornia l ic :lddcd 10 200 ml of W"Jicr,

Jond a formula for lhe anlOUno of chconical dissoled as a function of


Lime. Druw its y.tph.

&UMMARV

The indefinile imegral of Hfunction f(x) is a r:.mily of function;. each of which has f(x) as
its firs1derivative. If we c11n lind one amiderivative of /(X ). then the indefinite imegral i< 1hm
amiderivative plus an arbitrary con,t<lllt. According 10 'l11corem ~.2. the indefinite ioue~:ral or a
>Um of two functions is Jhe sum of their indefinile integral>. and muliiplicalive cons1an1S may
be bypassed when findina indefinite iouegrals:

j t/(x) -

g(x)l dx

f(x )dx -

j cf(x)dx = c

g(x)d c:

f(x)dr .

The mosr imponant in1egm1ion fonnul:l is for powers of x .

Other integration formula> Omt llli$e from difTerenJiatioo> of trigonomeuic. im-erse uigonorncl
ric. exponential. lo:arithmic. and hyperoolic function' are li>tetl in equations 5..1.
In this chap1er we hove studied three ways to evnluale indefinite integrals. (Others will
follow in Chapter 8.) Fir~t. cJuc tu our expcnbc in 4.lincrcmiation. some amiderhdthc~ are
immedialely recogni7allle. Second. sometimes an amiderivative can be guessed to within a
mulliplicative conswn1, and this constanl can then be adju<tcd. Third. a change of \':triable eM
oflcn replace a complex imegration J>roblcm wilh a simpler one.
Integration play< a fundllmental role in kinematiC< Sonce , -eloclly is the dem-ati,e of
po>ition. and accelenuion is the dcri-ative of velocity. it follows that position is the indefinite
integral of -elocity. arl<l \clocily i> lhe indefinite intearal of a<<elcmtion.

u(r)

/ (l(r)

eft

ami

,1 (1)

= Ju(t )

eft.

Many physical systems Me modelled by differential cquaJions. and the solution of n dil~
ferential equation usually involves oue or more inte~ration>. The differential equation for the
deHeclion of a beam require.;; four inlegrarions., re~~:uhing in four arbitrary con..~ant< 1ha1 are
dctcnnined by two boundary corl<litioll.S at each end of the beam. The Dirac-&:ha function is
most effectin~: in repre..enting point forco. on be-Am~
FirstorderdifTerenti~l equations an: separable iftheycan bccxJ>n:ssed in the form N (y) tly =
M (x) dx . Solulionsare then deli nod implicitly by

N(y)dy =

M (x) d x.

KEY TERMS

In reviewiug this chapter. you should be able 10 define or discuss the following key terms:
lmegrand

Ant iderivative and indefinite integnl

Ramp limction
Boundary conditions

Change of variable

Simply supponcd
Differential equation
Separable differential equation

Boundary-value problem

First-order differential equation


REVIEW
EXERCISES

ln Exercises 1- 28 e'n.huuc the i1ldctin.itc imcgraL


I.

3.

J
J(lx' -

(3x 1 - 4x + 5)dx
2

3x + 7x') dx

j.Jx-2dx
j(JX- Jx)dx
9. J
(x +I <l.t

2.
4.

5.

6.

7.

8.

5)'1

11.
13.
15.

17,

19.
21.

23.
25.

27.

/sln

11.

3.t dx

f.rcosx
jxJI"+Xdx
J ~ d.r
J
JXJ'
J dx
/e-'- dx
"
2

(I

14.

d:x

16.

18.

.r)Z

I
,/X(2 +

e3- 5t

I v't f

X
4,t

X CO<h 5x

10.

dx

I(;,
I(:,-2jX)dx

+ 2x - ;, ) dx

J + 3x7)' <lx
JC'+jX 5) d.\.
J(fi - 15) dx
x'
fi
x(l

J.tl1-xdx
2

Jx'(t- 2x'fdx
J ~tlx
2- x
/ <2+-/X) dx
7

T and the tcmpcratw-c of the cnvi.J'Ourucnt. If d1c watc cools to 50C


in 10 min, fi11d T as a funcLion of 1 .
+< 31.. Find the curve)' = {(:'()for which

f"(:c)

x 2 + I, und lh al

p:.tSscs through t!hc point (L 1 L) with slope 4.

* 32.

Find Lit<: cwvc . - f(x) for which J"(x) P""SC> throogh tltc two poinL' (I. 4) and (- 1, -3) .

* 33.

12x 2 and tl>at

Find the curve." = f(x) for which f"(x) = l 4x 2


that is tangent to the line y 4x + 4 at ( 1. 8).

* 34.

A boy lives 6 km ftom school. lie

+ 6x. ancl

decides 10 walk 10 >ehool at a

speed that is alw.ays proponionalto the square root or his distance rrorn
the school. If he is halfway 10 school ancr I h. lind his dis1ancc frorn
school at any time. How long docs il lake him to reach school'/

* 35.

If a ball is lhrown q:rtically upward '''ilh a speed o r 30 m/s. how


high will it riic'1

* 36.

A stone is l hrown vcrticaJ ly downward over the edge of a bridge


50 m above a river. lfthc stone strikcs lhc \vatcr in 2.2 s. '"hul was its

ioiLiaJ speed?
(n

ExcrciS<:s 37-44 evaluate the imJcfinilc integral.

+ 37.

J/ + .rx
1

f
f
f

dx

"/;;=s;=i';;n;'=== d X

20. /sin4 xcosxdx

... 39.

Jxe- dx
J--dx
I

* 41.

+ 45. A gmp1l o(ll1caccclcnu..ion a(t ) of an object b shown in the figure


helow. Find ils velocity ll(t) and position x(r) in lhe thuc intcrvl
0 S 1 S IS il' v(O) = 0 = .r (O), and drnw gmph< of coch.

22.

24.

x=

5x lnx

clx

26.

c/x

211.

J Sx dx

3
d
I +h' X

* 43.

<eeh'

29. If the numbct of batLCI'ia in a cullutc ll'iples in 3 days. when will


it quadJ\lJ)Ie it~ Miginal number?

30. Water at temJ>emture 70C is placed outside where tempemture is


- 20C. Newtons law of cooling Mates that the time rate of c.hange of
the temperature T of the water is proportional to thedilTerente between

./4 + 3cosx

.:.;:(2' -+:..,A::;);_
4 dX
x6

-:-r,=:;:::;;:;= d X
xJ I + 31nx

Par.tbola
(5. 2 )

Stl'ajgJu line

10

15

* 42.

jsi1'13 xc~' .tdJ.'

44.

fr.anxdx

Review Exercises

*'

46. Find the cquatjon of the curve that passes lhrough lhe point ( I: I)
such tlt:~l the sJopc o f the tangent line ac any point (x ~ y) is half the
sq uare of the slope of the line from the origin to (.t. y).

47. l11e equation y = .r3

48. A container in the fo rLU of an invened righH:in:ular cone o ( r:1dius


6 ctu and height I5 em is fuJI o f water. Water cvaporo.~tes from tJJe
surface at a rate proportional to the area of the surf;lcc. If the water
level drops I em in lhc fi rst 6 days. bow long will it take for half the
w:ucr lO C\ '!tpOrmc?

+C

or

describes a faLUily cubics where C


represents the distance of ils hori7.onta1 point o( inflection atx.wc che
x-axis. Find the equation of the curve that passes through the point
(I> 1) and intersects each of tltese curvc..c;; at right angles.

373

*'

49. Find deflections o f the beam in Figure 5.9 if tbc 111ass of the block
is M kg and M is so large tJ1at i.he mass of the beam can be neglected
in comparison. Is the bcaLU s trajght for x > 5? Would you expect it

50. Repeat E.xercise 49 if the mass is on the right halr of the beam. Is
the beam s uaight fo r x < 5'? Would you cxpecl il to be'?

to be?

* 51.

Repc~t Exercise 49 if the mass is on the middle ha1J of the bcaiU.


Are 1bc ponj ons of the beam 0 < x < 5/2 and 15/2 < x < I 0
straight? Would yoo expect them 10 bc1

CHAPTER

The Definite Integral

Application Peview

Currents in circui ts are often monitored by a controller. The controller Lakes aclion whenever the

current strays signi6caody from its expected value. The current i in the left figure below differs
from its steady-state value signilicamly at time 10 but <loes so for a very shon time imerwtl. so
short pe-rhaps, that it might be de.e med acceptable.

Steady ~smte

current

to

'o

Onlhc other hand, the currem in the right figure moves only half as far from the steady-state
value as in the left figure. but doe.~ so fo r an extended period of time. This might be deemed

unacceptable.
THF. PROBI F.M
Devise a way to distingui sh mathematically. for purposes of the controller,
bct\veco very short~ but very abnoo11al bclla\ iour of a function. and Jong Lcrm. but less dramacic
1

changes from the nonn . (Sec Example 6.15 on page page40 1 for a solution.)
'----

There are two aspects to calculus: ditlerentiation and integration. We deah with diller.
emiation mu.l its applicatioos in Chapte rs 3 and 4. Integration in Chapter 5 was synonymous
wjth aotiditl"erentiation and the indefinite irlte-graJ. ln tltis chapter we .hwestigate a new type of
integral called the definire imegral. Before doi ng 1hat. we introduce sigma notation. a compac t
notation for sums of terms. particularly useful for de.t inite integraJs. In Section 6.2 we discuss
four problems that motivate the concept of the defini1e iruegrl. l n subsequent sections we develop the definition for the integral and dlscuss various ways to evaluate i t. Section 6.6 presents
an applicalion of the tlefinile integral- fi nding the average value of a funL~Iion- :md Chapter
7 dc\'clops a muJtitudc of physical applications. The dcfinilc integral i:). by dcfinitjon. very
different from the indefinite integral, yet the rwo arc intimatel y rela1ed through the fundamental
theorems of integrdl calcul us (Sections 6.4 and 6.5).

16.1 Sigma Notation


One of the most imporwnt nolatioos in calculus. sigma n otation. is used Lo represent a s um of
terms, aJI of which are sim ilar in fo rm. For example, Lhe six terms in the s um

-+-+- +- +- +1 + z2
I + 32
I + 42
I + 52
I + 62
I + 72
374

6.1

Sigma NUI.lliun

375

are all fom1ed io the same way: Each is an ioteger di,ided by I plus the square of the next
imeger. If k rcprcscnl.5 an integer, we cao say that every tcm1 hns the fom1 k/[1 + (k + 1)1 ):
the firsuerm is obtai ned by seuing k = I, the second by seuing k = 2, and so on. until k = 6.
As k /(1 + (k + 1) 2] represents each and e,cry term in the sum, we can describe the sum in
words by saying. "Assign k ink/[ I + (k + 1)2 ]the integer value> between I and 6. inclusi 'Ciy.
and add the resulting numbers together." The not,uion used to represent this statement is

Lr +(k+ t)2

k-t

TheE symbol is the Greek capital letter sigma, which in th.is case means "sum." Summed nre
expressions of the form k/ 1I + (k + 1) 2]. anclchc 'k = I" and "6" indicate that every imeger
from I 10 6 is subsliluted inlo k /I I + (k + I)'2] . We call k /[ I + (k + I) 2] che general term
of che su01, siJ><:e it represents each and every teollthereio. The leuer k is called che index of
sunnnacion or V"driable or s ummation. and I and 6 are called the lim liS or summation. Any
letter may be used to represent the index of summation; most commonly used '"" i, j, k, I, m,
and 11 . Here i.s another example:
u

L = sJ + 6
11

+ i' + s3 + 9' + ro' + 113 + 123 + 133 + 143 + 153 .

nsS

In :.urnming a large number of tenns. i1 is quite cuml>cr~omc: to write them all down. One
way around 1his: difficulty is to write the first few terms co indic:a1e the panem by which the temu;
arc fom1cd, three dots, aod tbe last term. For example. to indicate the sum of the cubes of the
positive integers less than or ecrualto 100. we write

where the dots indicate that all ntllllber:< between 53 and too' are to be filled in according to the
same pattern suggested. Obviously. it would be prefernble to express the sum in sigma nota cion.

which is compact and leaves no doubt as to the p-11cn1 by which tem1s arc formed.

I EXAMPLE 6 . ,
\\'rite e-clC'h of the following sums in sigma not;.ltion:

(a)

I
2J

(b)

.,fi

16

16

169

+ 3 4 + 4 5 + 5 6 + ' .. + ~
32

128

4096

+ J3 + A + ./5 + ... + JiO

SOLUIIO:\

(a) To delem\ine the pauem by which terms are fom1ed, it is often advantageous to write
values of the variable of summation above the terms. If we use i as the variable with
i = I corresponding to the tlrst term. we wJile
i ::l

i ::2

i:3

ia4

;. ?

16

169

-2 3 + -3 4 + -4 5 + 56
- + .. + 14 15

TI1c gcncml term is i 2 /l(i


I

+ l)(i + 2)]. and

16

169

-2 . 3 + ++ -5.+
.. +14-. = "C""
3. 4
4. 5
6
15
~ (i
/= I

(b) If 11 is chosen as !he in~e.< of summation and 11


rt= )

11=4

tt=S

tt= ?

16

32

64

128

40%

+ J4 +

.... l)(i

+ 2) .

= 2 to com~spond to the first rerm,

n= 2

J2 +

;z

+ + JIO=

j5

The rcprescmations for the sums in Example 6.1 in terms of sigma nooation are no! unique. ln
fact. there i~ an infinite number orrepre~em:uiort~ for each sum. Consider. for exam ple. the sum
repre$tolltd by

I f we write (')til son\e ()frhe. tenn~ in the surnma1ion. we fi1ld that

the same sum ~ts ohao in Example 6.1(b). This sum can also be represented by
6

2.1+6

,E.JTTI

;l nd

\Ve can transform any c.1nc of these representations i nto nny other by making a change of variable
of summation. For example. if in the summation of Example 6.1(b) we set i = 11 - I, ohen
n= i + l . and
2''+'
zi+.l
=

Ji+l'

.jii

For the limits. we find that i

= I when n =

2. tHld l = 9 when n = 10. It follmvs thnt

Similarly. the chm1ges j = n - 4 and m = 11

+ 9 tnmsform

iUlO

2j +6

j~2 .JT+ii

and

6. L Sigma Nol:ttion

317

I EXAMPLE 6.2
Change each oflhe fo llowingsurrumllio ns into representarions thm are initiated wiLh the integer I:
(a)

26

2/3

i2; i + I

i=-1

+ 2j + 5

102 /

_L

( b)

J=-3

sin (j

+ 5)

SOLUTION
(a) To i.niciate the summation tlt I , we want n = 1 when i = 4, so we set n = i - 3.
Then i = n + 3, and by subsLiHHion we ha\le
26

;2/3

L ;2+ i + I = L

(11

11= 1

t=-l

(b) lnlhis case, we set 11 =


101

.L,

}=-:~

j' + 2j + 5
sin (j + 5)

+ 3)2/3
+ 3)2 + (11 + 3) + 1 =
(n

23

.i + 4. Then

106
= .L
(II n:l

+ 3)2/3

.L "' + 711 + 13 .
n= l

j = n - 4, and

+ 2(11 - 4) + 5
sin (ro + I)

4)

(n

23

.L"
106

n;l

-611 + 13
sin(n + 1)

If we examine the results of Example 6 .2 a nd the summations immediately prei:cding this


e.xample, we soon come to (ealize that there js a very simpJe way to cha:nge variables. To
il lustntte. consider once again
10 ')n+l

.L -.fii .
n=2

Should we wish to iniliatc the :summation witl1 I rather dum 2. we lower bo1h limits b)' 1. To
compeosate, we replace each 11 io the general lenn by n + I; lhe result is
9

2 (11+1)+1

2'i+3

~Jii+l - ~ Jii+l'
Similarly. for slmpllcity in the summation
10 ( 11

tt= l

it would be advisable to lower each

11

+ 4) 2

e"~"'

in the generaltenn by 4. This can be done provided thm

we raise each limit by 4:


J()

142

")

" " (11


4)L- 1!11 +~

n :=l

"" ~L-

R"

11 eS

Eve-ry summation represen1ed in sigma not.uioo is of the ronn

.L" /(i),

(6. 1)

where m and n are inte.gers (n > m). and .f (i) is some fu nctioo of the index of sumJnation
i . In Example 6.2(a), .f(i) = i 213 j (i 2 + i + 1). m = 4. and 11 = 26; in Example 6.J(a),
.f(i) = i 2/ l (i + I) (i + 2) 1. m = I, and tt = 13 . The following properties o f sigma notation
are easily proved by writing ou1 each summation.

THE OREM 6 . 1

ff f (i) and g ( i) a~ fu1tctic:>rl8 of i. nnd m and


tl1cn
n

are p()Silivc it1tcgcrs such that u

L J(i) + L g(i);

m.

\ 6 .2<1)

"

L cf (i } = c L
if

:>

2:)JU) + g(i)J =
l=m

11

f(i)

(6.2h)

c is a constam independent of i .

Comp;,are Theorem 6. 1 with n ,cocem 5.2 itl Sectior'l 5. 1; notice the s imi1uri,il.ls tx:twccn

propenics of sum mati011S i11 $igma notation and those of indefinite integrals.
\Ve cmphasiGC thnt Sil!imu r1ot ution is $imply 11 conci se~ymholi~m used to represent t l sun
o f tem1s: it docs not evaluate l he sum. In the foUowitlg discussi011, we develol'> fom1ulas for
:,tuns chat pro'e useful in future wor k.
Summation
i represe;u~ che ~urn olf t he fi rst n pOSitive:: integers:

.L;'=

"

I > = I + 2+ 3+ 4 +

00

+ (11

- 1)

+ II.

I~ I

If we write the tenus on the right in reverse order. we have

2: i

= n + (n -

I)

+ (n

+ ... + 4 + 3 + 2 +

- 2)

I.

i =l

Addition of these t'vo equations gi,cs us

"

2 I> =

(11

+I)+

(11

+I) + (11 +I)+

000

+ (11 + I)+

(11

+I)=

1!(11

1).

i l

Con seq uentl y.

I:i =

+ I)

11(11

(6.3 )

1=1

This resul t can be used to develop formulas for 1hc sums ofthe squares. cubes. and so on. of the
I>O~itive intc&ttrs. To find the .sum of the :..quares of the ti rst
expansion and simplifica1ion

; 3 - (i - I ) 3

11

IX>:Sltive integers, we note thtu by

= 3i 2 - 3i +

for any integer i whaL<oever. It follows that


n

" 3
- c;- 1) 31 = I: (3i 2 I:
r;
,_,

3i

+ r) .

1=1

But if we wl'ite the left-hand side in full. we tll1d

L" [i 3 i=l

(i - l):'J = [13 - OJ] + (23 - 13)

+ [33 -

2:'1 +

... + [113 -

( 11 -

1)3).

6.1 S1gma Numtion

379

Most of these terms cnncel one another. lenving only 11 3 : thnt is.

L" (i 3 -

(i - I)'] = 11 3.

i=l

Thus,
II

113 =

L (3il -

3i

+ I)
(using 1bcorcm 6.1)

i =l

i=l

i=J

~
11 ( 11
=3 L..., o2 - 3

1=1

We can solve chis equarion for

+11

(using formubl 6.3).

L?= i 1 :
1

~ .,

I [ 3
L-J'=-n+
1=1

I)

311(11

I)

11(11

+ 1)(2n +

11=

t ;'

(I- 1) 4 = .a;3 - 6(1

(6.4)

A si o1ilar procedure beg,ioning with 1he iderHity /

I)

+ 4l-

I yields

= 112(11: 1) 2

(6.5)

i -1

These results tan also be established indcpenc.lenaly of one anoaher by mathematical i11duction.
(See Appendix A for proofs of formulas 6.3 nd 6.4 using this technique.)

EXERCI SES 6.1

in E.x.erciscs 1-10 express the swn in sigma notation. lnltiatc dlC swn
nwioo with the integer I.

tan 1

8. -

tan 2

Lan 3

tan 225

tan 4

+ - -1 + - -2 + - -1 + ... + :-:--=;;1
I+2
l +3
I +4
I + 225

> ,
I
I
I
9. 4 + 5 + 6 + I + - + - + .. + s 9'

zs

10. 0.9 + 0.99 + 0.999 + ... + 0.999 999999

I
2. 2

2 3
4
- ,... - + 4
8
16

S
32

+ ... +

10
-1024

In Exercises I 1-1 S verify by a change: of wriablc of ~~tnnnuuiOI'I 1lu11


the wo s u mm ation~ ;uc identical.

16
11
IS
199
+
+ - - + .. + = - - =
14 +15
15 +16
16 + 17
197+ 198

,.. ,.,

3. -

4. I + J2 + _.13 + 2 +

II.

./S + J6 + J7 + ,,ii + 3 + .. + 12 1

5. I + 2

+ -23 +

3k- k'

7. -

1 4

67

10 11
- +-58
9 1 2

14 15
4 14 4 15
+- + ... + .,..,..,--.,.-__
13 1 6
4 13416

" '
14.

2:: ..,.,
1=1)

,,

')II

.n 3JJ

23

I .

+9

JJ + m

.-=0

13. ~ ( - 1)" 112-+


1020

6i

,_ 1

2:: -./f+5
:CI

i1

L li-S
L 2-m- m

k 2

- - + .. + ::-:,..--,----:---:-o
2 3 4
23 4 5 .. .. 16

6. - 2 + 3- 4 +5 - 6 + 7 - 8 + ...

L
211 +I
"...
101

12.

!)

J9

') )

"'16(- 1) I

L-

... l ,

~ 729(~1 -

2) !

., + 8 . + 17

380

Cl~er 6

22l

n.e Definite InTegral


221)

15. "
' -:--o~ rZ - lOr
r = l$

u=IO

>1<

u2 -

29. Prove formula 6.5.

25
30. Is

L (j(i )g(i) l

[t,.,

C<]ual 1o

oa l

ln Exercises 16- 25 use lhc:orcm(,.J and formulas 6.3-6.5 1u ev.-.tualc

"

17. 2::: (2i' + 3j)

"

20.

L (4m .,,_,

2)

k= 2

"
2::: (11 + 5)(" -

21.

4)

Usc the t'ormuJa in Exercise 3t to sum the finite geometric series in


E:tcrciscs 32-35.

23. 2::: (1 3 - 31'>

n=IQ

., 24. 2:::

L::
,_, i<i - 3>'
II

<,'- 5}

" 22. 2:::

1- r

"

u=l

24

i =i

+ 32 '

2u

<k + 3)(k + 4)

.. 25. 2::: (i' +2i- 3)

26. Vriry T heorem 6.1.


"

27, Pinda rormulafor"

L- k(k + I)

, /lint:

k= l

~ - - -k(k + I} - k
k +I.

28. If [(x) is a ltutclion or X. de lined I'or all X . simplify Ute sum


2::: t/<i>-

.r<; -

i(; + 32 + (id + .. + I ~ 576


I

3 + i) -

27

+ 81

- ... - 19 683

34. 40(0.99) + 40(0.99) 1 + 40(0.99)' + .. . + 40(0.99) "


35 . .Jo:99 + 0.99 + (0.99}3'' + (0.99)' + . . . + (0.99) 10
36. PJO\'C lhai

*'

L j (i ) 5 L" IJ(i )J.


/;;;;1

i= l

37. Expccss dte following summation in sigma notation:

I
I
I
I
I
I
I
I + - - - - - + - + - - - -2
4
8
16
32
64
118

1)1.

i-1

S+

~ J). I -

l=n

~=s

+ ttr 2 + at 3 + + or" - 1

(a) If S. representS this swn. express S,, in sigulu notation.


a(l- r" )
fb) P
' rove that Sit =
.

19. 2::: (k1 - 3k' >

g(i}] '!

There is a fi rl>t tcrnt t l. artd every term thereafter is oblaincd by multi


plying: The prcooding Term by r (caiiOO lhc t.'Ommon rmio ).

J=

Sl ~ l

18.

a + ar

"

16. 2:::<3" + 2)

[t,.,

31 . A finite geomdric serits i:s a sum of 1crms of Lhe form

the sum.
12

J (i) ]

+ ... +

I
.
4096

6.2 The Need for the Definite Integr al


In this section we consider four inherentJy diffete11t problems: one on area. one on volume. one
on blood flow. and one on work; but we shall see that a common method of solution exists for all
four of them. This common then1e leads to the definition of the defi nite integral in Section 6.3 .

Problem 1
We first consider rhe problem of finding thearC<O A .in Figure 6.1 a. At present, we have formulas
for areas of very few geometric shapes - rectang les, t.riangles ~ polygons. and circles, and the
&11311;1#1.81

Are:a u11del' 01 Clll'\'e

)'

J = .f(.t)
)'

=:::,

""'""' )' =I (X)

r--

A 1 A2

.4

~,

Xt X2 -"i-1 X;

a= xo

~.
x11 - t

6.1 The Nood for W; De611ile tmegral

381

shape jJl Figure 6.la is not one of them. \Ve cao. however, 6Jld a n appro~imatioo to the area by
consuuctingreclangles in the follo\ving way. Be1ween a and b pick n - I poilw; x 1 x 2. x 3
. . . Xn-l on the x-axissuch 1har

a = Xo < X1 < -"'2 < <

Draw ,errical lines rhrough each of rhese

11 -

Xu - I

I poims

10

< x,~ = b.

inrersec< rhe curve y = f(x) and

form rec'tangles. as sllown iu F igure 6.1 b.


If A; deno1es <he area or <he i"' such ret~angle (i = I , . .. , n), lhen

A;

= (heigh! of recumgle)(widch ofreccangle)

f(x;)(x; - x;_,) .

The sum of the-se n reccangular areas is a n aJ>proxima liOn to the required area; thar is, A is
approximalely equal to

,,

,,

LA; = L /(.t;)(x; i:::::l

X;-1).

i:::::l

Jf we let the number of these recrangles: gee. lar,-ger and larger (and at the s:m1e time require each
to have smaller and smal ler width tltat eventually approaches zero)~ the approximation appears

10 ger better and bener. In fac1, we ex peer <hat

A = lim '"'A; = lim '"'f(x1)(x1


n-oo L..,;
i=l

n- oe L

x1_ , ).

(66)

i=l

Problem 2
If the area in Figure 6.1a is rotated around the ..\:.axis, it traces out a vo lwne V . This is
ccrlJiinly no1 a Sl.i:'lndan.l shape for which we ha,e a volume fom1 uJa. ~1nd we lhererorecnnsider
finding an approximationlo V. We to.kc the rectangles in Figure 6. I b (which approxi male 1hc
area) and rora1e Litem around Ihe x -axis (Figure 6.2). Each A; rraces out a disc of volume V1

(i = l, . .. , n}, where

V1 = (surt<>ce area of disc)(thickness of disc)


= lf(rndius of dic)2(tl>ickncss of disc)

An approAnnacionlo lhe rc.qu1rcd \v l ume is then

..

L v, = L: r.l/(.r,)j (x,lei

Xt- 1).

i:l

1\~uin we feel i111uhovcly thJt u~ the number of discs becomes larger and larger (nnd the width
oreach npproachcs lcro), the approximation becomes be ncr nud be ncr. and
II

li m "'V;

n-C"'o:: ~

D!ooJ
~MUfti)

Ct,,..,<'Cctiun of

blood,.,,.,

n...

II

= ,_oo
li m "'Jr!f(,r;)]!(x,
L.,

x,

tl

(Cl.7 I

hrl

Problem 3
When blood nnw< throollh u vein or ancry. it encounters resi'<lnce du.: to friclion with the walls
of the blood vessel and du~ to the vi:.<v>ity of the blocxl it>elf. A> a'"'"" the 'ciO<.;ty of the
blocxl i> not ~"OO>tant aero~~ <'ros>Seetion of the \C>S<:I; blood no'" more qui<LI> near the
centre or the ve,..cl than ncar its walls. h has been >h<>" ol that for lamonar blood flow in a 'essel
of circulr cl"ll"ction (Fi~urc 6 3), the vcloci1y of blood b ll"<n by
u

v (r) = c( R2

r 1) ,

0 ~ r < R,

where c > 0 is acon; ttull. R i>the radius oft he blood \'Cli.'iCI, and r isr.-dial distan<'<' measured
from lite centre of the ve.<sel. We wi,h 10 find the rate of blood nuw through the \<CS.cl. that is.
the I'Oiumo ol'bhw~ nowing lhrough lhc cross-section per unit time
H u were conSianl over the cro-section, then flow per unit 1ime would be the product of
v and the Cf<).\S';;ecCinnal nre.t Unfortunately. thi~ i-; not the case.. bJI we can ~till use the idea
tlmt flow i< vel oct I ) multiplied hy area. We divide the cros.<section into nngs wi1h mdti

0 = rl) < r 1 < r2 < < r,_ 1 < r" - R.


Over the "' nng. the \'Mation in v is small anJ v c.tn be appn~>im.ued hy u(r1). The RO\'
through the ;"' ring can therefore be appro<i nulled by

Ft = (nrea of ring)(\ elocity m outer rndius of ring)


2

= (1rr1

:rr;_,)v(r,).

An approximtuion to Ihe required Aow F is the sum of these F1 :


"

II

L F; = L (rrrJ- 1rr/.
-

1)to(r;).

1=1

and il ....,, re<onablc th,u as the number of rings increases, so docs the occutllcy of the
I$. \\t anticipate thai

appro'<imation: that

"' CJTr; -rrrf_,)t(rtl

lim "'F1 = ,._


lim, L
" oc. L
iel

(6.81

i=l

Problem 4
A spring is lixcd horizontally into a wall at one end, and the other end is free. Consider finding
the work to stretch the spring 3 em by pulling on its free end (Figure 6.4,.).

u
Unurerched
posirioll

l-3 -

o~xo

I
... -y-1'\... - ..... - -/!- .... , _ ...
-'-"J~J
Strerched
posiriun

Let us choose an x-u is po.o;ithe to the right with x = 0 :lt the JX>Sition or the free end of
the spring when it is in the unstrctchcd positio n (Figure 6.4b). Tn order to c.alculatc the work
to Stretch the sprirl!h we must know sonu.:thing about the forces invo.l ved. h has been shown
experimental ly that the foccc F that must be exerted on the free end of the SJ>ring i n order to
1m1in1.nin a stretch}( in the spri ng is proportional to .r.:

F(x ) = kx .

where k > 0 is a constant. Thi~ then is the rorce that will perform the work. The. basic
dcfinilion o r work W done by u constam force F acting along a straight-line segment o f length
d is \V = Fd. Unfortuna te ly. our force F (x) is not constant~ il depends on x. and we cannot
therefore simply multiply rorcc by disltlncc. \ Vhat we cnn finc.J, however, is an approximnlion
to the required work: by d ividing the lengdt hetween x = 0 and x = 3 into ll ~uhitlletvals by
11 - I poincs.r1 .x2 . .. , ;t n - 1 such tluu

0 = ..to <

.t1

< x2 < < .c,.-1 < x, = 3.

\Vhen the spring i~-o s trclchcd bccwccn x, _ 1 a nd .r;. the fOI"CC necessary to maintain th is .stretch
does not vary g reatly and can be nppro;(. inlated by F (.:.;i) . l r fo llows then that the work necessary
10 Sll'e tch the spri 11g from Xi - l to x; is appro.xinleltel y equal to

IV; = F(x1)(x1

,r 1_ 1) .

As a result, an approximation to th~ total wmk requi red to pull the free end of the spring from

x = Olo .r = 3is

L \ V = L F(.~1 )(~1
1

Once again we cxpcctlhat as


required work W . and

11

x 1_ 1) .

becomes indefinitely large , this <tpproxhnation approaches the

1 1

la l

lim "'F(x1)(,t 1 = u-limoo "


L..,' W; = n-oo L..,
j=l

.1'/ - 1).

(6.91

i= l

Each of these four problems on area, volume. blood How. ~md work has been tackled in the
way. and the n1e1.hod caa1 be de~cribcd qualitari,1e ly as fo l lnw~ .
The CJU<HHily l<l be calculcued, say \V . can nnt he oblained for the ohjecl (; given because
no fo nnula exists. As a result, 11 smallec objects., say G;, are constmcted. The G,. are chosen
in such a way !hat lhe quamily IV can be calculated. exac1ly Ol' appl'oximately. fol'each C;. say
IV; . T hen an appl'o ximation fo l' W is
~anlC

I:w;.
i=J

384

Ch:.peer 6 The [)refinite ln l e~l

If the number of G 1 is increased indefinitely, this approximmion becomes more and more
accurate and

IV

lim L IV;.
-oo
l=l

II is this limir-summcuion prt~cess that we dis~uss throughout the remainder of the chapter. We
bcg.in in Section 6.3 with a mathcmatkaJ descript ion of the process, and by doing so, we obtaiJl a
unified approach to the whole ide~. We then discover that there is 11 very simple wny to cnlculatc
these limi1s. At that point we will be ready to u se the technique in a multitude of applications.
including the four problems in this section.

16.3 The Definite Integral


The four probl cn1.s of Section 6.2 have a comnmn theme: the lintit of a summa tion.

By means

of a ~ummution we approximated sontc quantity (urea, \'Oiumc. blood flow, work). nod the limit
Jed. at least imuitivcly, to an exact value for the q uantity. In thjs section. we iJwcstig.ntc the
mathematics of the limit summaLio n - but only its mathematics. \Vc c oncentrate here on what
a definite integral is, and how to C\'aluatc it; interpretacion of the dc-f injtc imcgral as arcu. vol ume.,
work. and so on. is ma<le in Ch!ipter 7.
Th define Ute definite integral of a function f (x) on an imerval a ::; .r ::; b (Figure 6.5).
we divide Ute imerval into 11 subintervals by any 11 - I poims:
ll
D~ ~

XO

<

X1

<

X1

< <

Defining the dcfinilc inl ~ral

X;

or /(X)

< <
from J

X,t- 1

(I

IU

<

.r =

X,t

= b.

f)

Next we choose in each subintc.rvt\I Xt- l ~ ..t ~Xi any point

x; whatsoever. a nd evaluate

f<xi> - We now form the su m


/ (x:)(xo - xo)

/(x;)(xl - Xo)

+ + /(x.~)(x, - Xu-t ) =

L" f (x;')(x; -

.r;_,)

i=l

L" /(xi) t.x;.


j_ J

where we haveset
subintcnals.

t:.x;

= X;

-x, _,. Wedenme by 11/\x;ll


llll..rdl =

ohelength ofthe longest ofthett

max lt.x;l.

l l ..... n

It is often c-alled the norm of the particular partition of a ~ x ~ b into the subintervals
t.x;. With this notation, we are ready to detine the deJlnite integml of .f(x). his the li mit

of the 'llmmalion abo' e as the number of subintervals beconl(-.s increa~ngl) lar,;e arw.J tYCr)
,;ubintenal shrinks: to a 1KMnL An c:a.-;:icr w11y to say thi$ i~ tu lake: the: limit 3S the norm uf the

ponition opproachu ~cro. In 01her " ord<. we dehne the definite hllel(rnl or /(x) " ith respect
a to .1 I> as

to .1 from .r

1>10)

l)rovtded that the limit <XtSIS. If the limit exists, but is dependent Ott the choice of subdivision
~x, or stur points

:rt'. then the dclinhe integral is of liule use. We stipulate. there.fore. that in

order for the definite integr-dl to exist. the limit of the sum in equation 6. 10 must be independent
of the nl3Jlntr of subdiision of the immal a !: x !: band choice of s~ar points in the
!<-ubintcnal\.. r\ t first 'igl'tt thi~ requirc:tne'nt might ~m r.uhcr severe. ,jn<.~ we mu't no" du~ck
th!ll all ubdivisiOii> nd toll choi"'"- of star point> lesd to the SillllC limit before concludin~
thut the dclinitc intcgrul c.xi-;ts. r\u1unutcly. however, the following thco~m indil:ute~ that fur
continuOtt' furu.:lions. thi:ot i-; unuece~'IVY A proor of thi'\ thcon:m can be found in ndvuuccd
book~ Oil

mathenu tical n.nal) sls.

THEOREM 6 . 2
If a function j (.t ) i~ comi nuou5on a f'inilc interval 11 ~ x ~ b, then the definitt: intc!!ral
of/(.<) with I'C>pcot tO X from .1 =a tO ,t = h c<ists.

For a continuou~ (unction~ the dell ni le intcgnd exiJ~f <;;~ and any choiceofsubdivisinn cmd star
point' kudsto its com:.:~ val ucthrough thclimi ting JIAA'\:. We call j'(.t) on the lclilliutd oide of
cq umion 6. 10 the imcgntnd. and a aotd h the lower and Ul>pcr limit~ or Integration. re:~pcctivcly.
11le >Unt
J(,\ i) tu, i>ealled a Ritnt:>nn sum. and beeause ofthi<. definite int<aflll6.10
i< lw calkd the Riemann integral. The imegml ws nan1ed aflcr CiemtM nuthcmmician
G. 1'. U. Riemann (1826-1 866). "ho uttroduocd the notion of the definite integral as a >um.

I::=,

I EXAMP LE

6.3

EJiuJte the definite integral

Sill 1 110\ Sine< j(.r) = t 2 i<eontinuous on the inten'al 0 < x < I . tho definite intcgntl
CJ.bb. and we ma) choose ny subdivision and tat IXIinto in tiS c aluation. 'Tht: oimplest
partitiOn is intOII cqu,tl >UbintervuiS Of length 1/11 by the points (Figure 6.6)

Oernk

in!q;nl of JfA) A ~ (n.-.m X


(~

xa I
)

t,

: (1, I )

j
II

o... .. Jt .

\\'c. choose (or ss~r poinL~ therighLcnd of each subinlervttl: that i~ in .t,-1
Then. by equation 6. 1o. we have

x; =,r1 = i fn.

~ X;. wet."hoosc:

Sx

I 2 3

11

; -I

x, t= -

"

\.t; =I

II

'

Since all subintervals have equal length 6x;


1/11 . the nonn of the panition is ll!:>.xill = 1/ 11 ,
and tuking the limit us Ut:>.x, II -4 0 is tantamount to letting n -4 00. Thus,

1
t

x 2 dx = lim "
11 -1"10

L._,

i= J

!...
ll

)2(-1) =
11

lim

n-I'IC

1L

II

1=1

1f we oow use formula 6.4 for 'be. sum of the squares of the tirst n positive iotegers, we obtai.o
=

.
I F1(11 + I)(2n + I}
lim --'---'--'.:.._--'--'

u - :.o n 3

TI1is example illustrates that even for an elementary funt.:t..ion such as j(x) = x 2 . e\ aluation of
<he de6.ni<e imegral by equation 6.10 is quite laborious. ln fact, had we not koowo formu la 6.4 for
<he sum of the squares of the integers. we would not have been able to complete the calculation.
Jmagine the magnitude of the. problem were the integrand equal to f (x) = x (.t + I ) - 213 . In
other words. if definile integrals are to be at a ll useful. we llllLI\t tlnll a simpler way t o eV'ctluate
them. This we do in Sc-etion 6.4, but in order to stress the.definite intcgrnl as a Limjt summation,
we consider one more example.
1

I EXAMPLE 6 .4
Evaluate ~1e definite integral

1'

(5x- 2)dx.

- I

s s

soumor-<

Since /(x) = 5x - 2 is continuous on lhe imerval - t


X
I, Ihe definile
integral c;o<_ists, a_nd \VC may choose any partition and star poims iJl its evaluation. For rz equal
subdivisions of Ieng1h 2/11. we use points (Figure 6.7)

L-=I.I.!::I!J.i.~

Definite
iutcgmJ uf f(;c) = Sx- 2 from
.t;:::; - l lo.f;:: l

y
/(x)

X;

=5x - 2

- l

2i

+ -II ,

i = 0, ... , II .

If we choose the right end of eac.h subinter-val as a star point, that is,

:r; =

.t , = - I

+ 2i / n ,

!hen equation 6. 10 gi ves


1

(5x - 2)dx

- I

Ooce again all subinter:vals have equal length l!.x; II C..r; II -+ o with " --> <Xl.
1

1
_1

(5x - 2)dx =

lim

L
"

n -oc

[ 5 ( -I +

2/ n, aod 1berefore we may replace

2i) ](2) =
-

11

- 2

If

lim

L" [20i- 1411 ]


U2

n ....,.OO .

t= l

t= l

\Ve can break the summation into two parts, and lake constants outside each summation to obtain
1

{5x - 2) dx =

- I

20i " 14] =

"
lim [ ~
II~CO !- t1 2

l :::::: d

"' ~ ll

[20-;- ~" i - -14~" l ].

lim
r1- 00

ll:::l

11"

!1::=: 1

ll

.1:le i

\Vith formula 6.3 for the sum of the first 11 po:;itivc intcgcr.i,
1

1
- I

{5x - 2) dx =

lim [ 20 11(11
, ....... oc,

n2

I) _ ' \] =
Jl

lim [ I0 - 411
n

n-co

J=

-4.

Note 1ha1 in Example 6.3 1he value of the deli nile integmJ is positive, and in Example 6.4 it is
negative. The value or the de.finite integral in Exercise 8 is :tero. In olhe r words, the value o f a

definite integral can be positive, negati ve, or zero, depending on the limits and the integrand.

I f! fi13f-i3

J.

1,'

(:\t'

d,~

2)

+ 8.

9. Ev1llumc lhc dclllitc integral

( ' x"dx

]_,

J' .\":.

d x.

'

10.
Con"-idc IJ-.e

{ tl)

dellni~e inlcgral

1'

2 ' d.>: . Shuw thm whe n

Xa

0 S A' :5 I i!!o 'i'Jbd, idcc,l inl<J n C'4ll\l.l t> ul)in


~crval~. nnd. ).Jur polnt$ ur" .::hoien tlS rif::IUhand e-ndpoints
n Clk"h :-;ubn1Ctvll, equm.itMl 6.10 lc:u.ls to
IIIC IIIICI'V.)I

1'
0

I"

2-( d,.,. =

lim - "

.,_1'00 "

L-

= ct.

,o:, = alt .
1

x, =- cdr

x1

clt'} ,

x. olr" = b .

Sh\\W th:,u wtth t hCclll\k'<! l>f .o:; - x 1 ir\ \he Jl~ ~u\)ill\efv.M
X1-1 ~.tC ;S ;t.; . ~uatiQon6.\0t,_iv~.:,

2" ",

'"'

{ h ) V~clhe n>nrlula i n Exc:n.:i~o;e 3 1{b)or!Scc.:tion6. 1 1ucxi)I'C:SS

Ute:-~u nun.,rti ()ll i n c lo....;c;d form.

(l)) UI\.C the: rct~uh of E'(c:rd~ ~~ in s~~\'i.U\' (). I U) wrilt \\1~


sumnl:ttitm in c\OS(:d ftnn, a nd hc1\te show \hll
(c) U,:~c L'116pilnl',s ru le h) c\Wu .tte thl s limil, und hertcc fiml
Ihe \';.~ hu.: o f th e dcfini1c in1e5:n tl

II . U.se lhc 1cchnique <>'' Exercise 10 to evaluutc

i)

e' tlx.

,... 12. Usc 1.hc fOI'Il HIIU

L: ~ln/0 =

,_,

sn

(11

+ 1)0 .

118

~Ctn-

. 0

s.n -

tocvuluutc

sin A' ti x.

15. Sl\ow that the ddm\tc \ntc&cu\ of tht ful\(:\\~' 'n' Cli.cu.:-i"M.: 5'1 "f
Scct iOJ\ 2.4. dOU 1\0\ edst 01\ "''Y \ 1\\ Ct\'l\\ l l S X ~ b v.\'!U\W<''I.'Ct.

16.4 The First Fundamental Theorem of Integral Calculus


In Section 6.3 we demonstrated how 10 evaluaLe definite iotegra ls using detloilioo 6. 10. lnte
grands x 2 and 5x - 2 in Examples 6.3 and 6 .4 are very simple polynomials, as arc the integr.mds
in q ucstjons 1-9of Exe rciscs 6.3, but in spite o f this , calculations were frequently laborious, and

388

Ch.ap1er 6 The Deftni1e lmegral

invar i.ably requi.red surumatioo fonn ulas from Section 6.1. We promjsed a very siJnple technique
that would replace these calculations, and this is the substance of the first fimdamenwl theorem
of imcgml calculus.

THEOREM 6. 3

(First Fundamental Theorem of Integral


Calculus)
If .f(x) is co111inuouson the i.nterval a ::; x::; b. aod F(x ) is ao aotiderivativeof f(x)
thereon. then
1>

j (x) dx = F(b) - F(a).

(6. 1I )

PROOF Since .f(x) is conti nuous on a ::; x ::; b. the delinite i111egral of .f(x) from x = a
to x = b exists and is defined by equation 6.10, where we arc at liberty 1.0 choose the ll.x; and
xt in any way whatsoever. For any choice of t:lx; , a convenient choice for the xt can be found
by a pplying the mean value theorem'' t imes to F(x) . once on each subinte rval x; _ 1 ~ x ~ x;.
This is possible since F'(x ) = j(x) is cominuous for a ::; x ::; b. The mean value theorem
s1a1.e s that for each subinte rvaJ, there ex ists at least o ne point C; bel ween X;- 1 and x; such that

i = I , ... , 1t .

x; - x;-1

But F'(c;) = j(c;).so that


= I , ...

, n.

lf we. now choose x7 = c; . then

f<x:>t.x; = F (x1)

F (x 1_ 1) ,

and equation 6. I0 gives


b

j(x)dx =

'L, J<x7J t.x1 =

lim

lim

"

'L, I F (x;) - F (x1_ 1)J.

JJ<>.<rU- o .

na ~ l- o.

I= I

1=1

When we write out all te1m s in the summation, we find that many cancellations ta ke place:

f(x)dx =

=
=

lim

na~ - o

{IF(x 1)

F (x0) ]

+ IF(x2) -

F (x 1) 1 +

+ iF(x.,)

- F(x, _ 1)J)

lim {F(x.,) - F (xo) l


116.\'ti _,.O
lim

{ F (b) - F (a) j

114.\'ti-O

= F (b) - F (a).
If we imroduc.c the notation

{F(xJlt

to represent the difference F (b) - F (a). Lhen Theorem 6.3 can be expressed in Lhe form

f(x) dx =

{!

j(x) dx

(6. 12)

This is a fu1uastic resuh. No longer is it necessary to consider limits of summariollS in order to


evaluate definite imcgrals. \Vc simply find a_n antidcrivativc of the integrand, substjtucc x = b
and x = 11 . and l\ubtrnct. For i nstance. to evaluate the definite i ntegral i n Example 6.3. we
easily wriLe
I

--0 =3

Note that had we used lire indefinite integral x ' /3 + C for x 2. we would have had

Because the arbitrary consram al\vays \'anishes in the evalmnion of definite integrals. we need
not use the iodelinite integral in this conte:<t: any antidcri\ativc will do.

Sintilarly, for Example 6.4, we obtain

1
1

(5.t - 2) dx

{52

= ..::._ -

}t

- 1

I EXAMPLE

2.<

- 1

6 .5

Evaluate the following definite integrals:

(b)

JJ

.Jx

+ 4dx

- 2

SOLUTION

(b)

/_: J.v + 4 dx

I EXAMPLE 6 .6
Evaluate

SOLUTIO:>! Sinc-e - 1/.r

+ 3x 1/4 is an anlitleri vati\'e for

l/.r2 + 3x 3 for I 5

x .S 3,

_ { - I + ~ } = 1~2 .
{ - ~ + _34x_.. }~ ={ -~3 +243}
4
4
X

I EXAM PL E

6.7
I

CDn che definite i1ucgral

f_,.t
1

dx be t:\"alualed wilh Theorem 6.J'!

SOl l "TION No! ~on 6.3 rcquii\:S ohe io11csnond 1/.t' 10 tx: cominuous 001 lhC imeNal
- I :5: x : I. find this is IH\1 the case. The function is discontinuous m ..r = 0.

Titerc iS

i1

difficully wilh Tiocvrem 6.3. his SllbiiC. btll ionp<>I"IMI. The lhCCli"Cill SlaiC> 1118110

cv.iluute delirtite itlleg:ral~ of continuous function~. we use antideri\1ltives. Dut how do we know
that contin u~ functions ha\e an tiden,ali\et~.? We doo"c )C.I. This fact will be es:cabli, hed in
&.~,ion

6.5 when we ,crif) lhe seoond fundamelllollheorcno. You mighl ask why oho <eeond

fundnmemal theorem is not proved l1rs1. Would it not he more logical fi~t co establish exi~ten cc
of antideri\ ative~. tmd then u~e this fuel lO prove Thtorem 6 .3'' From a loJic point of view. the

answer is )C>. However. fruno a proctocal poinl of 'iew. "Theorem 6.3 os w useful we wanl oo
she ill.'\ cry pv.sible cmphs;is. To prmc lhe .ccond funda!IICIIIOI!hetlrcm fir>l would tlclr-~t
from the imJ>Ortanc-c amtl ~impl id ty of Theorem 6.3.
Bdorc ""move on 10 ohc serond furlililmenllll lheorem. we presco111he following 1heoo:ms.
\\ hi<.h dcs<.Titx: I><Opcrtoet of lht definile in1egral.
THEORI!M 6 .4

If f (x) i<conlinuoo<on a ::; x ::; h.lhen:


(i)

(ii)

L"
1b

f(., )d x = -

j(x) d.t

J.bf(.o: )d.f:

1'

f (x)d.o:+

ll.o)

f (.o: ) d x .

(( i1h)

THEOREM 6 . 5

If f (:r) and g( <) are con1inuou~ on a !:: x ~ h. lheol:


(i )

(ii)

{ ' l](.1)

1.

+ g(.<)] d.< =

kf (x)d.<

1''

j(x ) d x

+ f.bR(.) d.-:

1b

f (.r) d x,

I( 14a)

(6

14b)

\\hen k is a COO\Wlnl.

Propenie 6. 14 are :uoulogou< 10 5.3 for indc:finile inlegrtlls. "Thcoreon.~ 6A uold 6.5 con be
pruv..-d using d1her Theon:no 6.3 or equ,uion 6 . 10
We can al<o esl~blish lhe following property.
T H EOREM 6 .6

When f(x) is eonlinuous on a ::; x ::; b and m ::; /(x) ::; M on I his imerval.

m(b - il)::;

1b

f(x)dx::; M (b - a ).

(6.15 )

6.4

Th~ First Fun<bmenul

Theorem of lntegr:>l C:>lcuhlS

3'31

PROOF ByCQualion 6.10. we can write


b

1
a

f(x)dx=

<

lim L
n<>.,n-o 1a
.

f(x;Jll.x,
l

L Mll.x

lim
1<>; 1-0 .
1 1

= M

"

lim

IAI-0 L..,

tJ.x1

= M

(b -a)

lim

nA .;I ~ O

= M (b - a) .

A similar proof es!ablishes the inequality involving m .

I EXAMPLE 6.8
Use Theorem 6.6 to find a maximum possible v-dlue for

~OI U'1101'1 Clearly. sin .r 2 ~ I for all x. and on the intef\oal I ::: x ::: 4. The largest value
of 1/( 1+x2 ) is l/2. Consequenlly. (sinx 2)/(1 - x 2)::: 1/2 for 1 ::: x::: 4and. by Theorem

'
!. +

6.6.

si nx 2

~---::
, dx

<

2(4 -

l)

Velocity and Speed Revisited Once Again


= 3r2 -

61 - 105 I'CJ>resents the velocity (in metres per second) or a I>Anicle


moving along the x -axis beginning at time I = 0. \Ve ca11 e~-tsily calculate the definite in regnal
of t.'(/') beLwcen any 1wo 1imcs1 say t = 0 and 1 = 12. By doi11g so, we get our first glimpse
o f defini1e imegrdls at work in applied problems.

Suppose that v(r )

( 12

Jo v(r)dr =

r'l (3r

lo

6r - lOS) dr = [ r

3r 2

105r )

= 36.

Realizing that integralion is a limit summation, and what is being added are proch.JCIS of velocilies
v(f) multiplie d by small lime incrc01cnts dr . we imcrprct 36 as the displacement of the pa n icle
at time r = 12 s relati\e to its displacement at r = 0 s. Although we do 1101 have enough
information to determine where the particle is at any gi\'entime. we can say that at r = 12 s. it
is 36m 10 1he righ1 of where it is at t = 0 s. In general, when v(t) is 1he velocily of a panicle

moving along the x -axis.

/,b

v(r) dt

(6.16)

392

Ch.ap1er 6 The Deftni1e lmegral

is the disp lace m ent of the panic le at time 1 = b relative 10 its d isplac-eme nt at 1ime 1 = a. lf
the ddinite integral is positive, Lhen at time 1 = b Lhe parLicle is to Lhe right of iLS position at
t =a ; and if the integral is negative, then at t = b, the particle is to the left of its position at
I

= a.

Speed is the magnitude of velocity. lf we integrate speed between the same li1n its, we get
a differeot result.

1'

Jv(t) Jdt

1''

2
l3t - 61- 105Jdt.

Since 31 2 - 61 - 105 < 0 forO :;: I < 7,and 3/1 -61 - lOS > 0 for7 < I :;: 12, we d ivide
the integration into two parts,

12

1
7

Jv(t) Jdl =

2
- (31 - 61 - IOS)dl
3 + 3t 2 + 1051

1-1

1114 .

1"

2
(3t - 6t - l05)dt

2
3
1: + 1t _ 31 _

105tr

Since Lhis integral adds products of speed Jv(t) J and time increments dt. which we interpret as
dis tance trave lled, 1114 m muse be the dis tance cra,e lled by the particle between t = 0 s and
1 = 12 s. In genen>l.
(6. 17)

[' lv(t) ldl

is the distance travelled between times 1 = a and 1 = b.


These ideas are reinforced by graphs of velocity and speed in Figures 6.8. Because velocity
is negative for 0 < t < 7, the pa rticle is moving to the le ft allhese t imes. Products v(l) dl
are negative during this lime interval , and therefore contribute negatively to the definite integral
of v(l). Ute panicle scops a t/ = 7 s, and then moves to the right from 1 = 7 s 10 t = 12 s.
Products v(l) dt then comribute positively to the definite imegral . with the ultimate result being
36 m. At time 1 = 12 s. the panicle is 36m 10 the right of its position at 1 = 0 s. On the oth er
hand, s peed is always positive, and therefore products Jv(l) Jdt always conuibute positively to
the integral of s peed.

1Jil!l!.lll3iiJI1!

Velocity function v(t) = 3t1

6t - 105

M#ltclll;l:wJ.iil11l

Speed function lv(t )l =

1Jt1 - 6t -

250

250

200

200

150
100

ISO

too f.--~

50
2

10

12 I

50

- 50
- 100; -- - -

10

EXERCISE S 6.4

In E.xercises 1- 40 evaluale lhe defioite inlegral.

3. j ' (4x 3 +2x)<ix

4.

1'

(x'

+ 3) dx

2.

J.' (x

- 2x+ 3) dx

(x' + -~-}lx

6.

s. {

1
1'"
- .l

-o

I.

-o I
--, il.t

x-

sinx d x

12 I

1051

6.5 llle $o:(md Puodamem.al TI)C()tem of Jnrepal C~h:\ll tlS

7.

j'

(x

I - x' )dx

8.

-I

~-' (

1
2

2.r) dx

29. J, '(x + 1) dx

3(),

:c

1
f
5

+ 3x' + 2) dx

9. J,' (x'

II.

13.

1'

.< (.<

10.

+ 1)2 dx

12.

x2 + 3
- --dx

+ l)dx

-l

cos2.rdx

Ji -

16.

18.

' (.< 2 - l)(x 2 + I)

dx

20.

1:

3cos.r dx

1'

2'' dx

26.

sin x cos x d:r

J:rfJ Sl:c x tanx dx

J', t!

csch

* 41.
* 42.

1'

e ' d.<

28.

U(l)

38.
40.

x dx

-I +.r 1

1'
1'"

dx

cu.b 2.< d.r

1 + 4x 2

dx

= - 12 + 31 - 2, I = 0 10 l =

2. I = I to I = 2

45. V(l) =l ; - 31 +21 . 1 = 0 10 1 = 2

46. U(l) = I' - 31 2 + 21, I = 0 lO I = 3

In E.'<t.rtiscs 47- 52 usc Theorem 6.6 to find rn:t.IC.imum and minimum.


"alues rot tho i111cgrnl.

dx

-: I

- tlx

dx

.W. v(l) = - 11 + 31 -

.. 47.

> X

_1

11(1) = 31' - 61 - lOS, I = 0 10 I = 5

49.

27.

[I

11(1) = ~1 2 - 61 - 105. 1 = 010 I = 9

* 43.
*

~~~

25.

L:

36.

In F.xcrci<e< 41-46 11(1) represents Jhe ''cloeity of a particle moving


along lhe .r-axis. Calc.ulalc lllc dcfinile inlc'jll':lls of U(l ) and lu(tl l
becwccn the twQ times shown. l.merpret udl number and ploL (or
dntw) a graph or lhc velocity function to corrobonttc n:.sull.$.

(x- l)' d.t

dx

j'.t~.+
lldx
_,

dx

ll/2

24.

./~'"=?

3"dx

.rl.r + II ti.T

I)'

12.

J.l.

.rJ.r'- 1

- x") dx
39.

,
1

1
1'

ix l dx

I'll

3.5.

37. {
( .r 22

'

Jx)dx

32.

- 1/1

(x'-

_, x'< x' - x ) dx

ix ldx

\3.

14. 1

1s. j '

19. /,
2

x(x'

)
!,

17. {

1'
.,
1

31.

1'
1'

393

51.

1o
J.

~/"

s:in.t"

- - dx

I+ xl

:t

sin x

:r f 'Z

--.,dx

0 2 +x

1'

1
1

;<r f l

48.
" SO.

I +.<
( 1ft

2.\'
~

,,, 10 + ...

( l +4x ) cos (x 2)dx Sl. [ '

s.inx

- - dx
dx

J 4 + x' dx

16.5 T he Second Fundamenta l Theorem of Integra l Calculus


The first fundamental theorem of integral calculus in Section 6.4 allows us 10 use muidcrivativcs
to evaluate definite integrals of continuous func tions. \Ve now show that continuous fum::,t ions
always have antiderivatives.
When f(t } is cominuous for a

::; f::; b, i ts de.finite imcgral


!.b ,{(T)dl

is a number. If b is changed but a is kept fixed , the value of the defini te integral changes; for
each value of b, there is a new value for the detinite integral. In Olher words, the value of the

definite integral is a f'tulCtion of it.s upper limit. Suppose we replace b by ;c. Ulld denote the
rcsuhig lionction by F (X),

L'

F(.f)

f( l ) flt .

We now show thal the deri,,.t ivc of F (x) i< j(x); that is. F (x ) is"" nntidcrivmive of f (x).

THEOREM 6.7
Calculus)

(The Second Fu ndamental Theorem of Integral

When / (x ) is continuous for a 5 x 5 b, lhe func1ion


F (x )

1'

((\.18)

j' (t) d1

is differentiable f orti ;S.r :S b. and F' (.}() = f(x ) .

IR()()Il If .\' is any poim i n ahc opet1 inter\'31 a < ~t < b. then h can always be cho:sen
s uflic iently :-~m al l that .t
IJ is also in the interval a < ;t < h. By equa tion 3.3. the d eriv~:ltive
of F (~) 01 thi< .r is defi ned "'

,
. F (x +h) - F (x)
F (x ) = hm - - -'---'-'h

-o

= h-oh
lin> -I [1.<+h /(1) d t

1'

lim -1

h-oh

[1Hh

f (t )dt

c;

I !..<+h

= lim -

h -0 "

f(t) dl

1 ]
J(t)dt

(by property 6. 13a)

.c

f (f ) d t

(by properly 6.13b).

Accoo'ding to pmpctty 6.15.


mil 5

<+A

j'(f )dl :S Mh,

<

where m und 1\1 are the minimum and llUL'<imum o ff (x ) o n the intcr\'al between

x and x + II .

Division by h gives
I
m :S h

1'' +1> f( t ) dr :S M .

Consider wh!ll happens as we let II -. 0. n,c lim it of the mitkllc tcm1 is F' (x). Funhcrmorc.
the numbers m and M m ust approach one another. and in the limit must both be equal to f (J:) ;
they are minimu m and maximu m '~lues of J(.r) O<l the inte" al between x and .r- h. and It
is a pproaching zero. We conclude therefore that

F'(x)

lim

x+il

f(t)dt

.f(.r).

h-o '
This a.rgumcm e-an also be used to establish that F (x ) has a right-hand derivative /(a ) at

x=

a , and a left-hand d erivative f(b) a t

x =b.

Acron.lin~ tO Thoorcm 6.3. we cvnlutu e c~e delluite Integral of a conthltlous fun CL.ion over

the interval o ~ .r ~ b by calculating the d i frtrtnte betWCCil va.lues of uny antid~ri"'Mive of


/(.t) fll .x b and :u x = tl. Thea~m 6.7 euabhsl\c:1 the fact that couinoous functioM
have antic.JeriYIIive\:, h dues not, h\M.\C'\Cr, ) ield an antii.k:n\lltive or /(.t) in a form U$C(Ul (or
enJtwiotl M c.lefinitc in~ral or /(.t). lf"'c wtrt to usc: the a~idtrt,mivc of equation 6.1 8
in eqw_tiorl 6 .11. wt \\OUkJ obtain

j'"

{ (x) d.T

f."

/ (t ) dt -

!..

J(l) <It

= j'"
/

(1) tit.

Thi5 h. ccrtamly 1rue. but n<M \'tty tlctprul . To useequation6 11 toe,aluatc the. definite int~&ral of
a fURCiiOIIt {(.\ ), WC I'ICcd .jR ~ntidcri\lltt\COf j(A ) \YfiltCll in lCM'I'\S O( function~ thal WC already
know. Some ntegraful$ have ea~lly computed amidcrivativcs such as thusc i11 Chapter 5: other"$
have an t idcrivntivc$ that arc c;on1cwhm more CQiliJ)Iicnled und require the i1ltes,nnion technkjuc'i
of C haplCI' 8 10 C'<press them in ter~ns or wellknown fu nc1ions. There are some functions.
however. that do not have anlidcrivatives that can be exprc~ as the "um ot" a finite: number or
\\CIIktlOWn functio n~. A ~imple e..~runplc is.,.- , ' . Thcore"'' 6.7 gta"Ji'ltct' that this function
has an 41ntideri\<;dl'-e; it jus1 C""-J.OilOt be ex~ as a linite ~m of "Well"nO\' n function),.
Symbo1ic;~.11y. we may write the resuh o f Thcorcnl6.7 in the fonn

1'

-d

d:c ('

[ (1) tit

f(K).

We usc lhi on the followint four c.amplcs.

I EXAMPLE

6 .9

Evaluate

:!...1' /1~
dx

~cll.l'TIO'

dt .

Meotding to 6. 19.

This is \"alid f01 -1 ::S x <I

I EXAMPLE 6 . 10

F.~tluaiCd~

SOI.l flO'

1.

':!

sin t
--dl .
I+ I 1

For this problem we setu = 2< 2 und in,oke the ehuin rule,

-d 1. ___d, _ -dd< f.'' -~int


s in/
d] du
- d t = [ -d } '" du
+- I -tit
1
''

tf~

>ill /

I - 12

sinu ( 4 x)
I

+ "'

rl

= 4x sin (2x
1

+ 4x

11

...

E~alua\e ~ J: \\ +

1 111
1) dt.

SOUIT\ON We can ~o\ve lhisprob\em by reversing the limitson the integral, which according
to equation 6. \.3a, i\)ttoduces a negative sign. TI1us,

\n the ~o\\owing example, the variable x appears in both limits.

d f2.~

Eva\uate cos \21) + I) dt.


dx ~!
SOUIT\ON Since the it\tegrand is continuous for all real numbers, properly 6. I3b permits us
to write
lx

x!

co~('lt 3 + l)df = fa
.r~ cos(2t3 + l) df +

1lx cos(2t +
3

l )dt

{or an~ real number a whalSOever. To find the derivative of the second integral on the right,
we set u 2x all<\ \l~c the chain rule (as in Example 6.10), and for the derivative of the first
\1\\egm\ on the right, we reverse the limits (as in Example 6.11) and then use the chain rule wilh

u = x1 .

dd
X

'

.
cos~21>
+ \)d!

d Jxl cos(2t 3 + l) dr +-d


= -dx a

xl

14< cos(2t

dX ,,

+ l)dt

\_dd JKcos(2r 3 + t)dt] du + [!_ f. vcos(2t3 + l)dt] dv

u a

dx

dv

dx

= - 2xcos (2u 3 + I)+ 2cos(2v3 +I)

= -2xcos('2x6 +

I)+ 2 cos(t6x 3 + 1).

"fbe Natural Logaritbm Function


In Section I.IJ we rev\ewe<l propenie.~ of exponential and logarithm functions. Fil$l came the
exponential function (IX: and \he ~aritnm function log.. X is iL~ inverse fun,1ion. The logarithm
\)1 .{ \() \)ase, 0 . \0&, X. \S 1\ power. the (:~Ower tO raise 11 in Order tO get X~ !hat i '
Y = hl!la X

if

aY

Based on this de~nition. it was strightforwat'd to derive pi'Openies of the logarithm function
(equat 1ons 1.66) based on corresponding properties (equations 1.63) for the exponential function.
The natuml logarithm function lox uses base e, where e is the limit of (I+ 1/n)" as 11
appr~aches mfinny. We saw Lhe advantage of In x , as opposed to lo&1 x. for dHTe rentiatio n in
Secuon 3. 11; In x avoids an extra con slant.

The lalurallogarhhm func tion c.an be introdoccd indcpcndcmly of the exponential function
using a definite integral. Fo.r x

>

0 we define

ln.r

!.'' ~dr,

(6.20)

ru\d with Lhb definition we can det ivc all J>rOpercies of the logarilhm function. We begin with its
> 0 , ic folh:>ws that ln .\' > 0 for x > 1, <tnt! property 6.13a
x < I. According lo equaliOI''I 6 .19. the dcrivati\'e. of In .\' is

graph. Since 1/1 is J>05itivc fur 1


implies thai il is 11egmive for 0 <

!!_ ln.r
dx

=!!.dx. . f.' ~dr = .<~ .

(6.2 1)

Bccnu!oC 1 /~t > 0 for :c > 0 . it follow~ that the dcr'i va1ivc of In ).' is positive . nltl therefore it is
tUl increasing function. The sec(md tlerivalive is nega1ivc.

d'
-1

dx-

ln x

-""";j',

.r-

so thai the graJ)h is COf'lcave d0\\11\\'Nnl. The g raph mu.~l therefore look like that i 11 Figure 6.9.
Whm is not dear m thi s point is that the grdph is asymptotic to chc negati"e y.. axis. ahhough

this lllight ~ec.::m n:<Sonablc on the ba.o;is that I I I be<..'Oll'l& i n lillit~: :IS I ...:,.

o+.

)'

5 :c

-3
-4

\Vhen u(x) is a di fferelltiable funct.i on o r x I equation 6 .2 1 and the

ch~li n rule give

tl
I du
- lnu = - - .
dx
11 dx

(6.22)

By making s pecific choices for u(.r ) . we can derive prope11ics _1.7 J Cor lhe l'litturallogarithm
fun ction. First. for u = -"l-" , where xl > 0 is a iixctlnumbcr. 6.22 gives

dx

In (x 1.r) =

-I x, =

,,,,,

-.
x

(6.23)

Equations 6.21 and 6.23 show that ln x and In (x 1x) have the same derivative for all x > 0.
Theorem 5.1 implies that these functions <1u1di ffer by ~ t most a con><ant: that is,
In (..r 1.t )

= lnx + C .

lf we set x = I, and note that 6.20 iJnplies that In 1 = 0. we ob1ai.n In .r, = 0


Consequentl y,
In (XtX) = lnx

+ lnx,

+ C.

and for x = x 2 we obtain identity 1.71a.


(6.2~ )

Differcntiati(lll fonnula 6.22 implies Utat for x > O,

- In (I)
d
dx

When this is added to 6.21 ,


I

.\'

.\

- - - = 0.

Because this is valid for x > 0. Theorem 5.1 implies that


ln x+ ln ( ;) =C.
Substinttion of x

= I gives C = 0. and therefore


(6.25)

We can now verify property l.7 lb.

In(::~) = In [x,
=

In

.tt

= In X t

CJ]

+ In (~)
x,
-

In .t2

(by 6.241

(by 6.25).

For fixed x, > 0. equation 6.22 gives

-dx In(.r' 'l


d

d
= dx
- (x 2 In x ).

Since this is vnJ id for all x > 0 , Theorem 5.1 once again g:i \'e"S

In

+ C.

(x ' ') = x l lnx

If we set x = I, we obtai.n C = 0, and for .T = x 1


(6.26)

With lhe logarilhm function defined by 6.20. chere is n n:numl way 10 inrroduce rhe number

e. We define<' as the number whose narumllogarithm is I :


ln e

= I

!.
I

< I

-dt
I

I.

(6.27)

The resulting exponentiaJ function ex would sat isfy propen.ies 1.69 since aiJ exponential functions satisfy properties of this type. Furthermore, it follows fro m 6 .26 and 6.27 that

In (ex ) = x In e = .r.

(6.28)

A~~

6 .6

V:1.1ues

399

11tis is propeny L72b. We can also establish L72a by setting y = e 1"' . When we take
logarithms of both sides. and usc 6.TI a nd 6.28 ..
ln y = ln (e1' ') = ln.r(lne) = lox.

(6.29)

Tile graph in Figu re 6 .9 indica1es thai for aoy gh'Cil y -vaJue Oil the c urve, there is only ooe vaJue
of x th3t gives that y. In other words. if h:1 y = ln x as i n 6.29, it follows that y = x. Since
)' = e ll'lx . we have x = etll~ .

EXERCISES 6.5

In Exercises 1-20

1.

3.
5.

1'

<lifferenli~H e

2.

(31 + I) dt

[ 'sin

f.''
1

llle deflnile inlegrdl with res:pec110 .1 .

9.

ll.

t
x

{
' dt
I JiT+1

IS.

["":r cos (r ) dt
1

14.

f.''"
<;<IU

J.z.t; ./i.d/

16.

-I - dt

..)1""+1

[./A ./i

dr

.r:

+ I) tit

.{A

4.

(1 ) dt

6.

(21 - t')' dt

~- t 3 rostdt

t ..tT+T
{_, +4

dr

sin(3r+4)dr

(3Ji - 2r) dt

j'_,,tan (3/ + l)

8.

10.

.+-

;;>+:idr

r~
4X

(
t

dt

18.

19. 1 z., rlnl.d/

20.

17.

~, !

7.

13.

tle4r

In (1

4? _ 4 ,:

tfr

21. Verify Lhut whc::n a (x) and b(x) arc dilfc.rcntiublc fun<.:tiom; of .'(.

-d.
d

I) dt

J1

Lb''' f(t)dt = f[b(x) Jdl>


d g~J

/[a(x )] du
-d . .
X

Is equation 6.19 a spc:cia1 case of tbis result?


2X!

til

12. /_, ,

22.- 28..

sec (I- t)dt

Use lllle resul1 of Exercise 21 to redo Exercises 8. 10. 12, 14.

16. IS. and 20.

16.6 Average Values


The average value uf two numbers c and d is defined as (c + d)/2. The average \'alue o f a
set of 11 numbers .ro. )1 ..... y,. is (y, + Y2 + + Yn)/ n. We would like to etend this idea
to define the average value of a function f(x) over an interval a =S .\ =S b. By beginning with
some sin1ple ftmc tions we can see how 1.0 do 1his.
The runclion .f(x) in Figure 6.10a is equal to I for 1he tirs1 third of the interval shown. 2
for the second third. and 3 for the la.'it third . We would expect its average value over the interval

0 =:: x =:: 6 to be 2. The func-tion in Figure 6.10b takes on the same function values, namely, l ,
2, and 3. btll nm on the same s ubintervals. Tlt e fact then it has \-alue 2 fhr 2 :S .x :S 5 and value
3 for 5 ~X ~ 6 suggests that its average value .should be ::iOmc.what lc$S than the average value
of 2 for ohe fu1Jctioo in Figure 6.10a. T he aver.Jge value ofthe function ill Figure 6.J0c should be
even less than that in Figure6. 10b. Thcsechree runctions indkme thal l\\'0 factors are importanl
when considering average values of functions: values that the funct ion take.s on, and lengths of
the i nterv-~ l s Oil which they take these values. Perhaps what should be done to calculate ave~ge
values for these fullctions is to add together t.he products obtained by multiplyu1g each of tbe
fu nc tion value.(; by the length of the interval in which it has this v~t l u e, and then d ivide this s um
by the leng th of the overall interval.
For Lhe fu nction in Figure 6. 10.1, this yie lds an average value of

6- 0 11 (2 - 0) +2(4- 2)

+ 3(6 -

4)] = 2;

400

Cllap1er 6 'n1e Definiae lnteg.rol


FIGURE 8 . 10c

Al"troge \'alues or rhr-ee piece-.~ise cons1nn1 function>


)'

>'

for 1he func1ion in Figure 6.10b.

I
- f l (2- O)

6-0

+ 2(5- 2) + 3(6- 5)] =-II


6

and for 1he function in Figure 6.10c,


I

- [1(4- 0) + 2(5- 4)

6-0

+ 3(6- 5)1 =

-.
2

This procedure is applic.1ble only to a function whose domain can be subdivided irllt>a lini1e
number of subi01ervals inside each of which me func1ion has a constant \'lllue. Sud1 a function
is said 10 be piecewise constant. Wha1 shall we do for fune~ions mat are not piecewi e consranr?
By rephrasing the procedure above, il will become ob.-ious. The same three 3\Crdge values are
obtained if we udopl 1he following approach [illusarmed lon hc limcrion f (x) in Figure 6.10c}.
Di' ide 11\e inacnal 0 ~ x ~ 6 imo three subinlervals 0 < x ~ 4. 4 < x 5 5. and 5 < ..r 5 6.
Pick a poim in c.tch subirrLCn1ll; call the points x: .
and .rj. EHlluate f(.r) a1 each poim
and multiply by the length oflhe subinterval in which the poirn is found. Add lhese resvlas, and
di,.ide by ahe length of the imcna llo obtain the average ,-alue.

.A;.

ucxn(-l- O} + Jcx;)(s- 4} + f(x;)(6- 5)J = r1(-l) + 2(1} + 3(1)1 = .

Bur this is rhe procedure usa! 10 define 1he delinite ncegral of a lunction; it lacks the limit
because lhe function is piecewise conswm. In o1her words. fOI" a function f(x ) rhr.l has a ~aluc
a1 every poim in the interval a ::; x ::; b (but is not necessarily piecewise consrano, we define
iLS average \'alue as
I

avernge,aluc = - h-

I EXAMPLE 6.13

lb
0

11

What is rhe a\erage value or the function /(.r)

SOl

f(x)dx = - - lim " f(x,)A.r,.


h - 0 I AXII-O L_-

= x: un the interval 0 :s x

liTIO'\ 8) equarion 6.30.


uverdge value

112

= -2o

2
l {XJ
- } =

x 2 dx = -

-J

(8)
-

23

:s 2?

r6 .101

66

I EXAMP LE

I\\'CI"l3C Vo\h!C.i

401

6. 14
Find the average value o f J (x} = sin x on 'the im"rvals (a) 0 !:: x !:: ,y / 2. (b) 0 !:: x !:: rr,
and (e) 0 !:: x .S 2rr.

t~lllji::WfKlil

Aver~

v.1!1.1e ()( sin.t


I

SOI .UTTON

We calculate average values oo these imervals as:

y
(a)
2>r

( b)

(c)

-I

1"/l
1'

I
n2
1

-I

rr
-I

2Jr

2
rr

2
rr

sin.rdx = - 1 -eou }~fl = - (0 +I)=

2
11

I + I) = 2
sin x tlx = -I {- cos .r }~ = -(1
rr
1r
tr

1:t'. sin x dx =

- I 1- cou}~ = - ( -1
2JT
2;r

I) = 0

TI1c gr-Jphof sin x in Figure 6. I I also ~ugge.sts that the average values in parts (a) and (b) should
be the same, and that the average value in pan (c) should be zero.

I EXAMP LE

6 . 16

App lication J>review


Revisited

ln 1hc Application Prc\'iew we questioned how n controller mig.hl monitor current inn circuit.
One way would be for it to const.,ntly measure the average value o f the cuJTCnt o'er some hal
interval of time. Suppose lhc current in the circuit is as shown i1l l'igure 6.12, and the controller
eMstantly measures the average vnlue of i (I) over the p revious 0.1 s. If the a verage ' 'a lueever
rcachc~ ISO%
steady-state c urrent. the c <HIIrollc r takes corrective action. At what time. if
any, docs the concroller react'!

or

OJ'!J!I'h

Abn~>mul

ctan-cnt in d ccmc

SOLUTION

Clf'(tnt

If we deno te the nverngc vnlue of i(r) ovu the 0. 1 s before time 1 by f(r}. then

i . 10 + 200(1 - 1.9)

-i (l )

30
\

. tO - 200(1- 2. t)

20

= -0.I 1 [

'

i (f) dt.

t-o l

For I .S 1.9, 7(1) is a lway> equaltu 10 a nd the controller takes no oction. For 1.9 < I !:: 2,
i(r) is increasing and so also is i(l):
1.9

2.1

i (l )

= 10111.
IOdt +
14>. I

J.'

110 + 200(1 - 1.9)]dl l

1.9

10 [ [101} ;~, + {10011

20(50/ 2 -

1901

- 370r J; .9]

18t ) .

Titis reaches 150% o f steady-state c urrent in d1c interval I. 9 < I S. 2 if

20(501 2

Solutions o f this quadratic equation arc I


therefore the controller reacts at 1.97 s.

1901 + 18 1) = 15.

= 1.83 and I = 1.97. The first is unacceptable, and

...-._
'lbeorem 6.7 can be used to establish the nexltheorem.

402

CblpiCI' 6

n.c Octio.ilt l nregr;ll


THEOREM 6.8

(Mean Value T heorem for Definite Integrals )

Jf f (x ) is comi nuow;; foro ~ x

5 b . I hen there exis1s al le;.~st one number c between ll

and b such that

J.bj(x)dx = (b I'ROOF

a)f(c).

(6.3 1)

By Theore.m 6.7, the function

F (x) =

J.xf (t) dt

is con tinuou~ for a :S x :S b. and has derivative f(x) for a < x < b. Mc~n value theorem
3.19 applied to F(x ) guarantees at least one number c between a and b such that

F (b) - F(ll) = (/1 - a) F'(c).


By substirution.

J.bf (t ) dt -1"f(t)dt = (b -

a)f(c) .

Since the socond integral vanishes. we have

J.bf(x)dx = (b -

a)f(c) .

By writiJtg equation 6.31 in the form

!.b

f(c) = -1f (x) dx,


b- a a
Theorem 6.8 states that the function must take on its average value at least once in the interval.

I EXAMPLE 6 . 16
Find all values of c satisfying Theorem 6.8 for the function
X

:S

f (x )

= x 2 on the interval 0 :S

I.

SOLUTION Substituting a= 0 and b = I in equation 6.31 gives

, 1' , {"'x33}

x dx =

(I - O)c =

= --0 =3

Of the two solutions c = If ./3 for this equation. only the positive one is between 0 <llld
I. Thus. only c = 1/./3 satis(ies Theorem 6.8 for the funct ion j(x) = x 2 on the interval

o:s x:sl.

..-..

I EXAMPLE

6 . 17
Find

all values or c satisfying T heorem 6.8 for the function / (.<) = si n x on the interval

rr / 4 :: x :: 3rr / 4.
SOl u-1 ION

Substituting a

311
;r )
- - sin e=
(4
4

= rr / 4 and b =

\~{'

3Jr /4 in equat ion 6.31 gives

sin xdx = 1 -cosx l;~~

./2.

.Tf4

~i nc =
Jf

TI1crc a rc two angles between

c=

Sin- 1 (2../2/Jr)

..-..

7T /4

and 3;r /~ with a sine equal to 2../2/rr , "'unely

= 1. 12 !llld c = 7T- Sin- (2../'i.f ;r} = 2.02 .

EXE RCISES 6 .6

In Exercises 1- 20 li nd lhenvcroge value of the func~ion oven he inLCr~

20. j'{.r)

= L.r j.

0 :S .r :5 3.5 {See P.xen::isc 68 in Section 1.5.)

vaL

1. f(.r) =

3.

x' - 2x. 0 ::; x ::; 2

=x'- x.

J(x)

o s .r s

4. .f(.,)

= x,

5. f(x)

= ,/.\"+~,

! .r !

= .\ "'- I, 0 $X$ 2

9. J(>')

=co<x,

10. f (x) ~ cos.r,

II. f(x ) = lxl .


12. f(x)

"' 22. T he vclocily v of blood flowing 11woug.h a ci(tular H~in or ati CI')'
of radius R at a di.stanc.x: r fl'(wn lhe c:cnLte of the biO<>d vessel is
v(r )

s x :52

= 11. o ! x ! 2

14. f(x) =

l.r' - 41,

0 s x :53

-1 $ .t $ I (Sec Exercise n in Section 2.4.)

= sgn.x .

-I $ .r S 3 (See Excn:i>e J7 in Sectioo 2.4.)

17. f(x ) = h(x - 1).

18. j(x)

19. /(x) =

lxJ,

= 2x

24. f(x)

= x~- Sx.

26. f(.r)

27. f(x) =

= il(.r -

23. }'(.<)

0 S .r S l <See Scaioo>2.5.)

r 1) .

i ..

28. f(x)

cos.r.

-2 $

= x '(x

Iii* 29. j(x) =x../x'+

0S

x S 3 (Sec Exercise 68 in Section 1.5.)

i*

O~x~tr

1 ~ .t

+ 1).

0 S x S 2 (See Se-ction 2.5.)

x$

0:!:: X $ 1f{ 2

./.\"+!.

4) ,

30. f<x)

0 $ x :5 2

- x',

25. f(.r ) =COS .\.

-3 ::; .r $ 3

IS. f (x) ~ sgnx.

In Exl!rti~cs 2:\-30 li nd all val ue~ Hf c ~~ i~l')'ing cqmttion 6.31 furl' 1he
fm1<.:1ion j(x) on thcsr>ccificd intc.v.ll.

0 :$ .<!: rr/'2
-2

= c( /11

(Soc Problc1n 3 in Section 6.2.) \Vhut is the average value of ~1(r ) with
1\!'Spc.:Cl IQ r '!

-:r/2 :$ x :$ rr/2

13. j'(x)- lx' - ~ I .

16. f(x)

VcriJy th ~lthis is Ihe snn'lc as Ihe tl\'<:wgc of the velocity (\tndion over

0 $ .t $ I

8. f(.t)

- t.

the time incer val f t ~ 1 ~ /~ .

- I $ X $ I

7. f(x) = ,, - I ,

.t

h
1

0 $ .T $ I

6. f(x) ~ ,/.\"+~,

21. If a p.1r1icle moving tllong the x-n:<is i$ at position x 1 al timc 11


;md 3t pr.~ti t ion .t2 >H time lz, it~ average velocity over this time imcr\'al
is

I,

= lfx' + 1/x',

O~x~l

osxs2
1S x S 2

ages J(x) aodlags behind it.

\n 1\Yan~ a\)\)\\<:11.\i.on~I~\\wo\'lin% Ctt\\\ic tunclion"&, suchas \hedaily\)rice


o~ to\d., o-r \.\"11! \'a\\ICS o( stoc.:. ;md bonds,\\ ts advanlageoustodefine

a n\\.w\nt a\'etate. \\ ts \\\e a\'Cta<&c o{ a function O\'cr al\ imcrva\ of


\w,ed kn?,m L,00.\ o\ >~ariab\t \)OSi\ion. '\''ht \en~th L moving avemge
o~ a iun.c\ion f \x) a X i'il

f\x) = -1

fl

32. j(x) = sinx, L =27r

3l. f(x) = sinx, L = rr

34. f (x) = x3, L = 2

l l f(x) = x2, L = I

.. 35. /(x) = lx\, L = I


36. f (x)

Jtt) dt.

= 1-lx\.
0,
\

lxl< l
, L =I
lxl > I

* 37. J(x) = h(x- 1),

x-l

L = I, where h(x - I} is the Hcavisidc

func1ion
~8. f(x)=h(x-a)- h(x-b). l. =b - a.whereb>a>O
and h(x - a) is the Hca1<isidc function

\n'C:~.ettist> 1\-1% ca\cu\a\c the movin~ aYcra~c o( f(x) for tl\c %i"cn
\en~\\\ L. \l\Qt,ordrow, f(x) ar(flx) loi\\us~rate\\(1\11 J(x) a'cr

\ 6.1 Cnan~e ot Var\ab\e in the Definite Integral


"fhe fits\ fu11uamenta\ theoremof integral calculus indicates that to evaluate the definite integral

-;:::.x== d ,t
Jx + S

- I

-t

we should first find an antiderhc.tive for x I.Jx


<:ase du =dx. al\d

= Ill -S du = f <u'fl- 5u-tl2) dtt

f .Jx +5
x

+5. To do this we set u = x + 5, in which

dx

JU

1 - \Ot1 1/ 2 + C = ~(x T
= ~~~~'
:J
3

5)312

IO(x

+ 5) 1/2 +

C.

Consequently,

d.\

-1

-4

..{i"'+5

= l ~(x + 5)3/1- tO(.t + 5)1/2 }-1


\3
_,
[~(1)3/2 - 10(1)1/l]

= u(4)3/2- 10(4)1/2] \6

= --

3
An alternative a111>roach, whic,h usually rums out to be less work, is to make the change or
varlal\\e u x + S directly in the .<Jefinite integral. In this ca<c we ~gain replace

II -

J x + 5 dx

-fii du.

by

, II

In ~Cldition. we <et>ll\Ce the limits x = -~ and 1


-1 by lhMc values ol' 11 that COtTe<pond
IO lhe.e
of-~, namely, u
I anclu 4. respe~ll'cl). We then obtain

""'"ts

- l

_ ...

r::-7< dx
V..l

1.

r.:5 du =

II -

Vll

2
= { JIIJ/
l-

1.'

10111/l ] "I

(u

1 2
'
-

5u- f2) du

= [~<~>''l- I0(4J't2]- [~oi'l- IO(IJ ''z]


16

3.

That this lllCthod is gc1crally acccptahlc is stated in 1hc l~lllowing theorem.

THEOREM 6 . 9

Sup1)0se f(x ) is condnuou


""db = g({l). TI1Cil

0 11 (I :; A :;

b.

l ft f

1b

f (x) dx =

a11d

we se1 x = g(u ). where

= g(a )

f3.

g(u ) is

(g(u)) g'(u) du,

if g'(u) is continuous on tt :; u :; {3, and if when u is between tt and


between t1 and b.

I EXAMPL E

c1

6 . 18

Evaluate /,
1 (

.:.X

SOl UTIO'I

If we SCI u

!'

X-

dx .

= ./X -

I. then du

f'

I
I }
2{ I
- ;; - u2 - 3u3 J'i-

r.:.Ji , dx
2 (.., ,, - I}

I) 4

u+
I +
--2(11

h- o u1

= [I /(2 ../X)J dx, and

I) du "" 2

J' ( 1 + 2 + -;1)
,/i- o u2

3
11

du

1
;-2 [1 +1+~]
+2
[
I +
I
+-=-- -]
3
./2 - 1 (Ji -1)2
3(./2-l)l

ljUIII. JjF 1!!IL.J

= 21.2.

Whcu follows is a discussion leading to the well known formula for the area of a circle. Although
you should be a ble 10 follow lhc culcu 1mions. i1 is highly likel y 1hm you will not undcmand dlC
soLU-cc of son..: or the ideus. Do 110t be alm,ed: in time. we will deal with all lt~pccts of the
derivari on in delai l. For tlOW, si mpJy take the e;cample as an illustration of 1he power of i111egral
calculu~, and Hn indictuion of thing.<; to come.
Whe n we study applications of definite irucgmls in Chapter 7, we shall sec thm the area of
thedrclc in Figure 6.13 is gi"cn by the definite integrol

Area nf o

A=

41'

Jrl- xl dx.

()

The i.ntegral itself actually gives the tlrst quadrant area; the 4 provides a quadrupljng factor
for the olher three quadranLS. ll is noLpossible to guess an amiderivativc for J r 2 - x 2. In
Section 8.4 we learn thai an approp>i me change or variable i s to set x = r sin IJ. The integrand
becomes

ah~ohue vnlu~ being nece~ary to ensure po."ilivity for


ha\'e

A= -1

!.

J cos2 0. Since dx = r cos 0 dO. we

II:

r i<Josll rco.OdO,

8,

where 01 and Oz arc values of 0 corresponding to .r = 0 and .r = r. Whcn x = 0.


th< equation x = r sin 8 requires that 0 = r sin 8. We choose Be = 0 us the solu
cion of thi> equation. (There arc others, an<! in Section 8.-1 we shall sec why this choice
b made.) S irnilarly. when .1 = r. we choo>e the >olution 82
rr/2 uf r
r sin 0 .
Then

r0 '' r

A = 4}

eos91r~9d9.

Fe< 9 on the onten-al 0 ! 8 ! rr f2, co< 9 is positive, and absolute \-alue< may be dropped.
A =

To find an acmiderivathe for co'

f"'l co<29d8.

4r' Jo

0. we solve double-angle formula

1.461> for co,2 8.

cor(} = I + co<20
2

Thus.

('''1(I +en~ 28)dO=

A = -lr Jo

2r

("/l

Jo

(I -;- cos20)d0

= 2r (o+ ~i29[' = ,,:.


the formula lor the area of a <Irde. II<! reminded that )OU are not expected to soh-e oth<r problems like this yet. In time, yes. but r1 0 t now. What we hope is that the e:xamplc ghc$ you a

glimp>c ol' thc powerofintcgml Cltlculu> and the imponancc of the tools tllal we have developed
in Chapters -~ lllld 6.

EXERCISES 6 .7

16.1

1.

J,'

3. /

>'(3x' - 2)' d.r

2.

17.

0
__
x_

i.IX

'- Jx + ;l
6.

7. [ ' 1

y ~dy

' J + .jii
. !, "' '"
I

dx

(X- 2)
2

sin3 x
dx
, ,., ( l +cosx)''

.
1 Jx
-~

!,

dr

12

21.

r-;1

,11

10.

12.

\(77-::-'-tlx
I +x

14.

X+ I
dx
_, (x'+ 1..1 + 2) 1 '

!,

J2 + 3"nxrosx dx

(.r

'

+ l)(x..;;

!.

...

+x

I)

.
d.\

tf.r

1~. f'J( x')' ' dx

dx

' - .~ + ..

1
1

(lnx) 2

:t! l

--dx

22.

.. xlnx

- 3

Kel X

< ,/4+3 tW\X

J:

/(.t)dx = 0:

and lhal if f (x) is an e,cn function.

(" /(X)<Ix
J_u

= 2!." f(x)tlx .

tfr

l..l. Show tl'"'t if f (.t ) ;, an otl~ rw"tion. then

f.''

~tl

_, (.t + 2)'

1'

20.

(
},
"T

13.

~.

vii

I I.

J.'z~d:

_, ,
1

dx

24. Show ulgcbruically thut if f (.t) i_s a. continuo u.s (unction with pcriOtl J!. I hen

(I

+t

f (x ) d x

[ ''

f (.< )ti:t.

.. 26.
27.

II

.f\.fl.

!
f

_,J.rz-

, (j - 4X -

U=X

,
I - x
u- = - 5+ x

o .,.,--,-x-== dx,

f.

nile imcgnll.

X')'"

co> 1(;r/2) oos 01 dO= :r


used i n C(llcula1ing

i!<>

f.~'

!!lin(}

1l1i .. in1egrc~l

II =.;;
-

dx!

28. Show Lhut

In ExcrciKCs 25-27 u-:c Ihe suggcs1cd subsliuuion 10 cvalu.al c I he deft-

., 25.

6.\'

..

t"'J~i ;nOO

I -<OS

I/>

(2;r - 1/>)

d~.

powel' from a half-wave

antenna.

SUMMARY

The dctinitc integral of n co ntinuous function f (.r) from x


a to .r b i s a number, one
that depends on the runction f (x) and the l imits tl and b . We h:we d cfined the definite integral
by sulxliviJ ing the int:r\'al a ~ x ~ b into 11 pans by n
J points a = x 0 < x 1 < <
.t,_ 1 < ,.\'J1 = b, and choosing a point .tJ in e."lch subinterval .x1_ 1 ~ x ~ .\';. l lte definite
integml i s then the l inlit o rche ~ummat ion

"
L
[(x.. ) t>x
IO.Asi-O

J.bf (x)dx =
where D.x;

l im

1,

1 1
~

= x;- .Xi-1 Since all calculations in thls li mit take place on the ,\' ....dxis. we regard

the dctinitc i ntcg.r.ll 3s an integration along the x -axis from .r = a to ,\' = b.


The first fund.amemal1heorem of imegml calculus allows u.'\ tu cakulate defini te i ruegr.els
of COnt.inuous functiOn$ using Ulltiderhati\'CS1

[(x ) d, =

{! [ (x)d{.

This presupposes that a continuous functio n has an anrideriv.Ui\'e, ~ls \'Crified in Section 6.5 . It
wa:; :;hown that when the Udinitc in1cgral i.s given a \'ariable upper lin1il x . the resulting function

F (.)

= J.' [(r } dr

is an antid erivativc of /(.): that i s. F'(x)


The average \>llue of n function

= /(x}.

(x} over nn intervnl a

-1-

b- a

b is defined ns

f.bf(.r } dx.
a

The mean value theorem for definite intcgrals guaron tccs the c.x istcnce of at least one numbe r
takes on its average value.

c between a and b , at which f(x )

[(c) = -1-

b- {I

f.bf(x) dx.
a

Evaluation o f a definite integral with a co mplex integr.md c-an sometimes be simplified wilh
an appropriate change of v:uiable.

Cba:pecr 6 Tbt uetin1te ln!c~l

408

KEY TERMS

fn reviewing thi s thapler, you should be abl e t o define or discuss the fo llow iHg key Lenns:

General 1erm

Sigma notation
Index of summmion or ~}ria bJe or summc1tio1l'
Finite geometric series
Definite imegral
Upper li mit of integration

LimitS of summation
Norm
Lower limi1of imegration

Riemann sum
F irst f undamental theorem of integral calculus

Riemann integral
Second fundamental theorelll
of integral calculus

Average value

Mean \'lllue theorem for t!cfinilc integrals

Piecewise conslant
C hange of vari able
REVIEW
EXERCISES

In E'(ctd:scs 1-20 C\"alu:uc dlC' definite integral.

1
"
1
s. 1'
3

(x' + 3x - 2) dx

I.

(.r' -lr)i/.x

-I

9.

J'

(r' - x')dx

J.' (.r - l.r)i/x


f -l I
6.
1

4.

JX)dx

(-

.;; -

x(x + 1) 2 dx

8.

10.

1~ c:osx dx

f.$J.i+lll.x

12. J.' x!x2

13.

{ (/X+
;;

14. [

17.

1'

2
x + I
(A"+ I)'

f."f'
1

19.

18

)dx

tiX

cosx
llx
(I+ sinx)'

16.

IS.

( I + xlil )2
xl/3

tlx

20.

I dx

xJ.i+ldx

J.'

(x- S)dx

(b)

-3 X

=5

= X

x,

,rx:;:i.

-2 :5 x

0 :5

,f

:5 - I

:5 I

0 :5 x :5

7f /2

t!l'+lill

28.

1-l

+ I)' dt

29. [ ' ,r;>+\dt

30.

1"

dt

!.'

12(1

I COS I

x(l +

1x1 )' llx

31.

11-.-,--dl
I

r-t3 L-

fx +

21dx

J.'

(.r

+ 3) dx

+I

32.

sin! l dt

- \ -l

Jn Exercises 33-..\4 v(t) represents the velocity of a panicJc mo\'lng


along Lhc .>axis ~ Ca!cul:uc the definite imcgmls of v(l) and lv(t) l
between the tw<, times shown. lnlcl])rt't e-.K...h nwnbcr physically ami
drJ.w :a grJ(>h of v(/) to corrobor.ttc rcsulls .

.;. 2 1. Use equal ion 6.10 1c.1 <:valuate the fniiO\vins deJinite inlegral<;:

(a)

25. J(.r )

27.

-x 2 ../1 - x dx

J'
-

I
- d.Y

Ill Exeteises 27-.32 difft'r"~ll iale L11e inlcgml with rt:st.x::ct Lo ..-.

f'-

!,'

(b)

0 5 .x 5 I

26. f(x ) = cos' .<sin2 x.

II.

= ../x + 4,

24. f(x) = 1/ .<2

x l (.c' + 3) llx

-l

+ 3x)dx = - J

In E."<ercises 23-26 find the average \'fllue ofd~ runction on 1he inter

23. j(x )
1

(x 2

\'Ul.

.t

- l

- I

15.

J.'

(a)

~d.t

(x + 1) dx

7. [

-I

3.

2.

~ath of the foll c"''ing ans"m is incom:ct. btlt do so


wi1hou1 C\alualing 1hc definite imegral. T hinl nb(lut what the definite
integral reptCWMIS.

22. Prt)\'C that

33. v(l) = / J - 61 2 + li t - 6 . I= 0 to I = I

34. V(l) = l l - 61 1 + II/ - 6. I = I IO I = 3

Rt\'iew Exercises

In ~crtiotc<t 35-12 C\'alu:ue the definite integral.

35.

'
!.

J7.

J >'Jt-A'cl.\

J'

x 2(4- x')' dx

40.

-I

.t'

-.,...-..,-,,.,-... tl:r

(x' + I)

J9.

* 36.
JS. [

ll+.rl

JA

! X-

41.

1
I

25

./X
. dx
X-.)

f.' J2+x+ ./X


1

42.

dx

409

CH APTER

Ap~tlication

Applications of the Definite


Integral

Ptcvicw

The figure on che lefi below shows gas in the cylindel' of a se3nt engine, diesel engine. or internal
combustion engine. Jc requires work to move che piston to the len and compress the gas in the
cylinder. On the other hand. if the gas exp8nds~ it does work in mo"ing the piston lO the right
p

'>''"

111<: figun: on the right ubovc is the Rcwki11<


litr m1 iu~~liz~u st~am engine; it represents
the relationship between pressure P and volume V ofg& in th<: cylindel' during o ne cycle. \Vater
on low 1cmpcrature and pres-sure (J>Oi nt A ) is heated al cons tmll \'Oiume (along path A B ). Along
BC che water iJ> con\'cncd to steam and expands slighlly, and the expansion contin u e~ along
CD. To complete the cycle. the steam is cooled and condensed to wtuer along D A.
THE PROB I.CM

Determine the output or the steam engine during one cycle. (Sec page

440 for the solution.)


In Chapter 6 we defined the definite integml of a function

!.
u

f (x) as the limit of a sumnuuion

f (x)d.x =

lim . L f(xi) ll.x, .


14-t. ll-o
l t

ihc limit ~ununati<>n on the right tldinc:!r< the value of the: Udinitc integral on the: left. I n
this chapter we thi nk of thi$ equation i n the rc\C::r.$t: d in::ction in unkr to evaluate gcunu::tric ami

physic;tl quanti ties. E.ach quanti ty is expressed :.1S a limit summation of1he form in thisequmion.

The limit summution can immediately be imerpre1ed as a delinite integral. The defin ite integral
can then be evaluated by means of an antideriv,nive or the imegrand.

17.1 Area
We have formulas for the area of very few shapes - squares, rectangles. triangles, and polygons
of any shape. since they can be divided i.nto rec1angles aod 1riangles. Consider fi ndiog lhe area
in Figure 7 .la. Each of us has intuitive ideas. about this area. ln 1h is sect ion we take these
intuitive ideas of what the area ought to be, and. make a precise mathematicaJ definition o f what
the area is .

410

1.1

Mjlclii;IWA,

;\rc.a unt.lcr curve y = / (:<)

Ale3

Appro.l im.llion Q( Jrc:a under CIJJ'\'c y

FIGURE 7.1b

411

= j(x)

P~

~/(.<)

1"--A, Az

A.

1\;

b .<

-"t

.\'1 -1

,\'2

x,

x,-1
{J

=x..

Re.:aJI from Problem I of Section 6.2 that the area in Figure 7. La can be approximated by
rectangles. Specifically. we partition the imerval a ~ x ~ b inco 11 parts by points
tl

<

Xo

X1

<

X2

< <

Xn- 1

<

X11

b,

and constmcl rcctang.les ns shown i11 Figure ?.lb. The area A1 of 1he jlh rccumgle is

A; = f(x;)(x; - x;_ ,) = f(x;) llx;.


and an approximation to the required area A is therefore
"

i; l

;= I

LA;= L f(x;)

L\ X;.

I f we let the number of rectangles get lai',\Cr and larger. and at the same time require each
rectangle to have >maller unu srnaller w idth that evemually approaches zero, we feel that better
and better approximations can be obtained. In tact, if we take <.l1e limit us the norm lllu;U
of the partition approac hes zero, we should get A. Since we have no formal dc6nition for the
area o f odd-shaped figures, we take this opportunity 10 make our own. We dclinc the area A in
Figure 7.1a a$

..

(7. 1)

Area A then has been defined as the ljmit of a sum of rectangular areas.
The right :;ide of equation 7 .I is strikingly sjmiJar to the definition of the definite integral
off(x) from x = a 10 x
iJ (equation 6.1 0):

II

.f(x}dx =

lim

f(xrJt.x; .

11 <1<111-0 I

The only tlit'fen:::ncc is the absence or s in cqtmtion 7.1. But when we rcl:all that in equation
6 .10, xi may be chosen as any point in the subinterval X; - 1 ~ x ~ x;, we sec that by choosing

X; - X, .
b

1
0

"

f(x)dx

= l<lxd-O
lim L
.

f(x;)t.x; .

1- 1

IL follows Lhal area A of Figure 7 . Ia may be calculated by means of the definite imegr.ll
(7.2)

412

Chapter? Appijcaionsor the Oefinilelru egr:~l


ll is imponant to reali ze that equation 7.2 does no1 imply Jhat a deli nice incegral should ailv-J ys
be 1hought of as an area: on che contmry, we will lind chat defi nice i111egra ls can rcpresenc many
quantities. What we have said is thac the area in Figure 7. 1a is defi ned by the limit in equation
7.1. But this limit may also be interpreted as the definite integral of j(x ) with respect cox
from x
tt cox
b ; hence, the area may be ca lculated by che defin ice imegral in equatitHl
7.2. Since definite incegrals can be evaluaced using anciderivacives, it seems chac we have a very
simple way to li nd areas.
h is simple to extend this resuh to chc problem of finding che area in Figure 7.2. Wl ch our
incerpretation of equacion 7.2, we can scale chac che area under the curve y = f (x), above the
.r -ax.is, and between the vertical lines x = a and x = b is given by

FJ!I"il..;;pfW

rwo CUt\'tS

A1't:a bet\\etl'

''ertical lioe;;

at)d lWO

Since the a1-ca under the cur\'C y = g (x) is !li"cn similarly by


) = g(x)

it follows that the requ ired area is


A= A 2

the

A1

1b -1b

g(x)dx =

f(x)dx

1b

lm<gral ror

(7 l )

We now luwe two formulll,; for finding areas: equation 7.2 for the area unde-r a curve., above
and bcrween two venical lines; and equation 7.3 for the area between two curves

x~axi s.

and two vertiC:ll lines. Note thai 7.2 i< a spedal case of 7.3.
area uttdu a cu"-e

[f(x)- g (x)]dx.

AI

1his poin1 we could solve

a number of :uea problems using the!;:e rwo results, but to do so \would strongly suggest that
integration should be approache.d from a "fonnula' point of vjew; and jf there is any point of

view thAt we wish to adopt, i1is completely the opposite. By the end of this chapter we hope
10 ha;e developed a sufficiently clear undcrscanding of the limit-summation process that usc of
imegration in siLUations 01her than those discussed here will be st~<~ightl()rwartl.

y = f(x)

~10

illustrate how we arTive at the correct definite integral for area problems without memo(ind~ng the arcu in Fisurc 7. 1n.

rizing the fom1ulas in either equation 7.2 or 7.3. consider ugain


f <.r)

~-\_~----+,. x=i l_ b x
dx

We draw a rec1angle of width dx at position x as shown in Figure 7.3. The area or th i~ rect>mgle
is
f(x) dx .
\Ve visualize this rectangle a~ a representative for a large number of such rectangles bel\veen 11
and b. Tu find the requira.l ar'e.a we add togrth~r all such f('.Ctangulcer are.as anti take the limit a.s
their widths approach zero. But this is the co1ce.pt of the definite inte.gral. so that we write for

the limit-summation process


Uillflllf"' ~ Ul 1nt~gml f():
arc; bel ween 1wo cur\'e.. arll.ll\lo<'
\-c;nit"~l l inc::.

A =

J.bf(x ) dx.

Limits a and IJ identify x-posilions of fir& and last rectangles. respectively.

.--c
._ f(x)

Sin1ilntly. fo rthe area in Fit,:urc7.4 we draw a rec-tangle of width dJ,: and lcngtl1 f(x)- g(x)

r-...
'---rr.-

and , lhc:rcforc. or it rca

[f(x) - g(x)Jdx.

f (x)

_..,.=g(x)

~ > ':

...___,-.,,],g (x) :
a

{J

___;; X =

cb:

To add areas of all such rectangles between a and b and. m the same time. to take. the limit as
their widths approach zero. we once again use the detjnite integral.
b

A=

[f(x)- g(x)] dx .

7.1

Area

413

For area problems, then, we start with the area of a representative rectangle and proceed
to the required area by summation with the definite integral. We express this symbolically as
follows:
area of venical rcc1angle

x posidon of
A

widlh of
vertical
rectangle

length of vertical recmngle

last rectangle [ y-coordinate


of upper end
..
f
of rectangle

,........._...,

y-coordinate]
of lower end
of rectangle

X -poSitiOn 0

dx

(7A )

first rectangle

I EXAMPLE 7 .1

.........
Find the area enclosed by 1he curves y
lith!liJ1

=x2 and J = .r 3.
IUJ.<t!Ji"fJ l!il

A~.'l

bounded by J = x' and y = x'

Es1ima1c of aroa bounded by y

= x'

andy = x'

.t

SOLUTIO!\ The area of the reprcsclllativc rectangle in Figure 7.5a is


{' '

hence,
A

~'l,.3)

1'

(x' - xJ) dx

d.t .,

= x~3 -

x4 }'
4

12

II is easy 10 check whether 1/ 12 is a reasonable answer. This area is contained inside the square
of area one in Figure 7 .Sb. It is casonablc thnt d1c area bounded by Jhe curves is onc-lwclflh
that of the squnre. We could have used this argument to ball park the nns wcr before maJOng any
calculo.1iolls.

In the next two~xt\ntples\ expressions for areas of rcprcscnta6ve rocut.uglcs vAry within the region
specified. l n S\teh cases. we set up ditTerem imegrals corresponding to difterent represenuuive
rc.c.tanglcs.

I EXAMPLE 7.2
Find 1he area bounded by the x -ax.is and the curve y

SOLUTION Areas of rectangles between x


[(x 3

= x l - x.

- I and

x) - O] dx.

x = 0 (Figure 7.6) are

Area OOur.dOO by the c:unes

)'.r' - xandy=O

,T

whereas between ~r = 0

~nd :c

= I areas a.re

[0 - (xJ- x)]dx.
Con!::equently,

A = ! O(x 3

x)dx

_,

= { : = =

+f.'
0

(-x 3

L f-:
+

+ x)dx

~l

(~4 - 2~) + (-~4 + 2~)

We could have saved ourselves some calculations in ohis example b)' nooins that bcCtlUSC of the
symmetry of the diagrdm (x 3 - x is an odd func tion), ohe owo areas are idenoical. Hence. we
could find the left (or right) area and double i~

A=2

r' 'lo (

o (x 1 -x)dx=2 { :._ _ ~
4

- 1

_,

=-2 -1 -?' ) =?1


4

I EXAMPLE 7.3
Find theareaof thetriangle with edges y =

x. y = - x/2. andy= 5x - 44.

Areas of rcprescnlative rectang les oo the left and right of x = 8 (Figure 7.7a)
are. re.specti\oely.
SOLUTIO~

[x- (-x/2)l dx

and

[x - (5x - 44)]dx:

therefore,
S

A=/.

[x - (- x/2)Jdx

+f.

II

[x- (5x - 44}Jdx

3[,8xdx + f."(44 -4x)dx

= -

7.1 Ar<

,,5

~ I x,,s+ {44x- 2<2)~'

= 48 + (484- 242) - (352 - I28)

= 66.
Fi g~.-ue 7.7b

provides a quick check. ~lbc required are.a would seem to be somewhere between a
third ancJ h<Jif that o f the I 5 X I I rectangle w ith area 165 square uniL~.

m:MJ
bounded by y

= x . .~ = -.x/2.

area

and y - 5:t -44

c.'r trian;lc

y
,(11. 11 )

(I I, I I)

: )' =5.< -44

,\

'
(8, -4)

We see from the examples above that the lengtlt o f a re presentative rectangle in equation 7.4 as
" upper y nti nus lower y" is '"'l id whether th e rectansle is in the first quadrant (Figure 7.5a),

the second "''d fourth quadrams (F igure 7.6). or partially in the first a.nd partially in the founh
(figure 7 .7a). In fact, it is valid for rectangles in all quadrants. Remember this: we use it in
many appliccnions.
FICURil 7.8

)!

; g(y)

dy

X ./'(}')

\
X

There is nothing special about vertical rectangles. Sometimes it is more convenient Lo


subdivide an area into horizontal rectangles. For example, 10 tind the area in Figure 7 .8. we
draw a representative rectangle at position y of width dy . Its length is j (y) - g (y), and
therefore ilS area is

lf(y) - g(y)J dy.

Adding over all rectangl es gi\'CS

1"

= " lf (y) - g ( y) ] dy.

(7.51

Corresponding to equation 7.4, we could write that for horizon~a l rectangles,


a rea of horizontal rect~ng le

width of
horizontal

A =

y -posi ti011 of
length ofhorizonral rectangle
rectangle
last rectangle ~[=-_r--c-.tx_>_rd-::i-n-atc-~--x--cO<
>r-::d::-in-,_,-~c-=-] ,.....-,

y-posi tion of

of right end

ofleft end

of rectangl e

or rectangle

d y.

(7.61

nrst rectan$le
I EXAMPLE 7 .4
Find the area bounded by the curves y = ~x
ijlci'i;l
hy .\'

fJI1

JX""+'l4, x

Area bounded

JY.

r =o

and

+ 14, x =

.jJ, and y = 0.

SOLUTION Subdivision of the region (Figure 7.9) imo vertical rectangles results in two
imegrations: one to the left and the other to the right of the y-ax.is. On the other hand,throughom
the required regi on~ the area of a horizontal rectangl e is

[J)i - (y~ - 14)]dy,


and therefore

A =

[(fi - y2 +

14)dy = { -2 _vl/2
J

-l /

+ 14y 'l' 0

J6
3

64

+ 56= 40.

- t4

In choosing between horii'..ontal and vcnical rectangles. consider two objec.ti\'Cs :


1. M inirni ze the number of integnuions.
2. Obtai n si mple dcfillitc i ntegrals.
For instance, by choosi ng one rype of rectangle we may obtain only one definite integral. bm

it may be very difficult to evaluate. If the orlter type of rectangle leads to two simple definite
i ntegrals, then it would be wise to c.h oose the fiwo simple integr..tls.

I EXAMPLE 7 .5
Find the area enclosed by the curves

y=

~x 2

- 16'

X=

6,

:0: 0.

SOLUTION Examination of Figure 7 .I 0 indicates LhaLhorizomal rectangles necessitate three


integrals. l.n addition, we would have to solve equati ons y = x 2f 15 and y = xf ~xz- 16

fCM' x in ternlS of y . \ Vc 1hcrcforc Opt fOr \'Crtical rce-mnglc.s.

16

~ x' ls+ Jxl 15

16

x']dx

x'3 ~ s6

=~
(~)
+ (So +72)- (3+ ~)
15
3
3
= 76.2.
MOiiC4il.IWAIM

The mean value theorem for defi nite intcgmls (Theorem 6.8) shltes tha t there is a number
belween a a nd b suc h lhat

1b

j(x)tlx

= (b -

tt) j(c ).

It has a very s imple inte rpretation in tenns of area when f (x) 2:: 0 fo r a $ x $ b. Since
f (x) 2:: 0 for a $ x $ b, the definite integral may be inte rpreted as the area under the curve
y = J(x), abovethex-axis, and between venicallinesat x = a andx = b(Figure 7. 11). The
right side is thea rea of the recumgle of width b - a aod height j(c) shaded io Figure 7. 1J. The
mean vaJue 1.heorem guar.mt.ees at least one point c betweeo a and b f or which the rec.tangula.r
area is equal to the are-a under the curve. For the curve in Figure 7 . I J, the re is exactly one such
point c; fort he curve in Figure 7. 12, there are three choices for c.

raiiJMQ V

.,

I:Unil;li'U[EI

A tC3 IIIICI)lf'elrh)ll nf

mc.ln va101o theotctn fi)t dcfilii ~ ite&ills.

meal'! v,llue lhe\)l't"'

Three points Sllllsfy the


f(lt 1~i ~

fl iiH:Iion

,,

EXERCISES 7 .1

(d) Repeal p311 (b) \\ith cighl rccwnglcs all of cq~d wid th~
denot ing the sum ofthcnrcu.">o f thc( i~hlrce&ani:,lcs by As.
Show O*' the gr1ph the extra prcci:o;ion of As over A,..

ln E>:crciscs 1-16 lind 1hc QI'Ci.l of 1hc rtgion bounded by the curves,
I . y'

x' = 4y

4X.

2. y = x'

:l. yx 1 = -' )' = 5 - .r 2

4.

+ 8.

_v(y - 2),

X =

= 4x
.t

4 8

+ )' =

18. Repcut E'(ercis.e 17 wi 1h t..hc c.n;a bounded by 1hc curvcs y =


. \.) + 1. x = I. x = 3. and y 0.

S. t = 4y - 4y 2. y=.t - 3. y= I. y=O
6. ,v- r'~, .t ... I,
7. 12y

8. y

= 7-

x'.

= .;:r:t:i.

9 . .v = (x - 1)'.

x-

y
y

This discussion is continued in Exercise 48.

12

2, y = - x

= 1/ (2.<)

In Exeu.:isc:s 19- 22 sea

= (x + 4) 2/8

19.

x=O. y = O

12 .

.f

+ )' =

13. X=

x,

.r = O

I. X +)'= 5. y

7 ,1'. )'

S<t

= 0.

=2.< + I,

y = 2.<

+6
t:

14. X,\' = c, y = x 2 .\' - 1 (~m~Ucran:a)

JS. y = r 2(2- x 2) . y = ~l.J .


16. x

"'

(but dn rl{ll ev.tluate) definite iniC...t!(s) ft)l'

t"- ".

y'!, 4.t = - y1 , y = -I , .)' = I

+ y 1 :;; 4

.1. 2

+ y2 ;; 4.t

(interior to both)

2 1. ,<2 + y' = 4. x 1

+ >'' = 6.>

(interior to both)

+ y2 =

22. x'

)' = ,T /4. X= 0

1//4 -

X =

20. x l.

10. y=sinx (O Sx S n ). y=O


11. y=.\' 5 -

u~)

the mua of the rt'iDn bounded by the cur"cs.

16. x

=y2

csmalkraroa)

2J, Find lhc m~.l Of lhc rcgiM bounded by y l =.tax and ~z = 4a)',
where a > 0 is ll<."Qn:stanl.

y ~
In Exerdses 24-35 find the urea of th.: rt."Q,ion bm..mclc<.l by the <:urvcs.

= 1.1'1 + I. x + (y- 1) 2 = 4. y = 0 (above d>e .t axis)

24. y = x J Jx + 3. x= I, x =6. y= - x'

17.

(a) Find 1hc nrca A of 1hc region OO..mdcd by the curves )' =
16 - x 1 y = 0. x = 1. and .r = 3. rn the remainder of
thii: prohlc1n we approx1n\ah! A by n."'Ciangk.'Smtd show tlutl
the :tocurucy or the approAim.uitXl irx.T\:J.SC~ as the numbo.:r
or roct.anglcs increases. To do this you will IK:OO a hll'gc
gmph or the runction /(x) = 16 - x' on lh< intcrv.tl
0 .t 3.
(b) Ollhcj!r>phdrJw twon:.:llln~tlesofcqual width( I unil)and
w11h heigh1;; determined in the same way a;; in Figure 7.1 b.
If A 2 denotes 1hc sum of the W'e&S of these two rccu n ~lcs.

25.

Ui. ,1'

= x3 -

r =

s s

whal is A l? What is the error in the ~ppro>.i rmuion or A


by A:'! lllustr:uc this error on the gmph.
(c) Re.pcm part (b) wilb four rttlanglt!S aU of width one-h:df.
dt::m) ling the sum of the an~as Oflbt:: fo ur rcct.:~ ng.lc s by A 4
Show on tbe grnph the c;<lra precision of A4 over i\2.

= y 1 + 2.

x = -(y- 4) 1. )' = -x + 4. y

X, X

+ ,\' + l = 0.

=JY'+T

x =-l. x = l
x
(x -2p
1' = 0.

29. y=sin' x. y= l/3. O S x ::; 2n


" 30. y = ln.x : . y =l-xz. ,\'= I

*
*

31.

lxl '1' + lyl'' '

32.

y'

= x2 (4 -

= I

x')

=0

7.2 Volumes of Solids of Re,olmiOI'

33. y 1

34 .

= x' (9 + x)

.r' =x 2 (x' -

.. 35. (2x - \') 2

I 49. Show thm thc CW'\'CS

4).

=5

.v = x' + 16

= x 3, x = 4
boun~

* 36.

The tangent ll:nc u1 a point on the fist qund~u\t pan of the pnrnboln
.r = 2 - x' makes a lfiangle wilh 1he posi1ive X and .)' axes. Find
the point for which ltM.: area of the triangle i~ smulle~t.

37. Ri.."J)C!tl txcrci~

36 fo r the curve: y =

2-

and

204y

= 13x' - 1

11trec r"gions. Find Ute area of the largcs1region.

P be :t poinl on the cubic curve y = j(x) = ax~. Lcl thc


umgcn1 1inc u1P intcn;ecl y = f(.x) again at Q. and let tl be the area
of ohe resion bounded by)'
/(.x) Md ohc line PQ. Leo 8 be lhe
:m~a of the region defined in the same way by starting with Q instead
of f'. Show lhat 8 is 16 limes.., lurgc as A.
51. 'T'bet>nraboln y = ax 2 ::u\dthc circlc .l.' 2+ (y - r ) 2 = r 2 i ntersect

50. Let

x4

In Exercises 3&-45 it is necessary to use a .:.akulator or computer to


lind poinls of in1ersec1ion or 1he curves. Find 1he area or 1he region
bounl.!cd by the cut,cs (to thn::c decimal places).

i.

419

in hvo pOitHS as Shown in the fiui'C below. Find the ,~atuc of a that
nlukcs. the a1ca i n:,.idc the parabolu uud below lhc h\)riwrnal linc Uuough
Ihe points or imcrscctior' as le~rgc us possible.

38. y=x 3 + 3x 2 + 2x+ l. x=O . .r=O

!II 39. y =x' - 4.(, y=2 - x - x'

iii$ 40.
i + 41.

y = x' - 5x2 + 5, y = 0
y=e~:, y =2 - x 1

~ * 42. y=cosx~ 4y=x+2

ii* 43.

44. y = x ' - 3x2

ij . 45.

2
y = - -, y = x 3 +3x - l, x = O
x+2

X=)';-

+ 4x -

y'- 2y ,

.r'

2 . .x = 4 -

X=

./1YTT

** 52.

-t--t- SJ. Show lhlll

46. For whal values or m do the cur\.tes

)' =

.X
2
3x

+I

I+

)' = mx

s.nd

1hc urc::a of the region in the figure below is

--m2

f'Q
r,.

[/(x) - mx - bj[J

where x 1 and x Q arc x -eoordintucs of

bound a region with finite area? Find the area.

Prove lhat the result in Exercise 50 i'\ valid for any cubic y

f (x) = ax 1 +bx1 + ex + d.

+ mf'(x)j d.r ,

P and Q.

)'

.r ;

47. Find a point (a.b) on the curve


x f Jx 2 + 1 such Lhatthc
region bounded by lhis..:ur\'c,the XaAi:s, and the li..nc ,\:=a h-as aref;l
equal to twice that of tl1c region bounded by the CUI'\'C, the .\' iixi.s, a nd
1hc tine y = b.

y = mx+b

48. If 2' (n
I , 2 . ) ree~anglcs (all of equal width) are d!llwn in
Exercise 17to ~~pproximate A, and A ;!It denote,; the ~"'" of I he :irea~
of these rcct:mgles, s how that

++ 54. Suppose that the horiz.oruul lirlc )' = h intersects the parabola
y = (tX1 + bx + c (a > 0 ) in 1wopoints f> and Q. Show lhat thca.l'ca

or the region so bounded is lwo-third:'i of l11c lcnglh of P Q rnullip1icd

.A..n
~
~nd

I
A - [2-"--l

3- -I-)]
+ -6l ( 2n-)
+ 2ZN-J .

hy the distance fmm 1he ver1cx ofrhe par-dbOia to 1he Mril'(l.JUill line.

i! **

SS. A circular pasture h:u~ radiu~ R. A Cflw i~ ried 10 a i\1!\kC a1 rhc


edge of the p!l.Siurc. \Vh~U length Of rope ptnnit,; the COV.' 10 graze em
haJJ the J)~sLun;.']

bc&lcc tbut ij.ro A r = A .


11-:>00

l7.2 Volumes of Solids of Revolution


l.n &c1ion 6.2 we d iscussed 1he idea of r01a1ing nm surfaces around coplanar lines to produce
volumes of solids of revolulion. To 1\nd 1he volume genera1ed when the region in Figure 7. I a is
revolved about thex -a x.is. we again approx inuu.e the region by n rect.aogles as in Figure 7.lb- lf
each orthese rectangles is rotaLed amund the x -axis, Lhe n 11 discs are fonned. Since the radi us

420

Ch411)11er 7

Apj)I K'alion~ <,fthe

l.>tfinire lnre~rol

ApprudrmtJAg \-olUnlt- of solid of rC\olutioo b)' discs

of the i 0 ' disc is f (x;) (Figure 7.13), i1< volume is g iven by

V; = ;r (/(x;)f(x; - X;_,) = ;r(/(x;Jf LH; .


An al>proximation to the required \'Oiume V is therefore

"

1=1

1=1

2:: v, = 2:: ;r[J(x;)f 6x;.


If we let the number of rectangles get larger and larger. and at the sante time require the widths 10
get smaller and smaller, i1 ~em~~: rea~1bl e that we will obtain better and bener:1pproximation.s.
Furthermore, if we take the limit as the norm of the portition approaches >.cro, we should get
what we think is V. Since we ha\e no fom1al definition for such volumes, we make our own.

We define
V =

lim

l ~.'t; l-0

"
I>r[f(.t;)f
.

6x;.

(7.7)

I= I

But we can interpret the right side of this definition as the definite integral of rr[f(x )]1 with
respect 10 x from x = fl to .T = b; that is.,.

Consc.qucrllly. we: can c.alculatc the \'Q)umc or t11C solid of rcvoluli()n ~cncratcxJ
region in Figure 7.1 a around the x-axis. by ntcans of the definite integral

by rotaLi ng Ihe

(7.8)

The volume of the w lid of revolution has been defined by Lhe limil in 7.7, but ror evaluation of
this limit we use the del1nite integral in 7.8.

WijUJII.I#j
Ei" Integral ftX
,ohl'l'oe -.bel, area befv.~ '""
0
C"W'\'eS is ro1att<l aJ\JWld ,\ -axb

lnte:r.tl for
''Ohlme \lt.f:tt"

:a~

undtr

~ C'U"'t

h rotated ifUWld .\' JXi>

)' = /(X)

-~
.T

To avoid memorizing 7.8 as a formula, we use the technique imroduced in Section 7. 1.


For the ,olunK' obtained by rotating the rcg.io11 in Figure 7. la about the x -ax.is. we oonMrucl a
rectangle of width dx at position x, as shown in either Figure 7.3 o r Figure 7.14. When this
rcc1ang,le is rOlmcd around the xaxis.the "olumc of chedisc generated is

;r[f(x)JZdx.
where dx is the thickness of the d isc and ;r If (x) ] 2 is the area of its flat surface. TIis disc is
pictured as representing a large number of such discs between a and b. We find the required
volume by adding volumes of all such disc.s atld taking the limit as their widths approach zero.
But this is 1he concept of the definile illlegral. .so lhat we write for 1he limil-summation process

where limits x =a and x = b identify Xpositionsoffirst and last discs, respectively. 'ntis is
called the disc meth od for findi ng the volume of a solid of revolution.
A slightly more general problem is that of finding the volume of the solid of revolution
genercned by ro1a1ing a region bount.led by 1wu cunes and l\\'0 veni<."al line( as shown in Figure
7. 15 around the x axis. If a rcptc$Cntati,c rectangle of width dx at position x is rotated around
ahe .\'axis. ahe \'Oiume formed is a washer. Since the outer and inner radii of this washer are
/(x) and g(x). respectively, its volume is

To add \'Ulumcs of all such washc.r~ between a anti


widths approach zero. we usc the definite inteSJ111

b. and a11hc same lime take limias as thc:ir

(7.9)

422

Ch:ttwc:r 7 Applications ur the De-finite ln tc:~rJI

I EXAMPLE 7 .6
Prove tha t the volume ora sphere or radi us r is 4lfr 3/ 3.
ljlflll;ljFJII'ill

A sphere or radius r is formed when the semicircle x 1

+ l ::;:

r2 (y ::::. 0)
is rotated around the x -ax.is. The voJume of the representative disc geoc-ratcd by rotating the

Volume t'

SOLL"TION

recrangJe in Figure 7. 16 around the x -axis is

Since the volume formed by the left quarter circle is the same as for the right quan er circle, we
calculme the ' 'o lume generated by the right qua rter and double the rcsuh:

I EXAMPLE
lii!Ji);f!#f.&fJ

7.7
4

Prove that the volume of a right circu.la r cone of base radius r and height h is ,. r 2 /r / 3 .

\'Q)umc of

<l rii:.hl d l'\:uJar cone

SOLUTION 11te oonc can be gcncrdtcc.l by rotaling the ui anglc in Figure 7.17 around the
x -axis. Volume of the representative d isc formed by rowting the rectangle shown is

y
r.-t

:t=;;
-~

Ty

1
r

hence.
J
X

I EXAMPLE 7.8
Find the volume of the solid o f revolution obtai ned by rolllti ng the region bounded by the curves
y = I - x 2 and y = 4 - 4.r 2 around (a) the x -axis and (b) the line y = - I.
SOLUTION
(a) Let us ballpark the answer before we begin. l fthe rectangle in Figure 7. 18a is rotated
around the x -axis. it produces a cylinder with radius 4 and height 2, and therefore of

fDL!i!lil PJ Et.:JfM
of solid of rc:vol\ltion

E.;;titnarion of ,otume

I l!i!lil IE UW
rutatod around the

.~

\ 't'llume v..tlen are;a i'>

u.;,is

4 )'

y=4 - 4xZ

volume 11' (.1)1 (2) = 3211' cubic u ni~s. We migl11 estimate that the requ ired volume
would be about half of this. Let us lfiod out. When the rectangle in Figure 7. 18b is
rotated around the x-axis, the volume of the washer formed is

Because. of the symmetry of the region. we ro(atc only the right half and double the
result:

V = 2

L'

15JT(I - 2x

+ x')tl.r

= 3011 {x- 2.;) + ~ L= 16rr,

exactl)' <>ur e.~1imate .

IU=: lit*I 'fAtl '

Volume
-'htn :ltta i~ rotated srour-.:1 the:
line y - -I

(b) W hen the rectangle in Figure 7.19 is rotated around the l ine y = -I. the inner
ana ower radii of the wa her are rt and r1 respectively. Now r1 is a leng th in the
y-dircction, and in Section 7.1 we le..'"U"ncd that to calculate lengths in lhe YdirectiOn 1
wetakc ul>pe.r y minus lowery. Hcr.cc. TJ = (1 - x l)-{-1) = 2 - x 2. Similarly,
r 1 = (4 - 4.r 2) - {- 1) = 5- 4x 2. The volume of the washer is therefore
...

')

., ...

.., ..,

"'

(JTri. -nrj)tlx = [rr(5 - 4rt - ;r(2 - x-ntlx = 3rr(7 - 12x

+ 5xJ )tlx.

Consequently.

'

21
1

V =

3rr(7- 12.r 2

+ 5x') d.JC

= 61T {7x- 4xl

+ xsJ~

= 2411'.

We can also rotate horizonlal rectangles around vertical lines 10 produce discs and washers as

illustrated in the next two examples.

I EXAMPLE 7.9

Find the volume of the >lid of re,ohnion when the region enclosed by the curve.< y
y = 0, y = I , and x = 0 is re,ohed around Lhe y-axis.

In x,

SOI.UTTON When the rectangle in Figure 7.20 b ro1a1ed around 1hc y -axi,. the 'olume of
the dise fom1<d is
rrx 2 dy = TT{~~') 2 tly = ne 2Y dy.
The required volume is lhcrcfore

-JT (e-' - 1).


2

dy

11-- - - - -"'7-'y = In x o1x = e>'


x- -

I
I

I EXAMPL E

7. 1 0

Fi nd the volume of the solid of revol ution obtainc<l by rotating the rc:gion end ose<.l by the curves
y = x' - I 1111d y = 0 around the line x = S.

'loilCil IICJ

tine x

SOl UTION When the horizontal roctanglc in Figun: 7.21 is roUJtcd around the li11c
the volume of the washer rom1cd is

MaiQrol liU.)liUcl lltc

=5

x = 5.

-1 ~2 --x1 -l
.L'\

r,-

Since .r1 and x 2 are .t-coordinate-s of poims on the curve }'= x 2 - 1. we solve this equation
for x
:I:.JY+T. Si11cc x 1 > 0 a11d x1 < 0. we set .r1 = JY+T >llld .r2 = - .JY+i.
nu1s, the volum ~ nf the washer can be expressed as

;r [ ( s

+ JY+i)' -

JY+J) '] dy = 2on- JY+I dy.

(s -

The required volume is therefore

= /_~ 20n- JY+T dy = 20tr { ~(y + 1)3/l [ ,

Wa~her!: in F.xantple 7.1 0Me not asslraightforwardas in previousexilmples.

40;r

Forson'le problems,

w1c~hcrs

arc totally inappropriate. Conitler, for exampl e. rotating the rcgiun in Figure 7.22
around the x -axis. ~ch of the re(.;tanglcsshO\\'Il yields a 'vushcrwitha vol ume fommla different
from the others. As a result. use of washers requires six definite in te~..-als and unl y one of these
is easy to set up. Determination of this volume seems to lend itself to the usc of horizontal nithcr
rhan vcn ical rcc1angles.
Vr.!ume (,y

: x=/(y)

'
To see that this is indeed true. we divide the interval a

a = Yo < Y < J2 < <


In each subinterval

}'i -I

Yi- t

+ Yi
2

.1'- < >'

!S y $ y;. we tj od 1.he midpoint

Yi

b into 11 pans by the J)Qints

= b.

.:JCtliJ,I 0 1 M

App1u:\i11mtion oi voh1me of

Sc.llil of te\d lltion by t ylindrical ~hel h

and construct a rectangle of length / (yj) - g(y,) and width y,- y1_ 1, as shown in Figure 7.23.
When this ; "' recmng le is rotfiled around the x -axis. a cylindrical shell is fo rmed. Si nce the
length of the shell is / (yj) - g(yn . and its Inner a nd outer radii arc )'J - t and)';, respectively,

its volume is

(try~ - rryJ_ 1)[/(y,') - ~(yt)] = rr( y;

+ )'1- l) (y;

= 2Jryj ]J (y;") where f).y;

- .Yi-t>l / (.v,) -

g(l~)J

~(y;J ]

f).y; .

y; - Ji - 1. When we add the volumes of a ll such s he lls. we obtai n an approxi-

mtltion to the required volume V :

L 21f)'/ 1/(yiJ -

g (y,) I b.y;'

i:l

IJ we l_et the nurnber of rectangles g.e~ larg,er and larger. and at the same time require the widths
tO get s maller and smaller. it seems reasonable to c,<pcctthat approximations will get beucr !Uld
bellcr. As the norm ))t.y d) approaches zero, the limit should yield what we think is V. We
therefore defi ne the required volume as

li m

11-"
. 11-0

..

L 2rryj[.f(.v,"> - ~(yt)Jf).y,.

t7 10 1

Butt he right ~ide o f thi~ definit ion is the definite imcgral o f' 2rr y[f (y) - g(y}) with respect to
y from y = a toy = b; that is,

"

2ny[f(y)- g(y) ] dy =

lim

L 2nyjlf(yi) - g(yj)]6y;.

l dY; II~O .
1=1

426

Chapler 1

ApplkatiQnS of the Oc:llnitc hue:rnl

Consequently. the volurne of the solid of (evolution genera1ed by

ro 1~ tin g

che region in Fig

ure 7 .22 around the x-axis is given by the definite i megrnl

1b

V =

2ny[j (y) - g(y) ]dy.

(7.11)

"
l 11 practice we develop the intcb,rntl in 7.1 ] by drawing a rectang le of width d y at position
y as shown in Figure 7.24a. When this rectangle is rotated around the x -axis. the vol ume of the.
cylindriC<ll shell generated is a1>1>rox imately

2ny(f(y) - g ( y)]dy.
We obtain this by picturi ng the shell as being em along the rectangle and opened up i nto a sl ab
with dimensions dy, f (y)- g(y), and 2rry as i n Figure 7.24b. Thickness d y of the shell
corresponds to the thi ckness or the slab; lengt h / (l') - g(y ) o f the shel l cotresponds to llull
of the slab; and inner circun1ference 2rr y of the shell corre~ponds to the width or the slab. Jf

we now add the volurncs of all such cyJinclrical shells, aJlcl m the same time take the lim.ic as
I heir widths approach
the dcfinitc intcgral

1ero. we obtai n the required

Tt fo llows that
v =

vol ume. Bul this l i mit summation defines

2rry(f (y) - g(y)]dy.

1h

21l)>[.f(y) - g (y )]dy.

"
The method Llescibed is called the rylindrical shell m ethod for find ing the volume o f a soliu
of revolution.

MjiHil;lii)ElifM
up integruJ

Using differentials to

f~>t <.)'Lit Kiric!~l

Mjtflll;lilfM'ti:W

$e1

.v
b '"~ ..

Volume of <.:yt inclri<.~Jt

shell or thi<;kocss d y

sbc:lls

-----.-

1---.,.f(.Y)-gl y)

---1

dy

I
.X

I EXAMPLE 7.11
Find Lhe volume o f the solid of revolution when the region endosed by x
y -axis is revolved around the x -axis.

= 2y -

y 2 and Lhe

7.2

\\)l urne~ofSvl idsofR~'olutiolt

Vulumc when :u'Ca j;,:

427

EMim:nc oi \'Oiunlt oi
~olid

\ll' rc\'Qiucion

)'

( I , I)

>

SOLLTfON Thcvolumcof thcc.ylindricalshell formed by rolating therectangle in Figure 1.25a


a.round the x -11xis is approximately
( 2Jry)(x)dy

211y(2y - /) dy

211(2yl- yJ)d.v .

Hence.

1
2

2
2rr(2y - /)dy =

3
- y''4 J0' = 211 (~4) =
3

2
211 { '
3

811

We can check tha t this is reasonable by rotating the rectangle in Figu re 7.25b around the x axis.
TI>e cylinder su generated has vulume 11(2) 2 ( I)
4Jr . That the aoovc volume is two-thirds of
lhis is acceptable.

I EXAMPLE 7 . 12
Usc cylindrical shells to calculate the volume in Example 7 .10.
SOLLTION \Vhen we rolate lhe rectangle in Figure 7.26 around the line .r = 5. lhe volume
of the cylindrical shell is approximately

2Jr(5- x)(O- y)d.r = 2Jr(5- x)( l - x')dx = 2Jr(.r 3 - 5x 2

+ S)d.r.

Total volume is therefore

'
1
_,

= 21r

2Jr (x 3

s,v1

+ S)d.r =

~' - -5~- - x2Jr ~ :....


4
3
2

(!4 - 3~ - 2~ + s) - 2n (~4 + ~3 - 2~ - s) =

TI1is is the result obtained l>y the wushcr method in Example 7.10.
JIIli:~flii.EI)I

-I

Volu.ne wtJen

CI'CII

is rotaed around the line " - 5

+ s.,. } '

40/T

_,

42S

Ctupte' 7

A pplk":u ioru of iht:. Odi:ni 1e In !{gr.ll

Even though the region in this example is ~)'mmetric about the y -ax_is. the volurue gen erated
hy the righ1 hulr is l~s than that generated by the left. For this rcal)on we c~Uli\Qt integrate

from x

= 0 to x = I nnd double tlle resul!.

If rotation were performed about the x -axis. we

could indeed integrate over cithc:r hulf uf thc intcC\'otl a nd double. Finally. fOC' rotu.tion abo ut tht
)'-axis. we would integrate over only o ne..hulf o.f thc interval and neglect the other half in order
to elimin:u e du pl ic~lti ons .
TI1e volume in Example 7.12 was also calculated with washers in Example 7.10. The
volume io Exampic 7. J I could also be done with washers. but not so easily. T ry it. Was hers
cnnnot be used in the following example.

I E XAMP LE

7 . 13

Figure 7.27 s hows a plol of the fun('t ion f(x ) = x - 1 sin.:r on the interval n/2 !: .x .::; n .
Fi nd the vQiuruc o f the solid of revolutiOI\ when the area bounded by this cune and lhc lines

x = tr/2 and y = 0 is rou11cd round the .raxis.


D3l 1J.. iiflEEII

SOI .L"TION \Vhcn we rolatc I he rectangk: ~ how 1 1 a rQund Lhc )'axis. the volume of the representarive cylildricJ\1 shell i~ a p ~)foxima~ cl y

Vohune

\\hen area i~ ml.ncd umand lhc:

y aXiJ.

x)

sin
2n (x ) ( - - dx
.

)'

0.6
0.4

""z={

sin x

The total volume is therefore

0.2

V = f :r2nsinxdx= 2Jr

J~/2

;;

2Ir sin x J.r.

I-

COs.T }"
.'t/2

2Jr.

Equations 7.9 and 7.11 provide two methods for calculating volumes of solids of revoluti011:
washers <lnd shells. Where applicable, both methods give the same results: as they should. We
ill ustnue this for the region bounded by the curv.cs in Figure 7.28. M indicated i11the figure. each
curve dcti nc.s y as a (unction of x. and x as a fu.mction of)'. \Vhcn this region is rotHtcd around
the x -axis, cyl indrical she lis can be used to calcuhue the ,olume of the solid or revolution.
II=

1
d

2try[q(y) - p(y)ldy = 2n

)'q(y) dy - 21r

,.

)"

y = g (x)

or
X=

1J

q(y)

1"

)'fJ (y)dy.

(7. 11)

Suppose we make thechange of,nriable ,,

q (y) in the first imegral. When th.i s equa1ion

is solved ft>r y in terms of x , 1he resuh is y


g(x). and theref'ore dy
x = 11 when y = c and x = b when y = d, we obtain

d yq (y) ely

= g' (x) dx. Bc~'llw;e

lb

g(x)xg'(x) dx.

"

Now 1he product rule for c.liffcrcntialion gives

and therefore
,
I d
I
2
g(x)xg (x) = - - lx[g(x )]2I- - [g(x)] .
2dx
2

So

yq(y)dy

1" {I
2

{~x[g(x)f lb- ~ 1b lg(.r)lld.r


2
fa 2 o

-I blg(b)l2

2
2

Similarly, the change of variable x

<

I
2
= -1Jd

- d
-d {.r[g (x)fj- -[g(,r}f dx

yp())dy

2 - -I
I
-a[g(a)l

11b

1''

[g(x)f dx

- ac - [g(x)l dx.
2
2

p(y) o.n 1he second imegral ill 7.12 lends to

= -2 M 2 -

11

1
2
-ac
--

[f(x) )2 dx.

Substitution of these into 7. 12 gives

V = 21f

2
I
-bd
{ 2

1b

- Jr

1b

-I ac2
2

[g(x)]l d.<

"

-I

1''
1b

2.

+ Jr

[g(x)J' dx

} {

- 21r -I t11P - -I ac
2
2

-I

1b

2.

[f(x)] dx

[f(x)]l dx

{1r[f(x))2 - 1T[g(x)J2} dx.

"

But this is the integral obtained when the washer me.thod is used to find the volume.

EXERCISES 7.2

In Exercises 1- 12 use the disc or washer mcthcxl to find the ''olume of


the solid of re1<olution oblainod by rotating the region bOunded b)' the
curve~ uboul Ihe line.

1 .f

+ y' =36,

3. y
4.

= x' + 4.
y2

X -

5. X -

= 2x

= 16.

X :

20, about X

1 ::::::: )'ll X _,

5, about X =

=0

=Q
I

.r = 0,

6 .f + y + I = 0, 2y = x - 2,

about y = 0

7. y=4x 2 -4.t , y:x>. aboul y=-2

8. .t

= 1y -

y1 -

2. x

= - 5,

9. )' = 5 -X, X = 0 .
10. )'

11.

.r

12 . .Y

= .t 2 &

2 \', )'

cscx~

)' =

= In (X+ 1).

about

about X

=0

=6

= l.r - :c 2 , aboul y = 2
0, .r = n / 4, x = 3it/ 4. "boul ,. ~ 0
)' = L. X = 0 .

- .X\

= 0. )' = 0 ,

i.
!*

JJ. _r=.\'3 - x. y=

35.

ih

36. y

y=

16. x
11

y = 2~, y

.r = 3.r- x'. '' = x'- 3.r.

=0.
about

= i.

19.

20.

y = x 2 ,r = - x 1 x

21 .

y= - /9 - x. x=O. y= O.

=-

23. y

=0 .

(x + l)y

=sinx.

24. y = 10 - xl. xly = 9.

./X-1
= J4- x . .\' = x3+ 1.

* 38.

the \'Oiumc of air di:,.plucx:d.

(a) l fa.phcrcofmdiusr issliccdadiSianceh from its centre.

39.

.show 1ht11 the volume of the smaller piece is

(b) Use the result in pan Ca) to find the mlio of lr tor in order
th<~t tbc ~mallcr piece will have \'()lurnc cquuJto OIH>L
hinl
of the sphac. Gi''C your answc;l' 10 four dec-imal places.

(a ) Find mcubic polynomial )'

aboui)'=O
alx..t . r : 0

oboutx"' - I

abouty = O

Cn)

about X= 0

29. y

x = -2, x = 2. y = - I .

about y = -I

2, y = 0, about)' = - I

30. x=J4+ 12y 2 , x- 20y= 24, y=O. ilbOut y=O

= a,tl + bx 2 + C.A' + d LO fit

the thn:c point$ (0. 0). (4, 2), nd (6, 0).


(b) Deacnninc the anlOUm of fill required 10 bui ld the C.lllbtmkmcnL.

l. about x- - I

.1'

abotll x = 0
= x2 -

V = - (r- h ) (2r +h).


3

~o emOOnkmenr is 10 be huill around the circular wadins, pool in


figure (a) below. ..lgure (b) shmYS a crossSCC1icm of lhc cn1tmnkmcnt

)' = (X - 1) 2 - 4, 5y = 12X, X = 0, (X ?. 0). about


x=O
27. x 1 - y 1 =5, 9y=x 1 +9, 9y+x 1 +9= 0, (-3~x~3).

lx' - 11.

=0

* 40.

* 26.

28. y =

about y

During one n:volution an .Urplanc propeller displaces an amount

In E:<.ercises 25- 32 me the most appropriate method to fi nd the ' 'olume


o f the solid of rc,olution obtained by rotating the region bounckd by
the cun~s uboutthc line.
25. (x 2 + 1) 2)' ~ 4, y =I .

= 0.

of uir thnt can be cakularcd as a volume of :J solid of rcvolulion. If


the region yielding the \JoJumc is that bounded by the c urves 64x
y(y-4) and 6<1x
y(4 - y) .andisrotatcdaboutthcx -axis, e<~ culate

=0

x =4

0 ::': x ::': 2".

aboot y=- 1

.T

about y

22. y =X . X.'' = 9. X+)'= 10. (X :2: .1').

about y =-1

, y = 16-.r 2,

about X = 0

lx 1. y = 0 . abooL y = - I
.1' = J2 - x. )' = 0, about y =

18. y = 2 -

aboutx=O

A tapered rod of lcoglh ~ has circular c rOSSSCCiions. If lhe radii


of its ends arc a ;tnd b. "-ildt is the \"Oiu.mc of the rod?

+ .r = 4,

./i,

* 37.

14. y = - /4 - x, .r = 0, y = 0. obout y = 0
15. )' (x - I ) 2, y
I. ;~bout .r = 0

about y = - 1

34. J=e " . y= 4-x 1,

about X = 0

Ill Exercises 13-24 usc the cylilldrical shell method to lind the volume
of th1.: solid of t'c ,o huion obtained by rotating the region bounc.lcd by
the curves abou1 the line.
13. )' .- I

3, y = 0,

In l-crcises 33-36 acalculatororcomputcr is needed to find the poinLS

abo<~t y

= x -

of intersection of the curves. Find the \'Oiume of the solid of revolution


when lhc region bounded by thccur.cs is rotated about the line. t.-orrccl
to three decinlal places.

about x = 0
2

y=(x + l)t/ 4, y = -(x + l )', x=O. aboul x = O

32. .

=0

about y

2. y' = 5 - x. x = 0.

* 31.

I '
1- - - - 6m- I
-

4m - l
(b)

>tt

JL Find the volu1nc of the donut obt ai~lCd by rot:tting the cirdc io the

figu.rc below about the y-axis.

I
L

t-- . -1

a- b u

** 44.

a<-l!X

'

A right cifc.ul(U' cone of beigln H and bi1se rudius r has its ..erte.\
at the centre or a sphere or radius N r N < H ). Find that pa11 or the
volume of Ihe sphere inside the cone.

** 45.

Devise a way to calcul:ue lhe \'Oiume of 1hc solid


when the region in the firsl q u:~elrnu l boun ded by

++ 42. A cylindrical hole i.s borod lhrough thecCI\t.roof a sphere, the lcnglh
of the boJc being L. Show that no matter what lhc n:~diu~ of thc. ~phcn.::.
the \'Oiume of tbe sphere that remains is alwa)'S the same and cqu"lto
the volume of a SJ)here of diameter /....

**

43. \V:uer haJr tills :1 cylindrical pail of r.~ius o and heigbt L. When
tile paiJ ls rotated about its axis of symmetry witb anguJar SjXed CtJ
(figW"C following), tbc swfacc of the water assumes a patabolic shape,
tbc cross-section or whicl1 is given by

y = I - x 2,
is rotated about Lhc line y ::: x

= 0.

+ I. Him:

<f rt\'Ciution

)' = 0
Distance fonnuJa I .16.

46. A sphere. of icc crcrun is to be placed in a cone of height t unil


(figure.bch.lw). 'Vh~ r.u.lius of 1hc sphere ghcs the ll)()SL voluulCof icc

cream inside the cone (as opposeO 10 abo"e the cone)


ha<e angle 2(/?

lOr a cone

with

where g > 0 is the acceleration due (o gravity and H is a constnnl.


Find the !)peed w. in tc.rms of L.a. and g . a t which water ~pHis over
the tOJ>.

l7.3 Lengths of Curves


Formula 1. 10 defines the len!,'lh of the straight-line segment j oini ng two points (x 1, Yt) and
(X!, )'2) . l11e fonnula s = riJ gives the length of Lite arc o f a circle of radius r sublended by
an angle 9 a1 the cenlre of the circle. In this section we derive a result that will theoretically
enable us to find I he length of any curve.
FIGURE 7 29a

of a

Finding lhe length

Appm:;:imatint the lenglh of<)

MiirliiJMJf110:J
I:W\'C

CUI'YC

by

~ghl

line

:;cgiiKOI!:>

y=f(x)

B<b. J<b))

A(o, f(o))

A.=Po
X

x,_ ,

Consider findi ng Lhe length or the t une C y = .f(x) in Figure 7.29a joining points
A (a , f (a)) and B ( b , f (b)). To find i ts leng:th L we begin by approximating C with a series

432

Cb<lpltr 1

Appt~1iorn of Lhe Oefini1e l ntt~r:IJ

of straight-line segments. Specifically, we choose 11 - J consecutive points on C between A


nd n.
A = P0 . Pt. P2 , . . , P, _,, P,, = B ,

and join each P;- t to P; (i = I. .... 11}by means of astraig,ht-linc scgmc.nt, as in Figure i .291>.
If coordinates of P; are denoted by (.r 1, )';), then the length or the line sewncnt joining P;- t
and P; is

liP;_, P;ll = }(x;- X;-,) 2 + (y;

- Yi- t)2 .

For a l~e number of these line se~:ments. it is reasonable to approximate L by the sum of the
lengths of the se.g me.nLc;:
n

11

i- 1

i= l

L li P;_, Pill = L .,f(x,- x,_1)2 + (,r; -

_,.,_,)2.

In fact, as we increase 11 and at the. same time dec-rease the le.nglh of each segment"$ we expect
thcapproximalion to become more <:~nd more accurate. \Vc thcrcrorc define
n

lim

IO..t., 1-0

L liP;_, P;ll =
i=l

where l:J..t; =X; - Xi-

t =

lim

116-'rll~o

L }(x; .

x;_,)2

+ ()'; -

}'H)2,

(7.1 '"'

J:l

When we note that J- t = f (x,_t) and )'; = /(x;) . we have

"
L
j(l::.x;}
l "-'11-o

lim

+ 1/(x;)- / (.r;_ 1)12

I=J

t ;,

~m

+ [.f(x;)- /(x;-o) ] 2 ~x;.

I~Ki u~o r=
. l '

(7. 13b)

D.x;

In order for 7. l3b to be o useful definition from the point of view of c.aJculation4 we must find
a co,wenient way 10 e\>"3luale the limit summation. To do this \ve assume that f ' (.r) exists at

every point in the interval 11 ;:: x ;:: b. and apply the mean value theorem to /(.r) on each
s:ubinrerval .T;- 1 !:: x :S .r; (f igure 7.30). The lheorern guamm ee.~ the exi~e nce or a1le.:.1st one
point :cf between x,_ 1 and :r, such that

f(x;) - f(x;- o)
f(x;)- /(.r; - o)
(x,) =
=
.
x; - 'i -1
llxi

Consequentl y. using these poims

xt.we cru1 express t11e letlgth of C in the fonn


n

L =

lim

L/

~~~1-o.

1+

[/'(x;>J' D.x1.

l=l

Mta~ \afnc lhcorem npplicd to

r lillt stgllltnt

(7 I ~ )

7.3

UnJihS of Cllr\'eS

433

But the right side of this cqumion is the definition of the definite integral of the function
lf'(x)F with respect to x from x =a to x = b:

Ji +

Thus we can calculate the length of C by the definite integral

b'Fdy)2

= ,r1 + l ;t J dx.

17.15)

In this o.lcrivation we assumed that f'(x ) was defined at each point u1the iotcrval a 5
x :5 b. necessary for the mean value theorem 10 apply. But in order to guarantee existence of
the definite integral in 7.15, Theorem 6.2 requii\Cs continuity oft he integrand /I + (dyfdx) 2
Consequently, to ensure that the length of a cun-ecan be calculatetl by me<~ns o f 7.15, we assume
that f(x) has aoontinuou> first derivative on the interval a :5 x :5 b. When we study improper
uuegrals in Section 7.10, we shall be able to IVeaken the continui ty requ irement.

.........

I EXAMPLE 7.14

Fimlthc length of the curve y

;e..._
CW\'t

SOLC1101\ Since dyjdx = (3/2)x 11Z. equation 7.15 gives

t.cnvb ol

y = .vl'! froan :c = 0

=.1 3/ 2 from ( I . 1) to (2, 2J2) (Figure 7.31 ).

10

x = ~

L= {

1-t-Gx'0Ydx= [ Jl+ ~xdx


I

= -(22112 - 13V 1)
27
.

The result in equation 7. 15 is clearly m eful when the equation of the curl'e is expressed in the
form) = j(x) If the cur\e is defined in the form x = g(y) (Figlll"C 7 32). a similar am1lysis
gi\'CS

e t(i)

Lc:OI!Ih
ui Q.a"'C with ~WIIi\'111 il'l fm
f

= ~1.))

For example. we rna) writetheeqution <>fthe cune in Example 7.14 in the form .1
in which case dxfdy ~ (2/3)y- 113, und
I. =

!.I \II + (2 )2
2./i

- 1 2./i
1

dy =

-y-1/)

4
\

+ 9y21l
9y:!Jl

d\ .

J1 \/t'"" 9~y-ll>dy

!,2/'i
I

1./i

.j4

+ ~y liJ
Jy l/3

_v,

lf we set u = 4 + 9y 21J,then du = 6y- II3J y. und

l.

= JI

!.

12

1.1

du
.jii6

22

= -18I { 3-2 ul/2 } 13

= -1 (223/2 27

I Jlfl).

y l/1

'

"

MiUUIIshiiii.S.

UsiPg

di fftl'l:~mlah.

inJC~.~I

10 SCI up

fOI' k:ng.th

ur a C-llf\'C

y =j(.\)

Tv

. :)
.. ..

,,

-:dx:., ..

x x+dx

/)

lntcgrnls 7.15 and 7.16 can both be interpreted geometrically. This interpretation will be useful
\\'hen we $tudy pardJ'l'letric equations of cur\'eS in Section 9.1 and line integrals in Chapter 14.
Figure 7.33 shows the curve C of Figure 7.29a. r\1 a point (.t. y) on C. we draw the tangent
line to the curve. If dx is a short length ulor>g the x -axis at position .x. the length of thm pan
of the tangent line between venicallines at x and x + dx is given by
../(dx)2

+ (dy )1.

This kngth along the tangent line closely approximates the length of the curve between the
same two venicallines. the approximation being more accunue thcshoncr tltc length dx. If we

picture a large number of these t<mgemialline segments between a ruld b. we can find the total
length of C by adding togethe-r all suc h le ngths and taki ng the limit a.' each approaches zero.

But this process is represented by the definite integral , a1td we therefore write
x=IJ

L =

!,..,,

J(dx )2

+ (d)')2

L =

or

>~

../(dx)2

+ (dy)2.

(7.17)

y c

The first integral is chosen when it is more conveniem to integrate with respect co .x . in which
case dx is wkcn outside the square root. We get

f.

{b /

f. \ I +

(dy)z
tlx
dx.

which is equati on 7 .15. The second corresponds to a more <.Otwcnicnt integral with respect to
y. in which case d y i::: taken outside che s:quMe root \Ve obLain

L
which

I E XAMPL E 7 . 1 5

i~

1+

=[

(dx)
dy

d)
..

equation 7 . 16.

Find the length of the curve 24X)'

= x + 48 between (2, 4/3) and (3, 43/24) .

SOLUTION It is easiet' to express)' in terrns of x (rathel' than x in tenns of y).


y =

x4

+ 48

24x

x>

-24 + -X .

We rtow use the fac11ha1 small h:ngths along the curve arc approximated 1.>~

+ (dv)
,1 :~ dx = \

(x' 2)1dx

.::IIa. c !miJ1 l.c:ngth or


<.une 24.vy = ,\'.. + 4~ fron1 .1:. = 2

ra .x=3

)'

j I + -x'6.1 -

+ i" -

4
:;I + --;;
dx =

= - 2 J (x' + 16) 2dx

{ I
1, - -1(., .s + 32x'
\ 64.<

+ 256) d .1

= .r' +, 16 dx.
8x-

8.<

'Tl~e

.rl

tot I length of the curve i' therefore (Figure 7.3~ )

= ~8 {xlJ - ~ I' = S"


9
X

EXERCISES 7 .3

In Exercises 1- 10 lind the length orlhc cunc.


I.

8x'

=)"~ from ( I, 2) 10 ( 2_12, 4)

2. 3y = 2(x 1

3.

14. x' -

+ 1)112 from ( - 2. 10./5/3) to ( -

= 2(y - 2) l/l

IS. y = sinx from (0. 0) to (n, 0)

16. y =In (cO$ X) bclwccn the lines x = 0 :md x = tr/4

I. 4_12/ 3)

17. _r =ln.\' rrum ( 1. 0) lo!lnyolhCrpOi!llOnthccurvc

f10o11 (0, 2) tO (2. 3)

4. y = (x - 1)'/' fro m (2, 1) to (10, 27)

x1

5. ,. = .
4
6. y

7.

+ -3.r1

.r =

8. 36.ry

10. y

3.v'

19.

x = y' - 2)' from (0, 0) to (0. 2)

20.

x' {4 + )'' {9 = 1 Coomp~e length)

+ -,from (1, 129f;2) to (1 /2, 33/ 64)

Sx

= x' + 108 from (3. 7/ 4) to (4, 9 1/36)

y'
I
9. x = -20 + -71'

bCI.wccn the lines \' = - I and)'= - 2

= x'fS + l/ ( 12x' )

= x2(1- x ' ) (eoonpletc length)

18.

from (I. 7/ 12) lo (2. 13/ 6)

= (I!"+ e-' )/2 between Lhe lines x = 0 and .x =


x'

.v' = I fro on ( 2. - ./3) Lo (3, 2./2)

21. Find the lc11g1h of the curve 3x = jY(y - 3) from ( - 2/ 3, I) to


(2/ 3. 4) .

* 22. Fi1kJ the lcnglh of the cuf\'C x'll) + )'V~ =


*

23. I r n is w1y number other lht1n I or -I. and 0 < tl < b, find the
lcngUl of 1hc curve
.t "-t 1

)' = - -

"+1

from ( I, 17/60) to(2.30n/480)

be-tween
In E"<crci.;c.s 11- 20 :O:CI up ( htll do not (\'lllu:nc) a definit.c inl.cg.ral ror
the length or tJ1c curve.

= .t' fro111 (0, 0) to (I, I)


y = Jx' - 4x from (I , -I) to (2, 4)
x' - y 2 = I from (I. 0) to (2 .J3)

X=

13.

+ .,..,..----..,.,..--,,.,.
4(n - l )x -

a ;md .t =b .

* 24.

lf 11 is any number SR."'ii!Cr thlln 1/ 2. a n~ 0


!cngch of the curve

II. y

12.

1.

)' --

x i~ +

4(2n - I)

between x =a and x =b.

< 11 < b, fi nd the

+ -::---:-,.,...-;::-:-7
( 2J1 + l)x~ -

17.4 Work
When a body mo,c< a di"ance d along a stmight line I under the aceion of a constant force F .
which am in the ~me direction as the motion (Figure 7.35), the worlc done on the body by F
is dcfi ned ""
(7.Jg)

IV= Fd .
OefiDJtion of \lt'Od: done

:J.

b)' a co, ,.:u~nt

fM~ .:~lona

<lone by n \ olric.hlc force

o qN1&}at line

vj

Direction
Q(morton

r1

.r,

x.

b : .r,.X

Oirec1ion
ofron.-e

Forc.es oRrn ..ary cllher in magnituUe or direction., or both, and whe.n ~c:h forces act on
the body. calculacion of the work done is 1101 as simple "' u'ing equation 7.1 8. ln this section
we u>e dcllnite integral; toculculmc work done by forces !hut alway) act along the direction or
motion, but do not have: constant m~:1gnitude. SpccifiClllly, let us con~idcr u particle thai moves

along the x -a~is from ,f


tl to x
I;. where b > a. under the action of some force. Suppose
the foccc alwuy ttelS in the positive XdircctiOJl, but its >i:te, which we denote by F (.r). is not
constant along !he .r is. To find 1hc work done by 1hc Io ree :ls the par1icle mo,es from x = a
to x = b. we cannoc simply multiply force by distance. [Where would we e\'llluate F (x)?]
ln~>~cad. wedhide the intcr,al a ~ x ~ b inl.() 11 wl>imcf\als by point> (Figure 7.36):
a = .to <

In each ;ubinlc:n.d

-'t-l :;

X1

< .t1 < < .r,. _. < ..r,. = h.

x :; .t;, \\t. choose a poin1 ,,,. If lcnglhs of subintenals arc

small Md F (.t ) i<eonlinuou.~. !hen F(.r) does 1101 '""Y S"'tly O\'Cr any given subintenal. In

such a si1ua1ion. \\C may approtima1e F (x) by a coostanl foree F(tj) on each subinten"al
.<;- 1 :;; x :;; .<t h follows 1ha1 work done by !he force ovenhe i"' subinlernl is approxima1ely

F(xt)(x1

x1_ 1) = F<xi) llx,.

Furthcr111ore. an awruxintation to the total work done by the force a~ the particle moves frum
X

= a 10 A = IJ ;,

I:" F<xn t:.x

/: I

As " become larger w1d each !:>xt approaches zero, !his approximtion bccoi11C5 bcucr. and
we 1herefore define

17 19 1

Bull hi< limi1 n13) ai'O be imerpreted as !he definile in1egral ofF(<) \\11h respec110 x from
x = a 10 x = b. Conscquemly. IV may be calculaced wi1h !he derinite imcgt:ll
W =

1b

F(x)dx.

(7.20)

It is simple enough to inll!rpr~t pans of the integral in equation 7.20. and in :so doing we begin to
feel Lhcdcfinilc in1cgral m work. The integrand F(x) is the forccal l>osiljon x. and dx is a small

dis1ance along the X axi>. The produc1 F (x) dx is therefore inlci'PI'etcd as !he (approximale)

7.4

Work

437

work done by the force a long d.t. The definite integral then a dds over a ll dx's, bcg_inning at
x = a and ending, at .x = b , to give tot~ I work.
Wlmt is important al>out equation 7 .20 is not the particular fonn of the definite integral, but
the fact that work do ne b)' a force can be evaluated by means of o definite integral. As we $01vc
work problems in thi s section, we find th;~t the form of the definite integral V'Jrics considcrnbly
from problem to proble m. but the underlring fact remains thlll each problem is solved wi th a
definite integral.
With lhc exce ption of work problems thnt in\'olve emptying tanks (Ex"mplc 7.18), we
recom me nd that you ;1lway~ malie a diligram i Uu~t rdting the phy,ical ~et u p a 1~ome imcrmeditue
stage between start and finish. Determine forces at this position in order to set up the work

itnegr--Jl.
In the derivation of 7.20 we as:,umetl Cllnl inuity of F (x). anti this guarantee< existence uf
the dc.fi nitc integral.

I EXAMPLE 7.16
Find the work neces!!>ary to e-x pand a spring from a strelch o f 5 em 10 a s1retch of 15 cnt if a force

of 200 N stretches it I0 em.

SOLl'TTON Let the s pring be s tretched in the positive xdirection. nd le t x = 0 corresi>OIId


to the free end of the s pring in the uns trctchcd position in the top half of Figure 7.37. In the
bottom half of this figure we have shown the ~pri ng s Lre rched co an i mermedime posilion.
j;mm:;J

d.t

-I IO: O.OS .r O. tS

'

According to Hooke's lt~w. when 1he spri ng i.~ srre1ched an amounc x . the re~tori ng force i n
the s pring is proportional to x:

F_, = - kx ,
where k > 0 is ~t constant. The ncgati\'C s i.s:n i 1tdi catcs that the force is in the negative X
di rectit)n. Since F,
-200 N when x
O.D O 111.

-200

-k(O. IO).

and it fo llows that k = 2000 N/m. The lbr.:e required to counteract the restoring force o f the
spring whe n it is slretched ar'l ::tmount .r is therefore

F (x)

= 2000x .

Work do ne by this rorcein stretching the s pring front positiOt x a further amount dx is approx
im a tcly

2000x dx,
and hence total work to increase the s tretch from 5 e m to 15 em is
0, 15

W =

o.o;

2 }0.15

2000xdx

= 2000 { :....
2

= 20J.
o.o;

438

Cl~oU~Ptcr 7

AppH~3tiuos

uf l_bc: J)cfinitc lmcgr.J:I

We could check on 'vhctbcr this answer is rcasonabl.c as follows. The minimum force during
stretching is at the beginning when the stretch is 5 em where F = 2000(0.05) = 100 N.
Maximum foroe is at 15 em where F = 2000(0.15) = 300 N. Work should be bet ween
I 00(0. 1) = I 0 1 m1d 300(0.1) = 30 J . In essence, we are applying Theorem 6.6 from Section

6.4 .

...-...

I EXAMPLE 7.17
A cable hangs venically fro m the top of a building so thm a length o f I00 m, having a mass of
300 kg. is hallging from the edge of the roof. What work is re<:)uircd to lift the cmire cable to
the lOp Of the building?
SOLUTION When the cable bas been lifted to an intemcd iate po int w here its lower end
is y metres above its original position (Figure 7.38) , the length o f cable sti ll hanging is
I 00 - y m. Since each metre o f cable has mass 3 kg, the mass of I00 - )' m of cable is
3( I 00 - )') kg. h follows that lhe force that musebe exesced to overcome gravity on thi.!> much

cable is 9.81(3)(100 - .r) N. The work that this force does in raising the end of the cable an
addi tional amo um dy is approximately

9.81(3)(1 00 - y) dy,
and total work to raise the entire cable is therefore

=[

Mi11Uil;l4dlili8

Building Ill

00
9.81(3)(100 - y) dy = 29.43 { JOO.r -

-~ Loo =

147 150 J.

Work to raise a c:abJc

dy

tlnitial posilion of end of cable)

I EXAMPLE 7 . 18
A tank in the form of an invened right circular cone of depth 10m and mdius 4 m is full of
water. Find the work requiJ'ed to pump the. wate.r to a level I m above. the. top of the tank.
SOLUTION

A cross-section of the tank is shown in Figure 7.39. Suppose we approximate

water in the tank wit.h circular disc-s formed by rotating the rectangles shown around the }'axis.

7.4

M;;ll#lii.IWAIM

\\i)r-1;:

U9

Wod: tu empty a litn.k

I
10

t- -- 4 ,

(4,10)

'"\
- -d-;-,Y-ll- 1

~~~ ~:tx
y

The force of gravity on lhe represenuuive disc at position y is its volume 1r .v'l d y, multiplied
by the density of water ( 1000 ks,lm 3 ), multiplied by the acceleration due to grJvily ( - 9.8 1),
-9.8 1(I OOO)nx 2dy.

h is ncgathc bc.x'.ausc gravity octs in the negative }'direc tion. The work done by (tn equal and
opposite Ioree in lilting the disc to u level I m above the top o f the tank (a distance I I - y) is

The total work to empty the tank is therefore

10

IV =

o (JJ

/.

To express x in tenus o f)' . we note that x and )' are coordinales of points o n Lhe straight line
through the origin and the point (4, I 0). Since the equation of this line is y = 5x /2. \\C obtai n

10
lV

98 10n /.

( I I - y)

MijlflillfiloM Wo1lz lo
ernp1y a tank
)'

10

dy

J569.6rr

(l ty2 - yJ} tly

- L' 1
4

= 5.75

106 J.

observations are r't04.eworlhy:

I. Tn E,~tmplc.~ 7.16 and 7.17, the d ifferential n:prescnl~ distance moved and the imegrand
represents force. a di reet application of the discussion lcadi11g to equatio11 7 .20. rn Example
7.18, the ,liflt:(elltial is pa1't o f the fon.--e. a_rld the distance moved is part of tho integrand.
This. is why we suggested earlier 1ha1 i1 is not advi.sablc to use equation 7.2.0 as a formula.

1he integrand is not always force and the differential is not always distance lllOVed.
2. ln each of these examples we set up the coord inate system~ it was not given. We cao use
any coordinate system w h atsoever~ but once we have chosen our coordinates, we must refer

l-4-

everything to that system. For instance. were we to use the coordinate system in Figure
7.40 for Example 7.18. the de~nite integral would be

dy

10

10

II
= 156\1. 6;r { ___r_
3

1\\'0

C;')

IV

(! )

= _

10

- 10

( I - y)98 11l1r

[2

S(IO

+ y)

]2 dy.

Evaluation of this detinite integral once again leads to W = 5. 75 x I0 6 J.

O l:tp$C1' 7

1\r)plic;:uions of tbe DcHnilc. hllt:J.!f:l

Heat Engines
Jn the Application Prcvicv.. we introdu,~cd the problem of con"crting heat to mcciU\1'ical C1\ergy,
With the law-. of thennodynarnics. we can ~tud)t lhis conversion in hern engh\CS. Figure 7.41

Application Preview
Revisited

shows gas in ncylinderclosed on Olle end wi1h a pis1o11 on l.he o1her. The force on 1he pis1on face
due 10 1hc pressure of 1he gas is F = P A, wheo'C P is 1hc pressure of the gas in 1he cylinder
and A is 1hc area of Ole face of the pis10n. If lhe pis1on is moved to the right a small amount
d.x by the 1>re.~sure of the gas in the cylinder. the work done is F dx . As the pis1on face moves
from position x 1 10 position x 2 the total work is
IV

L.,

P d.r

AI

Cylinder

x=O

&n! li

IF=

Pis1on

dx

IV =

!.

v,

(7.2 1)

PdV.

Positive values of this integral correspond to the situation described ~1bove when the gas is
expanding: ncgali"'Cvalues correspond to decreasing volume. Although we dc"clopcd furmula

7.2 1 <>n the basis of the cylinder and pi>ton in Figure 7.41 , it represents the work done by
expanding or compressing gases for any shape. In order to cvaluute the definite intcgml. it is
necessary to specify P as a function of \1 .
Figure 7.42 show,; the stales through whi<;h t he~ in a helot engine might pass during one
complete cycle. Begioming at poi on B and pnx:reding along C1 to A . lhc volume of the gas
in the engine is increa.<ing whi le ics pressure is decreasing. The gas is doiug positive work. the
amoulll given by the integral in 7.2 1. where I' is defined in terms of by the equation f<>r c2.
As the gas retums to !>.late 8 along C1. integral 7.2 1 defines the work done on the ga:s, The
diflcrenc'C of these imeg,rals rcprescn!S the net work done by the gas during one cycle of 1hc
cng.inc. In Section I.J.3 we imerpre1 the imcgrations a..~ line incegmls. and cominuc discussions

=rx::z 24

wo1k <hJle
by heJt el&ine fu1' one cycle

P A d.r.

A'l

Rut A dx is the change in the ' 'olume of gas in 1hc cylinder due to the change dx in x. If
we denote this change by dV . and let V1 and V2 represent the volume> of gas in the cylinder
:u posiLions ..t 1 and x2 , re.s-pcclively. then

M#lic-III,J fJ ,JJ \Vl)rk dunt'


by gas in nlO\'IIIS piston

G<>s

L''

frorn lhat point of view, For 1lOW, notice. thai subtt;_"lctio'


II

or the intcgrnls rt:pl'CSC"l t~ the area

enclosed by C 1 and C 2 . In ocher words. we can calculaoe 1he outptot of 1he engine by calcuh>ting
the tH-c<l enclosed by the curves.
The Htmk.im: c_\'clc rcprcscnu. nn ideali7..ctl steam engi ne: the pressure and volume orsleam
~uoi ng a comph:tecycle migholx: lho><: in Figure 7.43. Wmcr mlow tcmperawrc and pressure
([>oint A) is hentcd at conscant \<Oiumc (along path A 8 ). Along BC the water is convened co

<ttllln and exp;md< slightly. and the e.1wnsion continues along CD. To complete the cycle. che
s1eum is cooled nnd condensed 10 water along 0 A. If the gas is expanded adinbmically along
C D (temperaourc is held conslanl). thcn P and V are rcl;ucd by PVY = k, whco'C y > 0 and
k > Oureconscants ( y
1.4 for air) . Using point C in Figurc7.43, k
10l (0.02) "'
2
4. 2 x I0 . To decennine che output of this pnniculr e ngine. we calculate the are11 enclosed by

j j#ltclll;j..

I(}<

Jhntioe cycle (l)t' srtam

t~sine

p 8(0.0 I. I 0')
C(0.02. 10')
P\f/ = k

5 X 10"
10"

C,.
A('"'
O".O.,I,..,.
J(f
"'')

- -=- D(0.104.v tO")

0,02 0.04 0.06 0.08 0.10

the cur ves:

IV

(0.02- 0.01)(1 05

10 4) ,

0.10'

(kV- u - IO')dV

O.o>

= 900

+{

k V -().4
- 0.4

10 4 V

} (UOI

= 2.5

10~ J.

11.02

EXERCISES 7.4

1. A s pring tcquire.s u 10 N lOree to stretch it 3 cn1. Find the work. to


increase 1hc stretdl of the spring from S em tv 7 em.

10. In Exercise 9, fi1ld tf1c work 10 empty the umk through an outltl

2 m abo' 'C the wp or the umk.

2. Find Chc work to increase the sutlch ol'lhc: spring in Excrt i.~ l fn.m1

7 em to 9cm.
3. A c.age or mass J\{ Kilograms is to be lifted from the bottom of a
mine Shitt't h metres deep. If the nu~s of the cable used to hoist the

>11

cugc is m kilograms per mc1re, find the Y.'ork done.

4. A unifom1 cable of lcnJ:;t h 50 m and mass I 00 kg homgs vcrli.cally


from the top of a bul.lding 100m high. How 1nt1Ch wo1'k is required lo

gel 10

111

or the cubic on top of the building?

7. I low muchworii i ~n::qtt i red to litl the end of the chain in Exercise
5 a di ~IW1<.:c of 4 m?

11. How much work is required 1.0 lift 1he wm,ct in l hc Lrough of Exercise I I loa height of 2 m abo"c rhe top of the trough'/

* 13.

A 1\A!l ilngular swimming pool full of wutcr is 25 m long and l Om


wide. The depth is 3 m foc Ihe flrs1 I0 m of Jcngth, then d-ecreases
linearly to I m m the shallo"' end. I low llluch work is required to lower
lhc level of lhc wmer in Lhe pool by 112 nr?

5. A 2 rn chain of ttl~L>: s 20 kg lies t;m 1hc flOllr, If fric:-tilm between llOlX'


and chain is ig1101t.:d, how muc-h work. is rcq\ircd to lif1 one end or the
ch01in 2m s1migh1up? Assurnc thut Lhc ~"'USI>Cndcd portion of the chuin
makes n t'ight angle with I he ponion on the Door.

ti. H(.nv much W()ri: is rcqui red to lit\. tJ1c <:lld of the <:hai 1~ in E.<crcisc
5 Dill)' I m'?

H. The ends of a trough nrc isoscck:~ lriangles with wic.lth 2 111 und
depth I m. T he trough is 5 Ill long and it is full of warer. How much
work: is required to lilt all of the walcr to d1c top of the ltough']

f! 14. How much

work is required to <:npLy the pool of Exercise 13 over

iLs ctlgc'?

15. The force of repulsion between two point charges of like sign, one
of size q and chc other of :o.itc Q, has magnitude

M. A 5 m chain of mass 15 kg hangs vertically. It is ,cquircd 1o Hrt


the lower CJHI of the ch.ai n 5 m ~) thal il is level with dlC upper end.
Calcula1c the wotk done IISill each of the coordinate S)'Stems i n the

fig ufC below.

qQ

4:rr~ur t .

where fo is u consumt and ,. is the djsumcc bclwccn Lhe charges. It' Q


is J>laccd at the origin) and q is moved along th:: x -axis from ..r
.t
5. find the work done by lhc d cctrosw.tic lb rec.

9. A tank filled with wntcr bas the form of a pamboloid of revolution


with vcrtic:i:ll axis (figure below), If the depth or the mnk is 12 n1 and
Ihe diamclcr of the lOp is 8 ttl, li nd the wurk in pumping lhC wotcr to
Ihe top of the tank.
)'

= 2 10

16. Two posi1ive charges q , and ftz_ arc J)lnced ati}OSitions K = Sand
.t = - 2 on the ~t-axis. A lhird positi,e charge q'J is moved along the
.taxi:-; r.omx =I tox =-I . Findlhcworkdoncby lhcek ctrost.alic
fo.ccs of q 1 and lJ l 0 11 q3 (see E.l<en::ise 15).

17. fthc chai n in Exercise 5 is Stt'C(ctlc.d ouc sLraighl on the floor, and
ifthccocllicicnl of fric1ion bctwecu floor and chuiJl is O.Ol. whal wotk
is required 10 raise one end of lhe ch<tin 2 m? TilC fon.-c or fr iclion on
thou 1x1.11 or lhe chain on lhe lloor is 0.0 I tin)CS the \.\lCighl of the chuin
on tlle lloor. Assume, unrcalisLica.IJy, lhatLhe ~usptnded portion of 1hc
chain 1nakcs a right ang.lc- with the poL1ion on the Oo<.lf.

"' 18. Two similar springs. each 1 m Joog in an unstrctched position. 1\t'!vc
spring COilS! ani k ncwwns per metre. ll!c sprin.!f-':> arc jnined together :tl
P (figure below), and their rrcc ends arc fas lcncd lo two IXlS~ 4 m UIX!lt .
What work is done in moving the mitlpuinl P a distance b mcln.:s lo
the right'!
CrOSS-SOCii()n
in X)-planc

19. If lhc f<W<< .,r a <m<<l-oow lin .-1ons) i< proponi<>nallo lhc draw
(in mctn:.)). ;md it i) 200 N a.ta full drawo( SO em. what wod. is required
10 fully dro-. 111c a-ossbow'?
20. A hudel ol waler woth ma 100 kg is on the ground auacnoo 10
on~ end of a cubic with Ul.W per unit length 5 kglnL Tl~ other end
of Ihe c:Jblc i< 3110CIIOO IO 3 1VIndi3SS I00 m ab<wc Ihe huckct. If lhc
bucl.cl il) I'Ui:,cJ U.lll ~on~ant ~d, water runs o utthrmg h u hole in the;
bollom a u OOr\c;;cant r:ue co 1he e.xtcnl that the hucket wouM h:He mat(
80 kg when h rcaclx:~ lhc top. To further comJ>h<.:atc ntallct:t. a pigeon
of m;rs~ 2 kQ l:rnds on the bucket when i t is 50 m above the ground. He
unn>e~ialc ly begins tailing a ba!b. splashong wu1cr over lhe su.lc Ollhe
b<Jdc1 '" the rutc of I ~Efm. Fmd the work done by the "indl.rn in
rJi<ing the huckcl 100m.

i .-

1 1. A he:mi.sphcrical unt "-'ith di:un.etrll pl.!l.ne': Ul tht

lf)JJ

The ron-e ncrdcd 10 lllOI'C lhc pi<lon i< F = p A. """'"'


A is the on:a of 1bc r""" of the pc.>lon. If 1bc ~ius of 1bc C)tindcr is 3
1.1111s. find 1llc work dooe in mo11n1 1bc p!ston from a poont 10 uno IS 10
::a point 5 units frun lhc: ei06CJ end.
2 <OnSIJinl.

25. A tovo:ns wutc:r i). wppJicJ from the: w;,~.tcr tower bhO\o\'n below.
The 1o""" is a cylinder or length 5 m and rodiu< 3 on cnpp..'d on top
and bouom by hcmi~phcr~ or rctdi u~ 3 Ill. Tile wtucr b pumped from
a well 50 m hclow ground If 1he diao11Cicr of lllC pipe from ptoonp 10
tank b IOcm. how much worl is n:.quin:IJ w Oll lhc: lazlk initially?

has radiu~

5m.

l lcrnb.phc:rc:

Cal Show 1h01 of the depth or oil in the lank i< It "''""" hen
the 1anl; is ooe-balf full b~ ,otunlC. then h m11>1 wlo>f) 1llc
cqu.;~;tion

IOcm
Submcniblc pump

Find h 10 lour dimol pl3cc<.


(b) Find 1hc work 10 empty lhc h:tlrfull lank i f 1hc oil has

clcn<ily 7:1(1 kglm'. and lhe oullel i< a11h<' 1<'1' of Ihe 1ont.

"' 21. Nl'wton't IIIH\'tr.ml /(1\4 of gmvitario11 ~a t e__~ that 1hc: force of 31
tracaion bch\ICCill\\O pomt masses m and M has mag.niludc

tn E.<crdsc.s 26-.l() C'd!CUialc lhe "or!< done by a~ U!l<cn tllrough the


C)'Cic:: ~K,\\n in the figu.n:.
cyt.l~ in the fi1urc below ..onW~ o(tv. o isothcnnal

26. Tile Stirliug

GmM

prot'(':$5(:$ and

F=--

r' '

.. ~ere r os lhe ~o>Uon bc1"ccn 1llc masses and G = 6.67 x 10

11

awu

~ ,-olumc procx!:)SC.J,

100000 p

f' m,AJ 1 h1con~L

(a) If At """"""IS the omssof1hc earth and"" "'J!:Inlllas a


po.nt tn.'K40 conrenrraled a1 iu; cen1rc, <how that \ t1ott't
uru\~allawof gJ3\1tJiion at the earth's ~>u:rf:tec n:dt.K'C'C to
F mg. when: g ~.82 m/< 1 A<Suonc lor the caku

lauon 01 M thallhc earth ,, a sphere With r.ldiUIO

60000
(0.2.60000
,

23. A n,ht circulucyJindcr ..-ith hocuontalllXIS hl.t radiUS r mctr~S


llftd len~Lh It tllC:t.rci. I( it i~ full of oil with dcn~il)' p '-ito,r41n~ per
cubic mcuc:. how nkld Uf'l. U n:quin.:d 10 empty lhc llJ.nL lhrouJh ~n
oulld allh 1op'>

U. A~ i~conllred '"a C)1inder. closed on one end "ith a pi.,.on oo


ohe ocher 11 1he l<mperauue is held c:onstano. lloen P V = C , "he"'
P d the pc-cuurc of the g_u in lhc cylinder. V is ilS volume. and C 1.1

~06.100~)

PV:4,-

(,no l.m

and mcn clen<ol) 5.S2 x 101 ksJon'.


(b) u.. the origin! bw F = GmM /r 1 wi1h 1hc CC>nh rc
gurdcd 111 a point rnass to calculutc the work required to
lift u mass of 10 kg f(ocn the Cilt't.h's sutface too. hcig.lu of
10 kon.
(cJ Calculate the "'OC't 10 pan (h) u~ing the Cl'!1tlana gra\'llll
honal foroe 1- = mg in par1 (a). I~ 1here 1 ~s.n licant
dffcrence?

~ -4,

1
11(0.6, 20000)

0.6

0.4

0.2

~~

27. 11.c: Erk.non cycle in the ligun: below c:on~iM~ of two isolhc:rnal
procc:sst!l and two con.sta.nt-prCM~Urc PIOC~.

P mom. IOOOOO)
100000 ~
ClO.O~. 1000001
~000

PV = 4

c=~,"'----

0(2. 10000)

11<0. 1. IOdXh

0.5

1.0

1.5

20

1llc Ouo qdt for the intcm;~.l combustion engine i.n tbc figure
below OOOSiSIS or t WO adinbatic: proc:CMCS uncJ IWO <;(HlSUlJl(VOI UI1lC

2-~.

JX'OCt<:(es.

Jl. A g:as is confined in a cylinder, closed on one end with a pi:stoo


on the othc:.r. If the ~u:s C.\pands udiabatically. then the prcs.surc P and
\'Oiumc V of the gM obey the law

C(0.0002. I 040000)

I 000000 p

v'1'

c.

wht.-"re C is !t con~t!lnt The (orc..-e on the p iston fnc::e due to the pressure

of the gas is gi ..en by F :: P A, whe-re t\ is the are-a of the IOOe of


the pi:>ton. Show that if 1hc pi~ton is n~vedso a.s h) reduce 1hc \'Olumc

600000

occupied b)' 1he s~ls from Vo to Vo/ 2. then the work done by the pisaon

is

200000

0.0008

* 29.

The Di~ul cyd~ in the figure below consi$U or two ;:adiabatic::


processes. a consuuu-prc-.ssurc process and a c;on>turu-volurnc proc'C.SS.
I'

2000000

8(0.0002. 2300000)
Cl'O.OOOS1S. 2300000)

., Jl. A drop of liquid wilh initia1 mass 1\f kilogrd.lll.S falls from r~
under grav11y an d evap<.n:U\!S unironnly,losing maAA m kilogr.Jm( each
s.ccond. Ncgltins :Ji.r rcsistaocc. what work is done by gr.1vily on the
drop before it ~tr~mpletely e:.,.uporate:s?

.U. A di\ing bell of mas" 10000 kg is; auached to a chain of rna.~


5 kglm, and the bcU sits on the bottom or the ocean 100m below the
sut(ace (figure below). Uow much work is required to lift the bell to
deck level j m abo\'<! Lhc surfncc? Take into accoonl lhc f:K1 thal v..hcn
the bell and chain are below the surface. they weigh less than "'ben
above. The apparent loss in weight is equal to the weight of water

1500000

di>plae< by the bell and chain. Assume that tl>e bell is a pcrrL-ct cube.

1000000

2 m on ~<:h s i de~ thc:rdorc, when completely submerged, it d ispl:tees


8 m 3 of wJ.tcr. Assume also that the chain displa~s I L of wutcr per
metre of length.

30. The Brayton cyflt for a gas tutbine in the figul\! below consists or
two adiahsuie pr~"~ !lnd two C'OClSiltRI prt"ure pr iX't'sses.

I'

1000000

8{0.0001. I 040000)
C(0.006. 1040000)

600000

0.0015

0.002

17.5 Ener gy
Many engineering systems ""' analyzed on the basis of energy. and there arc m8Jly kinds of
energy -

potenrial, kinecic. thermal. :uKI e leclric~l. 10 name a few. Oflen one k:ind of energy is

uansfom>cd into another. or others. ns a system evolves. We consider u number of examples of


this here.
_in lifting a mass vertically. work is done agait\St gravity. \Vc say that the mass gains
gr:witational potential energy :lS a result of the lifi. rhc amount being equal 10 the work done.
For in'\tancc.
..
when a mass m ls HftcU an amount h > 0 against gra\ity in Figure 7.44a. lhc
force is constant (pro,ided that h is relatively small), and the worl: done is W = mgiJ. \Vhere

g = 9.8 1. We say that the mass has gained gr:witational potemial energy as a result of the lift.
G PE

= mgh .

(7.22)

MillillJiliiiil!t

ll@tUII;IMffW

Lifling a mass againsl gnwil)' rt"sults in a gain in grnvila1io-nal pocential e.nerg)'

final
position

Initial

FinaJ

p~

posit ion

P~ ]

,,

"

lllil.ia.l
posiUon P1

pOSiliOn P,

"o
I

Po

We also say that the difference in gravitational potential energies between initi al and final
positions P 1 and P2 is mgh. In Figure 7.44b. we have shown the same situation. but initial
position Pt is at heig)lt h 0 above some other reference po int denoted by P0 . At position Pt,
the mass has GPE mgho relative to Po; at position P2 , it has GPE mg(h +Ito) relative to Po.
'!be d ifference in these GPEs is sti ll mg(h + ho)- mgho = mgh .
We have memioned gains in GPE. differences in GPE. and GPE at one point relative to
another. and this is what is i.mpon.anL Ofteo, however. it is conven.ieot to talk about GPE at a

point, r.tther than the difference in GPE between points. This can be accomplished by choosing
some specillc point as having zero GPE and saying that the GPE at any other point is the
difference in GPE berween the two points. Por instance, if GPE is set equal to zero at Po in
Figure 7.44b, then GPE is mgh 0 at Pt and mg(h + h0) at P2 . The GPE at any point below
P0 is negative.
If m is allowed to drop from position P1 , it loses GPE as it falls. It picks up speed, and
enetg y is associated with the speed. This is called kinetic energy; it is deli ned by
KE

= ~mv 2 ,

(7.23)

where. v is the speed of m . Provided that ail' drag on m is ignored. whatever GPE m loses. it
gains in KE. To put i1another way. the sum of its GPE and K.E is always the same:

GPE+ KE = C = mgh

+ - mv 2

(7.24)
2
This is called consenalion of energy. Energy simply changes from gr.tvi1ational potentjaJ to
kinetic during the fall. We apply conservation of energy in the followi ng example.

I EXAMPLE 7.19
A stone is thrown verlically upward with speed 20 m/s at the edge of a 50-m cliff. How fast is
it travelling when it strikes the base of the eli ff'?

SOLUTION Let us take the base of the cliff (Figure 7.45) as our place of zero gmvitational
potential energy. When the stone is initially thrown upward, its GPE is mg(50) and its KE is
m(20)2 / 2. If we substitme these into 7.24, we obtain

50mg +200m .

Substituting this back into 7.24 and replacing h by y gives


I

mgy + - mv 2 = 50mg +200m

2g y + v = I OOg + 400.
2
'Ibis equation relates speed and height. At the bottom of the cliff y = 0 , in which case
2

v = IOOg

+ 400

v = jJOOg

The stone therefore strikes the bottom o f the cliff at 37.2 m/s.

+ 400 =

37.2.

J20m/s
Clill'
50m

L __

_J_

y =O

(GI' E= OJ

We can also solve the problem using the technique from Section 5.2. Since tocceleration of
the stone is a = d ufdt = - 9.81 ,

u = -9.8 11

+ c.

(Here v is velocity rather Lh!ul SJlL'ed.) If we choose I = 0 when the stone is thrown upward.
then u(O) = 20, ancl this implies that C = 20. Hence.
dy

dt

A second

in tegn~l ti on

= v=

-9.8 11

+ 20.

ghcs
J = -4.9051 ~

Since y(O) =50 . it follows llmt

+ 201 .L D .

D = 50, antll

)' = -4.9051 2 + 201 + so.


The sronc wikcs rhe bonom of the clilt'when

y(l)

-4.\10512 + 201 +50

The po,itive solulion is I

-20

I=

)400- 4(- 4,\105)(50)

-9.81

= 5.827 s, and a1this time


L'

= -9.81(5.827) + 20 = - 37.2 m/s.

The energy solution related speed v to posi tiol y; the solu liOI\ using i11regra1 ion rehue!ii \'eloci1y
10 rime 1. Which is preferable depends on rhe [pi'Oblem.
~

I EXAMPLE

7 .2 0

Show thlll wll\:11 11>J)ring is stretched (oc ""'"i>"<Cd) lUI :unount X. th..: potclltilLI energy stored
in 1he spring is kX' / 2. If ann~ 111 is auachcd tOthe end ol'the>pring. the spring is stretched an
amount X , mKl the tllC:L'Rt is then released , what is itsspeed at the in.strml the spring isunstretched?
10e force net-e.~sary to mainwin stretch x in the .spring o f Fig ure 7.46a is kx ,
when: k is the spring cons1ant. The work to stretch !he >J>rill!l X i~

SOLU110N

IV

x kx dx =

l b is is potential energy stored in the spring.

{k'}
....:_ x=
2

1
- kX2 .

446

Cbaprel' 7

Appli~ti<-os of lbe

Dc6nirc

huct;r~l

jiOjlclll.l4'fltl.fW Cak':Ulatiou
ene r~y Slorod in ~ spri ng

o(

ConKrvatiou uf cn<4'g}' fut

~ibn2ting 11'13$$-Spl"i_ng_ S)'SICill

1--- x --J I
x=O

MjUUIJ;I+Ci.HW

polcntiul

1--- x -

x =O

dx

when spring
unstretclled

when spriog

mh.tret<.:hed

Suppose now rha1 the mass is mtached w lhe spring, the spring is stre1ched X. and the nli.tss
is released. During lhc subsequent motion to the left (Figure 7.46b), potc.ntial energ)' stored in
the spring turns into kinetic energy of the mass (provided tJ1ere is no friction betwe.en m and the.
surface along which it slides. and tJtere is 110 air drag 011 m ). Consenrarion of energy equa1jo1
7.24 still holds. but mgh is replaced by kx 2 / 2:
I

C = - kx 2 + - m v2
2
2
Initially, the stretch is

X and the mass is not moving. so that

c=

-kX2
2

~knee.

- kX2 = - kx 2 + - m v2

2
2
2
\Vl1en the spring is unstretchcd. x = 0. in which c-ase

This is the speed of m each time it passes through x

Ill

= 0.

EXERCISES 7. 5

_I. Jfthc stretch of a spring is doubled, by what factor docs Ihe potential
cocrgy in 1he spring change?

2.

(o.) Mass m in the figute below is pulkd a distance x 1, co the


1igllL of il$ equjl ibr'iunl pOSition. h iS Ihen given S-l>t:t:J Ito
10 1he righl. Find an equmion rel:uing speed or 1he mass
and stretch {or compression) in the spring thereafter.

"' 3. When thccrossbowof Exe-rci)c 19 i.nScction 7.4 is fired, I he potential energy stored in tJ1c drow is tmnsfomK:d into kinetic energy of tbc
arrow. lf the mass uf th.; arrow i.s 20 g. what is the speed of the arrow
a~ i1 lc:wcs 1hc cros~IXn\'?
;>f.

4. The force of reculsiorl between point charges is deltr1ed i n Exercise


15 of Section 7.4. If Q is placed a1 the orig:in, and q is moved along
lhc .\'axis from x = r to x
r j2, whl t is the gain in electrostatic
energy?

... 5.

(b) Usc the equatjon in part (a) to fi nd the maximum stretch the
spring experiences.
(c) Use Lhe equalion in pal'l (a) to find tl1e rnaxirnurn speed

ex pcricnccd by m.

(a) A 50-m chain wilh mass 100 kg ha.ngs from 1he 1op of a

bt1iJding 50 m high so that its !(Wier end just touches the


ground. \Vhat gravitationaJ potential energy is stored in the
chain? (Take ground level as zero potentiaJ.)
(b) Usc energy cons iderations to determine the work to lift the
chain to the top of the bt1ildjng.

7.6 A uid Pro..,.,ue

6.

447

is p.mg . where 0 < 11 < 1 is consmmnnd 8 = 9.81


Use energy considerations 10 sho'vlhat when the enass is at
positi<ut x, its: .!>peed is given b) the ctlu:uion

(a) If the nws min the figure below is:s1owlyallo-.v00 1ocom..


pte". the spri ng~ \\hal is 1he uhim:ue compression?

(b) If the mass i.s droptxd. O:Jld stn.kcs the spring with spocd
t '<l whut is the maximum comprc:ss.ion c~pcricnccd by the

spring?
(b) Usc lhc equatiOn tn p;ut (a) to dctcrmmc where the mas.s

comes to an inslantanoous stop for the iirsllinlC.


( <:)

\Vlmt

i~

the: linit or the cxprc~hm in part (b)~ J.t _. 0?

ls lhi..s to be c:<pe<:tcd?
(d) \lih:J l is the limit ot' the e:<pression in pa rt Cb) ~I' _. I ?
I~ this tu he CA(kX."h :d?

10. Two springs withcoostams k 1 and k: :en; joined together. and then
one cud i~ fuMc:ncd to a wall (fizurc below). h is ~Q\vn in ph)'~i~
that when a horizontul rorc;c is app1i()d to the fr~c <nd. the rutio of the
stn:tch~

s 1 2nd s1 in the two springs is mvergcly proportion;llto the

r.uio of their spring const~n&s:

+ 7. A punidc is 1110\"<ld along 1hc .rwci$ by u for: F(x) in the:

direct1on. Use Newton's secocxi lllw to show t.hnt 1f F(.t) 1S the total
l'c\f\..-e <tn the ~rt iclc, then lhe \\ICN'l: do;lC by F (x) is cqu:~l t o the ch:msc
in kint.:lic: t;!.!leflY of the: particle.

8.

(a) Newton's unh~rs~ lo.w of g r.witatior. is sto~ed in Exercise

22 or Sec1ion 7.4 Suppose M rcptL~ncs the mass of the


can h. \\'hidl we regard as a sphere with radius 6370 km ami
me.a.ndcnsi1y 5.51 x 1()3 kgtm>. Cel.culotc the g_r.,l\'itation::al
p<Ncmial energy gained by a IO..kg

1na'\~

liflcd rrom 1hc

s,

s,
Show thai if the rn::..: end j,. fn(W...'t.l s.o u.. 10 produce :110131 "li"CICh in the
SJ)rinp or L . the l)(ltcntial cncr:.y :,torcd in tht Sl>rina,s i~

k.k, L 1
2(k,

+ k:)

canh's surface to a hcigh1 or 100 km. Treat the canh ns a


point mus coo~n1r:ued :u its centre.
(b) IJ lhe m3ss is dropped (rom this heighI. wilh what sp.""Cd

9.

((;!)

docs it strike the carth'l


Mass m in the figure for Exercise 2 is pulled a distance .t4
10 Lhe right of ilS equilibrium position and lhl!:n reteased.
As it 1110\'C:S to the left. ((iclioo bc.t\ft-cn m and the sutfacc
on wbich i1 slides ret3rds the moaioo. The force of fnction

17.6 Fluid Pressure


IFI!Il1
ti0t1

of llrtS)\U C in

When an object is immersed in a Auid, i1 isae~ecl on by Auid forces . These forces are independem
of the object. and are 1berefore a property of the ftuid i1self. They always act perpendicularly
10 the surface of 1he submerged object. We use the concept of pressure 10 describe these nuid
fore~. We define prtssure a t a point in a Huid as the magnilllde oflite fiuce per tmit area that

l>ctemlil\3
;a

tlllid

)'

Suri:.:c of fluid
X

l=1L_
T t:::::=3
t>y

would act on a s urf.1ce a t that point in the fl uid. Because pl'essure is the magnitude of the fluid
force at a point. it is the refore a positive quantity. E:~perience s uggests that pressure depends on
two factors: depth below the surface of the fl uid a nd the type of fluid itself. In Qrder to di$COver
the prcci.l)C dependence. we. considc.r a :small h>OrizomaJ disc o f the. fl uid (Figure 7.47).

Suppose we denote b)' P(y) the functional dependence of pressure P on depth y. Then
l>"'&;urc at I he bottom .,rthe di,;c i~ P (y) aml prc,;urc at the top or1hc disc is P (y + 6y). If
the fluid is stationary. then the sum of all vertical forces on the disc must be zero. There are three
vertical forces acting on the d isc: fluid forces on its top and bottom races, and gravity. Since

pressure P (y) , which is force per unit ;uea, is the same al all points on the bouom of the disc,
it follows 1ha1 the force on 1he bottom of 1he d isc must be A P(y), 1hat is. the producl of the
area A of the bouom of 1he disc and pressure at poitus 011. the bonom of the disc. Sintilarly. 1he
fluid force o n the to p of the disc is -A P (y + !:iy ) ; it is negative because it is in the negative
)'direction. Finally, if p is the density of the lluid (mass per unit volume), the force of gr.vity

448

Ctrape:er 7 Applic:uionsor lhC: Oellnile huc:t r:d

on the disc is - 9.81p (A Ll.y) . Since the sun> of these three forces must be ~ro, we set

+ Ll.y) -

AP(y) - AP(y

9.Sip(A L'ly)

0.

Rcarrangcmc11t of this equation yields

P(y

L'ly) -

P(y)

Ll.y
and if we lake limits of both sides as oy

dP

dy

= -9.81p ,

0, we obtain

= - 9.81p .

This diffc1-cntial equation foL 1-'(y) i:i inunediatcly iJJtcgrablc:

J'(y) = - 9.8 1py + c.

= 0] . C must be e<Jual to

Since nuid pre:;~ure at the surface of 1he Ruid i' equal to 1.ero [ P(O)
ze.ro; hence~

{' = - 9.8Jpy.

(7.25 )

Since - y is a mea.<ure o f depth d below t he surface of the fl uid. we have shown that

p = 9.81pd .

(7.26)

where d is always Laken as positi,c.


An illuminating interpretation of furmula 7.26 is suggested by f'igurc 7.48. Abu.,c a point
at depth d below the surface of a fluid, we consider a column of Ruid of unit cross-sectional
area. The weight of thiscolumn of fl uid is its volume multiplic<l by 9.81 p :

IV = 9.81pV.
But V is the product of the length of the column, d , an<l the (unit) c ross-sectional area: th;u is,
V = (l)d = d, and hence

w=

(7.27)

9.81pd.

A comparison of equations 7.26 and 7.27 suggests that pressure. at any point is precisely the
weigh1 of a ootumn of nuid of unit cros~&ctional area abnve lh ~ll pOint
We now consider the problem of determining total force on one side of a flat plate
(Figure 7.49). immersed venically in a ftu id o f density p . If the area of the p late is subdivided into horizontal rectangles of width dy. then pressure at each poi nt of this rectangle is

113'l'!Jirn fK'D'

r.)rc...~.s ure

wci~lll

flu id

of a <:vlumu

o(

i,;

Mjlfjii;J#"ll Fol('c 011


lla1 ~ulace ~ubmergctl in a lluid

Surface of fluid

)'

Surface oi fluid
X

b ....

dy

,,,

{/

.....

=.f(y)

f'.x = g(y)
x2

J.

""-! )'

76

,..,lbd Pttssnre

449

approximately P = -9.Sipy. Thi s is an approximation becau;eslight variations in pres.suredo


occur O\'Cr vertical displacements wi1hin the reetnng.le. It follows that force on the representative
rectangle is appi'OXimatcl) equal to iiS area muhi1>licd by -9.8 1py.
-9.&1py(x 2

x 1)dy = -9.81py(f()') - g(y)]dy.

Total force on the plate is found by adding forces on all such recwngles and taking the limit a.s
their widths approach zero. We obtain the required forcc.lhercfon:. as

1b
,,

(7.28)

-9.S ipylf(x}- g(y)l dy.

Once again we do nOI sugge..-:1 that equation 7.28 be memori1.ed bec..~ause it is a~sociated wilh
the choice of coordinates in Figure 7.49. For a different coordinate systc1n. theddlnite integral
would be correspondingly dilferem (see Example 7.21). What is imponam is the proc<!dure:
Subdivide the surface of the. plate into horizonlal rcctanglc.o;, fi nd L11c force on a rc.prc-.sentativc
rec1angle, and finally. add over all rec1angles with a defini1e imegral.
N()(e that \"er1ical rec1angles canno1 be used without f11nher discussion since it is 001 cvidem
ho\\' to calculate the. force on such a re.ctangle. Consideration of verticaJ rectangles is given in
Exercise 8.

I EXAMPLE 7. 21
The "enical face of n dam is parobolic with brendth I00 m and height 50 m. Find the total force
due 10 fluid pressure on the face.
SOI.I.j11QN l.f we se1 up Ihe coord.inale system in Figure 7 .50, we see that the edge of 1he
dam has an equation of the form y = kx 2 . Since (50. 50) is a point on this curve. it follows
that k = 1/ 50, and )'= .r 2 /SO. Area of the represe.ntative rectangle is 2xdy, and i1 is al
depth SO - y below 1he surface of the water. Since density of water is 1000 kg/m'. force on
Ihe rcprescnlativc rectangle is approximately equal to

(9.81}(1000)(50 - y)2xdy

19620(50- y) ,J50Ydy.

98100../2

Total force on the dam must therefore be


F

1~ 19620(50 -

98 100../2

liii .. Jil;l') :l'!J

'I

r:'luid

y)5../2y'l1 dy

ooyl/2

fun.~ <tn :1

fo JO (50y 12 -

2ystl l so
- -= 6.54 x 10' N.

dam

y 31l)fiy

450

Cblp&er 7 A~ icl liOM Of lhe l)t/inile ln1egral

I EXAMPLE 7 .22
A tank in thefonn of a rightcircular cylinderofradius 2 rn and le1lgth lOrn lies on its side. I f it
is half-filled with o il of density p kilograms per cubic metre, fi nd the force on each end of the
tank.

141411;1
on cll\l (If half lUI lXI cylindrical

tant
' 01 UTION Force on the representative rectangle in Figure 7.5 I is :lpproximatcly equal to
the pressure 9.8Jp( - y) multiplied by area 2x d y of lhc rectangle

9.8Jp (- y)2.vdy = -1 9.62py j4- y2dy.


X

TOtal force on each end of the tank is therefore

F =

1
-z

- 19.62py/4 - y 2 dy

= 19.62p { 3~(4- / ) 312 }. -2 = 52.32p ~0

Marine engineers arc building buoys made of wood and concrele. Both are cylinders with
diameter 25 em. The length of the concrete cylinder is 90 em. Dcnoitics o f the wood and
concrete cylinders are 580 kglm 3 and 2900 kglm 3 , respectively. 1l>ey have asked us 10
determine the lcnglit of the wood part of ~ tc buoy if lhctc is to be I rn al>ove water.
SOLUTIO:>~ To solve this problem, we need Arehimedcs' pri11ciple. h is discussed in
Exercise 18 and Stalei< that the btt0)'3lll force on an object when immersed or partially
inmterscd inn fluid is equal to the weight of tho fluid disploccd by tho objecL Suppose we
denote the length o f the wood cylinder by L (Figure 7.52). For tile bwy 10 float in this
position, the magnitude of the force of grovil)' on the buoy must be equal to the buoyant
force. The force o f gravi ty on the buoy i <

The buoyant force is equal 10 the weight of the water displaced by the buoy
~rm in

M;tlclil;l . . .ifM

Wood

IOO cm

I Surfa..-.o of
water

Concrete

25cm

When " 'e e<JUate these two expressions and cancel


obtain

ll g(25/200) 2

from each term, we

9pcm
The wood cylinder should be 6.45 m long.

= 6.45 .

Conau lt ing
Fi~urc 7.53a >hows an inclincdtubc reservoir manome1cr. used to measure pressure

1uri

aCI(m\ on lhe Mlrf:lce of the gauge liquid in the reservoir. The r~noir is a C) Iinder with
diameter D and lhc tube is a cylinder with diametc.r d. When pressures on liquid surfaces
are the -.arne. the <urface.. are at the same level "" sho" n. When e<tr pre,..,ure i' e<cned

on surface A. the surface faUs an amount H (Figure 7.53b). and hqUid '' pushed 1nto
the tube caw.ln ~urface 8 to ri~ an amount h (both rdativc. to their pu.'itilHJ.~ in Fil:ure
7 .S3a). Our problem is to find an expression for the ema pressure in temlS of length /.
and to determine design parameters that can be modified to make the ntanomctcr more
effcc.1i1e.
AJ'!al)~i.;.

o(

p.nauc1m in aa ioclinWwb<:

ttienOtr mmometer

1---D---1

Surr:~<e A

G:10go hquid

ll

I [

Surr~~ee 8

SOl l TIO:\ Suppose we let the cxtr pressure on surface A be denoted by 6 P. To


lind a fonnula for 6 P in tcnns of It and H . we consider pressure at point Q. From the
tX>int or' oew or the re.ervoir. the pressure at Q is 6 P + pgk. "here k is the det>th or
liquid in the resei"\Oir and p is its density. From the point or\ iew ofthe tube, the !lteure
at Q i' pg(k + H +h). Since these must be the same

6P

+ pgk

= pg(k + H +h)

6P

= pg(H +h).

Bccau'" the 1olwne or liquid in the manometer has 001 changed. we can -.ay that

In addition. h = l.. sin 8. so that

452

Cl:llplcr 7

ApplialiOn5 o(lhe l)etinih: ln!egral

The effectiveness of !he manome1cr is in wsing L to predict 6 P . Most desirable is for


L 10 be large for small changes in pressure. The larger L , the less likely errors in iiS
measurement will affec11he calculalion of 6 P . \Vhon we write

we can. ee that L increases asp, dID, flll<l sin8 all decrease. In o1her worti<. 1he gauge
liquid sho uJd have small dens ity, the.tube diameter should be small rclntive to the reservoir

diarnctcr. and angle IJ should be as small as possible. This i> thcorerically >peaking of
course.
Let us discu~s each of these factors briefly. The mano mete r will be tnel.lSuring the
1>re..<.sure of a nuid (gas) on wp of surface A. 11 follows !hat !he liquid should be imm i ~ible
with !he ftuid. II should also develop a reasonable meniscus so thm length L can be
mc:a.ured satisfactorily. and it should suiTer minimallossdue loevapor&ion. Hydrocarbon
liquids turn ou1 10 be mos1suitable with lo-west densi1ies around 80% 1ha1 of wmer. Use
of such liquids increases effectiveness of the manometer by 25% compared to water.
The l'lllio tl/ 0 should be minim ized rnathemalically. There are limil< 10 how small
d can be and how large D should be. Tube diamele(, in practice, should exceed 6 ll\lll in
ordertoavoid ex<:<:.'ll>ivecal)illaryeffectS. ThesiiUalion d = D and IJ = JT /2 eorre;ponds
to aU-tube manometer. For given 8 P ,

Considerthe,iiUalion When tl

1.

= 6 mm . f)= 60 mm <ay, and 8

t.P

= p~(l/ 1 00 + 1/./2) =

1 39
'

= n / 4. Then

(t.P)
pg

Finally, angle 8 should be minimi7ed. A praclicallower limil i< JT/18 radians: below lhis
angle, 1he meniscus becomes indis1inc1 and it is difficult to get on accumtc readiJlS for L .

EXERCISES 7 .6

1. A tropical fish tank has length I m. width 0.5 m, and dcplh 0.5 m.

.6.

f ind the force due lo water prcs..'turc on cac.:h of the sides and bouom
when lhe rant is rutl

2. The
sb:~ pc

,~icaJ

4. A square plaJe. 2 m on each side. lla5 one diitgonal venical. If it is


on& half submerged in water. what is the force due to water pressure
on each side or the pJatc'?

S. A cytindric:d oil lank or r~di us r ~nd hcighl h has iiS a.xi!\ "~nical.
lf the density off the oil is p, find the fo rce oo the bouom of the tomk
when it is full.

surface of a dam e~_poscd to the waler of a lake has Ihe

J()() HI

--J

TilC \'Crticnl ttnd o( :1 WaiCf trough is an isosceles triangle with widlh


2 111 and depth l m. f'ind the force of the water on c.acJ1 ~oo when Lhc

trough is half full by volume.

sbown below. Fi.nd the fon:."C of the wa(cr on the face of the dam.
y

J.

* 6.
y= 45x 1()6

A rcclangular s wimmi ng pool full of waler is 25 m long and I0 Ln


wide. The deplh is 3m for the first 10 Ln al the deep end. decreasing
linearly to I Ln a t the s hallow end. Find the for<.'C due to lhe weight of
the water on each of lhe sides and ends of the pool.

7.6

7. The vertical race of a dam across a river has the shape of a pambola
36 m m;ross tl:lC top and 9 m deep Cll the (;CUtrc. \Vbal is the forc.:c that
the river exerts on the dam if the water Ls 0.5 111 fronl the top?

f'lllid Plessnre

453

Fluid
surt~e
X

* 8.

Show that lht force due to Ouii.l pressure on the vertical n..'tt.angle in
the (igurc below i.s

9.8 1p
:!

F = - -h(1- - -)

whccc p is the density of the Ruid.

Surface of Ouid

12. Tn Exercise 6, find the force due to water pressure on cadt part of
the bollom or tlte pool.

_.. 13. Tbe bow of u londing barge (figure bdow) <:onsis.ts or a rcctangu~
lar Oo.t plate A mctrc.s wi.de and B mc1rcs long. When the batgc is
stationary, lhis plate t'IHl.k<.-~'> An :lng,!c of !T /6 rodians with the s urfo.cc of
the wntcr. Find the .naxiJmam force of the walcr on the bow.

* 9.

or

A fhtl plate i n I he ~h<lJ!Ie


a (rapezoid i~> submerged vertically in
a n uid with densily p. The pi3LC ha~ 1\VO parJilel vertiCJ~I sides
l englh~ 6 and Ra nd a third side of length 5 that is peqx:ndicular 1.0 Ihe
parallel sides and m a depth or 1 below the sucface of the fluid (figure
below). Find lhC force due 10 nuid pressure On each Side of the pla1e.
using both horizontal and vertical rectangles (sec Exercise 8 for vertic.al

or

reclang_le.").
Surfw.-e of Ouid

*
8

14. A. W'.tter mnk is to be buill in Lhe ronn of a recwngu1ar bo.x with all
six sides welded along 1heir j oins (figure below). Sides ABC D and
E FG H arc to be square. and all sides except A BC D arc supponcd
fro m the oulside. II' the tnaximum force Lh:u A BC D can wilhs:tand is
20 000 N. what is the largesl cross-section 1hat can be built. ass\lming
that at son)(! s1age lhe tank will be ful l'}
_,;:~":..__ _-,iF

10. The base of a trianguJar pbth.:. of length 11 . lies in the surface nl' a
lluid of density p. The third venex of lhe lriangle i:.; al depth b below
lhe surface (figure below). .Show that ~~ fon:e due to fl uid pressure on
e!lCh side of the plate is 9.S lpab1 j 6, no matter \Vhmlhe shape of the
Lriong.Ic-.

A f--

+--{B

1
- -a ---~ Fluid
surfuce

S/7

11. Set up(butdo not evaJuate) a definite integral(s) to find the rorc:edue
to water pressure: on each side or tbc flat verticaJ plate in the foiJowing
figure.

x IS. A cylindrical oil tank of radius r and length h has ils ax:is hori ~
:rontaJ. lf the density of the oil is p, lind the force on each. end of I he
lank whe-n il is half full.

16. A right circtdar cylinder or radius ,. and height h is immersed in


a fluid of density p with its axis vertical Show that the buoyant IOr<...'C
on the c:yHnder due to the pressure or tbc fluid is equal to the weight
of the fluid displat-cd. This is known as Anhimedes' principle. and is
vaJjd for an object or any shape.

454

Ch:lpter 7 J\ppli<.':uiol)1 oftbe Oetinite ln{egr:.l

17. Tile towel' half of u cubic.a1 t.an_k 2 n1on ea~:b s ide b occupied b)'
wa1er, and lhc uppet' half by oil /density 0.90 gtcm3).

Vacuum

-1

(a) \Vhat i:) tbc force on each side of the tank due to the pressure
of the w~tcr mtd the oil?
(b) If the oil and w~t.er are stirred 10 cre:n~ a u ni l()rm mixture,
docs lhc force on cacb side change? If not. explain wby
JlOt. If so~ by how much docs it increase or decrease?

l 8. Archimedes pri11dp!e states that the buoyant forc.::c on an object


when inuucrsc:d or paniillly inuncrsc.tl in a fluid is cquaJ to the weight
of <he lluicl displaced by <he obje<:L
(a) ShtlW that if an object flouts partially :)ubmc-rgcd in watc.r,
the pcn;cnt.agc of the volume of Ill: obje~l abl.wc WdlC-1' is

Mercury

i * !3.

(a) A block of wood (density 0.40 g/cm 3) is cubical (0.25 m


on each side). lf il Ooat:s io willcr. how d-eep is ii.'S lowc.st
point below the SUI race? RcfCI' lO Archju1c-dcs' principle in

Ex.crcise 18.
where On and

{)~~,~

are t_he densities or 1he ol>jccl and '''aler.

(b) Re~a1

(b) l f the densities of icc and water arc 915 kgtm 3 and
1000 kg/m 3 resvectively. show that only 8.5% of Ihe vol
ume of an iceberg il> ab..we \Vater.

* 19.

A wood pole w ith radus I0 em and leng~h 3 m is to be used as a

buoy. ILhas density 500 kgtml . To make it float vertically a concrccc


cylinder with radius lOem and h:og,th 30 em Vi auaclu.:d to one end. lf
the density o r the concrete is 3000 kg/m 3 , how much of the buoy wm
be ab<we wmer?
20. \Vhat :s-hould be the length of the concrete all(tcbmcnt i.n EAcr<:isc
19 ir exactly 1 m or buoy is to be above water?

* 21.

p::rrt (:t) given th:u the block

i~ a

S:J)here o f radius:

0.2Sm.

respectively.

A "Quare Jog 20 em hy 20 ern and length 2 m ha~ been lloating


in water for a number of yc:!lr!O. Water grndua.lly pcrtue.;;ued the log so
thul the dcosi.ty of !.he Jog is no Ionge..- coostanL h ''ur:i_c s li.ncurly with
depth beginning with the ck:nsity of water at the edge deepest in water
lO 500 kgtm3 at iiS top edge. Detcnnine lhe heigh! or log protruding
from the water.
22. II' a full lube of mercury is inverted in a large comaincrol'mcrcury,
the level of mercury i.n the lube wiJJ faU, but i~ wiJJ slabjJi-zc al a point
higbcr than that in the container (following fig\lfC). This is due to the
fact that air pn:.:ssun:: acts on the surface of the lllcrcul)' in the container
bul not on the surface or the mercury in the lube. The extm column of
mercury, of height h , creates :.1 force at A that counteracts the atmospheric pressure (ransmiucd through the rncn:ury in tbc container to the
lube so lhal the tola l pn:s~un: at A is equal to the total pressure at B.

(a) Show that il' tltc dcns.ily of mercury is 13.6 g/ml. then the.
atmospheric pressure at the surface of the mercury in the
container is 1.33 X Jo' IJ N/rnz. pro,,idcd thai h is measured
in metres.
(b) If h i~ measwed as 761 rnm~ tlnd the atmoS-pheric pressorc.

x 24. A mnk is to he bui 11 in the fCinn of a right circular cylinder with


horizootaJ axis:. 'The ends ore to be joined to the cylindrical side by a

continuous weld. One end or the to.nk o.nd the::. cylindrical s ide are sup
ported from ~he ()Ul:Sidc. l11c remaining un:suf>P{li1Cd end c.:ao wilhsumd
a IOta I force of 40 000 N less 1000 N for each metre of weld on thm
C1tcl. What i~ the maxinltuu ro-~diu!> for the t:lnk i f il is: to hold:~ nuicl
wilh dcnsily 1.019 x 10' kg/ro 3 '1
._ 25. Find the ratio LJ R such that the (ore~ due to ilu.id pressure 011
the r'CCI.angular ~nc.l se.rnicircular pans of ~he l))ilt C in the tigute below
m'e equal.
Fluid surface

* u.

(a) A spherical shell with in1tcr radius I m and ouiCI' radius 2m


has densiLy 2 kg/m3. It is cut in ha1f by a plane through its

centre. The flal edge of one of the halves is placed carefully

on the Sllffacc of a large cont<.lillcr of '"-:.ncr. How far docs


the shell .'l-i nk~

(b) Repeal part Ca) ifariny hole is drilled through the shell m
jls uppc.r most poi.t:lt.

17.7 Centres of Mass and Centroids


Everyone is a<.:quainled with the action of a let:ter-totte.r or seesaw. Two children of unequal
masses can pass many hours roclOng:, provided that the ch ild with grea1.e r mass sits closer to 1he
fulcrum. In this section we discuss the mat hemal ics of th e seesaw. This requires a detlniLion
of moments
masses, and moments lead to the idea of the centr e of mass of distributions of
masses, l umped or continuous.

or

1.7

Centres of !\las!': and Cenl:roi(l~

455

To discuss che mathematics of a seesaw, 've consider in Figure 7.54 a uniform seesaw of
length 2L balanced a t its centre, with a child o f mass m at one e nd . If a second child o f equal
nulss is placed m the other end, the ideal seesaw siruation is cre~ued. If, however, lhc mass of
the second child is M > 111. then this child must be placed somewhat c loser to the fu lcrum. Th
find tbe exact position: we must detemlloe what ntight be c.alled the rockinR power o f a mass.
A little experimentation shows that when M = 2m , M mus t be placed halfway between the
end and the fulcrum: when M = 3m. M must be placed a distance L /3 from the fulcrum: and
in general, when M = am , M must be placed L I a from the fu lcmnt . Now the rocking power
or the child of mass 1n is constanl, and ror each mass M = 01~1 we have found an equa l and
o pposite rocking power if M is placed at L f a.
MjiCJileJ+ft!lj

Momc.ot::; of children on a scc.saw

/, - - - - 1 - - - - 1. - - - 11

~lr&======r=======~
....

Clearly, then, rocking power depends on both mass and distance fro m the fulcrum. A
1iu1e thought s hows that the mathematical q uantity that remains const ant for the various masses
M = tWI is the product of M a nd distance to the fu lcrum.; in each case, this product .is
(am)( L I a) = m L, che same product llS for the child of mass m . It would 11ppear. then, that
rocking power should be defined as t he producLof mass and distance. We do this in the fo llowing
detl nilion. and at the same time give rocking rnower a new name.

DEFINITION 7. 1
The first moment of a point mass 111 "bout a point P is the product md, where d is the
d irected d istance from P to 111.

If directed distances to the righc of point P in Figure 7.55 are c hosen as positive a nd
distances to the left ate negative. then d 1 is positl \'e and d1 is ne.g ative. ~fass m 1 has a positive
first moment m 1d 1 about P , and mz, has a neg::n ive first moment m 2 d2 I n Figure 7.56, we have
p laced fi ve childrc;n of masse.'5 111 1. m2. 1113 , illJ . and ms on the same seesaw. A sixth child of
mass 111~ is to be placed somewhere o n the seesaw so that all six children form. the ideal seesaw.
M::Ciriilljlfl.7'3.

Oclinition of

ntOITl("nt

of mass abotU a point (In a line

- - d2----1f------ dt- - - -

"'z
ljiclil.l&i*

Dctcuniual.ion

o(

pt):)ition., of -clti lt.lr~n Cor idea.! seesaw

456

Ch:"'I)Cer 7

AflplicatiOn'-Oi'lht l.ld\n11t lnlegr31

To find the appropria te position for the sixth child, we let x be the directed distance from
the fulcnmt to the point whene this chi ld should be placed. The total first momem of all six
children nbout the fulcrum. choosing dismnccs to the right as positive and to the left as negative,
is

lll t(- L )

+ m~(-3L/4} + llll (- L / 2) + m.(L / 4) + ms(L ) + 1116(x).

We regard this as the resultant fiJSt moment of all six c hildren auempting to turn the seesaw
- clockwise if the momc.nl is positive. countcrcloc;.kwisc if the moment is ncgati""<=. Balance

occurs if this rcsuhant first moment is zero:

0=
\Ve may solve this equation for the position of m 6 :

- -(-llll t +3m 2
41116

+ 2m;

- m , - 4ms).

Placing the fuiaum for an ilkAJ ~w

I 4Cc:IIJ;14&il

'

xz
.l

'"t

"'2

x :O

....

"'J

m.,.

"'s

,6

We now tum this problem around and ploce the six children at distances from the len end
in Figure
If the mass of the seesaw it>elf is neglc~1ed. where ' hould the fulcrum
be pi need io ord er to create the ideal seesaw? (Sec Exercise 16 for the C<lSC when the mass of
the seesaw is nut ne&lected.) To sol ve thi>l)l'oblem. we let the d istance that the fulc n un should
be placed from tho left end be represented by
In order for bolancc to occur, the total first
moment of all six children about the fukrum must vanish: hence.

"''I""'"'

7.57.

x.

1l1e ~OhHi()n of this equation is


I

ru;, DnJI

Mocnen1 01

a mn(li about a line


)'

/ I
A~~'y

M~m ;x,.

(7.21) )

where M = L~al m; is the lOtalmass of all ~ix children. This point rat which the fulcnom
creates a baloncing position is C<lllcd the centre or mass for the six children. It is a point where
masses to the right are balanced b )' ma~< to the left.
In the remainder of this section we extend the idea of a centre of mass of point masses nlong

a line (the ~saw) to 1he centre of ma~s of :1 disaribution of point ma~;se!l: in a plane. ;lnd then

Ill

to the cemrc of ma.-ss of a continuous distribution of mass. Our first step is to define the first
moment oh point mass in a plane about a line in the plane (Figure 7.58).

DEFINITION 7 . 2

The fi rst moment of a rnass m about a line I is rnd, where d is the directed distance from

I to m.

Once again directed db;tances are used in calculating lii'Sl momentS. and therefore distance..'
on one side of the line must tk: chuscn as posichc and distances 0.1 the Qth~r side i:ll> uc~ativc.
For vertical and hori?.onLal lincs, there i s a llltturul convention ror doing Lhil>. Discunces to the
right or a vertical line arc chosen as fX1Sitive. and dblartcc~ to 1hc h:fc arc negittive. Distances
upward from 1 hori'I'Onl~l line are positive, and distances downward are negative. In particular,
when a rU\SS m is locotcd at posi ti on (x , y) in the xy ~plane, i ts first moments about the.x - and

y-axc$ arc my nnd mx, respectively.


Firs1 moments nf usystcm of n point masses m 1, 1112 , , llln located at I)Oints (x 1, y1) ,
( ,( 2 .

y2 ) . . (x, , y.) . rcspec1ively (Figure 7.59a)<lbouc che X and )H lXCS nre defined :lS chc

sums of the fi rst mnmen1s o r the ildividual rnasses about the..~ lines:
n

firs1momem of sys1cm abou1 x-axis

= "'II/')''
L.- ' ,,

(7.30.1)

i-1
n

first momem of sys1em aboU1 y-axis =

Lm;x;.

(7.J0h)

i l
IOU,jil;l}il~Bl:J
rn:~o;<;.e.~

Systeln'l of n poin1

cmbedrlo1 in n pl::11e

>'

Y.
ml

m 1(x 1 y 1)

m,.<x~,

...

ml( x'l:.. YJ)

)',,)

my.x3 yl)

Place
:r

x =r

Whal is lhe physical mea11ing ofdlCSI! firsc momems? D<> lhey. for insumce, play the same
role 1ha1 fiJ'SI n10mcnL~ did for the seesaw? To see chis we imnginc 1hm each point mass i~
embedded in a chin piMtic place in the xy-plane. l11e plate itself is massless and extencls 10
include all 11 masses m; . Pictu"' now Ihal chc pbue is horizonllll. and ll sharp edge is placed
along the y-axis. Docs 1hc plate ro1a1c aboullhis edge. or docs il balance? II is clear lhlll lhc
plate balances if the fir.;c momen1 of the system <tboucthe y -axis is equal to zero. and rotmes
otherwise, 1hc direction depending on whether che first mon1cnt is posi1ive or ncga1ive. Si rnilarly,

chc place balances on a sharp edge placed nlong Ihe .t -oxis if 1he fim momcn1ofche syscem :>bout
chc x-axi< vanishes. In gcncml. chc phuc balnnccs along any s1migh1edge if chc li rs1 momcn1
of 1he system aboucchncedge vanishes.
We defined che centre of mass lorn disnibotion of children Olllhc secaw <IS chc poinc <It
which 10 place 1he fulcnuu iu order 10 oblaiu balance. Analogously. we dcllne 1he centre of
mass (x, Y> of che distribucion or poi111 mas.<es in Figure 7.59 a as che position 10 plat'C a sharp
poinl in order 10 obc.atn balanct:. Renlt:n\bt:r lhat t he plastic it~ell' i.s m..~sh:ss a nd onl y the: pui11l
masses m ; can crca1c moments. IJaluncc occurs at ll poinl (X.)") if balance occurs about every
snaighcline 1hruugh (:r,
In panicular, balance must occur abou1 1he lines ,r = x <md y = y
Jl'll'llllel co lhe and x -axes as ill Figure 7.59b. Since oolance occur.; about x = ;r lhe lotal

:n.

.v

first momcm of the syscem about !his Unc vanishes. we obwin che condicion Ihal
0

= m,(x, -

n + m, (,r, -

X) + .. . + m,(x. - x),

which can be solved for X,


I

x = -M """
!--mx.
I

=I

I.

(7.31 )

= z:::;

where M
= 1 m; is the total mass of the system. Note that rhis equation is identical to
7.29. Similarly. for halancc ahout )' = y. we ti.nd that y must he

=M

"

(7.32)

Lm1y;.
i -1

\Ve h.ave obtained n unique point (X. y) bused on conditions of bahmce about the lines x = X
afld y = y . Doc:s tbi1> oo:e~rily huply lhat b:1hu)l.:e l)i,:c;urs ;ti,>Oul evety strajgbt line throiJg.h
(X,)')'? The arlswcr i~ yc.~ (~ee F.xetcise 39).
Every planar point mass disuibution ha.'~i a centre of mass (:f. Y) defined by equations 7.3 I
<trtd 7.32. Our c.lerjv:-~tion has showo lbat lhe Grst ux ,tn::nt of the ~yscem abotu any )h\e through
f) nlUSI be equal to 2etO. This poirll is sigrlitlc.ant in another way. If a l)i.lrtideof mass M
(the total mass of the system) is l ~ued at the <.~en tre of mass {X. y), its firs t moment aboul the
y -axis is MX. But from equation 7 .31. we have

rx.

fltfX =

'Lm;x;,

(7.31)

i= l

and we cuncludt: lhill 1he flrsl munlenl of Llis lk .tititlll.S par1idc: M <&bout 1hc y -ax_is is C.:.\.(u.;tJy

!he same as !he firs! momcn1 of 1hc sys1em aboul !he y-axis. Similarly, !he firs! momcn1of M
about the .t-axis is M)i. and from 7 .32.

i'4 Y =

"
'Lm,y,.

(7 . 3 ~ )

i:l

Thus, the centre of mass or a system of poim masses m; is a point at which a s ing le particle of
mass ;\1 = L7
=J nt1 has the same first mome.nts abom the X - and y -axes 3S the system. It can
be shown further {Exercise 39) that the first moment of M about any line is the same as the first
moment of the system about that line.
fn sunumll'y. we defir\ed the centl'e of mass or a system of poim masses as a b0:1la11ce point.
We fourld as a re~u h I hilt the cemre of rnas.s is aJXlitH a1 whic h i:I .Si1lgle particle or nH\1\S equalro
1he tolal mass of 1he system has the same firs1 moment about ~lny line as the system i1selr. This
argument is req~r.sib1e. \Vere we to define the centre of mass as a point to place the mass of the
system for equi\lalent fi rs momenL~. it would be a balance point. Tn other words, we have two

equivalent defirti(ions of the ce.mre of mass of a system of point masses- a balar1ce p<~int OJ an
cquivaJem point for first momcms.
MiiiCiil;l.fll[l'ill
m:u;~ of a ihin plait

Centre of

Y = /(x)

d..-

We now mat-e the tra1uition from a discre[e .s.ys[em of panicles 10 a cominuous distribution
of mass in 1he form of a 1hin plate of cons1an 1 mas< per uni1 area p (Figure 7.60). In order 10
find the mass or the plate, we proceed in exactly the~ame way that we did for areas. Wedi\1itfe
the plate into vertical recta glcs. the mass in a repre~'icnlati ,e ~ct.anglc of width dx at position
x being
p(f(xi - g(x)]dx.
To find the total mass of the plate, we add 0\er a ll such rectangles of e\er-d iminishing widths:

M = J.b plf(x) bX

g(x)] dx.

(7.351

Bt\sed on our d iscus.don for systems of point masse.' we define the centre of ma.s..-.; of a
continuous distribution of mass a..'l that point {X. f ) where. a partjcJc of mass M ha.'l the same
6rst mon1ents about che x- and y-axes as the distribution. Ln algebraic terms , we nOle that
Mx is lhc firs! momcnl abotollhc )'axis of a panicle of mass M al (x.
To !his we ntUSI
equate the first momem or the original distribution about the y-axis. Now each point in the
.-epresemalive .-eclangle in Figure 7.60 is apptoximalely 1he same dis1ance x from 1he y-axis

n.

approximately. because the reccangle does. have Hnjte. though very small, widlh. The tlrst

moment, Lhen, of Lhis rectangle about the y~axis is approximately

xp[f(x) - g(x)]dx.

7.7 Cenlrest,f Mass MdCen1roids

459

We lind the lirst moment o f theent i.r e plate about they-axis by adding (irst mome nts o r aiJ such
reclang les and tak ing Lhe limit a.s their widths approach zero. But once again this is Lhe process
dell ned by the dell nile integral. and we obtain, therefore, for the llrst moment or the plate about
they-axis,

1
1"

xp[f(x) - g(x) ]dx .

Consequently,

Mx

(7.36)

xp[f(x) - g(x) ]dx,

a nd th is equatio n can be solved for X once A1 and the int.egr-.,d o n the right have been evaluated. Equation 7.36 represents for continuous d istributio ns what equation 7.33 does fo r discrece
distributions .
To lind y we must equate the product My to the tirst moment of the plate about !hex-axis.
If we consider the representa tive rectang le in Figure 7.60 we see that not all poims therein are
the same distance from the x -axis . To circumvent Lhis problem, we consider all of the ma.ss of
the rectang le to be c.once.ntrated at its cenlre of mass. Since the centre o f mass is the midpoint
of the rectangle - a point distant If (x) + g (x) 1/ 2 from the x -axis - it follows that the lhst
moment of this rectangle aboul the x -axis is
I
- lf (x)
2

+ g(x)]p[f(x) -

g(x)]dx.

The total tlrst moment of the plate about the x-axis is the detjn ite integnd of thjs expression.
and we set
b 1

MY =

- [f(x)
2

+ g(x)Jp[f(x) -

g(x) ]dx .

(7.37)

This equation is used to evaluate y.


Equations 7.36 and 7.37 can be memorized as fonnulas for and y. but it is easier to
perform the foregoing ope1ations memal1y and anive at these equations. Besides. for various
shapes of plates. we ntight use hori zontal rectangles o r combinations of hori:wnta l a nd vertical
reclang les, and in such c.a.ses 7.36 and 7.37 'vould have to be modified.
On the basis of dellnitions 7.36 and 7.37 fo r (x , y) , we can show that the llrst moment of
M a t (x , y) about a ny line is the same as the lirst momem of the plate about that smne line.
This implies that the plate balances alo ng a ny line through (x, 'J) a nd therefore at (x, Y) .

I EXAMPLE 7.23
Find the. centre of mass of a thin plate. or constant mass per unit area p if its edges are. detined
by the curves
M:IIU)il;iij,QID C~nt.rt of
ma;;s of plate. bounded by y =

2- x 1 , y

y = 0.

= 0, and x = 0

SOLUTION

X =

0.

2:: 0.

Using vertical recta ngles (Figure 7.6 1). we li nd that

)'

If (x, y) is the centre of mass of the plate, !hen Mx is the llrst moment of the single panicle
of mass M about the y -axis. This must be equated 10 the lirst momem of the plate about the
y-axis. Since xp(2- x 2 ) dx is approximately the lirst moment or the rectangle in Figure 7.6 1
about lhe y ~axis, the following integral g ives the first moment of the plate about the y -axis:
X

Hence we set MX = p, aod solve this equa1ion for X:


X

p
3
3
= - = p - - = - -.
M
4./2p
4./2

To timJ y. we C.:-alculatc the fir:.a ntoJlleJll of tht plate about the X -axis. Since the cc:nt.n: or mass
of the rectangle in Figure 7.6 1 is y/2 units above the .r-axis, it fol lows that me lirst moment or
this re<:tangle about the x -axis is (y/2)py dx. When we integrate this to find the first moment
of lhe pla1e about the x -axis, and eqwue it to MY. the resuh is

P {

= -

IG./2p
15

-4.r3 + -xs },;2 =

4.r -

TinL>.

16~p 3
15 4./2p

Y=

I EXAMPLE 7.24
per unit area p i f its edges are defined

ma~s

Find the centre of m>lSS of a thin plllte of constam

by the curves

y = x1

4.

SOUf fiOX Using vertical rectangles (Figure 7.62) yields


n~~ o (

2.x -X:

plait bounded by y
:W.ld

y=

,t: - J

1
2

=p

P()"l - y ,) dx

12

- 1

= p1
X

(x 1

4)]dx

Y 2." - x2

[(2x - .r1 )

- 1

_,

(4

+ 2x- 2x 2 )dx =

p {4.r

+x2 -

2.<' }
3 -

= 9p.
t

I f (X,)) is ~>e centre of' mass ol' the plate. then first moments of the plate and M about the
y-axis give

1
1

Mx =

xp(y2 - y 1) dx

=p

- 1

=P { 2x

d.t

1
1

(4x

+ 2.r2 -

2, 3 ) d.t

- 1

+ -2x3
- .r
-2 }'_, =
3

9p
2

'lbus,

1'
To find

9p I
2 9p

= -- =

y. we usc rnomcnts about the ~\" ax is to write

M)'

L~ ~(y, + )o'2)P(Yl -

Yo) dx

i L~ (yj - )';)

dx

= '!.1 ' 1(2< - x 1) 1


2 _,

= 2p { -

x
3
4 +x -

(x 1 - 4) 2 1d.r = 2p 1

}2

4x _ , =

21p
2

(-x 3
-1

+ 3x 1 -

4)dx

Consequently.

21p I

y=- 29p = -2
Looking t\l Figure 7.62. it would apJJCIIr to balance at the point ( 1/2, -3/2).

I EXAMPLE

..

7 .2 6

Find first momentS of a thin plate (J{ constant ma.~ pe.r unit tll'e a p about the llne.s (a) )'
(b) y = -2. lind (c) x
-2. if i[s C!dgc~ are defined by the tur,e~

= lyl' ,

.<

= 2- y

SOI.! ,ON
() Since the mass is distributed synlnlCtricallr ~li.>Out the x axis (F igure 7.63) , the first
momc.nc about y = 0 is zero.
(b) Since the ccnlre of rr1ass of the plme is Orlthc .ra:c.is, its first mornent about the line
y = -2 is

iUEi'LJ i&!il

FiN """'
mcnt) of pl.&k' abotl! lines

y
,\' : -2

(2)(rnass of plaoe)

y =- 2

.<

= 0.

X=-)' :}

2(2) (mass nf plaoe above ohe .<axis)

i'

p (.<z- x 0) dy = 4p

1'

(2 - / - y )dy

"' } '
= 4p {2y - -y; - ::...
3
4 0

17p

(c) The first rnoment of the !>lace aboln the li ne ;r

= - 2 is tw ice the flr'St moment of

ios Ul>p<:r half. Since the x-coordi11aoe of the centre of m ilS$

or

ohe reprcscnuuivc
rectangle in Figure 7.63 is (.rl + x 2 )/2. the r.lisranc:e from the line x = - 2 10 lh et'Ctllrcof mass oflllC rectangle is 2 + (.< 1 + .r1) /2. The li i'SI moment of the horizontal
recta11gle about x = - 2 is therefore

( 2+

x, +-"2) p (.t 2 - x1)dy.


2

auclthe first moment o f the pl ate is

1'( + x,+.r-,)

p(xl- x ,) dy

=p

1'

[4(.<2

- .r1)

+ (x j - x~)jdy

= 4p

Cr, - xr) dy + p

fu'

1 1

'

= -17p + p
(4 -4/+y' - y6)dy
J
.0
17p

= =

+p

88 1p
105

4 yl

4 )- --

3 1

1(2 - y ) - (y ) ]dy

v'

7 1
11 }

+ '- - '-

It has become apparent through our discussiOIIS and examples that in calculating the cenrre of
mass of a thin plate with cons tam mass per uni1t are-~ p. p is really unnecessary. As a conStant
it is taken out of each integration and cancels in the linal di"ision. Titc location of the ccmre of
mass depend' only on the geometric shape of Lhe plate, and for thi> reason we could replace all
references to mass by area. In p<tnicular, the mass of the plate M can be replaced by its area A,
first moments (of mass) Nix and MY can be replaced by first moments (of area) Ax and Ay,
and equations 7.36 and 7.37lhcn takc the form

Ax = [ ' x(f(x) Ay

~ I

- [f(x )
2

~(x)J d.t.

+ g(x)][j(x) -

(7.38)

g(x)]tlx .

<7.39)

It is customary when using first momcms of area tO call (x. Y) the centroid of the area mthcr
than the cemre of mass of the plate. simply because all references to mass have been deleted .
We emphasize. howe\'er, that the statements in this pal"dgtaph apply only when mass per unit

area is constant.

Figure 7.64a <hows an automatic valve consisting of a plate L metres 1vide and H metres
high that pi'Ois about a horizontal axis through point A . Water creates pressure on pans
of the vnhc above nnd below A. If the force on that part of the valve below A is gre;>ter
than on that pan above A, the valve rcmaios closed. If the fon:e is greater on that pan
above A. the v.tlve opens to release water from left to right. ])e,ign;pecifications require
the 1>ahe to open when the depth of water is D . Our problem is to locate the !Xl"ition of
A for this to happen.
QURii 7.84b

W:lltr

\Vater

V.tve
A

--~-------ff~LL~---- Y
SOLUTIO!'\

First we should point out that it is not the forces of the water on the top and

bonom parts of the valve that del.errnjne whether the valve opens or closes; it is momenLS
of these forces about the pivotal axis through A. \Vith this in mind let us consider a small

horizontal strip of widLh dy and length Lon the face of the val ve (Figure 7.64b). The
force. due to water pressure on it is pg( D - y) Ldy when water depth is D. If the
required position of the pivotal point A is h metres above the bouom of the valve. 1heo
1hemomeot of 1his force about A is (h- y)pg( D- y)Ldy . Moments will be positive
for areas below A and negative for areas abo1e A . The total moment on the valve is

loll (II -

y)pg(D - y)Ldy = pgL

L"

!ltD- (It + D)y

+ y 2]dy

}'2

)'3 } /1

= pgL { hDy- (h + D) - + 2

= pgL ltDH - (h

+ D)Hl2 + 3HJ] .

The \'alve is on the. ve.r ge of ope,n ing when this moment "anishcs,

f/2 Hl] .

0 = pg L hDH - (It + D) l +

The sol01ion of this equation ish = (3DH - 2H 1 )/(6D - 3 H ).

EXERCI SES 7 . 7

K. y=x3, x= y3

In Exercises 1- 5 find the centre of mass of 1he thin plate with COI\Siant
russ per unit ~a.
I.

l.

Semip3r.tbolic

- a -1

a- 1

Pambolic

9. )"=X, )'

T
,,

T
h

10.

X=

y>- 2)'. X +)'= 12

11.

y = J2-x. x+y = l

12. X = 4y - 4y 1, )" = X - 3. )' = I, )' = 0


13. .t 3y

3.

= 2.t. 2y =X+ 3

4.

= 8,

)' = CJ -

x'

+ 1-1. The edges of a thin phnc wllh consuant milS.~ per unit Brtll p a..re
defined by lhc CUfVC> )' = l.tlln . .1' = X + 2, ond )' = 2 - X . fi nd
its first momem about the line x

- - (1 - l

= - 5.

15. Show thai the cenuoids of regions A and 8 in lhc figure below
ha\'C coordinates

T
h

II

Quartet-circle

XA =

(~)a.
11 +2

Xts =

(~)a.
Ylf
211 + 4

YA

= (~)v.
4n + 2
=

(~)b.
211 + I

5.

lr>of
n~l

(o.b)

Semicirde

Y:_;::/ A

In Excn:.i~ ~I) lind thcccntroido(thcn:gion bounded by lhc<:w,'CS.

6. y

=x1 -

7. ~,. =

'P
x

V JA. I '

I, y

= - x:! -

v= 2-

x'

2x - I

16. Find lhc centre of mass of the seesaw in Figure 7.57 if the mass of
the seesaw is not neglected. Assume that it has unifo rm mass per unh
length p and len gth 2L.

464

Chapter 7

Applic.1tioos of \he t)c:l1nite hueg.ral

In Eltcn:i.scs 17- 19 find the centroid of the n:gioo bounded by the

29.

curves.
I
i

= J )' + 2. )" = X , )' = 0


18. \' + .<2 = 0. X = \' + 2. X +

\'

y ='2

17.

(above y + x 2= 0)

+2=

0,

19. y = J2- x . 15)' = x 2 - 4


20. Find the ccmre of mass of the thin plate in the figure below if it
has constant mass per unit area.

* 30.

rt--;:::;.
=f ~~~I
,,
1

Par.lbolic

Parabolic

7...~.w il1n

dJ.,
,-

:r,----...1-:..J

IL___ --+'

.T

df?

ln E._'\crci~s 2 1- 25. a pl:ltc with cons1ant mass per unit ~rca p is


bounded by lhc gi,cn ~UI'\'CS. Find its lin.t nK.>mcnt aboul the suggc.)tcd

~ --~ -31.

line.

l r--

- d -

21. x + 2 = ,>2 , y - .x about x + y = I


22. y = 2x . y - 2x . y 4 - 2x 2 .'' 2:. 0 abou( y x
23. y x' - 2x. x + '' 12 about 3x ..- y = I
3
24. X = ) . X + y = 2, \'
0 about X + )' + I : 0
2
+ 25. x = )' - 2y, x = 2y - )' 2 about x + 2y = 4
26. If a region A can be S-ubdivided inlb n subregion$ A ; (i
1 . . . , 11) such that the centroid of each A; is (i ; . y1) . show that
tho <'Cillroid (X, Y) of J\ is ghen by

Channel

- I
b

d -

i:I

"/2

"'

In Excteiscs 32-35 lin<! coortlinalcs of the centroid oflhc tt&ion ace


r.ttc to th rc~ tkcimal plu~-cs.

3' = 'ALA,)',.
,_,
In Exercise-s 27-3 1 usc l1~ tcclmique :.ugge~1cd in E1crcist 26to find
1hc ccmroid of the region.

* 21.

i!* 32. y = x' + Jx' + 2x +I . x = 0. ,\' = 0


i * 3J. \' = x 4 - 5x' + 5. \' =0 (alxwc the x-axis)
i 4 34. ~ =y' - y 2-2y. .r = J2.v + I
iii. 35. y = x'- .r. y =./X
"' 36. Pto'-c the following tltl'o~m oj' Pappt~s: lf a pb.nc region is tC
\'Oh'Cd about a coplw1ar tlxis not crossing the rcgiM, lhe \ "'IUJtlt' ge1\
er.ucd is equa.l t o ~ product or the !lte".) (lf the region :lnd lhe circum
tCrcncc or the c ird c: described by the centroid of the region.
37. Usc the .rc:sulLof E:tc(cisc 36 to find 1bc volume of tbc donut i.o
E;'(crcisc 41 of Section 7.2.
38. Use Lhe result of Exercise 36 to lind the \'Qiume in E:.:ercise 42 or

L - - - - --, ,

~ion7.2.

L-r.=::::~,,
, ::::::~--;x

39.

(:.t) S how lh311hc lin;a momcr.t or I he syslcm of point masses i.n

Figure 7.S9b about any line is the srunc us the fitst momcnl
or u txJintma:ss M m, at (X.):) about lhat line.
Him: Usc fonnuht 1. 16 for the <.li.MaJK:.C rrom a poinl to a
lioe.
(b) Does it follow t11a1the fir M moment of the system about any
line through (X, )i) is zero]

* 23.

L7-l

*
X

40. A lhin (htt pl!tte or ar c<a A is htmcrscd vertically in a Ouid of


den.siLy p . Show that Lhe LotaJ force due to tluid J)l'eSSUI'e on each side
of lht! plodC is equal I() Ihe product or 9.81 p' A. and the dcplh or the
centroid of the pl:~ tc below the surf:~cc or 1hc Ouid.

7 S Mument.'l u( lne11i.1

4 1. Tflc. dam in the figure below is .J Ill his,h. 10m wide. and (t n)d.rc;s

thick. His mudc of ooncretc "tith density 2400 kg/ml. Uttamine the
minimum value o f a if the dum is not to overturn about poiJn A. when
d=4 rn.

-Du.n

4 nl

** 42.

The gate A 8 in lhc ligurc below is 525 n1m wide und is held in
iH d osed pohltioo by o; vcnic.al cable and by a 525 mm hinge located
along its 10p edge B . Poe" a deptJl d = 1.8 m of water. determine
the minimuo1 tcnsiOtl in the c.ab lc tju ied to prevcl\t the gate frmn
opcmng.

I
I

\VQfet

485

18m

Walet

I
<Jmn

/J

l:Onm

(a) Asswnc thm u seal cxius 2.1 B. so th"-1 no w\'ltCt pt"Cs~ u rc iti
pt'CSCIH under 1h e dam.

~ 43. Showth3t thc centroid of a 11i;!nglc with ' 'crtkcs: (.t 1 y1) , (.t 2 y 2) ,

and

(.T,.

y, )"

)'=Yo +>~+y,
(b) Assume tha4 no seal c).ists at B. so that full hydrostalic
pressure is present under the dam from .A to I] .

17.8 Moments oflnertia


NewtOn '.s second law F = mt1 is fundament~ll to the s tudy or trans bHional motion of bodies.
For roLationaJ motion of bodies. il.S coumerpart is r = l et, where r is torq ue. a is angula r
accclcrmion. and I is the momml ofinertia of t he body. T he kinetic energy of a body of mass m
moving wilh velocity u is m v1 /2. The kinetic energy o f a body rotating with angular velocity
w is I w2/2. Thus. for rouuional motion. there is a quantity called the monocm of iucrtill of a
body lluu i~ analogous to ma~s in 1ranslation111equations. In this sec1 ion we define and cakulmc

momen1.s of inenia.
'fo define moments or incniu o f bodies. we bc.gin with the moment of inertia of a point

mass.
DEFINITION 7.3
The moment of Inertia or second m omem or a poim onn.<s m about a line I (Pigure 7.58)
is the product md2 where d ;. the directed distance fron1 I to 111 .

ln pat1icular, if m is a1position (x, y) in the xyplane, irs moments of inertia about the X
and )' axes are 11zy 2 and mx 2 respectively. For a sys1em of 11 pan icles of masses m 1, m l . . .
m, located at point$ (..r 11 Y t), (x2, y.,) , . .. , (x, . J J1 ) as in Figure 7 .59a, mom en(~ o r inertia
of rhe system abom rhc x . and y -axes arc sums of 1he momcnr.s of incnia of rhe pat1iclcs about
the x- andy-axes:
II

l'll ()lliCilt or inertia llbU ut .X .l\XiS

1111yf.

<7A0ul

L m ;:l:l.

(7.40b)

;:I

momem or inenia about y -axis =

l l

The transition from the discre te case to a c o ntinuous dis tribution in the form o f a thin plate
wi th constant mass per unit area p is not al wruys so simple as for 11rst moments. f:irst consider

Cbapcn 7 Applications vf lhc Dcl'inile huc:grnl

400

the moment of inertia of the plate in Figure 7.60 in Section 7.7 about they -axis. The mass of
the rcprcscntath-c rcctangJc is

p[j(x) - g(x}]dx,

and each point of the rectangle is approximately the same distance x from the y-axis. The
mont<: on of inertiu. then. of this <~:<:tangle about ~te y -axis is approximately

The moment of inert ill of the plate about the y-axis is found by adding moments of inertia of
all such rectangles and taking the limit as their widths approach zero. But again this process
defines a definite intcgrul, and therefore the moment of inertia of the plate in Figu.re 7.60 about
1hc yaxis is
(7 .4 1)

I EXAMPLE

7.26

Find 1he momem of inertia about the )' axis of a thin plate with
if its edges are defined by the curves

y =

AI tH H f) 110$ 1

Mon~nt

of

y= ~-

.-t3,

con~ 1 am ma~~

X=

per unil area p

0.

SOLUTION Since the moment of inertia of the vertical rectangle in Figure 7.65 is

inertia of plate about y -axis

") = ~2-.<

tbe moment of inertia of the plate is

x-

.T

In the forst integral we sctu = 2 - x. in which case d u = - d.<. and

8u'lfl

= p {

I EXAMPLE

8115/l

2u1/l

-3- - -5- + -7-

}2

256J2 - 319
210

p.

7.27
Find rhe mom em of inerticl about the line y = -I of a lhin pla1e of constant mass per unil area
p if its edges are defined by the curves
X

= 2)' .

7.S M()rl1t.nts of lne1tin

[fj Kclii;IWI!fj.

467

,\ (cxneru of inema of p!ille about the ltne J = - I

SO l UTIO 'I/ Since the directed distance from the line y = - I to a ll points in the horizontal
rectang le in Fig ure 7.66 is approximately y I ~ the moment of inertia of the recta ngle about the
line y = - I is approximately (y
I ) 2 p(x2 - Xt) dy . It follows that the moment of inertia
of the plate is

=p fol <-y'+3yl+ 2y) dy = p { - -~5 + y}+i}: =

FJOURE 7.87

inen&a

o(

!OC'Iangle

~tomc;nt

~tOOul

vi'

.r-..1xh

28p
5

In Examples 7.26 and 7.27, and in the discussion leading to equation 7.41. 1\'C c hose rcc wngles
that had lengths parallel to the line about which we required the moment of ineotia. This is
Jt()( j wa coincidence; 1he use of perpendicular rectangles is more complicated. Consider, for
instance, findj ng the mome-nt of inertia about the x-axis of I he plate in Example 7.26 . To usc
the vertit;:tl rectangle!\ in Figure 7.65, we firs t require 1he moment of inenia of suc h t1 rettangle
about tl1c .T-axis. The mass of the rectangle, as in Example 7.26. must be multiplied by lhc
square of tile distance from the x -axis to the rectangle. Unfon unarely. different p oints in the
rectangle are at different d istances. One suggestion might be to concentrme all of the mass of
the rec~;mgle ar it!\ centre of mass a nd use the di:sro.nce from rhe Xaxis 10 che centre o f mass.
This is incorrect. T he cemrc. of mass is a point at which mass can be conccntrarcd i f Y.'C arc
discussing fi rst moments. We are discussing <eoond momcnrs. (In Exercise 12 we show that
the moment of inertia o f the rectang le in Fig ure 7.67 cannot be obta ined b)' concentrating its
mass :u its ceorre of mass.) What are we to do then'] To use this type of rectang le, \VC rnusc first
de,e lopa fonnula for it.s momenl of inertia. To <Jbtctin this fonnula. ICI us consider the moment
of inertia about !hex axis of lhe rectangle of width lr and length Yl - y 1 in Figure 7.67. If we.
subdivide 1his rectangle into smaller recta ng les of widrh tly , rhe moment of inenia of 1he ciny
rccranglc abou1the x -a.x is is approximately

The moment of ine.nia o f lhe long, ,ertical rectangle can be oblained by adding over :lilt he tiny
l'tclangles as llleir widths dy ctppruach lero.

(7.42)

468

ChapiCr 7

Appli<:3Jjons or the t>etiniiC l nt~~r:ll

We can use this formula to state that the moment of inertia about the x -a~is of the vcrtica.l
rec.Langle in Figure 7 .65 is

~(y~ - y:) dx

~[(2 - x) 312 -

x ]dx.

The moment of inertitl of the plate about the x -axis is !herefore

r=

1'

!:[(2- x)lf2

- x9]dx

o 3

= !: { _:(2- x)S/2- x'ol' =


3

LO

( 16J2 - 5)p

30

1lte alternative procedure for this problem is to use horizontal rectangles that are parallel
to the

x -axis and obtain two definite integrals:

1'

I=

y2pytf3 dy

J,''\,2p(2-

y2)dy.

In summary, we have two methods for determining moments of incnia of thin plarcs:
1. Choose rectaJlgles parallel to the line about which the moment o f inerda b required. in
which case only the basic idea of mass times distance squared is needed.
2. Choose. rectangles perpendicular to the line <1bout which rhe momenl or inertia is required,

io which case fomtula 7.42, or a similar formula, is uccdcd.


As for fi nding cenu-es of mass. we could. in the special case of unj tOrm mass disuibuLions. drop

all references to mass and talk about second moments of area about a ~Jle.

I EXAMPLE 7 .28
Find the moment of inertia about the line x = 2 of a place with mass per unit area p if its edges

IOjlcJilii4Dm)
~lomeot of
inertia of 1>1ate abom line x = 2

arc defined by llte curves


X=

SOLUTION We lirst d ivide the p late into horizonta l rectangles of width d), and then subdivide
thi:> rcctang1c into smaller recta ngles of widlh dx (Figure 7 .68). Since the directed distance

x=2y - y 1

F==!==~

y'- 2y.

:dy

from lhe Une J. = 2 to the ti ny rectang.le js x - 2, the moment of lnertia of the tiny rectangle
about .\: = 2 is
x=2

(x - 2) 2 pdydx.

lt follOWS that the moment or inert ia or the IOiilg ho rizontal rectangle HboUL X
X

x,

II

.}x'

(.r - 2) 1 pdyd.t = pdy - (x - 2)'

x,

p
3
3
= - [(.r? - 2) - (.rt - 2) l dy.
3
x,

The moment of inertia of the entire plate is nQow

( =

1
0

p
3

- ((X2- 2) - (XJ- 2) ] dv

(
,
Jp Jo
[(2y- )'- -

"3 lo (24y -

2p (

1184p

105

'}

3-

2} - (y-- 2y- 2) Jdy

1 2y~

2p{
2
J
= 12y - 4y
3
.

+ 8y3 -

+ 2.y

2 is

12y'

J2yj
5

- --

+ 6y j

+ Y'6 -

- y 6) dy

y' }l
7 0

22. rhc polar 1nomcnt of incrt1a or a rx>int nutSS m at (.\' . y) i:s de lined
the product or Ill and the square ol' it~ di,t.ancc rrom the origin,
Jo = m(.~ 1 + y 2). Fon l>: 1hin pl01c (wid> con>IWU '"""' pcruni1nrea)
i n the fi gure below. k:t I,. and I> be it$ Ulon'IC:'ntl: ol' ir-.enia obout the
.r~ and y~:o;c~. Show thnt J0
1,, + 1,..

In Exercises 23- 25 the

)'

c ut\'C!<>

define

!L plntc

wilh

Ma :->'\

per uni 1 u rc:L

equal to 2. Find its moment or i ncrti~ abott the line UC'-'\Iratt to three
decimal places.

:1$

i + 23 . .r= 1 - .<' .x=y' .


il + 24 .\' = .r1 - X . )'= ,jX,
il + 25. y = .t 3 -

' y

= fi ,

aboui )'=O
abOUl X :

""""' y

=0

.. .. 26. Find the ~(.'COil<.! ltlOUX:nl o r urea or .. rce1arlgk;. about its diugonal.

l7 .9 Additional Applications
Volumes by Slicing

Jr we ca11represent the arc.a of parallel cross~sections of a volume a" a function of one \'ariablc,
we Cl.Ul use n definite integral to cttlculatc the volume. I n particular, when we use the disc or
washer method 10 detem>ine the volume of a solid of revolution. parallel cross-sa1.ions are
circles. In the following example. parallel cross-sectio ns are squares.

I EXAMPLE 7.29
A uniformly tapered nx! of length 2 111 has square c.~ss.cctions. If the arec:L'S of its ends are

4 cm1 and 16 crn1 as in Figure 7 .69a, what is the volume of 1he rod?
MMiJII;Iftl);W
f.C\:Iii>IU

Volume of capered rod with squ&re <:tO!>~

A(.t)

~f
~=~
' ---- -- ~~~ - - - - ~

,C:::::.
r
:--~-- - -
- ~-

/~-...

- - ~ ~ ---

...

~ - -~ -- --

~. 1

l = ~~
X ~-::---Jl
dr

.1

.~-

If we define an x-coordinatc perJ>Cndicular to the squa,c cross-sections as in


Figure 7.69b. then the side lengths of the cro~s-sec1ions at x = 0 and x = 2 lli'C 0.02 m and
0.04 m, rcspecti\'cly. Since the rod is unifom1ly taper-ed. the side length of the cross-sec1ion a1

SOl U110N

x is

0.00 + x (

0.0-1 - 0.00)
2

+x
= O.o2 + O.otx = 2 100

1l1c area o f rhc cross section a t x is rhercfore.

If we construct at x a slab of cross-sectional area A (x) a nd width dx, the volume of the slab is

7.9

AJditiQe~,!

Ar,clk;:llions

471

To obtain the volume of the rod. we add volumes of all uch slabs between x = 0 and x = 2.
and mke the limit as lheir w_idths ::.pproach zero:

Area of a Surface of Revolution


If a curve in thexy-plane is rotated abuutthe x - or )'-axis (or a line parallel to the x - or )-axis)
in order to produce a surface, w'Ccan calculatc thcarcaofthis surface usi ng lel'\glhs along curves,

discussed in Section 7.3.

I EXAMPL E

7 -30

........
If that part 1>fthc parabul a )' = x 2 between x = 0 and x = I is rotated around the y-a.xis, fin<l
the area of the surface of revolution traced out by the curve (Figure 7 .70).

SOLUTION We approximate length along [he parabula corresponding toll change dx in x


by the tangential straight-line length:

(dv) dx = J 1 + (2x) d.
2

I+

-
dx

If this straight-line segment is rutmcd around the y-a., is. each point follows a circular path Q( radius approximately equal to x; therefore, the area traced om by the line segment is approximmely
equal to
(2rr.r) (

J! + 4xz dx).

We find tOilll surface area by adding all such area~. and taking the limit as widths d.~ approach
1.ero:

A =

I
/.0

2JCxJI

+ 4x 2 dx

= 21T

{ (I

+ 4xl)3/ 2 } I
12

(S./5 - l)lf

472

Olap:er i

Apphc:tlliotti ol' lhc Definile hltcgirll

I EXAMPLE 7 .31
Find the area of the sutface of re' olution traced out by rotating that pan of OlC curve y = x 3
between :x = 1 and .x = 2 about the x -ax i ~ .

SOLUTION We approx imate length al<)ng the cubic corresponding to a change dx in x by


the umgcntial straight-line length:

I f this smt.ight linesegmenl is rotatcdabouLthc..\' -a:<.is (Figure 7. 71), each point follows a circular

path or rmlius approximately equal to y; therefore, the area traced out by the line segment is
approximately equal to
(2Jr y)

( JI

+ 9x' dx) .

By adding over all such areas. we obtain lhe area of the surf~1cc.

( 1453/2 - lolf').~r

27

)'

Rates of Flow
In Problem 3 of Section 6.2 we considered laminar blood flow in a circular vessel . Specifically,
velocity of blood through the cross section in Figure 7.72a is a function ofmdial distance r from
the cc.mre of the vessel:

where c > 0 is a constant and R is the radius. o f the vessel. lf we cQnsuuct, at radius r. a thin

ring of width dr as in Figure 7.72b, then the :urea of this ring is approximately (211' r) dr. Since

v docs n()t vary gtea~.1)' over th i'i ring. the amount ()f bloocJ Ocv..vinR through the ring pc;r u nit
time is approxin1ntcly v (r ) muhip1icd by tl1t: :uea of the ring:
v(r)(2:>rt' tlr).
We can find the total flow through the blood vessel by adding (lows through uH rings and taking
the l i mit ~~ widths d r o f the rings ar>pcoach zero:

= Jo{ R v (r)2trr dr = 2;r ffo" rc(R

UOJHIII I Q
..c-1

:n:M

<.:in::ulo bt~~~..d

\'CS-

or ki i t~ R

= 2tr< \fl- 2r-

1 1

r ) dr

r' \"

Mij l,.jll UUD!lih

R~t~.e

through~ ~;i o:~tlnt

\'~s;.e l

hloud

Jt cR'

4 f0 = -

2-.

of biQOt.l llQW

Elongation of Rods
~looke~ htw s tates that forces exerted

J,, purticuhil'. suppo:\e x

rroOOO'- :
X

by ~llritg> nre proportional to ~tretch and compression.


measures distance tot he right in Fig~m; 7.73 m'M.l ,\" = 0 corresponds to

the right end of the spring when the spring ls lU\SifCh:hed and UIK'Qrnpre~~. Hooke's law then
Slcucs that a llm::e F applied lo the :-.pring :ts s hown causes stretch

WhC'n spring
unHrctChc(l

whNe k is the spring cons1am.


Figure 7.74 show 11 rod of n8turnllength l.. When a force F is applied I<> the right end of
the rod. the rod a<.:t"' li kc a l'\pring in 1h~t t the rod ~rc:u.:he.~. It stretches only lllinutely even when
F is large; but it docs :stretch. h is l)hown i1lthe area of SU'C:Il.th of nUlterials tha t the amount o f

stretch is rclmcd to F by the equation

FL
X = --

A F.'

or

where A i~ the (coosU'hlt ) cross.~t in.,al area 1he md and C is Yoong.'s nOOlllus of elaslicity.
a consta nt that de1x:nds on the matcrinl of the rod. h1 effect. A f./ L play the role of spring
C<lllhtUnt

k.

t====:::J---- - L- - -

x=O

PIIO'ii"

Now
the rod i> 1urned ,enicall) and hung from ils lop end (Figure 7.75). Due 10
iiS "eigh1,1hc: roJ stretclu, and U>ing 7..13, we can calculate how m...:h. If we consider a small
length dy at positaon y . the force on each cross =tion in !hi< elen~nt is approximately 1he
S3ltl<. and equal to the weaght of that p an of 1he rod below i1,

pg(L - y) A,

where p is the dca\,ity or lhe nuuerinl in the rod. Accordiug tO 7.41, lhC clement d y stretches

by
P!i(l. - y) A dy

pg(L - y) dy

TOtal stretch in the rod i< therefore

I.

d,v

1
1.

' 2 }t. =

PII(L - ,1')
=.:..:.:,,---::..:dy
= -pg { L 1 - E
E
.
2

Thi may be <omcwhal urpri<lni that sm:tch does noc depend"" the crou=tional aoea
of the rod. This i< eplnined by th<: fact 1hat the weigh! of rod beloo. any cross-section is
proportional 10 CI'OO>:>S<:.:tionl un:a A. bul S!relcll in 7A3 i in\i.'l"<el) rroponional lu A . The
t"o factor> compcns.att. To get an ide-.1 of the magni1ude or thi< swtch. suppo:.c that L = 2 m.
p = 8000 t~n1J. und E 2 x 10 11 N/m 2 Then

P/11. 2
8000(9.8 1) (2) 1
-- =
= 7.85
11
2E

2(2

10 )

O1
111 ,

Suppo.w now that cros<-Scctionnl nrcn A of !he rod b nO! constunt: the rod is mpcnxl. say.
with circub1r CnJ'<SCCiions (Fi ~urc 7.76). Ra<Jiu< of lhc lmgc end is r ond ihC rod wpcrs 10 a
point ac y
L. 1llc f()rcc ()n crosssections of the elemenc of width dy at position y is a.gain
the weight of rod below il. IIMII:I y.

where ... i.< the rndiU> or the rod "'position y . Since""' equation of I he ide o( the rod is

y
1he weight below )' is

= --L
(x r

r)

The c Iemen I d)' lhcrcforc sue1chcs:

This is onc-1hird 1hc s~rc1ch for lhcnoniUI>Crcd rod. and hence 1hc 101ol strc1ch of the lltpercd rod
is pgL 2/(6 ).

Green's Functions
(;n>en 's functions are widely used to solve. engineering prublerns, especially in the presence
of quantities repr=mcd by Dirac-dclm functions (see Scc1.ion 2.5). We illustrate wi1h lhc

following problern for static delkctions of n taut string of negligible mass. constant tension r .
and length L. wilh ends fixed al x = 0 and x = L on 1he x -ax is:

dly

- r - , = F (x) .
d.K
y(O)

= y(L) =

0 < x < 1..

(7 .44n)

0.

(7.-l-lb)

Quami1y FC<) is the load per unit .<length on the siring. To lind y(.t) we would inlcgnttc tl1e
diller<ntial equmion 1wice and use 1he end condil ions to evalua1e 1he c<>nstanls of imegrn1ion.
For insrance. if F(x) k.r(x- L), where k < 0 is a consmm. integralion gives

I'C.t)

k(Lx3
- - -x' ) + Cx + D .

= r

12

The end cotldilions require

= y (O)

ThesegiYe D = Oand C

L'') +

0 = y(L) = -k ( -l:' - -

= D,

12

CL + D.

= - kL 3(12r),and therefore

Green s function=> provide an alternative way to solve such problems. TilC Grccns function
for this problem is

G(x; X )

= -[.I (L
Lr

- X ) I1(X - x)

+ X (L -

x) ll(x - X)].

0.45)

where l1(x - X) is the Hcaviside function (S<.'<:tion 2.5). Think of X as a parame1er thai can
take on any value be.tween 0 and L . Given x and X , G (x ; X) represent~ the deflection in the

srring at position x if a unit force in the positive y-direc1ion is applied at position X . Given
c.hm the lorad on the S.lring is F(x ), the OeRcction at any point .r is given by the definite intcgrnl
y(x) =

iL

G(x : X)F(X)dX.

(7.46)

We reason as follows: If G (x; X) is the dcncction at x due to a unit force at X , thco


G(x; X)F(X) is the deflection at x d ue to load F ( X ) tfX on that part of the string d X

and X. ln1egra1ion from 0 10 L gives The deflecTion at x due to the enTire load on 1he string.
Whe.n f(x) = kx(x - L ) . as above.

y(x) =

l,

- (x(L - X ) h(X - x)
Lr

+ X(L - x ) h(x - X)]kX(X - L ) dX.

It is always nece.~sary To subdivide the imegrdtton imo two parts. one from 0 to x. and The mher
fromx toL Sinceii( X-x) = OwhenO < X < xandil(x - X) = Owhenx < X < L.
we obtain

y(x) =

.!:_ ('
Lr lo

X(L - x ) X (X - L) dX
4

_ k(L -x) { X
Lr
4
kx

= -

12r

LX

}'

+ kx

Lr

+ .!:_ (L x(L- X) X(X Lr

J.,

{2LX

L X

L )dX

}L

(2Lr- x' - L').

This is the >ame solution as was obTained by solving 1he differemial equation and evaluating
ronstants. The difl'eremial equation and end condiTions are built imo the Green's function. The
definite integral in 7.46 lakes care of the loading F (x) .
If the string is subjected to a concentrated i()ad of F neMons attached a1 .r = L /3,
then F(x) = - F 8(.t - L/3), where ~(.t - L/ 3) is the Di.rac-dclt<l function. It is a very
complicated proccs. to solve the diO'eremial equation for such a load. Use of d1e Green 's
funcLion i s paniculatl)' simple. The deflection is agai n given by the integral i n equation 7.46.

where we use the property in Exercise 34 of Sec! ion 7. 10:

yC< )

11.. ~,tx(t- X) h(X- x) + X(t- x) h(x -

~: [x(L -~)~~(~-x) +~(L - x)h(x -~)]

~; [ 2<" (~ -X) + ( L -

I-;(L
" -Jr"'

x ),

.t} "

X)](- F }8(X - L /3) tfX

(< - D]

0 <X< L/3

L/ 3 <X < L

The graph is shown in Figure 7.n where we hcwe filled in the removable
X=

L/3.

M;;JIUII.i!JOO

Dcllt:ction ol' 3otri o~ due: 10

L
_ 2Fl.
91

<.1

puiut kl.kJ at x

= l.fJ

di~cont inu i ly

;.H

1.9 Addi t.iOn:ll A ppt i<.~hCH'IS

477

EXERCISES 7 9

J . Verify that the sutfoc~ area of a sphere of rndius r is 4,7 1'!.

2. Find the area of the curved s.urfacc of a riglu circular cone of radius
r :l!ld t.:igbt h .
3. 0 1kulclle the mlc ofOowufblo(ld thn;tugh a cin;.ular \'tS..."CI of rclCiius
R if the clocity profile is (a) v
f( r )
cR J R' - r ' and (b)
v j(r ) (c/ R2 )(R2 - r 2) 2

4. Find the vo.luroc o f the

9. Find the area of the surface of revolution fot'mcd by rotatitg about


the x -axis that pan of the curve 24XJ x<~ + 48 between x
I and
X= 2.

... 10. Find lhc area of tbc surface of rcvolut.iou gcncmt.c d by roWLing Ihe
curve 8y 2 x 2 ( 1 - x 1) about the x-axis.

11. Find the area or tbc .surface of revolution generated by rotaling the
loop or Lhe curve 9y 2 x(3- x) 2 about (a) the .\~-uis aod (b) the

pyramid i n the figure bcJO\Y.

x-a,;.is.

12. Thebase of;.l solid i.s. theclrcle x 1 +y2 =r 2 .andevery planesec~


tion pctpCndicular to the x -axis is an isosceles o1a.nglc Cfigutc below).
Pind Ihe vo lu111e of Ute !;Oiid.

f---- - b - - - 1
5. The a mount or wlllcr conswned by a t Olllmo.nity varies throughout
the day. peaking. nalurJ IJy. around meal hours. During 1he 6-h period
between 12:00 noon (t = 0) ond 6:00 p.m. (t = 6), we find that the
number of cubic mc.tres of water OOi lSUtncd per hoor at Lin"tC t is given

by the function

/(t) = 5000 + 21.65t' - 249.7t 3

+ 97.521 4 -

13. The end of t.bc rod at x = 0 in the figure below is rigidly fixed. If
a force with ltlag_nitudc F is applied to the right end. how lung is the
rod'?

9.680t' .

Find Lhe total consumption during this 6-h period.

* 6.

The number or bees per unit area at a distance x from a hi ,e is given

by
p(.r)

600000
= -(Rl + 2R ' .r - R.r, - 2x l ) ,

0 5 x 5 R,

l.

31rr R'
where R is the maximum distance trtl\'elled by tbe bees.

(namrnl length)

(a) What is the numlxr of bees iu the c.:olony?


Cb) I low l'nuny bees art within <1 distance: R/ 2 of the hive?

* 7.

If lhc radius of !he bloo<l vessel in Figure 7.72 is reduced to


because of art e ri osel~is, the velocity profile is

v(r) = c(R' - 4r2 ),

R/2

l4. In the figure below, a mass ,\'1 is placed on a vcrt i c~ll rod. If the
Jcogth of the compressed rod is L, wbut is its length iJ M is removed
aml thc rot.! is turned horizontally'? Assume tl~t M is so large tllat the
weight of the rOd c:ln be neg l ~ted in co.nparioon.

0 5 r ::; RJ2.

WhiSt J..'Crccntagc oflht:. nonni:il Oow (1CC R4 / 2) gc~ through the hurd~
ened ves~l?

I
I

* 8.

A tree tmnk of diameter 50 em (figure below) has a wedge cut from


it by two planes:. The lower plane is perpendicular to the axis of the
trunk, and togctbcr the planes m.akc: .rn ongJe 1t / 3 rndians. meeting
along a dia.mc.ter of the cLroular cross-section of the trunk. Find the
volwnc or the wotlge.

.t: l.S. \Vhal is the aJl.S \Io'CT to Exercise


inlt)

14 if the weight of the rod is takc:o

account?

16. Suppose the rod in Exercise 13 is turned venkally so that its top
end is fixed . !tnd I he l()I'Ct:: F puUs Yt::rlitaJiy downward on the lower
end . How long is lbt:: rod if 1hc weigh1 of lh e rod is also l.a_k cn in10
:~c co u n l '?

17. What happe ns when you attempt to find the stretch of the rod in
Figure 7.76 ir the pointed end is at y = 0 and the bugcr end is at
y = L?

1 ~. Suppose a mass M is auachcd to the lo\\-cr end of lhe l-!!pc:rcd rod


in Figure 1.76. It is so large that the mass or the rod is ncglig:ible by
com.parison. What happens when you attempt to find the SLrCtc:h in the
rod?
19. A tapered rod or kngth L has square cross-sections. The squares
on the ends ha\'e dimen"ioos a and b (b > a). 11le rod is placed in
a vcnical posi1i011 \Vith Ihe larger end below the smaller end. and the
smaller end li.xed in position (figure below). What is the length of the
rod in this position'l

where E ~nd I are mat('rial con$Lants, und F(.r) is the lo:tding. The
Groen's function for this problem is
I

tJ

G(x: X ) = - ( x - X) 1/r(.< - X ) - -61!: 1


61

Xx 1

+-

Z1

fa) Use formula 7.-16 10 lind deflection> or the beam when


F(.t) = k. wherck < 0 i<aoonstant (pclfllop<thcwciiht
of the beam itsell).
(b) W'bcrc is y(.\') tJ minimu.m?
y

I
I

* 21.

20. What is the length or the rod in E:<crc:isc 19 ir the rod is turned
upside dowo'?

2 t. Repcul Excrci:sc 20 ir u mass M i.$ di5tributcd O' "Cr the bott.orn of


the rod.
22. Repeat E'tcrcisc 19 if 3 mass M is di5tribulcd 0\"CT the boiiOIH or
chc rod.

(a) Re1>ea1 F..xercise 26 if1he lnading F(.r) = - F J(.T - 1./2)


i!i:due tosconcentraled lo:ld F :u :r = L / 2, :tndthc weigh!
of the board is negligible in comparison. Usc the property

ln Exer-cise 34 of Section 7.1 0.


(b) OrJw ag,r:.~ph of the di\1ing board . Is any part of i1 ~trnight1

28. When both ends of a beam are clamped borizontally. deflections


musl satisfy

d'r

Et -

+ 2J. Elc.:trons are nrcu from an electron gun at a target Wgure below).
The probability thai an electron qrikcs the target in a nng of unit
ala distance .x tfom the centre C\f the target is given by
p - /(x) -

- R5 (R3
3Jf

3
- .t ) .

d:r4

are:t

0 ;!: .< :!: R.

y(O)

= F(x) .

= /(0) = 0 = y(L) = y' (L).

11le Green's tUnclion fN this JrOblem is

G(x: X) = (x - X) 1 h(x - X)
6 /

Whut pcrccnta~'C of a cascade of electrons hit~ wiLhin a di~anee r from


Ihe centre of the t:us,ct?

x'
6EIL'

+ - - ( - L ' + JLX'- 2X')

::r=

t
I

~(X'2LX' + L' X)
211.'
.

1.. . . . ,.,. ,.

(a) t,;~ furrnula 7.46

Elc:.:cron gun

LO

F(.r)
k. where k
of tt.e beam it~II) .

24.

if

F(x) =k.whcro:k < Oi>a<'Oil>lant.

Orow o. graph of the deflected string. Is it symn1ctric a.bout


L/2 with minimum at x = L/2? 1;) thi:; to be
cxpocccu?

.\" =

25. Find Lhe solution for fonnula1 ..&..& ir the string i< subjected to the
consttlnt loading ofExcn!i.se 24 und :1 CO:lccntr:\lcd loa.dof F newtons :u
the centre of the string. Usc the property in Ex~rcisc 34 ofSc:clion /.10.

* 16.

{b) \tri(y thu.l y(x) ha~ a minimum 'here i1 ~hould be c~


pc<:ltd.

(u) find thc solutionorromlulu 7.44 fordeflections of the: 5 trin~


(b)

Static deflections y(.r) of a diving board ol"lengLh L and fixed end

x = 0 (figure below) must satisfy


<i' l'
/ - = F(t)
(/Xa

y(O)

'

0 < x < L,

= y'(O) = 0 = y"( L) = y"'( L),

tirul deflection~ u( the bcnm whet!

< 0 is a co11qam (pcrtmp('hc wcig_ht

29. Rcpc:u Ew~isc 28 if chc looding P(x) - PJ(.T - L/ 2) is due


to u COI')CC.ntrmcd load F ru .t: - ~/2 and me weight of the beam is
ncJ:,Ii~iblc

in compa.rison. Usc the propeny in

E>.erci~

34 of Sa:tion

7.10.

30. The vertical wall of a rnclangul:tr container Otkd \\ilh water h~


a \'Cr1ic:d rccl;lngular slit with tlcig.ht h :..nd width w. The upper edge
of the slit is H uniu below the surfnoe of the walcr. t:sc: the modified
c./fili from Soc1ion 5.S 10 show thaathe volume
ToniccUi law tJ
rate al which water runs out of the slit. assumjng th<lt thc conlajncr is
kept full. is

2j'[gcw [( H + h)'' ' - H"'J.


3

31. Twurig hl circular cyfind cr.s. cach ofnuli ~ r. have: axe:~ lhat inter
socl ~ I, right angles. Find Lhc volume common (O the (wo cylinders.
:t

docs it totkc for the waicr Lo C\'itJloratc co mpletely '? Nole Lhac Excrci:w:.
19 ln Section 5.5 is this same problem gi ..cn a co nuaincr tha i is a right
circular cone. Now we ask you torepcalthcproblcm with no knowledge
of the shape of the contain<:.r.

12. What is lht: vulumcufair lrapptxl in lht: a llic u fa hnus.c: if chc mnf
has sh<Jpc shown in the figure below~ ihc peal:_of 1hc roof is 2m all(m;:
the baM!.

\l
_I

10

'f..f<

* 33.

A c.ertaiu popuJation is being started at lime t = 0. Once born, the

throughout the year, pcrhaiJS of N / 12 al t11c lir.;Lof each monlh . 1n the


hmcr case she would incur costs due to popc.t'\\'Ork. dcllvcry chtrgcs,
~nd so oo. Let us suppose lh:atlbc rcta.iler decides 10 order i.n equallot
sizeS:, x. aJ equally spaced imetvals. N j:r; times pea year.

proOObilily 1h:il an indivillual livc..'\ to an age 1 is p(t) . F'o r time 1 > 0.


lhe birth mit: i~ r(t) individual~ pt:r u nit ti nte. Show that at tinte T the

num.bcr of individuals in the popuJuLioo is

:M. An (lJ)pliancc tttailcr must be concerned with he r inventory costs.


For cAarnple, gi,cn Lhot she sells N refrigcrat.orS per year. she 1nus1
decide- whclhcr LO onltr the yi':tr's supply :lt one time. in which c:t.~e
a large stor.tge area would be necessary, or to m:tke periodic orders

N(T) =
~

34.

(a) Ifeach order has fixed eosts of I; dollars. plus f dollm for

p(T - l)r(t)dl .

eac-h refrigett~tor. whaLare the total ye arly o rdering c osls1


(b) Bct"'cen succd!:ive deliveric,;, the rct:tiler't:: stock dwindles
from x 10 0 . Tf she assumtS 1ha11he nunlherof fef,ige l'atON:

(a) An orange is spherical with radius r . S uppose it is cut into


sJicc.s of <:qual width. lflhe lhickness Df each slice is 1, find
the area or peel on each slice ir thickness of peel i.s h .

in stock dcc.ascs linearly in time. tlJtd the yearly stocking


cost per refrigerator is p dollars, what are the yearly stock-

(b) Find the \'01 ume of p...~ l in c:~eh slice.


~*

ing t:(x;;I.S?

(..:) If che rerailer's LOt;tl yearly in"e n1ory cosc~ are her onlering
costs in pari (a) p lus her stocking cost-. in part (b), what

35. The ciepch o f '>'~".:Her in the ~.:on t ainer in the figure below i~ originally
10 cl'l\. Water evnpor.:~tcs from Lhe surface 01 orate proportional to the
are:.~ o f till:: surface. II' the wacer level d rops 1 t,; ll'l inS days, how kmg

\'alut; o l' X mi ni m i:~-..c~ inventory cosH;?

l7 .10 Improper Integrals


A ccording to Ihe first fundamemal rheorem of integrAl calculus (Theo rem 6.3). defini te i megr-.:tls

are eva luated with anciderivatives,

f(x) dx

{!

f(x)dx

1:,

pnwidcd 1ha1 / (x) is coruinuous nn tl $ x _s b . In Lhis sec.:liOn we investigarc what Hl LILl


when .f(~t) is not continuous o n a ~ x ~ b, o r when cithc.r a. orb is infinite.
Consider whether a reasonable. meaning can be given to !he integrals
oo 1
/ ,I

--:;j d X
,\

and

""
/,I

I
r.;
.y ..\:

dx.

Both integrals are. improper in the sense lhat theil' upper limjts are not flnile. and we therefore
call t.hem improper integra ls. If b > 1. the re is no d ifHcul1y with evalual ion aod ioterpretatioo
of
b 1
- dx
and

!, x'

480

Clup1er 7

Applkali(utS (Ifthe l)cfltlile lntqnl

Clearly,

l1 ~dx
xl

!,
iiilcliJ ;I

Area under

1 -~X lb =

~dx =

{2J.fl 1

v X

and

= 2../b- 2.

We can illiCrprct thc first imegmlns the area under the curve y = l/x 1 , above the x -axis. and
between the vcnical lines x = I and x = b (Figure 7. 78). The second integral hase <actly 1hc
same inlerprclalion bUI USI.'S 1hc curve y
1/../X in pla<c of)' = l/x 1 (Figure 7.79).
In Table 7. I we hiwe lis1ed v,llucs of these definite imegrals t'OI'I"esponding 10 various values
of 11- his clear 1ha11hc two in1cgrals display complelely different chumctcrislics. As b is made
very large. 1hc intcgrul of I f,r1 is ai\\"<~YS less than I , but geLS closei'IO I as b increases. In Olher
words.

1
I

y=z
.<

~dx

lim ( b
11- o::. } I xl
h

M:JICIIJ-J:WA)!U

curv< )'

.T

= I.

Geometrically, 1hc area in Figure 7.78 is always less 1han I bu1 approaches l as II approaches
infinity.
Contntst this with 1he intcgntl of 1/ ~.which becomes indefi nitely large as b increases:

..\ton lHitr

= If ./X

lim
b-.QG

!,

r.: dx = oo;

.y X

)'

1ha1 is, the area in Figure 7.79 can be made 11s large HS desired by c-hoosing b sufficient!) large.
TABLE 7.1

I
y;; -=

Jx

(I

J"

2XI dr

b
100
10000
I 000000
100000000

' I

ti T

..;;

18

0.99
0.9999
0.999999
o. 999 999 9')

198
1998
19 998

On 1he basis of these calculalion.s. we would like to say thllll hc im1noper in1egral of l/ x 2 ha>
value I, whereas the improper imegral of I f.fi has no value. Tn 01her w()(ds. if lhese improper
in1cgrols are 10 have values. 1hey should be delined by 1he lim its

I d.:c =

<><
/ , ,v2

lim

b-+oo

/,1' I

x2

00

dx

/,
J

r.:d,t = lim

v..r

b--:>J

/,b
I

r.:dx.

y X

[n the second case the limi( does not exist, and we intervrct this to mean that the improper
intcgn1l docs not exist or has no value.

In general. !hen, an improper integml with an infi nile upper li ntil is defined in terms of
limils as follows.
DEFINITION 7 ,4

If j(x) is cominuous for x :=:: tl, we delioe


00

1
11

provided that the limit exists.

.f(x) dx = lim
b -+oo

1b

f(x)dx,

lt

(7.47)

7.l0

lmpmpct"lnt<:gr2ls

481

Jf the limit exists. we say that the improjler imeg.ral is eqWII to the limit. orthat the improper
integral cmwerges to the limit If the. limit doe.s not exist, we say lhaCthe
no vHJuc, or chat it diverges.

I EXAMPL E

i mpmJ~r int~gral

ha:-;

7 .32

Determine whedter the following irnprQpcr integrals converge or diverge:


{ ""

(")

--~--, (/X

roo --;::=_.=2=::;: (/X

(b)

(x - 2)3

}3

(C)

lo i1 + .r 3

SOL U110 N

(a) Using equation 7.47 gives us


1

{ ro .,----.., dx = lim {''


(x -

}3

2) 3

(x - 2) 3

&-:x; /,

-I }b

(/x = lim {

2(x - 2) 1 3

b-oo

b~~ [~- 2(b ~ 2)2] =

2'

and the improper integral therefore converges to L/2.

(b) Once again equation 7.47 gives

f ""

2
x

dx = lim f b

Jo ,/1 + x>

ijUIII;I#I

ElillM

(c)

A1ta undC:I'

""'~ y = (2x' + 6)/(3 ..-'

'

b-oo } 0 -/1 + x 1

dx = lim

~ ~~~ + x 3 )

b-ea 3

and the improper integral thel'efOI'C divel'ges.


Though we cannot at present lind an antiderivative for f(x } = (2x 2 + 6)/(3x 1 + 5).
we can solve this problem by interpreting the improper integral as area. T he graph
o f f (x) in Figure 7.80 indicates that the curve i~ a~yn>p!Otic to the line y = 2/3.

+ 5)

Clearly. then. the improper integral

!,

oo

'

2x 2 + 6
::---;,----: d X

Jx2

+5

can have no value since the area under the curve is larger than the nrca of a rectangle
of width 2/3 and inli nite length.

\Ve can tleline improper integrals with infinite lower l imits in exactly lhe same wuy as for

improper integn1ls with infinite upper limits. S pecifically, if .f(x) is continuous for x ::; b. we
define

[~ j (x )dx = ,~':!"" { f (x ) dx,

(7.48)

provided that the limit exists. For example.

- oo

(x - 6)5 dx

{ -1 }5

liJn
. (/x = lim
,,~ -oo a (x - 6)'
,,.... -oo 4(x - 6)4 "

1
[-~
+
]
~-oo
4
4(a - 6) 4
lim

\Vhen J (x ) is (,."'QillintK)u:, for all x . we de tine

!""

j(.x)dx =

-oo

lim
a--oo

1<

f (.r )th

1b

lim

f (x } dx,

b--oo ('

C1

provided that bolh limits exist . The number c is arbitrary: e:<islcnce o f the Iin"'i ts is independent
oft: (see Exercise 33). For example, if we choose c = 0. then

! "" .,. ,.+- '_

00

x1 ) 1

(I

d< =

Lb +

10+
{ -1
+

lim
, __ ,.

lim [
, __ "" 2( I

xl) ,.

2( 1

1
I

= -2

+-2

lim
+ x 2)2. d.< + b-oo
o

lim
,,__,. (I

+a

2)

lim

"~""

2(1

(I

x 2)2

dx

-1
}b
+
x2) 0

~] + b-oo
lim [~ 2
2

2( I

+ b2) J

= 0.

I EXAMPLE 7 .33
(a) Is it possible to paint ~lC area bow>d cd by the cuncs y

1/ x, x

I. andy= 0?

(b) Rotate the area around the .<-axis "' ronn " IH\1 is somet imes ca lled Gabriel's hom.

Does it have: finite volume?


SOLUTION
ljlclill~

ostea undt

.r =

P2inti1lg

1/ x

(a) 'Painting is posslbte if the area of Lhc region lxxmdcd by the curves is finite (Figure 7.8 1). Since

oo 1
-:dx = lim
.t

!,1' -: dx =
1

b- ()0

.l

lim (In ( x i}~

b...,.oo

lim In I>

b-oo

oo.

1hc area is not til\ icc. and cannot 1herdorc bt:: painled.
(b) We use discs to determille whether Ga briel's hom has fini te volume:

!.""
I

1r y 2 dx =

lim

l~oc

lb"
I

.\'2

dx =

lim
to- <

~---"-}~
=
.t
I

lim
b- -:

(rr - Jr)
=
/J

If.

1l>e volume is litlitc. Co11sidcr this now. The horn can be filled with paint. but the
nat c;ross-scctional a1ca inside the horn cannot be painted. How can this be?

( EXAMPLE 7.34

tL'B!J. i1iJ I

f.M:ape

\'Ch.!cil)' i)( J)I'OjCcliiC ffOf11 CIU11f !>

grnvilatiumll field

x x+dx

Earth

Find the escape velocity of a projectile from tlile earth's surface.

SOLUTION Suppose the projectile is fired from the earth's surface with s peed 11 in the.<
direction (Figure 7 .82). If the projectile is to escape the e arth's gravitational pull, its initia l
kinetic energy must be greater tha n or equa l to the work done against gravity as the projectile
tmvels from >he earth 's surface (x = R ) 10 a point where it is free of gmvity (x = 00). Whe n
the projectile is a distance x from Lhe cenue orthe eanh, the force of auraction on it is

F(x) =

GMm

where G > 0 is a constant, m is the mass oflhe projecli le, an~ M is the mass of the can h. This
is known as Newwn 1$' unh,asctl law u.f~n1dtatiun. The. work dole agains1 gravity in mwelling
from x = R to x = oo is

f oo -GmM-

IV =

.xl

fbGmM

dx = lim

b-oo R

.l

- - dx
,r2

}b .

= GmM
R

GmM
GmM]
- = lm1 [GmM
-- - x
R
b- oo
R
b

= blun

Oo>

Since the initial kinetic energy oft he proj ectile is m v2/ 2. it c>capcs the ~ra>itatiomtl pull of the
eanlt if
I

- mu1 >
2

GmM
R

1ha1 i~ ~ if

Hence J2GM/ R is the escnpc velocity (notice chm ic is independem of che mass of the
projectile). tr we take the mcmt nodi us or the cmlh as 6370 km, its mean density as p
5.52 x 10) kglm 3, and C = 6.67 x 10- 11 , \ VC obtain v = 11.2 km/s. (Compare this with a
308 \Vinche:;torthat fires a 150-grainllltllet with a muzzle velocity of only 0.81 kml..)

A second type of improper imegrnl occurs when the imegmnd is disconti nuous at

tt

point or

points in the intcnal of intcgmliLijl, For example, inlegnmds of the illlJlruper integrals
5

1
= dX
],I -;=:
.;;:-=~
cnch have infinite discontinuities at
calculale
1

1
J
s

.;;:-=~

dx

1
l .,..-----,-:c:; d X
],1 (X - 1) 4

an~

= I . I f c is u number between 1 and 5. it is easy

A.

= j2.;;:-=i}s = 4- 2..;c::l

and

'

dx- {

- 1

(x - 1)'

3(.r - 1)3

10

15- -:-:--I~
f, 3(c - 1)3 -

192 '

and one possible interpretation of these dclinite incegrals is che areas in Figures 7.83 and 7.84.
If we let c 31JI>r()ach I from the right in the fim integml, we find that

1~
5

lim

c- 1* c

VA -

dx =

lim

c-1 '

(4 - 2..1C=I) = 4:

MjtUij;h.FJ!FD.

Area lllder curve J

.l'

)'

\ = - -

- Fi

y= (.< - I}'

256
I c

'

I c

"

= 1/ (,\: - 1)"

484

Chl\~er

Appl io~,.~ i<"iS of

the Detini1e l n1esr.-1

and in lhe second imegral.

[ 3(c

~ 1)'' - 1~2] = oo.

II would seem reasonable then to define the area under the curve y = 1 /~. above
lhe x -~xis. and belween lhe venical lin<::' ~ = I anu x = 5 a' 4 square uni1s. On the mher
hand, Ihe area bounded by y = I1(x - I)', )' = 0, x = S. and x = c becomes increasingly

large as ~r = c 1110\'~ c.:loser lox


I . and we canno11herefore assign a value to this area when
c = I. In other wo rds~ if we define

{~dx =

f.

lim

t -1 1

;
1

r.:--:

.yX- I

r.

s
dx = ,_
lin1,, j , s
1 (x - 1)'

dx

and

1
(.I' _ I)' dx,

Ihen Ihe lim improper in1eg ml has '' v:1luc of 4 , whereas lhc second has no value.

111cse improper imcgrals are examples of the general situalion described in the following
Figure.~ 7.85.

definition and illustraled in

rmtfJ.I D bt11
11'1'1pt'Upc f

intesmls

(or function~

MjiCJIIaiWMo@M

with infinite di.c:c.onlinoitie'i

{/

IJ .t

J'

/,X

"

"

DEFINITION 7 . 6

If .f(x) is continuous :n e very point in the interval a

f(x) dx =

lim

1b

c~a +

b except at x = a , 1hen

f(x) dx.

(7.50)

provided lhal the limi1 exis1s (Figure 7.85a). If f(x) is continuous a1 every poin1 in the
i111erval a ~ x ~ b excepl x = b , then

1
1>

f(x)dx

= lim

c- b-

1<

f(x)dx,

(7.5 1)

provided til at 1he liruit ex.is1s (Figure 7 .85b). If .f (x) is coninuous a1 every point in 1he
x $ b exceptx = d where a < d < b.then

interval tl ~

1
b

11

f(x)dx =

lim
~d-

1c

f(x)dx

I)

provided that both limits exist (Figure 7 .85c).

lim
1.> -'?4/+

lh
t:

f(x)dx,

(1.52)

I EXAMPLE

hnprQpct lnl ~ls

7. 10

7 .36

485

Detennine whether the followi ng imJHOpcr i ntegrals CtUl\'ergc or di'tergc::


2

(a)

lifi.a!: m3

Ill;

Improper
imctr.ll of /()"" 1/ (x - 2)'
)'

<

;--= dx

1_,'

(b)

- 2 (x - 2) 3

.
1...

"'7i tl.t:

5 ....,,
'

(C)

SOl UTION

(a) The gr:111h of ohe integrnnd (Fiure 7.86) shows a discominuioy at x

)\_
:2

_, (x - 2)3

1r

tl:r = lim
c-2

1
-;--~~
d.r
_, (.t - 2)3

fino

<-2

d ...

= 2. Hence,

- 1
2( x - 2)1

Jc
_

[2(c~ 2)1 + 3~] = -oo.


1

The improper integral therefore Uiverges.

kiJAII;I 'ill U I
integral o( / (.r) ~

hflJ)RJper

1/:-:_.

(b) Since rhe discorltinu ity is interiOI' to the interval of integnltion (Figure 7.87), we use
equaoion 7.52:

1c

-_,

lim
..!._ dx
c-o- _ 1 .-: 4

}''
-o { 3.c3 _

lim ['

c-o~

_,

lim

..!._ dx

}t: x1

{ )'
-1

.-

+ lim

c-u 3x 3 c

lim

c- u

s ince neither of these li1'11its exists , the improper imtgral di\'erges.

llili!Jil -r:JilJI Imprope-r


ime;r.tl of I (x) x 1../X=i
y

}' = -

(c) Since the i ntegrand is c.Jiscontinu()US onl y at x

I.

S --:=...
=

-' -

Jf we seo11 = x -

I c

= dr

.,;;t=l '
I . Ihen

lim
<-I

(S

Jc

= L (Figure 7.88).

..;;t=l

dx.

d 11 = dx, and

Thus,

f.
I

s --;:=x= dx =
.,;;t=i

lim
( ' - I t-

:(.x- I)J/l
3

+ 2..;;t=il s

L.,

28
= lim [28 - :3(c - 1)3/2 - 2JC=i = 3
]
c~l +
3
and the improper integral comerges.

linproper

1<1111

Various I)JlC'S of ornrroper in1egrals may occur in the san"' problem. For e.ample, Figure 7.89
shows 1h.111hc integral of {(.1)
l /(:r2 - I) from .f
-3 10 .f
on,oh es lhe u:.c: of
five limit:..:

/(XI - 1/V.: - ll

o(

'

)I

1"' --

dx

.\'~ - I

-3

.r

-3

1
-

= r-lim1
+

1'

dx

-2--

_ ,~t - I

11!

lim

r-

t- 1

r -,j"

+ c--"'
li m
fc

-- dx
x1 - I

lim

1
- dx
- I

+ clim
I

1.

<

--dx
2
-I

- - - tlx .

b-oo JO X1 -

If any one oflltc~c limi 1~ fails 10 c.'<ist. thcn 1hc irnpropcr integral tli\crgc~.
If an ioucgrund hu ,ojuonp di<eontinuity, such a. Ihe fuox1ion /(A ) i n Pigure 7.90, 1hen the
ionpm per integrol off ( 1) from x = a to x = b is define'(! in term of lwo limil>,

['
hn;>rlli)Cf
1111tt1fl!l c/ lll:l"-"liO'l ..,,,,. JI.I!)J)
ciM. tlltll uil)

f.

b f (:r) dx

lim

r-d

I.ef(x)dx +
a

lim,
c-d

1b

{(A ) dx .

but 1here i no quc>lion inlhi c:se that the improper intevrul c:on'e~e>.
c'

cl

.x

EXE R CISES 7 .10

In E ll.crd~ 1- 1~ dr:tctnbnc whclhcr the imprurx:r it\lcgrnl cnmert:e.Si


<.-. di\'CflC'l Hnll tht v.thtc ll w- cadl cc..1:vergcm in1e""l.

I.

J.

5.
1.

9.

II ,

(A

1'_, J.t'

4)' '' '

dt

-l

1_,' 'ro.r'
+
(

1'o

)l o/T

( I - 1)>11

d,T

f."" r,;;;-:-i

r/1:

'I ')' tl.f

1- .t

'

1_.

r
- ::u

ro. t dx

..f\"n dr
'+ ~

x'

I-

.ji':::X

= 01

,,,

ll. Rcpc:IE\crci<elllif .l

l ,and x =I?

tlx

1- x- '~< rcrbc:cdby,r -1- x->.

lJ. A h>Ciioo J (A) qwJorO<-. I' pm~lh<lt.T oktortyfwoct""lpdl)


x

d.r

, J x' -

1/", _r

(b) If !he region in p;~rl (a) iJ rol~tcd ubuutll~ lil'w; y = I, i:s it


possible to ~~~ign u nun'b..Y to rcpl't-S.Cilt it1 volume?

on the

inlcn~ l

~ 0 and

ir it s.isf~ two ctwKlitiom: f(x)


f(.r)d:r = I.

f.

(a) Show thJI

h ofth< followinJ ruocliOOI qtwliftcs

6.x

f(x) = (I

12. r l
-:ztl:r

0 ror

[I

p<Jr:

2.<

+ 3x 2)':

j(X )

(I

~ .<) l

- ~-t

14.

1'"

16.

(b) If a v:~rible

,T

L:

18.

4)2

d.<

x hilS pdf .f(.r) defined for .r

0 , then

the probt:ability ch,u .\ lie-; in .tn intcnl11 I is the definite


;o.,~.ll of J(r) o<r I . In paotlcul21. lhe (>rob:lbobly lllOI
x is grt'ater th;tn or ~uul

tn

i~

. in.td.t

[ '

.r'

,fil4

19. If /(X) j, '' C.'()lllinUOU!o odd function.


/(.l)tlx

boundet.Jby thccur ...c.. y- 1- .\

l t. Repe:;.txcrcise20i(\1 = t-x 1 1~is rt'pl.ttc:dby) = 1-x - lt l.

_..., (.t' - 5) , .

8. [

10.

(a) I ~ il possible co nuign o 1H1mbc:r to represent che urea

(r! - 2) d x

-"' (.r -

"

t x
J.:
11.

4.

20.

r
~ dx
'
(.t - 4)'/)

13. f l
.;; dx
15.

2.

Pix

Jx

~a)

Calculate P(.T
i<i\ il

/. ... /(.< ) d.\.

3) (or U<h pdf in p:rl (I).

necessarily trm: th.H


24. Verify Lhal

J.oc ~ tl .t convcrgcil' p >


I

I anddhcrges if

p :f I.

SunHnr-ry

25. The force of repulsion bc1wecn two point charges of lite .sign, one
of ~icc q um.J the other of size unity. has magn itude

or the llH)Sl important funt:tk"lfl~ in physics :md engineering is


1hc l)irac-<lelw jitllt'liOII ,j(x - (I) (imroduccd iJl Scc1ion 2.5 a> U1c
limn of the lLnil pulse function shown below):

11- 34. Om~

H.r - tr)
where 61>is 1 cons1m11and r i~ the tl i~t ~l ll Ce L~WCCI\ the cha.-gcs. 'J'he
potential V m uny IX>int I' d11c 10 charge q i~ de fi ned U." the work

1"'

26. Veri fy thm iff(x) ij:; coruinuOttS 011a 5 x ::;: /) exccp1 for a fi nite
discontinuity !U d (Figure 7.90}, then the inlJWOper imeg.rnl or I (.r)
fron1 x = a to x = b nw.~t converge:.

= .t(3 -

f(x)dx

- "\)

lim

.t) 2.

1.,

f(x )l(x - ll ) d x

1""

1"

U-

-- -

f(x)S(x - a) dx

i'

>

-;:::;.T:I

=d

,r;r.::T

31.

= f(a ).

func:.tkm ,;u v.:du:!.blc:.


X

30.

31.

_,
1

,r=x
.l' P~ (x- a)

- .o (x2 + 5)' Jx

o-o

33. Show thaJ exis1cnce of the limi1,; in cqwnion 7.49 is iRdepcndem


of the choice or c.

SUMMARY

f(x)P,(x - a )d.t ,

1'h is ir; Ihe fundamental opcr.uional prOilC.:It)' tha( 111akcto; the l>imc-dclla

Ui,ergcnt intcgrul.

!,

1'"

t -<1 - e.>

J:

show lhal

- IJ

In Exercises 29-.\ 2 detcnninc wllCthcr the impro(>er i11tcgwl c."OI\\'CI'gcs


or di\ctgcs. Hint: Comp.arc 0.1ch integral 10 a known convergent or

29.

11).

f(x ) [tim P,(.r - u)] dx.

f {x) f l im l', (x - a)]dx = l ion

->.}

f(x)dx

tr QO

(lim! P, (.r -

Assuming Out the ordl..-, of iiHC:gr!lliO!\ and the proc:cs.-1 of Lak.ing the
limit as l approaches zero can be interchanged, 1hou is,

,.- 28. ls cquut jo,l7.49 cquiva.lcnt to lhc foUovring equation?

! .,.

For a continliOtiS runccion f(,\').

n:quircd U) bring 1hc unit charge to P from infinily a!ong the sunig.h1
hne joining(! and I' . Find a fonnulu for V.

+ 2'7. Find the length of the loop of the: curve 9y2

487

"

a+

..

In this chapter we used definite integrals i n a wide variety of ~lpp licatio ns . \ Vich dill"erc ntiuls
and rcprescn1a1ivc clemems we were able 10 avoic.l n1emori2a1ion of fon,mlas. To summarize
the use o frepresemative elemerus in various app li cati on~. consider the regi on

R in F igure 7.91.

To li nd 1he area of R . we dmw recwngles of length J (x ) - g(x) and wid1h d.r. Areas
[f (x) - g (x)) d.r of all such rccwngl cs arc 1hcn added 10 find 1he lowI urea:

[f(x)- g(x)] tfx .

y
)' = /(x}

-11{l'C

<188

Ch~tcr 1

AI>Jlhc:nion.:: of ll'le 1Jeftni1e lnttVJI

If R is rotated around the x -axis to form a solid or revolution, the rectangle generate ~ a washer
wi th volume {JT(/(x) J' - JT [g(x)J2 ) dx . and therefore the total volume is

If R isrotatcdaboutthe yaxis. thercctangletra<:esoutacylindricalshell of volume 2JT.<lf(x)g(.r)) dx . and the tOial volume i<

1b

2JTx[/(x) - g(x)) dx.

First moments of this rcc.taug lc about the x-and )'"axes arc

2tf(x) + g(x) ][f (x) -

and

g(x)J d.x

x[f(x)- g(x)]dx.

and these le-.ld to the de.finitc integt;tls

1b

l
- {lf(.r)]~ - [g(x)J') dx
2

xlf(x)- g(x)]dx

for first moments of area of R about the x nn.d y axes. Second moments of area of region R
about the x. :md y .axes arc
and
where in the first integral it was necessar y to usc ronnult1 7 .42 for the second momcm of chc

rectangle.
Tite: key. the.n. to use of the tlefin ite integrdl is tO Uivitle the region imo ~mallcr clements.

calculate the quantity 1\.'(juircd for n reprcscntati\'c clement, and then use n definite integral to
add over all elements.
An important point to kee p in mind is that for lengths in the y -dircc:tion. we- a lways toke
'"upper y minus lower y: and ror length s in lhe .x -direclion. we use " larger .x minus smaller
x : This way. many sign errors can be uvoidcd when calc ulo.ting lcu.r:tlu. F'or instance. note tha t
this rule i!i u!ied 10 tlnd che lengch of a recmng l e (in areas, vol umes. fluid pressure). 1he radius
of a circle (in ,olumcs. surface area). depth (ftuid pressure}. and some d istances (work}.
An improper i ntegral is a definite integral \Vith an infini te l imit antUora point of discontinuity
in the imcrvul of integration. The value o f an improper l mcgral is defined by first calculating
it over a finite interval in which the i ntegrand is coltillUOus, and therl lett.i ng the lim.it o n the
imegn1l approach e ither infinity or the point of discontinuity.

KEY TERMS

In re\iewing this chapter. you should be able to tlcllne or c.liscuss the followi ng key terms:

Area
Volumes of solids of revolution
Washer method
Length of a cur"e
ncrgy
Conservation of energy

Flrst momeot of mass


MomenL of i oertia or second moment
Are-a of a s urface of revolutio n
Improper integrals

Represemative rectangle

Disc method
Cylindrical shell method
Wuti<
Kinetic energy
Fluid pressure
Centre of mass
Volumes by slicing
Green's function

REVIEW
EXERCISES

In Excn:i~ 1-5 calculnlc ror the region hounded by the curves: (a)

Surlo..-.: ot water

il,. area: Cb) chc ''olumc10 oft he sc~id" of rcvt.llutinn obtuinc.d by roUiting
x ;md .raxes: (c) ilS ccr1Croid: and (d) i1s second

the rq:.ion : bo\11 the

moments or area about the .r and )'axes.


2. y =x3, )'=2 - x , x=O

1. y=9 - .t', y = O
3.

X =

y2

4. 2}' = .t . )' + I =

I. X = 4- ~)'2

X , ,1'

=0

5. 2x + y = 1, )' - 2X ; 2, X = y + I. ,\' + X + I = 0

In Exercises 6- 10 And the ''olumc of the solid of rc,olution obtained

18. Find the surfa<.-c arcu of the vohutlC of the ~ol id of revolution ob-U!.ined by rotatin the region bovndcd b)' the: curve.-. 2x + )' = 2.
)' - 2x o 2. and .)' o 0 nboullhc Xoxis.

by rotating the I'Cg.ion bowt<kd by the cu_r\'CS abol.lt the l ine.


6. .' ' ~

l.xl + 2. y

uboul y

olxJut .r

7. X - y' . .r - 4,

s. y = x1 - x. y

3.

= 0. "b<>ul y = 0

9. y.r'= 1..r= I, .r= 2, .r=O,


10. .v

In

19. ,\ tunk in the (onn ofun ill\'CI1Cd rig.ht Cil'(ulurconc Qf tiCJ>th 10 Ill
and mlli us ~ m is full of water. liow nux.:h 'A'()~\ i" n::<juinxl to lower the
v.:dcr lc,el in the t~mk by 2 m by 1>1.nnping WIIICf out through an outlet
I m :tbO\'Cthe top ol' the tank'?

=4

:obouly= - 1

= x + 1. )' = ./4 - 2x, )' = 0.

Exl'T\:1~

abou1 y

20. Show that i( it ta l..c~ \V un i t ~ of work to ~trctc: h ;i ~i ng rmm


cc)Uilibrium to n c:cnuin stretch, thl!n i 1 n.~uire~ 3 \V n'IOC\: un it~ of work
to double th:t.t :Un:tc:h.

= I

In Exc.:rcii'C..;; 2 1- 28 dctt:rminc whether the imjU't'ljX:r' intt'&I'J.i con vc rg~~

ll-15 fine.! the li1'Sl ami t:c:cond ttiOffK'niS o( area ol' lhe

or di"erg...'S. E\alwte each convcr~c nt i ntcsral.

region bounded by the c urves ::tboul thc line.

21.

r X "'
~ Jx

2'2,

12. 2x = ,fY. J'ix = .,fy=2,

x = 0,

13. 3y = 2x + 6. 2)' +X= 4, )' = 0.


14. 3.r = lt + 6, 2y

+ .\' = 4 ,

15. .v=x1 - 2x (." 5 0).

~~

y=O.

nbotu x = I

l 3.

abool y = - 2
abotu J =

= 2x. y=4 - 2x.

"*

~
ubot~x

25.

=I

16. A wmc.r tank in the fonn of a right drettl:tr cylinder with rndius
I m und height 3 m lms its axis: \'c;nicul. If it is half full. how n1uch
w(:ll"k is IU)uired to empty the tank tltt\)Ugh :t.n uullcl at the lop of the
tank?

" l7.

(x + I )'

dx

24.

---<IJ:
j3 - X

- 2 ./ 4

r~
, dx
(.< + 3)

2{). [

~~ .dx' + 4dx

2ll.

-X>

X
-

dr

x'

' - l - dx

_, Fx

rx

_, Cx' - I)'

dx

29. A unifbnnly tapered l'<xl or length I m tus ciu:ular cross..s:ections.


the: rudii of its ends being 1em and 2 em. Find the vol11mc: ofthC' rod by
(u) u~ing ahc fact that c vccycruss Stx.~l i t'n is d n::ular;uld tb)con~i dcring

+ 17. A ctir tu n~ orr a bridge into a river and submerge$. t f the I.Qs)S of
ils front and rear side windo\lo"S an: 2 m below the surface of the W'Jtcr
(sec figure). what i.s the fon:c due to water pn:ssurc on cuch ~Aindow ?

f l
0

>

>+:

tbc rod US Uvohnnc of UbOlid of I'CVO!ution.


30. In Exercise 16. how much wort:. is rct1uinx l to empty the tank ifi ls
U:tiS i~ hc.u itOnU.Il und the Oll(ict is ttl the top of the U111.k'?

Techniques of Integration
CHAPTER

Application Pre\'iew

Shown below arc a tractor and trailer in two positions. Borll mtciOr and trailer begin on rhc
x -axis (position A). The tractor then rurns co the right and moves so chat the pio.or poinr where
tractor and trailer are coupled wgether follows a curve whose equation is Y = g(X). Tn1c1or
and trailer are shown at some time later in posirioo 8.
y
\Path followed by pivot point

I
Path rou""ed by po1nr
l>ct\\CCO r"r w~l;

Position A
Give.o that the pivot point moves along the curve Y = g(X), find tile
equation of the cune follo"ed b) the point midway between che rear wheels. (See Example 8. 13
on page 511 for the solution.)
In Otapter 5. we discussed three 1echniques for evaluating anrideri,aties. Firsc. since
every <llfferentiation formula can be rc lilted as an ancidiffercntiatiorl formula, 11 foUo1~s 1/Jalthc
more competent we are aJ differentiation, 1he more likely we are to recognize an antideri,ative.
Our ;econd technique was called "udjuMi ng constt\111$." It is npplicublc co sutliciently simple
intcgmnds thot we can inunedintcly gucs~ tl1c unCdcri vati~c 10 within u muhiplicati I'C constnlll.
his then a matter of adjusting the constant. The third techu iquc WIL~ 10 change the vari11ble of
intcgl".uion. By a suitable transformation, replace a complicated amidcri\'ati\-e wi1h a simpler
one.
In this c~a~1er we develop additional tectJ!iques applicable to more wr.tplex imegnuion
problems. Al the same time we t.dc lhe opponuni1y 10 review lllc apptkations of deli11ite
integrals in Chup1cr 7.

THE PROBI E\1

8.1 Integration Formulas and Substitutions


Every differemialion formula c:1n hc re<lated as an amidifli>ren1ia1ion formula A pamaflist of
frequently encoumered formulas is a< follows:
,11+1

u"du = - - + C,
11 +1

!_ du
Ll

490

= In lui+

C:

-1:

(Kial
(S.tbl

~I

+ C;

f e" tlu = e"

lnlej!ratkK'a fQrnmllS and $Uth(iUlli<IRS

(8. 1cl

Ill"tlu = ll"log., e + C;
cos u du

!S. Idl

= sin 11 + C;

sec2 u d11 =

1011

11

<N le )

+ C;

f sinudu = - cosu

(lUI)

+ C;

(8.1g)

+ C;

f csc 2 udu = - cotu

(8.1 h)

+ C;

sccu lanudu = scc u

(8. 1i )

j c.scucowdu = -cscu + C;

t8. lj )

= In I sec ll l + C = - In cos ll l + C;

f collltlu

=-

1an "tlu

ln lcsc u l

scc 11 du = In I sec u

+C

= ln l sinu l + C;

+ tanu l + C;

csc udu =In lese u- cot ul + C;

f J1f

(8.1 n)

1
- -- , d11 = Tan- 1 11

+ C;

(8. 1p)

sec-1 II + C;

(8.1q)

t/11

l +u
I

11Jul - I

J
I

du =

cosh

11

t/11 = sinh

11

+ C;

( ~ . 1 r)

sinh

11

du = cosh u

+ C;

(8. Js)

+ C;

(8. 1!1

Jcsch~
sech lllluth
csch

(8.1m)

tS. Io)

u1

scch 11 tlu

(~ II)

+ C;

(S.I k)

II

= Sin-

491

11 colh

tanh u

ll

tlu = - coth 11

+C;

( ~ .l u1

11

tlu = -sech u

+ C;

(8. 1vl

u du = -csclt u

+ C.

(8

IWI

All of 1l1ese except Fom1ulas 8.l k, I, m, n are restatements of d ifferentiation fonn ulas from
Chapter 3. Formulas 8.1 k and 8.1 m are ver ified in Example 8. 1 below. fo1mulas 8.11 and 8. In
are similar.

I EXAMPLE 8.1
Verify formubiS S.l k and S. lm.
SOLL 'l10 N

ToverifyS. Ikweexpresstanu intermsofsinu and cosu:

tan udu =

sinu

- - 1/ u =

cos u

- lnlcosrt l + C

= In lsecul +C.

If ime~'nllion to In I cos ul is not obvious, II)' the substitution v = cos u .


To obtain the antiderivali,e of sec u requ ires a subtle trick. \Ve firs1 multiply numerator
and denominator by sec u + 1>1n u:

sec 11 d tJ

l fwc now set v

sec 11

sec2 u + sec u tan u


sec 11 + 1.11111
du =
d u.
secu + t.anu
sec u + tc:utll
/

= sec tt + Lt'Ul u. Lhcn dv =

f~cc rttht =

(sec 11 tanu

+ sec2 u) d u. mH.I

j ~dv = ln lvl + C = ln lsccu + ,,;nu l + C.

We could list many other illlegrmion fonnulas. but one of the purposes of this chapter is to
develop intcgrat.iol techniques that eliminate then~ for excessive memori zation of fonnulas.
Our emphasis is on deYelopmerH of amideriva1ives rather 1han memorization of fonnulas. und
we prcl'cr to keep the li ~t of formulas ~horl.
In Chapter 5 we dcmonstntlcd how a change of variable could sometimes replace a complex
imegration problent with a simpler one. ' Vhen the integrcllld comains x Jfn, where n is a positive
inleger, it is often uscful loset u = x 1f n, or, equivalently. t = u'1 \Vc illustratc ln the followiug
example.

I EXAMPLE 8 .2
Evaluate the fo llowing indefi nite integrals:
(a)

' 2- ./X
r.: dx
2 + v~

(b)

x ' IJ

-~~ d~

1+

(c)

x zn

..fi
+ .r 1/ l dx

SOUITION
(a) lf wcse1 11 =

2-

..fi or x

tJ , 11tcn

..fi
r.: fix = /

2 + v

dx = 2u du, and

2 -u
(2ulfu )

Z+ u

=2

2u -

11

2+u

du .

Long divisiOil of -11 2 + 2u by 11 + 2 immediately gi\'CS

!2- ../X

r.: dx

2+~A

=2

!( - u +

= 2

u'
( -2 + 4u

4 - -8- ) dtt
u+ 2
- 8 In lu

+ 21) + C

= - x + 8../X - 161n (2 +

..fi) +C.

S.l lnregr:llion Fonnuras an.l Subsrinnions

IJ we set 11

(b)

= x I/ J or x = u3, then dx = 3u2 du, and


X l/3

-----:::-::
l +x 213

=J

11

(3u du ) = 3
l + u2

dx

l +u-

du.

Oivision of u3 by u2 + I leads to

dx = 3

_x_'/"7
3!-::3
I

+ x

(u -

c ~ In

+I

= 3
=

ju

du

+ 11) + C

~x 213 - ~ In (x 213 +

I) + C.

(c) The purpose of the substitution u x'l is to rid the integrand of fractional powers.
Tn the presence of both x 111 and x 1fj, we set u = x 116 or x = u 6 Then dx =
61t 5 du, and

.
rx
J

.
_:.......,.,.
l +x 11>

dx

6j(u - 114 + u2 -

J+u 2

(6115 du)

(and by long division)

117

liS

= 6( - - -

I+

+ u-.)du

113

+ -3 - 11 + Tan - 1 11 + C

EXERCISES 8.1

ln Exercises 1-22 evaluaJe !he indefinite integral.

f
f
s.
f

I.

3.

7.

xl

5 - 3x3

d.r

(x' + 2jl/l
41

+g

t> + ~I + 5

J+
(x

15.

d.t

4.

sinO

13. )
t/IJ
cosO- I

14.

x+ 3
t="7-:;

v 2.r+ 4

e'

- -,- dx
I +e.._r

e'
--dx

J + '-"'

18.

J ~JS)I
(x

l )(x' "- 2x) Jl tl:t

~-

21.

x'
tlx
( I + x')'

.
f

(x - 3)lll

dx
(x - 3)11 + I

~ond

22.

lhc longth of the curve y

In (>S .r) fmm (0. 0) to

./X, .4 dx

+.r

(r./4, - (ln2J/2).

x+ 2
x+ l

- - dx

dx

20. lran 3.rdx

dt

2J.

I I.

d.r

12.

x'+ 2

- ,dx
x +I

24. find the area of Ote region bounded by tJte curves

x1 +

X+

I '

\'= - -

X+ 3y

= 7,

493

" 25.

(a) (3
Fond the
ccwu
Itt.: reg;
X )l/l
. X ;d oI ,f und
y =on0.bound ed by the cun"C>; .v

(b) What i$ ils SCC.'Of'ld mornen1 of area ::tboutthe 1inc .r


.. 26. If / (x ) is a continuous e"en func1ion. pro"c thal

!"
n

f(x) d x = 2

{"
lu

j'(.r) tl x

* 27.

(x '

ov.du;~tc

3.~ + 4)3/z d.r

32. S ho w 1hm the Subsritulion u

i nk'grnl
ofu.

0.

t ..

/ x

rc:tistuncc force acing on it lhut ir\ newtons is cqul.ll toonc-halfit.:sspccd


(in nleu..:s pel' second). Assuming th.u the 1>amtroopcr lhlls ' 'CrticuJiy
downwurcJ a11d th:llthc: parachute is a!n:.&dy OSlen when the ju rnJ) lake:.
place. chc dill'l.:rcntiul cqu:aiorl <k'scribing his motion i.s

33.
2 Show lh.at when the qu~1dt;,11ic c + bx - xl factors as r + bx = ( p + .t)(Q
- .x). thcn chhcr o f the substi1utions (p .,. x ) u =
2
Jc + bx - --. or (q - x )u = Jr + bx - x l: repl~'CS lheintt2,ral

1 =;;:::;1===. d::r
./c + bx - .r'

with lhc intcgrd.l Q( a l"'dtio11a.l ru11c1ion of 11.


,..
._

34. If tt . IJ. ~md 11 n.rc constants. C~Jaluatc J .c(a

+ bx)" clx.

3S. The: rurK:tion


f (x )

= 1962- "

J xi + bx + c replaces the

(b) If tho probab ;lity that x ~ 3 ;s 0 ..5. "hul is ;..1


2-8. A p:.ratroopel' und his porachu1e fall from rest wiLh a combined
111illl.'$ of I 00 ~,g. A I i;illy irbtant d ul'ing dc:socnt, 1he panachulc: 1\i.Js an oir

- x =

+ bx + c. dx with the imctral o f a ration~\! functi-on

interval x ~ 0. See E~crci~ 23 in Section 7.l0 for the


definition of a 1x1r.

200tit

/(x)d.r:

Jx>+ 3x + <I tu

.r .,

( a) Sh(lw that the functi<H1 /(.r) = ;.e - lx. whc.Tt A > 0 i~ a


constanl, qualifies as a probability Ucn$i1y func:tion on lhc

dt

11

= I?

and iJ f(x) istH'Ontinuou.sodd flmclion.

! ""

3 1. Usc the ..,b,.;tution

v21r 0'

<

-oo < x < co.

where tL ;md a arc cunstums, is (.'UIIed th~ normal prvbobil ily clcusi1y

where v is velocity ancJ 1 is ti rnc.

f,,,,.,ion.

Co ll5t--q~ntl y.

(a) find vas a function of 1.

r:

(b) If x mc:t.')UfC.S v~rtkal di.StancC f:tllcn, then V

= d .'( /dt .
Use I his 10 fi nd the position or1hc paratrooper a.s a tt mction
oftinle.

f (x) d x

17

29. E,-..Jumc /.'

.J 1 + cos x tlx . flhu:

30. Ev3lwtc

fot any n

x(3

+ 2x)

cos .r = 2 co<' (x / 1) - I.

tf the expected v.IIue or x is defined ns

} "" .<f(x ) dx .

> 0. Hitu: Find A and B in order dw

show thai /it is the expected "aluc.


1

--,---::-= = x

.r (3 + 1x )

Bx"

+ 3 + 2.r'

,.._ 36. Evalunte, i f p oSS;lblc.

l..t ,rl ...


"' !.tl ~/.,, whc. .. IJ

f "':U f

-=

18.2 Integration by Parts

, ,

-nstanL
"'"

One of the most powerful techniques for tinding aUidcrivotivcs ;s that called mtcgrauon b.
paris. It resul tS from the product ntle for difh:renllal on:

d
du
- (u v) = u tfx
dx

tiu

+ v- -

1
~x

If we take antiderivmi ves of both sides of this equation with respecl to x we obtain

-d (uv)dx =
dx

u -d v. dx+
dx

du
v-;j:dx.
x

J....,. - .... - J.......


t.t'l"l<>o< .~~-11<>;1

c..., '""'"...,"""''b~
r" n.. ~..........,..
'""'"'" ';oy

h '"'t'' " ' "" "''" """"''~'"-"'"""''"..._

~be lcl o >olok .o~' ol ,:l), I t "'~"'" ob:. non::- o.-\~11' ,-.,,.
" '!:''"'"''"''''~''"'"
'" '" ~II U::<f 1t "'"'' <h 4 o'olho)" ,., u ~\1</ " - ll>eo< """'c..' ' "" ,, . . ,.,,. .,(<9v0 t l"'-"'40 > <'<:(>.WI...,. \\> ~ T.........
" ' '" ~...... .,,.. ,,;~ ......,. ;, ...,,..,.," lh<' . ...........,.

:,""'l<'""'

~<lc

.f"''"

Wilt\

'l'!ot: ,.,..,...,,., 1~ , ,.. o;"->to""' - ' d t- M> u ...., ol'lr '"""' l"'"eT"l ;..:. ~'"'~""! '~'~""' lh e ~-a~\ ""' ~
" '"">'""'""'1'1-. It !.;""""''~" ' " k>l d \.' he'''""'''~' ,....,,.e>lfo;o;n"'f """' ol/do" lul~IC.'""d ''''''"
c""' ltii<'P.flllr mm.Wll) , Tl"" ~'"'"'i'\ ,.

ti(IY,ffl>"t~!> tll<>t .,....

,,. .\'. ... d ...

wi ll cl,.,;ry ohc10e Oo;lca!'. 'l\ u- ;., ,..,~'"'''"" hy 1:0.0" "'" .,.., ~hi.fi \)t\)l)l..:m woe\\\"\\~\ ,k\\\'I'C: u 'o'k) 1,1 o,>
ln$vs;h u w toy that u d t> = ;t~" d ... ' l'h e t o: llt'i.: ('-""' l i<J":<"<ibi l'o\ic:c;:

" =

u e,x,

,,. ...,.~ rt x~
diJ=f! d ,OL',

'' =

c~ .

tltJ -

u -

,\' tt',

II -

l.

dt~

x d.'(~

d u a c.l,\ ,

,\ ' ,

du = cl.\' ,

Poo11u la 8.2 tl lt'rl gh't'. .

J .H ' tl,'l:

('~ .

Je'~

Ae '' -

d.'\: .

We fltt \'t" lhert= lh re r'C'fllltced im egrm inn of X i 'A whh l o\\t:.l;.t~ltit...>~\

\)r fx, \\ -..\t:fu\\\c: s\n\ l\\\ \\\:.Mi.\\1\,

The tin11l s;n lution h. thc f <;l"l..lfC

.H~

' t I,\ --

,tt'

"'+C

t'

'l'hil. tXaHiplc i llm~umc).thut iutc.grutitm by l)l\r\$ f t \)\l\<:.1:-Sl\ " 'm\tUt\\\ w \\\\ \ WO()t\\tt';)', i;)\'\e
mcnrnl II\ICJ)totiOl (frotn dv to u) u'U a ~ct..n\U hu~~mllun \\m\ \1. \'\4..)\lC{\\\\y $.\"'\'lk'r '''~'' \\'\e
u i~illul. We l"hould ubo nou~ thm i n tile e:vnhmti nn of the m enu\\ ln\Ctt (l\\\\\\ w~ U\\\ \\0\ 'n,~\\\~
11 consrAm of lnrcgrmioo1 . Por cxumplc. lnlhC io'l<l~ttlliOol frool\ du <! 1 c.lA \<l ~
we <\\u

11111 wricc

v = r'

= ' .

+ I ). 1hod we done ~o. 1hc soltollo WQlo\d \l&'e \)ti:>Cteile<l. 1\.S (\!\\"'~~.

j.r ~" tl.t = .del'' + D) - tu' + 0 ) ''~


= x (e'

+ V) -

e' - Dx

+C

= xe' - e" +C.


Constant D has therefore disappeared rrom Lhe eventual solution. This always occurs, and
consequentl y. we do not include an arbitrary constant in the mental intcgr.nion for

v.

I EXAMPLE 8.3
Evaluate the following integ,rals:
(a)

l nx dx

j .<cosxdx
1

(b)

(c)

e:t sin x tlx

SOl UT!OK
(a) Tmegration by pans leaves no choice for u and d o in this imegral; they mus1be
u = In x ,

du = dx .

in which case

du = -dx~

v=

.T.

Then we have

= x ln.r- { x !.. dx = xlnx- x +C.


.

(b) lf we set

du = cosx dx.
du

= 2x d.r .

u = sinx .

an<l

jx 2 cosxdx = x 2 sin x - j 2xsinxdx.


lntegmtion by l"'ns has therdore rctlucctl thc power on x from 2 to l . To eliminate
the .\" in front of the trigonometric function completely. we perform inlcgrution by
pans once again, this time wjth

u =

= sin x d.Y .

dv

"'~

Then

d u = dx ,

11

and

-cos~t }

jx
1

j -co>x d.<)

cosxdx = x2 sinx- 2 ( -xcosx-

= x 1 sinx +

2xcosx- 2sinx + C.

(c) lf we set
II

= c',

dv

= sinxdx .

then

d11 = e' dx,

p'

sinx dx

= -

= - ex cosx -

and

COS.l',

J- e'

cosx d.t.

Integration by part:; appears to ha\'C led nowhere~ the integration of e"

cos x i:s essentially a< uimcult a.< that of e' sin x . rr. however. we persevere an<l integrate by pans

once 3gain with

du = cosx dx,
I hen

v = sinx ,

and

= -' cosx + tr' sinx-

tr' sinx dx

e-' sinx dx .
By transposing the last

This equation can now be solved for the unknown integral.


term on Lhe right to the left, we find that

2 l e-' sinxdx = e-'(sinx- cos x),


and 11lerefore,

e-' sinx dx = - e (sinx - cosx)


2

+ C,

to which we have added the arbitnU)' constant C.

The 1echnique in pa1t (c) of this example is often called integration by rc1>roduction. Two
applications of integmtion by pat1s enabled us to reproduce the unknown integr.tl and then solve
the equation for it.

I EXAMPLE 8.4
Evaluate

x>

J9+x 2

dx.

Solution Methods
(i) lfwese.t u = 9+x2 , thendu. = Zxdx, and

r.:=x=
3 ::::;; dx =
J 9 + x2
=

x2

J 9 + x2

(xdx ) =

~ (~uJfl18u'' 2)
3

!-"fo

_- _9d
_u =

J9

+ x2

dx= l x 2
=

~(9 +

J9

+ x2

~j

<u'll- 9u- lfl) du

~(9
+ x 2 ) 312 3

+ C =

(ii) lfwese.t u = J9 + x z, then d u = (x!J9

9)9 + x 2 + C .

+ xz) dx. and

u>
)dx =J(u 2 -9)du = - - 9u
3

x 2):;z- 9/9 + x1 +C.

(iii) If we se1

dv =

J9 + x 2

dx,

then

du = 2xdx,
and

x /9 + x2 - 2

= x2/9 +

xz-

[i<9 + x

~(9 +
3

2 3 2
) ' ]

+ C

xz)J/ 2 +C.

+C

1l1is cxumpleHiustcatcs thm more than one tc..;,:hnique may be eJiccti\'e on an indefinite integral,

and 1he answer may appe>lr differenl dependi,lg on the lechnique used. According 10 Theorem
5. 1. lhcsc sol utions can only ditrcr by an alklilive constant. To illusmue lhis in Example 8.4,
note 1hnt the solution o btained by rnctl10d (iii) can be written us

J9 +x2 [x 2 - ~(9 + x2) ] + C

~J9 + x2 [-l8 +.r 2J + C

= -h+x 2 (-27 +(9+x 2))+ C


3

= -9J9 + x 2 + 3~ (9 + x 2) 312 +C.


which is 1he soiULion obutined by lhc olhcr two methods. h is also worth noting thatlhc 1hrce
methods work ' 'cry well provided lhat the power ou x in the uurncrmor of tlle in1egrand is odd.
\Vhcn it is e.ven. they fail miserably. ln such cases, trigonometric substitutions (Section 8.4}
come 10 1he rescue (see Example 8. 12).
hncgnnion-by-pans formula 8.2 applies to u1dcfinirc in1cgrals. For dcfinile in1cgrals. it is
replaced by

b rb
luv).1 v du .

1. ud~ =
I EXAMPLE 8 .5

(8.3)

~
.a

Evaluate

r.

r.:-;--o d x.
"' ....

SOLUTION H wesc1
I

d v = -..;; ::::= dx.


8

U =X.

then

v=2./x + S.

du = dx ,

f'
1o

./x + 8

dx = j2xJ.r +

s./12 -

s}o

2{

and

-1
4

2-J.r+8dx

~(x + 8) 1 J:
3 2

= ~(4J2- 3.J3).
3

EXERCISES 8.2

In Exercises 1-18 evaluate the inde(inile integral.

Jx ~inxdx

2.

J. J x'1 1n XIIX

4.

I.

j z scc' (z/3) d z
7. j Sin - x dx

5.

6.
8.

J
J

9.

x e lX dx

II.

./X ln (2x)d.r

j xJ3 - .lx

13.

15.

j x'Jx+5 dx
17.

10.

j ,f'f+X
x>
dx

12.

J <x- 1)21nxdx

e'

14.

j Tan -

cos (In x ) dx

16.

.j2 +x

dx

~ ~dx
2+x

J co..~; xdx
J
J+
x'

.../5

3x 2

(/X

2+ x

xdx

t' 2.' (':OS

3x d X

18. j tn(x 2 +4)dx

19. Cm\lcr lhc: C\'Uiu..llitlfl of A.5~ dx. Wcrc ..-c k u~ int-.,nlic:ln


by p;ub.th<:
"old cvontuolly be o(tlk: form

'""'"'<'

J.c

t'

d~-= J\.t't' ~ B..t"t' +C.t 1,.,. + Dxzt' +E.rt:' + f"~' +G.

l)iiTcrcmm.: thlscqu~toon , nnd thereby olltnlll A. 8 . C. D. r:.


(.ith no 1ntqn1tioo).
20. 11\C ract Ol 01 dum

IS

Shown ill tile

li ~UI'e be low~

d F

27. /(1) = ~

2ll. /(t) -

2?. /(1) =sin t

30.

1:

1Ct> =,_-

31. SOO.' lbalthe funClion

where the (UI\C

tQ;;;cquthm

for positi\'t>const~.mts a and fJ qul'llirlc-; u~u probabilily dcnsily func tion


on the intcrvul x ~ 0. Set F.xcrcil'>C 23 111 Scclion 7 ,I 0 for the defini1ion
of' a pdf and E1:crciC 26 fo.- the dclin1tion or r (o-).

k= l n 20l.
100

Find the tocol (<JR.'C. OJII th:: du1 We 10 w:!tcr prcs,.urc when the witter
lc,C"I on the dam u. lOO n1.

Ia the sudy o( signa/sand t'OfltlnunhmitJtr~ in dccuica.l cnghlee:ring_.


UIC Fo.uintran.o/<>mro(o hJJXhon/(t) ..

"bct'C ; it the: complex nun'lller whme sqlurc t) -1; thl.l ls. i1


-I .
In F.~en.:i!.es 32- 35 Hnd the Four~t>r tnmifomr uf d..:; fut'ld.._)n /(1) .
t

21. If we so:t 11 - x -' ond tlu = rlx. then dfl = -x-> tlx w>d

l. 111 < L/2


I (I) = o. 111> t.J2

J2.

v = .r,and

0.
l.l. f(t )=

Subtraction off .r dx from eh side ur this cqu~tion now 8)\f'


= J. WhJ.t is wmng with the afumen.t?
1

.u.

35. /(t)

n1f ,\

j (X)

CO> -d.~.

22. f (.r) ., '

24. /(X)

=I

23. f(x)

= ' '

+ 25. f(x) = 2A 2 - :1.4

!.\

26.. 1"ht gtlll fmt'1iol1 f"(n ) for n > 0 i,. <krined by chc i1nsnopcr

in ~Jl

r (nl =

J.

ThcLtlpltlU lrtmifomrora function j'( l) is lhe runclion F(J) defined


by the improper inlc&,ral

F (.r)

= 1"" ,-" f( l) dl .

={

O :;S I :;S T

a> Oaconstanr

I.

a<

0.

I>

36.

J8.

}":u~ si11.t dx

Tw,-

I <

./Xd.r

J~. lJsc lhc sub:s:lltulion x - ~in 2 0 .and imc:~mhon by I}Ut11i to find a

rormula (ot J.'.r.(l-.r)nd.r .

z- ~ dx

Show lh;C when n i< a (XJ"'Iive integer. r(n) = (n - 1)!.

ll t'OilSUtlll

Ia F.>.erciscs 36--.l& evalurue lhc indcllnk! anU!rol.

whcrr 1.. > 0 is 11fi~cd~'OO.Stant and n i~ a PQiitivc intct.~l' In fl>.crci e~


21-25 C\UIIt..lttlhc~c inlt:Jnlli ror th~s i~1 function/(.\') ,

1 - I(T.

1' > 0

0. t <:a

The ~mn lt'IK'J i~ atopicofm:sthc:rroti<:s fllltb1lC'nt.-ltoitll bnnclk:'


mc:rgi111l..-nnJ. W.;. 11n: cons....)) t..vdl'u:Jiing integral I ot the (onn
und

Il l> T

U+I( T . - T $ 1 $ 0

/(t)- e al.

I . > 0 a <.'011"Uml

<

r ,,

L- (11

2r)'

t'lJ

.-" co~xdx = ~m.x ""

+ cos :c

U-IJ ~I

L
, .n

~-h
(- I Yn!
XJI-b--t

(11 - 2r- I)!

+ c.

where Ln / 2J i ndicates lhe ftc)C)I' function or Example 2.22 in


SecLion 2.4.

' 8.3 Trigonometric Integrals


0.fr&:l'lliati011 (oml.UII~ (or trie--onomc:etic (UI'l( t iORS 6U8,gtH thn::c p;tlnAJi Of lhCSC fU.ocli()~:

si"'! and cosulC~ lln,cnl anJ ~.nt. and coung_ent and eo..~u The den' ' t: ufeither function
in a i''cn pair '"'"I\'Olll meN fu~ion... ofthal pair Note thai imqv~lkMl fomnllu8 le-n abo
subsuntiatc thas p;airina. ~a<"h indefinite integral gh-ec fur~on" '"' the un'IC: p.tiring. Because
o f I hie. a flf'lt 'tcp in C\'t'f)' ill l tgnd im.o lving lri~unonlClri c fu ncliono; i( 10 rewrite the integrand
so that each lcnn in,-.:d ve~ only one palr. Forexanlplc:., lo cvnlu;nc

IOil .t:

+sec
..t cot1 ,.,
J

sin x

d.r.

we would fi1-sc rewrite it in 1hc form

f(

c:,c .r

c_osx) cb .
Slfl$ .K

COot X

The hr~ term cont.tins CXKCC IIIlL~ a11d C()lt\f\..~,ts. anU lhc ~c.'tlnd h.~ ~i1\CC nnd eosnts. We now

considi!r in1tgrnlc invohing eac:h ,.ir.

Integrals Involving Sine and Cosine


We frequently ""''"'"" int<gmlsof tho fonn

cos" x sin"' x dx.

whcrem und n ntU) or muy n01 be ir11ege.rs. The kC) to evaluntion of this I)'J>eofinte&rfll i~recog
nition or LJw fuc:t dnu wh~:n cithc:r n = I or m
I. the intcgrutiutl bc."C,.'Vrttc tilrc,ightforward.
ro illu:sm-tte thib, cunbitlcr diiTcn.:ntiution of .siniJ+ 1x by I)Qwcr rul e 3.2 1,

~SinJ +I X=

d.\
'T11 i~ n1c11n~

( 11

+ l )o;i n" \'CO~\',

thtll difl"crcniarion o f 11 power of sin .r leads to a puwcr of

~ in x

one lower than the

original. nmlliplicd by oos.x. In on.!er to I"C\' CrM: the procedure and amiditTcrcntiale a power of
.'iin .t . \.\C need facwr of CO\ .t . h thtn becomes a mauer or Nisin&th< power on (in x by one
and divtdin_g by the: nc:w 1)0\\cr, thitt i:io, we c-an say
~n xco,xd:c

.
f

I
. ~o
= n+l
- - s1n "'

co," .oin xdx

1
= --cosHI X+ C.
II + I

~ C,

II

#<

-I.

!limil,trly.

II#< - I.

(!>.4h)

Forrttul:l~ R. ll4uuJ 8.1k COilHl.ill the II = -I cuses for lhesc integrals. or c.:ourse. a ~ubscimci nn
can alwtlY" be made to C\'tllumc imcgmls 8.4a and b. For exumple, if in 8.4a we set u = sin x .
1hcn du = co x dx, 1\Cid

J stn x cos u/.f =

:.in'H I .\

---~c.

ju" tl u

11+1

Also useful a.re 1hc formul.o<

Jsint~xdx

I
= - si n fiX

COS fiX

dx

I
- - COS fiX+

"

II

C.

and

(8.-k)

Trigonometric identities ar~ used to write imegrands in rhe forms conlai11ed in equ:Hions
8 .4 . Paniculurly helpful ore

sin 2 x+ cos 2 x

I EXAMPLE 8.6

(8.5a)

=I,

sin2x = 2sinxcosx,

t8.5b)

cos2x = 2cos2 x - I.

(8.5<:)

cos2x = 1 - 2 sin 2 x .

(8.5d l

Evaluate the fo llowing in1egm ls:

(d)

J
JJ

(g)

j cos x dx

(a)

sin 3 x dx
sin x cos 3.r dx

(b )

(e)

(h)

sin x coss _r dx

(c) /

CO>: Jx d.,stn 3x

(f)

sin2xcos6 2xdx

sio x dx

sin2 x cos2 x dx

SOLUTION
(a) With idemity 8.5a. we may write

sin 3 X d.r

sin .r( l - cos2 x) d x

(sin x - cos2 .r sin x) dx ,

ttnd the second tenn is in fom1 8.4b. Thus.

(b) Once again "'e use 8.5a 10 rewri1e the i ntegrand as two easil)' integrated 1em1s.

f sin3 xcos;xdx = f sillx( l -cos 2 x ) cos5 xdx

= j ccos;xsinx -cos7 xsinx)dx


I

= - - .:;os6 x
6

+ -cos8 x +C.
8

(c) This integrand is already in a convenient form for in1cgrotion. AdjuSLing conslanJS
leads to

sin 2x cos6 2.r dx =

--cos'2x +C.
14

(d) Using 8.5a. we obtain

JsinXC()I).J_f(/x

/ sin112 .rcos.r(J- Sin 2 .t)dX

= / (sin 112 xcosx - sin512 x cosx)dx

= - sinl/l x - -

s in 7/ 2 x

+C.

(c) Agai n we use 8.5a.

cos3 3.r
--- {.~'
1 =
sinz J.t

co::s:.:>:::~:...
: (:_:l~-,..-:':...i':...'...:3::.1~) dx =
2

sin' 3.r

-I

l (coS.Jx - cos3x)

dx

s111 3x

I .

3 si n 3.<

3son 3x +C.

In these five examples. at least one of the powers on the sine and cosine functions
was odd, and in each case we used identity S.Sa. 1n che next three exnmplcs. where
ntl powers arc even. idcntilies 8.51.>-d arc useful.
( f)

Double-angle fonnula 8.5d can be rearmnged as sin2 .< = (I lhererore

lin 2 xd.t =

'b:)/2, and

~~(l-cos2x)dx.

Fonnuln 8.4<.1 cnn now be u~cd on 1hc second 1cm1.

I
(g)

2
si11 x dx =

~ ( x - ~ si11 2x) + C.

Wilh double-angle formula 8.5c rewrinen in the f{)(Ol cos2 x = ( I + cos 2x)j2, we
ohtai1l

If

4
cos x tl.v = 4

2 If

( I +cos2x) dx = 4

( I + 2cos 2x

+ cos2r ) t/x .

When .t is replaced by 2r in S.Sc. 1hc rc.<ull i~ cos4x = 2 cos' 'b: - I. This can be
ocarranged as cos' 2t = ( I + cos 4x )/2. Md 1herefore

) dx .
j.cos x dx= 4lj'( 1 +2 cos 2x + l+cos4x
2
4

We now use 8.4d on the t wo cosine 1crms.

cos x dx =

I ( 3x
I
)
4 2 + sin2x + 8 si n 4x

+ C.

(h) In this imegral we use idcnti lics 8.5b and d.

I EXAMPLE 8.7
Evaluate

>in x cos x tlx

~ J( - ~ '') dx =

sin Sx cos 'b: dx .

(>in x cosx) 2 d.t

054

~ j sin2 2x cix

i(x- ~

sin 4,r ) +C.

S.J Tnguuometric lnl<:J.;r.tls

503

SOLUTION Unlike Example 8.6. in which alluigonomctric functions had tho~amc argument.
OtiS integrand is the proc.Juct of trigonometric functions with c.liffcrcnt arguments. Our first step
is 10 rewri te the imegmnd so tlwt we do not huvc such a product. Note thac a sum i~ obv iously
integrable. but not a product. The product fomtula
I

in A c:us 8

+ 8) +sin (A - B)J

2rsin (A

can be used to athrant.1ge here. Wf! c.an rewrite the intcgnmd in the fomt

l sin5.rcos2.td.<

=I~(sin 7.<

+ sin3x)dx .

and now integmtion is easily handled by 8.4c .

cos 2x dx = ~ (-~cos

sin Sx

7x -

cos 3x)

cos7x- - cos3x +C.


+ C = -14
61

1\vo-i ntcgration by parts (and reproduction) can also be used to evaluate this integral . tl\lt the
above method i muclt simpler.

..-..

Integrals Involving Tangent and Secant


For integrals irwol\'iug tan~4.:nt nnd $Ccam, we have tWQ nhom~lti''C$: Rewrite lh intcgr;tnd in
tcrnts of s ines and cosi nes. or express the integrand in terms of easily integrated combinacious
of tangent and secant. h i~ ~ua lly more fn1itrul to in\'C.Sligatc the second possibility before
expressing the integrand compl etely in terms or sines and cosines. l1l this regnrd we note that
combinations or tangent ru>d secant that can be intcgrnted inunedintcly arc

I
I

11

tan" x scc x clx

sec"xtanxdx

hm

1
""

II+
sed' x

+ C,

--+

1/

# - I;

i8.6o)

0.

(8 .6b)

II

1/

Once again these rcsuiiS arc sullgcsted by differentiations of powers of tanx and scc.r . Alter
natively. the~(: integrals can~ e\uluutcU with ~ubsti tutio n :s: u = tan .t ror 8.6a t~nd u =:sec x
for 8.6b.
To recmnge integnmds into these integnble combinations, we use the identi ty
(8.7 )

I EXAMPLE 8 .8
Evaluate the following integrals:
(a)

(d)

I
I

tan2 x dx
4

tan

(b)
2

3x sec 3x dx

(e)

I
I

tan 3 x dx
3

(c)
3

1an x sec x dx

(f)

I
l

sec x dx

.sec4 x
- 2- dx
Lan

504

Ch:apcc:r 8

Te<hniqucs of l ntcgr~1 ion

SOLU'n ON

(a)

Jtm2 x d x =

(b)

Jtan3 .r dx = /1a1u- (sec 2,v - l) d x


=

(sec2 x - l )dx

tan.r - x

+C

(se<J x tan x - t.an.r) dx

= - <ec2 x + In l cosx l + C, or - tan 2x + lnl<os.t l + C

(c)

sec" .r d x =

sec2 x ( I

(sec1 x

+ tan2 x soo2 .r) d.r

+ -I h lll J x + C

= tan x

I
- tan5 3x
15

Juut~x scc3 x dx

(e)

+ wnl x) dx

+C

= J mnx(scc1 x - l)scc3 xdx

= / (sec5 xtan.< -sec3.r tnnx) d,<


I
I
5
= - sec.\'
- -:sec 1 x
5
3

+C

sec' x

(f)

- 2- d.<
Lan

sec2 x

-,-

/ ( hi ll ~ X

+ sec-, x )

d.r

= - - - +an x + C
tanx

I EXAMPLE 8.9

---

Evulualc

scc 3 x

d.~: .

SOLUTION We usc imcgmtion by parts IIIlO reproduction. By selling

u =

sec ~t.

then

du
and

= secxtanx dx ,

u = tan x,

fonnula 8.2 gives

Jsec:'

x dx = secx 1anx -

Jsecx 1an2

x dx .

Trigot~unlC(Iir

8.3

lnltg.rah

505

Since tan2 x = scc1 x - l. we can wri te

sec3 x dx

= sccx tanx

= sccx tanx-

J
J

sccx (scc2 r - I) dx
secl x dx

+ ln l secx + tanxl.

\Ve now solve for

I
2(>CC.tlanx

+ 1n i :$Ccx + tanxl) + C.

lntcgnmds in the following example c;uutot be mmsfomtecl by means of identity &.7 imo forms

8.6. and we therefore express them in tenus or si ne~ anc.l C.:(J:,inc:;.

I EXAMPLE 8 . 10
F.\'lllllilte the following ime&mls:

(b)

dx

sec x U\nl x

SOLUTION
<a)

1
tan 2x dx
;;ecJ 2.t

(b)

=
:;;;;;

sin2 2x cos2.v dx = -I sin 3 2.v


6

+C

cos' x d . _

'"'

sin" x

.'( -

cos x (I - sin x ) d
l

sin ""

j( oo! x - cosx) dx = - - - -sinx + C


I

sin .t

SIW .X

Integrals Involving Cotangent and Cosecant


Since c.Jcriviltives Qf the c.v langenl a1n.J cosecant p;ir
-

C()( X

dx

=: -

o;c2 .f ,

d
- csc..r

dx

- cscxcotx

are analogous to those of the tangent and secant pair


d

dx

tan.x :;;;;; sec-? o>,. .

tl

tl.x

secx = secxtanx.

a d iscussioo parallel to that coocerniog the tangent and secant could be made here. Aside from

the ract that cotangent replaces tangent and cosecant replaces secant, there are also sign changes.

Examples are given i n the exercises.

506

Chap;tr 8 Technique(,; of l11tegrJtion

EXERCISES 8 .3

t/J2 arc constunts, is P = Vi. Show that the :.m::ragc powc:t supplied
10 lhc n:s;stor ;s P = (Vmi.,f2)cos (,PI - .Pz) .

In ExC'rcises 1- 20 C\'aluatc the indefinite integral.

I.

3.
5.

7.
9.
II.

13.

15.
17.

cos3 :r ~in .\' d.r

J ~c
J +
J
J
J
J ,;.o
J
J

1 _,. clx

tan 5 x

co.' (x

4.

Jc~1 xc.:.ot.\' tlx

6.

2) dx

-I4- dt

8.

cos2 xdx

10.

sin 1

sin; y cos' y d y

dO

14.

l + cosO

cosO sin 20 dO

16.

s-inl .\' coss .r dx

IS.

/t::m-xdx

* 33.

cosx
- dx
sinJ .\'

JJ~;m:t:
J
r x
r----;e
r
.

Unns)1

esc' f)

dx

t/0

001

s.cc2 X dr
J l +tan x '

~ J.rt' dt.

n ,c de vaJuc ot' a pcriot.lic wavcfonn f(t) with period pin the lbcOt)
or signal$ is defined as
FJ,

j 3+4csc'x
~
dx
em .r

WI,

whccc the intc.rval 0 :5 t :5 T reptcscnlS any numt'Cl' of complete.:


Q.\,Cillmi..-.lSOftJlecurrenc. Showthat/nn~ is 1/ h ti mesthe~rnplitudc
of the current.

sin

line is: giv<:n by

= j(t ) = A CMM + 8 in

S4:.c1, 3.r can3x dx


t:lnJ X &cc X
2

pow~r

where A, B. and w :II'C con~l :'ln l~. and 1 i~ lime. Tht root-mea n-~u.arc
(rms) current I rrm is de tined by

I!""

=-

j(t)dt .

-p/l

In Excrci$C3 34-37 find the de v<1luc lOr the wuvcform.

1
$in .r dx

J~.

* JS.

20. ( esc..3 X dx

Sl'C" X

T he ahem:uin,g, current in :l

scc'' .r dx

12

19.

2.

f(t) of Ex<:rc;se 33
f(t)

= A .,. 8 cosw. A. 8 . w constant<

36. /(1) = sin1 WI . o.> constllnt

COI .t'

37. /(1) = I , 0 < I < I , f(t + I) - /(1)

2 t. Find the area of the regi<M'l heawecn the x -u.xi~ and

y = si n x

for

0!:X!:11' .

22. Fmd the \'Olumc o f the ool.jd of ~votution obtained by rot~ling the
region bounded by y = tanx . ." = 0. and x = rr / 4 aOOut thc line
.\'=-I.

23. Evaluate

J.:r JI -

sin 2

.. 26.

* 28.
* 30.
* 32.

cot.$zdt

J
J
J

cos_o

dO

I + sm8

cos6x coo 2x dx
I

sinx cos? x

dx

~..-

xdx.

25.

l 7.

J +,;no +
(I

J
**

J
J

cos3 0

~.::..,.:..,d(}

J+su10
4

sin

....;

c;o:s

* 29.

3 1.

J~c;xdx

x d.t:

2
t'(X5 2..J: sin Jx (l .t

The power rcqujn::d to majntain ctuTcnt i ( f) = i111 cos (wl + </> 1).
where i IW > 0 and </J1 arc c.:onstanls. through a resistor that has voltage
ac;rossits tcrminaJs V (t ) = Vmcos(wf +c/J:). whcrc ~" > Oaod

s;n' 6 + s;n3 9 + .. ) d/J.

39. If n is a ~rive even Lntqzocr. show thai

S<-'1!' .r d.r

In Excn;ises l -1-31 evaluate the Lndcliuitc intcgnll.


. 24.

38. Evaluate

"

"' '

= L-

l ( n/ 2 -

, _

I ) tan 2' +l X
'_r + I

+ c.

40. A SCI ofrunclions [ 1(:r), / 2 (X) . .. is s:.~ id to be ont.-,normal over


an i.ntcrval a !: x !: b if

1:

f, ,(x)j",.(x) d.r =

o.

I.

m = "

-1- n

ShQw that the set of funclion::;

r- sin x .

""

= coo.r.

,n r

.;;; cos 2.r.

J;i Sln3.\' 1

is 011hono rmal over the interval 0 ~ x ~ 21l'. Orth ononnaJity of


these [unctions is the basis for Fourie,. series, a topic; o f fundamental
important'C in aU branches of e ng ineering.

18.4 Trigonometric Substitutions


Physical and geometric prQblems arisin from circles. ell ipses, and hyperbolas often give rise lO

imegmls involving the square roots t.l~ - I>~A' , Jni + Jiixi. Hnd .Jbi.\"1- a l . Trigonometric s ubstitutions replac.-e them w ith integrals invol ving uigonometric functions.

Consider firs t two intcg.Til l$ that involve s.quare I'OOlS of the form J al - b l .rl :

f ~.;"F.j =_,.2~

(i)

=I

,
J

(ii)

and

dx

r:=~'T2 dx.
../4 9x

lntcgml (i) is evident from diffcrcmiJniOil of the inverse trigonomelric func tiollS in Seclion 3.10,

(See also for mula S.lo) ' Ve now show that a substitution can be made o n this integral that is

Ja

2 - b 2 x 2 . i ncludi ng
applicublc to much more difficult problems invo lving the square root
(ii ). lf instead of the squMe root J'f'="'X! in ( i ), we had the squurc rl)()t J 1 sin18 . we

could immet.li~lLely !\il'nplify the lllllf rno11o Joo..:;;.2 ()


I c;ns 81. 11til' prompts us to m.jj\:e the
substilution x =sin II, from which we h<l\e dx =cosO d O. lntegnll (i) then becomes

f JJ-

j -1cos IJI

.fl

dx =

cos9 d 9.

To eliminate the a bsolute. va lues. we need to know whether cos is positive or negative. But
this means lhtlt we must knov. the possible values of() . There is a problem. The equation
x si110 does not really derlne 6; for any give11 x, we do 1tot howe a unique 9 , but an inft1tite
number of possibilities. We must therefore restrict the values of II , an<.l we do this by specifyi ng

that /) = Sin- 1 x. In other words. 11lthough we have used the cqumion x = sin 9 to change
vari11bles. am.lwill continue to<.lo so. it is reall y thecquaJion 9 = Sin- 1 x that properly <.leli nes
the chm1ge. Since IJ so defined n1ustlic in the imerval - rr /2 :S 9 :S rr /2 ( Ihe principal values
of the inverse sine function). it follows that cos/J ::: 0. und absolute values may be dropped.
Consequently,

f ../1-x

r.:='==:;;d.x
2

0 05 9
- - - do = j tto = 9 + C = Sin= j-lcos8
1

x +C.

For imcgral(ii), thc substillnion.r


sin Q is of lillie usc sitlCC 1hc squ11re roo1 J 4 - 9 si111 0
does not simplify. lf, however, the square root were.../4 - 4 sin1 6 , it w oukl immediate!)' reduce
10 ./4 - 4 sin 1 8 - ./ 4cos20
21cos91. We em\ obtai n this result if the subsliltllion is mod
ificd to x = (2/ 3) sinO (or, rnore properly, 9 = Sin- (.3x /2) (. Since dx = (2/ 3) cosO d 9,
we have
l
(2/3) cosiJ
cos9
r.=;p;;dx =
dO = diJ .
../4- 9xl
J~ - 4si n'e
3 2lcos01

21

Once <tg<tin absolute values may be d ro pped because 1


-;== =.=;;dx =

)4 - 9.t1

~
3

ld9

;r /2

~3 + c

:S 9 5

;r /2,

3
~s;
.. - '( " ) + c.
3
2

For imegrals containing square roots of the form .j,p . + b2 x 2 , comi.ider the pair of integral'
(iii)

- - dx
I

+ .x 2

Integral (iii) obviously has answer Tan - t x


I

(iv)

+C.

1--+

(3

.....,-....,.,- dx

5.x 2) 3/ 2

Altcm:nively, if we had I + Jan2 (J instead of

+ x 2 the two terms in the denominator would simplify to the one term sec2 8 . We therefore

substitute x = tan 8 or. more properly. 0 = Tan - I x , from which we have t/.:J.:
With this sub5titution. we vlm1i11

I-

+ _,-2

l -scc'
O
sec'

d,t =

dO =

d ()

. _,

= () + C = lan

= scc2 8 d8 .

,v + C.

2
The substiuu io11 .v
tan 0 in (iv) yield< (3 + 5x 2) 312 = (3 + 5 1an Bill. l'or simpl i fi2
eation. WC need n 3 ill (r()tl ( Of the tan 8 :und ll()t il 5. 'nliS C.Oil be BCCOillplishcd by modificntion
of our substi1ution to

fl'o m which

Then

=
i:Jic:lil;i
di
x = J'fl5tt V

TtiJI'Igle ft'l fi1

Ir.

3v5

I
-()dO
=

I,.
3v 5

sec

cosfJdO

I G sin II +

3-v 5

C.

To express sin () in 1enns of x ~ we t.lrnw lhc triangle in Figure 8.1 10 fit the change of \1atiablc
./Sx;./3 = tan /J . Since thehy)>lllenuse ofthetriangle is -. ./3 + 5x'. we obtain

I .,. ,.-+___,):. . ,. .,-,:


(3

5.r 2)'12

dX =

./S X

3.J5 ..j3 + 5.<2

For i ntegrals conlaini l squnre rooiS of the fonn

+ C = --;:-i';"-17' =::=i + C ,
3-/3 + 5x'
Jb'x2 -

<l 2 consider

I -;: :;:

== dx.
-/x 2 - 4

If we had J4sec2 8 - 4 in.">le.ad of ~, an immediate simpliflcatiOl would occur. \Ve


therefore subs1i1u1~ x
2sec 8 , f rom which dx = 2sec8tan(Jd(J . O nce again 1hc roal
substitutiol is 8 = Sec - I (.r / 2). in which case 8 isnn angle in cilherthe lirst or third quadrttHI.
\.Vith lhis change,

and

I
lli!I!] "ID 1'ti"ngk:: tu fil
x = 2scc9

-;=.==== dx

Jx1 -

=1

2secO t<m fJ
2tan6

dO

secO dfJ

= In I secfJ + 1an OI +C.

To CXPf= ian II i n lemlS or X we <lr,\w lhc triangle in Figure 8.2 I() lilthc. c hange of variable
0

x /2

= secfJ.

where D

Si nce the third siclc is

--;:=:i=:::::; dx
J xl -

.Jx1 -

4, we h~ve

x
.Jx2- 4
= In - + - ' - - 2

+C

= In

lx+ )x 41 +D.
2

= C- l n 2.

We have illustrated by examples Lhat Lr igo nomelr ic substitutions c.an be useful in the eval
2

uation of imegrals involving the square rooiS

J a' - b'xl . .ja' + blxl,

and

Jb2x2 - a' .

8A TriJ,.'(IOC)metrk Subs1ittui1.1ns

509

Essentially, the metbod replaces tem' s under the square root by a perfect square and thereby rids
tbe iotegrand of the square coot. To obtain the trigonometric substitution appropriate to each
square roo1, we suggest the following procedure. \Ve have been working with the trigonomcrric
identities

I - sin 2
1

e = = 2 II,

+ tan 2 8 =

sec 1 8

scc2 9,
ran

8.

To determine 1he uigCNlomerric substinuion approprione w>say~ J t1 2 - b2 x 2 , we mentally set


a h = I andobL.ain ~. \ Vc:. nq l t: that l -x 2 resembles l-sin 2 8. and immc:diatdy set
x = sinD. To simplifY the original square root J a 2 b1x 2 we then modify this substilUtion
to

Si milarly. for the square root .Ja1 + b1 x 2 we initially set x = tan9 . having mentally set
a = b = J, and not<! that I + x 2 resembles l + tan2 (J. We then modify the substitution to

b tan(J

Finally. for the square root .Jblx2- al. we initially set x

= secfl and then mo<lify to

Thcsqunrcroots Ja 2 - b' x', Ja 2 + b 2 x 2 ,and ./b1x1 - a.2 arc replaced, respectively,


by

Jal + a 2 tan' 9 .Ja2scc' 9- a 2

Ja1- a.2 sin2 11

.Jll1 sec2 e

.Ja 1 tan2 (!

= .Ja1 cos2 fi

= a l cos Ill

= al sec Ill

= lll tan Il l

= a oos9 ;

= ascc8;

= atan ll.

In each case it is our choke of pdocjpal vaJues for the iover:se trigonometric fun.ctioos that
enables us 10 neglect absolule vt1lues .

We now consider more complex examples.

I EXAMPLE

8.,,
Evaluate the following imegrals:

(a)

x 2 (9x 2

+ 2)

dx

510

ChapterS Techniques orhue:.raiion

SOLUTION
(a)

rsnrn ,1'!'0

If we SCI X = (.J2/3) lan 9. Ihen dx = (.J2/3) ~ec2 8 dO, and

Trit n&1e IO Iii

.< = (./'!/ 3)1:m 1J

3
= ,fi
2

0011 8 d(J =

3
,fi

J .

(esc- 8 - I) d8

= - (-cocO- 8)- C
2,fi

3.<

= -;, [.J2 +
2v2

()

= --I 2x

3.<

/3 - x 2dx

= /3cQsOdO,and

(3sin2 8)/3- 3siri 8(J3cos8d8)

= 9

v2

J
= J
J
=

.r

( Figure 8.3)

3~Tan- '( 3xt:::) + C.

2v 2

(b) lfwcsel.t = /3sin0 , 1bendx

/x

3
r;,n-( v2
~)] + C

sin2 11 J I - si n1 0 co. 8 d8
sin2 8eos2 9d8

~ j sin1 20 dO

~ (e- ~ <in48) + c

(by 8.5b)

J(' -~os<W) d~

(b)' 8.5<1)
(by 8.4d)

~Sin-(~) - ~(2sin28cos29) + C

.vf:i

(by8.5b)

32

2s;n-(.!_)
- ~(2sin0cos0)(1- 2sin 11) +
.,/3
1

16

=~Sin-(~) - ~(~) JJ;/ ( 1 - ~ ) + C

(b)' 8.5b,d)

(Figure 8.4)

$..f Tri&oSHJmc:l~ SubSdnuions

I EXAMP L E

511

8.12

Use a trigonometric substitution to eval uate t he integn\1 in E xa mple 8.4.

SOLlJfJON Jfwesetx =3tan6,thendx = 3sec2 1id6,and

.r

--;:;;=:===;; dx =
./9 + xi

f .Jq+

21 tan 8
,
3 sec 8 d(l
9 llln2 9

= 27
= 27
= 27
= 27

tan 3 sec2 ()
,.,
dO

f
f
f e eG 8- sec8) +
SCCQ

tun 8 (scc 2 8 - I) scc 8 d 8

(scc 3 tan

()

scc8 tan 8) d8

sec)

../9 + xl)

(by 8.7)

-27

(by 8.6b)

../9 + x1 + c
3

(Figure 8.5)

~(9 + x2)l/ 1 - 9h + x 2 + C.

I EXAMPLE 8 . 13
= .((x) lo llowed by the
puim miUway bctwc..-cn Lhe rear w heel:; of u uuilcr given thut Lhc pivvt poi nt uf the traclor and

The ;\pplicati<Jn J>review J>osed the p<lJblem of li nding the pltlh ) '

Ap plication P rc,iew
Revisited

trailer follows the c urve

Y = g (X) (l'igure 8.6).

Pivot

.r
SOLUT ION T he key is to notice that the
)' = f (x). Consequently.

dy
dx

li ~>cjoining

Y- y
X- X

= ---=

(x. y ) to (X. Y) is always tangent to

g(X) - y

X- x

This is a d ifferential equation that must be solved for y = f (x) subject to the initial position of
the midpoint between the rear wheels on the X ax is. Tb solve this equation we must e liminate
X. T his c an be done by leuing L be the tixed distance be-t ween (x , y ) and ( X , Y) , in which

case

Given ~(X} . we cun theoretically solve this equation for

X in 1C111lS of x, y, :uld L, mld


substinue imo the diRerential cqumion to express dyjdx in tcnns of x and y. To do so for
given g(X ) could be a fom1idable task if g(X ) is at all complicated. We shall consider only
the simplest possible situation in which the pivQt poim begins at the origin and moves up the
y-<1xis. l n !his case, X is always zero. and L 2
x' + ( Y - y) 2 . From this equation.
Y - >' = J L' - x 2, and the differcmial equat ion for y = /(.<)is

dy

Y- y

dx

X - x

-=

Integration with respect to x gives

y = Wc nowsct x

y= -

j L' - -~'

d.~ .

= L sin 8 andd.r = L cos Od() .

L cos O
L . () L cos9 d9 = - L

Sill

= L1- cos 9 - In I esc 9 -

= -L ( JL' L

e<>t 011

I -sin
. e 8 dfJ = L

+C

x')+c

L - .J.2- .rl

+C

-~

= - ./LZ = L ln

(sin 8- c.' c9) d8

t
.JU_ _;__I
+ In --_;_
X
x

= - ./.2 - x2 - Lin

= -./ L2 - x' -

$ 111

L -

+ ..Jr;L"'1,-~x:-ri
L +.JU

I+ C

J Ll

- xl L

LIn

x' - L In

L+.JL 2 -x'

+ )L' - .r'

J [.2- x' + C.

Since y = 0 when x = L. it follows that C = 0 . and

y = L In (

EXERCISES 8.4

In CxcrciliCs 1-20 cvnluutc the indefinite mtcgral.

I.

3.

5.

7.

,,.J2.~'

d.r

~ x' dx

10

)1 - x2 tix
3

2.

Jx ./4+x'dx

4.

f J9
f J4'=""il
I

x'

6. J

8.

5.rl

9.

dx

j-

1- X

f ~ <IX
f xJx' +

j L' - x2.

12.

J J4 -x' dx
X

dx

;:,)~!' dx

14.

dx

16.

13.

(2 -

IS.

x2J
. 2.r)

+7

x+S tl x

l Ox 2 + 2

dx

10. (

x - S

II.

x./sx' + 3dx
1
- 2 dx

_,L_+;,;..f_.~:_'_.;,x_' )

J.Jx 1 - 16
dx
x-"

x 3Jx2 - 4

dx

17.

19.

!7

d;

j (4x' ~

2
x +:r

20.

9)3/ z dx

a - x

l li.t:

* 33.

f --

j Jx 2 - 4dx

= ..!.. In a
2a

+xl+ C.

a - .\'

= a 1b 1 is iTa b.

23. Find the area of the region common to the c-iR::Ics x 2 + y2. = 4

x 2 + y 2 = 4 X.

"' 24. Gi_,cn tbal lhc equation 2x 2 + )'z - 2ty


descrihes an elli1:x-:e. liod il:ot area.

+ 4x -

35.

37.

22. Verify th atthe area inside the ellipse b 1 x 2 + a1 y 2


:mel

-3-- d .l

21. Show t.hat if a :> 0 . tllcn

J
*
*

Jv'Y'+
~dl'
4 .
x' +
J

IM.

4)'

+3 =

L.

(b) Find

x'

34.

j JI+ 3x1 dx

)'"''I~, dx

-x' "'(74x-;,,-'___,l

.\

J./x' -

.16.

-,=;;
x=
' = dx
5

= x :: ( l - x

).

39. Fi.nd lhc lcnglh of the par".Li>ola y = x' froru (0, 0) lo (I , 1.) .
* 40. At wh::tt distance from the.centre of a ci.rclc sh ould a line be drown
in order l h~l t lhe second rnomcnl or ::src:~ of the circle :dJOUl lh'd line will
be c.qual to Lwit:c tl1c $(,:Ond n.10 1nc nt of the c i n;lc ilbout i2 line tlli'Qu: h
i1S ccnlre'!
+ 41. When wttte r in the soi l 1110ve.s lO the surft~ce of the soil, depth z
beiO\\i lhe surface i.s related to suclion !lead x by 1he equaLion

(a) lf tllc boot st.arlS at Q and the string ~J wa)'S rc.nlaills st.raig.hL
shO\"' th::tt the cqu:ttion of the curved path ,V = f(x ) fo1
lowed b)' the boat must s.&uisfy the difTen:ntiaJ equation

,/U

- ., dx

~'!K dx,
V :> 0 is aoon$:tant, ::.nd K = kf(cx
z(x) = -

(the y-axis) towing his sailboat by a su-ing of Length

X -

38. Find lhe lt"nglhof Lhetur'\'e 8)'~

25 . A boy initiaUy m 0 (hgurc below) walks along the edge of a pier

dy
dx

whe re

1+

4md c > 0 being. COit:;tants. Show that if .t(O)

;;;;o

z(x ) = H - ,/Vc(k + V) Tan-

+I) w ith k ::> 0

H, then

(li"C'
Vk+V x ,

wbcrcw; i.f t (H,. - L) - L.thco

z(x ) = L

y = f (x ) .

k
[ Tan- '
./Vc(k+V)

- Tan -

~c
- - (Hw k+V

L)

,rvc]
+v

f~
--- x .

~ k

When .:a tJcxiblc cable o f consLa.nt T03SS pc( un_it lc ngtb p h:mgs
bcLwccn two fi>c~ point> A an~ B ( figure below). ~1cshupe .1" = f(x)
of the coble mUSl satisfy the diJferential equati011

'+ 42.

('h )''

pg

I+ dx
-'-

+ 26. The parabola x ; y '!. di\idcs the circJc x l

+ y 2 ;; 4 inll> two

pan~. Fi nd the second moment or area of the smaller JXJ11

where 8 > 0 and H > 0 arc conSiants. lf we set k = pg/ H and


p = d.r/dx. then

a1Jou1 1he

or

.t axis.

/I + p' d x

= k.

27. Find the centroid of lhe region in Exercise 26.

r .. 2S.

Find the hoti20ll1nl line th:n divides: the ellipse b Zx ?. + a 2 .\ ' 2


a i b 'l into two parts so Lhllllhc area of the lowtt part is [wicc the areu
or Lhe uppe.r pan .

dp

h follows that

J/ I+

* 30.

Example 3.39 in Scctioll 3.13.)

A cylindrical o il can with horizomal axis has rad ius r ancl length
Jt . If the densily o r Lht! o.il is p, lind lhe forre on each end of 1he can

when i1 is full.

In Exercises 3 1- 37 evaltlalc the indefinite intcgml.

* 3J.

32.

j (7 - x')"'

Lowest point
dx

= kx +C.

P'

(a) Evaluiatc the int.cgra.l sbownto h nd p


d }fd x .
(b) lntcg.tate.once more to find the shape of the <>able (see also

29. lr a lhi n <:in.::ular plate has rmlius 2 onils and GOns1ant nU~ss pc;r uni1
area p . find i u~ OlOOlCIH of ioenia aboul any 1a1lgent li1le to its edge.

2x 4 -x 2
,
(/X
2x +I

dp

on cable
X

$14

C~S

Te<:hni q ut:sof l ntc~atiun

il ,t t 43. Repeat Exercise 21 of Set..1 ion 7.6 if the log il' circ.ular

with mdius

IOcm.
~ 4-J. Find the area in:,ide the Joop or the strophoid y 2 (a
x 2(t - x).

,.. 45.

+ .r)

(b) Use l"'rl (a) 10 prove lhal

I.

x +I +JX'+l

(a) By rotionu.lizing the dcnomlnutor, show thm

.-.-+- 1-+-..j',=.,=+- 1 =

1
I - - ---:=== d.f

**

x+l-JX!+i

46. Sho" ''"''

= I-

_
,2fi + 2~ In ( I+2,fi)

l ' ~+x

- - dx =rr.

2x

I - X

- 1

18.5 Completing the Squa re and Trigonometric Substitutions


Trigonometr ic substitutions reduce integrals containing square roob of the forn-1 /a2 - b'lx'l .

J n 2 + b 2x 2 , and Jb2 x 2 -

ol to trigonometric i meg.rals. For imegralscontainingsquare roots

of 1hc form -/axi+ bx + c we can a~ain reduce 1he inregral to a 1rigonomctric imegrol by a
rrigonome1ric substirurion. if we firsr comple1e lhe square.
Consider 1hc integral

+
I .,. .-;:+- - -,.'----:,-;,~/'

d ' .

5}' -

2x

(.r2

rf we complete the square i n the denominator~ we obtain

1----:,+---- +
1

---,,..,. d.t

2x

(.r2

5)3/2

=I

+ 1) ~ + 4)312 dx .

[(.r

Had the denominator been {x 2 + 4) 31'1 . we wou ld have se1 x


the denominator [(x + I)' + 4 )Jf! to sel

X+ I =

= 2 ran 8 . II is narural. then, for

21ml9,

in which case dx = 2 sec' dO. and


Triangle to fit

x +l

x+l

= ----;=.===='
= ==
4 /.r 2 + 2 r + 5

+ c.

(Figure &.7 )

Consider another exumpl e.

Complcrion of the square

le~ds

I / 4+

10

x - x 1 dx =

J- (x - 1/ 2)2 + 17f 4dx.

l f wenow seL

.J0
x - - = - - sin8,
I

then

JT7
= --cos9d8.
2

dx
(IJicJ

Ji72c w9 -cos8
.ffi
- - coo 0d9= f d0
f / 4 +x - x dx= f ,/- -174 sin 0 +17Ji7
4 2
2
2

= ~ fcos2 8 dll = ~

2 - I

-,.

TriJ111Ic 10 t11

f C+ C:

29
) tJ(J

Ji'isin O

I 28) + C = -(8
17 +sin OcosO)+ C
= -17 ( 8 + -sin
8

17 - (2x -l ) + -J7[2x - 1] 2/4 + x-x2 + C


= -Sin

2.' - I

..m

= .!Zsin-(
~
8
11

(J

..Jo1 _ (lr - ql

..m

~(2x -

1
) +

..m

(Figure 8.8)

1)/4 + x - x 2 + C.

r\ s a fi nal illustralivc example, we consid~r

- 2 ..J4 + x - .t2

. J2x> + 3x

6 dx =

If ~

./2

If

3x - 3 dx =
.r + 2
.
l

./2

/(
\

x
3) 2

x +4

dx .

57
16

Irwc seo

J5"i
.{ +-34 = -sec8
4
'
then

/57

dx = - - =8 wn 9 d9
4

and

.J2.r 2

+ 3.r- 6

dx = _I_

../2

f(

../fi/4) sec IJ - 3/4 .Jfi stc8tan 8 dO


j(57/ 16)sec2 8 - 57/1 6 4

= _1 f (../5'i/4)scc8- 3/4 .f5'i St:<: 8 tan (J t!O


./2
(.J5'i/ 4) '"" 0
~
= -

../2

[E
. + l

Triangle 10 fit

= ../S'i e6

zj
+3
(J

ill

,
~(4x+ 3) - 57

= 2 .J2~2.r 2 +3x-6

2 9 -~scc8) t/8
f( .Jfiscc
4
~

.Jfi

"'tan

~v2

8- 4v23"' In sec II + tan Ill + C

= .Jfi [2../2.J2x2 + Jx

6]

.J5'i

4../2
3

4x

+3

- - ../2
- In 1 -../fi
--,=~
4 2

57

= / 2x' + 3~ - 6 2

2./'i/ 2x 2

+ 3x

57

--../fi
-,=~--

3 In 4x + 3 +
./'i

+C

(Figure8.9)

2.J2J2x2 +

3x- 6 + D ,

516

Chapcer8 Ta:hni-ques of lmep;nion


where

D = C

3
,.In 57.
8v 2

We have shown thm completing the square i s the nawral generali zation of the technique
of trigonometric substitutions. It reduces integrals comaining square rOOIS of Lhe fomt
,Jtu2 bx c to trigonometric i ntegrals.

I EXAMPLE 8.14

Find the volu me of a donu1.

SOU.TfTON

A donut is generated (as the vo l ume of Lhe solid of revolution} when the circle

(x - a/ + / = b2
.l:tlijii;I:W:lllil,

(a > b)

i s rotated around they-axis (Figure S. t O). Cyl i ndrical shells for vert ical rectangle< yiel d the
volume genermed by the upper semicircle, which can be doubled to gi,e

[}.)fml

volume b)' Shells

a +b

V=2

2:;rxydx .

a- b

If we sol ve the equat ion of the circle for

We now set x - a

= b sin 0. iJl

y=

Jb' -

which case dx

(x - a) 2 . then

= b cos 0 dl). and

.</2

V = 4;;r

(a + bsin fJ)bcos8bcos8d8

-;r/2

1 (t+
2

= 4nb

IOjiciii.J!j .
donut by washers

- .T/ l

cos20 )
2

+ b cos2 8 si n8] dl)

With horizontal rectangles (Figure 8. 11), the washer method yields

Volwnc uf

x.

We can solve the equati on of the circle for x

- x, -J ~

Jy
a

Thus

= a Jb 2 -

y2 ; hence.

ld thi..) simplihc11 W

v=
rfwc ~t } =

b sin9, 1hCII fi)'

2;or

f.b4tt/b' - >'ly.

= bcos8 d8. and

= 4rwb1

{e + 2I }11
sin 2/J 0

4trnb2 (Jr/2) = 2tr 1ab:.

EXE R C I S E S 8.5

11. Ql\e of lhc galt'S in 111 d1n' l5 ctn:u!ar -.ith r.t.diw 1 m. I( the rate

In Fte~iJa 1-10 evaJW!C the ndclinhc inqnl


I.

3.

s.

.r

J21 + 6.r - .,.z

<.... +

~) ,,,, dy

Jx 1

12.<
.r+l

'1, -~' +
9.

dx

' I J.r
I
f
s. I '
.

4.

d.<

+2.< +2
I

lx - x 2 -

(4x - ,Tzpt:

5-4.<

Jllz- 4x2- 8

dx

dx
(u) hy compla i na 111< "'I'"'N nnd .._;ling .< - 3/2 = (3/2) ' "' 0 nd
{h) by multiplying nt~rucrau~ und d~IMJill lllUi or by - I . comp1cuns 1hc.
.quare. and scuins x - 3/l = (l/2) ""'Q. (<) Expl.,n th< <lifT=ncc

(/.f

2.r-J

6ydy

11 dosed arx:ltbc stulaa: o( lhc va:a.k:r i' 3m al:lo\1: the: top of the: ~~r~.
find 11\e fon:e due to WUII:r pn.:5t-t.fC Oft the @RIC

x -6x- 13

111 the rwo atSwerS.

tlx

J:r

14.

j v x'- 2x- 3d<

16.

15.

(2x+5)/2.r-J+8x - 12

J J'lx
.x

x'

d\

d.\

l 8.6 Partial Fractions


l'klrtiaJ fi'ctiofls is 8 method lhlt W< apply tO rational fUn<.'tiOOS, integrals of the: form
N (x)

-D(.\) "'

(!>.'\)

where N (x} and O (x) me polynomials in x, and the degree of N (x ) is less than I he degree
of D (x}. When the degree of N (x) is greaten han or eq11al1o lhat of D (x }. we di vide D(x)
inlo N(x). For example. in the integral

+ 4x 3 + 2x + 4 dx.

::...__~~.:.....=.:....:._

x' +I

518

Cbopltr S

1~.:htuque-; of lnqtali().l

1he 11Ume-rmor has degree4 and the denominator degree 3. By long division. we obtain

+ 4x3 + 2x + 4 d .\.
x1 + I

_
-

I( +
X

+ -,-x - )
x' + I

d .<.

Tite first two tenns on the righl, namely. x + 4. can be integrated inunedi.ate ly. and partial
fmccions can he applied to the remaining term, J.' /(x 3 + I) .
As a general rule dtcn: lfrhedc:gree of N(.r) io;;grcatcr thal'l orcqual to the degree of D(x) ,

divide D(x) int<> N (x) to prooucea quotient. J>Oiynomial Q (x) and a remainder polynomial
R (.) of degree le~<.' than th<lt of l)(,t); that is,
N(x)
D(x) = Q (.t)
where dcg

< dcg

R(.r)
D (x )'

D. hncgral8.8 can then be written in the fonn

N(x )
D(x) dx

Q(x) dx

R(x )
D (x ) dx .

TI>e first i ntegral on the right is trivial. This lea, es intcgnuion of the nuional function

R(x) dx ,
D(x)

where deg R < c.k..>g D.


To use partial fractions on integrAl 8.9 we must fac-tor 1hc denominator D (x). In Section
1.2, we Slated 1hat every real polynomial, a11d specifical ly D (x ) , can be factored i nto real li1ear
factors CI.Y + b and irreducible real quadrntic factors ax' + bx + c . A quadratic factor is
irredudblc if b 1 - 4ac < 0. In the complete factori:tation of D(x) , these factors may or may
not be repeated; that i s~ the factorization of D(x )

(ax

+ b)"

COiltaill S

(ax 1

and

tem1s of 1he fOt_n l

+ bx + c)",

where 11 > I is an integer. \ Vhen 1t = I the ftlCti'lr is nonrepe<liOO, anti when n >
repe.a1ed (it has 1nuhiplicily n). For exa mple. in 1he factori?lltion

I, ir is

+ 1)'(3x2 + 4x + 5)(.2 + 1)2.


+ 5 are nonrepeared. ruld 2x + I and x 2 + I are repeated.

D(x)

= (x- 1)(2x

x - I and 3.t2 + 4x
his worthwhile noting here that D(x )
(x - I )(2x - 2) docs not have two d istinct
lincarfactors since we can write D(x) = 2(x- 1)1. Ukewise. D(x) = (x 2 + 1)(3x2 + 3)
docs not huve distinct quadratic li1ctors: it has n repeated quadratic fuctor D(x) = 3(x 2 + 1) 2 .
Distinct factors must have differem zeros.
Having factored D(x ) , we can sepanue the rntit>nal func tion in 8.9 imo fracti onal compo
ncms. We call this the partial fracl ion d ecomposition of the imegrand. We illustrate with the
rational function (x 2 + 2)/l"(x- 1)(2x + 1) 3( 3x 2 + 4x + 5)(x 2 + 1) 2 ) , and the n stategen
era I rules fo r ull deconposit ions. The partial fraction decompositio n o f this ptlrlicular nuio na1
function is

= .- ,---1 + ::-2,-+.,.....,.1 + (2x + 1)2 +

(21:

+ 1)3

Gx + H
Lt J
+
-,,'
2-:
+-:-1
+
(~2
+ I jl'
+ 4x + 5

Ex+ F
2

3x

where A , B, . . . , .I are con!)tants. T he fi rsLterm com:sponds to the nonrcpcatcd linear factor

x - I , the next three conespond to the repeated linear facto r 2x + 1. the fift h term corresponds

to the oo nre peated quadratic factor 3x 2 + 4x


quadrat ic x 2 + \.

+ 5, and the last two te rms result from the re peated

Let u.s now slate general rules for the partial fraction decomposition of rational functions
R (x)f D(x). "lbere are three rules:

8.6

:1. For each repealed or nonrepemcd linear fi-.ctor (a.r


cem1s in1he decomposition:

P.trli:Jl fr:1Cl!OilS

5 19

+ b)" in D(.r). include the followi ng

A,
A,
__:_..:...,..
+ A2 + ... + .,...--....::....,.~
a.r + b
(ax + b)2
(ax + b)

(8.1 0<H

The number of terms corrcspoml' to the p()wcr non (ltX + b)" .


2. For each repc,ucd or nonrcpcmcd irreducible qwKlralic Factor (llx 2 + bx +c)" in D(x ),
include the following 1erms in the dceompo,;ilioo:

B1x + C 1
__;,:.:.;__:......:..:..._
+

B2x + C2

B,x + C.,
+ . .. + -:--::-"--.,.:-....::.....,,...

(8.1 0bl

+ bx + c)"
AS!linthe nuMber of tcmts corresponds tOo the power 11 on (ax 2 + bx + c)".
ax2

+ bx + c

(llx~

+ bx + c) 2

(ax~

3. The complete de<'Omposilion is the Stllll or alllcrms in S.JOa untl nll tcmts in 8. lOb.
What is imporiMI here is to realize thnt all tcnn, in expression 8.1 Oa arc immediately i ntegnJble.
Temts in expression 8. 101> ure unlikely lobe mcnt:JI intcgnuions. but il" b
0. lhcy can
integrated with a trigonon'tCtric :substilUtion. If b-:): 0. they can be integrated by co111plcting the
square tmd usi ''S n trigonornetric .substitution.
The following examples will clarify the above rules.

oc

I EXAMPLE 8 . 15
\Vhat form do pan ial fn1<.:tion decompositions (or the lUI lowing rmional funct ions take:'?

x2

+ 2.r + 3

+ 3x -

x2
(b)
(d)

x' + x 3 + x 2 + x
3x 1 - I

+ 5)(x2 + 2x + 3)(2x -

(Jxl

1jl

SOLtmON

(<>) Since 3x 3 - x 2 - 3,, +I


(3x- I )(x- l)(x + 1), we have nonrepeated linear
factors. Rules I and 3 give the partial fraction decomposit ion

x2 + 2x + 3
........;..:........:...,=__:__::__
1
2
3.r

3x

= .,----- + - - + - -.
3.r- I

X -

X+

(b) The f'IICIOrizntion.r 1 + x 3 + .r 2 + .r


x(x + I )(.r 2 I) has two nonrcpeatcd linear
fiocl(>rs nnd a non<e-IJeaiCd quadratic factor. The three rules lead to

x2 + 3x - I
x' + x 3 + .r2 +
(c) Since x~ + x 4 + 2x 3
decompositi{)n i::;

= -

+ 2x 2 + x + I

__ + +I

X + J.r - X
--,-____,,;:__:....;::_..;:..._
1

.r5

+ .r4 + 2.r3

(d) Si.nce x 2

3x 5
(Jx2

+ 2.r2

Cx

+ f)

= (x

I )(x 2

I)2, I he partial frdction

A
Cx + D
F.x + F
+
.r+J x' + I + (.r' + 1}' .

+ 2x + 3 can be fac tored no further. the deC()mp()Sition is

+ 5)(x' + 2x + 3}(2x

..-..

+- + ---,--X + I
.r 2 + I .

- 1)2

Ax

+ lJ

Cx

+ x' + 2x + 3 + -+--=
Jx2 + 5
2x - I
(2x - I}'

To illustrate how to calculate coefficients io partial froetion decompositions, we usc""" (b) of


Example 8. 15. \Ve bring the righl si<lc of the decomposition to a common Ucnominator.
x 2 + 3x - I
A
8
Cx + D
--:---:-----:,.---- - + - - + ----,-2
.<4 + x J + x + X
X
X - I
xl + I
A(.t + l) (x 2 + 1) + Bx(x 2 +

I) + x(x + I)(Cx + D)
x + x' + x::! + x

anti equate numcnttors.

x2

+ 3,t -

I = A(x

+ l)(x 2 + I} +

8 .t(.t 2 + I) + .t (x

+ I)(C.r + D ).

There are two methods fo r finding the constants A . 8. C . and D .


Method 1. First, we gather together terms in the various powers of x on the right side o f the
e<1uation :

x2

+ 3x -

I = (A+ 8 + C)x 3 +(A

+C +

D )x 2 +(A + 8 + D)x + A.

Now. Exercise 35 in Section 3.8 states that two polynomials of the same degree can be equal for
all values of x if nnd only if coefficients of corresponding powers of x are idemicnl. Since we
have equal cubic polynomials in the equation aboe. we e(Juate coeOkients of x 3 x 2 , x , a nd
.\" 0 (meaning tem1s with no x's}:

0 =A + 8 + C,
I= A +C +D.
3 =A + 8 + D.
- 1 = 1\.

.x-3 :

r' .
X :

xo :

The solution of these four linear equations in f<>ur unknowns is A


-I, 8
3/ 2, C
-I / 2,
and D = 5/ 2. Hence. the panialfraction deco mposition of (x ' + 3x- l)f(x ' + x: + x ' + x)
is

- x/ 2 + 5/ 2
.
xl + I
x' + + x l + x
Nlel_
hod 2. In this method we subs:riuue con\'Cnient "'alue.~ or x inro the equation
=

x3

3/ 2

- -+
--+
't
X+ I

x 2 + 3x- I = A (x + 1}(x 2 + 1) + 8 x(x 2 + 1) + x(x + 1) (Cx +D).


Clearly,

,'C ~

0 is most convenient~ since it yields the value of A:


-1

that is. A

= A(l )( l):

= -1 . Convenieru al~o is .t = - 1:
(- 1) 2 + 3(- 1) - 1 = 8(-1)(2) :

it gi,-cs 8 = 3/2. The ,-alues

x = 1 and x

I + 3(1 ) - I
4

+ 3(2)

= 2 yield the equations

= A(2)(2) + 8( 1)(2) + 1(2) (C + D) ,

- 1 = A(3)(5)

+ 8 (2)(5) + 2(3)(2C + D ) .

When A = - I and 8 = 3/2 are substituted into these, the resulting equations are

C+D=2,

4C

+ 2D

= J.

8.6 Par1inl F~c ti OI!S

521

= 5/2.

11>e sohuion of 1hcsc is C = - 1/2 and D

11is also rossible rouse a combination of methods I and 2 for finding coefficien1s in a 11an ial
fraction decompo5ition: 1hat is, substitute some values of x and equate some coefficients. TI1e
total nlunber must be equal to the numb:r (Jf unknown coefficients in the dccomJXl."iitiun.
Once we have completed the panhll fi-ae1 ion decomposition of a nuional furlction, \\'C can
intesnue the function by finding antiderivatives l\f the componenLtractions. For example. with
1he partial frac.tion decompo5ition of (x 2 + 3-t - I )/(.~' + x l + x 2 + x) ,;, is very simple to
find i t~ indefinite integral,

x' + 3x- l

-,--~-~- d.r

. x' + x > + x l + ,{

! (I
-x

=I

+ -3/2- +

- x/2 + 5/2 )
(/.~
x2 + 1

x + I

31

I
-dx
+-

- I-

.t

+ I

X - d.t
dx - -I I - ,-

3
2

I
4

.f

+ I

>

= - ln lx l + - In lx + II - - In (,r- + I) +
Other imegralions by partial fraccion
example.

decom~i1ions are

5!

+-

-l~n-

-2-I d x
x + I

x + C.

ilh1strated in the following

I EXAMPLE 8.16
Evaluate the followi ng indefinite integrals :
(a)

I + ~Tl +
1-+- dx
x'

5 dx

(b )

(c)

x>

(d)

_r'2 + 1
2x> - 5x2 + 4x -

I
1-t-dx

1 d.r

X' + 8

SOl UTIO'I
(a) Since x' + 6x 1 + 5
(.<1 + l)(x 2 + 5). the pnrtial fraction dccomposi1ion of
2
x j(x' - 6x + 5) has the fomt

Ax + 8

X
--~-

x + 6x + 5
2

x +I

Cx + D

+ --:--,-x' + 5

\Vhen we bring the right side to a common dcnomimuor and equate numerators, the
~ultis

x = (Ax

B )(x 2 + 5)

+ ( Cx

+ D ) (.r 1 + 1) .

We now multiply out the right s ide and CQU11te coefficients of like powers of x,

0 =A+ C.
0 = 8

+ /),

I = SA + C,
0 =58+ D.
Tile soiUiion of ohese equations

therefore

x
~-~-dx =
x' + 6x 2 + 5

c~

A = 1/ 4. B = 0. C = - I/4. anti D = 0. and

I(

x /4
- x/4)
-+- dx
2
2
x + I

8 In (x 2 +

x + 5

I) -

In (r + 5) + C.

(b) Since 2.r'- 5x 2 + 4.< - I

= (2x- l)(x - 1)2 . 1hc par1inl frac1ion dcconli){)Silion

tukes the tbn'll


,, 2 + I

--:-____:~__:_

A
__ - __:..:..._
+ -8- +

2t' - 5,t 2 + 4x - I

2x - I

x -

.,........:~

1)2

(X -

A(x- 1) 2 + B(x - 1) (2x - I) + C(2x- 1)

(2x - l)(x - 1) 2

When we equate numerators .

.x2 + I = A (x - 1)2 + B(x- 1)(2x- I) + C(2< - I),


Wc nowse1x

=0. x = l ,nndx = 1/2:


X=

0:

I= A + 8 - C,

X =

I :

2 = C(l),

111CSC give A

= 5, 8 =

- 2, and C = 2. and 1hercforc

2
x
l
~-~,.----dx

2Jrl - 5x2 + ~x - I

I[

2x - I

:::: -ln l2.r2


(c) Since- x 3 + 8 = <.<
I

- x3

+8

= A(l / 4) .

5/~

x= l./2 :

+ 2)(x 2 -

2.r

- -2x - I

2
dx
(x - 1)2

11- 21nl.r -

Jl -

- +C.
X - I

+ 4), we se1
8x + C
+ ....,.-- - -

= --

+ 2) (x2 - 2x + 4) x + 2 xl A(.t 2 - 2x + 4} + ( Bx + C)(x + 2)


(x + 2)(x 2 - 2.< + 4)
(x

2x -,- -1

and now equctte nurm:r"cors:

I
We set .r

= A(.<2 -

2x - ~)

+ (Bx + C)(x + 2) .

= -2 and equale coefficiems or x2 and I:


X=

-2 :

I = 12A,

x' :

0 =A+ B .
I = 4A

I :

= -l/12,andC

11-.esegive A= 1/1 2. B

_ l _ d,

x3 + 8 '

= / ( -1/_1_2
x

+2

- In lx
12

+ 2C.

= l /3. andlherefore

-x/ 12+ l/3 )dx


x 2 - 2.r + 4

+ 2 1 + -I

12

4 - X

(x - 1)2

+3

dx.

8.6 Parlial Frat::lions

1 =

For 1he inLegra_l on 1he right. we set X -

dx =

I
Mjlclll;lij:HfW
ht x - I= v'3tan0

~x~- 2x+ 4

./3 sec2 0 dO, and

I
x 3+ 8

I
12

+ 21 + -12I

_2_ In lx

+ 21 +

_2_ In lx

+ 21 +

- - dx = - In lx

12

I {;;
12-v3

Triangk to

x-1

12

141
r-;

12-v3

J31an

e.

523

in which <..-.ase

(I + ./Jum &) r., 2


-v 3sec 8d8
3sec2 &

I (3 (w +

./3 tan

o) d(J

./J In I cos 81) +

(x-

./J
I)
I
I
= - In lx + 21 + - Tan- ' ----r:; + - In
12
12
vJ
12

C
./J

.Jx2

2x + 4

+ C

(Figure 8.12)

(x- I) I

= - ln lx + 21 + -./J Tan- 1
12
12

--

./3

- ln(x 2
24

2x + 4) +

where D = C + ( 1124} In 3.
(d) Do not be misled into partial fractions in this example; the nuional fmlction is imme-

diately integmble:

x2

- ,- -. dx = - In lxJ
X' + 8
J

+ 81 + C.

There arc a number of useful devices that can sometimes simplify calculating c.oefHciems in
partial fraction decompositions. \ Vc indic.ate two of them here. First.. if ax+ b is a nonrepeated.
linear factor of the denominator of a partial fraction decomposition, then its coefficient in the
decomposition can be obwined by covering up" this te.ml and substituting x = - b/a into
what remains. For example. to find 1he coef(icient A in
X
.,----___,.,,.,.,..--~

(X - I )(2x

+ 3)

A
= -- +

x - I

8
2x

+3

'

we c()ver up the x - I and substitute x = I into xI (2x + 3). We obtain I I (2 + 3) = I15.


This is A . To find 8 , we cover up 2x + 3 and substitute X= - 312 imoxl(x - 1) . The result
is 8 = - (312)1( - 312 - I} = 315. '!bus,
X
115
------ = -(x - 1)(2x + 3)
x - I

315
2x + 3

+ ---'-

Remember. however, that cover up can be used only on nonrepeated. linear factors. h could be

used to find A, but not 8 or C. in


(3x - l)(x2

+ 2)

A
3x - I

8x + C
+ --,--'--:x2 + 2

Second, when the denominator consists of only one repeated linear factor. say (x + 3)4 ,
rewriting the numeralor in powers of x +3 c.an sometimes give the parLial fraction decomposition
very quickly, especially when the numerator is a linear or quadratic polynomial. For example,

to lind the partial fl'action decomposition or (x- 4)/(x + 3)4 we write

x - 4
(x +

3) 4

(x

+ 3) -

7
4

(x + 3)

(x +

(x

+ 3) 4

l ,ikcwbc,

(.< + 4 ) 1 - 8x- 14
(X +

I
= (X

4)6

(x

+ ~)

(.t

8
4)' - (X + 4) ;

8(.r

+ 4) + IK

+ 4)

IS

(.t

+ 4)6

We hve chemcal problem 10 soh.,. T'"' subslaoc.,., A and B re.ICI to fl'"" a lhinl
<uiNane<o Con"""' a liB) that 2 grams of A react With 3 grams of B to produce 5 grantS

of C. When 20 grams of A and 4() grams of B ate originally brought together, J gran"'
of Core focmed in the fil'\l hour. Our problem i> to find tbc amount of C P""""" in the
llliJ.CUn.: at uny time.
'iOI l 110' We need to assume something about the rate at which A and B combine 1<>
11ivc C Con~ultsti on with chentical engineer. sugge.IS that for many chemical rcaClion~.
the ntle at which chemicals react is proponioc~al to the amounts that me pc'e<elll in the
mixture; the more chemicals, the faster the reartion. To e'(pc'eS)

thi~

algebf:ucally. we

let x he the number of gnuns of C at time 1 in lhe mixiUre. Some ot' this cunte from A
and some l'rom B. Specifically, 2x {5 grams came from A and 3.<{5 11rams can"' 1'1<1111 B.
Thi< menn ~ that there are 20 - 2.x /S gram< of A and ~0 - 3x /5 gran1s of B remaining.
The rutc at which C is formed is represented by its deriY.Jtivc d.rfi/1 u11d hecnuso it is
proportionll to the l\OlOUillSof A ;md B present in the Rlixture. we write

2K

= - (50 - x)(200- Jx)


25

= k(50- .r)(200 -

3.r).

where we hnvc set 1: = 2 K f25. Notice that we multiplied the """"'"" of II und B
on the mtxture rather than add them. By adding them. we would ha\C the UMCccptable
sillUion dx / ill being greater than zero even when 01\C ofthei'C&CUUIIS \anishes. Sinc-e
1 0 when J\ and Bare brought together, .x(r) must also satisfy .r(O)
0. In addition.
x(l) = 3. We 11<e partial fractions to write lhe differential equation in the fom1

or

J:dr

I
= (50- .r)(200dx = ( 1/ 50 Jx )
50- ,,

.l/ 50 ) th .
200- J.r

ll >s ,;epomtcd, and sol utions arc defined ompliettly by


I

~1-"- C = - [-ln(SO - x ) .,- ln{200- 3.<)).


50
Absolute value< nrc unneces<ary because x eannot exceed 50. We now solve this equation
for x by writins
20050(kl +C) = In (
,
50- X

3x)

nnd C"<ponemiming,

200 - Jx =
50- X

ot!'"'''

8.6

whci'C D

= esoc. Cross multiplying gives (50 -

be solved fo1 x

200-

50De 50kr

3- Dej()k1

= 200 -

626

Jx. and this can

. The initial condition x(O) = 0 requires D = 4. in

which caw

x=

x) De501a

1\ll'lial FrJction

200 - 200e50k1

3-~

200{1- ~)

_.,.

J - ....,-- 1

grams.

Sincex(l) = 3.

3 =

Mji j ; : :
Amount oi
subs&ana ~ in dacuttellll

200(1- ~)

k = _!_In ( 191).

3- 4esot

Thu~. the number of grams

50

188

of C in the mixture after I hours is


.
_ 200 (I _ e' .. (191 /tllltj
(I) 3 - 4e' .. (191 ( tlll) .

50 .......

We could plm this function or draw it using techniques from Chapter 4. What is
iltlcrcsting to note i; thm with only the differential equation and some physicalt"CUSoning.
we can get a very good idea of the shape of the gtaph. The graph begins at (0. 0). and has
horit.olllalasymptme x = 50. The differemial equation shows us that dx f dt.the slope
of the graph is a maximum at t = 0 when x is smallest, and decreases as;. increases.
The graph can therefore have no critical points or points of inflection. II mu.sl UJ)J>car ns
in Figure H. I ) .

Nuclear scientists arc approaching us Lhis time. The volume (in cubic mcLrcs) of a plug
now-reactor to operate for 90"k frac tional conversion is given by the integral
I

V = 393

022

(0.096 - X /2)
(0.024 - X }(0.024 -

3
X /2) 1

dx.

Evaluation i$ often done numerically because it is claimed that the integration i.s too
complex. We are a>kcd toe\ nluate the integral exactly so as to reduce error-. in numerical
intcgmtion.

SOl I 110'

I
f
2
=

FirM. we consider the indefinite imegt111

(0.096 - .r/2) 3
dx
(0.024- x)(O.Ol-1 - xf2)l

= -21

(0.192- x) 3
1
'x.
(0.024- x)(0.048- x) 2

Let "' 5et a = 0.024 in which ca5e panial fractioos lead to


1

(80 - .r)

f[+
2
I

(a - x)(2a - x)ldx =

= -I [ x - 343aln Ia - xl - 216a2
2

2a-

343a

216a
a - x - (2a - x) 2

+ 324a In l2a - xl ] + C.

324a ] d
-

2a - x

Consequently, the volume of the reactor should be


V

2(3~3)

[ 0.022 - 343( 0.024) In 10.024 - 0.0221

216{0.02-f)1
O.O.fS _ 0.02

+ 32'i(0.02-1) In

O.O.f8 - 0.0221

= 0.023m3

EXERCISES 8 . 6

l.n ExCt'ciscs 1- 16 cHtluatc the indefinite integral.

I.

.!.

5.

7.

9.

J ++
J--d
+z z
J X d.r
Jx .r' +;I- +
J-,-1- dx

., .r 2
d.'(
.r- - 2.r I

t'

13.
IS.

16.

6.

(X - 4)'

3-

3>

M.

1 d.<

10.

l z'+J:
- 1tl'l.
)

12.

z +5:

f
f
f

.r' +

;xl + 6 d.<
3t+4

z' - 3z1 +3t 1 - z

14.

I''+ 3)' 2 + 3,1 t

J'+2.!.- 4d
J +v+l dr
J x'+ 2 .
r

.r 2

2.r8 '

y'

.1'' - 6y .

(.t '

2)

I ,., +
I + ++
I x'+ r'.
11 I 31

dl'
2

y' ~ 61

.1' Sy'

ku'.

V(i )

dr

4 -

wbcn: V

= ~- Om you inl<rpr<l V

form

til

ph)6i<olly?

= 0 10

A c-.u ofm1"s I!)(X) I.:J .uan~ from rcq m un ntcrliCCCon and mo'-es
in cbc positive .r..duecttOn. The ~nganc 4!'(Cri.S n conslant Ioree of m.'l8"
nitude 2500 N, nnd air friclion Cuscs a rcsisti'"c force whose magnitude
in newtons is cquotl to the Mtuarc lll' Lhe speed uf the car in mcli'Cs per
second Newton's second la\V 21vcs rhc rollowina dtlfcrcnual c(juation
for the velocity or the car:

= 2500 -

dv

mvdy

= mg -.tr12,

where J is distance faJien by tht: raindrop. find lhc \'elocily o r the

raindrop when il strike< the <:lrlh ir .. ralls from helghl h.

* 18.

tlv
dt

= mg -

where g = 9.81, and k > 0 is a COMtam. SCJXIrale the di1Teren1inl


equation to ~how that if the rt1indrop exits v~rtically downw.1rd from
Ihe cloud wilb vcJocily v.,, then

2 dx

1500-

19. Wbcn a r.tindrup with mass m ralls in air. it b. actL"C.. on by &r.JVity


and also by a force due 10 air rc~as 1ancc that i~ prnpnrtit'lnul to the
square of its instantaneous s peed. Accordins to Newton's &ec.'\Ond luw,
the diflCrontial equation describing the velocity of the raind1'0p is

20. lf"ewishtoknowlht ,doci(yoftheroindropinExcrrise 19uit


5lri.kc::s the car(h, it isp::fcrabk:: to find \Ciocily I.\ a function of disua.nce
rallen (iosle3d or lime). 8) pre>Sine lhc: difi'CJ<Oiil cqu.aion in lhc

+ 2xl -lx2- 4.\" + 4

1/2.

du

+ 3

<'

(b) Find lhe position of the car uso. (unction of time.

'"d,

dr

17. Fmd the l<ngth or the CUN< ,. = In (I - .<') rrom


X=

dr

.r'+ 6
x4

4.

11.

(a) By scp>roling !he diiJcrcntial cquolion show 11\:u {I)


50(1 - .-1")/( 1 + .-1'5).

.,

v.

* 21. The cxpoocntial growth of bacteria in the two c~crctt;CS of

ec
tion 5.5 is unrcal.i.slic in the Jong tern\ when there jj t\ lintitcd roo.l
suppl). Tbc logistic model inlroducc). a quanlily C cilllct.l thc c atn
mg capal'ftJ for the crwironmenl in which lh.: b.lCCeria fin: h\'ing A'
Ibe nur.nbcr N(l) of b ac1 cri;,~ appro:.chcs
iii srowth f':l.lC mus& slow
down. The logiSLlc model to describe this is the din"crcntial ct~umion

c.

dN
dt

= kN (I_N)
c .

"""' N ;,. moll. d V / dt i> >ppro<irlutdy equal to l IV.


oi1U> pr<><rV~nJ o.otl) npo'><1!ial ~tll. Tbe fac:IOI' I - N IC c~~U~<.~
dN /dl ..., 0 a< \ - C Sol\e lhi< c!illcrentoal <quo.to for IV(I)
.. t>t" ' k ,. t. C tOO, tlld N (O) = tOO.
1'<'11 that

.. 22.

Sbo~ Uu~:lthc sotuton 01 1he ioi,tic nnJel

1Uil1.tl pe1'lliiJ.1ioo N (Oj

in h urd e 21 for an

Jl.

.r + S.r'- .r'- :!.r +I

x3

-L

x.a xt .... x

JJ. Drin~ the initial ""''~of now on u pipeline of lcn;th L . the


\CIOCtly of l tH: flow t' mull Sllh St )' the tlll(erertllUI eQUI1bOn

= N0 ''
I+(

di = T '
where g

23. l'ullltcccnuuiucfthercg.unbool'dcdbythc""'V<'(' +2) 1)


~ -.r.
0. I '= O.ao'<l (.t. )';::: 0).

= tl.S I and H~ is 1hc cFI'cclivc heud l)l' lhe Iinc, given by

II, - II (1 -;"r ),

14. Suppose th.;u N rcrrcse~ tbt 1unbc:r of pcopk: irl .a JX!puL&trt'n.


and .r(l) the nuolkr ohm arc inf<ct<d by 1'001< di= :11 any 2iY<n
tin:~C t h i< oftu a-._mncd tlulthc r.uc ol infc<tJon is prupJnionul to
ttc ptodo<t of the nwnb.:rof inf:led ud not .nfco:tcd.

wtL-re II istheconst.anl head al\d t11 u lhc final vclocit) in the pipclir.c..

(a) Using ttc iti:ol oonditi"" u(OJ 0 . sll<>w tl!a

(''f + r)

Lc1
I = - In
2!111

d.r

d1 = k.r(N - .r).

-.tllT'l: k > 0 is u cocblanL Titus. .x(z) mu!ll ~atnry thi~ ditrc.-cntiul


equation subjcclllo an initiJ) condition such
pcflltlllS . .t(O) = 1.
1b.:n:fvre.one infc:ck.'tl pcn.on i~ introdu,cd into 1.hc pupul:alion at l~ mc
I - 0 . Hnd .r(l).

!l."

~ ~~.

dOl

c
~ONu ) , 11

IV-

dx

''l- '

(b) Solve the equuion in pa11 (a) 10 obwin 11os o func-liott of/.
34. lnthcstudyoffriclional flu id flow in .educt. lhc ro11owlng indcfinjt c

ir11cgrnl is encountered:

25. Cth:mk:al ~.tcttoft') such <1.'5 1hm. in Project 12 an: 'ailed $.'Ct'lfld,.
ordu rC'actton< (b<uu.se of the .t 1 tern1 on til;! righl). In gl.!ncrul. lhcy
Ulkc 1hc (orm

d.r
- = k (a - xl(b dl

~).

Sohc thi.l diiTcn:nl1011 CGu.:alion in the c.01:o;cs that (a) a

.; b. (b) (I ,l b.

16. The clocty P of " "Jt<r, llowtng from a tap tbat IS wd<koly l...,.ed

oa. v.uies inll.i.I'Iy l.ilo.<Ording ID

do

a -dl - r 0

~tcuc.J)SU.Ic

In 1::><'1\:t><'ll 27-J2 cvaluutc the in<Jclinitc incq;o"al.


17.

JS. l(an 1nli...-o.grud is a ri.hoaal furt11on ('If sln .r and ~A. il can be
reduced to a r.uimaJ fu~uon of I b) I~ ~-liiUiion

'.

a > 0 is a conMnl ~nd v0 is the


u(l) """~the inotiol con~tion v(O) - 0.

w-f'IC:I'C

wt\!re k ~~ a ccns~an1. Eva!uGIClha ID&...'111.

vcltdt). find

Th1\ ~ of\cn called thl; Wck:f,lrot'"

(l.lb~ lilulion.

Show lhal

wtlh lhi~

Alh~q ltution

d.l = - - , ({/,
I

\:OS.\

+t

A' + .t+ 2 d
,\ J

l \ J ... '

.'C

In Exercise< 36-39 use the sub<titut;on ot' l!~ercise 3S to ev>luatc the


itlcJral

19.

JO.

I
1

1
(< 2 I S)(.r 2

-~
-, d-
t +r

+lr

~ J)

37.

dx

38.

I __

I: .__ d:r

I - 2ca;x

J9.

I
I

3 I Ss.n .r

dx

sin t+cost

d:t

40. Show thai the answer 10 Excl'ci~ 36 c::tn be expresOO in lhe usual

form, In Isecx +

tanxl +C.

(c) Show Lhal

. .. 4 1.

:..'" + :,:.n-
)

Js -

1
4 <'0S.C

d.c - :Tan 1(3han ;:) + C.


2

( -coo.<
...... )
2

is ulso Zlll usllidci "ali\'c of' (S - .J c:m :c) 1 Usc: i11o C\':tl
uall! lhC deli nile in1c:gr3l in pan (b).

(b) Wh!tt lt:ll)f)Cn:o. "hen theanaidcrivmivc in part (a) ii: u:!d lO

eva!ue:u:

"~ 42. Evaluouc

!.''

5 - 4 cos x

Evaluutc

dx?

+x +3

I . + .... + +
+
+
+'
+
I
J ''

2.<'

2x ; 8.< 2
,, . 4 :Jx 3

11.< -

s dx.

3.< S d
X.
2x - 12

18.7 Numerical Integration


To evalwne 1hc definite integral of a cominuous functio n .f'(x) with respect to~ from x =a to
x b. \\'C llll\e used the fin;t fundamcnlllltheorem of imcgral calculus: Find an anti~erivative fOI'
j (x ) , subo;titutc the limits .r = b a nd .r = ll . nnd subuact. The e valuation1>rocedure de pends
on o ur ability 10 produce an antidcrivative li.>r j (x). When a fu nction f(x ) is complicated,
it may be difficult or even impossible to fi nd its antidcriv<Ot i\'C. In such a case, it may be
necessary coal>proximatc the definite irltegral of f (x ) on some interval a :::; x ::S b , rnchcr than
e\'alua tc i1 analytically. We d iscuss chree methods for doing thi.": the rectangu la r, Lrapczoidal ,
m>d Si mpson~ rules. Each method di' illes the inte rval a ~ x ~ b i nto subinterv>lls and
approximates j (x) with an easily integrated func tion o n eac h subinterval.
If j (A') :::; 0 on (I ~ .1 ~ /> (Fig,yrc 8. 14 ), the delinitc intcgnol of j(x) w it h respect to x
can be imerpreted as the orca bOunded by y
/ (x) . y
0 .r
ll , and x = b. We have
carefully pointed o ut that it is not ah, ays wise to think o f a definite integrnl as area. but for our
discussior1 here it is conve nient 10 do so. Ou r problem is to a,)(Jroximate the area i11 Figure 8 .14
when it is d illlcult or impossible to find an antidcrivoti\'C for f (.t }.

lnilii'J

JllfW Odl1litc
wi ~~Cn I (x) ~ 0

integtnl ns. atJ


.1'

y =J(.r)

(I

b ,\'

Rectangular Rule
The tirst mc1hOO is lO rctunl 10 dd i nilion 7 I for area in 1cnns of approx imating rectangle~. \Ve
subdivide che inter val a ~ x ~ bi ntonsubincervalsb)'pointsa = xo < x 1 < < .t'A- 1 <
x11 = b. For simplicity. we d1oose 11 equal !rlubdivisinn..~. in w hich Cil.<oc

.rr

=a +i

("- (/ ) .
-~~-

a nd <lcnotc tltc w idth of ellch sub intc." "' by h = (b - ll )(n . Area under )' = / (x). bovc
y 0. and between x = a ond x b , which i~ given by the definite integral o f f (x ) rrom a
to b. is appl'Oximated by the re<'!M gles in Figure 8. 15.

(h

J.
(I

"

"

i=l

i=t

f (x ) dx"' 'L, J (x;) lt = h 'L, J<.~;) .

(8 . 11 )

This is the rectangu lar r ule fo r approxjmntiug the dc~njtc integral: we ha"c replaced the a rea
under y = f(.K ) with 11 rectangles. Another way o r looking ltl it i& toll<ly t hat we ha\'C replaced
the oi g:inal function / (x) by a function that is c.onstant on each subinterval (Figure 8. 16),
but the COI'IStant value varies from subinterv<~l to s ub interval. Such a function is called a step
ftmcri<m. The de(inite integmJ of th is function from x = a to x = b is the right side of 8. 11.
It approximates the de li nite integral of f (x).
Graphically, it is reasonable to expect that an i ncrease in the number of subdivisions results
in an increase in the accwacy of the approximation.

8.7

Appf\,,,,.._ .., of area

M4flil.l. I'

M j t I I I! I I 81 W

N.itnenc.al lnl~ti on

ln!a-pt~Uhon

521

eX rect.ut-

by re.:unglh 01 t-.lW ~ idth

{(b)

j(,,,,

-'l .1.'-4

/ C.<tl
.t,,...l b
\' , .. 1

A,-1

-'1

..t1 XJ

/C.,)

f(x, J

I ,,,

'

x,_,

I EXAMPLE 8 . 17
U..:" subdivcsiO<l ot the inter\ at t !f .x !f 3 inco 5. 10. l!lld 20 equal pan< co approimate the
W:linicc incegr l

with the rectangular rule.

SOl U'IIO'

Wi th 5 cqlotlports, h = 2/5 1111d

/,l

sin .x tlx ""

sin

5 i l

Xt

= I + 2i / 5. Coot<cqucmly.

(r + 5i)
2

~5 (m ~5 +sin 5~ +~in~5 +>in ~5 +

1.370.

With 10 equal pans. Ia = 2/10 = 1/ 5 and , = I


1

10

>in

3)

i / 5. so chat

;)

S L:: sin I+ S

= -5I(si n 65- + sin -75 + ~in 5-8 + :,iu s9- + ~i112 + sin -II5
+ sin -12
+
5

13

sin -

14

+ sin -

+ sin 3

1.455.
With 20 equal pans v.c ha'c
llll

- "\' <i n
10 L
il

j)

I+10

I (
II
6
13
29
)
= j(j
; in j(j +sinS + sin
+ +sin j(j +;in 3
10

1.494.

Wheo we c.ompare these results with the correct answer,

f.

sinxdx =

[-cos,t)~ =

-cos 3+ cosl = 1.5302948,

we sec (hal as n increases from 5 to 10 10 20. the upproxlmation i mprovcs. but


increased even fu rther to give a reasonable approximation to this integral.

i1

must be

Trapezoidal Rule
Regan.lil'l area. it is clenr in Figure 8.15 1h~u were we to join succe~s i ve poim~ (x;. f (x;)) on
the curve)' = /(x) with straight-line scgmcnb; a:t in Figure 8. 17a. the a1ca unUcr this broken
straight line would be a much bette r approxi mat ioll to the a rea under)' = J(x) than that
provided by the rectangular rule. Effectively, we nnw ap1)roxil'llt1te che ;.&rea by n uape1.oids.
Miji?Jii Jj

'fl

A pproxim~iun

or orc.a

by crapczoids or equal widlhs.

X; ... ,x;

-iilt--

xJIl

.t

x,

x.. ... l

Since the are<~ of a trapezoid is its width multiplied by the ;wemgc of its parallel lengths, it
fo llows that the area of the i'" tr-dpczoid in Figure 8. 17b i, g iven by

where again h

=x;- X;- 1. As a result. II~<: area under y = /(.r) can be apwoximatcd by the

sum

h [/(x o)

+ /(.ru) + f <xz) + /(x o) + .. . + ::....f<x::.:,)__:


:... +..,.:f:...<:....:
x:...--'"') ]
2

~lf(a) + 2f(xo) + 2f (xz) + + 2/(.Yt) + + 2/(.<n -o) + / (b)J

I [

[(a)+ 2

?=
f(.r;) +/ (b)
,_,
n- o

We wl'ite the.refol'e thm

J.

f (x)dx"'

1 [

f(a ) + 2

E
n- 1

f (x;) +/(b) ,

(8.1 2a)

8.7

N um erical lruegr~(ion

531

where x; = to+ i h = a+ i(b- a)/ 11 , and call this the t1apezoidal r u le for approximating
a definite integral. Note that ir 8.1 2a i!'.i written in the form

1
I

'

j(x)dx"" h

f(a) - /(b) ]
2

+ h L"

(8.1 2b)

f(x;),

t l

the summation on the righ1, except for the (irst two tenns. is the rectangular rule. Tn other words.
theexua numerical calculmion involved in using the trapezoidal rule rather than the rectangular
rule. is mjnimaJ. btu it would appear that the accuracy is increased significantly. For this reason.
the trapezoidal rule supplants the recta ngular rule in most applications.

I EXAMPLE 8 . 18
Use the trapezoidal l'llle to a pproximate the definite integral in Example 8.17.
With 5 equal partitions,

SOLUTION

:sin x dx "" Z
2/ 5 [ sin I

/.1

+ 2 { ; sin

I +

2i)
S

+ sin 3 ] = 1.5098.

With 10 equal parts,


sin x dx :::,

;s

[sin I + 2

t,

sin (I +

i)

]=

+ sin 3]

1.5252.

Finally, with 11 = 20, we have


19

sinxdx "=' -1/ 10[


- sin I + 2 f;s in ( I+
10
2
/.
1
l

+ sin3

1.5290.

As expected. these approxiruali.ons are sig:nifkaotly better !han correspondiJlg: results usiJlg lhe

rectangular rule.

Simpson's Rule
MjtCjll;l []10 Approxi
.nation of c.ur\'e by pru~bolas fo
Simpson's 1\lle

)'

.\' =/(x)

y = ax2 + bx + <

Xi- 1

X;

Xi+!

'lbe rectangular rule replaces a function f (x) with a step function; the trapezoidal rule replaces
j(x) with a succession of linear functions - geometrically, a broken s!l'aightline. So far as
ease of integraLion is conceroed. the next simplest fuoction is a quadratic function. Consider.
then, replacing the curve y
f(x ) by a succession or parabolas on the .subintervals Xi - l ~
x ~ x;. Now the equation of a parabola with a vertical axis of symmetry is of the form
y = ax'+ bx + c with three constams a , b. a nd c to be determined. If this parabola is to
approximate y = f (x) o n x; - t ::; x ::; x;, we should have the parabola pass through 1he e nd
points (x;_ , .f (Xi - t)) a nd(x;, .f(x;)) . But this imposes only two cond itions on a, b, and c .
not three. To ta ke advantage o f th is flexibility, we demand that the parabola also pass through
thepoim (x; + 1 , j(x;+1)) (Figure 8. 18).
In other words. instead o f replacing y = j(x) w ith 11 parabolas. one on each subimerval
Xl-l :S x ::S x; , we replace it wilh n / 2 parabolas, one on eac.h pair of subintervals x ; - l ::S
x :5 Xi+l Note. that this requires n to be an even integer. These three conditions imply that a,
b, a nd c must satisfy the equations

axf_ 1 + bx; - 1 + c = f(x; - t}.


tu:;'
axf+ ,

+ bx; + c

+ bx;+l + c

(8. 13a)

= j(x;).

(8.13b)

= / (Xi + tl

(8. 13c)

These equations de1ennine 11tc values ror tL . h. and c, but it wi II not be necessary to ac-tually
solve 1hcm. Suj)IX)se for lhe momc.nt that we have solved equations 8.17 for a . b . ald c, and
we continue our m~i n di scussion. \ Vith the parabol'<l y = oxl + bx + c replacing tht.: CUf\1C

y = f (,v) on the intOI'\'BI Xi - l S. x S. x,..,. we nppmximate the area under)' =


that undet the parnbola. namdy.

j"H(ax
X,

a
3<4+1- xf_, )

+ bx + c} dx =

(.r) with

11

+ z<xf.. ,- xf_,) + c(x, .., - .r,._ ,)

~"l +l-

~ ~

(ax2

+ bx +

_ ..,

(2a(.ri+t

+ Xi+oXi -o + .r12_ 1)

+ Jb(x;.._, + X;- 1) + 6c].


2/t , x;+l = XJ+ h.and:t;-1 = .\'; -h to write everything

Now.ifwcuscx;+J- Xi -1 =
in temlS of It and .t;, we obtain

, .,
1

Xi- I

211

c)dx = -{2a (Cx,


6

+ /1) 2 + (.r1 + il) (.r1 - h) +

+ Jb( (xr +II) + (.r,

(x, - 11)2 j

-II) J + 6c}

= ~ (t~(6xj + 21o1 ) + 6bx, + 6c).


But if we add equmion 8. 13a. equmi011 8.1 3c. and four tin1cs equation 8. 13b, we find that
f (.r,_ ,)

+ 4j(XJ) + j

( XJ+ o)

= a<xf_, + 4.r/ + xJ.. ,) -r b(.r;_ ,


= a [C x1 - h) 2 + 4.vJ

+ b [(.~1 = t1(6x1'

/o)

+ 4x; + X!+o} + 6c

+ (x1 + h)lj

+ 4.r1+ (.q + h)]+ 6c

+ 21,2) + 6bx; + 6c.

Thus. the area under the parabola may be written in the form

~ .,

(ax-+ bx

+ c)dx = - (.f<x1_ 1) + 4j(x1) + j


3

.l'J -1

(x1+ 1)] .

Md the right side is free of ll , b, and c. The same expression would have resulted had we solved
equation~ 8.13 for u . b. and c tlnd then evaluated Jhc integral of ax' + bx + c from Xi- 1 to
.~'i+ J . The derhcuion above, howe"er, is much simpler.
V.'hen we ac.ltl all sueh integrals over Ute u/2 subintei'V'ills .X;- 1 ~

~ x,.+ 1 between

.x = a and .r =b. we obtain


"[
3 f (xo)

+ 4.f(x,} + j

(xz)]

,,

+ Jh [/Cxll + 4.f(x3) + f(x,)] +

+ 3 (f<x, _1) + 4f(x, _ 1) + f(x.l]


h

= 3 (fC.ro) + 4/(x, ) + 2/(xz) + 4j'(x 3) + ...

2j(x,,- 2)

+ 4f(Xn- o) + f(x.)].

In other words. the definite integral or f (X) rrom X =


b

j(x)dx "' J (f(a}

a. to X = b can be approxi mmed b)'

+ 4f(x,) + 2f(x2) + 4/(x;} +


+ 2f(x,_2) + 4.f(x, _ ,) +

f(b) ],

(8. 14)

S.7 Numeric311nttgrntion

533

where x; = a+ i lz =a+ i(b - a)/r. This result is called Simpson's rule for approximaring
a definite integral. Although llle formula do~s not display it explici tly (except by counring
temlS), do uotforgetthatll must be au eoen imeger.

I EXAMPLE 8.1 9
Approximate the definire integral in Example 8.17 using Simpson's rule witlt
II

11 -

10 and

= 20.

SOLUTION With n = 10,


. -6 T, 2 S .tn -7
1/5 [ S .ill I T, 4. Sill
3
55

+ 4 Sin. -8 + _.. 2 Sill. -13 + 4 sm. -14 T,


5

55

. ~]

Sin.)

- 1.5303085.
With II

20,

1/ 10[

- - sin I

II
13
+ 4 s in -10
+ 2 sin -56 + 4 sin -10
+ + 2 s in -14
+ 4 sin -29
+ sin3
5
10

- 1.5302957.

........

1ioblc 8. 1lists the approximations in Examples 8.17- 8.19. The correcl answer for the inrcgral is
1.530294 8 (to <e,en decimal places). Tt is clear char each merhod gives a bener a.pproximmion
as the value of, increases. and that Simpson's rule is by far the most accurate.

Rectangular rule

Trapc7oidal rule

1.5098
1.5252
1.5290

11=5
II=

10

1.370
1.455

II=

20

1.494

S;rnp<Ott'< rule
1.530308 5
1.530295 7

In practice, we use rhe re<:tangular, trapezoidal. and Simpson's rules 10 approximate delinire
intcgn1ls thnt cannot be handled anal)'tically, and we will not therefore ha,e the correcr answer
with which to compare the approximation. We would still like to make some sratement about
the accuracy of the approximation, howeve-r, since what good is the approximation othen\ise"?
The following two thoorems give error estimates for the trapezoidal nd e and for Simpson s n ole.

TH EOREM 8.1
If .f"(x) exists on a ::; x ::; b, and T,, is the e tTor in approximating the definite imcgraJ
off (x) from x = a to x = b using Lhe uape:toidal rule with n equaJsubdivisions,

r.

r J<x)dx -.:
1(1
b

1 [
2

,_,

J<a) + 2 L:J<x;) + J<b> .


i= l

then

II

I < M(b- a)3


-

(8. 15)

12172

where M is the maximum value of l.f" (-~) I on fl ~

b.

TH ~O R~M

8 .2

s.

Iff""(,) niJl'on n < #'" < b. t'll'd


is the cllllt" in appro:tim..dilll: lhc ddinitc iategral
or f( T) from .r o to""- h u..'ins Si.mpsons rutc with n equal tubdl\i~otu,

M (b- a )'

IS. I :5
wl1ere M i-: the mn:c.imum value of l/

IS011 1

"(.t)l on a

11

~ .r <b.

Proofs or eheJoe 1hecw1" $ cru1 be found i" books 01'1 1\tuneri~nl nno.ly4:i~o>. Note thnc becAuse
Clfll\C 11~ faceorin thcdcnominutorof8 16. theaccuracyofSiml,-.011~ rule incrt.a~s n'I.K~h RlOre
raJ)ldly ll'un d<'CS thM-1 of the tr..pe.~oiUal rule.
For ohe func:lioo>/(.r) = sin.r in E>:an11>les 8.17- 8.1\1.

F<r) -

t(.r) - sin r.

and

- >Jn.r

Con,equendy. in bolh Ctbe' "~can ~tak that M = I. and thc:rdore

T.

<

1 -

For 11

0 -

1)

12n'

.2_

s.

and

- 3,1

:5

(3

I)<

ISO" '

.as, .

= 10 .11HI n = 20. we find dwt


2
3(10)1

rr,.J

< -

- 3(20) 1

< 0.0007

und

IS"ol ~

o.oon

ruld

<

S I
lO

8
4S( 10) '

;S 45(20) I

< 0.000 01 8;
< 0.0000012.

Dift'erence"- lxtwcen the CorTect V'Jiue fort he integral and the aPJ,.O'<inuuionl\ li-..ted in Tab1c8. 1
co<robonue ~= predictions.

f EXAMPLE 8 . 20

..

Wha.t &.-.. th~ nlHAimum po'~:tb le error 1n ~ing the:


to oppro'<invue
3
(

tra~1oidal

}, ~I + xl
SOLU flO.\

rule \\ilh 100 equal whdivisions

dx?

AccurJ ing l<> formuho 8. 15, if T000 is the maximum po"iblc error. then

I T100 I -<

M (3 - 1) 1

12(100)2

2M

J~--:-:x 10 '

where M is the muximum of (the absolute -.lluc of) the second dcri\'lMi\c of ollC inocgr-.md
I/ J"i"=Xl on I < .r < 3. Now

Instead of maximizing the absolute ,aJuc of this function on ahc intcr\111 I .S x .S 3. which
would rc:quirc: nnolhcr deri\ati\ c. we note: that the maximum \':tlue of the numerator is obtained

for .r

= 3, and the nunimum '41fue of the denominalor occurs al

.- = 1. It follows therefore

thai the 'ccorod dcrinti\'C cannot possibly b" larger than


3(3)15 (3) 3 - 41 4(1- l )~f'
-

1(, /i

Thus, M muso be less than or equal to 1179/ ( 16./2). and we can ,tate that

2
( 1 179
- ) < O.OOJS..
IT"<l I < 3 x ro -16./2
that i~. chc c1ror in u-..in~ the lntJ>C.toid~tl rule wi~h 100 cqu.c1l :,ubdivision~ to upproxirmue the
defini te integral cannot be any lorgcr thon 0.0035

I EXAMPLE 8. 21

How ntan) equal sulxli' isioo of the intcl'al 0 ::; x 5 2 guamntcc an error of less than I0 -s
in the appmximatk>n or lhe defini te imegml

l.

dX

l l' -'

using Stmpsons ml e?

SOLliiON According to formula 8. 16. the error in using Simpson's rule with 11 equal >ubdivision~ Lo approximate this definite integral is

IS, I ~

M (2 - O)s
18011 '

'

"hereM is the ma~imum oftl'oe(absolute valucofd'IC) founh dcri\1lti\c of c- ' on t he interval


0 ~ x ~ 2. lt is~~ short calcuhHion to fint.l

Instead o f maxrmi1ing the absulu1e \aluc of this function, we note that


value o r I (when X = 0). Fun hcrmurc. because 13 - 12x 1 4.r 11 3
h;n .t "''"'imumat A = 2, 11 follows that

/11

:s 413 + 12(2)2 + ~(2)'1( 1) =

e_.,.: h&\ a maximum

:s + 12.r1 + 4.1

I.

which

-160.

Con;cqucrnl).

15.

< 4 60(2) = 3680

J80n
45n'
The error is less than 10- 5 if 11 is chosen sufficiently large that

3680

-5

10

45n'1 <
lh:lt

h. if
11

3680
(
> 45 x 10

) ,,.
5

= 53 5

Since n must be un e''en inlt!ger. I he required accuracy is g:unr.mtood if, is chosen g.rea1er thi.ln
or equal to 54.
~

FIGURE 8.19
til)g w()rt_ t()

Numerical techniques are indispensable in situations where the function t() be integrated is not
known, but what is avail:lble .is a set of tabu.lated '"'lues for the funct.iQn, perhaps experimental
data. For example, when the truss [n Figure 8.19 is subjected to a force F at its centre, pQint A

Appro:ti

d.etlect ltll$S fJ\)fn

deAccLs an amount )' from its equilibrium position. Forces ( in kilonewtons} re.quired to produce
deflections from 0 t() 5 em at iotcrvals of 0.5 em arc listed in Table 8.2.

tabulaled values of dtlle<'1ion


F

y =O

0.5

1.0

1.5

2.0

2.5

3.0

3.5

4.0

4.5

5.0

1.45

2.90

4.40

5.90

7.43

9.05

10.7

13.2

15.3

18.0

The work done by F in deflecting A by 5 em is the definite integral of F with respect to y


from y = 0 to y = 5. The trapezoidal nlc and Simpson's rule can be used to approximate
th[s deftnite integrol even though we do not have a fonnula for F as a function of y. Indeed,
formulas 8.1.2 and 8.14 do not usc the form of the function to be imcgrated, only its values at the
subdivision points. We have these points in Table 8.2. SiJtce there are II points and, therefore.
10 subdivisions on the interval 0 :$ y :$ 5. the trapewidal rule gives
5
/ ,I

1/ 2
F(J) d y"" - [F(O)

+ 2F(0.5) + 2 F ( I.O) + + 2 F (4.0) + 2F(4 5) + F(5.0)]

~[0 + 2( 1.45) + 2(2.90) + ... + 2(13.2) + 2(15.3) + 18.0]

= 39.67 kN em = 396.7 J.
Simpson's rule gives

rs

1/ 2
), F(y)dy"" "'3[F(O)
I

= 6 !0

+ 4F(0.5) + 2F(1.0) + + 2 F (4.0) + 4F(4. 5) +

F(5 .0)]

+ 4( 1.45) + 2(2.90) + ... + 2(13.2) + 4(15.3) + 18.0]

39.54 kN em

395.4 J.

What we lose en this type of application is the abllity to predict a maximum possible error in the
approximation sinee we cannot find M for formulas 8.15 and 8. t6.
The error bounds i11 formulas 8. I5 and 8.1 6 arc somewhat idealist ic in the sense that they
are error predictions based on the use of exact numbers. For instance. they 1>redict that by
increasing 11 indefinitely, any degree of accuracy is mtainable. Theoretically this is true, but
pmctically it is otol. No mauer how we choose to evaluate the sununations in 8.12 and 8.14. be
it by hand, by an electronic hand calculator, or by a higb-speed computer, each calcu.lation is
rounded off to a cenain number of decimals. The final sum takes into account many. many of
these approximate numbers," and must therefore be inherently inaccurate. \Ve call this roundo.O'cn'OI' and it is ;cry difficult to predict how extensive it is. h depends oo both the number aod
nature of the opera1i01\S involved in 8. 12 and 8. 14. ln the approximation of a detin_ile integral

by the trapezoidal rule or Simpson's rule. there arc two sour= of error. R>rmulas 8.1 5 and
predict errors due to rhe methods 1hemselves.; round-off errors m;1y also be appreciable for
_large n .
We should emphasize once again that allhough we have used area a.s a convenient vehicle

8. 16

by which to explain the approximation of detlnite integrals by the rectangular. trapezoidal, and
Simpson s rules. it is not necessary for f (x ) to be nonnegative. All three methods can be used
to approxjmate the de(init.e integral of a functioo j(x). be it posit ive. negative, or sometimes
positive and sometimes negative on the interval or integration. The only c.ondhion that we have

imposed is that .f (x) be continuous. In view of ow discussion of improper integr.ds in Section

7.10. even this is not always necessary.

8.7

Numerical lote{;l<l60il

537

EXERCISES 8 .7

In Exercises l- 10 use the U".tpezoidaJ rule and Simpson's rule witJt 10


equal subdivisions to approximate the de finite integr.tl. Jn each ca~e.
evaluate the integral analytically to get an idea of the accuracy or tJte
approximation.

J.

,
1

iii

2.

tanxdx

iii

4.

JX+idx

Iii

6.

- dx

3.J.'
II 5.1'
_,
Iii

t.'OS .t i/X

iii

1/'!

II

- dx
x +x
1, ~

9.

Iii

3.4

tf X

i*
->

1
-J

7.

}, Jx + 2

{ ' -:==
l=

4.6 3.8 3.6 3.6

5.8

3X dx

6.0 7.0 6.8

2 1. An aerial photograJ)h of an oil spill shows the J)tl.Uem in the figure


below. /\ssum.i1tg thatt..he oil slick has a uniform deptJt of I em. e.~ti mate
dte 11urnber of cubic met1c.s or oil in Lite spill .

350 rn

1
8. [" ' - - 1 dx
3+x

Jo

180m 110m 2<Xl m 440 m 210m 180 m

10.

50m -

In Exerc ises 11- 14 usc the trapc~oid:d rule and Sirnpson's mle with
10 eq ua l subdivisic)ns 10 appro;<jrnme the defi njtc intcgraJ.

~ II.

Iii

12.

'
t~dx

li!

14.

- - - dx

~ 13.

+ .r3

1'
1_,

e'' dx
sin (x' ) dx

i * 22. The numerical technjques of this sc.'i:tion can aJso be used tO approxi(n!tlc (Ottny improper intcgrnls. Cons ider

IS. Show graphically thal if y = .((x) is <..'On(.'aVC downwmd on the


intcnaJ (' 5 x ~ b. then the lrapc1...oidal mle underestimates the
definile integral of .((x) on this interval

(' e'

}" .JX dx.

J6. What happens 10 the t::ITOrs in 8.15 and 8.16 when I be number of
paT1itions is do ubled?

(a) \Vhy can lhc uapczoidal mJe and Simpson's rule nol be
used directly to approximate tJtis integmr?

17. 11te definite integral

..[X replaces tJtis int~


proper integral with an integral that is not improper and use
Simpson's rule witJt 20 equal subdivisions to approximate
its value.

(b) Show lltal the change of variable u =

is very illlJX)I'tanL in mai.Jtematical statjstics. Usc Simpson's I'Uie with


I6 to evaluate the integral for tl 0 and b
I.

(c) Could you use lherectangulal' l'uleon the illlJ)I'Opet integral?

~ * 18. Use Sillli)Sons l'ule willt I0 equal intervals to 3J)J)I'OXimate tlle


ddinite integral tor the Jengtl\ of Lite pa1abola y .x 2 between x
0
and x = I. ColllJ)tl.re the answer to lllal or Exei'Cise 39 in Section 8.4.

i .

23.

(a) Sbow 1hn1 the definite in1cg:ral

41J/81- x'

L = 3

~ * 19. Use the U'apezoidal 1ule and Simpson's rule with 10 equal sulxli

visions to approximate tJte definite integral l'or Lite lengtJt or the curve
y = sinx fromx = 0 1ox = n / 2.

I * lO.

The swimming pool thai follows has an average deplh of 1.8 m.


It is to be drained and filled with din. To cslimate the volume of di11
required, measurements across the pool arc taken at 1-m intervals. Usc
Simpson's mle to find the estimate.

9-

5x dx

L of lh e t::Uipsc 4x 2 + 9y 2 = 36.
Sbow 1hn1 wbcn we scLx = 3sin 8. lbc 6-intcgral is no
n:prcSCniS lhe length

(b)

longer itnpn) pcr.

(c) Usc the trapezoidal mle and Simpson's nde with eight equal
subdivisions to approximate the 0 -integraJ in pan (b).

!.""
1

se1

equal

G ubll. ivi~ i ons

on the

I
--4 dx

'

I + X

rc~ultin&

intcg_rul.

x'

- 1

x' + I

w .r

ro,. the function labuhucd in

o.s

1.0

I.S

20

l.S

1.8

2.0

: . I

2.6

2.&

) .()

O.M l.ll lAO U i() 1.7$ Ull 1.?') 1.62 1.:11 0.9-1 0.42

J.O

i ntc gr~ l

[I
[/)

- dx

- o.s

1.6

i~ a cubic polynomial. c vuJu ~:~tion or 1hc


of f (.x) lhwn x = tl 10 x .e b by Simpson's rule

mlc?

.. 29.
- 1.0

...

In l~xcrci~>e~t 2-9- 32 how nw1y cqu:ll :-.ubd ivistoft-. of the interval of


integration guar.antcc an error o f k.'$S: U"n 10 _,. in lhc :approximncion
or Ilk' dclinilc integral using (a) the lr.lpc.toid~l rule ~n<.l ( b) Sirnpson's

the followi ''&table.

,.'

1.2

nlways gives the t:XJtcl answer. l llustnue with an cxumpfc.

-:;-.,.---:;-,....,. d X

.r'1 +

1.0

dc linilc

26. U:;c the trupczuidal rule and Simpson's l'ulc lO ~ppn1xirnutc the

xc

E.~rcisc 26 for the irucgrul from ,\' = I h.l x 3 lOr lh~..:

+ 28. Show tluu. wh<:n J' (.t)

25. U:~:e lhc technique of E:c:crcisc 24 to appro>. imatc

dclinile imc&ml frum

27. Repeal

runction tabu hllcd below.

= 1/ t . a~xl usc th e uupci.Oidul rule 300 Sinlpson~ rule with 10

f.
ii 'if

1,.

J.S

2.2ll7 0.)9S 0 O.lll 0.310 0.)3-1 1.$1 1) 0.123 0.021 - 0.())7

1,0
-<).~,

Jl.

(!l r

d .t

30.

32

1""''

co~.r tl.t

J .r +2

dx

SUMMARY

Anti tliffcrentimion i~a rnr ll'l0r0t.:OI1lplic:Hcc..l pr occss thun dinercnti~ci on~ i1is not possiblc toshlte
a set of rules and formulas 1ha1will suflice for mos1 func1ions. Ce11ainly, 1here m-e imegration
formula!) that we must learn. but most or those given in this chaplcr ha,c been dift'eremiation
formulas listetJ in Chaptcr3 written in tcrnlS orintegn:lls nu her thanderhacives. \Ve have~(fesscd
1he impor1ance of knowing 1hesc obvious imcgra1ion fonnulas. b<u beyond 1his. it becomes no
organi1..3tional problem - organizing a difficult integral into a form that uti lizes these simple
fonnulo.s. The three most importnnt lhnique.s for doi ng chis are substitution..~. integmtion by

p:arL'i. 4111cJ paniul fmt:tions.


1l1crc is a "uic1y of subs1inuions for 1hc eva luation of indefin ite integmls. many suggested
by the form of the integrand. For t:xample, a tc.rm ir1 x lin (n ao i11teger) suggests the subStitution
u = .x l/ 11 , 1'ri onomctric substitutions are most important. They eliminate .square roots of the
form al blx 2 and Jb 2x Z- tl 2 and !he general roo1.Jt1xl + bx + c.
Lnccg ratio n by parts is a pO\'c_rful _integration technique. It is used to evaluate ancidcrivative-s of trans<.ende-ntal full(tions that are mullipl ie.d by powers of .r, it leads to the method of
imcgnuion by reproduction. and it is used 10 develop reduction fonnulas.

1l1e method of partial fr.actions decomposes complicated rational function..o,; into simple (r.tC
tions that arc e jther inunediately integrable or .an1enable to vthcr rm:thuds. such as trigonometric
substitutions.
Even wi1h !he techniques !hat we have s tudied and tables, there are many functions that
either cannot be antidilfereotiated at all or can be antidiffereotiated only with extreme diftlculty.
To approximate defi nite integrals of such functions. numerical techniques such as Lhe rectangular
rule. the trapezoidal rule. and Simpson's rule are essential.

KEY T ERMS

Jn reviewing this chapter, you should be able co defi ne or discuss the following key temt~, :
Integration by parts
fmegralion by reproduction

fntegrotionb)'(>arts formula
Trigonometric substitutions
Rec1nngulnr rule
Simpwn, nolo

Pani:al fraction decomposition


Trapezoilial rul<

R EVIEW
EXERCISES

In Exercises I- SO cwahll.ltC the inddinile intcgi'UI.

I.

j ../2 - xtlx

27.
29.

I
1. (
d ..
(X+ 3)1

"'+
3. ! - d3. r
.r

4.

j -x'-+d3 x

31.

I ,
I
I x../in.t
,

x' + 4x cosl.\

Jx'+
Jx
--d
x' +I

' Jsin! x cos x Jx


3

9.

11 .

13.

I S.

/~an' (2x) dx

12.

14.

tlx
.r
.l.r 2 + 4

16.

x ' In d.r

37.

x 1 + 2x- 3
' d

,/X+S x

43.

l l - rl.t

dx

45.
X

(x' + 1) '

tl.r

18.

j (.t'+x ' 1) ' d.r

j x+ 1 dx

1.0.

2.1.

/ cos-.. dx

17.

19.

21.

, x' )' dx

(x + 1

x3 - 4x

x'

d<

(I+ J.r'J .

;::!'Y
rc+

/ Jx1 ..- 4 tlx

dx

25.

J~in -t cos2t d.\

24.

je

26.

3"' cos 2x dx

.Jx' + 4x

dx
5

ol -

.\': +.,-

x
I

.rl

+ 4,\"

.\'~

+ xJ

- 4

dx

-I -d.<'

J ../16

J.r + x'

d.r

.x zT,ut- .( (/.'(

Jh
-.roxdx
42. J
I dx
.r(9 + x')

44.

.r 3 + 5.r 2 + 4,r '

j .r l xI '.r+2
dx
l I 4.r

46.

.\'1+'2
dx
.r 3 + 4.r 2 + 4x

j Jx' + lx + 4 dx
x' + x' +
4A'

47. j xsin- 1 xdx

4H.

J./cot.xes<:' Adx

49. j ln (./X+ l)r/x

so.

dx
, (4.r - x 2)'12

( I

iii

52.

1'

il

54.

[_, I
- - dx
..-

si11 x cos sx dx

-6x
- . tlx

ln Exercises Sl-55 usc lhc lrupczoidal rule and Simpson's rule with
I0 oqu~;~ l p~n1 ititns to apJli'O~imate the deli nile intcgrnl.

.f

40.

51. ~ -sin.\' tl.t


X

23.

38.

30.

36.

39.

r -,fi

I
I

j .r3 + 3.r 2x'+ 3x + 1 dx


x'
41 . j ---tl.r
1 + 4.r

dx

/ x'J4-.< tlx

32.

:.<."<:

8. / xsin .\"tlx
10.

J ../4 I J.r! dx

I
J

dx

. Jx+3

2~.

J
34. J -

- - - tlx

j +cos!
~in x
d
x .x
35. j x 2 (3.r ) d.

.r+ l

6. (

tiA

I - sin 2x

JJ.

s.

s dx

I 53.

d.r

li ss.

_ lnx

, (l+x')'

clx

J,in.tcl.r

I+ ,~

540

Chapte r 8

Ta.:h,iques of lnteg,ration

l n Exercises 56-71 evaluate tbe indefinite integral.

56.

* 58.
*

60.

62.

64.

f I v'X
f +
f
J ../X
J
x / 3 _

X'1'

16

dx

* 59.

dx

l
(3x - x 2)3/ 2

sin

57.

dx

x cosx sin 3x dx

*
*
*

61.

63.

65.

J
1
1
J
f + + +I

66.

In (I +x ) dx

* 68.

esc' xdx

10.

72.

I
x3.Jx2
-

9 dx

3x

x(x 2

+ ccls2x

sin' x >< 3x dx

dx

:w::

x 2 Si11 - 1 xd:r

~69.

* 7il.

f
f
f

x 4 + 3x 2
x2 -

3x

1x

+5

+7

dx

dx
(
?
.,./
?
x.l .4 - -~- )"' ~

: d.r

x+Jx i+ 4

(a) Use, the substitution z 2 = (1 + x) / 0 - x) to si11ow du~t

l - x

1) 2

* 67.

dx

l+.x
- - dx

sin (lnx) dx

J
J
JJ1 -

2Tan-

1+

-- - , 1 - x2

1- x

+ C.

(b) Evaluate the integral in part (a) by multiplying numeratm


and denominator of the integrand by .J l + .:r . Verify that
this answer is the same as lhat in part (a).

CHAPTER

Parametric Equations and Polar


Coordinates

The figure below shows a mechanism in which rod A 8 , pinned"' A , ro~ates counterclocl:wise
about A. Slider B moves along rod C D.causing it to rotate also. End of rod C E is confined
10 slide along a horirontal ljnc. lengths of the members A 8 , CD. and C E arc as shown.

Application Preview

- - d>l, - -

THE PROBLEM

~ the solution.l

Detcmtinc the path followed by jOint C. (See Example 9.6 on page 54 8

In this chapter we introduce parametric represemations for curves and polar coordinates.
Parametric repre!tenBHion~ of curves offer an altemati\'e to the explitil and implicit forms used

in Chapters 1- 8. Polarcoordiruues provide an ahern:1tive way to identify points in a plane. They

arc more efficient than Cartesian coordinates in many applications. In pan icular, we shall sec
how they provide a unified approach to conic sections.

19.1 Parametric Equations

A curve is defined C>plicidy by equations of the fonu y


f(x ) or x = g(y) . and implici tly
by <ln equation F (x, )') 0. A third method is described in the followi ng defini tion.

DEFINITION 9.1

A curve is said to be ddined par.tmetrically if it is given in the rorm


x

x(t),

y(t),

(9. 1)

541

542

Ch;tptet 9

Pttramc1rk EqlUitions and Pol:trCoordinatcs

VariabJe 1 is c.aiJed a paramewr ~ it is a con necting li.Jl.k between x and y. Each value oft io
the int.c rval a ;=". I ;=". tJ is substituted into the parametric equation s x = x (f ) and y = y(l)
in 9.1, and the pair (x, y) = (x(f), y(t)) rep resents a point on (he curve. \Vhen the interval
for 1 is unspecified, ,..,.e assume that it consists of all values for which both x{l ) and )'(I ) are
defi ned.
Gra.phing calcuhuors :md compu1ers arc adep1 at plotting curves defined parametrically.
Given the functions .r(l) and y(1), and the in te rval a ~ 1 ~ {!,they plot sufficiently many
points (x(/}. y (l}) and join them with ~1raight lines to give nn excellent rendering of the curve.

I EXAMPLE 9 , 1

Plot the curve


X=

y = t

2- I,

+ 5,

-6 ;=".

I ~

5.

What does it look like? Verify this analytically.

SOLUTION '111e p lot in Figure 9.1 appeac.> to be a straiglu line joining lhc points (8, - I) and
(-3, 10) . TI1is is easily verified by sohing .1.
y = I+ 5:
y = (2 - x)

=2 -

+5 =

=2 -

for f

-x

x and suhstiuni ng into

+ 7.

y
tO

6
4

-1

\Vhen x(t ) and y(l} in equatio ns 9.1 are linear fu nctions, the curve is always a str.tight line or
I ine .gment. J a altl fj are finite. they define ends of the line segment: i rthey a re nol fini te. the
whole line results. Line segments can also be delined by no nlinear futlctions (see Exercise 31 ).

I EXAMPLE 9 .2
Plot the curve
X

2
1 -

21

)' = 3 -

+ 4,

- 2< I

21 ,

2.

What docs it look like? Verify thi~ analyt icall)'.


SOLUTION The plot in Figure 9.2 appears to be a pambolajoining the points (1 2, 7) and
(4, - 1). To veri fy this, we solve the second equation for 1 = (3 - y)/2, and substitute into
the first:
X

(~)
2

(~)
+4 =
2

13

-2 + -4 .

c~ it
)'

s
4

3
2
o r---~2~~~--~
6----S~--~,o~--~
,2~x
- I

I EXAMPLE 9 . 3

Dis.:uss the curve defined pammetrically by

.< = 2oosll.

SOLU110N

y = 2sinll,

0 :: 9 < 2;r.

If the given equations arc squared and added. the result is

an implicil ddinilion of the curve:. TI1c give11 equations lhcrcforc define lhc circle parameui
cally. Seldom is it possible to give a geometric interpretation for parameter r in equations 9. 1.
Parame1er 8 for a circle is an exception: it can be imerpreted as the an1e in Fi&ure 9.3. Values
0 :: II < 2;r describe the complete circle in o counterclockwise direction begi nning ot (2. 0).
Additional v-Jiues of 8 duplicate existing points. For example. using 0 :: 8 < 4Jr traces the
circle twice. Specifying an intcrvnl of length less than 211' describes part of the cirelc. For
inqance. 0 ~ 8 ::S 11 gives rhe upper semicircle y
J4- xl .

(x. y)

- I

-I

I E XAMPLE 9 .4

Find an implicit definition for the curve

.\" =, _,J,
and plot the curve.
SOl.l.ilON

By adding and subtmc."li


.. ng the given equations. we have

x+y=2t

It follows from the fir:,t Of ChC.SC that I


(X
y)j2, fUld \ VhCn thiS is s ubs tlttllcd into thC:
second. we obt.a:lill an impl icil defi ni tion of the: curve

J -

.r =

(X

+ )')3
4

UniOrtunotcly, ~s is always the c:ase when an eJectronic <..levice is used to plot parametric equa
tions, Figure 9.4a docs r101 iru.lic~HI~ w hich vul ucs oft give which points o n the curve. \Vc can
remedy chis by rclatitlg graphs of x (I) and y(t) it\ Figures 9.4b lltld c to the c urve i 11 Figure Y.4a.
f or example, beginning with t
0 . Figl U'CS 9.4b Hlld c give .t: = 0 andy = 0 . and therefore
rhc point (0, 0) in Figure 9.4a. As t increases. from 1 0 10 t
I. values of x illcrca'\e from

010 a maxi mum value 2/(3./:3) alI = Jf./:3, and lhen docn:ase 10 0. Simullarleously, value,;
of y increase .steadi ly from 0 to 2. l11is gives the first qumJrnnt pan oft he curve in Figure 9.4a.
As 1 increases beyond I. values of x decrease through negative numbers while y continues
to increase. 'l'l lis is refleeted in that pan of t!he graph in Figure 9.4a in the second quadrant.
Becduse x = t - t 3 a ndy = I + t 3 arc odd functions. repJacing t by - t reverses the s igns
or x and)' . This means thaL<.:orresponding to c;.lch poim (x . y) on the graph for which 1 > 0,
we I'UUS( have the point( - .t, - y ) correspond ing to _, . This sive~ third - and founh-quadrant

1>ans of the curve in r.igure 9.4t1.


1D1,inJ ~
p!CJC of ;r = f

- t\

I!C "lE:m Pluc

LIJ.!l!ll -:::ml Plor of

P.anuncuic
= f + ,~

I,SX

of

,l'= t+t'

.l = l - /)

- 3 -2

-J

-2

It is worthwhile nocing that wcrecakulul ors an<.Vort:omputcrs not programmed to do panunetric


ploL,, we would draw J)llnunelric curves precisely liS ouliincd in lhis example. Piece the curve
IOelherfrom separa1e graphs of x(t) and y(t). This woold belhecase if .r(l) or y(t). orbo1h.
contained unspecified parc.lrneCel's. The following example &san illustration.

I E XAMPL E 9 .6

Dr..tw Lhe cur ve defined paramelric.ally by

x = acost.
where a > 0 and b > 0 are constants.

y = bsin2t.

9.1 PJJ-amelric Equalions


ljUjiiI4:H.i.A

J)arametric plol of x

= 2cost. y

= sin21

liilCJil;l#&iffi

545

Graph of x = a cc)$1 , y = b sin2t

"
0.5

-2

- I

- 0.5
- I

SOI .UTION Because a cos 1 and b sin 21 are peri.odic w ith periods 21T and IT. respectively, it
follows that a ll values o f x and y are obta ined for 0 :::; t :::; 21T . We begin by plouing 1he curve

using .specific values for a and b , say a

= 2 and b =

I. The result is shown in Figure 9.5a.

'lbe required c w"Ve can be obtained by changing scales on the x - andy-axes (Figure 9.5b).
To conlirm this, we draw the curve using the technique of Example 9 .4. We drw grphs
of x = a cos I and y = b si11 21 (Figures 9.6a and b) . Value I = 0 gives the point (a. 0) in

Figure 9.6c. As

increases from

= 0 to 1 =

1C /2,

values of x decrease from a to 0 and y

increases from 0 to b, and then decreases from b to 0. This gives the 1\rst-quadrant pan of the
curve in Figure 9.6c. As 1 increases fi'Om IT/ 2 to IT. values o f x decrease from 0 to - a and y
decreases from 0 to -b. and the n increases from -b to 0. T his adds the third-quadmnt part of

the curve in Figure 9.6d. Continuation leads to Lhe full curve in Figure 9.5b.
Mjlclii;I4:Hifh

Graph of x =a cost

Mijl#jii;IK&Jlj

Graph of y

= bsin21

2n-

Mijlclil;l#!l~

2rr

Fi.rslquad:rnor gmph of x =a cos t .

= b sin 2t

.t

i::UcJII;IiJICiiif!)
Fi.rsl and rhird-quadnun gr:mh of
= a cost . y b sin21

)'

)'

- (/

(I

-b

Mii!lc:lll;;l*4

Paramctrjc equations arc frequently used in l)rob1cms concerning the motion of objects in a

Pa1h of a

plaoe or in space. For example, suppose a stone is thrown horizootally over the edge of a cliff
100m above a river (Figure 9.7). If the initial speed or the stone is 30 rnls, the path followed by
the s tone is described parametrically by

',

'
100

\
''
\

y = [()() - 4.9051 2 ,

.r = 301,

'

'

where r is time in seconds and x and y arc in metres. For these equations, I has been chosen
equal to zero at the instant of projection. and r = .JJ00/ 4.905 is the time at which the sto ne
strikes the. river.
\Vhen pal'"Jmeter t is eJlminated from this paramettic .representaljon of tbe trajectory, the
resuhing equa1ion is

y = 100- 4.905 (; ) ' = 100- 0.00545x2 .


0
It clearly indicates that the stone follows a parabolic pa1h. However, if we we-re to discu..ss 1he
motion of the stone as regards:, say, velocity and acceleraLion. we would find it necessary to
return lO the pammetric represenlaLion lhat describes lhe molion in lhe horizontal and ''enical

directions.
We say that equations Y. l d efine y parametrically as a function of x if the curve so deli ned
repre.."'enbi a func1 ion; 1ha1 is. if every \ler1ic-al line 1hat imersects I he curve does so exactly once.
The pantmctric c.qualions in Example 9.1 therefore define a function parametrically. but in

Examples 9.2-').5 they do noL However, each of the curves io these latter examples call be
divided imo subcurves thm do represent funcLions. For instance, in Example 9.4, each of the
subcurves
I - 1 3

)' =I + 1 3 ,

I 5 - lf ./3;
t -1 3, )'=I +t\ -J f ..{j ~ I ~ 1/./3;
X = I - 13, )' = I+ t 1 ,
I ?_ Ij-../3;
defines a function, and docs so parometrically (Figures 9.8): together all three curves make up
X =

X =

the original curve.


.M:IIfliiJIDrn.,

M:lldii;J ..:lrnll

M:llfiii;J'41t:W

)'

0.5

-2

1.0

.r

0.4

-{).4

-J
- 1.0

-4

- t

- 0.5

When equations 9.1do define. y parnmctricalJy as a function of x ~it is straightforward to


find the deriv-~ti,e of the function. If we denote this fu nction by)' = f (x), then the chain n1Ie
applied to y = f(x) . .r = .t(l) gives

dy
f>r(wided I hen that t!x j tll

:F

dydx

d1
dx d1
0 . we may <()Jve for

dy
dx -

ely
dt
dx
dt

(9.2)

This is ca.IJed the parametric l"ttle for differemimion of a function defined parame.tricaJiy by
equations 9.1; it defines the derivative \If )' with respect tOX in tenns of derivatives oft he given
functions x(l ) and y(1) with respect to 1.
For example., the curve in Figure 9.9 is defined parametrically by

x (l) =

12

2
- ,....--t(t2 + I )

v(l) ,
-

+ 1'

It <.an be subdivided into two pruts, one corresponding to vaJues of I > 0 and the other to
value._.;; or I < 0 and each defines )'as a funclilln of .t'. The derivative of either function is
1

dy

dx

dy

-2(311

dt
= d.r
=

(tl

dt

I)

+ 1)1

-21
(1 2 + 1)2

3t 2 + I

rJ

Neither function hus a rclntivc maximum or minimum. and chis is consistent with the facr that
d yfdx never vanishes. The derh!lltive is undetincd fort = 0, as is y. \Ve c:m also calcuhne
d 2 y/d.r 2 in spite of the fact that d yfdx is in terms of 1 rather than x. Once agai n it is the
chuin rule thm comes to the rc.~cue:

= d (dy )
dx

dx

d ( d y ) dt

= dt dx dx =

d (dy )
dt d.r

dx
dl

(r '

+ 1)1
-2f

This derivati ve isposicive for t > 0 and nega1 ive l'or r < 0. agreeing with che fact that the curve
in Figure 9.9 is com.:avc upward when t > 0 and com.:.uve downward when/ < 0.

Milfdll;l41h

Plot of X = {f :

+ 1)- 1,

)'

= "211(1 2 + l } r

)'

7.5

5.0
2.5
- 2.5

- 5.0
-7.5

1>0

~0.2~~~
'(:OlR'--.;x

I EXAMPLE 9 .6
Appli c11tion Preview
Revisited

Find Plll11metric equmions for the p.1th followed by joint C of the mechanism in the Applic.nion
Preview. Plot the path fM I,
1/ 2 111./2 = 2 m. and d
I m.

J!IW 81

--<1>1, -

~OU.7110N Coordi1l8les of B in 1cnns of the angle of roUition 0 from the positive


(Figure 9.1 Ol are

xa

I, cosO.

.VB

x-a~is

= 11sinO.

Thec.quation of line B D is)' = m(A - d). and ,j nce 8 is unthe line,


/ 1 sin

Reeau~ poi11t C(x,.

9 = m(l 1 cos 9 - d)

/ 1 sin 9

m=

I,

coso- d.

y, ) lies on thi< line. il follow<o 1ha1


/ 1 sin9

Yc

= I 1cos (J

Funhennon:, the leng1h or CD is /z

d).

- t / '' ' -

w 1h11
?

(.r, - d)

+ y,l = 122

29
/ I2 Mil

.,

(r -d)' + (1 cos ll -d)l(x, -d)

=I, .

This equalion c.on be wh cd for x,:

.~,

= d

l1(tl - I, cos II)


,jdl

+ If -

2d/ 1 cosll

Siw:e -'r is ulwys Je..s lhun d. wechoo,e

The y-coordina~e of C h now

i _

li(d- I, cos8}2
d 2 +I~ - 2dl, cos II

1,121>in81

-.j~J"""2- =+==i;;l~=-==2<=ll=t=co=s=o

We can sec that ) (- is positive when

8 is an ang.lc in the first or second

QU(ld r<lll t,

and Yt is

negative when 9 is in the third or fo urth quatlmnt. Hence


M)lciii;I!JLMJ

Motion of C

Yc

i tl 2sin 0

j J l +I; - 2d/ 1 cosO

When/ 1 = J /2./1 = 2. and d = I. parametric eq uations for the path followed by C are
Xc

2(1 - ( 1/2) cos i:l]


2(2 - ~osO)
= I - -;,;=i=;=T.f==~~:;=;:::::7j = I .
.jl + 1/ 4 2( 1/2)cose
../5 - 4 cos0

)'

--

-t

(I /2)(2) sit> 0

.JI

1\ pi~ of the curve for

........

+ 1/4

2 sin 0

2(1 /2)cos0-

../5

4cosil'

0 :::; () :::; 211 is shown in Fig ure 9. I I.

I EXAMPLE 9 .7
The tire o f a car rolling a long the .faxis witho ut slipping picks Ul> a stone a t the origin. The
path followed by the stone is c alled a cycloid. Show that parametric equations fo r the cycloid
in tel'nts of the angle I) through "hich the tirc has rotated sittce picking up the stone are

x
where

= R(8- sin 8),

y = R(l - cos8) ,

R is the radius o f the tire. Plot the cycloid for 0 :::; 8 :::; 41f. In wh<tt direc tion is the

Slone travelling when it meets the road?


SOLUTIO'< The x coordinate of P in ~lgurc 9. I 2a isequal to II 0 B 11 minus 11 PC 11. Since
length I P Bll nlong the c ircle is equal to m0 B II and 11 PC]] = II P A II si n 8
R sin 9. it
follows tha t

x = R8- R sin 8

= R(8- sin O) .

Furthermore.

)' = I ABU -IIACIJ = R - R co>8 = R(l - co>9 ).


(}e\.-eluJ)4nc:nt of PJlh folluwed b)' stox

e~ught iJl b'Cad

of a

Tire

C:

0
:::&1~

PJttl (\)JIO'AW by SU.l.'1!' for two revohcictg cf tirt

2fl

2nR

4;rR X

ti.l'~

The plot in Figure 9.12b for 0 ~ 9 ~ llr shows the P<llh of the stone for t\\0 rc\'OIUiions
of the tire. (h '-''8< ploncd for R = I and scales were then chnnged fi-om I to R.)
The slope of the cycloid is given by
dy

d.\'

R(sin 8)

dyjd()

= cl.r fd8

R( I -cosO)

sin()
---co
- s..,
(!'

,...I

h is undefined "hen e = 2mr . V31ucsof9 at which the Stone meets the road. Using L'H<lpilill's
rule, we calculate

lim

8-2n.'T

~in9

--:--

1 -cosO

coo9
=
sin 9

lim

- oo

sin/l

and

I-

cosO

00.

Tllel>e show that tl1c stone is n1oving vertically clownwurd as it n1ccts the road, and then 'enicully
upiV'drd as i1leavcs the road .

........
Many of !he applications of imegrntion in Chapter 7 can be adapted 10 curves defined par.1111elrically. \Ve illu<trate in the next example and the following coMuhntion project

I EXAMPLE 9 .8
Fi nd the area bounded by the curve in Example 9.5.
~OI.UTIO'\ Because of the <ymmct~ of the curve. the area is four times that in the first
quadrant (Figure CJ. t 3). If we use venical rec~a.ngles of area )' dx. then the required area tS

A= 4

[, .

ydx - 4

10

bsin2r(- .asint)dl - <lab 1 ~1 sint sin2tdl .

:rfl

With identity 1.48a,

A = 4ab

/1 1

1
0

-(- cos3t
2

+ cost) dt

lnlegrals 7. 17 in Scc1 ion 7.3 d~line the length of a curve. \Vecxpress them in a slight ly diiTercn1
rorm,

i"

J(dx)l

+ (dy) 2 ,

't~t'Ve defined

Length t>i a
panl,netl'iCillly

where A and B are !he ini1ial and final poitus on the curve (Figure 9. 1~). When the cune is
defined paramenically by 9.1 . chis inregrdl becomes

8(b. dl

l(dx)
- + (dy)
- dt.
2

L
Ma. c)

=[

fi
Ct

dt

1\I.Jl

d!

Verification of rhis rormula is lcfr to Exercise 49. We use ir in rhe following projcc1.
.(

In a roll ing mill, the right circle in Figure 9.15a represcms a cylinder of radius R that rolls
around a second cylinder or the same radius. Auached to poim P on the end of rhc nghr
cylinder is pan of a linkage P2. Many question~ are being asked about I he motion of
point P. one of which is the disraoce that ittrovels as the right cylinder rolls once around
the left cylinder. These questions can be answered if the equation for rhe curve followed
by P can be found. Ourtask is to lind i1.

D1stancc travelLed b)' a pomc on une cin:le as it rclls around MOther cirtle

r.
(

,I

.I

(>=angle IVQP

z
'110~

Suppo<;c we let 0 be the nnglc though which the centre of the right cylinder
rotnlesn~ shown in Figure9. I5b. Atthi< position. we can say thm the x -coordinate of P L<

SOl l

X=

nouu + WVIJ = Rcos(J + II PQUsin,P.

Angles 0. ~. ancJ p are related by the equations

(lf/2 - fJ)

+ t/> + p = r..

<ll1d 8 + 2p =If.

\V1,cn thc.e are soh eel for p und results urc equated, we ubtuin t/> = 3Q f 2. Furthermore, if angle II is bise.:tecJ m S 10 di,'ide triangle P QS into rwo congruent tight-angled
triangles, we <ee th.u I P Ql /2 = R sin (IJ/2). Hence,
X

= R CO; 8

+ 2 R >in (8/2)>in (38/1.)

= R(cos 8 -

cos28

+ cos8) = R(2cos8- co;28).

The .)'-coordinate of P ;,

y = IIUQIJ- UQWIJ = RsiniJ- UPQJI<-os


= R;in8 - 2R>in(8/2)co>(J8/1.)
= R(sinB +;inti- sin28)

R(2 .sin9- ; in 29).

\Vc have \hercforc found parametric equationsfor thepath of P,

x=

R t2co~e-

cos28), y = R(2sin 9 - sin 28).

According t() fom1ula9J,the distance that Ptravels for one complete rotation of the
cy\inder is

J: )R - 1sin6 +2sin 20) +R (2cosB- 2cos 26) dB


1(

L "'1

= 4.Ji.R

J: )1 -(cosOm26t sinBsin26)dB = 4Ji.R /,' ./1 - eos8d6

=~JiR ~ l - ll - 2sin2 (8/2)ld8 =S Ri" sin(8/2)d8 = 16R .

- (l+u)''l = (l-u)

19.
\ , X =1

'l. x

+I .

X ::

--

, )'

l -11

y :=. 31 - I

=11 t )I t

~.

) =I

10. .t ==

- I

~ ..~=I +1cosl, y ::::.1+1sint,

+ COO I, ) = - I - \\i11 I.
G. X = 1 + \!l. ~ = I - I I I

~. X ::::. \

O~ t ~ iT

J_,2+ 31 + 5,

%. ll=l+3c<:>\\l , )' =-l+hint, ()~I ~If

y "' t + t

I
I - - ,
I

\'
.

t
I - I

21 1he poinl corresponding 10 1 = 4.


11. Find poini(S) on llx: cum:
13

= - - 31,
3

t - 1 ,
-\- \ .

where 1be slope of the Iangen\ line 10 the cUI'\" is equal to I.

y : 1> +)

- n/1 !:II< Jr / 2. 9~0

\1 . x='loolfl. y = 1sin1 0.

\\\ e~c<eise I :1-7.() asRume \hal y is 1\efmcd parametrically 8S a func


\~ ()("raM{\<><) d~ Jd~.

n. ,. = ,, + 31
\4. X = -

"-

IS. x ~,>
\fl. X

.~

~ . .x = - l+si\\1, y = - 1 - }co~l, ()!1 ~n/2

I\, X = 11

)' = t l + 21 - 5

1\. Find cqu:uions ror 1hc wngcm and nomutl Jines 10 the curve

() 5 I 5 7r / 4

1. x :'least, y =~sinl. O S I <l.'t

x=

0 ~I :S ln

~. x=-1+4cMt. y=3+4sint ,

1\l.

1+ 11

1. ~ ~ ,, -

, . \' =

I, }

-, UI

--

x = l'+ -I .

y= l - I

= ,fl=1 . .\' = Ji"'+i'


* 2.5. X =2u ,5, )' = 7 - 1411

24. X

" '26. x c

.,z

1- 2u

+ 3. )' 1v - 4

y- hl 4- 3

~~ ~',

17. x ; s ' 2 - J 2' l . y=s2+2s

18. X = (21 + 3)'' , Y =

lJ.

+ l)J\i"=l.

In E>.cnc~ 23-26 assume I hat y is Jc~ncJ pan~ metrically ns o func


1
tion of x. 2nd 6rd d~fd:r arYl d yfdx .

f+6

l7. \.) llla\:.l dirf..:te'lQ.: b<t\II'CCR lhc IWO CVI'IQ

) = 2t

1 -

and

= CO~ I.

)":

2~

1 - f?

28. What curn. ~~ dc.-.cribcd by the par~unc:tric cttuation'

= lt+arosll.

= k+bsinll.

0 ~ II< ltr?

9. 1 Pa.ramelric Equalions

*
*

29. Find pwamctric e<Juations for a circle with centre (II: k) and radius,..

= x , + (x, -

ln E;(crciscs 45-47 find Ibe le.ng lh C)f the curve.

= )'o + (}'2- )'o)t.


* 31. Show that the eq uations x = 2 sin1 1. y = 4 co..c; 2 1. which are not *
)'

x 0)1,

linea1. define a straight-line segment

32. Draw tJte IOI!owing curves and determine whetJter they are related:

(a)

X=

secB,

(b) x = cosh 4t ,

x=

(c)

)' = tanB, -rr/2 <


y = sinJul>

HI + ~).

y =

1 ?;

x+ 1
x-2

*
*

35. x 2

34.

+ y 2 + 2x -

4y =

* 36.

+ y 1 +xy =

45. x = 3+ 4 cost, y = -2 +4sinl t

0:;: 1 < 2rr

46. x= ~-'sinl,

~ 1

y=e-'cost ,

O ;S t

47. X=1+ln 1, y= l - lnl.

I :;:1 :;:2

48. Set up. but do not evaluate. a definite integral representing tJte
length or the ellipse

x = ti cosf:J,

*
*

In Exercises 33- 36 find parametric equations for the curve.

33. )' = - -

e < rr/2

Ht-D,

44. Thccycloidx = R(6-sinB).y= R( l -cosB).O :::; 6:::; 2rr


of :\ample 9.7 and the .t-axis

30. Sho w that che straight line through cwo poincs P1(x .. )' 1) and
P2 (x:. y 2) has pw:amctric c..-quati(lns
x

553

5y 2

4 - x 2 + 2y 2 = 0

J = bsin Ot

0 !:

8 < 271'.

49. Verify formu la 9.3 fot the length of a cmve.


50. Tite equalions x = 12 + 21 - I, y = t + 5. 1 < t < 4 define
a curve paJ'atnetl'ically. Find JXtrametric equations that describe Lhis
curve but have values of tJte parameter in tJte intervals (a) 0 5 1 S 3
and (b)O:::;t :S I.

* SJ.

Suppose x(l) and y(t) in equatjons 9. 1 arc continuous on a :;:


IJ and have derivacives o n a < l < ~ , and that x' (I) ::/: 0 on
a < l < IJ . Show that Cauchy's generalized mean vall1C theorem
(T11corcm 3.18) implies that cherc exists a number c between a and~
such that
y(fi) - )'(<>')
y'(c)
f :;:

37. T\vo pankles move along str.1.ight lines t 1 and


metrically by

1 ;

= 1-

f, :

= 41- 5, y = 21-

1,

)'

I ~

1,

I,

t 2 defined para-

0;

x(fi)- .<(<>')

I ~ 0;

x'(c)

Interpret this resuJt gcomctricaiJy.


where 1 is time. When are they closest togetJter?

* 38.

lf x

.\'(1) and )'

= y(1) define)' as !'t funLIiC)n of x. show lb!ll


2

dxd y
-til tft2

52. A p<'trticle lrnvd s m:ou nd the tirde x 2 + .Y1 4 cou ntertlockwisc


at constant Sf>eed, making 2 !'evolutions each SCCt)od. If the JXI.tticle
0, fi nd J)arame.tric equations ro1 it'>
statts at point (2, 0) at time 1

dyd 2x

pOSiliOn in lennS o r I .

til tft2

e~r

53.

(a) Pind lhC :irea under o ne arth o r l.hC cycloid in Ex!'IUJplc 9.7.
(b) Find 1he length of one arch of lbt:: cycloid. W h:u docs it
reJWCSCnt physically?

54. Draw or piOl the strophoid

Jo ..xercises 39-42 fi nd lhe area bounded by lhe curve.


39. 'Oteellipse x =

*
*

tl

cost , y = bsin l. 0

t ,5 2Jt

y=

40. Tite astroid x = t':Os3 t . y = sin3 t . 0 .5 t :;: 2Jr


41. The deltoid x = 2cosl

+ cos21. y

= 2sinl- sin2f. 0:;:

I:;:

2rr

i*

= t l cos 1, )' = b sin 21 of Example 9.5

t ')

+ ,, ,

55. PIOI lbe p<Hh l0UC)wcd by j oinL

in 1.he rnech~mism of Ex~Hoplc

9.6when l 1 = 1,1, = 3,andd = 1/2.

i*

43. 11Je curve x

and fi nd points at which il'O tangent line is h01'i1..ontal.

42. 11tedroplet x = 2cost- sio2t. y = sin t , 0 ,5 t :;: 2Jr


In E.xe.J'Ci..c>es 43-44 fiod tlte volume oftJ\e solid of revolulio n when tlte
area bOtutded by dte curve(s) is rotated about the x -axis.

I (I -

56.

(a) Find the x -coordjo.ate of s lide1

E in ExaOlJ)Ie 9.6.

(b) Pl(l4 .\' as !I function of 8 when l 1


1/2 rn, l1
2 (O,
/3
4 rn, <I
I m. and )'
2 (O . From the graph
es limale Ibe length of Ihe Stroke C)( E (l.he lcng lh Oflbt:: ljnc
scgmcnl aJong which E moves).

554

Chapter 9

P~rnmetfic

F.quatioos ~nd Pol~r Coordinates

ii* 57.

End A of shaft:\ B with length 4 em (figure below). tnO,cs around


the circle of radius I em counterclockwise at 60 rpm. As it docs so.
a slider at end 8 mol'es back and forth along the line y = -3 em.
Assume that there is no binding when A is at positions (0, I) so thai
the slider moves between quadrants three and four at these times.

Slone

y
X

* 59. A string is wound around the circle x 2 + y2 = r1 in the figure


below wilh one end at (r, O). u the srri ng is unwound while being held
Wut. the curc thm the end traces is called an imolut~ of the circle.
Show that parametric C<Juations lor the jnvolule in tenns of Ihe itnglc 0

shown are

y=-3

.r = r cr:;s B + r8sint9,

.4 struts at position (I. 0) aJ tiuiC I = 0, fiml


and plot a formula for the x -coordinate of 8 in tcm1s of 1
for two revolutions of II. Hint: It is ncccssar)' to piece a

,r = rsinB - r 8cos8.

(a) Given tbat

func.tion together.
(b)

Estimate maximum lcfl nn<lriglu positionsofthcslidcrfrom

,.

the gmpb in part (a), and then find these posttions exactly
by using the facti hat \'elocity will be lcro there.

\
) String

(c) Is tliC velocity function continuous as 1he slider passes he


tween quadmnts lhrcc and four?

.r

(d) Usc a gnlph of Ihe velocity function to estimmc maximum


speed of lhe slider.
(c) Use 11 grnph of the ac.:clcr.ttion func1ion 10 cs1ima1c maxi

In Exercises 60-62 lry to druw 1hc curve and tltcn pl01 it.

lllUill tD(I) i.

" 611. x = cos b. y = sin39 (cu" c of Ussajous)


58. lf the slooc ill E.xample9.7 is embedded in lhcsidcoflhetirerather
than the tread. its path iscallod a rrcx:hoid (figure following). Show d1at
if the distance from the ccnlrc of the lire to Ill.: slonc is b. parametric

equations for the trochoid arc

= RIJ -

bsinb.

31
312
61. x = - .
l' = - I + I3 .
I +/'

* 62.
y

=R-

bros(J.

x = cos 3 0. )'

= sin 3 B

(folium of Oesca1cs)
(astroidor hypocycloidoffourcusp4;)

* 63. A C'OW is auachcd 10 tJte side of a silo of mdius5 m will>a rope of


I0 m. Delennine the g:ra;;ing area of the CO\If.

19.2 Polar Coordinates

MJICJII.(.jbJW 1 {\)lac toot


'-~il'l<'l~ of a poinl

0 ~,
Pole

Pol:~r

axjs

In 1his scc1ioo we: introduce polar ooordinatc:;, an a lu;mativc coordinate sysccm for chc plane.
Many problems thai ha'le 'ornplex oolutions using Cartesian coordi n:ues hcc<>mc much >implcr
in polar coordina1es.
Polar coor dinates arc t.lefi ned by choosing a poi111 0 in the p hmc called the pole and a
half-line originating a1 0 called lhc polar axis (Figure \1.16). If P is a poim in lhe plane, we
join 0 and P . Tile firSt polar coordinate of P. denoted by r . is the length of line segme111
0 P. The olher polar coordinate is the angle 8 through which the polar axis musl be rolalcd
to coincide with line segment 0 P. Counterclockwise rotations are regarded as positive. and
clockwise rotations as negat ive. ln Figure 9.17. position OQ is reached through a posit ive
rolation of 1C / 6 mdians; therefore, for point Q , IJ = 1C / 6. But clearly we could arrive at this
position in many other ways. We could, for instance. rotate the pol ar axis counterclockwise

9.1

14CciiJ;J4.1tM
dinntc~

of two specific

Polar coorpoi n l ~

'~ W~J lill Relation\hips be;w~ pol[ll' Olnd Cat1tsi<ln


c\JIOC\Iinatcs
)'

p
r
)'

Pol~ I' CoorditHHt$

555

through any number of complete revolu1ions, bringing it back to its original position, and then
ro1n1e a further Tr /6 rudians. Ahcmalivcly, we could rotate in a clockwise direc1ion any number
of complc1e rc,oiUiions. and then a funhcr -l l;r/6 rndians. In olhcr words, polar coordin:uc
9 for Q could be any of the value.s Tr /6 + 211 Tr, where II is an integer. Possible values of 9
for point R in Figure 9.17 are 3n / 4 + 2nn. For poim Q. r = 2. and for R. r = I. Polar
coordinates r and 9 for a point are wriuen in lhe fonn (r, 9) so that possible polar coordinates
for Q and R are (2, n/6 + 2111r) and ( I, 31l/4 + 2mr).
This situation is not li ke that for CarteSian coordinates, where each point has Oltly one set
of coordinates (x. y). and e,cry ordered pair of real numbers specifies one point. With polar
coordinates. every ordered pair of real numbers (r, 8), where r mus1be nonnega1ive. represenls
one and only one point. but e,ery point has an infinity of possible representalions. We should
point ou1 lhat in some applications this is not a dcsirnblc simmion . For instance. in lhe bmnch
of mathematics called tensor tmal_\sis. it is necessary that polar coordinates assign exactly one
pair of coordil,atcs to each point This can be accomplished in any region that docs not contain
the pole by demanding. tor ins1ance, that -H < 8 :S If. When we use polar coordina1es w
find areas in Sec lion 9.4, we mu~1 also be particular abou1 our choice of B. For now, however.
no advantage is gained by imposing restrictions on 0. and we therefore accept the fact that if
(r, 8) are polar coordina1es of a poinl. so are (r. 8 + 2111r) for any imeger 11. N01e also 1ha1

pol.w coordinates for the pole arc (0, B) for a:ny fJ whatsoever.
If we imroduce imo a plane bolh a system of Canesian coordinates (x. y) and a sys1em
of polar coordinates (r, 8), then relmior\S exist between the 1wo. Suppose 1he pole of polar
coordinmes and lhe origin of Canesian coordinales are chosen as lhe same poin1. and thm lhe
polar axis is chosen a<S the positi\e x a:~i~ (Figure 9.18). In this cn:;c, Cartesian and polar

'

Pole

Polar axis

x coordinates of any point

P are related by the equa1ions


x

r;'[l "'"I[W

= r cosO,

= r sinO.

( ~A)

l>ol:t.r coor-

dinatti. of point (I. I)


)'

( I . I)

Theseequa1ionsdefine Canesiancoordinalesof a point in termsofiiS polar coordinate<: I hat


is. given i1s polarcoordinalcs (r. II). we cMcalculale i1s Canesian coordinmes (x. y) by means
or 9.4. For example. if polar ~'00rdi na1es of a poinl are (3. 2) .then iiS Cartesian coordina1es are

x = 3cos2 = -1.25.

y = 3sin2 = 2.73.

Equations 9.4 implicitly define polar coordinates of a point in terms of its Cartesian coordinates.

For instance, if Cartesian coordinalcs of a poinl arc (I, l) (Figure 9.19). its polar coordutatcs
nnast satisfy

I= rco<ll.

I= rsinll.

Tf we ~quare and add these equations. we have


I

+I

= r-) cos--,A
u

'A
+ r-' srn-"

"
= r-.

and therefore r = ..fi. It follows thtu cos 8 = sinO = 1/ ..fi. from which we get 0 =
Jt/4 + 211.1r. Thus. polarcoordinalesoflltepoimare (../'i. Jt/4 T 2Jr!t).
Equations 9.4 definer and II implicitly in tem>s of x and y, but obvious!) it would be
preferable 10 have explicit dclinilions. There is no problem expressing r explicitly in terms of
x andy,
r

Jx>

+ y2,

(9.5)

but the case for 8 is not so simple. If we substitute expression 9.5 for r in1o equalions 9.4, we
obtain
X

cos 8 = -;=:;===:<
+ y2

Jx2

sin8 =

(9.6)

Ch:i~er 9

556

Pararn etri~.:

Equations and Polar COOfd:in.11es

Except for the pole, these two equations determine all possible values of e for given X and y .
What we would like to do is obtain one equation. if possible, lhat defines 0. If we diride the
second of these equations by the first, we have

tan B -

)'

(9.7)
X

and this equation suggests that we set

8 = Tan- 1 (~)-

(9.8)

[JEI!JI Polar coor

dinah:< of (-1 , I)

P(-1, I)

Unfortunately. neither equation 9.7 nor 9.8 is satisfactory. For instance. given the point P with
Cartesian coordinates (- I , I) (Figure 9.20). equation 9.7 yields tan 9 = -I. the solutions of
which are 9 = - rr/ 4 + 111r. These are angles in the second and fourth quadrants. so only
half of them are acccp13ble polar angles for P. Equation 9.8 gives 8 = -rr /4. which is not a
po.siblc polar angle for P .
We suggest that all angles satistyi ng 9. 7 be found, and then a diagram be used tOdetermine
those angles that arc acce1>table values for B. We illustrate in !he following example.

I EXAMPLE 9.9
Find all polar coordinates for points with the following Cartesian c-oordinates:
(a)

~
dinat~s

' Ell

Polar 000<-

of four poinu

'

(3, -I )

(- 2, -4)

( - 2, 3)

(c)

(3, -I)

(d)

( - 2, -4)

../5. Angles that s atiSf)

tan 9 = 2/ 1 are I. II + lllC . Since the


point is in the first quadrant (Figure 9.21 ), acceptable values fore are I. I I -r 2mr .
Polar coordinates of the point are therefore ( ../5, 1.11 + 211rr).

il, 2)

(b)

SOLJ.;TI O~

ta) r = .j I; + 2 1 =

y
(- 2. 3)

( I, 2)

v'iJ. Angles satisfying mn 8 = - 3/ 2 are - 0.98 + 111r.


Since the point is in the second quadran t, possi ble"aluesfor 8 are (11' -0.98)+21111' =
2. 16 + 2mr. Polar coordinates are therefore ( Ji}, 2.16 + 2mr).

(l>) r = .j( - 2)2 + 32 =

= .j32 + (- I )2 = Ji(i.

Sir~re \'alues of 8 satisfying tan 9 = - 1/3 arc


- 0.32 + 11 rr . and the point is in the fourth quadrant, it follows that polar coordinates
are (./10, -0.32 + 2nrr ).

(c) r

(d) r = / ( - 2)1 + (- 4)2 = 2../5. Since angles satisfying tan8 = 2 are 1.11 + mr.
and the poim is in the third quadrant. polar coordinate~ are (2../5. 4.25 + 2mr).

The resulls in equa1 ions 9.4-9.8 are valid only when the pole and origin coincide and rhe polar
axis and positive Xax is are identical. For a dilferent anangernen1. these relarions must be
changed accordingly. For example. if the pole is at the point with Cartesian coordinates (h , k)
and the polar axis is as shown in Figure 9.22, equations 9.4 are replaced by

,.3 CUf'\'tS i11 P~latCOO\IIn :Hes

[ii ClJII ;I ?I

557

Polar coor

dinates with oole at {h. k)

PoJar

a.tis
(h, k) .C,----'=-----:-:--:---,
Pole
Horizontal
li11c
X

= h + rcos<e +a).

= k + rsin(e +a).

(\1.\1 )

You will verify the.<: equations in E.erci~e 13.


The usual choice of pole and polar axis is that in Figure 9.18. ami unless otherwisestipulatc<.l.
we assume this to be the case.
EXERCISES 9.2

ln E.xcl\:i:sC:s L-8 plot the poim having the given set ofCartc.:~ian coo~
dinatcs. aud find all possible polar coonlinatcs.

Jn E~crcises 9- 12 plot lhC point hll\'ing lhC gi\'CI1 SCI or pohu coonli~
mucs. and find i L$ Cartt:tian coordimucs.

1.(1. - 1)

2. (- t, J3)

9. (2, ,T / 4)

10. (6, - .T/6)

3. (4. 3)

4. (-2./3, 2)

II. (7. t)

12. (3. - 2.4)

5. (2. 6)

6. (-1. -4)

7. (7. -~)

8.

13. Verify thai polar antJCartc~ian<:oordinatcs as shown in fit:urc9.22

arc related by equations 9.9.

(-~. 2)

19.3 Curves in Polar Coordinates

A curve is defined explicitly in Cartesian coordinates by equations y


f<x) or x
.~(y),
and implicitly by F(x, y) = 0. A point is on a curve if and only if its Cartesian coordinates
(x, y) satisfy the equation of the curve.
Annlogously. n cur"e is defi ned explicitly in polar coordinates if itS equation is expressed
in ci titer of lhe forms

r = f<8)

9 = g(r),

or

(\1.1 Of

and is defined implicitly when its equation is g iven in the form


F(r, e)

(9.11 )

0.

A point is on a curve if at le>~st one of its setS <>fpolarcoordinates (r, e) smisfie.~ the equation of
the curve. All sets of polar coordinates for a poim need not sutisfy the equation. For example,
the origin or JlOie has polar coordinates (0. 8) for any 8 whatsoever. But only those t'OOrdin;nes
of the fom1 (0, (2n + I);or) sotisfy the equation r = I +cos 8. Likewise, the polar coordinates
( 1. rr) satisfy the equation r = sill (e /2). bm the coordinates ( I. 3rr) of the >arne point do not
satisfy this equation.
Often we are required to tmMform the e.qu~u ion of a CUT\"e from Ci-trtesian coordinates 10
JlOiar coordinates. 011d vice ' 'et'Sa. To transform from Cartesian to polar is straightforward:
Replace each x with r cos 8 and each " with r sin 8 . For example, the equation x 2 + y 2 = 9
Je.o;cribes a circle centred at the origin wilh radius 3. In polar coordinates. its equation is

9 = (rcos9) 2
Consequently, r

+ (rsin9) 2

= r 2 cos 2 9

+ r 2 sin2 9

= r 2.

= 3 is the equation orlhi.s circle in polar coordinates, a much simpler equation

than x'+ y 2 =9.

I EXAMPLE 9.10
Find e<juatioos in J>Olar coordinates for the following cut\cs:
(a)

2x

3y = 3

SOUJ110N
(a) For 2.< + 3}'

3. we obtain

3 = 2rcos8

+ 3r sin0

(b) FO< the equation ,,? - 2.~

= -2x +

or

3
::----::--:-~---::

2 cos II

+ 3sin II

+ y 2 = 0. we have

(x 2 + yl)

= -ZrcosO + r 1 = r (r- 2cos0).

Thus.

r =O

r = 2cosll.

ur

Since r = 0 defines the J>Olc, and this point also satisfies r


it follow> that we need only write r = 2 cos II.
(c)

For the curve with equation x 2 + y 2 =

Jx 2 + y1 -

= 2 cos 0 (for 0 = 1t /2),

4x. we obtain

r 1 = r - 4rcos9 = r(l - 4cos0).


nms.

0=

r (l - 4cos9) = r ( r - 1 + -!cos O} ,

from whic-h we have


r

= 0

or

r =I

- 4cos8.

Again the tlOie satisfies the second of these equation~. and therefore the equation of
the curve in J>Olar coordinates is r
I - 4 cos 0.

To tmnsfom1equatjons of curves froOl polar to Canesiao coordjnates can some1imes be more


difficult. t>rincipally bec<'ttL<e we ha ve no sub<:t itution for 8 . If, however. the e<juation invohes

cosO and/or sinO , we use equations 9.6.

I EXAMPLE 9 . 1 1
find equations in Cartesian coordinates for the following curves:

(a) r

+ cos 0

(c)

SOl UTIO~

(a) We use equations \1.5 and \1.6to wri1e

Jx 1 + y 1
and multiplication by

= I

.(
+ -r=,=;=.

Jx2 + y 2 gives

Jx' + y''

r1

= 9sin 28

(b) For r 1 cos 211 = I. we use double-angle formula 1.46b to write the.equation in terlllS
of cos II rather tht1n cos 21/. and then usc equations 9.5 and 9.6:

= r 2(2 cos2 11 - 1) = (x 2 + y1) (2x 2 x+ y2 -

1) =

(xz

+ yz)x

x2

y2

+ y2

(c) This time we use double-angle formula 1.45 on sin 29 :

Example.< 9. 10 and 9. I I illustrate lh<ll equations for some curve.< are simpler when expressed
in polar coordinates. These polar representations can prove very efficient in producing grdphS.
whether we are plouing with an c.lcctronic device or dmwing by hand. We illuwatc wiOt the
curve r = J + cos 0 of E;'\amplc 9.1 J. To plot this curve by graphing c.ulculutor or computer,
we supply the function r ~ I +cos II and the range of values of 9. Because I + cos9 is
2Jr -periodic. the range - Jr ~ 6 ~ 1r suffices. Other values of f) create duplications. The
result is shown in Figure 9.23: it is called a cllrdioid.
To draw the C>~rdioid by hand we first create a Cartesian coordinate system consisting of a
hori1ontal 9 -axis and a venical r -axis. On this set of axe< we graph lhefrmcrioll r = /(B) =
I + cosB (Figure 9.24). This is 1101 Lhe required curve: it is a graph of the function /(8).
illusrrating values of r for various values of 8. It represents an .. infinite table of values.. for r
as a function of 9.
141.111 1

UfE 1

Ploc

Q(

C'ardioid in polar cooNlinatcs


r

1.5
1.0

0.5

.o 0
_ ,.

- - 17-- -1- ---::;r

--r

"

I)

1 + cos8 , we now read pairs of polar coordinates (r, 8) from


Figure 9.24. ime.rpreting r fiS radial distance and() as rotation. Suppose v.-e begin with the two
points (2, 0) and (I, 1t /2). Figure 9.24 indicates that as 9 increases from 0 10 rr /2. values of

To dmw the cardioid r

r decrease fi-om 2 to I. This means that as we rotate from the f) = 0 line 10 the (J = 1t /2
line in the first quadrant, radial diswnce.c; from the origin become smaller. This is shown in
Figure 9.25a. As rotation is inc-reased from ;r / 2 to

TC ~

Figure 9.24 shows that r.tdial distances

continue 1.0 decrease, event.ually reaching 0 a1 an angle of 7f radians as io Figure 9.25b. Notice
that! he Line(}
tr is tangent to the curve atlhe pole, reflect i.ng 1he facLihat Lhe pole is attained
for an angle of 1t r.tdians. Consideration of Lhe graph in Figure 9.24 Lo Lhe lefl of(} = 0 leads

10 that part of the cardioid below the I} = 0 and I} = 1r lines in Pigure 9.25c. The symmetry

660

Cba.pte 9 Plln:uuctric Eql.Ultions :lod 1\11::. C<xudio.ates

of Figure 9.24 about the r-axis is rcAcctcd i1l the symmetry of Figure 9.25c about the. 8 = 0
and 9 = 1r lioes. Since the function r = .f(8) = l + cos 8 is 2rr -periodic, only values of 9
in the imerval - 1f < B ~ 1r need be considered . V~llues outside this imerval retrace previous
poinL~.

Kn!J!ffi(fl?fj"W
r

= I + COlt 0

1'0)111

llittu~

l>lor of
tO

= I+ cosO from 0 to

flt'lt of
r

,T

= I + <:o:tfJ

from - rr

10 tr

0.5

0.5

0.5

J.O

1.5

.T

2.0 0

0.5

t.O

t.S

2.0 0

I EXAMPLE 9.12
Plot and draw the curve r 2 = \lsio28.
SOLUTION Tite explicit ~efinition of the curve is r = JJsin 28 . For >in 28 to be nonnegative, 8 must be restricted to the intervals - rr =" 8 =" - ;r /2 an.d 0 =" 9 =" 7C f2. When these
arc ~ubm i ued to whatever electro nic device you use to n1ak.e (X.llar ploLS. the resul t is as shown
in Figure 9.26.
Lacking a device tbat does polar plots we can dmw the curve by first dmwing a graph of the
function sin 211 in Figure 9.27a. A graph of the function r = 3Jsin 211 then follows (Figure
9.27b). Reading pairs of polar coordioates from this graph gives the c.urvc in Figure 9.26. It is
called a lemniscate.

Mjltjll;lii1EilA

" -2

J\:,lar plot of lemniscate.

2"

-I
-I

Ui11!'11;1HtJ

Cnttesino gmph of sin20

Cmcsi:ll) graph of r = 3Jsifl 20

FIGURE 9.27b

,.

" 0

iT

I EXAMPLE 9.13
Draw the cwve r = Ia - b cos IJ 1. where b > a > 0 are constants.
SOLUTION We begin by drawing the function - b cos8 in Figure 9.28a. A shift of a units
vertically gives the graph in Figure 9.28b. Absolute values lead to Figure 9.28c. Interpreting r
and fJ as distance and rotation leads to the curve in Figure 9.28d. Angles at which r = 0 are
(J = Cos- t (a/b).
MjiCJii;JM:fW

Cnrte~'lian

graph of -bcoslJ

1::11Hii;I4EJ:IM

,.

- b <OS I)

Cllt11!'sian graph of a - bctyj>lJ

a+b

tl - b..:os O
- ;r

iT

"2

iT

0
- rr

-b
ljU-1 1114:11:fM

rr

- "2

2
a-b

Cartesian graph of r

= Ia- bcosOI

,.

ljlcliJ;I4:FJ:f!il

Polar graph of r

= Ia- bcosOI
iT

a+b

- rc

"

IJ

"

To 11nd points of intersection of two cwves whose equations are given in c~utes ian coordinates,
we solve the equations simultaneously for all (real) solutions. Each solution represents a distinct

poin1 of intersection. Fo1curves whose equations me given in polar coordinates. the situation is
somewhat nlOI'CCOillJ>Icx because we have mulliplc n~amcs for points. To lind points or imcrscc
tion. \VC ag:.1in sol ve the equations simultaneously for all solutions. E:.lch solution represents a
I>Oint o f inteosection: but as poinls have many sets of polar coo,d inates, some o f these solutions
may represemthe same point . In ~ldd ition. it may also happen that one set of pol ar coordin:.1tes for
a point of intersection satisnes one equation. whereas a different .set satislie,s the other equation.
Pa.nicu_larly troublesome in this respect is the pole. \Vh ich has so matly SClS of polar coordjn:.ucs.
Tite best way to handle these difticulties is to graph the c urves.

I EXAMPLE 9.14
Find points or inte1section for the curves r = sin (J and r = 1 - sin (J.
SOLUTION
deli ned by

If we "ct sin 9

I - sin 9. !hen sin 9

I / 2. All solutions of this cqu:Jtion arc

~ + 2mr

e = ( sn

6' + 2mr

where n is an integer. Graphs of the curves ill Figure 9.29 iJld i.c ate tllat these values of f) give
the p<>int.s of intcn;ection ( I /2, Jr/ 6) and ( I / 2, 5Jr / 6) . Thcligure a lso indicates lhatthc origin
is a point of intersection of the cw,'es. We did not obtain this poi m by solving r = sin 8 and
r = I - sin (:l because different values of (:l yield r = 0 in r = sin 8 and r = I - sin 9. To
obtain r = 0 from r = sin 8, 8 must be one of the values 1m, whereas to obtain r = 0 from
r = I - sin 9, 8 must be one of lhe values Jr/ 2 + 211Jr. Thus, both curves pass through lhe
pole, but the pole cannot be obta ined by solving the equations of the curves simultaneously.

lnrersectioo poinrs of polnr cur\tes r = sin fJ and ,. = I - sio 6

I#lt111;1"fFF

2"

0.5

:r

0
- 0.5
- t

-1.5

-2

Slopes of Curves in Polar Coordinates


When a cune has polar e qua tion r = /(8) , a ::;: (! ::;: {J , substitution into equations 9.4 gives
X =

j(/i)cos8,

y = j(/i)sin8,

a :0

(:l

:0 {J.

(9, 12)

Tht$.C: an: parametric eqlutions forthecu"'C where the panunercr t< the polar angle 8. F..quation
\1.2 gives

tfy

(t,}-< =

dy
dO
d.\'

f'(fJ) sin 0

+ f(O) CO$ fJ

= f ' ((l)co.< O- /(O)<in O'

t/9
Thi:,. formula cktinc~ che~/CJf)(t ofthe tauge1111i11r tollt'Ur1't', "'hid1ha~ polar equatit.Ml r

I EXAMP LE

9 .16

Find points on the cardioid r

f (fl) .

I +sin 8 at which the tangent Ime is horizontal.

\01.ll riO'\ The graph ofthcC'.Irdioid in Figure9.30 indicates three poinL~m which the wngcnt
line is horizontal. To find them we u:,e cqu.tcion 9. 13 to wricc

tfy
(cos8)sin8 +(I+ <inB)cosll
0 = - =
.
tlx
(cosO ) cosO- ( I + sin 0)si n 9
Sin(.'C the nunltntlor mu!\.1. ''tini~h. we sec

0 =cosO sinO+ cosO - <in 8ro<B = oosll(l

+ 2sin0).

Frum cosll
0. we ehoo.e 0
.? / 2 and from I + 2s1nO
0. we take 0 = -7</6
:uxJ -S;r/6. Thus. poinl 'i\ IU which the cardioid h..-..~ a hori7.0f'lc.al tangent line ha\e Cunesi.u1
'OOrdinmes (0, 2) and( ./3/4. -1/4).
Mit;ili.!M&Iol

lotlh m wl1i.:h 1angen1 NM H>

:= I + !lind~~ liui?oeiUI

., "
2
1.5
I

'

0.5

"

lf

' 2

Lengths of Curves in Polar Coordinates


The lellgtlt ufa Cllr>t> defined parametricall) b) x = x(l ) an~ y = y(t) is given by formula
\1.3:

_ J.#
/(dx ) (dy)
I..., I -t11 + -dt
2

1 dl .

I f we subs1i1ute from cquBiion; Y. 12 in1u thio formu lu with 1 rcpluccd by(). we obllin

1.. =

f.
f.

11
"lf'(IJ)CO>IJ- f(IJ)sin fJtl

11

"lf'(IJ)Il

+ lf(IJ)Il diJ.

+ 1/'(0) siniJ + /(IJ)cos/}J2d/J

564

CIJ:Ipter 9 Parometri< EqtWions ood I'IJiar Coordiootes

= j(8) , a :;; 8 :;; fJ ,

n 1us, we may write for the length of a curve r

(9.1-H

I EXAMPLE 9 .16
Find the length of the cardioid r = I - cos(}.

SOLUTIOK Accordi ng to equation 9.14 (see Figure 9.31).


L =

co.~ 8) 2 + (sin 8)2 dB

12.' .j(I -

_ [ ' J2J1- cosBd8


2;r

=h
=

FIGURE 9.31

;r

1-

21

2.'

sin

[t -2sin 2 (~)Jde

(B
2' )dB= {

~ngth of polar cun c

2 -2cos

(e)}
= 8.
2
27

r = I - ros 9

I EXAMPLE 9 . 17
The plate cam in Figure 9.32 rotates about an axis through the origi n and perpendicular to the
plate. T he follower moves back and fon h along thE' x -axi< a< po int A on its end re mains in
contact with the cam. S uppose r = a + b cos 8. wh.ere a > b > 0 are constants, is the polar
equation of the edge of the cam. and th at 1.h e cam rotates at ti> revol utions per secood. Show
that the follower e.xhibits s imple hannonic motion (called a harmouic cam) and fi nd a form ula
fo r its ve locity.

M@lclil;l*fW

\t.loc:it)' of foli(JI.\'Cr in a plate cam

r = li + bcor-0

Cam

As the cam rotates, the value of r represents the x -coord inate of A ; that is,
= ,. = a + b cos fJ . If we choo:ie time t = 0 \ttthcn the cam is in the position shown. the n
e = 2rrwl' and x (t ) = a + b C()S (2rrwt). Th.is represents simp.le barmon.ic mot.ion for the
motion of A. The vcl ocily is

SOLUTION

~t

dx
dt

u(t)

-2nbw sin (2nwt).

EXERCISES 9 .3

In E.xerci1;e,; 1-lO tind an equ:uion for lhC curve in pol:l.r coordin:deS:.


Druw, o.nd lhcn p l ot~ tbc curve.

In Exercises 22-25 find all poiniJ: t)f intersection for Ihe curves.

22.
l.

X+

2. y

3.

2y

x' + y

25. r = I,

r = l +cosO
r

= 2 - 2cos0

r = 2oos20

= 3
In Exercises 26-29 find the Sh)pC or the: curve tll the gi...cn value or 0 .

4. x' - 2x + y' - 2y + I = 0

6. x'
7. x'

r' = 8t'OSUI

cos8 ~

24. ,. = I + cos O,

= -x
2

r = 2,

23. r =

+ y' = x

9. (x2 + yZ)l

=x

8. x

10. y

26. r=9 cos 2ilnt 0=tr/ 6

+ 2_r2 = 3

+y

27.

l x' + y' x

,.z = 9 sin 28 Ill = -5tr / 6

28. r = 3 - 5cos0 at 0 = 3Jt/4

= 1f x 2

29. r

= 2 cos(fi/2) at 8 =

tr/2

3(). Fiod tl1c slope of the tangco l )inc. to the curve r ::;;;:: 3/( l - sinO)

at the point with polar toorclinatcs {6. rr/6) in two w(lys: (a) by u:>ing
9 . 13 ~ (b) by 11nding the CllUation or 1he curve in Canes-ian coordinates.
:tnd caltol::iting dy f dx.

In Exercises 11- 20 find 1he equation or 1he c-ur"e in Canesian coordi


~te!:. Dra w~ !\rul Lhen plot, I he curve.
II .

r =5

13. ,. ~ 3sin 0
15.

,. = 3 + 3sin0

17. ,.z = - 4 cos 2(}

19. r

=5csc 9

12. 8 = 1

* 31. Show that if /(~) is on even function, then the curve r


is synlm<:Lric about the lines 0;; 0 and()
with lWO C.UillJ)feS.

14. r 1 ~ 4:s.in20

16. r

~-

=2sin20
= cm 2 (J esc(~

21. Dl'aw aod I>IOIIheCUI'VCS (a) ,. = 2 + 2 sin 9. (b),. =


and (c) r = 4 + 2 sin9.

(or

X o.~.X is) .

= [(8)

Illustrate

In Exercises 32- 39 dn1w. and 1hen pl(>(, the cune.

r = 3 - 4cos tl

2.(). r

~ 1t

32.

*
*
2 + 4 sin e.
*

r =

sin 30

= sin48
= tf'
r = -2 cos(0/2)

33. r = <:os20

,.z = (J
37. r = 2 sin (9/2)
39. r = I +cos ((J + ll' /6)

34. r

* 35.

36.

38.

I'

40. AI wtt&ltiJ1K.1HilXI po~hions is the speed oft he follower in EumJ)Ic


9. 17 nla.Ximum Md minimum?

41. Find thelengthofthccardioidr "' 11(1 +sinO).

~2.

(O

49.

2
(.t - o)

isaconSianl.)

44. Cu"<es wiUI equation> or tile f<xn r "' a( I CO>O) orr "'
t1 ( I ::i: si n (:1) (o > 0 a constanL) arc culled mrdioUls.
(u) Dr..tw all such curve$.
cqua1ions for the can:lioids in Cartesian coocdin..-ues.

(b) Find

t1 2 sin

(a) Dm\\ all such <.'tuvcs.


~6.

(u) Dmw dlC cun'CS r

R the equaLion reduce.$ to r =


2a c:os9. Does 1llis reprcsem lhc ..:nti r~: circle?
(c) Do you need bolh equations in port (o) to desc-tibc the entire circ1c when a > R? If .so, which part of the d rclc
COINspomJs to which equation?
(d) Rep.eat pan (c) when a < R .
(a) A pa Ltol boat at point A in the figure below <pots a <uhitll}o
ri nc. subrrscrgiog ut point 8 31;t lime thnt we. cull/ 0 . n,c
submarine. unawnrc of the: patrol bont. rollows :.1 stmight
line path lll conslant s:pcc:d t1 alal :,omc angle t/J 1-clativl!
to fJ A (unknown to the txtlloltx:xLt). Tile patJ'ol boat hc.ads
dirc<tl y wward point 8 at speed V > v foo k/(t + I')
unit~ of time arri\i_
llg,Utpoint C. Show that the submarine
and patrol boat an: cquidistaJII rrotn 8 al l "' k/ Cv + II).
(b) We set up u sy~tc111 of pol:ar coordinates with 8 as I)CJ)c
and 8 1\ os t>Oinr axis. Let the distance 18C I be dcnou::d by r,,. Suppose that the patrol boa1 now follows
the Jogarittunic. $pil'lll ,. = r0~1~., still al speed V. when:

(t

(b) Find equations for the lcmniscatcs in Cartesian coordinates.

R' is

(b) Show 1h~ when a

A > 0 de Ones a particulal"iine of force. Plot or draw lincs


of force for A "' I , 2, and 3.
(b) The cquipolcntiu11ina for the dipole :li'C defined hy r 2 =
8 cos(). where B is u consunt. Plo1 or druw cquitX)tcntial 50.
lines for 0 "' ::1: 1. ::1:2. and ::1:3.
~3. Draw, and then plol. thcb((oli1111 (x 2 + ,v 2) 1 = x1 y.

45. Cun~ wilh e,lutaliM~ of 1he fonn r 1 = a 2 cos }J) or f 1


2U (a > 0 a conslant) are called lcnmisctltc.,.

+ .' "'

r = acos9 J R- a'sin'O.

(a) The electrostatic (.'h:trgc distribution cOC'ISisting ot' a charge


q > 0 at the point wilh polar coordintes (s, 0) <~nd a
charge - q ut (s ~ 7f) is culled a tlipolc. When s i:s very
s n\Ull. the lines o( f'orcc (OJ' the d ipole arc defined b) the
equation r = A~i n 1 8, whc.rc c.u.c h value of the con~lanl

(u) Show that the fXllar cquivalcm ror the equation of I Ill.:- circle

= /V 2jv2 - I . Show that the paticll boat mwt inter

cept the s.ubmann<:.

= b acos fJ and r = b a sinO .

where a and b arc posithc constants. in ahc three cnscs


a < b,a b~<1nda > b.

(b) Mnd cquali<HH: for 1hc curves. i C;;~J1Csia coordillaiCS.

(c) Compare the$<: CUI'\'es with ahc cardioids of Exercise 44


whcna.;..;.b.
47. A curve wilhcquuionof thc fo nnr = asinn(J or r = arosn(J ,
where a > 0 is u const1nt and 11 > 0 is nn integer. is eta lied u rt)se.

P..uh of subm~rinc

" ad'u

Show tl\:11 the :.c h:a:. '' pct.d~


It

48. Show thai the roses r = Ia sin n91 211d r = loc-Os nO I. where
a > 0 is a con."'-ant anc.l n > 0 is ar~ iruc.gcr. have 211 petals.

1 - -- - k - - ----1

19.4 Ar eas in Polar Coordinates


l ll Section 7. 1 we usetl defi nite imeg.l'als lO fi1tl cl.l'eas bounded by c urves whose cqualions are
convenient ly expressed in Cartesian coordinates. In t his seclion we indicale how lO find areas

bounded by curves whose equations are expressed in polar coord.inatcs. We require 1he formula
(9.15)
for the area of the s haded sector of the circle i n Figure 9.33. T h is formula " ',suits from the fact
that the are a of the sector is the fmctional part (82 - 8 1) / (2tr) o f the area 1T r 2 o f the c ircle.
Consider finding the area of Lhe region in Figure 9 .34a bo unded by Lhc radial lines 8 = a
and 8 = fJ and the curve r = .f(li) . We divide the region into s ubregions by means of 11
I

radiaJ Jjnes

e = e,.. wht:l't:
a =

lio <

lit <

e,

< .. . <

e.,_, <

lin

= fJ .

9.4 Art..'lS in Polar Coonlin:ues

1J

..

\lj;J#F(I

Area

bound~d

br curves using po1ar coordinates

M4lrl11;14F('I;')

567

Approximating area with sectors Qf circles

iT

2
r =f(())

9=9,

Polar axis

Polar :l.xis

On ~lat pan of~le curl'e r = f(8) between ()= 0;_ 1 un~ 0 = 0; , we pick any poim with
polnr coordinates (f (8i), Bt} as in Figure 9.34b. If between the lines 6 = ()1_ 1 and 8 = l)f
we draw the arc of a circle with centre at the pole and radiu~ f (9t) . a sector is fonncd with area
~A;

I
2
= 2lf(8
1 )I D.8; ,

whe re 60; = 0; - Bo- t Since this sector approx imates thut pan of the o-equired urea between
the radial linc,s 0

- 01_ 1 uml e =

0;~ we can say that an upproxintution to the required area is

Jl

II

l= l

1;1

L .:l.A, = L 2 lf(9i) ]1 M,.


By increa;ing the number of sectors indelinitely. and at the same ti me requiring each of the
0.8; to a1>1>roach zero. we obtain a beuer and beller approximation. and in the limit

area

L
~ If (8,')]
11<>01 1-0
2

lim

Ml; .

l; l

Q
'fil CEfW
fQr polar c:oon.lin~tlc5

But this limit is the defi nition of the. definite integrol of the function (I /2) l/(6) ]1 with respect
to 0 from 0 = a to() = {) . <Uld we therefore write
$a."{Qr llJeOI

(9.161
In order lO arrive at Lhis integral in any glven problem. without memorizing it. we use the

procedure adopted for definite integmls in Canesian coordinates discussed in Cllap1er 7. We


dmw at an1C () a reprcscon01ivc sector of angulur width di) and radius r (Figure 9.35). The
Mea of this sector is

U areas of aU such sectors from angle a to a ogle fJ are added together, and 1he limit is takeo

as their widths approach zero. 1he required area is obtained. But this is the process defined by
the definite integral, and we therefore write equation 9.1 6 for Lhe area. Definite integral 9.16
~ 8 ~ {J.

exists when f (II) is continuous for a

568

~ramerric f.'4111ions nnd P'l)l:)!' COI.)C\:Iin~ucs

0 1ap1CI' 9

I EXAMPLE

9 . 18

Find the area iMide the cardioid r = 1 +sin 8.


SOT UTI ON

ljltlil.l ~ A1'ta int>ide


cnrdiuld r = 1 + 11in 0

TIIC area of the repre~cmative sector in Figure 9.36 is

-r 2 d(J

.<

= -2 ( 1 + sin8) 2 dfJ ..

and we must add over al l sector. imcrior to the cardioid. Since are;~ on either side of the
fJ 1r /2 line are identical. we calculate the area to the ril!ht and double the rcsuh. To find the
orca to the ri ght of the line() = n / 2. we must identify angular positions of the fir~t a11d la>t
scctoN:. The first sector is at the pole, and the cqu:nion of the e<~rdioid indicates thm r = 0
when sin (I = - I , that is, when (I = -n/ 2 + 2nn. But which of these values of 8 slll!ll we
choose'? Similarly, the last sector occurs when r
2. in which case sin 0
I, and could be
any or the values n /2 + 2nn . Again. which shall we choose~ If we choose a = -;c /2 and
fl = 1l / 2. then all values ore in the interval -Tr /2 5 () 5 7r /2 yield poiIlls on the right half

of the cardioid with no dupliC"'dtions. Consequently.

area = 2

~(l 1

- ( 1 + sin /1)2 d8 =
- /2 2

~/2 ( 1+2sin8+

_,, ,,

38 2wsll = {--

(I+ 2sinfJ + sin1 9) d8

- . /2

cos2t1 )
dB
2

sin 28 }'12
-4
- .T/2

I E XAMPLE

I -

!'12
=

3JT

........

9 . 19

Find the area common to the circles .r1 + y'

= -1 and .r2 + y1 = 4.r.

SOLUTION F.quutions for the circles in polar coordinates are r = 2 and r = 4 cos 9 , and
they intersect in the poi 11" with polar coordinalcs (2. Tr/3) (Figure 9.37). If A, is the tll'CII
above the x-axis, outside .rz + y2 = 4 >1nd inside x 2 + y2 = 4x, then the area common to the

::::::::1 AI'U COR~


mo11 tv ci~~ .x: t ,.: - 4 ttld

,'(: + ,.: = 4X

circles i !r" I he area o f e ither circle less twice A , :

"2

area

= J1'(2)2 - 2A , .

The area of lhe reprcscm:uive element is the <.lifTcrencc in the areas of two sectors:
I

- (4 cos 0 )2 d0 - - (2)2 d f) = 2(4 cos2 0 - l} t/0.

Since all sectors in A, can be itlentified by values of (I between 0 and n /3. the required area is
0

r'3 2(~ cos 8- l)dO = "" - Jor"'3

area = 4n - 2 lo

[2( 1 + cos20)- !l dB

,..
2
EXERCISES 9 .4

In Exercises 1- 10 find the area of tl1e region eoclosed by the curve.


I. r =3.sin8

3. r

= 2 sin28

S. r 2

= -cosB

2. r = -6cos8

7. r = 4- 4 cos9

4. r 1 = 2 sin28

9. r

= s in 38

6. r =2-2tos9
8. r = 4 -2tos9
10. r = 2(cos6

+ sin9)

9.5 DetioiLiMs <.' f O.)tlic; Sections

= 4 + 3 sin 6 but outside r


19. Insider= II- 4cos01

In Exercises 11 -21 find the area of the indicated region.

18. Insider

II. Outside r = 3 but inside r = 6 sin 6


12. Inside both r
13. Insider

S69

=2

Inside Lhe bifolium r = sin 9 cos2 8


21. Bounded by 0 = 7T and r = 0. 0 ~ 0 ~ 7T

20.

= I and r = I -sinO
**

= 2 sin28 btu outsider = I

22.

(a) Show lhtu in pOlar t(M)rdinales 1he SlrOphoid

14. Inside both r = 2 + 2 cosO and r = 2 - 2c-os0


JS. Inside both r =sin f) and r =cosO
16.

Inside botl1 r = cos9 and r = I - cos8

where a

>

tl cos 28

sec 8 .

0 is a rorLc;tanl. takes the. form r

(b) Draw or plot the curve and fi nd the area inside itc; loop.

17. Insider = I - 4 cosO

19.5 Definitions of Conic Sections


In Section 1.4 we used conic sections to illustrate the algebraicgeometric inte,.play of plane
analytic geometry. They can be visualized as curves of in1.e rsec1 ion of a plaoe with a pair of
right circular cones (Figure 9.38). Certainly, this suggests why Lhe conic sections are w named,

(a) Circle: Plane ))Ctpendicular to


axis of cones

(d) Hyperbola: Plane cuiS


bo.lh cone.c:

(b) Ellipse: Plane cut~:> completely across

one cone but not perpendicular to axis

(c) Pair of straight lines: Plane passes

Lhrough ve11ex and cuts thmugh bolh cones

Parabola: Plane cuts only one


cone but not oompl e~ely across

(c)

(f) One straight line: Plane passes


through ve11ex and touche-s both cone-s

but because of the thrCC""dimensional nature of the cone. an analysis of conic scc.tions from this
point of vie\V is n01 yet possible. In this section we use plane analytic geometry to u.-elop
definitions for parabolas, ellipses, and hyprbolas.

The Parabola
DEFINITION 9 . 2

A parabola is the curve traced out b y a point that mo\'eS in a plane so that its distances
from a fixed point called the focus and a fixed line called the directrlx are always the

same.
Suppose the focus of a parabola is the point (I'. q) and the directrix is a line)' = r parallel
to the x -axis as in Figure 9.39:1. If P (x , y) is any point on the pambola. then the fact that its
distance from (p, q) must be equal to its distance from y = r is expressed as

J<x -

fl)l

+ (y- q )l = (y- r(.

(9.17)

With the absolute values. this equation includes the case of a directrix abo\'C the focii.'S. as in
Figure 9.39b. If we square bOih sides of the equation and rearrnge tcnns. we Obtain

\Vccan wh'C this equation for)' in tcmls of x: the result is


I

Y = - - - ( ( . r _ 11)2
2(q- r)

+ (ql

_ ,.2)].

(9.1Sl

We could rewrite this equ:uion in our accu>torncd form y = ax 2 + bx + c for a parabola. btu
the present form is more informative. First. the line x = pthrough the focus and perpendicular
to the directrix is the line of symmetry for the pambola. Second. the parabola opens upward if
q > r. in which case the focus is above the d ircct.rix. and opens downward if q <: r. Finally.
the vcncx of the Jl<1rnbola is found by senins .t = p , in which casey = (q
r)/2, halfway
between the focus and directrix.

JA [i]

lth:

1!?'1

n.J PM.tbob with dim:-

bei()V. (OCU\

FITW

P.mbol

wilh direcuh aiJO\'t rocu...

Direcuix
f'OCU>

(p , q)

P (x,y)
['3'B'~114W:lil

Verux

l,.lmbob

will vcnicol din:cui'\:

.<
x= r;

.t

Dirc!Ctrix

Directrix

A similar analysis shows that when the direcn'ix is paralle l to the )'axis (Figure 9 .40). the

equation or the parabola is of Lhe form

X = .,--J __,.. (()' _ q)l


2(p - r)

+ (p2

_ 1'2)).

(9. 19)

9.5

IJd 'initU.MH of Coni.: Seo..1ioru

571

Given the focus and d irec trix (parallel to a coordinate axis) of a para bola, we can easily
fi nd its equmion: use fonnulas 9. 18 o r 9.19. or follow the algebraic Slcps lc,ld ing from 9.1 7
to \l.l8. CmW<!I-,;ely. given the equation of u pantbo la in the form y = axZ + bx + c or
.t
ayl + by+ c. we can ident ify its focu und d irectrix (see Exercises 52 and 53).

I EXAM P LE

9.20

Find the equation of the parabola that has focu s (2, 4) and d ireCtrix .< = 6.
SOLUTION If (.< . y) is M Y point on the pambola (Figure 9.41 ). the fnct that its distance from
(2, 4) is equal to its d istance from x = 6 is expressed as

x=6

J<x- 2)2 + (y- 4)2 = 6 - x.


If we square both sides >md simplify. the result is x = ( 16 + Sy - y1 ){8.

---

The Ellipse
X

DEFINITION 9.3

An t llipst. i!) the curve cn:tcec.J out by 1a point that moves in a plane so that the ~um of iLo;;
distances frOI'n two fixed J>Oint... calledj'od remains conl)htllt.

-JjUiil.l-,11
tentlt<o of 1wo fo...i

Cllipj,(' im

The CCJLIHlion of an el lipse is simplest when the l'oc.:i lie on cithel' Lhc X or )'uxis and ~are
cqu idi~tant from the origi n. Suppose the foci >lrC

(c, 0) nd ( -c, 0) (Figun: 9.42). and the sum


of the distances from these foci to a point on t he ellipse is 2a, where a > c :!; 0.
If P (x . y ) is any point on the ellipse. Ddinition 9.3 implies ~lilt

J<.< + c )2 + yl + .J<x _ c)l + yl =

2a.

(9.20)

If wc.trdnsposc the second temtto the righthand side and square both sides, we obtain
(.t

+ c)2 + /

= 4a1

4a )(x- c)2 + y2 + (x - c)'+/,

and this equation si mplifi e~ to

(?- c.r = a)(x- c)l + yl .


Squaring once again leads to

a'' -

2 ll

cx + c 2.r 2 = a2 (x 2 - 2cx

x 2 (a 2

c2 )

+ a 2 y2

= o4

+ c2 + y 2)
-

or

a 2c 2

D ivision by a 2 (a 2 - c 2 ) g ives
(\1.21)
It is c ustomary to denote y-inre r-cepts of an e ll ipse by b (b > 0) (Figure \1.43a), in w hi ch
case b2 = a 2 - c2 and the equation of the el lipse becomes

or

(9.22)

Milnii -1'4[]

[&Jip;.co

"'lth W'fi<'lll , ,.,jor a"'i~ :md h~HI


:rut l lllfl\(11' tl ~.,.

" '

Tl'le line segment acros.~ 1hc ellirlSC and t hroug h the l'oci is called the m l\iOr a xis of the
cllirx.-;(.; i l l1as Jengdl 2tl (sec rigure 'J.43 a). The midpoi nt or 1hc m~or <'l:<is is ctllled the tenrre
f)f the ClliJ)St:. T he line scsn1c:n1 nc::ross 1he. elliJ)SC, ll'lrough iL
'\ G(:nn~. n.nd J>cltendicu ht to the
m ajor ~I;< is i.s (.-ailed rhc mhlor axis: it h~S: IMgth 2(7. N(I(C l h.atthe line segment joining: ci1hcr
C11<.f ol' the minnf nxis co a f'ocus (Figur~ 9.4:lft) ht~s 'elgth n. and 1hc ~riMgle fomlCd SJ'Ie<:il;es

rhe relationship nmo ng tl. b . and C' , namely aZ


bl + c2 ,
A ... imilar (lllld)'~i~ s ltows that when the rod or ch~ dlip!'oe 're Oil the Y-IiXh, equidi~tOI.Ill
from the origio, the eqwuion o r the ellip:ic il) agnin in fonn 1J.22. In this case. 2b is the length
o f the mnjor axis. 211 is the length o f the m illornxis. culc.l b 1 <t2 + t1 (Figure 9.43b).
What we!'hould rcm c:nber ischnt an equation offornl 9.22 ;lwnys spccifies nncll iJ>SC. F oci
ttre Oil the longer Hxis 1111d carl be located using c l = ln2 - b1 1. The length o f Ihe m:ljo r nxis
represent..: cJ1c ~u m ofchcdisuul cc:~~ from 411\Y point on the dli~c 10 the foci .

/ EXAMPLE 9.21

Draw the ellipse J6.\' l

+ 9y2 = 144, irldi>Ca ting its fod.

SOI .lJIIOI'\ If we write the e llipse in the form ~ 2/9 + y 2/ 16 =< I. its x and y-imercepL~
are 3 and 4. A .<ketch of the ellipse is therefore as shown in Figure 9.44. The foci must lie
on llw Jll,,i.< af distance.<
=< J41 - 31 =<
from the origin.

.J?

"' b in equation 9.22, then x 1 + y2 "' a 1 , a nd this is the equation for a circle with mdius
a and ceotreat the origio. But iJ a = b. 1he d istance from theorigi o toeach focus of theeiJ ipse
must be c = 0. In other words, a circle may be regarded as a degenerate ellipse whose roci are
If a

at one and the same point. h is also true that when c is very small compared 10 half the length of

9.5 Dcfinioions of Conic Secoions

141.111

14 I

EJiilhC

when c is rm.1ch less than a

,.

the major ax is (Figure 9.45u), the ellipse is shnpc!{l very much like a circle. On the other hand,
when these leng1hs are almost equal (Figure 9.45b). the ellipse is long and narrow.
When the centre of an ellipse is at point. (h, k) and the foci lie on either the line x = h or
)' = k (Figures 9.46), the equation for tl1e ell ipse is somewhat more complex 1han 9.22. If 2a
and 2b are again the lengths of the axes of the ellipse, a calculation similar to that leading from
9.20 to 9.22 gives (sec E.m cisc 55)
(9.23)

FIGURE 9.45b
Ellipse
when c is approximau:ly equal 10
a

Ahcrnalivcly, 1he curves in Figures 9.46 are 1hosc in Figures 9.42 and 9.43b cransla1ed h
units in the x -direction and k units in they-direction. According to Section 1.5. equations for
the translated ellipses can be obtained by replacing x and y in ..- 2 fi1 2 + y2 f b 2 = I by x - h
and )' - k , respectively.
IJhlll.l

txlfii

fiGURE 9.46b

Ellipse wilh <>:ntrc 31 til. k)


t a X

Focus

(/1, k b)

(il, k+r)

~---(ll, k)

(h - n, k)

...

I EXAMPLE 9 .22

573

Focus
(II. k-bl

.Y

(IJ.k- C)

Draw the dlipo>e 16x2

+ 25y2 -

160x

+ 50y =

1175.

SOLUTIO:\' If we coml>lele the squares on the x - and y -terms. we obtain


16(x - 5) 2

+ 25(y +

(X - 5)2

100

(y

1) 2

1600

or

+ 1) 2
64

= I.

The ccnlre of lhe ellipse is (5, - I), aud lcngiiiS of its major and minor axes are 20 and 16,
respectively (Figure 9.~7 ).
ElliP5oC 16.t 1 + 25r! - 160x + 50y = 1175 dtawn by <:omplcl in~ th:. ~qu~rt)

'

(- 5, - I)

(15. - I)
(5. -9)

The Hyp erbola


DEF I N ITION 9 .4

A hyperbola is the path troced out by a point that moves in a phlne so that the difference
be1ween its distances from I\\'O fixed poi n t.~ called ,lOci remains COtlSUuu.

Uk.e the ell.ipse. the simplest hyperbolas have foci on e ither the x - or y-axis, equidista nt
from the origin. Suppose the foc i are (c. 0) (Figure 9.48) and the difference in the distances
from P (.r , y) to these foci is 2a. T hen Definition 9.4 implies that
(9.24)

This equation can be simplified b) a cakulation similar to thai Je::1ding to 9.22~ lhe result is

y2

xl

---- I

a2

w here 1>

=c2 -

lti@ Q

[]0.

b2 -

(9.25)

a 1 . The hyperbola has x intercepts equal to a.


ll)l)Ct'boln

Wit h f~xi Oil .\' &XI~

Wllh I()CI t)tl y-:n:i~

)'
(

' y'
-- 1
o 1 - b'l-

-c

y2

.\.2

---=I
2
2

-a

\
When the foci"'"' on the yaxis (Figure 9.4')). the equation oftl>c hyperbola becomes
(9.26)
is the co nstant difference in the distances from a point (x, y) to the foci, and a2 =
c - 1> This hyperbola intersects the y-axis m b.
1ltat p11utofthc line scglllcntjoining the foci of a hypcrboh.athat is bctwt:en the two branches
o f the(..urve is called the transveru a~ is: of I he hyperbol:l; it has length 2tl in Figure 9.50a and
2b in F ig\lfe 9.50b. ihc midpoint of the tn1nsver:;c: axis i~ calh:d the crmrc of the hypt:rbola.
T he line segmcm perpendicular to the trntlSVCrsc axis. thro ugh its cc1Hre, and of length 2b in
Figure9.50n and 2ll in Figure9.50biscalled theconjugateaxis. J\Symplotes of both hyperbola.~
are the lines y = l,xf a.
What we s hould re membe r is that an equation of form 9.25 or 9.26 spccities a hyperbola.
where
1

211
2

The foci Lie on the exteosion of the tr ansverse ax is aod cao be located usiog c2

= a2 + b2 .

The length of the transverse ax is repre.-;ents the d ifference of the distances from any point o n
the hyperbola to the foc i.

9.3

Dtfi"bli~Ov/ Cuh:Stctioos

575

FIGURE e.SOb

-~~Treub ver.)c---.
axis

/'

I EXAMPLE 9 .2 3
ll)lXfbol
1(\r:- 9J:- I .U <hov.n1 toci

..
.

4 )( /

..

. :

.'

_, : . ]

s .(

Draw the hypcrbol 16.tl - 9y 2 = 144, indicating its foci.

SOLUTION If we CJ<prc~ the hypcrbolu in the fonn .r 1 /9 - yl / 16 = I. its xintcrccplli


are 3. With a<ymptote< = 4x /3. we oblain Fil!ttre 9.5 I. TI1e fuci lie o n Ihex :!X is ;U
distances c = :!: 4 l + Jl = :!:5 from the origin.

When the centre or. hYP"rbola is at point (11 , k) and its foci are on !he lines ,,= h or y = k ,
equations 9.25 and 9.26 arc modified in exactly the same wa) a> cqu..tion 9.22 "as modified
for an ellipse. We ~plac-e each x by x - h and eoch ) by )' - /.: (<ee also Exercise 56).
Consequently. equations fO< the hyperbolas in Figurcs 9.52 lite

II'BiE
Hyperbob:' \1-'llh

and

I.

PIOURR e . &Zb
ttflol~< a t ( It ,

t
b

l'J' :
..

'

.
/

k-

k(x-~)i/b
;;<x - h)
0

(h. k + <)

..
/

. .
: ch. k>
.. ...

h. k +b)

....

(ll.'k.- b)
\,,

576

Ch"pter 9 Pnrfmelrk Equ:niun.s and Pol:w- Coordinmes

.........

I E X AMPLE 9 .2 4

Draw the hypert>olu x 2 SOLUTION


[!I!I.;J g[EJI

,r: - y= + 4.r
10) =
co.n1detiag rhe IOCJU::tre.->

llyp<tbota
5 \It i l\ \ II by

If we complete squares on x. and y-tcrms. we find


(x

+ 2)~

- (y - 5) 2 = -1 6

(y - 5) 2

.:..:....-,-:.... -

(.r

+ 2) 2

I.

16
16
The celllre of the hyperbola is ( - 2. 5), >tnd the length of i" transverse texis (along x = - 2) is
8. If we solve the equmion for y, we obtain

(-2 9)

H. sj,~:

y 2 + 4x + lOy = 5 .

:.,

y = 5

(-2. I)

_.

J<x + 2)2 + 16;

asymptotes of the hyperbola arc then y = 5 (x + 2). Tbe hyperbola can now be ske tched as
in Figure 9.53. Its foci are at the points ( -2. 5 + 4J2) and ( -2. 5- 4J2}.

..-..

y: 5- (X+ 2)
)'~5+ tt ~ 2)

ff P i$ a point on a <.:oni<.: se;:(.;tion, thc.[Qcal rtulii al P arc the lines joining P to the foci
(l'igurc ~.54). As a rcsuil, a parabola hils one fo<O!II n1dius at C>lch po int, and an e llipse anu
hyperbola each have two. One of the propenic;; of conics that makes them so useful is the f>lCt
that the nornud line to the conic at any poiru bi ~ec1s the a ngle between chc fOcal rndii. For the
parabola, the norm>ll bisects the angle be tween the fOClli n1dius and the line through P parallel
to the axis o f symmetry of the pnraboill. We w ill verify these facts in Exercises 59 and 60. To
obcain one physical significance of these results. suppose each conic in Figure 9.54 is rotmed
about the ~t -ax is to rorm a surface of revolu tiOil, which we regard a~ n rnirror. lt is n lnw of
optics Ihal when a rny (lf lighl slrikc:.s a re flec11ing surface. the i1nglc be tween incident light and
the nomull to the surface is always equal to the ang le hctwccn reflected light and the normal.
Consequently, if a beam of light trave ls in t he negative x -din.--ction und strikes the parbolic
mirror in Figure 9.54a. all light is rcftectcd toward the foc us. Conversely, if F is a source of
lig,ht~ <lll light striking lhe mirrnr i. reflected l)<lnllle l 10 the x -)x is. If e iLher focus or rhe ellipi:e
in f igure 9.54b is a light source. all light strikiing the. elliptic mirror is rellected towa rd lhe o the r
focn~ . Similarly, if either focus o f the hyperbola in Figure <).54c i~ a source, a ll light striking
the mirror is reRe<:ted in a direction that wou ld make it seem to originate at the other focus.
Convtrsely, i f light that i s dirtcctd 41t one focus first strikes the mirror. it is reflecced toward the
other fcx:U<S. This is pr<:<:isely why we have parabolic reOC(;tors in nutomobile headlights and
sear<hlights, parabolic and hyperbolic reflectors in telescopes. and el lipticcei linw; in whis pering
rooms.
Thus far. we ha\'C defined parabolas. ellipses, and hyperbolas in terms of d istances: for the
parabola we use a foctls and a dire<:trix, and fo r the e llipse and hyperbola two foci. In Section
9.6 we. show that clli~ and hypc.r bolas can 2'tlso be defined in terms o f a focus and directrix.
I ij lorIlli I l Ul=.:l

Htpsc a.s

Ulf111BCMJN
bokt \l) ;.1 r( fl(CCOr
)'

Hyper

9.5 l:kfiniaiun.. ofConio.: So..:tion"

577

EXERCISES 9 . 5

rn xcn;.iscto: 1-14 idcn1ify Lhc CI.JU<tLion as rcprcscming a smtighl line.

-i-

40. Find chc heighI of Lhc parabolic arch in Lhc (jgurc below.

a cirde. <t t)arabola. an ellipse. a hyperbola, or none of Lhese.

1. 2X

+ 3y =

y2

3. 2x-y=3

s.

Sx

2y

II -

7. y 2 - x

+ y2 -

4. X~

+ y1 =
2

6. 2x

= 14 -

+ 3y

2. .r1

l.t

-----T~

+2

3y + 5 = 0

8. x 2 + 2:c

x2

3.r + ly = 2-5

4 :

= 3y + 4

1 ----L ..

I:

~-=-~ _r;_ --'------'

1 - - 5 - -1

9. y 1 + x 2 - 2-r + 6y + I 5 = 0

10. x' + 2y 1 + 24 - 0

}$

11. S + y 2 = 3x 1

ll. x 1 + 2y 1 = 24

J.3. y '=3x+4

14. 3 - .'(

= 4y

41. Exph1in how (In dlip.._....: o m he <lnlwn w ilh a pic..:c uf ~fring, IWV

LaCks-. alld a penciL


~

42-. Find the cquatio.n of lhc ellipse traced out by a point that

lllO\'C.S

lhm the sum of its distartOCS fro m (4. 0) is alwoys equal to lO


(a) by u.slng equation 9.22 with su.itable \'a1ucs for a and b and (b) by
estabJishinS and simplifyins, an equation sill\ilar to 9.20.
.$(1

ln Exercises 15- 36 draw the curve. rdcntify foc i for each ellipse and
hypc.r bolu.

15.

x-

)' = 2x2 -

17. x 2

16. -

25

,..

25

36

X+ .\'2 =

+ 3y 2

= 16

22. 2.\2

+X-

3y + 5

w.

19. - - - =
21.

+ '- = I

7x

23. 9x' + 289y = 2601

24. y = 10(2 - x 2 )

25. 3.r 2 - 4y 2 = 25

26. yz-.rl=S

27. y = - x :r: + 6.\ - 9

28. 2.r' - 3y2 = 5

29. 3.r' + 6,r 1

30. x 1 +16y 2 = 2

3 1. y2 - 3x2

= 21

= I

34. ''- 6x- 4y

16)' + 13

44. Show thai the equation of c-.'Cr)' straiglu line. cvcryc.irclc. andc\'cry
conjc; scc:-tion discussed in 1b.is section can be obtai.11. 1::d by uppropriaLc
choices of constants A, C , D, E. and F in the equation

llx'

=0

* 46. Show that the cquution of the tangent line to the hyperbola b1 x 1 a:yz = a 2b: ala poh1t (:c0, y0) is b 2.'t.\'o - a 2y)'() = a 2b 2.
Fi11dthl~poim P on lhoupanoft!K~ellip.se2x 2 +3y 1

= 14 i111he
first quadrant where the t.:J.ngent line ru. P is perpendicular to the line
joi ning P and (2. 5) .

24y = II

36. 9x 2 - l6y 2 - ISx - 64\' = 'H


37. Find 1he equation of a hyperbc.ll:"l th:u
(1~ 2) Pnd bas nsymp(()tc.$ y = 4A.

+ Cy' + Dx + y + F = o.

+ 45. Show that the cq1aation of the tangent line to the ellipse b2.t: 2 +
n 2y2
a 2b 2 at a poim (x0 , y0 ) is blxx0 + a 2yy0 a1b 2

* 47.

35. 9x 2 + y 1 - l.S.r - 6y = 0

~ses

lhrough 1he JXlint

J K. l-Ind the equation of an ellipse through Ihe poin1s (- 2, 4) and


(3, 1).

* .l9.

3 2. X= - (4 + y)'

+ 2.r +4y 2 -

33 .r '

43. Find tJ1e equation of the hyperbola traced oul by a poim Ihat ltlO\'es
so t.hnt Lhc difference between lt$ distan$ from (0. :1::3j is u.1wa.ys
CQuOI IO I (a) by using equation 9.26 with suitable vnlu.::s fo r a ::md b
and (b) by eslabli.shiog, 3_nd simplifying an equation sim.il:.l.f 10 9.24.

16

.r'

:>:

18. 3x = 4y 2 - I

=- = l

y-'

)-

48. Find lhe area insi.d c the ciHpsc b 2 x 2 + a 2y 2 =

49. Among all rccw.ngJcs thatcnn be in.sc1ibcd inside thccllipsc.b1x 2+


a 2 ) : 2 = a2b 2 and ha\'C sides par~dtel to the axes. find the one wilh

a 2 fi2 .

Jargcst possible area.

* SO.

A p1olmt- spheroid is t~ oolid of revol ution obtai ned by rouning

un ellipse. ubout its major :u.is. An qb/atc spheroid is obto.iucd by


rutaLing the ellipse about its minor ax~ . find volu10cs for the prolate
and oblau~ !>ph eroicl~ gener-ated hy the ellipse b'!.xz + a 1 y'!. = o 2 b 2 if

f ind Ihe wid1h or 1he ellifMiC arch in the figure below.

" > b.

- -'-r n'l--- -'

11'---

St. A sharp noise originating at one tocus F 1 or an c.Uipse is re.ftc.cted


by the ellii)Se towa1d the otJlel' rocus F1 . Explain why aU re.ftc.cted
noise an'ives al F2 aLexaclly the same Lime.

578

+.

Chapter 9 P:lrametric Equations and Pol:u Coordi n:l.l t:~

52. \j~ cquatiun 9.18 lO show lhul when u JW.r..tbo)u is ,o,,rriiiCll io the
form J = (l.T-: + bx + C, the (ollowing formulas identify its focus
( /> q) ao>d directrix y = r:

p=--.

2ll

= 4a
- (-

pan.1Uc:l to the Xax.is. lli111: Or..aw the tungcnalinc


IIPFI = IIRFI.

~8.

\\11eO a beam of light tr41-clling in the ncgati\'C X -directiOn (figure

below)

1+4ac - b ).

~trike~

a pard.bolic mirro1 with cross section represented by

= a)'l + c! all light r&)'l) nrc rcflcetcd to ahc foc us F of the mirror.

ShO\v that aJI pho1ons th:::H p&SS simuhnncously through X = d arrive


ot Fat the s~mc time.

Whal are formulas ror the rocus (p. q) and directrix x = r ror a
pantbola nflhc type x = ay' +by+ c?
54. Use the formulas ln Excrc::iscs 52 and 53 to identify the focus and

* 53.

x=d

directrix foro.ny pl.rllbolu.s in Excn::.i.sc:s 15- 36.

'*

P R ul P u.nd ~how

tbat

55. Sho\\' that when the centre of an ell iJ>SC is !U point (h , k) 3nd its
foci are on 1he line .\"
lr or y k . Dclini1ion Q.3 1e:ld!: to equation

9.23.
56. Show that w!).:n the ccmrc

foci dJ'C 00 the line

= Jt

or a hyperbulu i~ at point ( /1, k) and it~

Of)'

= k, Oc(jnition 9.4 JcruJ.s IO equal ion

9.27.

57. Pro'"C th:ttlhc nonn:.1lli:nc to the paro~bol~ :c = ny 2 in the figu~


below bt.socts the angle between the focalr-,ldius F P and the line P Q
~~

59. Prove thai the oonnal line 10 an elljpsc or hypcrbol3 bisccls the
angle between the fOC<J.I rodii.

**

60. Prove th3t the nonna1 line to 1hc parabola ) = ox 1 + b:r + c


at any polnt P bisects the angle between the foe a) r::tdius and tbc line
d>rough P parollcl to d1c y axis.

s:tS~nern lhrough the focu~ of a parabola wilh e nd~ on the


is c:llcd afo~:(l/ chore/. II is an established rcsultlhat l:;mgcnts
to a par.lboln at the ends ofo focal chord arc perpendicular to each other
ond inter-sea on the:: diroctrix. Prove this fo.r the parubolu .r = ax 1 ,

u: 6 1. A line
1~r::~bola

9.6 Conic Sections in Polar Coordinates


In Section 9.5 \Ve defined parabolas using a focus and directrix and ellil>~es and hyperbolas using
two foci. In this section we show that all three conics can be defined using a focus and directrix
t1nd

that~

in polar coordinates.. one equacion represents all three conics.

Let F be a fixed point (the.[ucus). and I be a fixed line (the d irectrix) that doe< not pass
through F, as in Figure 9.55a. We propose to find the equation of the cun" troccd out by a
poim Lhal moves so thal itS undirected distances from F and l always rema in in a consuuu ratio
~ called the eccentricity. To do this we set up polar coordinates with F as pole 11r1d polar n is
directed away from I and perpendicular l<J I a< in Figure 9.551>.

I@ 13 I 11'} l'!lfi1!W

r/ P (r;ll)

F
Focus
dOire<;trix

r ~ Polar axis

- d J,ole

9.6 Cooi..: S1ion8 in Pobr Covdm.\le.S

579

lf (r,IJ) arc l>olarcoordinaccs for any p<.>inc P on che required curve, chc face chal the racio
of the discances from F and/ 10 P is equal co i s expressed as

(\1.28)

_....:.__ = f .

+ ft'OSI/

\Vhen this cqumion is sol\'cd for r. we hflvc


(9.29)

r =

- cos9'

and chis is chc polar equation oflhc curve cmced ouc by che point Ccrcainly, chis curve should be
a J>:irabola when E = I . \ Ve now verify lhis. and show that the curve is (Ul ClliJ>Se when < I
and a hyperbola when f > 1. To c..lo this we trartsform the cquatio1l irHo the u~ual Canesian
coordin;nes x
r cos& and .v r si nO . From cqumi ons \1.5 and 9.6. we obca in

artd this equation simt,l ifics to

jxl + J ' -

+ x).

1\1.30<1)

.,
+ )'"., = E"(d
+ x).

(Y.30b)

e(d

or when squared.

"'

.r
When E

= l. lhc x2-1crms in9.301>canccl, and lhe cquacion reduces 10 lhal for a parabola:
(9.1 1)

as in Figure 9.56a. When E

x2

1. we write

+ y =

(I -

d'

E" (

E2).r l -

+ 2(/x +

2di 2,\ '

+ y2

,
X")

or

= E2d2.

When E < I , we divide by 1 - E1 :

and complcce the square on che x-1erms:

(9 32)
Comparing equal ion 9.32 w ich 9.23, we conclude 1ha1 9.32 is Lhe equacion for ao ellipse wiLh

centre at position X

= d E2/(I -

2)

on the X.axis. Since one focus is at the origin, it follows

thm che ocher must also be on the x-axis at x

= Zd 2 /(1 -

2) as in Figwe 9.56b.

580

Cl~tt 9

1>.-r.l:m('.lrk: Equa.iuns :n1 Pol~r CoordiB11CS

W3F

1;1

:= =.,

E<:otOnric-

~ =:. I leads II) p:ar;\bolil

ity

II1E ur-

M:ilclil ;I__,.~ Bccenll'iC


ily to <: I lrnds 10 ellipse

.,

&x::enmc-

ity ~ > I I~ to hypefOOia

'

Other
\

f<W:US

Owtre

......
A simil:1r calculation shows that when <

tifl

X+-,-( - I

> I , poi nts on the right

y2

~2- I =

h~lf of the hYI>erbola


2

ul

~2 -

)
I

( ~.33)

are obtnined "' in Fi gure 9.5(><:. Equation 9.30a is not satisfied by points with x-coordinatcs
le:;_< ~'"" -d. :and therefore points on the left half o f the hyperbol a. shown dotted. do not satisfy

'J.2'J.
We have shown thnt cqumion 9.29 defi nes an ellipse when 0 < e < I. a pnmboln when
< = I . and a hypcrbohl when e > I , nnd thi $ provides 11 unifying approach to conic sections .
A ll three conics can be ~tudicd using it focus and a directrix.
II is clc;\1 that eq uation 9.29 can yield only u parabola that opens to the right a11d has its

focu:- on the x -~-xis, :111<1 an ellipse and hypcrbokl with roci l)l'l the x -axis, one ~u the origin. 'fo
obtain paraboiM thm open to the left, or up>or down. :u1d ellipse...; and hyperboh1s wilh foci on
they-axis. we must chunge the position or the directrix. The conic sections in Figures 9.57
have directrix to the right ol' the focus. They hllvc equations

r =

<Y.34a)

1lle conic sections in Figures 9.58 cm<J 9.:59 hav~ e<JU<Uions of the form, respectively.

r =

C.:\llliC ~t kuH in rhe 1\)l'n

\) f

ed
---"7

- E sin 8

and

I'

r = cd/(1 + c co~O )
y

II
Polar
axis

l
X

ed
l + esi n(/

(\1.34b)

Co~t.cSec..'lions in

9.6

M Jt~JII.I Dl
Ct-.lic ~tions i n cbe fOI'tll of r = ~d/ (1 -

ln.... Ill
f

PoiMCov lill.'lC<.S

581

H!J l:W

s-inO)

P~Iar

Polar
~is

:!.>:110

'
~

Coni~; section~ in the form

[:][.fifii

of r ~ (.(f/( 1 + f sin c~ )
.1'

)'

Polat'

~ /axis

'

,,())at'

axis

'

axis

In Figures 9.56-9.59. one focus of 1he coni c is chosen'" the pole. In 01her wonl,, the
simplicity of equatiOn> 9.29 and 9.3~ 10 describe conic sec1ions is a direct COIISequence of 1he
facl lhtH the pole is a1 u focus, unO the directrix is cilhcr pantllcl or pcrt>cmticular 10 1he polar
axis.

.........

I EXAMPLE 9.25

Dn1w the curve r


SOLU1101-i

EJiiplloC r

IS/(3

= 15/(3 + 2 co<O) .

IJ we write the equation in I he fonn 9.34a.

r=

+ 2<os0)
y

l + G)cose'

the eccentrici ty E = 2/3 indicates llllttthe curve is <1n ell ip>C. 801h foci lie on the .t -uxis,
and one is at the origin. TIIC ends of the major axis occur when 8 = 0 ond () = ;r, and for
these values r
3 and r = 15 (Figure 9.60). It now follows that the cc.ntre of the ellipse
is at .t = -6, and its 01hcr focus is at .< = - 12. If b > 0 denotes half the lcngdt of the
minor axis (il is also the ntaximum y value on the ellipse. occurring when .x = -6). Lhen
b 1 = a 2 - c 2 = 92 - 62 = 45. Coo>o;equently, I> 3./5, and the ellipse is as shown in
Figure 9.60. This information now penniL' u.s to write tht t:qualion of the ell ipse in Cartesian
coordinates:

(.t + 6) 2
.:...._:-:-:....
+ i- =

81

45

I.

582

Chaplet 9

P:uMlctrit Eq-u lions :uuJ IVa Coordinates

A n ahcmati,e approach in Example 9.25 would be tu substicute r = Jx 2 + y 2 and cosO =


x f J x l + yi into the polar cquatiun fo rlhe conic. simpli fy it to (x + 6)2 / 81+ y 1/ 45
I, and
then drnw the ell ip>e from thi s equation. We il lust rate this method in the fo llowing example.

I EX AMP LE

9.26

Draw the cunc r

SOI.lfi'ION

= 2/(3 - 4 sin 8 ).

l fwcso t r

= / x"+ yl

u11dsin 8

= y/Jx 2 + yl, thcn

Oi\'ision by Jx2 + y2 leads to


3/x2

+ y2 -

3/.r2 + y2

4y = 2

or

= 4y + 2.

If we now square both sid we ohtain


9(x

+ l) = 4 + 16)' + 16/
9x 2

or

1y" - 16y = 4.

If we complete the square oo>thc y -te mts. we o btain

36

64

= 4- - =
7

Oi\'isio n by -36/7 yields the cquutiu n

xz
- 4

Hyptl'bola

= I.

7
)'

8
7

3.r

\' = - - + -

fi

This equation describes a hyperbola with centl'e (0. - 8/7) and y -intercepts eq ual to - 8/7

6/ 7 = -2/ 7. -2. Its asymptotes are


8

Y =

{'7.if

- 7 1'1 4

Jx

- 7 ./7.

The hyperbola is shown in Figure 9.6 1. butonl)' the top half is desc ribed by r = 2/ (3- 4 sin 8 ).
The equation 3 /x 2 + y 2 = 4y + 2 does not permit y :::; - 2 .

..-..

I EXAMPL ~

---

9.27

Fifld t\ polar r1:prcscnuuion ror the ellipse


( ,( -

1 )2
.$

w:n

111

(X- 1) :/~

,.

'I.

1b li ntl n volar repc.sc.:ntution fo t' the e lliptse ( Ftg:ure9.6'l), we cou\c.i ui'C the "~""'
l)(t)lll't.OOt'di ~uUeJS dC'6ncd b)' ,'1: r t.'t:OS8 l'l tld )' r toin 8, but the: TCSUhit\$ equation WQUid 1'\0\
1Je ,;implc. Try it. We krlQ\v that~~ ~im1>lc polar reprcsenuniot must r<:-M"\ i t the \l01e \s <;h,.Js t''
")OI.UI IO N

ll1p;e

+ .Y~/Y =

yl

+9 -

ill t' rocus Qf the ellip~ ~n(,1 Jm)IU till iS e ither J.)l,U'lll\cl Of jpcndi(,;u Jaf (()the d in:-t,;1.TiX. f"ot th \'S
e llipse. foc i are Q.n 1hc line x = I m di~ance; c :l: 9 - ~
..JS front tl'le .x <\<t.b;.. Let
us d~e the pole :11 pnsitio n ( l ./5) l'd the l')Uhn axi~ p;:~rnllcl to the x i\x.is. and th\.-rcforc
parallc:l co che directrix. Aet.."\')fc.l in~LO cquatic.t n 9 .9, po\au- tultl Cr.tnc..,.hm <:(xr~J\,,t\tc.s tm: rduted.
by x = I
r cos() and y = ...tS r sin f). If we ~t;_lbsti.tute tht..- se i nto the cc,mttl \)" ft)t' the
ellipse. \\' l>bt;.lill

9r 2cos:2 (J

4(5

+ 2 .../Sr s i1l (J + r 1 s.in 1 9) =

36.

'fh is CCJUalion co\n be cxprcs:s(."ti o..;;; a quat.lt~\tic equation in r:

Sol utioulro fot,. lll'e


r

=
=

- 8JS .sitl 6 :l: J320 s in!9

+ 64(9cos'l9 +

4 sini6)

+ 4sin'6)
-8J5 inO ./576(co<1 0 + sin1 Q)
1(9eos1 9

Since r musa be tmnneg,ati\'C, we must choose +3 .u\d not -'3 , t\nd therefore
r

=
4

EXERCISES 9 .6
hllhC~tSCS

3
I. r=
I + cos O
3. r=

s.

J- 10 dnn\' thC COI'I/c K'Ctinn.

3 - 3 sin8

2.
4.

,.
,.

1.6

3 + 5cos8
16

5 + 3cos0

7.

' "'
'"'

9. r=

3 - 4sin6
I
2 - 2tos0
St:<:. ()

3+6scce

6.

~
I' =

8. r=
10. r=

- 3 sin 0
I

2 + sin @
4 c;.c

7cscfJ -2

In Excrci.')Cli 1 1- 16 find the Curtc$iitn equation for the cun'C.


II.

r=

13. r =
15. r =

12.

1 + 2cos0
4

6 - 3sin9

14.
16.

r=
r=

r=

a polar cqu.-1tion in
9.29 or 9.3.4 for the conic.

16)1

(a) Show that the ecccntrtcity l of an e11i~ or hyperbol<i is

always equal to the di ~tancc from its (,'Cmrc to either focus.

3
J-sin8

In E.~tetciscs 17- 20 find

11. x1

'l l.

t6

(v +I }'

19. (x -1) 2 + -- - = I
4

divided b)' half the IC'ngth of the majOC' Of' tral\svcrsc t).iJt..
3+cu~8

20.

22. A circle hus bccndcscrib.:das ad.cgcner.nccllipsc in the sense lh:u


its foci cointi&.~ What happens to the x-entric:ity of an ellipse if the
distance between its foci approaches zero? Hilft: Sc.e ExCt\."lSC 21.

5+5cos0
one of lhe four forms of

x'- 9(y -

the ccccncricilics or the cllir~cs in Figures 9 .453

'"'d 9.4Sb.

I - 3cns 6

t8. 4.r 2 + 9y'

Oi~uss

(b)

..-... 23. Paths of cclestinl objc:rtS<.'an be dc~cribcd by ...:quations ufthc fonn


r a/(1 + toosO). v..hc.-c a > 0 is a constltlt. To fi nd the time to
h"Jvcl from orcc: poinl on lhc palh 10 nnolhcr. 11 is nccc~~ry lot."'o;haa.tc
the integral

= 36

I=

(I

Do so in the= >hut (o)


( > I.

2} 2 = Q

+ <cusO }'

= 0. (b) 0 <

d9 .

< I. (c)~

= I. and (d )

Information on Transla11on and Rol.adon of A~e~ ha ~ been placed on the Text Enrichment
Sile: www.fu'a rsomul.cllltextltrim.

SUMMARY

Tn Chapter I we defined curves explicitly a11d implicitly. In this chapter we added a third

description - the parametric definition


.~

l'(l ),

)'

= _y(l),

~ ~ I ~

/J.

\Vhen such a curve defines a function, the der~vati,e of the fu ncLioo c-an be cakulaced using the

parametric rule:

dy
dx

dy
dt .
dx

dt
second- and highcr-QrcJer derivatives can be Ct'l_lculatec.l u~ing the chain rule. The length ofslch
a curve is defined by the deli nite integrcd

Polar coordinates pru,,idc tul uhc111utivc way tQ identify the positions of points in tl plane.
They use distance from a point, called the pole, and rotation of a half li ne. called the polar
axis. Many curve; that have con'lplcx cquati\)11 .~ in Carte!)ian coordinate~ can be rcprcscntc.d very
simply LL~i lg polar cootd irlatcs. Particularly sirnplcarc rnollileavcd roses, ('ard ioids, lemniscc\tes.
und some circles. Polar coordimttcs also J>rov ide a unified approach to parabolas, ellipses, and
hyperbolas. Each curve can be described as rhe pmh tmced out by a point 1har moves so that
1hc ratio of its distances from a fixed poin1 and a fixed line ren10in consranr. 11lc cuM: is
an ellipo;e, a pan1bola, "'" hyperbola depend ing on whether 0 < < I , = I , or > 1.
rcspLctivcly. Ell ipses und hyperbolas cun ulso be defined as curves traced out b )' a point th at

moves so rhutlhc sum tmd difl'crcnce of irs dismn<'CS from two fixed poims remain constant.
When a curve r = f((J ) in polar coordio1arcs encloses a region R , Lhe definite integral

rfl
~u<e>l 2 de
Ju 2
with appropriate choices of a and p can be used 10 llnd the area of

R.

= /(9). et ::; 8 ::; {3 are


y(9 ) = [(())>in e,
f)

Parameuic equations l'or a polar curvc r


.r(9) = f(IJ) cose,

(II

:::

:::

p.

Its ~lope i~ given by


dy

dx

.('(O) sin 0 + f(IJ)cosB


f'(O)cosO- f(O)sin8

anc.l its lclgch can be calculated with the Oclinitc integral

KEY TERMS

In re,iewing this chap!cr. y<:>u sh<:>uld be able to dclinc or discuss the following key terms:

Parametric equa.liOI'lS

Panunetric.ally defined function

Parametric n1le

Polur (\<>Ordinates

Pole

Polnr axis

Slopes of curves in polar c-oordinates

lengths of curves in pQblt coordinates

Areas in polar coordi1u1tes

Parabola

F<>cus

Directrix

Ellipse

Major axis

Cemre of an ellipse

Mielor axis

H)'Jlefbnla
Conjugate ax i~

F.cceHricity

REVIEW
EXERCISES

In EAct'\:iM::- 1-.30 draw. oU'ld chen plot. the CUI'\ \:,


I.

r =cosO

2. r

3
I + 2 sin II

3. r =

5. r =

4. r =
6. r=

+ sinO

t' =

3
2+ sinll
2

4+t .

8. r = 4c.vs8

13. (x 2

+ y 2) 3 = x

+ )'

= 4cos30

2 0. r = sin 10- eos 2 0

21. X 1

27.

2.0

18 . t 1 +)'I

+ ,)'1 -

+ :ly '

22.

- 6y = 0

24. y'

r=

sin1 0

26. r

+X + 2y =

0 30. .t

= y' + )'
=

sinO. y = sin 20

In ExL!f'Cisc...; 3 1-35 fi nd 1hc UI"Clt ofth!; I'Cgion indicated.

+ 4 cost .

31. .,,.;de ,. = 2

t .:S ;r

+ 2 cos 0

31. htoidc r = 4 htJI OUl~idc r = 4 sin 20

33. Common lor= 2 and r 2 = 9cos 20


34. Insider

= s in2 8

35. Conunon 10 r

= (OS (0 /2)

28 . .t '
1 ;>

= X

x' - 3y2 + 4 = 0

3x + 2,1' = I

29. d = e-t. y = Int .

II. x ~sinl t, )' =cos t . 0:0 t ~2lT


.\' :::: - 2

19. r

3c:o~

+ y = .fx' + y'

25. r (2cos0 - sinO)= ,l

)' = 5 - 3t 2

+ 4 =>in 1.

J:

2cos0

12 . .\' -.: I

17.

l6. r =

L~. 2x '

2cost. y = 3sin t O ~ t <2>r

10 .~=

i 2 = 4cns2 0

7. r + 1=2sin8

9.

= -$in0

IS.

= I + s in 8 and r = 2 -

2 s in 8

586

Chapcer 9

Par:lmelric Equ:~~ions and Polar Coordino1es

In Exercises 36-37 assume LhaL )' is dcHncd as a funclion of


Hnd d yfd x and d 2 yfdx 2 .

> 36.
37.

= 13 + 21 ,
.x = 2sin u ,
X

= 31 - 13
y = 3cos u
)'

I n Exercises 3~0 fi nd the length of the curve.


~

38. r =2+2cos6

x, and

J9. x= t 2 ,

,\'= 13 ,

* 40. x = e' ens t ,


* 4 1.

O,S I ,S I

= e' sin 1 ,

0 .::; 1 .::; ;r /2

Find I he equal i on of I he langenL li ne Lo I he cun c r


aLl he po inl w ilh polar coordinales { I, rr / 6) .

=2 -

2 sin 0

CHAPTER

10

Applicati on Pre,iew

Infinite Sequences and Series

The amount A of a certain chemical in a reactor decrc.1ses exponemially in ti me


A(/)

where A 0 is the amOtml at time 1

= Aoe"'.

= 0. and k

<

0 is a constant. Afler timeT , the amount in

the reactor i$ AoelT at which time an additional amount Ao is added, resulting in an amount
9

Ao + AoekT = A0 ( I + tfT) mthattimc. This amount then decreases exponentially unti l an


additional amount Ao is oodcd at time 2T, and so on, and so on. Once the runounl of chemical
reaches a criticnl lc,,cl L in the reactor, the process must be u:mtinatcd.
TilE PROBLEM
Find an equation dtal <fetem1ines the time at which the pr~JCcss must be
tenninatetl. (For a solution to the problem. see Example 10.38 on page 662.)

Sequences and serie.~ pl~y !!!l imponant rule in many areas of applied mathematics. Sr.quences of numbers were first encoumered in Section 4 . 1, although we did not use the tenn
sequences at the time. Newton's itcr.:ui,c procedure produces a set of numbers .\' 1 , .t 2 : x:.h . . .
each of which approximates a root of an equation /(.t) = 0. The fillit number is chosen as
l>OillC initial approximation to the solution of lite equation. and subscqucnl nurubcrs defined by
the formula

/(x.)

x,+, = .~, - f'(x. )


arc bcllcr and bcllcr approximalions. This ordered oct of numbers is called a sequence. Each

number in the set corresponds to a positive integer, and each is calculmed according to a stated
formula.
A series of numbers is the sum of the numbers in a sequence. If the numbers arc .<,, x 2.
XJ, . , the corresponding s~r its is denoted symbolically by
co

L:: x, = x . + xl +

.l)

+ ....

n=l

where the three <lOIS indic-,ue that the addition is never-ending. It is all very well 10 write an
expression like tbis, but it does not h.-e meaning. No matter how fast we add, or how fast a
calculator adds, or C\'C.n how fast a supc.rcomputc.r adds, an infinity of numbers can never be
added together in a fi nite amount of time. We shall give meaning to such expressions, and show
tl1at

they arc really the only sensible way to define many of the. more common tnlm;ccndcntal

functions. such as trigonometric. exponemial. and hyperbolic.


The fir.;t two sections of this chapter wtd Section 10.8 an: devoted to sequences und the
remaining ten to series. This is not to say lhat series are more important lhan sequences; they
are noL Discussions on series i.nva.riably become discussions on sequences associa1.ed with

series. We have found that difticulties with thi s chapter can usually be traced back to a failure to
distinguish between lhe two concepts. Special attention to the material in Sections I 0. 1, I 0.2,
and 10.8 will be rewarded; a cursory treatment leads to confusion in other sections.

587

I tO. I Infinite Sequences of Numbers


Sequences of numbers are defined as follows.
DEF I NITI O N 10. 1

An lnHnlte ~equence of number~


intege.rs.

i~

a functi<>n

whose domain is the set of positive

For example. when /(11) = 1/11. the following numbers are associated with the positi,e
integers:

2'

It

3'

4'

1ne word ittfinile simply indicates that an infinity of numbers cs defined by the sequence, as
there is nn infi nity nf posit i\ c i11tcgers.. but it i1tdit'Hte.." nothi ng nbout the nature of the numbers.

Often. we write the numbers

.f(n) in a line scpamted by commns.


( Ill. Ia )

.f(l). [(2) ..... /(11), ...

and refe1 10 this arTay as t.hc sequence mther 1han lhe rule by which il is ft>nned. Since this
nouttion is somewh:u cumbersome. we adopt: a nouuion sintilar to thtu used for the sequence
dcfoncu by NcMun"s ilorativc 1>roccdure in Section -1.1 We sci c 1 = .f( l). C2 = /(2) . .. ..
c,. = f (n) , ... . and write for IO. Ia
(1 0 lbl
Th~ fi r~l number c 1 is called lhc first ttrm o ft'hc: sc:quen<.:<:, c2 lhe second term, and for general
It , Cu is called the n*h ter111 (or gcneml tcnn) of the sequence. For the extunple t1bove, we h.ll\ C

(' = -2'

Ca = I,

I
C)= -

ecc.

ln some applications, it is more. convenient to de line a sequence as a function whose domain


is lhe set of integers larger tilM or eqcolltosome lhed integer N . and N
IXJ>ilivc. ntgati vo.
or zero. L ater in this chapter. we find it convenient to i1litiate the assigrlment with N = 0 . For
now we prefer 10 u~e Detini1inn 10. 1 where N = I , in which ca~e we have the natura l silllation
where the fif!)t tenn of the sequence corr~spond~ to n = I, the second tenn to n = 2, and so

.,,"be

on.

I EXAMP L E

1 0. 1

The general terms of Four sequences arc


I
(a)

2"-'

"

(b) - -

11 + 1

(c) (-l)"lu- 31

\Vrite out the fi rst six tenns of each sequence.

SOLLliON The firsl six cerms of these sequences are


(a)

I III
I
I. ;; -4 ' 8
- ' 1(:, ' .' 2;

(c)

-2, I, 0, I, -2, 3;

"

'

(b)
(d)

3 456

2' 3' 4' 5' 6' 1'


I , - I , I , - I , I, -I.

The sequences in Example 10.1 are said

1.0 be defined explicitly; we have an explicit formula


for the nth term of the sequence in te rms of n. This aiJows easy dete rmination o f any term in
the sequence. For instance, to find the one~ hundredth term, we simply replace n by I00 and

pcrfonn lhe resulting ,uithmetic. Comms1 1his wilh lhe sequence in the following example.

10.1

I EXAMPLE

lntinircScquen-.:eso(N'umbers

589

10.2
The first tenn of a sequence is c 1 = I and every other tenn is to be obtained from the fonnula
C11 +t

Calculate cz. C3. c. and


SOLU110N

= 5 + J2 + c.,

> I.

cs.

To obtain c2 we set 11

~'1+1

II

I in the formula:

= cz = 5 + .../2 + c1 = 5 + J2+1 = 5 + ../3 "=' 6.732.

To find c3, we set 11 = 2:

c3

= 5 + J2 + c2 = 5 + j2 + (s + -..13) = 5 + J1 . . J3 ~ 7.955.

Similarly.

c. = 5 +

c:s =

Jz + c3 = 5 +

j7+ l1 + ../3 ""' 8.155

and

5+ .../2 + ' = 5+ / 7 + j1+ h + ../3"' 8.187.

When the tcnns of a sequence arc defined by a fommla such as the one in E>arnple 10.2, the
sequence is said to be defined r ecursively. The terms for a sequence obtained from Newton's
itcnnivc procedure: arc so defined. To find the a()(Jib tcnn of a recursively defined sequence. we
must know the 99"': to find the 99"' , we mu:st know the 98111 ; to find the 98"', we need the 97"':
and so on down the line. 111 other words. to find a tem1 in the sequence. we must first find e"ery
tern1 that preced<:$ it. Obviously. it is much more convenient to have an explicit definition for c.
in tenns of 11. but this is n01always possible. It can be very difficult to find an explicit formula
for the 11"' tem1 of a sequence that is defined rccursi,ely.
Somction<:$ it is impossible to gi,e an algebraic formula for the tcm1s of a sequence. TI1is
is illu>trated in Exerci>e 31.
When the general term of a sequence is known explicitly, any tcmt i n the sequence is
obtained by sub>tituting the appropriate Ydluc of 11. In other words. the gcncraltcnn SllCCifies
every tem1 in the sequence. We therefore use che genernl tem1 to abbreviate the n01a1ion for a
sequence by writing the general term in braces. Specifically. for the sequence in Example 10. l n.
we write

_1_, ,"'- =I,~2 ..!..~8 .. .. , -2''-'-,, .. . ,


{2"I

indic-.ue 1hat1he firs1 cerm corl'ts:J>Ond ~ 10 the inreger n = 1. and 1ha1 fhere is
an infinite number of tcnns in the :sequence. In gcncml. we write

where 1 a11d

len}~ = C1. C2 . C3 . ... C11

( 10.2)

If. as is the case in this sectjon, the 11rst term of a sequence COITesponds to the integer n = 1,
we abbreviate the notation further and simply w r; te. /c.,) in place of (c., }j"'.
S ince a sequence {c,.} is a function whose domain is the set of positive integers, we can
represent (c.,) graphically. The sequences of Example 10. I are shown in Figwes 10. I.

Mjt.lll I ihi!W

c,.

z~-~

':1

c,

JO

.-..

In- 31

II

(j, = (-1)1

. . ....

S 10 12 14 n

4 6

2 4

-5

4 6 8 10 12 14

6 8 10 12 14 "

c,, = (-1)11

..........

(', =- I-

8 10 12 14

-I

-2

- tO

fn most (lpplications o f sequences we are inccre.sted in a number called the limit of th e

sequence. lmuilively. a number I. is called 1helimi1of a scqucnce lc.. l if as we go fanher and


fan her ou1 in 1he sequence. 1hc 1crms gel m"bitmri ly close 10 L nnd stay close to L . If such a
number L exis1s, we write
L

lim

~~-

C1r,

( HU>

tuw..l say thnt the lOequence {c") CcHwt rgt s to L. If IH) such 1lumber e xis:t:t. we say that the
sequence c.locs n01 huvl! a I imil. or that the sequence dhergcs.
For the sequences of Example I0.1 , i1is evident thai:
(a)

(c)

lim - - = 0.

(b)

lim ( -1)"1,

(d)

.,--.oe

211- 1

ll-o0

- 31 docs no1 cxis1.

II

lim

n-ee n

= l.

+I

lim (-l)"+l dOCSIIOieXiSI.

n-oc

Note how the poinlS on the g raphs in Figure.s IO. Ia and b cluster around t he Hm.its 0 alld I as

11

gc1s larger and larger. No such clus1cring occurs in 1he rema ining 1wo figures.
h is usually. but not alwa)'S, easy to determine whether an CXJ>licitly def-ined sequence has
~limit. an<.l wlmt thalli mil is. h is likt: lindirog lhc limil ul' a function /(x)
00. For
example. if we divide numcr:uor :and denomin~IOr ol'lhe .cquence {(111 +n - 3) /(2111 + 11 + 4) I
by
I
3

"" .<-.

,z.

+ 11 - 3
-"" 211l + ll + 4
lim

11

.
= n-oo
hm

I + - - -1
n
I

11

..'

2 + -n + -, z

Seldom is it obvious whe.ther a recursively de tined sequence has a limit. 1::-o r instance, it is not

a1 aiJ clear wheLher 1he recursive sequence o f Example 10.2 has a limiL In spile of Lhe fact
that d iffere nces betweeo successive 1.errns are approachjng zero. and tenns of the sequence are
therefore getting closer together, the sequence might not have a limit. For instance, the sequence
(Ji?} does not have a limit~ yet it is easy to show that differences between successive terms get

10. I

lnRnite StQuences of NumberS

591

smaller and smaller as 11 increases. (See Exercise 30 for anolher example.) There are ways to
veri!)" 1hat a sequence has a lim.it, and some of 1hesc will be discussed iJl Section 10.8.
When a sequence arises in applic11tions, i1 may be pe&c1ly clear thm the sequence is
a>1wergem. Such is oflen the case in the field of numetical analysis. where recursive sequenc~
commonly arise in the form of ilcr:uive procedures. We have already encoumered NewiOns
i1era1ive procedure as one example. When this me1hod is applied to 1he e.quation

= x3 -

f (x )

3x

= ()

wilh initial appmx imalit>n Xo = 0.7 10 find lhe TOOl oelween 0 an<) l, the SetjUenl"e <.>blaine<) is

x, = 0.7.

Xu+ I

f (xn )
f' (x ,)

II

.\'Jt - - - -

2:: I.

The first three 1crms of 1his sequence are illustrated in Figure 10.2. and 1bc IangenI Jjnc eonsmootion by which 1hey are olnained makes it clear 1hat 1he sequence must con v~rg~ to the sulution
of che equation betv.een 0 and I . In other words, it is not nece~sary for us to verify convergence

of the sequence algcbroieally; i1is obvious geome-lrieaUy.


l!n:.., __ .r;g

Graph1ca1 illustration of SQ:Jnencc ITom Newton( ilcnuhc pi'O(..'Q:'harc

0.75

y=xl - 3x+ I

0.5

(.< ,,f(.<,})

0.25
0.2'-- 0.25

-0.5
- 0.75

To find 1hc soluLion of 1hc equn1ion we cvalua1c 1crms of the sequence algcbraic.tlly until
they repeal:

=0 .7.

0 .20 5.

x5 = 0.347 296 355.

X6

x,

Xl

X )

0.342 ,

... =

0 .347285.

0.347 296 355.

ObviouS!). X6 = 0.347 296 355 must be clo~e II>1he root of x 3 - 3x + I = 0 be twee n 0 and
1. How close can be veri fied wilh the zero intcmtedialc value 1hcorcm from Section 1.1 1. For
imaance, ro verify that 0.347 296 is tm tlpproximation 10 the ~olmion correcdy rounded to six
decimal places, we calculate

f (0.347295 5) = 2.3 X

I 0- 6

and

j(0.3472965} = - 3.8

1()- 7

The f<lCithat func.lion values are opposi1c in sign verifies 1ha1 0.347 2% is corrcclto six de<:imal
places.
Because Figure 10.2 illustmled lhm the sequence detined by Ne"~on 's itemlive procedure

must converge to a rool of f (x ) = x 3 -

3x + 1 = 0. it was uonecess.ary to veri.fy existence of a

Lirni1of the sequence algebraically. Are we proposing thai a graph of 1he func1ion f (x) should
always be drawn when using Newton's method to solve equations? Not really, although we are
of the philosophy lhm pictures should be drawn whenever they are helpful. Numerical analySIS

have proved dwt under very mild restrictions, the sequc tcc dcfiotd by Newton s method always
,onverges to a TOOl of the equation pruvitk:O that the initial approximation is s ufficiently close

tO that root Sec Exercise 68 for fun her discussion of this point. In pmcticc. we nppt'oximatc
the "olution o f an equntion j(:c) = 0 using Newlon'"" method exactly us we d id i1'1 Section 4.1 .
!lll d as illustratc.d above. \Vc set up the: appropriate sequence, ch(.)()SC an initial approxim.uion,
and itcnuc to find further approxinlliOI\s. We do not ,erify, algebraically or geometrical!)', that
the sequence has a limit. If terms get clo~r and cl()ser together, we ~'Uspcct that the sequence i,:
convergent, that is, thattcnns arc gcUing c1oscr c.u ltl closer to the limit. \Vc do not know this for

sure. 11or do we try to verify it. 'The real problem is to sohe the equation j'(x) = 0. We luwe
wh~H

we believe is olo ipproxinuuion to the solutio n, und we verify this d irecLly w ith t he 1.cro
imcm1ediah: value theorem.
Another wily to sol ve for 1hc same 1''001. of x 3 - 3.r + 1 = 0 is to rewrite the equation in
the form

= -3I (x3 + 1),

and use this to dctine the followi ng recursive sequence :


I

Xt

3'

(x, ... 1).

, 2: I.

1l\C initial term wa~ chosen somewh.m arbitnuily. In practice. it should be M close to the root
a~

po.ssibl e. The fir:oa eight

tet n l~

of the seque1lCC nrc

. , = 1/3.
x, = 0.345 679.
3= 0.347 102 19,
x. = 0.34727295,

x5 =

-" =
-"1

0.3~7 293 53,


0.3~7 29601.

= 0. 3~7296 31,

.r, = 0.347296 35.

t~ct

that .\'] and x 8 a gree to six decimal places leads us to belieYc that the sequence
converges and that its limit is appruximlatCI)' equal10 0 .347 296. \Vc cannot be ccnain o f this.
nor do wcuucmptto verify it. Whm we wan! is an approxima1ion to !he root of x 3 - 3x + I = 0
between 0 a1ld I. We c.an ve.r it)' directly that x = 0.341296 is an approx imation accurate to

The

six decimal places. as above.


This method of finding the root of an equation is often called 1he lllelhod ol' succcssi.-e
apprnxlmutlons or fixed -point Iteration. Whm we do is rearrange the cqua1ion ./'(x) = 0
into the form .x

g (x)~ and detlne

-<t = A.

a recu~ivc sequence.
Xn + l

= g(x,),

II ~

I,

where A Ls some inili aJ approximation to the root (the clo~r the better). \Ve iterate hoping Ihat
lenn s of the SeQuence get closer togethe r. Jr they do. we terminate iterations when we s us pect
chat we hove an approximation to t he solution or the eqmuion with che rcquire<l accuracy. and

verify this directly with the zero imermedime value 1hcorem. If terrns do 1101 seem to converge.
we may try a di fi'erent rearTangement or the equation into the forn .\' = g (x), or a nother
method.
We can visualize the method of s uccessive approximations geometricaJiy. l.n terms

or the

example above, we are tlnding where the gmph of the function .f (x) = x 3 - 3x + I crosses the

x axjs (Figure 10.3). There are three poi nts. aod we have been concentr.u ing on the i ntercept
between x = 0 and x = 1. When we rewrite the equation in the fonn .r = (x 3 + 1)/3. an
allemative inte rpretation is possible. We are.searching fo r x coordinaLes of points of intersection

of the curves y = x and y = (x 3 + 1)/3 (Figure 10.4).

10.1

ljC.,JJ);J.IoJ

593

A'' - lx + I - 0 as :ca>ordi.nak's c {
= ". y =(A + 1)/3

Sotu1~s of

puinh of iJllc:n.a:lion u( i.:UtVC$ ,\'

7..5

lnfinile Scquc:nccs of !"\umbers

'

J'

-3

-I

- 2.5

--........-

3 .t

- I

-5

-2

To show how 1he me1hod o f successhe approxima1ions works, we have expanded 1ha1pan of
Figure 10..1 between x
0 .33 a nd x
0.35 ( Figure 10.5). To fi nd x 2 we substitute x 1
1/3
into (x 3 + 1)/3. IIi~ the y-eoortlinate of the point A (>n the c nrve y = (x 3 + 1)/ 3. I f we
proceed horizontally fro m A to line y = x , then point B has coordinates (x 2 , x 2) . To find .<3
algebraically. we c\aluatc (x 3 + I )/3 at X2 . Geometrically. X3 is t11e y-coord inate of point C.
I f we mow: horizonta lly to the line y = x, coordinates of D arc (x3 , x 3 ) . Co ntinuation leads
to tlte sequence con\'erging to the required tOOL
We illu<trate the method again in the fo ll<>w ing example.

[11

0.35 )'

0.34

0.335

'
I EXAM PLE

... , 0.335

0.34

0.35

0.345

10 .3

Use successive approximations to approxima te the solution of j(x) = .t 3 + 25x -SO= 0


between x = I and x = 2 with error less tha n 10- 6 lll ustrme the sequence gmphically.

SOLU110N

The equa tion can be wrine n in the fonn x = (50 - x')/25, leading to the

sequence
X1 = I ,

.t ,.tl =

50 - .'"

"

25

?; I.

The nc.xt 15 terms are


X1

xs

xs
X11
X 1.i

1.96,
1.77.
1.776 3,
= 1.775 93,
= 1.775 948 2.

=
=

= 1.70,
X6 = 1.778 2,
X9 = 1.7 75 8 1,
Xa1 = I. 775 954 ,
Xas = 1. 775 946 9,

.lJ

x. =

1.80,

= 1.775 1,
x ao = 1.77600,
xu = 1. 775 945 ,
Xa6 = 1. 775 947 4 .
X7

The ..,rms ocrt:iinly appear 10 be 11'-~ling closer 1ogcthcr (llgn:cinglo six decirnal plac."<:> a11his
stage). therefore suu<Ning th.111hc sequence ha< a limil. We do fl(ll \crify !his. \~e hlwc \~h>ll
we fed i< an appnl\imation 10 the required solution of x' + 25.r - 50 = 0 wnh suffitJCI11
accuracv. To \trifv ch:u t.nS 941 is: a soluti on ''ith error lb~ chan 1 0~, \\C ~the zero

inu:rmcdiatc vulu;: ;hcorcm with the calculations

f( Lns 9~8) = 2.s x 10-'.


The tenn..: C\( the se<tUCtlcC urc ~ho\\11 in Figure 10.6.

Miili]lhl?411

3
2.5

I.S

We are bei n~ prcsentcd with 3 ehtmieal problem. A precipitate rcsts at th< bottom or a
beater th.11 holds volunt< V o f a n101her liquid. The mother liqu id IS 10 be remo...W in
tlu: followinloJ way Volume ii of pure wa1cr is ndd<d to the healer and the liquids are
mixed. TI\Cn V uf the mitcure is removed. -leaving volume V of J-l mixture trut t."'Olldi&
less n1othcr liquill thlu1orignally held by the beaker. This is called a dcc;lnL11ion. Our
problem i'\ to determine how nMny dec,mt.atjon.') are neces~nry before the umount of mother
liquid i ~ lc.'s thlll I % uf iL<- originul amount.

c.

SOLUTION What " e need is a form uhl


for the amou11t of mother li~uid in the
beaker after the uh decanunion. After the first addition of pure water, the concentration
o f mother ltquid i V /( V + V). Ati<r mix1ure is rcmo,cd, the nmount o f 1nolher liquid

remain in: ic;

c, = v -

v1

vii
V+ii

= .,..,.--~

V + ii"

After the addition of pure water for the (11 + I)" time. the conccntrJtion or mother liquid
is C./( V + V}. A fter mixture is remO\ed. the amount of mother liquid remaining is

c... , = c. - c. v_

V+V

= (

v-)c. .

V+V

c2 = ( V +V c.=

V"+l
(V

V)"

c,-= (~)c=
V + V

v>

- .
(V + V)2

<

V"

=>

( V .,.

100

1
nln(~)
< ln(- )
v+ v

v
+ Vl'

<

V)''

(V

100

-1n100

" > ln LV /(V

100

V) l

TI'IC snlallcst .suc.:h integer 11 scnbf)'ing this inequdlity i') lhe minimum number uf decunt1\
dons required. For insunu..'C. if V

100 cml and

,, >-

V = 200 cm

lniOO
In L10(1/3(l0]

1hcn

4.2

Fi''e c,lc(mlt4\1tons \\QUid be required

In F~erdses 1- 20 dccc:.rminc whether lhe 5equcncc t-s con,cr:!enl ur


divcrg('r\1. t1nd h n)iiS for tcn.. ergcnt soqucnccJ.

I.

f;I

l . ~~-

4.

3. (Jf

{(~f' }

' {(-::,)"''}

?. , ... (ti~T ) l

II.

12.

- 1,.,

nl

I
I

-r 1

1<- >J,;i'+l\

('-+

=
C', .-'

c.
u . r, = 4. c,,,, ::: - --;.

...

+ I

~. fG
r"J
(- I)"
9{ .

IJ. <1 = 2.

n ::_l
,, ~

p~!.-2\
"' - s
II!. \nc "I

{" ~ ' }

16.

I S. 211 ~ 3
17.

{ , , I Sn - }
n 2 -t- 211 -2

19.

11::1

20.

l +

.-!!-Tun-'ll\
1

11 2

+ :. + 2}

10. {Tan ' n I

Ia t-...M!f'Ct.....:S 2 1- !S Uraol W\ ophtll fcmnul.l f,wIN! ,trKr.ll tt.nn ur the

~qucncc

In cuch cuflC tSMnne thot lhc rcmainin' tcnns

,,,mern suttc:,tcd hy lhC: given t(nns.


21.

3 7

i ' 4'

-s

7 10
22. 4. ;j
9

IS
1(1

31

13

16

16'

25'

19
36' ...

ronow 1hc

2J .

'

ln2

In 3

.n

ht 4

- .jj' ,/4'

- h15

i 46. f (x ) = x' - 120-<+20; x, = 0: '""'-' = (x'+20)/ 120


li 47. /(.r ) = x ' - 2.< 2 - J.r+ l ; x, = 3: use.r=( 2x 2+ 3.r -

,/5

24. I , 0. I. 0. I. 0 . . . .

25.

l)/.<'

I , I . - I. - I, I. I . -I. - I. .. .

i 48.

Ill Et trtiSt!> 26-19 show hQw L'HOpitnl 's rule can be usod to C\ttluatc

28.

{ ~}

{nsin(;) I

(a)

nlC 11.

111

x3 -

ji _. 50. .\''

i a.
i
+ .30. If a sequence converses, I hen d1 0"erl.!necs bct"'ecn s ucce.~i,e rc..ms
in th:: sequence n1u,;a U1)1)I'Oaeh 7.Cft), T he COil'\a'SC is not alw;lys lr~o~e. i
The (ollowingc'C :unplc is an il luscnuion. Show til:." d iffercrt<;"~ bctwccn
i ""
succe~s i\'C tennsof thc sequence {In 11) approoch zero. batlhc u:qucnC'<'
J l.

.a9.

itscl f dhr l'g es.


priniC integer (greater

33.

Xt c

lx:lwc.cn X

= x' -

2x- I;

45. f (x)=x' +6x+ 3;

bciv.tctl .t = O aml .t = I

.x

= 0 illld x =

between x = 0 and .x

(Cl) Use. Newtoo$ ilcr;:u.lvc proc\!<.iun:: wilh .t 1

.t:" -

I 10 upprox.

ISx

+2

0.

U~;e.

i! Od

1\\.-u'ton 's method to ll:I)J'>fOxinunc the roo1 between 2

J.

(d) Wh:u h:tppcns if d1c seqm:nce in pan (b) is used to upproxim:ne t h~ roo! he1ween 2 ai\CI 3 with x 1 = 2 and x 1 - 3'?

S6. A supchun is d ropped rromthe 1op nr:abuilding 20 n'l high. Each

I= 0

(II) 1r J. denotes lh<: dir;t"r'K.'C Lro~vclled by l hc ball be tween the


, and (11
I ) 111 bounces. Hnd a formuJ~' for d .

= 0

(b) If ' denotes the lime between the n' 11 and (n .. I)"
bou!r'ICc s, lind o formula for r

22 :::: 0
IIC

22 - 0

x1

= 2;

x , = - 1;

57. A \logsib :ll ~1 rartnh\,1\1~ pal.icmly walchinc. for hi ~ llla~ter lO return


from the fields. When the fa rmer is I klll fro m UlC fannh ou~, the dng
imme-di31Cl) lakes Of( for the farmer. When he reaches the fanner. he
tums .:~nd ruru) bud:: 10 the fam1h o~. whereupon h:: ag.:~ in tums and

I'U ns 10 the rann~r. The dog continues lhis fmntic action tlltilttk: runne-r

rtl(l\(.8 th<: (anlnhOUSc. lf the dog runs twi<:c as nu t as the fanner. tlnd
liM" disult!CC d,1 run by rhc dog fi'Om the 1)1)in1 when ht reache-s the
Can ner for the 11 111 time 10 the poinl wllCI"' h e reaches the farmer for the
(n + I )" tintc. Ig nore any nccelcnuions o f the dog in the 1urns.

.. SS. lbc equilateral tnanglc in the left figure OCiow has pcrin}(:lcr P. II'
each side of lhe triMgle is divided into th1-ee equal pans. n equilatentl
tri o.ngt<: is c..b'uwn on lhc middle scgmenl o f each side. and the figuru

In E:~crci scs 44-48 iiJustrate that the method or successive approximations with the suggested reanangemcnt or the C<JUalion j'(x) = 0.
along with the inilhd approximatio n x 1 leads 10 a S<:quence that t-onverges to a root of the equation. Pind the root accurate to four decimal
places.

44. j(x)

0 and.\' = I

ti1ne i1sttil:cs tlliC g.round, it rebounds to 99% of the heigh! from \\hich
it fe ll.

li 38. x, = 2. .. ' - 3.t+ l =0


i 39. x, = I , _,s- :U + I = 0
i 4(). x , = 0. ' ' -lx + I = 0
li 4 1. x 1 = 4/ 5 , X;- 3.r + I = 0
i 42. x, =0.85, .r' - 3A + I =0
! 43. x , = -2. .<' - 3.< + I = 0

Iii
Iii

X =

= 0 und X =

between

(c)

x 2 + 3.< + I = 0

If 34. x , = - 1.5. x ' + 3x +


i 35. x, = - 3, x 2 + Jx + l
i 3(). x , = 4, x 3 - ).' 2 + x if 37. x, = 2. x 3 - x 2 + x -

btiWCC'I\

St . x +4.t' - 5Dx1 + 100x - 50 = 0

S.a. e' + e - T - lOx = 0

= 3 a11d .\' = 4

.x' + lx + I = 0

- 1,

1= 0

between

( b) Usc d.:: nlClhod <lf s ucccssi'c approAimtJ.li(ll!'l$ wilh x 1 = I


and .l'11 ..-1 = (x!
2)/ 15 10 appc"oxinlate the n'Mlt. in part
(u), uccun.tc to six dccimlll places.

term?

h.us a limi(. Approximate any limil th:n exists 10 seven dcc:irnal pl(l(,:-es:.

J2. x 1 = I ,

= (1/ 2)(1 +x'> "'

acc:-umtc to six dc<:im:~l p ltn~ .

l n E..-<crciscs 3 2-43 U.M: Newton~ itcrJ tivc procedure with the g:i\'Cn
initial ~pp rox im;t lion .TJ to dclinc i.t scqueMe of <ip(UW:i mollions tQ a
sol au ion of the equa1ion. Oe1crrninc gntphicnlly whc1hcr 1hc sequence

i
i

)X +

7= 0

illl<HC the l'u ot, lx:;tw..:.cn 0 and I , o f

List ils first I0 tcnns.


r1 111

+ ll x -

- 3.\' 2 -

th arl I ) wf'k:-n a ll such prime!'> an: listed i n 3$<:Cnding t~ iu.

(b) Can you give lt fonnula for the

6x1

.. 55.
IC'Oll Of a SCQUCtlCC is tll C 11

use .r

In t .xcrd scs 49-54 find a 1\:iifl~lgcmc: nt o f tltc cquaaion thut lcadli.


tltrough 1he n~1tlod nf .succcssi,c appro:<int;llion,;, to a fourdccimal
pl:.tCC :lj)Pf{U,im;'UiOl'' l(l th.; f(H.H o r the: l.'i.JUUI.ion.

lhc hrnit for the scqucn(.'C.


26.

- >'- 1: x, = 0:

/ (x) = 8.< 1

usex

= 2 + 1/ x

usex= - ( l/6}(x 3 + 3)

tr.tnsformcd il1tQ the middle figure, wh:ll is the pcrinletcr P 1 of thi.s


figure? rr each side of th is ligu(c is now subd ivided into tht'Cc equal

S~ uc ll..""eSOfFmll.:t ion~

10.2

portions and cquil:ncr.al triangles ~n: similarly t;On.structcd to n."Sull


in the right figure. what IS the pe rimeter P: or this figure'! If th i~
subdivision proceJS 1S continued indcliniu:l)'. wh:,u is the perimctcl' fl,.
after the n ~obdivision? \ Vhal is lim,,_...., P,J?

i + 59.

A stone of n\OSS 100 is thrown vertically up\VUrd \Vith speed

20 mit;. Air cxcr~ u rc~ist i ~ (Qn.."C on the ~ton e prQJXllt i,mal to it.:;
:;.peed. anti h a~ nutgnitutlc 1{ 10 ~when the ~pcctl of the stone i~ 10
nlfs. It can be du.lwn thatlhC hci~hl ,\' (1n metres) nbov<: the projection
point u.tu.irled by the 51()1)(: is g.i"cn by
,1'

th:!.Lis, F,, i~ :a liltcarcmnbinatioel of c, and theM lCtn\So imnx:diatCl)


bcfurc c,. . T he bo. ba. .... /)~,~ are f1xed ccn~tams. Cl'hc nmning
avet'aSCI'S in Ex(rci&e& 63 and 64 are cn\lsal FIR filters.) If cit =
(11{(11 + I) ). Af =2 wilhbo= l.b 1 = 2.nndb, = - l .culcululc
& hcfit~t I O t.cr~nsor(F.,} ")'.

66. Repeal E>coci:<e65 wilh Cn = ((1 / n 1) sin (n/ 3)). M = 3. b0 =


I, b 1
-2 . b: = 3. b3 = - 4 . Li~ttc:nn~ 1\')urldc<J 10 four decimal

plllCC~.

67.

-98. 11 + 1 18 1 (1 -

(a)

, -1"') .

WhCI'\: I is time (mCtlSUrcd in SCC.ood~ \\lilb 1


0 Ill the inSt!ll\t Of
pw.tection).
(a) n~c Litnc takc11 fo the !ICII''ICto fettu' tl co its projeclion point
c:.un be obwincd by scning y
0 a'Kl s.ohil.S the equation
few 1. Do so (COITCCt iO two dcc::imal ploc.os).
( b) Find the Lime for tile stone to rcu1rn if ail' rc~isumcc is ig~

597

= g(x)and
UlC &en 11gure below is o g.rO.J>h of 8 (.1'), lhcn the light figun:
exhib its gcomcui c:.nlly the se<wcncc o( U()l>roxinultions of
(I determined by the mcti"M\d of ~uccc;;sive '4)j>roxin1ations.
Showthat il'!>iS IIlconlyf004 of lheequ91iotl.l'

)'~X

)'

)'

,q (x)

norcd.

!! ...

60. When the beam in the figure below \'ibmtCii vcrlic::olly. tJ\.'::rc W'C C-cr-l<.l.in frcqucncic~ of vibr~tion . called nt.l lunllfrr.qut:ui'it ~' of 1/u: $,)'.Htm.
11lcy arc ~o l u t ton s nr the cqumion

(h) lllu~( I'11C gmphically hO\' the .;,cquence defined 1.1)' the
nlCthod of nc::ccssh-c '-pprc.ni m.'lli~ con"crgcs to the root
.r = ex fbr the cquution .1' = g(x) if g(..;) ib !.U shown in

the l-en nsun:: below.


di,idtd by 20n-. P'ind 1hc two smallest naturu.l frequencies.

J~,

IOm--1

'-l-~1123
x,

.+ 61. If/\ , i~ the orca

or the figure with J>Crin'!Cicr

P~ iA

E<crcise SS,

lind a forrnula for ..\,.


-+

62. Whut3rc the nc'(l two tcm'S in the sequence I. 11, 21 . 121 1,
Il l Z21. 3 12211, 131122ZI?

il.

63.

(a) PJot the (ir~t 20 IC11n!< of the ~juc:ncc: (C,J}, when: C"
( 1.02)" + O.S cos (rr tt /4

+ n {4 ) .

(b) In digital signal proc<:!::.ing. the M.."'C..llll'll{X' {AJ1}) defined


in tcnm of c. by A, = (c, c,_, + c.,-:)/3 i~: C.1llcd

u c:uu~u l tllrcepoint
terms.

rwmin~

uvc:ruJ:er. P1ol its first 18

il.+.

64. Repeat part (b) of Exercise 63 with u scvcnpoinc I'Umling ti\'CJ.'llgCt'.

.t

65. Suppose that ternlS a sequence {c,. } I'C prcscnt a discrctCli lllC
:;ignul. A FIR (tinite im1)Ulsc resp<:w'lse) fillet' is o ~equcncc {F,J) whosc
1crm.~ ate lirk:a combi nation$ of the lt:l'llll> or fc:,,}. Jt i:, eaus 01l if F,, is

or

or the rorn1

J' g (x)

~
I

123

.\'I

(c) lllus ora1c graphic.llly tha11hc nlCthod fnils for ohc funcoion
g(x) in rhc rig..ht figure abc.we.
(d) ll!lSCd on the rcsulls of p:rts (().H C). what determines S\1(.' s..~ or ft.ilurc of the rn<:Ul\Xl or .succcssi,c appro.ximruiOor\S?
Part (C) t>ro"idcs a proof or the COI'TCCt an~wcr.

(c) Pron: that \\hen .'f: = g (.\') h;.~ a root .r = &.the nlCihod
or succ:cs.sivc upproX..imut.ions wilh an initial approximation
c.1r x 1 alwuys romc1gcs tu CT i( lg'(.r)l :S ll < I on the

intcr\'_.l lx - I .:S. lxt-al. lnot h cr \\onl~. whcnan<:ql&tt


rion /(),') = 0 is rc~mangc..-d into the form x = g (x) for
the I])Clhod or Mtc.:c;cssivc 31>1Jroximations. SUC.:CaiS 01'fuilurc
depends u' whCA.hc.r lhe OC.rivativc of g(.x) is between - I
~nd I near the required rw r

>t>t- 68. Suppose that /'' (.~) exists on nn open inl.cl'val containing a root
J.' = Ct of the cqufttion /(X) = 0 . Usc the rcsuiL of Excrc i ~s 67to
prove that if f ' (o ) :ft 0, Newton's i tcrnti.vcscqucncc ~tlw;tyscon vc rgcs
to a provided the initial approximation .t 1 is chosen sufficiently dO'iC

too. Him: first use ..xercise 67(e) to show that Newton's sequence
converges 10 !> i l' on tlte interV"IIx - Ct I :E lxo - ~>I. Iff"/ (/') 21 :E
fl < I. (In actuaJ fact. Newton's method often works even when
/'(a)= 0.]

SM

Ch:ap1er I0

Infinite Sequences 3 111J Serit-s

110.2 Sequences of Functions


In many applica1ions, we encoumcr sequences of functions as oppo:.cd 10 sequences of numbers.
A scquc.nce of functions is 1he assignmenl of functions 10 pooitive in1egers. For insrance. rhe
first five functions in the sequence (x 2 + IOxe-x } on the imcrvnl 0 :: x :: I arc
x 2 + IO.u- , x 2 - IOxe-1.', x 1 + IOxe- 3'. x 2 + IOxe- 1', x 2 + JOxe-s.'.
l@lijii;IWIfi

Scqucoo: (x'

+ lOu ~ 1

fur 0 :!: ;r :!: I

Sequcoo: (;r'

+ lQu

~ ~ for - I

:!: .T 5 0

-50

-100

- 150
- 200

-250
They are ploncd in Figure 10.7a. As 11 ge1s larger and larger. values of x 1 + IOxr get closer
and closer 10 x 2 for all x in 1he imerval 0 :: x :: I. and we say 111a1 tlte limi1 func1ion for this
:.cqucnoc orrunclions is I (x) = .<1 . We wrilc lhal

lim (x 2 + IOxe- "') = x 2,

0 :: x :: I.

u-:x;,

This is also 1ruc for x > I, bu1no1for x < 0. Figure 10.7bshows 1ha1 for x < 0, 1hc funerions
take on increas.ingly larg.e negati\e value,s as 11 increase.s.

1t is impona.m 10 realize thm a sequence of functions comaios many sequences of 11umbc.rs;


simply subsrinne a ''alue of x imo 1he func1ions. For example, if we set x = I in the sequence
(x1 + 10.u- "xl. weubtainlhesequenceofnumbcrs ( I + JOe- ) . Theyaretltey-courdinate.~
of the poinLs a11he endsoflhe curves in Figure 10.7a. and lhescquenceclearly converges 10 I. In
faCI, for any nonnega1ive \'llhle Xo. Ihe sequence Of numbers fxt + IOX()I'- nxo } is visualized as
heights of points of in1ersec1ion of1he curves in Figure I0. 7a wilh the line .t = .ro (Figure 10.8).
Each such seque-nce has limit x~.
FlOUR& 10 . 8

ScqU<'ttce of poin1s obmined from a SC<JUCOC'C of fuoc-lions

~ "= (

I EXAMPLE

10.4

.........
Plollhe fir>t five func1ions of the sequence {(x - I)/[II
x

+ 11 2(.r -

I ) 2]1 on lhe inlenal -5 ::

5. \Vhat is the limit function for the sequence'!

SOLUTION The first live funclions are plotled in Figure 10.9. Geometrically, and alge
braically, il is clear thm

x -1
lim --....,.-,.----...,-, - 0
n+ '(x - ! )' '- -

n -;.oo 11

for all x.

The sequence in the next example is indispensable to fuiUre discussions.

I EXAMPLE 10.5
Sltow thai the limit of the sequence of functions {lxI"f u !j is ~cro for all

. lxl"

x: that is. verify tlutt

- oo < x < oo.

ltm = 0,
,,...,.(),J
IJ !

SOLUTIO:-< We can see this intuitively. As 11 increases by I. a fixed. extra 1-rHioctor>tppcars


in the numerator. but an ever incrc:tsing n1ctor n occurs in 1he denomimltor. When n surp:tsses

x. ~>e fra<:tion will get smaller and smaller. Get us prove this analytically.
Suppose that x is any fixed value, and Iet m = Ux IJ , where Lx J is the Hoor function
of Exercise 68 in Section 1.5. It is the lal):est integer that docs not exceed x. (If x = 3.4.
then [13.41J = 3; if x = -22 .6, then Ll -22.611 = L22 .6j = 22: nnd if x = -4. then
ll-411 = L4 j = 4 .) Since lxI" is lx I multipNcd b)' itself II times. aml 11! has 11 multi()lications.
we may write l.hat

lxl"
11 !

If 11 is chosen greater than m =

lxl"

(!::!) (~) (lxl) ... (!::!).


I

1t

Ux 1J , then

( lxl) ( lxl) ( lxl) .. . (~) (~) .. . (~).

11!

M =

c~l) c~l) c~l) .. . (';;1 ).

(~)
(m~)
m + J
+ 2 ... (~)
n

ut+l

Suppose we let

a fixed conM.ant Then

lxl"
n!

lxl ) ( lxl )

<M ( m+l

' m+l

...

( lxl )

m+l'

ll

+ 3~ .. . . n i n the denominators by the s mallel' i11tcgc:.r m + 1.

where we have.reph-t(.;C(} m +2. m


Sl nce rhere are

n- m

brac.i\ctccl l enn~ on the right, all cqual 10 each ocher. we have

lxl"
l )" -"' = M ( -lxl
- ,_ M ( -lx- -)-'"
u!
m +I
m ~ I

( -lx
-l- )"
m

+I

Now. M ll.tl/(m + 1)]_,. is a fixed number as fur 9S the l imit on, i !' concerned. Funhcrmore.
lx 1/(m + I) is also (.'.01\.'itant and ts IC"ss t ha11 1. Cott~tJ ent ly,

lx l"

. t\lf
hm - - < l1m
~~ -.o.,:. It!
- 11-c

( -lx
-l-) -m ( -lxl
- -)"
m - 1

. ( -lxl
= M ( -lxl
- - ) -'" ltm
- -)"
m + I
n - Oo m + l
= M

(_k!_)_,.

(0)

m +l

= 0.
Since lx In/ 11! ~ 0. it follows that lim,,._00 l..r I"/ 11! = 0.

EXERCISES 10 2

In E.:<l!.rci.ses 1- 14 h(:r) i'> 1he: ndl l<!nn in a seqocnc:c of functions


~ f,(.r) ~. Pine sraph;tOflhe lin;~ fi \'C funetiOM in the scq u cnc~ . Octc:.r
mine we,cther Lhc sequence h.:-.s a limit.

ii!

2. j.,(x) =

3. j.,(x)

ii!
i

4. j,,(x)

, , 0 S. ,r S. I

l+n .t 7

.0 < x < I
-

nx 2

- - - ,0::; ,r
I I II.<

= .!_ + .!_ ,;,,


.1'

i
il
il.

l+n:r--
Il l X
~

1. f,{.r ) "' " 'x"(2 - x) . 0 :::: x ::;: 2


H, f,{.t) = ll'.r"(l - x'), - I S X S I
2 + IIA' 2
9. j~ (.<) ~
' 0 :::: .r:::: 2

tlX

1::

,. I . j.,(x)

:r
11 . j~{.t) =(si n.<)''". 0:::: x S. :r

~~ p

s1

(...!...), I ::;

1 + tJX
10. j~ (.<) "'(>i n.t) 11" . () < .t <

il.
X

II X

5. j.,(x) =i7x"(l-x).O ,S .r,SI

6. j.,(.r) = IIA ( 1- .<).0 S..r S 2

S2

f,.(.c) =

13. j~ (.<)

~ ~ 14. f,{x)

C"7
;"" )''"

.0

r)' 0

in
= ( -;-

1
"

<.f

<

= n:txe-"t, 0 ~ -~

<

:T

A 5_ ,T

<

oo

IS. Porwhru valut>of.r dO<"hCS<'!""n<.'Coffur.:t;un/(1-.t')/(1-

.r)} ll>vc Hmil1

110.3 Taylor Polynomials, Remainders, and Series


,D'f( . !ti aLl .JJ'!U Q-.c;it1;).
ri(.)rK \ )( 11 pendulum

Figure 10.10 shows a pendulum consisting of a mass on the end or a string of lcngllt L . If the
nta.~'i is pulled slightly to ' he side a nd relcitsed. it ~wing~ b.."lCk und forth tQr :)Ome time to come.
11lc position of the mass c.an be described by the a11glt 0 that the slri11g ntake.< with tl!C vct, ical.

By ~natyzi ng the fon:es ~1c1ing 011 che tn~:\s. iL can be shown thai when Hirre.l\istance h; neglet.'ted,
0. as a function of time 1. must sati~fy the c.lil'f ert!nti al equ:.ttion

-d' &
+ -g
2
d1

smli = 0,

( 10.4)

where g = 9.8 1 is Lhc accelcr-.llion due to gravity. Thus, O(t) is a func:.tion whose second
derivative is -.rs/ L times the sine of itself. This is a very difflcull differential equation to solve;

in fuct. no t-ombination of the simple functions with which we are i11miliar satisfies equmion
10.4. What is sometimes dune in cx>~mplcs like this is replace sin (I with~ simpler function
f (8) that simultaneously fl(>proximmes sin 0 and for which the differential equation

d 1(J

dt~

+ L /(8 )

= 0

t 10.5)

is solvable. !\Ithough the solution of equation 10.5 only approximates the solution of' 10.4,
if [(8) :lp(>roximatcs sin iJ very closely, the solution of 10.5 may he s ufficiently close tO the
solution of 10..1 to give real iriSight into the n101io1Lof' the pendulum. In p!uticular.as we shall
show shurdy. when I) is very s mall , sin 8 cru1 be approximated by 8 itself. Tn other words. for
snutll oscillations of' the pendulum, equation I0.4 can be repi;K:ed by

h is struightforwnrd 10 verify that


8(1 ) = A

cos/ft + If/
B sin

is (I solution of equation 10.6 for any conswnts A and B whcltsoever. Tf motion is inititHed at
time 1 = 0 by pulling the mas. an amount 80 to the right w1d then releasing iL then 8(1) must
satisfy rhe arlditional conditions

9 (0) = 80 ,

These imply that A

9 '(0) = 0.

= 80 and B = 0, and hence


8(1 J

= Oo cos

Jft.

This function yields simple harmonic motion for the pendulum, as we expect.
IJI this section we show how to approximate complicated functions by polynominls. Tite
intcr,tal u11 which the approximHtion is requlrcU ar\d the a<;curacy of the approx.inn1tion dicuue the

degree of the polynomial. 10 begin with. we recilll the Mean Value TI1eorem from Section 3. 14.
Tt states thai when

f (p\')

i.s contimtous on chc closed interval bc.tween c and some given value

of .r ~and f '(x) exisc.s un the. open interval between c anti A' .there ex i_sts a number between c
~nd ..t ,

C:llll it Zo such that

f(x)- j(c} = f'(z,) .

.r- c
or.
f(x} = f(c )
This theorem

W'dS

+ J'(q,)(.r- <') .

t 10.7)

a corollary tu Cauchy's generalized mean "al ue theore.m in Seuion 3.14.

We gi\'C a direct proof of it here because a similar but more compliemed argument leads t.o an
e.:<teusion co_
tlled Taylor's Remuincler Fqrmultl. a rc~ult 1h~1t is fundamental to our :;cudic.'i in this

chapter.
Con,ider the following fu11c1ion of .V for Axed c and x ,
F(y)

= j(x) - f(y )- ( x - )') tf (.r)- .((c)) .


.r -

Because f is continuous on tlte closed iotetval between c and .x. f(y ) is conti nuous for yin
this interval. Funhermore.

f ,,(y} = - .f' (y)

+ -I-[f (x) - j .(c)].


x-c

Since f'(x ) exists on he open ime"al betw"'n c and x .so also does F'(.v). Finally, F {l-) =
0 = F(.t) . Rolle's Thoorem (Theorem 3.17 in Section 3.14) implies that there exists a numb<:r
:0 between rand x al which
0

F '(z(l)

= - /'(~0) + -

.t -

- [f(x)- fCc)],
~

and this gives<'quation t0.7.


Suppose now thal f(x ) und / ' (x ) arc contiuuous un Chc ciU-..'-C\1 inter' a1 betwee11 C and x,
and ,{" (,1') e<i~t< ,..,, the OllCII interval b<:twccn c and x . Consider the following function of .v.

F(r)

= /C-1 ) -

f (y)- ,('(y)(x- y)-

(xr--cv)!
- -

[.f(x) - { (c) - / '(c)(,t - c)J .

Because /(.) :111cl .f'(.r) are continuous on the closed interval between c and x. F (y) is
COI\tinuous for)' io chis iHelVill. Fun.-hemlore,
.
F' (y)

= - f'(y)- /"(y)(.r - y) .._ f'(y) +

2(.r - y)
(.r -

c )2

'

(/ (.r) - .f (c ) -

'

(c)(x - c)j .

Since f'(.r) is corninuous on the dosed interval between c and .r. and .f''(x) .:xits on the
open intc"l b<:tweerl c and .r. it follows th~t F' (y) exists 011 the open interva l. Finally,
F (c) = 0 = F(x) . tlnd therefore Rolle's T hcureon implic.' the existen~-e or a number : o
between c and ,\' at which

'
2(.1' - "')
.1 "(z,)(x - :.o) + j'( ~,)+
)l [f(.x) - j (c) -

= F'(z o) = - /'( Zrl -

(.r -

'

(c)(.r - c)l.

Thi$: c.an be rearrangetl into the fonn

f <x)

,
= f(c ) .._ f ' (c)(.~ -c) + -J"(~J)
-(x - c).

(10.81

By a:;.~un1ing that f (x) . ,{' (.t). Md f " (.r) are coouinuous 011 the closed inter\>II b<:tween c SOld
x. andf111(x) exists on the l)j)el\ imerval, it cm he "'huwn in tl ~imi lar w~ty 1h.u there e.x.ist..~ a
number ,: 1 bel\vccn c and .r such that

f (x) = f (c)

+ j"(c)(x -

("<c)

c) + ~ (x - c) 2

l"'<z2)

(,r

- d.

( IO.YI

These re.~ults can be extended indefinitely if we assume that f(x ) has a sufJicient number of
derivat ive.~ on the c1used intenal between c anU x. The cc.mlplele result is c<uuaillCd in the
following theorem.

TH EO REM 10. 1

(Taylor's R e mainde r Formula)

If f (x) and its Jirst 11 derivatives are coooio uous on the closed interval between c and x.
and if f(x) has an (ll + l)m deoivative on lhe open interval between c and x , then rhere
exists a point t.n between c and x such Lhat

f " (c)
f "'(c)
f(x ) = f(c) + f'(c)(x - c)+ - -(x - c) 2 + - - (x - c) 3
2!
3!

+/

(11) ( )

c (x - c)"
ll!

+ ...

J <t+l)( )
(11

'"

+ 1) 1

(x - c)H.

( 10. 10)

Th-e notnti(tn f(" )(C) I'C'J't'~CilU the 11*' derivative U( j(.t ) C:VIIIUa t< lU ,\" = C .

In rf1c renmiru lcr o f thi.; II<I:'C(ion, we: &."l>lU11C lh<tl /(.tt) h~l~ der iv.lti\'c:~ o f t1ll ()l\)et6 o n the
do~cd intcrvalllcawccn c ancf St)nl<: v.:lluc o r x. Tayl<us remuindcr fo n n ul;a ctul dlCtl be writtcu
down for al l v;~ fue~ of n . f'or n = 0. I, 2. arnJ 3. we:. OOC.nin

f(.<) = j'(c)

+ f'(t<>)(-< -

j'(.<) = f <c )

f''(.,. )

+ l f - < x - c)' .

f'(c )(.< - r)

f<x) = [ (c )+ /'(c)(,<- r)
.
./(<)

( 10 l l 111l

<') .

/""(c)

+ 2 ,(-< -

= .{(c)+ f'(c )(. - c ) + -/

(10 Ill'!)
J'm( t.'J;)

c)'+ ""')!(> - r ) 3 . ( IU ll c)

"(c)
'
J"'(c )
2 !- (x - c)'+
3!

- -<x -

c)

'

).
--(,. - (',
+ -/ ""(:,)
4!

(I O. IId)

The Ja$1 terrns iu these (.'qu tu ton$ are called n,ylo rtmulndus, an<.l when the y l)re denoted
R(). R 1 R: . a1Jd R \. rc~> pcc l ivel y.

(1 0 12a )

/(x) = /(c)+ flo,


/(.r)

= [(c)+ /'(c) (,< - c )+ 1?1

( 10. 12b l

j"'(c )

f(.r) = [(c)+ /'(c)(.r - c)+ ""'2!(x- c) 1


/(x)

= /k)

by

+ 111 ,
j""(c)

("(c)

c)+ -

-'- / ' (c)(-r -

-21 <x - c)' + -3!- (x

(I ll 12c)

c)'+

II; . (1 0 12d>

amJ in scncrnf,

j(.r)

= j (c) +

Ji">(c)

+-

/ "(c)

f ' (c)(x - c) + - 2-!( x - c)2

--(.A' -c)"

n!

+
( 10 13)

R11

We c:tll R, 1he rcn'lllinder for rhc simple I'Ct\Wtl rhar, if we drop the rcmaimJeno from equations

ro. r2, rhcn wluu remains is a seque1c::e: ofpolynonlial appro:<i mations to f (x) . called the Taylor
poJynontiufsu( / {.r) about c :
f (x) "' P0 (.r) = j(c) ,

( 10 14a)

f (x) "' P1(x) = /(c)+ / ' (c)(.r - c ),

(1 0.1 4h)

j' (,t') "' P2 (.r)

= j(c) .,. f ' (c)(.r -

f (x) "' P3(.r) = j(c)


The

,m

{"'(c)

c)+ - - (.r - c) 2 ,
2!
j'"(c}
,
f'' ' (c)
f ' (c)(x - c )+ - - (.r - c)+ - -(.r- c)).
2!
3!

Tt1ylor pofynomittl P,1(.t' } i:i il polynomial of degree

11 . WhatlmpJ>tll~

( Ill 14c )
( 10. 14<1)

i n pn1ctict: i~

thar as n irK:rea.o;es, the pulynorniuls P,1 (.r) apJ>roximutc /(~)more nnd IIK>r'Cclnscly on some
finite or intinirc intcrv1ll (.~Onti-lining c. The fOl lowing examples illustrfltc thi~ .

I EXAMPLE

10.6

Find Lhe fi rst six Taylor polynomials for sin x about .r

= (). Plot Lhe polynomials and sin .r

show how the approxi mations improve as more terms are included.

to

004

Ch;ac)ccr 10 Infinite Sequences and Series

SOU J T10l\' 111e value of f(x)

=sinx and ils firs1 five cteriva1ives a1x =0 are

f(O) = 0, j'(O) =cosO= I. .f"(O) =-sinO= 0, f "'(O) =-cosO= -1,

J'')(O) = sin 0 = 0, J lll (O)

= cosO = I.

The first six Taylor I)Oiynomials are

P0 (x) = /(0) = 0,
P,(x) = /(0)

+ f '(O)(x- 0)

= x.

l'2(x ) = ((0)
'

+ / '(O)(x -

+ -2!- (x

0)

J"(O)

- 0)2 = x.

P3(X)

J"(O)
/'"(0)
<>
2
/ (0) + / '(O)(x - 0) + ~ (X - 0) +~(X- 0) 3 =X - ' !,
3

P,(x )

X-

xl

J!
x3

Ps(x) = x - 3!

x5

+ -.
51

Because cvcn-onlcrcd derivatives of sin.t' vunhh. even-numbered polynomials are the same a.'i
lheir odd predecessors. We have shown polyolomials P1(x) . P3 (x), and P5 (x) along wilh
sin x for posili,c \'alucs of x in Figure 10.1 l. We have included P7 (x) and P9 (x} to further
emphasize the following fac1s. The higher the degree of the polynomial. the more closely il

spprox ima1es sin x for small values of x , and the large~ the interlllll on which Ote approxintation
is n::onablc.
T'.t)'lor polynomi..tb. of sin x

I EXAMPLE

10.7

Find the first live Taylor polynomials of <''' about x

"' 0 . Plott he polynomials and e' .

SOLlJTlON Since all deriv01ives of e at x = 0 are equol 1o I, !he fio'St five polynomials are
Po(x) = I,

P1(x) = I+ x.
x2
P2(x) = I +X+ - ,

2!

T X

_yl
.2!.. -

~ 2!

'"'

~
1

3!

x"
+ -.
4!

They arc illuS(I'nlcd in Figure 10. 12. When X > 0, !he polynomials arc ruways less tbao e' and
approach e' as more tem1s are included. For x < 0. the polynomials are altema1ely higher and
lower than e t . gradually getting closer and do~r to er.

I EXAMPLE

10.8

Find lhe lir$1 five Taylor polynomials for In X about

I . Plo11he polynomials :1nd In X .

SOLUTION Since
.!_ 1

= I,

= 2,

~.tx ln .\'~t=l

.t x-1

d3
- 3 lnxlx= l
d.x

2
=-I
x' =

d'

- 3
.\'3.

- -I

x 2 x-1 -

-1

I.r= = - 3!,

lhe fi rs1 five polynomials are

l'o(x) = 0.

P1(x) = x l', (.x)

I.

(.x - I) - 2
~(x - I ) 2.

I
I
.
P3 (.r) = (x- I)- - (x- I ) 2 - -(.r- J)'.

P4 (x) = (x - I)--(.< - 1)2 + -(x - 1) 3 - - (x - 1)'.


ntey are ploned in l, igure 10. 13. For 0 < x < I, the po lynomials are greater lhan Jnx and
approach In x as more te rms are included. For 1 < x ::; 2 . the polynomia ls are alte rnately
higher and lower thao lox . gradually geuing closer aod closer to In x. The plot of 1he Taylor
polynomials on the interval I ::; x ::; 3 in Figure I0. J4 indkates that they do not approach In x
for x > 2.

1#11~111;1418*""'

Tayk>r pol)'fttllni,ds o( In x on 0 < ,\'

,.,

p>

zx

IS

-2

IJ (!liiJI T.i).1ur pul).110n1ials of ln .t Qn I < ,\' :$ 3

p f'~
2

Po

-I

'

PJ

Inr

-0.5

-l

To make this clearer, below is 11list of the first 30 Taylor polynomials evaluaJcd at x

= 2.1.

A t fif8tthcse numbers get closer together, but then they separate. \ Ve have sh.own them in Figure

10.15.
0.0
0.59948
0.610 38
0.58443
0 .531 23

1. 1
0.877 87
0.875 94
0.90632
0.96462

0.495
0.609 92
0,(,0569
0.56994
0.506 23

0.93867
0.871 92
0.883 17
0.922 33
0.991 79

0.572 64
0.61254
0.59599
0.552 32
0.47675

0.894 7J
0.871 92
0.893 3 1
0.94164
1. 023 75

T8)'10t' J10l)rnOntials of InA' <"''llhUIC:d Ul X = 2. 1

~, (2 . 1 )

0.8
0.6

0.4

..

O.l

10

IS

25

II

The plOIS in Examples I 0.6-10.8 support our earlier contenlion that as 11. incrc11ses . the T11ylor
polynontials of a function approxi nll:ue the function more nnd more closely on some fini te or

infinite interval. We would like to be able 10 verify analytically whal we sec geometrical I)'. To
do this

\\C

rctum to the Taylor remainders. The.: u 1h remainder is

(10.15)
where Zu (which, as Lhe notation suggests, varies wilh n) is aJways between c and x . We have
wrinen R,(c, x) to emphasize the fact that remainders depend on the point of expansion c, the
value of x being considered. and the choice of 11. The sequence of remainders {R.,(c, x)) is a
sequence of functions. Jt is these re.mainders that we drop from Taylor's remainder formula in
equations 10.1 2 Lo fonn Taylor polynomials in equations 10.14. The polynomials in 10.1 4 will
approximate f (x) more and more closely as 11 increases if the neglected remainder.; R, (c, x)

""' smaller and smnllcr. In Olhcr words. to verify algebraically thai the stquencc of Taylor

Pol> nomjal~ ~'pproxin-.aces a functi01\ more and more d osdy with inc~asin1 11 on an Lnterval
/. il lS s.flkient to >how that for each .t in / ,
lim R.(~. x)

-"'-

(1<} ' 6

0.

lnletval I h. cnlkd tho;: lnttrvul ur COilVCI'gt:fl(t. It i~ tcdiQIH tu \\fit< t;I'OUI~ of equations


like 10.1 4 ;u'ld SU) thtu . the more and more tems that nre. adllctl. chc beu.c r un..t bcncr i.s the
approxinuuion. We ilwtt\1 u nutation to ~h\nd for th is. \Ve wrile

j(x) = f (c)

j"'(c)

('"(/')

+ f '(c)(x- <') + -2!- (x- c) 2 + - -,-(.~


~.

-c)'+ .. . ( 10 l7l

The . . indtcate thitt a1klillonal temls of the scune fonn arc to he nddcd indefinhcly. This ls

callccllhc Taylor <erii'S of f (x) about the point c. "Be clear Oil" hot we mean by I 0.17. because
it cannot be h\~cn litcmlly. Tlk..-rc '" an infinil ) of tcrTn$ on the rig,ht '\itlc or the equtnion. tuld
Ustitution nf a \Slue for A (und c) gi\ c~ an infinity of nurnhet's to odd. No matte r ho\\ fB:-tt you
add. your calculator Bdds. or a ~uper computer adds. an i nfinit) or nunlbc"' cantlOI be added in
a fi ttite amount or time. F..quuuon I 0.17 does not therefore mean th1U odding the tem1~ on the
ri~t gi\t> f(~ ). h n~earb >ll) one of the following 1hree equivalent lUllcnwnb .

J. The mon: and n>O<c torn th.u ore add..-d on the righ~ the <lo:.cr wl<l closer tl~e >Unl gets to
/(.r) .
2. By adding >ufhcic:ntly many tcmts, we can make the sum on the nght close 10 f(.<) as
dC$ircd.
3. If the Tl\) klf sc:ric~ iii ln.tnc.uted m ony nunlber or 1Cl'I'US. the ~ ...ulling T'Ay lo r PQI)1t1Qillial
Ol>proxin>Aics .((X): tho lt\rgcr n. thc bencr the appC"oxin\1\lion.

To u.c lhe tcrmino lug) from Sccduo 10.2. we say Lhat thc Tn~IO< I>Oiynuminls P,(x) converge
to f {x) , meaning agoi n lh>llthcy get closer and closer 10 f( x) . the IU'l!CI the \l~Ue o f" We
also say tha t the Taylor serit> converges to [(.t}.
When c = 0. the Tn) lor stric.< bec()mes

/{xJ

/ {0 )

+ j'{O)x + ["(0) x2 +
'2!

/"'(0) x'
3!

+ ....

( llllh)

We callthi the ~ln<luurio "'-'ries lor f (x ). h consists o r po"crs of x nothcr thom x - ..-.
To oblaJn the Tu) lor (or Macllurin) ....-ies for a function f (x) . " evaluIC II <lcthathe.
c (or ' = 0 ), '"lei u>e fomnla 10.17 (or 10 18). To ,hQo.Y 1hat the scri~
oonvcrgec< to / (.<). "e slto\v th>t1 Taylor remainders appC'Cia<h 0. an<l we mustlll<ln interval /
on which this hilppc11>. Thi' can be <!lli1e lli01cuh beclluse R, (c, x) is tlclined'" terms of :~11 an
unknow n value exccptlhat it must he between c and x. 1l1is problem is USlMll)' circumvented
hy fi ndi ng a mHximum v~tl ue ror IR,((', x)l . and $howing that this nuudmun\ vulue appro01ches
zei'O. We illustrmc in lllc next three cx.tmplcs.
of f(x) at x

I EXAMPLE

10.9

Find the M oclauri n sc rie~ for Sin :c mu.l ~w lhat it conn~rgc~ tu ~in ,., for al1 .\ .

SOLL,ON

U5ing the derhativcs of sinx in Eumplc 10.6. \\C Ctln wrote TaylorHcmainder

fonn ula for <in x and c


~in x

- 0.
xl

x5

x1

= x - - + - - - + .. + 3!

5!

where

R.(o, x)

7!

d"

dx"

(sin

"
r)l
:._
+ R (0 x)
,-o !
11

11

But the (11

+ I)'" derivative of sin ,< is sin x or cos x, so that

I-dxn+t (s i11 x ) I..-=:,.


d"+'

I : ; I.

Hence.

I R., (0. X )I ::::

lxl"+'
(n +I) !

But acoonling to E.xample I0.5. lino, .... 00 lx I"/n! = 0 for any x whatsoever, and therefore

lim I R.(O, x)l = 0

11 -

0()

lim R.,(O. x) = 0.

n- oo

1be M aclaurin ~erie.'\ For ~in.:t lherel'orc converges to sin x ror ali A. and we ma)' write

sinx

..r3
_.s _.1
=x--+---+ ...

3!

""

S!

7!

( - 1}"
+ I)! x 1n+l .

-"
- L.., (2n
n =O

and Lhis is true for all x; that is, the interval of con\'crgcnce is - 00 < x < oo . Let us not
forget what we mean by w1i 1ing the M aclaurin series for sin .x. Tf the series is1runcatec.J at any
value of 11, a Taylor polynomial is obtained. These polynonoinls approxi mate sin x more nnd
more accurately for small x. and the interval on which Lhe polynomials approximate sin .r wi1h
any degree of accuracy geL> larger and larger. We illustrated this graphically in Figure 10.11.

Tn a similar way il can bc shownthallhe r..1aclaurin series for cos .r is

~ ( -1) " 2n
.<2
OOSX = L.., ( ) 1 X = I - - ,
211
2
n=O

x'

_.6

+ 41 - -6 1 + ,

- 00 <

< 00.

Sometimes it is necessary 10 consider points on either s ide of the point of expansion separately.
This is illustrated in the next example.

I EXAMPL E

1 0 . 10

Find !he Maclaurin series for e" and show that it converges to e' lor all x.
SOIX TION Since

d"
dX

(e-')1

.1'(1

= e\=0 =

I.

Taylor's l'elllainder fonuula roo ex and c = 0 gi,'eS


x1

xJ

x"

+ x + -2! + -3! + + -11! +

R.(O. X).

where

dn+l

R.,(O, X) = d .t"+I (e")

x"+

xn+l

1- ( +
..~:-

Now, if X < 0, Lhen X < Zn < 0, and

IR.,(O, x)l <

....

11

J} l.

e"'.....:;-...,...
(11

I}!

whic.h approaches zero as n bc<:on1es infinile (see Extunple I 0.5). If x > 0. then 0 < z;11 < x.
C'llc.l

l.rl+l

c (II + I) ! .

IR.,(O. x)i <

which ;1gain hu.s limic zero us 11 ~lpproaches infinity. Thus. fo4' any .r whatsoe,er. we can sar
thm lim,~-oo Rr~(O. x)
0. and the Maclaurin series fo r elf cotwc-rgcs to ex:

x2

e...

oo

x3

= I + x + -2! + -3! ~ =

~ x.
L..- 111

-00 < X < 00.

- ~u

Sc1>ara1c discussio11s for x < 0 and x > 0 ~1re reflected i11 how the Taylor pol)nomiols converge
to er in Figure 10. 12. For .r > 0. polynomials are always less thtln e-'' ~nd incrc~lsc toward et .
For x < 0. polyr1omials ure alternately largu arld smtlller thnrl e~~: ) gradually appn.mchirlg e e.

The following ex11mple is more Uinicuh. Renwi11ders <.lo not ap1>roach 1.ero ror all .'< so 1hnt the
Taylor series does not corwer-ge to 1hc functiom for all .r. 'Tlle ptOblem would have been e' en
more ditlicuh had we not suggesc.ed va_lues of x to consic.Jer.

I EXAMPLE 10.1 1
Find the Taylor series for In .t aboUl the poim 1 and show that. for 1/2

.x

2. it tonverge$

to lnx.

SOLLJTION Using the derivativ~ of In x at c = L in E,'runpl e 10.8, we obtain Taylor's


remainder fonnub1 for In x !}Od ~ = I ,

I
21
( - 1)"+ 1( 11
ltu = (,r - 1)- - (x- 1) 2 +....:(x- t)'+ +

2!

3!

n!

1) 1
(,r-l)"+ R.,(l. x).

where

d+l
(x - 1)"+ 1
(- 1)"11! (x- I)"+'
R.,(l, x) = - -(lnx)l
= (:;.,)+ I (11 + I)!
dxn+l
.... .;.. (n + I ) !

(- 1)" ( x - 1) "+
n + J

and

z, is between

I ancJ

Zu

x.

If I < x :::: 2. then the largest value of x - I is I. Furthcmtore. z., muSt be lnrgcr lhftn 1.
It follows that

IR.,(I, x)l < -

11+ 1

(~)n+t
I

11+1

and therefore
lim R.(l , x )

-""

= 0.

If 1/2 :;: x < l. then - 1/2 :;: x - I < 0. Combine this with .r < z,, < I. and we can
state that -I < (x - 1)/z., < 0. Then,
I

IR.,(I, x)l < - n+ l

and

lim R., ( I . x ) = 0.

n -'>00

Thus, for 1/2 ~ x ~ 2, the sequence of remainders ( R. ( I , x )} approaches zero, and the
Taylor series converges to In x for those values of x:

ln.t (.r - I) -

1
it'
-

I) 2 +

)(_' -

L"' ( - I) (.\'- 1}".

I)

s ,\

+
~

2.

II

11 1

111i.s ~cncs actually cotwcrve.s lO In .r Oil the lat1!cr intcrvotl 0 < \' ~ 2
in Figure 10. 13. We will !!.~tow this a l~ebn\icn.ll) in E"t~uupte 10.22.

i' S

\\C saw gn\JJhically

llle abo'e e..xaruplts wtr~ chos.tn for thetr sintplictl) . It i~ l1R1ple to find 11"' deriv~ti'e"' of
'in ,,. r'. and lnx: that ~amdcrsappmoch u:ru fuiiO\'" tmmcdiacely. Yuu nli.t) di~ot;rc:c thlt
the examples wue sin1J)Ic. but 1 h1~ ()Illy ser\'ts to em)>hastve tt\e fc~llowin& point. If ' hebe ure
1he siRlJ>Iest J)OS-'liblc CX1:4nlplet, 1\fKi thcy arc 1\Ut simple, imttgine 1hc Uinicuhy in finding :.nd
i h OWillS that Taylot St:l'ic=,. or other function..\ convc:tgc: to the: function. The problem is liO grct
chat we do our uuno.'l Hl avuhl cnlculating de.rivalh c~ mtd con:ro.itlcnng. rellla iltler~. n ased on
discus~ioets iel Section lOA. we will find tlhernat i' c~ in Scclit)l\ IO.S.

EXERCISES 10 3

16. In Soctton 4.4 we ).\.lk'd lhr 'Cond cktiw;a.tive teSI Cor ckKnnttll,.
wht'thcr a cnbcal po~nl \ q .... ...tch /'(s.) - 0 )l1dds a llllth\C

In E"-CI\:UCS 1-S usc 'lo~ylor" rc:nuin.krfunnula to show ttul the= 1o~y


kY ~ries fOt' lhc fw-.:tm /(X) aM the pcMIII ndiealed (.'00\CfJ,C\ IO
/(A) ftv nil x In cadi usc. piOilhc fnt J.i'< T~ylor" polynlwnials to
tJ~ ~ the) approxirtm:Lc /{.t) llll.)ft' c.:lm,dy us l.hc dc'rcc of tt.e

rnuumam or u rd.lli\'C nlirumurn Uw: rll) lor's rt::n'lal.adet rontulo~ to


\\:rify this I'CSfllll when {'(.\) lAd r(A) :tfC <.'01\liftU.()Il$ ()'n ltn 011)..-on
lfiiCfV;tl ront.2:i.ning x 0

ptJi ynuntiOIImcruscs.

j . I, j(x)- <><K aboooo .r ~ 0

i + 2.

j(.r) = ..'' nbout x = 0

il + J.

j(.r) <;n (IO.r) oboot r - 0

iJ

4. j(,o) <;n.r

$, j(r) I"" 000u1 r

;.boo).

.r- n / J

17.

~~lend

the l"C"IJh of fJCoo:lr.c 16 ttl vcriry the cx1rcma IC'-t C)( Fx

etcise 3S 10 Sc<.:tion 4.4.


' Ul 'Olcrc i:o an inlcgtal fotm fN" Ihe rcn\;\indcr R~ (c , x} in Tu)'IOC''s
1tnutindcr (omJla ttut is sometime:" more tl-w=fulthan 1hc: ~olerivuh\C
(()rmir.Theorem 10.1 : If /Ct ) .a~dil~ li~ 11 c.l.--ri n:.tivc~ an:cotui~

ro 1~ (bo;oJ interval k'l"ccn rand ' . then

/(.r)
In Ewciscs 6-1.5 find ohc T"ylor ...,<> (Ct' d'OC lurt.1;M/(> ) uhOUI
r, PI(! I enough polynumi:~~1s a.nd II"K: (unt.i iun IO dcta-n\il'lc the
JncC'rvol on which the series convc.rgC$ to lhc function.
trk: rxnl

R.,(c,x)

7. j(.r) = ~>3x.c=O

iil
i
i

8. j(r)-.,n.r .c ~ .T/2

i,
i.

T ...

whc:l\:

Il l 6. /(.r)sa'' .c= O

U~

/"(c)
+ --(<)'
,, !

R,(c. ~).

= .!..J' (.< -l)"j<-)(l)dl .

the following Ot-tlinc: tO Pf'O\C lhi..; rtfi;Uh.

<t s._ <h&<

=I

I<>

11. /(.t) - . /(l+lr) .c-0


12. j(.l)

/"(<)
<) + ~ (.r - <) 1

n! ('

'/h)~ 1/(1-x).c -0
10. /(.t)"' 1/(2- x ) . c

= /(<) + f(r)(A -

1/(1 + 3x) 2. r

=0

j , 1.1. /() .... .... c = 2

14. /(X) & ./1+3x.c=O

IS. /(.r)

= 1/(4 + .r)0 ' c = 2

/(<) ,

, .(,)dl.

(b) Usc intcgndoon by 1 n with u = /'(t ). du = /"(1) Ill,


dv = dt. unO 11 = 1 - .x on lhc integral in Jlllr1 (:1) to
oblliin

/(x)

= j'(c) + j'(c)(.r- c) +

1x

(x - 1)/"(l)c/1.

1(1.4

f (c)

+f

"

r<<>

" _ , , ~~

' (c)(.<- c)+ 2 !(.< - c)'

lim - -

.r-<t

' ['

+ 2T (

(,x -

6 11

(b) U~;c I~' Hi'lpit:~rs mlc 10 s how th:;at for C\'Cry p@;iti,c integer

= J"(t), i/11 = f"'(l )tll,


( c) Csc intcralic.mbypatt!>withu
.
du = (.r- t)tlt. wu:l t l = - (l/2)(,t: - 1) .. IUll-bmin
/(X) =

~wcr Series

,'(N

= O

t ) 1f"'(t ) ilt .
(c)

Prove. by nlstheuu!Lk'al ir.duction. tMt J<"'(O)


,

(d) Co111inuc this p roc:t:~S foobUtin lhe i mcgral fo-rm (orTuylor's

0 for

I.

rc:m:t.indcr romndu.
.... 19.

(d) Whtn i ~ the Madmu in M!rics; (or f (x) 1

(3) On1w t gr.11)h o f the fun c1ion

j(X)

.-J.'

0,

I 10.4

.I

"' 0
'

X=

f~ ) Fur what \':11Ut5 (lf' .t di.l4:~ Lhc Mad;,uari11 !)Cries o f /(:<)

c;:onvergc to f (x )?

0.

Power Series
Our work; Ofl T~1ylor series in Seerion I0 .3 begun w ith 3 ftulCtion and d evelo ped Lhe Tayk>r (Or
M aclaurin} ser ies (m Lhe function, a :;c:quence of p olynomiab that approximate the: functi on

rnore a_nd more closely ~lS more and tnorc temtS are h\cluded. 111 th is section we bcgil w ith a

series called ~~ power :-.eries aud nsk two <Juestion~: FOC' what values of .t docs. the powe r series
converge. :and to wh~at func tion docs it <.(tn\el'ge'J
1\ power series jn .t is an (intinile) series_ or the focm

""
I;

a11.t

11

= " O + o,x +

.x1 + + n".\'

a2

11

+ .

(1 0.19)

rt~o

w here coefficien ls tlu are constant". For exmnple,


00

L
x" = I + x + x' + x3 + + x" + ..
,,. o
and

arc power series in .r.


' lb My thtu the Taylor series
f (x ) = f (c)

+ f' (c)( x

- c)

f"(c )

+ -- ( x 2!

c)2

+ ...

of a function f(x ) converges to f (x) is to suy thatthc sequence of1'11ylor polyoomials I P, (.T) 1.

where
l',(x)

f( c ) + / ' (c){,t

-c) + +

J<l (c)

n!

.f (x) ; that is, as

(x -c)".

more a nd more terms or the series are included. the closer and
closer the sum gets 10 f (x). We use the same idea to dellne what we mean by convergence
of power series I0.19. Power series I0.19 is said to convege 10 a function f (x). OJ' have sum
f (x ), o n an inlerval I if the limit of the sequence { P, (x) I of polynomials where
converges to

P,(x) = ao

+ a 1x + + a,x"

a~/(.tJ forcach.A in l . lthuchacascwe\\ritc

...

j(x)

a:

Lo"#'' tJc + a.x + ..o

Cln-t

+ . \

in I.

(10.:!11)

and ""l 1ha11hc puwcr smc>eonvetg<S 10 f (x). or has.um/(.t). Oneo ~gain wh:u \\C mean
is thai a s nmre Hnd more term!! on the ri:Jtt arc mcluded. the clnliC:t and clo~er the '(um gee~~; to
/(x) on che intenol / . \Vc call I the interval of conYer~tnce for the p('t\\er ~ries.
Thcl'c ~rc l\\O tbpct'tS to co.WCJ.Cill"'C for J)O\VCI M:rics. intCI\~I

tlnd ~~oun1 /(x). h is

relatively s imple to find the interval of convergence for a power scric~. hut much more difficult
tO find i1S IIUIH,
Oatc the ca~k"'' and perh~tps lhe: n10St imponaot. l'(nvcr scric~~o is

tr

"" ti.A" = a + tu
L
n-.o

+ a.r 1 + ax' + .

''here a is some non1ero oonstanc. h is so imponant in .application.~ th.u it i~ &''en a special


n.unc: it &._, called a gronlC'trlc <cries. A geonte:llic ~ries i~ a ~ ail"\ in which every t.enn af111!T
the fi~t is obc.dlnc:d by multiplyina the previous tern1 b) t~ JanlC nrnoueu. called the ~ommon
ratio. For geometric ~es 10.21. the conrnon ratio is ..r: each 1erm i~ x tin\ts the ~'ious
tenn. To ~~....v th.n the scrie~ C('W\\~ we must s.ho\v that the sequence of fl''lynomials

P.(x) = a - a~ - ax 1 +
has a limit l l'wcmuhiply P.,(.<) b)

.< 1'.,(:<)

... + a \'n

x. lhcn

= ax + ax 2 + axJ + +ax"+'.

\Vhen thc:sc are s ubtracteU.

f rom which
(10.2.:: )

Thi< i~ a simphhed lonn fot P.(x). Since


hnl

n-"'"'

.r+l =

0,

- ) <X<

I.

.< = I

does not exist..

oc.hcrwi~

' ' fo llows 111111 whcnlht sequence Il',(.t)} is restricted 10 the iollcrvnl
lion P.,(x) =
tl-"'10

l..rl <

1-x

}xl <

I . i1

has o limil.

I.

We eanlherclhrc w rite that

"' x, = a + ax + ax 2+
"L..
n 0

1-x

xl <

I.

( 10.2.1 )

l11C gcomclric :.er;c. has wm o/(1 - .t) o n the inoerv<ll (of cQnvcr,col<'t') xl < I. It~~ olQI
have a s um for l.t I 2:: I.
Figun::s 10.16 how some or the polynomials P.(x) and a /( 1 - x) when a = I. They
illusu-a1e how polynomials approximale 1/(1 - x) more closely as 11 increases. Pol)nomials
are defined for a ll x and I I (1 - .r) is defined ror all x I. bul cuncs y = P. (x) approach
y = 1/(1 - x) only for }x} < I. For x > 0, polynomials approach 1/(1 - x) from below;
ror x < 0. lhey oscillalc uboul 1/(1 - x). bu1 gr.1dually approach I /(1 - x) .

10.4

tii1l

[;]Ill .
PA11i"l 51an:-. 0( &Olltnelric series I

~a Series

613

FIGURE 10.16b

+ ,, + x! +

Y
J

y=

} = I ->

r:-;

-2

2X
-I

- I

-I

-1

-J

-3
FIOURE 10,1tiCI

FIOURti 10.16c

)'

1Y

J!

3
.V & /', (X)

Y = ''u(x)

I
y= l - x

-2

-2

=-.
1 -.\'

2
- I

2X

- I

-2

.~

- I

-2
- 3

-2
-3

L:-o

Obviousl y, 1hc in1crval of l'tmvergence for a powc series


lt ... x" nlwl'Jys inc lucJc..o.; the
' 'aluc x = 0. but whut other l~i bilitic,, arc there? For geometric seric.:; 10.23. the imena l
or convergence is the intcnal - I < .r < I. The termi nology itself, interv1tl of conw:rgc~tcc,
~ uggests th.tll the values of .l' fo r which a pmver series convctgcs form ~omc kind o f inte rval.
Th is is indeed true. as we sh~ll soon see.
Evet)' Maclaurin series is a power series ill x. F.xaml)les 10.9 and 10.1 0 developed Maclaurin series for sin .f and e:c ,

sinx =

x3

x - -

3!

xs

+ -5! +

e' =

They arc power series that add to the fu nctions with intervals of con"crgcncc - 00

111C power series

< .x < oo.

,
L:: n!x" = I +x + 2! xl + 3! x 3 + .. .
n O

con,-e~-ges only

for x = 0. This is a direct reswlt of 1he titc11ha1

lim 11! l.x l" =

00

rt-+()0

ror any given nonzero vaJue of ~t. How, then, could the addition of more and more te.nns eYer
converge? These examples have il lustnued 1ha11here nrc a1leas11hree possible types of imervals
of co,wergcnce Jor power series L~ ( t 11 .:tn:
1. The power series c.o nverges only fo r

x = 0;

2. The power series converges for aiJ x;

3. There exists a number R > 0 such that th-e power series converges for
for lxl > R ,and mayor may not comerge for x = R.

lx I < R, diverges

614

Cbapcct 10 ln(hlilc: Scqttelll.:es o~.tkl Series

These arc in !act the only possibilities for ao intcn'lll of convergence. ln 3 we call R the
radius of convergence of the power series . It is half the length of the interval of convergence,
or the distance we may proceed in either direction along the x -nxis from x = 0 and expect
convergence of the power series, with the possible exceptions of x = R . Tn order to have a
radius of coJwcrgc.ncc associated with evc.ry power series. we-say in J aod 2 above that R = 0
and R = co, respectively.
Evc.ry power series L~~ anx" now has a radius of convergence R. If R = 0, the power
series converges o nly for x = 0; if R = 00. the power series converges fo r all x ; and if
0 < R < oo. the power series conve'llcs for lx I < R. diverges for lx I > R, and may or may
not converge for x = R . For many power series the radius: of convergence com be t--akulmed
according to the following d>corcm (sec Scction!0.!2 fQr a proot).

THEOREM 10.2
The radius of convergenc-e of a po,ver series L~, a nJ:n is given by

R = lim

n-..oo

1!!.:!._1
a,,+l

or

(10.24al

R = lim - -.

( 10.24b)

n-..oo ~

provided that either limit exists or is equal to infinity.

\Vhcn we know that the radius of convergence of a power series I:: o 0 11 X 11 is R, and
0 < R < oo. we know all values of x for which Lhe series convel'g.es with the excepLion
of two values of x. The series converges for lx I < R , diverges fo r lx I > R , and may or
may nol converge for .x = R. Some series converge al bolh cndtxlints. some at neilher,
and some at one end bm not the other. There is 110 simple test that c.a n distinguish :unong
these situations for all power serie.s. E,-ery power series must be checked individual ly as tO
whether it convcr-~cs at the.endpoints of its interval of convergence. At x = R1 a powerseries
reduce.') tO a ~eri es or numbers. Sections I 0.9-10.12 comai n te.'\lS that dc1em1ine whether series
of numbers converge or diverge, and thc.reforc testing whether endpoints of po'ver series with
fini1e radii of convergence can be included in tile intervals of convergence will have 10 wait until

we have covered tbis material. In Lhc mcanti.mc, we will call the intcrvaJ of convergence the
open interval of con,e rgence whenever \Ve have not tested for inclusi()n of endpoint~.
When Thcorc01 J0.2 determines the radius of c-om.crgc.ncc of a power series, iL also determines the open interval of convergence. We il lumate in the following examples.

l EXAMPLE

10. 12
OJ

Find the open intcrvaJ of comc.rgcncc for the power series '

1) "

~.t".

L-, n5~'
,_

SOLUTION

Since
( - 1)"

I I

n 5 2"

R = bm

-a,n-oo lln+J

(11

1)

" +lim 25 ( n

n-oo

= 25,

I )5"'+2

the open interval of convergence is - 25 < x < 25. The seri.es converges for lhese values of
x, diverges for x < -25 and x > 25, but we do no!. know whether it converges for x
-25
or x = 25 .

...-..

I EXAMPLE

10.13

(11 !)'
L"' -x".
(211)!

Fiud the opeu imcrval of convergence for the power series

n=O

SOLL'TION Since
(11 !)2
.

= IJJU

,_oo

2
(2n)!
.
[ (11 !) (211 + 2)(211 + 1)(2rr) ' ]
=
11m
[(11 + 1)!]2
-oo (211) !
(11 + 1)2(n!)2

(211
(211

+ 2)!

+ 2)(211 + I)
(11 + I )

lim -'----'-'--::---'= 4.
2

"-""

the open jntervaJ of convergence is -4 < x < 4.

I EXAMPLE 10.14
oo

Fi1Jd t_hc ope-n inte-rval of converge-nce tOr the power series ""' - .t".
~ ~~~~

n=l

SOLUTION Since

lim - - =

n- oo

:1i{iJ

lim
n~oo

'j 1/ n'J

lim n = oo,
n~oo

the series converges for aU .x. hs lnterval of convergence is - oo < x < oo . which is not j ust
its open inten r:-11.

I EXAMPLE

10. 15

oo
I
21 1
Find the open imerval of cor1\'ergcncc for the power series '""' - - -x
r+
1

L- n 2 2 ~

n- J

SOI.lJTION Sioce coefficients of even powers of x are 0, the sequence {a.,ja,.+1 1 is not
defined. We cannot ~tere fore find its mdius of convergence directly using Theorem I0.2. htstcad.
we write
00

L -n 22u- r
I

llt+t

n =l

aod set y =

x1

i_n the series:

According to equation I0.24a, the radius of convergence of this series in y is

Ry

n l2n
I

= ,,_
lim00

1) =
2

II +

lim 2 - n - oc (
11

2.

-,-----,-:-;;-~

(n

1)22n+l

.JY.it follows thallhe rdius of convergence of the poweseries in x is Rx = ..fi.


The ope.n interval orconvergence is Lhereforc - J2 < x < ../2.

Since x =

61&

Chlipltr 10

lnfiniJC Sequcni!~ S :uul Series

Sums of Power Series


Finding the interval of convergence of a power series is o nly half the convergence problem .
The other ha lf is find irlg the sum of the j)O\\'Cr series. \Vha t f unction does 1he power serie-s
ap)>roximatc as more ancJ more terms arc inclucJcd? Sornclimcs it is pobSi blc to rel ate a given
series to a series with known sum. Four very important seties with known sums are
tl

- x

= a

..1. (U.

+ a;t 2 + a.:c3 + ,
xl

xl

e-' = l +.r+-+
-+

2!
3!
sin ,\

x3 x'
x'
=x- - + - 1 - -

cos X = I

I EXAMPLE

3!

7!

.\'l

.\'6

-I <

-oo <

+ .. .

-< < I.

<

00,

-00 <X < 00,

-2! +--+" ' -oo


4!
6!

< X <

oo.

( 10 2 5a )

( 10 251>)

( I 0 .25c)

(1 0 .25d)

10. 16

"' ( - 1)"+1

Find the sum of the power series """


L
, ...o
SOUJ'T'TON

x" .

.,.,
-

We wrilc

f(-1)(~r )"
,. ,..0

= - 1 + - x - - <2
2
22'
a nd note that this is a geometric series with first lenn a =
therefore replace x by -x/2 in equation 10.25a. we ll!l' e

00 ( - J)TI

?;

+ -23'<3

I and common nuio - x /2. 1f we

-2

- 1

,< = I - ( -x /2) =

2"

This isvalid fo r - I < -x /2 < l , or, - 2 < x < 2.

I EXAMPLE

10.17
00

Find the sum of ~tc power series

L
,~

(-

I) 112"

xm +2 .

(2n)!

SOU 'TION T he 'cries "'"' be expressed in the form

~ .<:...-__;1):.....
"2_"
L

n 0

(211) 1

2>1+2

2~

(- 1)" ( {;;

= x L - - v2x
n

)2n

0 (211) '

(..fixf
2!

(../2x)4
4!

+2

10.4

his x 2 times the Maclaurin series for cos x with x replaced by

F\Ni't:r SeJies

617

Ji x . In other words,

"" (-I ) 2
L
xm+l = xl cosJ2x .
.,:o (211) !
11 11

''alid for -oo <

./2 x

< oo. or -oo < x < oo.

Other methods for Onding sums of power series arc discussed in Sect ion 10.6.
For many power series I here is no known funcrion to which the pO\ver series converges. In
such cases we write

.f(x)

L"" a,.x"

(10 26t

nO

and say that the power series defines the value of j(x) at each x in the interval of convergence.
For i nstance ~ a very important series in engineering and physics is

which converges for uiJ x . It arises so often in applications thai it is given a spe-Cial name, 1hc
Bessel,(lmctiOit r~filtefirst kiurt o.furrter zero. and is denoted by J0 (x). I n other wonJs. we write
"'

( -t )"

l o(.r) = "\' :u
,x2",
L- 2 '(11 !)n=O

:Uld suy that the power series define.s the value of J0 (x) for each x . We get an ide" of what
Jo(x) looks like by ploui ng the polynomial approx imations obtained by adding more and more
terms of the series. Figure I0.17 shows the first five polynomials for x 2:. 0:

P, (x)

I'

xl
l'z(x) =

22'

xZ

PJ(.T)

- 22

P,(x)

I - 22

+ 2'(2!)2'

;cl

"'.4

xiS

.\'('

2'(2!)2 - 26(3!)1

+ 2'(4 !)2'

They arc even functions, so their graphs would be symmetric nbout the )'axis. As the
number of tenns is increased, the IXllynomi<lls approximate l o(x) in Figure 10.18 more and
more closely.
\Vehavecalled 10.19 a power series in .r . II is also said 10 beapouerseriesabomO(meaning
the point 0 on the x axis), where we note that for any inte-rval of convergence wh&.ISOC\'Cr. 0 is
always at i1~ centre. Thir.: suggests Ihat power serie~ ahout other !)Oints might be con ~idered. and
this is indeed lhe case. The genera] rx>wcrseries about o poill con the xaxis is
00

L a.,(x -

c)" =

an+ a 1(x - c)+ a 2(x -

u=O

and is said to be a power series in x - c.

c)2

( 10.27)

811

C'b:.(llc= 10 Inti nile Sequen-es tl!nd Strib


li#lic.III;I.JT:)(nl
~ itxl l)( UC'1Ir.r lf:-1\)

PJ(X)

B ~sel 's ftrntliOI'I l o(X) of lhe fii'OI

P;(.A)

0.&

0.(>
Y ~ Jo<xJ

0.4

P 1(X)

0.2

' l o.._x

6 '
-I

-().2

-2

-().4
Pz (.<)

P, (.X)

A power series in x-c has an illtcrvt\1ofcowcrgcncc alld a radius of co n vergc ncc analogou ~
to a power series in x . l rl particular. every power series in x - c has a 1'-adius of convergence
R :tuch that if R = 0. the power series c-onverges only for.\' = c~ i f R = oo. the power scrie.."
rowergcs for all x~ and if 0 < R < 00. rhe series cowel'gcs for lx - cl < R , diverges for
lx - cl > R , ~lnd may m may not converge for x = c R . 1bc radius or convergence is
again given by equatiorlS I 0.2.J-, provided that the- limits c.xist or urccqual to infi nity. For power
series in x - c , then, rhc point,. is rhe cemre of the inlel"\a l or comergence.

I EXAMPLE

10.18
Fi nd the open i ntcrval of convergence of' Lhc power ~erics

SOLU'f ION

B)' writing the power series in sigma ncuarion,

"" 2''+'<- + 2)".


I:
tr=l

we can usc eq uation I0.2-la (or I0.24b) to calculate its radius of C<Mwcrgcncc:

Since this is a power series about - 2. lhc opco1 inrc<Val of convergence is - 5/2 < x < -3/2.
We can do better. This is a geometric se1i es with fi rsttcml4(.t +2) and common ratio 2(x +2).
According to C::<(Uation

""

1 0.25t~,

L 2"+1(x

-+

il:s sum is

+ 2)"

4(.< T 2)
- 2(x

+ 2)

+ 2)
2x + 3

-4(.r

valid for -1 < 2(x 2) < 1


-5/ 2 < x < -3/2. This is now known to be the
interval of c.onvergence, not just the open inLe.rval of convergence.

EXERCISES 10.4

In Exc.Jx:iscS; 1- 25 fi nd the open intcr\al or convctgcncc for the power


series.

z. L"" ,,z.t"

"' I

I.

L
- x
,.,
II

I: (ll +I l)l x"

..L
-..

7.

9.

L (- l )"n'(.r + 3)"

6.

2"

I: -.jti (X + 2)"
1

-~ c - ~ y

I: 2"

8.

-""

It + 2

30.

I: 4"- .r'"'
"' -1 (X I:
,.,
321J

. I: -n+l (2.t)"
neO

12.

L""

"' (- 1)'
I:
--
(211)! "

32.

L=

( - 1) .\''btt-1

14.

3'

.\")...

' 34.

0 .JU+T

(- e)".r

"'

9 ,}
Ill
+ 3''
_4 _,.~ + -x
+ + -x
+
y>
321

,(
15. -9

16 .t + l2x1 + 33x3 + ... + ,".~" + ...


I

36

,.

(- l )"
'bH2
32n+ l(2n + I)!'

"' (- 1)'
I:
- II! x

' 31.

"' .,
I:)
- x"

I: ~<x + l)"

33.

,,- on!

"' ( 3)"

'

=Q

I I.

"' <

...

"' 2"
36. I: - (.r-1 /2}"

w+

35. I: (.r+ ll '"


(211 + I) !
0

37.

n!

L"'

.....

( - 1)"

G" +,Q

22'' (211 ) ! X

49

6-1

_.. 38. If m is n nonnega1ive integer. the Bessel functi on of order


the first kind is defined by the power Krics

+ 10) 6 + -(x + 10 ) 7 + -(x + IO)'


I ( )

r~rX=

+ .. . + ,,
.!. (.t + 10 )" + . ..

"'->

11at )

')

(- I)"

2 ~~ n !

(11

m of

!n+.11r

+ m ) !; C .

+ 8(3x) 2 + 27(3x) 3 + + n 1(3:t)" +

J ,

20. 1+->
5
"'

3'

27 .

- .c + .r + - x + +
4
I6
I

.. 21.

L"" (x + S)"'

11

17. -(x

19,

29.

.,

(x - 4)"

I: (- l)'.r''

10.

II.

I: (- e )' A~

=<I

Ool n - 1

18. 3x

I)"

..

~'-

11 ~0

>

L n>
_!_x11l

o.>

- (.t - I)"

28.

u-1

13.

"'

L"' " 23".""

~-

'10

s.

26.

,.

3.

In Exac iscs 26-37 find dli! M.nn of the power series.

+-.<
25

( 11

+ 1)

3;,

+ +--<

(a) Wrioe oullhe first fi ve ternlS of Jo(X). Jo (.<) . nnd J,. (.<) .

.
1 .r - +

(b) Find the i nll!t'V:tl ofcnn~rgc:nce (or t.meh J,..(.r).

~---

4-

I:-
Inn '

.\9. 'fhc hypc..gcornctric series is

11=2

<x/3

I+-.\ +
I'

.. 22.

00

<t (

+ l ){j(fl +

I)

2! y(y +I)

+ a (a + l)(a + 2)/J ({J + I)({J + 2) X3 + .. ,


3 ! y (y + l)(y + 2)

') '

"-'-'-.\''!

' 23. L-, (311)!

oo

I:

25.

2 - 4 - 6 - (2n)

t
357 .. . (2n+ 1) .

oo I I 3 5 ... (2u

I:
n= l

2 21' (2n) !

+ 1) 1' X

where a. fj . ~nd )' arc all conswnls .


(a) Write this series in sigma notation.
(b) Wh..al is the radius or convcl'geoce of the hypel'goomeuic
SCI'ies ir y is not 'l.e.J'O or a negative imeget?

620

Ch:lp~e r

10 Infinite Sequen<:es an~ Series

110.5 Taylor Series Expansions of Functions


Let us summarize what we have seen in See1ions I0.3 and 10.4. Given 11 function f (x) and a
point c, lhe Taylor series of f(x ) about cis
oo f (n)(c)

f (x) = "
L.,

n!

it= O

(x - c)" ,

and the series converges to f (x ) at all values of x for which Taylor remainders approach zero.
Gi,en a power series I;~ a., (x - c)", there is an interval of convergence inside of which the
series has a sum. If this sum is f(x) , we write
00

L a, (x -

f(x ) =

c)",

,f

in the interval of convergence.

n=O

Clearly. Taylor series are power series. A power series would be a Taylor series if a.. were
equal to J<">(c)/ n1 If this were the case, then power serie.~ and Taylor series would be one
and the same. The following theorem allows us to pro\'e this.
THEOREM 10.3

If /(x) = I;:Oa. (x - c)" , and the radius of convergence R is greater than zero, then
each of the following series bas radius of eonvergence R:

"'
tcx) = L:
ll(t,(x - cr-,
n =O

f (x ) dx =

"" a
" --"-(x -

L., n+ l

c)"+ 1

+ C.

(10.c8bJ

n=O

Due to the diffic ulty in proving this theorem, and in order to preserve the continuily of o ur

proof. Note th~t the theorem is stated in temts of radii of convergence


f'dther than intervals or convergence. This is due to the lactthat in uiJl'eremiaLing a power series

d iseussion. we o mit a

we may lose the enc.lpoims of the origi nal imervaJof convergence, and in integrdting we may pick
then up. It could be srutcd in terms of opc11 inte rvals of convergence , however: tcrm~bytcnn
differentiation n1ld intc:gr..Hlon of power series preserve open intcrvaJs of convergence.
The next theorem implies that p<>wer series and Taylor series arc o ne and the same. that

every power series is a Taylor series, the Taylor series of its sum.
THEOREM 10.4
lf f (x ) is the sum of the p<>wer series L:::':.o n.,(x -c)" with R > 0. then the series is
the Taylor series of f (x).
PROOF When we set x

= c in

00

f(x)

L t, (.r -

c)" = a0 + o 1(x - c) + a 2 (x - c)2 + .

n=O

we obtajn

obtain

f (c ) =

Cl() .

If we dilferemiale the powe r series according to Theorem 10.3,

\VC

10.5 ThylorSeries E.~J,:lnsiOlS of F11nc1ion4;

When we substitute x =

621

c, the result is
f'(c}

If we uil'fcrcm iat.e the power series for

=a,.

f' (x), we obtain

/"(x) = 2a2 + 3 2a3 (x - c)+ 4 3aJ(x - c) 2 + .


and substitute x

= c.
f"(c) = 2az

or

Oz

f"(c)

2!"

Continued differentiation and substitution leads to the result that for all11,

ll,

The power series j(x) = L:~0 a,.(x- c)" is therefore the Taylor series of j(x).

The following corollary is an immediate consequence of this theorem.

COROLLARY 10.4 . 1

If two power series L::";o a. (x - c)" and

L:;;";Q b,. (x

- c)" with positive radii of con-

vergence have idcmicnl sums.


00

""

n= O

n=O

I:O.<x- c)" = I).<x - c)",


then a.

= b, for all tt.

Theorem J0.4 shows that Sections 10.3 and 10.4 were dealing with the :>amc problem but

coming at it from different directions. In Section 10.3, J(x) and x = c were given, and we
developed the Taylor series for f (x) about x = c. Theorem I 0.4 shows that this is the only
powc.r scric~ for /(x) about.J." = c. In Section 10...1~ a powc.r sc.rics, L~o a,,(x- c)n. was
given. and we determined it.> interval of convergence and sum / (x). TIICorcm 10.4 indicmcs
that the power series is actual ly the Taylor series of /(.r) about x =c.
This equivalence of power series and Taylor series simplilies the problem of finding the
Taylor series for a function f (x ) about a point x = c immeasurably. lnsteud of finding / 1">(c)
and showing that Taylor remainders apJ>roach ~cro (as we did is Section 10.3). we can proceed
a~ rollow~. If, by any method whatsoe\'er, we can find a power series L::,0 o,~(x- c)'' that
has sum f (x ), then it must be the Taylor series off (x) about x = c. In the rcmnindcr of this
scctiQII we show how easy it is to do this. ln essence. we take series with known .)Utn)). such as
l 0.25, and construct other series from them.

I EXAMPLE

10. 19

..

Fi nd (>l) the Maclaurin series for 1/(4 + 5x) a11d (b) tile Taylor series about 5 for l/(13- 2x).
SOLUTION

(a) We write
----- =
4+ 5x

Sx)

---,---:-7"'

1+ 4

1/ 4

Sx

1+ -4

and interpret the right side as the sum of a geometric series with first tenn 1/4 and
common rntio - 5x j4 . Equation 10.25a then gi"cs

+ 5x

= t(n(-S~tr

1-~' 1

n~o

=L
<X

( - 1)" 5"

lxl

x" ..

4"+1

IJ-1)

<I

< 5

( b) By a similar procedure. we have


I

- - = :---::-:---::- =
I J - 2x

3 - 2(x - 5)

3 I - J(.t - 5)

~(1/3) [~~.. _ s>]"

00

I -

1/ 3
2

- (X -

5)

< I.

lx- 51 <

3:+, (x - 5)"'

n ~o

Tn both examples. propenies of geome~ric series gave not only the required series. but also
their intcn-als of convergence. To appreciate the simplicity ofthcscsolutions. we suggest using
Taylor remainders in an attempt to obtain the series with the same inter\'als of convergence. You
will quickly abon.

Addition and Subtraction of Power Series


Atx:ord.ing to allc rollowing theorem. C(NWCrgem power series can be <Jddcll and
their common interval of convergence.

THEOREM 10.5

rr f(x)

.,..,

~ub t m clcd

""

L tl.(x- c)" aol<l g(x) = L b.(x- c)" have positive radii of converu=O

gcncc. then

f(x) g{-t) =

L"" (a. b.)(x - c)".

(10.2~)

n=O

val id for ever)

x that is common to the intecvals of convergence oflhe two series.

We use 1his result in 1he rollowing example.

I EXAMPLE

10.20

Pind the Maclawin series for f(x) = 5x j (x 2

3x - 4).

in

SOl l n oN

\Ve dec.lVOtpo"C {( 'f") m1o its pan ial fntcliOiliS.


Sx

3.r - ~ = ~ + ~

/ (.r) = .r> -

and upand c:a"h o f 1~ tcmb in a Maclaurin .eries.


~

1.<1 <

.r - 4

1
-

+ .r

I - .\' +

x1

x'

+ ,

I' I <

4.

and

I.

AdcJition of these .~~cries within their common imervo:tl of convergence gives th<: Maclaurin series
ror / (.r):

--,-- 5.,..__ - (- 1-

x 2 - lx- 4

<:- ).,.

~..~.
..

(1-.r + , ,_ ,\) +)

..

(-I- ~) .< + (I - :,) .r2+ ( -1- :, ) ' +

= f:(c-1)"- :. ].

1.<1 <

I.

Differentiation and Integration of Power Series


Perhi\ J> ~ 1he rno~ l J
lO\\ erful ccchniquc rur gcnent ting Taylor series i ~ 10 UitTcr'Cntiutc or intcgra1e
known expansions uccnrding to Theorem I0.3.

J EXAMPLE 10.21

f tnd Maclalln n scnch for che fOllowing functic ns:


X

(a) CO<X

x)'

(b) (2 -

SOI.o TIOI\

(a) We dcrit:<J ohc Maclaurin series for in .r in E.omple


inx

.r 3

.rs

.r 7

3!

5!

1!

10.~

= x -- + - -- + .

We ohcn suucd lhatthc Maclaurin series for cosx could be derived in a simil ar way.
Tcrno-hy-tcmo dif'fcrcoHiation of ohc sine series is fn<tcr.
1

co... r

x' x' + ...)


= -d(<in .r) = -d ( . r -x- +--dx

3!

d.r

xz

=1 - -

2!

x4

+ -

4!

5!

x6

- -

6!

7!

+ ...

- oo < x < oo.

(b) We begin with 1he Maclaurin series for 1/ (2 - x).

2- .<

~r

.ts

.\'2

= -2 + -+-+-+
.. .
22
23
2'

l.tl

< 2.

Tcrm-by1Cfm diil'ereruialiOrl of 1his series gives

l.rl <

wilh open interval of convergenc.;e

(2 - x):

'

- -"""'""1 = 2'

3 2x
_2_
_
4

2. Anothe r differemimio11 yields

4 . 3.r 2
2_5 _

+_

+ ... ,

lxl

< 2.

Mulliplication by .r /2 now g ivc."i


2

.1'
2<
3 .2t
4 . 3.1
--...,.=
-+--+
- + ...
(2 - .r)'
2'
2'
26

~ 11 (11 + 1) "
= L...t 2Jr+3 X ,

lxl

< 2.

n l

I EXAMPLE 10.22
Find the Taylor series about I for In x.
SOLUTIO\' No1ing that lnx is an an<iderivative of 1/x. we lir.;t expand 1/.t in Taylor
seri e~ about I :

- =
x

(x - I) + I

= I - (X - I)

+ {X

- I)2

{.t -

I)'

+ ... ,

lx - II <

I.

ff we inl egr-ate this series term by 1enn, we have

ln lxl
Substitution of x

[x - 2~(x

+ 3~(x -

li

I )3 -

~(x - I )' + .. ] + C.

= I implies thm 0 = I + C; that is, C = -I . and hence.


In lxl

(X -

=L
)

11=1

I) -

I
- (.r -

( - 1)11+1
(.r II

I)

+ -I (x
3

- I) -

II
I) .

According to T heorem 10.3. the mdius of convergence of this scties is also R = I; that is,
the open interval of coowergence is 0 < x < 2. We can therefore delete the absolute values
around x.

Comparison of Lhe wlutions in Examples I0 . I 1 and I0 .22 indic-ates once again Lhe advantage

of avoiding the use ofTaylor S remainder formula.


1

Multiplication and Division o r:Power Series


lll Example 10.20 "'" a<l~e~ 1he Maclrturin>etries for 4/(.r - 4) and 1/( 1 +A) lo obl><in 1he
Maclauriol series for Sx /(.r 1 - 3.r - 4). An ahen>ati;e procc<lure mig hi be 10 muhiply Ihe 1wo
series since
5x

x! - Jx - 4

= 5<

(-~-)
(- 1
-)
x - 4 x +l

S.r (
-4 I

x'
) (I + .r4 + .r'
42 + 4) + . . .

+ X, -

+ ...)

The rule~ of algebra demant..l cht~ t we muhiply every tenn oftht: fir~ t .stri~, by e\ery term of the
secor1d. tf we do this and gro up al l producs w i1h l ike powers o f' x . we obtain

....,.._.:;5~'-- "'

.r 2

3.r - 4

Sx
-4

It is clear that the coefficient or ~\'


mula I0.22:

11

[I + (-I + 4~) x + (I- ~4 + ..!_)


x'
4
1

is a finite geometric series to ,vfljc:h we can apply for-

L - .. . + (- l)" ]
( - I)[ I - -J + 4
42
4"

(- 1)"

I [

(-!)"+' ]

'
I+ !

'

Consequem ly.

it
5x """ (- I) "4
...,..---7--5
x 2 - 3.r - 4 = - 4 "
L._;

[ I - ( - -;I)"+'] .r"

u-0

~ (-1)"+1[I - (-!)"+']..-"+'
4"+1

~
n=O

~ [(-1)"+1- 4+
-~-] x"+'

L._;

n=O

f [(- !)"- ~. ].r".

n =l

For thjs example. then. mult ipljccu i.on as well as addition of po,ver series leads lO 1.h e M aclaurio
series. C learly, addi tion of power series is much sim pler f or this example. but we have at least
demonsu-ated that power series can be multiplied together. That this is ge.nerally possible is
s ta ted in the following lhcorern.

626

Chaptt r 10

lnfiniiCSefrtlencCi ond Seric:.

THEOREM 10.6

If f (x ) = E~0 a.(x - c) and g(x) = E:'. obnCx - c)" have po~i tive radii of
convergence R 1 and R, , respectively, then

""
!Cx)g(.t) = Ld,(x
- c)",

( 1 0. 30~)

n =O

where

d,.

" a; b.,_, = a b.
=I:;
0

+ a 1b.,_ , + + a, _ 1b1 + a..b0 ,

10.30b)

;=-o
and the

I EXAMPLE

10.23

radiu~

orconvergcn<.-e is 1he snl4lller c.>r R 1 and

R2

........
Findthe Maclaurinseriesfor .f(x)
SOl UTI ON

= [1 / (.r - l)j ln(l-x) .

If we i ntegrate the Maclauri n series

1 -x
we find that
- In I I -

xi =

xl

xl

x'

x + - + - + - + + C.
(

=0 . we. obtain C = 0. and

By sc.:uing .r

In ( I - x) =

x2 x3 .r4
-x - 2 - 3 - 4

We have d ropped absolute value signs since the radius of convergence of the cries is I. We
now fom11hc Maclaurin series for f(x ) :
I

- - ln(l - x )

x- 1

- 1

- - ln ( l - x )
1 - .t

= (1+X + X"., + X\ + .. ) ( .t + .\"2l + .l3J + ...)

( I) 1 ( I I) 3
=L"" (1 +:;+ - + + - )x.

= x + 1+ 2 x + 1 + 2 + 3 ' +

II=

1/

Since both of the multiplied series have radius of convergence 1. so also does the M aclauri n series

for (x - I) - I In ( I - x). In other words, its O<pe.n intervaJ of convergence is - I < x < I .

I EXAMPLE

, 0 .24

Find the first three nonzero tenns in the Mae i:Juril'l series for tan x .

L:O:{) Or ;r" , then b)' seni ng tan x = sin A' / cos .:r

If tan x =

SOLUTION

we have

00

sin .r

= cosx L:: a,:x

11

=O
\Ve now substi tute Maclaurin series for sinx and cosx:
'

....

x - -.J + -' - -.7 + =


~

3!

5!

7!

.z + ....-- - ....-~ + ) (a + a1x + f h X 1 + ) .


0

,,

1- -

2!

4!

6!

According 10 I he corollary tn Theorem lOA. 1wo power series can be i<.lemical only if cnrre.

sponding coefficients arc equal. \Vc therefore muhiply the right side and equate coefficients of
like powers of x:

= a1 :
ao

0 = a, - - .

3!
0

which im plies a2 = 0:

2!

a,
liJ -

2!

Oz.

0
=a,-+-.
2!
4!

a,

5!

which i mplics (IJ

from which a 1

1
=as--+-.

2!

4!

= - - - =
2!

3!

= 0;

from which fls

I
I
I
= 5!
- + --- =
6
4
1

15

The firs t three nonzero terms in the Madaurin :scric.'\ for tan x are therefore

= x

I ;
3

2
15

+ - x + - x 5 + .

We could obtain the same result by long d ivision of the Maclaurin series for sin x by that of
co:; x shown below. Long tlivi~ i on can protlure a few lCmls or a Maclau1i n ~cri es. btu seldom
docs it suggest a panen> for o.ll tenns in the series.

).J

).J

:r-6+
2;->

15 .....

l..S
15+

628

Cht~er

10 lnfinilt. Stquenc~s and Se-rie'\

Binomial Expansion
One of the most widely used power series is the binomial expansion. We are well acquainted
with the binomial theorem, which predicts the product (a+ b)"' for any positive integer m:

(a+ b)'" =

L"' (::)a"b'"-".

( 10 .3 1)

tt=O

With the usual definition of the binomial coefficients,

(Ill) =

m!

m(m- l )(m - 2) (m -

(m -Jt)!n !

11

11

+ I)

n!

the binomial theorem become~'\


(11132)
Even when m is nOt a poshive integer, this fom1 for the binomial theorem remains almost intacL
To show this. we consider the power series
00

I+

111(111 -

l)(m - 2) (111

+ I)

11

n!

n=l

x0

for a ny real number m exce1n a nonnega1ive integer. TI>e radius o f cotwergence of this powe.r

series is

. 1111(111-

1)(111- 2) .. (111 -11 +I )

R = 1om

"~""

_ll T_

lim
11-o.:.

(11 +I )!

II!

1)(111 - 2) (111 - 11)

111(111 -

=I.

111 - II

The open interval of convergence is therefore Jxl < I. Whether the serie.~ converges at the
end-points x = I depends on the value of"' For the time being, we will work on the interval
lx I < I . and at the end of the d iscussion. we will state the complete resuiL Let us denote the
sum of the series by

f(x) = I

"'

+L
,_,

m(m- l )(m- 2) .. (111-

11

+ 1)

11 !

.n

lxl

'

< I.

If we differentiate this series term by term according to Theorem 10 .3.

(x ) =

..

Loo

111 (111 -

1) (m - 11

1) ,_1

(11- 1)!

1-<1 < I.

and then multiply both side..' by .X . \ Ve have

. , . _ ~ m(m - I) . . (111 -'f(.)- L..


(n - 1)!

11

I) x"
'

J,t

< I.

n l

If we add these rcsuiLS, we obLain


00

'\' m(m - I ) (m - n
f'(x} + xf'(x} =
L..
(n - I) !
ll=l

--':(._11_1_-_ 1.:,)_ _ _(:..m..,...-_ n_+_...:.J) x" .


+L"
00

I) , _ 1

11=1

(n - I) !

\Ve. now change the variable. of sunm1at.ion iJ1 the first sum:

, _ L m (m -

+L

I) (m- 11) "


X
11!

<

(I

+ .t)f(x) -

n:O

""

n:l

m(m - I ) (m. - "


(11 - I)!

I) "

X.

Wh.en th.esesummations are added over theircommon range, beginning at n = I , and the n = 0
term in 1he first summation is wriuen out separately, the resull is

(I

+ x)f'(x)

=m+

Loo
II =

- 11

I} ( "' -

11
--+1 ) x

(11- I )!

II

111 (111 - I) (Ill - 11 + I) J


+~
~ - '- --'--- - - , : _ - - : _:_ X"

Ill

m(m - l ) ( 111

rt!

n=l

= mf(x}.

.f (x ) m ust satisf)' the differential equation

Conscq ucn1ly, the function

/'(x)

f(x)

Ill

1+ x

I11te-gration immediately gives

lol.f(x)l = mh> ll +xl +

or

f( x ) = D (t

+ x)"' .

To eva luate D . we n01c that from the original definition of f(x) as the sum of the power series,
/(0) = I, and tltis implies that D = I. Thus.

f(x ) = (I + x)"' ,
and we may write fi nally Lh:n
00

( l +x)"'= I

+L

I+ mx

111 (111 -

J}(m - 2) (111 - II

n!

I}

x"

m(m - l) 2 m(m - l)(m - 2) 3


X +
X
3!

2!

tiO.JJa)

+ .. .

( 10.33bl

valid fo r lxl < I . T hi s is calle.d the binomial expansion or (I+ x)'" ; it is the M<lcla urin
se.r ies lor ( I + x)'". We have >'eritled the binomial exp&lSiOtl ror m any re al number except a
nonnegative imcgcr, but in the case of a nonnegative intege-r, the series terminates after m + I
term~ <"md is thererore valid ror these v"cllues or m als.o. \Ve mentioned earlier that the binomial
e-xpans ion may also convc.rge m the endpoints .:r = I , depending on the value. of m . The
co111plete result s tates that I 0. 33 is Vlll id for

- oo

<

.\'

< 00

if m is a nonnegative. integer,

-I < X <

if, !: - I .

-I

if -1 <m < 0,

<

.\' ~

- I ~ X ~

ifm > 0 but not an illleger.

It is not difficult to gcner~li<e this result to e xpand (a+ b)'" for re al m. If

write

(a

+ b)"' =

a"'

( + ~b)"'
I

lbl < Ia!, we

630

a..-10 lnlioi1t "-"'"'""'' <;ones

ond now pnd the bracketed term by means of 10.33:


(a

+ b).. =

a.. ( 1 - m ( "
"ii )

+ 111(111 -

1} (")
ii '

m(m - I )

,
aM- b2 - ...

21

=a"'+ ma'" 1b +

2!

+ ...]

lb, < lal.


lbl < lal. 110 ~~

which. as we pre<licte<l. ~ equ.lli<HI 10.32 e>ceptlhallhe ..,.-ies .we~ not terminate. We reoom
mc.nd usc ofeither ofcquatioos 10.33 over I0.34; 1his neccssi<a< ..crea<ion of1he I. We illustralc
in lhe following cxnmple.

I EXAMPLE 10.26

Usc the binomial expansion 10 find I he Macluurin series for .r /(2 - .r) 3

<;OJ l'llOX

We demon<trale how 10 use both of equations 10.33a and h. Ry 10.33a.


X

(2 - x)'

.r)-l

= 2~ (I-~) = 2' I - 2
=

~[I+~ (- 3)(-4)(-5) ... (- 3 -11 +I) (-~)]


2}

~
n=l

.r [

L""

=2)
- I+
-

X [

-11 -

_
-

L..

n=l

~[

+ 2) (-

(- 1}"(3)(4)(5) .. (11
II!

">l

"'

(2)(3)(-1) ... (11


.
2'J+ 1u!

+ 2)

"

l)".t"]
2"

~ (11 + 1}(11 + 2) A"]

L.,

2+1

n:l

.r

~ (11

= .,~ + ~

X
,_+
?l

-L""

+ l)(11 + 2)
2n+4

11(11

nH
X

+,x
I)

,,. ..

"=2

1-11 < 2.
With Equation 10.33b. we \\rite

.v

x (

.r)-3

(2~).=
.r
2 ' ( 1 - -' )=211-2
=!..

[r +

p .

x [

(- Jl

3x

(-~)+
(-3)(-4) (-~)' + (-3)(~~)(-5) ( - ~Y +.
2
2!
2
.
3 . 4 .1

=23 l+2+2,2!"'

J . 4 . 5 x'
23 3!

+J

,4 .

s . 6 x + .. ]

24 4'

""' (11
= "
L

+ 1)(11 + 2) x" ~'


2'+

n O

""

- "'11 (11

- L

2JI+J

1) "

.\

lxl <

2.

n J

This resull was also oblaincd iot Example 10.2 1 by d i ffcrenl ialion o l' 1he Maclaurin series for
1/(2 - x) .

I EXAMPLE 10.26
Find Lhe Maeb.1urin series for Si n- 1 x.
SOL UTION

By the binomial expansion, we have

= I + 2'~ .-2 + ..2...x


+ :>. 5x" + 3 . 5. ?_.s + ...
212!
233!
2'J !

lxl <

I.

lntegnuion or Ihis, :.cries gives


Si n - x

= (x

35

23

222! 5

2'3! 7

- -x' + - -xs + - -x 1 +

3 5 79
2 1 4!9

+ ..

+C.

Evaluacion of boch sides o f 1his equ>1tio n at x = 0 gi\'es C = 0. >\ccording 10 1l~eorcm I 0.3.


the radius of convergence of this sedes must be 1. and we can write
. - I

Sut

x = x

L""
t~ = l

""
= x+L

,. 1

- '

oo

+L

11 = 1

I . 3 . 5 ... . (211 - I) :1Ml


.1
2" 11! (211 + 1)

I 2 3 4 5 ... (211 - 2) (211 - 1) (211)


2 4 ... (211)2"11! (211 + I )

(211) !
2n+l
x
(2t1 + 1)22>'(n!)2

lxl

< J.

2n~o
~

1t145a~KJt4~iC7~tliii---------.::-:::~~=:~:=~~=:~=:~~---------------------------------------,:IttJAI~-~Sf~+~4iz
--------.... '
po;"'
u.- rity),.,- '\o<CI'Ico~ f
IUno~;rH,..
---- ----------- --II .

Fcn,.~,._
I ln.
,.
......... - " ihu.J rile IVI-...:_-1
u lxut tlh,_ rnd,c.aff"d
aun,,

- -- - -

I he

J(.t) ..

I.

/ (r) - -

1..

3.. ..,.. 2

z. / (.) .;;., --!...._


c"" (o 1)

.I. /'(.<") -

4 . .l"'f.t) --

.I (.x)

r'

..,

~tinh ..-~

10. / (,!') - ,,.<

IJiluut ,. =r

utxmt ...1

:z..

1 , l.o 1

-:

/1. / (.) = .~ ' I .J.rt- :!A I


I

12. /(.. )

--~

+ J.J. /(. ) 2r

'+ .'f

14. j(:c) ~

IIIXJUI ... -

2J.

~-

I I

19.

J'('C)- In'

t1boul o

= ()

ul><>ul

= -2

:c'

3"=4X

aha-

I
C' - -

(2 -

,l ) '

"'""ll \ - '
~ - '

))'

J (<)

2$. j'(:c) - Two '

o.twut.t -0

2t j' ( 'l - .;:t'i1

bc.)Ul

:c

...r.+J w""'' ' - :

l 7. /(.<)

29. j' (()


aboot .\ = 2

C'\

'"-' - o

j ( r) c - -(>

Go

."'-"1 c.
~ s- 4

211. /(t)- ( I

ubu111 .r

l!.bl-.ut X

b.'"' ~ 2

/(r) .. ln(\-t ) )

12. / (<)

about .t ~ - J

111-o.t.a co

11. / CC')- H I .~~) l1:

21. /(f).__..!,__
(..\ 1 ltl

utw""' A' .--. 0

J.r

IJ. / (A ) - cu... tu
(,&~}

.."""".#c. u

I
/(t) . . .1'1( 1 + h ')

20. / (X) - 1/<

u1>oo1 .r o= .I

7. /(.1)-= r - l

9. /

uhottl ~- ()

ub!)Ut .t _

r "'

"'- f(.) ,. .-'

+ .. J

.J

!A)' '

(1+

,,~\ \ '

')'

30. .f(x) = ' (I - xl' '

abouo x 0

IJ1 E'cr.:o.><:> 31- 33 find lhc fo~ four nootcro latll$ in lhc M~><lautin
~nes for the functiOn.
31. f(.r) 1an2.\

J2.

(x) = ""'"

In F..crci<e> 47-48 usc Tayl.,.ric< to lind I fom>ulo


ti,cofthc function at A - 2.

33. f(.r ) ' in .r

JJ. Find the M aclcmrin sc:rit$ for <.v$2 x.

x _ 1r _

36. f(x) =Sin

(x

49. Show th:rl C\'Cn~onkr dcriv~Ui\'Cti of ,\' 2 sin l.\ at

JS. f(x) In

+3

.'C

= 0 arc cquul

to :zero.
)

50. Show th.tt oddonkr dcrhJ..thc ort

2x 1 +4
37. /(x) ' -1 4.1'

r.,. then"' deri'"'

=3+x

47. /(.<)

In E"<crciscs 35-~R find the Maclaurin s~rics ror the func1ion.


lS. j(.T)

= (.1 + 3x)'

45. /(.t)

+ xJ.fi]

I
r.
[ I - ;.f,r 2

.: 31 X

= 0 v:nish.

fi"'

Sbow llw Bessel fu:~tions of ....


lu"l (d<finod in Exercise 38 or
SccuM 10.4) satrsfy the propc;rucs 111 l:.lercoses 51-52 .

39. Pro'C 1he cnmllory to Thco<cm 10.4.


40. Pro"c 1h:u if J. pov.er seriu wilh p<Kili\e rudiusof convcrtc:n has
~um rem.
uo,..(x- r) = O.then a"= 0 f<Y all ''

E.:

.. t. If. tJu1 il\,!; II WOI-J..ing day, OflC JX:I"$0R drink$ (I'UIII U fOU111Uit) ('\GI')'
30' (on lhe !lvcc<age). then the probabllity thal cx.ncdy 11 J>Coplc drink in
:l time intcrvnl Hf length 1 '-CCOn(L<t is given hy the Pni .t~tm lli\trilmlitm:
P.(r) - I ( -I )
n! 30

"

-;====:~--: \. I - 2t-U

54.

L:npq-l = L:np(l - p)-.


,.,

e' - I

(b) If fJ is ahc: probability lhat a single Uic will cumc UJ) (i. is

o,

8) )

2!

3!

+ BoX + - x + - x + .

1he co.cO.cienl< 8 1, 8 2,

the un,wcr in 1>an (a) what you would ex peel'!

u,, ... :lre C!iiiCIJ lhc IJt!rnOulli

nmn/K>n. Wrilc lhs cqlHIIIOO m the fOnn

4J. t-1nd 4hc Maclaunn .;erie.< for Ihe error/lmNitm err( \ ) den ned h)'

= (e -

? }.' ' , -r dt
= ---

.fiio

I)

( I + BoX + o,, I l + .. ) .
2

u:d $l.lb:slitulc ahl;. M~l.lutin ~ric ror t:" lO find the fint

4-1. Find Madourin .scrics for lhc Fmn<l inr~rals C(x) and S(xl
lk:fUlCd b)

J.':.'m

(rrt 1/ 2)dt.

fic Bemoulli """"""'


(b) Silo\< llullhooJd llanoullr numb<ro all llfli>h for n 2: J.

ss.

~lha.t

,...u i-1/U

'
In ExcrciM:( 45-46 u;;.e Mttclaurin series lO fi nd a f<N ulul:a for the ,rn
dcl'h':ltivc Of the fum."' ion Rl .-= 0.

L P.(/1 )<".

,..-11

(a) If we ddinc f()- ICc' - I) 01.1:


0"" /(0)
I.
it turtl> out 1ha f (x) 11. MiOCIrin :.erics e>.PIl>iO<l
-...idl I)O:)ith-e ndiu.> uf c0f1\<rgt..ncc. \Yhen d1h e:<p~sion
is o.prcsscd in lJK: ronn

-"-- =

(a) Wh~t it~ the wm o f this series'?

S(.t} -

+ Al

lhc coefficients f', ( p ) arc <alled 1hc Lt~11d1r polp!oo!Jtrls. Find


P,.(p). P1C/l). P , (p).and Pl(/l)

.,.

crfu )

12 i~ expanded in u Maclaurin

...,

or

..

+ x l)

<erieto in x,

c-'1~.

.&1. A cat.:un C)lpc:rinx:nt is to be pcdonncd until it U su.c(uL nx:


prob.ll>lit) 1lwt il will be !itJC>CcSS(ul in any given aucrnp< io p (0 <
p < I) .and thm:forclh< probobili1y lht it will foil it q w I 1'
The cpccuxl number time th>l tbc CJ<pcrirncnt must be pctf'onncd
in Older 10 bcwccc..rulc:an be lhown to be "'P~nlcd by the 111finitc

.,.

53. If the function { 1 - 2tv:

2_

" (X)t' .
"'J
.f.-

f'<w a definilion of .f,(x). sec Exercise 38 in Section 10.4.

110.6 Sums of Power Series


Theorem 10.3 provides an imporcanttcchniquc for fi ndingsums of power :;cries. l fascri~ with
unknown sum can be reduced to a series with known sum by differentialions or in1egra1ions.
then the unknown sum can be obtained when 'lhese operations are reversed.

I EXAMPLE

10.28
00

Find the. sum of the series

L (II + I) .<".
..0

SOLUT ION Without the factor n I, the se ries wou ld be geo me tric. lntegrmion of (n l)x"
removes the fac tor. This is the idea: now lcL us formulate it muthcmatically. 1l1c nu.lius o f
convergen!X' o f the series is

I~
+2

R = Jr-oo
lim
n

= I.

If we denote the sum of the series by S(x) ,

"" (II + I ).r"'


=L

S(.r)

"'""'
a nd use Theore m I 0.3 to integrate the se ries te rm by temt. we o btain

J s (x ) tlx

But the series on the right is a geometric series with sum .r /( I - x), provided that l.r l < I,
and we. may therefore write-

r~(x) d.r =

l.rl <

- - +C.
l - x

I.

If we now differentia1e this equation, we obc:-li n

S (.t ) =

( I - .r)( l) - .r( - 1)

.,

(1 - .<)-

--.

= -:-

(I - .r)l '

and therefore

oo
'"' (n
L..,
11 =0

+ I ).r" = .,..,-....,-::
( I - x )l '

The. open intel'vaJ of convel'gence. is - I < x < I. If we sel x = I . the series l'educes to
L:: .o (n + I) = I + 2 + 3 + , which clearly doe.s nol have a sum. Al x = - I. we
have I - 2 + 3 - 4
- I < X< I.

...-..

+ , which diverges. aJso.

The intervaJ of c.onverge.nce is therefore

10.6 Sum~ of Powt:t Strie~

I EXAMPLE

635

10.29

Find the sum of the series

L"" -t x" .
tt l

II

SOLUTION We can remove the !ilc10r J I 11 bydiffcrem iation. and thcrcb)' produce ageometric
series. We proceed as follows. The radius of c onvergence of lhc series is

. I

R = hm

I / II

n-oo U/(11

+ I)

I = I.

If we denote the sum of the series by


"" I

= 2: -x".
n

S(x)

I)~ I

and differentiate the series 1em1 by term . we have


S' (.r)

"" 1
= L""-

-'

This is a geometric series with sum If ( I - x), so that


'
S(x)
= -I-

1-x

lmegration now gives


S(x) = - Ln (l- x) + C.

Since S(0) = 0. it follows that

0 = - .ln (l) +C .
Hence, C = 0. and S(.r) = - In (I - x) . We have shown therefore that
I
L"" -.<
"R:l

- In (I - x) .

II

lllc open i nterval of convcr~cncc is - 1 < x < 1.

I EXAMPLE

10.30
00

Find the sum uf 01e >e,ies

+ I ) 2,
L (-1)"(211
.r .
(2n)!

..

SOLUTIO!\ Widl\lUtthc factor 211

I. we sre a cosine series (note the x21> and the (211)!).

The fuc1or can be eliminated by integ:nnion. We now kno\v how to proceed. The r:tdius of
comrcrgcncc of the :series is

( - 1)"(211+ 1)

R = lim

(211)!
( -I)"+' (21!

+ 3)

+ 2) '
(211 + I j2 (2n + 2)
lim
n~oc
2n + 3

[(21! + I)
..... 00
(21!) !
IIIU

(2.1!

= oo .

(211 + 2)(21! + 1) (211)! ]


:.._
21! + 3

. "--- --:-.:---,--:-....:...:_

If we denote the sum of the series by S(.r) ,


~(-1)"(211 + 1 ) :),,

S (.r ) = L.,

x.

)
(211 !

..0

and use TI1e0rem I 0.3 to integrate the series term by tcmt. we obtain

.
J

S(x) dx

=L

""(-1 )" 2
- - .r +
(211)!
lt.CJ

00

+C

= x

1)"
L0 -(-(211)!
- x'ln + C = .r cosx + C.

,~

Oifrcrentiation now g ives

S(x)

= - .qinx + cos .r.

EXERCISES 10. 6

In ExcrciM:S l-I S find the SURl or !he power series.

,.,

.,

I , z:,u-1

~o 2.

3.

L"' (11 + l)x"

"' (rr2 ~ 211)x"


s. L

..,

"' <- r x'" ..


I:
ln + I

"
Lnzx.,-

- - t'
:L'
II+ (
"""
"' (-1 )'

,, 8. L: -n-_,.~
II

'""

I )xN 2

II.

f (" ~ ')x"

f_ (-I)"(Zn + 1) x"' ..

12.

"

n l

ra- 1

~I

7.

L 11(n -

f:("+
, +2}"

10.

11 2

11-2

el

""

9. L n3"x~'

13. ;C

(- 1)"(11 + 2) XlA

f: (-1 )'~ (211


1

15.

:1

(211 + J)!

(211 + 3)2" X "'


n!

- I) l.o+l

(2n !

f.

14.

(2Jt)!

~I

110.7 Applications of Taylor Series and Taylor's Remainder


Formula
When n function is approximmet.l by its u th.-degrc:e Taylor polynomial,

j"(c)

[Cx } "= [(c ) + ['(c)(x -c)+ - - (.r - c)2!

+ +

J ll (c)
n!

(.r -

c}".

rhe erro1 in doing so is the Taylor remainder,

R (c x) =
"

'

p+ll (- )
(II

(x -

I)1

I') II+ 1
'

where Zn is belween c and x. The .smaller R, is. the better 1he approximation. Because z,1 is
c.lo is replace r<"Hl(z.,) by some l~rger value,

unknown , we CIIJ\1101 find R.,(c , x). Wha t we

thereby obtaining a maximum value for the error. For instance. <.'Onsider using: the rirst three

terrns of the Maclaurin s~rie~ for e- to approximate ex on the interval 0 ::; x ::; 1/2. Taylors
remainder formula, with c = 0, states that

where
atl d z is betwccrl 0 al'ld x. Although z ls unknown -except that it i s. bo;t\\'t;o;:n 0 Rnd ,'(.we <:iul
say Omt bec.uuse only Lh.e \'Uiues 0 ~ x !: arc u ndtr (;l)ll ~ idcr.uiun, :::. mt1st be. l i:tis than ~. h

f(~IJ OWb< IIUtl

e 1/1,'< )

R1(0. X) < -

-- 5
6

..,/i( 1/2))
6

< 0 .035.

1' hus the (JtuMJn.ulc fur1c1ion l + x + .t 1/2 a wroxilmuc.s t ,.( on the intcrvnl 0 ~ .t ~ I /2 wlth
error 1lO gn_'lucr than 0 .035.
rn the follo w ing cxnmple, we detcrmh'X: the 11 umber M te:nn~: of a Mach\Uli l'l series l'eQUhef.l
10 S ll<:ll'\)ll(CC .il Ct rtt'ill 8CClll'aC)'.

I EXAMPLE

10.31

I low nlHII)' IC0111i i n Chc Mttela urin ,scriC$ fot In (I


10-o for tlllY x in the intcrvul 0 ~ .t' ~ 1/2'?

SOI.l..'1 10N The u' 11 derivutive o f In ( I

+ .'( ) g.u.UtUil \.CC a l fU I\(.Uti OI\ CITOr ort u~ t han

+ .r) is

(/"
(-1)11 + 1(u - 1)!
lol( l+.<) =

d.t
(x + I)"

'

and therefore Tay lors remainder fonnuht wi th c = 0 swtes thM

In (I +X)

xl

xl

"' .r-+- -4
2
3

w here
tJn+ l

R,.(O r ) = - -+- 1 1n(l


dxn

,\'n+l

+ x)l <~. (II

( - 1)"

+ I)!

( - 1)" n!

x 11 + 1

<z.. + 1)+ (n

+ I) !

11+1

-:-,-( , ""'+-,-I)i-,(;-:-'.,"+-,-I'"''"'
) +-"1
".t

Since z" is between 0 and ~\' and 0 ~ ,\' .$ 1/2 . we can state that ,r mus t be. less th;.u\ or equ.\1
to 1/2 . and :" must be greouer than 0. Hence.

IR.,(O, x)l <

(11

+ I)( I )+ '

(l/2)tr+l

"'

11-lis is less than 10-6 if

(11

I) 2"+ 1

< 10 _ 6

or

(11

+ 1)2"+ 1 > tri'.

A calc ulator quickly in<.lictlles that the s mal lc::;t vulue o f n for which this is lr\le is n = I S.
Consequently. if In (I + x) is t>il!Oximmed hy the 15'-dcgree I>Oiynomisl

on the interval 0 ~

x :::;

I/2, Lhe truncation e JTor is less than 1()- 6 .

1\vo)oungekx:trical engineers are having adisa&>reelllCilt and we miN ;.culc the argumtru .
Figure 10 19 bhO\\b tWO charges o( equal Size q, but of' Oppo>ite >iiJI>. di>tllnCt d apan.
When d i' <mall, the: ronfigumtion is called a dipole. One engineer argues that when
point P is \'try for away from the dipole. so that r is \try muclllal'gCI' than d . the charges
efTeaivel) cancel one another. and the po1e111ial due to them is zero. 'Ole other cnt.meer
duagr>. We ntu>t dedde which of lllem is a.vrect.
S II L 110' Th< potenualat P due tochargeq i~gi\en b) the formula V = q / (471 Erl 'o).
\\here fo i,acon>tant. Similarly. the: potential at P W.,tochargc -q is V = -q/(471fDT ).
TIIC potential at P tlue to both charges is

ljl . 81

~s
-

,, ___ (q- - -q)

4tr~o

--J/2 - - - - d f l -

r1

r2

'Ole ca:ine low applied to triangles P R S and P Q R gi'es

d'

rf = r 1 +4

ri = r + -d'4 1

rdcos8,
rdcos (Jr- 8)

d2

= r +-.,..
rdcosO.
4
2

Hence.

TI\C hinnmi:al exprm!ooion cun be used lO write

t=r:::;==;;;:::;:::==:=.=.
= ---;::==.;====
J rl + d 1f4 +rtf cosO
dl
d
r

1+-,+ - rosl/
4r-

~ [1 + (~
+ 'r!. cso)]
4rl

112

= r

_ (-1 / 2) (-3/ 2)
2!

(d' _~<o o)' + .. .J.


-'rl

When d i' very mlk.'h lu' than r . <o th;u term< in d 2{ r2 , tfl / r 1 etc.. are nqjli,,ble
comp.~red

10

d / r.

r"F:=:;'f';':=l=:===.=~ "" -,1 ( Jr' + d'/4 + rd ro;l/

~ + :!._
cos
2r

Simil:uly, / 'T':="T,";';I:===;==,; "" ( I


Jr 1 + d'/4 rtf cos& r

2(/r co

e).

II).

Hence.

v"" _q_ [:(I + !!._ c~o) -: (- !!._ e)] = qd II.


41T(o r

2r

2r

co

co<
41flor l

This shows ti1Uilhe potential due t<> lhc dipole does not vanish at large diSUIJICcs from iL
h i< <mall becau<e d is <mall ond r i< large. hut it is 1101 '"ro. Only when P i< on the
perpendicular bisector of the line joining the charges <so that fJ = rr /2) is the potential

t.ero.

limits
We hove customurily used L'llopitnl's rule to evaluate limits of the indctcnuinme fonu 0/ 0.
M:>elnurin a.nd Toyl<>r series can wmetimes be used to od\'antogc. Consider

:< - sin x
lim - - : : - -

... -0

.\ ,

Three upplicntions o f L"Hopitul's rule give u limit of 1/6. Ahema<ively, if we substitute the
Mnclnurin ~ries for sin x ,
A -sin.\
lim----,-- = lim - 1

x>

r-u

x~o

x>

.x -

~"

- -X3!l - ,\-5!S - ) ]

Here is another example

I EXAMPL E

10.3 2

'-S

E\'aluate lim
"
"here c > 0 is a C(l(IMam (see al<o Eercise 58 in Section 4.11 ).
4-0' i!t/1. - I

SOU :TION

We be~ in by makinjl thc change of \ariablc u = 1/A in the limit:

)_-s
hm

A-0 (ltC

/1.

=
-

lim---

..-.""" ~n -

We nuw expand en into it) Muclaurin seril!.),

>..-s
lim - ,;..--

-~ e<l4-

v'

= lim ...,..-- - --;:-;-----;:---

-x (l +cu+~~,2 + .. ) - 1

If we now divide numerator and denominator by u5 we oblain

>. $
;.-o- e</1 lim

I =

,.!!_~ c

t?

c'

c'

c6 v

-v4 + -2v3 + -3! 112 + -4! tJ + -5! + -6! +

o.

Evaluation of Definite Integrals


In Section 8.8 we developed three numerical techniques lor approxim;uing definite irucgrals
of functions f(x) that have no obvious antidcrivatives: the rectangular rule. the trapezoidal
rule. and Simpson~ rule. Euch method divide~ the interval of integmtion into a number of
subimern ls and approx imates f(x) by a more elementary function on each subintcnal. The
rec~angularrule replaces f(x) by a step function, the trapetoidal rule uses a succession oflinear
functions, and Simpson's rule uses quadratic functions.
Another possibility is 10 replace f(x) by a rruncated power series (a polynomial) 01er the
entire intelYal of imcgmtion. For instance, consider the definite imegr.ll
1/2 .

srnx d

- - x.
X

where (sinx)fx is defined as I au = 0. The integml defines the area in Figure 10.20.
'.. liiil
~

;:)!

=x 1 sinx

sin x

y; -

0.8

0.6

0.4
0.2

o.s

1.5

Taylor's remainclet fornoufa for sin x gi1cs


3
1
x5
d~
/
sinx = A - : _ - - - .. "- -(in ,o)

3!

dx

5!

,.

:.._ '"- R.(O..o)

...o 11!

I' here

R..

d"+'Cin.r) /
x~'
dx"+'
:.. (11 + I)!

co. x) =

and Z., is bellveen 0 and X . n 1erefore.


~in T

X~

xl

- r =I --"-- + .r- Rn(O.x)


.
3!
5!

d"
r " -. When "e take definite imegrdls.
where the term befon: .1 -oR. (0. x) is -(sin
x )/
d.r"'
r-o nr
12

1
()

1 sin.t
-d.r
.r

1'1l [
0

.rl

I - -

3!

xl

.rJ
+- .. + .t'-I R, (0' .r)] d.t
.S!
x;

= { x- . ,
3 3

+ -5 . 5 ,

= ~-

(l/2)l

J J!

- ..

} 1/2

(1/2)5 - ...

5 5!

(I

11/ 2 I
- R,(O.x)d.x ,
X

1'/2~R.(O,
0

x) dx.

Now
2
I/

I R,(O, .I)d~
.
-:

IJ.u
. ld"+(sin .r) I
Stnce
.I

I<

~ :;::::!u

d;\JI+I

{'/2 -;I R,(O, x) dx

Jo

1 2
'

11-: d"+ n+l


(sin x) l
~

d~

x"+

x=., , (ll + l ) 1.

Id.1..

I, it follows that

x"

1/2

<

1
0

(11

l) !

n+ l

={

dx

(11

~)(II +

} 1/2

I) !

7(1-1

(1/2)"+1
+
_;_:17)(.:...n_+_ l_)! .

Thus. if we write
112 sinx

1
0

(1 / 2) 3

3 3!

- -dx "' - - - X

( L/2)S

+ - - = 0.493 107639.
5 5!

then 1he maximum error is

(l/2)6
6. 6!

(l/2) 7

<Jr

depending on whether we regard x + .r 5/5! as a fifth or sixlh-dcgrcc approx imation


7
for sin x. Since ( 1/2) /(7 7!) < 0.000000 222 gives a smalle.rerror, we can say that

x 3/3!

1 2

0.493 107 639 - 0.000 000 222 <

1 .sin x

1
1

that is.

0. 493 107417 <

12
'

dx < 0.493 107 639

sinx

+ 0.000000 222;

dx < 0.493107861.

ConsequenUy. using only three terms of the Macla urin series for (sinx)/x . we can say that to
five decimal places
l/2 sin x

- - dx = 0.493 11 .
X

An easier anal)sis is given in Example 10.54 of Section IO.I J. It uses alternating series
ins1cad of THy lor's rcmninder formula .

Differential Equations
Many differential equations arising in physic.s and engineering have solutions that can be e.x.prc.sscc.J only in terms of infinite se-ries. One such equation .i s Bc.sscl's c.fiffc.rcntial equation of
order zero lo r a func tion y = f (x) :

.ry"

+ y' + xy =

0.

Before considcri ng tl1is somewhat d illicult differential equation, we introdure the ideas through
an easier example.

I EXAMPLE

10.33
Determine whether the differential equatioll

dy
- - 2y
dx

has a solution that can be expressed as a power series y =


convergence.

L:oauxn with posiLive radius of

SOI. U'n ON

1r

.,

Y = f(;P() =

= tro + a 1.r

ll11 .\'"

+ a;:.r 2 + ...

nc(l

is to be a sohni~""'fl of the d i1YC:rcntial


difl<-rential equation:

cqo~tio",

(at + 2a2x + 3rl ).t"2 + 4(l,x.J +

we may

~u~tiiUlC

the power series into 1hc

(flo+ tt 1,.: + (12x 1 + .. ) =

x.

We now ga1.hcr together like tcrm.s in the vaious )>owen; of .t :

0 = (a , - 2rto)

+ (2ll, - 2l! ,

- l)x + (3a 3

Sin<.:c tl~ power :<eric_'\ un the right IUt.s sun1 ~ro. ib


40 in Sect ion 10.5). <ltld therefore we must sel

2a2 )x 2 + (4a, - 2a;,)x' +

cocflic:icnl~ ntu$1 tlll vanish (sec Ex~!rcisc

a,- 211<)

= 0,

2az - 2 i4t - 1

=0 ,

3a, - 2a, = 0,
4<J., - ""'' = 0,
and so on. These cqumions imply th1lt

", = 2au:
I
I
a, = - (1 + 2a = - (I + 4a0) :
2
2
1)

a,

= -3 (12 =

a, =

- llj

- ( 1 + 4a0):

3!

.,,

= :_(1
+ -~<o>
4!

The pauern emerging is

+ 4ao).

a, =

,,

2.

Thus.
/(x)

= a.v + ?a
0x
\Ve can fi nd

2"-2

= a0 + 2llo.< + 2 ( 1 + 4a0)x 2 + + -ll!- (1 + 4ao)x" +

>

~(I + 4ao) ( 22!\ 2 + 3! x' + .. + 2


4
23

1/..

+ ..) .

lhc ~um of 1hc series in p:trcntheses by noti ng 1M 11he M aclaurin seric~ fOf eLt i s
1
1! A

(2x)l

(2x)3

+ (2x) + -2!- + -:l!- + ..

TI1erefore, che solulion of the differenli.alequatioll is

y = f(x ) = ao

+ 2aox +

= - -J - -X +
4

4l (1 + 4ao) [ e2 < - I - o')

'.
-J ( 1 + 4ao)e-'

J
X
= C e-'' - - - -

Ustng pcit'VC'r SC'tiC-l. to .M>Ive t~ dtffC'rC'ntial equ.-u10n in E:.:an1pk 10.3l i' ccnainty not the most
expedient method A fe.r simpler method will bt: di{.(I.Und in So;tion IS.l But 1M C'<am:pic
dc..UJ) llu, tnlf 1h~ ~dtwe b) whK'h pow-er u:~ rc u~d .o soi\C U.ffC't'tncial equations.
'Vc: QO\'\ .-pply the Jm)l;aJUI'l." In ~-.e~,. differential equ..Mk>n Of ou,Jet lC:fO,

I EXAMPL E

10. 34

Find a pnwer~eri~'t1Wlolution ) ' - L~n,.x". With pc:'l~itt\C Nldiu~of (X'M"f \ 'et'l:,enc<. for n c.sscr s
dift"crcntinl cquotihn of order ret(',,

xl'

+ )'' + xy = 0.

SOI.lf rtON f11 thi-: ex:.mple we uba~ldml the nc.a.:.n ion of Ex.un1pte 10.33. anti muintain
sigma lOLaliorthi'OU$hOut. When we substitulc y = L~ n,.A,J 1 uHo the \lifferentinl equation,
wcobtnin

o ft

..' L

...

1!!!1

l )a .l'"'- 2

u (n -

"" na .r"
+E

+ x E"" o,,.t"

n=l

...

L
,,<,- 1 )11.~"- + L na.tH
,, ::

In urdr.r tn bnn& these th~e MAntntatio.ns together Cb one. And combine teml;; itl like powC'rs <>f
x. \\e lowef the i\Ckx of " uun1ation in the last tetTn by 2:

\Vc nuw cumbitlC the three Mlmmations over their comnwn intt i'\UI, bcillnnin~ at n
write scpat\\tcly the" = I tcrtn in the second summation.

2. und

<v

a,

+ L: l n(n -

f)a ,.

+ tw + a"-:lx" - 1

1
Butt he Oflly \\U) d power ..ene:t Cd n be equal to zc_ro
zero: th.at ~.

i~

for all or~ cocfhtiCiltS to be equal to

n(11 - l)o. - no,. + a._, = 0 ,

,, - 0:

11 ~

2.

Thus.
Dn-'2

l in = - n 2-

a rccur~ivc I'Ch~tion tlcfinulg the unknO\\U cocOicient

2,

(1,

of x" in lCI'tn< of the c.c.x:ffic:ient a,_:!

of x"- 2 Since a 1 = 0. i1 follows 111>11

o =a, =a,
Por "

= 2.

For 11 = 4.

Forn

= 6.

al

=-

" =
116

= a$ =

ao
22"
a1

ao

"o

= 2 2 = 2"(2!)2.
42
24

a.
62

-ao

= 2"(2!)262 = -

ll()

26(3!)2 .

TI1c soluliun is therefore

T he function deflna.l by the infinite series

-oo < x < oo.


i s called the zero-order Bessel function of the lim ki nd.

EXERCISES 10 7

In Exc1dSl'S 1- 10 find a maxi111um possible error in

~ing

the! ghcn

ICntHi of the Taylor series IC.l<lppruxinli11C Lhc (unclion on the iniCrvill

~*

IJ.J._,

- 10.

11

sin.r tfX

14.

- rl

d .r

$pCCificd.
('2

I. e"' I +.r + :._


2

.fJ

+ -6

3.

~J

xz

,x l

t~ l+x-r -+-

+ 4. '"' I+ x

++ -6
2

for -O.OJ ::; x ::; 0

for

lx l .:: 0.0 1

.r l

5. ~in .t ~ .r- J!
..\'1

6. COSX

x~'

4!

(I -

17.

r - 1)

19.

r-n.:>

20.

r-n

for 1.r1::; 0. 1

~ I - - + -

21

to cv::Jlu:uc I he limit.

lim

16. Hm

cosx)'

I H.

3x'

('' +e-
e1

lim
y-

(t

eosx
x2

Ji+X-1
X

~)
X

lim xsin (

lim

I -

lim- 15. .--o


X

x < 0.01

for 0 .:S
xl

ll'>C series

14in x

l.
'

ln Exercises l5- 20

forO .:S x .:S 0.01

l' )

I)

--

In Excrt:i sc~ 21 - 24 d ctcnuii'JC whctc the M ac:h&u'in krics fu r dtc func-

Lion may be lt'uncntcd in otd<'-1' to gwuante.c the OCCUI"acy intlic:-ated..


7. In ( I -

* 8.

x)"' - x -

( I - x)'

J ... 2

xl

0::;

"' I + 3.< + 6x 2 + 10x 3

x ::; 0.01
foo

*- 21. sin (.t/3)

l.rl < 0.2

f:

22.. 1/

Ofi

Ji"'+"XT Oil 0

' 23. In {I - .r)

9.

l.rl < :rI 100

for

10. ln.t

for lx -

"o

..

I I~

I I.

1
0

sin x

leSS lhan JO-\

< .X < 1/2 with 4,:1ror lc~s thun I 0-*

l.rl <

1/3 with error less than 10

cos 2 x on lxl < 0. 1 with cn'Or lcs~ than 10- 3

In Exercise:, 25-30 find a series solutio-n in powers of X ror the llilicr-

1/2

dx

0 11

CIT()(

(.r- I)- - (.\ - I)2 + - (.\ - I ) ' - - (.t- I)4


2
3
4

Cllliul cquution.

In E'(crci:;cs 11- 14 C\<Jiuatc the imcgrult'OrTIIOLhttc decimal ptvc.cs.

l!i*

~ 24.

lXI ~ 4 w ith

iii

12.

'
"
1

cos (x2 ) dx

25.
27.
*

>J + 3y

=4

xy' - 4y = 3x

29. y"+y=O

26 y" +y'=O
*

+ 2y' + J
xy" + y = 0

28. 4xy"

30.

= 0

*-

~ll.

FirKithc naiUI"ll llo~unthm of0.999999 9\>')9 tH.'Curut.e to IS d.:ci

ll\3.1 pl:l CCS.

... J 2. In $pt.'ei~t l rclotivity theory, the kiRctie cnergy K or lin object 1110\...

ina with St>c~d

t~

f>,

is dcrlned b}

Po+ iPoVJ [I+ A:J + c~ ~ k) M:+ ] ,

where /J) = k Po/c~ .


where t' is: u con~tunt (the " I~C:tl o f fight}, mo i-. the rest m:tsJ. o r lhc
objccl, ;;~ntlm is ils rna~.. Y.hcn ntQ\'ing wilh spc:ctl v. 11"11.: nla!isca. m.
and m 0 arc related by

.. J5. Th<: ellir.>e b z.r 1 + a :y 2

tl 1b 2

<;unbel\:f)I<:SCntcdptli"JIIletrkully

by
1(

a: fJCc.>~l ,

= b s;nt.

~ I

< 2JT .

(a) Show that the lt:ngttl or the <;if\;Untfetcncc of tlx: clli 1)~ i~
definct.! hy th.: d..: finite intcg_nd

a'

Usc the binomial cxpunsi<mto show th>tt

- 1)2 .

(h) UI'C the binomi.al ex.sxmsion to shO\v that


und hcn~c. to u first uppro.~mla liOll .
class:ic~l CXJ)J'ession m,,vl/ 2.

kinetic et~rgy is delincd by the

JJ. Statznntton pressure P0 and pressul\: P arc rclutcd by the Much


number M in incompn:.-ssible flow of an ideal gas by the t.qu.ution

-p = 1+ -2-.
when: k is a con~an t. For cornp-euible flow, the

so dl<U

10

7kl -

3k'
)
(i4 - .. .

a first approxirmuion. L is the circumference of

u circle M r.'ldius b.

J6. T he well fi.mcl ion for leaky aquifcN. is defined by the convergent
im proper integral

kMl

l'u

/, = 2nl> ( I -

rel<~l.iot

is

P
o= [I + (k--2-1) M ]""-"
1'

(a) Show thm if t-~-:/(.f.u,\) is rcpluccd by a Mnclaurin ~Series


in fJ 1 /(4Ci.x). :lnd uucgration i.s done: tcnnbylcnu.

Assuming that

c~ I)M' <I,
exp:and the l ~ttc:rrei1Uion to sh..-,w 1hat for sola! I M. P,,j P in compt"('SS
ible now <.'an be appro~ ima tcd by P11 f P in ineomprcssiblc t\>w.
._. 34. rhc ligui'C below :cho~ unifonn . IWo dimc:n~ionul, c:ompn:$~ib!c,
adiitbtlti<.: no\Y or a fricaionless Ouid ~ round aciretll or obJCCt. The prc&-.ur~ is Po and tJe ' -cloeity is V0 i n Lhc undis!Urbcd llow to the lclt or
Ihe objeCI.

(b) Show tlwc the c:<.poocntial integrals

uti~fy

the recursion

rclution

,.. J7. PI(HH:l:"s law for d1c enc.rgy density \1.1 of blackbody ntdintilm of
wnvekn~th A stales 1hm

Altho ,;~agnation point S. the velocity of O>c Ouid is zero. and we


lei Pt be the :uagnatic>n prCSSilrc. 11 i$ known thlll

P,

Po

I + (k---I)
2

']'lt>-o)
M0

when:: Af o V0 fc 0 is Ibe Jvfnch nu1nbcroflhe flow ( C() is the velocilyof


pressure propagation in the undj sturbcd flow). and k > 1 is a t.'Onstant.

S;rciJI.

ell/{i.t.'/) _

wht.:rc h > 0 is Plancl. 's con.stam. ,. is the {constuu) speed of Light.


und T is 1CillJ>Cr.lturc. also ;~ssumcd (.'Of\Siant. Show lh:.lt for long wave
lengths. Planck's law reduces to the Rayleigh-Jeans law:

8;rkT

(b) Usc 1M binomiul cxp<tns:ion 10 $how l~l~ when d is very


mud less Ihan ,t , can be "pproxim atcd by

or

" 38. In !.he figure below, charges


q > 0 and -q coulombs one a
di $ti.Ulel' (/ ~lfXII't. Tnc: confiurmion is called nn cf~clfh'<lipolr.
(a) The electric field :11 pnin1 P due to 1hcsc churgc..;; i~

,.
G

where

4nlo(x - d/2)
1)

E "

4n l 0(.r + d/2)2 '

is a cort,.u.mt. Verify tOOl can be cxpn."'1cd


..
in

lhc (an n

d)-'] .

- ( I +2.1:

+ 39. Liquid Oows in the scm icircuhir Ru1nc l1\ the figure below.
(a) f ind an expression for the nnio of the <.TOSs-scctionaJ areu
A of 1hc nt'"' ltl the pnxluct of lr and d .
(b) u ~ the 1\:SU!I io pari ( ll) 10 Hn<.l an upproxunation ror the
ratio thuc indudc~ ~enns of order 9 2

.' <!))

II

---r ------ r---q

1 l 0.8

II

....!!!!.__
.
2n ~o-\'J

- - -- X- - -- --

Scmicin.-ular
numc

- - - d -- -

Comergence of Sequences of Numbers


fu Section 10.1 we introduced the basic ideas ofconvergeot.-e for se.quenccs of number:;. Examples were so sl mplc that we had no di fficuhy t.letcrrnining whether sequcr1ces converged ur
diverged. As this is not always the case. we discu ~s two impona111 ways to show that a sequence converges. Our concern i s panicularl y with recursively defined sequences. since ir is

usunlly Sltaiglnforwrtrd 10 dccermine whccher an cxplicic sequence is converscnc or divergelll.


For inM.tUX"C. consider the scquenct!

Ct =

I,

of Example I0.2 in Sec1ion I0.1. hs ne.<l four 1enns are

C2

= ().732,

CJ

= 7.955,

Co

= 8.155,

cs = 8. 187.

\ Vc suspect that the sequence has a limit. Why'! Because terms arc rapidly getting closer and
c loser 10 each other , or to put it another way. differences bet ween terms are rapidly approaching
zero. But this does not guarantee t'Onvergence. Differences between terms of the sequences

{Jill and {In 11 } get smaller and smaller as

11

increases, but neicher sequence has a lirnil.

The following two definitions lead to Theorem J0.7, which can be very useful in verifying
convergence o f sequences like this .

DEF I NITION 10. 2

A ~cq uen ce fc,.) is: said to be


(i) incre...slng i r

(',.+1

(ii) nondecreaslng i r c,..+l

>c.
~

for alln ?: 1:

c, for a lin

~ I;

11 0 JSa )

CI O.:l5h

c,,

for all tr ?: I;

( 10 15cl

s c,,

for l;lltu ?: I.

1 IOJ~d

(iii) decreasing i f

c,.+l <

(iv) noniucreasing if

c.+l

If a sequence satislic.s any one of the:,c rour propenie.s . it is said to be monotonic.

DEF I NIT ION 10. 3

A :rocquetlce fc"J is said tll have a tl upper boond U (he bounded abo\'e by U) i f

(1 0 J6a l

for all11 :;: I. It h<1s a lower b<mnd V (is bounded below by V ) if


(1 0 J6b
for a JJn ~ ), (f (I ~Ctllli!IX."e ha~ both an
boundt'.d Mquence.

Upper IJ.o~u'ltJ

and 3

fOWC

bound. it iS- ~tit)(() be 3

Note that if U is ar\ upper bound for a sequence. tlle:tl a11y number gre-ater than U is al~o
m1 UJlJ>Cf buund. lf V i s ;1 lower bound. so too is any numbel' .mmllcr than V .
Let us illu!'ltra.tc th~:~ definitions with some simple e:<plic:it ~uence!'l before stating our
fil":'t convcrgellCC t heorem and applying it to the ctbove recursive sequence. n te sequem.:c

-~-, }
{ 2-

= I .~2 - 4~.~8 . ...

is de.creasing, has an UJ>J>et bound U = I . tul d a l ower bound \1


"

;;-:j:"l =

12

is illCI'Casing. has "" upper bound U = 5. aod a

H- 1)"+1)

2 3 4
low~:r

= I. -

1.

= 0. The sequence

4
:s

bound V = - 2. The sequence

I. -1 . ...

is not monotonic... has an upper bound U = I , and a lower bound V = - 3.


The abc)\"e recu._n.;ive ~equence C t = I, Cn+ l = 5 + J2- c11 appears to be increasing,

\1

= 0 is o bviously a lower bound. and U = 10 1111Cars LObe an UJll"'' bound. The reason

why we would verify these conjecture...,. is contained in the following theorem.

THEOREM 10.7
A bounded, monotonic sequence has a li miL

FIGUAI 10. 21e

Grnph of inere<~>ing and bounded ~uence

c,

c"

o~\
08

0.7 t

tlilEli I Graph of ,lecrC<Jsing and bounded sequence

I 4!<)II.

0.5

0.4

0.3

0.~,~

0.6 f

___ ._._._._._._. _::.:.: . :.: . :___


...:_::....:~

0.1

l
5

LO

~--~~--~----~--~
0
5
10
Jj
20- I!

20 n

LS

To expand on this SllltCIIlCill somcwluu,co11Sidcra sequence /c11 I whose terms arc illustrated
graphically in Figure 10.21 a. Suppose that the sequence is increasing and therefore mono10nic,
and that U is an upper bound for the sequence. We have shown the upper bound as a hori7ontal
line in the figure; c 1 is a lower bound. Our illluit ion suggests that because the 1erms i11 the
sequence always increase, and they never exceed U, the sequence must have a JimiL Theorem
10.7 confirms this. The theorem does not suggest the value of the limit. but obviously it must
be less than or equal to U .
Similarly, when a sequence is decreasing or nonincreasing and has a lower bound V (Figure 10.21b). it mus1approach a limit that is greatenhan or equal to V.
Another way of stating 111eorem I 0.7 is as follows.

COROLLARY 10. 7. 1

A monotonic sequence has a limit if and only if it is bounded.


We are now prc:.parcd to give a complete
C1

I , Cu + l

fllld

typic"tl Uiscu~sion for the J'CCursivc sequence

5 + J2 +C.,.

( EXAMPLE 10.35

Verify 1hat tl>e sequence


C1 =

I,

Cn+ 1

= 5

+ J2 + c,,

It

> J.

has a limi1. and find iL

SOLUTION The first live terms of the sequence arc

c 1 = J,

c2 = 6.732,

c3 = 7.955,

C4

= 8. l55,

C~

= 8.1 87.

'lbey suggest that the. sequence is increas ing; that is, CJr+t > c, . '10 prove. tltis we use mathe
malical induction (see Apperldix A). CenaiJlJy. the inequaliry is vaHd for n = 1 since c2 > c1.

Suppose that k is an integer for which Ck+ 1 > C> . TI>en

from which

It follows that

5 + j2

+ Ck+ , > 5 + Jz + ck.

The left side is Ct+2 and the right .side is Ct.-+ I Therefore. we have proYcd that ct+2 > Ck+ l
Hence, by mathematical induction C11 + 1 > c,1 for aU n ~ l . Sjnce the sequence is increasiog,
its first term c1
I must be a lower bound. Cenainly, any upper bound, if one exist.s. must be
m least 8.187 (cs ). We can take any number greater than 8.187 and use mathematical induction
to te.<t whether it is indeed an U)>per bound. h appears that U = 10 might be a rea.<Qnable

guess forM upper bound for this sequence, and we \'erify this by u>duction as follows. Clearly,
< 10. Then

c1 < 10 . \Ve suppose thai k is some integer for which CA

Ck+ i

= 5 + J2 + Ck < 5 + J2 + 10 = 5 + .v'J2 < 10.

By mathematical induction, Cn < 10 for Jl ~ 1.


Since the sequence is monotonic and bounded. TilOOrem I 0. 7 guaramees thai it has a Hmi(.

call it L . To evaluate L, we roke limits on each side of the equation defining the sequence
recun:dve ly:

It is imponam to note that this cannot be done umil the conditions of Theorem 10.7 have been
checked. Since 1enns (;, oFthe sequence approach L as 11 -4 oo . it follows Lhat 5 + j2 + c.,
approaches 5 + .J2 + L . Furthermore, as 1t """7 oo, C11+t must also approach L. Do not make
the mistake of saying that C11 + 1 approaches L +I as 11 """7 00. Think tlbout what limn-, 00 l'rr+J
means. We conclude therefore that

= 5 + ..J2+L.

I f we tnmspose the 5 and square both sides or the equ(llion. we obtain the quatlrc.ttic equal ion

with solmions

L =

I I J29

Only the positive s2are root satislies the original ~ation L = 5 + J2 + L defining L , so
that L = (I I + -./29)/2. The othcrroot, (II - .J29)/2 ,.. 2.8, can also bcc.liminalcd on the
grou11ds that aUtenns beyond the first arc g1eatcr than 6.

.-.
Sequences do not have to be mono tonic tn be convergent. Convergence can occur for other
reasons. The following example iiJustr.atcs a second common way for sequences to converge.
Agaio we have chosen a recursive sequence as illuscration because for an explicit sequence. the

limit is usually ob\'ious. Considet the recursive sequence


I

Cn+l

- 2 + -,

II :::

Cn

I.

The first six lemlS or the sequence ilre

C1 = 2,

Cz

= 2.5.

c3

= 2.4,

C4

= 2.417,

Cs

= 2.4138 ,

c6 = 2.4 1429.

The terms seem to be clustering :uound a nunnber close to 2.4 14, one larger, one smaller. one
larger. and so on (Figure I0.22). UnforwnateUy, lack of monotony precludes the possibility of
using Theorem 10.7 to discuss convergence. In Theorem 10.8, we discuss properties that imply
convergence for sequences of this ty pe~ but it is helpful first to illustr.ttc these propet1ies with a
spccil1c example such as the one above. What are the properties that lead us to belie1e that this

II 141 t

1.5

0.5
0

6"

II

!'!t:qucncc con\ CfJtc..''t fon"'t. lhc 1ern1.., arc not moi'\Utonic.:; 1hcy arc ''ul,.down-updo" nupdown."
f-low 1.lo we =-~)' thi< tH.Ithenwticully? Differences bct'"ccn ~u.:ec.,~hc. tc.1 n~ h1 the ..equcnce urc

c, - <o = 2.S- 2 = 0 .5 ,
Cl - q

= 2.4 -

2.5 = -0. 1.

Co - C} = 2417- 2A = 0017,

cs - c, =
c0 - <s

2.~138-

2AI7 = -0.0032.

= 2.4 14 29- 2.4138 = 0.00049.

The fua that lheJe difference~ are mlternatcly po"\lti\'t and ne,ative unplief. that lhe \erms in
the ~eq ucr1cc {t,, t arc upo-cJo\vn-up-down-updO\\n. Thit. i~ nOt cnou:,h tO gua.nliHcc oonvcr
genre.-~ however. For c.xt~mph~ . the ~cqucncc I. - 1. 1, -1, I , - I. ... is up-down-up-<tow1r
up..down. hut it del(..; not C<)ll\'Cr;e. 11'1c 11dded feature of the ieque,,cc ~bove is \hat u.bsolutc
""l<ocs of <he dill'crcnc:cs

lr, - rol = 0 .5,

l<'l - c21 = 0. 1, \c, - q\ - 0.011,


I<'< - <ol = 0.0032 . I<- csl = 0.0004<)
seem to foml .- \1'-'Cteluirlg sequence with limit ero. We ha\'C l\aid ..seem.. bccllU:\e wt: ha..-c:
not proved thm the~ I'W\)()Cni~:, hQicJ ror all ditTcn::nces, 0111) the htllt live. \Vc Sl'h\11 pro.. ide

a gcnc:rul \C.'Iifieation after ~tati,,g the nc.-xt theorem. Huwtver. h h the "'~cJown.up.-do'vn
up-<lown namn: of 1he <equcncc toge1her with lhc loci thai ab<olute \'\llue< of tho: dtlf=~
dccrca. and approAch 7cru that lead m; to bclic\-c lhat the ~nee: lc" t ha.~ a Hmn Thi' is
fonnalired in the followon& theorem.

THEOREM 10. 8
Suppose a 'equen<:e IC'.,I ha< the following propeot o<s:

1. Dltrcrencc> C'.,+ 1 -c., allenme in sign.


2. Ah..ohuc v:~luc' of the<e differences leu 1 1 -c.,I t~re dccrca<ing.
3. 1\bSOI\IlC Vl\IUCS Of lhC diiTCrCilCCS IC.o+ 1 - l'\ upprouch 0 .

Then I he sequence ~ C11 1 converge~. and its limit lie~ betwt~l\ a1\y two ~uccc~sive tenn.s in
the sequence.
A sequent-e 1ha1 s:~1 islies conditio n I of this theo rem i5 s:~id 10 be ;Jn oscilla ting seque nce.
hs 1erms are up-downup.dOI>nup-dom. When 1erms also Slllisfy condi1ions 2 and 3, 1hey
oscillate aboul a limil - one bel01>, one abo\e. one below. one abo,e. and so on, but gr.!dually
lhey ge1 closer and closer 10 the limi1.

10.8

Convcrgcn~c of Seq 1.1~n<:c:s of Numbc=r"

651

We now verif)' that the sequence c 1


2. c+
2 + 1/c. on the previous page is
oscilh11ing and cowergcnt. The dillerence between the ( n + l)d' and n' h terms is
Cn+O -

Cn

(2

+ -I) c,.

2+

-(c. - c, _ 0)

- I- ) =

Ct~Cn - 1

Cn - 1

Since a11terms in the sequence urc clearly I>Ositi,c. the denominator of this expression is IX>Sitivc.
h follows thm the difference c,,+ 1 - Cn must tuwc the sign opposite to the prcviuu~ t.liffcrcncc
( ' 11 -Cu- t~ lhtu is. the ditl'crcnces c,. 11 - C11 nltcrnale in sign. Furthermore, since 2 is a lower
bound ror all tern" in the seq uence. it rollows that l'or II ~ 2,

ICn + l

c, I

- Cn-ol
= lenCnCn0

<

!c. - Cn-d
(2)(2)

lc., -

Cn- ol

But if each difference is less chan one-quaner ahe prC\'ious difference, che differences must be.
decreasing and have limit zero. 8)' Theorem ICI.S this sequence has a limit L that we obmin by
taking limits in the recursive definition:

L = 2+

=
Of the two solutions I

-.
L

J2 of this equmion. only L = I + J2 lies between co and C2.

Notice 1hat we did not use mathematical induc1ion to \'erify that the sequenc.eaboYe is oscil-

lutingand convergem. T his is chara<:tedstic of rOscillating sequences; nw.chemalical induction is


not required to verify the three propenies of Theorem I 0.8. It mny be ncccssal'y to usc ind uction
10 prove ancilliary results (such a.~ a lower bl)UIHJ fOt the sequence)~ but induction is not needed
to verify !he propenies of Theorem I 0.8.
An oscillating sequence resulls when the method of successh't appro~:i.ru_ations is applied
10 the equation f (x ) = .r 3 + 25x - 50= 0 for the root between x = I and ,t = 2. With

x. =

I.

Xtt+l

.50 - x 3
25

ll

?: I ,

the next 15 temJS are

x1
xs

= 1.96,
= 1.77,
1.7763,

Xs =

=
x,. =

XII

1.775 93,
1.7759~82 ,

1.70,
-'6 - 1.7782.
.\"') = 1.77581.
"12 = I. 775 954,
.tos = I. 775 946 9,
X3

X4

"'

~\'1 0

,\' 13
,\'16

1.80,
= 1.7751,
= I.776 00,
= I. 775 945.
= I. 775 947 4.

1llC terms arc indeed oscillating but appear to be appn.xtching a lim_il. Were. we gi ven this
recur!l.ive sequen<.-e without reference to 1he equ<ttion .\' 3 + 25.\' - 50 = 0, and us ked to discuss
iu; convergence, we would verify the conditions of Theorem I 0.8. In the comext of solving the
equation, however, we would noc. do 1his. \Vc have what we feel is a potential candidate for
the solution <>f the equation x 3 + 25.r - 50 = 0 hel\veen X = I and .r = 2. Tn verify thm
I. 775947 is the solulion c.:orrectl)t rountkd to ~ix decimal places, we use the zcro intermediate::
value theorem with ahe calcula1ions
/ (1.7759~6 5)

= - 2.7 x 10- 5

/(1.7759~7 5)

7.3

I Q- 6

Now thru we know whal iL means for a sequence 10 lm\'e u limil, and how 10 find Iimil.s,. we
can be more precise. To give a mathematjcal definition for the limit of a sequence. we stan with
our intuitive description and make a succession of par-i! phrases. each of which is one step closer
10 a precise definition:
A sequence {c" } has limit L if terms gel arbitrarily close to L, and stay dose to L, as n

geLS larger and larger.

J\ sequence fc,,} has limit L if ccrms can be made arbic,;uily close co L by choosing n
sutllciemly large.
A sequence I c.. I has limit L if differences Ic., - Li can be made '"bicrarily clo5c to 0 by

choosing It sufficiently large.


A sequence fc,l has limit L if giYen any real number ~ > 0. no nl3tter hQw small . we can
make differences Ic., - L l less than f by choo;;ing 11 suftieicntl) large.

Finally. we nnivc at the following derinitioOn.

DEF I NITION 10.4


A sequence /c,} has limit f. if for any given > 0. there exists an integer N such that
for all 11 > N ,

lc.- Ll <

f.

This definition puts our imuitivc idea of a lin1it in pn..-cisc terms. For those who have
Sludied Section 2.6. 1101c the sim ilarity bctwron Dcflni tion I OA and Definition 2.2. for a beuer
undcrsllutding of Definition 10.4. it is hcll>l-,al 10 consider its geometric itttetpretation. The
inequality lc11 - Ll ...:: e. when wriucn in the form /, - e < r, < L, +~.is interpreted asH
hori1.ontal band of width 2f around L (Figure 10.23a). Definition 10.4 !\.'quires thm no n~mer
how $JUall ~ , we can find a stage. deJ'\Oied by N ~ beyond which aU terms in the sequen-ce arc
contained in the hori1.omal band. F,Jr the :sequence and in Figure 10.'23a, N musl be chosen
a.~ shown. For the same sequence. but a smaller i. N must be chosen correspondingly larger
(Figure I 0.23b). Proofs of some resull< in the rc.a of this chapter require a work ing knowledge
or th is dcfiniLion. As an example, we usc it to verify that H sequtfK"e cnnn01 have two lirnils. a

fact thm we have implicitly as.<umcd throug!IO<ul our discussions.


liill?lil.l J (!] '""'

FICURf! 10.23b

,.

'

I.+"
L

L- '

1---.--=--.---.-----i- ~ v .... '


1-~~--+-c.._

.-

______

12

:nil terms in band

; all tcr111s in band

4 N 8

:: ~ ~.;;.:;:.;:..;;.~..;;-;;.~..~-~-~-:i2-~--;;-!:!-::;y2~...~-:::-

20"

t6

20 ,,

TH EO REM 10.9

A sequence can have ac most one limit.

PROOF We prove this by showi ng that a scquencccannoc have two distinct limits. Suppose
lo lhcconlra.-y that a sequence I c., I has two distinct limits L t and L 2 where L 2 > L 1 a11d
let L2 - t 1 = 8. If we set ( = IJ/3, then according to Dell nition I0.4. there exists an integer
N 1 such that for all n > N.,
lc, - Ltl < ( = &/3;

that is. for 11 > N t, all tenns in the sequence are within a distance 8/3 of Lt .

But since L 2 is also supposed to be a Jiolit, there exists an N2 such that for 11 > N2 , all
terms in the sequence arc within a distance f = 8/3 of L2 :

lc,. - l-2l < E =


Mjlflii;IMIFZU

lllustmtion of proof that a soq.cncc c:annot h;.w c rv,;o limi1s

L1 + t

-----------------------------1]-i\ifi.o,;;,; ---

- - L,

-------- .... --- .... ------ .. ] in here ....

1L _ ;

................................, .............. .

L 1 +e

tem1s : ---
---------,[All
--

~/3 .

- -- ------ - ----- - - in here ---- ~ ------ - -- - - -

- - L,

..

L 1 -e

-----------------.

.'

----------~--- --- - ------

N,

N,

But this is impossible (Figure 10.24) if L 1 aod L 2 area distance 8 apart. This contradiction
therefore implies that L 1 and L 2 are tlte same: that is. /c.,) cannot have two limits.

The fo llowing theorem, which states some of the properties of convergent sequences, can
also be proved usi ng Definition 10 .4. Only the first two parts. howc\'Cf. arc. straightforward (stc

Exercises 58, 66, and 67).


THE OR E M 1 0. 1 0

If sequences Ic.) and /dn I have limi!S C and D. then:


fkc,) has limit k C if k i.< a conant.

(i)

( ii)

/c,

(iii)

/c, d.) has limit CD.

(iv)

/c.,fd,) has limitC/ D provided thai D =F O.andnoneor thed,.

d.} has limi tC D.

= 0.

EXERCI S E S 10.8

In Exercises 1-24 detcrmjuc whether the statcmenl is tn.1c or false.

Verify any sli.ncn-.cnt tholl is true, and ghc a counterexample 10 any


~tatemem

that is false.

I. A sequence can be increasing and nondocteasing.

4. A decreasing scqucf'loe

~mtsl h ave~

mu.~ tu~

h as~

llm.i t, it tnust be bounded.

12. lf a sequence is bounded, bul not monoLonic, itc::.annot bave a li.tniL

lower bound.

13. If u sequence has a Limit. i1 must be bounded.

o. lower bound.

S. An inCre::t$ing sc.qucncc with a lower bound th ust have a limit.

6. An. inctcasing sequence witJl an upper botmd must have. a li.miL


7. I[ a sequence divcrgc-.s.lhcn cilhcr Lim11 ..., 00 c,

.10. lf a scquc nc:c is monotor:Uc aod

11. [f ;.l scqu-cnc:c is bounded and has a limit, il tnust be monotonic:.

2. A monotonic sequence must be bounded.

3. An increasing sequence

9. A sequence c an be both nonincrcasing and nondecreasing.

(X)

or lhn11 .,. 00 C11

14. If n seque nce is not monotonic. il cannot have a UntiL

J5. lf fJU 1cnns of n scquenee (c,.} ltre less th!m U. and L


lhnn ... (XI Cn cxis.:s. lht::n L ttmSI bt:: less lh:~n U .

- ()0.

8. A sequence cannot be both increasing and decreasing.

16. A sequence {en) has a Limit if and onJy if (c,~} has a limit

Chapter 10 lrUll'nte Stqut:ni..~!: and .series

654

.17. An usciJJating sequence UJU.:5LCQO\'Crgc.


Ut If ltnl\S of !l sequence are :l1tcrn:uely pOSilive and neg:u.ive, lhc

if a ::icttUCIIC:C coovcrg~. lhc-n diJTcrcoc:.c-s be


tween successive 1enns in the sequen1..:e mus1 appr"Oach zero.
(b) The following example illustrates thaL the converse is 110t
lruc.. Show Lha.t i.lifl'cn:n<.:c:s beLwt:-crl :;ucce-:,~ivc tcn.ns of
Lhe sequence {Inn) approoch zero. butlhe sequence itself
diverge..'i:.

.., 43.

(a) Pn.>\'C tbal

soqucnc:c is osciUutiog.
19. A sequence (t:rr} ofpositivenul_
nbersconverges if C,.--tl < C., f2 .

* 20.

A sequence {c.,} of numbers con\crges if CO?+ , < c,. / 2.

* 21.

l'f an increasing sequence hus a limit L , then L must be cquallO

Lhc smalksL upper bound for the M:qucncc.

In Exercises 44-47 show that the sequence i$ convergent and find its

'*' 2:2. If an i11finitc nwnbcrof tcrnls of a sequence all have the same value

limit.

a . Lhcn eill1cr a is the limiLofthc sequence or the sequence has no limit

*
*

23. If absolute values of differences bctwocn terms of an oscillating_


scquc.Jtce decrease. the s.cqucncc cOfi\'cgcs.
24. If tcnns of an osciUatiJlg_ soquc11cc. appro~ch zero, then terms must
be altcntatcly posit.ivc and ltcgati,c.

ln Exercises 25-41 discuss, with p1oofs, whether the sequence is


monotonic. and whclhcr it has ao upper bound. a Jowc.r bound. an<.l
a limit

* .26.

Ct

28.

29.

30.

31.
J2.

33.

10

3.

c l =I,

= ./S + c,.
c... .,. = ./5 +c.,

c+

c,

= 5,

=2.

= - -- .
4 - 2c,

c, = I,

C11;-1

Be''

16

+ 37. Co= I.

Cn+l

4c,
= - - -.

Cu-tl

39.

c, = 2.

40.

c, = 2

41.

('I

It!::

22.

(Ill ) = - - -.

3+ c.

= 0.

<"n+l

c.,+

Cn-1 =

J +?

+ c,

If ~

I,

Show thm the sequence

i~'\: conv.crgen1

+ 5c.,

rz. ~ I.

and fintl il!: limit Him: You rnay lind it helpful

tt)

Ct -

../26

I,

c,,

, :::. I.

is con\'ergenl and find its limit H i11t: Show lb:tl 4 :$; c,. :$;
2, and thea consider (C11 -tt) 2 - (c11 }!.

ll~

pr<we

tb.-.1 I/2 :S. c,J .::: 1 for 11 ::;: 1. Do this by mn1hcmatical induction.
S I. Show th;U the sequence

u:::.

5 for

11 ::;:
'1:"

52. Show th;U the sequence

c, = 4.

Co+!

= J2D

Jc.

II

> I.

is convergent aod find its limit.

II~

* 53. Show lh~il the sequence

II .2:_

4c" docs. not converge.

"~

'*'

-c,.
3 -c,.
= - - -.
5- 2cn
4

c,. ....

11

_s 2 lOr 11 :::_ 1. Do Lhj:s by llliUlli.:IHitt.ic.:itl induc;.tion.


49. Repeat Exercise 4-8 if c 1 = 0. ln this case you might w:.mt to show
thm I _!: Cn ;: 2 forn ~ 4.

54. Sbow LhaLLhc scquc11Cc i_u Exc.rci:;c 53 diverges for aoy lirst term
c>ecpl Co = 0 and Ca = 1/ 2.
55. Show th;.lt the sequence

c, = I ,

Sc.,

Cn+ l

= 2c.,- 1

1,

is e,-onvcrgcnt and find its limit. Uiut: Express c,..+ 1 in the fo rm


5/(2- 1/cn) and show I hal 2 ~ C11 ~ 4 for n ~ 3.

C11 +1 =

= 1,

.,

-Cn

CJ = I,

42. Show that the sequence

c,

2:

fl

~ Cn

16 - Be;'

3c.
= - - -,
1 + c,

c,..+, -

.,-:--:-~

4+c,

tl~

C.,

47. Ct = lO>

* 50.

Coo+ l

c,

= 12+ - .

c,....., 1 = ---,
2+ c.

c 1 = I,

thut I

=0.

36.

Cn+l

is con"crgent and find its limit Hint: You rnay 1 nd it hc::lpful ro prove

11 2:;

~~~

Cnof. l

fl ~

3- ""

34. Co = I.
35.

* 46.

8 - c,
5

= --- .

c..,+ =I +

C'u-1 = - - - ,

Cn+l

+ 48. Sbow thot Lbe sequence

ll ~l

./6 + c., n?:


c, = 3. Cu+ l =I+ ./6 + c. ,~
c1 =I. Cu l = 4- ./5- c. n.~
c, = 4, c..,_, = 4 - Js - t., , n?:
CJ

c, = 2.

45. c, = 20.

=~
(c' + s).
12

= 0.

21. Ct =
*

= - (c~ + 12},

25. Ct = I.

44.

n?::,l

is monoLonic and bounded. Find an approximatjoo Lo iLIO limit acC:UI'ate


to five dec:imall)lacc.s.

56. Show Ihat the scqucnc:c

c.= 3,
is convcrgcnl and find its li mit

10.'1 lnftnl!c Sclk S or NU!llba'S

f. 57. Show 1h111thc scq uen~

c,.

= 7b-c.""-c....,-1'

c, =a,

where a and b Hrc nonzero conslant.s di\'CI"g\.~ unless ab


Prove parts ( i) ;.llld (ii) url l!COIUTI 10.10.

* SH.
+.

59. Find bounds for the sequence


Ct

= 2.

= I,

Cn + l

64. A pttrttcu larbrccd ol"rabbiL-.; gm'o\'S 3nd repmduccs according lo lhc


foUO\\ing schedule. Each k m"!c rabbit becomes an adult and prod~
n pair of babies (one male and one (anafe) at lhc age of 2 months and
poducc:s t!xocdy one pair every monlh thf..--rcar'ler, Su1>1:10Se we begin
with Qnc pai.r of udult rabb.it..s who produce a pair of b:tbics ~ t the e nd
of the first m-omh. If no rabbits dit.:.. find :a se<1ucncc {Rn) rcprcscntiog
the rrunl.bc-t of adult rem:tlc r-abbit~ afler n ll10!Hhs. Do you rct:ugrti:t.e

il?

+ <'
1 + 2c.

65. Prove: that the $Cqucncc

II ~ I,

c, = d.

60. Oclcnninc: whctllcr lhc following sequence is monotonic, has


bc.n.utd~.

Ct :

:.mc.l hus a limit:

- 30,

c: = - 20.

CH+I

s+

c,,
2

+ 3'

II

2: 2.

111:*

(~\)

(b) if l im -"" c., ;; L > I. there exists an irucge.r N such


Ihal for all n a:, N.
Cu

11~
i-

>

ltl. - Dl <

whcnc,'<!rn > N1 ,

2(1CI + I)

I<. - Cl < 2 Dl ~

>

whcrtcvcr n

N 1.

L., there ex i.s.t~ an integer N such lhtll


(e) L:sc IIICSC rcsuJIS 10 pro' c plrl (iii) of 1'heor<m 10.10.

c. < L +

I.

Thi~ n..-su!t i~

usccJ in 11k:on::m I0.1 5. Sc:ctitln 10. 10.


63. The Filwmu.:ci sctfucm.:c found i 11 many ~~rca~ of applied malhc
mltics is defined by

* 67.

t non olon ic~

In this cl.c:rcise we outli ne tl l>roof for I)Url (iv) of Theorem IO.t O.


(nl \b'i fy lh!ll when <1.

I
I
c.
C
<1, - l>

(u) E\'uluatc the list I0 tcnns of dtis sequence.


(b) Is Lh<: sequence

I<. IJd, - Di + IDI c.- CJ.

lc., l < ICI +

n.:sull-; en: u~cd in 111C(lr"Cfl1 10.1 (), Scc1ion 10.11.

62. Prm-c that i f Jirn,._ 00 Cn


fnratll tt ~ N ,

(b) Show th:u given any e > 0. Lherc c.xis-1 posili,c intcgcN
N 1 N 2 mld N1 such thai

,,

1.bc.

Verify lhol

Jc.d,. - C Dl

L+l

1-

.J..

66. In lh~ ucrdsc we out li ne .u JHoof of parl (iii) of11cor~ ttl 10.10.

lhU.I ror all II 2:: N.

<

/o

Cnn =

(a. b . and d all positive constants) is incrcasi~g if an~ 011ly if d <


b + /t~+bl. Wholhopp<ns"hcnd = b + a + b 1

c.. _,

+ ti l . A sequence (c,,l has only posttivc terms. Prove Lhat:


(u) if litn.-ro cll - L < 1, there exists an integer N such

c.

&55

-# 0 nnd D <f' 0 .

1<. - Cl

ld,.l

ICIId.,- Dl

I Oil d., I

(b) Show lh:al given any > 0. there cxisl posili\'C intcBcni
N, . N7. and NJ such that

is il bounded. and docs tl have

a li mi t?
(c) Prove thut

ld.,l >
n

2.

Jc, - Cl

(d) Vclify 1hm an ex plieir fonnula for ciJ is

<

IDI

whci'IC"\.'Ct tJ >

EIDI

\\hC:OC:\'CC' II

<I DI'

Jd,. - DJ < .,.,:,...:......,.


~ICI + I

(c) I)Cfine a sequence {h11 } ~ the ratioorl crn~ in the Fiho1\t4Cci


sequence
Ca+l
b,.. = - -.

N"

> ;V!

whenev.crn > N3

(c) Now pro''C p:trt (iv) of11'14:0I'Cin 10.10.


(iS.

Find an cxplicil fonnuln for 1hc rccurti\C $cque.ncc

c.

Is lbjs sequence (tJOnOIOnk? Docs i1have a Limi1?

C1 = I,

C: =

2,

C11 +1

ll

:! 2.

656

Chlllp4tr 10 Infinite Stq\~ncto; and St~

I t 0.9 Infinite Series of Numbers


len I is an in linite sequence of numlx:rs.the

Infinite series of numbers arise in two ways. When


expression

L"" c. = c, + cz + c

+ +c. +

( 10.37)

II= I

is called an infinite ser ies of numbtrs. or simply a series. From the sequen{-e,~ of Example
10.1. we may form the following serie of nuunbers.

I EXAMPLE 10.36
Write oulthe first ~ix terms of the following ~eries:
00

"" <- 1)"1, 2:::

II

L n+ l

(al

--

(C)

It ~ I

31 (d)

!J= I

SOLUTION The first six terms of these >Cries are shown below.
oo

(a)~ - =
L., 2n-J

n=l

(b)

1+ -2 + 4- + -8 + -16 + -32 + ...

""
I
2
3
4
5 6
2:::
-"= - + - + - + :- + - + - +--
11 + 1
2 3
4 )
6
7

=
~

( C)

L (- I)" I

II -

31 = -2

I - ()

'T'

I - 2

+3-

...

n-l

L"" ( - 1)"+

(d)

= I - I + I - I + I - I + .. .

n= l

Infinite serie~ of numbers also ari~e when specific values of x are ~ubstitmed into power or
Taylor series. Fo r e,;ample, if we subst.itute .v = I into the-po,ver series
1 xn/(n~), we

L;:

obtain
I

-2 + 2-. +- + ... + -+ .. .;
2l
J . 23
11 2"
if we set .:r = -2, we obt~1in
"' (-2)"
L:
-=
n 2"
nal

( -1 )"

- 1+---+-.. + + --.
2
3
4
11

As mentioned earlier in this chapter, i t is not possible to add an infinity of numbers ina 11nite
amouo1of 1ime. and therefore expression 10.37 is as yet rneaoi.ngless. To auach a meaojng we
1ake the same approach as we did f or Taylor series and power series. We illustrate wi1h a simple
example, and then give a fonnal definition. Consider the infinite series of Example I0 .36(a).
00

L: 2-
n=J

1+ -2 + -4 + 8- +--.
=

If we stan ad din(!. tcnns. a paucrn won emctge11-. lnJeeU. ifwedcnu1c by

S, the s um of the finst

n ttrn\S of this scr~. \\C' find that

s, =

I,

(SUi11 Of first l \.\'0 IC-rms)~

S: = I + -2 = 2'
I

IS

I
s, = 1--+-=
-4
2
4

(sum of fitStthrcc terms).


(~um uf fir~l four term~).

s, = 1+ 2 + 4 + ii = s
S,

(:-;tun of first tivc terms).

= I - 2 + 4 + S + J6

and 50 on ' Vc see that the sum ofthe first

11

cerms of the series is gi\'cn by th.e formu la

2" - I

-2"-1- 22"-1.

11 -

As we add nl()fC 1.\rHJ more tcnn~ C>f this scr ics t<>g.cthC1'. dlC sum S, get~ clo!.Ct ond closer to
2. h is ahvd)'S les<lhan 2. bul S4 can be made arbitrJrily close to 2 by choosing 11 ~ufllcicnll)

large. If this scrk:s is 1.0 ha,e a sum, the only re-asonable ~m is 2. In practice. this is precisel)
1
wh..tt we do; \\C c.L.:finc the sum of thC! 1-eries L -;! 1 1/2"
to be 2.
l -et us now luke this idea luld dctitle sum:-, for general infinite serie.~. We begin by dctlning
u sccJtte1lCC

IS,, t o~ fo llows:

s, = c-,.

s.

= c, + ('~ + c, + ... + c,.

or

L:O

called the SC()tlrncc parlhtl !\UIUS of t he .serih


C;,. 'the u 1h term oft he SCQUC:IlCC ,S,.1
represent& the sum of the first II terms of the series. lfthis.equence has a limit, say S.lhen the
nwe temlS of the ~es thai we O<kl together. the c l<l'r 1he sun1 get$ to S . 11 ecms reasonlihlc.
then. to call S lhc $Urn of the .cries. We therefore n~nke the folkl\,ing definition.

DEFINITION 10.6

LetS, = E~-! c~ ~the ,w pa1tiuJ ~um of a series L~ 1


sums IS.} ha' hm11 S.
S = lim

-oo

we c all

c,. If the !tequence of partial

s.,

S the s um o f the series and wrile

if {Sn} does not ha\'C a limit. we s ay tffi.tt the scric~ doc~ not have a sum.

If a series has sum S. we say that the series connl'l:es to S. which means that itS sequence
or partial sums eomerges to S. If a series does n()( have a sum, \ve say that the series di\'erges,
which mcan.s that iL~ saJUC:nce of partial ~ums diverges. Partial sums arc: to infinite scric:.s o f
numbers what Taylor polynomials ore to Toylor series. Before proceeding with examples we
need to m~ke one comment abntll 1em1inology ami one about notal ion. l n the next ft!\\' seer inn~
we will be concentrating on se.ric.s of numbc.rs a.'i opposed to power series or series of functions.
\Ve shall usually write serit'S i ns1ead of St'ril'sofnuml>ers; the context will always ma_ke it clear
when we arc djscussing series of numbers. 'Vhenc\rcr the fir.,ttcm of a series corresponds to
n = I, we shall drop the limits n = I and 00 on the sigma. notation when the notation appears
in text. For instance. we shall write
c, in place of
1 c, . Limil!'l will always be retained
when sigma nomtion appears in a displayed equation. or when the lower limit is not 11 = 1.
According to Definition 10.5, the series 'L 1/2"- 1 of Example t 0.36(:o) ha< sum 2. Since
every term of the series L 11/ (n + I} in Example 10.36(b) is greater than or equal 10 1/2. it
follows that the sum of the first lltems iss. :::_ 11(1 /2) = 11/2. As the sequence of partial
" "'" b th~refore unbounded. it cannot possibly have a limit (see the corollary to Theorem 10. 7).
The series docs not therefore ha\'e a sum:_it di"erges. Examination of the first few pania_lsums
or the .cries -1 )"(11 - Jl in Example 10.."16(c) l~ads to the re.~ult that for II > I.

L:::.

L(

II - 4

- 2-

S.,=

. if 11 is even,

~~~~.

if tt is odd.

Since the seque nce {s .. } does not have a limit.thc series di\'erges. The panial sums oflhe series

L (- J)- are

s, = 1. s, = o. s1 = 1. s. = o.
Since this sequence docs not have a limit. the series docs not have a sum.
Perhaps 1he m~t imponant serie~ of number,;, and cenainly 1he serie_,:; that occurs most
frequently in applications. is the geometric s~ries. \ Ve discussed it in the context of power
series in Section 10.4. bm it is worthwhile rcpeoting the ideas here, uneneumbcrccl by other
considt:rations. Gromctric seric.'i arc of the form

L"" ar"-

= a+ or+ ar 2 + ar 3 + ... .

(10.38)

n=J

Each temt is obtained by multiplying the preceding lenn by the sa.rne conSl<tnl r. Ihe common
ratio. IJ {S.l is the sequence of partial sums for this series, then

S.n = o

+ ar + ar 2 + + ar n- 1.

I f we mulliply this equation by r.

r s. = ,,.

+ tlf 2 + ar 3 + + ar".

\Vhen we subtract 1hcsc equations. the result is

Hence, for r

I , we obtain

Sn =
Furthermore, when r = 1.

a(l - r ")

1- r

10.9 Infi nite Serie~ of \lumber;

659

and therefore the su m of Lhc fustn terms of a gcomccric series is

.S., =

{ a(l - r")'
1- r

,.

:f:.
( 10.3\ld)

r = I.

na.

To detemline whether a geometric series has a sum. we consider the limit or this sequence.
Cenain ly. limn- co na doe.~ not exi~l. unless uivially a
0. In addit ion. limn-oo r'' = 0
when lrl < I, and does not exist when lr l > I. Nor does it exist when r = -1. Tious. we

may ~tate that


lim S,.
ll-00

lrl <
= { 1: r '
docs o10t exist, lrl > J.

The geometric serie.s the refore has the sum

""

l: arn-1 =

(10.39b)

1-r

Jl =l

if lr I < I . but otherwise diverges.


The series in parts (a) and (d) o f Example 10.36 arc geometric. The first has common
mtio 1/ 2 and ohcrcforc converges to 1/(1 - 1/2) = 2: the second has common ratio - I and
therefore t.lh crges.
We encoumer the ha rmonic series

~I

( IOAOJ

L..., - =1 + - + - + - + ..
II
2
3
4

n l

q uite ofren in our work . h does no1 have a .s'Um , and we c.an show this by considering the
followi1g panial sums of the series:

s,

= I,
I

2'

s! =

I+- =

s, =

Sz

+ -3 + -4

>

- +- +-

Ss = S, + - + - + - + - >
s,~ =

Ss

+ -9 + -10 + + -16

2'
I

+ 8 + 8 + 8 + 8 = 2'

5
I
I
I
> - + - + - + .. + 2

16

16

16

(\

Titis procedure can be continued indefinitely a11d shows thm the sequence of panial sums is
11u~ ham1onic series therefore diverges (see once again the corollary 10 Theorem
10.7).
In each ot' lhe ex:11nples :lbOve. we used Oefi ni1ion JO.S for (hC~urn of a series 10 dctem1inc
wh~her the series converges or d ive rges: that is, we fo rmed the sequence of partial sums {S11 J
in order to consider i ts limit For most examples, i1 is either 1.00 di fficult or i mpossi ble 10
unbounded.

cvalua1c S, in a simple form. and in such c.ases consideration of the limil of the sequence {S,. J
is inlpractieal. Con..~queruly, we must de\-elop a ltema tive ways to de-cide on the. convergenc-e
o f ascries. Wedo thisinSections 10. 1()...10. 12.
We now discuss some fajrly simple bul im port ant results on convergence of series. U a
series
c,. has a (injte number of its terms a_ltered i n any fashion whatsoever. 1he new series
converges if and only if the originaJ series converges. The new series may co nverge to a d ifferent
sum, but it converges if the original series conve rges. For exampJe, if we double the 11rst three

660

Chaplec JO

ln fi <~ile Sequences Ol.k lSeties

terms of the geometric series in Example l0.36(a), but do not change. the remaining terms, the
new series is
I

16

2+ 1+-+-+-+ .
It is not geometric. But its 11 111 partial sum, cal l it S, , is very closely related to that of the geometric
series, call it r,,. ln fact, for J1 ~ 4, we can say !hat sll = r. +7/ 4 (7/4 is !he total change jn
the firstlhree terms). Since lim, .... 00 T,, 2, il follows that lim, .... 00 Sn 2 + 7/ 4 15/4;
thai is, !he new series converges, but its sum is 7/ 4 grea1er !han that of the geometric series.
On the other hand, suppose we change the firs! 100 terms of the hannonic serie.; , which
diverges, to 0, but leave the remaining terms unalle.red. The new series is

LOO tenus
I

0+0+ +0+-101 + -102 + .


We know tllltt the sequence of partial sums rr,, I of rhe harmonic series is increasing and unbounded. The 11'h parlial sum S, of the new series, for 11 > 100, is equal to T,, - k, where k is
the sum of the first lOO tenus of the hannonic series. Bu1 because lim,_"" T, = co, so must
lim. .... 00 S, = oo; that is, the new series diverges.
The. following theorem indicates that convergenr series can be added and sub[l'acted, and
mulriplied by consmnts. These properties can be: verified using Definition 10.5.

THEOREM 10.1 1

If series [:;'!, 1 Cn and[:;':, d. have sums C and D, then:


00

L kc,. =

(i)

(when k is a constant).

kC

(l OA Ia)

~~=I

L"" (c,:!: d,} =

(ii)

C:!: D.

(IOAibJ

11= 1

Our first convergence test is a corollary to the following theorem.

THEORE M 10.12

If a series

PROOF

E:;;-:1 Cn converges~ then lim11 -l>~ C

If series

11

= 0.

L c. has a sum S , its sequence of partial sums

has limit S. The sequence


0. St. S2. . . . , Sn-t , . ..
muse also have limit S: it is the sequence of partial sums with an additional tenn equal to 0 at
the beginning. According to Theorem I0. J0, if we subtract these two sequence-s, the resulting
sequence must have limil S - S

S1
But sl

0=

sl =

c l. Sz -

= 0; that is.

0, S2- S 1,

Sn - Sn- t. --> 0.

sl = Cz~ and so on; that is, we have shown thal limll~OO Cn =

0.

Theorem 10.12 states that a necessary condition for a series ~::;c. to converge is that the
sequence I c, I or i L~ terms must approach 'l.ero. What we really want are sufficient conditions to
guaraJHCC convergence or divergence of a series. \Vc cnn mke the conlropositive of the theorem
iUld obtain the following.

COROLLARY 10.12.1 ( n"'-Term Test)

If li mn-co Cn

0. or does nm exist. Lhen Lhe series

L:, c, diverges.

Thjs js our first convergence test. lhe n'h-term test. 11 is. in fact. a test for divergence rather
than convergence. stating that if limn-co Cn ex..islS and is equal to anything but zero. or the limit
does nor exist, then the serie.c;

L c, diverges. N ote well that I he n lh -tenn test nel'er indicates thm

limn~oo Cn = 0 , we can conclude nothing about the convergence


or dive rgence of
C11 ; it may converge or it may d iverge. Por example , the ham1onic series
l/11 and the geome1ric series
I /2" - 1 both satisfy the condition lim,_, c., = 0. yet one
series diverges and the other converges. The n'h-terrn test therefore may indicate that a series

a series com:erges. Even if

diverges. but ituever indicates that a series converges.


In particular, both series L (- I)" )11 - 3) and L ( - I)"+ 1 of Example. I0.36 diverge by
the. tJ th -term test.

To uoderstaod series, it js cruciaJ to d.istlng_


uish clearly among three entj6es: the series
itself. its sequence o f panial sums. and its sequence of tenns. For any series L c,.. we can fonn
its sequence of tenns lc,. I and its sequence of partial sums )S,. I. Each of these sequence" may
gi\'e inform:ttion about the sum or the series
c,, but in very different ways.
The sum of the series is defined by its sequence of partial sums in that L;cn has a sum only

if IS.J has a limit. The sequence of pa11ial sums therefore tells us defi nitely whether the series
converges or diverges, provided that we cao cva.luate S11 in a simple form.
The sequence of terms I c, I, on the other hand, may or may not tell us whether the series

di\'crgc.s. If fen I has no limit or has a limit o ther than zero, we know that the series docs no t
have a sum. Lf {c11 J has limit ?..ero, we obrain no information about convergence of the series.
and rnus1 contioue our investigation.
The interval of convergence of a power series (or Taylor series) is all values of x for
which the series comcrgcs. ll is de-lerminetl. except pos:sibly for e ndpoints. by the radius of
con,crg_encc. As we uneovor corwcrgcncc tests for series of numbers in this and tbc next chrce
sections, we will be able to discuss endpoints of intervals of convergence, and consec.Juently, be
able to convert open intervals o f c.onvergence to intervals of convergence. As a tirst example,
2
we show that the open inte.rval of convergence for the series
1x " f(n 4") is its interval of
coJwcrgcncc: that is, cndpoims arc no t included.

L:

I EXAMPLE 10.37
.,.

Find the inte-rval of convergence for the power series "" - -.t1n.

L
n4"
n=l

SOLUTION If we set y = x 2 then


00

I:

n=l

>
I
2n
=
~
, 4" .
L
--~

II::::: I

- y" .
n4"

Accord ing to equatio n I 0.24a, the rddius of convergence of this series is

Ry = lim

~~ ~oc

(fl

+ 1)4"+1

I)

11 +
lim 4 ( ll

n~oc

= 4.

Consequently. lhe radius of c<..Hwergence of the powe.r series in .r is R'( = 2. and the open
interval of cQnvcrgcncc is - 2 < .t < 2. At x = 2. the series becomes

""

I
'"'
L._; ll 4n (2)2"

, =1

00

= '"'
L._; H'
r1= l

I he harrnonic scric~. which diverges. The interval of convergence i:\ therefore - 2 <

I EXAM PLE

1 0.38

x < 2.

Sol\c the prubfcm in the Applicmion Preview.

Ap plication Prc\ic"
Revisited

SOl l'110N Were we to plot the amount of chemical in the reactor. the result would be
somewhotl ikc that in Figure 10.25.

il

3T

2T

We need to find a tbmmln for the amount of chemical in the rcac10r m limes 1

= 11T.

taki ng for grunted that the amount Ao has been injected at this time. We alrc.ndy know thul
A(O) = An. and A(T) = Ao( l + ek 7 ) . Duri ng the time intef\al T < 1 < 2T , the amoum
Ao( l + el.. 1 ) decrea:-.es exrxmentially so thai duri ng thi.s time interval

+ ek1)eklt-T> .

A(l ) = Au( I
As /

-> 2T - . this approaches

lim 11(1)

llo( l

+ ~,.)~' 12'~' -T> =

ll o( l

+ ~o)~r.

r ~l'l'

Once the injection of' Ao n1 t


A (2T)

= 2T

is 11111de.

= Ao + Ao( l + ,/' )e ~r =
1

A0( 1 + er

+ eu r).

There is a pomern emerging here lhnt we could verify hy mathematical induction. but suppose
we accetll that
A (11 T }

= Ao( I + ekT + e 2kT + + e"" ).

Thi::, i:') u geometric sunt, arHI if we usc fvnuuta I 0.39a.

. -

A(n T) - A0

[I - (e''~')n+l] I -

t 1'

Ao

[I - e<"+t)k:r]
I -

kl'

We can now say that the process should be teminated for the smallest integer 11 such chat
I _ e<+ l)kT ]

Ao [

I - ekT

> L.

This COil be solved for


II >

_I_ In[I - ~(I -

kT

Ao

ekT}] - I.

10.9 In ~ nile Series of 'iun>bcls

663

EXERCISES 10.9

In Exercises 1-10 dctcrmone l'<bclhct the series convcrgcsordl\'ergcs.


Find llr sum of each con,-.:rgont series. To get aj~ding for a series. ol
is helpful to write out its first few tcm11. Try il.

"" + l
Ln

l.

=I

2n

"'
'2:=(";)

3.

"'

i:(
. :J
=

4.

=1

s.

J1AU

'2:
,.-2
3

?;\ nZ+

9.

"' 7"+)

II.

26. find a simplified formula fonhc iii'C3 A. on Exercise 61 ofSCCioon


10.1. What i' lim. .., A.?

L"' COS~IiT)

""-here 11

10.

28.

~ I

is an inttger.

s. =

I:Tan .,.
il- l

0.666666 ...

+ 2r + 3r1 + ~r ' + .. .

<hew that

T. - S. = r(T. - nr"
Sol \'C t11i~ equal ion for

12. 0. 1313131313 ...

).

T,., a.nd e :~lc the lirn.i~ a.s 11

00 tO

>hOW lhal

...

14. 43.020 502 050 205 ...

lJ. UJ7 346 346 346 ...

,, :::_ 0,

(a) Accordong to equation 10.39a. lh.: n"' partial sum of 111<:


gcomceric series I + r + r 2 + r' + .. is
(I r)/ (1- r) . 1fT. dcnOic~eht:n" panialsumofthcscrics

In Exen:i~ 11-1~ \\C ha\'c given o repcalln& dc:oimol. E.pocs. lhc


dc:cinl31 ilS a sconlC:lric ~n~ and u.sc: fonnula t0.39b to express it" a
rational number
II.

+ .r + x2 + .. I

...,

3"

25. What distance docs the dog run from the time when it sees ohe
fanner until the fanner reaches the farmhouse in Exercise 57 of Section
10.1?

27. Find all values of .r satisfying llr inequality

eo 4n +3"

I:

24. Findthctimctakcn forthesupabsllonE\cn:ise56ofSoction 10.1


to come to rest.

'2:
.. ~

....

6.

,.z_ I
~~

7.

t'

L
s.,+

2..

2.l. Fond the total distance tta\'I!Ued by the supcrlxlll in Excn:ise 56 of


Section I0.1 before it comes to resL

""' n r"L...

In E>M:o5C> 1~17 compkl llr staiCmCnl and g"" a short proof 10


sub<tane.iatc )Our claim

= .,.,.-....:......,.-::
( I - r )l'

(r( < I .

(b) Verify lhi< rcsuh by setting S(r) =

:L;,nr

and in

tcgrJtins,.
16. If

C 11

C<'IWCfSCS Wld

L,d,. di"crgc8,1hCil L: (en+ d.) . ..

ln Exercises 2\1-32 usc I he result ol 1:.\erei<c 28to find I he sum of lhc


series.

In Exerci<e< IS-21 delennin< whclhc:rlhe<c<oc< OOO<tfi<'<O<dl\'te<


Find 1hc sum ol each COO\'I!fl:COI .eric>.
"XI

18.2:

"),) 3., - l
19. "
L... -2"-

2" + 3"

4"

=-1

"' z +4" - 8"

11.

'L

~"'

oo
II= I

11(11

+ >'

Him: U'C partial fn~etions on 1hc 11 lcrm and find 1hc: ~ucncc of

partial

'-Un1~.

2
5

25

125

625

-1

l + 2' + 21 + l' + ...

30.

-+-+-+ - + ...

Jl.

3 + l7 + 2-13 + 2187 + .. .

12
~
1'12
768
32. - ~- +- + -- "'

2S

125

625

JJ, T"''O people Atp o single coin IU JCC who com firs~ flip a hcuJ.

.. n. Find the sum or the scric.5

'L

29

SllQ\V lim\ the pwbubili1y that the Ot:!~ll).:f)On 10 Oip wins lhc ~~~n~ i'
represented hy Ihe ~ric~:

-+
2 -8 + -32 + .. +--+",
2,_,
Whal is the sum nf thiS series?

JJ4 TY..'O people 1~ .a di~ to K'C: who can fint throw o sh.. Fn'd the
iXUt.lbilily INIIhe J)Cr'tl "110 lhiOW> fii'SI wins lhe ~ne .

n.c

44.
Laplace lfllll~furm of A ruDCtion I (I) \\QJ; deti~ by en imJI'OPO' inrttrnl in the ins&NaiOI'b to ~tn.iK' 27- 30 in Sectioo 8 2.
Show tha ~ben / (t) lS 3 COiltlll.UOUS h.MlC"hOD Wtlh ~00 p. its
l...fipl.tee transfonn is gi"cn b) the: tW1nU.I') tl'lk..",rll

1'

In E.JI.C't'Ct!!CS J.S-38 show that lhe inten-al of ..-:on'c:r'encc of the power


~riel i~

the'""-.:., the open intci'V.d of convergence.

F (s) -

I - e

I''

/(I)P "1/1.

+ 36.

.\5.
1

37. ~2"(~) <x - J)

-:
'--

fill

38.

....,.

L n 3" x"
1

L: <->")H
-C

11 45. SuPtXl~ the \ Oitugc V !n applied In the crcuil in lhc left fi,eurc
below is ns shown itl the l'ight llgui'C. Cunc1Hs und vol uagc~> in lhc
~'i rcuil :.rc initially zero. ~rhc t\!ci Hict p:hMl~'> cwrcnt rrcd y (with no
resl"tar.cc) in the fotward ditcction, but p1-event~ currem ln the rcvctsc

dirccton.
(a) Show th.kt the din'tiCI\ltUI cquJtioo dc"ribing the volugc
across the: c;ap:acuOt IR til 11n14: tntcrval 2(n - I)T < t <
(lii - I)T. n; 1.2..... 1

.Y.>. One o( Y,L"tro"s pnn11/n~ dc5crihes a ti.tec bc:t\\ CCn r\chil ~"i ar.d
a tortchc. I.A:r.o ~l aum

lhtl tf lhc tortoise ~ z.i\CI\ .a head ~11 . then


no a~ let" l.()w b"-1 \dlillc-1'\.U't'., he ct.r. nevl!r c-.ltch 1M h.n"'~ - He
R'uurn." foll0'4~ In trdcr to wrch the tor"10is.c. Ac;.hill." n~ ""'
""'l c up the l<&h of the he>d st~n. But "hi!c he " nonn& thl>
d..ta..1cc, tt...:tonu...c ""'nub-u (urtherdii>l.ance. While Achlle. R\h~Co> up
lh'd'-UlOC. lhc tt'floi~CO\'Cn 1 furth::rdi)t.lllCc. :~nd '-',C'f1, 11 hll klWi

that Ach 1ll~ '' hl"li)' lllJl..in.tUI>dblana:eovcrcd by lhc te>ru.liK. :md


therefore can nt\a" cmch 11~ tonoise. If the 101101~ b Jl'\.11 .a tn:;,d
stun oflcngth L Jnd Achtllcs runs c > I tin'ICS as fast a; the l0f10i~.
o'iC inllnllc M..-ric~ lu ) how thut Achill~ doe$ in f:K-1 c:a~ch the lnrlt~k:
and 1h:.u thc th11tllnc-chcco' ('''Sindoingwib CL(c- l ) 1.

40. Pand 1hc lime bti\\CC'1l 1.05 ond 1: 10 whtl\ lht R\IRUIC nJ\d hout
lund~ nfuclocL. pi)tnl in the '\JnlC din:clio!'l (<~ ) hyrc.msoning wa1h inllnile
selit.:$ ~;;in t;tcrd..c 3911nd (b) by linding e'(pr~onJ for the un_aul.u
<~~pl!acc:n-.cnu o( the hand" as (urw:tlons of lime.

1
11\

-+rV=<>V,
dt

+ R:

ond a=

R,c

Hint: Usc the fJ.ct Ihilt lhc CUNUI lhnw.tth R 1 mu~t be the

R2 nd C tn order to find the


v!Jltngc :.1cro:o..~ R 1 .hen Lhc VOIWJc ..cro~ C is V.
(b) Sohe the: tliiTcrcntial e<.Jl&ution in pJtl (a) by mltltipl)ing
11 by t r r. By tlcnmi ng II~ vnlt tag~ :u.:ms-: the cap.tcilor Jtl
time 1 2u"/' by V., . s how thtil for 2(n - 1)1' < t <
(211 - I) T .
~um of the cuncnts thcou&h

av ( v.
v = -r+

,,, 211

Wh."lli$ l'llt = (2n

41. Repel~ I F..erci~ 4() (or Ihe instnl between 1()-50 cnl I0 S "hen
the h..lnJH'OOftCidlO.

42. A child llil< ullorl:<: ruo"""' or iJ.:.ntical cubtcal bkl<:l...._ "htdl>he


Slaci<S as hc>wn tn the foiiOfiOS fi~n l hc lOP bloc~ prot~ ; tiS
length ~\('t"tt\c ~cond Noel.. which protrude~~ i1~ lcn&th ovtr the ahird
bloc&.. whch prorndes ill ltnt.lh over Ihe: fourth blocL. ud M> 0"'
Assuminst 1hc cenu-c of n-wss of each bloct is 31 its gcon-.ctric centre.
s how lhut Lite (.1..1ltrc l:'f mru.~ of the top n blt)o.;.ks 1~,. d1n.x:1ly over the
edge t\l' lh.: (11 + I ) 111 hl'ICk. Now deduce thai if a $uff'w..ic.nt ruunhocr C'f
blocb i~t J)iJCd, t h e 101) block CWl be 1n:.uJc lO protrude Ult (~I 0\C:f Ihe
bouom block a~ dcsifod ~~itl1o u t1 hc stack falling.

~ r = R1 R:C

l}'ft

I) T

--

mr

R,

(c) Sho\V th.u th~ diiTcrcntiul t \(uulioo describin the voltage


aero~~ the c:apacitur in the ti me uuctval (Zu - I) T < r <
2nT is

dV
-tit

+ t1 V

= 0.

where

tr

= --.
R,C

Solve the difftrCitl1al C4lll:llion 1n 1')81'1 (C) nod lNe the fac1
lin
V should he- \' ( ( 211 - I)T) tL,.:occlculatcd
that

iopon (b) toshuwlh>tro<(.!t

(1.1)

1-.(111-IIT I

I)T < I <. :!111',

V =

-a;)-'']

["r\' + ( \'

(c) Set 1 = 211 T and \1 -

sbow that \f11

=:

fJ

II- 1Jo-IITI.

v. in Ihe full<'lion in pan (d) to

V.,_, + Q,

WhC.I'C

............

43. P I'OVC lhc (CliiClwing rc~ult.: If E~ 1 Cn (l()n\'c rgco:, then ils lctms
((Ill be gwuped in ;my rnMncr, und the resulli ng <~CI'iCS is, convcrgcnl
wi1h tht.: <Otun c l'.llm

n~

lhc orig,in:-}1series.

and

Now find an cxplit il I'Cll'ltlull for lhc SC(JUCil(,.'C {V"}.

4(). Hi~ <,; ust om~ry to li$WOX: th01t w hen a d .mg is ~dJ.tim:slt::.n:.d t<.l liM:
humnn hody, it will hcdiminmcrl cx,..oncmially: th:u i~, i f A rcpn:s.cml\

l.hc aiJlOWll Of drug in the body. then

(n - I ) T < r < ni.

A -= At...-- .tJ ,

(b) S.ket<h graphs of these functions on one set of ax.cs.

where k > 0 is a consl.3nt :md Ao is the. :tmoum injccccd :u time.


l = 0. SUJ'Ij)O~ n StiCC~AAi\'C i njcaio,ts<~of :lmoum Au llrr: :\rlm ini!>.tr:retl
at ~q u all y spaced time int~nats T . lhc first injcct ton at timet - 0.
( :1)

(c) Wh:at is the:: amoom or dtuj.\ in Lhc:: bo~y itruncdia.tdy :!.ftc..

the: u"' injccl'ion fo.r very l ~r;c n: thai is. what i&
lim A,.((11 - I)T)~

Show th~r the amount of dru; in the body ~t timc 1 bctwo:n

lt~'>O

110.10 Integral, Comparison, and Limit Comparison Tests


Ttl Scclion 10.9 we derived he nm lCml f~. u lCSt for divergetlCC of a ::;eries of numbers. Tn
order to develop fnrrher convergence and divergence tests, we consider two dasses. or ::.erie.~ of
numbc::rs:

1. series with terms that are all n01megative;


2.

serie.~

with bo1h

po~ i li\e

a nd t\tgarivete.mrs:.

A series with terms t.hat are all nonpositive is the negative of a serie-S of l )'pe I. and therefore
1 ~:..-s:t appl i ca bl~ h> ~t:':rit:':s of lypt:! 1 is c:asily ;ublpletl to a series w ith nouposilivt:': lt:':ru1s.

any

DEFINITION 10. 6

A series

L:,

Cn

is said tobc nonne.g ative if each tcml is nonnegative:

For example, the harmonic series

1/ tJ and the geometric series

Cn:::

0.

l/1!'- ' are both

notm cgativc series. \Vc have already seen that both series have the.propc.rty that 1irn11_ 00 Cn = 0.
yet the harmonic series di\erges and the geometric ~cries cotwerges. Examination of terms of
these series te\'eals thm I / 2n- l approaches 0 much more quit'kly than does l /11. In general,
whether a nonnegative series does or does not have a sum depends on how quickly its sequence of
terms {cJ, } approaches zero. The Study of convefgeoce of t)t)rUlegative se(ies is an iovestiga1.ioo
into the quest ion: How fast m ust tenns of a nonnegative. series approach zero in order that the
series have a sum'? T his is not a simple problem; only a panial answer is pro\ided through the
tcsrs in rhis secrion and the nex1. \Ve begin w ith the inlegr<lltcst.

THEOREM 10. 13

(Integ r al teat )

Suppose 1ha1 the tcnns in 11 series :L::"=t Cn nrc denoted by c. = .f(n), and .f(x) is a
continuo us. po:;itjvc, decreasing function for .t ~ J. Then the series converges if and
only if the improper integral 1"" f(x) d.r converges.

PROOF To prove !his resuh. we re1un1 10 the ddlni1ion of the sum of a series as !he li mil of
its sequence of p:mial sums. Suppose thsllhat lhe improper imegral converges to value K . We
know thal lhis value can be imerpreled as the area under the curve y = f(x). above the x -axis.
and to the right oflhelinex

I in Figure 10 .26a. In Figure J0.26bt..heareaof thenth rectang le is

666

Ctwpter 10 l.o.finile Soquench and Scnc.s

Comparison 3rea for inteuof<il est

y
~/(x)

......___

"= /(1)

=/(n)

A= /<2!

lA =/(4) I

A - /(3)

n-1

f(n) = c,,andclearlyitisless thanthearcaundcrthc curvefromx


the 11,. partia l sum of the series
c. is
Cz

11-ltox

+ .. + /."

f(x) dx

+ [(2)

< /(1)

Now

!.

'"- .. . + [(11)

1
3

f(x)dx

<

= 11 .

+ + c,1

= f(l)

= [(I)

S,, = c 1 +

f(x)dx

11- l

+ { [Cx) dx

[(I)+{"' f(x)dx

=/(I)., K .
What this shows is that lhe sequence IS, I is bounded [since f ( I) + K is inde1>endem of 11].
Because all terms of the series
arc positivc. the sequence [Su} of partial sums m us t be
increasing. It follows by ll~eorem 10.7 that the se<1ueoce (S. I must ha,e a limit: that is. the
se-ries
c., converges.
Conversely. >UPJ)()SC now that the imprOJ>er iouegral <Ji\erge.<: the area under the cut\oe
)' = f(x) . above the x -axis and 10 the right of the line x = I, is "infinite.~ This time we draw
rectangles to the right of the vertical lines at .11 = I, 2, ... (Figure I0 .27). Ttoen

L:cn

+ Cz + + c,J
/(1) + /(2) + ... +

S, = c,
=

>

!.
!.

f(x)dx

1;

/(11)

f(x)dx

1+l

+ .. . +

J(x)d.\

+1

f(x ) dx.

Since lim,...,. 00 J ;r+l f(x) dx = oo, iLfollows that lim~r-+oo Sn = oo, and the series therefore
diverges.

...............

----

A j'(U

)' = {(\')

.. /(J)

To uM: the integral test it

A /ll)

~ /(11)

r , -r

n+l

is roXSSary 10 a.ntidiffL-renliatc the fttnoi~._,..t

/(\)~ obttunct.l br

rcpl~ cin " '$ in (.', "ith .t $, When the mltideriVi,tivc aJ)pcarl) obv1ou~. and other conditions of

Theorem 10.13 arc met. theintegra1test muy bethee.l,iCbt wcly m dcddeon convergence of the
the ..natdtrh-:Khe i nol obvtQUJ. 11 n'Q) be bcuer IU ll')' another \e~rot

<ties;
f i!XAMPLI! 10.39

"h<

(11)

"

t...., "' +
I

"'
1
L:
-.. n II\ u

\ b)

--

...oumo'i
(u) Sh1ce j (x)

J.

rt;.

I /(x

+ J) i~ C<'minuous.I'USitivc. unU I.Jccrcllsinl

1r

- 2 -lfv =
-t

"' ,u.-n
L
,....,,

(C)

+1

If

11i-1 .r\"'I =--4


2

for

x~

l ,tmd

1f

it follow. tll!l.t ~'" .eric


1/ (112
1) """''<'!;<>.
(h) Since th t~ series heein..~ with 11
2. we modify the irueg.ra\ test by con,idtring the
iml>ropcr imegral ot' f(x) = 1/(x In.<) over the interval x ? 2. Since J(.t) ;,
~ithc:, CQIHinuous. and dc.crcluting for .t ~ '2. and

!,

...

11 follow. tllllt

1
- - d,,
,'( hl.t

lin (In,\ )

li"' = oo.

E;;.1 1/ (lllnll) diverg<$.

(c) Since xe- ~ i~ coutinuous, po~ilh-c. and llccrcasing rur x ~ I , anJ

!."' ,. -

d.t

I-xt- - e-)"'1

2.
-

che scric" conn:rgc.~ .

We mentkmcd before th;n mo \Cty important seri~ are geometric ~erie~; and hatm\>nic scric~.

Convergc11Ce of geometric seriL>\ was d1>eu"-"'!d in Sc~tiun 10.\1. The ham1onic :-enes be: Ions
loa lype of sen es called p -scr lcs, defined as follow.:
I

1+ -2P + -J f) +" '

( 10.42)

\Vhen fJ

I, the p-series becomes the hannonic series. which we know diverges. The series

diverges for p < 0 by 1hc n'"ceml ICSI. Consider lhc case when p > 0, bul p oj I. The
function l jxP is cominuous. positive. and decreasing for x 2: 1. and

l.

oo 1

- dr
xP .

-(p - l)xP-1

p <

}oc. = { 00,I

--~'
fJ -

p>l.

According to the integral test. chc p -series CO>Il\'CI'ges whe11 p > I and diverges when p < I.
let us summarize rcsullS for geometric and p -scries:

Geometric Series
:

:Lar"- =
1

p-series

tl=l

L _1
0.:

lr l <

{ ra 1

lr l >

dhcrgcs.

I.

= { ~mergcs.
dtvcrgc.'l.,

rt= l n.P

fJ >

"

< I.

(10.43)

h is unfonunace 1ha1 no general fonnula can be given for che sum of the pseries when
p > I. Some interesting cases that arise frcq,ucmly ru-e cited below:
0.:

L ~ "" I.202 056 1103 I '


11=1

( 10.441

II

Many series c-an be shown 10 converge or diverge by COm(laring them 10 known convergem and
divergent series. This is lhc essence of the following two tests.

THEOREM 10. 14 (Comparison Tes t )


If

0 ,::

Cn ,'$ On

for all n and

d" ~ 0 for all n and

L::,

On

cooverges, lhen

2::. d, diverges, then L:.


1

Crr

L=-: c,J converges.


1

If Cn

>

diverge);.

PROOF SupiX>.5C fi~ thai La. conYc.rgc.s and 0 ::=. c. :5 a, for all 11 . Since a, 2: 0 1hc
sequence of partial sums for the series La, must be nondccreasing. Since the series converges,
1hc sequence of partial sums must also be bounded (corollary 10 TI..:orcm 10.7). Because
0 ~ Cn ~ n, for all 11. it foiiO\VS that the sequence of partial sums of L Cn is nondecreasing
and bounded nlso. 1l1is sequence therefore has a limit and series
converges. A sintilar
argument c :~n be made for the di\'trgem case.

Len

Theorem 10.14 Slates 1ha1ifthctermsofa nonncgnti,c series L c,. arcsmallcrthanthoscof

aknown corwergent series. then L c" musl co1werge: if1heyare largerahana known nonnegative
di\'ergent series, then L c,1 must diverge.
In order for the comparison ces1 w he u~eful . we require a ca1alogue of known convergent

and divergent series with whic.h we may compare Other scric,s. The-geometric and p -scric-s in
equation 10.13 are extremely useful in chis respec1.

I EXAMPLE

10.40

Dc.tcmtinc whether the following series COll\'Crgc or djvergc:


1)0

(a)

(b)

..:L,

(c)

~ 212 + I
L- 15114 - 14
a !

SOL UTION
(a) For this se ries. we note that when n :::, 3,

In 11
II

>

II

Sin<.:! L~l If n diverges (harmonic scrie5 with tirst two term...: c hanged to ~ero), ~l

docs L~, (In 11)! " by the cornparisol tesL 1llUS


can a.lso be. "'CrifJec.J wi'h the integral test~

(b) Fonhc serie.<

:L (2n 2 -

I)/ ( 15n 1 + 14), we note 1hm

2u 2 ISn"

Since

L 2/ ( I5112)

L::.2 (ln u)/ n d i,e rges. This

l
2n 2
2
14 < 15114 = 15112

= (2/ I5) L II 111 cowcrgcs ( tJscric< wi1h I' = 2). so cloes

1he given series by the comparison IC$t.

(c) For this series we n<nc that the i1cqualily

+I

2n2

15n 4 -

<

3
112

14

is valid if and only if

BUll his inequali1y is valid if and only if

which is ohviou<.;ly true for u ;::, I. Consequentl y.

211 2

15n 1

+
-

14

=:;

3
--:;
n '

und since L 3/n 1 = 3 L 1/111 ~.:onvcrgcs. ~u does the given series.

1\vo obSC"'lllions abou1Example I0.40 are worthwhile:

J. To use the com!J..lt ison test we must first have a s uspicion as to whether the given series
converges or diverges in order 10 discuss 1he com.'CI inequality: Ihat is, if we suspcc~ thl\tthc
given series converges, we search for a convergent scri c~~ for the right side o f the inequality
!:. and if we suspect that the given series dh'Crgcs, we search for a divergent scrie:t for the.

rigl11 siclc of the opposite inequality :::; .

2. Recall thlll when 1hc 1cnns of nnonnegm ivc series nwrooch zero. whether 1hc series has
a sum depends on how fas t t hese. te nus approach zero. The only differcnt-e in u d, terms
of Examples 10.40(b) and (c) is !he posi1ion of the negmi1e sign. for very large 11 this
difference becomes negligible since each ,,til term can, for large n. be closely approx imated
by 2/(15n 2). We might then expect similar analyses for these examples, yet they are qui1e
d ifferenL In addition, it is natur.ll to ask whe re we obtained the factor 3 in part (c). The
answer is: " by trial and error."

Each of these observations points out weaknesses in the comparison test. but these problems
can be eliminated in many examples with the following test.

THEOREM 10. 15

lfO

(Limit Comparison Test)

=" c,. and 0 < b,.. and


.

c" = e,.

lun -

bn

n->00

0 <

e<

( 10 .45)

00.

00
00
00
then series ~n=l
~
cn conve ro-es
~
bn c.liverges
o if "......-r1=l bn conve ro-es
o , 1nd
~ diverges if 4..,..n=l

e,

L:

PROOF Suppose that series


b,. converges. Since sequence (c,. j b,.) comerges 10 we
can use the result of Exercise 62 in Section 10.8 and say that for all n greater than or equal to
some i nteger N.

or

Since. the. series L:~N (e + 1)b, = (


(b y the comparisoot.est) . H ence.
divergent ca.-;e.

Cn

< (f

l) b,. .

J) L:~N bn converges. so also must

L::,N c,

L;:,c, converges. A simjlar a.rgumeot can be made for the

If lim,_ 00 c,.jb,. = t. then for vel')' large t1 we can say that Cn ::, tb,.. h follows that if
(b,.) approaches zero. / c,. j a pproaches ~ero lje times as fast as (b,.) . Theorem 10. 15 implies
then that if the sequences of nth terms of two nonnegative series approach zero at proponiooal

rates. the series converge or diverge together.

L"

The limit comparison test avoids inequalities. To use it, we must find a series
bn so that
the limit of the r.uio Cu/bn is llnite and greater th an zero. To obtain this series, it is sufficient in
many examples simply to answer the question : What does the given seies really look like for
very large n In both Examples 10 .40(b ) and (c), we see that for large 11 .

Consequently. we calculate in Example 10.40(c) that


211

2+

1511 - 14
t = lim
=
2
n~oo

1511
Sioce

I EXAMPLE

L 2/ (1511 2 ) cooverges. so does L

11

2(2+ ,:2)

.~"'oo n ( IS -

1511

-14)
114

(211 2 + 1)/(1511'1 - 14 ) .

10.41

Dete nnine whether the following series converge or diverge:

(a)

~ J11 2 + 211 - I

J!=l

nS/2

1511 - 3

00

(b)

2"

2:: 3" +5
11=1

= I.

10.10

lntel)r.ll. Con>p<orii<JO. and limit Compori<oo Te<t>

671

SO!.(.;TIOI'
(a) For very large 11, the 11"' tenn of the series can be approximate<! by

Jn + 211 11S/l + 15rt -

II

11 5/ 2 -

J.

PI ;/2 .

We calculate therefore that


.J11 2

t=

lim ,s/2
n -~

+ 211 +

II /I+~ - _!_

1511 - 3

= lim _ _,\:..__-:':::'-...:':.:.'1
--::-""' . 11 3/2 = I.

n- oo S/l (
II

II 1/2

Since

1+ 113/2
- I5 - - 3s12 )
11

If n 311 converges ( pseries with

p = 3/ 2), so does the given scr ico by the

limit comparison test.


(b) For large 11.

+I
3" + 5

2"
We <".lieu late therefore that

2''

3"

+I
+5

2"(1 +~)

5)
3"

= ..~~ 3" ( I + -

= n-o..lt m ( 2 ) "

3"
= I.
2"

(2/ 3)" converges (a geo metric scrics with r


comerges by the limit contparison tc~l.

Since

I EXAMPLE

2 / 3) . the given series

10.42

"'

F ind the interval of comergence for the power series "

11

SOLliTIO); If we set )' = (x - I) 2 then

(" + I} 32"

211(11

(x - 1) 2" .

'The radius of convergence of the series in y is


II

R,. = lim
.

n- o.::

(11

l)l2

n +l

tun
n-oo

+ 2) 3
=
(11 + l)'

2.

The radius of convergence of the power seoies in x - 1 is theoefore Rx =


interval of convergence is I - J2 < x < I + ./2. At endpoints x = I

series becomes
00

"
L

n=l

(n

+fl 1)3 .

./2. and the open


./2. the power

I Ff i? z!Hfi!i SC,Bi.M

' ...i:-1
, 211 .- '
~

4,

..

"' t~
,. .,

.. bi~ ,,

....1

..L. , !..:!:..!L!!.
~
Ln"
" ..
, '

6
" ,:,:.;-=-,.
. ";:::.,-.+
=s

' __I:, ;,::---,-;-;,--,..,( 211


1)( 2n +

L. ~

-t I

- Sn 2 - .111 -

2:,~
,

:~-~

19.

. .. ,

2.~~

=t
L: u- ,z+ 4
$.

" f:~

II 'T' \

21.

L:'Vi:11
,._ ,

II

I)

r::~

9.

-.t

ro.

Itt II

.jill + 211 - J
1/

l,J,

f;

11

+ .4$

N
I '
;T
15. ,/_,
"
- ' '" ( n- )
2"

,_,

"

II

r-,-.

u.r~
11 ,,, + 3

+ 2 11 i J

211 .. -

L "2;. - :...
.,_,
~

..,

14, L
"
16.

t'J.

n- Z fnn

v'n"+T.....
n -

II '

..,

11

21!. ""'
-tt(\tw)"
L
-t
I

29. " L.., (Inn)~

10.11 Limit R atio and L imit Root Tests


rn rhis sec(i()n we consider two nddilional lC$tg to dcu: rmint whethct nunnc.:gatlvc sctic:s CUt\\ 'Ct iO
or diVCflr!C. The tir~t tc.'!t. the lhuit r.,tio tc.st, indkatcs whether a s.crle:) L. c,1 rc.sen,h\ts. a

..:comctric series for lnrgc

11 .

TH EO R E M 1 0. 1 6

(Limit R ati o T est)

Suppose that c, > 0 aod


Cn+l
,
I1m - - =

,.....,.00 c,
Then:

(i}

(ii)

(iii)

L.

( 10 .46)

L::, c, converges if L < 1.

E:, Cn diverges if L > 1 (or ir lim,...,.

L:

1 Cn

00

Cn+l fc,. =co).

may converge o divege if L = l.

I:CtUII;I41WJrl
SchemohC f()r pnxlf ot' li.mit ratio
L

FIGURE 10 . 28b
ttsr

f.+ I

l.+ I
-2-

I'HOOF
(i)

By the re.<ult of part (a) in Excrcise61 of Section 10.8 (sec nlso f igure 10.28o). we
can :say 1hat if lim, _ oc Cn+dc, = lJ < I , there exists an inreger N such 1hat for
all 11 ::; N .
Cn+ I

c,

<

L+ J
2

Conseque.llly.

("11+1

Cl/+1. <

C'/H J

I)
(L-L
- - I)
2
(L- +I)
2

< (L

<

r ,v ;

CN+I

<

f N+l

<

(L+ ~ y
(L +~ y

--

CN

-2-

CN

and so on . Hence.
CN

+ CtH I + CN+2 + ''

<

CN

+ (-L-L
- I)
2

- )
+ (L- 2+1

CN

CN

+ .. .

Si 11ce 1he right side of this inequality i~ a geometric series with common ratio
(t + 1) / 2 < I, it follows by the comparison test that L~N c. CQil\'Crgcs. noereforc. L~. 1 c,. converges t'lso.
(i i) By the res ult or part (b) in Exercise 61 of Section 10.8 (see also figure 10.28b). ir
lim,._ 00 c,.+.Jc11
L > I , there exists a n integer N suc;:h that for a ll n :;: N ,

Cu+l
L +I
- - > - - > 1.

c,.

'Vhen llm11 ~ec c,+ 1/ c, = oo, il is a lso true Ihat ror JJ greater than or equal 10 some
N , Cn+lfc, must be greaterthan I. This implie.>tlult for all II > N . c. > CN . and
therefore
li m c,. "' 0.
,_00
He1~e-c L: c~ lliYctgc~ by then'" -term lest
(iii) To shO\Y that the limit ratio Lest i!\ inconclusive whe.l L = I . consider the two p series
I I 11 anti
I I 11 2 . For each ocries. L = I , yccchc llrst series tlivcages anti
the second converges.

L:

In a oo nrigorous way we cao j ustify the lirnjt ratio tes1 fro m the followi ng standpo int. lf
lim,_,. 00 Cn+ 1fc, = L . the n for large n, each term of series
c, is essentiaiJy L times the
tcnn before it; that is, the series resembles a geometric series with common ratio L. \ Vc would
expect convergence of the series ir L < I and divergence i r L > I . We migJn also anticipate

L"

674

Chaper 10 lnJini!e Sequerlces andSeries

some indecision about ~he f... = 1 case. depending on how 1his linti~ is reached. since 1his c.ase
corresponds to the common ratio that separ.lle.s convergent and divergent gcomeuic series.

I EXAMPLE

10.43

Determine whethe-1' the following series converge or diverge:


oc

2"

00

I: ,4

(a)

I(H)

I: .,...--::---::---::-----,:1 . 3 . 5 . . . (211 - 1)

(b)

na l

(c)

SOLUT IOr\

(a) Forohe series L 2" I 11 4

(11

L = liJn

+ 1)4
2"

= Lim 2 ( - " - )' = 2,

~~ ~oc

+I

n''
and the series therefore diverges by the limit ratio test.
(b) For ohis series,

(11

+ 1)")1)

I 3 S

L = lim

(211

+ 1}

17 100

n-oo

100

= lim ('-' _+_ I)


n

n -Poo

- - = 0,
2n

+I

I 3 S .. . (2n - 1)
aod 1.he series therefore cooverges by the Limj1 ratio test.

(c) Since

L =

lim

= lim

n"

n-.oo (

fl +1) " = e > I


1J

n!
(see equation 1.68, Lhe series

L 11n / n ! diverges.

\ Vc present one last test for nonnegative series; Lhere arc many olhers.

THEOREM 10. 17

(L i m l tRootTest )

Suppose thao c., 2::. 0 and

lim ~ = R.

(10.47)

n->00

Then:

L: c, converges if R < 1.
L::,
if R > I
if limJJ~oo ~ =
(iii) L:
may converge or d iverge if R = _
I.
(i)

(ii)

1 Cn
1 Cn

diverges

(or

oo).

10..11 l;ntil R.atiy ~ nd limit ROoJI Tc ~

675

PROOF
(i) By the rcsull of part (a) in Exercise() I of Section 10 .8. if lim,._., .\(C;;
there exis ts an imcgcr N s uch th<\l for alii! ?:: N ,

= R<

l,

R+ I
2

Consequently.

R+l
2
"' ~CN+I <
1

"'VcN+l

<

R+

1.

R+l
2

or
or
or

< (

CN+l

R +2

I)NH ,

and so o n. Hence .

Since the right side or this inequali ty is a convergent geometric series. the le ft side
must be a con"crgcnt series also by the oomparison test. Thus. L~N C11 converges.

and so must

L:!: 1 Cu.

':jc.; = R > I. there exists an

(ii) When lim.,_ .,

R+ l
~ > -2- > 1

or

imeger N such that for all 11 ::: N ,

( "l t

> L,

a s in part (bl o f Exercise 6 1 in Section 10 .8. When lirn,._ oo ~

oo. it is a lso

true that for '' greate.r than o r equal to some N . ~ mu..'\t be greHter thml I. Rut it
now follows that lim,._ ., c. # 0, and the series dive rges by the " '" ccrnl tesc.
(iii) lb show thai the test is inconclusive when R = I . we >how that R = I for the
two p series
1/ 11 and
I j n 2! di"crging nnd converging. respectively. For the
harmonic series, let R = lim,l ..... ;);> ( l/ n) 11" . lf we take logarilhms, then

In R =

In [ lim (.n!.);] = ,._lim In(.n!.)


::.o

n~ oo

= -

II ,

lim--

,,-oo

lim l /n

n- oo

In(.!.)
=
n

- lim

n -+oo

In n

(by L'Hi>pital's rule)

0.
Hence. R

I for this divergent M:ries. A similar a na lysis gi ,e.s R

convcrgcm series 2:

I EXAMPL E

I/ =

II 11 2.

for the

10.44

Dctcnninc wh~.;tlu.:r the following ~ric.s con,crgc or di,crgc:


co

(b)

""

u=l

(In11)"
11

SOLUTION

(a) Since

R =
theseri~

~m
n...,.oc

It+

'].,.

~-

()
11

II +

= n...,.oo
lim ( -

Jt

"

t!

> I,

diverges by 1he limi1roo1 1est.

(b) For1his .cries.


11
= ~~ lim
[ - -- ] '/" = lim
-oo (lnn )'
-o.:

R
If we .set L

= lim,

1_ 00

1111
"

Inn

u 1fn, then

In L :: In ( lim n
n--oo

1
'")

= lim !l
n-oo

= lim ..!. Inn


11....,.;:oo

ll

(by L' H6pitaf's rule)

= o.
Thus, L

= I , anc.l il follows 1hm


R = lim

n fn

n-oo In 11

= 0.

The series therefore converget'.

We hnve developed six 1e<1s 10 determine whether series ot' numbers converge or diverge: 1he
n lhterm. imegral, comparison. limit comparison. limit ratio. and limit root tests.
The form of 1he n 1erm of a seric~ oFien suggests which 1cs1 should be used. Keep 1hc
following ideas in mind when choosing a ICSI:
1. The limit ra1io lest can be cffce1iveon factorials. produc1s oflhc form I 35 ... (2n- I),
and cons1an1S raised to powers involvi ng 11 (2". r" . etc.).
2. The limit root rest lhrives nn functions of
I 0.4-1).

11

raised 10 powers involving n (sec Exampl-e

3. TI1c limit compari son test is successful on rational funcL.ions o f n. ant.l fmctiunal J)QWCrs as

well (.jii,

:/n/(11 + 1}. e1c.1.

4. The imcgmiiCSI can becffeclive when the 11"' term is easily imcgnued. Logari1hms often
require the integraltesl.
By dehllicio n, a series L c, co11verges if and only ifits scquenccofpattial sums {S,. I co,werges.
The difficully wilh using this dcfinil ion to discuss convergence of a series is d"ll S, can seldom
be c.valuatcd in a simple fonn. and therefore consideration of lim11- ro S,. is impossible. The
tcslli above have the advantage of avoiding partial surns. On the other hand, they have one
d isadvaotage. ,1\.Jthough they may indicate that a series does i11deed have a sum. the tests io
no way s uggest the value of the sum. The problem of caJculating the sum often proves more
difHcuh than showing that. it e.x isLS in the fi rst place. In Section 10 .1 3 we discuss various ways
to calculate and approximate surns fo r known convergent series.

10. 12 Abs(l(ure 3nd Cvndi1i(lf1aJ Com'trgeoc:e,. Al lem.lrh~.g Series

EXERCISES 10.11

In Exercises 1-20 determine whclhcr the series converges ordiverges.

I.

:>0

"'I!"

L n

2.

~~~

3.

f:")
,.

4.

(n - I )(11 - 2)
n 2~

oa l

7.

f:Jii+T

n"t tf 2

L
0>

9.

6.

Lnl"
,._,

f,

(2n)!
(n!)'

s.

3- " +

('0

10.

z-"

I: 4 "+~.
I I ,_ )

:0

'
L 3"" '1"(11- I) !
1

2:
"'

12.

1 +1/n

e"

2-4 ... (2n) ( ' )


=I 35 ... (211+ I) n 2
~

24 ... (211)
41 .. . (Jn -r l)

"' cr

lilt

16.

L (n + I)<+'

(11 + I)"

15.

17.

19.

21. For ..~rhat integer values or a is ahc series

u- 1

- Jn + rr

14.

"'

n=l

II.

I: ,~!
,,~,

-1 -

5.

13.

677

.. =1

'-,:;. , . . + 3

2:

5. , ,

.,., n1 2"' - n

n=l

"0

18.

* 20.

n'+l

n"-t'

II"" I

-L2:

2" +n ~J"

4'

""' (?Lll._,n
)'
- -) '

= (311)!

~ (n !)'
L (lin)!

..,

L"
.,., 4

con\'-crgcnt?

110.12 Absolute and Conditional Convergence, Alternating Series


The convergence tests of Sections I 0. I 0 and 10.11 are applicable 10 nonnegative series. series
who.~e 1enns are all nonnegative. Series with infini1ely many positive and negative terms are more
complicated. II is fortunate. however. that all our tests are still useful in discussing convergence
of series with positive and negative tenus. What makes this possible is the following definition
and The<Jrcm I 0. I 8.

DEFINITION 1 0.7

A series ~ 1 c. is said 10 be absolutely con,ergenl if tloc scric; of ab:;olulc valu~

L:;~,

lc,.l converges.

At first glance it might seem that absolute convergence is a s1mnge concept indeed. Wh;u
possible good could it do to c.o nsidcr the series of absolute v-J.Iucs. which is quite ditlCrcm from
the original series? Tile fac.t is that when the secie' ofabsolule 'alues converge,;. it auwmatically
follows from the next tbeorem !hal the original series converges also. And since the series of
absolute ,alue,s has all nunnc:gati,,e tcrrns. we can usc the comparison. limit COlllJ)arison. limit

ratio. lim.it rooc. or integral test to consider irs convergence.


THE OREM 10. 18

If a series- is absolutely oonvergem. then i1 j(; convergem.

PROOF Le.t {S,. } be the sequence of panial sums of the absolutely convergent series L: c.
Deline sequences {Pn} and (N,. }, where P,. is the sum of all positive tenus in S,., and N., is
the sum of the absolute. values of all negative tenus inS, . Then

Sn = P"- Nn.

The sequence of panial sums for the .eries ufabsulute value..< L: lc.l is

and thi~ sequence must be omndccrcasing and bounded. Since c<teh of the SCIJUCnccs I P. \ and
IN., I is nondccrcasing '1nd o pan of I P,. + N.). it follows that each is bounded, and therefore
has ;o limit . say. p and N . JeS(leCii\'ely. As a result. sequence I P. - N. J =
has limit
P - N. and scrie$ L Cn comcrgcs to P - N.

rs.}

I EXAMPLE

10.46

Show that the following series are absulutel) cotwergem:

""

" ' (-1}"11


L.., (11 + I)2"

(a)

nl

(b)

1-

---+-+-+-+
" + -15'; - oo
22
3l
4'
5S
66 ------77
g8
9?
1010

SOLIJTlOK

L 11/ [(tO + 1)2"J. We use the limit comparison test

(a) 'l'lle series of absolute values is

to show thm it converges. Since


II

t =

(11

lim

+ 1)2"
I

11-0v

=I ,

2"
and L (1/2)" is convergent (a geometric series with r = 1/2), it follows that the
series of absolute values converges. The gi\'c:n series therefore converges absolutely.

(b) The series of absolute values is

R =

lim
II """TCV

the series

1/ n". Since

I ) t/

11"

1
lim - = 0,

IJ --..:0... II

1/ 11" converges by the limit root tesl. The given series therefore con-

verges absolutely.

In Example 10.45, absolute C'omergence of the gi,en series implies comergenee of the series,
but it is c.u stomary to omit such a statement. It is important to realize:. howc\'cr. that it is the
given series thnt is being analyzed~ and its corl\'Crgcncc is guaranteed by Theorem 10.18.
\Ve now ask whether series can converge without converging absolulely. If there are such
serie-s. and indcc:.d there arc. we must devise new corwcrgcncc tests. We describe these scric:s a.s

follows.

DEFINITI O N 1 0 . 8

A series that converges bul does noLconverge absolutely is sajd to ton verge condilionally.

Tile: rnost impontnt type of series with both posil.h't and llC&Itivc term-, i~ a1l a1ternaLing
A' 1.he nnmc \ugge.~n. an alttrnating S-~~tries IUs tcmn that arc altcmuc.ely posn,,c and
negative. For cumpk.
~~s.

1- ---- + -s - ...
2
J
.j

is an altcrn:uin scriez.. c.tllctl the altern ating harmonic sc:rh.-s.


Given all al 1 crnmin~ series 10 cxominc for convergence. we 1lrs1 test for ubsolute COIWCf'
gcnce a.\\ in Hxftmple 1(),45(a). Should this fa il .. we check ror cnndit ionul comcrgcnce with the
4

followil1CSI.

THE O REM 10. 1 9


An a hcmating !.Cric5

(A lt e rnating Series T ot)

L::;:,c., COOl\'Cl'CS if the sequence of o~utc \ alucs of the tcnns

{lc.11 is dccrcmna Jnd Ita' limit <xr<>.

If IS. Its the scq~~~:ncc of partial sums of the senh, then diiTcrcnccs ol' partial sums
= c,.. UC"c:ause.lhe c, ahen,.ue in sign. so aJs.o t!o the dtnerc~ S11 - S~~:-l
This means llut the partial'"""' IS. I form an O<ici llming qucnc-c. Sirl<'C abwlulc values
IS, - S. - rl = c. arc dccre"sing and approach zero. i1 follows by Theorem 10 8 of Section
10.8 that the sequence {S,} of partial sums has a limi t.

PROOI

are S,, - S,_.

I EXAMP L E

10.46
Determ ine whether the following $Cries converge absolutely. C(M1\ 'tf1tc conditio nall y. or diverge:

.,.. ( -1)"+1

(a)

"

..,

(b)

II

il

+ 51;

j,,l

L, ( -l)

II

(c)

lfl

SOLL'1101\'

E(

(,r) T he lllicmating Mmlonic series


-1)""' 1/ n is 1101 abwlutcly con""'J!cnt b:ause
the series of aboolute , . ,...
l / II di''"'"'' Si""" rhc -c<JUI:rltc absolute values
of the term.< 11/n I i< decreasing with limit zcro, thc scrie>
(-I )+'/n converges
wndition.oll) .
(b) For thi ahcnwting aeries. we firs t consider tho: ..:ti<' uf uhM>Iulc 'luc'

~
L,

or

-./rn'' -+,--,5-,.
tt

11)/2 .

n=l

\Vc:. use the limit t.-ompurison tc:M on this ~rics . Sin<.:e

,,,+ 511

11/1+ ~

Jill

lim

lim

' 2 II . tt 1/l
n J

-oo

= I.

,liz
and
l/ n'12 dherges, so does
../11 2+ 51!/tt 3 fl, The original series does not
1hcrcfore convc c a~lutcly. We now reson to the altcrnatjng series test. The
sequence I II + 5tt/
or absolute v-.llues ofthr: lernl'> of the >erie> i> decreasiM

L:

"'''J

if
)(11

1)2

(11

+ 5(11 +

+ I )3/2

I)

Jn + 511
2

<

,,,,

\Vhen we square and <.:ross nmltiply, the inequality bc<.:onlC-S


3 2
11 (11

+ 711 + 6)

+ 511)(11 + 1)3
= ~~~ + 8111 + 1811 3 + 16n2 + S11;
<

(11

that is,

n'

1211 3

1611 1

+ 5n

> 0,

which is obviously valid because '' 2: I. Since lim,._ .,. Jnl + 511/ n 312
0.
we conclude that the altcmating series
(-I)'' ( J ,,! + 5n / n312 ) converges con
c.Jitionally.
(c) If we apply the limit rntio test to the <cries or nb~lute values L 4" / (n 53"). we have

L:

L = lim

(11

+ 1) 53"+1

n-,.o.::~

411

lim

~ ( II+
- '-' - I )

.,_ ,., 3

u 5 3"

Since L > I, the serie:. L: 411 /(n' Y') diverges. The original alternating scri~:.'\ does
not therefore converge absolutely. Cut I.
4/3 implies that for large n, each tcnn
in the series ol' ahsolute ,,,,lues i~ approximately 4/3 times the term thin precedes it ,

unO therefore

c.c1nnot po:-;:-;i bl y exi,st, and the give11 .seric,s diverges by the 1r1hterm test .

'Ve h.a'e lloted .several tirnes lhC:"'I 1he e'seltii:ll q ue~tt iOl fM convergence c,f a lormegative series
is: Do the lerms approach 'l.cro quic kly e nough to guarantee con\'crgcncc o f 1he :-;erics? \Vhh a
series that h:ls infinilely many (X)\itive and neg-mive Lerms. lhis ques1ion is i1lnppr01>date. Such
a series may converge because Qf a paninl cancel ling crrcct~ for example. a negative term rnay
offset the effe<, of a large positive term. This kino of process may produce a convergent series
even though the series would be divergent if all terms were replaced by their absol ute values. A
sptcifi~ example is the alternating harmonic series which converge-< (conditionally) bcC!lUSC ol'
this cancelling effect. whereas the harmonic series itself. which has no cancellations, <liverges.
Absolute and condilionnl convergence are panicularly important when discussing e.ndpoints
of inlervdlsof convergcnceforpowcr scrie..~si n a:enncnf lhec ndr~oi nts of1en leads to an allemal.ing
series. We illustrate in the following examples .

I EXAMP L E

10.47

oo
Find the inlerv-al of COil\ergence for lhe power s;e,ries "

L
,,;;:0 (11 + 1)2"

x".

SOLt:TION Sittcc the rad ius ol' convergence is

R = lim

(11

+ 1)2"
I

n~ oo

( 11

+ 2)2"+ 1

2)

II +
= lim 2 ( 11~ 00

1t

+I

= 2.

the open intcr,al of oonvcrgcncc is - 2 < x < 2 . AI ..r


2. the scric~ i:. L:~() I / (n + I).
the harrno11ic series. which diYcrges. Al ).'
- 2. w obtuin lhc alternating hnrrnonic series
L:~o ( -I )" j(n + 1). which converges conditiOilally. 11-.c interval of convcr~cnce is therefore

-2 :,;;

< 2.

I EXAM PLE 1 0.48


.
Find the interval o f convergence for the power scnes

~ n(- 1)"
L- (ln + )3 (x
5

n= l

- 3) .

SOLuTION Since the rudius uf convergence is


11 (- 1)"

= -oo
li n1

(211 + 5)3
I)( -l)"+'

-,----"'C.,..,-7-~:::c;-1

(11

(211

= n-oo
lim
(11 +

1)(211

+ 5)~

I.

+ 7)'

the ope11 interval or convergcr~ce is 2 < x < 4. At .x

2. the ~crit:~ hecc mes

Sint:c:
II

e=

lim

u-oo

(211

+ 5)3
I

lim

811 3

-oo (211 + 5)l

= L.

= ( 1/8) I: 1/111 COil,erges ( /J = 2 series}. so also docs I: nj(2n + 5) 3 (by


the limit compc.tl'ison test). At x = 4, 1he power 1\erics becomes

and

I: 1/(8112)

""

n: l

11(- 1)"

c211

+ w

This is tHl ahcnlalitlg series that converges absolutely. as indicaled in the d iscussion of
The intcrVltl o f convc~cncc is t herefore 2

..-...

= 2.

::0 x ::0 4

In SecLions 10.10- 10. 12. we have obtai ned number of teslS for determining w hether serie.~
of numbers con"erge o r diverge. To test a series for co1wergence. we s uggest the following
procedure:

I. Try the , .. tenn teSt for divergence.


2. If

rCfr I has li mit zcrQ and the series is !lOll negative, ny lhe comparison) li mic 4;()111 1J(lri~on ,

ljmit r-dtjo, li mit rool. o integl'al test.

3. U {c,} has Limj1 zero aod 1he series conwins both posi1ive a nd neg:a1ive terms. 1est for
absolute c.onvergence using the tests in 2. If this fails and the .series is ahcmating, test for
condilional convergence with the alternating selies test

Each ofrhe comparison, limir comparison, liotir rario, lirnir r001. imegml, and aheruaring series
1es1s requires condir ions 10 be sarisfied for allrerms of I he series. Specifically, 1hc comparison,
limil comparison, and limi1 roo11eS1s require 1ha1 c. 2:. 0 for all n ; 1he limil ralio 1cs1 requires
c, > 0; !he integrallesr requires /(11) 10 be posirive. conlinuous. and decreasing; and !he
ahernaling series 1es1 requires llcnl\lo be docreasing and {c, Jio be aherna1ely posilive and
negative. None of rhese requiremeniS is essential for all11; in fac1, so long as !hey are satisfied
for all terms in the series beyond some point. say for 11 grea1er 1han or equal 1o some integer
N . the panicular 1es1 may be used on the series
c. The original series
c, then
converges if and only if
Cn converge .
Before lea,ing this section. we pro,e Theorem 10.2. We ''erify 10.24a using 1he limil ralio
1es1: vcrificalion of 10.24b is similar, usi ng 1be limi1 root 1es1. lflhe limi1 ra1io 1es1is applied 10
1he series of absolute values L la,x" 1.

L::::':,..

L:::,..

L:::,

(1+11
l.t n-Oo)
lim - .
a

11

Assuming 1ha11imi1 10.24a existS or is cqual lo iJllinily, !here arc 1hrcc possibi li li~:
(i) If lim, _"" la, {a ,J = 0, 1hen lim,_O>O la.+ 1/a, l = oo. Therefore, L =
and the power series diverges for all x # 0. In other words.
R

00,

= 0 = ..,_CO
lim ~~~ .
OnTI

(ii) If lim,._.., la, ja,., 11= oo. then lim,- 00 la,+JJa,l = 0. Therefore,
the power series converges absolulely for all x . Consequeml)',

L = 0. and

I I

a.- .
R = oc = lim ,_00 o,. ... ,

(i ii) lflimn-oola,ja,+ 11 = R.1hen J;m, -oela.+,Ja. = 1/ R. In 1hiscase. L =


l.rl/R. Since 1he power series converges absolulcly for f. < I and diverges for
L > 1 it follows 1ha1absolurc cOta, ergence occurs for JxI < R and dive,-gencc lo r
J,{I > R. This implies 1ha1 R is 1he radius of con,ergen<:e of the power series.
EXERCISES 10.12

In Excrdse:s 1-1 4 dclcnnine whether lhe series com-crges absolutely.


COO\'Cr&CS COIJ~i l iOMII y. or d i\ CI'gCS.

""' (- I)"_)_
t. "
.. _
L..,
n + l
o

2. L

"'
3. L
"I

.....,

(- 1)

* I I.

f: c-t (-"

13.

L: <- ~r- ~

11

II' + I

211

4.

11

S.

w +n+ l

9. .;;.., (-1)"+'
n=l

14.

( -I )

, /n 2 +J
n' +5

L" cos (nlr/10) Col .,


+ Su

11 1

.. ,
L: ;;"
-..
L

..L

In Exerei~~ 15-22 find the iruerval orcom-ergeat:'eofthe power !:"eries.

""

L-

12.

tll

00

CO> (lilT /2)

7. " ' (-I)" ,

"

11=Z

'

"' (- 1)" ~
L..,
,.,

3"

15.

. 17.
'-'

-""

n+l

( - "- )
It + J

10.

n <in ~" / 4)

110

...""

- I (.r - I)
112"

.. 1?. L: -n,-(2x)

f (-1)"+' ~

lf=l

II

21.

n -

+ I

:L -x~~
11=2

l.n ll

16.

IX.

20.

-L J;;<x
(11

+ 1) 1

"'

,., ,,

+ 2)"

L" ,fii':;:r'
I

-o
11

""

II "t*

\lr-1

I
22. Z::: - -(x-2)'
n 2 ln n
lf=l

10.13 Eucl 311<1 Appro<imote Values (Qt Sums orS<ries or Numbers

* 25.

23. Discus; convcrgenc:c of the series

683

Discuss convergence of the series

~ sin(nx) .
L.,

n'2

n =l

* 24.

L c.

converges absolutely. then


absolutely ror all integers p > I.
Prove that if

oo

E c~ converges

II"

2::: <-t) {11 + 1)+1'

110.13 Exact and Approximate Values for Sums of Series of Numbers


In Sections I 0.9-10.12 we have concentmted on whether series of numbers converge or di1erge.
But this is only half the problem. Tite comparison, limit comparison. limit ratio. limit root.
integral. and alternating series tesL~ may detemtine whether a series converses or diverges. but
they do not detemline the sum of the series in the ease of a convergent series. This part of the
problem, as suggested before, can sometimes be more complicated.
U the convergent series is a geometric series, no problem ex iSIS; we can use formula 10.391>
to find its sum. It may also happen thm the ,<h partial sum S. of rhc series can be Cltlcularcd in
a simple fom1. in which case the sum of the serie.< is lim,_ 00 S. Ca.<cs of rhe Iauer type are
very rare. By subsrirming 1alues of x into (lOWer series wirh known su ms. we obrain formulas
for sums of series of numbers. For insrancc. in Example 10.10. we ,crificd rharrhe Maclaurin
series for ex is
::01

t!r

L -x".
II =0

- oo < x < oo.

11!

By substiauting .t = l , we obtain a series that con\'erges to e,


ocl

e= n=G
l:= 1+ -I! + -2! + -3! + .. .
n!
Another illustrdtion is conrainetl in the following example.

I EXAMPLE

10.49

Use Example 10.22 to show thai the sum of !he allemaring harmonic series is In 2.
SOLliTION
lox =

According to Example 10.22. rhc Taylor series abour x = 1 for In x is

<:<>

u=l

( -1 )"-H

I
.,
(x - I)" = (x - I) - - (x - 1)-

II

+ - (x 3

I)' - .. ,

with open interval of convergence 0 < x < 2 . At x = 0, the series becomes the negatjve
of the harmonic :;c.ric:-, that clivcrgc.s. ami at .t = 2. il becomes the conditi onally convcrgc.nt,

:.lltemating harmonic series; that is,

In 2 =

L"" .{-1)"+'
:. _. :. . . _ = 1-1

II

-2 + -3 - -4 + .. .

Convergence of the Taylor series at x = 2 does nol, by it.self, imply convergence to In 2, a.s \Ve
are suggesting. his. however, true, and this is a direct application of the following theorem.

THEOR EM 1 0 . 20

ff the Taylor series L~0 a, (x - c)'1 of ct function f(x) converges at the e lld J>Oint
x = c + R of its intCI'\101of convergence. and iff(.\) is continuous at x = c + R. lhcn
the Tayk1r series; evaluated at c + R oorwerges to f(c + R) . 'n'e same resuh is valid at

lhe ocher endpoint.,

x = ,. - R,

This cxomplc su ~ec.us another possi bi lity for sum ming convergent series of numbers. Find

pc)\\~r series w ith k1lO\VI\

s um that reduces to the given series of numbers upon s ubstitutio n of

a ' "alueof x . \Vc illu.'irnuc thi"- in rhe 11cxt two examples.

I EXAMPLE

10 . 50
Find

""

L -1"" .
n= l -

SOLUTION

Th isseriesrcsullsifwesct x = 1/3i n thepowcrscrics


00

L nx".

S(x) =

~~~

The radius of convergence of this series is

1-"-1

= ,_oo
lim
+I

= I.

11

If \ Ve divide bmh s ide." by .r.


00

-S(x)
X

= L ilA' "-~
~~~ ~

and integr-.>te according to Theo rem I 0.3.

j ;sex)

eo

dx

=
n=l

This is a geometric se ries with sum

~S(x) dx =
X

l- x

lxl

< I.

Differentiation now gives


I

( I - x)( l) - x(- 1)

- S (x) = ----;'-'--:;,.--'- = -:-(:-,- x"")2


x
( I - x)2
Thus,
00

S(x}

="
L

11

x"

= .,.---....,..,.
( l -x) 2.

11= 1

When we set x = 1/ 3,
co
"

Tl

3"

11=1

.,.,..--/,..3.,.,.,...2
(I - 1/3)

IO.l:l E><.lt:l and App!u...:in'utc. Value> fur Sums ofScrie)Qf Nw.nbers

I EXAMPLE

6$$

10.51

Fin<l1he sum of 1he series

L"" (211( -+1)"1)2" .

n=O

SO! VriO'\ There are many power series Ihal reduce 10 lhi< series upon subslinuion of a
specific value of x . For inslllllCC, substiruli()n ()( - I /2 . I, and I / J'i in I() 1hc foll<>wing power
series. respectively, lead 10 1he given series:
oc

L 2n +
,
1

=0

( - 1)"

oc

x"

I.

"L

'

n-<l

(211

1)2" x" '

\Vhich should we consider? Although it is not ~he simplc.s t, the third series looks most promising:

x correspond.> to Oe coellicient in the denominator ' "~gcsL~ that we


can find lhe sum ()f tbis series. We 1heref<>re se1

1he fac~lhalthe power on

S(x) =

./2(-l)" x""+' .

L
211 +I
nc(l

To find the radius or convergence. of I his series, we set y = .r2 , in which case

,.=o

./2( - I)" .Tbt+l


211 + I

- 1)"
=Xu=()
L"" .J2C
y".
2n +
J

l11e radius of convergence of the y-sel'ies is


.J2c- I) "

= n-limoc

Rr

211

1)"+1

J2C211

= lim

,,_ oo

211 + 3)
( 2n + I

I.

+3

llle rndius of CQn,erge.lce for the power series in x is therefore. R, = I also. lf we differentiate
the series with re.<pccl tO x.
00

S'(x)

00

= L .J2(- I)"x2n = L -J2(-x 2)" = l ".:0

rr~o

./2
(-x 2)

J\ntidifferemimion now sives

S(x) =

Since S(O)

J2

I + x2

d .x = J2 Tan- x +C.

= 0 , it follows that C = 0, and

J2< - J)"x2"+'

;O 2JI

= J2Tan- x.

lfwcnowsc1.x = J/J2.
00 J2(- l )" (
2n + I
Jt;O

~2):1n+ l =

v~

Con<equcntly.
""
( - 1)"
( I )
~ (2n + I )2" = hTan - -./2 .

./2

= l +.x2 '

I EXAMPL E

10.62

:;::
tJ.o;c the tim 20 1crm~ of tht: ~ icg

r:;:-_1 ( -

\ )"+ 1/ n~ tQ c:~t\nm\c \\~ $\.\n, , ()\)\.;.\ \\\ \\\\ cnQr

c.~ l fnuue.

SOLUTION Th<

"""or 1hc firM 20

1crm' n f 1ho series is (\.901 11%5 . Si11Coe llle scr\cs

altcrmuin~. uncl ubsoh.uc: valucJ

i->

uf terms urc d~cre :uing wilh Hm\l :ti!l\l. lht: ~W.\!\i.mut\\ \'Kl!l\~\.\')\\!
l.lUUr ;"this estimm~ i.o,; rhe 2 1,. term, 1/21.1 < O.()(X) \OS. n)u!\.

-:. 0.)()1 593.

In practicaJ situations. we often have to decide how many temls of a seri.es to take in order to
guarantee a certain degree of accuracy. Once again this is ea.-;y for alternating series whose
1erms satisfy the conditions of the alternating series tesl.

I EXAMP L E

1 0 .53

How mtmy tem1s in the .series L~z -l)a+ J/(n 3 Inn) ensure n truncalion error o f less thal

lo-s?

SOLUTION llccau:;e ab:;olute \ a lues of terms arc decreasing and have limit :tero. the maximum errur in trunc.uing this alternating series ,.,,hen 11 = N is
(-l)'V-t2

+ 1)3 In (N + I ) .
The absolute v;due or this error is less than Jo- 5 when
(N

(N

+ 1)3 1n (N +

< 10-5
I)

()r

A calculator q uickly reveals that the smalle$l in teger for which this is va lid is N = 30 . Thu:s..
the tnu'K."":.tted series has the required accuracy after the 29th term (the tirst term corresponds to
II

= 2. 1\0t II =

I ).

In Section I 0.7 we illu>tratcd the use of Taylor's <>.:mai ndcr fonm<la to estim>uc the error when
definite integrals a re approximated using Tuylur series. Ahenutling series sometimes provide
an easier nhe-mative. \Ve redo the example of Section I0.7 to demonstrate.

I EXAMPLE

10.54

llpproximate

112

sinx

--d.x to live decimal places using the Maclaurin series for sin x .
X

SOl Ul iO 'i

lfl

Using Lhe Maclauri n series for sin x , we obtain

sin .T
--clx =
X

l {1

L
0

1(

xJ

x - 3!

xS
)
+

5!

dx

I
(I / 2)
( l/2)s
(x--==--+~}'12
=
+
- ---.
2
3 . 3!
5. 5!
3 - 3!
5-5!
3

(I

This is a convergent ahemating series. To find.-. fl vc deci mnl>lPi)roximm ion, we calculate pnnial
sums umillwo :,ucccssi\C sums agree to tive dcci m<:ll:>:

s,

= -2'

52=
S3

( I/ :2)J

s, - 3.3!

= 0.493056,

( 1/ l )s

= S2 + - = 0A93 108,
5. 5 !
( 1/ 2) 7
7 . 7!

S4 = S> - - - = 0.493108.
Consequently, to five decimals the value of the integml is 0.493 II .

688

O>:tpter 10 Infinite Scqnentes and Series

Truncating a Series Whose Convergence Was


Established with the Integral Test
Suppose now that a series
the integral

E:t c. has been shown 10 converge with the integrai !Cst: that is,

converges where f (11) = c... If the series is truncmcd after the N '11 tcmt, the error cII+ 1
c11+ 2 is shown as the sum of the areas of ahe rectangles in Figure 10.30. Clearly. the sum
of these areas is less than the area under y = f (x) 10 the right of x = N. In Olher words, the
error in truncating the series with the N~Ja term must be less thau

[XJ f{x ) dx .
GURE 10.30

( 10..181

Approximariog the sum of a series usi,,g the inregral test

~(x)

........_
/ 12)

/(3)
2

'.v=Jill' + I )
r .Vl

\
3

r,,.,1=fiN+ 3)

I
N+ I

= j(N >2)

I
N+2

I
N+j

I EXAMPLE 1 0 .55
oo

Obtain an error eStimruc if the series

11(ln ll)
2

4 is 1ronc.arcd when ,, = tOO.

SOl l'TION The error cannol be larger Ihan

r"" .r(lnx)'
___:1___,. d x

---=---,..
< 0.0035.
3(1n I00)'

}, 00

\Vhcn convergence of n series is established b)' the comparison, limit comparison. limit ratio. or
limit root les:ts. we often estimate the tmnca1ion error L:~N+ I c,, by comparing it 10 something
that is summablc. We illustrate this in the following two examples.

I EXAM PLE

10. 5 6

ln the fir:sl parag.r(tph uf this section we indit:atecl that e is the sum of lite ~rie.)
oo

L n(

n=O

Use the t\rst 10 terms to find an approximation fore.

E<~~::K:l w.d .'\pJ'rv~irnJK<:: Vu lue~ fv Sums vrSc.ti~ ofKumbcrs

IO.J 3

619

SOLUn ON ll1C sum of the first I0 terms is


I

'\' - = 2.7 18281526.


f...J H!

m::;Q

The truncation error in using this as an approximation fore is

-+
-I I! ++ -13! +"'
10 1
12!
= _1
10!

(I + ~II + _J J .J2
1_
+ _ _1
__ + .. )
11 12 13

< _I ( I
LO!

+~+

_I_
11 2

II

(n geome1ric series)

II'

(using equation J0.39b)

LO' I - 1/ 11
II

+ -~-d ..)

< 0.000 000 304.

10 . 10!

\Ve may write, therefore, that

2.7 1828 1526

<

"" ~
I:

< 2.71828 1830.

n:O ll!

and to six decimal pla~cs, 1: = 2.718 282.

I EXAMPLE

10.57

How many te.rms in the con\'ergent serie..~ E ;! 1 n / (u + 1)3"1 ensure a truncation t rror of leS!>
than 10- 5 ?

SOLDnON

If this series is tnmcated after the N "' teml. the error is

""

'\'
L.,

n=N+I

(11

" I )3"

N + L

= (N + 2)3V+t

N +2
(N

+ 3)3N+2 + ...
(a geomc.tric series)

(using equation 10.39b)

= 2. 3"' '
Consequently. the error is guar.weed to be Jess than 10- 5 if N satisfies the inequality
_ I_

< 10-s

2 JN

or

In (105/2)
N > --'ln-3'--'-

Thus, I0 or more terms yield the required accuracy.

= 9.85.

690

Cbapttr 10

lnOoite Sequences atl\1 S~des

EXERCISES 10.13

ln Exercises 1-LO va'ify that the swn of the scrks is as indicated.

il *
>

""

::1

19.

L: ~~~,

1:

(5 tcrnlS)

Ln~'J

20.

fi *

n~l

(- 1)"

2. '""' --'--'L.., (2n + l )!

sin 1

::1

il + 21.

"""

L"' ~sin(::)
2"

JJ = I

iii.

-X~

( 15 tom" )

ll

211- I

22.

(20 ten"')

L:J"+''
(- 1)"
24.
L: - 11-

* 6.

2"'

-:s

L 3" +II

2J.

!.

(20 Jcrms)

"=2

( 100 Jcrms)

11 1

"" (- IJ"+'z"'+3

L:
2

(211)!

"'

2'

~= 1

n3''

.. 7. L: -

* 9.

"' ( -1)"

L:

+I

211

eo

i.

~t=2

5.

(tO terms)

II~ I

"'

= -8( 1 + cos2)

.. 8.

= ln3

In Exercises 25-27 llo\V many 1enns in the series guaranlee an approx-

iJnation to Lhc. sum wilh a ltu!lcalion cnor of lcs.~ Lhan I 0- 4?

L"" -112

1
-11 =ln2

26.

u=l

(-1) "

L 3'"(2tt + I )!

3sin

G) -

n J

10.

""
L:~=
2
2'
I') ~

"" e-" sin1 n is ll'lllle"dted after the Jot. terlll.


* 28. Suppose Llle SCI'ies L

n :J

* 11. Find Ute Maclaurin series ror Tan - ' x and use it to evaluate

Obtain an error estimate by (a) using I0.48 and (b) using_the fact tJHU
e-" sin2 n < e- ". Which gives lhc bcuc::rcstitntHc::?

(- 1)"

{)0

L: 211+l .

12-. Find the Maclaurin series ror x / (L

In Excrci.scs 29- 36 C\<uluatc thci ucgral comet 1(1 three dccimaJ l'lucc.s.
Compare the \vork in Exercises 29, 30, 32. ond 34 to thoJ in Exercises

+ ..-:1 ) 1 and use it to C\'aluat..e

11- 14 o f Section 10.7.


1

i * 29.
il

31.

ln Exercises 13-14 appl'oximalc the sum of the scics if it i.s ttun<:atcd


il 33.
after the Nit! tcnn. u~c 10.48 Ill find au error c;o;timal.C.

oo

13. '~;""' ( .

"2

-X~ n
)4 . N= 10 il l4. '""' - , .

i..J II~ + 1

,.-l

11-1

(!

il.

N=:i

il.
In E:xc.rciscs 15- l6 use 10.48 10 .::stimatc Lhe error when lhc. series is
uun::atcd atkr the Nih lcrm.

lS.

l
L"" -,--,
+J
1'1=1

N = tOO

16.

L., -; sin (.!.)


Jl=l II

il

I * 32.

+ x~

1.
IJ

11

sinxdx

- I

e-'' dx

f" -x -1-tn(l -.t)dx


~.~

1.
J'

cos (x 2 )tf.r

0.3

! * 34.

v'i""+?dx

1/2

30.

1/ !

1/2

.t 6 - 3.\'3 - 4

dx

37. In deu::nnlning l ht.: radialt;t! JX.lWCr from 01 half WU\'C iU1lCJ1J1a, it ht

N = 20

!.

'"

I - cosO

II

<10 .

Find u two-dccinlldplac-e upp_ro.ximation foe:- Lhi.s intcgrut

ii*

33. A

't'cry impon ant runclion in engi nee1i ng and phy~ics is 1he error

function crf(x) defined by


Cl"f (x) =

( - 1)"

18. '~;""' - - (20 terms)


L- n4
~t=l

tl

proxhnlltion lO the sum of the SCF"ies. In ct~ch case. obt.ajn lin t::-.Siim:uc
Of the ln.tnCfJi iOn Cm)r.

(3 terms)

36.

1.
1 .t" + I
1.

i :t

necessary to evalua1e

In Exercises 17-24 usc lhe number of tenns incJicated to find an ap-

i "'

35.

sin.\'

1
--'X
X
'1' -I- dx

CaJctdatc crf (I)

r e-' dt

2
., fit Jo

t'OIT<:ti 10 thrcc

decimal spaccs.

'" 39. This cxcrci$C shows that we must be


<ltX"W<H.:y of u rcs11JL C01l llidn the :series

S = 3. I 25 100 I - O.OOOO!X) 18 (1

\'Cf)'

Cllr('ful in prcdict..ing1hc

(c) Tr f., :;: S. - S arc the d ilfercnccs lxtwc:cn lhc sum oft he
series and its finu (our partia l sun~. show that

E. O.<XXl 1002.

+ ...!_ + - 1- + - 1- + .. ).
10

JO'

10'

F., = 0 .00001002.

(u) Show llotlhcsum ofthisseries is cxJCll)' S = 3. 124 9999.


l 't> l\\'0 dccim;,sl p l :t~:Cll. Chen. the valu~.: <If S i .. 3.12.
(b) V~rify thai the fir~llbttt J)ltrtiol ,;ums ofthe scri.:s tltX:

s,

=3. 12.; 1001.

s'!

= 3.tzsooo92.

E,

0.000001 002.

E,

0 .000000 I(X12.

What \:all you Wy about the. ~Cur"J.c:y or $ 1. S1. SJ,


nnd S, us lii)JJI'OXi llltdiun~ 10 S'!

(d)

s.

Jr sis npproxin\<Ued by nny o f s. s~,. SJ. or to two


tb:inl:"tlt.. the res ult is 3.13. not 3. 12 us in float'l (a). Th\1$, in
spi te of the 1C(Ur~y l>fedictcd i n J)W"I (c ).
SJ. and
S.1 du not predict. S C(HTCC-tly 10 two dccimJ.I pluccs. Oo
th -ey f)l"tdic. S COirtCd y tO three 01' rour docitnul places!

s,. s2.

s, = 3.125000902,
s., = 3.1250000002.
SUMMARY

All infir1i1e sccw etx::c of numbers is the ssigruncnt of numbers to posili\'e integc:rs. l n most
ar>J'Jic:uions of :-cqucncc~. lhc prime considcnuion is whether th~.: scquc1tt."C ha." a lillliC. If the

se<tucnce has: its te-rms defined explicitly. then our ability to mkc limits or continuous runctions
(l imil$ ao infinioy in Cha1>1er 2 ;md L'H6pilnl's rule i n Chnp<er 4)

C>U> be: very hel pful. IF ohe


sequence is defined recursively. existence or the limit can so.nc1imes be established by shO\\'ing
thm the sequence is monoconic and bounc.ledl, or that it is oscillatOI)' und cowctgeut.
Arl cxptcs:,ion of the fonn

""
L:c,

= c1 +

c1+ + c,. +

n l

is called an inlirlite series. \ Vcdcfinc the sum o f this series as the li iJlit of its sequence of partial
sums { 5~, J. provided t hat the sequence has ki Iimit. Unronunatcl)'. for m ost series we cannot tinc.J
a simple form ula for 5'11 and therefore analy.sis of the limit of the stq_ucn<.."<:" [S1, ) is impossible.
To remedy this. we. developed various convergence tests that avoidc.d the sequence { S., ~ : n 1b
ccrm. cumpariSOil. limit comparison, limit rcn io, limil root. integral, and altcn1a1ing series tests.

Note the sequences 1hat are a-ssociated with a series

LC

11 :

{S,I

sequence of pantia 1sums for the dcfinition o f a sum;

{c.!

se<}uence o f tenllS fol' the nhternt test;

{c.,/ b. I

sequence for the limit comparison tc~t:

sequence for the lim it mtio re..i\t;

sequ..:oce for lhe limit root tchl;


sequence for the altcmaoing series oes1.
Depending on 1hc limils of these sequen ces- if ohcy ex is t - we may be <lble 10 infer
sorncthing about convergence of the series.
Jnli nite sequences and series or func tions are importalll in applications - in particular,
power serie~. ( r\s ~cienLi!lol..<i, you will ~ee other lype..,. of series: f:ourier series, for example.) We
considered situations where a power series was given a od the s um was 1.0 be determined. We
11
saw that every power series
Cl11 (x - c) has a radius of convergence R a nd a n associated
inte rval o r c.o nvergence. If R = 0, Lhe inte rval of convergence consists of only o ne point
x = c; if R = oo, the power series converges fo r all x; and if 0 < R < oo, the interval of

eotl\Cil,>ence om"'t be one ul' fouo possibilities: c- R < x < c + R, r - R ::; .r < c + R.
c - R < x !: c .,. R. or c - R !: x !: c + R. The 11ldius of convergence is given by
lim.,_< !a,{a,+ tl or lim.,_""'"' - ''" provided that the limits exist or arc equal to infiotity.
If 111 each point in the intcl'\>tl of <'lll\'ergcuce of the power series the \1llue ot' a function j'(.r)
is the same liS ohc sun\ of the series, we write j'(,<) = L;a,.(x- c)" and call /(.r) the .um
of the series.
We also considered situatious \\here a fuuction f (x) '"" I u poilll c are given, and a.~k
whether .f(.r) has a power series cxp;111sion about c. We saw ohat there can be at most one
power ~coics cxpno1sion of f(x) ab<lut c with 11 positive mdius of convergence. and this series
must be its Toylor series. One way to verify that J (x) docs inCIL'Cd have a Taylor series about
c and that this series converges to f(x) is 10 show that the sequence of 'fa)lors remainders
(R,(c, x)l ex ists >Uld has limit zero. Of!en. however. it is much easier to find Taylor series by
adding. rnu hiplying. differentiating_, and integratillg known series.

\Vhen a Taylor series is cmncmed~ Taylor's remainder R,.(c . ..t) represents the truncation
error and, in spite of the fact that Rn is cxprc.o;sed in tenm: of some unknown point Z11 it is
often possible to calcula1~ a max imum "aluc for the error. Sometimes R, (c, x) can be avoided

ahogether. For in:)tance. if the Taylor seties is an ahemating series. then the maximum possible
truncat iOt1 error is the value of the next tern1.
Power series are oflen used in s ituatiOn$ that requi re approximations. T~ly lor series provide
polynomi al approximations to colllplicmed funct ions. and they offer an ahc.nuuhc to the numcri<:.altechniques of Section 8.8 in the evaluation or definite integrals. Power serie.s nrc also

useful in situations that do not require approx iona1ions. They arc sometimes helpful in evaluating
limiiS, and they are the onJ) way to solve ma1ly differential equations.

KEY TERM S

In rel'ieo.ving !his ~hapoer, you should be able 10 define or discuss the following k~y 1erms:
Sequence

Series

Infinite ~uence of numbers

Term

Explicit sequence-~
Limjl of' a sequence

Divergent sequence

Recursive sequence

Con\'ergent sequeolCe
Method or successi\'C approximations
or JixctJ.. point iteration

Taylor Remainder Formula


Taylor J>Olynomials

Taylor remainders

Tt\ylor series
Power series
Common ratio
Open intcrv~l.l of COIWCrgcrwc
Sunt."l\ or power series
Nondccrcnsing sequence
Nonincreasing sequence

Maclaurin series
Coomct ric series
Radius of convergence
Binomictl expansion

Upper bound
Bounded sequence
Succcs:;ive approximutiOtlS
Sequence of partial sums

Lower bound
Osci llming sequence

Divergent scric:s
,,u. tenn test
l.n tegr~l

tesl

Compa.Jison test

Limit ralio test


Absolutely convergent series
Alternating series
Alternating series test

lmcrval of cOflvcrgcnce

lncrcasi ng scquellC..:

Decreasing sequence
Monotonic sequence

Inti nice series of numbers

Con\'ergent series
Hurmonic series
NooJ\cgativc series
p ..se1i es
Limit comparison test

Limit root. test


ConditiooaUy convergent series

Alternating hannonic series


T runcation tJTOr

l~cviev.

I:.U:cd<eS

693

REVIEW
EXERCISES

lrl Exet'C.ises 1-6 discuss.. with ,d) ncccs.sury prtlOfs. whether the.! st
queoce is monotonic omd h~ls ;m \lppcr bound. a lower lx1und, :t.nd :1.
lim h.

+ I. { ' - 5n
n2 + .Sn
~

l.

3.
.. 4.

ra-o

3}

+
+4

2 1.

~ I

23.

(l fn)}

= 1.

5. Co= 6.
6. Co= 6 .

2:
"" 246 .. .

<" +

= 15 + Jcn- 2,

c. . -rt

2
= 6 + -.

25.

, ~

27.

11 2

29.

s)'

L ( - 1),, Inn
-

28.

..

11

= 2. :'ccurul.t 10 S t.loc.:irnal place&.

3 1.

C()O\'Crgc'/

33.

2,\1+2 1t

30.

Y'

/1+ c;,

32

11 2

L"" (n + 1) 3.<"

n 2:.

I.

10. Usc lhcdcrivolivcof lhc rwoclion f(x) = (lnx)/.t to Jli"OVClhal


tloc sequence (In 11 / II) is decocasing ror 11 ::: 3.

,.,
0)

13.

15.

f: (ln r~

14.

d 6.

..... ~

37.

16. t

..

Jn

(- 1)

tJl + 511 + 3
, .. _2,+5

,._,

-n3

18.

L
,_, J;eo> (M )
"

"'
I:
'
,,.,
u:

2~,

,z + 3
n:tn

(- 1)"

It"

00 ~
L:
'' - <-~ + 3)"
n -1

~r-2

L"' n3x.t.

38-

?II

L
II:

:...xJn
II

39. /(.r) ~

JT+?.

40. /(.r) =

r-.

aboul x

obooo .r

=0

=0

41. /(.<) =COb (.< + :r/ 4) about x = 0

(".~I)

~,

t,cos-(D

.x"

In E'(etdscs 39-47 find the power series cxpan&on of the fllnc.tioo

t
,..
f

abottllh-e indicaled poinl.

f: ("+ I)
n=l

12.

5 211

L-;;r
,_,
11 1

17.

:1/l + 2
+ 4rl

L
... I
L: ,..
...

u= l

"" -I <x - 2)"


* .35. 'L:

In Exercises 11 - 30 dctcrm.incwhcthcrthescricsconvcrge:sordivergcs.
In the case of a c.onvc.:rgcnt :)CriC:.l> Ihi! I h::~s both positive :tnd ncgati,c
tenus, i odice~ l c whether il .:c.uwc.,;es ldbbOI\Hc:ly or c.:omlitionally.
-

10'
s J+ t

n:l

n3

I + ;;~ r

34. :L ..!.._, ..

"""

9. Find an explicit definition fo r the sequence

n- O

L"" ---.
n+ 1 <,
+I
110

- c'
"+'"'

,....

f:

3 6 9 .. . (3n )
(211) !

L~ (- I )"~'
"'

C)

' ''

lR E.'(etdsc~ 3 1-38 find the interval of 001wcrgcncc for the J)O".''c r $c;

8. For what '':.llucs uf k doc~ tltc sequence

II.

f:
=

11-1

.r +(. - ( .x+ 4

c~~- =

24.

+5

26.

L:

L -;n-I Cos
,_,

I
L"' 1'3inu

00

I 7. Use Newton 'g iccnllivc procedure ond the met hod ohucc~"Si"c approxirn:ttions lO uppro.ximutc the root of the eqw.uion

lx:1wccn .\' = I urtd .r

22.

"" "l + I

L:

11: 1

n :: l

c +I -- -4ti
-'
5+
1

(ln)

, .... 11

~~ ~

c,.

00

20.

IJ!

11= 1

Il l + I

c,

f:~" eos-f)
,,.,
n
n: l

c. _. =(1 /2)./c! + 1.

c , = I.

19.

42. j(.r) = .r In (2x

+ 1)

43. j(x) = >in x. about X


44. .((x) = x/(x'

45. .((x) =

e' .

:about x

= " {4

+ 4x + 3) .

about x

=0

=3

about x

=0

694

* 46.
* 47.

Chapter 10

l nRnile Sequen.:e> and ~ries

f(x)=(x+l)ln(x + l).
/(x) =

aboulx=O

* 50.

Find the Maclaurin series for f(x) =

J 1+

sinx valid for

-rr/ 2 !: x !: rr/ 2 by firs! showing that /(x) can be written in

x e'' . aboUI x = 0

the fom1

f(x) = sin (x / 2)

* 48.

How many 1cnns in lhc Maclaurin series for J(x) = e-' guar
antce a lruncation error of less Ihan I0 5 for all x in the interval

Why is the restriction - rr / 2 !: x !: rr / 2 necessary?

0 !: X !: 2?

* 49.

Find a power series solu1ion in powers of x for the differential


equation

y"- 4y = 0.

+co; (x / 2).

51. On a calculator lake the cosine of I (l"'ddiun). Take the cosine of


this result, and take it again. and again, and again, .... What happens?
lnterprel what is going on.

CHAPTER

11

Vectors and Three-Dimensional


Analytic Geometry

Application Preview

The figure below shows a boom 0 A carrying a mass M. The boom is supported by cables A B
and AC .

Til F. PRORI.F.'-1
If lensions in Ihe cables mus1no1 exceed 20000 N, wha1 is 1he maximum
mass that can be supported by the boom? (Sec Example 11 .10 on page 719 for the solulion.)

Chapcers 1-IOdealt with single-variable calculus -difl'erem iation and imegration of functions .f(x) of one v"riable. In Chap1ers 11-14 we study multi\'llriable calculus. Di<eu<.<ions
of lluce-dimcnsionol anolylic geometry and \"CCiors in Sections 11.1- 11.5 prepare the wuy. In
Sections 11.9-11. 13 we diffcrontialc and integrate vector functions. and apply the rosuhs to 1hc
geometry of cuf\eS in space and the mlltion of objecls.

111.1 Rectangular Coordinates in Space


1llc coordinate of a point on the real line is its directed distance from the-origin. Cartc.sian
coordinates of a poinl in a plane are i1s direc1ed distances from 1he coordina1c axes. In space,
Canc:.."ian coordinatcsarc directed distances rrom three fixed planes called the coordinatt' planes.
In panicular, we draw 1hrough a poim 0 , called 1he ori~in , three mutually perpendicular lines
called the x-. y-. and "axes (Figure II. I). Each of the axes is coordina1izcd with some uni1
distance (which need not be the same for all three axes). These three coordinate axes determinethe three coordinate platk:.s: The: .t y-coordinare plane:: is that planca:mtaining the .x- and y-a.'\e.~.
the yz-coordinate plane contains they- and ~-axes, and the x::-coordina1e plane contains the

x- and z -axes.

If P is any point in space. we drw lines from P perpendicular to the three coordinate
planes (Figure 11.2). n 1e direcled dis1ance from Lhe yz-coordinale plane LO p is parallel LO
the x -axi.s, and i.s called the x -coordjnate of P. Similarly, y - and z -coordjnates arc defined as
direc1ed dis1ances from the xz- and xy-coord ina1e planes 10 P. These three coordina1es of P ,

695

mttl.LLY'mtW ""'~ '"' ,......,....

.,;._.,. tat

<6ol.o ~,.,. I }<'' " , .,._ .,~..,..,

HI! .til I MIWM

J\

P<'"" i

" P""<'

N-u

M i!.iii

C.......,o,!i- 1<1 > .;.( ( -

HIM M

>!

rlo(k; po h"

(2. ~ ,4)

<I

o
(:l, ~. - I)

wrlltCn (,t:, J' :). Me Cillle(l dtc Curltsf.an C)r n!C:hlnJ:ul:r cool"dinat~ll ()I' P . N<>l e 11\tlt ir we
dr.1w finc;:s l hiUII~~h I' ah:n e J)O:I'"J.lic:lklicll:..r 1u d~ :o:c!ll, t lu~n ch ..-ll.ie'<.::lo:-.1 J h.tan..:<S fhml 0 to
fX.ll/11.) Q/'# ll(('f'l'>CCdOI1 Q( lh<:...C pt: rpcndk:ll lll ri!< Wilh dt ;;t iC~ tlt"C Ubt""l lhe CMICll iflll en;:\n_Unf, lCII ~\f
p (FiJ;_Ut'C J J , ,l),

By L'ilhcrdclinilion. t"ach poi.nc in S!ni(.' C h111.s 11 uniq1.1c o rdc1cd se1 ofCane&i<m <:-o ordin<llC-5
( ,,._, )'. ,::) ; COJlvcnrefy, every o rd ered (riJde t li' "CJI n umber:-. (.t',
i ~ 1he M!:l nf ..:oMd imueJ. l'\ ' lt
o ne: :md <m fy vue ptNn1 m SJXK.~. F"or es;tnlj)le, PQilltS willl <:()()rdill;Ues (1, 1, I) . (2, -3. 4),

>, .:)

0 . d,

- l ),mMI ( -2, 5. 3) 01rc $h-vwn i n F igUI'C I I A.


T he ~oordiu.ulc .!>ySicrn."' in F igure.\ I I, I - 11.4 di'CCtLIIOIJ righlhiu.td~(l coord.in~ l!>)'St~ms.
lx.xu use if we curJ lite l ingers on our righ t hund from the positive .f din.."Cti ~lll toward ~he IXJ'$ilhc

) "(lirc:clivn, lhcn the d iUntb JJViiU.'< in t he ~,fS it.i \C ,z...Ui fcct ior'l (Fig.uce 11.5). The c.:.ordinate
" Y'Ienl in Mgure 11.6, tl n l hc (lfhc r l'ulnd , iii n ldl handc:d eoordl"nlllt llYilt.nt., ~i n<:e t he lhumh
of the tell h.nnd' poinl'> in the po.silive zdircclion w hen the fingcr!i of t his hand om: curled frum
rite fJO...itivc x-<.lin.x:tiou 10 the positive y-<.litctt ion. \Vc ~I WH)S u ~>c right-handed sys.te1n" in this
t)()(\k. H~ s hc)uld cvcryc)J)e.
SutJI}(l.-.c we con::.tr ucr for nuy tw(l f.K>int!) P 1 u1id Pl wilh t-"OOl'd ioutej (.r 1 y 1. :: 1) tuul
(.r:z, )'2, : 2), r-cspc~ llvc l.)', ~ bo.t w ilh.sid c.) p:r:eltel W ille coordi n ~uc plun ~:-.. :uu.l w ith Uuc ,_,cg.men
f j P1 ns d itgoml (f-'igure I I. 7). Ucccmsc C1i anglc.s P 1A 8 and P 18 P:. ure righl ~ang.led , we c;m

write

NP,I'zN' = IJI',BN' + 1181', 11'


- IJP,tln '

+ !IIIJII 2 + ll lii'JI 2

IEI!l'.T
tli,,IC toy-.rc.n

I!JJ:
IWO l)('!inh in 1\f'A'C

y
X

.\'

In other word< the length of the l ine segment joining, two llOinls P,(x ., .1'1- : ,) and
P;(.r;. }'2 :1) i>
!I ' )

This i~ the 3naloguc ot'rormula 1.10 for the length of n line se~tment joiniug t"o points in the
xy plane.
JU>I as the X and y-axo d ivide the xy-planc into four rcgioos clled qut>dmnts. the .ty.
yz -. and x: -coord inatc planes di,itlc xy:-space imo eight region called <>ei:UtK The region
where x . y., lllld ,Zw\:Oordinsucs nrc oil positive is called 1hc fir"lt OCIIHlt. There is no commonly
accepted w.~ty to numbel' the remaining .seven ocran1.s.

EXERC I S ES 11 .1

I. ~ a Cil<l<sl.. coordinate system lJld show the points (I. 2. 1),


( - 1. 3.2). (1. 2. ). 13.~.-5). (- 1. - 2.-3). 1-2.-5.4),
(8. - 3. -6).ond

(-~.

IS. 1r P and Q the fi~ure h<l"" II.IYeCOMlmotcs (x1 y 1 ltl ond


(xl. y 1. z1). ~how that coordir.aes o(ll\c poi ru N midwy bct"-'\Yil P
and Q are

l. -5) .

.!. Filld lhC lcnJih orahc lioc: scgtnctltjoinittg the poi.n~ ( I. - 2. S)


Oll<l (-J. 2. 4).

J. Prove thtll the triangle wuh '"ni<cs 12. 0. 4../2). (3.


""'' (4, - ~. 4 -fl) IS riglan1lc-d ..d i>o>ock:s

I. S./i).

4, A t ube ha~~i dc:suf1cn~lh 2unil:o,. Wh:al tlrecoo di n:llcsofilscornc~

i r unc ~lm~,:.l'itr. al thcofig.in. lhrccof'its faces lie in1hc coordinulc1>1:anc~.


and one CC'II1lCr h;u. all thn:c coordinate,: positi\c 1

S. Show ollUI the (undirc<l. pcrpcndicull!l') distancts fron n poim


(.c. J'. l) to li!C .<. )', un~ zaxcs are, <lollti-,ly.
+ zl,
./' + :'. Ull<l ,~... + y'.

J.'

In E"<erci...c( 6-9 find the (undircdcd) dulanCX:s from the

pom~

to co)

111< cngu>. <b) the .CJJus. (c) the )' <is. ond Id) the 4>-<i>.
6. (2, 3. - 4)

7. ( I , - 5. -6)

II. (4. 3. 0)

9. (-l. I. -3)

16.

10. Prove th.lt the 1/V'ce poinu (I. 3. 5) . ( - 2. 0. 3) . .,,, (7. 9. 9) n:


collinear.
It. Rna lll.lt point in the XI -pbnelllat is cquid>tant fron1 the poo ""
(1. 3. 2) 1111<1 (2. 4. ~) .00 h.., a y-coordmalc equal to three time ;,.
\' "'(()01\llf\31(:.

Jl. f-ind an equ1uion describing ull points that oro cquid i ~ltl iH l'on
the J1Ciirltlft (- 3. 0. 4) uud (2. I, S). Wh:11 <IW$ 1his cqu11tit'n tk looCiil>c

Ia) Find the mdpoinl of the hrc: S<&m<tll joining the poin~
P(l. -1.-31 and Q (3. 2. - J ).

""n

tb) If dli! lill< "''!mtnt joinin' P .1M Q is extended i~


length beyond Q ll> potnt R. find the <voolmates of

1 17. The fOUf'-sidcd object 1ft 11\c: fj,,uc below is a trtl'flhrJrc:m. (f the
rour \'(f"IICCS or the (('lt1h<ron arc as sho\\ 11, Pf'O\\: lh.u lh ( chn.-oc lines
joining the midpoims ofOWQSiiC edgej (one of which i-. PQ) ncc1 u1
liJ)I'inlth:H bisect!\ cuh uf"llh.:nt.

scomctrktllyt
U.

ld, ./)

Ia) If (J3 - 3, 2 + 2J3, 2J3- I) nnd (2J3. 4, J3 - 2)


utc twu \Ctticcs of an cqtti laLeral trianj!lc. und ,(the lhu\1
vcncx lies on tile '>axis. find the 1hird ''enu
(h) Can )C)U lind a third vl;!rtCX on tJ~ .r -u i~?

a.a. A birdhouie " b~:ult tronl a 00-<


I/ 2 m oo exh side with a roor
~ ~~~ tn the figure 10 the

riglu. If the dlst21tee from eacb


Mrner or the roof to the peak i.IO
31-1 m. find coordinates or the
nine come~
ide~

or the house.

(T he

of the box are parallel to

the coordinacc )>Innes.)


X

R.

2~

l b. r.O)

c.

18. Let A. 8 .
and D be the \ Crttcc... or a <tU,adril.ttcral in ~p;tte (not
planar). Show th~1 1 th..: line o;cgmcnt~ joining midpOints of
oppusitc sides of the 'l' u~dri lm cna l iniCI''iCCI inn point th;tl biscc t.s each.
nc(."CS~::ui ly

698

**

Chapter I I

Ve(lOCS an<l Th.ree-l)n'l'len~on al A(UI)'Iit.: Gcornr::try

19. Generalize the result of Exercise 15 to prove that if a point R


divides the: 1crHtlh

P Q so

ii PR\1

thM - ~ RQft

l't

= -.

r,

when:: ,., anti ,.2 arc

L.

positive integers. then the coordinates of R are

:c=

r 1x2 + rzx1
r,

+ r1

r,yz + r2.\'t

)' = ,., + ,.,

z=

20. A man 2 m tall walks alons the edge of a straisht road I0 m wide
(figure right). On the olhcr edge of tbc rood stund$ u ~Mcct1ight 8 m
high. A building runs parallel to ~"' ruad and I rn from it. If Cartesian

coordinates are set up as shown (with X and .1-axcs in the plane of the
road). find coordinates of the tip of the mrm's d\OOow \vl\,en he i.~ at the

position shown.

11.2 Curves and Surfaces


Au equation involving the x - and .\-coordinates of points in the xy-planc usually spccilics
a curve. For example , the cqu:nion x 2 + y 2 = 4 describes a c ircle of radius 2 centred at

C:,r:J!TjT-JDOI lu 1he XJplace. x' + ,.: = 4 describes a


\:irdc:

l(j(#\ii.t#j.....
.t 2

+ >-' =

4 ~t:ribcs

Tit spa.
a cylindt r

the origin (Figme 11 .8). We now ask what is defined by an equarion in,oJving che Cane.<ian
coordinates (x. y. z) of points in space. For example. the equatioo z = 0 describes all poims
in the xy-planc since all such points ha"c a z-coordinate equal to zero. Similarl)', y = 2
describes all poinL~ in the plane parallel to and 2 units co the right of the x ~plane. What docs
the equation x 2 + y 2 = 4 describe? In other \\'Ords. regarded as a restriction on the x -, y-, and
z-eoordinates of points in space, ratherthan a restriction on the x- andy-coordinates of points
in the xy-plane. what does it repre.<ent'? Because the equation says nothing about ~ - there is no
restriction whatsoever on z. In other words. t:hc <-coordinate can take on all possible value.~.
hut x- and )-coordinate,; must be restricted by x 2 + y 2 = 4 . If we consider those points in the
xy-plane (~ = O)lhat satisfy x 2 + y 2 = 4, we obtain the circle in Figure 11.8. In space. each
of these points has coordinates (x, y, 0). where x andy still satisfy x 2 + y 2 = 4 (Figure I 1.9).
If we now take any point Q that is either directly above or direcdy below n point P(x. y, 0)
on this circle~ it has exactly the same x- and y-coordinutcs as P~ only its .:-coordinntc differs.
Thus the .t- and y-coordinatcs of Q also sati~fy x 2 + y 2 = ~- Since we can do this for any
point P on the circle. it follows that .t2 + )'2 = ~ describes therigln-circular cylinder or radius
2 and infinite exlent in Figure 1 1.9.
Byreasoningsimila.r to that usedabo\'C, we can show that the equation 2x+ y = 2 describes

the plane in Figure I 1.10 pamllcl to the ~-nxis and standing on the Straight line 2x + y = 2,
z = 0 in the .t)' J>Iane.
Finally. consider the equation x 2 + y 2 + ~ 2 ~ 9. Since Jx'i + _r'l + z2 is the distance
from the origin to a point with coordinates (.t. y. z) .this equation describe> aiii>Oints that are
3 units away fTOm the origin. In other words, x 2 + y 2 + : 2 = 9 describes points on a sphere
Q X.)'. :)

.. . -- .-..
~"

of idllius 3 cerure.d at 1he origin.


It appears tbm one equation in the coordinates (x, y,

~) of points in space specifies a


su11ace. The shape of the surface is detem1ine<J by the form of the equation. If one equation in
the coordinates (x , y, z) specifics a surfac~ itt is easy to see what two simuhaneous equations
specify. For instance. suppose we ask for all points in space whose coordinates satisfy both of

the equatioos

x 2 + y 2 = 4,

z=

I.

By itself. x2 + y2 = 4 describes the cylioder i11 Figure I 1.9. The equation z = I describes
all point.s in a plane parallel to the .typlane and I unit above it. To ask for all point.s that
satisfy x2 + y 2 = 4 and z = I simultaneously is to ask for all points that lie on both swfaces.

I J.2 Cunes Md Swf:~<es

ri'I'!l!l

Plane with
tquar~

2.\

+y=

699

Curve of imersection of

<ylindt l' .<'+ ) J = 4 ~~~d plane ~ = I

~/-----------2---/

..---.-----r------

/
.r

Coosequemly, the equations x2 + y2 = 4 , z = l deseribe the cun"e of intersec~ion of the 1\VO


surfaces - the circle in Figure l l .l J_
The equation x = 0 describes rhc yz-plunc; rhc cqumion y = 0 describes rhc .r: plnnc.
If we pur rhe rwo equations together, .r = 0 and y = 0. we obrain all poinrs rhor lie on both
rhc )'~plane and rhe x: plane (i.e., the z-axisj. In orhcr words, cquarions for rhc z-nxis nrc
X= 0, )' = 0.
fiJtally, x 2 + y 2 + z2 = 9 is the equation of a sphere of ra<lius 3 centred ar thc origin, and
y = 2 is the equation of a plane parallel to the x z -plane and 2 units to the right. Together, the
equations x 2 + + z1 = 9, y = 2 describe rhe curve of inre.rsecrion of the rwo surfaces rhe cin:lc in Figure 11.12. Nore rhar by subsrituring y = 2 inro rhe cquarion of rhe sphene. we
ca n write ahcmuLivcly that x 2 + :: 1 = 5. y = 2 . This pair o f equations is cqujvalent to rhe
original pair because all poinrs rhar satisfy x 2 + l + z 2 = 9, y = 2 also sarisfy x 2 + : 2 = 5,
y = 2. and vice versa. This new pair of cquarions provides an alrcmarive way of visualizing
the cUJvc. Again y = 2 is the plane of Figure I 1.12, but x 2 + z2 = 5 describes a righr-cin:ular
cylinder of radius ./5 and inlinire exrentaround they-axis (Figure 11. 13). Our discussion ha>
shown thnr rhc cylinder and plunc inrcrsccr in rite some cunc as rhe sphere nnd plane.
In summary. we have illusrrnrcd rhar one equnrion in rhe coordinarcs (x, y , z) of a poinr
specifies a surface: rwo simu lraneous equarions specify a cun e. the curve of inrersecrion of the
awo surfuccs (pro,.ided, of course, rhat the surfaces do inrcrsccr).

ljlflll;f" lJ'

LEM

Curve vf inlc...,
kt'1ioo o( q:~lrre x~ + y! + :! = 9 and

inler~lion of cylin~r .\"~

pl<lnt y = 2

:tnd plane

Cuf\-c of
..&.

z: =

r = '2

z
)'= 2

.t'2

+ x.? = 5

\
y
X

In Owplcr. l -9 we k.Wl'Cd I() appreciate tbe vuluc or p&uuin m\d drUWII'!J (.'\If'\~ in the
xy plane. Sumeun,t$ t1 plo1 ot dnawing 5ei'YCS as a dc,t.;c by W'hich 'We c11n interpret algebraic
stac:emcncs ~nw:tnclly (MJ~~;h ns the mean 'aluc lheorc'nl or the interprctdlion of a critical point
o( u fu nctic.Jn a-. a point "here lhc.tange.u: Ji11e m the grnph of 1h~ function is horin..wunl. \'trtkal.
or does. not c-~i:,.l). Sunk: tint~ tl'k.-y ptay a n inccy al P'-V\ in the wtutM.m or a probk:m (sut.-h as
~hc:n 1he d~finite intcar' i~ u.~'\J to fi nd areas. ,oJ,tmcs., ccc). SOflletimcl o plo1 or drawing is a
c:omplete solution to a problem h,uch as to dctt.-rmifte whether a ~ i\Cil furli!tion llCL') a.- inverse).
\Ve wi II fiud th.u ph.MIIr1g Ut1d <Jmwmg. .:,.urfiiK.."eS can be j uo;;t it" usdul for t'nult iv~\riubl c cal(:ulus
in Chapter> 12 1-1.
Oncufthc most hel pful techniques for drawing a surface is to im~agine the intersection oft he
surface wilh varinus ph,rtes - in particular. the coordinate pl anes. forotll th c~c cross-sections of
the -.urt'acc, i1 1~ somc1irnc." 1>0~<i blc 10 vi~uali?c the entire surf'ut.-e. Ptlrcx:lllli)IC, if we intcrM.-ct
the surface: = ,., ~ yl whh 1he yzplunc. we obtain 1he r~u"~1bob1 .: c: )' 2 .r: 0. Similarly,
the paruboh:1 .: = .r1.) = 0 is the itHcooec."lioncurYc with the xz~planc. rhc~r>c curves. s hown in
1 might be: ~hapcd as shown
Figure 11.14a. \\Ould lcaJ us to ~u~pect tha11he: sul'facc z ~ .\' 1
in Figure I J.l4b, To verify lhi~ \\'C ilncrsect lhe surface with a plane :. = k (k a coo..\'"t.ant),
giving the curve

+ )'

, .' + y - J:
.: = k.

:= /...

1be Iauer equation~ intlicjue th.u cross-sections of:: = x 2 + y ~ with plane:) : = k are circR:s
c-entred o n the .:~axis \\ith ro.1dii .Jf that increase a~ k increa.su. Tht~ cu-tainly cunfinns the
sketch in Figure 11.1 -lb.

tLiEl!I
~

= ,f: + )'~ W\lh

cn.. ~ ~lhlllll or strt'acc

lll tH IMII I.\lll

of

,\':0 and H \:OI>rdin.a" p!~u~~e~.

lmersections of the surface y = z + x 2 with the .vy-. x;:-. and y:-coordinatc planes give
1wo parat,..lla( ;uld u 'trai~~tlnline. :"roohc,w1 i11 Figun:: II , ISil. Thc,~o;e reall)! do n01 help us visualize
the SUrface lr, hOWC\(r, WC intersect thebUrface \\ith plane., Z = ~ , \\C obtain the parabolas

= : + .r

:: = k

or

y = xz+ k

z=

k.

These p-.uabol:ls. s hown in Figure I I.ISb, indicate that the s urrxc y


as in Figure I I. ISc.

= : + x 2 ; hould be drawn

\urfact } = ;: +
l

y = z.

/ \ X= O

Ill usn

Cross-mons or

Crossso:t!OOS or
1

x1 with coonlillale pla~es

surt.Jee J = ;: "'" x

wilh ~bnes

z= t

tioo of surface y

= : + x:

I
... ...

..

We Ciln sometimes ..build'' surfaces in much lhe same way Ihal we "buill" curves in single
vari<~ble calculus. For Jhc surface z l - x2 - y2, we firs1 draw the surface z x2 y 2 in
Figure I 1.14b. To draw z = - (x 2 + y2), we corn z x 2+ y2 upside down (Figure I 1. 16a),
and tinnily we see that z
I - x 2 - y 2 is z
-(x 2 + y 2) shifled upward I uni1 (Figure

= +

l l.l 6b).
Uluw..rioo

UIUS:rlllio

of ~urf:r:e : = -lx' +>~)

of surfxe :

=I -

x' - y'

z=J -x'-y'

~y

( EXAMPLE 11 .1

Draw lhe surface defi ned by each of 1he folltl'.,ini equations:


(a):= J4x + 2) -A 1 - , ,, - 4 (b) .v = 1 - ./x 2 + z'
SOI.l'110\
(u} If we square Ihe equation, and at the same 1imc complete squares on - x 1

- y 2 + 2y , we have

+4,t

and

or,
(x - 2)

+ I.V- 1)2 + : 2 =

J<x-

I.

(x,

Because
2F + ()'- 1)1 + ;:l is the di>toncc from a point y. :) 10
(2, I, 0). this equation stale< thai (x. y. z} must always be a unir dilonce from
2 = I defines a sphere of radius
(2. I. 0) Ii.e., rhe equation (x - 2) 2 (y - I ) 2
I cenlred a1 (2. I, 0) ){Figure ll.l7a). Because the original equation requires: 10 be

+:

nonncgati\'C. the required surracc is the upper hair or this sphere - the hemisphere
in Figun: I 1.17b.

702

CNpltt I I

\ b.:-tOr$ nud Thrci!-Oin'le:nSionol Anal)'tk

Mijlt111;11

<- -

2)

G eon~l)'

mHfl'W

Uii!l~lil;l4

Sphere dcK1ibcd b)

QI"iJ!W

llcmisphcrc de-

+ o - n' + z' = '


:

,T

(b) If we intersect lhc surface y


J x2 + z' with the x y-pl;uc. we ob<ain the broken
(i.e., bent) straig ht line y = lx 1. z = 0 in Figure I LI Sa. 1ntersections of thesurfat:e
with planes y = k (k a co nstunt) gi, e

Y =

J.l1 + z1

or

y = k.

y=k

1l1ese define circles of rndii k in the planes )' = k (Figure 11. 18b). Consequentl y,
>' Jx' + z2 deli >les the ri&ht-circ'lllar co>le in Fi&urc I LISe. 1l1c surface >'
I + Jx1 + zl can now be o btained by shifting the cone I unit in the )'direction
(Figure 11 . 18d).

_mn rw

~J

)(.'(.1iun

or surt':iw."C y

L B'Ut&AL m.J Cross


~"liOB~ c f ~urt!!ce y - Jx i + ,: !
wilh pl:a"e ~ J =k

oo~s

= ./xi + .:1

wi1h ...y-p!~

'
K3M\

IT::._
dc:!oCO'bcd by r -

Con~

J.t: + zi

CUll(

dcscribtd by ')' ~ I

y= J xl + :1

y= 1 + \lxl + ::l

I
-.

+ Jx: + ~:

- ---------

---y

11.2 Cauve:s ak.1 Suf:.cet

703

Cylinders
Suppose that / is a suaight l ine and C is a curve that lies in some plane (the x y-plane in Figure
I I . I<Ja). A cy lind er is the surface traced out by a l ine that moves along C always remaini ng
parallel to I (Figure I 1. 19b). The right-circularcylindor in Figure 11.9 is gcncnued by moving n
vertical line around Lhccirclcx 2 y 2 = 4, 1; = 0 in the xy-planc. A lthough we might not likcto
thi nk of i t as such.the plane in Figure 11. 10 is a cyl i nder. The surface i n Figure 11.15c is a cylinder; move li nes parallel toy = z, x = 0 in Fi gure 11. 15<1 along the parabola y = xz, z = 0.

M:ttt'llla

l)efinilion of a cylinder ul\ins tin.e :md CUf\'c

When one of the coordi nates x . y . z is 1nissing from the equaLion of a surface. a cyl inder
results. The right-circu lar cylinder in Figure J 1.9 i s an examp.l e (~ is 01issi ng). For such
cylinders. a l i ne parall el to the axi o f the missing variable (the line.< = 2, y = 0. say. for
.r 2 + y 2 = 4) p lays the role o f/ , and the cross-section of the cylinder with the p lane of the
remaining two vnriables (circle .r 2 + y 2 = 4, ~ = 0 in the x y -plaoe} plays the role of C. 1\JJ
cross-sections of the cylinder with planes perpen<liculflr to the axis of the missing vari;~ble are
identical to C.

111C equation z

= ..r2 is free of )'. Euch CI"OS$~a:tion or lhi~ surface with a plane y =

is 1hc parabola z = x2 i n the plane y = k. Consequently. z = x 2 is I he equmion for the


parabolic cyli nder in Figure 11.20. The surface yr = I, x > 0 is the hyperbolic cylinder in
Figure 11 .21 . All cross-sections in planes parallel 1o the y z-plane are hyJ>erbol as.
, . 1 t311-lj 11m

FIGURE 11.20

Hypetbl'llic cytindu y: - 1. ;r > 0

lz=x'. ,

-- ~::___

)'

'
Quadric Surfaces
A quadric surraee is a surrace whose equaLion is quadr.ttic in
such equation being

Ax 2

x, y, and z.

+ Byl + Cz2 + Dxy + Eyz + Fxz + Gx + H y + /z + J

Lhe most general

= 0.

( I 1.2)

For the most pan~ we encoumer quadric surrac.cs whose equations are o f the fom1

or these equations with ~' . y . and z interchanged. Surfaces wilh these equations fall into nine
major classes, depending on whethc.r the constaltS are positive. negative, or zero. They nrc
ill ustrated in Figures 11.22- 11.30.
The names or these surfaces arc derived from the fact that their cross-sections '"c ellipses,
hyperbolas. or parabolas. For example, cross-sections of the hyperbolic paraboloid with plattcs
z k are hyperbolas x1 la 2 - / IIJ1 = k. Cros~scctions with planes x = k are parabolas
z = k1 I a2 - y 1 I b 2, as are cross-sections with planes y = k .

ljldiiiWIWfM

Elliptic f.) 'linder

M@lciii;I:WIWIM

Hypctbol i~ (.~Iin der

lijiUII;IWIWJM

Parabolic <.:ylixler

'
.1.'2

{12

a _,'

}'2

+ :-. )

IJ2

.....~.- ... b
y

)1

----y

A'

Millflll;l

Elli psoid

'Eiliplic p!antbolokl

Mjl?lll;l Cta:ia

E.WI){ic ~;one

l.

MjUiii;I:WIWJ:M

Hy~bol ic

l)(.traboloid

Miilciii;IMJ~

EJiiplic hyperboloid

Mijtclll;lj-'llil!ll

ot' l)!le Jt.hect

'

Elliptic

11.2 CurVes and Surfaces

705

ln applications o f multiple integrals in Chapter 13. it is often necessary to project a space


curve into one of the coordinate planes and l:ind equations ror the projection. To illu:;uate,
consider the curve of intersection o f the cylinder x2 + z 2 = 4 and the plane 2)' + z = 4
(the li t'St octam part of which is shown in Figure 11.31). Since the curve o f imersection lies on
the cyljnder x 2 + z2 = 4. its projection in the xz-plane is the circle x 1 + z2 = 4, y = 0.
To llod its pr~jecti on in the xy-plane., we eliminate z between the equations 2y + z = 4 and
x 2 + z2 = 4. The resuh is x'2 + (4- 2y)2 = 4, or x 2 + 4(y - 2)2 = 4. This shows that the
curve of intersection lies on the elliptic cylinder x 2 + 4( y - 2) 2 = 4, and therefot'e it projects
omo the ellipse x1 + 4(y - 2) 2 = 4. z = 0 in the x y-plane. The projection o f the curve
in the yz-plane is that pan of the line 2y + z = 4, x = 0 between the points (0 , I. 2.) and
(0, 3. -2).
,li!ilciii;IWIFIM

Projcccioos (lf a cur\'e in 1he coordionlc l>l>l!lC'S

lx'

+:2=

4.1

2y + - =4

Projection
F-plane
2y+ :_::4

Projection
in x:-planc
X

Proj ection
in .\)' plane

Graphing calculators and computers can plot surfaces provided that the equation or the
surface is solved for z in tenns o f x and y. (This may n01 always be convenient.) We have
shown some computer-generated plots in Figures 11.32together with values for x andy specified
in generating the plots. With the complexity of lhe expressions for z . drawiog these surfaces by
hand would be a fo rmidable task.
To appreciate the shape of a ploued swface, it is often necess:uy to vary the po int in space
from which the. surface is viewed. Computers usually have this ability; graphing caJcuhllOI'S
may not.

DM!W

FIGURE 11.32b

Computer plots of surfuces

:, :: 3 ~x2 + y~ - x~ - y2

: : X( )' - J)2e-(\' + !"Y4

- 2SxS2.-2SyS2

- 5 S X S 5. - 5 S .\' S 5

CHAPTER

Calculus Preparation

' 1.1 Introduction


Over the years, I have l earned that the .nos:t f requent reason for students fHi l i r1g to achieve
optimum results incalculus courses is inadcqlw teprepararion. Smdcncsnwy undersrand calculus
(.;Oncepts completely. Lhey may have every formula and every rule memorit~ed _perfectly~ they
may e ven have a c lear idea of the proca.lun::s required to solve prob lems~ but because they
lack skills in a_lgc.bra, geometry, and tr igonorn etry, they cannot put their krlowledge to work.
h i s unfortunate that they never get to dernon~wne their calcu lus knowl edge becc.lll~ of poor

mathematical preparation . To obta in the-be!\1 possib le grades in calculus, it is essential to have

excellentalgebmic skills and a good gmspofthe elementS of analytic geometry and trigonometry.
In this chapter we ghc yoo the op)>Ortunity to test your skills and knowledge in these three an:as.
and, should it be necessary, the meart~ by which 10 make improvements. Titere are also ~tions
lhat may con1ain material unfamil iar to many readers. Topics in these sections are esse.ntialto
some oFthc applications of calculus; your instructor will indicate whether they are required For
your course.

l lte calculus course at your institution may, or may not. spend time Qn the review sectio1s
in this cb1lpter. Jf it docs not, your instructor may ad,'isc you to review ccr1ajn sections or1 your
own. DO l'f! You could regret ignoring review material in this chapter in order 10 get to caJculus
i n Chapter 2 more q uickly. Each review sectio n opens w i th a diagnostic test to d~ tcrmine your

knowledge of the material in the section. Give yourselr the suggested time to take the test. no
longer. You must not only be able to solve the t est questions. but you m ust also be able to do so
reasonably quickJy. Do NOT use a calcubnor unl e~s spe;cific;:ally instructed to do so. Answers

are provided m the end of the section along with marks for each question. Assign yourself
pan ial marks For partially correct answers, but try to he objective in doing so. It is difficult
to be specific as lO what constitutes an acceptable score on the diag.nol)(ic tests. Ce1tainly a
score o f less than 50% indiC>Jtes thai detaile<l Sludy of the section is required. A score of more
thar'l

soc&, but not much JTt()re, would also suggest the need

fo r careful review. ~fart.: ~ in the

80~ 1 00%. range indicate a

good working knowledge of material in the section, but it could be


beneficial to gi"c the. section a quick rcading. Jl'aying Jtrlicular aucntion tO ,,arts corresponding

10 incorrcclly answered lCSI queslions. ln addition, not only is il helpful to refresh your memory
on concc:pu leamed some lime ;lgo. it is also wi.se 10 become familiar with lhe terminology,
nolation, and conventions set forth in these sections. To improve your skills o n, and knowledge
of, dlC material in a rc,icw sectinn. read the di~cussi ons and examples thoroughly, tr) as m;lny
o f 1he exercises as yuu c.:an. unU then n:tukc the diagnost ic test. If you arc conscientious in your
work, we are confident that you will do much b-etter the second ti me. Yo u w ill be well rewarded
for talc.iog the time to do this; your calculus sr.udies wiJI be so much easier. ln fact. the 1ime you
spend on calculus prepa.rarioo now wiiJ more than compensate for ex 1ra time that you would
s pend on solving calculus proble ms late r. Be lieve me; I have Laught t.housands o f s tudenL.s j us t
like you.

11.3

Vt<"IOfS

707

EXERCISES 11. 2

In Exercises 46 SS find !.!quations for projcctiC)n;; of the Cttr\'C in 1hc


In c:.1ch ca:w: drow the curve.

In Excniscs 1- 3S draw the s urfac.:c defined by the equatiou. Whenever


possible. confirm your dr.twing with a plot genel'ated by a computer or
graphi ng c.alculator.

I. 2y + 3z

2. 2 - 3y

3.

y ~ x' + 2

4.

5.

.r' + z2 =

6.

7.

x' + 4y 2 =I

10. x=Jy' + z'

.1.1. X= j l - y'

.12 z= 2 -x

= y2

.14. x =

+ z2 = 0
x' + y' + (z- 1)2 =
x 2 + z2 = y'l

17. .r 2
19.

- 2x

48.
4
49.

*
*
*

+ y'

)'Z

20.

z + 5 = 4(.r 2 + y2)

= y 2 +I

23. y 2 + z 2 = x

24. x 2 + y 2

+ 4z2 =

25. 9z2 = x 2 + y2 + I

26. ()' 2 + z2 )~ =X + I

27. zl

+ 4y2 =

29. x 2 - z 2 = 4

51.

x' + y' = 4. x + y + z = 2

53. Z = .r 2

33. z=y 2/ 16 -.t 2/4

34. x 2 + y 2/4 -

= 3, x' + z2 = 3
+ y', X+ Z =I

+ y 2 + z1

= I , )' =

y2

orthe curve in l.he

* 56. z = x 2 - y2 z = 2x + 4y in the xy plane


* 57. x 2 + y 2 - 4 t.! = 1. x + )' = 2 in the XZ-plane
* 58. y = z + x1, y + z = I jn the xyplaJle

30.

z'/25 =

=z

z ~ Jx' + y' , z ~ 6 - x' -

55. X~

ln Exercises 56-61 find tXJIUUi()nS [(ll' I he proj ecli<)n


specified plane. Onhveacb curve.

28. y - z 2 = 0

31. z = x'/4 + y'/25

z' - 9x' -

x' + y 2/4 + z'/9 = 1


32. x2 = zl + 9y2

35.

x 2 + y 2 = 4, x

54.

= I

18.

22. x 2 + z 2

21.

50.

52. y 2 + z'

z' + 2

16. 4z = 3j.r 2

IS. z=y+3

47 .

=4

9. l = 2(.r 2 + y 2 )

.13. x2

+ y ~ 3. 2y + Jz ~ 4
.t + )' + z a 4, 2x - )' + z
x' + y' =4, z =4
x' + y' ~ 4, y c x

46. x
0

z = ,\'J
x' + y' + z' =

8. y2- z2

xy. yz. and ,\'Z(,.'OOrtliatc p lanes.

59. x = j 1 + 2y 2

16y 2 = 1

*
*

+ 4z 2,

x'

+ 9y 2 + 4z 2 =

60. z = .r:! + yz, z = 4(x- 1)2 +4(y 61.

x2

y2 -

2y = 0 ,

z2 = x ~

36 in lbe yz-plane

1) :! inth cxy-p lanc

+ y~ in tJtc x z-planc

In Exercises 36-45 draw the curve defined by the equations.

36.
38.

40.
42.

x' + y 2 = 2, z = 4
37. x + 2y = 6, y - 2z =
t. = x 2 + y 2 x 2 + y 2 = 5 39. x 2 + y! = I. x + z =
z = Jx' + y', y = x
4.1. z + 2.< 2 = I , y = Z
2
2
t. = J4- .r 1 - y 2 x + y - 2y = 0

43. z=y.y=x'
44.
45.

xz + z2 = 1, y 2 + z2 = 1
z = .r 2 , z = y2

In E.xercises 62- 71 dmw whatever is defined by lhe equation or c:qua

tions.

62. (.r - 2)' + y' + z' = 0 63.

65. X+)'= 15, )' -X = 4

67.

z = lxl

z = x', y = z'

69.

x = ln (y 2 + z')

z = lx - Yl

71.

x=

v 64 .

..jX + ,jY = I, Z =X

66.

z=

68.

70.

x = 0, y = 5

I-

(x' + y 2) 113

2, y = 4,

z' -

1= 0

111.3 Vectors
Physical quantities th at have associated \Vith them only a magnitude c an be represented by real

numbers. Some e.xamples are temperature. density. area. moment of inertia. speed. and pressure.
They are called scalars. There arc many q uantities, however, that have associated with them
both magnitude and direction, and these q uantities cannot be described by a single real number.

Yelociry, acceleration. and foroe an:: perhaps the most notable concepts in this category. To

represe-nt such quantities mathematically, we introduce vectors.

DEFINITION 11 . 1

A vector is defined as a directed line segment

To denote a vector we use a leuer in boldface type, such as v. In Figures 11.34a-c we show
two vectors u and v along a line, three vectors u, ,.. and w in a plane. and three vectors u. v,
and w i n space. respectively. lt is customary to place an anowhead o n a vector and caiJ this
end the lip of the ector. The other end is called the tail of the ector, and the direction of the
vector is from Lail to tip. A vector the n has both diret:lion and length .
lifiCJII;Iijm!F'RW
along 1he x-axis

,,

lilf#Jil;l:ilmflllll

Vectors

Vectors

FIGURE 11.34c

in

Vectors

Sp.1CC

)'

'

Definition 11.1 for a vector says nothing about its point o f application (i.e .. whel'e its tail
should be placed). This means that we may place Lhe Lail a nywhere we wish. This suggests the

following definition for equality of vectors.

DEFINITION 11. 2

Two vectors a re equal if and only if they have the same lengLh a nd djrection. Their poinLS
of a pplication are irre levant.
l:ilc-lii;IWI414 Equality (}f
ve<:lors u and v bul ooc of W and u

For example. vectors u a nd v in Figure 11.35 have exactly the same le ngth and d i.rection.
and are therefore one and Lhe same. Allhough the vecto r w in the same figure is parallel to u
and v and has the s.arne length, it points in the opposite d irection and is not, therefore, the same
as u and v .

Components of Vectors
X

We realized in Chaptel' I that to solve geometric problems, it is often helpful to represent them
a lgebra ically. ln fact. our emi.re developrnem of singJe.va.riable calculus has hi.n ged o n our
ability to represent a c.urve by an algebra ic equation and also to draw the c urve desc1i bed by
an equation. We now show that vectors can be represented algebraically. Suppose we denote
by PQ the vector from poim P to point Q in Figure 11 .36. If P and Q have coordinates
(x., y 1, z 1) a nd (x2 . y 2, z2) in the coordinate system shown, then the length of PQ is
( I I.J)

Note also that if we.start at point P . proceed x 2 - x 1 units in the x~irection. then y2 - y 1

uniLS in the ydirection, and fi nally z2 - z 1 units in the zdirection. we arrive at Q. In oLher
words, Lhe Lhree numbers x 2 - x 1 y2 - y 1 and z 2 - z 1 c haracterize bolh Lhe direction
and the length of the vector joining P to Q. Because of this we make the following agree
ment.

ljlfJI1141FI

Veor &on' poin1 P 10 poinl Q

Jl

D EFIN I T IO N 11 .3

If the tail of a vector ,. i~ at P(x,. Yt z 1) and ill; tip is at Q(x2. Yl 1:2) . then v shall be
represented by the triple of numbers x2 - x 1 n- y1 z1 - z 1 In such a ca.<e we enclose

the numbers in parentheses and wrile


! I I AI
The equal sign in 11.4 means "is represe nted by." The numt>cr .t2 - x 1 is called the .t
COiliJ)Onenl of '' )'l - )'1 the )'Component. and Z2 - z. 1 the Zromponent. Vec1ors in the
xy-plnne hne only a n X and n y-component:
v = C.t1 -

x,. Y1 - y,),

where (x, y 1 ) nnd (x2 , y2) are the coordinates of the tnil nnd tip of v , Vectors along the x -nxis
have only an xcomponcnt x1 - x 1 where x 1 and x 2 are the c<x>rdinate.< ofthe tail and tip of v .
\Vc now ha,c an algebraic rcprcscmation for vectors. Each vector has nssociatcd with it uset
of components that can be found by subtractilllg the coordinates of il< tail from the coordinate.<
of its tip. Conve-rsely, gi"c.n a set of real numbers {a, b, c), there is one and only one vector
wi1h these numbers a' componentS. \Ve can vi suali7e thi' \'&tor by placing i1s tail atrhe origi n
and its tip at the 1>0in1 with eoordi nat~ (11, b. c) (Figure 11.37). Ahemativcly, we can place
the l:lil o f the vector at any po int (x1 , )'1 , z 1) :and its tip all he 1>0i111 (x1 + a , )' 1 + b , z1 + c) .
It is worth emphasizing once again that the same componenlS of a vector are obtained for
any point of application whatsoe\'er. ~r example, the two vectors in Figure 11 .38 are identical,
and in both cases the COilli>Oncnts (2. 2) are obtained by Sllbtracting the coordinates of the tail
from lhooe of the tip. What we are saying is t hat Deli nition 11.2 for equality of vectors can be

!)tated algc:brditally as follows.

THEOREM 11 . 1

Two vectors are equal if and only if they have the same components.
T.lil of a ,ccor

ca11.

be pl.aet:\1 m any po-;m

IUlit

w wm

1\\o equal

(a. b. ")

}' (I. 4)
/
( - 1. 2)

3
2

(4.2)

I (2. 0)

2 3 4
X

\ ClQI")

Chap1er I I

710

I EXAMPLE

Veck)J'$ and 1bre~ -Oi men -:ion:.l An:aly1te Goometry

11.2

liUijil;l mfi1JI Con~ncnts


o( ''c.l.i or with (cDgth 5 ontJ ansJc ~
rdltJians \\i lh .t'-a;..:b;

Find the components of a vector in the x y -plane that has length 5, hs tai l at the origin, and

makes an aogjc of n:/ 6 radians with the positive. xaxis.

SOLL'TION Figure 11.39 illustrates that there are two such vectors. u and v. From the triangles
shown, it is clear 1.hat

liOP ll = Scos(rr/6) = S.J3/2 and IIP QII = li PR ll = Ssin(rr/6) = 5{ 2.

Consequenlly, Q and R have coordinates Q = (5./3/ 2, 5/2) aod R = (5./3/ 2, - 5/2), and
p

" = (s.JJ.
2 2~) .-

,.

I EXAMP L E

11 .3

I:JIHII;IWikUM

Comc:x:nlents

of ol \'t1t)r t,i\'e.o i 1.~ length ;II)({ itG


dire~t:tiun

Find the components of the vector io the xy plane that has its tail at the poim (4. 5). has length
3, and points directly toward the point (2, - 3) .

SOLUTION lo Figure 11 .40 II P Q II = J22

+ 82

= 2Ji:7. Because of similar triaogles,

we can wri te that

p (4, 5)

3(2)
3
liST II
IIQRII
or IISTII = 2.Ji7 =
=
gPS II
II PQII
.JI7'
T

Similarly,

II PTII = HPSII II PR II = 3 (S) = ~


IIPQII
2J17 Jl7

1--l.JR
Q (2, - 3)

Since liST II and II PTil represent differences in the x - and y-eoordinates of P and S (except
for s igns), the components of I'S arc ( - 3/.Ji7, - 12/ .Ji7) .

..-..

U nit Vectors and Scalar Multiplication


If the x - . y -. and t-<.::ompon~nt.s uf a vector ~ are (v..-. Vy. u J. often ca11ctl tl1e Cartesian
components of v, tben these compooents represent the differences in the coordiuates of its tip
and tail . But then. according to <'<Illat ion 11.3, the length of the vector. which we denote by 1''1,
is
( 11.5)

In words, the length of a vector is the square root of the sum of the squares of its componcms.

DEFINITION 11.4
A vector v is said to be a unit vector if it has length equal to 1 unj1; that is. v is a uojt
vector if

(11.6)
To indicate thm a vector is a unit vector. we place a circumllex" above it: V.

II.J

I EXAMPLE

Vecloni

711

1 1 .4

What is the length of the vector from ( I, -I , 0)

10

(2, -3. -5)"/

SOI.l'110N Since thewmponents of the vector arc ( I, -2, -5) , its length is

/ (1 )2

+ (-2)1 + (-5) 2 = J30.

\Ve now htwe vecturs . which are directed line segments. and real numbers. which are scalars.
We know thut scalars ca n be added. subtrnete.d, m ultiplied , and divided , but c~m we Uo t he ~me:
wilh vcc[Ors. and can we combine vector~ and scalars? f1l the remainder of this section we show
how to add and subtr.tct vectors and multiply vectors by scalars: in Section 11.4 we define two
ways to multiply vecto rs. Ea~o:h o f these ope~a tiuns can bt: approached either a lgc:bmically o r
geometrically. The geometric approach uses che geomelr-ic properl ies of vecto.rs. nlmely. length
B!Jii]
GeonlCfnt
illustNII1on of scalar mul1ip!icatioo
uf ,cctun

and direction: the algebraic approach uses components of \'\:<:tors. Neither method is suitble
for all ~i tuati ons. Sometimes an idea is mo re easily introduced with a geometric approach;
somelimes an :lgebraic app1oach is more suitable. \echoose whichever we feel expresses the
idc:.u mure clearly. But. whenc"cr we take <1 geometric approach. we are careful to fo11ow it
up with the. algebraic e<Juivalcnt; conversely, when an algebraic. approach is take.n, we always
illustmte the geometric s ignificance or 01e resuIts.

To introduce multiplication of a 'rector by a scalar. colsjdcr the vectors 11 and v in Figure


11.41, bol.h of which h:tve thei r 1.ails at 1he origin ; v is i n 1he same direction as u blll i s tw ice

a:, long as u . Tn s uch a ~itualion we would like to say that vis equal to 2 u a nd write ,, = 2u .
Vector w is in the opposite dire:clionto r cmd is three times (\S long as . a nd we would like to
denote this vector by w - 3r . Both of these siMtlions arc rct~li zcd jf we adopt the following
defini1ion for multiplication of a vector by a scalar.

.\
DEF IN ITION 11 .6
lf A > 0 is a scalar and ,. is a vector, then ). v is the vcx::tor thut is

it1 the same direction a.,.;


,. and A times as lo11g as v: if), < 0. then ).v is the vectorthat is in the opposite direction
to vand l!.. ltimcs as longas v.

This is u geometric definition of scalar multiplication: it describes the lcngch nnd direction
of !..v. We now show that the components of), v arc ). times lhc components or , .. In Figure
11.42 we show u box with 1\occs l>arallelto the coordinate plm-.cs and A\' !L~ diagonal. and have
given vceton componell tS (v,, v1 v, ). From the pairs of similar triuolglcs 0 A 8 mod OC D .
and 0 8 E and 0 D F , we can write that

nocn
v,

neon
v,\'

nOD II

= no an =

nDFI
v,

Ih i

Jvl

= !...

Hent.-e

IJOCII = lo.o

IJCDll

= Al>p,

II DFII

= >.v, .

where II 0 C 11. II CD II, and ll D F II are lhe comJ>Onents of Av. In other words, lhe comJ>Oncnts
of >.v are !.. times the comJ>Onents of ,,:
.l.v = J,.(v_, , vy . v, ) = (.l.v_, , !.. vy . >.v, ).

( I 1.7)

712

Cbarer II

VMorS and Three.l)imen ~ic:mnl An::.lylic Georne1ry

ljtciiJ.I}i '\l!tJ

Cum,,ouc::nb of

l.v are A limes lhe m nponc:nJ:,. Ql v

/)

To multiply a vector by a scalar. then. we mulliply each componem by the sc<1lar.

I EXAMPL E

11 .5

Find componems for the unit vector in the stunc direction as v = (2, - 2, I).
SOLl'TION 11le lengthof v is lvl = )(2) 2 + ( - 2)2 + 12 = 3. According to our deli nition
of multiplication of a "ector by a scalar, the ,ector ~ '' must ha,e length l ( j that of v) and the
same di rection as \' . Consequently, a unit vector i"n the same direction as " is

v=

I
- \1

~(2
-2. L) = (: _: . ~) .
3 '
.
3' 3 3

This cxan1plc lllustratcs that u u11it vector i11 thc same di rection as a gi"en ,ector ,, is
( 11.8)

v =

I EXAMPLE

11.6

Find component~ for the vecto r ofleng th 4 in the direction oppo_~ite that of v
SOLIJ'TION

Since

lvl =

(I, 2, - 3).

Jt + 4 + 9- ./14. u unit vector in the same d irec.tion as v is


I

-v
V= -114
.

The vector o f lc.ngth 4 in the opposite direction to ,. must therefore be

( -4}v = ( - 4 )

54

,. =

( - 4 ) o. 2, - 3J = ( -

54

54

8
.- ./14'

12 )

54

\Vith the O\JCrt\li~' of SClllnr mu\\ip\ication. we cnn s imp\\fy the solution of &nmple l \ .:>. The
vectorthtn poims from(-'. 5) to (2, - 3) is v
-'2., -8), ~1\d therefore the unlt vector in this
d irec.tion is

=(

"=

1
~(-2,-!\)

"

~ .... ""

'l " 17

" 11

=(-2,-S) =

=(-1 , -4) .

The required vector of length 3 \s

.\

Addition and Subtraction of Vectors

litilllJ

In Fisurc I 1.43 we show two parallel vec1or.s u and v and hwe plncod the Ulil of von the tip
o r u. 1l would ltCCin lli:)\Urt\1 to denote the ~tor that has its U\i\ t\l the tail or u anO its Gp at
the tip of v by u + ". Por instm'lce, if u <\l\d ,, were equal. then we wnuld simply be saying
thtlt u + u = '2u. \Vc use this ide~\ to define addition of vectors even when the vccu)rs arc not
parallel.

ucklition o{ vec1ur'

DEF INIT ION 11 . 6


The sum of two vectors u and v . denoted b~ u + v . is the vector from the toil of u to the
tip of v when the tail of v is placed on the tip or u.

lk<:ause the three \'CCtOrs u . v . 8f'ld u + v thc:n form a tr\an.1t (Figure \ \ .44), we taU this
h'i:.U'It;Uhu addition ur \'CCt()rS.
NtliC that were we to place tails o f u aml v both at the same \><li nt (Fi~ture 11.45), md
complete the par,\llelo~ram with u al\d v as sides. the diagonal or this pan1Helogr.~ m would also
represent the ,ector u + v. Thi s is a n cy_uivalc m method for geomerrica\\y findlng u + ", and
it is called parnllel~grtmt 11dditiun of vectors.
Alg.ebrait-aU~', \iet'\ Ol'$ arc added component by oomtXJilC:I'It: \hal h;., if u
( u \:, Ll y , U : )
and v = ( v,~., v>, v:), then

( I 1.91

To 'erify this we simply note that differel\ces in the coordinates of P and Q in f igure t 1.% a~e
(u, . 1.1 r u J, '"""differences in those of Q and R are ( v, , v 1 v~) . Consequently, difference.~
in the coordina1es of P and R "'""' be (u , + v,, u y + 11y . u: + v,) .
It is 1101 difftc\ll t lo show (see Exercise 26) thm 'ector ad<liti<.>n and scalar multiplication
obey the followi"S ntle.s:

1J

\'t\.."t~ ~..10

lbelt

[l

Tu '"'''

~:\)mpuncn1s

( tt

+ '' = \ 1 + u~

+ v) + w =

u ..,. (v

\ 11.1\la\

+ w);

A(u + v) = AU+ Av;


(!-

+ t~) v

= ;_,. + t~ .

( 11 IObl

(1 1.1 0<;)
( I I.I Od)

lf we denote the vector ( - 1) by -v, 1hen1hecomponents of -v are the nega1ives o f those of

v:
X

- v = ( - l)t = ( -I )(Vx, Vy, v, ) = ( -Vx, -Vy, - v, ).

(11.1 1)

Titis vec tor has the same lens th as v , but is opposite in direclion to ' ' (Figure 11 .47). When v
is added to - v , the resultant wetor has components tha t arc a ll 'lCro:

Ui!II!JJl"UFD

v + (-v)

Veci,,J.
1hc :ouunc lcng:lh ns v, bul
i ~ ~'.i i lc in direction to v
- v

h:.1~

= (v, . v1 .v;) + (- tl, , -vJ, -v:l

= (0.0,0).

Titis vector, ca lled the zero vector. is d enoted by 0 , a nd has the propct1y t ha t
(I I I ~)
for any vectOI' v wh.nts.QC\'er.

To subtmct a vector v from u. we add - v co u .

DE F I NI TIO N 11 . 7

The d itl'erence u - v between two vector.)

u. and v is the ec>r

.}(

II -

\'

+ (-

( 11.13)

v).

In Figure 11.48, u - v is detcn>tincd by a triang le, and in Figure 11.49 by a pamllelogram .


Alternathcl)', if we denote by r the vector joining the tip of v to the tip of u in Figure 11 .50,
then . by triangle a tldicion, we ha,e v
r = u . AUdition o f _ ,, to c~tch side of this equation
g.iYes

- v

Titus r

+ '' + r =

- v+u

or

+r

= _ ,. + u .

u - v. muJ u -vis the vector j oining the tip or v to the lip of u . Definilion 11 .7

implies that vectors are :iublt'd.ctcd t.:omponent by component:


u- v

M41Uil;l -._ . .,

(u,,, " >' u :) - ( vx. Vy . v;)

Emi!J

u - ,.

[l

II -

= (ux - Vx, ll y - Vy, 11:- V:. ).

c;,ll be ob1ained by lldding U nnd

can be \:btaino:t b)" ...:kl11'.g u

- v \\oilh llifl)i.ular ~i!di1i on

-v

:~nd

wi1h puallelogrom a dd it ion

n~;
bo obmi11ccl
subtrac1ion

( 11 . 14)

u - \ 'c:an
di rc<:~ l)

with

cril.1n~ lat

'
I EXAMPLE

1 1. 7

.........
Jru = (1 , 1, 1) , v = (-2,3,0), and w = (- 10, JO, - 2), 1intl:
(a)

Ju

+ 2v- w

(b)

2 u - 4 + w

(c)

lui + -

4
w

11

I L)

\'.1<)rS

715

SOI.lHOI\'
(a)

3u

+ 2>' -

w = 3(1. I. I)+ 2(-2 . 3.0) - (-1 0, 10, - 2)

= (3, 3, 3) + (- 4, 6. 0)

+ (10,- 10, 2)

= (9, - I, S)
(b)

2u - 4v + w = 2(1, I, I)- 4(- 2, 3, 0)

+ (-

10, 10,-2)

= (0, 0, 0) = 0
(c)

Since lui = J1 2 + 12
lu lv

+ j;j"' =

+ 12

.J3 and lvl =

-./3(- 2. 3. 0)

= (

/(- 2) 2 + 32 =

Ji3.

,fl3 (- 10. 10. - 2)

40
40
8 )
-2../3 - ./i3'
3J3 + Ji3'
- ./i3
.

Forces
\Vc have already mcnlloncd that quantities such as lcmpcra turc. area. and density havea.~~iatcd

with 11\cm only a magnitude and arc lhercfore represented by scalars. There are many quantities.
however. that have associated with them bolh magnitude and direction. and these arc described
by vectors. The mo~t notable of Ihis group are force..;. \ Vhen we ~pea k of a force, we mean a

push or pull of some size in some specific direction. For example, when the boy in Figure 11.51 a
t>ulls his wagon. he exerts a force in <he direction indicated by the handle. Suppose that 1\c pulls
with a force of 10 Nand 1ha1dte angle between ~tc handle and the horizontal is ;r/ 4 radians. To
represent this force as a vector F 1 we choose the coordinate system in Figure 11.51 b. and make
the agreement that the leugtlt u.fF1 be t(fi.alt<J tire mag11itutlc of the/tJIY:c. Since F 1 represents
a force of 10 N, it follow~ thou the le ng1h of f ' 1 i ~ 10 units:. Funhennore , bec~t us.e F 1 makes an
angle of 1t / 4 radia ns with the positive. ,\'a anc.l )'IIXCS. the difference in the :r:--<.:oordinatcs (and

the y-cooroinates) of its tip and tail must be IOeos(Jl' /4) = sJ2. The components of F 1 arc
therefore F1 = (SJ2. SJ2). Tf the boy's young sis1er <lr<~gS her feet on the ground. then she
effectively exertS a force F 2 in t he negat ive x -dircc1ion. lf the magnitude of this force is 3 N,
lhe.n ils \'CCIOr reprec;enl~u ion is F 2 = {-3, 0). Finally. if 1hecornbined weight of the Wltgon
and the girl is 200 N. then the force F3 of grnvity on the wagon and its load is F3 = (0, - 200).
FIGURii 11.61e
\\'<lgl\Jl t\elt~;.

Bl)) pulling

a fon::e in lhl'eeliOn UT hilndle

FIGURii 11.61b

bh\.~ti()n

<lf fo.tttS

e\.erttd by hOY ami g,h1 dragging, U.eir feel

F,
10

2!:
4

J:ll311;1iR..rmJ

Remlu n1

In mechanics we replace the individual forces I<' t. F2 and F3 by a s ingle force that has the same
effect on the wagon as all three forces eombined. This force, called the resultant force of F t.
F2. and F3, is represented by the vector F, whicb is the sum of the >-ectors F 1. F2 , and F 3:

(s/2, sJi) + (-3, o) + (0, -

200)

= (5./2 - ' sh- 200J.


The manitudc of this force COITeS(JQnds to the length of F,

IFI = / <sh -

3)2

+ (Sh - 200)2 =

193.0,

and must therefore be 193.0 N. hs directiool is shown in Figure I I.Sic. where

9 = .fan- ( 200r.

sJi) =

5-v2 - 3

1.55 radians.

By the .\' -. y-. and z-components (v,.~.. Vy, v:) of a vector v. we mean that if we start :ll a
point P (Figure 11 .52) and proce.ed v.t units in Lhe .x direction, u) units in the y-direction, ond
v, units in the : -direction to a point Q, 1hen v is I he dircclcd line segmcm joining P and Q. To

1!l!L1
"'"'"'---.[W""'

M4Cijil;ljUA1
CCI)rne-1ric illus.h-,li(l)
\}( <:\unponcnti of a \ 'eCtor

Unit

\'tct()I'S

y
X

phrosc this another way. we introduce th.ree special ve<:1ors parollel 1o the cOO<dinme axes. We
define i as a unit vcetor in the positive xOircction. j as a unit ,,.ector in the po:sitivc y-dircc.tion.
and k :ls ~\ unit vecror in the positive z-dircction. \ Ve ha\'e shown these vectors wilh their tails
a1 the origin in Figure 11.53. and il is clear thai their components a"'

i=( l, O, O),

J =(O, l, O),

k =(O,O,I).

I 11. 151

But no1e. 1hen. thai we can wri1e the vecwr v = ( Vr . v1 u,) in the form

,. = (v,, 0. 0)

+ (0, vy. 0) + (0, 0, v,)

= v,(I. O.O)

+ vy(O, 1,0) + v, (O,O.I)

Jn other words, every vector in space can be written as a linear combination of the three vectors i,

j, and k (i.e., as a constant times i plus a constant times} plus a cQnst.ant Limes k). Furthermore,
the constanlS multiplying i,

1. and k are the Canesian components of the vector. This result is

l tJ

\'t\.'tOfl>

c.<1ually cl ellr gcornetrically. In Fisurc 11.52. we have >hown the vector v from P tO
define points A ond 8 as shown in Figure I 1.54. then
v = PQ = P ll

+ BQ

= PA

717

Q. If we

+ AB + B Q .

Bul because PA is a vec1or in lhc I>OSiti,e x -c.lh-cction nnd has lengt h u.,.. il fOllows thai PA =
= v>l and OQ = v)<. "nd thencl'orc
.

v.ci.

Similarly, AD

v = v,i + v>j + v,k.

(1 1. 16)

To say then t hat u,,. llyo and v, arc the x . y , and :-components of a vector v is tOsay that v
\:all be wriucn in form 11.1 6.
Some authors refer to Vx. Vy, and 11: as the se.\lar components of the vector v. and the
vectors v.~.i. u>.j, a nd v~ k. as the \'eetor components of v. By compouem. we ah\'ll)'S mc~m
St(t/(l f CQIIIJ)(}IItlft,

Vectors in che ..ry-plane have only an.\' am.l a y-comp()nent. and ea1l rherefore be written
in term:. ofi ~lnd j. I f ,. = (v,,. uy). then we write cqui, alcntl y lhilt v = uJ + v,j (Figure
11.55). Vcccors along the x-nxis have on I> ill'l .\'COmponent nn<.l cian the refore be" riucn in the

form v = v,ri.
We use chis new notarion in che follow ing example.
~

spox.-c .:i!n be ex-prtsscxl u

cc:unhil~lfion (){ i. j , ~od

Vecl~in
11

in
as

Vt<'IM

abt- .rypiJnc CUll be t'<P'tSSQJ

linti!.l'

a I in~111 c(wnbi,lalion uf

i wkl j

,r

I EXAMPLE

11.8

lllC force F cxencd on a point chalb'C


Coulomb's law as

F
Ln[i)"!'Ji] .mC!II Elea n~t~ic
f\)r<.'e of chall.~ if: on ch~e q 1
Qr

~
------r

q,

coulombs by a charge (/l coulombs is defined by

= q,q\ i N,
4 n f or-

where E"o i!-: a pf.l.;.icive conscHnt, r i.;. the d istan ce i n met.re.~ between the charges, and ;. i~ a unit
vector in the direction from (/2 to q , (Figure I I .56). When q, mld C/2 arc both positive chargc.s
or both negative charges. then F is repul sive. and when one is positive and the other is ncgmivc.
F is altrdttive. In particular, supJ>O"O thai charges of 2 C and - 2 Care placed a1 (0, 0, 0) and
(3. 0, 0). rcsl>ectivcl y. and a third charge or I Cis pl;lccd 11~ (I, I, I) . Accordi ng tOCoulomb's
l<~w, the 2 C charge will exert a repulsive force on the I C charge, and the -2 C charge will exert
an attractive force on the I C charge. Fi nd the rcsuluuH of these two forces on the I C charge.

SOLUTION If F 1 is the fol'ce exerted on th<' I C charge by the -2 C charge (Figul'e 11.57).
then

(I)( -2) ,
r,
4Juor l

= l <-2) 1 + fl +

where the distance I>L't\\'CC!n the charge~ is r


( 3.0,0) l\>() , 1, I ) is ( -2. 1, t),andthcrcfo

,.,, r-c t"barge 111 ( f , I, I ) duo 1$)


2C 4;h..usc a1 (0. o. 0) .1t1d

- ZC ~:b~ll.t' .u (J, 0. 0)

i = ,/6 ( -21 + j

<
(1. I. I )

1::

= ../6. The v<!ctor From

+ k).
A

Consequently.

F, =

-2
I
...
. /76( - 21 + j
4Jrfo(6) vo

Simil~~rly. t h e fhrce f'2 exerted on lhe I C

r, =

( 1}(2}

k)

'

-i:---(21 -

12,/6 lr fo

J - k ),

d uugc hy thc c.:hargc m 1hcurigin is

4JUu(3) . ./3(1 +

i+

k)

I
...
...
~
r.;
(l + j + k) .
6v >X Eo

The rcsultanl oftltcsc forces is

I
"' = F , + F2 = 12 von
(( 2 + 2 vr.
2)i + (2 v "'
2 - l)j + (2v2
tu
17

l )k l N .
A

Fl

Sup1>0se nutnber or lbrces F 1 F'2, . ... F, act on the mass M in Figure 11.58. n,e resullant
of these forces is E' = l''t + }1'2 T + F,. h is a principle of statics that "the ma.~s will remain
motionless i I' the sum of all rorces on it is zero:' rn such cir(:umstances. the m:ts:s is said to be
i 11 equilibl'irmtuntlt r rlt~ acrlou of rite .forces.
F = F,

+ Fl + + l' =

0.

( 11. 17a)

This is a vector cqua1ion. II is cquivalcmto 1hrcc scalar equations obluincd by invoking 1hc
principle 1ha1 vectors are equal if and only if l.heir componcn1s are equal; 1ha1 is. if I' =
F, i + F,j
. + F.k, Jheequivalenl 10 11. 17a is

F.~: =

0,

Fy = 0,

The following examples use !his ptinciple 10 advumage.

(I 1.1 7b)

11.3

I EX AMPLE

VeC"lOfi

719

11.9
1\vo cables A B and A C are tied together at A and auached to a \'ertical wall at B and C a.~
shown in Figure ll.59a. Detemtine the range of value~ of the magnitude of the Ioree P for
which both cables remain taut when a mass of 100 kg hangs at A. Force P acts o nly ir> the
dircc.Lion shown.

SOU J"n OI'> Suppose we establish the t'OOI'dinate S)'Stem in Figure 11.59b. For equilibrium
when both cables are taut, the x -components of all forces acting at A must sum to zero. as must
t he )'Components. If magnitudes of the tensions in AC and
then

U41CJIJ.IJ51MfW

For~."e c:~cnc:d

on IW\)

BC are de noted by T.-.c and Toe .

c.lblc~

anached IO a wal l hy P and hM:iinJ,! rna~

8
y

tan Q=

l4

t on

I'

tOtHg

*'"-

'

IOOkg

Since tan9 = 3/4,it follows thatcos8 = 4/ 5andsin8


and sin = 3/./10, and therefore
4P

0 = - -

TAll

+Tc+ -

-JJO'

"

The minimum value of P can be detem1ined

0=

= 3/5. Furthermore. cos= 1/M


3P

3TAn

- + -5
JTO

- IOOg .

by decrea<ing it to the poilll when T,..c becomes

zero:
4P

TAH

0=--+5
JTO'

0=

3P

3TAH

./10

- + --

-IOOg.

By elimi11ati11g 'l',n and solving for P we obtain P = I OOg/3 N . By setting TAn = 0, we


obtain the maxlmun1 value of P.

4P
0 = -5

The second of these gi,es P

+ TAC

3P
0 =- 5

1 00~.

= 500g / 3 N . Consequen1ly, both cbles rerru1in llmt for I OOg/3 <

p < 500g/3.

...-...
I EXAMPLE

11 . 10

A )l)Jiicatio n

Preview

Find the maxiJnum mass that can be supported by the boom in the A pplication Preview (shown

Revisited

again in Figure 11.60a). Assume that Lhe mass of the boom is negligible in compariwn to Lhe

mass.

SOL UTION We use that fact that the s um F of all forces ac ting on the encl A of the boom
must be ~ero . Titcrc is t he weight W (Figure 11.60b), tensions T AB and T AC , Olld a reaction
R by the boom iiSCif. They arc R
(0. R. 0). W
(0, 0, - M g). a nd

T ,,Q

5.8)
AB ) = T,.n(4.8. - \1.6,
= TAR ( IABI
1
2
J4.8 + 9.6 + 5.82

T M

AC) =
= r,..c ( IACI

TAr(-1.2 . -9.6, 5)
J7.2l

+ 9.6-l +

5l

r,R (2.4, - 4.8. 2.9)

6. 1

r.c< - 1.2. - 9.6, 5)

13

Hence.

F =(O. R.O)+ (O. O, - Mg)+


MJiclil :l

TA/1 (2.4, - 4 .8, 2.9)

.
61

TAc( - 7.2. - 9.6, 5)

13

.'
C(-1.2. 0. S!

9151)1

w ~-Ms

\ Vhcn we equate <:OnlJX>ncnts of this vector to zero.

2.47;.,,

7.27;.c

6.1

13

---.,..-'=

= R-

4.8TAH
6. 1

2 .97~'" sr,..c
\ '
0 =- Jrg+
+--.

9.6T,.c
13

6.1

13

The first and third of these can be solved fo r TA11 = (3/2 )Mg and 1',tc = (1 30/92) Mg.
Since the tension in cable A 8 is l<,rger th;ult hat i n AC, we require
40 000
M < - - : : : . 1359.

lMg < 20000

3g

Mnximummass that con be SUJ>JX>ttc.d by the bomn is 1359 kg.

EXERCISES 11 .3

= li + j :tnd v = - i + Jj, find cornponcn1s c.lr the \'tctor in

If u = ( I, 3.6) . v = (-2, 0. 4) , llnd w = (4,3, - 2), fi nd<ompo-

If u

ncnts for the vocto.r in Exercises 1-10.

E:~crc i~

I. 3u - 2v

2. 2w +3Y

3. "' - 3-u - 3v
5. 2\\t- 3''

4.

7. ( 15 -

2lwl)(u + v)

9. l2u + 3v - wi\\

11-14 i!nd illutlralc the: \'t:Cior gconK:trieally.

12. u - v

14.

lvlv - 2fVIw
8. l3ulv -l-2vlu

v+ ,;

6.

10.

\' -

\\'

l''+ wl

11'1 Exc.rcisc.s 15-24 find lhc Cartcs:ittn t.'Oillpon<: niS rorthc .spatin.l VCCIOr

described. In each case. dritw 1he vcc1or.


15. Fmm ( 1,3, 2) to ( - 1, 4, 5)

11.3 Vectt)rS

16. With length 5 in the positive x-dircclion

and (2, - I, -2).

17. Wi th lengLh 2 in the negative z-direction

J8. \Vi lh 1a iJ
(1.3, 5)

fll

( I, I , t). Jenglh 3. 11nd p<)inling 10ward lhc pOinl

32. Newton's universal law of gmvitation states tJtat the Coree of at


traction F~ in newtons. exened o n a mass or m kilogr-.uns by a mass of
.M kilograms is

19. With positive y -co mponent. length 1, and paraUel to the line
through (1 , 3, 6) and (-2, 1, 4)

x- and

y-couJpOncnts.lcng tb 10. and

z-

22. H as ilc: taB at the or igin, makes angles or TC /3 and TC / 4 l'adians


wilh the positive x - and y-axes,l'espectively, and has length 5 / 2

23. From ( I, 3. - 2)

_ OmM f

r '

'

where G = 6.67 X 10 - 11 is a constant. r is Lhe distance in mches


between the masses. aJld i' is a unit vector in tJ1e direction from m to M .
If poim masses. each or 5 kg. are situated at (5, I ~ 3) aJld (- I , 2, I ),
wha t is Lhe resultam l"orce o n a ma.c;s of 10 kg at (2 , 2 ~ 2)'1

20. ln tJtesamedjrection as tJte vector from ( 1, 0, -I) to (3, 2, -4)


but only half as long
2J. Wi lh p<)Sil ivc and equal
<:OltlpOnt::nl cquaJ tO 4

721

* 33.

Illustrate geometrically Lhe triangle inequality for vectors

lu + vi

S. lui + lvl. Prove the rcM>It algcbmically.

34. Detennine to the nearest newton tJte lelt.c;ions in tJ1e c.a bles A C and
BC in the figure below.

35. Detennine to the nearest newton tJte lelt.c;ions in tJ1e c.ables A C and
BC in tJ1e figure below.

10 lbC rnidpOinl or I he Unc scg,(tJCtll j oini ng

(2, 4, - 3) !tiKI ( I , 5, 6)

24. Has ils tail al the o rigin, makes equal angles with the posilive
coordi nate axes. has aJJ positive c..-omponents, and has length 2
25. lf P , Q , and R arc lhe points with c..-oordinates (3, 2, - I).
(0, I, 4), and (6, 5, - 2). respectively, find coordinates o f a point S
in order tltat PQ = RS.

16. Prove that vector addition ruld S('.alar nmltiplication have the prop
A

erties in equations 11 .10.


27. Draw aJJ spaliaJ vectors or length I that have e<JUaJ x - and
components and tails at the origin.

.v-

28. D 1aw all Sl)atial ve.ctOI'.S of length 2 LJtal have Lheil' tails at theol'igin
and make an angle or TC / 4 wiLJ1 the r>ositive z -a.xis.

*
*

= 3i

29. If u
+ 2J - 4k and v i + 6J + Sk, fi nd scalars A and
p so Lhat the vector w =
18j - 32k c.an be written in Lhe fo rm
w =AU+ pv.

si -

30. Find a vectot T of length 3 along the Langent li ne tt) the cut ve

x' at d1e poim ( 2, 4) (figm-e below).


)'

* 36.

For tJtc cables in Exercise 35, it is known that the maximum aiJo w
able tension in t-able AC is 600 Nand that in BC is 750 N. Determine
the maximum fo rce IFI that may be applied al C.and the COITesponding
angle 0.

37. A 200-k:g t"l'ate is suppotted by the !'ope-and -pulley an-ange11.1ent


in the figure below. Detenuine the magnitude and di1-ection of the fotce
F d1at should be exe~1ed on the fl'ee end of LJte tope.

Y =x Z
X

* 31.

Use CouJomb'slaw(sceExample l l.8) tofindtJ1efon.-conachargc


of 2 Cal the origin d ue to equal charges of 3 C at tJ1c points ( 1. 1. 2)

722

Chapter II

Vn."tor'!O and Tiu-ee-DimensiMal Ana.lytK; OtOt'l"'tlr}'

(Him: Sec cq~ ati on 7 .43.)

38. A 16 kg. hori:oonoaJ. lrianguJar plalc ABC is supponcd by three


wires in the figure bclc..1w. Dctcnninc the tension in each win;,

D
L
-~

......... l.. .. . .. .. . . .........

Vectors u . v. a nd w arc s~i d tel be linear ly dependent if there c:xi'!.t thn.-e


""alars a. b. :1od , . not all zc1'0, such that o u bv +tw ~ 0. J ( thi~
cqu:.uion can only be ~ati~ticd with o = b = c = 0. the \'CCtors arc
S.'lid to ()c li nc:lt'ly indcpendcnl. I n Execi.;cs 42-45 d etermi ne whether
the vc<.'10f'S ~u'C lim::;-rly dcpcndcm Clt' l i nc:~ul y independent.

39. A container with wcig.h1 360 J\ i~~: suppo11cd by CiJblc~~:. A 8 and

A C in the figttJC below. Knowing that F


60i N. dctcnninc the fon:c
P = pj that must bcaJ>plicd 31 A 10 maintain thcconfigUI-ation. Wh31

43.

arc the tensions in the cables?

= ( I, I. 1). v = (2. I, 3), w = (4, 2. 6)


u =( I , 1.1). v = (2. 1,3), W = ( 1, 6.4)
u = (-1 ,3, -5), v = (2, 4, - 1), w = (3, II , -7)

42. u

44.

45. U = (4,2.6). V = (1.3,-2). W = (7. 1,4)

46. Usc YetLOrs 10 show 1ba1 the line sc.-g.lncnl j oinjng lbe midpOiniS
o f two sidc,s of a tri1mglc is pan tUei iO Ihe lh ird sid e and i1s Je ng1h is
One-hal[ Ihe le ngLb Of I be tbjrd Side ( fi gun: belOw).

40. A 200-kg mass is h ung by means or 1wo cables. A B a nd AC.


whic h wc a u achcd to the top o f a vertical waH (figm c below). Deter
mine the rnagnittldC or a horizontal Coree F perpendicular to the wall
that will hold the weight in place. What arc the tensions in the cables?

47. Use ' 'ectt)t'S 10 show th at the mcdia1L10 of a triangle ( figure be low)
all mee1 in a poinl wilh coordinates
x
(

2m

41. T wo bars. A 8 and 8 C. arc pinned at 8 as welJ as at each of IJ1e


ends A lmd
(figure follows). lnjli:tHy C!ltb b::.r is o r length l. !UKI
p0in1 8 is !tLa d iSh(nce II !1bovc Ihe line A C. 11Je b:\JS !trc identicrtl,
each hitvingcross-scctiona l an:a A and Young's modulus E. A verticrtl
Coree whh magnitude F is a pplied at 8 . Show th at the displacement
y or 8 is related to F by the oquation

+ x 2 + x .l
3

+ )'2 + YJ

'

A (x 1 y 1 ~ 1 )

= -211 (h L

y) (

Jy'-2hy+ L'

- I) .
y

Z1+ ~2 + ZJ)

48. I( n point nu1~5es m, an: located ut pointa (XJ. y,) 1n the X)'J)Iunc,
equ::ttions 7.31 and 7.32 define the ccnlrc o( mu' (X. Jj o(lhc&ystcln.

-80nu

Show 1h:u thi.'se h~o o Ca1:lr equations are rcpn:-ccrucd b) tM one \ector

equ:uion

~\l(;r, Y) -

49. lf u a u 1 i J- u,j and \ 1 v, i + ~,.,j.:t!howth.alc'uy'-octor w


iD tbc .typbnc can be wriuen in the fonn '" l u f- P'" ~-idcd

tNtuJv,

u,v,

L m, r,

5.\. Two win:> AC lltw BC ""'att-'<htd to lht top of pole C 0 tn


the fi~ bc:lov.. The force c.~cru:d b) the: pole Q \crtk..al. or.d lhc

~0.

50. Two iden1ic:al s,prinss ...,,th 4'0C\Uont k OtCJ<Mncd 11 point C 1n the


figure be-low and 1\:i\'C 1hcir ends fi'(cd at ;l and B. In lh1s position the
springs ;tiC untn:tchcd and uncompn:sscd.
(a I If a "'eight W is au.o<hcd w thcio join. nd lowly kllocttd
until the ))MCn\ I~ in (:quilibriUIIl, Ond 4n a.tUJhOO relating
\V. L. S. . and the diJo.luncc ." l nnn C to D .
(b) f i nd on aprmximntH
"'n for W '" ltnn~af )'when .Y i-: \'Cry

s:rn.all compnred co L.

25-00 N force 3pp1ied :'II C i~ in 111. neg:11ive Xdn.."'Ciion. llc:lcmunc


the l.cnsions io the wires :tnd the \'Crtical force exenocl by the pole.
... 54. In Ox: left figure below~ the X)'plunc is the: inta(acc b<:I"'CC'n two
n~tt..-.ri::.l~ thutlx>th tl'lu\~ nitlighl . If a l'>~y vf Iight Mri"cs the; t~u f~t.: in
a tJjrcc:tion defined by the unh \'.;(I Of" lJ = (U,., U1 II), then WI1\C
Lhe light h rcnecrcd a long a \'C..: lor V. lJ~ M.unc t' rcfraclc.'tl dlooi (v.
lhc lhrtt "'CCicm U. V. and ~ all he in a phtne lhaau. perpcnd11.'Uiar 10
the xy.plane.

or

(a) If the an&lc of incidence i in 1he ri:;ha Oe,urc ~low h. cqwt


to 1he angle of refteainn . find oomro~a~; for \ '" 1cm1~
of tho or it.
!bl II the angle or rcfr.o<.1itxl 0 ;, rclalcd to l.hc: Unj;k: or in
cidence by n 1 '"'n i
11 1 ~in tl. where 11 1 and n 1 art I he

51. In the figure below, two sprinss (woth <on"""u .1; 1 ond k 1 and
unstrctchcd lcn~ths /)arc H\cd "' poinu A ond 8 . They ""'joined
to a Slcc\C lhal ~!ide:~ &lUll&, lhC: .ldAi~ find the: n:iUilalll fora: of lbc

ndices, o1 refraclon oftht-1\lo.O ma1enatc. tlndccll1't~nl<

!iXlO&.S on lhc ~;;fcc\"C al3n) poena hctwa:n 0 and C.

of W.

52. In the lollowng hgurc, collnrs A and 8 are connected by a 440.


mm wire and $!ide fn:cl) on friction lc.u rods. one: u!ong the: y Ws. a.nd
tt-= other palllllcllo Lhc:: IIAi-. and p.a....in.e, thmuJ:,h the x ati) 2~ 1nm
from the ori.gin. If ;t force F of m;.1anhude 4~0 N i'( applied to collar A . '*'* 55. Vecrors v 1. \'2.... "' are dr.s"-n from Lhe ccn1rc of a reaular
determiM the fOI"CC P rcqum~d to keep collu 8 w. the poG-tion shown.
n.s.dtd polygon in the pJ!Ute to each of 1U \"ertces.. Show th:u the sum
W'hat is the tcn~ion in du: wire'~

()( Lhc:SC

\CctOr$

is the

L.CI'O \'-'(tOC.

l tt.4 Scalar and Vector Products


The Scalar Product of Vectors
In Section I 1.3 we learned how to multiply a vector by a se<llar and how to add and subtract
\'Cctors. The next question naturally is: Can vectors be multipJicd? The answer is yes, and in
fact we c.Jcfinc two prQIJucts for vector.>. one of which yields u ~Jar and the other a vct:tor. The

first is defined algebraically as follows.


DEFINITION 11 .8

The scalar product (dol product or inner producl) of two vectors u and v with Carte> ian
components (u 1 . u,1. tt :) nnd {Vy, u>., U: ) i~ defined a.s
!11.181

If u and v have only x - and _v-componems. Definition I 1.8 reduces to


(11.19)

and if they have only x -components. it becomes


(I 1.201

It is >traightforward to check that

i i= J J = k k = l ,

( I l.~lal

i . ] =j . k = i . k = 0.

(11.11bl

and that the scalar product is conunutati\'C and distribuli ..c:

u . v = ,, . u.
u (Av

I EXAMPLE

+ p w)

= 1-(u v) + p(u w).

(11.22al
(J

1.22bl

11.11

If u = (- 2. I. 3) and v = (3, - 2, - I). evaluate each of the following:


(a) u " ;

(b) 3n (2u - 4v).

SOLU110N
(a) u v = ( - 2}(3}

+ (1)(-

(b) 3 u (2u - 4v)

2) + (3)(- 1) = - II
(- 6 . 3. 9) (-16 . 10, 10)

(-6}(-1 6)

+ (3)( 10) + (9)( 10)

= 2 16

By taking lhe scalar product of a vector "' = (u.r. vy. v:) with iLSelf. we obtain
( 11.23)
Because Definitio n J 1.8 for the scalar produc-t of two vectors u and v is phrased in terms of

the components of u and v, and these components depend on the coordinate system used, it

11....1 S:.ab'MJ VC'I.10r Prodn.:ts

725

fo llows; 1h111t chis dcfinitton also depends o n 1he fiK.'t that we have used C.'lnc,sinn coort.linmc<.
Were \\'C to USC a different Set of c;oordimttCS (~UC'h as polar (.."OQn)inutes). tht n the dcii nition
of u v in tcmlS of <ornpooems in thclt COOC'tlitH\te ~) ~ten'l n1ight be <hffercnt. r or chis reason
we now fintl a gconu.iric definition for the sc.-.ll :lf producl (which i~ therefore irltlcpcndent of
coonlinate sy~lcnlS).

TH EOREM 11 .2

If (\\ O nonzero vectors u and v are placed ta il to rail. and 8 is the angle berween them

(0

~ 1t) . then

u v = lullvlc<><8.

lH!Ll

U \ 1 = lUI \'

li![IJI Pr"ar tbu

l'KOOI

I 1.24 1

ll1e =in< law appl ied to the triangle in Figure 11 .6 1 giws

~osd

IQKI! = PQI' + II'RI' - 2IPQIII'KI cosO


or

lu - I = fvl + Jut - 2fvllul cosO.


Co nsequently.

lu llvlcos9 = - (lul1 + v'1


2

lu - vl1 )

and if (u, .u,, u~) and (v,.v, . uJ are theCanesi>ln rornponems of u and v. then

lull>'! cos9

I
= 2f(u!
+ u;, + u~) + (1; + vJ+ 11~)
- l(u, - v, ) 2 + (u> - v,i + (11: - vJ 2ll

= u . \'.
All imm."XXiate
.
consequence of th ~ result is the fo llowing.

COROLLARY 11. 2 . 1

1\vo nonle!O vectofs u and v are pe!J!CIIdicular if and only if


u . ,,

o.

ill ~51

For cxantple. the '~ctor< u = ( 1. 2. - I) and v = (4. 2. S) nrc perpcndiculnr since


u v = (1)(4 ) + (2)(2) + (- I)(S) = 0.
Expression 11.24 docsn't ju.'>l tell us whcdtcr or notlhc anglo between two vectors is ;r/2
radians: it ~-au be used to dctcnnine the un~lc bct\\<Cll auy lwo nontero \Ceto~ u and v. whell
they ore ploccd tail to tai l. We first sol,c 11.24 for cosO :
<.'Os0

u . \'

= - -.
lulll

(1'.261

Since pn nci11al values of the inverse CO$ille flli1Ctionlic between 0 and :;r. precisely the r,lllgc
ro1 1:}. we can write lhat

( u.v)

8 = Cos- lull vl '

( 11.27)

I EXAMPLE

11.12

find the angle between the 'ector< u


SOl l'TION

= (2, -3, I) <U>d v = (5. 2, 4) .

Accordirlg to fomurla 11.27.

10 6
~ Cos-(~)
~
cos- '1(
+
)
lulbl
J4 + 9 + I )25 + 4 + 16
4

= cos-t(3~) = 1.25 r.adinn>.

I EXAMPLE 11.13
find the angle between the lines x + 2y
u~

CJL M

The a.ljft
hctwccn lincx iu tl1e .\') Pbl'l<
u ~in,~.: ""-"" ulonr. the hnt ..

.\

~x- 3y=~

3 nd 4x - 3y

5 in the X)' plane.

SOI.l'TION Sinccthcslope o Fx+2y


3 L< - l/2,ave<.1orulongthislinei> u
(-2,1 ) .
Simil~rly," vector along 4.T - 3y
S is ,, (3. 4) . If 8 is the angle betwccr1the>e '"''-WI'S.
<Uld iheret<>re hetwterlthc lines (Figure 11.62). lhcn

(-6+4) =Cos- (-2)5

(U\')

9=Cos- --=Co>_, fC../25


lull vi
, 5 25
The acute angle between 1he lines
rr.sult.

i~ tr -

1.75

r.

Sv

= 1 .75 r~diuns.

1.39 nlllians. Forrnula l .(M) gives the Mtme

The Vector Product of Vectors


Jn many applk:alions we need to fi nd a vector perperu.Jicular to l\\'O given vee t o~":'. For ins.tancet
con~itkr

finding a ve('tOr r

and ' ' =

(v., vy.

= (a . b, c) perpendicular to two gi,en vectors u = (u.r. u 1 . u : )

vJ. The coro llary to Theorem I 1.2 requires

a, b , tlnd c to satisfy

+ lm,. + w , ,
= r v = tw, + btr1 + cv,.

0 = r u = (tu,
0

\ Vhc:n we solve the:,c equations. we lind that then: is nn infinite number or solulions all represented by

a = s(u>v, - u,v,.).

b = s(r~,v,- r~,v;).

c = s(u,vy- u,.v,),

where .r i~ any real number. In Oher words. a"y vcc1or of the 10n11
r

= S(lt>V: -

1.1 : Vy U : Vx - Ux V: , 1.1, v 1. - II yll,)

is perpendicular 10 u = (u,., u,., u:) and v = (v,. v,., v:). When we chooses = I. the
rcsuhing \'ector is called the vec10r product of u 11nd v .

DEFINITION 11 .9
The veciOr product (cross producl or ouler product) of two vectors u and v with
Cartesian components (u, . ll y, ll:) and (v. . vy. v, ) is defined as
( 11.28)

Toclimin4ttc the need for memorizing the exact placing of the''~ componcn~ of u and T in
this definition. we borrow the notation for determinants from lincur aiJ:_'Cl'lr.l A brid discu:;.sion
or detcnnin.tncs aod their proper1ies is Si\'Cll in Appendix 8 . We ~et up I 3 X 3 d~temlinant" ith
i.j, and k ucn~~ the top ruw ant.! the component"' of u and ,. tk.T\)'C) the ~cond dJld third rows:

1 k

u,

"'c,,

v,

U::

v,

In actual fact. this is rtol a detcrminanl, since three entries urc vectors atH.I six at'C .scalar$. If
we ignore thi> foci. and fllll'l)' the roles to r expansion o f a 3 x ~ detennulant along its first row
(nmnclv, i time~ lhc 2 x 2 dcacnninnnt obtuined by dcletill}; the '''wuncJ colun111 cor\ta ining i.
minus j timt~ the 2 x 2 dctC'I'T'Ilinam obt..ained by deleting the row and column containing j,
plus k time< the 2 x 2 detenll!nant oht.lincd by deletinsthe ro\\ nnd column conwini ng k). we
obtain
j

= "-'
~J

But the value o( a 2 x 2 determinant is

I; ! I= ad - be.
Consequent! y.

lt r

II )'

It :.

v,

Vy

l':

and this is the san1e as the right side of t<Juation 11.2& We may therefore write. us a mernory-

saving device. that


i

u xv=

"
c,

u, u,

111.:?9)

II:

"'
the general rules for expansion of a dc1em1inant along

so long a:." e evaluate the right s1de u.~ing


its tirst row.
For example. if u =(I. I. 2) mtd v = (2. 3. -5).then

U X \'=

-1

= (S -

-s

6)i - ( -5 - 4),1

+ (~ + 2)k = - i + 9) + 5k.

It is strnightforword to verify thai

=j X j = k X k =

j = k. j

0.

111.30u)

k = i, k X j = j,

111.30b)

and that the crOSb product is anticoonmutativeand d istributi\c:


U X \'

u X (l.v

= - \'

+ p w) =

X U,

l.(u x v)

tll"\lal

+ p(u

x .,),

(I Ulbl

Our prcliminal')' analy/)i~ indicalcd that u x v is pcrpcndiculut to bolh u and v. The


following theOrem l'ellllcs the length of U X V 10 lengths of u nnd V .

T H E O R EM

11 . 3

If() is the angle between two ectors u and ,., then

fu X vf = fulfl sin 0.

(J

1.32)

PROOF Since 0 is an angle between 0 and 11. sin 9 must be positive. and \\e can \\Tile from
equation 11.26 that

sin9=JI-co 2 9 -- \/; -

( u v)' fulfl -

Consequently,

iul1 fvf1 >in1 8 = (u~ + u~. + u~)(u; + u; + v;) - (u~v., + 11>111 + u,v,)2

= u;(v~ + v; + v:) + u!.cv; + v; + v;)


2 2
2 2
2 2
+ u2( v,2 + v,1 - v,2) - ( u,v,
+ tt,.v,.
T u,v,
0

+ 2u,v, uyvy + 2u,u,11, v, + 2uyv1u, v, )


= (u~v~- 2u 1 vru~v0 + ~~~~;) + (u;v~- 211,11,11:11: + u~v~)
2
211.\VA ll ) t l ) - u,.VX
2 2)
+ (li 2.(Vy-

= (u 1v0
=

u,u1 ) 1 + (u,v0

u, v,)1 + (u,v, - u1v,)2

lu X vf2

or

fu X vf = (ulfvl si n9.
l!LJ

TI.c:.ce aJc.
t~ dirciun:> pap:.mlicuW 1\) 1"'"
~hcc1 \'101~. -.me i.n tl1e oppu-.ite
Uiro.."1ion It> the otber

L
t

,,

We now know that u x v i pcrpcndicuiM to u and v. and ha> length fulfv(>inO. where 0
is the ant:le between u and '' Fit:ure 11.63 illu>trtes thm there are only two dircx:tioft~ that are
pcrpendieular ICI u and ' '.and C>ne i~ the neg;uive Clf the C>rher. U l u~ denC>Ie by ,;, Ihe uni l \'eCIOr
along that direction which is perpendicular to u and ~ and is dele-rmined by the righthand rule
(curl the: fingc:.rs of tl1c right hand from u toward \' and the thumb points in dirc:ction ,.).

We now <how that u x v always points in the direction delerrnined by the riglu-hand mle
(i.e.. in direction ,;. rmhcr 1han - io). To sec this. we plliCc u and v 1ail1o 1ail and es1ablish
a ooordinme >Y>Itm with 1his common 1>0irt1 as origin and 1he J>O>ilie .rais alorlg u (Figure
11.64a). U tthe plane dctcrmi~cd by u and '' tile Ihe .ry-planc. In this coordinate systen~. u h'!;S
only an x -componeru. u = llx iCux > 0) . and ,, ha~ only x- anll y-component~ , . = Vxi + v,.J.
The cross product of u and ,. is therefe

uxv= tt,

v,

tly

0 =
0

l(rVyk .

For v in Figure l t.64a. Vy is cleouly positiV"e and Lhcrefore u,~,. vy. 1he oomponen1 of u x v. is
also posilivc. Bullhcn u x vis indcc.."ddclcnnincd by Lhe righLhand rule since '\\ = k.

tJ:

URI! 1 1 . a4b

T1~ ~irt\:tlOI'I of U X v i\ tl.l) dtni~ b)

the

ri&J'1

ux ' ;u, v, k

kc

UX \

PIOUftR! 1 1 . 64c

lwbJ ,.le
~

U,\'yl !

"="'
U /

/"

Jt

.. X ,. H \ "

''

When lhc tip of v lies in the second quodrant of tile .t) plane (Figure 11.6-lb}, u x v is
once again in the ~ithe ~-direction. tile dir-tion of ;,._ When the tip or ,, is in the third or
founh quadrant (Figure II .6-lc), v1 < 0. and u x ,, therefore ha> a ncgmivc :-component. But
in lhb c.ue ,;, = - k. aold the direction or u x I' is once again decermined by che righc-hand
rule.
\Vhm we hac now established is thi' following coorclinute.frcc definition for the vector
product or IWO \'CCtOr< U and \':
u x v = ( u 'JvJ>inO)w.

(I U.1 1

The Unlll'eCIOI' U dCIIO<'S tile direction or U X \' and thl' (:l<:'lor uiJvJ<in 9 is i" ll'ngth.
The rocctho~cthe 1cctor product u x v is pclllCnJicular to both u and v makes ita powerful
tool in mall)' opplication,, We >111111 .ce ~me of them in Section' II.S and 11.6.

I EXAMPLE 1 1 .14
l'ind the cross product of the ectors u =
b indeed perpendicular to u ami v,
Cm<<prodact
N tv. \\ \'f\.~0(( "" ptr~hcwliW ro bocb

'

SOLL"110~

Uoin~t

i + 2j a11d ,. =

3i - 2j + k, nnd show that the result

formula 11.29.
j
2

- 2

li = 2i - j - 8k.
?

We hac shown the ectors in Figure 11.65. We can cheek that u x ,, is perpendicular co u and
I'

U1ing II 25
(U X 1') II = (2i-

(u x v) I'= (21 -

jj-

8k) (I -r 2j)

Sk) (3i - 2j

+ k)

+ (-1}(2)- U,
= 2(3) + (-1)(-2) + (-S)( I) = 0.

2(1)

EXERCISES 11.4

lfu =

zi -

3j + k. v =i- k .anu ,. = 6i -

zi + Jk .c\'lduatc thc

find dinxtion aJlJic.) (or the vectors in l!xcrd~~ 37-40 .

scalar or rind the components o( lhc \'C\:tor in Excn:kcs 1-10 .

37. (1.2. -J)


I. U Y

2. (Y w)u

3. (2u - 3v) w

... 2i . u

s. l2uJv w

6. (3u - 4W) (2i + J u - 2v}


(105u + l40v) (I05u + 240v)
8

l105u + 240vP

7.

W \~

39. (- 1. - 2. 6)

+ ,. w -

II). u v

(u

V X \1'

13. o (v x w )
15. ((3u) x w)

+ (u

x Y)

+ Jl. Verify Lhe rcsuhs in equations 11.31.


~

11. (- Ju) X (2Y)


14. fi X \f
16. u x (Jv - w)

J l . Show th;&t the cross produ.ct is not associ:nivc:


U X

+ w) "

1ru = (3, 1,4), Y = (-1, 2,0),and w = (-2,-J,S),cvaluatc


the scala, or find Lhe componems or lhe \'e\."1.0t in Exercises 11-20.
J:l .

38. (0. I. -3)


40. (-2. 3. 4)

.0.

(>' X

th~1t

is, in gcncrnl

W) ~ (U X \ ') X W .

("') If u :F 0 . .show th31 if the conditions u ,. e u w ond


u x v = u x W ~n: both .)ati.iftcU. then \' = w.
(b) Sho"' that ir one of the conditions in pari {a) b
lhc rnher i< nOt. then ' ' c;JnnOt be equa110 w.

:14. The scalar u ,, )( w


{tnd w.

i~

~atbfictl

but

calkd the latlor utpll' prOtlutt of u . v.

(o) Find u v x w if u (6. - 1.0). >' ( 1.3. 4), and


w
(-2. -I. 4).

wx u
17.
Ju x vi

= u x ,, . "" .
wl c!\n be interpn.."'ted a..; the volume o f

(b) Pn:we that u . ,, x w

18. (u x w) - (u x v)
19. (U X V) X W

+ (u x

(2u

(c) Show th at lu v x
the p.lrUIIdcpiped with u , v . and

+ ''))

20. o x (v x w)

(dJ Verify that thrtt nontcro ''CCtors u . Y, and w all lie in the
s;~mc pl;~ne if and only if u v x w = 0.

ln Exercises 21 - 24 dctennine ' "'lelher lhC \'ectors are perpendicular.


11. (1. 2), (3, 5)
23. (I. 3,6). ( - 2. I, - 4)

w as <:otcnninal sides in

the figure below.

22. (2. 4). ( - 8. 4)


24. (2. 3, - 6). (- 6. 6, I)

[n Exercises 25-30 find the angle between the v-ectors.


25. (3, 4). (2, -5)
27. (4. 2, 3). (1.5.6)
29. (2. 0, 5). (0. 3. 0)

26. ( I. 6), ( -4, 7)


28. (3.1. -I) . {- 2.1,4)
30. (1.3. - 2). {- 2. - 6.4)

'

rn Exercises J 1- 33 find componems for rhe \"eCI(I(.

31. Perpendicular to the ve<tocs ( I. 3. 5) and (-2. I. 4)


.~ 2. Perpendicular to 1he y -axis and the \"ector joining rhe point'
(1,4, - 3) and (1.5, 6)
33. Perpendicular to tbc triangle with '-cniccs ( - 1,0. 3) . (5. I. 2).
and ( -6. 2. 4)
J4. Verify lhe results in equations ll.22.
+- 35. U.sc cquali.on.s l l.23 to pro'-c
1

lu + Yl + lu- vJ = 2l ul + 21 1

This is of1en called lhe parollelogmm lm,. Why?

In Exen.."'i~ ~6 pnwe 1he identily.

+ 45. (u x v) (w x r )

= (u w)(v r ) -

(u r )(Y w)

46. u x (' ' x w) = (u . w)v - (u . v)w

sinA
... 47. U*\"CClOfSIOpr'(.J\'Cthesint:le5w - -

slnB
sinC
= -= --

ror lhe
a
b
c
triangle in tile figUie below. flint: Note that PQ + Q R + RP
0.

Cros:s this equ:uio..n wilh PQ :.nd take lengths.

= (v.u tJ,., v:) ~the vec"lors i:


j. :lncl k ~)f'C called dirt'Ciion fmgle."' C't , fJ , arn1 y of , .. ShOw lh:H

36. The angles between a vectot' v

y = Cos- '

M
("")

= Cos- '

; .
("')
11

)'

48. Show thaothe vector (I>' Ill + luiv) / llulv + l luI is a unit vector
thal bisects lhc angle belwccn u and v .

J 1.5 Planes an~ Llrle$

731

111.5 Planes and Lines


Planes
A plane in space can be characterized in various ways: by t\\'O imcrsectill lines. by a line and a
poim not on the line, or by three noncollinear points. For our present purposes. we use the fact
that given a PQilll P (.ro. Yo. zo) and a vector (A, B. C) (Figure 11 .66). there is one and onl)'
one plane through P that i.s perpendicular to {A , B, C) .
ljhlil;l1
l

[I]

A pla.nc is chorocrcrizcd by n poinl in it ond a ,ector pcrpeodicubt 10 it

(A. B. C)

)'

To find the equation of this plane we note that if Q (x, y, z) is any other point in the plane.
then voctor PQ = (x - x0 , y - y0 z - z0) lies in the plane. But PQ must then be perpendicular
to (A. 8. C ) ; hence. by the corol lary to Theorem I I .2.

(A. 8 . C) (x- .To . y -

YO~-

Zo) = 0.

Because this equation mus1 be satisfic<l by every point (.~, )', ~) in the plane (and at the
same time is not satisfied by any poim no1 in the plane), it must be the equa1ion of the plane
through P and perpendicular to (A, B. C ) . If we expand the scalar product. we obtain the
equation of the plane in U1e form

A(., - xo)

+ B(y -

)'o)

+ C(z -

~o)

= 0.

This result is worth stati ng a> a theorem.

THEOREM 11 . 4

The equatiom for the plane tho-ough the poilll (xo, Yo Zo) perpendicular to the >'IX:tor
(A. B. C) i,
(I .J.I)
A(x - xo) + B(y - )'o) + C'(~ - Zo) = 0.

Equation 11.34 can also be wriucn in the fonn

Ax + By+ C z + D = 0.

(1 1.35)

where 0 = - (Ax0 + By 0 + C z0 ), and this equaoion is said to be linftlr in x. y. and z. We


have shown then that every plane has a linear cquaLion, and tl1c oocfficic.nts A. B, and C of x,
y. and z i.n ohe equatior1 are the components of a vector (A, B. C) that is pcrpeodicular to the
plane. lnsteadofsa)'iog that (A , 8 , C) is perpendicular to the plane Ax+ By + C z + 0 = 0.
we often say that (A, B, C) is normal to the plane or that (A, B , C) is a normal vector to
the plane.

I EXAMPLE

11.1 5
Find an equation for the plane through the point (4, - 3, 5) aod normal to the vector ( 4, - 8, 3) .
SOLUTION

According to I I .34 , the equation of the p lane is

4(x - 4) - 8(y

I EXAMPLE

+ 3) + 3(z -

5)

= 0

or

4x - 8y

+ 3z =

55.

11.1 6
Dete.m1ine whether 01e planes x

+ 2y -

4z = I 0 aod 2:r

+ 4 )' -

8z

= I I are parallel.

SOI.UTION Norma l vectors to these plmJes are :-< 1 = (I , 2, -4} and N 2 = (2, 4, -8).
Since No = 2No, the normal vectors are in the same d irection. and therefore the planes arc
parallel.
~

Lines
In Section I 1.2 we indicated that space c urves can be described by two simultaneous equations
in x, y, and !. , and thal such a rcprc.sc.ntation describes the curve as the imcrscction of two
surfaces. A straight ijne results when the surfaces are pl.a nes. We shall discuss this fu rther, but
we prcrcr 10 begin our discussion of straight Jines with \'ectors much as we did for plaues. A
straight line in space is characterized by a point on it and a vector parnJJe.) to it. 11tere is one
and only one line through the point P (x0 , y0 , zo) and in the direction ' ' = (v,. Vy. v,) i n
Figure 11.67.
If Q (x, )', l) is any point on this line, the.n the vecto r r joining the o rigin to Q has
components r = (A' , y, z:) . Now this vector can be expressed <LS the sum of r o, the vector from
0 to P , and PQ:

.Jiii!lfJIIJiUIIfM A line iii


dW<Iclcriud by a point Wl it and
a \'ee' IOI' along il

r0

+ PQ .

But PQ is in the same d irectio n as v ; the re fore. it must be some scalar multiple o f v (i.e..
J)Q = '''). Conseque.ntly. the vector joining 0 to any point on f can be written in the form
r

= r0 + f l',

(I

1.36)

for an appropriate ,alue of I . Because the com ponenL~ of r = (x. )'. z) are also the c.:oort.Jjuates
of the point (.r, y, z) on e, this equation is c"lled the vector equ ation of the line ( th.rough
(.ro. yo. zo) in direct ion v. If we substitute components for r . ro. and v. then
X

(x,

y, z) = (xo. )'o. Zo} + t(vx , Vy, t>,) = (xo + t v,, Yo+ IVy , Zu + tv:).

Since cwo vectors are equal if and only if corre.o;pondingcomponents are identical. we can wri1e
that
(l l ..l7a)

)' = Yo+ uyt'


Z = Zo

V: l.

(11.37b l
(1 1.37c)

'lllese three scalar equations are equivalent to vector equation 11.36; they :ue called parametric
equations ror line e. They i1Jus1rate once ag:ai_n lhat a lioe in space is characteri:ted by a point
(x0 , y0 , z0 ) o n it and a vector ( vx, vy. v, ) a long it. Each value of 1 substituted into 11.37 yie lds
a point (x' I z) on
and conversely, every point on is represented by some value of/ . For
instance, t = 0 y ields P , and 1 = I g ives the point a t the tip of ' ' in l"igure 11.67.

e.

I I.S

Plane~ :!Ad

Lines

733

1r none of v,, vy. and v: is equal to zero, we can solve equations 11.37 for 1 and equate
chc three cxpl'cssions co obtain

x - .ro

y - )o

: - Zo

(11.38)

These are called symmetric equ~lions for the line t through (x0, )o. : 0) parallel to v (Ux, Vy, v,). n terc arc o nly two independent equations in 11.38, wh.ich therefore substantiate;;
our previous resull that cunc (in this ca:.e. a line) can be described by two equation; in x. y.
and z. \Ve could, for im:tance, write
v. ()' - Yo) = v,.(x - xo)

or

= v1 x0 -

v.\.x - v.t y

a.nd

vz(Y - Yo) = vy(Z - ~o)

v~..:

v.~}'o,

U::Y

= v>::0 -

u~y0

Since the first of these is linear in x and )' and the second is linear in and z. each describes a
plane. The line has been described as the curve of intersection o f two planes.

I EXAMPLE

11 . 17

Find, if possible 1 vector, parametric, and symmetric equations for the line through the points
(-1. 2, I) and (3, - 2. 1).
SOLUTIO N A vector along the line is (3, -2, I) - (-I, 2. I )
(I, - I, 0). A vector equation forthe line is
r

(4, -4 , 0), and so too is

= (-1 ,2. 1)+ 1( 1.- 1.0) = (1- 1,-1 +2.1).

Parametric equations are therefore


X

I -

I.

)'

- I

+ 2.

I.

Because the z-comporlent of every vector along the line is 1.ero, we cann01 wrire ful l .symmetric
equations for t he line. By eliminating 1 bctwecin the x and yrcqua1ions. however~ we can wri te

y-2

+ I = - -,
-I

If we set x + I = 2- Y~: = I . or x + y = I, .;: =


or the pla nes x + y = 1 and z = I (Figure 11 .68).

t.

I.

J, we represent the line as the intersection

l.ir.c: Jq:ttsc:ntcd as intc.ncction of l\lio places

FIGURE 11.68

.r+y= l

vr

(3. 2. I)

/' /
X

,-------!

(-1. 2. I)

"

7'" '

I EXAMPLE 11 . 18
Find the e<~umion of the p~ulC containing the origin and the line 2x + y- z

= ~. x +: =

5.

SOl uno~ We can easily firt<ltwo more poims on the plane. Rl< instance. if we SCI X = 0.
then the equmions or the line rc<1uire:
5 arrd y
9: tuld if we SCI ~ = 0. lhe11 x
5 and
>' = -6. T hu< P(O. 9. 5) and Q (S. - 6, 0) . as well il$ 0 (0. 0. 0). arc poin1> 011thc plano. h
follows diClr that 01' = (0. 9. 5) >11rd OQ = (5, - 6. 0) arc vector.- in dlC plane. and a \CC!or
mmnal to the pJanc is

01'

OQ

"I=

0i
9j 5
5 -6 0

(30 . 25. -45).

111C voctOC' (6 . 5. -9) is also nom,.oltu the plane. and the equation of the plane is

I EXAMPLE

(6, 5, -9) (.< - 0. y - 0. : - 0)

6x

+ 5y -

9:.

11.19

Find ..,ymmetric equal.ions for the line


A WC(Of

luni- IJIC he\)( i"~f").C(ttCn I)(


IWO pltf'l(.\> ("II) be ObU r<'\1 tty
\h"''''$ aormal
pl.lne

\Wfot\

to lhC'

x + y - 2t = 6. 2t - 3y + 4z

= 10.

SOLU"IION 1b Iindo) mmetric e<luaticxl<, we roquircn vector parol lei 10 !he Iincand a point un
it. llyscui rrg.~ =O andsolvingy-2: =6.-3y-4: = IO. wcobtainy = -22.: = -1 4.
Conscquerrtl)'. (0. -22, -I~) i< a poim on tllC lirlC. ro ~ rld a vector along the lirlC, we could
0 011<1 sol"ing x + )' = 6. 2x - 3y 10 for
tirklanothcr point on tliC lirl<:, " 'Y by setting:
x 28/5. y = 2/5. A ve<:tor along the line is therefore (28/5. 2/5. 0) - (0. -22. - I J )
(28/5. 112/5. IJ ), un<l sv io (5/ 14)(28/ S. 11 2/5. 14) = (2. S. 5).
Altcrnati,-cly. we know thut (I, I. -2) un<l (2. -3. J ) arc vector that are nornull to the
6 and 2x - 3y + J~
10. 1Ukl u vector aloug the line of inter>'<.'Ction
plane&x + ) - 2~
of !he planes ntu>t b.: perp<:n<licul~r to both of these \'CCtOC'S (Figure 11.69). Consequent I).
VCCIOf along lhe line or intersection is

-2

2 -3

= (- 2, -s, - 5).

Symmetric equations for the line are th<rcfore


X

..

)'

+ 22
8

:+

IJ

=-s

EXERCISES 1 1 6

fn l:!xcrciM;S 1-10 li rld the C\lu:uion fur lh~ plane.

6. Cvntoining the line> (x- 1)/6


1)/3 (.r 2)/4 = '-t S

1. Through the 11<1inc ( I, - I, 3) ond nonnot 10 the \\lOtor (4. 3. -2)

7. Contaimng the lrn<: .1

2. Throuth the PQint (2. I. 5) und noniiJ)tQthc ''ector joininz (2. I, 5)


ond (4, 2, 3)

point ( 1, - 2. 4)

='

S. Cllfllllining the lines X

z= 6I- t

> 1 2z oo 4, ~X

r y 1 3~ 6 ond the

+ 2y + J z = 21. x- y + 6:

x"' 2+ 3r.,r= 4. z"' - 3+5r

J. Cunraining lhc poirus (I. 3, 2) . (-2, 0, -2) . ( I, 4, J)

4. Concaining chc point (2. - 4, 3) trrocl rhc line (.v - 1)/3 = ()'
5)/4
+2

S. Conwinrn& the In,.,. x

= y{8 = ( Z + 2)(2 ond (x +

= 2y = (l + 1)/4 oncl X = 1. )' = 21.

9. Contuining lhr li1'K'S 3.r + .ay


3z = 19. 3. - 2y - 9t =-58

= 13 an~

= - 6. x + lr + t = 2 and 2) +

10. Containing ohclinc x + y- 4z

= 6. 2x

+ 3y + Sz =

10 and (a)

perpendicular to the X)'~pla nc. (b) perpendicular to the .\'lpl:l.nc. and

(c) perpendicular 10 ohc yz pl;rnc.

11.6 Geonl<lri< ;\ pph<ntionl ol S<:abr and Ve.:aor Pro<lll\:ts

I I. The aculc angle belween 1wo inlcrs:ling plan~ is de lined as lhe


aculc angle bc1wcen their normals. Find the acule angle between lbe
planes x - 2y + 4z 6 and 2x + y
+ 4.

735

22. Does 1he line (x - 3)/2 = y - 2 = (z + 1)/ 4 lie in lhe plane


x - y+2z=- l ?

= ;;

* 23. Show thai a vector perpendicular 10 1he line Ax + By + C = 0


in ll;c xypl:mc is (II. B).
24. Show 1hu1 if" plane has nonlet() in1crcep1s a. b. and c on 1hc

In f.<crci~ 12- 21 li>wl \"CCior.t>arumctric.tnd synunctric(ifpos>iblc)


cqualions for I he str"Jighllinc.

y-. and t -nxcs. lhcn its equal ion is .r fa + y fb

y-2

5 = -3- =

Find cqua1ions for 1he four faces of the 1eu-ahcdron in Exercise 17


of Scc1ion II. I.
26. Find equations for 1he nine planes ronningthc sides. bouom, and
roof orthe birrih<>ll!SC in Exercise 14 of Section 11.1.
27. Verify 1ha1 rhe equal ion of lhc plane JXI'S>'"ll lhrough 1he lhrce
poims P, (x1. y, ~ 1 ) . P2(x1 J?. <z). and P; (.t; . J ;.t;) can be writlen in1he form P1 P P1P2 x P 1PJ = 0. \\here /'(x, J. t ) is any
poin1 in lhe plane.

<+~

* 28. The region S.,1 in the xy-planc bounded by Ihe Slr~ighllines x =

- 2

0, .t

18. Through 1he poinl (2. 0, 3) and parallel to 1he line x = 4- r.


y 2, l = 6 - 21
19. Through the point of intcl')cction of the lines

and

.r-1

(b)

y+ 4
3

-- = -- =
6

I. y

= 0, .andy = I is " rectangle wilh unit an:a.

(a) Show 1hm the region in lhe plane )' = l lh31 projects onto
S, 1 is also a rccaunglc, bul wi1h area ./i.

x - l
)' + 4
t -2
--=--=
2
- 3
5

t- 2
4

Show Ihal I he rea ion in the plane X + y - 2t = 0 lh31


projws on1o S,1 is a parallelogram wi1h area /6;2.

(c) Gcncra.lizc 1hc resuhs of pans (a) and (b) 10 show llrar if S

is tho area in a plane Ax+ By + Ct + D = 0 CC f' 0)


that projects on1o S, 1 . then ~e area of S is sec y. where
y is the octile angle berween k and the normallo the gi\-cn

and p;~rollcl1o 1he line joining the pointS (I . 3. -2) and (2. -2, lj
Ul. 2x - y = 5, 3.t + 4y + t = 10
21. Through til<! poim (-2. 3, I) and parallel to the line .t - y =
3. 2x-y+ t - 2

plane.

111.6 Geometric Applications of Scalar and Vector Products


t!'J1J r, m m. Cvmponent)
af n v1or c-an be tOund by drawins
perptnc.!iculm from the tip ,)( the ''lor

10

the

C'QOf!.lin:ltt .l'<:ttt

.r-.

I.

* 25.

12. Through the point (I. -I. 3) and paralic! to til<! \'<!CIOr (2, 4. - 3)
13. ThrO<>gh the poin1 (-I. 3. 6) and pamllello lhe \"CCior (2, - 3. 0)
14. Through the poin1s (2, - 3. 4) and (5, 2, - I)
IS. Through the poinls (- 2, 3, 3) and (- 2. - 3. -3)
16. ThrO<>gh the poi>lls (1. 3, 4) and (1 . 3. 5)
17. Through the poim ( I, -3. 5) and plnllcl to the line
X

+ t/c =

Components of Vectors in Arbitrary Directions


We have defined what is mcam by Canesi~n coonponen1s ( v,. u>. U: ) of a vector v. They are
scal"rs thai mulliply i. j . and k so 1ha1 ,. = v.i + u1 j + v:k. They can be represen1cd in 1enn<
of scalar produca< as
Vx

= V i1

tly

= V j,

V:

= \' k.

(11.39)

Goomeuically. 1hey can be fo und by drawing v. i. j. and k all at the ori~in and droppiu~
perpendiculars from the lip of v 10 the x-. y. and z a.,c~ !Figure 11.70). We now generalize
our definition of a component along an axis 10 defi >te Ihe componen1of a vec1or in an)direc1ion
whatsoever.

DEFINITION 11 . 10
To dcrinc the component of a vector v in a direction u. we place v and u tail to tail at
a poinl P and draw lhe perpendicular from the lip of v to the line conlaining u (Figure

11.7la). Thedire<:lcd disaance P R is called 100 component or v in the diree1ion u . If R


is on the same side of P as 1he lip of u . P R is 1aken as 1>0si1ive; and if R is on 1hat side
of P 01>posi1e 1o rhe Iii) of u. P R is negmive.

In Figure 11.71u the component of v in direction u is positivc. un<J in Figure 11.71 b the
COil1J>OI1Cnt i$ ncg:tli\'C. Note lh!Jllhe length of ll is irrclt,..~uu: il is Ulll) the tlirection or ll th:U

~u
R

Sl ulion
"'l:cre '-on~! (t( v u~ dutioo of
ll i" r~p1\'

determines the comporn)nt of ,. in the dircctiOII u . If 0 is the angle between u and . then

IP Rll = 1.-1cosO.
The right side of this cqlllltion looks very much like the scalar product of v and a ''ector thot
makes an angle 0 "ith v. h tacks onI> the lcr>gth of this second vector. ClcMiy. u is a \'ector
that makes an angle 0 with v . but wecunn01 write Ill' Rll = lllul cosO. since the length of u
need not be. I. lf. ll0\11\!\'er. U is the unit \et:h)( in tht Nlll\C: direc-tion as u . then we can write
( 11.40)

In other words. we ha,c the following theorem.


TH EO REM 11 .5

The component of 3 vector v in a dirtetion u is v U. ''here U is the unit vector in

dirtttiOII U .
This result agrees with equations 11.39 for the x-. )'.and :-oomponents of v.

I EXAMPLE 11.20

f"ind componenL< of v
(n)

= (I , 2. -3) in din:ctionsspecifled by the following vectors:

= (-1.3,4)

(b)

u = ( I. 3. - 2)

(c)

(4, ~. 4)

SOLUTIO'\ Components of " in these directions are

(-1. 3. ")
7
(3 ) Y U = ( 1.2. - 3)
= --

,/26

{b) v u = ( t.2.-3)

(1, 3.-2)
~

, , 14

,/26

13
v 14

(4. 4. ~)
(C) V U = ( I,2, -3)
r. =0
~v 3

TI1i< l.iSt rc;utr means that ,, i< perpendicutu to (4. 4. 4) .

Distances Between Points, Lines, and Planes


There are three distMccs bel ween geometnc ObJCcts in a plane - betwceo> points, fron> n point
tu a line. lind between parallel lines. They were all co,C1C<J in Section 1.3. Distance formulas
1. 10 and 1.16 handle tl><: fir<~ two ituation,, Distance between parallel line.<can he calculnted
by finding n point on one line and usio1g 1.16 for the distance from this point to the 01her line.
ln >pace. there are six distances ofintercst - point to point. point to line. point tOplane. line
to line. Iine 10 plane. and plane to plane. Fom1ula 11.1 gives distance between points. We can
develop a fonnula for the distance from a point ( xo . .l'l :tl to a plane Ax .,. /Jy +C4 + 0 = 0
very similnn o formula 1.16 for distance from n point to a line in the xy -plane. ~ R be the foot
of the perpendicular from 11 point P (.r1 y,. : 1) to the plane in Figure t 1.72. an~ Q(x. y. z)
be any other point in the plane. The lcng01 of vector I'R i> the di>hllll'C from P 10 the t>lane.
and this length is the component of PQ in direction PR . Hence. IPR 1= I'Q PR. A normal
vc.:too to the plane is (A. B. C) . and thercrorc

- = .....,<;,;.A;;.
~8~.C~)~
PR
I
2
2

JA + 8 + c>

.,.,
MJHiil tWFMiM

Thu)l,

\Pit\ = IQ .
\

Si,ce t'<.)

( ,\: -

( A.B.C)
Bl +

J A1 +

ci .

.x,. y - Y :. - : ,) .
\I'R\

\(x-

J A~~ll~;~

<t Y - Y ;- ; ,) '

c!\

\A.< .... 8 )' + C:- (A.< t + B y,+ Ctt) \

=
n ecause

JA'

B1 + C '

Q is '" the plrulC, \\C. can i'Cpbce Ax + Ry + C t \'lith - D. ru'\d the fon\\u\a then
\A.<, + ll ~,

+ Ct , + D\

J A1 + 8 1 +

(\\ All

C~

Notice. as we sug.ge(tl. the $iml\~rhy of thh formu\ l\ \0 \. \{).

( EXAM PLE 11.2.1

...
Find the distancehoon the point (I , 2. 5) tOtl'IC p\onex

SOL\f f\ON

+ )' + 1~ = ~ .

1\ceo<ding to \ \ .4\ . the distance i

I1 + 2 + 2(5)

- 4\

Jt+ l +4

../6'

With diswnce fon11ula 11.41. it is s1raightforwa.rd to tj nd the diswnce between two paraiJel
planes. o r be tween a plane and a line pamiJel 10 !he plane.

I EXAMPLE

11 .22

Show Lhat the line x = 3


I, y = I - 21,
paralle l. and tlnd the d is tance be.t ween Lhem.

z = 4 + 31 and Lhe plane x + 2y + z =

6 are

SOLUTIO\/ A vector along the line is v = ( I. - 2, 3) nnd '""'tO< noron:ll to the phu1c i>
N = (I, 2, I ). Since ,,. N = 0. the vectors >Ire pcrpcnd iculnr. ao>d this conlirms that the line is
parallel to the plane. (We cou ld also cool firm this by substituting from the equations of the line
into the plane, 6 = (3 + t ) + 2( I - 21) + (4 + 31) = 9. ll coourodiction. nte: line and plllnc
do nOt therefore intersect.) To lind the distance between the line ao>d plane. we find the di>tancc
from (3. I. 4). a point on the line. to the plane:
13+2( 1) + 4 -61

.J I + 4 +

= ,/6'

f'OJ' the diStance between pomlllcl planes. choo;;c '' poim on one plane nnd find the db tancc to
the other plane. There is also n formula (>ec F.xcrci<c 31 ). This lea,cs two distance' to line and line to line. We illu tt11tc each by example.

I EXAMPLE

11.23

point

.......
Find the distance from the point (1. 3. 6) to the line

f:

(rUin u PQ!Ill tu a lifl(

.t -

1=

y- 2

:- 4
3

SOIU I 10'1 Clearly. Q( l , 2, 4) is n point 011 the line. 11nd therefore the requored diuance d
is the component of I'Q in the direction PR (Figure I 1.73). Uy cqumion 11.40, then. d =
IPQ. 1>R1. To find PR we need a vwor in the: direction
Since v = ( I. 2. 3) is u w.:tor
along f. the vector

rn.

P( t ..l. 6) t

vxi'Q =

I
0

3
-2

- I

b pcrpcrldicultv to bOth PQ and v . It now foll(l\v that u vector nlong PR is

(v x PQ)

.i

- I

.1

- I
3

si + 2.i - 4k.

Finally, !hen.

PR

and

(8. 2, -

4)

J64 + ~ +

= (0.-1.-2 .

16

(4, I, -2)

(4, 1.-2)1

J2i

J2i
=

J2i
7

Cross products can also be used to find the distance from a point to a line (see Exercises 29 and
30). The above method p:lmllcl~ that for Other c.Jistanees in this section; it uses SC<llar pnxlucts.

I EXAMPLE

1 1 .24

Find the (shoo1est) distance between the lines

e, :

x - 1
2

X=

-I + I.)'= 21.

= 3-21.

11 .6 Geurne11ic Appli<':l!ioos of Sclbr ml \'etior l'ruckls

I ~tclii,IWII'L

S~Oftcst distance between nomntcl'\('('ting lines

SOLt.:llO:\ There will be a point P on t 1 and a poiru Q on 2 such that line scgnreru P Q
is perpendicular 10 f 1 and f 2 (Figure I 1.74). The length of lhis line segmenl is the shoncsr
di~ance between the lines. There is no problem li11ding a vector in the same direction as PQ:
cro"s ,cctors (2. 3. 1) and (1, 2, -2) along e, nnd t 2 rcspccrivcly,

!~
I

~I = (-8.5.1).

2 - 2

Now comes the hard pan to visualize. If we take any point on f 1 say R(l, -.'l. 4), and
an) pornt on t 2 sa)' S{-I. 0. 3). then the comi>Oilelll of vector RS along I'Q is the length
of PQ. (If you are cot cominced. consider the follo\\ing Ul'Olument. lr is easier 10 see th..llthc
component of PS along PQ is the length of l)Q w rhat fi'QI = IPS PQI. Bur PS = PR + RS.
and therefore.

l'Q = I(PR + RSJ PQI = IPR IJQ + RS PQI.


Since I'R i~ perpendicular 10

PQ, it folr..>IV) thllt l' R 1'<} = 0. and


~

IPQI =IRS PQ/.


The right side is the component of RS along PQ.) Consequendy.

PQI = 1<-~ 3, - I ) . (-8.


J905.

1),_- vr.=

10.

Thi~ pro.:edure faih \\-h<!n the lines an~ pamlld, bvt in lhi~ ea."'-'. \\e C'al1 picl: a point oo one

line

and 1111<1 llle dtsunce to the other line a, rn Eumple 11.2J.

Areas of Triangles and Parallelograms


Vector product~ 11nwi~e a simple way to find areal or' tri<tnglcs !llllll>amUclogrdnt<.
THI! O RI!M 11 . 8

If A, 8, ond Core vertices of a triangle, the11


I

:w2of a ABC- ZIAH x ACJ .

(II ~2)

I'R<x n Area of uiangle ABC in f'isurc JI.7S i,


I

21-'CII BDI = 21-'CII ABisi nll

= 'iI iM! x ACf.

739

ljtciii.IA

Area of u ui2ngle in

US

Attil

of a

>

Forthe triangle wit h verlites A( I, I, 1) , 8(2, -3, 2), and C(4, I. 5) in Figure 11.76.
AB X AC

- 4

= (-16. -

I. 12).

0 4

The 11rc<t of the triangle is. therefore.


Art.a of a

~1(- 16, -I, 12)1

~ J256 +I+ 144 = ~J401.


2
2
The following coroll~ry io n din.'Ct consequence of Theorem 11.6.
2

COROLLARY 1 1. 6 .1

noc area of a parallelogram with cotcrminal sides A U and AC (Figurc 11.77) i>
area =

lAB x ACI.

(II 4 1)

EXE RCISES 1 1.6

10. Along the liner = 1- 1, )' = 3 + 21.<:


I. (I. 0) . (~. 2). (2. 6)
2. (- 1,0,3). (5. 1.2). (-6. 2. ~ )
3. ( I, 1. 1). (-3.4,-2). (- 1.-2.3)
4. ( 1. 2,.1). (J,5, 10), ( -~.-4. - lt )

lAF.xcrci~s 11 -28 find &he dlsunce indicated.

II. Froon the poont (3. 2) 10 UIC line 2.v

+4

12. lleh'I.CCn the l i1le<.t'

In E.:<cr..:iSC$ S-6 find lhc an: anl'ltw: J>~r.Li lclogr;nn w1U1given ''crti<:es.

5. (I. 2. 3). (4. 3. i). (- 1.3. 6). (2. 4. 10)


6. (1. - 2.4). (3.5. 7), (4.6.8). (2, - 1,5)
hl

E.xet\:~

7-10 nnd II-.;: con ..f}Oil4.'1tl or V = ( I . - .a., 3) in lhC dircc

tion ~x.c,:ifi~

7. ( I, 2. - 3)
8. In the direction from (- 1, 2, 3) to (4, -3, 2)
9. Perpendicular to the plane x + y + 2< = 4 in the direction of
increasing z

= 4 - 31 in ohc diroction

of illCrcasins x

+ 3y = 18

= )' nnd 3.r + 7 = 3}'

13. From ohc poi no(I. 3. 4) to the plwlC x

+ )" -

2z

=0

14. From1l1c point (-2. 3. -5) lothc plltnc 2t + .r + 4z.:; 6

IS. BciWCCil lhClincx = I - I. y = 2 + 31. Z = 4- 2/dlld O>e


plnnc 2.< + 4y + 5~ = 10
16. Bctwc'Cn OlC line x + y + < = 6. 2.r - .>' + : = 3 wt<.l diC plw\C
2.1 - iJ -< =S
17. f...UMlhc li1K" X

.3y + z = 4
1
_8. From the line .r
1

x+y-2z=

= 3()' + 4) = I - 61. )'

:C -

+ 41 , t

l to the pl:a.,~ 2t -

= - 1 10 the plfnC

19. Sctwccu lhc pJuncs 2x + 3y - t = JS and 4x


20. Between 1hc 1>lanes

x- y

+ 2z = 4 and 3x -

21. Fronlthe. p-oin( (I~ 2 , - 3) (0 the line x = 2(y

* 22.

FrOIIllhe J)Qim (3, - 2 . 0) I.Olhe lincx

+ 6y 3)'

= - 6 - 31 forro a trium:dc and tind its a rea.

+ 6z = 10

+ I) =

= t . ." = 3 -

'l t = 7

In E.:<.crciscs 34- 35 \'Crif)' 1.ha1 ~ anti \Y arc pcrpcndic::ulut. and Lhco

(z - 4 )/2
4+t

21. :t

find scalars). and

23. Pr<nnlhepoint(l,- 1, 2)to thelinex+.r-z= 2,2.t+ 3y+z=4

* J4.

* 24. From theP<Jint ( I, 3. 3) tolhclinex = 2- I .V = I + 2J, z = 3


25. Bctwccutbc tincs(x- l)/2 = (y+3)/ 3 = 4-z:mdx = -l+l ,

p so lhal u = A.i + pW.

u= (2,1) : v= ( l/./'i..l {./'i.). iv= ( l/h. -t / ./'i.)

35. u = 3j - lj : v =

(1 -

2j );J5, \V = (2J + J)/J5

y=ZI. t = 3-2t

* 26.

Be-lweentheliJtes: x

)'=

* 27.

= 1, y = 3t -

I, z

In ExcrciS~..-s 36-37 verify that U, V, and \\-> arc n:mluaUy perpendicul ar~
and then lini.IM:alar!, ; .. p,and J. :so then r
Aii + p V + # U' .

= I +2t and:c = 2t+ I ,

l - l.t=4 + 2t
Re1ween the li n e~ x
(t - l)/3

(y + 1)/ 2

+ y - :z

* 28.

8clwl!cnthc lincsx+ J.'+ Z=


3, y - 4z = 5

= 4 . 2x -

z = 4 a nd x

2.x- .r +z = 3and 2x- .-.+z

36. r = (I, 3. -4); ti = (2, I, 0) / ./5. v = (-l, 2, 3)fjl.i, \V =


(3. -6. 5)/,J70

* .\7.

k; ti = (f + j+ k)/ ,/3,
' = (i- j);.J2

* 29.

Show thatlhc distance fro111 point P( l. 3. 6) to the line x - I (y- 2)/2 = (Z - 4)/3 in Figure t 1.73 can also be round as follows:

IPRI = IPQI sinO = IPQII vl sinO = IPQ x vi.


Us<: the Lcchniquc

(lf

E~crc:: i sc 29 to find the UisLaflcc:. in E.xc.rcisc

J)eCpcndicular. and lind scalars A atld p so that u = Av


sure to recognize that \' a nd w a re not unit vectors.
~ 39. lfu

*
1:

latent! form a par..dldograrn.

= ( 1. 0. 1), v = (J. l.- 1) . w = (- 1. 2. l ). und r =

4(). Find the equation


fTOOl

* 32. Show that the lines joining the midpoims orthe s ides of any quadti

or a plane normal lO i - 2] + Jk a nd 2

units

the poi11t (1, 2, 3).

f[ Q, !J, cmd C (altJ)()Silin: C.:OII:)Iatll:i) an:. lhc inten:Cpt!, of <t pl<t!U.:


wilh lhe X . )'. and zaxes. and pis the length of the perpendicular
from the origin to tbc plane, sbow tha1

41.

33. Showthatthethreelines (x-4)/3 (y-8)/4 = (z+ 7)/(-4):


(x + 5 )/3
(y + 2)/ 2 ~ (z + 1)/2: and x
I, y = 5 + 1.

Be

Pm"e lhal l he (uodirt...>cled) distance belwee.n two parallel pl anes


= 0 and A.t +By+ Cz + D, = 0 is

Ax+ By+ Cz + D,

ID,- D,l
JA' + B' + C'

+ pw .

(-2 . -3, 4),veriry lh at u , v , and w are mutually perpendicular, a1ld


find scalurs A. P: and p.. so that r ~ Au + pY J.LW. As in Exercise.
3-8. lake inLO accoun1 1ha1. u. \'.and w are n()l unil VCCU)(l\.

24.

* 31.

v = (i + j - 2k)/./6,

+ 38. lfu = (3. 2), v =( I, - J), w = (6, 2), veril'ythm v and w ure

where I) is the at~gle between PQ and line f .

* 30.

r = 2i -

-1'2 = -a'l

+ -b 2 + -c l .

111.7 Physical Applications of Scalar and Vector Products


Work
ljltclilphWr.t!M Work
done hy a fort."t along a line

In Sectioll 7.4, we described work as the product of force and distance. Now that we have
represemed forces by \'ectors, we c;1n be more precise. I n panicular, if a panicle m oves along
the line in Figure 11 .78 from P to Q, then vector l'Q represents its d isplacement. lf a constant
force t acts on the panicle during this motion, then the work done by t is defined as

II'

F PQ.

( 11.44)

It is imponant to keep in mind exactly when th is de6nition of work can be used: For
cons tant fo rces acting along straight lines, and by constant F , \Ve mean Lhat F is constant in
both magnitude and direction. Note that when F and PQ are both in the same direction. then

the angle between the. vectos is zel'o. In this case.

IV = IFIIPQI cos(O) = IFIIPQI.

742

Chlpter II

VMors ooo ThrteDill'lensionll Analytic Geometry

and 1his is essentially the equation deal! with in Section 7.4. When F and PQ are not in the
same direction.

II'

= IFII I'Q icos8.

Since IFI cosO is the component of F along PQ , this equation simply states that '~hen f and
PQ are not in the same direction. IPQI should be mulliplied by the component of F in direction
I'Q. The componem of F perpendicular lo the displaccmem does no work.

l EXAMPLE

11.26

If the boy in Figure 11.79 pulls the wagon handle with a force of 10 Nat an angle of ;r / 4 radians
with the horizontal. how much work does he do in walking 20m in a straight li ne?
'>OI.UTIOI'\ The force F e~encd by the boy has magnitude IFI = 10. and poirus in a direction
that makes an angle of rr / 4 radians with the displacemem \'eCtor. If d is the displacement vector,
then

IV = f1 d = lflldl co.~O = 10(20)

cos(:) = :~ =

100i2J.

When motion is along a straight line. but F is not conslanl in direction or magnitude or both.
we rnu~l u.~c integration. Tile following example illustmres suc.h a situation.

l EXAMPLE

11.26

The spring in Figure 11.80 is fixed al A und mO\'CS lhc sleeve frictionlcs<ly along the rod from
8 to C. If the spring is unstretched when !he sleeve is at C. find the work done by the spring.
\\~ done

by o spri ng pulling a sl:ve

alotl~

a lirte

r ./~-I

C'

~x- 1-J.c

1-

lm - - -

SOLUTIO!\' If we >d up a coonlinate >)~nt as shown, then at position D the fon.'C F exerted
by the spri11g on the sleeve h.~s masnitude

where: k is the spring co11stam. Since: the spring is always .su-ctchcd during mOlion. the dirr:.c.tion
o fF is along the vcct01' DA . Clearly. then. F cl1unge; in both magnitude and direction H> the

sleeve moves from 8 to C. For a small displaC'.emem d x at position D . the amount of work
done hy F is (approximmely)

[/

F (dxl) = 1Fidx(cos8) = k ,, (I- x)2


,

k (l - x)

1/2

I -

/(1 -

x) 2

I I]

+- - 4

] d.c
1/4

1-x

dx r,;=====c;;=:=';'T.
/( 1 - x) 2 + 1/4

Total work dCIIle by the ~llnng ~~ tl~ $lt."CVe nove.-: rrom

IV =

1'

"' k

I - .c -

'2
"'k { .< - :..._
2

= - (3 4

lo

mu~t

k(l-x) [1

1'[

2./(1 -

x) + 1/4

1- x
2/(1 - x)2

+ -I J( I 2

.t) 2 -

1/4

the.rcrorc be

d.<

] dx

1 /~ } '
0

../5) J.

Momcncs
Momc n1~

play o fuuJanlCnLal

ro~

in s uuctural

t ~ ginccring.

\Ve CI1COUUcred them in Scctioel

7.7 in the context of centres of mass. but lacking vee tOt'; we could not treat them l'uUy. (There
was n hint of vec1ors when we defined the firs t nlOment of an object as itS n\SSS rnultiplied by
some tlirtcwd disllm~. c.lirc(.'\Cc.l diSt<.lllCCS flOW bci11g, associated with \'e<::tors.) \Vc discuss Hrst
n. onu~nts

or fo'Ccs here .~.uhcl' tlmn tina mon-..:1\l~<> of masse~.

7,7, the force of ~&l'aVity Oil the

llltl.'i;~CS.

Force~

were present in Section

bill by pushi11g VC:CI()f'$ \1ld force.< intO the b..ckg.rounc.l,

we conccnl mtcd on the l)(tlancc ronccpl . Other


npprooch.

~pp lic:utio n~

of n10nu.:nt~ re<tuitc u vectorial

When a force is toproducc!l de.~ ired eHcct. "here to apply the force "'"Y beJU&tos imponant
a" the size of the force. For example. it is easier ~to in~rt St..-rcws using a screwdriver whh a large
hfulCJic thnn ll :,~nail one: a lon,g prybar can be. more cfrecth'e in remo,int; !>pikes than a s hort

one: it i s easiel' to spi l a roulette wltcel by 1"-111 i11g on it:) e4..1ge r\uhc:r than somewhere ck>sc to
iL-: ce,ure. 111e directio.l to a pply a force is also important. \Vhen ~!!pinning a roulette ''heel,
we don't pull uway from the centre of the wheel (or toward the centre): we pull in H llircc.:tion
pc:rpendiculn:r to the line joining the centre of the wheel and the point of application of the

for~ e.

r,, tuming o rc,olving door. till! be;~ direction is I>Crpclldicular to the plane oF the door. These
ideas arc captured in I he following definition.

DEFINITIO N 11 . 11

111C n10rnenoo f" force t', applied 01 a point Q. about a r>Oint P (Figure 11 .8 I) is deli ned

M = r x F.

where r "' PQ.

(11 .4~)

The magnitude of l\'1 is IMI = lr ll F1 sin 8. making it clear that M depends on both the
point of application ( Q) and the dirccoion of F (ansic II rclaoivc to r ). h i< a maximunl when
F is perpendicular 10 r . 111e direction of M is a1erpendicular to both r and F. 1r r ;u1d Fare in
the plane of the p.tgc in Figure 11 .8 1. then M point< out of the paj;e towards the reader.

, _____,. ____.,{
:...._

IJ

Unit'i or momcm arc newtons muiLiplicd hy metre.~ ( N m). the same a') for work. For \\tOrk.
I N m is ca.llcd a j oule. ' Vc do not assocjatc joules with morucm~ moment is a vector, work is
a scalar.

I EXAMPLE

,,,27

.......
2] + Sk N , a pplied at the point (I. 2, -I ) about the

What is the momem of the force F = 3l poim (J, I, 4) . a ll distance-' in metres

SOLt;TION The momc.nt is


:\1(

I EXAMPLE

,1 , 28

i
- 2

- 2

-5 = - 2i + j

+ k Nm.

.......
What is the moment of the force F =
(3, I, 0) . a ll distances in metres'?

J- k N, applied at the point (3, - 2, 3) about the point

SOLUTION 1l1e momem is

.i
M =

-3

0
0

k
3 = 0.
- I

Tt is zero tx::c-ausc F is along the same line as

ll'.

(This is like trying to close a door by pushing

toward 1hc binges.)

........
I EXAMPLE

,1.29

The horizontal rectangular plate in Figure 11.82a is supported by brackets at A and 8 and by
a wire CD, where D is directly above 8 . Tf the tension in the wire is 200 N, determine the
moment about A of the force exerted by the wire on point C.
FIGURE 11 82a

Plate supported by hinges at A IUld 8 and wt:n: C 0

FIGURE 11 82b

lvkment of lens-ioo in wire about A


t
D(O, 0 , 240)

I
I

240 "'"'
8

y
T

SOLUTION
is

C(320, 300, 0)

With Lhe coordinate system in F igure J J .82b, the tension in the wire acLing on

200(-320, - 300. 240)


= 8(-16, -15, 12) .
)32()2 + 3()()2 + 24()2

Since AC

(0.08. 0.3. O).thc required moment;,

i
M

C"(!

or J

1.::::l

focc~

'"1!EII

Momenl

.tbout a line

AC X T

I EXAMPLE , , .30

0
\16

= (28.8. -7.68. 28.8) N

nt.

In discu<sing rigid bodies it is u<eful to define the moment ora force about a line. When a force
F acts at a point Q. its moment about a point P {Figure 11.83) is M = r x F = I'Q x ~. If
t is any line through P . we dctinc the moment ofF about f as the contponcnt of .\<1 along l.
If\ is n unit ,ector along f. then
moment of F about t

pL - -- --Qq

0.3
0.08
-1 28 -1 20

= (PQ x F) . v = I'Q x . v.

I II -l6 )

The parentheses are unnecessary since the expression makes et~>e only if the vector product i.>
performed firSI. This is a .<calar lrit>ll' worluc,.

Find the moment of the force in F.antl>le 11.27 aboutthc line through the 110ints P (3, I, 4) atld
Q(4. - I, 3).

SOLUTION According 10 the calculations in Example 11.27. the moment about P is i\1 =
- 2i +J+ k. The montcnt about the line through P and Q is the component of M nlong P Q.
Since a unit "ector along P Q is PQ/ II'QI. the moment about lhc line h
( -2 . I . I)

( I - 2 - 1)
' ./6

ola

(~

about a line i., im.le-pcn

\kat ul pui nt eltQ)tn on I inc

The moment oft he force F about the line in !Figure 11.83 is indq>Cndcnt of the choice of point
P on Cu.<ed in its detemtination. Tn show this. suprose that R is any other point on t (figure
11.84). Then

moment of F about l = l'Q x F

v= (l'R + RQ) x

v= l'R x F v+ RQ x

v.

Since PR x f is pcrp<ndicular to I'R. and v i> 1>arllel tO I'R. it follow> that PR x f' is
perpendicular to and their scalar product is zero. Consequently.

v.

moment of I' abo<ut t = RQ

v.

and thi' is formula 11.46 with P replaoed by R.


The moment of u force about u line is not unique bc:c.uusc the unit vector\ ulong the line
is dctcnnined only as to sign: that is. if V i~ a unit \'ector alon the line. so is -V . In other

words. moments of forces about lines arc dea.emlincd only as to sign. This could be remedied
by specifying a direction along the line. but this is not nom>ally done. It is agreed that when a
number of momerns are rcquin:d about a line. the same dircct.ion will be 1akcn along the line in

all calculmions.

I EXAMPL E 11 .3 1

Whnl arc lhc mo>mcnL> about the,\ -. y-. and

~-axe>

due 10 the for<'C in &ample 11.29?

SOJ.lno~ Since A is on the .r.axis in Figure 11.82. and the mome nt about A is M =
2S.Si - 7.6 j + 28.8k N-m.che moment about the .r-axis is 28.8 N-m. the .r -romponenc of
~I To find mom..'<~L< about the J- :ond ~-ax~. "" lind the moment due co the tell<ion T about
8 , the origin, <ince '' hes on both ae<. It i~

i\1 11

= nc x T =

Io:n

J.J ~

c2s.s. - 30.72. o)

N . m.

- 128 -120 96

Con<equcnUy. moments about they- and :-.axe;. tltC -30.72 Nm ad 0 Nm. respcccil-ely.
AIO conrlm1ed i< the moment 28.8 N m about the x-axis .

.........
EXERCIS ES 11 .7

In ,.c~i~ 1-5 c::3lculcu" lhl= nliOCnoent or the


point

fon:~

:about the giV'(n

= l I 2j 01 ( I. I. 0) .OO..t (2. I.
i .. Jk (0.0.0) abwt (

.\ F -1. F ~

S. F

5)

V PQ )( 1-'

Jl j + 4k at (I. I. l}.b<lut (2. 2. 2)

=6i ~~ (0. I, 3) abwt (2. 0. 0)

~ f - 2i + J.l - ~li 01 (I. ~.l) abovt(o)lhrliO<:thtoushthepoinu


3. 2) o.nJ (- 2. 4. J). (b) the coordoR.Ilc o.<cs.
2 3t.) 4 - 11.: I t 51

= +

(<J the looc

zt

f =

4l !k 01 (6. - 2, I) >bout lhc lone X -

1ft the

+: -

2.

,..,..,.,.,ncnt< (\(thr ,.,..,..,t ahooc

the ongon duc too fon.-. = (f '., J-1 f~) ac:tongat a p<lont P (r, J. ;).
whru :u-c 1~ rnon1enu. dUC' tn &bout the .r ,\'. and l3Xef.'l

II. Sul)pO\C:th.tta fort"e F Kl~:ill a point Q. and I' ioc.;. pointonal1nc


l. Prot-.: lh-lt ..~ PQ x F " papcndicvbr to (. ll1cn the m<tlCnt o(
F about ( " ' ""'
12. P1'cWc: thtl if a force F r.ch at :a point P. and
P. then the: n1(MilC111 due: kl F .rboul t i~ 1cm.

:, :
1-,

""rtc' q, onJ q, an: plr<d in thr 'l)plonc :ll

16. Whrn the mek<t on the ~8""' b<low pas< <IO<C to th.o '""""""'
i.t ::Uir.let\o'd co tbt 2Stttord b) a gr:l'\1t.ll.on.lil fOra- \\ath

~trotd. 11

tlitemKJ, r is the ,fi,t:Jnc:c from 1hc- rC'Cirr:cc to Ike Centre o( the GSrtf"'Od.
und G is Newlon's gru"it:uional const4nt Dcrenninc 11\c work dlC
rocl must do against this force in Otd;l!r 10 foil()!.\ the slroight-linc
polh '"'"' A to 8 .

y=: - I

9. F :; i + j - k l:l (- I . -1. - 2) alxlullht line x - )


x-l> T3l = 4

=(If,. M M,)

J~, - Y
F1

t'.:

f''<>hOn< (5. S) ""'' (-2. lJ, rl'<"'to>cly A lhonl """"'" dw!lc q,


<ntO\cdalonglhe r -:-~<from .r = I to 1' = - I hnJt~1013lwort
don~ hy the electr01lt:l11C fOil...,. Ol f/ o and f/> on f/)

tO. If \1

F,

l!'

0100otudoGm ol/f r', whe...,mand M orcthrm>e<oflh<-~

7. F :;(oi -sj+ k >t(-l.J.I)aboulthe linc(.>-J)/ 2- 1+1

It

+ F1 j + F, k oct< ot point

14. R<P""t Example 11. 26 if the pnns h"" on un<trctd><d length I


th:tt i< k thon the l<nsth of AC

In l:.xcrct'""' ~ calallatcthr niOfll<nt olthr '""" abotathr tncloe>te<l


lin<(<).

:t

..

= xo - x.

1.3. 0)

IS. 1.\l pc><i"'"

(t.

= Fo i

v-

v,i + v.j + t.k on: the wmponcnu of a


I'(At.l'tO:t) nd
tor olocg ltnc t. Show tho! if Q(.<, . .''o -.>''any point on t.
then the lllOm<nt or .. olx>u< t i.e the ...,... triple product

I. F = 2i + 3j - 4k ~t (I . 3. 2) 3bOUI (-1. 4. 2)
l. F

IJ. SufliXO<c th-lt ~ fon:e F

io~; a line chmus.h

fl
l

Rndc<

<=>---,,

...

17. rr F j, a con<l:lnl (l)n."C, S-hoW tffill thc "Ofk done by ... o n {HI objccl
around any d()otcd polygQn i< ' c.ro.

1HO\ in&

11.7 Physical Applications of Scalar and

18. Two springs with constants k and 2k arc joined together at C and
have theirothcrcnds fixed at points A and 8 in the figure below. When
their joi n is at C. neither spring is t.'Omprcsscd or stretched. lf the join
is puJicd aJong the shaight line CD perpendicular to A 8 , what work
is done?

V~1or

Produces

747

21. Rod A 8 is held in place by c-ord AC in tl1e figure below. If the

tension in the cord is 1500 N. determine the magnjtude of tJ1e moment


about B due to the tension at A .

c
A

I
I
I

,.

I
*

3.6 rn

D
2.4 Ill

2k

8-

L- -

*
J9. A s mall botJI h:~ ng.S from d::IViiS, One Of whkh iS shown in the
figure below. The tension in the rope A 8 AD is 41 0 N. Oelcrmine the
mornenl aboul C of 1bc rcsulcanc LOree or 1he three censions in 1bc rope
exeT1ed on I he davil f.ll A .

22. A preens concrch:: waJJ scc1i0n is 1e1upor.:triJy hdd by two cables


as shown in lhc figure below. H lbt:: lension in Cilblc 8 D is 900 N.
dccemline (a) the moment aboul 0 or the force exer1ed by cable 8 D
at 8, (b) the (tJOmc.nl due tOthis l()I'Cc !lboul the z-axis. and (c) chc line
BC .

*
* 10.

A 200N fon-e is applied to lhe bracket ABC in lhe figure below.


Determine lhe mome nt of tJte force about A .

23. 11te 6-111 boom 0 8 in the figure below has a fixed end 0. A cable
is sttetched from the nee end B to a I)Oinl A in Lite vertical waH of tJ\e
xz-plaoe. JftJ\e tension in tJ\e cable is 1900 N, fi nd the monlent about
0 of tJ\e tension at B.

_..----1 }-

4m

2.4 rn

)"

748

Chal)(er I I

Vectc'l'f: and Three-Dimensiooal AnaJylic O~'lf~tlry

Z4. Por<c< of 700 N. I000 N. :md 1200 N arc applied to the bnlckct
in tbc figure below. The 7()().N fom: is directed toward poiru E. The
IOOO.N fom: i< pontlk:l lo lbc A'Jplanc. :md lbc 12()().N for<c i< in tbc
yzplanc. Wh>t i lbc 101al momcnl of all dm:c forces about A ?

y
30. n1c 575mm vertical rod CD in Ihe llgmc be low is welded to the
mMpoint C of the 1250-ltlm rod A 8 . Dclcnuinc the 111orncnts aboul
A 8 o f (a) Ote 1175-N force F 1 and (b) Ibe 870-N force F, .
'(100. 150. - 50)
X

25. A for<c F oct<ala point A with coordinaaes (a . a . a). Show"''"


the sum or 1hc mamcnls of F abau.t the coordinate axes 1s zem.

26. The re<:lllngul.1r p late in the figure below is hinged 9t A and 8 and
~upportcd by a cable that p~tSsc-s over a frictionJess hnnk 111 E. I( the
tcnl\iOn in chc cable is. 1349 N. determine roomencs nbouc the coordin::uc
axes nf I he rorcc CXCI'Icd by the tltblc (a) a1 C :-lnd (b) 91 D .
)'

0.90 11\

31. 'n1e frame AC D jn the figure be low i~ hinged aL A and D and


supported by a cable that passes through a ring ac 8 and is attached to
hooks al G and H. Knowing that the 1c:n~ion 1n tl1c cable is 1125 N.
dclcrmjnc the moment abom d1agonaJ i \ D of chc force exerted on the
framcby(a)portion BH oflbccoblcond(b) ponioo BG ofllx:cable.

280mm

27. 111C nlOillCIH ai){)UI lhC x ~a.xis or the l'eSullanl fi.)rtC: or the three
t e n.s i t)n~ in the mpe in Exercise 19 e.xerted OllLhe davir Ill A m us.1 nol
exceed 37S Nm in absolulc value. What is the Jargesr aJiowahJc c c n ~ i on
in 1he m pe when a

= 2 m'J

* 28. wruu i< the l::argcu allowable distance a in Exercise 27 when the
tension m lhc mpe i~ 300 N?

29. The forc.-c F inthe(ollo\\ing figun:h:lsmagnirude 125 Nandac.1,in


a dirccrion perpendicular 10 the handle

of the rOf'C'C llboul thc cootdinmc axes.

BC ofrhc:crank. Find momcnl~

J2. Tho fon."CS f 1 ~nd F l in <q>ace h3\C lhe '31c magJJintdc. Prove
lh:u the rnoment of F I !\boul lhe line or :action or f 2 j<t; equal IC) lhc
moment of F1 aboul the line of ac1ion of F 1,

I J1.8 Hanging Cables


Ha11ging cab~~ arc used in many etl,ginecring applteation-s- ~ptns10n bridge& transnUssion
lines, aerial gondolu, and so on. They are Rexible members eapoble ol' "ithstandng tension
force.< bot ll<~ hear force<. The shape of a hangmg cable is dcttnninoo b) the loads 1ha1 it
supports. be they cooc<nlrated loads. distrit>u:ted loods. or a con>binatioo of these.
Suppose the luoo per u11i1 lcngth in the x-din:ction (includinc the weight uf the cable ilo;clf)
of rhe c"ble in Figure I 1.85 l~ w(x) Nlm. It is imponam 10 re.:tiiZ.C thnt thi10 i~ not the load per
unit length as rne.hurc:.tl along the cabh:. For simplicit). we hn'c cho~n u ct.xndinatc sysLem
where the lowc~t JXlil'll of the cable is on the .raxis. 'nm;!c force$ nee Oil that 1,;1n of the cable

between .1'
0 ond an m'bilrury J)()int .\". u horizontnltcnsion T0 au .r 0. u umgentiul tension
T ut X. tux.! the \crtical loud. They are in cquilibriun1 :,.o that we ntny cquntc horizomal emU
''crtical componen~ of their resultant to 1.cro~

= - 7i1 + Tcos8,

0 = T>inll

-1'

u(t)dt .

Ill - - ,

If we eliminate T, we olxain

r o = ~o w(t)dt .
Since tan 9 is the ~lope of the cable at x. \\e can \Hitc

Tolly
- =
dx
Since this equation

i~

1"

w(1)dt.

valid nt ~v~ry point on the cable we may tlifl'erentiutc with respect lO .r ,

tl2 y
tlx 2

w(x )

= ru

i\vo intcgnuinn, 1\f thi differentia l cqtnuion give he <h,tpe 11f the cubic once d>e lo.t~ w(x) is
'peci lietl.

)1X) /

"-----~--i__A(-

To

- - -- - - 1 - - - - - - - - ..
X

Thc<impl~t load i< w(x) = u> , a constant. Uniformly dL<tributcd lood<a long the horizon
tal wouiJ be rcalitcd i>a >U>JXn>ion briJgc where diC weight u l' the c.Jble would be negligible
compan..-d l<!lhal of th~ roadwa). T... o integratiOIIS ur 11.48 give

y(x) =

wx 2

-=r
:z,o

.T

+ Cz.

Thus. unifonnly loaded cables are parabolic. The fact that y'(O) = 0 implies that C, = 0. and
therefore y(x) = wx 1 /(2To) + C 2 The position of the xaxis dctcnnines C,. If the origin is
chosen as the lowest poim ofthecable, then C2 = 0 a nd y(x) = wx 2 /(2To).
When the >uppons A and 8 of a cable have the same elevation. the distance L between
them is c alled 111c spun or I he cable, and 111e \'eJtical d istance It from the supp011s 10 the lowest
point of111c cable is cnllcd the sag (Figure 11.86).

.--- -1.. - 1~1-----. 8


h
I

- -- -- -+- -- - - -- X
\Vc now cun~ider a l<ible carrying a load uniformly distributed along the cable i~el r(rather

thall ill the horilontal direction). Such would IJc 1hc case for a cable hanging under its O\vn
weight. Let w newtons per metre be the constant weight per unil length <\long the curve,
and choose a coordina1e sy~tem wirh 1he y -uis through the lowe~ point on the cable (Figure
I 1.87). Since lcng1h along the cable c.orrc.sponding to an increment cf.r along the x-axis is
,J I + (dy jdx )' dx . the weight of ~>is much cable is w,/ 1 - (dy/dx) 1 dx. The weiglu per

w,f l + (</.v/ dx)l.

unit length in the .rdirection along the cable is therefore


this for the function w(x) in equation 11.48:

M:llfiii.IWIW:fJ

1. -(dy)'
r-

(II .19)

dx

1it

C::hle: with a lood tmlf'cnnly dKtnhtnOO alt'lllS lhe Cilhle

We >ubstitutc

( r. y)

------+-------- X
Th is differential equation defines the ~hape ofthccu.--c. To sol\'c it. we first set v = dy/dx
and dvjd.r

= d 2 y/d.r2
dv
w r-~2
-dx
= -J1
+v
T0

-;:==::::; du
-/1 + lJ2

w
= -T. d.r.
ij

a separated differential equation. Solutions are defiICd implicitly by

-;:==:
' ==;; d "
-/1 + .2

In the integral we substitute

w.r

T0

+D

Since

v is

= utn 0 and d v =sc:c2 0 dO.

= ~
t/.r
To

J-

1
-sec2 8d8 = J sec8d8
sec 0

f
=In

I
--;==;==, dv

Jl

sec8

+ vl

+ tan 81

w.r

= -

To

+D.

= 1n 1J 1

+ v2 - ul.

the slope of the cable and the coordinate system has been chosen so Lhat

u(O) = 0. i t follows that D = 0. Consequently,


UJX

ln (Jl+v1 +v) = -

To

==>

y' (0) =

When we square

Ji"""+V2 = - v + e""' ''. the r<:>.ull is


u= -dy
= jbo/To
___-,__I
2~1A/To

d.r

_I

(<'""fT>_n-'"'
1'' ).
'

Integration now g.he~

0
= 2w
To (e"''"' ... e-""'~') + C = T
roo;h(~)
+C.
w
To

( 11.51))

Thi:. cun.-e i!'- ca1 k."'C.l n caltn:lUJ' Tf we choose IIlii!' .\'axi-. 10 pa~s ahrough 1he minimun'l poim or
the cable (Fisurc 11.88), 1hen C = - Tu/ w and

Mjl 111;1418

C'tblc wilh

>

To [ cosh (wx)
=-;;;
To - I ] .

ori~tn

cbown ut IO'A."Cil

( 11.511

~r.l

Tho imponant propcrtie$ of the ea1enary are deri,ed in the next two erunple.<.

I EXAMPL E 1 1. 32

Verify 1ha1 the length of etllenary 11.50 from .t


= ( Tu/w) >inh (w 1'/ T0).

0 10 any poim will> .tcoordin111e X i<

.t

SOl UTI ON The length of tllC Cdlenury from x

s= fl + (~'Y dt = {

= 0 to an arbitrary .r is
l + sinh

(;Jdr

=f ~, ( ;Jdt ={ ~~inh(;:)[ =~sinh( ~:).


I EXAMP LE

1 1.33

Verify 1ho1 the 1cnsion T >1t any poin1 P(.r. )') on tile ca1e11ary I 1.50 is related to To by the
equatioo T = T0 ..,.. wy.
SOI.l."ffON The ICn>ion at any poim in 1hc c~bl e i given by 1hc first of equations 11.47,

(dxd}')2

I +-

I EXAMPLE

11. 34

A urli(orm cable wi1h mau per tutillens.th S kJV'm i-: ,\U!(pended bcnvt..ocn the two points A and
8 in l::j~ure I l.SCJn. 11le ..:.~>-.'lll i~ 150m a.,u che sag i~ 30 111. Determine lhe lcn~th ot' t.he c::~ble
and chc maximum anU mirtimum tertsiQilS ht it.
<

ll

'l

u.,,~nl

a!J[i

\!:lble

~ystan

wtlfl ti\'eA l>fUO .lllld ~g

,,~ '[_/8

C~on:udiru::1 c

applied tu c-M: I ~

R(1j,J0)

SOLl.i"llON If we adQI)( the coordioul!c <y<tcnl in Figure 11.89b. the cquotion ol' thc catena')
U\ke~ rorm I I.S I . \ Vilh Jk)irH

(15. 30) Orl the curv\!,

3~:

To[ (15w)
To - I]

30 = -;; cosh

If we set z

= cosh c~:v)

I.

= wf To. the cqwuion


/(z}

can be solvcU numerically for

z.

= cosh (75z}- 30z -

For instance. Ne\\IOI1s icerative procedure wilh ::: 1 = 0.0 1

and
cosh (75<.,) - 30z,. - 1
75sinh (75~.) - 30
gi,cs the "l'flr<>Xinttiun< :1 = 0.010 17. :J = 0.010 16. and Zt = 0.010 16. Consequently.
the minimum tCJlsion in the cable is To= w/<. = 5(9.SI )/O.OIO 16 = 4828 N. t\ ooonli ng 10
E'ample 11.32, the length ofth<C(lble is
Zn+l

= ;:, -

= -2Tw0 sinh (75w)


Tn

i\~X'Ording to Example

2
.
smh
[75(0.010 16))
0.0 10 16

= 164.9 m.

I 1.33. the maximum teniOtl, at x = 75. is

T0

+ 30u =

4828

+ 30(5)(9.81) = 6300N .

If a uniform t11ble is pulled very. very tight, il becomes ulmUS\ horitontal, ond it "ltuuld be
1>0ssiblc 10 consider its weight per unit length

w alon1: the curve ss a unifoml weight I>C:r unit

length in 1he x-direc1ion. To show lh,tt thi< is the case, we eXI)(CSS cquatiOtl 11.51 fOe' the
catCI'I.HI) in the forrn

.l' =

~ [ 00~1 (;: ) - I] = ~ I[ (I + ~;Jl + ~;.::

.L . . - ) -

For a very tight cahlc. T0 \\ill he very lai)!C. su that if we retain only the

Maclaurin series.

y"" To (w2~2 )
w 270

fir><~

ll

two terms of lhc

= wx2.
2T0

\Vh:u we arc saying is 1hat as the tension in a catenary is increased. the more closely it can be
approximated by a parabola.

II .8 IIJnging C~blts

A cable, with unifom1 weight per unit length w. is to bc~uspcndcd over a chasm with both
ends of the cable at the same ebation. Design specification for rhe cable requires that
lension must never exceed T,.. We are to determine the muximum allowable horizonral
pan for the cable
SOU., 110:\ We can take the equation for the cable in fomt 11.51,

'~here T0 is the tension at rhe lowe 1 poim in the cable, provided we use the coordinate

S)stem in Figure 11.90. If IJ i.qhe sag and L is the span, then

(wL) -

IJ = - [ eo~h w
2T0

I .

Sptn of hanging cable


)'

Maximum ten;on occur:. at A and according to Example II.JJ is g11-en b)

Since T,. i< specified and w is knoiVn, !his equruion implicitly defines L as a funccion of
0
T , and to font! the ma~imum \aluc of L. we M:l dLfdTo = 0. Implicit dillerentiutiOit
g1\'es

u dL)
(wL) + To~illh (U!L) (-wL
2To2 + 2TodTo .

0 =cosh 2To

2To

and thi< implic< that

dL
dTo
If "C >ct:

=!_
111

[wL
2To

IL L/(2To).llti> equation

z-

coth z

-COlli(~)]
= 0.
2To

become.

=0 =

z lanh z = I.

This equation connot be 50lv~ cxaetly, bu1 when i1 is solv~ numcricolly 1hc r<suh is
~ = 1.200. ll now follows !hat

wL
- =
2To

2.40070

1.200

L =-~

To rcpl:.:e T0 we note 1ha1 T., = To cosh ( ~)


span is
L

2.400T., scch (1.200)

= To cosh 1.200.
=

l.32ST.,

Thus. maximum

153

754

Ch.ap1er II

Vec1ors arld 1lu-ee. DimerlSiMal Arlalytic Geornetl')'

EXERCI SES 1 1. 8

I. A cable with ends at Lhe same elevation has a s1>an of 100m and a
sag of 5 m. II is subjected to a uni fM m horizontal load of I 000 N/m.
Determine Lhe minimum tension in the cable.
'2. Cable A 8 in tl1e figure below supportc; a unifonn. horizontally dis
tri btued load of 1100 N/m. n1e lowest point of the c.able is 3 m below
supr>ort A, and the suppo11 at 8 is 6 m higher than tll e SUJ)I)()rt at A.
Determioe Lhe rninimmn and maximum tensions in d1e cable.

6. Use the fommla i n Exercise 5(a) and tlle lwo-tcnn approximation


in J)art (b) to calculate tl\e length of tl\e cable in Exe1cise I .

7. Complete Exercise 6 for the cable in Exercise 4.

8. Tile cenue s1>an or L1le Vel'l'a7..ano-Nal'rows b1idge consists of lwo


unifoml roadways sus1x:nded fi'Om four cable$. The design oftlle bridge
includes d1e effect of e.xueme temr>eratule changes, which cause tlle
sag of Lhe centre span to vaJ'Y from h w
115.8 m in wi1lte1' Lo
h~,
J 18 .2 In in SUinJUel'. If the SJXln is 1278 Ill, use tl\e tWOtCI'In
apJ)I'OXimation i n Exercise 5(b) to dete1mine the change in Lhe length
of the tables at d1ese telllJ)Cralure exu-emes.

~ ~Sfhll InsrrrT
II OON/m

9. BefOI'e being fed into a 1winting press located to the l'ight of D


in tll e figure below, a conti nuous sheet of paJ)Ct having ma.c;s pel' unit
lengtl1 300 g/m passes over rollers at A and 8 . Assuming that tl1e
curve t"onned by tl1e sheet is parabolic. detennine (a} tl1e loc-ation of tl1e
lowest poi 1ll C and (b) tl\e maximum tension in tl\e sheet.

10. A 40-m rope is stn mg between the roofs of two bui ldings. each
14 m high. The ma:~ i mum tension is 350 Nand the lowest point of
the ('able is 6 m above the ground. Determi ne the horizontal distance
between the buildings and the mass of the rope.

- - -40111- - 3. Cable A 8 in lht figure below suppOriS a unj fomr. hc)riZOnlally diS
tribtrted load of w newtons per metre. The lowest poi nt of the cable is
I m below the suppo11 at A. and the suppo11 at B is 2 m higher than
the support at A. Determine a formula for the minimum tension in the
cable in tcnns or w.

2m

_L
ltn

-,- b!::bl::b:b"==l,d
w newtons pc.r metre
- - -20 m- - -

4. '[be centr e span of lhe George Washington Bridge consists of a UJl i


form roadway suspended from four ('ables. '0 1e uniform load supt>Orted
by each cable is 142 kN/m along Lhe hori1..ontal. lflhe spaJl is 1050 m
and t11e sag is 94.8 m . determine Lhe minimum and maximum tensions
in each cable.
5.
(a) Show tll at Lhe length of a cable L11aL SUJ)J)()riS a unifo1m,
horizontally diSll'ibuted load Of W, from iiSrninimum point
toapoint P(xp , yp),isgivenby

II. !\ 50msteel measuring Jape has ma<s 1.6 kg. lftlle tape is sltelched
between lWO J)OinLc; at the same elevaLion until d1e tension at each end
is 60 N, determine the span of the tape.

12. An aerial trrunway cable of length 150m and with mass per unit
length 4 kg/m is suspended bclwccn two poi niS at the same clevali on.
l f the sag is 37.5 m, find the span of the cable and the ma.~imu m tension
in the cable.

i. *

13. A counterweight D of mass 40 kg is auached to a cable tl'lat passes


over a small pulley at A and is auached to a SUJ)I)()I't at 8 (tiglll'cbelow).
'llle system is motionless. Knowi ng that L
15 111 and h
5 Ill.
de.tennine the lengtl1 ot" tl1e cable from A 10 B and tl1e mass per unit
lengtl1 of lhe cable. Neglecl tl1e mass of tl1e c-able from A to D .

xp

L ( P) =

Show lhat tl1e value of this integral is

L( P) =

x;

WXp

I + ( - -)

>-

r(l

'T.0

+ In
2w

I+

W.<p) 1+ WXp.]

--

Til

To

(b) The expression in pan (a) for L( P) is unwieldy in per


fOmli ng calculatiOnSi nvolvi ng the lenglb of ll cable subj ect
lO unj fom loads. In prac1icc, it is often approxi mal<..d by -.
1 *-*- 14. A uniform cord 900 mm long pac;ses over a frictionless pulley al
fii'SLexpanding
+ (w2x~f T02 ) as an infinite series aod
8 and is allached to a rigid support at A (figure below}. If L = 300
i ntegmting term by term. Show that thjs lc-.tds to
mm. determine the smaJier of the two sags for which the cord is i n
cquiljb1i um.
.
2 YP
2 .YP
L(P) = X p I +
Xp
Xp
+ ... '

/I

J(

), S( )4 ]

conoergcnt for ypf.tp < 1/ 2. In si lu:~t ions where yp is


much less than x p, lbt series is truncated nfler the first two
tcrrns.

11.9

i*

IS. To the left of point 8 in the figure below. the long c-able rcscs on a i
roughhori;.r..onml surf3.("(:. lf thecablehas mass per unit length 2.7 kglm.
decermine the force F to maintain equilibrium.

**

Diffe-rentiatiot and Jn1egr.uion of V~ors

755

16. A cable of uniform weight per unit length w is suspended between


two points at the same elevation a distance L apart Dccennine the sagto-span ratio for which the maximum tension is as small as possibJe.

lt=3m
8

_I

-2.7 n -

I I 1.9 Differentiation and Integration of Vectors


In Section ) 1.7, vectors were used to represent forc.e s and moments; they can also be used to
describe many other physical quantities, such as position, velocity, ac-<.-elerJtion. electric and
magnetic flelds, and nuid llow. In applications s uch as these. vectors seldom have constant
componems; ins tead~ they have components that a re either fu nctioos of posi1ion. or func1 ions of
some pa rameter. such as time. or both. For instance. che sp ring force in Example 11 .26 varies
in both magn itude and direct ion as the sleeve moves from B to C. Consequently~ components
of the vector F representing this force are functions of position x between B and C :

When a pa nic le moves a long a curve C in the xy-plane de (ined parametrically by

C :

IIKJI Posi1ion
\'e<:lor of J)<li'Licle moving ~dong a
cur\'e io the xy-pltmc
MijiCJII-h

X = X(l) ,

y = )'(1),

a ::; I ::; {J ,

its position (x, y) relative Lo the o rigin is represented by the vector r = xi + y j (Figure I J .91 ).
rllis vector is called the poslllon vee!or or dlsplac.e ruent vector of the panicle relative to the
origin. We wilJ have more to say about it in Section 11.13. Note. however. that if we substitute
from th e par.unelric equal ions for C. we have

= x(T)l

+ y(t)] ,

r= xi+ yj
X

which indicates that the dis placement vector has components thm are functions of the parameter/ .
In this section we consider the general situation in which the components Vx, vy, and Vz
of a vector v are functions of some parameter t.

\' = Vx(l)t + Vy(I)J + Uz (l)k,

( 11.52)

and show how the opera1 ions of d ifferentiation aod iote.gration can be applied to such a vector.
In Sections ) ) . I ) and I I . 12 we w ill use these results to d iscuss Lhe geomell)' of curves, and Lhis
will pave the way for an analysis o f the motion of particles in Section 11 .13.
Because" in I J .52 has components that are functions oft. we say that \' itself is a vectorvalued function of 1. and write v = v(t). Each o f the component functions vx(t ). Vy(l).
and Vz (l) has a domain. and the ir common domaio is caiJed the (/omai'' of the vector-wdued
function v(t) . G iven thaL this domain is some interval Cl ~ t ~ fj, we express 11.52 more
fully in the fo1m

,. = v(l) = vx(t)i

+ v1 (t)] + u, (t) k,

a ::; I ::;

/3.

( 11.53)

To di l'fere.ntiate and imegl'ate vectol'-valued functions. we fi rst require the concept of a limit.

But because two vecLors are equal if and onl)' if their componentS are equal, we can s._.:ty Lhat
' '(/) is continuous at to if and only if

uAio) = ,_,o
lim u,(l) ;

= tlim
u, (l );
- to

uy(lo)

u, (lo) = t~to
lim u-(r)
...

(i.e. , v(r) is continuous at 10 if and o nly if its componenL' are cominuous at t 0 ).

For example. the vec1or-valued function

1)1 + ( 1/I)J

v(l ) = (1 -

1) - 1k

+ (1 2 -

is discontinuous for I = 0 [siJlce uy(O) is no-t defined) and for r = 1 [since v,( l) is not
defi ned] .
The derivative of a scalar function y (f ) is its iJlstantaneous rate of change:
dy

- = IJm
dr
h-o

)'(f

+h) - )' (1)


11

The derivative of a vector-valued function is also a rate of change defined by a similar Limit.

DEFIN I TION 11 . 1 4
The derivative o a vedor-valued UI1CtiOll v(f) is defined as

dv

dr

ILm

v(l + h ) - v(l)

11 ~o

II 1.561

provided thatrhe limi1 exists.

lo prdCtice, we seldom use the definition of a derivative to caJcu.late dy f tit for a scalar function J (t)~ formulas such as the power. pn.xlucl, quotiem. and chain rules are more convenient. TL
would be helpful to ha\'c corresponding formulas fordcri\'atives of vector-valued func.tions. The
following theorem shows lh~ t 10 differemia1e ;:l vecwr-valued function, we simply differentiate

its Cartesian components.

THEOREM 1 1 . 8

If v(l) = ll.r (/)l + v,.(t)j

+ uj

t )k , then
111.57)

provided that the derivatives on the right exist.

PROOF If we substitute d1e components of v(l +h) and v(t) imo DefinitioJJ 11.14. then we
have

-d' = 1.lfll { [vx(t + h)i + v". (l + h)] +


h-o

tit

lim { [ u.r (r + h) -

h-o

u.,(r) ]i +

Vz (l

+ h)k] - [vx(t}i + Vy(t }] + v, (t)k]}


h

[ vy(i + II )h

vy(I)]J +

[ u, (l + h) - v, (r)]k}
h

(accordi ng eo equaeion 11.14), and

dv
dt

[ .

= 11-o
l11ll

v,(t + II) - v" (1) ] ~ [ . v> (I +h) - vy(l) J~


o + hm h
.1
1~-.o
1l
v-(1 +h)- v. (t)]-

+ [ lim

h-o

II

(according to equation I I .54). Si nce each of the limit$ on the rigtu exists. we can write thai

dv
dv,. d v>.
dv. = -1 + -J + - ' k,
dt
dt
dt
tit

Tlu:or~m I I .S gives us a working rule for d ifferentiating vector\'alued functions: To dif-

ferenlhue c.1 vecwr.valued fullClion, we dift'eremiace its Cartesian componems.

I EXAM PLE

1 1.36

..........
If v(l )

= t 21+ (3! 3 -

SOLUTIO~

2t )]

+ 5k, find v'(3).

According to 11.57,

dv
dt

'

Con<tquemly. v (3)

. .

2/ i

+ (9/ 2

- 2)j.

= 61+ 79j.

The sum rule 3.9 for diffc.rcntiation of sc.a lar functio ns h,ns its counterpart in the sum rule for
vc<.:tor\'alucU functions,

du

= -

dv

+-

( II.SS)
dt
tit
dt
(see Exercise 22). There are three eypcs of produces associllted with vcctors: the produce of'
- (u + v)

a sctllar and a vector. the dot

produce or 1wo vectors, nnd the cross product o f two vecLors.

Corre.,ponding eo each. we have a produce rule for dift'crcntiation. but all resemble the product
rule for scal(lr fulcrioels.

THEOREM 11.9

If /(1) is a di.fferentiable funct ion and u(l) and v(l) are differentiable vector-valued
functions, then

d
- (!)
dt

d
dt

dv

df
v
dt

+ .f -dt ,

dv
dt

- (u ) = u -

d
(U
dt
-

X \ ')

For a proof of these results, see Exercise 23.

du

+ -dt v ,

dv
dt

U X -

( I 1.59a)

du
dt

+-

X V.

( 11.59b)
( 11 .59c)

I EXAMPLE

11 .37

..

If .f(l) =

,z + 21 + 3, u(l) =ri + r 2j d

3k , and Y(r) = r (l

(a) -(.fu)

(b) -

d1

(c) - ( u x v)

( u . ' ')

dr

+ j + k). usc 11.59 to evaluate:

dr

SOLUTION'
(a ) With I 1.59a,

d
dl

- ( {u)

du
= -Jf
u +[<11
ell

= (21

+ 2)(ri + 1 j

= (3t 1 + 41

- 3k)

.J..

(r

+ 2 1 + 3)(i + 21j)
A

+ 3)l + (-f11 + 61 2 + 6t)j

- 6(r

l)k .

(b) With 11.59b,

- (u v)

dr

dv

= u-

dr

du

+ -d r v

= (ri + 12}

= (1 + r 2 -

- 3k )

(i + 1+ k) + (i + 21J) (ri + r.i + rk)

= 3r1 + 21 -

+ (I + 2r1 )

3)

3.

(c ) With 11.59c.

- (u X \ ')

dt

=u

dv

X -

dt

+ -d u
(//

(/ I

+ Il'j -

1 k

i
=

12

(i

3k ) X I + j

- 3

+ k) + (iI T
A

21

l\

'

2/J J X (l i

+ lj + / k)
A

=
=

[(1 1

+ 3)l (3r 2 + J)i -

(3 +

(3

t)J + (I -

12)k]

+ 21)j + (2r -

+ i2t 2i - rJ + (I

- 21 2 )kl

3r 2)k.

rr vector-valued functions can be d iiTerentiated. then they C<lrl be antidiiTerentiated.

Furmally,

we make the follow ing statement

DEF INITION 11.15

A vector-valued function V(r ) is said to be an antiderivative of v (r) on the interval


~ < ( < /3 if

-d\'
= v(l )
dt
For CJ<mnplc. un antidcrivmivc ()( v(l )

fora

<

< {3.

l II 60)

= 2ti - j + 3t 2k is

If we add to V(t) in l 1.60 a ny vec tor with con.stant components, denoted by C , then V (r) + C
is also an antiderivative of ' '(1). We ca ll this vector the Indefinite Integral of v (r) , and write

v(l) dt

= \'(1) + C .

(1 1.6 1)

For our cumplc. lhr:n,

Because vector. can be differemiated oomponent by componcn~ it


be integrated component by component; that is. if v (r)

foiiO\~S that they

may also

= '' (r)l + v1,(r)j + t,(r)l: . then

(11.(12 )

I E XAM PLE

, 1 .38

Find the indefinite integral of ' (r)

1\c<<>rJin~; to equation 11 .62.

SOLL'110N

I
where c

= Jt=li + ~j + 6r 1k .

v{r)dr =

[~(/- 1) 1' 2 ... c,}- (e' + Cl)J + (21 1 + CJ)k

c,l + cJ- c,i. is a cons tant veC1or.

EX E R C I SES 11 9

(/

In Excrd.!tet I S lind lhc lar~CSI possible domain fOI' d.c \'et..lor\Oiue,l


l'unclion.
1. ' '(r) = r 1i t

2. v (r)

4. v(r)

S. v (r)

18.

Jil1+k

= (lin r)i ... (ro<r)j

3. >'(t) - (Stn

16. J,'[I( U >< ~) )

tli -

In (r ~ 4)(i

rj

(t' cos t)k

.f!

- (\1 \'
ti l

If /(t ) = 11 + ;1. R(l)


211 - 3r. 11 (1) = d - r'.i + 21k . and
2
V(l) .:;:. i - 21j + :\t k. lind l hC liC~Iar or Ihe C.:OIIIJk._lllCIII~ or lhC VCCIOf'
m f..K\!rci'\.C$ 6 21 .

du

tit
tl
8. dt [g(r)u (r)l
d

10. - (u x r v)

"'

d
12. -(Ju + 4v)
til
d
14. -l/(l)u + g(l)vJ
dl

!1. u

1/(l) u(t) ldl


1/(t]u vi dt

~: - ' I u(r)dr

22. PtO\C cqu..llit>> 11 .58.


l J. Verify dlC n.."'Sllhs in equ.uiOilS 11 .~9.
24. Prove th.:~ t forditrcrCHiablc (unCI1 011~ U(l) V(l). aOO \\'(I).

fi.

19.

I
I

l)k

+ j)

<"i 1 (cos' t).i -

[3g(t)V(I) - U(l)] dt

dv
dv
10. U X - - /(I)U - \
dt
dt

_ , l j(

+ (r +

17.

d
d!
d
9. - (u " ' ')
dt
d
II. - (2u . \ )
dt

7. - (j(l)\'(I}J

13.
IS.

I
I

X W)

du
= - V
dt

X W

+ U -ddtv

X W

+ \J V X

dw

dt

2~ .

Pro,c dmt it' .-difrcrentiuble t1.ulc tio n v (t) h.uscoMtom lel'tgth, lhCil
O, thc ''cUCM' t/v fdl i.;: pc:rpendicul.v
tu v .
26. If\'= v(s) .sat .. flcn:nli.abfc. \'\Xtor..vulucd luncttonMdJ = s(z)
is I tJiffcrc:nti.Jblc ~4\181' run(tiOft prove lh.it
11 ~any point :lf. which tlvftl t

tlv
dt

d'' tis
d

Ji

Thi' f'C) uk is cui ted the dinill IllIt' (or 41i rrCR'lli ~ itlft of \'CCk.lt'~v.l!ucd

functions.
U(l) dl
4 V(I) dl

27. Show that the following definition (or the limit of a vector valued
function is equivalent to Definition 11.12: A vcclor\'alucd function
\'(1} issaidlohavclimit V asJ;:~pproachc!'l<l ifgivcnanyt: > O. thcrc

cxiSIS a B > 0 such that [v(/) - VI < t whcnc\'cr 0 < [I - lol < B.

J J .I 0

P;munclri..: n(...-:t Vc..:tol.' Rcp.~.-c.s.eotatjoo s vf CW'\'cs

761

IJ J.1 0 Parametric and Vector Representations of Curves


In Sec[ion 11.2. we presente d curves in space as the imcrscction of two surfaces. For example,
each of the equations

describes a surface (the first is a sp here and the second a p lane), and together they describe the
curve of intersection of the s urfaces - the circle in Figure 1 1.92.

FIGURE 11.92

Cune of intcl"Se(;tion of a sphc;re and a plane is a t:irck:

Parametric Representation of Cun'es


Tn m any applications it is more conveniem w have a curve defined parametrically.

DEFINITION 11 . 16
A curve in s pace is defined parametrically by three functio ns:

C :

K3J3!1J:WiiFM Cur\'es
arc din:,clcd &oro i(li1ioJ poit~l 10

fma l poilu
8 (1 = /3)
fi~al \
pomt

point

= X(l ),

)'

= )'(1) ,

1:

= Z(l) ,

Cl

5 {3.

( 11.61)

Each value of t in the interval Ct. S I .S f3 is s ubstituted into the. three functions, and the
triple (x, y, :.) = (x (l) , y (l ), z(t)) speci fics a [l()int on the curve. Definition 11.16 clearly
corres ponds to parametric Definition 9. 1 for a plane curve.

h is cuswmary to assign a d ire.crion 10 a curve by calling thai point on C correspond ing co


I =

a the initial point and that point corresponding to I = fJ the fi nal point, and the direction of

C isthat d irection along C from initial poi11t to fina l point (Figure 11.93) . NOe in panic ular that
the direction of a c-urve always corresponds to the. direction in which the parameter increases
along the curve. Because of this. w henever we describe a c urve in nonparame tric fo rm but with
a s pecified djrection. we must be carefu l in setting up parametric equations to ensure that the
paraJneter increases in 1he a ppropriate direction.
When a curve is described as the curve of inl.e.rsect i.on of two surfaces. we often obt.ain
paran1etric equations for 1he curve by s pecifyi ng: one of x. y . or z as a function of 1. and then
solving Lhe equations for the other two a.s func tions of I. Consider.tble ingenuity is somelimes
required in arriving at a s uitable initial function oft. We illus trate this in the following example.

I E XA MPLE 11 .39

Find ,,.,..lnx:tric eqiJ<li.OIIS for ea<h of the following cuoe.':


(3) =- I = .r l + yl .r - .v = 0 directed"' that z lntrCIIses \\hen t unu )' are pllSIIiVC.
(b) .r + 2)' +: 4. 2.x +) + 3: 6 dircxte<i so lh.ll y incrc;~se< along the curve.
(c) .r2 + (y - 1)2 :::: 4 , t
directed so thai y inc:rcuses when x i' J>OSiti\'C.

='

SOl t '110N
(a)

'The~""'" of intersee~ion of lhe circular pamboloid : = I + x 1 + y 2 and the plane


y = .r shown in Ftgun: 11.9-1. If we choo!>C .r ..s the parameter dlong the: cuf\e by
seninjl \ = / ,then y = X anu: = I - .t 2 + ) 2 imply WI
.r

y = I,

I.

= I + 212 .

\Vhcn 1 > 0. so are x andy. tUld for these vulues of 1. t inercuse...~ as I incre~lSes.
that these arc actc:plablc parametric equation:\ ror the cu1'\e.
(b) 'The smaightlinc interscclion of the: t\\0 planes is shown in Figure 11.95. Ifwechoosc:
.v "-' the param<ter by scnina ) = 1 (thus forcing y to increase 1 in<rtaSC5). then
llli~ n1ean~

.r +:= 4 - 21,

2x+1:=6- l.

TilC Stlhll ion uf thc.licequ:uiun:. for.\ nnd z in ICI'Ill"i of 1 gives the J)HmmclricequUiiun}\

.r = 6 -51.

= 1.

-2+ 31 .

(c) 'Thceurveofilllersectionoflhe ri,ght-drculnrcylinder .r 2 +(y-1)2 = ~and the plane


x ,. show11 in Figure 11.96. We ._.,w ill Se.:tion 9.1 th111tngoMmetric !unctions
rc particulnrly useful forcir.:les. If we Ct ,r - 2co,l.thcn y a I :!: 2sin 1 . A SCI

:=

of parnrnctric equation'\ fm the curve is therefore

x 2ro! l .

y =

I+ 2sinl.

; = 2cosl.

~ 1 ~

2JT.

Any ron&c of values of I ol length 217 uaces the curve exactly once To chec~ that
these eqooti"'"' specify the correct direction olool!\ the curve. we note that I = 0
g ive the point (2, I, 2) und I = 1r/2 give< (0. 3. 0). S ince vnlucs o f 1 between 0
a nd 11/2 give one-qua11cr of the curve (rnthcr th!lll three-quarters). points arc indeed

gcncrnted in the requi'OO direction indi c a~cd b) the arrowhead in Figure 11.96. Ha<l
"" dlOscn the equations

.r ,.

2co>l.

y = I - 2inl.

= 2ro>r.

0:;:

< 2JT,

we would ha\ e generated the same set of points traced in the opposite direction.
l..ulC of incr-

CUJ\'t Of
i o~ u(

pM.tbOCutd ard pbnt'

: . I I rl t
Iy=t

' I
4
A'+

2.v + ~

..c + 2y+ t.4,

l2.n )" ' 3z 6

y:x

)
)'

;r

Cu''-'t C)( lnttrs.,ion (lf


c.irc,\ lar C)-bMi md pfanc

11.1() P.lr.unelrit: :1nd Vc~o:tor Reprc:Knlations of O af\'CS

7U

Because computers and grdphing calculators produ<:c excellem plots of parametrically de~ ned
duce:-dimensiotlal curves, lhere is incentive. to rcpresem cur\'eS parame:lrically rather than as I he
imerscction of two ~urti1ccs. PlotS or the curves in E,,amplc 11.39 are shown in Figure~ 11.97.

Conlptller ploiS of tbe- cuvts in E-<amp!e I 1.39

'
,Y

DEFINITION 11 . 17

A curve C: x
.r{l). y
y(l). ~
~{1 ), a ~ 1 ~ {3. is said to be a conlinuous
cuovc if each of the functions x(t ). y(t ), a11d z(t} is continuous fO< a ::; 1 ::; p.
Geometrk~1lly, this implies that the curve is at no point separated. Each of the CUI\e.s in
Exumpk 11.39 is therefore cominuous.
A curve is said to ben closed cune if itS initial and final pointS are the ..ame. Circles :uod
ellipses arc closed curves. Straight-line segments, parabolas. and hyperbolas are omt closed.

Vector Representation of Cunes


'll>c posit ion vector or displaocmcm vector of a poiou P(x. )'. :) iot space is

r=

(x,y.z)

=xi+ y j + : k.

We visualize it as the vector drawn from the o rigin to P (Figure 11.98). I( we consider only
110i nos thaolie on a curve defined l>arantctoically by 11.63. then fOI' Ihesc poinl we can write Ihat
r

r(l)

x(t)l + y(t)j

+ :(t) k.

a ::;

t ::;

fl.

( II .(\.I)

= a tO 1 = fl. the ti)l of th i~ V\.'1.1()( traces the curve C fro on initial poi ntlo
final point. We call11.6-l thc \'ector representalion of a curve.

As I varies from 1

\'('(.101

ttpre.:.rntation of ..t curw.

764

0 J3p iCJ 1 1

I EXAMPLE

Vcs;:h) IS Otld

Tb.r:ecDi.t.l)Cn.Si<lll:tl A J)3])'tiC <JcOillC:tl')'

11.40

Draw and plot the cut'\e with positjon vector


r = r(l) = (2cosl)i

+ (3sin1)] + (1/ IO)k.

0 :;:

I :;:

6Jr .

SOLUTION Whenwesetx = 2cos l ,y = 3sinl , and<; = (1/10), it is clearthatx 2 /4+


y1 /9
I. l11is means that the curve lies on the elliptic cylinder x1 f 4 + )" / 9
I. As 1
illcrcas.:s from 0 to 6Tr, values of z increase linearly from 0 to 3Tr / 5. What we bave therefore

is three loops that rise around the elliptic cylinder. Tc is pari of what is C(lllal an elliptic helix.
It is drawn in Figure l\.99a and plotted u1 Figure l l.99b. lf the curve is given width, it COu.ld
represent three coils of a spring or three windings of an inductor.
MillfiiiJ::uJ\I'fi'fU\.

Computer pJo1 uf sn elliptic helix

'

I EXAMPL E

11 .41

Draw and plot the c.ur,e. with poshioo vec.tor


r = r (l) = 1i + t 2]

+ lk ,

I ::::

0.

SOLUTION When we set x = t , y = t~, and z = I, then z = x and y = x 2. T hese imply


that r = r(l} describe$ points on the curve of imersection or the surfaces y = x 1 and z = x
(Figure ll.lOOa). Because t ::: 0 , ooly that half of th.e CUJ'\' e of intersection in the first OCI<ll)( is
defined by r = r{l }. The curve is ploued in Figure I I.I OOb.
IM[ioiiiCI1l!l[li;J:I3''r"-1lil!ti;]
il!'!II Cur..e r = ci + ,~j + rk.

l:llfiii;IMIIJE'i11\. Ct)mputer
plu1 ll f cum: r
+ t 1j +

= ti

tk

y
X

EXERCIS E S 11.1 0

In Exercises 1-10 lind (J:at"dltlCtric ~snd \ector reprcsemmions. for Lhe


Draw and plot C3Ch curve.

cur\~.

I . .r + 2)' + 34

=3 dirocled so thai z inaeases along

= 6. )'- 2l

the Cur\'C

2. x~

+ y 2 =2. z = 4 directed so 1ha1 y 1rtere~es in 1he fii'SI OCI<IIH

3. .r 2 + yz
is pos iti,.c

= 2. x + y + t =

I d u"tcted so th:u )' dee:~as.t.~ when X

origin

+ 2.r 1 =

6. t =

I )'

= Odircclcdsothat z docn:as~

*
*

9. x
10.

=...fi,.

= y 1 directed ~way frorn lhc ori&in in the firMOC:lanl


z = J.r.! + y2 y = .c2 directed so thai y docTea'\C.S in the fin..~
l

octant.

ln Exerc-ises I l-IS draw and plot the C\IT\'C with the shcn positiM

Jx : + y:. )'

= :c din.."Cicd so Lh:lt y

incr~ \\1lCn

11. r (l) = 1i + rj + 11iu <: 0


12. r (1) = (2cosl)i+(2sinr)j+31k.O :;;I :;; ~rr

= z directed so that x ck."Cte3SQ along lht curve


x is

13. r (l) = (1- 2)l + (2- 31)} + 51k


14. r (t)

J)O:)ili \C

7. t

y 1 .x 2 +)'l - 2.\'

x2

when x is positive

\'CC.lOI',

4. t ~ .~ ~ + )' l. :r l f- )' 1 eo; S dircc:1cd clockwise as vtc'"'Cd from the


5. l

* 8. z = /

= .~ + y. y = x l din.-..::lcd so t hai A incr~~ along lhc cunc

= (tl

- t)i + rj + Sk

15. r (1) = (msl)l + (sin1)j + (cosr)k. 0:;;

5 tr

111.11 Tangent Vectors and Lengths of Curves


If C i< a curve in the xy-plane (Figure 11. 10 1). lhen the tangem line 10 Cat P is defined as
the limi1ing posi1ion of the line P Q as Q moves along C tOward P (see Section 3.1). We 1akc
lhe sante approach in defining tangent vectors to curves in an arbitrary plane or in space. On
curve C <left ned by 11.63. we let P and Q be the l)()ints CQJTCSponding to the pamrnctcr alues
1 and 1 + h. !>osition vectors of J> and Q are then

Line
joinin~ lWU

poinb on a. \!Ut\'C.

y p

r (l) = .t(l )i

+ y(l)j + z(t)k

and
.r

r (t

+ h) =

.t(l

+ h)i + y(l + lr)j + ~(I + il)k

(Figure 11.1 02), and til<! v~10r joining P tO Q i~


L.imil

o(

vecror J(lining cwo roint' on a am-e

le:kh h> t.lngcnt \'tctoc to CUJ'\'t

z
p

r (t + h) - r (t)

PQ

r (l

+ lr) -

r (l).

I f we 1e1 II approach ?.tro. then Q moves along C toward P . and 1he direc1ion of PQ
becomes closer to wha t seems to be a rcaS011ablc definition of the U\ngent direction to C at
P . Perhaps then we should defi ne limh-o(r (l +h)- r (l)] as a tangem vector oo C at P .
Unfonunately. the li mit ector has length zero. and therefore
lim [r (r + h ) - r(l ))

h-o

= 0.

tf, however, we d ivide r (l + h) - r (l) by il, then the resulting \'ector


)"

r (l

+ lo)

- r (l)

II
is not equal to PQ , but iLdoes have Lhe sam e d irection as PQ. Consider, then, taking the limit
of this vector a') h approaches zero:

r (l +h) - r (l)
h~o
h

lim -'---..:....-....:....:.

rm JQ
\ 'W1; r

T'M~-".

I:JjW

Tr the limit \"C'(:Ior exiStS. then i1 will b<' tnngcm 10 Cut P Rm llCCording to equAtion l I S6.
this lin\it de:flne$ the dcri,-ativc d r / dr:

II ('1;11'\'t

h)

'

dr
dr

hm
-o

r (l + h) - r (t)

II

dx.
dy.
- 1+ - j
dt
dr

d :.
k,
tit

+-

provided thnt each oflhcdcrh:uivc..H/;c / t.lt, ely/ til . un(j d :ftlt C'\ists. \Vc haveju"t cstabH'(hcd
the follc>witg result.
y

.<

Ulll~m l \'t'Ctono n is i

TH EORE M 1 1 . 1 0
If r = r (t) = .\ (t)i + )l(t)j + l(l

)k. cr ~ I ;$: fJ . is th~ \c:"Ctur repr~cnla\iun uf a ..:ur\C


C. 1hen many pOinl on C m which x ' (t). y' (t ). a mi t'(t) all C'\h.t aHl do nOl "anish

:n C'.adt point

.;;imultAnonu_.;; ly,

012 o curvt. one i n the opposi1e


d ii'CC'Ii Oil IC) the Ol h!!t

dr
.
dy .
d: .
= -dt
= dx
- i + - j "- - k
dt
dt
dt

is a mngem vector to C (Figure 11.103).

A
7> i

-~ ~ dr

1'1lcrc urc two Ulngcnl directions at any point on a cun'C. One or these has been shown to
be d r / dt ; the other must he -dr f dt (rigurc II. I~). llow can we tdl which one is d r f dr"J
A cl~"f analysi,s of 1hc limil in 11.65 indic<tte~ the following (~-e Excrcbe 17) .

dt
)'

lfJii;IWIHioli

COROLLARY 11. 10. 1

T he tangent vocror d r f dt to a curve C: x = x(t ). )' = y (t).! = :(1). a ,S: r ,S:


always points in the direction in which the parameter c increases along C.

Smooth

/i.

D EFIN IT IO N 1 1 . 18

A curve C: x ~ .<(/) . )' ~ y(l}. ~ t(l). a :-:; 1 :-:; {3. is said to be a smooth cul'\'c
if the dcri\<athcs x ' (l). y'(t). and ;;' (t) arc aU continuous for a < I <: fj and do not
' 'anish simultaneously for a < I < fJ .
.<
[;%

MOIUIII.I..J

'

Since x(t). y ' ( t ) . and z:'(t) arc. the comp(ulc.n ts of a tangent vector to C. this dd\nition
implies lhat a long a smooth cunc . small c hanges in f produce small changes in the direction of
1he tangent "ector. In other words, the tangent ,ector turns graduall y~ or smoothly... T he cur\'e
in Figure 11 .105 is s mooth; that in Figure 11.106 is not because abrupt changes in the direction
orthe curve occur at P and Q. According to the foUowing definition, this cur\'e is piecewise
s mOOth.

DEFI N ITI O N 11 . 19

A curve is said (0 be 3 piec.ewise-s mooth cu_rve if 1t is cootinuous and c an be divided into

a fini te number of smooth subcuncs.


)'

I EXAMPLE

11.42

For !he curve in Example 11.4 1. 1\nd a tangent vec10r m the point (3, 9 , 3).

SOl .lJHOJ\

Since r =

A cangent vector 10 lhis curve a1 any poim on the curve is

tlr
dx .
dy.
tl~.

- = - i + - j + - k = i + 2/j + k.
dr
"'
"'
tit
3 yields the point (3, \1, 3). a tangemt vector at this point is r'(3)

........

I EXAMPLE

= i + 6j + k.

11.43

Find a tangent vector at the point (2. 0. 3) to the helix


lillclileliU

il!1 I

'TangCilt

x = 2cos l ,

YtC!Or h> it heliX

= 2sin/ ,

31

z =

1T~

I ~

0.

Is the helix smoo1h?


SOLUTIO~\

A tangent vector to the helix at any point is

do
d x- = - i
tit
tit

d1.

dz.
+ -
j + - k
dr
dr

= (-2sinr)i

(3)-

+ (2cosr)j + -

2n

k.

Since I = 2;r yields the point (2. 0. 3). a tangent vector at this point is r 1(2;r) = 2}
(3/ (2;r ))k (Figure I 1.107). Since x' (t ). y' (I). and l.'(I) are a ll cominuous functions. and they
arc never simultnncously zero. 1he helix is indeed smooth.

........

Unit Tangent Vectors


Wlten a c.::urve in che xy-plane is <.lcf1ned pnn.unctrically by
C : x = x(1),

y = y(r),

c. S

r S {J,

(1 1.67 )

a langcm vector to C is

1' ;

(11.68)

and thi~ tangent vecror poilus in 1he direc1ion in which 1 increases along C. To prod uce a unit

t.mgent vector to C m ony point, we divide T by its length:

..
T
dr/dt
I =-=
.

ITI

fl:llflil;l:i'il]
:)loug :l curve

'lilt* l.ellglh

8(1 :{J)

1
8

L =

//;....,--~.....
P (x(l). y(l))

(11.69)

We now show that if length along C is used as the pal"~111e1er by whit:h to specify its poinL'>.then
division by ITI is unnccessnry.
In Section 7.3 we shOwed that small lengths along a plane curve C can be approximated by
straight-line lengths alons mngent Iines to the curve, and that the total length of n smootb curve
from A to 8 (Figure 11.1 08) is

A (I

ldr/dtl

j(dx) 2

+ (dy) 2 .

.t

WiLh pa.rametri.c equat ions 11.67 we can wri 1.e Lhis formula as adefin iLeintegral wi1h respect

= a)

tot (see also equation 9.3):


X

(11 .70)

Funhcrmore, if we denote by s = s(l ) the length of that pan of C from ilS initial poi nt A
(where 1 = cr) to any point P (x(l), y(t)) on C (Figure 11.1 0 8), then s(l) i$ defined by the

integral

s(t)

(-dx)' + (dy)'

= {

dt.

dt

dl

! 11 .7 1)

h follows. then. thnl the derivative of s(l) is

ds = f (dx) '
dt

dt

(dy)'

, ll.n>

dt

But according to 11 .68, d r / dt is a tangent vector to C with the san>e length:

~~~~ /(~;)' + (~~)' ~-:


=

( 11 .73)

When this equation is multiplied by d t . it g i,es

= j(dx) 1 + (dy)l = ds.

ldr l

lilll-6 411 inA Snll.ll ~gtb,


along the talgent line to a curve in lhe
xy.plane are regarded a~
aiOilJ

the C'Uf\'e

~mal llen,ah:S

(11 .7-1)

This equation states that ds is the length of the tangent vector d r = clxi dy}, and therefore
ds is a mca.<ure or length along the tangent line tO C. In spite of this we often think o r ds as
:1 ntcasurc o r sm:tll lcngths along C itself (Figure 11.109), and that ds is approximated by the

tangcmial straightlinc length

11~1(

Note too thm if we use length

s a long C as a parameter, then the chain n de applied to

r = r(s). s = S(t) gives


d r ds

(I I 75)

ds dt"

...

(The chain rule is proved in Excrcist 26 of Section 11.9.) Consequently, equation 11 .73 implies
that
(/r

ds

d rfdt
ds/dt

d rfdt

~-:-:

ld rf dt l

( 11 .76)

\Vhaa we have shown, then, js that _


if we choose Jength a_lolg a curve C as the paranleter by
which to specify points on the curve (C: x = x(s), y = y(s)). then the vector

dr

= -

ds

dx:

dy .

-ds + -ds J

( 11.77)

i~ a unit tangent vecto r to C. In additio n. the corollary to Theorem 11.10 implies that d rfds
points in the direction in which s increases a long C . This suggests pcrt&aps that we should always
set up parametric cquatiOI\S for a c urve with le11gth a long the curve as paran>et cr. Theoretically.
thi!' i~ quite acceplable. but practically il is impos..,.ible. Formost curves we ha\'eenough difficulty
ju.st finding a set or parilmctric equations, let alone finding that set with length along the cunc as
para.me1er. Jf we 1hen use a parcune1er t oaher th:ln length along the curve, a unil tangent vector
is calculated according to J 1.69.
These resul1s can be exaended 10 space curves as well. When a smooth curve C has
parametric equations I I .63, equatjon I I .69 still ddines a unit tangent vector Lo C. but because
C is a space curve, drf dt is calculated according to 11.66.

Corrcs)>onding IO fomlUia 11.71 ror length along a curve in I he~\' y ..pJanc. lcnglh along a
smoolh curve in space is defined by the definite intcgrul:

2
,
/(dx)2
+ (dy) + (dz) dr.
!.
2

s(r)

dt

a \

(11 .7~)

ill

dt

These lwn r-esuhs inlply that


SnuJI

( 11.79 )

blilhs .llollg d~ot laJlgtfll htte t()

a cun't in

sp.~c aR" re,4.rdc\l I>

!>IR!tll 'en:th~

Jlon.f:. the CIII'\'C ll((h

'/;]S
c
A

dt

the thrccspa<-c !lllaloguc of 11 .7.t Once again we nre led to the fact thnt when s is used as
panuneter along C. then
dr
d:x,
"".
d z.

( I I ~0)
T
- i + -j + - k

= -tis = tis

dr

.r

I EXAMPLE 11.44

ltirl
y

d.~

ds

is a unit tangent vecwrto C. In addition. multiplication of 11.79 by ilt yields

tis

= /(dx}2 + (tiy) 2 + (dz)2,

(11$1)

indicAting that small lengths ds along C (Fig me 11.110) are defined in term.< of small lengths
Jtir l along the 1angcntline 10 C.

Find n unit umgen1\'Cctor to the cune

C: x

= sit11 ,

= 2oos r,

z = 2t/7r.

1~ 0

m the poim (0, -2. 2) .


SOLlJI'ION

A 1nngcn1 vector to C Ul any poi111 is

dr
-

tit
Since 1 =

1r

dx .
d y.
dz.

= - i+ - J + - k =cos rl- 2si nli+


~
tit
dl

(2).
-

7f

k.

yield.< the point (0. -2, 2}, a mr1gcn1 vector at this poim is
r'(;r)

= - i + (2/;r) k.

A unit tt\ngcm ve<.1or is then

i = - i + (2/ rr )k =

Jl -

I EXAMPLE 11 .46

4/Tr !

- ;r i - 2k
J4 + ;rl

........
Find the lenglh of that pari of lhe ctuve x = y2f3. x = z 2f3 between 1he points (0, 0, 0) and
(4, 8. 8) .

SOI.lJI'ION If we use x = 1, y = r 312 , z = rJ/2, 0 ::: 1 .S 4, as pai'M>etric cqualions for


the curve. then

L= 1' (dx)2
+ (dy)2+ (ti<
)2dr =
ill
dt
tit
0

fJ

I+

~ dt = { ~ ( I + ~

r'l

4
= 27( 19v'l9- 1}.

770

Ve(;(()I'S aod 11u-ee~Di metlsi ooal Atlalytic Geometr')'

Ch.apte. II

EXERCISES 11. 11

In Exercises 1-5 express the c urve in vector

ronn

and find the u nj1

ln Exercises LJ -14 find the leng th of the eurvc. Dn:rw c:it::h c urve.

tangent vector t at each point o n the curve.


I. X

sir\ I ,

2. X =I, )'

y = cos I,

= 12, Z =13,

J.l .

I, -oo < I < oo

x = 2 cost.

= 2 sint , z = 31.

12. X = 2 - 51, y = I

2:

0 :S 1

+ I , z = 6 + 4t,

27r

- I 5 t 5 0

3. x= (1- 1) 2, y= (1+ 1)2 , z=-1, - 3515 4


4. x

+y

= 5,

x2

5. x + y + z = 4,

y = z fmm (5, 0, 25) 10 (0, 5, - 5)

x2

+ y 2 = 4, y :: 0 from (2, 0 , 2) 10 (-2, 0 , 6)

*
In Exercises 6-10 find

t at the point.

6. x = 4cost, )' = 6sint, z = 2sint,


(2../2, 3../2. ,fi) .

- oo <

<

oo~

7. X= 2- 5t, y = I + I , z = 6 + 41. -00 < I < 00: (7. 0, 2)

10. x2 + (y - 1) 2
is oe.g ative; (2 , I, 2)

+ 5. directed so that y

increases aJong the

16. Find a unil ta ngent vc..-ctor 10 1he curve r = (eos t

+ 1 sinl)i +

( sint- tcosl)]. 0 ~ t ~ 2n . called aJl involute of a circle.

8. x 2 + y 1 + t, 1 = 4, Z = / .t:1 + _y1, din::t::led SO IJ)al X int::n:3SCS


when y is positive~ (I, I , ../2)

9. x = y 2 + I. z = x
curve: (5, 2, 10)

J5. ln Ddinjtion J I .IS why arc tb c derivatives assurned n(lt tO vanish


sirnullaneously? /l.im: Consid er the c urve .t
t 3, y
12 , z
0.

17. Show that the tangcnl vector dr/dt to a c ur ve described by equation 11.64 always poiniS in lh c direction in whkh I increases alonglhc
CUT\'C.

= 4, z = x. directed so that z decreases when y

!8.

(a) What haJ)t-,ens when equation 11 .66 is tLIOed to determine


2
12 y
13. z
t
a tat~genl vcctOI' to the curve x
- oo < t < oo. at t..he origin?

(b) Can you devise a way in which to obtain a tangent vectOI'?

111.12 Normal Vectors, Curvature, and Radius of Curvature


l.n discussing cunes we distinguish bet\veen two types of propenies: intr insic and not intrinsic.
An intrinsic property is one that is independent of the parameter used to specify the curve; a
properry rhat is wl illlri11sic is parameter dependent. To illustrate, 1he tangem vector T = drf dr
in J 1.66 is not intrinsic; a c haog:e of paramete r results io a c haog:e in the leng:1h of T . T he unit
tangent vector f , on Lhe oLher hand, is intrins ic; Lhere is only one unit tangent vector in the
direction of the c urve. l be length of a cw,e from its initial point to an arbitnuy point is an

intrinsic propeny; a change of parameter along the curve does not affect length between points.
Because le.ngth along a cune. is an intJ'insic propeny. it is custonHu'y in theoretical discussions to use i1 as the parc:lrnet.er by whjch to specify po i.nts oo the cune. When C is a s mooth
curve in the xy-p lane, parametr ic equations fo r C in ten ns of lengLh

C : x = x(s),
M:iUJII;I
T here is
essellli:.lly one Ol)f'Ul;tl ctitecliOtl ro
tl Clli'\'C i(l lhe X.fpl tli)C

0 ::; s ::; L.

take the form


( 11.82)

Normal Vectors to Curves


The. nom1al line. at a point

Normal
vector

y = y(s ),

s a long C

on a smooth curve

in the. xy -plane is that line which is


perpendicular to the tangent line to C at P ( Figure I 1.1 I 1). Any vector along this normal line
is said to be a no rmal vector Lo Lhe curve at P . S ince the unit tangent vector to C aL P is

Normal
line
X

i1 follows <hat

dy.

= - - i +
ds

dx.

ds

( 11.&3)

l!J

Til ere

~ a pl.ux of nonnal YCCtors to o


cunttn~

is a unit nuonal \'ector to C a t P (note tha i N = 0 ). Because there is onlv one din::ctior~
11ormal to C at P , every r'K)rmal ,ector to C at P nn~ ~ wme multiple )~N ol N.
The situation is qui1e ditierem for space curves (Figure 11.1 12). lf i is the unit mngcn1
vector to a sm001h curve C. Oleo there is an entire plane of nom>al vectors to C at P. In the
following discussion. we single out two normal vectors c;"tllcd the pdncipal nom1al and the
binormal. Suppooc !lull C is defined parametrically by

C :

= x(.).

= y(s) ,

= z(s),

0 :5 s :5 L ,

and Lh~u T i the unit tangent \t tlor 10 C defined by I J .SO. Because

X
)'

(11.&4)

i has uni1 k:ngll1,

= 1' T.

Jfwe use equation 11.59b to differen1iate this equation

\\~th

respect to s ~ we have

But if neither of the vectors 'i' nor d'i'f ds is equol to zero. then the fact lhallhcit seal or product
is equal to zero implies lh3tthey are perpendieul:lr. In other wonJs.

13' R
~ lrincip.al
notmal and bino1'mal II) 01 cuve

N=

ds

i.:; a nom1al vcctor to Cat any (.K)inL The unit normal \'CCI.o r in this direction.

dT / ds

=
X

( 11.86)

ld'f/ dsl

is called the principal normal (vector) 10 C (Figure 11.113).


Because N is defined in tcm1s of intrinsic properties 'f and s for a curve, it must also be an
in1rinsic propeny. h follows, lhen, Ihat no matter what parame1er is a~ to spedfy points on <l
curve. N is always the ~me. Bu1 how do we fin<.! N when a cunc C ~ spcx::ilictl in tcrm:s o r a
pam meter other than length along C, say. in the fom>
C : x
If s

= x(l).

= y(l).

= z(t) ,

a S 1 S {J?

( I I.R7l

=s(r) is length along C (measured from 1 = a), then by the chain rule
dt
dT dt
d s= dfds '

where d t f ds mu<l be positive. since both s and I increase along C . Consequently,

:'11

d T f ds
ldTf dsl

(dT / dt )(dt / d.r )


l(d'i'/ dt)(dt f ds)l

(dT/ dt)(dt f d.r )


ltff/ dt ldt / ds

d T / dt
7
:ld -i":.,..
{ d:-:
tl

( 11.88)

In other words. for any parametrizatio n of C whatsoever. the vector d i' f dt always poio1s in the
direction of the principal nonnal, and to find N, we simply find the unit vector in the direction
of d T / dt.
In the study o r space curves, a second normal vector Lo C. caJicd the binor mal (vector)~ is
del\ ned by

B= t

N.

( 11.89)

722

Chapter II

Vn."tor'!O and Tiu-ee-DimensiMal Ana.lytK; OtOt'l"'tlr}'

(Him: Sec cq~ ati on 7 .43.)

38. A 16 kg. hori:oonoaJ. lrianguJar plalc ABC is supponcd by three


wires in the figure bclc..1w. Dctcnninc the tension in each win;,

D
L
-~

......... l.. .. . .. .. . . .........

Vectors u . v. a nd w arc s~i d tel be linear ly dependent if there c:xi'!.t thn.-e


""alars a. b. :1od , . not all zc1'0, such that o u bv +tw ~ 0. J ( thi~
cqu:.uion can only be ~ati~ticd with o = b = c = 0. the \'CCtors arc
S.'lid to ()c li nc:lt'ly indcpendcnl. I n Execi.;cs 42-45 d etermi ne whether
the vc<.'10f'S ~u'C lim::;-rly dcpcndcm Clt' l i nc:~ul y independent.

39. A container with wcig.h1 360 J\ i~~: suppo11cd by CiJblc~~:. A 8 and

A C in the figttJC below. Knowing that F


60i N. dctcnninc the fon:c
P = pj that must bcaJ>plicd 31 A 10 maintain thcconfigUI-ation. Wh31

43.

arc the tensions in the cables?

= ( I, I. 1). v = (2. I, 3), w = (4, 2. 6)


u =( I , 1.1). v = (2. 1,3), W = ( 1, 6.4)
u = (-1 ,3, -5), v = (2, 4, - 1), w = (3, II , -7)

42. u

44.

45. U = (4,2.6). V = (1.3,-2). W = (7. 1,4)

46. Usc YetLOrs 10 show 1ba1 the line sc.-g.lncnl j oinjng lbe midpOiniS
o f two sidc,s of a tri1mglc is pan tUei iO Ihe lh ird sid e and i1s Je ng1h is
One-hal[ Ihe le ngLb Of I be tbjrd Side ( fi gun: belOw).

40. A 200-kg mass is h ung by means or 1wo cables. A B a nd AC.


whic h wc a u achcd to the top o f a vertical waH (figm c below). Deter
mine the rnagnittldC or a horizontal Coree F perpendicular to the wall
that will hold the weight in place. What arc the tensions in the cables?

47. Use ' 'ectt)t'S 10 show th at the mcdia1L10 of a triangle ( figure be low)
all mee1 in a poinl wilh coordinates
x
(

2m

41. T wo bars. A 8 and 8 C. arc pinned at 8 as welJ as at each of IJ1e


ends A lmd
(figure follows). lnjli:tHy C!ltb b::.r is o r length l. !UKI
p0in1 8 is !tLa d iSh(nce II !1bovc Ihe line A C. 11Je b:\JS !trc identicrtl,
each hitvingcross-scctiona l an:a A and Young's modulus E. A verticrtl
Coree whh magnitude F is a pplied at 8 . Show th at the displacement
y or 8 is related to F by the oquation

+ x 2 + x .l
3

+ )'2 + YJ

'

A (x 1 y 1 ~ 1 )

= -211 (h L

y) (

Jy'-2hy+ L'

- I) .
y

Z1+ ~2 + ZJ)

J. 2 Nonnal \ie(."rors, Curnrure, attd RJdius uf Cluvarure

J J

The significance of the fact that the binormal has constant direction can be seeu from a
drawing of the curve. Because the parametric equations imply that y x 2 and z y, rhe
curve is the curve of inlcrscction of these two smfacc.s (Figure 11.1 14). Since Ihe curve. lies in
the plane- yt ' = 0, and a normal vector to this plane is (0, - J, I), if follows that (0, -1, I)
i~ always normal to the curve. But this is precis:1y the direction of B. ln otl1er words, constant

B implies that the curve lies in a plane Lhat has B as normal (see Exercise 30).
c

Tangem, principal nonnal. a11d

binormal to the curve .r =1. y =11 z =r1 1;:: 0

z=y

', EXAMPLE 11 .47

........

Show that for a ~mooth cunc C: x = x(r). y = y (t ). a

~ t ~

f3

in the xyplane. the

principal normal is

x dxd y) (dyfdl ,-dx jdl )


= sgn(dyd
------.
dt d t 2
dt dt 2 ../(dx/d1)2 + (dy/d1)2
2

where the signum function sgn(u) is defined by

sgn(tt) =

I,

if 11 > 0.

0,

if 11 = 0.

-1 ,

ifll < 0.

SOl l'TION The uni11angcm vec1or 10 C is

(d.r fdt , dyftllj

T ~-c~5F~~~
j (d.r / dt)!

+ (dyfdt )Z

For simplicity in notation. we use a dot '" above a variable to indicate that the variable is

differemialed wi1h
nouu.ion.

l'eSJ'<'CI IO

1. For example .i: = dx/dt and

x=

d 2x/dt 2 Wi1h 1his

By equat ion J 1.88. N = (d'i: / dt )/ld'i'!dt l. where

dt

c.<..i>

:;:;;: d

tit j/(2 + j>2

= -t1 (

)c
.n +y.J)

.j.i:2 + .i>2

dt

"'t'

1
. 2 , , (- (.i:t + j>ji)(.ti + j>}) + (i-2 + j2)(.ti + ji})J
2
1(x + Y}

(X2

+ _jl2)l/'

(( - .i:l;t - .i: i>)+.i:2}i+j2x)i


.

+ <-.t:j.r _ .v~Y + .i2_;; + 5.2 y)JJ

If j,..\: - .i' ); is positi ve. then

yi - .i:j .

N= J.i2+ .\>~ '


whereas if )'~"-

.~ y

is n eg~u ivc , then

In other words.

-.

:'<

<...

>gn y.< -

.J' >( .vi. - .i-i. )

jx2+ y2

Curvature and Radius of Curvature


\ Vhen &englh s alolg a s~1ooth curve C is used at> the parameter by \\hich to idemify points
on the curve, the vector T = drf ds is n uni t tnngcnt vector to C. Suppose we different illiC

d 2 rfcls 2 . Since i has constant unit length, only iL~


di rection ~~~~ ~~~~nge: then:: fore. the derivative <ti'{tls must be a measure of the rate of change
o f the din.>cti on of i. ~inc-e is re>1lly our way of specifying the direction o f the ~urvc i tsel f,

T wirh

re.-.pect to s (()form

tiTJds =

we can also s.oy that d T j ds is a mcasun> of how fast the direction o f C changes. But exactly
how docs a vector d'l'/tfs that has both magnit ude rut<l direction measure the rate of change o f

the direc.:tion of C ~J We illustnue by example that it c-annot be the direction of di'f ds; it must
be its magn itude that measures the n11e of change of the di rection of C. In Figure 11. 115 we
show a number o f circles in the ..ry~planc. all o f which arc tangent to the ya:<is at the origin.

Parametric equations for the circle with centre


the circle )as rneasun.'<l from (2 R , 0) ) are

x = R
Consequeml y.

+ Rcos(s/ R) ,

( R. 0) and rndius R i n tcm>s o f lengths along

y = R sin(s/ R),

0 :S s < 2rrR.

Nonnal Vtct<m3;. Oti'VImiJ\". Md R3diu:sor'C111'Witure

J I . Jl

775

and

t!i'
ds
At the ntigin. f = ;r R. and

dl'

I,

;;; I.~R = R i.
W\ii111

Thus, for each of the circles in Figure II.IIS.lhe ' 'ector (/'i'/ds has """liYlhc same direction.
Yet the rate of c hnngeof the d ireclion ofT is 11011hesamc for each ci rcle; the direction changes
more raJ)idly as, the mdiu~ ofthecirclcc.lecreases. 'Ne musc: conclude. therefore. that it canno1 be
the c.lircction of d'i~/ds that measures the rate of change ori. Si nce~1 vector has only length and

Circlu

t.tntclt to d.o )' n:< i ~ m (0. 0)

direction. il m us t be the length o f <IT/d ;- !hat n1casures this rate of change. The c ircles in Figure
II. I I S certainly support this claim; the length of d'i'fds is I I R, and 1his quanti IV incre<L<eS
a s the radii of the circles decrease. This agrees wilh the fac1 that the ra1c a1 whi~h
tun~:;
increases as R decreases. According 10 lhc followi ng dcfini1ion. we call ld 'ftdsl cun'<lture
and 1/ltft(ds l mtfiliSOjCIITV<IIllrt!.

DEFINITION 11.20

If .r = x (s). y = y(.l). ~ = ;;;(s). 0 :S s :S L. are parametric equations for a smooth


c urve in terms of lengths along the curve, we define t he c urvature oft he c urve a1 a poim

as
K(s)

~ ~~ ~

p(s)

K(s)'

( 11.90)

its radius of clJ.Olaturt as


I

( 11.\11)

and its cirdeofcun'8tureauhatcirclc in dw plane ofT and N wilhccntrcat r (J )+ p(s)N


and radius p (s ).

TilC c ircle of cuiVIIturc is illus1ra1ed in Figure 11 . 116 .


l'orthe circles in Pigure 11.11 S wehavealreadyshownthat dT /d.t =

[!Ill
~ Cird e
of (lii'Vanrt of a coi\'C

'

'

Circle or

,1'

R-'L ConS<!quenlly,
for these circles, che curvature b nlways R-~ . and the radi us or curvmurt: i$0 R, the radiu~ of
1he c ircle. In 01her words. for a circle. the circle of cuf\uture is 1he c ircle i1sel f. i1s radius o f
curvature is its radius. and its cur\'atun: is the inverse or its radius. For the CI-\ SC when a curve is
ntM a circle. we show in Exercise 27 thnt at ;u1y point on the curve the circle of curvature is in
some sense the bc:!>t-fiuing circle 10 the curve at that point
Because cur\'111\lre and radius o r curvamre have been dciincd in 1erms o f intrinsic properties
f un<.l s for a cur"e. 1hcy llllL"t ai'O be. imrin~ic properties. ll follows. the n. thac no matter whm
pa.ramccer is used 10 specify points on u curve. curvature and radius of curv~turc arc always the
same. The following 1heorem shows how 10 ealculate K and p when the c urve is specified in
tenns of a parameter other than length along tlte curve.
THEOREM 11.11

When a smoolh cune is de lined pammetrically by

C : X = X(l) ,
ilS c urvature

I( (I)

y = y (T) ,

r=

a < I < {J ,

is g iven by
K(l)

where

Z = Z(l) ,

d rj dl and;: = d 2 r fdt 2

1;. x rl
I rP '

( l l.92)

PROOt'

I f S(/) i" fcng1h nlong C (mea~ u red from 1

JJ'rJ= J'r'i~ ~ = ,,-r,,,,,


ds

dt ds

= ,,ri;t~t l

ld s/dtl

= l<ti 'fdtl
lrl

ds f dt

(Set equation 11.79). Now. we Carl Wri(C r i n lhC: form i


have.
;: =

et). 1hen by the choil rule

= I rri: lhcrcfore. using 11.59tt. we

(~ 11)t + 1rl c~i


"'

dt

"!1

= ( ,, , 11)t + (l i-llcl'i'/ cltD ld T f "'


dtl

= ( cld1 1r. l) T

+ (

= (1 ill~:I) r X

lcdil'i' i)
N.

r. we gel

If we take the cross product of thi:; vector wi1h

( dtf, I rl )

l rl

J"T I). .

+ ( I rl di r

N. (since r is

Because i is pcrpcndicularto N. it follows that Ir X Nl

X :-;

P"nlllcl

to i')

=IriiNI sin(Jr/2) = I.,. and d>ercfore

Conscquenl ly,

~~~I =
and
K

(1) =

lr x rl
lrP

Fo1 the r:Hiius of eurvacure, we have (he rollowil\g.

COROLLARY 11 . 11 .1

When a smooth CUI'''e. is de lined in terms of an arbitra.ry parameter t .


p (t)

1rl 3
I.r x ..1
r

( I 1.93)

J 1. 12

I EXAMPLE

Konna:l Vccwrs. Curvanue. ;and Radius ttf Curva1urc

n1

11 .48

Fi.od curvature and radius of curvaiUrc for the curve in E-xample 11.46.
SOI.lJ110N

Aa;ording

K(l) =

11.92.

Lo

I( I. 2r. 21) x (0. 2. 2)1


lr x rl
=
1(1 , 2r, 2r)ll
lrP

(I

i
I

+ 4t 2 + 4rzp tz

21 21
2 2

+ g,zpn J(O, -2. 2) 1

= (I

2../2

= ( 1 + 8rl )3/Z '


I

p(r)

K(l)

(I

+ 812)3/2

2h

Note in particular Lhat as t incre-ases, so does


I I. I 14.

I EXAMPLE

IJ~

a fact that is ccrtaio1y supported by Figure

11.49

Show that for a smooth curve )' = y(x) in the xy-plane,

~< (x)

I.\"' I
=[I + (y')2jl/2 .

SOLUTIOC\ When we use x as parameter along the curve y = y(x) , pnrometric equations
arex x. y = y(x). Then

r=

i' = (O,y"(x)),

( l ,y'(x)),

and

, I

1<

y'
)'

0
0

l
0

r x r =

11

J'

k.

11

Titus.

K(X)

li-xi'l
Ii-1;

ly"l

ly"l

11- +,....::;<Y--7.,)- z.-l";n


"
1(1. y' )ll - :-:

At-cording to equati01ts 11.86 and I I.90. we can write ~lal

dl'
ds

( 11.94)

This is called the first Frenet-Serret for mula of differen1 ial geometry. We !lOW derive the
second or these fonnulas. DiiTerentiaLion orB = f x N with rcspecLLo s gives

dB

d"i"

- = d.<
d.<

+ T )(

ctN

= KN X N
d.<

ctN
+ T )( -

ctN

= T )( - .
d.<
d.<

........................... .. .....," ''"'

,_ ,

' ("'
)6c . .. l
ds
-

l( l

r r X r

'

;ui(//ICMC,
r(r)

r . r x r
lr x ;:p

The third Frenet-$erret l<~rmula expresses dNjds in terms of T and


Exercise 29.

(1 1.\17)

N; it is developed in

EXERCISES 11 .12

In Exercoses 1-5 find Nand B at eadl point on the cunc.

l+

I b) F = t 1 ,j is a \'<X lor that is defined at cad! poin1 P on


C. OcmMc by Fr and F,v lhC component~ of F in 1~ di

I. X = sin I. )' = COS I . Z = I, - 00 < I < 00

reCiiOJIS Tuntl Nn1 P. Find Fr and ( v as functions of t .

2. x = t ,y= l 1 z= t', 12:,1

(c) Express F inlctms of j und N.

3. x=(t- 1) 2,y=(t+1) 2 z =-t , -3:51:54

4. x + y

= 5. x 2 - y = z. from (5. 0, 25) to (0. 5, -S)


z =x. x 2 +y: =4. y?. 0, from (2. 0. 2) 10 ( 2. 0. -2)

5.

* 24.

Repeal Exercise 23 for lhc cu1ve C : x = 2 cos 1.


and lhc \ector F = xli + ylj.

= 6sinl. z

7.

x = 2- 5t, ) =
x 2 + y2 + z2 =

= lsint. -00 <

< oo:

z = 6 t 4/l, -00 < t < : (7. 0. 2)


z = j.r2 + y 2 directed so thot X increases

25. The ''CCIOI'S :il.and Bu-crc caku!aced aceach poim on chc cwve
1
1
.t =I, )' = t , l = 1 in b;11!1plc 11.46. If f' = 11i + 2Jj - Jk
is a "ector defint:d along C. fond Ihe compooeO!s of F in !he dirlions
t . N. and B. F..:cpres. r in lcr~ of t. N.and ii.

I +I.

K.
-1,
when y is positive: (I, 1, J2)

9. x = y 2 + I, z = .T
curve: (5. 2. 10)

+ 5. directed so that y increases along the

26. Calcula1eT, N,and li fotlhccarvcx =cost. y = sint. z = 1.


Express ~~e cc1or F
T xy'j k in1emlS of T, N.and B.

21. In this exercise we discuss ourclaimlllllllhe circle of <'W'VaiUrc is


the bcslfining cirdc lo Ihe cunc 111" poin1.

=4. .r = z. directed so that z dtcrca;c; when y

(a) Is i11mc 1ha1 Ihe circle of cur\'aiUre a1 a poin1 on n curve


p2SSCS tl\rougl1 thai poim?
(bJ Sho" 1lu11he cirdc of CUr\alurc and CUr\"e sbare !he same
tan~enr line .11 11'N:ir common fklirt.

In Exercises 11-18 find the cwvaturc and the t:tdius of cur.,.;oturc of


lhc t'W'\~ (if they C<151). Ornw eacto curve.
II. (.r -/1)' + L- k)'

= R', z =O,donlciAldcountcrdoclwO>e

12 1 = x~+ at. J = .lv+ b!, l; = Zo+ct.


C.lo. Y to- a. b. c-all constanu)
X

= I,

.1'

= t'. Z = 0,

-::o <

(c) Verify lhal d~e ctrcle of cun111urc

* 2S.

If ~ is Cbc a.~gle between i and


(figure below), show 11>a1

+ 14, :..' l'' CO\ /,~= e' sin /, Z = I , -00 < I < 00

and"""" hoc lhc 5llntC

curnuurc althcircommon pc'lilll

t < oo

?. 0

=xi

10. x' + (1' I ) 2


is negative: (2. 1. 2)

13.

'i' for a cunc in 1hc .f)'plallC

15. x=r.y=tl.z= r' . t ;:: O


16.

A -

2<.:o)/ , ,l' .._

2~nz.

z = 2bin/ .

l7. ,\' I + 1. )' ~:~ t l - I, l

18.,

- t 1.y

... ,. :-2t.

I+

~1

< 2tr

I, -00 <I < OC

:= t :=s

- 1

x +a'y' = a'b' (a>

2 2

19. Atwlichpoont>on thccllipscb

b)oHhe

c~llu~ a m:t~imum.. and a1 "'hich points is 1hccun'11WC"' minimum?

....--.......__ _".:!:

"""be

K(l) -

dyd1:r. _ tlx d'y


2
dt dt'
l d t dt

[e;r

(~~rr

29. The thi1\J Frener- ScTcl formulll iR

11. Show th.u 1hc l)nly Ctir\'C$ for which curvutur\! ls idc:nticolly equal

tiN
u' - KI..
-~r

IQ tCI'Q Ul\: Mr\&i ghl hnc5.

ds

22. WJuu happens to curvature at a poiru of inftcc1ion on the gmJ>h of


function)' = f(x)?

2J, Let C be. L11C CUI'\ C. X

J-r_t

10. Shuw ih<>tcUI'Oiurc fe><omoo<hCUI'-.: x ~ x(t). J - _r(1). a:=


I S fj. in lhc <Yrbnc
cxpre<scd in t~c fomo

= ( )' = T'J. in LJ\e: X)'-pl:inc.

(a) At each point on

C calculate tbc unit lanaau \tttor T :md

the principal nonnaJ

r.,.. N.)

= 2 sin 1.

r.

In Excn:iscs 6-10 find~ and ii at the point.

6. x
-!cos t. v
(2J2. 3J2. J2t .

)'

N.

Whol is

Jh

(Soo l:.xamplc 11.47

Verify this resull by showing that

N = 8 x T and tller'l calcularing

ciK fds .
JO. Show lhai JJ curve lies in a pbnc if and only if its IOI'Sion \"anishes.

780

Chaper I I

Vct.:IOI'S nnd Tlvte-Oimension:\1 ,\n~l)1ic Geonl(lt)'

111.13 Displacement, Velocity, and Acceleration


In Sections 4.8 and 5.2 we introduced the concepiS of displacement, velocity, and acceleration
for moving objects, but indicated that our terminology at that time was somewhat loose. ln
particular, we stated that if x = x(t) represe.nts the position of a panicle moving along the
.x-a.xis, then the instantaneous \'elocity of the particle is
dx
u = -.

( 11.98 )

dl

provided, of course, that 1 is time, and the accelemtion of the partide is

dv
d2x
(I = - = -.
2
dt

( 11.99)

dt

We illustrated by examples that given any one of x(l ), u(t} , or a(t} and sufficieru initial

conditions. it is always possible to find the Olher two. There was nothi ng wrong with the
lhey were correcl -

calculations in the examples -

btu o ur rerminolosy was not quite rorrect.

We now rectify this situation and give precise definitions of ,elocity and acceleration.
Suppose a particle moves along some c unre C in space (under per+taps 1hc influence of

\'arious forces). and that C is defined as a function of time t by the parametric equations

C : X

= X(l ),

)'

= y(t ),

Z = Z(l ),

I ;:::

0.

( I I. HX>)

The )>OSition of the ~niclc can then be described as a func.tion of time by i1s position or

displacement vector:
r = r(l) = x(l)i + y(t}} + z(t )k ,

1 ?. 0.

( 11.101)

The veloeity v of the panicle at any time 1 is defined as 1he time mte of change of its displacement
vector:

dr
( 11.102)
dt
Velocity, then, is a vector, and because of Theorem 11.8, 1he components of velocity are 1he
derimh-es of the components of displacement:
\1

=- .

dr
dx. dy. d~ .
- i + _. j + - k.
- dt - dt
dl
dr

( 11. 103)

\' -

Bot according to Theorem 11.10, the vector d rf dt is tangent to the curve C (Figure 11.11 7).
In other words. if a panicle is at position P , and we druur its velocity vector with tail at P. then

llrl

1[1& V<loxitv
il 1.lways La;ngenl tu curve along
'IA>bich the particle tr.avcls.

,. ls tangent to the trajectory.

In wme applications it is the length or magnitude of velocily thai is important not iL'S

direction. This quantity, called speed, is therefore defined by

+ (dy ) + (dz)
(dd x)
t
dt
dr
2

lvl =

( 11.1().1)

Equation 11.79 implies that if S(l} is length nl011g lhc lrajectory C [where s(O) = OJ, then
Ivi = d s I dt. In o1her words. >peed is the tirne r'llle of change of dis lance ua,elled along C .

It is imponant to underswod this differeoce betweeo velocity and speed. Velociry is the
time derivative or displaceme.nt; speed is the time derivative or distance travelled. Velocity is a
vecwr; speed is a scalar - lhe magnilude of ve locity.
The acceleration of the pal'licle as it moves along the cul've C in equations 11 . 100 is deli ned
as the rcu.e of chaog:e of velocity with respect to time:

a =

dv
dt

( 11.1 05)

AcceleratiOel,thcn. is also a vector; it is thcderivativeof ,c:locity. and thererorc its components a_re
1hc derharives of the comJXlnc::ntS of the veloci'ly vechJr. Alternatively. it is the:;ec(.'lolld deri\'ntivc
of displacemc.nt Hnd has components thou nrc the second deriq u ivcs of 1hc conlpotlcms of the
di~placeme.u ' 'tCior.

Ill lite s pecial cil~e i1'1 which C is a cur..e i1l the X)' plalle, defi nitio1B of displaccnt~H ,
velocity, SJX."-'tl. und actclcnlt ion bec:omc. rcspecti\cly,
r

= x(t)i + y(t)J.

( 11.1 06

dr
dx .
dy .
- = - i +-j.
dt
dt
dt

v=

(~~r + (~~)'.

11 =
a =

I II 106h l

( II

106cl

(II IOMI

dt

For motion along lhe ,\-axis,

(t)l,

( I I 107al

dx.
,. = -dr
- I
=
dt ,
dt

( I I IO?bl

I'

I I = ldxl
dt '
a

dv

= dt

( II 107cl

d 1r

--
d 2x,

= (/t l .
dt 2

(1 1.107d l

If we compare equations 11.1 07b and d ''~th equations 11 .98 a11d 11.99, we see thal for motion
along the x -axis, .t (l). v(t ). and 11(1) arc lhe conll>oncnts of the di SI>Iacen'ICnt. vel ocity, and
acceleration vectors. respectively. Oecuse these are the only components o f r (l). \'(I) . and
a (t ). it foll ow~ thnt cm'lsidcratiOl of the components of the \tctors is ccluivalc,lltu colsidc.n \tion
of the vectors thcmsclvC'.s. M.tr onedimensio\al motion, chen, we can dmp the vector notation
and work with components (and this is precise ly the procedure that we followed in Sections 4.8
and 5.2).

Newton's second luw ~:ribcs the effects of rorccs on the motiOrl of objects. h Sll\tC..~ that
i f an object of mass m is ~ubjl.-ctctJ 10 a fu rce .F~ then the time rate of c hange of its momentum
(111'') is equal to t':

}' = .!!_(mv).
eft
In

m~l at.se~s. the mal)~ of the

( II 1081

object is cunMarlt, l lld this equatiOl then yields its acceleration:

Jf I' is known as 11 func1ion of time

1.

dv

= m-

dt

J:'

= ma.

(1 1.1091

= I:'(I ) , 1hen 11. 109 defi nes 1hc accclcrotion of the

object as a function of time,

a (t}

I
- F (t},

and intcgr.ttion of this equation leads to expressions for the velocity ' '(!) and position r (t) as

functjons of time.

I EXAMPL E

, 1 .60

A projectile is fired at angleD to the hmizccnal w ith speed v0 (Figure 11 . 11 8). Find the distan<e
R from tllC firing place that the projectile strik.es the grounLI (called the mnge of the proje<.ti le).
What is the rna.t imum height attained by the p rojectile?
I;IAIMII
)'

------ R
SOlUTION Since the acceleration of the J>rojectile is dv j dl = :a = -gj. its velocity is
v(r) = - lid+ C. Tr we choose r = 0 :11 the inStant the projectile is fired, then v(O) = v0 =

~.~ocos9 i

+ Vosin ej . aold therefore Vo =


r(r )

Siolc< r (O)

= 0, it foliO\''' that

dr

v(O) = c . lntcgnuion of - = -gt j + ' 'O g ives


dt
I

,.

= - ;;tr j + vol

+D.

= 0 , and

The projectile sLrikcs the ground when r (t) =

Ri, in which case

\Vherl we equace comjX)rlcms.

11 =

tJuCOS81,

T he second o f these implies that 1 = (2uo/g) sin II , and when this is sub<;tituted imo the fi rst,

11 = vo cos O (

211Qsin8)

v~sin 28

The projectile mutins ma.timum height when the y-componcnt of its velocity is zero. 0 =
-gt + ~.~osinll
1 = (v0fg) sin 8 . The hcightof theshell auhis time is the y-componcnt

of it!' c.li~placemc nt.


I
- -g
2

(vosioli)

.
+ vosm
e

(vosinli) =
g

11. 13 Oispli'tcenl trll. 'kiOc.:ity, rmd Acceleration

I EXAMPLE

783

1 1 .51

A particle s1arts at time t = 0 from position ( 1, I) with speed 2 m/sin the negat ive y ..d irection .
II is subjected to an acceler:uion that is given as a function of time by

a (l )

+ 6 /j

17"7> i
vi + I

Find lcs vclucity and positjon ns functions of time.


SOLUTION

If a = d ' fdt = ( 1f .Jf-=i=) ) t +61j, then

,, = 2Jt+ii + 312]

+ C,

where C is some consla nt vector. Be<.-.ause the inil i.al veJocity of the prutkle is 2 nlls in the
ncgmivc y-direction, v(O)
- 2]. Consequently, - 2j = 2l + C
C = - 21 - 2j. The
ve locily. then, of the partic le at any time 1 ~ 0 is

v(l) = (2 v r -r
Beca use "

1 -

.+ (31"' 2)j.

nl/s.

2) i

= d r / d 1 , integration g ives

Since the pa rticle starts from position ( I . 1), r(O)

.
.
i + j

"' i + ],and

~-

= - i + D, or O =
3

I.

--i + j .

The displacert~ en l or the panicle is therefore


r ( t) "'

I EXAMPLE

[~(t +

1) /2 - 21 -

~} +

(13 - 21 + I )J m.

11 .52

The mu:;s M in Figure J I.J 19a i:; dropp<XI fronopoint 8 . Show !hal i f a nu~:;s 111 is fired from
a ny posit ion A dircclly a1 M a t the ins!<ln l M is released, m. will always coJiidc wi th M .
M#l[clll;lj lll)1iTW

MiiCCJII;IAII~

Mass

m is tired t'm m A at mess M al


8 <'IS M is dropped

Coor-

d i illll s y:sa~ m !'.l Moti)'7..C mo1io ns

of

nu~ ~ses ttl

nnd M

r
. M
Ill

,\ "-- - ' - - - - - - -- - - - x
h

SOLUTION We choose !he coord inate :;yslem in Figure 11. 119b, a nd lake time I = 0 a t !he
instant both masses begin motion. To show that the masses collide, we show that they have

the l>Utu~ 41iwlu~ent~fll vector l'or Mime time 1 . The oc.'\...:lerruiou o r each '"'' "II i<i. a
rmcgrntun of th1.s gvcs \'Citx:ilies v l' thC n~si!Cs,

"' = -'J.SII j + <..:.

== -9.8 ,~,
J

- 9.8 1/} + 1),

vw e

~inc.<>e M l~ dropped. iL" initWI \clocit) ii LtTO. , ,,(0) - 0 . This imphc.s lhlit I) = 0 . tf,.
IS fired at '"'.fie 8 with ~ v > 0. then \',.(0) e v roll 8 i - u ~in 6 j Thh, i mp4ics. t1wt
C

IJ co~ tl

l+

d r,"

Tt

sin f) j . lllltJ1'fttion~ of

,.,

_.

-t).8iiJ+vco~6i+tJ'i'l0 j

drM

and

dt

give

r.,. :::~aH:"C,.b-0t i - (-4.90$1 2 + t'S:in OI)j -E

and

ru

= -4.90Stl )+ l' .

= .. 0 and r,t~(O) "i + Hi. we tlh~t~i n E ~ = 0 and P hi + nj. -n,u5,


= '' COsO tt + (- 4.9051 2 + U:\il 8t)j und , , h i + (I-/ - 4}X)51l )j. Tllcma~

Since r...,(O)

r,.,

collide if nud ..-..hcu

Whe-n

\\'e

equate eomporlCil'-S,

from which

H
usin9.

I =

Thes.c art c;.'()tnpatible ~incc 1.\Jl 0


H / h . 'rhe
irl(.Teases, u decreases, and/or Q incl'ea~s. .

tin1c-

for collb.i1;n lt' occur incrca~.s ~~ lr

Tangen tial and Normal Components of Velocity and


Accele ration
Por some ty)>C:\ of n1otit)t1 it j.; incon\Cil ient to ~XI'I'C."" velocity w\ll tiCCclc muon of a I'Mnicle
ir1 tc:nul) of Ctmcsian componCIIL'i; :oomctimc~ it ~~ a11 a~l\taJll<~gt to rtl\OI\'C thc='c ,ccU.Ul\ intu
lhUl ore tnngcnl and llCM'Il131 tO the f"tllh of the p.1nicle. \ Vhen the trajcttory C of U
pan.i<:lc is , pecilirtl a> a fun1<>n of time I by I I 100. ib vclocil) ' - d r (ti t i< llln&~l l<l C.
and we can chercf ore wri1e
COnlpotlCtlfl\

I l

10)

In oLhcr words, the tangential component ot' velociLy is speed. l\nd v hil~ 110 aJmponcm
normolto 1hc lmjcctory. Diff'cr'Cnlitllion of !his equation gives the p<rrticle's uccelcration:

dv

= -dt

d'f
1
= (d
dt 1' 1)"
+ 1' 1di

lttt\) ld'J
dT/dl
'fdll

d 1 1) (
= ( dt
'' T + lvl dr

(:
1

1vi) T +

(11~~~ \).

(1 1.1 11)

Mj!tcjiJ;If'WI(B!J
~totm al C() IUJ")t~nu:

Tangential "'111..1

lli Ut;:~clc:til tiol'l

w e ha\'e therefore expressed a in temu of the unit t..o:tngcnt \'CClor j~ to C and the principal nonnal N (Figure 11.120 ). We call c/(lv l}/c/1 and lvlldT/ d t l the tangentia l and normal

<:omponems of acceleration, respectively. If ar and a1y dcnou:: thc:sc COOlJ)OJ\CillS, we catl write
that

( 11.1 12a )

where
y

t~r=a i~ =

t iN

= a N = lv

I"TI
dt

( 11.11 2b)

Note that lhe tnngeminl component of accclc rt~lion is the time rate of change of speed. Since
acccletmion is the rate of chHnge of vdocity. tl'lle normol component or ac.::cckrut ion mu$1 deter
mi11e 1hc nue: of change of the dircctiOt) of v. \ Vhat is si~ni fic.tnt here: i...;; that a is e.xpressed itl
terms ofT and N; it is not necessary to use the binomml 8 . The act.'Cierati()n \ector of a particle
is a lways in the plane of T and N.
To c:akult-1tc liN using 1 1. 11 2b is oflen quite complicated . A far c.asicr Jornmla results if
we take the s t'Siar product o f a as defined by I 1. 112a with it;elf:

2 ., . ., .
aT

+ 2CJ)'(IN .-.I N.. + a-,2v 'N' N.

= a~ +a~ .
since T N = 0 and T i' =

N N =

I. Consequently.

and because tl N is always positive (see cqualion 11.112b).

a.v = / la12

a}.

( 11.113)

Kepler's Laws for Planetary Motion


Based on 'Jycho Bnthe's ( 1546- 160 I ) stron<Jmical mcusuremcms, Johannes Kepler ( 1571 1630) postulo,ted three law~ of planetary motio n. The fi rst slates that planeL" move. in e.lliptic
orbics with lh~ sun at o)(:: focu:; or che ellipse. This can be prt.wt..1(1 with Ncwt<Ul'S second law
and Ncwcons uni versal law of gravitation. If Jll is the rnass of a planet, ~ewtons second law
requires the tiCCclcration of the planc1 1o suisfy F = ma. where F' is I he resultant of all forces
acling o n the p lmtct If \ve assume that the only fo rt.-c acting on the phmcLis the force o f attraction
or the Still. with lllliSS M' !hen F = -(Gm M I r 1) i . where is I he unit vector illlhe d it'CCiiOll
from lhe s un 10 Ihe plane!. 11 follows then thai

GM .
a = - -,-2-r )

( 11.11 4)

and the acceleration of Lhe planet always poil!lts toward the sun. Let u.s choose a coordinate
sys1em in s pace wilh origin m the sun (Figure 11. 121).

~fotioa

lijC.li);URJ tEJll

of planc.t around sun

c
,.
y
b

Planet
,V

Pick any two pointS on the orbit ofll~e planet and let these point.s and M define the xy-plane.
We first show that ~~e orbit of the plnnct always lies in the .ty-planc. According to property
11.59c,

d
dv
- (r x v) = r x dt
dt

dr
x ''
dt

+ -

= r x a + \' x v.

But v x v isalways the l.<fO vector. ond .so is r x a ( r a nt! a are parallel. see equation 11.114).
Thus,

tl
d1

- (r x v)

= 0

r x v = C,

(11.11 5)

wt~ere C is a constam vector. But this means that the tlisplacemem vect()( r and the velocity
vector ,. ore nlwnys perpendicular to C. In other words, C is in the ~- direction (or negati\'C
z-c.lirection), anti r and v are in I he .ry-plane; the plane1 moves in rhe .ty-plane. To show 1ha1
lhc planet follows a n d liptic path we find a fo rmula for r which represents lhe polar CQOrdinllle
in the .r )'plane. First we note that

d
C = r x v = (rr ) x - (rr) =
til
= r tir
- ( r x r )
dt
= r2
If we cross this with a
a x C

rr x ( tlr
- r. + r-tlr)
til
til

+ r, ( r

dr)

tit

(r: x ditit: ).

= -(GM I r 2)r.

M x [rl
- G
-;:r-r

(
r

dr)] =

x dt

[- (r x df)]

- GM r x

dt

We now use Exercise 46 in Section 11.4:

, G"[( dr). . . dr]

ax C = -

"''

r -

dt

r - (r r ) -

dt

Diffe.remiation off f = 1 with l'espect tot gives

di

di

r - + - r =O
dt
dt

,., dr

r -

dt

= 0.

11. 13 Oisphk.i:mtnl. Vdu.:rty. ~nd Au.:der:uion

787

Thus.

di

dr
GM d
-

axC=GM Jnccgn1tion wich respect to 1

dt
yields

x
where b is a constaiH vector. Since v X

c=

-(v x C) .

dt

GMr + b.

c um.J r arc bolh in the X)!plnnc~ so is b. Suppose lhC

.\ axis is chosen alo' S b (Figure 11. 12 l ). The doc product of with the equation above g.ives

r (v x C) = GMr r

+br

= GMr

+ lblr cose.

Thus.

r =

r (v x C)

GM

Since r (v x C) = (r x '') C = C C

+ lb lcos l(

= ICI2 it follows that


ICI 2/ (GM)

=
If we set = lbi/(GM). andd

-,+ ...,;ll""'b1:-:'/(:-'::
G::-:-M-:-:-).:....
1cos----:()

.,I

= ICI 2/Ih l. thcn


~d

r =

(11.116)

Accord i1lg to equation 9.34a. this is a con.ic section w ith the origin as a focus. Since planC(s
are kno""' to follow closed p<lths. the conic sect ion mu!<l he :111 elliJll'l!. The second ~tnd third or
Kepler\ laws are lliscussed in Exc:n.:ises 45 a11d 46.

I EXAMPL E 1 1 .5 3
i\ panicle isconlined to mo \'e in a circular p(llh of radi us
if a nd only if its position vector is r =
+ where

xi J1,

+ R cosw(t}.

R ;md centre (/r. k) in the .t y -plane

Rin w(t}.

and w(t ) is son1e function of rime / . Determine the fom1 of w(t) if the n=lerntion of the

particle is directed mdinlly towtmlthc centre or the circle.


SOLL'110 N The principal normal N at any point o n the circle is directed toward the centre of
the circle-. Hence, the acceleration must be a lo ng Na nd the tangential component must vanish:

d
0 = ar = - lvl
dt

=>

Ivi = C =

constam.

Si nce '' = - Rw'(t) sin w (t )i + Rw'(t } eosw(t)J, it follows that C


I

(I)

(I)

= -

W(l ) =

= lvl = Rlw'(t)l . Thus.

Ct

R +D.

l.n olher words. w (I) must be a linear fu nction of 1 for acceleration to be directed r.td ially toward
the centre of the c.i rc.lc.

Consulting

..

The mcwl roof of a structure is a hemisphere wi th a liirl:C m~ius ct = I0 metres (Figure


11 .122u) : the bottom of the roof i' II = 20 metres alx>ve the ground. During the winter,
large chunks of icc that form on the roof break on. slide down the roof. .ul<l fall to the
ground. An annular empl) t.one 11'1 to be cn:alcd on the: ttround in ordc:r to prc,ent hunlan
injury or f'r<lll"rty damage Our problen1 is to detcmune minimum width for the >.one.
SOl 11011; Once a chunk of ice breaks off. it picks up speed as it slides down the roof.
and omy leave the roof before it reaches the oouom edge. In order to determine ami nimum
radius for rhe empty zone, we ~hall find the furthest poillt at which ice can be expected to

hit the iroond. Chunks tht scrikc the ground farthest from thc roof arc thoc tht aoain
the gre.UCllol speed on the roor. ~mcJ therefore leave the roof earliest We ~hall a....:\ume that
there"' no friction bctwcen roof and icc in order to m.aximiu_ speed ln addition. chunk.s
that aurun gteatest speed arc one< thai. lidc from tile ''CI'Y lOp of thc roof.

lloof

I
I

II

AiKftlfM
)'

a
(x. VI

/
/

'

'

e
a

We consider. lhen. a ma$S m of ice starting li'om resl m tile top of the roof as it slides
dol\ n the circle in Figure 11.122b. It is acted on by @rfl\ ity and the reaction or lhc phere.
A~ long as the mass is on the roof. the r~a<...1ion of the roof on the mass i~ perpendicular
10 the sphere. h seems rcll'lonublc. 1hcn, to \\Ork with tnngemial and normal componem.s
of nl<llion 10 the sphere (or circle). When the mass i~ at position (.r, )'). tangential and
normal components of the weight W = - 9.81mJ are

Oisplxtmtnl, VtkJcily, rud Acctlfflllion

II 13

781

W = 9.81mcosii T +9.81msinON.
If the rcactilln or the ,phere on m is denoted by N
nulgnitude, then the tota l force on m is

- NN. \\ here N i' therefore il'

F = 9.81m rosO T + ( -N "'9.8 lm sinO) N.

1'l1e rna<.< lea'~ the &phere when N = 0 . To find where thi< happen<. we u<e
Ne\\1011 's ~nd low F = ma with F s abo,e, od a given by equation 11.111,

. (d vi) . (Jvl -di' ) .,

9.81mcooOT + (-N + 9.81msinll)!\ =

111

dt

T '"'m

dt

~.

When \\c equate componc:ms.

9.81 cos /I =

Now..1

"'

d
dt

- I" '

- N +9.81mstnll =

mJvJ

d i'

cit

a cosO and)' =a sin 0, so that

Hence.
9.81ros0

= :,

(-ll:~)

Muhiplicution by dll/ dl gives

dll

d(de)'
dt

d'e de

-9.81 cose - = a-1 - = a


dt
dt dl
2 dt
and we may integrate with respect to t.

-9.81 sin II

= a- (dll)'
+C.
dl

Since diJ/ dl "' 0 when II = rr/2. it follows that C = -9.81. and

-9.81 >in II = -a
2

(dO)'
dt

9.81

19.62

- (1

-~i n II).

To tuck le the nonnal components. we first calculate that

d
.
.
M
T "' -(acosll, asmll) = a(-smll, cosll) dt

di

T=

(oinll. -cosll).

Thu,,

dT
dO
- = (cosll, sinti) -d
dt
I

di"

dll

SubSIIIUiion

1010

-N

+ 9.81m sinli =

mlvll<ff/dtl gvcs
1

-N + 9.81m sinO = m ( - a dO
dt ) ( - dO)
dt = nw (d0)
dt
Replacing (d0/dt) 2 by (19.62/ a)(l- sinO). we olxain

N = 9.81m(3sinll

9.81m sinO = 19.62m(l - sin9)

2).

Clearly. N = 0 when II = Sin- 1(2/ 3). aod !his is the aogle at which the mass lea,cs
che <phere. h< <peed ac chis poinl is

I = _ 0 d8 =a { 19.62 ( 1 _
dt

!)

/ 19.62a.

Once the mas:s: leave!i: 1he roof. the only force acting on it is gravily, ;md therefore iL~
acceleration is

dr

dt

= -gtj

+c.

If we choose timet = 0 when 1he mass leaves 1he roof, !hen its velocily al this time is

y[19.62{;
~(sin Oi A

cosOj). where angle 9 is deflnC<I abo,e. Tf wcdcnulc chi> b)' vo. chen

(.; = v0, nnd in1cgnuion of


dr

-dt = -

gt j

+ v0

,.

r = --gt j + vul + 0 .
2

give$

Since the initial (X)Sition of the mass when it leaves the TOQf is ro = (I co:; oi + a :sin oj,
il follows chm 0
r0 , and che position of che mass afcer il "'aves chc roof is

111C """' hiL> lhc gr"und when !he )-<:Ornponem of r(l) b equal Ill- H .

- H =

.
--I r-, - J19.62a
- - - cosO t +a smO.
2"'

When we \ub>ticuce a= 10. H

= 20. sin 0 = 2/3. and cmO = ./S/3. we obtain lhe

quadracie equation
4.9051 2 +

109
80
( = 0.
3
3
.

the lllhiche solmion of which is t = 1.797. 1he x-coordinale of the rna" dtthh time i\

(2)

11%.2 3
,13

(1.797)

( j5)

+ 10 3

= 17.14 Ill.

This is I he minimum rJdius of the outer edge of the empl)' L.One. In other WOI'<.b. il mu)l

be 7. 14 meues wide.

11.13 Oi$pfaceman. Vckx:il)', :lf.KI Ac.-..:'Cierution

?91

EXERCISES 11 .13

In Exercise~ 1-S find the velociL~. speed. and acceleration of a particle


if the g.i~tcn equations represent its po!tition as~ function of time.

1. ,T(l) =

Jii'+l, )'(1) =I ./1 2 + I.

I~ 0

16. A particle tr.a,c b oounlcrclockwisc around Lhc circle (.t - lt) 2 +


(r - k) 2 = R 2 in the figure below. Show tllat thc speed oft he l':uticlc
ut nny time is I\'I = w R . where w = tiO/ tlt is culled lhc angular
spte(l or the particle.

l, X(l) = I + 1/ 1, ,1'(1) = I - l j t , I ~ I

3. x(t) =sin/, y(r) = 3cos1. Z(l) =sin/, 0:5


4. x(t)

=1l +

5. x(r) =

I, y(t)

=2te

e-'' . y(r ) = rlnr ,

Z(l)

t(r)

= 1/ tl,

= 5, 1 2:

:5 1011'

I :51 :55

In E.xerci~ 6-7 a particle ut (I~ 2, -I) starts from rc$1 ut time 1

a Y)

0.

17. AIXlrtidcl.l'"'d''c lsaroundthccin;lc.t 2

+ ,\1 = -&.countcrdockwisc

Hnd ots position as a function of lime if the given fWJCtion dcl1nes rts

m constant speed. making 2 r>:volution> each SC<'Ofld. If .r and y are

xcekrJtion.

measured in metres. whal is lhe \'clocily or1hc p.1r1icle v.Jfltn it is at Lhe

6. a(!)

= 3t 2 i + ( 1 +

I)J - 41 3k, I ~ 0

a (t)

= 3l + J/(1 +

I ) 1, I

7.

poin! (1 , - ./3)?

18.

(a} S hiJ\'-' lhal iran object tnOVc!t with constant speed in a cir

cular path or radi us

~0

(b) If a satellite mcwcs with constant ~(k.~ in a circula.r otbil


200 km alxwc the cartJ1's surface, what is its ~peed ? Him:
Usc Newton's universal law of gravitation (see Exe-r cise
32 in Sec lion 11.3) to determine the accelemtioo a or the
satellite. Assu n~ that the curth is a sphere wi1h radius
6370 krn <rnd density 5.52 x 10 3 kglm 3

In Exercises &-9 find the Hlngcmial and nonnal components of a<:celeration ror a partick moving with position defined by the given fuuctions
(where 1 i<tin\C).

8. x(t) = l, y(t) =t' +t . t<::O


9. X(l) = COS/, y(t) = sin / ,

R. the magnitude of its acceleration is

(nl ~ (vi2/ R.

t,

I 2:0

10. Show lhat the normal oomponcnt or accclcr.uion of a JX!nide can


be c>pressed in the form a,v lvi'/P KIYI'.

+ 19. i\vo pattidcs move along curves C 1 and Cz in the figure below.
If at some instnn1 of time the p.lrttcles arc: nt positio..-u P1 and P2 , then
the vector P 1P2 is the displacement of P2 with respeclto P0 Clearly.
OPt + P t Pl ~ OPz. Show llml when thiscquotioo is diiJtrcntilc:d
with respect to lilllC'1 we have

II. Fond the kinetic energy fo r each panicle in Exercises 1-5 of its
mass is 2 g. As:,.tmu: that x. y, cmd t etre measured in netres <old 1 i.n
seconds.
12. A particle starts Hl tile origin anti UlO\'~ along the cur,e 4)'
10 lhc poim

= .t

(4, 4) .

where V1' 10 and ,. P-;/ 0 3re velocities of P1 and P2 with I"CSpCCI IOa.he
origin. and v P:/ p 1 is the ve1ociLy or P, with rc~pccl 10 P,. Can lhis
t"quation be rewritten 10 the rorm

(a) If the y-compooe.1\l of its nccekration is always equal to 2


and Lhe )' -component of iu ''Clocity is initiall) zero. fi1HI
the :r -component or its accclcr:llion.

(b) If the .r<omponc.tn or 1Ls acceleration is cquallo 241 2 (I


be ina Lime) and l hc x -component o( its velocity i-.: initially

1cro. llnd the ycomponenl of it( accelcra11on.

13. A panicle mm'C.~ along the cur\'C .r(I)


t . y(I) = r l - 3t 2
1 ~ S 1n the .\')'-plane (where 1 is time). Is there any point 31
'"'hieh its velocity is parallel to its dis:pla~menr?

2t , 0 ::S

14. A p~rticlc moves along lhc

C\lf\."C

y = x l - 2x + 3 so thai i lS

.t ~llJXMlcnt o r" velocity is always equal to 5. Find iL~ accck:!ation.

IS. lf a particle stans at time I = 0 from rest at position (3, 4) and


experiences an ac:celei".J.tjoo a = -5t 4i - (2t 1 + l )j, find iLo; speed at
I = 2.

,t

* 20.

A plane nics on a course N30('E with airspt,:ed 6.50 km/h (i.e .


the speed o l" IJte p laoe relalhe to the air is 650). If I he air is mmi1l8
at 40 km/h due east. fi nd the ground velocity and speed of the plane.
Hint: Use ..xercise 19.

1 1. A pbc fti<S with peed 600 kmlh in "ill oir. Tho: pllft<;.. to fty in
a str.M&ht ltn<: IroniC tty A 10 city B. " here 8 i> 1000 l.m nonh-.C>t
of A W"- <hoold I'C its beanng if the wind is blo'""R from the u.N
SO Lmlh1 11ow long will ~"'trip take~
11. A ' ttaie,ht river i~ ~00 1n wide: a.nd the "'-aler ft~-s :11 3 ktnlh. I(
)OU can paddle yOW" cunoe ut 4 km/lt in still water. on what dtr.:ction
should you paddle if rou \Vi._h the canoe to go straight aero<( the ri"cr't
llow long wi!l tt tuLc to cro>S?

. ,. 23.

(a) In the figure bcluw u cwmon is 11rcd up an inclincU plum:.

If tho ;:peed ut which tl1e b311 is ejected front the cannon i<
$,~how th.stthc runge

R or 1hc ball i~ gh-cn by

30. Tho: Enslish lon&bowin medic,..! times """~lobe~


at 100 mor more. For an arrow 10 tr\-elaiiOO<Mtal ~i>WlCCOf 100m
with maximumhcilht 10 m. find the initial 'f"<'CCand antleof projectioo
ofth e arrow. lg.non: u.ir (ric-tion.

J I . The block of mass M in the h2ure belo"' slides on a thin film of


oil. The 6lm Lhickncss is h ont.lthe urcao(thc bJock ;,, rontncl '"ith the
fllm is ~>1 . When released. ma~s m c,;crt~tt lc:nsion in the cord. c-ausin&
block t\1 lO occclcratc. \Vhen the speed or JH is 11, the \'iSCOUS fcc
acting on ildue 10 tl>e hlm os F = 11 Avj h whcrCJL is the iscosityof
the: oit l~n d the speed o r l\1 ~ Q functioo of time 1 . 1\c.gJoct friction
in the pulley and ai r rcsi<taocc.

R = 2S2 co:.O in(O- a).

g ('0$2 Ct
(b) Wh:lt onglc

0~

-.here 8 is lhe accdcr.uion due to sra,ity.

film

9 nuimi>cs R
32. Water is.sucg from &he noulc of a fire hoQe ::u s~d S in abe figure
below. Show lhallhc m4lotnum hei&hl auaonahl< by the w:ua on lhc

building i given by (S- tt 1d 2)/<2!1S1). "ltcreg i>the occ<:lmtioo


due to ~" it)'.
Duildit'S

rxlrhclc c'<pcntnce .r 11 ,,. to


along the struoj;ht line )Qining the

24. Whal con ~t:m1 :.lC\!cltrJtion must a

tm,cl from (I, 2. 3) to

p, 5, 7)

point$. $UU'ting rrom rc.:K und covering the distance in 2 unit) or tinlC'?

25. Calcularc the ooronal component av oftheaca:lcmuonora panodc


~~>~ng~uliorb II 112and I UI3ifibpo:.itioni>i'enb) .t
t'+ I.
>- u' - I. <_,, +5t.l ~ 0 (1 bcingtimc:).

26. A J'311ocl< moves along the tun e X(t)= 2 + ~. J(IJ = I


0 "S 1 ~ 1/l.whertll~llme.l~ther~arimeatv.1Mchtt~accdenllm
o< pcrpcndiC.Uiar to its '"locity?
27.
(I) Show t11o1 roocion along suaigbt linc is tbe rcsuk in boch
of the followln,g: situations:
(i) The initial '-elocity is Ltro. and the ooceleratioo

33. A boy stands on a cliff 50 on lti'h that o'~rlool.> orhe 85 m " i;le
(fo~uro below). lfhee..,tl11\1wa >IOOC:at25 to/>. can he tbr"" itacros:.
the rivofl

b~constant.

(ii) The i.nitial velocity is noozcro. and the accclcr

mioo is constant :llld parallel to the i1\itial ''t~


locity.

I hi Can we 2Cotcmhze t11c rtSult.S or pan (a) and <tate thJI oon
(l:tnl :tCCClcr.tllon produces str:ughtlinc: 1'001100 ~

lll u~'lr:Uc.

I
50 \

_lj

23. A p.utide ~u~ frum ~ition rl) = (x0 Jo- lo) .s11jm.: 1 - lt1
~ith \Cie<h) v0 . tr il e:\pcriences con.s1an1 acce.kr41.H.Jn :. . " lO\Il' that
r

r0

Vo(l - to)

I
I
-a(t - to)

29. A looJJcr 8 m long h it> uppo:r end Olaiost a \tluc.al "all ond its
11)".\CI end on .t '"Joril.Oflta1 noor. SutJI)O:'IC lh;a the ~CI end )..lllb ~OS)
from the waH ;~.1 cm".ant ~d I mfs
(a) Find lhc velocity and acceleration or the middle point of
the lndde when the foot of t..he ladder is 3m n'Om the wall.

(b) Hc-.w faf.l doc.'i the middle pointoCthc laddcr~trikc the llo01'?

- - - -85---1
J.l. A g.oltcrcan dri\C3 ma..'tia'll.unofJOOm 1n the lllrOtl !!lc\d fciN."':ly.
fr'Om the Itt in lhc: f'i,:urc bciO'N. e&., he C\p.x:t \0 dear the ~m?

Tee

Oi1pl::~~:e nc,u,

11 . 13

JS.

(a) A projectile i.s htud :JianglcO to the horil'.ontal rrnma height

R "'

sin

rolls w th:u iu: centre ha1: <.'<'111'1..11H speed S.


x = 0 ~tltim c 1 0.)

(b) \'v'rn:t ang_lc n-.aximi"tc.S R

lh:11

nccelct'tUion?
.. 4().

(Q t'

illhktc:. Whut i:; the angle in


2.25 on'/

given v and II '?

p;,., (b) i l' v =

+ R ttlll 0 )

and

c~

in the

fi ~urc.

below represent C'f'OSSSCCiiCins or

le ft cylitxkr remains stationary while the right one


aolls (without !>liJ>t>il'8) around Lhc left one. and the cylitldcrs alwo.ys
I'CIU:Jill in c:m\ltlC't. If LhC l'ig.ht <.:ylimter pid.~ UJ) 0t ~p.;:d O( ditt at
tx:>int (R, 0). tl\c palJl that the dirt traces mil dunng one rC\'Oialtioo is a
u uxtioitl.

13.7 m/s tmd

n~e

(u) Show that p:tnnlctric equ:uj{)j\S fotthc: c:-anljod are

(d) Pfovc that the maxi1t1um height aunincd by the projectile is


R1 t;,m 2 0

4(/J

Circ;lcs

two cylinders.

(c) Suppose the projectile i'\ !l shOI. thm wn by un O lyn'lpic

h+

(A~m c

Cb) Wh;a.n urc nonnal antl lnngcntiul components of the uonc's

Find vclociL)'. speed. and accclcrJtion of the s:tonc if the Lire

(3)

"

:-~
I

JXU.h is <.'31!-cd a tmd w id (:14!c f:XCI'Cisc 58 in S ocli()O 9.1 ).

2~h
0 + -"-,

g
where g "'9.81.

793

" 39. Tr the stone in E'<crcisc 38 is C11ll)eddod i11 the side of t.hc ti re. it&

h n.bovc the ground with speed tl ( llgurc below). Show lhat


the ungc N of the projectile is gi"cn by the fonnula

v' cosli (
- - - <in(/ +

Velocity, ;~nc.l Al!l!clesoiW.l

= R(!co;:O -

cos20).

y = R(2 sin 6- sin20).

(b) Veri r) that if dle point of coruact moves Ill COfl..mmt speed
S, "'ith 1 ~ 0 " hen the ,:;1x:ck (ll' dift i~ picked UJ) 1 thcll

36. A c:::ulJl.OI'I is locmcd Ot'l n pi :Inc inclined '" ll'lglc ex to the hcv'i~ont al
(figUI'C belo w). lf a pt'ojcctile is fired from the C'MOOilat :tng.te fJ to the

0 = .1'1 j R.

J)lunc., prove thtu for the pmjcctilc to hit the plru'IC horiwrua:lly,

(c) Find ,clocity, speed. nnd accckration o( the st>cck of dil't.

/.1 "'Ton '( sin2a

3 - cos2u

)
(d) Whi.lt are norn"d <Jnd tangential cmnponcnts
nccetcnuiorr?

or the din\

t- 37. If r is the position vector o f a p3ttide Wtlh nlaSS m ll\O\ing under

1.hc a<:t.iOR or a IQr(e F. rnc tM tuc or F about the (IC'igin ls r = r x "".


The i.ln8''1:1r mon..._.,hml of m '1bout 0 i5 dell ned us 1-1
r x m v. Use
Nc'-'ton's second law in the fornl I I. lOS to show thou r = dHf d l .
}8. When a :c;tonc is embedded in Lhc tread of u tire and the tire rolls
(witholll slippi ng) u!ong Ihe .1{-axis (llgurx: below). lhc IJlllh llml il tmces
i.s called u (;ydoid (se-c Exurnple 9. 7 in Section 9. 1).
(a) Verify thuc if the cenu'C ofchc tire moves at constanl st:x..'Cd S.
with t = 0 when l11c stone ib al diC origm! then 0 = St / R ,
(b) PUld velocity. speed. and acceleration or the stone at any

time.

(c) Whut arc the normal and lanJ;cnlial wmponcnls o f the

.t.

4 1. SI'IO\v lhatl hc path of a J)tt.tli<'lc lies on a sphere if ats diSJ)l~mcnt


und \'Ciochy are: ut .....ays pcrpcrkficulur duri n~ liS motion.

* 42. SUJ'IXlSC in Excn;isc 22 that bcc::Luse of an inJured elbow. you can


puddle only at Z kuVh. lVI"" should b< )'OUr heWing to tnl\'cl suuight
10 01 point L kitomclrcs dov-nsucam on l.hc C.pJ.OSitc shore? Arc there
any rcblrictions on L?

stone's acc;clcrlllion'!
)'

43. Suppose that positjl)O vectots of a syste1n of n masses mi al'e


dc.nc)tt!d by ri and fmccs acting on 1hcsc (Uli$Sc-s arc Fi . Show tb!ll

if F
L~'= 1 Fi. then llle acceleration a of the centte of .na.~;s of the
syste.m (sec Sec lion 7.7) is given by F M a. where M
L.'=1 m;.
X

44. lf the fort'C acting on a particle is always tangcnl to the particle's


tnticctory. what can you conclude about the trajct'tory?

.io

45. Kepler'3 s~'"'CJml luw ~late~ that the line joining he 11.Un l( a l)la.r~:t
r;wccpsout cqu:.al ~rcas in <Xrua.ltimc iruc.rvals. To .show this. let A (t) be
the area ;;.wept 04.Hby the line beginning :11same li nlC ' in the rollowin&
li,gure :.nd t.:ntli ng ac lime t.
(a)

v~.; ry

(c) Usc p;,1 r1~ (u) and (b) tv o bluin

dA
- =
dl

that
A(r)

CI(J)

Docs lhis verify Kepler's second law?

I
..

fl

and U~ C(J\UIIion 6.19 10 ptO\'C that

dA

Position
of pJuner

-_,. -.
I

>'

= constal\l.

-,r'dO ,

(1,

Iii

t/0
ti t

46. K(p/er's 11u'rd /awsuucs lhm lhe squatcoflhc period o frcvolu tioo
uf a planet is J>ro portional to d1C ct.tbc of the lcnglh or the naj-ur ax is or
ib orbit
(a) U,;c pan ( C) or Exercise 45 to Sh(J"' thll if P i), the ti1nc
lnkcn f<:W' one compk tc rcvolutiOil, then

2rrab

Posicion of plancc

P =

ICI '

wrn:rc 2a :and 2/J arc lengths or the maj()f unt.l ntinor axes
of the elliptic orbit.

11>) Uecqumion 11.116to show that


and therefore

(b) Show thot = cos6t sin&j nd tlr f ilt re perpen~icul:tr. oWl deduc-e !'rom equation I 1.1151hat

**

b'Ja = ICI '/(GM ),

47. SuJlPC)U! lhal in Projcc..1 18 Lhc 1nass is g.ivc:n an mitial SJXCd Vq at


the lop of the sphere. Pro\'c thcu if u0 .::;: .fiii. ahe angl tJ tl.l \\'hich

"'
- )
( 3 + -Jag

the mass leaves the sphere is Sin- -2

SUMMARY

We hnvc now cstabli>llC<I lhc 1\Jundwork for muhi-.triablc calculus. We discussed curves
and surfaces in spa<...: wld introd uced ve<:tors. We described poi nts by Cnnesinn coord inales
(x, ,\' 1 z.) and then illustrated that <m equcuion F(x, y, z) = 0 in these coordinates usually
defines a surface. When a se<:ond qual ion G(x. y . z) = 0 also define$ a surface. the pair o f
simultaneous equations F(.t , y, t) = 0. G (x, y . z) = 0 dcscribesthccunc of intersection of
the two surfaces (pro\'idcd the surfaces do intcrsccl). It is often more useful to ha\'C pMitlletric
equ~llions fo r a c une, and thes:e can be obtHined by specifying one of .r, .v. or t al\ a functi011
of a paran-.eter 1 and solving the given C(Jmlti(>llS for the uthcr two in terms of 1: x = ~dt),
)' = y (l), z = <: (1).
The most common surfaces thm we encountered were planes artd quadric surfaces. Every
plane has an equation of t he f<mn Ax + By+ C ~ + D = 0: cotwer:<ely, e"ery such equalion
descri bes n pl:111e (pt'Qvide<l Hml A, 8 , a11d C arc 1101all zero). A p lane is uniquely de lined by
a vee lor 1ha1is perpendicular to it [Md (A. 8 , C) is one such vector] and a poin1 on it. Quadric
surfaces arc surfaces whose. equations are quadratic in x, y, and z. dlC mos1 imponam of which
were sketched in Figures 11.22- 11.30.
Every straight line in space is c.har.tcterized by a vector a long it and a point on iL (ContrJst
this wi1h the charactetization of a plane described above.) If (a. b, c) are the componems of a
vectol' aJong a line and (x0 y0 zo) are the cool'dinates of a point on it. then vectol', symmetl'ic.
and pararne1ric equations for the lille are, respectively,
(x ,y, z) = (xo. Yo. zo)

+ t(a, b, c);

Suntm:uy
X - Xo

715

= ' -(' <.o

x=xo+at.
y =Yo + bt ,
z

;u

cl .

Geometrical ly~ "cctors are defined as directed li tc scgmcnls; algcbrHiL-ally. they

me represented

by ordc.rcd sets of real numbers ( IJ.v . u, . Uz ), cal led their Canes ian components. Vectors can
be added or subtmcted geometrically using t(iangles or pal"dllelograms: algebraically. they are
added and subtracted component by component. Vectors c;tn ::tl.so be ntuhiplicd by scalar.-; to
give pa rallel vcc.:tors of diffc.rcnt lengths.
We defined two pnoducts of "eetors: the scalar l)noduct and the vector product. 111e scalar
product of two vectors u

= (u,,. u_,.. u: ) anti. v = (v... , v>, v:) is defined l-lS

= +
+
= iullvlc-osO,
ju;, 11~ + u1is the length of u and(! is the angle between u and v. If the
uv

u ,v.

ll yVy

ll ; U;

where lui =
components of u and v arc known, this equation can be used to find the angle 8 between the
vectors. The scalar product has many uses: finding components of vectors in arbitrary directions.
calculating di,;tant"eS between geornctric obj ects, and tinding mechanical work, arnong others.
The ,ector product o f two vectors u and v is

u xv =

ll x

Uy

l.l z

v,

V,r

v=

= lull,! sinO,i.

where\,, is the unit vector perpendicul ar to u and ,. determined by the right-hand n de. Becatt'\.e of
the.perpendicularity propcn). the vector produ ct is indispensable in finding vectorsperpclldic.ular

to other vector.. We t~<ed this fact when finding a vector along the line of imersection of two
plane.-.. a \'ecwr 5>eq>endicular to the planeconUtining thn.'C ghen poims. and distances be1ween
g<vnoctric objects. It can also be used to lind areas of triangles a~d parau:rosmm~.
If n cune is repr~med vectorinlly in the form r (l) .r(l)l + .v(t)j + <:(l) k. then a 1111it

vector ttu'lgentto the curve at any point is

'~' =

d r fd t
ld r f d t l '

1\vo unit vectors normal to the curve are the principul nornu11 N1111d the binorntal
d 'i'fdt

lcl'f /tlt l '

6:

B = T X N.

These three vectors l'on11 a moving triad of mutuall y perpendicular unit vectors a long the curve.
The curvature of a curve. defined by K(l ) = it X rl/lri 5. measures the rate at which the
curve changes direction: The larger K is. 1hc fascer the curve 1un1s. n,c reci procal of curvature
1
p
K
is e<1llcd mdius of curvmure. h is the radius of thtll circle which bc.st approximmes

the curYe m any poinL

If palllrnctric eqll>ltions for a curve rep=nc tile position of a panicle and 1


the velocity and acceleration of the panicle arc. respectively.
v=

dr
dt.

and its speed is the magnitude of velocity,

a=

I,.1.

dv
dt

d 2r

d./ 2'

i~

time, then

'nmgenti~l anti normal components of 'elocity und ncc..-cl eration or the 1~uticlc are defil)ed

by

{/

= -d t lvl

and

What these results say is that vclod ty is a lways tangent to I he trajectory of the panide. and
its uccclcnuion tii\Yllys lies in the plane or the velocity ve<,..'tor .and che principol normal.

KEY TERMS

l n reviewiilg this chapter, you shoukl be able to tlefine or cliscuM: the t'o llowitlg key term... :

Coorc.Jimuc ploUles
C.artc.siul ot 1'Ceta1\gular coordinutes
Left.. handed coordinate: system
C)' Ii nder
Scalars
l i p of a \'CCIOf
X.)' , and

:componcnls of v

Unit veclof
Parallelogra m addition of vectors
Rcsul lant force

Vt;t;;tor compOnents
Vec10r, cross, or ou1er pr0<1uc1
Nomttll or normal vector to the plane
Parametli < equo.ltions for n l ine

Origin
Rig.lH-haaKicd \:oordinate syslem
Octants
Qutu.lric: surface
Vec tors

Tnil of a vector
Carte~ian <omponents of v
Triangular addition of vectors
Zero ""ct or
Scuhl.r components
Sc-alor. dot, or inner prlM;:luct
Equation for a plane
Vcc1or equal ion of a line
Symmetric t<luations for a line

Compo nent

Work

First momcniS of forces


Sag of a cable

Spun of o <ilblc

Catenary
Continuous \'ector-valued function
Dcri \';Hive of a ' 'CClOr-valucd funclion
Antidcrivativc of .1t vcctOC"Indefi nite intcgml o f a vcccor.. valucd func tion
valucd function
Petra metric I'Cpre..~en ta liOil of curves
Conti11uous curve

)>osition or displacement vector

Closed c u rvc
Tangent vectors to ~.:urvcs
Piecewisesmoolh curve
Princ ipalnornml vecto r

0.1rwturc of a curve

Vector represcnutlon of curve


Smooth curve

U11i1 1a'gem

\ CCl OC'

to tl curve

Binormal vec1or

Circle of c urvuturc of a curve


1brsion of a curve

Radiu:., o f curvature o f a <.'\lt\ rt


Frenet- Sen eL fonnulas
Velocity

Speed

Accelera1ioo

Tangemial componeniS of veloci1y


and acceleration

Normal components of velociLy


and accelcraLio n

Kepler's laws

REVIEW
EXERCISES

In B'c'l:isc-f 1- 10 firw.J lll<: ~~hu. or ~h e ~a.kv oe the t.'t."I1Jl0l'IC11S of


Ilk: \ (:CKrif u = (1 ,3, -2), v (2,4, - I), w (0, 2 . 1) . and
r (2.
- r).

JJ, C~~~t~nliOg II'C 1if1 ,'1:


(2. z. 2)

I. 2t - 3w + r

X=)'= t.

o.

7. ( u

+ v)

)( (r - w )

+ v)

(r - w )

In

3. (u x '') x (r x w)
10. -

2r

V W

Eli.CI'(i~c,. 35-39

J~ . ll~tw~cn lhc !)Oint ~> ( 1. 3 .

+ 3(v + u )

x - y=S

.tt - y+lz 6
14. X - )' = $,

15. x 2 + y 2 + :; 2 6t

.r 1 +

)'l

+ z1 =

2x

+ ,l' =

~<

+ JO

ro
J7, X + ,\' ; 5, 2\'- 3)' + 6t =
18. ,1' :;::;: / 1 .}' = f, t = I }
16

38. f'JUm the l ine x - )'

.lx + 6y

j y >-!' z'

6.t -

I. )'

-2) ond (6, 4, 1)

:!. 2.,

+ 2,\' - t
+y+~

4 to lh.: p1al-.,:

l. b

-4 lO the plane

36. From lhC pomt (6. 2. 1) ''' 1t1c pl~ 6.1:

In E~t...'<'i'l'l'S 11- 2(i d rm.\' wh;'lh.ovcr diOotflllllion, or etW.III i Otl~. describe


in t:IX!CC.

2t. t = 4 - , , arK\

rind Ihe distance.

37. Fl\)m lhc l ine ,,. - ) '

13 .1

>' = 1 +

JJ,

4. Ju >c (4 v - w)

( u

!J. (u v) r - 3(v. w ) u

II.

= J., 3..v + 4.\ = 6 und d"~e !".....,.

+t

)'

J4. Contamint the lines :c =

:l. u (vx w )

J. (3u x .av) - w
5. Ju (v - ( \)r

~:~~~4

+t

+t =

..L

= 4

.l?. Fromrhcpuim (6.l.3) hHflcline.\' - y .,-z=<.. lx+y+4t- l


.&0. Find the ;~rca of the trinnglc "iltl
nod ( 6. 3. - 2).

'\X!t"'ic~s

( 1. 1, 1). ( - 2. 1. O) .

41 . If the poinlS in E..;crcisc 40 arc thruc \'C11ice' or a ~lldogl\\11\,


what <.\l'e ~sibiliti es
the fo urth vertex? What ute u1t:::&.,. C)( these

rot

=: l

P: r~l ldograns~

19. ,\'=1. )'= 1 }+ 1

20. ~ =

In Exerc:iscr.-42-"3 tin~ ttl( unit 111ngcn1 ''t~tor j, the tlriM:ipul t'M.lmULl

.r ..f. s = z

vector N. and the binorm:tl vcctOt' 6 ft'K" the <.'\lrvc.

21. p : 4 - .rl - 2yl


2

22. y ~t ~ I,

42.. .X: 2Silll . .\': 2CdSI.

.r =z

43 . .\' /',

+ z: = r. .r = c::
xl + y 1 = 2 7 + I

.V

2i2,::

~ J

I+ 4

= 1'2, z, :::

2J. y 2

44. If u parddc hnr; :.1 trajectory dclincd by

24.

1:. where 1 iG time, lind ils w:tocity, sprcd. und ncce\cr,uioc\ 31 lU\)'
tiJU<.' , Wll&~l Ul'l: IH'Ifl'n ll.l 6LIId lll.llB,CillHll C'OIIlpt)I\CJ\ UI Of itb vcloCi l) Otld

25 It

26.

=r

J' 1, .r r. 1

x ;;;

J. ,\ '

:K:ccl~o:ruti on?

,z = x 2 - yt

..s. A force: F = ,\' 2 (2i + 3j ) OCI.S. on ~l p:utide moving rrou x


10 .1:

lrl 8.\Cr'<=ii>th 27-.30 lind cttuarions for Ihe line.

-1 ulong the .t uxis. I l ow much \\C.)I'k docto it do'!

46. A 0011 rolls on :.t table I m hi&h with sl)t:(d O.S mf$ ( ll&urt beiQ'w).

27. l'hrt.,J:h l hc ""'"" (-2. 3. 0) and ( I, -2, 4)

2y+z =

28. l'hrough (6. 6. 2) 11nd pc:opcndicuh&rto the plonc S.r -

29. Parullelto lheline x - y

= 5. 2.r + 3)' + 6t =4 nd lhrough

rh: origin

JO. PcrpcndK:ulru lo lhc line A' = 1 + 2. ,v = 3- 21.


;,uci')C~ting lhis line. 3nd lhrough lhc poinl (I. 3. 2)

lm
::.

(a) With wh:tl ~1x.W docg it strike the. Ooor?


(b) Wh:u is it$ tli~p1ac.ement veciQr 1\:l:a ivc lO the point wht rc

J I. lluough lbe poiniS ( I. 3. 2). (2. - 1. 0). nnd (6. I. 3)

32. Tirrough the point ( I t 2, - I) and pcrpcndicttlar to the line J


l, .\' + y = 4

tl left the table whCI) it SLJihs the floor?

(c) Lf it rebounds in the direction shown bttt loses 20% or its


speed in the boum:c. find the positjonof its sct.'Ond boum:c.

798

Chap;ter ] [ Vee-rots ~n~~ Tltree-Dimension~l An~lytic Oeometlly

* 47.

In the figure below~ a spring ('~rith c:onstrull k) is fixed at A and


a:~tached to a sleeve at C. The s[eeve is. tree to slide withm~t friction on
a verti!ca[ rod~ and when the spring is horizotUa[ (a'l 8 ), it is tmst.retclhed.
If the s~.eeve is s~.owly ~owered~ the:re ~s a p<J:s~llion ;U wbich the vertica~.
c.omponent of dile spring foJt'ce on C is. balanced by the force of gravity
on llhe sleeve {ignoring the we]ghl of the spting Usellf). ] f tlhe mass of
the sleeve is m ,. ind an equati.on dete:mti ning .s in terrns of d ~ m , k,.
and g = 9.8 L, mirle accde:rration due to gravity.

.A 1.

-- e ----~----e----~-

B-1
s

C-j

48. find CaJttesian c:ollnponents of the spring force F on the sleeve in


Examp[e 11 .26.

* 49.

If a toy train tnt ve[s. around u~e oval tl'ack in the igure below with

constant speed, show thal ~ts acce~erration at A (the poi1U al wh~ch d!Je
circlll[aJt end meets the straiig[ilt sec.tion) is disc.ontinu.o~s .

CHAPTER

12

Application Previcn

Differential Calculus of
Multivariable Functions
The figure on the left below ~how~ gao; confined in a cylintler, closed on one end, with a piston

the uthcr. I r the pisiOn 1110 \'CS to the lefl. i.t compresses the gas i11 the cylinder. The volume
V that the gas occupies decreases and the pressure P that il exerts on 1hc piston increa~.
Conversely, when the pistun ll111Ves to the right. the volume of the gas increases and the pressure
Otl

on the piston decreases. The table bclou givc.s the pressure for v!'ui ous volumes of gas. They

have been plotted in the figure on the right.

54.3
61.2

61.82
49.5

88 7
28.4

72.4

37.6

I'

Cylinder

Gas

I!

Piston

60

118.6
19.2

194.0
10.1

JO

20

50

100

ISO

200

THE PROBLE~ I
What function rcpr=nts the data in the table: or. c.quivalently, were the
data poious to be joined by a smooth curve, what would be its equation? (See the discussion on
pa,ocs 882 alld 883 for thcsolutioll.)
Few quantities in real life depend on only one variable; most depend on a multitude of imerrclated variables.. ln order to understand such complicated relationshi ps~ we initiate discussions
in tlli~ chapter with derivmives of funeti011S of' more than one variable. Much or the theor)' and
n1any or our examples in\'olve functions of two or three variables. because in these cases \\e
can give geometric as well as anal)ti< explana.tions. If the situation is completely analogous for
functions of more \ariablcs, then it is likt:ly ~hat no rne lli()ll of Lhis fac[ will be made: on 1he
()ther hand. if the situmion is different ror a higher number of variables. we will be careful to
point out these dillcrcnccs.

112.1 Multivariable

Functions
If a variable T depends on other variables x , J , z, and I, we write 7 = f(x , )' , <;, I) and
speak of T as a function or x, y, z. and r. Ft>r example, 7 might be temperature. x. y. and z
might be Lhe coordinaLes of poilUS io some region of space, and t might be tin1e. The sLoppiog
distance D of a car depends on many factors: Lhe initial speed s, the reaction time 1 of the driver
to mo1e from the accelerator to the brake, the texture T of the road, the moisture level M on
the road, and so on. We wti te D = f(s, t , T , M , ...) to represent this functional dependence.
The function P = f(l, R) = / 2 R represents the power necessary to maintaill a current I
through a wire wilh resistance R.

799

800

0.3pltT

12 Differenli:ll C:lk:UI~ of Mnhi\'ari:Jble Funcrions

I jtdll,l u::&a f\ {um:tioo


f(x. y) of two ilhkl>cDI.Icm vari:Jblcs can be repn::scnted i!tXlfflctricalt~r

a.s a surtace wi'h equation

z = f(x. y )

z=flx,y)

More precisely. a variable z is :;aid to be.a function of two independent variables x and y
if x andy are not related and each pair of value.< of x andy determines a unique value of z.
We write 1: = f (x, y) to i11dicatc that z is a function of x and y . E.tch possible pair of values
x and y ofdte independent variables can be represented geometrically as a point (x , y) in the
xy -plane. The totality of all points for which f(x, y ) is defined fomlS a region in the xy -plane
called the domain of the function. Figure 12. 1, for example. illustrates a rectangular domain.
If for each point (.r, y) in the domain we plol a point l (x , y) units above the .ry-plane, we
obtai.n a sutface., such as the one in Figure 12 . 1. Each point on this surface has cootdioates
(x. y, z) that sati~fy the equation
( 12.1)
' = f(x 1)
.._

..

'

and therefore 12.1 is the equation of the surface. This surface is a pictorial representation of the
function.

It is clear that functions of more than two independent variables cannot be represented
I (x . y . z) is a f1mccion of three independent
pictorially as surfaces. For example. if u
variables, values (x, y, z) of t he~e independent variables can be represented geometrictllly as
points in space. To graph u = .f(x. y, z) as above would rc.quirc au -axis perpendicular to

Domain of f (x, y)

the x-. )', and z-axe.<, a somewhat clifticult ta~k geometrically. We can certainly think of u

f (.\', )', Z) as defining a surfac.c in four-dimensional X)' :u-space, but visually we arc stymied.
Although every function j(x, y) of cwo independent variables can be represented geomctricaUy as a surface, not every surface represents a function f (x, y) . A given surface docs
repre.<ent a function l<x, y) if and only if every vertical line (in the z-direction) that intersects
the surface does so in exactly one point. For example, a sphere such as x 2 + y 2 + z 2 = l
does n01 determine z as a function of x and v. Most. but not all, vertical lines that intersect the
sphere do so in two points. It is also clear alge.tDraicaUy that x 2 + y2 + z2 = 1 does not detine
z as a timeLion of x and y. Sol ving for z gi\es z = .j I - x2 - y 2 , two soludons for each
x and y satisfying x 2 y l < l .

I EXAMPLE

12.1

---

Draw the surface defined by the functiun I (x, y ) = x 2 + 4 yl .


SOLUTION To draw the surface z = x1 + 4 v 1 we note that if the surfaec is imersected with
a pl:me z
k > 0, then the ellipse .\' 2 + 4}':2
k. z = k is obtained. Ask increases. the
ellipse becomes larger. ln other words. cross-scctions of this surface arc ellipses thai expand
with increasing z. If we intersect !he surfa::e 1vith the y z-plane (x
0), we obtain the
pambola t = 4y1. x = 0. Similarly. intersection of the su,face wi~t the x z-plane gives the
parabola z = .t 2 )' = 0 . These facts lead to Figure 12.2a. A computer plot of the surface for
- 2 :::; x :::; 2 and - 2 :::; y :::; 2 is shown in Figure J2.2b.

liiCJII;IWf#fW Cro~~-:so.::tior~ of surface wilh .f t ~plane,


J.;: -plane, and planes .: k 1~d 1o $\uf;~ce ;: - .tl + 4 y!

IOUCIII;IAFWJN

'

Compauer plot of

12.1 Mllllivnriable ~nc i ons

I EXAMPLE

801

12.2

The ends of a ta ut string are tixed at x

0 and x
2 on the x-axis. At time 1 0 , the string
is given a d isplacement in tl1e y -d irection of y = sin(rr ~"/2) (Figure 12.3a). If the string is
then released. its displacement thercarter is given by

y = .f(x.t) = sin(11"x/2)cos(811"1).
Physically. this func tio.n need only be cons idered fort :;: 0 and 0 ::; x ::; 2, and is ploued
for 0 ::; 1 ::; 1/2 in Figute 12.3b. lnte1J>ret physically the intersections of this surface with
planes 1 10 (/o
a constant) and x
xo (xo a constant).

ltlilial diilpla<:emelll l'>f a s1ing


lh:u is lliplaccll 1hc11 fclea.;;;cd

IOJirlil;l
~li .;;-t)l :u;cmtu

'iEJIIEI:a

Cl'>lllJllllt'r pll'>l repfeilenling


of roims in Siring

)'

)'

0.8

0.6
0.4

0.2
0.5

1.5

SOLUTION Grid lines oro the surface are curves of intersection o f the surface with ven ical
x0 and I
10 . The curve of inte rsection of the surface with a pla ne I
lo
planes x
represe-nts the posilion of the string at time t0 . For I = t0, the equalion of the curve is
y (x , lo) = cos(811' lo) sin(l!' x / 2). Thus. t he string vibrates up a nd down always in the shape
of a sine c urve, its amplitude at time to being I cos(Srr lo) 1.
The c urve o f intersection of the surface with a plane x = x0 0 < x0 < 2, is a graphical
history of the venical displacement o f the particle in the suing at position x0 The equation of
the curve is y (x0 , 1) = sin(11" x 0 / 2} cos(811"1). The pan icle at x 0 undergoes simple harmonic
motion, the amplitude being lsin(1t' Xo/2)1.

Another way to visualize a function f(x , y) of two independent variables is through le1el
curves. Curves .f(x , y ) = C are drawn in the x y -pla ne for various values of C. Effectively,
the surface z = .f (x , y) is s liced with a plane z = C, and the curve of i.nte rsectio n is projected
into the x y -pl:me. Each curve joins all po ints fo r which f (x, y ) has the same value; o r it joins
aJI points that have Lhe same height on the surface z.
.f (x, y) . A few level curves for Lhe
surface in Figure 12.2 are sh.own in Figure 12.4; they are ellipses. Level cunes for the function
in Figure 12.5a are shown im Figure 12.5b.

ljiHI);IWfM

Le ~l t ur\'('S of th(' funttiuo j{x, )')

= x 2 + 4y1

.1'

C=36

X~ + 4y2 = C

-6

-3

802

Cbi~tr

12 Oifl'i!l'eiHinJ CnJ..:ulus orMul ti\'Qfi able Fl11."-'1iOil S

Mnfl!l.,lil Ea.=Ot*
f;u.'

CuUJpulcl' plol uf wr
f(.t, y) = .Y: - )'~

.f(x. )'/ = x'

z = .~ :- ) ~ detintd by

- y'
y

This Lcchniquc is used on tO))()graphical maJ)S to indicilte land elevation, on mari ne charts to
i nclicate water depth, and on climatic maps 10 indic;ue curves of constant temper;nure (isotherms)
and c.:urves of const<IIH barometric pressure (isobars).
EXERCISES 12.1

I. If J (x, y)
x 1y x sin y,eV"Iual< (a) J( l , 2) , (b) / ( - 2, - 2) ,
(c) f(x '
y, x - y'). and (d) f(x h , y) - f(x, .v).

2. If / (x . .v. z)

x 2y2

x'

+ 4zx2 ,

show thai j ( ll

+ b,a-

b. ab) = O.

24. f(x . y )

* 25.

= ln (x' + y 2)

f(x )y) = x 1

y1

" 26. A eloscd b<>x is 10 hove 10101surfnee urea 30m 2. l'ind a [(mnulu
for the vo lu me of the box in tcnns o f il.s length I and width w.

In Exu ciscs 3-6 fiud omd il lustn ttc gconlClricull)' the largest possible
domain for the fu nclion.

{a) A company wi~hcs to ton.stmc.:L u stur.agc t{ulk in the ror111


or a reemngular box. II' nuuerial lor siclcs and 10p costs

27,

S1.25/rn 2 :ll'!d nuucrinl j()r the bott.om COSlS $4.15/ml, firld


the coot o f building Ihe t.unk as a fu nction of it:o; length I ,
width U), and heig ht h.

4. j'(x ,y)
ln ( l - x 2 + y')
3. f(x , y ) J 4 - x 2 - y'
5. f (.<, y) = Sin- ' (x 2 y + I ) 6. j'(x. y . z) = lf(x' + y 2 + z')

12.xr- .r 2 v2

).
2G< + y

7. For whm vulucs of x ::tnd \' is the function f (.x . y ) =

(b) If the tank lliUli'Lholc.l 1000 mJ. nnc.J the construction cost in
tcnns of I :tnd w .

cc.wal Lo 'tcro? llluslraiC lhc$C valu<:!) as points: in the .xyplanc. Wh;U


is the largest domain of t.he function~

(c) Repeat pans (a) and (b) if the 12 edges of the tank mu:>l be
welded at a cost of $7 .50/tn o f weld.

* 28.
In Exercises 8-21 dntw the SLUface defi ned by d1c function. Plot the

$utface as a cJ'IOCk.

8. J (x , y )- y'

9. j'(.<. y) - 4 -

10. j(x. y) = x' +y'

f (x. ,v) = x 1 + ,1 1
13. f (x, y) = 1 - x 3

16. f(x . y ) = .<,r

18. f(x, y) = e- ' - '


20. f(x ,y)

= x'

- y

2y

II.

12. f(x ,y) = y+ x


14. f(x , y) = 2(x2 + y')

X -

IS. f (x, .\') = I - x 2 - 4_v'


+ 17. f (x. y) = ,v - .r 2
+ 19. f(x, y )

= lx -

21. f(x, y)

= .;r:,,-+
,--x'' - -.)'
''

.vi

A. rcctangu}arbox is inscribed inside the cl1ipsoid x 1 fa2

+y2f b2+

c = I \Vith s ides para1lcl 10 the coonli nate planes and comers on


the elli psoid. Find il formula f(l( the volume of l he IXlx in lcrms of x
t 2/

andy .

2?.

(a) A silo is l(l be bui 11in the shape of a righc-circul<lrcyli nder


sunnounu:d b)' a ri ~h~cir~u Ia cone. rr the t'ad ius l\f euch
is 6 m. fi nd a formula for the vo lume V o f the s ilo as a
functio n of the heights H and h ol' Llle cylinder and cone.
(b) If lhc t()tUI surface area or the silo must be: 200 m 2 ( not
indmli ng the bn~). fi nd v us u ru nc.tion of h. (lllC arc-IJ
of the curved ~uface or a cone is ;r r ./r 2 + h'i. )

30. The figure below shows parameters 1aken into accoon1 in 1heanal
ysis of the c.listtmcl.! Lnwclled b)' a long j ump..:r:

In .xe.reises 22- 25 dmw level curves


the values C = -2, - I, 0 , I, 2.

*
*

=4 f (x ,y) = y -

22. f (x ,y)
23.

.j4x'
x'

+ y'

f (x , y)

= C corresponding to

8 =ang le to the hol'izorual at which jumper leaves the gro und;


T = horizontal distance from lOC lo C..'Cntn:: or mass G al takco[f;
L = h01'i7.ontal distance frotn heel to centte o f mass on landing;
h =vertical distance between centre o f mass on takeoff and landi ng;

R = hotizorual d istance between cenuc o f mass on lakeolf and landing.

12.2

l.

= T + 1-

vl cosO (

+ - -8

2gh

sinO+ I sin'O + -;;>

where g = 9.& 1. Typical value~ ror


and 0.5 m, respectively.

D. rrom Lhat. in part (~).

ir takeoffspeed is increased by 10%'!

(k > Oconsbnl).

* .H.

A c.ow'$ daily diet <.-onsisL-s or three foods: hay. gmin. and s up~
plemenls. T he animal is always given II kg of hay per day, 50;"i of
which is digestive m:uel'ial a.nd 12% of which is protein. Grain is 74%
digcsti"--e aJld 8.8.-;t. protcitl. Supplements arc 62% digcsti\'C material
outd 34% protein. Hay COilS $27.50 for 1000 tg. whereas grain and
supplements cost SilO and $ 175, respectively, for 1000 kg. A healchy
c,o w's daily diet lllUSt coota.in belwceJ\ 95 and 11.5 kg of' digestive ma.
tcriaJ and be-tween 1.9 and 2.0 kg of prottin. f-:ind a fonnula. f't)r tllC
cost per day, C. or reeding a cow in tenns oftl1e number of kilo: nuns
or grain, G , and supplements, S. red 10 lhe cow dai ly. What is the
domain of this function'?

(c) What is the pcrccmagc cbangc in D. from lhat in pan (a).


if takeoll angle is increased by 10%1

/1
' ' ] ' ' " " " ' " " " "'

* 3 1.

A I<H1g l>iccc ofmetal ! m witlc is bent in two place!'i A and 8 (llg_urc


below) to fcmn <1 channel with three straight sides. Find a formula for
the crosssoctional area of the cl:l~nncl in terms of x, (J, and <j.

lOOe - '' sillX

(CJ ln1erpre1 physically the curves ot' i n1ersection of Ihis surface


with planes x =.to oud 1 = to.

(b) What i$ the percenta~e d tan:e in

32.

= J(.<, t) =

(b) Omw tbc surface T = J(.<, t ).

r . l., and h are 035 m. 0.9 m,

(a) Calculale 0(0.35, Y.O) .

;+;

803

lhc faces at .t' = 0 a.od x = n' arc both kept al tcmpcratur.::


O'~ C for time t > 0. If the initial temperature (at time
t
0) of the rOO is given by IOO~i n x. 0 .::; .\' .::; 1r . then
the tcmper:nure thercafler is

Using the formula developed in .E"<ercisc 35 of Section 1 J.13, the


tota11cngth of lhc jump is

D(O, v) =

L i.1n.its and Cootitmity

(a) A urtiform c ir-.:u_lar roc.l has Oat c.nds al X = 0 a:nd X = .tr


t)n the x -axis. The round side or the rod is: i n su l:_~t ed and

34. The Easy University is buying computcts. It has three models to


choose rmnt Each ruodcl A c.onlJ)uLc.r. with 64MB of memory and a 3
GB hard drive. costs $ 1300: 1oodel B. with 32 MB of memory and a4
GB drive. costs SI200; and economy model C, with 16 MB of memory
and a I GB drive, costs SIOOO. For rcasOI\s related lo aoorcdotalion.

tltc university needs at least 2000 MB or m\!mory and 150GB or tli~t


spoce. II' the compuler lab must hav-e 100 computers. set up :J t'onnula
for the CO&t of outfitting the lab in tcmts of the numbers x and y of
computers of models A and B. What is the domain the fuoction?

or

12.2 Limits and Continuity


The concepts of l imit and contiJ1Ui ty for muhivariab le functions are exactl y the same as for
functions of one variable; on the other ha nd 1 the 'vork involved with the applic.ation of these
con~epLs

is more co mplicated lOr nlulti,ariable futlclions.

lntui ti,ely, a function f (x, y) is said to have limit L as x and y approach x 0 and y0 if
j(x, y) gets arbitrarily close to L. and slays close 10 L. as x andy get arbitmrily close to

x 0 a.nd y0 . To say this in a pxecjse mathel));atica.l way~ _it is convenient to represent pairs of
inclependem variable~ a.~ poinL~ (x , y) in the X)'plane. We then have the following definition.

DEFINITION 12.1

A function f(x, y) has limi t Las (x, y) appi'Oaches (x0 , y0), wrinen
lim

(.,.y)--(xo.yn)

j(x, y) = L,

(1 2.2)

if. g_iveo any > 0. we can tind a 0 > 0 such that

lf(x,y)- Ll <
whenever 0

< /

(x - x 0)l

+ (y

- y0)2 < li and (x , y) is in the domain of f (x, y).

CHAPTER

Calculus Preparation

error 803
' 1.1 Introduction
Over the years, I have l earned that the .nos:t f requent reason for students fHi l i r1g to achieve
optimum results incalculus courses is inadcqlw teprepararion. Smdcncsnwy undersrand calculus
(.;Oncepts completely. Lhey may have every formula and every rule memorit~ed _perfectly~ they
may e ven have a c lear idea of the proca.lun::s required to solve prob lems~ but because they
lack skills in a_lgc.bra, geometry, and tr igonorn etry, they cannot put their krlowledge to work.
h i s unfortunate that they never get to dernon~wne their calcu lus knowl edge becc.lll~ of poor

mathematical preparation . To obta in the-be!\1 possib le grades in calculus, it is essential to have

excellentalgebmic skills and a good gmspofthe elementS of analytic geometry and trigonometry.
In this chapter we ghc yoo the op)>Ortunity to test your skills and knowledge in these three an:as.
and, should it be necessary, the meart~ by which 10 make improvements. Titere are also ~tions
lhat may con1ain material unfamil iar to many readers. Topics in these sections are esse.ntialto
some oFthc applications of calculus; your instructor will indicate whether they are required For
your course.

l lte calculus course at your institution may, or may not. spend time Qn the review sectio1s
in this cb1lpter. Jf it docs not, your instructor may ad,'isc you to review ccr1ajn sections or1 your
own. DO l'f! You could regret ignoring review material in this chapter in order 10 get to caJculus
i n Chapter 2 more q uickly. Each review sectio n opens w i th a diagnostic test to d~ tcrmine your

knowledge of the material in the section. Give yourselr the suggested time to take the test. no
longer. You must not only be able to solve the t est questions. but you m ust also be able to do so
reasonably quickJy. Do NOT use a calcubnor unl e~s spe;cific;:ally instructed to do so. Answers

are provided m the end of the section along with marks for each question. Assign yourself
pan ial marks For partially correct answers, but try to he objective in doing so. It is difficult
to be specific as lO what constitutes an acceptable score on the diag.nol)(ic tests. Ce1tainly a
score o f less than 50% indiC>Jtes thai detaile<l Sludy of the section is required. A score of more
thar'l

soc&, but not much JTt()re, would also suggest the need

fo r careful review. ~fart.: ~ in the

80~ 1 00%. range indicate a

good working knowledge of material in the section, but it could be


beneficial to gi"c the. section a quick rcading. Jl'aying Jtrlicular aucntion tO ,,arts corresponding

10 incorrcclly answered lCSI queslions. ln addition, not only is il helpful to refresh your memory
on concc:pu leamed some lime ;lgo. it is also wi.se 10 become familiar with lhe terminology,
nolation, and conventions set forth in these sections. To improve your skills o n, and knowledge
of, dlC material in a rc,icw sectinn. read the di~cussi ons and examples thoroughly, tr) as m;lny
o f 1he exercises as yuu c.:an. unU then n:tukc the diagnost ic test. If you arc conscientious in your
work, we are confident that you will do much b-etter the second ti me. Yo u w ill be well rewarded
for talc.iog the time to do this; your calculus sr.udies wiJI be so much easier. ln fact. the 1ime you
spend on calculus prepa.rarioo now wiiJ more than compensate for ex 1ra time that you would
s pend on solving calculus proble ms late r. Be lieve me; I have Laught t.housands o f s tudenL.s j us t
like you.

12.2 Limi(S and O.'tni1luiry

SOLUTION
the function,

805

Because points on the Tine y = -x are not with.i n the domain of definition of
c.an write that

\Ve

x' -

lim

(x.y )~(O.O)

y'

x +y

(x - y)(x

liJn

(X.)')->(0.0)

+ y)

+y

(x - y ) = 0.

lim
(X.)')->(0.0)

The-concept of continuity fol' multivariable functions is contained in the following deflnitjon.

DEFINITION 12.2
A functio n f(x, y) is said to be a continuous function at a point (xo, Yo) if

I.
2.

f (x, y)
tim

is defined at (xo. Yo)


j(x, y) exists

(X,)')-+ ( \'h)'O)

3. The value of the function in I and ilS limit in 2 are the same.

Error 804

All three conditions can be combined by writing the single equation

f(x, y) = f(xo,}'o) .

lim

(12.3)

(t,y)-+(Xo.)Q)

The function f (x, y) = (y2 - x 2 )j (y 2 + .~ 2) in Figure 12.6 is d iscontinuous at (0, 0).


The function is undeti ned at (0. 0) and the li mit as (x, y ) --> (0. 0) does not exist. Geometrically, a func tion f(x, y) is continuous at a poitu (x0 , y0 ) if the surface z = f(x, y) is
not separated at the po int (xo , Yo. f(xo , Yo)) . The function f(x , y ) = I - e - l/(x'+y') is
discontinuous at (0, 0) since it is undeti ned for x = 0 and y = 0. The surface has a hole at
(0. 0) (Figure 12.7). The function f (x, y) = sgn ( (x - 1) 2 + (y (see Exercise 47 in
Section 2.4) is discontinuous at (I, 2); it has value 0 at x = 1 and y = 2. and value 1 fo r all
other values of x and y (Figure 12.8). Thefunction h (x 2 + y 2 - 1) where h (.~) is the heaviside
function of Section 2.5 has value zero everywhere inside the circle x 2 + y2 < 1, does not have
a value.on the circle, and has value. ) outside the cicle. (Figure 12.9). his djscontinuous at each
point on the circle. The surface z = h(x 2 + y 2 - I) is that pan o f the xy-plane inside the
circle, and that pan. of the plane z = I above the outside of 1he circle.

2n

lij leiII ;l:j

l#(clii;IWfQ f unction
/(X, y) = I - t>-l/tA!+,.~I is

j(x, y) = sgn[(x- I)'+ ()'- 2J' J ;,

discontinuous m (0, 0)

disco!Hinuous .._, (I, 2)

lij!CCJII;j!j&JDI

Punccion

j(X. )')

:= I

~ ( 1.2)
X

)'

= h(x 2 + )'' -

I)

806

Ch.ap1er 12 Differen1ial Calculus of Mullivruiable FuncliOI)S

EXERCI SES 12. 2

In Exercises 1- 20 evaluate the limit, if it c.1.:ists.


I.

x2 - I

lim

+ 2y 3
Jim
(.t.y)(l.l) x.J + 4y.J
x3

2-.

(x.y) (l.- 3) X+)'

3.

2x - 3y

lim
(.t. ;) - ( 3.2)

5.

lim

9.

6.

lx' - y" J
,.

x - y

(x.y, ( 3.4)

x:! - Y:!
,.

lim

17.

lim

~.).~)-\ 1.1. 1 '

19.

20.

l2x -

,.

+ y

x-

(x..,)- (M ) 2.\3

(.t.s) .... ('l,2)

x - y
x - y

lim

34.

35.

zl

36.

(.l,y)-(t ,l)

+ y1 + 2y y 2 - 2x + 2y

Al>proach (I, 1)

Hint: Approach (1 , I)

JX+Y - .;x=y
y
sin ( x 2 + y 2)
{ 0.0)
x 2 + y:!

lim
\.l.) ) -

(a) Does 1he limh

sin (x - )')
exis1? Exphtin. Is

Um

.\'-

(b) II' we defioe.thc !'u nction eveywhete by giving it Lhe vaJuc


J along Ibe Line)'
.\'.docs the Urnh or lht: funtli()n t::XiSL
at (I, I)? Is the function continuOtLIO at ( I, I)?

+ y')

scc- 1(x2

x2 -

(X,)') .,.(I, I )

soc- ( . - .)
(<l.O)
x- + y

tim

x 1 - 2x

lim

NinJ:

lbt: fu nt LiOn cc)nlinuous a1 (I, I)?

+ 4 yl

(A,y),..(O,O)

Him: Approach (0~ - 2) along

X+ J

lim

(x.;r)-

+ 2)',.
+ 2)-

along stmightljncs.

3x3 - " 3

tim

x'

x 2 - 2x - y 2 + 2\'
lim
't
~
t<..,J- t' 'l x' - 2x + y- - 2y + 2
:~ l on g Slnaig:hl ljncs.

33.

,.

2
y -

32.

luu

T:ln - Jyzfx l

,.

X -

Tan - tx/(yz)J

r + z- + 1

( .l.;r)- ( 2. 1)

+ 4(y
3
..... y)- il\.-2) 3.< - ()'
slraight lines.
31.

x1 _ y1

Lim

( .1.. ,\)--(0. 0)

(.l v.:) .... (o.<t.))

18.

lun

14.

hm

..

lx'+>"l

(x.y) ( M )

* 12.

IS.

10.

,.

13.

16.

lim
(x.y.:) - (O.n/ 2. 1)

I I.

lim

Tan- (yzjx) 8.

lim

lun

x2 + y 2 + z

(.t . )'.~)- (0.11'; 2. 1)

(x.;.:)- (O.:rj 2.1)

xvz

lim
(x.>-.:) - ( 2. J.-I)

(.\.,\) - (Ul)

7.

4.

+y

).'

= L.

37. Givcamalherruu ic!tlddi.njtionfor

38. Is lht: fo Uowing: Sl.ah:menl lntt: or false? If a fu nt LiOn f(x. y) is


undefined aLevery p0in1on a curve C, then for any poinl (Xo. )'o) on
C,
Li au
. f<x. y) d(lCS nOI exis1. Explain. Give an exwnplt:.

li{n

/(.t. y, ?.)

(.t ..v.:) - (X(I"'fiU:)

In Exercises 2 1- 26 find aU points of djsc.,-onlinuity for the [unction.

r 2- I
21. /(x, y) = - - -

21. E"aluatc

+y

23. f(x, y) =

25. f( x ,y)

22. f(x , y) =

lim

(A,,\'),..((I,nJ

+ y-

(.t . y)~ (.tu.)liJ

.
I
24. j(x , y, z) = -

l - x2 - y 2

= lx- Yl

X\'

26. f<x.y)

xyz
x+y

= x ,y + xy-,

[em (X + y) - j I - sin2

(X

+ y)] where

29.

* 30.

x'- 2y 1

Hint: Approach (0, 0) along cubic cmvcs,

3xt + y1

.
li{n

(x - l )' + y'
,
,
.>(X - I) - 2y

{.l.y) {O,O)

(.l.;r) - ( 1.0)

Slrajgbt lines.

...

NinJ:

Approllch ( I, 0) along

0,

d (x, y)

= (0, 0).

(b) Show that .((x. y) is not continuous at (0. 0).

Hi111: Approach (0, 0) along pamholas.


lim

x' + )"'' if (X, y) ;0 (0, 0)


{

Jo Exercises 28-35 evaluate the limit, if il exists.

28.

(a) Sbow1hn1f (X, y) isconlinuous in each vari:Jblt:: sepanudy


m (0, 0) . ln Olhcr words. show 1hm f(x, 0) and /(0, y)
arc continuous a1 ;r, 0 and y = 0.

0 :E (1 :E 1r/2.

39. Let /(.<, y)

x2y2

**

40. Prove lhal:


(a) limu.yJ iM I (xy + 5)
2

(b) limv .., )- ( o. O) (x

=5

+ 2xy + 5) = 8

112.3 Partial Derivatives


We. now detine panial derivati\es o f multivariable functions and interpret these de.rivalives
a1gebr.tica11y and geometrically.

DEFINITION 12.3

The partial derivative of a funcLion

a[
- =
ax

hm

f (x , y ) wiLh respecLLO x

is

f(x +f).x,y)-f(x,y)

"X->0

( 12.4)

(1 2.5)

/).X

and the pan ial derivative with respect to }' is

of
.
f(x, y
- = l l lll
ay
'"''.... o

f).y)- f(x. y)
f).y

It is evident from equation 12.4 that the panial deoivative of f (X, )') with respect tO X is
simply the ordinary deLivati ve or f(x, y) with respect to x, where y is considered a constam.
Similarly, ofjoy is the o rdinary derivative of f(x . y) with respect to)', ho lding X constant.
for the panial derivative of a function or more than two independent variables. we again
permit one variable to vary, b tu hold a ll others constant. FOr example, the partial derivative of
f(x, y, z, t , .. . ) with respect Lo z is
.
a.{
f(x,y,z
- = 1JR)
az 4\z-o

+ f). z, r, .. .)-

f(x,y,z,t, ... )

f). z

( 12.6)

Hence . we d ifferentiate with respect to z while treating x 1 y, I, .. . as constanlS.


Other notations for the pa.n ial derivative arc common. l.n partkular. for OfJax whe.n
z = f(x, y ) , there;uealso

f,.

and

the lasLtwo indicating that the variable y is held consLant when differentia tion w ith respect Lo
x is perfonned.

I EXAMPLE

12.4

.........
Find ozfi!x and iizjay if z =sin (x 2
SOLUTION

Forthis fuoction,

az
= 2x cos (x2
ax

I EXAMPLE

12.5

+ y 3) + e'J' .

+ i) + ye'J'

and

az

ay

SOLUT ION

Since ojjax =

+ 3xz + 4 .

2x/J' + 3z.

~~I o. 2. 3) =

2(1)/2.

.........
Fiod offox at the poinL (I, 2, 3) if f(x, y, z) = x 2 fy"

.n'

= 3y-cos(x +r)+ xe .

+ 3(3)

= 7:

808

Ch<tpltl' 12

Differen1ial Calculus of Multi\'ruiable F\.ulctions

Fo r a fu nction y = j(x) of o ne variable, we detioed d i(fere tuials dx and dy in such a way


that the derivative d y / dx could be thought of as a quotient. This is 1101 done for functions of
more than one variable. Although we write the partial derivative

ax

in the form i!ffi!x (for

cypographjcal reasons). we never consider icas a quotienL


Algebraically, the parLia l derivative ofjii.~ represents the rate of change of f(x, y, . . . )
wilh respect to x when all other v~uiables in f (x , y, . . .) are held constant. For instance,
V = 7f r 211/ 3 represents the volume of a right-circular cone w ith height h and radius r, and
therefore V jiJr = 27C rh/3 represetliS the rate of c hange of th e volume of the cone as the base
radius changes and the height remains fixed. Similarly, v /oh = 1T r2 /3 is the rate of ch ange
of the volume of the right~circular cone as the height changes and the radius is kept fixed. \ Ve
shall learn later how to calcula te the ra te of change of the vo lume when the t'adius aod height
are both changing.
We can in1.erpre1 the parl ial derivalive of a function geome1rical.ly when the function cao
be interpreted geometrically, namely, when there are only two independent variables. Consider,
then, a function .f (x , y) that is represented geometrically as a swface z = .f (x, y ) in Figure
12.1 0. If we. intersect this surface with a plane y = y0 = a constam. we obt.aj n a curve with
equations

W::!!lc:lll;l#

aj

Geometric

iJlletpretation of pal'lial deri\'ati\'eS


of a fuoctioo j(x,y) of rwo
\'ariablcs

.Y = Yo,

z=

.f(x, Yo) .

( 12.7)

Because this curve lies in the pla ne y = y0 , we can talk about its ta ngent line a t the point
(xo. Yo, zo). where zo = f(xo, Yo) . The slope of this ta ngent is the derivative of z with res pect
to .x , but because y is being held conslant at y0, it must be the partial derivative of z with
respect to x. In other words, the slope of the tangent line to the curve in Figure 12.1 0 at the
point (xo, Yo, Zo} is af/axl(x,,.Jl>l Similarly. the partial de1ivative a flay e va luated a t (xo, Yo)
represeots the slope of the tangent line to the curve of illlersectioo of z = f (x, y ) and the pla oe
x = xo at the po int (xo, Yo. zo) .

EXERCISES 12. 3

In Exercises 1-20 evaluate of/ox and of/oy.


I. j'(x, y) = x'y' + 2xy

2. f(x, y) = 3xy- 4x'y'

3. f(x , y) = x'!y'

4. f(x , y) = xf(x + y ) - x/y

5. f(x, y) = x f(2x' + y)

6. j (x , y) =sin (xy)

7. f(X, }') =

X COS (X

8. j (x , y) = Jx' + y'

9. f (x, y) =

x/t' - y'

+ )')

11. f (x,y) = e>+Y


13.

24. of/ox at ( l , - I, I, - I) if f(x, y,

25. 8.{/&1 if .{(x , y, 1} = xj12

26. &fl&xif j (x ,y,z)=CoC' (L+x+ y +z)


27. o.f/oy at (1,2,3) if j (x ,y, l ) =Sin-' (>')'I)/ Cos- ' (xyt)

10. j'(x, y) = tan (2x' + y 1)

28. of fox if f<x, y, z)

IS. / (x,y) = (x + l) ln (xy} 16. f (x, y) =sin (.ve-' )

19. j'(x,y) = -

cosy

30. o.f/oz if f(x , z)

18. f(x , y) = ijl - cos' (x'y)


20. j'(x, y) = In (sec J x + y )

22. 8.(/oz if f (x.

z) =

= xyze"'+'
Tan - ( 1/(x'

23. of/oy at (I , t , O) if f(x, y, z)

+ z')J

= xy(x' + y' + z')'il

. .

+ y' + z')

= <z' / 2) Sin-' (x /z)+ (xf2)Jz'- x'

31. II

I (x ' y) =

of
x'yf(x-y). show that X ax

32. I f

f(x, y, z)

= (x' + y' + Z )/(xyz). show that

In Exercises 2 1-30 evaluaLe Lhede1ivalive indicaLed.

21. of/ox if f (x,y, z)

= x'fy + x sin (yz / x)

29. of/&1 if f(x, y, z,l) = x.rz In <x'

14. f(x,y )= ln(x'+y' )

17. f (x, y) = Tan - 1 (xfy)

11)

y' /I'+ (x /y) sec-' (I /3)

12. j'(x , y) = e>Y

(x, y) = xy<"'"

sin.r

z, 1) = Z1 /(.<1 + y 2 -

oy

= 3f(x, y).

af + y -af + z of
-

xax

of

+ )' 0)'

oz

= f (x,y,z).

33. Uf(x.y,z)=(x'+y 2)cos [(y+z)/x],showthm


of
of
of
x -a + Y,-- + z ,-- = 2.f(x,y. z).
X

a )'

aZ

12.3

the foiJowing ;:uc acceptable:

LOX.

(b) p =

(c) Sel y
2 . diffcremjate with I'C$J>CCI to x. and set x
I.
(d) Set x = I, differentiate witJt respect to x. and set y = 2.

35. Temperature at point.; (x , y ) in a semicircular plate defi ned by


x 1 + y'1 ,::: 4, y ~ 0 is given by T (x , y) = 16x 2 - 24.ry + 40y2
find.if possible.(a) T" ( l , l) , (b) T,_( l , l}.(c) T,( I , O).(d) T,-(1 , 0).
(e) T, (0, 2). and (f) T,-(0 , 2).

36. In tJte figure below. two iden tical bars A 8 and BC are pinned at
B as well ao; at A and C . Each bar is initially of length L. and initially

809

(a) p = <.-onstam. u = (2J. 2 - xy + z 2 )t . v = (x 2 - 4xy +


y 2)1 . w = (-2xy- yz + y 2)1

34. To evaluate a flax for f(x, y) at the point ( I, 2), state which of
(a) Dilferentiate f (x , y ) wiLh respect to .r holding y t onstaJH.
Md Lhen set x = I and y = 2.
(b) Sci x = I a nd y = 2 , and 1hen diQtn::.nliatc wilh rcspcc1

P<Utial Del'ivariw:.s

x y+z:t, u = x 2 y +t, v

= y1 z-2t 2 ,

w = 5x+2z

40. A gas-filled J)neumatic SII'Ul behaves like the J)iSLon-cyl iode~ ap


pruams sht)Wn below. AI one instant when Lhe piston is L
0. 15 1n
away rron the closed eod or the. cyli nder, the gac; density is unifOI'Ol at
p
18 kg/mJ. aJtd the piston begins to .nove away fm.n the closed
end aL a rort.c;tanL J'ate of 12 m/s. The ga.c; Olt)lioo is ooe~d i meosi o nal
and ptopol'li t)nal to distance fron Lhe closed end. It vaties l i1teal'ly
from zero at Lhe clt)sed end of Lhe cyl inder to 12 m/s a11..he piston. Gac;
density is always uniform throughot!L Lhe cyl inder, btU varie.c; in ti me.
Use Lhe equation of continuity in Exe~cise 39 to find Lhe density of the
gac; a.c; a functit)n of ti me.

point 8 lies a dioaan<.'C h above line A C. \Vhcn a vcnkal force with


magnillldc F is applied at 8 , the vc11icaJ displat-cmcnt x > 0 of 8 is
related to F by the equation

X) (

h -F - 2AE ( '-

f.
,JU-21rx + x 1

I)
'

when:: A is 1he cross-stt l ional an::a of lhc ~tr, and :: is Young's mc>dulus for the bars. Show thot F
vanishes for X = " -

<'-' - * 41.

a tax

Lh ' )'" j l - (I -

ln t-omplcx variable theory, two funclions u (x. y) and u(x, y) arc

said to be harmfJnic conjugates in a region R if in R they satisfy tJte


Cauchy- RiemaJtll equatjons

lo' fL 2) '"

au

T
h

Show lh!ll the fo llowing pairs or funCiiOnS arc barnlc)nic: conj ug!IICS:

(a) u(x,y) = - 3xy'+y+x' , v(x , y) = 3x 2y - y' -x +5


C

(x' + x + y 2 )/(x 1 + y'), v(x , y) =


- y/(x1 + y1)

(b) u(x, y) =

37. Can you finda fu ntliOn / (.r , y) SO ib!tl j~ (x , y)

f , (x,y)

1 1<'=---- - - - ---"'1
*

au

ox = Dy ,

= 3x + 4y?

(c) u(x. y) =ex (x co..c; y - y sin y). v(x , y)

= 2x- 3y a nd

38. SuJ)J)OSC tl, b, aod t: are dle tengd1s of Lhe sidc.c.: of a triangle
and A, 8. and C ruc the opposite angles. Find (a) tl,t(b, c, A) . (b)
A.(a, b, c), (c) "(b, c, A), and (d) A( ", b , t:) .

39. 11le equation of contioujty fot tJuec-di mensionaJ tUlsteady flow of


a cotupl'essible fluid is

ap a
a
a
at
+ ax (pu ) + Dy ( pu) + Dt. (pw) =

ui

= e~: (x sin y+

ycosy)

42. If,. and 9 are polar coordinates, then the Cauchy- Riemann equa
Lions in Exercise 41 for functjons u{r, 6) aJtd v(r , 6) lake 1.he for m

au

Dv

ar = ~ ae

ou

rat!

# 0.

Show lh!IL the fo llowing pairs or funCiiOnS smisfy IbeSt! cqumions:

0,

where p(x. y. t. t ) is the density of the fluid, and


+ tJ,i + wk. is
the velocity of the fl uid at position (.t , y, Z) and time 1. Dctcnninc
whclhcr the <.'Ontinuity equation is satisfied if:

(a) u(r , e)
(r 2 +rcos e)/( l + r 2 + 2rcose), v(r,e)
r s in e/( 1 + r 1 + 2r cos e)

= ,fi'cos(e/2), v(r,e) = ., fi' sin(e/2)


u(r , e)= lnr, v(r, e)= e

(b) u(r , e)
(c)

810

a,aper 12 Differefllial Calc.UI'US of ~luJiivnriable Func.LiMS

112.4 Gradients
Suppose a function j(x. y , z) is defined at each po int in some region of space. a nd th at at each
point of the regio n all three partial derivatives

&j
a.r
,.--.
ily '
uX

ilf
ilz

exist. For example, if f(x . y, z) represents the pres.ent temperalUre at each poiLU in the room
in which you are working, then these derivatives represem rates o f change o f temperature in
directions parallel to the x -, y-, and z-axes. respectively. There is a particular combination of
these derivatives that pl'oves very usefuJ in later work. This combinatjon is contaj ned in the
following delinition.

DEFINITION 12.4
If a function j(x, y, z) has partial derivatives of fox, i!jji!y, and of/oz at each point
in some region D of space, then at each point in D we define a vector ca lled the gradient
of .f (x, y, z). wline n grad .f or '\1 f. by

i'Jf ,. i'Jf ,. i'Jf.


+ - J + - k.
ax
iiy
oz

g md f = "i1 f = -

For a fu nction

(12.8)

(x, y) of only two independem var iables, we have

i'Jf.

i)j.

"ilj = - 1 + - j.
oy

( 12.9)

ax

I EXAMPLE

12.6

..._..
If

f(x, y, z) = x 2yz- 2xjy, tlnd "i1 f at (I, - I, 3).

SOLUTION

"

') "

')

..

Since "il.f = (2xyz - 2/y)i + (x z + 2x/)").i + (x-y) k. we have

" .fi(,.-,.31 = - 4i + sl- li.


I EXAMPLE

12.7

..._..
If .f(x, y, z) = Tan - t(xyjz} , what is '\1 f?
SOLUTION

(y)]' [

I
"il f = [

I+ (xyfz) 2 z

yz

z2 + x2y2

..
i+

i+

I
I+ (xyfz) 2

(x)] [ (xyjzjl (-xy)],


-

J +

I +

- 2
z

.:r z
X}'
,.
j .
k
z2 + r2y2
z2 + xzyz

= (yzi + xzl- xyk )j(z 2 + x 2y 2).

..........
Gradjems arise in a muJtiwde of applications in applied mathematics - heat conduction. electromagne Lic Lheory, a nd fl uid flow, to name a few - and l woof th e properties lhat make th em so
indispe-nsable are d iscussed in detail in Sections 12 .8 and 12.9. Examples 12.8 and 12.9 suggest
these properties. bul we make no attempt m a comple1e discussion here. For lhe moment \Ve

simply want you to be famj liar with the definition or g.r-ddjems and be able lOcalculate them.

12.4

I EXAMPLE

G1~djems

81t

12.8

liJtHII;IWfMIM Oradie.nt
of function dttining Q sphecc i;;
pcrpendicuht.r to the-sphere.

The equation F (x, y, z) = 0. where F(x, y, z) = x 2 + y 2 + z 2 - 4, detines a sphere of


radius 2 centred at the 01igin. Show that the gradient vecto r '11 F at any point on lhe sphere is
perpendicular to the sphere.
SOLUTION If P(x. y, z) is any point on the sphere (Figure 12.11), then the position vector
r = xi+ yj + zk from the odgin to P is clearly perpendjcular to the sphere. On the other
hand,

'11 F =

zxi + 2 yj + 2zk

= 2r.

Consequently, at any point P on the sphere, '11 F is a lso perpendicular 10 the sphere.

This example suggests that gmdie nts may be useful i.n finding perpendiculars to surfaces (and,
as we will see, perpendiculars to curves).

I EXAMPLE

12.9

'lbe function f (x, y) = 2t 2 - 4x + 3y 2 + Zy + 6 is dctlned at e very point in !hexy plane. If


we stan at !he origin (0, 0 ) and move along the positi,e x axis. lhe rate of change of !he function
is jx(O, 0) = - 4 ; if we move along they-ax is, the rate o f change is / y(O, 0) = 2. Calculate
the mte of change off (x, y) at (0, 0) if we move toward the point (I. I) a long the line y = x,
and show that it is equal to the component of '11 Ji<o.OJ in the direction v = (I, I) .
The difference in values off (x, y) at any point (x. y) and (0. 0) is f (x, y ) /(0, 0) = (2x 2 - 4x + 3y2 + 2y + 6)- (6) = 2< 2 - 4x + 3y2 + 2y. II we d ivide this by
the length of Lhe line joining (0, 0) and (x, y), we obtain

SOLUT ION

.f(x, y) - .f(O, 0}
/x2+y2

2< 2 - 4x

+ 3y2 + 2y

/x2+y2

'lbe limit of this quotient as (x, y) approaches (0, 0) along !he line y = x should yield the
required rate of change. We therefore set y = x and take the limit as x approaches zero through
positive numbers:

lim

x-+o+

+ 3x2 + 2x
J.r2 + x2

2< 2 - 4x

lim

x(5x - 2}

x-o

J'ix

- ../2.

This quantity, !hen, is !he rate of change of f (x, y) at (0, ()) along !he line y = x toward !he
point (1 , 1).
On !he other hand,

'\1 f i(O.O) = (4x - 4. 6y

+ 2)I(O.O)

= ( -4. 2).

and the component of this vector in the direction " = (I, I) is


A

"/t(O.O) . v

= ( -4, 2) .

(I' I)

./2

-4 +2

./2

-../2.

'Ibis example indica tes that gradients may be useful in calculating rtes of c hange of functions
in directions Olher than those parallel lo lhe coordinate axes.

812

Chapre-r 12 Difftttl'llial Cal<:ulu.(j of Mullivariable Fun<:tion~

I EXAMPLE

12. 10

The electrostatic potential at a point (x, y, z) in space due to a charge q tlxed at the oligin is
given by
q

V = - -,
4rr~0 r

where r =
F, where

J x 2 + y 2 + z2

Jr a second charge Q is placed at (x, y, ~). it experiences a rorce

F =
where

qQ
4rrE0r3

r,

r = xi + y,i + zk. Show that F = - QVV .

SOLUTION

\7\1 = "

Weshowthat V'\1 = -F/ Q.

(-q-0 ) = q0 (
-

4rr~ ,.

-q
4rrc0r 3

(xi

\7

4rr~

+ yj + zk)

x2

+ y2 + z2

EXERCISES 12.4

16. Tile C<IUOtion F(x. y. z) = Ax + By+ Cz + 0 = 0 defines.


plane in spat.'C. Show that at any point o n the plane the vector V F is
pcrpcndicuJar to the plane.

In Exercises 1- 10 find tbc gradient of the function.

+ xz + .n'

I. j'(x, y, z) = .t'y
2.

f(x,y,z) =x'yz

3. f(x, y, <)
4.

17. Usc the rcsuJl of Exercise 16 to iUustr<ltc thai a vector along th e

= x'yfz- 2xz

line

F(x,y,z)

f(x, y) = x'y + xy 2

5. j'(x, y) = sin(x

= 2.r + 3y- 2z + 4 = 0,

G(x. y. z) = x - y

+ y)

+ 3z + 6

= 0

6. j'(x, y, z) = lim- (xyz)

is V F x VG. Find pltr:imctric cqu!tliOnS for Ihe Li m::.

7. j'(x, y) = Tan- (yjx)

18. PI'Ovc that if j(x, y, z) and g(x. y, z) both have gradients. then
V(fg) = J'Vg + gV f. What does d>i< oemind you ol1

8. f(x,y,z) =<"' +r+:


9. f(x, y )

= l/(.r2 + y')

10. f(x, y, z) =

1/Jx' + y2 + z'

In Exercises I 1- 15 find the gl'adjenl of the functjt)O at tlle poinL

= .r:! + y 2

19. Rer>eaL ExanlJ)Ie 12.9 for Lhc functions (a) f(x, y)


and (h) f(x,y)
2xl - 3y .

20. Thccqu:~liOn F(x, y)


.r 3 +xy +y-4- 5 0 irnplitillydc fi.ncs
a curve in the xy-planc. Show Ihat at any poinl on Ihe curve. V F is a
nomu~l vctiOr 10 lht:: curve.

II. /(x,y)=xy+x+yot(l.3)

12. f(x,y,z)=cos(.r + y + z)ao(-1,1,1)


13. f(x, y, z) = (x 2

14. /(.<,y)

+ y' + z')' ao (0, 3, 6)

= e-x'-y' at (2,2)

IS. f(x,y,z ) =xy ln(.r + y) at (4,-2)

Draw the surface defined by the equations in Exercises 21- 22. AI what
points on tJte surl"ace is 'V' F not defined'?
21.

F(x,y,z)

= z- Jx' + y'- 0

22. F (x,y,z ) = z

-l.r- yl =

12.5 Highet-On:Jer Partial Dtivati vt..~

*
*

*
*

x' -

f (x , y) = I y'. fi nd \1 f. Find the point (x, y ) at


which V f = 0. and illustrate graphicaiJy the nalurc of the surface
z = f (x, y) at ~t i s point.

23. If

24. If the gradjenl of a funcljoo j (x . y)

i~

\1 f

= (2xy -

y )i

25. Repeat Exercise 24 if \1 f

28. I f / (X. y) ~md g(x, y) have fu'SI parti:l l derivatives in a rcg.ion

= \1g, how "'"

f(x , y) and

= 0 for aiJ pOiniS in some rcgjon R of space, wh~JI can we

29. I f V f

say ab(nll f (x , y, z) in R?

= (2x fy + l)l + (-x' jy' + 2)j.

26. If the gradient, of a funt1ion f (x , y , z) is \1 f =


2yz)j + (xy + y-) k, wh:>< is f(x. y , z)?

(xl + yj + zk)/ Jx' + y' + z'.

R of the xy -pl:>nc. and if in R, \1 f


g(x,y) rclmcd?

(x ' - x)_j. what is f (x , y) ?

*
*

27. Repeat E.'ercisc 26 if \1 f =

813

yzl + (x z +

**

30. Show that if the equation F(x , y) = 0 implicitly definesa curve C


in tbe.ryplane. then at any point on C Ole vocror V F is perpendkular
to C .

112.5 Higher-Order Partial Derivatives


lf f(x. y} = x 3y 2 + ye' ,then
ilf

iix

, ,

= 3xy-

+ ye'

and

Since each of Lhese partial derivatives is a functioo of x aod y , we cao t.ake further partial
deriva tive.s. The partial derivative of ilf f ilx with re.spect to x is caiJed the second p a r tial
derivative o f f (x , y) with respect to x, and is written

a~-(:~) =

a'f

ilx 2

= 6x y-'

+ ye'..

Similarly. we have three more second partial derivaLives:

Cf) = aya' axf =


a2j
a Cf) =
=
ax ay
ax ily
a
ay ax

'J aay (:~) = aoy'


a

'
6x-y

+ eX

'
6x-y

+ eX ,

2x 3

Note that the second pal'tial derivatives 2I ; ax y and a2 f I ay ax are identical. This is not "
peculiarity of this function; according to the following theorem, it is to be expected.

THEOREM 12. 1

If f(x, y). ajtax. ajj ay. a2 ffilx ay, and a2 ffily ax are all defined inside a cil'cle
ce.ntl'ed at a poim P . and are contjouous m P, then at P

iix ay

iiy ax

( 12.1 0 )

814

Chaplet 12 Difftol'entiaJ Calculus of Multi"ru;able Functions

lx(x, y)

Co1Tesponding to the subscript notatioo


tions fo r sec.o nd p:u'lial de1i vatives:

for

IJI fox,

we have the following nota-

Notice the reversal in order of x and y in the middle terms. In subscript notation. derivatives are
taken in t.he order in which t.hey appear (left to right, y tirst, X secood. in 1\'X). In 32I tax ay.
derivatives are done right to left, y (irst. x secood. Because of Theorem 12. I. the order is us ually
ine levam anyway.
Partial derivatives of o rders higher than two are also possible. l,or the function l(x, y) =
x 3 y 2 + yex above., we have

and

a [-aya ( -ax
I yxyx = -ax
,a'-1ay ) J = -axa [-ay
,a (6x 2 y + e") J = -axa (6x 2 )

= 12x .

For most functions with which we will be concemed, Theorem 12. 1 can be extended to say
that a mixed pan jaJ derivative may be calculated in any order whatsoever. For example, if \Ve
require a1ol;ax 3 ay1 where f(x, y) = ln(y>') + x2y10 , it is advantageous to reve1se the
order of d ifferem ia1 ion:

I EXAMPLE

12. 11

.........
Show that the function f(x, y) = Tan - 1 (~) satisfies the equation

SOLUTION

Since

at
ax

lly

-y

x' + y2'

l + -x2

a'f

the second derivative with respect to x is

a.r

Y)

y' - x2 =

ax 2

(x 2

,/ G)=
1+ -

2xy
+ y 2) 2

Since

x'

the second derivative with respect to

&2 f

is ~

oy

-2xy
.,
2 2 . When added, these second

(x+y)

derivatives canc.e l one another, and this completes the proof.

Tlte equation
(12.1 1)

12.5 Higher.Onkr Parlial DeJi\'a(i\'ts

815

for a function f (x , y) is one of the most importamequat ions in applied mathematics. It is called
Laplace's eq uation in two variables (x and )'). Laplace's equation for a function .f(x, y, z)
of three variables is

(12.12)
A function is said to be a ha rmonic function in a region R if it satisl1es Laplace's equation in
R and has continuous second patt ial derivatives in R. In panicular. the fm>ction f (x, y ) in
Example 12. 11 is harmonic in any region that does not con1ain poi.nLs on the y ax.is. The next
two examples illustrJte areas of applied mathematics in which Laplace's equation is prominent.

A third is contained in Exercise 30.

I EXAMPLE

12. 12

Show that the elecuostatic potential function of Example 12.10 is harmonic in any region not
containing the origin.

av
ox

4 JHo

- x

(x2

+ y> + z2)3/2

and

&'V

ax'
=

[ 2.>

4 n:o

(x'

i - z2

+ y' + z')S/2

Sin1 ilarly,

q [ 2y - x - z ]
4JT <o (x' + y' + z' )S/2 '

Addition of these shows that Vxx

q [ 2z' - x' - y' ]


4JT <o (x' + y' + z')Sfl

+ V yy +

V44 = 0. Since second partial del'ivatives are


continuous in any region not containing (0, 0, 0). V (x, y, z.) is harmonic therein.

I EXAMPLE

12.13

Figure 12.1 2 shows a 1-m by 1-m metal plate that is insulated top and bouom. Temperature along
cheedges x = O,x = I. and y = 1 is held at0C,whilethatalongy = Ois f(x) = 4 sin JTx.
Steady-stace cemperature ac poims inside the place is then

Show chat T (x, y ) is harmonic inside the plate.

8 16

Cln'll"lftl' 12

mrrcrc,.'lll i:ll C:IICIIIII!\ ofMUI Iil.'3riuble FIU ~Ii on s

ljilciii.IJ

EJD....1

Temperultlrt in u p!utc

O'C

O"C

T =4sin nx

SOLUTION

Panial derivatives of T(x , y) are

ar
ox = e-'
82 T
iJx2

aT
ily

a1 r
i) y2

C learly. iJ 2 T j iJx 2

4rr

[eT(I-y) - (!- .fi'(J -y)] COS itX~


r.

- e

= e'.,.- - nt -It (eTd - y) 4

e-;;-d - y)J sinnx.


.

= t!'' -e- rr ( - 1r'e:r(l - >> = e'T _

rr e -~t(l -y)) ~in,. x

(Jr2en< l-y)- rr2e- .T(J - y) ] sinJrx .


(~-tr

+ a2 T Jlly 2 =

0 . Since s.econd partial deriYatives are continuous for 0 <

< I and 0 < y < I, the: tcmpcrnturc: fu nction T (.\", _v) is ham1onic in this region. In
prJctice, we are not gic n T(.t , y); we must fi nd it; that is, we must solve Laplace's equation

.t

0 <X < I ,

0 < )' < I,

subject to 1he condi1ions


T (O, y) = 0,

0 < y < I,

= 0.

O < y< l.

T (l. y )

T (x, 1) = 0.
T (x . O)

0 <X<

4sin ll'.r .

I,

O < .x < l.

This is C'llllcd a IJutmtlar.ralt" problem: it is treated in books dealing w ith partial diffe..,ntial
cquatiOilS. Laplace's equation is a panial di ffcrcmial equation. that is, a differcmial cqu;uion

with partial <.lcrivruhcs.

Longitudinal Vibrations of Ba:rs


Figure 12.13 shows a bar of length L and uniform crosssection. If a longitudinal force (io
the x -d irection) is applied to each cross-seclion. the bar stretc hes a nd/or compresses. (The bar

acts like a very stiff spring.) Suppose we denote d isplacement of the crosssection normally at
position x by y(x, t) , where t is time. ntis allows for the situatjon when the applied force

F (x, 1) varies along the bar and is also a funct ion of time. h can be s hown that y(x, t ) mus t
salisfy the following panial differential equation. called the une dimc11Siu,wl11ave cquariun,
4

o2 y
i)[ 2

E J 2y
=
ax1

F(x, I )

0 < x < L,

> 0,

( 12.13a)

where pis the density ofthc material in the bar ~Ul d E is Young's modulus of elastici ty. It is a
cons tant that depends on the material of the bar; the larger the value of E , the more resistant the
bar is to stretch or compression. Given F (x , I) , p. and E. the objective is to solve the wave
equation fo r y(x, 1) , therefore g iving positions of cross-sectjons of the bar for a ll time.
Miiilfiiilii1ilill

End or bar
fixed at x = 0

x=O

Longimdinal vibr:lrion~ ~.,-..f a tur

Cr<)SSsec-Lion at POSilion X when bat


unsttctc.hcd and un..:omprcsscd

. ------ y

A- 0

Po:,ition of \:fu:ss-stction
nurmaJiy at x

x -----

By itself, tile wave equation hn.s many solutions; other conditions must be stipulated. Ncwwns second law governs motion (and it was used in developing the wave equation) . Our
experie-nce. with particle motion suggests that we require. two initial conditions, one specifying
the initi:1l po!\it ions of tnl."-S-Secl icms of Lhe b;lr, and a second specifying their velocil ie~. Tn olher
word'i, accompanyi.ng 12.1 3a will be initial conditions of the fom1
y(x, 0) = / (x),

< X

< L,

(12. 13b)

y,(x, 0) = g(x),

<X

< L.

(12.1 3cl

There \vill also be boundary conditions specifying what is happening at the ends of the bar.
Fo r ioltance. if end x = 0 is clamped in position a nd. therefooe . not allowed w move. y (x, 1)
satisfies

y(O, 1) = 0,
If end

x=L

> 0.

(12.13d)

o f the bar is allowed to move freely, then the boundary condition there is

y,,(L , t) =

o.

> 0.

T here are many types of boundary condjtjons that can occur at

{12. 1'\e J

x=

0 a nd

= L. These a re

two examples.
Lec us cake ;l very ~impl e illuslration. 11te. problem is much easier if displacement is no1
a function of time. in which case we solve for what nrc called static displacements of Lhc bur.
For this to occur, the applied force must be independem of time . Displacement becomes only
a function of pooition, y (x ), initial conditions arc. dropped. time djsoppcars from the boundary
conditions, and [he panial ditl'eremjaJequation becomes an ordinary differential equation,

0=

d1y
E dx2

+ F(x) ,

0 < x < L,

{ 12. 14 a)

0,

(12.14b)

y'(L) = 0.

{12. 14c)

y(O)

If. for irlstance. all cross-seclions arc 5ubjected to the same force. F (a CQf\Slam). then

Fx1
v(.r) = - .
2

d1y
F
dx 1 = - E

+ Cx +

D.

'l11e boundary conditiOrlSrequi re that

= y(O) =

D,

FL
.
E

FL C
= )',(L) = -E
+

C= -

1l1us. displa<.!efl'lent of the cross..section of the bar from its equilil>rium posilion x is

y (x )

FLx
Fx
= -Fx'
+
=
-(2L- x}.
2E
F:
2

Jts graph is shown in Figure 12.14. To get an idcu of the magnilude of these tlisplclc.:emems. we

fi nd tim for a 1-m steel bar with


righ1 end has tlisplaccmcm

y( l} =

E = 2.0 x t0 11 N/m1, and an ilpplied force F = 105 N, the

2 2

105
IO") [2( 1) - 1] = 2.5

thai is. the bar s1rc1ches by only 0.025 mm .

. pm .. rn fKD

S1a11C du.(lhccmtnt.; of 11 b4u bUb;ic<1cd 10 u longitudinul furx-c

The onedin>ensional Wll\'C equa1ion 12.13a is sntistied by longitudinal vibrations of bars.


Jt is ulso sati.sfic<.l by t.ran~ vcrse vibnuions of strings nnd rounionttl vibrutjons of burs. 1lw::...~
will be discussed in the r;:.xl!tt:ise:s.

EXERCISES 12.5

In Exercises 1- 20 find lhc dcrh :!live.

II. /1 3 ffiJ x 1 iJ}' ii'f(x.))= x'e'+y'e'

1. iJ' J/iJ.t' if j(x. y)

= x'>''- 2x1.r

12. 81 ffiJx' ir f(.r. y) =Tun-' (1/.r)

2. a' ffiJy 3 if f(.r. y )

= 2.</J + 3.r'y'

13.

J. iJ' J'fiJz' i f f (x. y. z) = sin (X)'t)


4, iJ 2 jji)yi)z i( /(X. )',Z) = X)'l<''~J-:

5. iJ' j/i)y i)x if f(x, y ) - Jx'

+ y'

6. iJ3 fliJx' iJy if / (.r. y ) = , +r- x 2/y2


7. iJ 3j'fiJ.I'l lll (I, 3} if j'(.r. )') = 3.r1)J- 3xjy
8.

a' ffiJx ay iJz., ( I. o. -I } if J<x . y. z} = .<2y' + x 2z2 + .r1z1

9. a' f/iJx' if f(x, y) = /I - x'- y'


JO. 8 2//iJz' if f(x,y.

z} = ln/rx,' +
-y~
,+-,~
,

a' flax ay' ii' j(x. y, z) = ""' (.r' + y' + z'l

14. iJ2 ffiJx iJ) ot (-2. -2) il' .f(x. y ) ~ Sin- (x 2 + _r 1) 15. iJ"'/IiJx' i)y' if f(x ,y)
16.

8')'/iJx' if f(x,y .

= x ' c'y' + l/y

z) = _,,_,.,o

17. fJ6 j'fDx 1 iJy1 iJz 1 if j'(.r , y. z} = if,<'+ i j y2 + 1/t'


IK. il4 ffiJ.r 3 il.v if j'(.r. )) =ens(x+y 3)
19. iJ'ffiJx iJy i)z ilt i f f(x, )', Z, 1)

20. iJ 2f/iJX iJy if /(X. y}

= j ,...x'"- +-:-)'2"'+- "z'',.----:.11

= Sec - 1 (X)')

az. + r -az
ay

xil.x

.f:

Jl. Whc11 the b!Jr in Figv1'C 12. 13 i$1utncd Y\:nkully 11.nd cl3mpcd :u
.t = 0 ( ftg 1.1rc be)()\.\) , static OOfle\:tions u f the btu mu~t s.otti~fv the
rono~\ling prol)k.m :
'

a:z + 2 n ' <Fz


a
- + y 1-

2t

= 2z = x 1 -

""

..

a.t ay

a-':r

+ )' + Uv/ , Show IMI


:: a'u , 1 a::n 1 a2 u
.r ~.r : '." a.r~ + " D;i

2:!, I f II ~

.t'

a'u
a:u
+ 2.x.r - + 2yz-- + 2x~-ax i)y
ay az.
ax ~z.

0 < " < L.

y ' (l.) = 0.

o.

th:tt would

- ,- - ,T x = O

= .r' - y' + 2.<y + >'


j (A, y) = ln(x 2 t y 1)

'

/(.<. y)

l4.

F(x).

when:: F(x) iilthe wcight oflh:u l);;ut ofrhc bar be: low the ( 1~'(:-iX:Ii\ln
be :u position x if the bar w.::rc u nsrretel~. find the lcng_lh
oru~e b$v n~ it Sll'ttt.:hc,; under its own weight.

o.

In E.xc:td$C.i 2J- 28 find a t'C8ion (if r>o..~iblc:) in whic.".h U~ function is


harmonic.

+.

'

)'(0)

(J1 u

2J.

d'y

= Ed-; +

25. j'(x, y) = xly! - 3xy

Un,;trek:hed

Strct,;hed

p<t!>ilion

)>OSition

26. j(.x. ,\'. z) = J.x 1y: - y'z + "''


27. j(... y, z)=

~ ZS.

1//x'+y'+z'

j (x. y, z) = .r'y'z'

:t. f..

.. Z9. If V(.r. y. ,::) t'Cf.u"C:Sew: the c!ocrrosuuic potential at a J>int


(.r. y. z) due 10 .. systcn ol' n JX'lint ch:u-gc.s ~~ pt'Nnl$ (XJ. y,. l t ) .
docs V (.r, y. z) satisfy L<~plact's e<ruulion'!
gt'JttiUiiOn.tll rxxcnrial m n point (.r, y, t.) in st>.\CC due co ~
U1til0 rm .sphc:ricuJ m u,s distribution (muss M) tlt (he origin i~ dc:tlnC'd

t< JO. The

:lS V = G M / r. w here G is a con.slant and r


,j.r'l
Show th ;~ t V (x. y. z) satis fies l o~p l acc't~ cq u:~rion 12. 12.

+ 33. Wltcu the bat i11 F,f:un:: 12.13 is sui>J'.~l(d to u fol'CC pc:r \Ulili.UCa
of anagnicudc f' oil i1s right end. anti no OLher forces act on dl<: bar.
<XJ,\I~UiOOS Jl.J4 ror &
ta!ic dcllccliORS become

+ yl + t l.

Oo

JJ. The tigur'C bdow shows o JJhiiC boun'-'cd by the lines .r = 0.


0. x
I , und y
I. Temperature alon,g the lirst Chrcc sides
is l:.cp m <f>C. whi le lluu ulon,g )' = I \'u_nc~ :sc:cording to f (x) ~
sirl(Jx-x)- 2si n(41r.r) , 0 := x := I. The tcmpenuureul Hfl)' point
inlcrior 10 dlC pl ~ l e is then

T (.t,y)

C(c~r- , -'"'')sin(3nx)
~- 0(e4 '

where C

(t>1" -e-

1
"")-'

antJ D

.v(O)
Find dispiOC'C il)COtS

34.

e- 4" ) sin(4;u).

' -

t-'") - 1 Show tlmt


I . 0 < y < I. mld that

x <

i l ollso ~al i:dic~ lhc bou ndary COI'ldit tons

7"(0, y)

= 0.

y'(L)

= FfE.

for tn)f>$-~ l i cms.und

the &cnglh ul'th-c har.

(a) S how then \\hen P(#r.,J) ~ 0. wa\"C equation l2. 13a is


:o...atisficd by fufk:ticms of the l'urtn
,I'(.<. I) a

= -2(t!"" -

1' (.r. y) is la<tNtllvnic in IJN: n:gim1 0 <

0 < .x < L .

(tlsinl.x + llco>I.X)(Csinc;.l

c> = f:/ p .

= 0 , T( l , )') =0 ,

where;,, A, 8, C, und D arc::~.rbi trat)' COns tant~.

7 (X, 0) ; 0. Ulld '/' (;r. I} : j (X) .

( b) Show that bound:'lry ct1nditions

B
T= .f(x)

oc

oc

~ 0 c;<;<;C), I).

=0

~Ultl

i.

= (2n -

12.13d and c require th:tl

l)n/(2L) . where 11 is an integer.

(c) If g(x) iscquiiiiO :,cro [bul not /(.<)], wbaLis

C?

(d) Lf f(x) is equal 10 r.cro [bul not g(x)), what is

D?

35. l fLheeods ofdte laut string io the figlll'ebelowatc fastenedat x


0
and .r
L on dte x -axis, and if che SLI'i ng vibtates i n the y-ditoction
only, small diSJ)IaccJnents y (x , 1) must satisfy dte one-dimensional

wave equation

Cb:.p1c..- J2 Differauial ~cul us ~fMnltiv:uiablc Ptl~Xtjoos

820

i ...
J'y
F(.r.l)
= c, a.r1
- + --p
'

0 < x < L.

>

0~

=r

where c 2
I {J, T is the (cQtl$1,.artl) tension uf lht: ~lrin:. itlld f) j~
the (constam) mass per uni1 length oJ' the strin,s. Function F(:r > t ) is
1he resuh or all forces per unit leng1h ac::1ing a1 pt)Silion x <lnd lime
l in the:. StJing (except rM tcosion in l11c string). AOC.."''tllpall)'il\g this
cqualloJl '"'ill be io.i1i:1l cooditions l 2. t3b :~.nd c specifying tbc position
and velocity of the Siring ;II 1i me 1 = 0, and 1he: bnundllry condi ti un~

y(O. I) = 0.
~ince

y(L, 1) = 0.

I >

J8. A tout string, has it~cnds lixcdaL :r - Oand x - Lon tbc .\' i)xis.
At time 1 = 0. the string is lllO\'Cd so as to take the shape of the sine
t un-e f(x) 3 s i1l(rr x 1L), aml lhen releasee!. 1flhe only fl'lrte Iaken
imo aCCllunt as acti1lg on tJlc striog is i.ts tcn...:;ioo. dK".Jl displacc nlCn L~ o r
lbc string mu~t satisfy

a'y
-- c' Dx
1'

y (x, l )

= y(l.. 1) = 0. I > 0,
)'(X . 0) = f(x) = 3sio(rrxf/.) ,
o< x
y,(x. 0) = 0,
0 <X < (.,

0.

consuuus.

3L /(8c). and L /(2c).

ih

(b) S how tha( the boundaJ)' conditions require 8 ::;;;: 0 a nti

). =nil'//..., wbcrc 11 is an integer.

39. Showthat whcnf(x) = 3sin(7r.t/ L)-2 sin(2;rxf L), inExercisc 38. Ute problem is satisficd by y(x. 1) = 3 sin(rr.t I L ) cos(n Cl/ L )
- 2 sin(271 x f L) cos(2;r ct/ L). Plot y(x. I) as o function of x for
1
0. L /(8c) . L / (4C). 3L/(8c). and L/(2c).

(c) lf g(x) i<equal w 1em [bul n01 j(x) ), wh;u i< C?

(u)

/d) If /(x) is equal lo zero [but not .~(x)). what is D?

JJ tJ1c length of the striog in Exercise 38 is L


X/ 100,

['~y(x,t)

sen e.~

Show that this f uoctiOI'l satisfiC$thc partial din<::rcntial equation. boundary conditioi\S. and the second initial condi tion

J l\,

+ F(x ),
dx

0 < X < L)

0= T --',;

y(L)

in Exercise 38. (Assume that diticrcntlati.oo and summatlon


OJ)a"Jtions c-an be imccchangc.d.)
(b) Suppo~ the fir-51 20 lcrnJs of Lhc. series an: used to ~pprox
imme y (x, 1) . PlOt 1hc sum orthese 1enns as a functiOl or
X rorthc time< I = 0, l/ ( 16c), 1/( ~c), 3/(16c), 1/(4<) ,
5/ ( L6c), 3/(8c) , 7/ {16c), and l /(2c). Ooc. lhc "rit~g

= 0.

\V"hcn the on1y force acting on the sLring (other tl1an internal tension)
is gnwity,then F(x)
- Y.8 1p. Show that the solution ror y(x) is
t~ purnbolu. What unckrlying assumption(s) UHlkc thjs problem and i1s

solution different from Example 3.39 ln Sc.."Ction 3. 13?


l7. A siring or length I.. is stretched tighlly along the x .. axis. Its right
end ls lixcdoo1bc x axis>and its left end at :r = 0 is looped around 1hc
y axis Wld is free 10 move thcrc..on without friction. The Lring is slowly
lowered under the influence of gr.tvity to Lake up a static position. 'fhe
s.hape of' lite string is defined by Ihe di !Terentia I equaaion
tfly

= T d- x-,

appear 1.0 retain iLS brokenl i oe~ha l>e ?

if*

41. The uoifor111 circular rod in the figure bc:low has flat ends at x
0
atJd x ;;;;; L . lf lhc round side of the rod is pcrfoctly ins ulated: beat
flows in only the. x-direc~ ion . When no heat sources exist wilhin the
rod. temperature T (x , t) at points i n the rod mus-t satisfy the one.
din'ttnsionol heat C.t)nduction cqumion

ar

a' r

at = k ax'.
- 9.8 lp,

y'(O) = 0,
S ol\'c his proble(o.

(-1)-1

tl= l

n.c problem

then is

= 0,

"'

y(.r,l) = -:-""
sin(2n - l);r.rcos(2n - 1)7rtl.
h>r 1 L., (ln - I) 2

Static dcOoctiOfls of the string i.n Exen:-isc 35 occur when F(x, t)

is ODI)' a funCLiOil Of X. and iniliaJ COOditiOil S ~lfC ignored.

lj2::; x ::;l.

disploccmen1s can be cx.prcSS(d in the fonu of an infinllc

y(O)

= 1 1.n, und

0 :::: X :::: l fl,

f(x) = { (1 - x)/ 100.

* 36.

r..

Sho'v lhal ,r(x.l) = 3sin(rr.t/L)cos(n ci / L) satisfies


this problem.
(b) PIOI. thc surface y(x , t ), interpreting c~scetioos physic:tlly. (Usc L = I and c = 2.)
(c) PlO< J (.t , I) asafunction ofx fort = 0 , L/ (8c), Lf(4c),

= (r1sinh + B cos)..x)(Csincl.t + D cosc!.1),


:tre arbi1rary

<

(a)

= O, the wlh'l::e<}ua tionissotisficd

where ;., A, 8 , C, ;:md f)

> 0,

y (O, 1)

both ends of 1he string: are attached to the x -::u::is.


(a) ShO'"'-' that whcn P(.t,t)
by a function or the fonn

0 < X < L,

<X

0 <

< f.. ~

> 0.

< L,

y( L) = 0.

x =O

Perfocl insulation

t2.5

where k > 0 is u constwH called the tllcmwl Jijfusi\i1y of the rod. Ac


companying 1his equal ion will be 1wn bOundal)' ronclilion;:; descrihing
what is happening Ill lhc ~nds of th-e rod. For iltS(IlnCC, it' the left end
is kept :ill.c mpcntlurc O""C and the right end is insulated. T (x. t) mu..il
s:tlisfy
T(O,I ) ~ 0. T,( L ,1)
0. 1 > 0.

+- 43. lf tbc ends

attemperalures

<X

i)r

a'r
= k

0 <

ax' .

T(O. 1) - 1;,,
T(.r. 0)

< L.

(a) Show thai the hea.~.conduction equation issat.isficd by fUJ')(:


tions or the form

L of the rod in EAc;i:sc 4 1 urc held

r: c and r;c for 1 > o. temperature T (.r . I )

fiT

In addition. then: wil l be an initi;al condilion describ ing the lcmpcrnlurc


distribution in the rod uttimc t = 0.

821

mu~1

satisfy

T(x. 0) = f(x).

.x - 0 and X

lligher.Qrdet P:ltli~l l>tri\':Ui \~.f

.'C

< L!

> 0.

T(/.,.t) = T,_.

> 0.

0 < .r < L.

f (.r).

A l\cr u \'Cry h.)t1g time. Lhc lcmpcru.turc Ill each IXJinL in the rod will
remain constant!. btu tCit'IJ>C-r.llurc will \':tty from poin1 to poim . Tent

perature ls snid lo have rc.achcd o. steady-Slate situlllion. ll ctlrt be foUJ\d


hc;.ll conduction equation. mxl
the initial conrlilion. Temperature becomes ::t function of only -f. which
s:ati:s.fic.s

by rcmovi na the.: 1crm OT /Ut fro m the


where

A. A. and 8 are arbilrary constanL~.

(b) Show that the boundary cc11ditiono roqu.ir<: () = 0 an~


}. = (211 - l)rr/(2L ), whcren is an integer.
(C) When the initial tcmpcr.nurc or the rod j;;; I (X)
X. so
tbat it increases linearly from O"C ot its left end to !... "'C nt
its light end. tcmp.:rnturc thcrc...Lilc:r is

d' T

0 = d.r' .

T( X. I )

= 8L
z ~
L.,

"

( - 1)"-1 _, , _l):'*'i<'> . (211 - l);u


t
Sill

(:!11
l) 2
2L

= 0 plot appn.u.i matc /(x) . anti do lhc n:majning

plots reflccllhe bound:try condit1ons'?

ii,.-

* 44.

If heal is iJddOO to t~ rod in Ext;rdsc 4 1. the l ~itl conduction

0 <X < L .

a'T

0 <

at = k i)x 2 '
T (O.t)
T (x.O)
(Q)

.t <

T (L.t) ~ 0.

L,

<X

t > 0.

~t=l

x(l, -

- l )1rx

+ F(x ),

T(O) = 1o,

= Tc.

> 0.

< X

< L,

m21s. Does the 1 = 0 piOI

T(L) =

ft.

Find T (.t) inchecasethat F(x) isaconstantv:luc F .

+ 45. The figure below .$how~ a bur o( length L and uniJorm cros.s~
$..'C~ion. When tOI'ttuc is app1it0 tv cn~~CCl i or\:,, d'IC.'lc cro~M."\:tion.)

'

I
I

Plolthc sum of the first 20 lcrms of lhis series as a ftmctjon


cf x forohc tioncs 1 ~ 0. I0. 100. 1000. and 10 000. Usc a
l m rod 1md k
1.14 x 10--4
approximate /(.t)?

T(L . I)

< L.

-r-n-n''tt'
. (211
'
S ill

,~

1211 - l)

T0

0 = k-d2

.r).aempcr.uure thcre:after is

L.,

d 2T

> 0.

(b) When the initiaiiCIIIJ>CrdlltrC ortloc roo.! i> /(X)=

,.

> 0.

lf F (x ' t ) is inclepcodcnt or I ' lhcn the steady-state u:mpctature of the


tod (sec E:<C(cis.c 43) musl satisl'y

Show th:U ror the: functions in part (a) or Exercise 41: the
bvundary CU~lt..litioo) rc:ttuirc thal B - 0 and A - ttlf / L ,
where n i< an integer.

81,'
7.(.l.. l ) = .

T(O. r)

.r

j(x).

where F (x , 1) is I he heal source tenn. I f temperJtures or the ends or


the rod arc held nt l'OC and r;;c. tllcn T (-', 1) must also satisfy the
boundary conllilions

42. If both ends: of the rod io E:~:ercise 41 nre held at temperature 0'-'C.
the problem fc~r T (x , 1) becomes

ar

T(L) = Tt .

T(O) = T0 ,

equ:Uion l:d.:$ LhC ronn

Show aNt this function satisficslh partial di ffcrcntial cqu:t


lion and IJoondllrytondilion~. (A&iunK: lhul diO'tn:nlilllion
and ~ununa 1ion operation<can he interchanged.)
(d) PIOl thc !!>UDl ofthe: first 20tc.m1S ol'thcsc.rics as a function ol'
x fotthe timeu = 0 , 10. IOO. IOOO. cmd 10 000. Use a J.m
rod and k = 1.14 X 10-4 m'fs (typical for copper). Docs
lhc 1

0 < .r < /.,

'

x=O

arc ron.-al corocacc. l..cc,r represent distnnec (rom the lc li. cndofth~ b;.lf,

J6. Show thtlt when c-(.r) i" unspecified, Sl:.11 ic ro&tuk>ns in JS):crcisc
45 crm be CXJ)I'CSS(d in the rorm

and J'(x. t) the angular displocemcnc of the cross-section at position


.\ and tinlC 1 from its torque--free 1,ositi01\. It Cllll be shown that when
r(x .r) represents Lht torque J~t u11it length ill positi{)."'' x o.nt.l ti rnc
1 . und p is the {constant) dcnsi1y of the bar. )' (.t. 1) must satisfy the

,I'(X)

= Cx- -1 f.' f." r (u) dudtJ.


fl

one-d imensional \\'8\'C equation

8 1y

2
18 y

= F.Jp

I)

where

r (.r, I)

"'
= c oX
:;-; + -p-.
ol
w here c !

0 < X < L,

t > 0.
C =

and is Young's n1odulus of elastiCity

1 ~.n~ r(.<) d.<.

or lhc rll<l

!erial in the bar under shear. Accompanying the panittl diffc,eMial


cquttliOI'l will be initial cor.diti(W'l.'; 1'2.13b anct c:: !>pecifying the ini tial
displacemenu and vclt) citics of t.Y()S.S scclions at tim e 1 = 0. b1 ~ddi

" -&.7. Twu functions u(.\". y) and u(.r. y } :~ru said to be harmonic con
jugaces i f they satisfy lhe Cauchy-Riemann \.XJUUtiollS of Exercise 4 l

in Section 12.3. Sl\0\\ that if u(x, J) and u(x . )') arc hlnnonie
..
con
j ug<ltes and ht:I\'C t.'OIUinuuus second pat1iul dcrivalivcs in v. region R .
th<'n each is ham1onic in R.

t ion, there will be two boundary conditions de~ribing end conditions.


Rx cl.ampk. if the lcfl end i:s damped so thru no rotation is pos.:.:ible, and the righl end is r.-ce to move. the bourldaty condiLiot'ls will be

12. 13d und c. T he problem i:s 1hcrcforc idc..'fllico.d to that ror longitudinal

,.;brolliom; of bars. Stark; romtiotJS occux when t (x ,/) is independent

43.

= .rl -

y 1 is hannon.K: in

UsctheCa uchy-R iernarli~ Cq ll alions in

E.Acrcisc41 orScG

(a) S how that lhe furo ion u(x, y)


the entire .ty plane.
4

of t , the time: deri\''llli\'C i.~ removed from 1he w~l\'e equ.-tion , nnd initial
(.'Ooditions arc deleted. Oi.'t.placcrnento; y{x) nltJSt thc:n sa1 i~fy

{b)

lion 12.3 to find a runclion v(x. )') :-to that 11 and v arc

d2Y

0 = d ,

'

y (O)

+ r(.<) ,

= 0,

hannonic conjugal\!$.

0 <

/(L)

< L.
,. 4!}, Repeat Exctc::isc 48 ir u(x, y) = ~ ' <::us y

= 0.

+ ' .

50. fot what \"alucs of n docs the run('Lion (x 2 + .'1 + <2)'1 s atisfy
e<tuation 12.12'? In what rcgio.1J arc tho! functions h!ltmonie'!

at k aSI ror the botu1~ll)' oonditions described ttbo\'C. SoiYclhis pmblcm


when c(~r) = x so Chtu to r~wc i1lCrUM:S linearly along the bar.

112.6 Chain Rules for Partial Derivatives


If y = /(u ) md u = gtr) , the chain rule tb rthe deri,ative d y/dx of the composite function
flg (x)! is

dy
d.r

dy du

(12. 15 )

du dx

Equation 12. 15 can be extended in terms o f more in1enne<.liatc \ariables. say y


1/ (S). s = lr(x) . in which case

= f<u) .

u=

dy
dx

dy duds

= ---

(12.1 6 )

duds <IX

For multi\'ariable functions. variations in chain rules arecount le,l\s. \ Ve discuss two examples
i n ronsidcrabledctail. a nd thcH show schematic d iagranlS that easily lead to chain rule.~ for even
1hc most compl icntcd functional si twuions.
SupJXJ'\e z is a function or u :u'ld u and each nf 11 and v is a function or x n1ld y ,

z = / (u , v) .

u = g(.x . y),

v = lt (x, y) .

( 12 17)

By the substitutions
z = j{g(x . y) . ll(x. y)l.

( 12 18 )

az;ax.

we express z as a function of and y, and can then calculate the panial derivative
However, if the fu nct ions in 12.17 are at all complicated, you can iJnagine how d iflicult the
composit.e functioo in 12.18 might be to differe111iate. As a resuh. we search for an altenmtive
procedure for calculating
name ly, the appropriate chajn rule. It is conLained in the
following theorem.

az.;ax'

12,6 Cham Rules fOt PattinJOerhmives

823

THEOREM 12.2
Let u = g(.<, y) and v

= h(.x , y} be coouinunus and have first partial derivat ives with


re-Spect LO x at a point (.x , y ) . and let z = f(u , v) have continuous first partial derivatives
i nside " circle centred at the poi nt (u, u)
(g (x . y). h(x. y)) . Then

=
azau
azau
+au ax av -.
ax

az =
ax

(1 2.1 \))

PROOF T his result can be proved i n much the same way as chain rule 3.20 was pi"Oved in
Section 3.7. By Definition 12.3.

Ol.
. f[g(.r
- = lim
ax
.:.x-o

+ 6.<, y), h(.r +

C..r, y)j - flg(x, y), h(x, y)]


.
C.x

Now the increment C..r in.< produces changes in11 '"'"'

+ D.x . y)- g(x ,y) ,

D.u = g(x

v. which we denote by

D. u = h(.<

+ D.x ,y)

- h (x, y).

Tfwc write u a nd 11 whenever g(x. y} and il (x, y) are evdlu ated at (.t , y), and s ubs titute for
D.x, y ) in the definition for 8:/lJx , then

g(.< + D.x, y ) and h(x

+ D.u, v + D. v)-

f(u

Inn

Ax

~., ~o

.
[ j {u + D.u. u + D.v) = 6"-o
Inn

1om

f(u . v)

[f(u

~" -o

/ {u , v

+ D.v)j + [ j(u . v + D.v)

- j'(u, v) ]

~x

D.u, 11 + t.v.) - j(u , u


Llx

C. v)

f (u, v

C.u)- .f{u, u)J


.
Llx

We assumed tluJL the derivative

ilz
- =

dv

Inn

f (u, v

+ D. u)

.Ott- O

- .f(u , v)

8 u

exists at (u , v) . An equivalent way to ex press the fact that th is limit exists is to say that

j (u , v

+ D. v)- .f(u , v)
D.v

where l 1 must satisfy the condition that lim Av- o fi1 = 0. \Ve can Wli te. therefore. that
/(11 , v

D.v) - /(11 , v)

[Zu(u, v)

+ eJ] D.v.

Similarly. we can write that

f(u

+ !J.u, v + D.v)

- j (11, v

+ D.v )

= (z,. (u . u + D. v)

where lim.!.\,_0 :
0 (proYided that D. v is sufficiently small ). \Vhen
substituted into the limit ror iJzj ax . we have

()~ =
;;u.<

lim

cu-o

lzu (u, v

~X-+0

lh ~ expression~

/J.u
D.v }
+ D.v ) + <21-D.x
+ [Zu (u , v) + <,] -6x
.

We now examine each pal'! of this limit. C learly,

D.u

au

D. X

ax

li m - =

+ 2]D.u ,

and

lim

~X-+0

D. v
ll X

ov
QX

are

In addition. because g(x . ) and h (x , )') are.comi11uous, ll u


Consequently.
1

Jim

A.t-0

lim

.4li ~O

=0

lirn

and

~.\-o

0 and IJ.. v

lim

0 as 6 .x

0.

= 0.

~~, - o

Finally. bec<1use iJ z/Ou iscontinuous.


lim z,, (u, u

a..--o

= o.v-tt
lim Zu(ll, v + ~ u) = z.(u, v) .

+ ~v)

\Vhcn all these results arc taken imo account. we

az

,-- =
oX

au

+ <.

Zu (ll , u) ,vX

ha v~

ou
{1<, v), vX

which complc~es the proof.

C hain rule 12.1 9 defi nes 8zfilx i n ten ns <lf dcriv.uives Qf the given Ji u>Ctions i n 12.17. We
could be m ore expl icit by i ndicating which ' '.ari ablc is being held co11stant i n each o f the five

derivatives:

uz ) a.u)

<lz )

ilz ) iJv )

ax )' = a,; .a...- ' + au ax

)'.

Fmm the rmint of view of ro tc~ of change. this result seems c1uite rea:-.onablc. The left :-.ide is
the rme of change o f : with r<"pectto x hol<! ing y consr.nt T he first term <<1:/iJtJXiJufiJx)
uccoums for the n11c of change o f z w i th respect to thoscx s tlwt ut1'e<t z through 11 . Thcscc()nd
term. ( UZ/0 U )(()v (OX ). llCCOUiltS for the nllC Uf chngc of Z w ith respect tO those.< 's thut llfl'ect
z through v.
tmal rntc of ctumgc is then t he. sum of the two pnns.

n,c

Consider now the fu.nctional situation


Z

= j (u , u) ,

= 8(X , )', s),

II

= il (x , J, S),

X = p (t ),

)'

= q(t) ,

S
r(t).
t 12.21)

By the substitutions
: = f[g (fJ(t ). q (t ) . r (t)) . h (p(r) . q(t ) . r (t))).
we express

11 2.22)

as a function of r alone, and can rtherefore pose the problem of calculating dz/d t .

I f we reason '" in the preced ing pmgraph. th-e appropriate chai n rule ror d~jt/1 must Hccount

fl"r all r:. affectitlg

z: lhr'ough u and

IJ.

th

'Ve obtain, then,


a~

tlu

oz tlv

-=--+-tit
Qll Lft
OV dt '
where we have written du f dt a11d d u/dt because 11 and u can be ex pressed entirel y i n terms

of' t :
u = g [p(t ) , q(t ), r (t)J,

h l /l(t ), q(t) , r(t)J.

Cha in niles for each o f du fd t and d vjd t (simil ar to 12. 19) yi eld

du
dt

au dx

uudy

iJu tis

<Ju dx

- - + iJy
-+as- d-.
ax dt
dt
t

ovdy

()v d s

-+ - + --.
(lx tit
()y d t
OS d t

Finally, Lhen.
d z = az ( oudx
tit
au ax dt

+ oudy + auds) + oz (ovdx +


oy dt

as tit

av

ax tit

av d y
oy tit

+ o v ds)
as dt

which expresses d z(dt in tenus of dei vatives of the g iven functions in 12.21.

(1 2.23)

'

12.6 Ch:lin Ruks for ParliJI Otrii:Jiil'eS

These two examples suggest the romplexities that may be involved in finding chain rules for
complicated composite functions. Fortunately. there is an arna7ingly simple method that gires
the correct chain rule in every iwation. The method is not designed to help you undcrswnd rhc
chain rule. but to find it quickly. We suggest that you test your understanding by developing a
few chain rules in the exercises witha discussion such as in the second example above, and then
check your resull by the quicker method.
In the first example we represent tl1e firnctional situation described in 12.17 by tl1e cl!ematic
diagram to the left. At the top of the diagram is the dependent rariabJe ::. whkh we wish 10
differentiate. In the line below z arc the variables u and v in terms of wbic/1z is irriciafly
defined. In the line below u and v are .r 's andy's illustrating that each uf u and vis defined
in tcm1s of x and y.
Here arc the rules to obtain the panial dcrivncive from chemmic diagrams, in gctJcral,

/~
1'

II

I
X'

/
)'

'
X

)'

followed by

a:;ax for the specific example:

1. Take all possible paths in the schematic from the differentiated variable10 rhe difl'eremiming
\'ariable.
2. For each straight-line segment ill a given path, differentiate the upper variable wi!ll respecr
to the lo\\'er \'ariable and multiply together all such derivatiles in that path.
3. Add the product~ together ro form the complete chain rule.

To calculate 8~/fJx from the schematic to rhe left. we nore ch1Jt there are rwo paths from z 10
x . one through u and one through u. For rhc path lhrough u we form rhc producr

and for tbe pa!h lhrough t,

az av
au ax

The complete chain rule is theo the sum of these products,

a:

ax

az au

az av

-fJu -ax + -fJv ax


- ,

and this result agrees with 12.19. The schematic diagram also indicates which ,ariablcs are to
be held consu111t irlthe derivatives on the right (as in 12.20). AU other variables onrhe same
IC\C( an: hclrJ COR5C.U1l.
For the scconr.l example in equucions 12.21 the schcmmk diagr.un is to the Jell. There llfC
si~ possible p211\.< from z to 1. so that the chain rule for d~/dt mu<l have sittemt_< We lir.d

a:.

dz
audx
dt = 01/ iJX dl

a: au d)

a~ <Juds
au a)' dt .,. au Js dt

azavdx ' fJ:r?vdy


+ /11 ax df T OIJ 0)' dt

a:auds
dt' as dt '

and thi~ agree< wilh 12.23. Norc roo thai if 1~hcn forn~ing >~ dcriVllliv~ from the ,o;chemllli,c di:o:
gram, there are two or more lines emi6natir.g ~rom a \1l_".3~le. fhen we obtarn a J>--11tlal den\-:Htl.!C'.

tf tl\ere is only one hoe, then '"'e ha,e an ordmo.ry derrVllll\e.


( EXAMPLE 12.1 4

825

....,.
Fiud chain rules ((J(

-a~ ) and
ax y

a:)
ay '
-

if
z = f(r,s ,x),

r =g(x ,y),

s= lc(x ,y).

826

Cbap1c: 12 l)iffcreOiinJ C.h:ufusofMtLhi\'a.dabk Fml1.'1.ioos

SOLUTION

From the schematic diagram to the left,

ilz ) ilz) ilr) az) !Js)


()y

:t

ilz)

ax

v
I EX AMPL E

12. 15

ar

t,._.ay

il:)

=-

X+

r.xaY /

ar) + -ilz) -ils)

ilr ''ox y

as

lis

r ..r

ox y

a:)
.
ax ''

+-

In Example 12. 14 it is essential that we iJtdicate which variables to hold constant in the partial
Ucrivathes. Tf we were to omjt the~- dcsignaLions~ the.n in the second rcsull we would have a
term azjiJx on both side.~ of the equation buc they would have different meanings. The tcnn
a:.j<J.x )y indicates the derivatiYe of Z with respect to.:'( holding y constant if Z were expressed
entirely in tern1s of .r and y; the term Oz./Ox),," indicates the dt:rivaLivc of the ghen runc;tion
f(r, s, x ) with respect to x holding rands constant.

Find dz f dt if

x = 2t

/I"'
y
,

,'(

I I

SOLUTIO~

+ -.
I .

y = t 2 e'.

From the schematic diagram to the left.

dz ilz dx ilz dy ilz

- = -- + -- + ax dt

dt

d)' dt

at

(Jx 2y 2 +sin y + r)(2 -

1/ r2 ) + (2x' y + x cosy)(2te1 +

r 2e') + x .

\Vhen a chnin rule is used to calculale a dcrivalive.. the result usually iwolves all intermediate

variables. For instance, the derivative dzf dt in Example 12.15 involves oot only 1, but the
imermediate variables x aotd y as well. Were tlzfdr required at t = I. value.~ of x andy for
t = I would be calculated - x( L) = 3 and y(l) = c - and all three values substituted to
obtain

dz
= [3(3) 2(e)2 + sin(e)
dr r=t

+ 1)(2 -

1)

+ [2(3) 3e + (J)cos(e)](2e + e) + 3
= 1378.6.

I EXAMPLE 12.16

Find 82z/8x1 if
?
z = st

. t,
+ 2 sm

s = xy - y,

= r +-.
?

)'

SOLUTION From the schematic diagram tOJ the left.

ilz

'

.\'

'/"'-

)'

:r

/ "'-

)'

as

az fi t

Now

y/x 2 ) .

'iJz/ax is a function of .f , t r. and )' ,and thcrdo re in order co find

)'

~1\

/"'-

II:

-ax = ;--,
+- = (2sr)(y)-"- (r + 2cos t)(2x i!s i!x
ot iJx

I EXAMPLE

/"-J

12.17

.(

)'

we form n schem~tic diagnun t'or

fJz/rJx. From this schematic diagram, we obt.ain

!:.(ilz
)as+ i ( ax8z )~8.r + i!x~ ( iJx!Jz)
as ax ax
81

= (21)' + 2s (2x +

<, 1, )'

yfx 2) ]()')

(.r 2 + 2 cnst) (2

[2.~y- 2sint(2x- y/.r 1)](2x- yfx 2 )

+ 2yfxJ) .

Temperature

T at poinL' in the aunoo;phere depends on both position (x, y . z) and time r:


T = T (.r, y. z, 1). When a weather balloon is rele..se<! tQ wke ten>perature readings. it is not

free to take readings at just an) )>Oint, only at those points along the path that the winds force
the blloon 10 follow. This pilth is a cur,e in space rcprc."'Ciltcd parametrically by

C : X

= x(t),

)' = )'(/) ,

= Z(l ),

I 2: 0,

f ngain bei11g time. Jf we SUbstitUIC fr()tn the C(fU<.it ions for C intO the temperature function.
then T becomes .u fm\Ctioll of I alone,

T = T(x(r). y(t), :(t), 1).


and this fu nction of time <Jescribe..s the te rnpcrature at puiuls a lung the path o(thc balloon. For

the deriative of this funclion with respecllo f , the schematic diagram yields
dT

aT dx

aT dv

aT dz

= -- + -dt
a.r <11 uy dt' + h cit

aT

+ -ut .

'l1le ques tion we pose is: Whal is the ph)'Sical difference between dT/clt and oT tat ?
SO LUTIO N Temperature at a point in s~e is imlepemlent u f the o bserver measuring it;
hence T (x (t ), y (t). z (t ), t) is the temperature at points on Cas measured by both the balloon
and any observer tixed in the XJ .: re ference system. Jf, however, these two o bservers calculate

the mte of ch3nge of temperamre with respect to time at some poi111 (x, y, z) on C. they
calcu lme different results. The observer foxed in the xyz -refere,lce system (not restricted to
move along C ) calcuhncs the rate of change: of T with respect to 1 as the derivative of the
function T (x, y, z,t ) partially with respect to 1 holding x. y. and~ constant (i.e .. the fixed
observer calculates 8 T jar as the rate of change of tempemture in time). The b<tlloon, on the
other hand, has no alternative but to take temper.tture readings as it moves alo ng
measurement of T a.-.; a func tio n o f I is

Tl x(t), y(t), Z(l), t ].

C; thus

its

Therefore, when the balloon calculates the time variation of temperature. it is calculatino d 1d
1
\t follo\\"S, then, that the teons
e

ar dx ar dy ar d:
ax dt ay dt a, dt

- - -r - - + - describe that pan of dT /dt caused by the motion of the balloon through space.

.........
Dio;cussions like those in Eltamplc 12.17 are promifl(nt in the tudy of fluid motion (gas or
liquid). Sometimes rates of change from the point of ..i.,.. of a fi.\ed observer are imponant;
other times. rates of change as measured by an observer moving with the fluid are appropriate.
Many imponant applications of the chain rule occur in the field of panial differential equations. The following e"<ample is an illusu-ation.

\. EXAMPLE 12.18
The one-dimensional wave ~uation

for function5 v(x, r) describes IJllnsverse ,;brntions of taut string": and _longotu~in,al and r_ma:

0!' meta1 ba \"1. SL-


funaooru
ttonaJ
Vlbr".IIIOI\S
n uw IIw if f(u ) anJ g(v) are tl'1c:e-dofferenuable
.
of u and 11 then y(x. 1) = f (X + ci) _ g(x _ cl ) satisfies the ~ave equluon.
SOLLl \OJ\ The sche.'Tiatic diagraDitO the left describes the functional situation

,.
/"'- t
X

II

where u

.l

- V + Ct ''""
-

= j(u) + g(v)

Cl The chain rule f()r 8yjal is

ay - CJy au -

ac -

au ac

ay au
av (lc

== cf(u) - ci(t).

The schcnmt.c Jiagram 'or a~'f'i)c loads 10

dy

ac1 ==

"

== ltr(u))c -r-1-cg"(vJI(-c)

II

a ("))au + !.... (3>) 3t


au as . ac a~ ac ac

at>

;)I

A simihn

~ak"Ulation

'

o"~'

cl (/"(11)

g.aves Oxl -

+ 8 ' (v)).

f"(ll ) _,_ g"(v). Hence _I'(X, /

) doe< incked <:Jti<Cy the

wa,c equation.

..-.

larl) in elec
~
u.ation is lO engineenng. part~
esled oo.- imponanl Laplace s eq .
I . The IWo-dimensoonal Laploce
We ha'e sugjl
.
O . l now and deflecuon or P ,l\es.
foci .IS Conn in polar
uosuuics. heat conducuon. "" . ,, II lo lbc next example. we '

coordinateS IS -
equation in c=an

coordinates.

I EXAMPLE

12.19

Find Laplace's equation in polar coordinates.


SOLUTION Cartesian coordinates are related to polar coordinates by the equations x =
r cos (J. y = r sin (J . The in\'crsc Lmnsformation i:;

II = Tan - (~).
The second of these is not always conect; it may nocd rr added to it. Since the derivation
here requires only derivatives of 0. and not 0 itself. the ll' is inconsequential. Suppose
V = f(x , y) is a functio nthat saLislies Laplace's equation a2 V jox2 + o2 V joy2 = 0. We
can express V in terms of rand II by writing V = F(r,O) = f(rco,/:i,rsinO). The
schematic to the left represents the functional situation where V = F(r, 0) and r and 0 are
expressed in terms of x and y. From it.

avar

av

() \loll

-+-.
01' ax
ill! ax

-=

ax

where

01'

ax
ae
ax

r cosO

Jx'

+ y'
I

+ (yjx)2

(-y)
x
2

Thus,

/ ox ~
)'

- sin II

--/'

s in II av
- - - - -.
8r
r

ae

TI>e combination of r and II derivatives on the right of this equation must be applied to V when
it is expressed in te-l'mS of r and 9 tO give the partial de1ivmive with l't:SpeCl tO X when V is
expressed in terms of X and y. To find 32 V jox 2 , we use the schematic IO the left.

av

/ "'-

x' + y'

= cosll -

ax

- y
=

av

il l'

,.

=cos II.

I'

/"'-

(a")

a (a")

a
ar
ao
or ~ ax + ae ax ax

)'

o2V+ ---sin II av
-

cosli - -

ilr 2

sin II o V)
----cosO
,. o1 oil
2

r 2 illl

a'v - -cosoav
+ ( - s .m li -av
or + cosll. oil or r- -1111iJ 2 V
2sm 0cosll iJ 2 V
= cos 2 11- - -

ilr'

2sinlicosli
rl

iJr ill!

a'v ) ( -

sin&
- ---2

,.

o11

sin II)
-J'

sin2 0iJ 2 V

+ ---r' iJII'

av
av
+
----.
ao
r
ar
sin 2 11

where we have assumed that mixed parlial derivmives are equal. A similar calculation gives

il1 V

- -2 =
ily

. ,

il1 V

sm-11 - iJr 2

2sinlicosli il 2 V

arao

cos2 0 il 2 \l

2sin llcosOiJV

+ -- -2 - --.,.--1
2
r

iJII

iJII

cos2 1i

av

iJr

+ - - -.

When these are added together. the result is Laplace's equation in polar coordinates.
(12.24)

Cnlculru ions i 1l this cxrunplc;: a ll ow U!') to emphnsi1..c < po int that we mode in Section 12.3. We
s nid that Jiltho ugh the deri\ativc dy/d x can be considered as a quOlic:lll o f d itTc:rentials. we
never consider a panial derivative as a quotient. To do so in e_,nmplc 12. 19 wou ld lead to errors.
We. calculated arjtlx = cos II. Nmicc that tl.t/ !lr = cos II . and t herefore to regard !lr/8x as
the reciprocal o f ilx (or is incorn.- ct

Homogeneous Functions
Homogeneous functions arist in numerous areas of applied rnathematk-s. A fu nction f (x 1 y, z)
is said to be a p osithely h omogeneous fu n ction of degree It if for eve.ry t > 0 ,
t"/(x. y .~).

f(tx,ty,J z) =

For example, the function f (x, y. z) = x 1 + y2 + z1 is homogeneous of d egree 2; the fu nction (x, y ) = x 3 cos(y/x )
x 2 y .ry2 is homogeneous o f dc&'TCe 3; and J (x , y, ~. /) =
1
1
.r2 + z1 (x y + yr ) is homogeneous
degree 4. Panial derivatives o f homogeneous fu nctions satisfy n1any idcmitics. Tn particu l ar~ their flrsr dcti\ 1alives satisfy F.uler's theorem.

THE ORE M 12 .3

or

(EulersTheorem)

If f(x, y. z) is positively hornogcncou.s o f dcprec 1r. and hns conti nuous first pa11ial

derivatives. then

ill

ill
i)j
+ .r:;- + z:;-

x:;-

/I~

1\

(12.26>

PROOF To verify 12.26, wedifferontime 12.25 with rcspecllO /. holding x , y, and~ constant.
For 1he deri\':Hive of tile len s ide we introduce \'ariables u = t .t , v = I y , and w = t z, a nd use
the schcmHtic to the left. lllC result is

= n.f(x . _y, ~).

o.:

<1)'

Q.\'

!Jf ott
&11 o1

1 \ y 1\

ilf ov
ol

ilf

ow

- - + - - + - - - nf

ov

ilw ot -

,_,/ (xy)
' ''

or

ilf _
n-t
.
::;- + Y of
::;- + z-;;- - 111 / (.1, y, z) .
l1U
uV
oW
When we scu = I, we obtain u = x. v = >
' w = z. and lhc equation above becomes 12.26.
.'If

The results of Exercises 31 - 33 in Section 12 .3 arc spec i11l cases of 12.26.

EXERCISES 12.6
hl Exercises 1- 10 we have defined u gcncrul functional situation and

:-::pccific example. P1nd the ( hain 1u!e: for the tndtcmed derivtuive
ill t.hc ge1lCrbl $ituCtliOI\, ond lhcn usc tll8t result I() calculule 1hc :..umc
derivative in the specific C):an'I:Jlc:.
t1

1. d z{t/1 if < = f(x . 1), x

2.

:~ ), if z =

=,!!(1): z =.u'/ (.r + 1). x = e"

f(x. ,r) . x = g(s. f) . .r = IJ(s. 1): z - x' e> +

y lnx. x = s'cost, y = 4 Scc-

8u )

(1 2

4. d :{t/11 if 4
f (x, y, v), x = g(u), y = h(u), v = k(u):
t = x 2y v3, .r = tt 3 2u, y = In (u 2 + 1). I' = ~~~

s. Jr
au)

II

= f (.<, Y s). ' =

6.

!:).

~(1), y =

h (r ), s = k(r, t);

"

= ./x' + y 2s, .x

+ 2s)

3. as 'i fu=f(x ,y,z). x=g(s, t).y=ll(s, t ).z= k(s, t);

if

if z

= I f(t + 5), y =Sin - (r

+ 5), s = tun ( r l)

= f(x) . x = g(y), y = /t(r,l) ; z = 3'+', x =

y' + 5. y =esc (r 2 + 1)

12.6

?,")

7.

uX

it'u =f(x,y , z), z=g(x , y ); fi = J !Jx ' +y' + z' ,

1,

z = x f .i

ilx)
'

:~),

9.
Z

=r

il' z

+.r , X

I In (s2

+ 12)

= J (x, y. u) , x = g(v) , 11 = ll (x, y ), 11 = k(t) ,


y = p(r); 1: = x' + y 1 + u 1 , x = v' - 3v 1 , u = l /(x 1 - y 1) .
v = e1 y = e:ll
10. dz f tlt if z

l n Exercises 1 1- 15 fi nd the derivative.

C:llcolus 10 solve this; probletn'!

19. If two sides of a triangle have lcng1hs x and y and the angle

between them is 9 , then the area of the. tri~ugle is A = (1 / 2)x y sin 0 .


How rast is the al'ea chi;j11g ing when xi$ I m, y is 2m. and () i:s 1/ 3
radian. if x and y are each i ncreasing at I / 2 m/s and 8 is decreasing
:u 1/ 10 radia_
n per second'/

= f(x , y ) , x = g (r) , )' = h (r ) , r = k(s ,l) ;

= l r + 5, y = 2r - 5. r =

831

-+ J8. The rudj~ und heig h( of u _rightcircular cone an:. 10 and 20 e-m.
respectively. Ir the rmli us is increasing O:tl I <.:m/min ant.l tire heighl is
decreasing m 2 t:mflni1l, how fast is the volunle ch:jng.ingt Do you need
multivari:~b l c

8. a,. if x = j (r. s, r) . ,. = g(.v). s = h (y. z). 1 "" k(y . z);


.
x = s 1r 2t 1 .r = >.-~. s = l / (y 2 + z2) . 1 = 1/.r' + 1/z'

Ch:'1in Rules for P"drlial l)ri\':Ui\es

20. When

:1

rockel rises from the earth's surfac.:e, its mass d cercascs

been usc ftu:l is being consumed nt the rate or 50 kg/s. US<: Newton "s
uni,c~ law of gr~ \li tati o n {sec E.u1U1>lc 7.34 in Section 7. 10) to dcLcrmin;;: how faSt lt!t force Of gr.Wily of LhCcanh OnlhCrOCkC( is Changing
wbcn the rocket is tOO km abo\'c lhc canh's surfucc :md climbing at
2 kntls. Ass:umc that dlC mass of the rocket at this bcigln is 12 X tif
kg.

* 21.

I f z = /(11 , u) , " = g (x . y). u = h(x, y). find the chain rule


forth~ second derivative 8 2 z f dx 2

11.

* '12.

Oetem1inc which of the foUowing fu.nctions are positively homogeneous:


ta) J~r .y) =x' +xy + 3y '

14. d 1 <./dv' if;:= sin (.~y), .r = 3<os v,

.r = 4 sin v

15. 8211 / &x Dy if II= y fJ.r2

16. Suppose that u is a dill'erentiable function of r and r


+ y1 + Show thm

Jx'

+ y 1 + z2 . Z =

X/ y

z'.

(aaxu)' + (aay")' + (aaz")' =('dl',') '.


*

= <.(t),

f (x. )'. 1:)

(d)

(cJ

f (x. J', z) = u ri: - xyz


f(x. ,r. z. r ) = x' + y 4 + z' + r '

(I)

.f(x . y. z, t) = e'+y' (z' + t 1 )

+ X)' -

2xy '

=x' sin (y/ z) + y ' + y /z


1

'*

23.

= / .r' + x.r + .t'' e 'l' (2x'

at

We can write that :d ong C,

I Tan- ( 2 Rr ~n 0)
V(r , e) = -l + 2
If
R1 - r 1

y
X

(a) Obtain Ute chain n tle defining dp / dl in term.< of ~p/Dt


with respect to l.
(b) fu pl&jn tbc phySiCaldiiJcrencc bciWc'Cn dp/dt and ap fat.
Z

- J_v' )

(a) Suppose that the circular plate with radius R in ttle fig_ure
below has its lower edge held at 0 V and its upJXrcdgc held
at 1 V. Show that the clocuos:tatic potcntjal

)'

and derivatives of X . y. and

- xyzr

<: 0.

Suppose l.ht dcnsiLy or lhc g<ts at any poinl i n the n:gion

time 1 is denoled by p (,t . y.z~ l ).


p = p [x (l ), y(i). z(r), r).

(c)

(hJ f (.r .r)

(tigure below)

.< = x (t ) , y = .r(r). <.

f(x, y); x 2y

tg) f (x , y ,z) = co<(.ry) <in(yz)

17. Considcr o gas that is movi.ng through somc rcgion D of space. If


we follow a pmtk.ular panicle.of the gas. it traces out son~~; cur, cd path

C:

(b)

83:2

Chnptt1 1!

OiiY'erentinl CC~I ..:U i lrS of~ l 11 1tiv~rio\ble Filtl.:livn.s

= R) but it

(b) The fu.nction in pan (Ot) is not ddinc.:d ror r

can be shown th;u '" r - R. \alues of V approach I for


0 < (! < ;r. nd l!ll!lroodl 0 for - Jr < () < 0 . Solve the
e,xpressiOJ\ in r)an (a) for r in tcnns
v and 0 . :md use

f )' =
+ (/)
-a_.
("!)'
ilx
) (8avF)' _
(af'
au

.+. 33.

lhc result U.> plot cc:tuipotcnti rtl curve~ rev V - 1/ 8. 1/4,


3/8. 112. 5/8. 3/4, und 7/8. Sci R = I to do this.

V(r. O) = -I (V1

V2)

u2

34. In many problems in clastitity theory. the Airy's stress t'unclion


~ (.t. y) mu~ satisfy the bih;&rnl.cniccquOI.ti on

ilx'

R - r

"" 26. If /(s) i5o di1Tcrt l1liub lc (unction. shmvthat /(X -)') satislk.'S
the t:quation

3y) + S(y- .<) sislics dte cqu>tion

au

au

ax

a~

ax' 8,1

a,\'

Usc Example 12.. 19 to show (h;,u, in IWl~tr <."ounl in:uc::, lhc equation c:.u1
~ cxprc:sscd in 1hc form

1-a + -11 a
> -' a
-2 ) ("'4
- +,.,' -a'<l>)
- =
( -ilra+
2
' ,. a,
r <JIJ
<Jr+
r <Jr
<JO'

" 25. If f(s) ;ond g (s) ore di Ncrcntiable functions. show lhm V j'(x 1 y 2) Vg(.<y) ~ 0.

rr /(.f)i~2 dil'f~nli001cful'l(.'1ion.show th:uu( .\' , )') =

- 0

iJ" <I>
iJ" <I>
;r' <I>
- +2- - +
-=
0.
1
1

sa tis lies 12.24 for,. < R . h rq :tn::scnLS potential in lhc circle in the
fi~:,un: in E:-:crd.sc 23 when pot.cnti:t.l on the upper edge is v, nnd thl.lt
on lhc lower cd~ is v,.

27.

I
-~ -

I 1 - V,)Tan- 1 ( 2Rrsin 9 )
+ -(V
1
1
rr

or

0 ; x - ucosv. ,. = u~n tJ :

+ JS. An obSt"or lt:t;vcls alorl.g the curve ,\'


1:. )'
31 3 + I. z
~~ + S. where .\' . .". and t nrc in metres nnd I ~ 0 i$ in seconds. If lhe
dcnity p or a ~as (in ks/t~>'J is given by p = (Jx 2 + y 2)/(z2 + S).
fi nd the time r1ce (')f chnnse of 1he dcn$ity of the gt'~ n.;; mc:lS:urcd by
the observct when 1 - 2 ~.

/(4x -

= ./.< 2 + y' + z.

J6. If /(r ) is u diffb-cntiablc function and ,.


.$hOWihat

3- + 4- =

s.

Vf

..., 28. If /(.>) ;;uxl g(t) urc twic.-c cJiO'~.!~-cn l iablc . show that the f1.1 nction
11(x. y) = xf(x y ) + )'S(,r + y) sa1isfies

+ 37. If f (.t', ,\')

a'u
a2u a2u
- - 2-- + - =0.
Ox'
il.r Dy
uy'

=0

f'(r)

~fine~ )'

=
dx

I (.r -

y)

+:

38. If l
P(.r , y )

J9.

(.\'" , y)

i.s

= /(.< 1 -

= - ,-(xi + yj +

~k).

a,. a function of .1: . show lt..-Jl

j;,f,! - 2.f..,J,J, + f .,.f,!

1,'

29. 1r I (s) :llld g (r) arc twice dill'crcntiobfe. sliOW ohao


g(x + y) satisllcs

0.

~ hu.nnonic (uncti.o n, show that the fu nction


y 2 2.ry) isalso hunnonic.

(a) Show ohat / (:<. y)

= ln(.r 2 + y2) s.ui,lio' L>pla<'<!'s

"!Ulllion 12.11.

30. Show that if f(v) is diO\:rcnli>l>lc. 1hcn u(.r. ,1')


satisfies

au +v iJll
ax &.r

x-

=.r ' /(.1'/ X)

2u .

(b) Transfor01 J (x, y) inlO polar coordinates Md show th.o


the fwlction satisfies 12.2.4.

40. Fi11d un idcntit)' srnisfted by the socond partial dcri,llli"\'CS or a


function f(.\' .'f. z.) that ill. po:sitivcly homu!icncuus of degree n .

In E"crd~s 31-JJ SUI)POSC that f(.x. y) sati s li ~s the lirst partial di r


fcre1llial <::quatioa. S how U.at wilh thc d l.all.<:: of independent v:uiabks.
run<tioo F(u . v) = j (x(u. v) . y(u. v)) mu>~satl>fy lhc:secO<Ki par
8

;~.*

41. II. is postulated in one of the tht:olics of lrallic tlow that the average

speed u :.H a poi.nt x on a struigiH highway hl.!ong the x -axis) is related


to the concentraUon k of tmtlic by U'c diffcrentitll cquru.ioll

tial differential equation.

au

a')z+(a' )z o;

31. (
ax

( ~:r +
32.

Cly

e:r

ay-

= (x

+ y)/2,

= (.T -

y)/ 2;

Y.11Cre 1 is tin1e8 and c

>

ak

= -c k -

ax.

0 ant.J '' :ue constants.

(a) Usc <:h:J.in rules for fJH/ iJx and au jar in the fu nctional
siou tion u = /(k) and k = g(.r,l) to show tha'

a' f a' t
- , - - , = O;u =
ax

II

au

ax + -al

u-

(x+y)/2,

11

a' F
au av

(x - y)/2; - - = 0

du
2 ,. -&k = 0 .
- ( u-&k + -&k) +ck
dk

ax

at

ax

Ch) The equation o( COOlinuity fO< unfli<

.aJ.

Oow ., ..,. lh>l

T.."tJ equl nn;;St.s tn ureconncacd b) spri~ N\in' equa.l 5prin:


k 1'iO that lhe: lnli.Orot'S arc (rec 10 "'tdc on 2 friclionlo.s: IOOie
l~ figure below). 1l1< "oils A and 8 IR h<d.

C01SU1111

uk
actu>
- + - - =0.
ar
a.
Utrre Lht..-.;;c hbt C'-'Ocqu:nioni to obtain the diffa'C:ntiJ.l cqult
1inn rclu1ins: ~x:cd und concentration:

1\

~ = - ck<,. -t)tl_

f----- '1

dk
I<) Solve tho diRcrential cquution in

hoo~_J off:[]-'
ooo
~8
1-

I"'" lb) rmu

X!

(a) U:;c Newton s~(lCld tuw h> '"huw th&tllhc difYcrunt iul ClJur"'
lions IQr Lhc motions of the "'a"'~'" til\!

/(k).

.u .

A Mad -clkk< lronl rc't a1 the Otig_in on o friction!(,.,, wire in a


Vc11J(UI pltl\t to the poil\l (.'1). y11) undl."t'lhc mftucncc uf &ra,ity Ul&ure
below). ,\~ '' tkx..~ ao. G'r.tvuation..tl poccnlla.l cncray ~ con,cncd into
~11'<11<. CO<li) AI(<. t). d~ bud""'- lc.l pc<.:ntiJI Cl1<.'fJY lfl$,\'
litH ~lr\<ht ~)' i< mtiJ/ lJ.tthiJ point. ~hen mg.\'= m 1 / 2. so
1&1 ~ ./fi). T" uncia <m>ll do<tr\Ce , (;lz)l + (d,\)1 ~<>nt

the """"' .ol (\,y) wolh ld()Coly v taM>~ tin~ ,'(d1)1 + (dy>'f
Jb'loe, lhC Wl.ll tilc'

.. j(dA ) j
I =

(.0

-~

d' .c,fdl'
(bJ The i:lcr-l..*l'<lnf.C ~or\ (rQtn thcorc::tic:al R'MX'hania.
fer this ~y1tc1n &.~'\:

lO IM1Cf1C the Cftlirc tUI'\C IS

!!.. (!.!:.) - !!!::.


a., = o.

+ ldy )i

dl a.<,

,,

1'" J'fiiY1

wtleR: .A1 .and :t:OR lt\cdi\ph)CCit\CRI:o. O(lhc lllJ.S)(:, (rom


d1C..-ir cquilibc'IU.Q\ PQSllJUn .\1 - d :Xa/dt: and j~ =

I 1

(dy)'

dx =

"'

""' 1''j1+()')

",g

iiA .

possible is ctlllcd the bs'fuhuttxhront!" probl;om II iJ IJ'IO\vn in the


calculu.) ~)( vJ.rituiott-. llut )' = f (A ) must :~o(i(Jbl'y th-.: tq\Jt~Lon

(aF)

aF

(c)

Obl~in

the cqu.Jtio

~quation$

1n pun ( a) Iron\ 1hc .. ulct-t:.-v.ani,t.:

in IU\ ( b).

.U. Suppos<:lbll the :cond..,.li.T J".llliJI ~iiTm:ial ~wt ion

= o.

az
3 ;:
~:
p-+q--+ax a.a, a, ,
1

-.here

= ~, . +Y(y')'

f (.v .)
{a)

th>l

(>")'

I ~1'.1 "

&

S :

0.

Ch) Show thot the curve th:ll <atisfoes the equation In pa11 (U) is
Ihe cydc1id dclir.cd plrtlmctrically by

x a(O - sir.O).

)'

a'z

"'>

QX

+ b)'.

lh< clungc ol van>bi<S

= CA +d)'.

v. tK:I'C a. b. c. and d un.: consHml1. Show lh.atlhe puninl diiTcrcnti:ll


ctiUltlioo in ~t unU 1 i-.:

11( 1 - cu>V).

when.: ll i'" u conMant


(c:) Show 1htt it d()C't; not nntcr wh;u poin1 on the ~)"<:loid the
bcJJ >latU fron>. the tunc to get to (.\o. l o) b
the

:o.Jmc.

a: )
= F ( A . .... 3ilz
- .,
, u.t

(I' q. and r are constants! os subj<ctoJ 10

f (.\ ) mw;~ satisfy the di A'crcnll>l <GUOIIun I +

Slll>W

m~

."

The problcRl nl' Ondins Ihe sh:tpc or wii'C thol moku I us ..,.n

d.c a.r - a.v

wt~re L i'" dell ned a~ Lhc "-in-c:tic energy Qflhc l wo


tess the cncrsy ~H,cd in the l'opri ngs. Shuw I hut

az

il

a: a:)

P- + Q- + R-,=0 s.l.z.:;-.-a
as'
asat
81
.s '
"hen.: Q1 - 4P R = (q 1 - 4tn' )(lld - bc)2

.as. s~ lha:t if a ..o luiJnn, =

j(x.y.:) ...r tl\:ttne...:lirncnsit'Wlal

l . ~lpbce eqwrion 12.12 c:an be cxrn:s.sc::d in the form u = -'(r), -.-hcr'e


r
y 1 :'.then /(A.>' t) nu>4 toe o( thdnnn

= /r'

f(x, y.z) = --;=~'=;=~

y=/{x)

Jx' + y' + z'

<x.. )',)

where C and D arc C011$Htllts.

+ D,

Cb:sptet 12 Diffen!.llial Cal.;;u l u~ of Multi\'ruiable Fun<:tiOr)S

834

112.7 Implicit Differentiation


In Section 3.8 we introduced the technique of implicit differentiation in order to obtaio the
derivative of a function y
.f(x) defined implicitly by an equation

F (x. y) = 0.

(1 2.27)

Essential ly. the technique iswilill'crentiateallterrns in the equation with respect to x ,considering

all the while tha t y is a function of x. For example. if y is deli ned implicitly by
x1 y3

+ Jxy

= Jx

+ 2,

implicit differentiation gives

2x/ +

3- 2~y3 - 3y
dy
-=
3x 2 y 2 + 3x
dx

dy
dy
3x 2 y 2 - + 3y + 3x - = 3
dx
dx

With the chain rule we can actually present a formula for d yfdx. Since equation 12.27,
when wriuen in the fonn
F(x , j(x)J = 0,
F

/"--y
I

must be valid for all x in the domain of the function f (x), we can differentiate it with respect
to x . From the schematic diagram to the left, the derivative of the left side of the equation is

.\"

dF
i'JF
i'JFdy
- = - + - -.
dx
ax
Q)' dx

.f

If we equate this 10 the derivative of the right side of the equation. we liod

F.r

dy

+ Fv=
dx

or

dy
dx

Fx
Fy

( 12.28)

For the function detined implicitly above by x 2 y 3 + 3xy- 3x - 2 = 0 , equation 12.28

gives
dy
-=
dx

2xy 3

+ 3y- 3
+ 3x '

3x 2y 2

and this resull is identical to that obtained by implicit differentiation.


S imilarly, if the equation
F (x,y,z) =O
dd i nes
yields

z implicitly as a

function of

( 12.29)

x and y, the schematic d iagram to the left immediately

&F aF az
-ax + oz ax

= 0.

aF + aF az =0.
a.v az ay

From these \Ve obtain the results

i'Jz = _ Fy
ily
F,

(12.30)

We do not suggest that formulas 12.28 and 12.30 be memorized . On the contrary, we obtain
result5 in this section that ioclude 12.28 a nd 12.30 as special cases. To develop these results we
work \Vilh three equations in five variables :

F (x. y, 11. v, w) = 0,

G(x, y, 11.. v, w) = 0,

H (x, y, 11, v, w) = 0.

(12.3 1)

We ;,IS_Wmc lhM m.:se equattOftS ddi-.e u. &.'. aiK! U' as function< of"' and ) lor "orne domain
of \\ti LK') of .1' .t1ld J' (aod do \:0 implicrlly). II nughr e'en be pos_,iblt 10 solve Ih<: s:ys1cm ~md
oblain oplictt definitions or the fun<'tiO~

= f(x. y ).

= /:(.. ). "'

= il(;t, y).

t 1 .u~

WepclbC (he prOOJcniQ(Iindinl: lhe si~ 6r3(ooQrcJtr r.aniaJ dcri\lUho of II c. anJ U' '"itb ~pett
10 x and \", SUJl)O~in~thoc it ij undesirable or even mpos(ible co oblajn the ~pl ic it form o( the
functi,lfl.l To do this, \\'t r)()tc that W\:fC' ~ufl.-: f 2.J2 l:D<Mll 0111d iiu hstituted in.ro 12.31. then

Fl-<. y, J(x. )), g(.<, y). k(.T. y)J = 0,


{;f t ,y,f( t ,y),/l(r.y).h (x,y)J = 0.

ll f< .y.f(x. \').g(x, v).hCx.y))

' to-OWd be identities in x 4ilfld) As re:Jalt \\C' could dJ ITe:rc:tu:i.uc: caeh cquabon "uh R$fXCt 10
.r. obtarnng from the ~hematic d~:ram
4

aF

aFau

aFav

aF<Ju

- a.
- ... a;t+au
av a..- aw a., = 0 .
il G

a;

ac au
J;; OX

't

i},!i. + ~ ~ _

ax

au tl.r

aG au
a; OX

aG aw

DH ilv i

il tlr

OUI a; = O,
IJH ou>

a.;a; = o.

or
F. ou

ilv

ut

i1 t

a+
F,-+F:
~
.r
il.c
"' a.. =
(; ~ + G iJl'
aw
"ax
;;- +G., -

" au + N av

''a;'+

Clx

We Ito.., in I2.J4 lh

~>eq,._~ tht

rule , I

oe cqu.uons in the thrtt un

~J;](<>ns are li~~ear in the

n PolrlK'lllar.

au
ax =
c~,ru.wrs:di

Jw
N ., () r

ICUSie(J ut

c.

G.

H.

H.

AJ)J'enW.x 8

~"""lls <Jut ax. Clut a

F./

G.,
Nv flw

Fv

- n,.

unlcn.,...,s, Mllutioru can be

F.
r~- Gs:. c.
- H,

Fw
c.

H.

a..,;a.\.

aod
Obclllned USing Cnamer

rkF, Ff
G, G,, Gw

H,

I Ic.
=

t. and

F.

N.

H, N.,
F,
C,
N,. N.,.

F~r
c.

(I~ l51

The two detcrmin:uus on the right of 12.35 involve orly derival.iYes of the given functions F ,
C , and H , and we have therefore obtained a method for finding au fil x that avoids sohing
12.31 for u , v. and w. We oould list similar fommlns for the remaining five derivatives. but
firs1 we introduce some simplifying notation.

DEF I NITION 1 2 . 6

The Jacobian determinant of functions F , G, and H with respect to ''ariables


and

iJ (F,C, H )

w is denoted by a

(1/ ,V,U/)

8(u. v. w)

and i; defined as the determinant

F.,

F.

iJ( F , G, H )

u, v,

Fw

c. c.

C,. =

H.

lfw

"

iJ F

iJF

iJ F

au

ilv

iJC

iJG

au

av

iJw
iJG
ow

() H

iJ H

iJH

ilu

av

iJw

tl2.36)

With thi notatio n we can write 12.35 in the form

o( F , G , H )
i)u

iJx

il(x, v, w)
iJ( F , G. H )

( 12.371

il(u . v. w)
The remaining derivmives of v = g(x, y) and
obtained from equations 12.34 by Cromer's rule:

iJv
iJx

w = h(x . )') with respect to .t can also be

IJ(F.G. Tf}
8(1t, x. w)

aw

iJ( F , G, H )

ilx

iJ(u,

iJ( F . G. H )

v, w)

iJ(u. v. x)
il( F, G, H )

112.38)

IJ(u. v. w)

A similar procedure yields

li(F, G, H)

ou
8y

iJ()' , II , UJ)
iJ( F . G, H )

!J(u.u.w)

iJ(F, G, H )

au
()y

il(u, y, w)
iJ(F. G, H ) '
il(u. u. w)

8(F, G. H )

ilw
()y

iJ(II,V,)')

iJ(F, G. H) .

~ 39
( L.

i)(u. u, w)

Rlrmulas 12.37- 12.39 apply only tQ the situati on in which equations 12.31 define 11 , v,
ancJ was function~ of_.. l:lntl y. h b.. however. fairly evident huw to construe~. fonuulas in other
situations. Here are the rules.:

I. The parrial derivative has:-\ Jtlcobian divided by a Jacobian (and do not forget the negati\'e

sign).

2. In the denominator, it is the Jacobian o f the runctions deiining the original equations with
respect Lo Lhe dependem variables.
3. The only d ifference in the Jacobian in the numerator is that the dependem variable that is
being differentiated is replaced by the independent variable with respect to which differen
tiation is being perfonned.

11.7

Implicit Oift'crenti;aliufl

837

The resuhs in equations 12.37-12.39 arc valid provided. of C0 Ur$C.lhal lhc Jacobian

iJ(F. G. H )
o(u . u. Ill)

J.
1""

0.

In actual fac-4 it is this condition that guarantees that equations 12.31 do define u, v . and
functions of X and )' i n the first place.
As a second example. the equations

define x and

.v as luncti011> ors and 1 . To find IJx/IJs w hen s


IJ(F. G)

iJx

as =

3(F. G)
o(x. y)

When s

IF
, I ~ -~
G, G,
= II
IF
,
F>.l
G, G,
x2

2/

II
I.
.+

?y

-4)'l
. - 4 )

= I and I = 0. the equations definin.s x nn<l y reduce

The first ghcs x = I

10

+ y +S =

0.

+ y 2 . which we substitute into the second:


0

= (I + i>2 -

2y

+ y + 5 = y + 6.

= -6. and .t = I + 36 =37. \\r,th the>e values. the panial derivative is


ax
a.r =

-1

174

I EXAMPLE

?y

-11
. 3 - -h
. -

2x2s

r +y 2 - 2x + l= 0,

Thus. y

= I and 1 =0. we first calculate

F,

o(s. y)

w us

- 12 1 = 65 750.
889

12.20

If x 2 y 2z' + zx sin y = S defines: as a funcrion or x and y. fond IJ:/IJx.

SOL1l ll0" If we set F (x. y. z)

= .t 2y 1z5

IJ( F )
1)(x)
=
3( F )

zx sin y - 5

= -

= 0. then

2xy 2z3 + z sin y


3x 2y 1 z2 + .\"sin y

IJ(z)

I EXAMPLE

12.21

The e()uations

x 2l z3 + uuw + 1 = 0,

x 2 + y' + z 2 + ,,:' + u3 + w2 = 6,

define u, u. and W as funcLions o f x .

u = I, v = - I, and w = I.

y.

and

z.

Find

aujaz

u + u + w = x + 2y,

whe n X

= I. y

0,

z.

= 2,

SOLUTIO"

If w e set

IJ(X , )',~.II, V, W) = II+ I! + U>- X - 2)',

then

F,,

iJv

az

r..

F,

3(F, G. 11)

G.

il(u, z, w)
=
il( F , G, H )

H.
Fu

il(11 , u. w)

G.

G,v
H, H.,
Fv F,.
G v G.,

H.

Hv Hw

Instead of expanding these

VIJJ

3u2

G:

dctcrntinan~.

3x2 y 3 z1

II V

2z

2w

uw

IIW
2

3111 3v
I
I

I
LIV

2w
I

and then substituting values for lhc va_riablcs. we

substitute first, and then cxp>Ul d:

au
Or.

- I

- I

0
I

= o.
2

EXERCI SES 12.7

In E:<crci.scs 1-' y is dc:fin:xl implicitly

de tine .t . y . and z ~ t'wlcLioll~ of u aJld v ~ C.nd ax f iJu ).., at the values


l. y
I. 11 = 1f /2. v
0. and z 0.

a func tlma of x. Find

a.~

.r =

dyj d.<'

2. ( x

+ y)2 = 2x

-+ lS. lf ttlccquation F(.t .J . 4) - 0 defines each of x. y. wKt


function of dlC od>er two. sht.lW that

(il.xa")r (ax) (ay)


il;: ' =

In Exercises 5-8 z il> deli ned impl icitly~ a function of -~ and )'. Find

a;.;a.r """ azta.,.


2

5. x sin z - ye;

i)y '

= 2x

7. zsin~ y + ysin2 x

6. x'z'+ yz+3x = 4
8. Ton- I (}'Z) = X l

:; z3

i<

16. lf z :

"'

COi>,Y , whcrc,x tt nd )'

z as 3.

-I .

~rc funCI I OR ~Of I

defined by

In Ex~rci!lc~ 9-13 rind the rcquirod dcri v:uivc. A.s5umc th:u the system

of oqut.~tions docs define lhc fu netion(s) indicated.


~. ilu{ax and ilv/ il.l' if' ' - )'2 + u' ... 2v'
2+ u 2 + ti
10, i)xfill if sin (x

+ 1)- sin (x -

11 . iJ / iiX )y,: il' x

r co~ tP

12. dt/tiA if .1 2 + y' l .l. llufa.\')J{ if X,\'11

xu + v + 4 = 0

1)

= rsin 4> cosO,


z' +

+ vw

2X,l'

- 4.

=t
,1'

= 1.
y

= 1..<2 + r' =

= r ,in.;l 1i110.

t =

x ' + y' - Sy = 4

+u

u v == )\

:rw +

14. Given that dle equations

x2

y oos (u v)

+ z2

= 0,

xy - sinu cosu + z = 0

x2 + y2

sio(uu) +2z2

= 2,

fond d zfdr.
17. Find

O.cotauh

if

=x2 + y

end

and )' nrc:: fu ncliOM (lf

II

and t dcfinc.'tl by

v=x-y.
J8. Fi nd ilz/ily), if z = u 3 v +sin (rw). and 11 and ,, arc [uneli~tlS
of x and )' ddi ncd by

12.8

y = 0 dcftnts z as a runction of .t

19. Gi,ol h:n z_3 - .\' .;: show that

(b) If rl-..; cqu,uious F (11. v,.<, )') = 0. G(11. 11. x. y ) = 0

20. I( the c:Qu;ations x = u 1 - tl. )' = 2.u v. define u and v u.s


(m\<.1ions o( X and)', lind azujCJ.t:.
(<t) Gi,~n t~tthcctluo.ltion t'~x

def1ne U 81ld U tlS funcli011S Of .\ and r 1 and abo define .\


u.nd ) ' as fun<.'tions of u ~md u. show that

+ y.)z:.+ 9x 1 = 2 dcfim.:s z oi.S

u furw:tion of x und )' . and x ns u function or y


ilzjax and iJ.r /iJz rociprl)(als?

;~ud

~ (11 , V)

z. arc

(b) Oi\~utt:uthe equ:uionst'x+yjz +9xl = 2: ,.\' 2 y+~q =


I define t us u func:lion of .'( . urld x n.s n funcl ion oft, ure

clz/tlx >nd dx/tlz rt<ipo'OCols7

a(.r . _,.J =

~'*"

24. Suppose Lhe SY$lcm of m linear ctlutllions in 11 unknowns(n > m )

Oxjart

llS

fwlCtions of u

Ulld

L a;jXj
J-1

v. arc {Ju jilx unci

i = l, ... . m

c;.

reciprocal~'!

.. 22. Givcn thutthccqumtOOsxl -2)' 1.s' 2t -2.H 2 ;; l ,.r 2+ 2y2s 2t +


S.u 2 - I dcllnc s und I as fua><trOllS oL< Wld )'. lind 821/8y 2

dclinc:s x 1 , x2 x,, os funct ions of .\'11,+! x,..+l


lh.Ul if I ~ i ::S m a nd m + I ~ j ::S n. the n

Odcfine u and

=:

0 define S

11Ad I US

Sh()w

v a,;: (unctions or s and t, and lhc C(llJ~UiOlS

H (s, I , :c, y) =: 0 , I($, I , X, y)


functiort'i of X and y. Show lhal

.... .t,1

-axax, =--o,,
D.

M Suppu.c d>ecqua<ion 1:(11 . v. s, 1) = 0. G(u . v. s.t) =

23.

8 (.v.y)

iJ(u , v)

(c:) Ghalthat lhc cqu:uion..; u :- v = 3x


;v. u - 2v~ =
.t - 2y dchnc u and v us functions or x and ,Y, and also

dclinc ,\' and )'

a(u. v)
a(x. y)'

8(11. 11) 8(.<.1)

-----il(s . I) il (x, y)

(3z' - x)l

21.

839

and .)',

3z' + x
't'

O irec i<H'I:tll>t rivum~s

wl'll.;.l'e 0
IO;j l .., ):,ll, and 0 ;1 is lhc sanlC as t.lctcnninant D cxccpl
lh.;Jl i1s ; 111 colu"\rl i~ rCI)Inccrl b)' 1he j'11 columil of )OIJ )..,, )(., .

I 12.8 Directional Derivatives


change uf 2 ftuu:tiuo /(.t. y. :) in
ru1 arhiWI')' dn6on v

Jfllfunc1ion f (.r , y, Z) is dc ~ncd throughout some rcgionol'space, lhcn at any poin1(x0 , y0 , <u)
we can calcolllte its i>tlrtial deriv~tives i)ffi).~ . i)jI i) .1'. and I i) z. T hese ~eriv.ltives defille
nates of chunge o f f (x, y , :) al (xo, )'o, <:o) in d irections para llel to 1he .r-. y -. alld z -axes.
f:Jut whtll i f we wtuH the ra te ofch~1nge of f(x , y , z) at (x 0 )~h .: 0) in so me <lrbitrary directi on
defined by a vccton (Figure 12. I5)? By the rutc of cha11gc of .f(x. y, z) in lhc dircclion v. we
mean the mte of change with respect to distance a.s measured along a line through (.to. y0 , z.o)
i 1l directi on v. Let us dcti nc s as a measure of directed di stance along th is line~ takings = 0
a1 (.r0 . y 0, zo) and posilivc s in lhc direc1ion o f v . Whm we wan1. lhcn. is the dcrivmivc of
f (x . y. z) with respeel 10 s a1 s 0. To express f (x . y. <:) in terms of s . we use panmetric
eq ua tions of the line through (.r0, y0 , zo) a long ' ' If \, ;; (v1 . v>, V::) is a unit vector in the
dirlion of v. th e.~~ paran1c1ric equa1i0r1s for this Iinc (see cttuarions 11.37) arc

ar

y
X

= x0 + vxs,

)'

= y0 + v1 s,

From the schematic d iagram to the le ft,

/ )'1"-:

I I sI
s s

tlf
ds

&f dx

of tly

\\'C

= ~ 0 + v,.t .

( 12.40)

obtuin

of tlz

=-+-+ox tis
oy tis
i)z ds

of

ax

Vx

a.r

&f

+ -oyVv. + -oz v,,

where all partial derivatives of f (x, y , z) are to be evalualcd a1 {.to, .Yo. zo) . We call this a
tlirccrional derivlllh1e. It is given an alternati v-e notatjon i n th e following definition.

DEFINITION 12.6
The directional derhati of a function
the ]>Oint (.to. yo. zo) is

at

D,J = -a v.
X

(.r, y , z) in the direction \ = ( v. , u1 , v) a t

at

at

+ -a}' v,. + -aZ ":

(12..11)

Now v.. , v nnd V.: are tl1e components of lhe unit vector V ir'l t he directio n of "~ and
1
iJff iJx , 8ff oy, and 8f/8z are the components of the gradient of f(.r, y, <;) . We cnn write,

therefore. that

D.[ = \1 I

v.

( 12..12)

CMS<!quently. the derivative (nne of c hange) of a function in '1ny g iven directio n is the scalar
product of the gradient of the function and a unit vector in the required d irection. We stale this
in the following theorem.

THEOREM 12.4
The directional derivative of a function in any direction is che component of the gradient
of the func cion in that direction.

I EXAMPLE

12. 22

.._..
Find D,.f at (4, 0, 16) if .f(x, y, ~) = x 3el'
(-2. I. 4) .

+ xz and,, is the vcctor from

(4, 0, 16) to

SOU.iTION Since
\1 f

t(M, t6)

2
[(3.< 1'1

= 64i +
and

\' =

,.

+ ~)l + X 3<'''1+ .t k ) ](<.O. t6)

64] + 4k

(-6, I, -12)
.,/36

I + 144

-I

m;-;(6. -I. 12).

"181

we have

D.[

=-

(64. 64, 4) . (6, -I , 12)

368
= - .JI 81 '

.JI8I
111c ract that the: Ocri\'alive is negath e rnear1s that f (x. y. l )

...-..

i:> decrea:,ing in d irection v .

T he directional derivativc g ives us insight into some of the propcnies of the gradient vector. In
panicular, we ru1ve the next lheorem.

THEOREM 12.5
The gradient \If o f a function .f(x, y, z ) delines the direction in which the func tion
increases most ra pidly, and the maximum rate of change is IV .fl.

12.8

O m1.h<:1\L ol 11

lun\:tioa poincs in the

~ol in:c1i~o,n i ii

whi<:h

the ftt:neti~olll im:rc:a'\Cl\ I'IH)':I nl(lidly

1J irec;lion:.l l>~n\'at1\e,..:

841

I' ROOF n1e01'Cil1 12.4 SI31CS lhal thc dirc~1ional derivnlive of j (x, y. z) in a direc1ion \'
is the cornponenl of 'V fin thai direc1ion. Figure 12.16, which ~hows componems of 'V fi n
various dircclions. makes i1 clc.1r 1ha1 D, f is grcniCSt when v is pnrallcl lo 'V f . Ahcrnmively.
if 8 is lhc angle~><:!""'"" v and \1 f . 1hcn

D.[ = \1f

v = I'V / llvl cos II

= I'Vfl cosO.

Obviously D,j is a maximum when cos II is a maximum (i.e.. when cos II = I orO= O)nnd
this occurs "hen v is pnrnl lclto \1 f. Fill!llly, when v is parallel to 'il f, we "'''c D,.J I\1 f 1.

and this <.-ompletc!i the pr(H_1f".

Nme that for any function .f'(,r . y, z) .


Df =
1

a.r.
Dx

i)j

DJ
-,
J - -Dy

IJf

Df
=a~
-.
k

In other words, the pnnial deri\'lllives of a func1ion are

iL~

dirccliona l deriva1ives along 1hc

coordinate directions.

I EXAMPLE

12.23

Find lhe dircc1ion al 1he J>Oim (I . 2. -3) in which the function f (x, y, z) = x 2 y
increases most rapidly.

+ xyz

SOLUTION ,\ ccoroing 10 Theorem 12.5, f(x , )', z) increases mos1 rapidly in Ihe direction
'\7 [(1,1,- 3)

= (2xy + y~ . x + x z. xy)l(l ,2.-1) = ( - 2, - 2, 2).


1

You might feel thai be<.-ause 1he deflnilion of 1he direclional derivmive D, .f does 1101 involve a
limit proces..;;. it is some: strang~ new type-of c.Hircrcntiation. To show that this is not the case, let

us re1um 10 thecalculalion of the derivative of f(x , y, z) m (x0 , y0, ~0) in the direction '' shown
in Figure 12. 15. \Vith paramet ric.: cqu.utions 12.40 for the line through (.x0, y0 , zo) itlong v. the
'"lue off (x , )', z) m any poirn (x, z) along 1h is line is .f(xu + v, s . .l'u + v.,s. z., + v,s ).
If we rake 1he difference be1wccn 1his value and j(x 0 y0 , ~ 0 ) and divide by the distances
between (.ro. y0, :o) and (.r. y. z). then the limit of this expre,o;sion ass- o+ should define
the derivative of .f(x. y . z) m (.:r0. y0. zo) in 1he diree1ion v; 1hat is.

.v.

D" r = hm

.f(xo

+ v ,s. Yo T v,s, ~o + v, s ) -

,-o

f(xo. Yo~)
.

( 12 -lJ)

lt can be shown that this limit (ancJ I his is pc.rbnps the form we might hn' c expected the derivative

to la~c) also leatls 10 the n.-.,.ult contained in 12.42 (sec Exercise 35).
Consider a curve C in sp!>ee 1ha1 is defined parametrically by
Rlllc:I.:A"
\:h;ut:t 1..1f . fU-"( ti\lll <Jiull);

!.1 \Uf"\'e

rJ
A

C : x = .r(t ) ,

y = y(r).

z=

~(t) .

a :::;

1 :::;

fJ

(Figure 12 .1 7). Imagine 1hat C is 1he path traced out by some panicle as it moves lhrough space
under the action of some syste m o f forces, and suppose that f (x, y, z ) is a function defined
along C . Pe rhaps the parLicle is a we.alher balloon and f (x, y, z ) is temper.tture at points

along its uajcc1ory C . In such applications we are frequently asked for lhe nne of change of
f(x) y! z) wi1h respect to dis1ance 1raveUed along C. U we use s as a measure of dislance
along C (laking.< = 0 at A). then Ihe required ra1e of change is d f f ds. Since 1he coordina1es
of poini.S (x, y, z) on C can be regarded as functions of s (although it mighl be dillicult to lind
these functions explicitly). the chain rule gives

df
tis

<If dx

ilf dy

<lf d!

=ax-ds- +ay-d-s +az-ds


=

(a[ax. ay.
a[ ar) (tlx
az .

rlz)

dy

tis. tis. ds

tir

'Vf - .
ds

In Section 11 .1 1 we saw that dr/d s is a unit tangent vector i' tO C. Consequently,

df=Vf'l',

tis

B m this equation states dull dj'fds is the directional derivative of f(x, y. ~) a long the tangent
d irection to the curve C . In o ther words, to calculate the rate of c hange of a fu nc tion f(x , y , z)
with respect to distance as measured a long a curve C. we calculate the directional derivative of
f (.\' 0 ) ' 1 z) in the direction of the tangent Vector (0 c .

I EXAMPLE

12.24
Find the rate of change of the function f (x, y, z) = x' y in the d irection of decreasing x at the poim (2. 4. 2).

x z along the curve y = x 2 , z = x

= 1 2, ~ = - r,
= -2. and 1he
ttmgcm vector i~ T "' (- I, -4, -I}. t\ unit tangent vector to C ut (2, 4, 2) in the direction
of decreasing x is therefore

SOLUTION Siuce parallletric equations for the curve are C : x = - t , )'


a tangent vector '" C a t any point i~ T = ( -I , 2r , - I ). At (2, 4, 2). 1

'

(-1, -4, - I }

=
viS

-I
r.;:(l. 4, 1).

3v2

The rate o f c hange of f(x, y. z) in thisdirec!ion is

'

'

f T = (2xv- z.x-. -xllt1 <1)

(1 , 4,1)
"'

-3 v 2

-I

28

3v2

- 3../2'

"'(14, 4, -2) (1, 4, I) =

ljhlll;lf.,I"EJl['W First and


~()nd tlirliorm1 l!eri,atives of a
iun<1100 f(x. )') ot' two v:ui t1ble~>

=j(x,y)

.....-....

~~---------------------------------------

ln preparation for maxima and minima of multivnriablc functions in Section 12.10. we now
d iscuss directional derivatives lor a fu nction f(x , y) o f two independem V'o riable.<. Suc h a
functio n cao be represented graphically as a surface~= f(x. y) (Figure 12.18).
Fo r a dircnion ,. at (x 0, y0) in tlte xy-plane.

DvJ='ilJ
X

,.

V,

where \l.f is evaluated at (xo, .Yo) . Algebra ically, this is the rate of c hange of f(x, y) in
direction ' ' Geome.uically. it is the mte of change of the height z of the surface as we move
along the curve.ofinteI'Section of the surface and a veticaJ plane containing the. vector v, or the
slope of this curve. Each directioo v a1 (x0 y0 ) detlnes an angle a with a line through (x0 y0 )
parallel to the positive x ~a.xi s~ and for this direction
\ = cos a

I+ sin a j.

\ Ve can write.

chen.

Dvf

='If v = ( axf i +

~i!y1 1)

(COSO' l +si n 0'

j)

i)f .
= !If
- cos a + - sma.
ilx
Q)'

(1 2.44)

rr D,.f represents the slope of tho curve o f intersection of the surface and the \Cnical plane
through v. then D,( D,.j') represems the r-otc of change of this slope. Now

D,.(Dv.f) = 'l(D,f)

Thus,

D,(D,.f) =
( 1 2.4~ )

We ca ll D,.(D,.f) the second directional derivative off(.<. y) at (xo, )'o) in direction v. I fi t is


posi1ivc, therl the curve.of i ntcrscction is concave upward. whcre.us if it is negative. the curve js
conca'c downward. \Ve w i ll fi nd these results usef ul in Section 12. 10 when we discut..~ relali\e

extrema ot' t'unctlons oi' two h1dcpcndcnt variable-~.


EX E RCIS E S 12.8

10. f(x. y) = x '+ y at (- 1, 3) along thccurvcy= - 3x 3 intloc


direction of dccn:a5ing .r

In Exercises 1- 8 calculate the dirt.aonal dcriv:uivc of lhe flU1CtiOtl at


the point und in the diroction indH::ucd.
I.

I I. J(.c. y. <)
.<J + z 2 ao (1. 0. -2) alon8 the cur y
I. t = - 2x inthctlircctionofi ncrcusing.r

J(x, _r, z) = l<' - y' + :c:' at ( I, 2, 3) in the direction of tl>e

vc<tor l'ronl ( I. 2, 3) to (3. S, 0)

12. J(.r. y.z)


x 2.r +xy 3z ot (2. - 1. 2) ulong ohccurvcx 2- y 1
3. :: = .\' in the d it'Celi<)n ol'incrcasing x

2. j'(.l', y, z) = x 'y + xz :u (- 1. I, - I) in the direction of the


\'CCior 1hat join...: (3, 2, I) to (3. I, - I)
J. j(X, )') = X~y + J' al (3, 0) in the c.lircclion tll' the \'e ctor fronl
(3, 0) to (-2, -4)

J(x. y) = Tan-

IJ. j'(x,

(.cy) at ( I. 2) along t.hc line y = l< in ollc

dirc<:tion of irw:.~rasing .\'


6. f(x , y) =sin (x + y) ot (2. - 2) along O>e line Jx
in the dit'eCLion of decrca$ng

+ 4y = - 2

7. f(.<. y. <) = x3ysinz at (3. -I, -2) along the line .r = 3 +


y=- I+
z = - 2 + 21 in t.hc direc1ion ofdct.'n:asing .t'
H. J(.r._r. z) = x'y+y'r.+t2.t at (I,- I,O)ai<Jngtlocl inc x+

I,

2_v + 1

9. J(x , y) = 2.<- 3y at (I, I) along the c".rvc y = x 2 in the


dircclion o( increasing X

.ry3 + z at. (I, I, - 3)

= 2xy + ln ( xy) nl (2, 1/2)

15. f (x. y,

z) = 1/./x' + y' + z' at (I, -3. 2)

J(x. y. z) =

17. J(:r. y. z)

+ 22 =2 in the diroo ion of decreasing 2

In Exe-rcises 9-12 find the ro~tc of change of the: function with respect
to distance tnrvcllcd along the cmve,

.r. z) = .r'1yz -

14. j'(x, y)

16.

'*'.

=0. x -

In Excrdscs 13-1& find the dinxtion in which the function increases


nlOSll'apidly at the point. \Vhnt is the rate ol' dumgc. in that tlirt."t.'tiun?

4. f(.r,y , <)= ln(xy+yt+ .r ~) uL( t , t , t) inthcdire<oion l'ron>


(I, I, I) t0\1/lll"d the point (-I, -2, 3)
S.

= x2 -

IS. f(x. )')

- I/

Jx' + y' + z m(1, -

3. 2)

=Tan - (xyz) ut (3. 2. -4)

= .<ye'Y at ( I, I)

19. In what dirocnion is1hc rare of ch!lngc off (x , }', t)


<St m lhCJXlint

20. {n WhlU directiOnS (if any) iS the

f(.r, y)
20?

= .rrt i>tn:!II-

(2, - I , 3)1

= x'y + yl !ll the pOint (I, -

r alt!

Of Change Of lhe funCtiOn

f) equa l tO (a) 0. (b) I. nnd (c)

.. 2 _1. In wh~l directjon:> (if C~ny) i~ Lhe rate of chant!c o( the rwk.1ion
j(x. )' . t) - X)'+ t ullbc poin1 (0. l. -2) cquul t o Cu) 0. (b) 1. and
(c) - 20?

.t.

* 22.

Must there 3hV'JYS be a direction in which the roue of change or a


runc1ion u1 poinl is C(Jualto Ia) 0 and (b) 3"!

t-

.,... 29. What is the mlc of chnngc of a function f (:r, y, z) in a direction


pcrpcndiculo.r 10 'V f?

:.: 30. Ttl.:: ~dte of e-h:tngc or a function /(.r. J) a1 a point (x0 y0 ) in


direction i + 2j is 3 and the rate of change in dirc.ction - 2i- j is - I.
Fin<.! it:i nuc ol' dwngc in direction .2i + 3j.

23. ln the dcriv:u.ion of 12.4l, why wa, i1 ncccs.swv to U:$C a un.il ''C<'t
V to dctcrntinc par.unctric equations foe- the line ihroush (x0 y0 Zo)
along v? In olhcr wor~ why co-uld we no1 usc the components of ,,
itself 1o write ponunet.ric ~u.uions for the line?

* 24.

... ..lt. Rttlc--s of chur~sc of a func.(ion f(x . y .;:) ut u point (xu, y0. to) i.n

dircetions i + j . 2i - k.and i- j + k are 1. 2. and -3, rcs~.,i,ely.


Whw is its partial deri\'tttivc with rcsexx-1 to l at the poiJn'?

How f~l is ttk: distance to tl1c origin changing with ~peeL to


distance tnwellcd along the cunc x
2 cos 1. y = 2 sin 1. l
31

at any poim on the cunr.e? \Vhut is tbe nne of change when 1


Woultl you expect this?

25. Find points o n the curve C : x


the r~te of c~nse o r f

= t. y = I -

(x, y, z) = .r 2

= 0?

+ xyz w ith r..:spcct to di51ancc

26. Repc:u Exercis.e 25 roc lhe Ctlrw c


runclion j(x, y , z) .r'- y' + '

iii

.._ 3~. F'md the SCCQ_n d d irectional dc.rivali\'c o( the functi_oo /(.1., )')

x.)y 1 at the point (I. I) ln the direction ohhc vector (I. - 2).

3.l l:;indtheseeonddirectionaldcri"ativeofthefunclio n f(.r. yl .z) =


,,.z + 2y 2 + 3z 2 at the point ( -2, -I. 3) in the d irection ( I, I, -1).

2t. ~ = t :n which

lr.wcUcd along. the cun"C: vanishes.

28. 1r WC k.now LhC r.1lC Qf d 1angc or a (UI1(;tion / (X, )', t) Ul u point


P on a cun'c C. proceeding in one di_rcction along C. wbut is the. r.uc
of change in the opposi1c diroction along C?

* 34. The palh followed by a ~one embedded in the trc:&d of a lire i<

a cyclnid given par.1mmiclly by x


R(8 - <in6). y
cosO), 0 ~()(sec Example Q.7 in Sc.:1ion 9 .1).

: z = .\'' X = yl and the

= R( l -

(a) I faw fast is the distance from Lhc origin changing with rc
spcct h' d i5tancc. U'avellcd along lhecurvc at the pointS ,...
responding to 0 = ;r /2 and 0 ;r ?

x = (cos/ +
cou)j , wh<!rc 1 is 1irne. Plotlhc path ulled an
(h) I-t ow fast is the )'-coc:vtlinalc changing at lhe~ (mini<.?
imolrnr ofa rin,.'lt!. Show th:l1 the rate of change of the dislar'l(.'t' of chc
(c) How fast is the .t coordinate changing at these points?
plrticlc from the origin. with rcspc~t 10 distance travelled, is :.fways
positive.
1- 35. Verify 1h:u expression 12.43 for D~ f lc:1ds co formula t2.42.
1:1. Tile path of a parlicle is dcnO<.'d J)3Jametrically by
1 <:in 1)i

+ (sin 1 -

112.9 Ta ngent Lines and Tangent Pla nes


Tangent Lines fo Curves
One equal ion in lhc coonlina1c. x. y. and z o f poi11IS in space.

F (x ,)", ~) =

0.

{ 12..16)

usually defines: a surface. (There are exceptions. The equation .t 2 + y'2 + :: 2


point, and xl
y 2 ~ 2 = - I defines nothiog.) When ea ch of the equa1ions

+ +

F(x, y, z)

= o.

G(x , y.

z) = o

()de fines a

(1247)

defines a su.rface.lhen togethetthey define the c ur,eofinlersection oflhe two surfaces {provided.
of course. thlll the surfaces do intersecl). Theoretical ly. we can fi nd parnmetric equations for
the curve by setting x equal to some fimclion of a parameter 1. say x = x (l ). a nd lhc n solving
equations 12.47 for y and~ in terms of r: y = y( r) and z = z(l ). 1be pammctricdefinition.
lherefore. Jakes Ihe fo rm

= x(l) .

= y(t ).

<:

= z(f).

a ::; I ::; /).

mASl

where a and/) specify the e ndpo ints o f the curve. Practical d i01cult ies arise in choosing x(l)
and solving for y(l) and z(l). For some exanwles, it might be more convenie nt to specify y(l)
and solve for x(t) and z(t) or, alternatively, Lo specify Z(l) and solve for x(t ) and y(l). We
consi dered examples of such conversions in Section 11.10.

l 2.? T.mgent l..incs JVh.l 1'tlng1:ll Pl:'tne$

845

In Se~tion II. II we ittdicated that when a curve C is defined parametrically by I2.4S, a


tangent vector l O C at any poin1 P is

lillctlj;l::l
line 10 a

ETT:JII

~urvc

T~l n,enl

in :)pace:.

dx. dv. d~ .
dr
( 1~.4 9)
- = -i + -- j + - k
dl
de
dt
dt
(Figure 12.19). lltc tangent line to C at P is defined as the line through !' having direction
d r/ dt. If (x0 , y0 , zo) arc the coordinates of!' and 10 is the value of 1 yielding P , then the
vector equation for the langent line at P is

+ u drl
-d
1 l~o

(x. y. z) = (xo. Yo. zo)

Tangent line
to C:ll P

112.50al

(sec equation 11.36). Parametric e-quations for the tangent line arc

x = xo + x' Uo)u,

)' = y 0 + y' (/o)ll,

~ = ~~~

+ ~~ (10)u,

( 12.50bl

and in the case where none of x'(/0 ) , y' (l0 ), and :'(10 ) vanishes, we can also write symmetric
cquatioots for the tangc.nt line:

x - xo

x'(lo)

I EXAMPLE

)" - )'o
y'(to)

: - zo

If 2.50c)

z'(lo)

12.26

Find equations for the tangent line to the elliptic helix

C : .< = 2cosl . .' ' =-!si n/ .

z=

21/Ir

at P (../2,2../2, 1/2).
SOLUTION

Since t

drl
'
(t/

t=tr/ .4

= 1t /4 at P . a tangent vectorto C at P is
= (- 2sini,4COSI,2/ iT}j,.,vf

= <-

J2. 2J2. 2/rr).

Synunetric equatjons for the umgent Jine arc therefore

..fi
-..fi

X -

J - 2../2

~ - 1/ 2

2/Jr
2../2
We have shown tlte tangent line to the helix in Figure 12.20.
UCiJII;Ij5"f'JJFJ!J

Tan~enl

line to :m elliptic

)'

beli~

I E XAMPLE 12. 26

Find equations for the IJ\ngeru Iine to the


P ( l/2, 3/4. 3/4).
~0 1

UTION

= 1-

' 1 ~

+ )' ..- z

= 2 at the pont

Para111ctric C<llllll iu n~ for the curve arc


,r

ince I

CUI'\O

r:: I.

= I /2 at p , a liAJent vector 10 the

,d.I

CUI'\t c'l

p is

(1. -I+ 21, -21),, ,,

(1 , 0. -1).

I , ..<1/1

Berouse 1he y-componcnc vanishe,., \\ e cannot '' rite full symmetric equations for the mngcn1
eq u ati on~ involving x and' Ahcrnutivcly.
flQinunctric equation" for the cnngctu li ne arc

Iitle, ahhough we coultl write pun i;ll symmeLric

The line is shown in Figure 12.21.

Tangent Planes to Surfaces


We now consider the problem of finding ~he equation for the ~ougcru plane at a j)l)im P un a
s1111'ace S (Figure 12.22). We dcfi ne t he tangent plane a! th,u plllr'IC which comains all tnngcnt
lines n1 P to cul'\'e~ i n S thrnush f' (j)f'Q\ided, of cour.e, that <uch n plane exi,ls). Suppo"'
thatthe;urlucc is ucfine<l by the cquuliun
P(x . y. <.)

= 0.

and that

C : X = X(/),

)'

J(l),

:(1),

is any curve in S through P . Since C is in S. the equation


P(x (t) , y (t) , z(t) l

=0

~ I ~ {J

is V"alid for 1111 1 in a ~ 1 ~ fJ . If F (.r, y, <) has L'Qntinuous first patill derivatives, and .r(l).
y(t). and t (l ) arc all ditl'crcntiable. we may differentiate this equation using the chain rule:

iJFdx

aFdy

oFdz

- +ily-dt- - az
- -dr
llx d1

=0.

1l1is equation, which holds at aii i)Oint> on C, and in particul"' at P, car>be expre.<.<ed vec10rially

as

O = (i)F, oF, IJF).

a., ay

a~

(dx, dy. dz ) = 'i!F. dr.


dr

dr

dr

dr

But if the sca ltlr product of rwo vector.-:; vani~heJ>, the vectors are perpendicular (see equa1ion

11.25). Consequently, '\1 F is perpendicular to the tangent ' "-"'tor <h/dt to C at P . Since C
is an arbitrary curve in S. it follows that 'i/ F at P is perpendicular to the tMgentlinc to every
curve C in S m P . In o1her words, \1 F at P must be pc.rpendicular to the tangen1 plane to S

Kil3" IJ

t.i

poinl /' oo
\'C<'h>D Ill

11.

EJ

Thnsent piJ11e ~ .,

'>urfm:e S C-<llllllilb aU lllnJC:Ill


10 I!Uf\'CS

fun(tion dd ininJJi "

)Ucfa!l't:

the lall~tlll j))!llnc: l\)

in S

is pcrpemli<ulu

tu

me ,li'(Ol,(.'(

,1'

at P (Figure 12.23). If the coordinates of P arc (,,-0, Yo, zo). then the equation of the tangent
plane to S at P is

0 = 'i! FIP (x - .to.)' - Yo. z - :o)

= F" (.ro . )'o. zcr)(x - .ro) F1 (xo. )'o. :o)(}' - Yo)

+ F, (xo, Yo. zo)(:

zo)

( 12.52)

(sec. equation I 1.34).

I EXAMP LE

12.27

........
Find the equation of the 111ngcnt plane to the sur1l>cc xy: 3 + y;:;2

SOLUTION

= 4 at the point (I, 2.

1).

A Vtttor pcrpcndiculur to the uu1gcnt plane is

llut then the vector (I , I, 5) must also be IJCrpendicular to the tungcnt plane. a111J the cquati\)11
of tbe plane is therefore

0 = (I, I, 5) (.< - I, y - 2, Z - I) = .<

+ }' + Sz -

8.

We have >hown in chis ~-ction chac if che equotion F(.1, )', :) = 0 dclit'oel> a $urfllec S. and
if chere i< a c.>~~gc:m plane > S ut a pomc P. then cllc ve<tor V F p rs nonn.ll co the tangent
plane (Figure 11.23). tc i' <u>tum,try to $Uttc in dtis 6icuaciO<t chac V Ftp nurnldlto the surface
itself ..\t P . mthcr lh.u1 to the L.ult;cnt plane to the ~urfat-c. Thi:ro. fact proves to be anDlher of the
important pl"UU)Crtieb of the grac.lie11t vector. anc.l is wonh Mating ,,sa theorem.

TH EO R EM 1 2 . 8

= 0 defines a s urface S. and F(.r. y. : ) hns coouinuous first


tmtial dcri1ativcs. thcnuc any poinc on S !he veccnr V F i> pcrpcndkulur tu S.

If du: cqunuon F(.t. y. :)

A geometric applicHcion of chis fact is contained in the followi ng example.

I EXAMPLE 1 2 . 28

.,.....
Find equation> for the tangene line at the point (I, 2. 2) to the o.:un-e C .t 2 + y 1 - z2 = 9.
4(x2 + yl)

= s:2.

SOl I '110N EqtkUion I2.49 indkBtes tlla.t to find a w.ngcnt I'CCtor tu C . we should fiN~ have
parameuic equations for C . These can be obtained b) fiN ;oh ing each oqutuion for .r 2 + y 1
and equating the result:

9This equmion implies thnt ::

x1 + y1 =

= 2,

z' =

5z 2/4.

the posicive resuh being required here. On C. then.

s. lltld rxu~ullccric equations for C arc


x

= ./Scosl,

y = ./Ssint,

= 2.

0 ~ I < 2n.

According to 12.4'.1. a tangent vector co C at ( I. 2, 2) is

dy . ....::
d)

dr
- .( dt d t

dt

(- Jssint. Jscost.O) ,.

tl.ll)

5 .,.

1~/,1)

( - 2. I. 0).

The tangent line therdorc h.~s equations

x -1

y-2

-2

- - = - -, ;: =

2, or~

+ 2y = S,

2.

The face d>ac gradiencs can be used to fi nd nonu als to surfaces suggests an ahcmativc
solution. It i clcM from Figure 12.2~ that if we define F(x , y, :) = x 2 + )" 2 + : 2 - 9. then

VF

F= x2 + >.2 + :l- 90

V F "''alua~od ut (I. 2. 2) i, perpendicular not only to the surf:.cc F(x. y. z) = 0. but aho
to the curve C. Similarly. if G(.~ . y. z) = 4(x 2 + yl) - S:'. then '\/(; ut ( 1, 2. 2) is :tho
(>crpendicul:lJ' lO C. S inCe a \CCtur ~\Ions the l::tngcm tine to Cat (I, 2, 2) j, pci'J>endicular to
bolh of these \<ClOts. il foUows that a \<Ctor along the tangem line i'

(V F x VG)ru.l.lt

= 1(2x. 2y. 2:) x (8x. Sy. -10:)1 11 u 11

(2, 4, 4) X (8, 16, -20)

8 I

-s

i j

= 8(-18. 9.0)
"'72(-2. 1, 0).
Once ag:.~i n . we h:wc obtained ( -2. 1. 0) a110 a 1angcrH vector to the curve. ~md equations for the
rangent line can be written down

a.." before.

Example 12.28 illuMr'dtes that when a cun e is d!.!lined as the intersection of l\VO surface'
F(x. y. z) = 0. G(x, y. :) = 0 (Figure 12.25), thcn u vcetonangent to the c urve is

T = 'VF

c '~ \

VG.

Thus to find a tangent \ectOt to a CUr\e we usc 12.49 when tbe CUr\e i> deli ned parametrically.
\Vhen the CUI'\e is defined as the i ntersection of two surfaces. we c.un either find par'Jlllet ric

equations and us~ 12 .49. or use 12.53. Note, too, that in order to find wngent lines
is not necessary lo have a direction nssigned to the curves.

c100 rn\llk

,>(

araditlliS of

"'uwrf~t:t<

Unit
tiOI'IrW \tcfllf' /() I <llffJCt

Tangent
pl.1nc
V 'xVG

'

,,s

if

850

C~

11 l)dYcu::nlial Cl l ~ulu) of ~ t ulliv!l.ri.ib'IC: 1:;-urK.tions

Mji~Jil;l

'LVlFA

I! I lUI? \Fi :0

A ~phcre

A p.uaboloid

a!IJO!\!Ji[j(P:i4JT
cylindet i' 11

h " .smooth S\ltfliCe

n smuolh surflk."'e

The ..urf-:acc o( me

pio.~~isc-.smovth ~u.rl'acc

111e surface bounding the cylindrical volume in Figure 12.29 is not smOoth: a unit nonnal
that varies continuous!)' over the surface cannot be assigned 81 pointS on the circles . 2 + y 2 = I,
z = I. This surface can, howe,er, be divided into a fi nite number of subsurfaces, each of
which is smooth. In panicular. we choose the three subsurfaces S1 : z = I, x 2 + / ~ I:

S2 : :=-l,.t 2 +y1 ~1 : S3 : .r 2 +y1 = 1, - 1 <:< I.Suchasurfaccissaidtobca


piecew~-sruooth

surface.

EXERCISES 12.9

In E.terciscs 1-W lind equations for the tangem line to the cun'e at the

In Exen:iscs 21-26 find an equation for tit<! tangent plane 10 the surface

point

&the point

I. J

=x z =0 at (-2, 4,0)
2

21.

2. x= t , }'= 11 , (;=1 3 at(l,1~1)

22. x

z+x - yat (I,I . O)

26.

6. x =2 - r2,y=3 1 21,<. = 1 t ( I,S, I)

= hos r, y ~ Join I, l

8. .t2 .13

9.

II.

5 at ( /i, - 3/ ./i. 5)

+ xy = 68 at (1,4)

X+)'+ 4 = 4, X -

10. x =

~-~ cos 1. )'

,l' =
~-~

x' + y' + 2y =

1 at (1 . 0. 3)

27. Show that the CUl"e x = 2(rl + 2)/3. y = 21 1. <. = 3r - 2


inlcrsccls the su-rfDCc x 1 + 2yl + 3zl 15 ol right ~nslcs 111tho !>Oint

(2. 2, !).

2 at (0, -2, 6)
sin t ,

z = 1 al (J , 0. 0)

= t 1 + l, )' = 21 - 4, z =

12. .r' +z' = 6.

I. - 2)

23.
(2.-1.-l)
24. x+y+z = 4at(l.l.2)
25 . .t = pin (iT ; / 2) at (-I. - I. 1)

4. J =.t ',z= xat ( - 2.4.-2)

7. x

= .' - y'z at (2, -

x 1y+>'z+z2 x+3=0at

3. .r = cosr.y = sinr. z = cosrat( I.O. I)


5 . .t 1 = y .

z = ,/x' + y' ll1 ( I . 1. ,/2)

+ 3 at (2. -

6. 2)

x+z = lat(z. -./5.-1)

+ y' + <' = 4, <' = x' + y' at (I. I. -J'i)


X= I. y <= I . Z = .Ji+/i at (-1. I .Ji7)
,r = I + cost , y = 2- sin I ,~= ./4+i at (2, 2, 2)

28. Vcrifythal l tM:curvc:.t 2 -y 2 +Z 2 = 1. xy+xz =2is ltmi}Cnl


to the. surf'"" .rn - .r'- 6y + 6 = 0 m the point (1. 1. 1).

29. Show that the cqWltion of the lanCllt plane to a surfaceS : z =


f(.t. y) at a point (.<o. Yo ~o) on Scan be wnucn in the fom\
t - ~ -

(x - xo) f, (.to. )'o) + (y - Yol/1 (xo. Yo).

13. x'
14 .

15.

16. x

= z2 + z'. y- z-

z' ot ( 12. -

14, 2)

+ 3)'' - 2)' + 5. t = 0 at (7, I . 0)


+ + 2y = 3, z =X+ I at (0, I. I)
X= t', )'=I, Z = ..fl+?' at ( I, 1, ./i)

17. X= y'

18.

19.

2x 2

y2

20. x = 1 sin/, y = tcost, z = 21 at (0, 2ll', 4rr)

In E:u:rcis.cs ~32 find 1hc indtc-a1cd dc.ri,'ati\'C for the funotion.


30. f(.r,y,:) = 2x 1 + y'z' m (3.1.0) " 1th respect to distance
alon the curve x + )' + l = 4. x - )' + ~ = 2 in the diroctioo of
inf."fcasing x
3 1. f{.t. y. z) = .rn +A.\'+ Xl + Jl at (I. -2. 5) papcndioulm
10 lhe surface

= .r 1 + ,.~

32 . .((x, y, z) = x' + y 2 - z' at (3. 4, 5) with rcspecllo distMcc


along lhc cun.rc x 2 + y 2 - ~ 2 = 0, 2x 2 + 2y 2 - z2 = 25 in the
dircclion o[ dccn::.asing X

12.10

33. IfF(." . .v);;;::; Odc.:linc.:oat;ur'c implititly in thc;.;y-pl:mc. prove


thm ot uny poin1 on the curv.: V

.t -Xo

...,a""U""'.--:G~.r7<1)--

... 35. Find all points ol'l the slri<icc z = ~r'!/ 4 - yl/9 tn which the
lanscnt plane i!-0 parulld t o Lht pJo:ulC X + y + z = 4.

!\la;\inmand

$51

Min1m~

cquulion!> for the Langc nt linc Lu C a1 P can be written in the fonn

F i s J>crpcndi<:ulnt to the cu1'\ 'C..

.. .~4. flind the cquc!lion of II'IC l~nJCnt phanc 1.0 the ellipsoid x 2ja1
y 1{ b1 + z 1{ c? = L at any point (XQ. Yo zu) on the ~ur(acc .

Rd~1ivc

act.y. t)

y-yC)

"

t -

a(F. G, N)
a(x.t. t )

tt)

"""'a"(li'
'. ""G,...:,1;71)..---

a(x.y.t)

1'

provided th:u none of the Jac.obi-.ms ''anishe.s.

f ind all points on the surface ~: ;;;::; J(.t'l' + y:) nl whic.:h the tw1gcn1
plane is parall~.l to the pi a~ x - )' + 2t = 3.

"' 38. Find all pointl; on the p:uabolnid l :c~ + y1 - I lrt whic h the
lt<lnllilllothc surfilCC coinc-ides with the t i n~~ joining the origin to Lhe
IXJint.

37. Suppose t hn ~ thccqu01t.ions F (A'. y. z. 1) = 0 . G(A. )'. 1., 1) = 0.


H (x. y, <, 1)
0 implicitly de line p;;u'amctric equations for a (.'Urve
C ( I hci ng 11\c: par.lm etct). If P (.rC~. _v0 .c;'>) is a point on C , show tlll:lt

,.. .., 39. Sho w lhatlhc ~um of lhc ink:rx:ept'l on t.hc X. )', and zu e~ of
the tungcOI pl:anc to t,hc stufacc ./X + ./Y + ./Z= /0 at i.l fly l)()int

* 36.

is

a.

I 12.10 Relative Maxima and M inima


\Ve now study rcl ali\'C extrema ol' functions of rnore chan O\C indcperKk:nt variabl e. M ost of the
Lli scussion will be (.'On fined to func tions f(.l'. y) o f two inclcpendem varinbles because we can
discuss the concepL"i geome tricully as well a~ a lgebraically. Unfonunately, not all results are
easily extended to func tions or more than 1wo indepcndcru variables, and we wlllthcrcfore be
careful to point out these limitations.
lJeforc beginning the discussion. we h1~ efly review maxintn- minima results for functions
j (x) of one variable. We do this because maxima- minima theory for muiLivariable functions i s
essentially the same as tha t for singlc.variablc functions. ln fact. every defi nition that we make
ml<.l C\cry resutc thnl we Uiscuss in this section hns its coulllCI"J>.ln in s.inglc variable theory.
Hence. tl S)'t\Opsis of singlc.vtuiablc resul ts is in order. Unfo l'tunatel)\ t>rovi ng re~uhs in the
multi variable case is considerably mo1"C complietlted than in the singJe. vcniable c ase. but if we
can keep central itle;;ts foremost i n our minds and <..'0 1\Stamly Jll<\ke comparisons wi th single
variable calculus, we will find that discussions are not nearly Mdillicult a.~ they might otherwise
be.
Critical points o f" function f(,v) are poi nts'" which f'(x) is either equal to zero or does
noc exist. Georne~rically, this means poinLS m which the gnaph of /Cx) has a horizontal tangent
lint , a verticnlcangcnt lirlc, or no u1ngcm line at nll. Critic.a l poims for continuous funct ions
can yield relative max.ima. relative ntinhna. horiro,ltal I)QirHs of inflcctiOl, vertical l)t)ints of
inflect ion. or jus t cornc.rs. There nrc two t~1s to detennine. whcthe t a <.:li tical point Xo yields a
n.:huivc naaximum or a rcl;ni\'e minimum. Tile firsHJc rivativc test states that if / ' (x) chan~cs
from a positive quuntity to a ~alive q ua n lity us x increases thro ugh Xo, then Xo gives a relative
maximum; if f'(x ) changes rrom negative to posithc. the11 a rclathc minimum is obtuinod.
Thesecond~derivative te.~t indiou es the na ture of a critical poi nt at which f' (.ro) = 0 whenever
f "(xo) ;f:. 0. If J "(.ro) > 0 . then a relative minimum is obtuinc:d. and if f "(.ro) < 0, a
relati ve max inHifll is found.
\Vc beg.i n our study of extrema theory for mu ltivariable fhlCtiolS by deli 1li1lg C1~ tical poi11ts
for functions oft wo i n<.lcpcndcnt vuriabks.

D E FINITI O N 1 2.7
A poitu (x0 , y 0 ) in the domai11 of a funct ion j(x , y) is said to be a c r itical point of
j(x, y) if

ofl
Qy

=0
(X(I . )\\)

o r if one (o r both) of these partial derivatives does not exist at (xo, Yo) .

( 12.54)

852

Ch:lpter 1'2

Oifftrtl'llial C(lculns ofMnhiv:~~riable Functions

There are two ways to imerpret critical points of f(x, y) geometrically. In Sec1ion 12.3.
we imerpre1ed off ox a1 (xo. Yo) as the slope of the tangemline 10 1he curve of imersection of
the surface z = f (x. y) and Ihe plane y = YO and of j8y as the slope or the 1angcn1 line 10
Ute curve of imetseclion wiUt x = xo . II follows, then, 1hat (xo, Yo) is critical if bolh cunes
have. horizontaJ umgcnLJjnc:s or if either curve ha.s a ''cJtical tangeot Jinc or no tangent Unc at
all. Alternatively, recall I haL1he equation of tJte taogent plalle to the surface z = J (x, y) at
(xo, Yo) is

z - Zo = fx(.ro. )'o)(x - xo)

+ J~.(.ro, YoHY -

Yo)

(see Exercise 2\1 in Section 12.9). lfbolh partial derivatives vanish.lhen the 1angent plane is
horizontal wi1h equa1ion z = z0 . For example. a1each of lhecritical poinls in Figure.< 12.3(}12.33, ilffilx = il.flay = 0 and the 1<1ngen1 pl~ne is hori'l.Oillal. The remaining functions in
Figures 12.34-12.38 have crilical points at which either ofliJx or iJf ji)y or both do not exist.
In Figures 12.34-12.37, 1hc surfaces do not have langcnt planes at critical poin1s, and i11 Figure
12.38, the tangent plane is vertical at each critical point. Consequently, (x0 , y0 ) is a critical
point ofa function J(x , y) if at (xo,)'o) the surface z = f(x, y) bas" horizontal tangent
plaoc, a vertical tangent plane, or oo tangent plaoc at aJJ.

1..08

liiiL~

horiu.mtal

ill

critical

T:angent plane

point

(0, 0)

DEI!IariF50U

h...-iwntal

a~

Tangent plane

critical point (0. J)

UO..cl!la0.41 ~ 'tangent plane hori.


z.oniAI tU critical poir11 (0. 0)

z = f(.<, y)
= l - .~2_1)' - 1) 2

:x

Critical poim Y
at whicb

Criticttl poim
a1 which

kf, O

/, =f, = 0

Mjl?lii;J4 lf.JQJI
hori.zontal

T<!lll:Cfll

plc:nc

critical points (X . 0)

z =j():, y)
= vl

lhi#Jil;lii

No lilnj!.cn( plane

at <.'ritical point ( I. OJ

Kill!l!l I]
tri1i<11J point

Elt:W

No tan~c:nt plane at

tx ..'()

<. f(x, y)

: ,J ;x - I)l + .vl

z =/(X.)')

I
points
ai wl1kh

j , and.f;

Criticil

do not

points

exist

al which

/,=f,=O

( I. 0)

"""

Critical point !II whkh


fx andJ;. do not exist

lx-yl

12.10 RcH:uiveMaxirua:-tocl M.lninKa

FIGU E 12.36

No 1angen1

t =f(x,y)
= lxl +

lii!I#JII;I!Uf*fM

pJ<tucs at critjctal poitlls

Venical tangent
(., J)

lyl

z = f (x, Y! = (y - 1)113

----.2:_

_,.~s---x

liUCJII;I::JI

No tangent plane

at crilical poiot (0, 0)

pln.nc at critica1 point (0, 0 )

853

,t'

Critical points
al whi,.h

f.. and./; do

poims

Critical point

which j~ =0
but/ 1 does

ttl

at whk'h

.fx o.odf.., do not exist

not exist

nol exist

I EXAMPLE 12.29
Find all critical points fo r the func tion

f(x . y) = x 2 y - 2x/ + 3xy


SOLUTION

+ 4.

For critical points, we first solve

iJf

0 = :;- = 2xy - 2yl


Q .\.

ilf

0 = - y = x- - 4.tv
.
0

+ 3y

+ Jx

= y (2x - 2y

= x(x - 4y

+ 3),

+ 3).

To satisfy these two equations sim ultaneously, there are four possibilities:

1. X = 0, y = 0, which gives the critical poi nt (0, 0);


2. y = 0, .r - 4y + 3 = 0, which gives the critical point ( - 3, 0) ;
3. x = 0, 2.r - 2 y + 3
4. 2x- 2y + 3 = 0, x

= 0 , which gives the critical point (0, 3/2) ;


- 4y + 3 = 0 , wltich gives the critica l point (- 1, 1/2).

Sincc[)f/o.r and ofJI)y are defined fo r "ll A' and y, thesearctlteonlycritical points. The plot
o f the sur face z = .f(x, y) fo r -I 0 ~ ' ~ 10, - I0 ~ y ~ I 0 in Figure 12.39 does not
really illustrate the critical points. Tn other words. computer plots of funcrions of two variables
arc not as helpful in dctc.rm.Uting critica.l points of functions as were. plots of function.s of one
variable in Chapter 4.

Critical p<)ints for fu nctions of more llum two ifl(lependem va,iable.s<:an be def"ined algebr'"ictltl)'.
bUI because we have no geometric rcprc.s cllation for such functions. tl1crc is no gcomct1ic
itHCI'JHCitttio n ror their (.TiCi<:al poinK For CXI, Illple, if /(X. )'. ~. 1) is tl ru iiCiiOn ofinc.lcptndenl
vari~~bles X. y . l . und f. then (.to, )'o, Z4), to) is. a critical point ol' .f()( , y . z. 1) if all four uf its
firsc~order plutinl dcrivati,:cs vanish 1U (xu. y0 , ~o. t0).

( 12.551
or if a t lcust 0 1\Cof the: partial d erivatives d oes; e'IQt cxi.o,.1 tlt the J>Oint. Note tha t hecnuse the p<u'ti\:\1
d erivatives of <1 func1ion are the COfll p011CillS of the @n\(.lient of 1hc fuucl ion. we C<\ll say that n
critical po irll o f a function is a point at which its grudie1\l i i either equal to ?..ero or undetl1led.

I EXAMP LE

1 2.30

Find all critical points for the func tiOil

SO L.t rrl(}~

r:or c riticniJ)()inl$. we con~ idcr the cqumion:,


IJf

o = a,, =

n,c partial derivatives nre clearly undefl ned for x

= y= t =

0, and therefore the origin

(0 , 0. O) is a critical po im. If .r. y. and : arc not all zero. theo\ the terms in paremheses cannot
v!lni~h.nnd

we must set

)'(. = 0,

XZ

= 0,

X)'

= 0.

If any two o f x. y. and :: vani, h. but the third docs not. then these equations are s.tisfled. In
other words, every point on the x -axis, every pOint o n the y-axis, an~ every pOint on the t-axis

is critical.

\Ve now t.urn our auemion lO 1.he classifica1ion of cri1ical poi nts of a funcLio n .f(x, y) of two
independent va1iables. Critical points (0, I} in Figure 12.31 and (0, 0) in Figure 12.37 yield

' 'hig)!" points on the surfaces. We d escribe this property in the following dellnition.

12 10 ReL"t i~ \1 a~ima )rtd Minim:a

855

DEFINITI O N 12.8

A functio n f(x, y) is SHid t<J have a relat h c maxiruunt j(xo. Yo) at a pc:>int (.to, Yo)
if there exists a c ircle in the x y-plane centred at (xo. Jo) such that for all points (.r, y)
inside this circle
(1 2 .56t
.f(x, y) !': f(xo. Yol-

The ''low .. pc:>intson the surfaces at (0, 0) in Figure 12 .30. ( I, 0) in Figure 12.34.and (0, 0)
in Figure 12.36 are relalive mininlil according 10 lhe.lhllowing.

DEFINITION 12 . 9

A function f(x, y) is said to have a relAtive minimum j(x0, .1'<1) at a point (.To, y0)
if there exists a circle in the xy-plane centred at (x0 , y0 ) such that for all points (x, y)
inside lhiscirelc
( 12 .57 t
f(x, y) :::: /(.ro. Yo)-

Every criLical poinl in Flgure 12.35 yields a relative mjnirnum of j(x t

x)

=0.

DEFINITION 12 . 10

If 11 criticnl poim of a function f (x, y) at which i).fj i)x = ilffi)y = 0 yields neither a
relative mnximum nor a relative minimum. it is said to yield a saddle point.

The critical point (0. 0) in Figure 12.32 therefore gives a saddle point. as docs each of the
critical points in Figure 12.33. Sa<klle poim< for ~urface' z = f(x , y) uredearly the analogue'
of horizonwt poims of inftectioo> for curves y = J(.r) . ln boch ctt-scs the dcrivmive(s) of the
runc1ion ''Clll ishes but there is neither a re l ativ-~ maximum nor a relative minimum.
The critical points in Figure 12.36 (except (0, 0 ) ] are the counterparts of corners for the
graph o f a function .f(~). They ;lre points at which one or both o f the partial deri1atives of
/(x, )') do not exist, but like corner.; for I (.t ) , they do not necessarily )ield relative extrema.
CriLi<.:al poinlS in Figure 12.38 arc Lhe analugwcs or vertical points of jnllcction t(x a function
/(.~) .

Oul' tliscussion has rnacJc it d ear lluu:


(a) At a relative maximum or mininmm of .f(x. )') , e ither ilffil x and il.f/ Dy lxlth
vall ish, or on~ or both of I he J)artial derivatives do not exis1.
(b) Saddle pc:>ints may al<;O occur where i).(jax = of!ay = 0. and poinL~ where Ute
derivatives do not cxi.St may fail to yield relati'-e e:<trerna.
In other words, every relative extremum of I (..r. y) occurs at a critical poi nt, but critical poinL'

do not always give relative cxucma.


Given the problem of determining all rela.ti ve maxima and minima of a functiOil .f(x, y ) ,
we should fJt"St find its critical pc:>ints. But how do we decide whether these critical points yield
relative nwxirua. re14itivc minimu. s~1Ud lc poi nt.s. or none of these? \ Vc do not ha\'C a practical
test thai is equ ivalent to the firstwdcri\'ative test eor func1ions of one variable. but we tlo have a tc~t
Lhm corresponds to lhe second-derivaLive test. Por funcLions of two independent variables the
situation is more compljcat.ed. however. since there are lhree secood~order paniaJ derivat i.ves.
but the idea of t.he test is essentiaiJy the same. l t det.ermjnes whether cenain curves are concave
upward or concave downward at the critical point. The complete result is contained in the
following theorem.

856

0 1:tprer 12

Differenl inl C::dcuhts vr Mttltivarilbk: Function$

THEOREM 12.7

Suppose (x0 , y0 ) is a "riLical point of J (x . y) at which IJ.ffox and 8f/1J y both vanish.
Suppose further that f x. [ ,. fu . }~y. anti .1~1 "rc all ctmtinuous at (.to, Yo) - Deline

If:
(i)
(ii)
(iii)
(iv)

8 1 - A C < 0 and A < 0, then f (x , y ) has a relmive maximum at (xo. Yo);


8 1 - A C < 0 ao1d A > 0. then .f(x , y ) has a rela~ive minimum <ll (.r0 , y0 );
8 2 - A C > 0. then f (x. y ) h>lS a saddle point at (.ro . Yo);
8 1 - A C = 0, no conclusion can be made.

PROOF

(i) Suppose we intersect the surli1~-e ~ = f (x, y ) with a plane parallel to the z-axis,
through the point (.t o, Yo. 0), and omoking an aJogle " with the line through ( x 0 J'o. 0)
parallel to the positive .r-axis(Figure 12.40). The slope o f the curve of intersection of
these surfaces at the point (x 0 , y0 , f (x0 y0 ) ) is given by the direcLional derivative

(see equation 12.44). S ince (.r0 y0 ) is a critica.l point a l wh.ich 'V.f =

0, it follows

that

for nil tl'.


In Figure 12.40 we have illus tmted the cri tical poi nt as a relative maximum.
\ VC verify that this is indeed the case? If we can show that each and
every curve of intersection of the surfooe with a venical plane through (x0 y0, 0) is
concave downward a1 (xo, y0 , 0). then (xo. y0 ) must give a rclHtivc maximum. But
But how do

to discuss concavity of a curve, we require the second derivative -

in this

ca..~e.

the

second direc-tional derivative of .f(x . y) . Acctording t<) equation 12.45, the second
directional derivative-of f (x, y) ;ll (x0 , 0 ) in the direction = (cos a . s in a ) is

a'1
a'J-1
--1
~0> < + 2
--.
ax <-!>
8xay
2

= A .:os2 "
MijiCIII;I#fW-IoU

ox,,y.)

OOSC<sin et

f
+ a'
1J y2

--1

+ 28 cos" si n et + C siol a.

Critical point yielding a r('lntivc. m:t.'4imum for the function

(.1\l.Ju)

2
sin a

where we under.<! and !hat here D,(D, /) is implici1ly s ut11xed by (.~0 y 0 ). ln order.
therefore,, to v~rify that (xo. Yo) gives a relative ma'ICimurn. it issul'ficient to show that
l>v(Dv / ) is ucgarivc ror each value or a in the interval 0 !! & <: 2%. However.
lh''Ctnt<;;c D,.( D,J) i~ unchanged if a is replaced by fX + Jr, it is sufficient to veri f)'
1ha1 D,(D,f) is ncga1ivefor 0 ::; a <: TT .
For any of these values or a except n /2. we ca11 write

D,. (D,f)
and if we set u =

= oos2 a(A + 28 10na + C 1an1 a),

UHl a ,

D, (D,f) = cos2 a(A + 2Ru + Cu 1 ).


II is evidcn11ha1

Q(u )

D,(D,.I }

<:

0 for a ll a

= A + 2Bu + Cu 2

<:

f. TT /2 if and only if
for -oo

<:

<:

oo .

Were we 10 drctw a graph of the quadmtic Q(u ). we would sec th~ll it cro~ses the
11-axis where

u =

-28 -/48 - 4AC


-BJB' - AC
=
2C
c

Bm because 82 - AC < 0. !here >Ire no real solulions oflhis equn~ion. a~d lhcrcforc
Q(u) never crosses 1he u -axis. Since Q(O) = A < 0, i1 fo llow lhnt Q(u) < 0
fQr all u . We have shown. then. the:1t

D,(D,f} < 0

for all a ;<-. tr /2.

When ll' = lr/2. D,.( Dvfl =C. Since 8 2 - AC < 0 and A < 0, i1 follows 1ha1
C < 0 also. ConscquenOy, if 8 1 - AC < 0 and A < O.lhen D,(Dv f l < 0 for

nil a. and (xo. )'o) yields a rtlativc muximum.


(ii) If 8 2 - A C < 0 and A > 0. a s imilur argume111lellds 101he conclusion tmt (x~. Yo)
yield:; a relative ntinimum ; the only di fference is that inequ.alidc::; nrc rcvcr~cd.

n - A C > 0. then Q(u ) has real dis1inct zeros. in which e<lSe Q(u ) is some
times negative and sometimes positive. This means then the curve of imersection is

( i ii) If

sometimes concnvc Ul>wan.J ant.l sometimes coHC8\'Cd ownward ~ nnd Ihe poinc (.ro 1 )'o)
therefore gives a saddle poinc.

(iv) If 8 2 - AC = 0. the classilic.otion of lhe point delennined by (x0 , y0 ) d epends on


wh ich of A, 8, and C vanish, if m1y.

To illustrate that we can obtain a relative maximum, a relative minimum, or a saddle point
for a critical po inl at which 8 2 - A c = 0, consider the three func tions I (x, y) = - y2 ,
I (x, y) = y 2 , and I (x, y) = y 3 in Figures 12.4 I - 12.43. The poinl (0, 0) is a critical poinl
for e ach function. aod atlh is poim 8 1 - AC = 0. Ye1 (0. 0) yields a rela1ive maxiJnum for
I (x, y) = - y 1 , a relative minimum for .f (x, y) = y 1 , a nd a saddle point for f (x , y) = y 3
In fact, every point on thex -axi.s is a relative maximum for f (x 1 y) = - y 2 ,a rei aLive minimum
for 1 (x, y ) = y2, and a saddle poim for .f (x , y ) = y3 .

C'riticlll

FIGURI!: 12.41

CriliCll

FIGURI! 12 42

points yie!d nH:ui\e 1'1\:lXirn:)

C'ritic:al

FIGURI! 12 43
p~ntG

poinrs yi eiCI rei :UWI} mil1ima

yekl saeld.le p:>inrs

'

= ~3

.(

/ EXAMPLE 12.31

Find a11d c lassify cril ical poims for e.ach of the followi ng fllllctiolls as yieldiHg relative
m;llxirml, rdarive mini rna, $addle point$, or 1tt)ne oflhe.~:

= 4xy f(x , y) = x' y 3

(a) f(x. y)
(h)

x-' - y'

SOLt:TION
(a) Cri[ical points of .f(X, y) are g.ivcll hy

Jf
0 = - = 4y -

ax

4x 3 ,

Solmions ofdtese equations are (0, 0), (I, I ), and ( - I , - I ) . We now calculate

\Ve could tabulare resulls 10 determioe the nmureoflhe c rilica) poi1HS.

ljUiildl4f&(W Diogrllm
sign <.'If fuucc:ion f(x, y) = .t" )'"

w show

Critical poi nt

8 2 - AC

Nature

(0, 0)

16

Saddle point

(I , I)

- 12

-1 2

- 128

Relali\'e maximmn

(- 1. - I)

- 12

- 12

- 128

Relative maxinrum

(b) For critical points we solve.

af =3x' v.
2
0= -.
~y

Every point on the x andy-axes iscritical. and al cac.h o f these points j(x, y) = 0 .
T he second-<lerivative test fails 10 c lassify these critical points. Figure 12.44 s hows a
value of zero fo r the fuoct i.o n on Lheaxes and Lhesigo of f(x , y) i.n Lhe [ourquadranlS.
It implies !hat lhe points (0, y) fo r y > 0 yield relaLive minima; (0, y) for y < 0
yield relative maxima; and (x, 0) yield saddle points.

12 10

Rel.-lhc i\!.1\IM:l ~WI \1inim:6

151

I EXAMPLE 12.32
1t .,j, ~r.h!htforwA.rtl
w \nif) that (0. 0)., i,. lhc onl\ critical poim of 1h~ function /(A t ..\) =
,
..
r - 6.ty- + ) and lhatthcquanllly 8- - AC of Theorem 12.7 is cqudlto Leroatthiscritie.tl
point. Show graphicall)
~lgcbfllicall~ ,.,.., tho: critical puim gi' a tldle point.
~

t<l

SOLLllo:- The \':&lueofthe function at (0, 0) is /(0. 0) ~ 0. Ther< oso r<loti,e minimumot
(0, 0) if/(<, ).) ~ 0 in ,o,occirdc aroond (0, G); chcre i>" rcloti'c noa<inouou iff (' .)' ) :;: 0
in some s uch circle: nod there is a saddle point if f (x. )') takes on nc~ative and positive values
in e\'~f) cin::te ccnlred 111 (0, 0) . Tl11: plot irl Figure l2.45 inlli..:.ue,.thm the la'l ,itu,uiol'\ l)rt.\oail~.
To show chis algcbruieally, we ftrst note chat values of f (x , )')arc i>OOiti,e a long lhe Xais
a1ld 1hc yaxi~ away fmm theflrigin. On the par-abola x = yl. value:~ of chc functiOil are

(y1) '

6(.)' ) /

+ y'

~ - 4y 1 ~ o.

Tbus. (0. 0) yodds 11 Sllddlc point.

Thi" compleie' our di,c u,,iun nf rchuhc extrema or function' or lwo independent \-.triable:' .
Qur ne<l ~Cjl >hould be 10 C\ICnd the theory to function\ of more thdll l\\0 \llriablcs. ft is a

simple mauer to gl'c: detinitiofb of relati'e tna.'<ima and minima for such functions: tbc:y are
olmost idenocal to Dcllnitions 12.8 and 12.9 (see E.'ercise II)). On !he other hand. to de,elap
a theorem for fun ctions of more than 1\\0 independent variables that i< analogous to Tbeorem
127 i' bc)ontlche >COIX: n f thi< book. We refer the ime,...ted re.oder Ill noon: ad,anced bol1ks.

EXI!RCISI!S 12 .10

In txcrcKCs l- 14 hnd nil Crtlic:ll pmnlt for 1he tunchon m\d c l ~'"lly

ca..:b u yickling 11 relative maximum. a rcltui"-c ll\1nimum. u s.Lddlc:


poim. or none or1htsc..
1. f (x. J )

= .r'- 2.ry + 2y1 -

6y

2. j(x, y ) 3.r y - xl - yl

J. /(A.I ) - .> 1 -Jx + ,l 2 .t. 2_1'


4. f(x . y)

r ' y' + 3.r

5. /(X, ,1' ) = A)' - A l

6. /(x , y) x,iny

+ )'1

7. f(x , y)

= xyu-<"2+>.:,

= .r' - 2Xy + y'


/(.t, _v) = (A l +) l ) l/l

ll. /(.t, y)

9.

10. j(x. y) = xyl


II . f(.r, y )

= 2xy1 + l.ry +.r' J'

ll. j(x. y) = ).t) + y 2

IJ. /(X,y) = ( I - .r)( l - )')(A+)' - I)

14. f(x. y) =.r'+y' - x 2 -y1 +1

In Exerci!SC~ IS-18 lind ;~II critic;.1l poin1~ ror 1hc fi J"lCiiQtt,

15, {(.~:, y, Z) :a .rl


16. f (.x. y . t .t) =
17. f (;r .r. L)

"4

+yl -

zJ

+ .),, -

2y

fXti nt in I) m whtch ciltu::r /, 1 or / ,y dC~es nut v.'l.ni~h.

2 1. Find aRd c:l;~<>sify the citic:al rxM:'II-5 for the funeaion J'(:r, v) =
~- b~ + V .
.

+5

x ' y 2 z ! + 12x l + 3x

.xyz

~ 22. Pint!. an<.I cla!;~fy the criticul poi1:ts cd' f (.~ , \') = -"" I :\( ... , + 'tr
u.s yicktinr, rcl oti, c n uxim;,, n.:l:..tivc minimu. ()ll,'~aW.!Ic (k)l ftl -j.'

+ .,.2.v:: - >'

"' 71 '!'he cqua uo-n 2.t '

19. G ive d<:tiniticwtJ< ftr a relali, c m aximum ~ttl a rel:alivc: minunu nl


(,~r :t rutk.:lion / (.r. y . l ) al u IJ'Qim ( Ko. _l'o, .to).

-;- 3yl

+ z1

12A')'

+ .&..f~

= 35 dcfuu.:s

lun.:l!Dn z = /(:., y ) . S how thathc: point x ~ 1 axt y = 2 i a


'-'l'itl('al point l'or t he functi(lo with va lueS M ( I. 2). I)I)C$ il >icht a
relutn.'C CMn:nlUm fur the fun ction?

(a) Showth..uthe(unctiM.f(x, y.t)-x~ + yl+zl - ,n z

24.

h u:o< a cn tic~tl pnint (0 , 0, 0) . \Vh11t are the.: Olhtr crilk:nl

poiat!>?

,.2

20. Suppose that /(.~: , ,\') ig hwmorridn the rtgion D : -'' ' +
< 1.
Show lhat /(.r . y ) cannOt h3\'C:I r'l:ln li\C maAimum or minim u;n {II any

( b) Vs<: th-e dc.finition in E:.:cn:i!c 19 10 show thul


hns o rclotivc minim u m '" (0. 0 , 0) .

f (x . ). z)

112.11 Absolute M axima and M inima


Ab..'mlut~ maxi ma and minima an: more importam than relalivc maxiota and mini1'1't.<1 when it
comes lO apJ>Iic~tions. ln this section and it l S~'tion I 2. 12 we discuss the theory of absolute
cx1rcma an~ considet' a number of applications. OtlCC uguin we ~gin with fut\C::1ion:; f (.\' . y)
of lWO indcpcndem \'atiablcs and hare our d~S<..ussion on 1he theory of absolute extrema tOr
f unctic>M of o ne vari<lble .

\ Vc f.carned in Section 4.7 thUl a f'm x:ttion /(.11) thut is corllinuou~ un tl finite int~;rval
x ~ b must ha,c an absolulc mll.x.imum and arl arn::olulc minin-mm on that inter\'al.
Furrhcrmo.-e. rhese absohuc cx-.rcma muse occur a1 e ither criti cal 1>0i1HS or at the ends x = a
and .x = b of lhe imcrvul. Consequently, It) tinc.l the absolme extteiiHl of a l'u nc t iot\ f(x), we

:$

cva luutc f(x) at all criti<ul poi11ts. at x :: a . tt1x.l :u x = b: the larg~t or thc~c numbers is the
ilbsolutc maximum o f/(;() on

a ;: x : b.

~\nc:J th..: snmllcst is the nhsolute miHimum.

The proccOurc is much the snmc tOr n function

(x. y ) thsu is contil'luous on a region R

thal is finite an(j includes all the poi nts 01\ its buuntlary. First. however. wec.leline exactly what
we mean by absolute c.x trcmn of f (.t. y) and COJl~idt.~T a rt \l mbc r of simple cxtunples. We will
then be ahle to m ake ger1-era l statements abGtu the nature ot'all ab~ohuc extl"ml\ , and l>rocccd
(0

the i tnJ){lrtam area ofapplicmions.

DEFINITI ON 12. 11

T he absolute m~inoum of a functio n j(x . y) o n ~~~ion R is j (xo. )'o) if (xu. )'o) is


in Rand

f( .r .y) S J(,lo.)'o)
for al l (x, y) in
in R and

R. The absolute minimum of j'(x. y) on R is f(,r n, )'I>) i f (xo. )'o) is


j(.r, y) :::. f(xo. Yo)

( 12 WI

lor all (,r, y) in R.

In Figures 12.46-12.51, we have shown six func tions defined on Lhec.irclc R : x 2 +y2 :S J.
"!be absolute maxima and minima of these funct ions for this region are shown in Table 12.2.

1\li!I(!IUie n1tt'< imunt

lfiGUAI! 12.47

Ab.wlu:-c. ma:drn unl

= z. Aboolufc. mil!.in)ul'l\ = 0

WOjUIIIi
mtam

Ff1"l"'W

1\~IUtC: nl<1(i

! . Al)'l~llute l'l'liAimum = 0

'

(-1. 0. 2 )

>'

FIGURE 12.48

Ahsol uiC ma-<i mlun

l . Ab.Qitl f(' mminwm = 1

Miitnll.t,mi"Ji'W

M#ltc-111 tptWjM

Al15.oll.lt: ma.'\imlno

Alull'l -

= I. Abwhue nlitlnll U.Al 0

../2, /\b$1)!utc:

A~lllllc

n1inu n um

lna.,:.i

= 0

&fJI!MfEW

f'unccion

P~ilion or

Value of
ahMll utc

/(1,\ )

ab.-.olule moaxirnum

x2+ y2
2

1+ x

Jcx-

- y1

1)2

+ .v'

4 -2r 2 - y2

Value. of
Po~i1ion

Ill<.\ X i Ill U Rl

ol
ab,ulutc minimum

n~ ini mum

E,et)' point on
.\'2 + )'2 = I

(0. 0)

(+ 1, 0)

(0. I)

(- 1.0)

(1.0)

0
0

(0, 0)

(1 , 0)

abwlutc

Every poim on
I - (x 2

+ y 2) 11:

(0. 0)

x' + y 2 -

Every point on

lx - Yl

(1/ J2. +I /

J2).

J2

)' = x.
1/ J'i <X < ljJ'i

862

ChApter 11 Differential CakuiU> ofMulti\'3Jioble ~01.,1iuns


For e:'ch of the functions in these ligures.. absvlw~ ~.l.lrenw occur 111 either a criricalpoim or
11 ~1111 0~1 rlrc bo.mulul)' of R. Thi~ result is true for an~ cu111iluw11s .frmctiou dtfirred vu a
jiuu~ re~:w11 1/rm .mcludes (11/r/w puiuts ou iu bo11uc/ary'. Although this result may seem fairly
ob\'tous geometncally, to prove it analytically is \'ety difficult; we \\ill be content to assume it~
\'alidity and carry on from there.

Absolute
e'\trema ot' a fu'M:\lun rot\\\nuoo,

an a region R and iu.

~xbty

Sometimtl> a drawing or plot of the su rfaccdcfincd b~ a functiun make. absolute maxima und
minima clear. This may not alway~ be the case. however. and we therefore tum our anent ion to
determining absolute extrema algebraically. Suppose, then. that a continuous function f(x , y )
is given and we arc required 10 lind its alx;olutc extrema on a finite region R (which includes
its boundary points). The PfC' ious discussion indicated that the e..wcma must occur either at
critical points or on the boundary of R. Consequently, we should first detennine all critical
points of f(x, y) in R . ande\'aluatc f(x. y) at each ofthesc poi nts. These values should now
be compare.! to the tna.Jlimum and minimum values of f(.t, y) on the boundary of R. But how
do we find the maximum and minimum value> of {(x , y) on the boundary1 If the boundary
of R is denoted by C (Figure 12.52). and it' C ha~ parametric equations .t = x{l), y = y(r J,
a ~ I ~ (L then on C we can expres. f (x. )') in terms of I. and r alone:
f[.t{l), ) ' (1)),

5 I 5

It

/J.

To find the maximum and minimum values off (x, y) on C is now an absolute ex1rcma problem
for a function of one variable. The function f{.t (l}, y(r) l should therefore be c\'aluatcd at each
ofils critical points and uti =It and 1 f1. A plot of fl.t(l},) (I)J could be valuable here.
If the boundary of R consists of a number of curves (Figure 12.53). then thos boundary
pruce.!ure mu~t be pcrfonned for each pllrt. In other \\O<ib, on e.~ch ~~ or t~ ~ndary ."e
express .f (.t, y) as a function of one variable, and ~he~ evalu.ate th.is funcuon at oLertUc:al potnts
an.! at the ends of that pa.n of the boundary tO whoch ot apphe>.
The absolute maximum of 1 (.t . y) on R is then the lnrgcstof all 'alucsof l(x. y) cva.luatc.!
at the critical points in.ide R, the critic~! poinL< on the boundary of R. and the endpoonts of
each part of the boundary. The absolute minimum off (x. y) on R tS the smalleSt of all these

'S<l<tndUty
o(

a rqton may C'(nuist of mure

than one:

C'UI"\-e.
)

values.

--+-:\:::=;:::::::~~---;x
~

I EXAMPLE

12.33

f( )

a < , < b

00
Recall that to fino.! the absolute extrema or a fuoeloou
X. conunoous
. : .b -,~
we C\'llluate l (x) atull critical points nml at the IJouno.lury po111ts .t = "and:' .-. . .
proceo.lure that "c ha\'e establi~hed here for l(x, y) .b much the :me - : :~~~~;:~ ~~

that for f<x )') the boundarv cool>i~ts not or two poont>, but of enure cwv
~
of

Ida . th~fore reo.luces to one or more exuent.a problems or uncuon>


l<x.y)ontheboUI ry
I( ) { l(x)l itisnotnecessarytodcterminethcnature
one "ariable Note too that for x ) or

hese .
of the critic;I points: it is necessnry only to evaluOte I (.t y) at t
poont<
~

, _I((

Find the maximum \ alue or the luncuon - -

.. )

~.r)

:c' -

2)

on therq;tOO

-2 < X < 2 , - 2 ::':: )' ::':: 2 .


.
.
. .
12 54 su c.ts thai the maximum value occu~
SOLUTION ,, plllt of the functo?n tn f'ioun:
hUc<li!CS of the square. We confirm this
. . h li I and thlfd quodnlniS. or on t e at a critical P'>'"''"' ....d 1 e Cnueal
. . nnonts
o t' f(r )') are 11iven b''
with the procedure oothne a )0\
,~

o =31
ax

4 . - 4rl

= >

0 =

a.ra,. =
.

4x - 4y.

I) and the \'a lues of f(x )') nt


.
(0 O) ( 1 I) and ( - I
Solutions of these equauonl\ arc . .
these critical point!\ :lre

17.1

.f(O.O) = ~

.f( I, I) = l!J
rj1 .f(- 1, -t) = [D

We denote the four parts of the bound:uy of

R b C C C and c. (f'igwe 12.55).


y 1 J .

12.11

I jjllli!IJS
-4-X )' -X"' -

m'l
2):

Ql)

A~llll e~13\:imaandMrnim:l

Nlilllll~
J!l:l
Regioo
and 1lS four bCtmd in~ cuYes

Complller plol of
X ~ 2. -2 < J $ 2

-2 $

,.

863

cl : y : 2

I
2
y

2
2

c": .(. - 2
X

Cz :x= 2
-1

c 1 :y = - 2

On C 1

y = -2. in which case

z=-Sx - x ' - 8,

- 2Sx S2.

For critical points of this function. we solve

0
The onl y solution is

dz

= dx

= -8- 4.r' .

x = -2~> . at whjch the value of z is


~ = g . 2 10 - 2''3

8=

l-0.44 ~

On C2 x = 2. in which case

= 8y- 16 - 2 i ,

- 2 :::. )' :::. 2.

Critical point< are defined by

0
T he only sol u1ion y

d~

=- =
dy

8 - 4)'.

= 2 defines one of the CQrncrs of I he square. and at this point

z = 13].
On

C, , y =

2 and

z = Sx - x - 8,

- 2 :;: x :;: 2.

For critical points. we solve

o=
i\tthc single point

dz = 8- 4.,3.
dx

x = 2 1/3,

z = 8 2' n

- 2"'13 - 8 =

l- 0.44 1.

On the llnal curve C.1 , x = - 2 and

z = - Sy -

16 - 2y2 ,

-2 :::. y :::. 2.

Critical JX>illls ;:~ regiven Oy

dz

0 =dy

The soluLion y

-8 -4y.

-2 define,:: another corner of ahe square a l w hich


l

8].

We have now evaluated J (x. y ) at all critical points inside R and at all critical points on the
fo ur pans ofthc boundary of R. It remains only to evaluate .f(x. y ) at the corners of the :;quare.
Two corners have already been aceoumed for: the other two give

[(2. - 2) =

l-40 ~

/(- 2. 2) =

l-40 '

The largest value of f (x , y) is the largc.:;t of ~1c numbers in the boxc:;. namely. I. and this is
therefore the maximum value of f(x , y) on R.

I EXAMPLE

12.34
Tcmpcnuure in degree.~ Cc l:du:-. at each point (x. y) incasemicircular plate defined by x 2

I. y ~

+ }12 ~

0 is given by
16x 2

T (x. y ) =

24xy

+ 40y 2 .

Fi nd the hon c;t and coldest points in the plate.


SOLUTION

For critical points ofT (x, y} . we solve

ar

0 = = 32.r - 24y,
11 X

DT

0 = - = - 24x
lly

+ SOy.

'The only solution of these equations, (0, 0}. is on the boundary. On the upper edge of the plate
(f igure 12.56), we set x = co~ I , y = ~i n t . () 5 1 5 rr , in which ciise

T =

16 cos /-24cos r sin/

+ 40sin 2 1.

0 =s t =s

1r.

A plot of th.is :Ji.mcli.on j n Figure 12.57 sbows !lhe absolme maximum at the reJative maxj murn

and the absoltue minimum ac the relati\e minintu.nL To locate them we. solve for crirical poincs
of this function:

0=

dT
dl

= - 32 cos 1 sin t

24(- sin2 t

+ oos2 1) + 80 sin I cos 1

= 24(in21 - cos 21).


LDI!'IIJ'iJ

""'tlll

Ma.:-tinu.un o~nd n1inimlllll

1emperJtu1-e" IJil a son ici~u lar domain

B!IJ

Plot of temperature on Scmicirtll

lar part of boundary

45 T
4()

35
30
25
- I

I X

20

1.5

2.5

If we: divide by cos 2 t (since C()!o;.2t


have

== 0 does not lead to a solmiot of this equati011). we


=

tcut 2l

1.

111c only solutions o f lhis cqutuion in the intcr,val 0 :$ 1 ~ Jr are 1 = rr / 8 and 1 = Sn /8.
\Vhcn t
nf8 , T IJ:LQ]:ond w hen t
5n / 8. T
145.0 At thec:nd.~t>f this pari o f the
bot'"dary. 1
0 ond 1
n. and 1' (1. 0)
and T( -I . 0) =
O n d>e lower edge
of the pb1tc. y = 0. in which ca-.e

= QB

T = 16x',

I.

-I:;, .r :;,

QB.

I.

@].

The only <ritical pOint of this function is x = 0, at which T =


The hottest point in the
pltuo isoher'CI'ore (cos(Srr /8) . sin (511'/8)) = (-0. 38, 0 .92). w here the temperature is 45.0' C.
and the coldest poi nl is (0, 0) with temperature 0 C.

...

I EXAMPLE 12.36

Find the poim o rt lhe firs1 OCt:lnl pan oftht plane 6..r +3y + 4z = 6closest to thepoinr (4, 6, 7).
Assume thut lines ofimcrscc.t.ion of the plane with thcc()Ofdi tate phlnes are part of the s urface.
~

Minim11n1 distlll'll.'e fnun (4. 6:. 7) kl p1"~ 6.\

+ .ly + 4:: =

6 in

fi"'' QI:Ulnl

:
(4, 6 . 7)

The distance D from (4, (>, 7) to any poi nt (x , y, z) in space is dcfl ncd by

SOLUll ON

l4!trl 1l.l

iiTW

We must min imizc D. but consider only llOincs (.t. y, ~) that smi<fy the cquatiorl of the plane
nr1d lie in 1hc firs t OCI!llll (Figure 12.5S). AI 1hc morncn1. D2 is 11 function ol' throe variitblcs
x. y. und z. btll thc.y are not all independent because of lhc planar restriction . If we sohc the
equation o f lhc plane for z i n lerms of x andy . and subsl itute.

1'1i111gula.

,tonuin for , t i')litl Ce fu K'fioorl


)'

o - = f(x' y) = (x - 4 ) 2
?

= (x - 4) 2

+ (y

- 6)2

+ (y -

6}

(6- 6r
- 3}
)
.4
. - 7
6x
(

+ 3v
; + 22 )

andy are i.ndependent variables. Now D is mjnimized w hen D2 is mjnimi zed, and
we therefore find the point (x, y) that minimi:res D 2 . The values of x and y that yield points
w here
X

on the first octant part of the plane are those in. the triangle of l, igure 12.59.

For critical point:; or D.1 we solve

8f

o = -.

= 2(.r - 4)

ax

il(

= - =

2( ,v - 6)

ay

+ =c6x + 3y + 22),
4

+ - (6.< + 3y + 22).
s

The solution or these equations is ( - 140/6 1. 174/61). an unacceptable J>Oint since it <Joe.os not
lie ill the triang le of Figure 12.59. T he point Ol the t riunglc in Figure 12.58 closest to (J, 6, 7)
therefore lies ulong one of' the edges of the tri.,ngle. We cun find it by minimizing f (.r. y )
along the edges of the tri:\llgle in Figucc 12.59. Otl C 1. x = 0 in which case

D'

F (y)

= f(O, y) = 16 + (y -

6)

22) '
+ ( 3y +
4

, 0 :S y :S 2 .

For critical points we solve


0

= F'(y) = 2(y- 6) + 83 (3-y + 22)

111e value of F(y) hen: is F(6/5)


later. On C2 y = 0 in which case

6
y = -.

=Iso 1. We will evoluatc D 2 at the corners of the triangle

D' :G(x) :f(x.O) = (x-4) 2 +36+ ( J.r +


2

II)'

.O s x

:5 I.

For cricieal points we solve


0 = G' (x) = 2(x - 4)
an tll>ucteptablc value. On CJ. 2.r

+ ~(3.t +

+y =

II)

2. in which cusc

= H (x) = (x- 4) 2 + (2- 2t- 6)2 + ( 6x + 6 ~ Gx + 22


=

(X -

4) 2

+ 4(.r + 2) 2 + 49,

0 :S

~ I.

For crilical poirus. we solve


0 = H ' (x)

= 2(x

- 4)

+ 8(x + 2)

X = - -,

agnin an unacceptable value. We now evaluate D 2 at the corners of the triangle in Figure 12.59.
/(0, 0)

= j329/ 4l.

f(l , O)=(ffi

J (O. 2)

(ill.

The poitu on the lirst quadrnt pan of the pla ne 6x + 3y + 4z = 6 closest to (4, 6, 7) is
therefore (0, 6/5, 3/5), where the distance is .J8o = 4.)5 .

..-...

12. I I

At>Witllt M;.,\ima anll ~1 in i rn :a

867

EXERCISES 12. 11

Jn E.:ereises 1-S 000 lhe ma,.imum and minimUin values of the fwu.:-

aion on lhe regaon.


1. /(X . J') =X~+)'"~ on

R : .r1 + y 1 $

j(x. y) = .t '+.r+Jy'+.l' omhcn:gion R boonucdby y = x+ I.


.)' = I -X , )' =X - I , ,1' = - X - I

2.

3. j(x. y)
X

* 4.
+

+ )' = 4. y + I

X.

y-

where P = x + y + z. S how l h:U A is m:aximized for fixed I' when


lhc Lti unglc is cqui.latcml.

6x = 14.
:;: 12. f ind lhe 1>0in1 on the surface z
(I , I, 0).

2?. Show that for any 1riangle witb interior angles A. 8 ~and C.
sin (A/ 2) sin (8 / 2) sin (C/ 2) 5 l/8.

Find the ll mximum \1alue of the runcticm f (A . 8 . C) =


sin (A/ 2) sin( B/2) sin (C/ 2).
28 . S h()w lh:H Ieos x +cosy + sin .r sin Yl ~ 2 for all .r :-l nd y.
Him:

2z_ - 6 (.;}o:;c;st to tbc origin.

I I . t-l nd lhe chonecl distance from ( - I. I. 2) to the plane 2c - 3y

= xl + y

13. Find the point on that part of the plane '

~ y

*
* 29.

A l>ilu is in tbc shctpc of u right-circular cylindc.r sunnountcc.l by d.


rigtu-circu1ar cone. If 1he rad iu~> of each i$: 6 m and lhe In aI surface
a.rcu 1.nus.1be 200m l (nol i.ncludin the base), whal heigh!.$ for the cone

closest to the 1>0im

+ 2z =

,t= J ~( : - x) (~ - .v)(~ -z).

:S A :S 3.

"' 9. Find ma'<imum and minimun1 vAIUC.S ol 1he function f (x . )' , z) =


2
.t)' z3. 011 that PJ1t of the plane X + .Y + l : 6 ror which (a) X >
0. )' > 0. l > 0 and (b) X 2:. 0. y 2:. 0, l 2:. 0.

+ )' -

8. j(x.)') = x '+y 3 -3x-3y+2onthccirdc.r1 + .r 1 5 1

+ lO. Find the poinl on the; plnnc x

y1

f (x ~ y) lx - 2yl
the CUI\ C j,tj + jyj
I.
+ 26. If P i' lhc ~rimNer of a rri:utste \Vith s ides of lengths x , y > and
z. the urcu or Ihe tria ngle is
Orl

j(x. y) = J.r'

4 in the fii'St

octant thaL b. closest to the point ( 3. 3. I) . for this qu;.:s.tion asswnc.


lhal the cur\'e& of interseelion of the plane with the coordimuc pl;mcs
or<: part of the surface.

14. 'The cla;IIOstatic p.>eential on c,act, puilll in L1le region 0 5 x .5


1, 0 ~ y S 1 i~ @:i\'<:n by \'(x. y)
48xy - 32.r l- 24 yl. Find ahe
maximum and minimum potentials in lhc region.

IS. \Vhcn d. roc.:tangu.Jar bo,\ is M:rll through the mail. the po:H omcc
dcrnand'= thm 1hc fcngah of lhe ho'C pfu" lwice the sum of il ~ height
and width be no more th!ln '250 em. Find lhe di me n sion~ ot' the box
satisfying this requirement that encloses dtc lurgc.st possible volume.

16. An open tank in the fonn of a rectangular pMallclcpipcd j.;; to be


built to hold JCX)() L of 3Cid. l(lhe COSt per unil ~re!l or lining lhC btl ~e
or lhc lo:tnk is three limes that of the sidc.s1 what dimensions m.inimilc
the cost of lining the tank'?

nnd cylinder yield IDaximum end osc-d volume?


30. Whal 'v-alues of x :.md y m:txim.i1..c lhc production function
P (.\" . y) - kx yt' . where k. a. und fJ arc p0$ithc CQn.stan~
Ia + /) = 1). when x and .'' muSI :>atisf)' A.r + 8 y = C. when:
A , 8 , :1nd C :uc

po~it i vc con_~tan l s7

3 1. A t on~ piece of mc1a1 I 1n wide is Nntat A anti 8. ::iS Shown in the


hgurc below. 10 form a ch:1nncl wilh 1hrce Slntig:ht si dcs. I f the bends
uro equidistant from the ends. where should they be made i 11 order to
obtain maximum po.ssiblc flow ol' Ruid along thcchunncl?
~ - ~

32. Find m:~xim u 1n and minimum valueG.of the function f(.r. y. z)


z2 .:;: I,

xy + .\"Z on the rcg.ion .r 2 + y!

J.3. F'ind mclxi.nun and minimum valu~-.t of the- rul'lclion f (:r ~ y, z) =


;. 2.Y<: on theregion x~

18. Pnwc that for uianglcs the poim lhal minimizes the sum of the
~uares: of 1he diS'lances 10 the venices: is: 1he cemmid.

19. find the point on tl1e curve x 2 - xy + y 2 - z 2 = 1, x 2 + y 2 = 1


c losest to the origin.

20. Find the dimensions of lhe box witb l!argeSl possible volume lhlll
can fit inside theeUipsoid .r1j ,1l + y 1j l)1+ z2jt?
1. assumjng thal
its edges arc parallel to the coordinate axes.

+ 2"2. Find the maximurn a nd mininlun values o f / (:<, y) = x 2 - y 2


on the c ircle x 2 + y 2 ;; 1.
23. f ind the maximum und n1inimurn vo.lucs of f (x , y) ;;::;: lx - y l
on lhc circle x 2 + y;: = 1

2S. t'1nd 1he m ax unum ~md mmimum va lues of

j(x. y) = x'y + xy + yon R : - 1 5 x 5 I. -I 5 y 5 I

+ )'Z + z2 = I.

7. j(x. ,1') x 3 + yl - 3.r- 12y + 2 on the ,quare - 3


- .' SJS-'

X_l"Z 01'1 t.lle "'phcte x 2

the mnximum a nd rninimtun va lues of j (x . y) = x:! =3x + 4y on region R lloonued by the lin<-'! x + )' = I. * <>n24 .theFindcurve
j.t I + l.l'l = I.

+ 2.ry - )'1 + 5 on R : ~.r + 9y1 5 36


6. j(x . y ) = x3 - 3x + y 1 + 2y on the ttianglc bounded by x
0. .l' = 0. .r + )' = I

5.

21. Find the maxim urn ond minimum \111.luesol'tht function f(x. )', z)

** 34.

+ y 2 ::::; 1

0 !: z S

I.

A company produces two produt"ts, X and

Y. each or whkh must


pass through three stages of manufac..iurc. In phase I. tlp to eight tmiiS
per hour of X tom be processed and up to four tmits per hour or Y.
For phase II, l b t numbers of uni1s per hour of X and Y are 3 and 6.
respcx:lively. whereas l b t IOlal number of unitS l h !IL can be h!mdlcd in
phase JJI is nine per two-h04H"shjfl for each of X and Y. The profil per
unit of X is $200 and per unil of Y is $300. How many unhs of each
product shouJd be manufacttlrcd for maximum profit?

868

**

Chap!C:J J2

Diffc:t'CO lial Ollrul u~ orMu.Hi,ruiablc FUIIo\.llt.lfl)

35. A cow's daHy diet consi~ of three r()()(.b: ha)', gruin. and wpplc
mcnts. The cow is alwt~ys given I I kg of hay per day. 50(bofwhich is
digestive material nnd 12t:'c of which is: protein. Gmin is 74 ~ dig.e~ai'e
maLerial and 8.8% protein. wheteassupplcments arc62~ dicsti\'C rna

~pace.

** 39,

1.9 and 2.0 kg or pt()(<:in. Octcnnine the daily amoun t ~; of grain and

and J

kept to a minimum.
xl

+ y2 ~ 1.l.

largc~l

**

0. TileSC incqualilics describe a pol)gon in the firs.t quadntnl

40. In Consulting Projecl :; in Seclion 4.7. '"t.e considered seismic


prOSp.'Cting witll tWO lllCdia. The figure belOW ShOwS the situation
for three media. t be problem ))c.ing lO predict depths t/ 1 and fl2 Show
tlWU. liM: time for a l:lig_oa.l lo be cotiucd by the wurcc. ~~ lluwgh
medium l with speed 111 , pass through medium 2 with spocd v2. tnl\'Cl
ll1ong, the interface between media 2 :tnd 3 with speed I')~ then through
lllcdiur.n 2 3nd cncdium I to the n:cciv.::r iii gi,cn by
I =

of the xy-planc. G i'-c an llfl!UDitnllo show that J(.t. y) is maximitcd


at one or lUC)te of the vertices of the. poJygon.

ui.s.nglc thatl\ai vcrlicc.s on the c irdc

+... 37. A rccumglc is sunnounted by an isOK"clcs triangle :ts shown in


the figure below. Find x, y. and 0 in or-d.e:r lh!tt 1he arcs of the figure
will be :lS large as pos.:~:;iblc under lhe rest riclion th:11 its perimeter mus1
be P.

34. 35. and 38 are examples of linear /Ut>gmmming

dimcnsionallitl('.ar prog:r.:tm111ing problem is to rn;::a.xinUu: Lhe function


j(x. y) =ex +tl,r w here c and d arc positi\'cconstants. Points lobe
oon~iderOO mUSI s:ati~fy m inequalilies ofd1e form A;x + 8;y 5 C; ,
i = t . m. where A; . 8 ;. and C; arc pOsitivccoosuuns. and x ~ 0

supplements that lhc C()W should be fed in order that lotal food cos~< be
36. finll thc arcJ. or th~

E~en:ises

problems that abound in applications of mathematic:-.. 111C gcucr.:t.l two

lcrialand J<I'.Oprotcin. TI-.:co.tof hay i $27.50 for tOOO kg. andgrJin


and supple menu co>L S110 un<J$175 for 1000 kg. A hcallhy cow's diet
mus\ contain b<::twccn 9.5 and 11.5 kg of digcsti,c material ondbc1 ~en

lfthecomputcr lab must haM! IOOcouJ.puters, how many ofeach

should the uni"cnity purchase in order \O minimilc cost?

2t/ 1 s;x8,

v,

Verify Lhat

1 i~ a

2tl2 StX02
t '!

J-

2tl,tan0, - 2tl, tan0,

o.inimun wlK:Il 01 a1tU Ql arc ghcn by

Sour
-. -ce_ _ _ _ _ _ __

Surface
X

,,,

Medium I
++ 38. The E.usy University ls buying compuLcr-s. h has three t:nO<icls to
choose from. Each model A CQlllputcr, wj1b 64 MD of memory and a
3GB hard dri,c. costs Sl300; model B. with 32MB of memory and o
4 OBdri\'C. C(k)l~ $1 200: and economy model C. wilh 16MB ofnlCillOI)'
anct a I GB drive. costs StOOO. For rea.~ons related 10 accreditation.
lhe university needs at Je'.Lq 2000 MK of memory and 150GB of disk

Malium2

,.,

Medium)

112.12 Lagrange Multiplier s


Many applied maxima and ntinima problems rcsull in coust roinl problems. In particular,
Examples 12.34 and 12.35 comain sU<:h problem>. In Example 12.34. to find extresn~ \'alues of
Ton the edge of the plate we maximized and minimi7..ed T(.t , )'} = 16x 2 - 24.<)' + 40y 2
$Ubjcct fir>t to the cQnMrainl ,r 2 + y 1
I. and then to the constraint y
0. Our method
lhcrc was 10 substilule from lhe constrain! equa1ion into T (x, )') in order tO oblain a func1ion
of one variable. In Example 12,35. to find the minimum <.li>tancc from (4. 6. 7) to the plan~
6x + 3)' + 4~ = 6, we minimized D 2 = (x - 4)2 + (}' - 6) 2 + (: - 7) 2 subject to the
Wll>lr-dinl 6x + 3y + 4 z = 6. Again we >Ubstitutcd from the <-onstraim to obtain D1 as a
function of two indcpc.ndcnt vurio.b lcs.
A nmwal question to as k is wheth er problems of th is type can be s olved without s ubstituting
from 1he constr'djnt equation. for if 1he cons1ra.i 111 equation is compl icated. subsl itution may
be very d ifficul t or even impossible. To show that there is iodeed an alte rnative. consider
the situation in which a function f (x , y, z) is to be maximized or minimized subject to two
constraints:

F(x, .v. t ) - 0,

G(x, y . : )

{12 ()(l..t )

0.

(12.60h)

Algcbr<lit.ally. we nre to find exltemc vnluc... nf .f(x. y , ::). considerilg Otll)' those \'a lues of x,
.)', and ;: that ~~li ;"i(y cquution~ 12.(,(). Oc:omdrically. we <.'~) n i1Hcrprct each \lf ll)t;SC co1\di tion~
~l) .specifying a sucfacc, so thul we are sc:cking extreme values of J(x, y. z), I.!Oe\o.:idering t>nly
those points on the curve of inccrsection C of the 5utfoces F (.\", y . z) :;;;;; 0 :.lnd C(x. ) 1. z) :;;;;; 0

(Figure 12.60).
Ex creme value~ of f(x, .V. t ) along C will occur either at cri1ieal points u f the funccio' or
at lhe cntl~ or the CUr\C. But whal derivulive t)f i.lel'i vacives of .f(x ' y . z) llJ'C \\'C talking about
when we say cJiaical r>Oim.s? Since we arc colCCmc<.l only with ' 'nlue..- of /(.t, y, z) on C, we
must lne(tn lhe derivative of f(x. y . z) along C (i.e .. the c.li_rec.tional deri\'ative in 1hc: umgem
dircerioo to C). If T. then. is a taJlSC" vec.ror to C. critical poit'ICS of j (x . y . z) a\olS C are

' 'alu.f:s of /(.r. )' :) t1h111g the CW'\~


(If iutci'\CUi.:lll (II l \\(1 i UI(:I<.'C:tt

'

g i ve~1

by

0 = Dr.( = "lf i,
or at poin1s where 1hc directional dcri\mivc is ml<.lellncd. t\t.:cording to equation 12.53. u umgcm
vector to C is
V F x '\IG. and lle1tce a u1tit tatge't vectOI' is

r =

+=

VF x VG

IVFx'ii'GI'

'111e tlii''Cclionnl derivative of /(X , y , z) ut I>Oinls along C is therefore ghcn b)'

D,-J =

V F X "10
\1 I. 1\TF X \TGI'

It follows. th~:n. that critical points of .f(.x , y. z) :ctre points (,,., y, .-;: ) that satisf~ the equation

\Tf \TF

\TG

= 0,

or J)()iniS ;u which the left side is not defined. Now vector V F X VG is perpendicular to both
'il F and \1G. Since \1 f' is pcrpcn<licularto \1 F x \1 G (their dot product is zero). it follows
that V f ft1USt lic in the plane or 'V F and V G . Consequently, lhere exist scalars A anU p.. s uch

tlu

"l .f

= (-)..) \1 F + (-J.L) VG.

or

VJ

+ >. V F + 1<'ilG =

0.

\1 2 6 1)

This vector equmion is equhalenl to the three sc~l ar equations

ilf

iJF

iJG

iJx
!Jf

ax

ax

aF

ac

ay

ay

-+A.- +J.L-

= 0.

I) 2.62)

a>' = 0.

( 12.62bl

- + ),- + J,L-

ilf

a~

. il F
i)G
+J.- +JL-

az

az

o.

( 12 6 2cl

and thc.<e cqumions must be satisfied m l1 critical point m which the dirccrionnl derivative of
(x. y, z) vanishes. Note. too. that at a point at which the directional deriYative of f (x. )' , :)
does not exisL one of the (><mini derivat ives in these e quations doc.1 not exist. In other words.
we have shown that c ritica.l po ints of f(x. y, z) are points that satisfy equations 12 .62 or

points a t wh i.c h the equal ions are unde (ined. These equatioos. however. cont.aio fi ve uoknowos:
x, y, z. !..~ and J.L To c.omplete the system we add equations 12.60 since they must also
be sarislled by a critical po inl. Equations 12.60 and 12.62 therefore yield a system of t\ve

equations in the five unknowns ..t. y. ::.A , and J.L ; the first three Llllknovms (.f. y , Z) define
a critical poim of f(x ." z) ulong C. The advantage of !his sySicm or equations lies in the
f~lct 1hat difl'erentiations in 12.62 i1wolvc Otlly l hc given functions {:tnd no ~bstitut ions from the
cc.m:o;traint e<IUations arc necessary). \Vhnt we huvc sacrificed isn sy.~; tem of three eqla tion~ in the
three unknowns (x. y. :) for asy<tem of fi,e equation,< in the live unknOMlS (x. y, z. )., 11).
Let us not forget that the originul problem wns to find extreme values for the function
f(.r. y. <:) subject to constraint< 12.60. What we have shown S<> far is that criticaiJ>oints at
whic.h the directional derivative off (x , y . z) vanishes can be fOtlllcl by solving equations 12.60
and 12.62. In mldition. critical points at which the directioo><ll derivative of f (x , )', z) docs not
exist arc points at which equations 12.62 are not defi ned. What remains is to evuluate .f(x. y, ~)
at all coi oical points and at the ends of C . If C is 11 closttl cww (i .c., if C rejoins it;;clf), then
f (x . y. z) 11eed.<to be eoaluarrd oafy at the criticttl poims. This turns out to be very important
i 11 prac1ice.
Through the directional c.lcri\ative and tangent vcc(()rS to curves, we have shown I hat cqua
tions 12.60 and 12.62 define critical points of a function j(x , y, z) that is subjc.ct to two
con.traints: F (x, y . z) = 0 and G (.t . y . ~) = 0 . But what about othet situations? Let us
say. forexllmple, thcll we require extreme vallle'S of a function j'(x, y, .:: , t) subject to a single
constraint F(~ , y. z. 1) = 0. How shall we find <'ritical points of this function? Fortunately,
a'i we now show, there is a very simple method thai yie lds equations 12.60 and 12.62. ancJ this
mcth()d general iu-s to 01her situations also.
To fi nd coitical points of f(x, y, ~) subject to constraints 12.60, we del inc a function
L(~. y .

z. I.,Jl)

= f( x, y, z)

+ 1- F(x. )', ~ ) + 11G(x, y, z),

and regard it as a function of five independent variables .r, )' . z. ).. and Jl . To lind critical
points of this function, we would tirst solve the equations obtained by sening each of the panial
derivatives of L equal to :r.ero:

il L

= ax

il L
ay

8L

0=- =

a:

0=

8L
ill.
i)L

IJf
ib

'i!f
ay

ar

,8F

il G

1J F

/J G

a.v

ay

u
a:

ac

+ ,.ax +JL -ax .


+ }.- + JL - .

- + ).- +ttCiz

az

F(..r. y, :),

= il"' = G(..r, y, z).

In addilion. we. would C'OilSider poinL~ at which the partial deri,atives of L do not ex
ist. Clcsoiy. thi means point' (x , )',~) >II which >Ill)' of the partial derivatives of /(.r . y . : ).
f(x , y , <) .and G(x, y , z) do not exist. Btll thesc are equations 12.60 and 12.62. We ha,c
shown. then. that finding critical point (x, ,\', ;:) of f (x. y, z) subject to F (X. y, :) = 0 and
G(x, )', z) = 0 is equhalem oo finding criLical point< (x , y, z,l., tL) of L(.r, )', z, )., Jt ) .
The twQ unk llOWns ;.. and Jl that a<:CQill)lany a critical pc.~int (.r. )'. z) Qf f (x. )'. :) arc called
Lagrange muJiipuers. They arc not a part o r the solution (x, y, z) to the original problem,
but have been introduced as a tonvenience by which to arrive at thut solution. The function
L (x . )'. ~. i. , 11-l is olien called !he L2gran1,>ian of the. problem.
The melhod for olher constraint problems should now be evidenL

Given a function

f (x, y, z, t , .. .) of n variables to maximize or minimize subject to m constraints


F1 (.x, y, z, t, . . .)

= 0,

F1(x, y, z.

1, . . . )

= 0, ... ,

F,,(x, y, z. 1, . .)

= 0,

(12.63)

we imroducc m Lagrange multipliers )q. Az . ... 1 Am imo a Lagrangia1l of n + m independem


wriablc:) .r . y. z ,t , . . . , )w 1 , A:!. ... ,).,, :

L (x. )' .

~- I . . . . )q, i.~ .

.. . , )..,) = j(x , )', z, I ,

+
Critical points (.r. y . .:. 1 .

. .)

ing critic.:al txJinLo; of L(,t. )',

of f (x, y. z,

z, I,

;n

0 = -

ih

IJL

. . .)

+ A 1F 1(x , y . (., I, . . .)

"- ,

.1.-.Fm(X, y , <. ,I , ... ).


I . . )

are then delermined by the equations defin-

ar +/q il F,
a1:,.
+ ,+ 1..,--,

= -

a.\'

ax

ilx
i! F ,

il(

F1(x , >' ~. t,

= -())ilL,m

i!F,.

( 12 65h l

!1 2.65cl

. . ) .

= F,.(x, y, ~ . t, . . .).

112.6Se 1

To use Lagrange multipliers in Example 12.35, where we were. minimizing D 1

4} t (y - 6) 2 + {.:- 7) 2 subject to 6.t


2

L.(x . y. <., }.)

12.(>4 1

... , A. >.2 ... , ).,,),namely,

0 = - = -- + .1., - + .. , + i. .,--.
lly
a.v
8y
a.v

+ 3)' + 4r. "'6, we detiuc the Lugranginn

= (.r -

= 0 1 + .1.(6x + 3y + 4~- 6)
= (x - 4}2

+ (y -

6)1 + (z - 7)2 ,.. /,(6.f

+ 3y + 4z

- 6).

Critical poin1s or L(x, y, z . .1.) arc defined by

= -aL
ax = 2(x -

4)

+ 61-.

aL
0 = -ay = 2(1. 6)

+ 3.1.,

iJL

o = az = 2(z ilL
0 = = 6x
[}A

7}

+ 41-,

+ 3y + 4z -

li.

The solution of this linear system is (x, y. <., ).} = ( - 140/6 1. 17 ~/6 1, 171/ 6 1. 128/61),
yieldingasbeforethccritic~l poinl {-140/ 61. 174/6 1, 171 /61) or D 2 .

I EXAMPLE

12.36

Find <he maximum and minimum \'al ues or the function J(.{, y . z}
xl + yl + tz = I .

SOLUT ION

If we dellne lhe Lagr.ngian


L(x , y, z, A) = x yz

+ l-(x2 + y 2 + z2 -

1},

= xyz on the sphere

872

Cli.ap1er 12 Diffe:ren1ial Calculus of Mulli\'ruiable Funclions

then critical points of L. and therefore o f f (x , y, z) . are deli ned by the equat ions

aL =

0= -

YZ

+ 2.!.x,

0=

xz

+ 2.!.y..

ax
aL
-ay =

aL = xy + 2.!.z,
az
aL
= x2 + y 2 + z2 a;.,

0= 0=

I.

If we multiply Lhe first equation by y and Lhe sec.ond by x , and equate the resulting expressions
for 2Axy, we have
,J2 _ _

'

(.

.2z
,.

~1

Consequently, eithe r z = 0 OJ' y = x.


Case 1:
z. = 0. In this case the equations reduce to

1~ = 0,

).,y = 0.

xy = 0.

+/

= 1.

The first implies that either x = 0 or A = 0. If x = 0, then y = I, and we have two critical
points (0, I , 0) . If)., = 0, then the third eq uation requires x = 0 or y = 0. We therefore
obtain two additional critical points( I, 0, 0) .
Case II:
y = x . In this case the equa tions reduce to
xz

2/.x = 0,

x2

2),z = 0,

2x 2

z 2 = I.

Theli rst implies that eitherx = Oor z = -2) . U x = O.then z = J.aodwe have the two
critical poims (0, 0, I) . If z = -2).,, then the last two equations imply that x = I/ J3,
and we obtain the four critical points
(1/J:\. 1/ J:\. 1/J:\)

and

(1/J:\, 1/J:\, -1/J:\).

Case JU:
y = - x . This case is sinti.lar to that for y = x, a nd leads to the add itiona.l four
critical points
and

Because x 2 + y 2 + z 2 = I is a surface without a boundary, we complete the problem by


evaluating f(x , y, z) at each o f the CJi tical points:
/(1, 0 . 0) = j (O. 1, 0) = j(O, 0 , 1) = 0 ,

j(l/ v"i, 1/v"i. l/ v"i) = j (l/v"i. =FI/v"i. - 1/v"S) = v"i/9.


f( tj ../3,tj ../3, -l / ../3) = f ( tj../3,:r:.tf ../3, 1/ v"i) = -../3/ 9.
The maximum and minimum values of f(x, y, z) on x 2 + y 2 + z 2 = I ;ue therefore J:\/9

and - ../3/ 9.
~

To compare the Lagra ng ian solution in this example 10 that without a Lagmnge multip lier, see
Exercise 21 in Section 12 .11.

I EXAMPLE 12.37
A company manufactures wheelbarrows at'' of its plants. The cost of manufacwring x; whccJbarrows at the i' h plant i s xf /c; . where Ci > 0 are known constants. The total cost or
manufacturing x 1 wheelbarrows at plant 1 .t2 at plant 2 . .. , x,, at p hmt 11 , is therefore

j(xt, . . . , x,) =
Tite produc tion enginct:r wishes to schedule a total of D wheelbarrows among the plant:;. Now
many should each plant produce if costs are to be minimized, and what is minimum cost?
SOLUTION We mu>1 minimize j (x,
= D. If we define the Lagrangicm

. . . , x,)

subje.c t to the cons traint 't

+ Xz + +

Xn

L(x1, . .. , X,,),) =

f(x 1 . ... , X,)

+ i.(x 1 + Xl + +

the n critical points or L (and therefore off ) are given by the

BL

8/

0= -

- -

Ox;

2x;

Oxi

iJ L

0 = -/)), =

Xt

+A = - + >.,

D),

I equations

=1, ... ,11,

cr

+ Xz + + x,. -

11

X, -

D.

These give x; = -c;)./2 for each i , and if we substitute these imo the last equation,
Ct A
c2l.
c, l.
- - - - - .. - - = D

===}

)..

- 20
= ---Ct

+ .. . + c.,

Thus, produclitln levels at the plclnts should he

c1 D

Xj -

Ct

+ + C

= I , . ... 11.

11

M i nimum cost is
2

-1
C't

Ct

C't

+ + Cn

+ .. +

1 (

ell

c.

c,D )z=

+ + c,t

Ct

+ .. . + c,

We are being approached by a hydraulic engineer who is fa bl'icati ng an open channel from
long p ieces of metal joined end to e nd. We take the width of the metal as I metre. a hhough
the solution is e.a.sily adapted to any wid th. Each piece of metal is bent to f01m the channel.
The engineer was once a student of this text and has solved Exercise 3 1 in Section 12. 1 I,
but he is not convinced that this is the. opti mum shape of the channeL For instance. why
is it necessary chat the two sides of 1.he channel be of the s.arne length'! We are to s how
that the solution to Exerc ise 31 in Section 12.11 is indeed the best of all possible channels
with a maximum of two bends.

a74

<:ropctr I~ Differenti;ll C...lcutus of ~tuhivariable Fuoc1ion)

SOI.liTlO:>; Figure 12.61a shows a channel with two be-nds bm allows for different
of the
of the channel and therefore dilfcrem angel, 8 and; . Volume of now
along
channeisl is maximi.ted when i!S cro;s-sectional area is maximited, and lllc nrea
for
thisthe
channel

lenl)th~

\ide~

where. inorder that boch ~ides of the channel ha>e the same height. the condition y sin 8
(I - .1 - y) sin t/)must be
If we rcard x, 8, and If> as independent
and thi; equation as a restriction on y, then A must be maxirniLed for \-alues of x. 8, and
if> in the box of Figure 12.61b.

smi~ticcl.

vuriuble~.

Ba!ding

Dcmlln

I"< of I11C1.1I to funn ch.Jnn<l

(I -

r -ylcos.p

...

~ 1 - ..r -.\)~mj

/)"_,_}

We shall find critical \'a lues of A inside the box. and then rurn ro the six faces oft he
One i-< the comple.ity of the

bo~ for maxima thereon. We shall ha\e two diflicultie<.

equations th:ll DlUSI be l>Qi vcd, anu the other i, to not lose sight of exn~tly where wear~ on
the solution proc~. For critical poims of A interior to the box, we lonu the U.grangcan

L(x. y. e. 4>. ),)

I Stn 8 cos 8 + xy Sill


. u"
= 2y
2

2
+~(I
2 - x- y) sinl/>t~tf> + i.(.l in8- (I -x- J)sintf>J.
Critical p<>Onts of this function are dcRned by the equations
0 -=

aL
a.,

=ysin 0-(1- x - y)sin if>c<>s'i'> + i.sintJ> ,

0 = 3L
()y = ysin 8eos8 +.ninO- (I -

.r- )}sintJ>costJ>

+>.(sin II+ sin 9'>).

o = ilill!L = 2~y'(cos 2 11
0

= ilL = ~(I
84>
2

-x-

= ()L
= ysinll 3).

- sin 11) + xyc051i + i.ycos li,


2

y)2(cos'4> - sin''i'>J - 1.(1- x- y)eos'i'>.

(I- x - y)sinof> .

( 12.66J)

(I ~.66b)
( I ~. 66c)

'.66d)
( 12.66<)

l2.1Z

l..a~tl~

Mullipliets

176

ll we sublrnc11he firs1 oflhese from the second, we oblain

One possibi lil) is sin I) = 0

I)

= 0. bu1 lhis yields minimum

OJ"Ca.

11le 01her

p<K\Ihilily i< lhal

.1 cosO

+ .x -

+ ),

= 0.

112 660

Wnen we 50hc this for'- and subslitu1c into equation 12.66<:,


I .,

.,

ly-(cos-8- sin 8)

+ xycos8 i ( cos8 -

y cos8(y- x- ycos9)- 0

= 0.

Thu. y
0 , .-hich ~:ive minimum area, or co< 0
equalion 12.66c from 12.66a. we find

= I/ 2 =

(I

1f

1-' If 11 e 'lllMtaCI

- ( I - x- y)<intJ> cost/> +(I - x- y)sin +A. sin = 0

sintj> [( l - x - y)(l- costj>) +A]= 0.

F.hhen in 4> = 0
if>= 0. givi ng rninimun1 A. or ( I - x - y)( l - CL") +A
When we subslitulc 1his i111o equation 12.66<1,
I

.,

.,

.,

0 = - (I - .r - yt(cos- sill")+ ( I -

This equation implies thal x


I

+y

x-

y) ( I -

= I. gh.-ing minimum A. or.

(cos1 - sill1 tj>) - cos2 4> +cos = 0

==>

cos = 2

= 0.

costj>)co~.

1f

'

F.ttuation 12.66e nO\' give. x + 2y = I . When thi< i< 'Ub>tituted inlo equation' 12.(16i.
e, two equations in )' and ). result. and lhe solution for)' is)' = 1/3. ThiJ 1n turn gh~
.t = 1/3. and the ace-d of the channel with the.se ,alue:. of. . y. 8 and i'

A=

~ GY (~)G) + (D G) (~)

+HI -i-3Y (~) G)=~We now tum 10 the six faces of lhe box defining pennissible values of x. I). and 4> . The
1hrce faces x
I, = 0, and 8
0 give minimum ,alucs 0 for area of lhc channel.
Thi< lea' es face> x = 0. = 1r/2, and (I = 1f /2. We need di>euss only one of the 1""1
1wo faces. since discussions would be identical for lhe 01her. Considoo lirstthen the face
x = 0. In this case 1he me1al has only one bend as shown in Figure 12.62a. Area or 1he

cros')-Secl ion of lhechannel is


I

A = - y'sin 8cos8
2

+ - (1- y ) 2 sin cos cf>.


2

objl:ccco che rcscriccion y sin tl = ( I - J ) sin . If (J mxl ~~ are caken n> illllCJ>cndcm
variables. this function must be n>Aximi'ild '"' the ;;quare in Figure 12.62b.
Our..... llW'

J.t.Wo. ul tn~~u c-Mud

du~

.' ~ &

---....-"!; ~;~ ;~-;; ~;.;~;;?


(I

) ') <'OS II

7rf21------,

.v&C_~

>

Fm.c we find critical point~ imeriOC" to the sqla-tfc., and then consider the f<ur lines
fonnin~ the boundary of the squurc.

For critical 1>0illls of A mtcrior to the ~quare, we

fom\ lhc ulgrangian

Cn ucal poincs of 1his functino are defined by tbe cquatoon<

= IJaeL

= IJL
- =

= -

IJ /,

iJ )'

= )'SinfJcosO .

(I - y}sin <f>cos!f> + >.(sill (}+ sin </>),

~)1(eo>-2 0- >in1 (1) + ). vco>fJ.

2
I

iJ

- (1 - I') (ells'<,$- sin -'-)- i.(l - )) cos4>.


2

V'

aL =

y sin O- ( I - )') sin,P.

IJ),

The second or chese implie>chat

>=

126M

~671>1
1 ~67.:1

cl 2(>7d)

0, which minimi1es A. or that


12 67~1

The third yields y

I, which minimi7es A, or
(I - y)(cos2 </l

sin2 </J)

- 2/..cos = 0.

(1 2.671)

When we subSiirute fi'om equation 12.67d into 12.67o. we olllain

Either sin 4> = 0. which leads 10 minimum A , or


y( l - y)C0$ 11- y( l -

y)cost/>

+ 1-. =

0.

( 12.67h l

When we ><>he thi> for), = - y(l - y)(cos8 - co,,P). and substitute into equation
12.6k

12.1 2 l-Mlr'.'$C Mul11plln"

y (cos 9 - sin 0)- 2y(l - y)cos 9(co~9- cos)=


2

177

o.

This require._,) = 0. a minimizing value.. o:r

cos2 9 - sin2 8- 2(1 - y )coo; O(c:osO- cos~) = 0.

'f>1

When we substiiUie ror ;, in equa~ ion 12.671.


2
(I - ) ) (<..,.2 >- sin >)
(rum

+ 2y(l- y)cosql(c'OSO- cos</>) =

0,

which y = I. a minimum. or

co2 - sin2

+ 2y cos(cos ll -

We now <Olveequation l2 67d for y


2 ,.

. 1 ,.

cos " - stn " 1

cc." '

. ,

<I> - '"'-

co "' )

= 0.

= >in~/(sinO +sin ).and substitute into 12.61 i. j.


2sin0cosll(cos 8- cos)
sin II

= 0,

+ in cf>

2sinlf>cos,P(co<8 -cos</>)

= 0.

+ sin tf>

sin 8

The-e imply thai

- sin1 e
2 sin 8 cosll

cos 1 9

cos2 - sin2 t/J

2 si n c:h<>s <,It

Thetullywa)rorthiscquationtoholdisfor O
The a rtll of 1hc channel for thei<le \ 'lll ues is

= <P = il/4.aldthisimplie>t1HII )' = 1/ 2.

~ GY G) + ~ GY G) = ITl

\Ve should now consider area on

the rour

~ide.

of the square. Along

= 0 nncl rJI = 0 .rea is zero. Con


>it.lcrotion or> = Jl / 2 is the mirror
imasc of 8 = Jl /2 For 9 = 1r /2. we

II

re <:tlr"idcring channels in 1hc shape

1-y

in Fi&ure 12.63. An.'>l is

wtlcre 0 !f )' !f I /2. For critical


pomt'). we w hc
0

2I ( / I -

)' ~) )
2) - ../r,:f:::;

)' = -.
3

The area or the channel " 'hen y = 1/ 3 is A = l../3/ 18 1. The area i> a:ro when y = 0
1/ 2. This completes the discussioo of the face x = 0 of the box in Figure
12.61b.

und y

= 1r / 2 of ltlc box. In Lhis case. the c-hannel ha.) Ihi:

Our final con~ideralion is face 8


s h<1pe in Figure 12.64a. Its area is

= xy +

2(1 -

x - y) 2 sin</> cos cf>,

where)' = (I Figure 12.64b.

.t -

y) si n <ft . This function mus1 be maxi mi~.L'd over lhe rectangle in

f':'l ,

Ch.lnRtl
mi-ttuion

u(

l'hlllnc:l with one

v~ica l

side

(1-x-y)eo<q>

(1 -x-y)sin

The associa1cd t agnwg.ian,


I

L(x,y. .>.) =X)' +

2(1-x- y)

sincos+ AI)'- ( I - x- y)sin'1.

hns critical points defined b)

ilL
0 = - = y - ( I - .r - y) sin 4> cos - A sin,

ox

= ilL
0v)'

= /J =

= .v-

8L

(I

2(1 -

- .r- y)sincns..-A( I +sin<,\),

(IUll!bt

x- y) 2(cos- sin")- ).(I- .T- y)cos. tl2.(>Kcl

ilL

0 = -()). = )' - ( I - X -

l') san 'I'


.t..

(12 .6Kdt

The fi rst two C<tuations imply thai J. = y - x. We substitute this into equa tion I 2.68a,
and into 12.68c, after removi ng the oxtra factor I -X - )',

0 = y- ( l - .v- y) sincos> + (y- x) sin,


0 = (I -

x- y)(cos2 </>- sin1 )- 2(y - x )cos .

112.68el

i 12.6Sfl

When we multiply the fin;t of ~1cse by 2 cos<,\. the second by sin 4>, and add,

2ycos - 2(1 - x - y)sin <j> cos2 +(I- .v - y)(c<ts 1 - sin1)sin = 0 .


We can u.se equation 12.68c.l to climinalt x from this equation,

This implies that

2cos- 2cos2

y = 0. which gives minimum A. or.

+ (cos2 - sin2 )

Equal ion I 2.68f now g ives

= 0

cos =

~)

1T

= -.
3

( I -.< - y ) G -

D- --> G)

2(y

y:: 3x - l .

When we sub:.titutc this in equation 12.68d.


I - ( I - ~ - 3.<

3.< -

Th i:; K>w J::_ivc.s )

A=

3- ../3

../3

1)2 = 0

,\'

We now consider A along t he f our


cdge$ Ofthcrcctangle in ftigun:. J2 .64b.
Alon.gt/J = O~tJld X = l . a rct~h.;nm i n

../3

2-../3
-2-.

I'M) \ \.Tiicl \id .t.'l

Ctl~ .l'
0 (sc~ Figure 12.63). This
leave:.$ Ihe case t/J
Jr /2. u rectangu
lar chnnnel ns shuwJJ i n Figure 12.65.
It~ nrc8 i.s

= 2I x ( l

Ill!I:I

im um. We hnve alrc:.dy <.Jcalt w ith the

]'(T ) (2) - ~

I
3-../3
(2 - ../3)+2 1- -3- -(2- ../3)

3- ../3

./3. and are.a ot' the channel is

2 -

( )
-:J-

- x),

(I

\ )/2

0 !': .r !': 1.

Cri1ical ,,oints of this fu nc1ion are g i, en

by 0 = I - 2t . 11-om which .\'

::I!

1/2.

A rcH of lhc channel is ~- Area is


-zero when ;r 0 ttlld .r I .
1l1i~ compl e1e~ our cuk:uhui on~: we have COtl.Sidcae:d utca of Lhe chalnel

for va1ue!) of

,, IJ . and interiorto the box of Figure 12.6 1b. over each face of~'le box. a long each e<lge.
<U>d ac each corner. The boxed numbers are ./i/12 . 1/8 ./i/ 18 and (2- ./i)/2. <he
largest of\o\'hich is ./3/12 . This is the: solutivn tv E~~rvise 31 i 11 Section 12. t l. confinltillb
Chill icprovides thcchtumcl with maximum flow from ull chunncls with a muximum of two
bends. \Vc m ight notice thtH if more bends or c urved sides were allow.:.'d. the. ~uca coukl
be in<.'t'Ca.~ed beyond this. For it\.~tance, if the channel is 5em icircular. iL~ are\ is 1/ (211').

EXERC I S E S 12. 12

In Exercises f 8 usc l.agrnngc mulliplu::rs 10 tind rnoxirrltnn und min


va lut~ o f the funclion s ubject 10 rhe constrJinls. In c:tch case.

imum

interpret r h~ cons:rm inl'


~ I. f(x. y)

= x'

gcon-.ctricnfly.

ll. {(.<, _)', Z)

+ 2y' + 4z' =

J. /(A'. y) = .r + y ubjeclto (.r - 1) 2 + y 2 = I

= x~ + y~ + z~ subject 10 x' + y' + z' = 9


f(x, y, z) = xyz subjt.'i:t 10 .r2 + 2y 2 + 3z2 = 12

4. f(x,

5.

y, z)

= .x' y + z subjctllO x' + y'"' I, t = )'

f(.r, )'. Z) =

,1,'

+ yZ + t 1 s ubjccl tn .r1 + y 1 + t 1 =

2z. X+

y+:= l

+ )' $ubjc<l <o x '+ y' = 4

-< 2 f(x . y. z) = 5x - 2y + 3<: + 4 s ubj0<11o x '

6. f(x . y, Z)

* 7.

2
"'X.)'t -X l ubjoct IO

x1 + )'1 = I , t = Jx 1 + )'!

.._ 9. Usc L.asrnngc muhij)l icrs to Mhc Exercise I I in Section l l . l l.

+ 10.

*
*
*

u~e L..r.tgrun,gc IUUitiplicrs

to solve E>.crcise ' 2 in Section 12.1' .

11. Usc Lagrange multipliers to soh'C

E:~crci sc

20 in Section 12 .11.

12. Use Lagrange mulli J)Iiel's Lo solve Exercise 26 in Section I 2. 11.

J3 . Usc L.agn:mge mullipLicrs 10 solve Exercise 27 in Section 12.1I.

880

Ch~()le

12 Differential CalculusofMulli\ari.-tble Ftllh:riou

14. Usc Lngrungc mulliptjcrs tosol\c F.:tcn::isc29 in Sct:tion 12. 11 .

t 15. Usc l...agrange nua lliJ"i e~ to $01,~ Exercise 22 in Section 12.1 1.

;~;. 27. f-I nd the max.Jmum va.luc of j'(.r , )'. c;) - ;2yt - ..r zy 2 subjcd
to constraints x 2 + y 2
1. t = / ."2 + y'l.

23.
,. 16. Usc Lagranie mull ipHcrs to sol"e E'tcn:isc 23 in Section J 2. J I.

Suppose I hal F(x, y) = 0 :~nd G(.r , )') = 0 define I\VO curves


ood C: in chc "" l~anc. Lea P(xo. rol :uld Q(Xo. Yo) be ahc
JX>ints on Ca amJ Cz lh:t.t minimitc the c.listtulCC between Ca mltl cl .
lf c. tnd c2 have. tangent lines oJ.t p and Q. !lilOW l.hallhc line p Q
is pcrpcndt-c;tllu.rto the~ tangcrltl incs.

c,

>11

1<

y -axe.~. Find coordir~mes of the ends of !he major and mi nor

2~. Usc Lagrange multipliers LO find Lhc point on Lhc first octant 1Xlt1 of
Lire i>lanc A.t +By+ Cl.
DCA . Bt C, D :lll po!>itivcc:orlSlW\L")
lhat maximizes the fu nction j'(.r, )' z)
;rJ' y1' i '. where I' tJ. and
,. arc pi)sirivc,: conllf lulli\.

... 30. The rnl iurn nf O tK1U1CS has pnrnrnctl'iC cq u atiOf\~

x =

+ t'

.r =

(<1 > 0)

(see Exercise 52 in Scctifln J .8 and P.xcrcisc 61 in Section q,l ). Find


the point in the first quadrom rarthcst rmm the origin in two ways:

2 1. Wlw are production lc,cls 0.1HI milliiHU!ll cost in Example 12.37


for the production of 500 whcclbam._)wS if there itrt: four p!ai)LS \~~o ilh
c 1 26. c2
24. <3 23. and c,. 27?

+ 4xy + 6)'1 = 14() describe$ cll i1)SC thol

nxcs.

19. Use L.agmngc mult iplierS tosohc Exci'C isc 17 iJl Scrtioll 12. 11 .

* 20.

e<}u:llion 3x'

has its centre at the origin, but major and minor tl.~cs urc not. :do11g thc-

.r- and

+ 17. Usc Lug runge mull ipHcrs t<Hohc E%.cn.:isc 24 in Section 12. 11.
ltl U:sc l.agr.Jngc multit;llicn; to whc Eu:n.:isc 25 i11St:"1ton 12.11 .

TIIC

(a) Expn:~s 0 2 = .r 2 + )' 2 ill 1crms of 1 and ma-<imi~e lhi~


function or one variable.

Zl. \Vhcn ;' thcnnQmtclcurructor is buill in the fonn of u ri &ht ~ci rcular
cylinder, neutron clifrusion theory requires its nKijus and height to sutisfy the equation

(b) Show th:u !tn irnp1icit C:ilu:Hion for the curve is xl

3axy wrd muxilnizc D 2

+ yl =

= x 2 + y 1 subj-ect 10 d1b cor~

strainL

where k. is a constn.nl. Find r nnd h i1'l tern1s of k if the reactor is 10


occupy 1lS sn'lall a \'Oiume as pOSsible.

* 23.

Find the pointS on the curve

x 2 + .ry + y 1

= I closest to ancl

rarth(:&t from the origin.

31. To find thcp(l intontllCcunc .t' 2- .ty + )'2+ ;:: 2 = 1 .r l +y2


1
chl!"C:it to the uligi n. we must rni nimitc !he fu r<.:~ itwr f(.x. ,Y. l) =
x 2 + y 2 + z 1 s ubjce-1 to the constraints dell ned by the equ.mions of the
cui'\'. Show th all hi~ can be d(lrlby(a) usi ng rwo l.agmngc n1ultipl icr;(;
(b)<Xp<es$in!l f (.r. y , z) in1ennsof .r ancl y lone, It = 1- xy,:~nd
minimizing this function subjecl to x 2 + )'2
I (with one Lclgrangc
nruhi tier); (C) c:<prcssiog f (.r, y , z) in lCI'IllS of..\' ntOtrc, u = I
x I - x z, ::and minimizing: thc$1! functions (ln llppropriatc i nrcr\'ols;
and (d) writing ,,.
cosl , y = s in/ along the cur\'C. expressing
f(x . y. z) in ccnns of t . u = I - s in t c:ost. ~1nd minimizing this
fuJ1Ction Oil app1"0priatc intcr, als.

32. Find rhc smallest aruJ l.argcst distwtces fro1n the ocigin to the curve
In F.xc1'Cisc,; 24-26 "sc l.agmng.c multipliers to find maximum nnd
mininmm \'a lues of 1hc fw\Ciion.
24. f(x. y)

25. f(.r,
2().

= 3x

+ 2.T)' -

+ 5 for 4.< + 9y := 36
1

y) ='' r + .ry1 + y r,.. - .s .r .s a, - .s .r .s

f(x. y , z)

= xy + xz for x ' + y 1 + t 2 := I

x'

+ y2{ 4 + z2/ 9 = I. x + .v + z = 0.

33. Find ahe Rla.<imulll "~"c of [(x, y, z) = (,ty + .r')/ (z'


~ubjecc l(l lhe consCr(!.int .r1(4 - x 1)
y1

.fl* 34. f'i nd 11-.:

+ I)

lntuimurn nnd mrninlUIIl values of the function j(X. y.

:) = zx:y~ + 2y::~ + 3z considering


ool) values of x. ,v. and z
2
~3tisfying the equation~

z = .r + .": , .r : + 3)': =

I. [)() this with

and without Lagr.mgc multipliers.

112.13 Least Squares


The basic tool of applied maahemaaics is lhe .fimction. Given a func aion such as f(x) =
.:r:l - 2.l' - 3 . and my value of.\' , say ..\" = 4, we calculate the vulue of the. fu nction at this .t
as .f(4) = 5. In many problems. lite function is noa given; ia mus1 be found . Whal we migha
have is a set of experimental data suggesting 1ha1 various quanLilies are related to one aoother.
but lhe daaa do nol specif y exacaly how ahey are relaled. For example, suppose a variable y is
known to depend on a variable x, and an experiment is performed Lo measure I0 values of y
corresponding 10 10 values of x. The resulls rue lisaed in Table I 2.3.

10

6.05

8.32

10.74

13.43

15.90

18.38

20.93

23.32

H .9 1

28.36

Consider the pro blem of finding that function y = y(.r) that best describes these data. Twu
conside.rations are important - ~impl ici ty and accuracy. We would like as simpJe a function
as iXJ.~S ibl e tO describe how )' depends on x. AI the same t.ime. we want the function to 00 as
accurate ns possible. Por exa mple~ we could find a pQiynvrnial of degree nine to fit the data
exactly; it would give the exact yvalue for each Yaluc of x. But this would hardly be a simple
repre:-;enuuion. ll would also be unreasomble rrom the fol1owingstandt>oint Because y value.s
are determine<..! experi mentally (wcussumc that xvalucs arccxact). thcy will be subject to errors
both ra11dom and systentatic. It is pointless to usc a ninthdegree polynomial to reproduce the
dahl ex~1ctl y. si nce it thel'efore abo repoduces the inherent errors. \VIuu we wanl is a ~im 1>le
func tion that best fits the freud of the data. To d isco\'er this rrcud. we plot the data o f Table 12.3
as points (x , y) in Figure 12.66.
Findin: the beslfitling line 10 data po:i1u:-. in Table 12.3

25

20
15

10

.I
2

10

We arc immediately impressed by the facr that although the points do not all lie on a straight
line, to describe them by a straight line would be a simple representation. and reasonably
accurate. We ~1Crcforc look for a linear ftu~"tion

y = y(x ) = ax

+b

t 12 691

to describe ~lC data in Table 12.3. Many lines could be drawn to fit the points in Figure 12.66
rea.._<onably accurately. and each line would be chamcterized by dillerent \'alues of a and b.
Our problem then is to find that line (or those values of a and b ) tho11 best fits the points.
Mathematicians llll\ C developed a method called least squares to arrive at a best fit.
We denote the x-v-<tlues in Table 12.3 by X ; = i , i = 1. .. , 10, and corresponding
observed \'a lues of y by 1 The linear function y (x ) =ax + b predicts \'alues)'; = y(x;)
ax; +b at the x;. Differences bet ween observed and predicted values of y are

)". - Y;

= (ax; +b) -

Y;

We defi ne a quantity S as the sum o f the squares of these differences fo r all.r;:


10

S =

2) y; - y1)

10

I:<ax; + b -

yf

(12.70)

i l

Given any a and b. S is a measure of the degree to which the points vary from that line - the
bette r the fit, the smaller the value of S. Ute method of least squares states that one way to

approximate [fle points in Figure 12.66 by a Straight line is to choose that line wh ich minimi7.e.s
the functionS= S(a , b). ln other words, find values of a and b tha t minimize S(a, b). For
critical poills of this fu nction we solve
tO

L 2r;(ax; + b- y

1),

i= l

as
0 =- iJb

10

2(ax;

+b -

)'1) .

i- 1

If we rewrite these equations in the form

(tA) a+ (t x;)
1= 1

10

= L XiY; .

1=1

{12.7 1a)

1=1

10
)
m
?;
x;
a
+
J
Ob=
?;
)';,
(

(1 2.7 1b)

we have a pair of linear c.quations in a and b. \Vith the data in Table 1 2.3~ we obtain

+ 55b = 1139.27,
55a + lOb = L70.34 ,
= 2.45 and b = 3.54. Since there is only one cri1ical point, and we
385a

which have solutions a


know that S(a. b) must have an absolute minimum for some a and
value~ musl mini mi1~ S(o , IJ) . The ~troight line
y = y(x) = 2.45x

b, it fo llows that these

+ 3.54

is therefore the best straight-line fi t (in the lcastsquarcs sense.) to I he data in Thble 12 .3. This is.

Application Prev iew


Revis ited

in fact, the line in Figure 12.66. II is important to remember that least squares assumes that we
have a prior knowh!dge of Lht Lypt: of function to be t.lelcrmined (i n this case: a l inc:-dr function).
and the method then proceeds to find the best such func tion.
Polynomials or higher order (quadratics. cubics. e tcete ra.) can be fi ue<.l to data poims by
least-squares (see Exercises 4- 7). Other types of functions can a lso be used, often by reduci ng
the problem to a straight-line :situation. ln the Application Pre.view we (Xlsed the problem of
llnding the tlnction that represents the tabulnted ''alues below, or the e quation of a curve that
approximates the points p lotted in Figure I 2.67a.

1"'SfKII

54.3

61.2

6 1.82
49.5

FIGURF 12 67<t

poillll)

for

pf <=S::W't-

72.4

88.7

118.6

194.0

37.6

28.4

19.2

10. 1

o~na

FIGURE 12 67b

and volume

Ptot of

lugarithm:o. of pre.bw't and volwue

f'
4

60

40

20

50

tOO

t50

200

4.5

1:!.13 Lensr SquJteJ

883

Thermodynamics sugges1s 1ha1 when 1he compression and expansion of1he gas is adiabatic,
pressure P and volume V are relmed by an cquaLion of1hc form P = b/ vn for some cons1ants
a and b. A direcl applicmion of1hc alxwc procedure leads ro complica1cd nonlincar cqumions
foHI and b. Instead. we take logarithms of1he equa1ioo P == b/ Va, and wri1e
fn P == in b - a In V.
lf we defi ne ne1v l'llriabies. p

= In P. B = Inb, v = In V, and A = - a. I hen


p

= Av +B.
=

This is the equalion of d s1raigh1 line in !he vp-piane. We han~ 1abuiared p


In P anJ
v "' In V below. and ploned p ag~ins1 v in Figure 12.67b. The fact 1ha1 1he poims seem
reasonably collinear is continuation of UlC facti hn11hc da1apoims iu Figure 12.67a arc adequu1ciy
described by a function of !he focm P b/ V0

t'-

lr I

3.998

4.12-1

-1.282

4.485

4.n6

5.268

Jl

In P

4.1 14

3.902

3.627

3.346

2.955

2.313

Filling rhe suaighrline p = 8 + A v 10 rhe dara in Tdble 12.51cads 10 rhe follo"ing equarions
8 corresponding 10 equnrions 12.7 1:

for A and

121.9758A

+ 26.92958

26.9295A

= 89.3605.

+ 68 =

20.2570,

the solurion of which is A = - 1.4043 and B = 9.6788. Coc~Se<Jucnll). lea>t->qWies c>linrates


for A and 8 give

P = - 1.4043u + 9.6i88

In P = 9.6788 - J.4043 1n 1'.

\Vhcu we: exponential~,

~nd this runctior.l approximates the ililla in Table 12.4. or lhe poinls in Figure 12.67a. It is
om~nl I~ notoce thai we did not apply rhe method of leasr squares directly ro rhe funcrion
P - _b I V and r~e data on Tnble 12.4. Were we 10 do >0. ir would be ,cry dinicuh ro >OI\'c
the resul.ruog equuuons fol' a und b. Try ir! Were we successful in doing so, values of a nnd b
would doffer from rhose above, but nor significantly.
Rcprc><:nration of data by other types of functions i> discussed in the exc:rci-.cs.

.,..

CI\,J)(tl' 12 Diffe'tnlitll Ctl l~'ttiUS of ~lulli\'a.riable r"\lllCiions

EXERCISES 12.13

l. The following table shows d\C ages M ( i11 months) mld the m-eragc test scon:s S (obl3incd on a suuxJurd i.ntdligcnc."C la.it) for children between
the ngcs of 9 ald 12 ye~;~.I'S .
A ge M (monlh!i.)

Find lcasi-S<(IH'I res cstitmucs for H li near function to descri beS= S(M).

2. The table below shows the a'cra,g,c ~y-:.tolic blood prcs$urc P of 13 chkJrcn.

Usc least S<tuarcs 10 fiml the cqm.uion ol' a straight line fiHill these datu.
3. Tbe PIOOll<:l iOII or sceel in the Unilcd Smtcs ror the ycars 1946-1956 is shown in the table below.

Find Ihe lcastsquarcs csti mate for u !incur function S


194<t. Ploc the data and line.

(a)

~o t lhc

= n1 + b to describe the dtlla by (u) tnking t = 0 in ycur zero and (b) wk_ing 1 = 0 in ycur

16 points in the lbllowing 1ahlc.

Du they scctn tu follct\...- a parabolic p~uh':t

y = tJX1 + bx + c il- thc equation or a l>arabola that i s lo apJU"O>:imatc lhc functic.)n ) ' = /(X) dcscribccl by these points.. then the
following sum orditrercnccs between obscrYed und predicted \'alucs is a measure of the tlCCuracy of the lit:

(b) I f

16

= S(a, b. c) = L (ax?+ bx1 + c -

y,)' .

where (..l';t J1) arc the poi nt~ in the tabk. To ftnl.llhc lx..">4 i))SSible fit i n the l ea~l Mtuares sense. we choose a .lJ, ami c to rHi11imizc S .
Show thnt S has only one critict~ lpoint (a. b.<.:) that is deli ned by ihc linear cqumicms

(c) Sol"c these cqmui(ltls f01 a, b , <tnd c.

il.

5.

(a) Fila

lca~r.sq uares

quadralic (as in Exercise 4) Lo the followi ng duw.

(b) Calctdatc the value of S al the t'fitkal vaJucs o f (~. b . and c.

i*

6. T he fo llowing lablc:: relmcs head H and discharge

Q (rom a pump.

Find lh c bcs1-fiuing pan::bola H

= a + b Q2 for 1hc d!tl!l.

11. 1)

ii!.

US

(a) l'it n k:acoH>quarcs cubtc: polynomial y = tu 3 + h:c2 + ex + tl to dat<~ ' " txcreisc 5.
(b) Ca lc:ul:ttc the v;,~luc of S at the critic.';ll values or a, b. c. and tl. H (\W d oes it <:nn)f>arC with lhat in ExCt<:.iliC 5(b)?

7.

il + 8.

l..(':i.':il Sq,r.ues

lA;$ squurcs ~:an

0..5

also be used with cxpc:mcnliul fw'X.'tions. Suppose. for cxwnple., that we wish to tit "' c:ur.'C to the data in the rollowing table:

1.0
ISO

140

1.5
230

2.5

2.0

3[>5

(a) To detcnninc whether Ihe d: tac;m be dCSt:ribcd by a ru n~lion of the fonn )'(X) :;;;: QeU~. we l~lkc le>a:aritlun:tun buth sitJ~ o l'lhe ~\U.tlion.
In y = In b + ox . JJwc OCfinc Y = In)' mxl 8
II\ b . IJ'lCtl Y = ox + B . and this is the equation of a stmigltt IiDe in the.( Y -pltmc.
rn other words. to test whether u set o f points eM be dcs.c:tibed b)' an cxponcntlul bt4 \ , we p10t )' = In y tlg.ai n ~t .t. and if these poilll.S
Cfttl be: ;:.ppro:<tnlO!ed b)' a :)tmight Iitle. then 1hc ori&iwl thila should be (ICS(rib:ablc by on ex llOnctuial func tion. Do this fot the d3l<ltl\

lhc lablc.

{h) find lcast-$tru.lrc" cs 1 i rnu1<:~ fot a and 8 . und hence find chc bcst-lluing CXIXM'ICntiul f<,r the oris ina1 daw.

9. In the Slttdy of'lc:mgshmc sand Lr~ on teaches , the following datn were record ed (tt the E.l Mmcnn lk'dl.il on the B~lja ill catifolllia. Longshore
energy tlux F is in uniL~ o l' jtJUies per meiJ'C per second. 3nd inl..mtn;cd weight U".lMSJ)Otl. \V is in units o f newtons: per second.

6.0

15

6. 1

9 .9

18
20.8

20

.lO

14.6

25.7

Show 1hat 1V can be rt pi'C!;entcd in the form W

= a Fh nnd fi nd least-squares estimates foro and b. Plot d ata poinl~> and the least.squares funttion

to show the adequacy ol'thc Ill.


10. 'l"hc follc)\"ing tublc: stxrws Lhc nwubcr of kilorncttes
~hi de

80

'iO

lOll

1'rud : I

2.23

2.20
2. 18
2.1 6
2. 13
1.9 1
1.96

2.05
2.00

Truck 2
Trud3
Truck 4
Trud 5
1iuck 6

2.35
2.37

2.21
1.95

2. 16

Usc thi:s inlbnu::.tion to solve

it

pet' liltt

I'CCOtdcd by six trucks m sp..-cd~ or80, 90, and 100 km.lh.

1.97

2.10
I .SO

1.92

Excrci~ $9

in Section 4.7. Ul:iC

w=

$20 und fJ

II. 'J"hc following, tublc n::prcscru.s o.:.nsu.s 11gure\ for Lhc populatiOt\ (in milli(lf~S) for the Unitcll SHtlei (t'l)lll 1790 to 19 10.

Show Lhlll thcse drua C."\!\ be reprcscn1cd b)' an CXiXlt\001itll fw\Ciion and find

ii ...

= $0.60.

12.

i1 ~ lcanSQURI'\!S es~ i mn tcs .

{u) 1'o lit a pow<:1' funt,.1ion .r


h.t'f lo dalu, we lake logauithms, oh1ainmg In y
hl b + a In x. When wt: set Y = In)' , 8
h~ b. and
X ; In.\'. thiii cq u31ion boc<ltllcS Y = 8 + a X. dtat for 11 straigtll line. 'tl1is lluggcst.s that we plot Y aga.i 11~t X to sec if d:th\ poi1llS
(X. Y) .m: rca~o n<~ hly collitoc:;.tr. Do su fnr the dula in the fnlluwing table.

(b) f'inrJ least-square~ cstiiiHitcs ror a and b b)' lining u straiBhl line

w Y a.~~ IUnctiun of X .

rhc follow in~ t.Wic n:prc)CIIlS the lcng_


t h o f timet (in second_)) th~t u.n uthh.:tc io:ould hold :i 1<*1 F (itl lli..'WIOI'b} in Ihe posi tion ~tlOYI'tl in th.:
figure below. how that/ can be rcprtscntcd in the form 1 =a F b. 3RII flllll lea~Csqua-cs cstimalt:iO for a and b.

ii + 13.

100
288

200
84

300

400

52

32

i.

14. The uurnbcr N ofbJctcria pc.r un it \'Qiumc in a c ulture a.Oer t houn i~ tabulated below.

(a) Plot t he data POints (t. In N) to s how that it is rcasom1ble to fi1 an equation or tbe fonn N
(b) Find lco:stsquurcs estimates fOe' a 1100 b.

= be'" 10 the data.

15. The folluwing tublc ghcs cNpcrimcotal valu<:.s of the pn;:ssun: P of 11. gi\'<:n ma..~ of gus t orrcsponding to vurious \'O)umes V. Usc lcust squarc.s
to 11nd c~timate.-. for oons.tams a and IJ il thcrmodynatHics s.ug~ an \X.IU~tion of the fonn P V"' = b to describe tin: data.

16.

(a) You arc given 11 pairs or ob:tcrvation~ {x1. )=';) aru.1 ru-e n:quircd 10 fi t a curve of lite for m y

lc:a;;.t

sct uar~.

= bj x 1 to them. By direct application or

find a romwla ror b .

(b) Apply the fO<mulu in part (a) to thedalll below.

(c) Plot dM: data points and lc--J...,lsquare..;; curve.

il.

17.

(a) tit the cun c y

= ln(IJ + a;r)

to the t'ollowing dala by com"t:n ing ttic ghcn relal:ion to linear rorm
~'=ax + b

umJ using

u.s dutu e'f; und .t1.

(hJ Can you ttSe lensc squares din."CII)' on the IC'Igarithm runction? E), plain.

il

IS. lkv.se a method for using least squares to obt~in a cui"\'C or the fonn
)'~

(JX

+b

to rcpl'':~cnt th c following c:hata.

8
1.197

il+

1. 11 8

19. A Cobb-Douglas production funct ion has the form P (.t , y)


laqy 1 " where P is the n1mbcr of items produa..-d per unit time .\' i.s the
number or employees. and y is the operating budget for thai time. The numbcl'$ k > 0 and 0 < q < I ore constunls. Fi_nd lctUtsquan:.s cstimutt$
I'Of' k and q &ivcn lhc followin& production daaa.

Workcn;, \'
Bud&cl. ,)' (dolldi' J
Production. P
/lim: Write P/.r

100
10000
800

110
9000
810

90

9000
720

100
12000
860

95
11000
810

105
9~00

800

110
10000
850

= k(xfy)(1 UJld tu._~o: logaritlnns,.

112.14 Differentials
lf y = f (x ) is a function of one variable, the differential of y. detined by dy = f' (x ) dx.
is an approx imation to the increment tl.y = .f (x + dx) - .f(x) for small d x . In pan.icular.
dy is the change in y l'QtTesponding to the change dx in x if we follow the tangent line to Lhe
curve at (x , y ) instead of the curve itself.

12.1 4 Difterentials

8S7

We wke the same approach in defining differentials for muhi,ariable functions. First consider a function f (x, y) of two independent variables that can be represented geometrically as
a surface with equation z = f(x, y) (Figure 12.68). If we change the values of x andy by
amounts t:.x = dx and t:.y = dy. then the corresponding change in z is

6.z = f(x

+ dx, y + d y) -

j(x, y).

Geomerrical1)'. this is the difference in the heighL~ of the surface at the poilm (x +dx, y + d y)
a nd (x, y) . If we d raw the tangent p la ne to the surt:1ce at (x, y), then very near (x , y) the
height of the tangent plane approximates the height of the surface (Figure 12.69). In particular,
the height of the tangent plane at (x + dx, y + dy) for small dx and dy approximates the
height of the surfcx;e. We define the d~f!erellfial d :. as the change in .: correspondin_g to the

changes dx and dy in x and y if we follow the tangent plane at (x, y) instead of the surface
itself. To fi nd dz in terms o f dx and dy . we note that the vector join ing the points (x, y, :) and
(x + dx, y + dy, z + d z) has componenL~ (dx, dy, dz), and this vector lies in the tangent
plane. Since a nom1al vector to the tangent plane is
'il(z- j(x , y)) = (-fx,- /y. 1),
it follows that the vectors(qucndy,

f x.- j 1 ,

I) anti (dx, d y, d:) must be perpend icular. CoJt:;c-

iJf

aJ

o = (-f"- J, I) - (dx, dy , d z) = - -a.\" dx- -ay dy + d z,


and hence
d: =

<Jf dx

ax

+ <Jf dy .

uy

(1 2.72)

Note tha t if y is held constant in the fu nction f(.t, y), then dy = 0 and 12.72 for d z reduces
to the defini tion of the d ift'ercntial of a function of ooe variable.
M;tltlii;JiJ'fKffi'W at i<> e:<acl difference in heights of o;urface at (:r, y) and at
(,< +d.<, )'+ dy)

Uji#JII;I4fi%U dz i" difference


in heigh1:i or snrfuce ;n (.l. y' and t;mgelll
plane at (x+dx.y+dy)

t
z = ft x .y)

(x,y),

; ;
, . ' dy

dx~

~ + dx, y+ dy)

In Section 4. 12 we indicated that we must be careful in using the differential dy as an


approximation for the change 6.y in a fu nction j(x) . Fo r the same reasons, we must be
judicious in our usc of dz as an approximation for t:..z... Indeed~ we haYc stated that dz is an
approximation for t:.z for small dx and dy. but the difliculty is deciding how small is small and
how good is the a pprox imation. In addition. note that if (x, y) is a c ritical poi.n t of the fu nction
f (x, y ), then either dz = () for all dx and d y, or d z is undefined. In other words, d z cannot
be used to approximate 6.z at a critical point

au

Oupter 12 Difft!:lmti:tl C dcul u~ uf Mlllliv:trinble FuncliOCH

I EXAMPLE

12.38

If the rod ius of a rightCi l"cu"ucone is chunged f rom 10 em to 10.1 em and the height is changed
from I m to 0.99 m, usc differcntiHis to approxim at ~ the change in its volume.

SOLU110N The volume of a cone of radius r and hei!!,h l It is &i ven by the f ormula V =

tu 2 h f 3. Tbe differential ofthis function is


dV

-;;-dr
ur

av
+alt

dh

2
- rrrh d r

+ - nrdh.
3

= IOO . anddlt =-I. then

lfr = lO. d r =0.1, /t

dV =

av

JT(JO)(I OO)(O. l )

I0011'

JT( IO)(-I) = - - em .

Equation 12.72 suggests the followi ng definition for the ditlCrential of a fun(.~li on of more than

two inde pe ndent variables.

DEFINITION 12. 1 2

II' a = j (x, y. z, 1, . . . , w) . lhen the diffe rt nlial of j (x , y. z, I , .... w) is defined as

111

d u = - dx

a.r

I EXAMPLE

ar
ar
+ -of dy + ilf
- d z + - dt + + - dw .

ay

az

at

( 12 7.\ t

aw

12.39

ljiCJII;J#ji[EJIB!]
Ufing
difl~-t :ti;al s cu :li)J)n>ximale <'.hoi~'#
in a~ uf 11 lriQnJ,IC

The area of the triangle in Figure 12.70 is given by the formula A


(!

= 11' / 3, a and b arc changed by J% and 8 by

percentage cluUlge in A.

ab sin IJ. If when

2%. usc diO'erentials to find the app1-oximatc

SOLU110 N Since

aA

aA
aA
db+ - d8
ab
ao

d A = -da + -

aa

2b sin e da + 2a

+ 2"b cos e dfJ,

sine db

the approximate pcrccmagc change in A is

(dAA ) =

100 -

-100 ( -1IJ sin fJ da + -I asi n fJ d b.o.. -I abcosfJ t/8 )


A 2
2
2

= 100 ( -da + -dh


{/.
h
1
3

+ cot !JdO)

1
2

Si nce a and b are changed by - % and IJ by - %,

ooC:) JOo(d:) = ~and ux{d:)


=

I 2.14 Differtmiols

Thus.

0
roo(dAA) - 3-+-+-COlO=
- +-c.otO
3 2
3 2

and when 0 =

Tr / 3.

roo(~)
= ~3 + 2~(~)(..!...)=
0.97 .
A
3
J3

.....

The approximate percentage change in A is therefore I'k.

When 11 rcsi>tance, R; (i

tance is

= 1, ... , II) are connected in series. then 1he re;uliant resis-

When they are connected in parallel, the ell'ecli~c resi5tance, R. is gl\en by


I

+ -R
R1

- = R

-1' ...

+ -.
R.

We are being asked the following question. If each resistance R, is changed by the same
mall percentage c, what is the percentage change in the scrie< nnd parvllel combinMi ons?

SOLL no~ Let U> consider the >eric:. c.ISC fio;t. Bee aU><; pen:cmagc change> in ihe R,
are small. we hall uo;e differenrial< 10 calculate the pereemage change in R.
iJ R
ii R
d R = - d R1 + .. + -dR. = dR1 + ...
8 R,
8 R.,

+ d R.

The percentage change in R is therefore

dR

100-R

IOO(d R 1 +

R1

.. +dR'n)

+ ... + R

.. --r--,I,_OOd_R_ _--:--.R,

( + -R,R2+ .. + -RR)

+ -....,....,,.-_I_Otld_R_:l:.._--:,........
R,

+ ... _

(Rr- +
R2

R._, )

R
-+"+-+I
R.
R.
1

Since the perccntagcchnngc ineach resistance Rt is c. it follows that

R!

- "7::::--1OO
~d_R.::",---<"'

R. (

IOOdR

R)

+ .. + -

= --...........:..--,,
+ ....--R,
R.
R,

...
R.
-R +1+ .. +R,

1+ ~ + + R,
R1

+ ... +

c
1

''r,&

-+
.. + - R.- + 1
Rn

= c. Hence,

889

890

Chaptu 12 lliffereooi3) Cakls(I(Multiv:uioble f'w1<tions

c R,

cR,

cR11

- -::-----'-----:::- + -=---=----::- + .. . +
R, + + R
R1 + + R
R1 + + R.
11

For the parallel case, we write that R = ( R1


dR = -

= c.

11

+ + -R1

)-t,

and take differentials,

11

(-1R + .. . + _RI ) (-_R'I d R, - . . - _IR2d R )


-2

I
I
)
= R2 ( -dR
R2 1 + + -R2dRII

II

The percentase change in R is therefore

dR

I OO -

"

- IOOR (R2
-dR 1 + + -R2dR)

_ !!... ( 100dR,) + .. . +!!... ( 100 dR)


R1

Rn

R1

cR

cR

R11

-R + +
1

R.,

= cR ( -R + + -R. ) = c.
1

Thus. whether reshtances are comtcctcd in series or parallel. small equal percentage
changes in the individual resiMances results in the 'amepercemage change in the resultam
resi:)lance.
EXERCISES 12. 14

In Excrciscs 1- LO finc.llhc diJlCn:.ntial or the funcaion.

8. /(.< . )'. l . I )= .l )" + )"t + <f +XI

I. f(x,y) =x'y-siny

9. f(x,y,t.w) = xy oun(zw)

2. f(.T. y) =

10. / (.< . .r. z. 1)

Tan- (xy)

J . f<x, y . <) = xyz - x 3 .-'

= r'>' "' '

II. A riglu-cir.cu1ar cone

ha.~

radius 10 em and hcig hl 2() em. If its

..-adius in<:'ea'Ses by 0.1 em ~ nd its height deci'Ca.ses by 0.3 em. use


4. f(,t . y. z) = sin (.<g) - .<2y2z1

differentials to rind lhc ctpproximme change in its volume. Compare


this with the aaual change in \.Oiume.

o. f(x, y, z) =In (.<2 + >:z + ')

12. When the e llipse b 1 x 1

6. /(x. y) - Sin- (.<.r)

7. f(x , y) =Sin-' (X+ y ) + c .,- (.t

+ )')

+ a-:y 2 =

a 1b 1 is r"O(ated: about the x-

uxis~ the volwne V of the S-pheroid is 4!T ab2 j 3. if a o.nd b ~tach


im.:-rcu.sod by I,~~ usc different ial~ Lo fi nd the upvrox.iu1atc p<:TCCnta!c

change in V .

112.15 Taylor Series for Multhariable Functions


Taylor series fo r functions of one vari~lbl e c:m be used to gcneraLC ~l"aylor series for multivarinble
fuoc1 i.o ns. For simplicity. we once aga.in work w ith func1i.o ns of two iodependent variables.
Ex 1.e nsions to functions of more th ao two iod!ependent variables will be c lear. Suppose that a
runction .f(x, y) has continuous pan ial derivatives o r all orders in w me open c ircle t-entred at
the point (c, d) (Figure 12.71).

12.1j

Ta)'kr' ~ '~'' Muflhariabte 1-~al'l(.uon s

891

Finding the Ta)'lor series of a ronct1on /(x. y) tbool a poinl (C'. (/)

FIGUAE 12.71

y
~"

'

...

/ ~

(r,d)

~..

.......~
X

Parametric equations for the line through (c, tl) in direcrion v = (vx, vy) are

x=c+v,t ,

y= d+ v, t .

If we substitute these valu~ into f (x , y). we obtain a function F(t ) of one variable,

F(t ) = f(c

+ v,t, d + v, t ),

which represents the \'alue of f(.t, y) ol points nlong the line through (c. d) in direction v. If
we expand lhis function into ilS Maclaurin series. Wl.: obtain

F(t)
'll~e schematic diagmm to

F (O)

F'(O)t

F "(O)

+ - -t 2 + ,...

( 12.74)

2!

the left gives

/ '\.._y
I
I
I
I

,
a Fdx
F (I) = -a -d

a Fd y
-d = f,(x. y)u, - J,<x . y)u,.

+ -aJ

and therefore
F '(O)

= f,(c. d)v, + J1 (c,d)v1 .

'The schematic for F'(t ) gives tbc second derivative of F(t ),


F'(l)

/ '\.,y

F, (t) = -a I F'
(1 )] -dx + a
-IF,(r)J -dy

ax

ay

dt

= ilx lfx(X,)')Vx

= ,r.. (x, y)v;?

dt

+ .f,(x. y)Vy]Vx + aylfr(X. J)Vx + f 1(x,y)v,.[v>

+ 2 1r, 1 (x, )') v, v1 + J, (x , y)v;'.

and therefore

F" (O) = J,.(c , d)v;

+ 2f, 1 (c ,d)v,v, + f(c,d)v; .

1\ similar calculation gives

F"' (0) = fxu(C. d)v;

+ 3/xry(c. d )v;vy + 3 /xyy(c, d)v,v~ + f ,._.,,(c, d)v~ .

and the p.aucm is emerging. \Vhe.n the~ resuhs are subslilulet.l into 12.74.

F (t)

= f(c- u,,t , d + u,r)


= f(c, d)

+ [j, (c, d)v,, + f 1 (c. d)v,.Jt


?

+ Lfxx<c, d )u.; + 2/xy(c, d)v., Vy + .{yy(c, d)v~J ~~ + .

(12.75)

892

Chaplet 12 Diffel'ential Calculus of Multi"ru;able Func1ions

We 11. ow let ,. be the vector from (c. d) to point (x . y). so that Vx = x - c and Vy = y - d.
andatthesametimesetl = I. Then F( l) = f(c +x- c,d + y - d)= f(x,y), and
12.75 becomes
1

f(x,y) = f(c,d) + [f., (c,d)(x - c)+ f 1 (c.d)(y- d)]+ Z!lf (c,d)(x - c)

+ 2/,y(c, d)(x - c)(y - d) + /yy (c. d)(y - d) 2 ] + .. . .

( 12.76)

This is the Taylor series for j(x, y) about the point (c,d). It gives the value or the
function at the point (x, y) in Lenns of values of the function and it.s derivatives at the point
(c, d) .

I EXAMPLE

12.40

Find the first six nonzero terms in the Taylor series for f(x, y) = sin(2x + 3y) about (0, 0).
SOLUTION

We calculate that
j(O, 0)

.ft<O, 0)

= 0,

= 2 cos(2x + Jy)I(O.O) = 2,

/y(O, 0) = 3 cos(2x + 3y)](o.o1 = 3,


f xx (O, 0)

- 4sin(2x + 3)')](0.0)

0,

f,y(O, 0) = - 6 sin(2x + 3y)J(O.O/ = 0,


[vy(O, 0} = -9 sin(2x + 3y)](O.O) = 0.
f xxx (O, 0)

-8cos(2x + 3y)](O.O)

- 8,

fvxy(O, 0} = - 12cos(2x + 3y}J(O.O/ = -12,


fvyy(O,O} = -18cos(2x +3y)](O.O) = - 18.

Im (0, 0} =

-27 cos(2x + 3y)J(O.O/

-2.7.

Formula 12.761hen gives

sin(2r + 3y)

0 + 12< + 3yl +

= (2x + 3y) -

~! 101 +

~(2x

3!

;!l - 8x 3

- 36x

y - 54.r/- 27il +

+ 3y) 3 + . ..

This series could also have been obtained by substituting 2\' + 3y for x in the Maclaurin series
for sin x. This is not always an alternati ve.

Summary

8$3

EXERCISES 12.15

I. If f(x . .r) = F(x) G(y), islheTuylorscrics of f(x,y) about


(0, 0) the product of the Madurin series for F(x) and G(y)"l

10. Jl

+ X)"

about (2, I)

II. t" sin (Jx - y) aboo1 ( -1. 0)

1. What arc lhe cubic 1emts in 12.76?

12. (.r + y)' In (.r + .1') :\00111 (0. I)


ln Exercise~ 3-8 lind the Ta)Jor series of the function about the point
by usi11g Tuylor :.cries foc fw!Clions of one vo~iablc.

IJ. 1an- (3.< + 2y) aboul (I. -I)

3. CO~ (X)') Obolll (0. 0)

14..r ' .1 10 aboul (0. 0)

5.

7.

x 2 yJT+X aboul (0. 0)


I

l + .r+y

:1001.11 ( 3. -4)

4. , ,. ,, :lh(l<ll ( 1. - I )

6. In (I+

.r' + y 2) aboul (0. 0)

.\'\'~
8. - - 2 :tbo(- 1.0)

IS. \Vh!!.l :are the lCMI\S in the "lilylor series for!): function

I+ y

t.

16. Express 12.76 in sigma notation. 1/im: Think about an opcrntor.

l_n E>itrc-i:;es 9-14 rind the T02ylor ::.cries of the. rurtclion up to ami in

[ (.\

eluding quudrmic tcrntS-.


X

9,

X)"

.( + )"

, about (- 1. I)

j(x, )'. z)

aboullhe poinl (c. d, ~) corTCs pondi_og to those ln C<t\l::t.lion 12.76','

a ]"

a + (y - b) - a)-

ax

ay

whach is: expanded as a binomial to ~r.tte on funct ion~;

f(x, y ) .

SUMMARY

\Ve began Lhe sludy of mulliv.lriable func1ions in this chapter. concenrr.uing our :mention o n
d iffere ntiation and its appl ications. \Ve introduced 1wo rpes of derivatives fo r a multivnri
able function: pania l derivatives and directional derivatives. Partial dcrivatiYcs arc directiona l
deriv~tives in directions parallel to the coordinate axes. The directional derivati1e o r a fu nc tion
J(x, y. z) in the direction ' ' is given by the fom1ub D.J = \1 f \, where\ is the unit vect or
in the direction of'" and the gradient \1 f is evaluated at the point m which Dv f is required.
This rormula leads to the f;tet lhatlhe gradient \1 f(x, )'. l) poims in the direttion in which
J(x, y. Z) increases most rapidly, and l'i7 fl is the (maximum) rate of change of /(.r, y, z).
A :second property of grndicnt vectors (that is related to the first) is that if F (.t . y , ~) = C, C a
constant. is the equation o r a surfatt. then at any point on the surface 'i7 F is perpendicular to the
surface. 1'his propcny, along \,.;th the fac t that perpendicularity to a surface is synonymous w ith
perpendic-ularity 10 its tangent plane. cnabks us to find equations fo r tangent planes to surfaces

::.nd Ulngent lines lO curves.


We illustr-..tted various ways to calcuhuc partial dcri,atives of a multivariable functio n,
depending on whether the function is defined explicitly. inlJ>Iicilly. or a:s a composite fu nc t.ion.
Since partia l derivatives are ordinary deri\'atives wilh other v.ariables held constanl, there is no
difficulty calculating partial derivatives when the funclion is defilcd explicitly; we sintply usc
!he rules from singlc-<triablc calculus. W hen the panial derivative of a composite function
is required, we use a schematic diagram illustmti ng func tional dependences to develop the
appropriate chain rule. Partial derivatives for functions defined implicitly are calculated using
Jacobians.
Critical points of a multi,ariablc function arc points a t which all of its first partial derivatives
\fanish or at which one or more of these partial derivalives does nol exist Critical poinls can
y1e.l d relalive maxima, relative min.i ma. saddle points, or none of these. For func1ions of two
indcpc.nc.lcnt variublc~. a :5(.-conddcrivntiYC t~t cxiMs that may determine whether a t:ritical point
at which the panial deri,atives vanish yields a relati,e maximum, a relat.i\e n1inin111m, or H~addle
point. Thi$ test i~ analogous lO thai for functions one \lariablc.
A continuous function of two independent \'ariables defined on :.t region that includes it<:
boundary a lwuys takes on a maximum value and a minimum ,aJuc. To find these values we
evaluate the function ateach of iLScril ical points and compare these numbers to the maxjmum and
minimum values of the function on iLS boundary. Fi.n ding: the extreme values on the boundary
involves o ne or more extrema problems for a func tion of one variable, the number of such
problems depending on the cornplexiry of the boundary.

or

11lc:rc are two tne1hods for finding extreme value< of a function when the variables of
the fwx.'tJon arc wbject to '-'Ut\\trainL': solve the constmint cquatil>ruro rnr dependent \uri abies
and cxpren the &"en fu<etion on t erms o f indepen<knt 'anablc. or ~~>e La~rangc multipliers.
la,grongc rnuhiplu:n. climina1c tht necessity for s.ohhlg th.! con"raint cqwuions. but they do.,
on the a her hand. iive a lorser system of equations to sohe for critical points
D iO"erenti,l l~ o f multivdriabk- fuoc"tions can be used to upproximatc c hnnges in funcrion\
"hc:n 'mot~ll c hungts are made to its itldepe,dcnt varitb lcs. Tuylor <erlc~ c..u1 a lso he u!\cd to

approxinuuc muhivarinblc function~.


The method of le<~<l squares fi t> a func tion y = .f(x) o l kno wn form to a set of data. It
minimizes the ~u no of the .1quares of the differences between measured alld predicted values
of y .

K EY TI!RMS

In revic\\ ing thi ch,tpcer. you should be able to define or di<eu.<.' the folio" ing kc) terms:

Limi1

Domain
Continuuu. fur>e11<1n
Gradient
Laplace's equation
Olain ru le
Euler's 11lc:orcm
Jacobian deaenuinant

Pllnial derhatl\c
Second partial derivati\c

Harmonic function
Pusili\'cly homogcntou< function

Implicit diffcrcntlatioo
Directional tkrivaahc

Tangem li 1le to a curve


Normals to s urfnc~

TalSCH pi "''e 10 Hsurface


Sm()()(h surfm;c

Piccewise-srnoo1h surf:1ce

Criticotl poirn

Relative muimun1
Saddle 1><>
Abwlutc minimum

Relati"e mini rnun1


Absolute ma'<imum
Constmint pro~k:rll>
Lagnmg,ian

Lagrange muhipliers
Leas1 squares

Differe ntial

Taylo< .etl<>

R EVIEW
EX ERCI SES

12. arjox), if X:: r <:<,.IJ. y:: r <onU


I.

ilflilx if f(x. ,1') ~ 'I>'- Sin

2. 112 fl8y 1

3.

if /(x.

l ' . l) - In (x 2

iJ1 fli1x 1 iJy if /(.r.

y. t . r) -

4. iJz/OA i f t 2' 1 Tun

s. Buja_v il" l4 co-: -'


6.

(.ty)

+ )'' +

x 3e>'-

xzt 2 -

+y = 'x
+ .\ <:os (.ru) + :z =
1

13. aDjax) ,, il' .r

r. 1)
sin (x + y

+ t + 1)

Sx

9.

X)

= 5, 3!1 -

2V

+ x 211 :

II. ar./at )., if :Z


II COS(Ill ), tt

= xy- x'- y'. x = tr'. y s , .-


.t 2 - y 2, X = 2u - 3u2 + 3uut,

I' ! - 21

15.

D1ujDrl)" il u -

.t 1 - y 1.r 1 1 -

+ -'

rt.'tto.O r

r cosO.

tmdx~ - y

20. IJujar), if u

= r cos4>

r !\in 8

,1' ~

3. l

+ ..u =

.ryr 1

= J. 2.\ :- .'x ty~ =

\Sin

(n). r

Vtll\ 1

21. If 11 = (x'

)'

inU. t

r sin 0
1/ l l

21 . ..l.\' = .Jt!

Ill. 1) 2:/a.r' if.rt-.<':'+ .1 2 = )

19. ti) Jdx ir y t - .t ll2 + '5\

2r 1

il'flhil t or /(u. > = '1./V- 1./U

10. df/tltof /(>. y)

at4 j ar )6 ir a = x l - ylxl, .r =

17. th/dl i f z = y- ' '


-tlnl

7. dyfdx or x=>'+3~'-~y4

+ 1 1 -

14-.

r.i~9

16. t/ 1ujdt: if u .tj;} - :,jx 1 X 11

dfjdrif/(x,y) = x'+y'-n.x= t'+3t, )

& auJ3z) if u 1

= f'>ill9 ro>U. y

+ y 2)1 1 +sin (.r /t)J. <hmnhat

a.. au au
x - + y - + z - = 211.
ax

8y

az

-lt

u'r'- 21. y

* 22. 2If u = 2.\' z - 2 3v


()

.r 1 -

() a

~x l

+ .xv. .show lh~l


'azu

+2.rv - - + v1 .

~x ~y

~y'

hl E~cn:i~c$ 37-40 fir'ld a11 crilical t>oinls (~ Lh~ funclion ~ux.l cl~"Sify
each u."$ yielding 1.1 rcl!uivc n\Uximvm. u rchuivc 1ninimtun, or a saddle
point

= 2u.

+ 23. II' l (s) is a dlffcrcntioblc function. show thot/ (3.< - 2)) S31isfics

at a[
2- +3- =
~x
ar

37. /(.<, y) = x'

0.

38.

= x 1 'in.'' :11 (3. -

26. f(x, .1', z) = .x' +


( 1,0, 1) tO (2, - 1,3)

I ) in Ihe dil>'clion ,,

)'' + z'

= 0 in 1hedircc1ion of increasing x

+y1 =

z1

43. Find ma>.illl ll!ll nntlmini11uun vttl u c~ of 1hc func1ion f(x, y. z)


+ 3y - 4z on 1hc .SJ>herc .r 2 + )'1 + r. 1 :S 2.

2.\'

.U. Find the poinL~ on the cutvt: x 2 + .t"


t'nrthc.sl rrom the origin.

31.

portion~aJ lo~~

=17 + I. )' =/

35.

+ .v +z =O. 2.1 - 3y - 6z = 11 ut( l,l. 2)

36.

ct.

47. lf lhc cqu:lli lln 11 =:; j(:< -IH) defines


t $h OW thai

I. t

=11 + 51

z = '." '' + y' = 2 "'( I. I. 1)

Ul

(2. 0. 6)

11

implicitly a~ :t (un-ction

Of X tlfld

all
dl

34. r

1 doscst to and

-16. CiCilCnalit.c 'Rc\'tcw Excrt:isc .'\2 in Otapu:r 4 to inc.:or-por:uc a Lhird


crop. say surlflowcrs. with n yield of,. dothtrS per hct:t.urc a1ld u pro-

x'- z' = y' at (- 1. 3. 2)

In Excrci$C$ 34- 36 find cqu._Jti()rlS fo r the wngcm line to the CUf\"<!,

+ y2 ;;;;

4-5. Find tiH: poi tu(s ) uclthc surfiu,:.c z:2 = I + XJ' c.;lo~:sL 10 1hc ori~i n.

+ I ( I . 0. 0)

' F +a'-F) + 12 F(x. Y) .


+ .1 2) (a
()x2
i)y l
.
=

-t

.1

.<'

In l!xcrci~cs 31-33 lilld thccqu:uion of th~ 1angcnt p!:ute to the ).urf~c..:.

x'

(x

,. 42. Find m:u:imum ~lnd minimum v.alucs or Ihe func;tion j'(x. )')
xy on the circle
+ y 2 :S I.

+ 30. J(x. y, z) = 2.<.\ 'Z- x 1 - ; 1 ru (0, I. I) along 11><: curve x 1 +


y 2 + z_'J ;; 2. )' ;; ~ in lhCdirlion of in,n;.~:>ing X

= .r 1 + y 1 111 (I , 3. 10)

lh:.lt

01 (I, 0. I) in lhc d1rcc1ion l'ronl

the di1cetion of dccrcttsing )'

33.

tr f( ,<. y) = (x' ~ y1)F(x. y) whore F (x. y) = x 1 f y - y 3fx.

-a'f
+a't
-=
f.lx l
ay.!

29. f(x. y) = In (x + .1) ru (3. 10) olong the curve y = x' + I in

3 1. z

41.

= (2. 4)

28. f(x.y , z) = x ' +.l'-2t ot ( l, - t, 2)>101lthcline.r- y + t


2x + 4)' + 2

-1!1.

= x2 - xy + y' + x - 4y
f(x. )') = (.r 2 + r' - I ) 2

~ocril'y

21. f(x.y,z)=zr;,,., 1 (x +y) i11(-1.2. S)nhcdII.'<:IIOt1J"'r


J>Cndiculu.r to the surf~: - x ! + )'! wi1h posili\'C :-contponcnl
4t

) 't:'

3~. /(.<. y)

In Exercises 25-30 lind the dircclion:.l dcriv.ativl!.


25. J(.r, y)

I <-. y> =

+ 3)"- 6x + 4

au = 0.
ax

T U-

4.~. P'ind tht llrs l ~i:< IH1n2:tro ICnUS in lhC Tay!OrScl'ics ro1X~ sin (.\'Z)')
nb0111thc point ( 1, ;r / 4) .

,. -4?. Find the best possible l ine, in lhc.lcast-squares scnM:. lO f11 1hc dahl
in the fo11owing tublc.

Iy

I .2

4,{)

8.4

12.2

15.6

19.7

23.0

26 9

CHAPTER

13

Application Preview

Multiple Integrals

In the figure below. a freshwater ma~":>h is drd.inell lo 1he ocean through an automatic title: gone
thai is L me1res wide and 0.9 m high. The gale is held b)" an L-melte-long hinge <\1 A and
bears on u sill a1 8. The wo1cr level in 1hc marsh is 1.8 m, and !he dens il\' of oc.c an waler
is 1030 kg/m3 wmparetl 1o 1000 kg/m 3 for the fll:l>h \\"dler in !he marsh. TI1e marsh and !he
ocean bolh crea1e forces on the ga~e, and when waler levels are !he same. 1he ocean crca1cs a
greater force, thus keeping !he gale closed. As I he ocean level falls. !he force i1 exerts o n the
gale decreases. and evemually the gale open>.

I
I

1.8

Marsh

A h Ocean

I
0.9

THE PROBI FM
AI whal dcplh of ocean wa1cr will the !ide ga1e open? (See Example 13.8
o n page 9 14 for !he solution.)
The definilc in1egrol of a funclion f(x) of one variable is defined as the limit of a sum of
1he form
/(x~) t.x,

+ /(x;) 6xz + + f<x:) t.x.

(13.1)

where the JOml or the 1>artition approaches tcro. We have :.cen 1ha1 dclinile imcgrals can be
used to calculate Mea, \'Oiumc, work, nuid rorce and moments. In spite of the fact that some of
the.'ie are two- and three-c.lim(nsionul concept~. we have been c.arerul to emphasi.te that a Uelinite
integral with respect to x is a'' imegm1ion along the .\ axis, and a definite integral with respect
to y is an imegmtion along, the y-axis. In other word:.. independent or how we interpret th~
result of the integration, a definite integral is a limit summation along n line. Generalizations
of these limiting sums to functions of two and three independent variables lead to defini tion~ of
double and 1riple in 1egr~ls .
1

113.1 Double Integrals and Double Iterated Integrals


Suppose a fu nclion f(x, y) is defined in some region R of !he xy-plane thai has fi nite area
(Figure 13.1). To define lhe double imegral of f<x. y) over R. we fir.;t di"i,le R imo n
subregions of areas ~A ., 8A 2 , .. ~D. A,., in any manner whatsOever. Jn each subregion
6A ; (i = I. ... . 11) we choose an arbi1rary poim (x7- y7) and form the sum

f(x~,y~) t.A ,

+ f<x;,y;) t.A2 + + f(x~,y:) e.A,.

"

= L

i=l

f<xr . .v7) 6A; .


( 13.2)

The norm of the partition of R into subareas 8A; is the area of the largest or the subareas,
deno1ed by li t. A; II = max;= I ....... t.A;.

896

13. 1 Oocble l nlc~rah .llhl 0 1)ubleltc:nuc:d lmqntls

R ino Snlalkr region.) hJ dc.finc


lh< Wubl< inl<'"'l of /(x. 1)

over R

SS7

Suppose. \Ve i.ncrease the number of tem1s in 13.2 by increasing the numbeJ of subareas
C. A; and dccrcasitlg the norm DC. A; 11. If the sum pproachcsa limit as the number of subareas
becomes increasingly large ancl each subarea shrinks tO a poinl. we call the limit rhe double
inttgrw of /(.t, y) over lhc region R, and dcnQic it by
CI.U )

The noU1tion II C. A; I

- 0 docs not necessarily require that every Ll.A1 shrink to

point. We

impI icitly assume. however. that this is always the case.

In some texts the R is placed below the iJ1tegral signs as in

jj

f ( x,y) dA .

We will usc the notation in equation 13.3, but be aware of the aheOJative.
Tf the limit in 1 3.3wer~dependemon thechoice ofsubdivision C. A; orchoiceofstarpoints
(~r, , Yi) , double inregrals \VOllld be of little use. We there tOre demand that the limit of the sum
be indcpcndclll of the manner of :subdivision of R and choice of star poiots in the subrcgjons.
The following rheorcm i ndi c-.t~t cs that rorcontinuous functions this is ahva)'S the case.

THEOREM 1 3. 1

Let C be a closed, piecewise-smooth Cuf\l'c that encloses a region R with finite nrca. If
f(x, y) is a continuous function inside rund on C. lhen the double integral or j (x. y)
over

R exist!'\,

For a continuous function. therl . the double integrnl exists , and any choice of subdivision
and star poims leads to the same value through limiti ng process 13.3. Note that cominuity witS
also the condition rhat guaranteed existence of the detinite integral in Theorem 6.2.
\ Vc cannot ovc.rcmphasiz.c the fUc.t that a double integral is simply the limit of a sum.
Moreover, any limit of form 13.3 may be interpreted as the double integral or a function f (x , y)
over !he regiou defined by the .O.A;.
The following propenies of dottble iJltegr:~ls are easily proved usiog definition J3.3:

f(x, y) over R exists and c isaeons~ttnt , then


Ji cf(x,y)dA = c Ji t(x,y)dA.

1. If the double inlegral of

2. IJ double integrals of f(x , y) and s<x , )') over R exist, then

Jiu<x.y) + g(x ,y)JciA = JL t<x,y)ciA + Ji s<x,y)c/A.

(I \ .5)

3. If " region R is subdivided by a piecewise-smooth curve imo two parts R, and R2 tll<ll
ha"e at most boundary points in common (Figures 13.2). and the double integral of/(.<,)'}
O\'Cr

R exists. then

!'{j R

f(x.y) dA =

j'fJR, [Cx.y)dJI +jJ.R: j (x.y) dJI.

(13.6>

4. The urc.u of a region R can be obwined by integrating the function / (x . y) = I over R:


nrca of R

=!i dA.

(13.7)

In spite of the fact that double integrals are defined as ljmiLS of sums. we do not evaJuate them as
such. Just as definite integrals are evaluated with antiderivativcs, we evaluate double imegr.lls
with double iterated integrals.

lilrlii;IWFH>h

A regio11

R
y

Double Iterated Integrals


We have already seen that a fur~ctioll j (x. y) of two ir~depMdcm vari!lbles has two first-order
partial derivmi,es. one with respect to x holdi11g y cor~ stant, and one with respect toy holdi ng
x con.-aant. We now reverse this process and de line ''partial" integration of j(x, y ) with respect
to .< anu y. Quite naturally. we define (t p<lrtial antidcrivativc off (x. y) with respect to x as
an antidcrivati' e of /(.t . y) with respect to x . holding y constant. For example. since

-:;-(.T

(1.\'

+ .r y)

.,

= 3..<"

+ 2xy.

x 3 + xl y is an antidcrivativc with respect to ,T of 3x 2 + 2.r y. Bu t so is .r 3 + .r 2 y + y . In ti1ct,


for any differentiable function C (y) of y. ,r l + x 2 y + C(y) is an antidcrivutivc of 3.r1 + 2xy
with rc>pect to x. Since this expression represents all ami derivatives of 3.t 2 + 2.r y. we call it
the partial indefinite intcgl"l of 3x2 + 2xy with respect to x. a11d write

Similarly, the partial indefinite integral of 3x 2

+ 2X)' with respect to y

is

where D(x) is an arbitrary diO'crcntiuble function of x .


fn lhis chuprer we ure concemed only wi th partial definite imegrals. l...imits on a partial
defin ite inlcg.ral with rcspccllQ ..r must nol depend on .x , but may depend on y. In general, then,
a partial definite integral with respect to .v is of the form
h,;)

/(x, y ) dx:

(1].8)

soJ

and a partial definite integral with respect to )' is of the fom1


h(x)

(x , y) dy .

(IHl

s<-l

E<lch of these partial definite imcgrals is evaluated by substituting the limits into a corresponding antlderivali ve. For ex<1mple.

{(x

+ 2), + xe-'+') j - {(x-) +xi' }


??

I \ ,I Oo<lblt lnt~roh n l Double lttr:lled lnrqrol<

Itt

Once antidiffcrcntiarion in 13.8 is complciCd and the limitS 5Ub5tituled, the: rc..uh is u function

of y alotte. It is the n po&Sible to imegrnte this funct ion with respccr I<> y between any tw<> lintil$,
<ay from y =,. toy= d :

l {l 'lJ

f(x . y) d.r d y .

f'Y)

I n pmctice we omit the brace. and simply write

"
1
'
(
.)
1

y)d.t dy.

j(T,

(11 I Ut

< 11)1

unde~onding that in the C\'aluaLion \\C proceed from the ittncr integral to the outer. This is

called a d ouble iltr.lled in tegral first \\i lh rc.pectto x and then with respect to}' (or. more
concisely, with respect to x andy). Double itemted integr\11< with respect toy and .r take t he
form

1...1.,.,'"' I <x,

(IJ. II r

y) dy d.r.

I EXAMPLE 1 3. 1

E'aluatc each of the: followins doubk ircratcd integrals:


(a)

~01

(a)

}~

'1'-'

(x

+ e>) dy dx

(b)

U110N

fu'['-'cx + ,-')dydx = fo't.r y- ''1;- t~.<


1

= fo'tx 1(x=

I) +

r 1 -

.r) -

r)d.T

fo'c-x' + r-- r)nx

f-;' _,- -,L

- (-j-

I - ' ) - (<' -

1
-

I)

= - - ~- ~-
3
(b>

[{ .;/+rd.r dy=[I.Jxl+yll:dx= [:CH - Nldy


(1

()

(- y+./2y)dy=<Ji - r>j ydy

- 1

= ( J2 - I) {

)~'

r.

- 1

I - 2 ,/2

Cl\.1p1er 13 Mnllil>le lnttirals

900

EXERCISE S 1 3. 1

1'

J. y+> (x 2 -

I.

.t)') dx

_, 1

(2xy

G ,.

+ 3.\')dydx

y~) dy dx

- 1 - rr

L{

9.

II.

(x

+ .1t

d)dx

11l' ../x + ,.dy


0

17.

dx

~.-~."

dx dy

1'f.'
-1

3 drdx

~dydx

, 1- y

J.'f'
0

J'dydx

ff
I

ax

If u(.<. y)
u(x . y).

'f. dx dy
X

In Exercises 32-35 you nrc given one o f tbc \"Ciocily conlpOOcntsfor


two-dimensional. SteadySiale. incompressible no w V = II(X. y)j +
II(X, )')j . Usc thccominuity cqwuioooi'Exen:isc 31tofonuall pos.ible

16. [ {
0

18.

tr

20.

!.
0

, , dyclx
(.IJ +.I )

24.

If~'''+)' x' / x' - yl dx d)'


./iy

C(mx
f"1"

* 35.

v(x. y) = sinx cosy

Stream fullCiions (/(x. y) for lwo-dimensional. stcad)'Siale. incom


p~ i b!c

tlydx

Oow :.ali~f)'

av = - v(x, y).

iJ.Y

x../.<' + y'dxcly

!.'f~
0

~ 34. v(x. y) = x / x 2 + y'

(8 - 2x 2) 311 tly dx

-A

_,

22.

-I - dxdy
l~y2

av = u(x. y).
-:;0)'

whc~ v ~ u{x , y)i + v(x, y)j i:, the ,clocity or dte flow. In Exerxcosydydx

cises 36-39

fin~l

nil &trcam functions ror the Oow with si\'cn ,-clocity.

36. v = xi - yj
37. v

26.

= kx. where k is a conslanl. fond all poosiblc funcl ions

values for the other component.

-1

ay

33. u(.<, y) = Tan - (yfx)

v l - y-

I
19. { [ ~d)'tlx

dy d;c

-I-, d)' dX
I + y

y 1 dydx

ilu
ilv
- + - =0.

I
(X+ )')

1'1'/x'+

dx dy

In two-dimensional. Sli:adySiate. incornprcssi.blc now.lhc:;-clocity


ll(.t, y ) j + V(X, J)j . which

A" COSy

.\-

* 31.

Lt -~r.x'f,x=:'+=)"''
0

(xy + x\ )dydx

'

12.

. /i+Yd xdy 30.

-1 -1

- 3y1 ) dydx

tf .

23.

8. 1_,'1"
!.'1"
10.

* 29.

* 32. II(X. y) = + )'

{f'

21.

!. e'-r

,l y- .rdydx 28.

V Of lhe now has IWI)COmponeOIS. \'


must s:nisfy the continui1y eqruuion

lt:

15.

(I

,1

xdxdy

)'

11
1,1,.
- 2 .t"'

011 + y)' dx dy

6. {.

13. , , ./i (x 1 + 2xy


14.

'-

(x1 +

7.

4.

~ -I

xsin )' dy dx

5.

2.

?:1.

-l - , /IS-11!

J.'J.:
i'J.'/2
1'1s

3.

d_~

,1 ../18-lY'
1

.__.:

()

ln E>.crciscs 1-)0 C\'Uluutc the doub!c itcnucd intcg.r..tl.

=(.<' + y')l -

2xyj

' 38.

v = - y,/x ' + y'l- x)x' + y'J

* 39.

v =- cosx siny i + (sinx cosy+ x)j

113.2 Evaluation of Double Integrals by Double Iterated


Integrals
According to Theorem 13. 1. i f a function f (x, y ) is conti nuous on a finite region R with a
piecewise-smooth boundary. 1.hen double inLegriil 13.3 exists, and its evaluation by means of
that lim it is independent of both the manner of subdivision of R into areas IJ.A ; and choice of
starp<.>ims (xt, yi) . \Ve now show that if we make particular choices of D.Ai . double imegr.lls
can be evaluated by means of double iterated i ntegrals i n x andy.

Mjl<:iil IAfWM

PM >( ll\il1 ()uuble i n ~egrals <:al be C\tthnted wtt h dooblc it('utcd i n~grals

.Y

'

))
) j-)

(Xj, .'))

Yz

Yt

.v. =c

b = x,

Xu-1

Cono,;ider first a rectangle R with edges par.d lei to the x- and y -axes W' :;;hown in Figure 13.3.
\Vc dividc R into smaller rectangle.~ by a ncl' "'""'rk of n + I vertical lines and m + I horizontal
I ines idem ified by abscis..~.a e,

= Xo

<

.\'1

<

X2

< <

,\ 'n - t

<

X11

= b,

nnd ordinates.

c ~

)'o

< >" < J :l < < Y"- < y., = d.

If the (i , j)th rectangle is thatrcctanglc bounded by the lines x = x 1-to x


Xi ~ y
YJ -t ,
and)'= )'j. then its area is D.x; D.y.;. where: 6.:ri = x ; - .\'i - 1 and D.yj = Yi - Yi- t We
choose as Stt\0" )>Oint in the (i . j),. rectangle tine upper right corner: (xt , Yj ) = (x/. )'J). With
this rc.ctangul~r ~ubdivi:..iun of R and choi<.:c or.stur points, Definition 13.3 fo r the double integral
of f(-r , y) O\ Cr R takes the !O..m
(13.12al

Since II t..r, lly,; II -> 0 il' the no nm


can write that

llll..r; II

und lilly; II i ndi 1idually upproach zero. we

/hf (x, y)dtl =

(1 3. 12hl

SuPJ>OSC we choose to first perfom1 the limio on y and then the limit on x . and therefore write

Since. x; is constant in the limit with l't:spect to y. the y -limit is the. definition of the de li nite

integnll of f(x, , y) with respect to y r.om y = c to y = d; that is.

902

Ch:aJlll!r 13

MUIIlple lnegf21;:

Conse<Juelllly.

!'{JR

f(x , y)tlA =

t {J<~

Jim
uo...jn~o .

t=l

f(x; , y)tly } A.r;.

Because the te-rm iu braces is a functjon of Xi alone. we c-an iHerpret chis limit as a dcfinilc
integral with respe.cl to x:

j tcx , y)dA =

t {1J

f(x ,y) tly } dx =

tt

f <x,y)dydx ,

( 13. 13)

a double iterated ime.gral. By reve~ing the onler or taking li mits. we can show similarly that
the double integral can be evaluated with a double iterated integral with respec.t to x andy:

( 13.14)

We have shown, then, that for tho special case of a rectangle R with sides parallel to the
axes. a uouble i111egral over R can be evaluated by using uouble iterated integrals. Co1werse.ly.
e very double iterated integral with constant limits represents a double integral over a rectangle.
The double iterated integral simply indicates that a rectangular subd ivision has been chosen to
evaluate the double i ntegral.
\ Ve have just st.med that the choice ora double iter~:uec.l integral LOevaluate a t.louhle integr411
implies that the region o f integration has been subdivided into small rectangles. We now show
tllat 1hex- Hnd )'-integrations themselves can beinLerpreted geomeuically. These interprelations
will simplify the transition 10 more difficult regions of imcgratioJt.
Tn the subdivision orR into rectangles. suppose we denote the dimensions of a repre.<ienuuive
rectangle at positio n (.t , y) by d .t and dy (Figure 13.4). In ~1c inner integral

1J.f
v

(x. y) d y dx

of equ;uion 13. 13, .t is held constant and integration is performed in the )'-direction. This
(panial) definite imegral is therefore interpreted as summing over the l'tctaogles in ~te vertical
strip of width dx at position x. The ljmits )' = c and y = d identify the initial and tcnninal
positions o f this \<ertical strip. It is imponam to note that we are not adding the areas of the
rectangles of dimensions d.x and d y in the strip. On the contrary, e-ach rectangle of area d y dx
is multiplied by the value of f(.t . y) for that rectangle.

/(.t , y)dy dx.


and iLis thesequ<lntities thHt are added.
lillclil;hlikJ

Atlditioo pt\)CtSS iJ) a lloubl~ il cra.~cd integral wilh rcspccl 10 J tlltd x

d "" 'rTTTr r l-rT"TT"TTTTTTrlnn

dy
(x. y)

"

dx

0\'C<I.

a recwoglc

1l1e x-integration in equation 13.1 3 is interpre~ed a~ addingo,.erall strips sll\rting at .r = "


and ending at ,r = b. Tile limits on ,r therefore identify positions of the first at\d last strips.
Although our diagram illustrates finite rectangles of dimensions d.r and dy and finite strips of

ljlclii.IWfi.W Addi1io1l
pmces.~ in a ck')ub!e ttr:rru:ed 1llte
gral with rcspen 10 .r and J \no-er
n rcc:nm,gle

width dx . we must keep in mind that the imegrations take limil~ as the$e dimensions approach
zero.
Analogously, the double iterated integnll in equation 13.14 is interpreted as adding over

horizontal stri 11s. as shown in Figure 13.5. Inner limits indicate where each Strip starts and stops.

and outer l imi 1~ indic~tc the pos it ion~ or first und last ~t l'ips.
The lrftnsilion now to rnorc general regions isquite !\traightforward. For the double. imcgml
of /(>. , y) over the region in Figure 13.6. we w~e a double iterated integral with respect to y

ily
()'. y) .

(!

J (x, y) d y dx over rectangles in o vert ical strip.

and x. The )-imegration Adds the quantities


We write

"<)

ti-t

IJ '

.f(.x, y) tly dx.

,d x)

where g(x) n11d h(x) indicate that each \'Crtical strip starts on the curve y = g (x} and ends
on the curve )' = It (.r) . The .r.integrntion now ndds ovet all strips. beginning at x = a and
ending at x

= b:

f.j(

f (x , y )c/A

l r.rtgrllliOII

with rc>PXt lo y and x idcb flf~


in~ide " \'(r1il:.al ~p. ;md lh:tl

;bf."l.
<) / (x . y)tly dx .
,t,~ (X)

(I

\.1 S)

fJ

A double iterated intcgrul in the reverse order is not convenient for this regionhcc;tusc hol'ilOillal
strips neither till start on the sumc curve nor ni l cn<.J nnlhe :-.arne curve.

over all ilrith

Forthe regio11 in Figure 13.7. we obtain

!1

f(x . y) dA

= ''j'hi>) j'(x ,y} d.rdy.

J
,.

( 13. 16)

~(y)

The limits on double iterated integrls h<"e been intorprctcd schematically as follows:
tix

"

Y g(x)

.X

With these interpretations on the li mits. ~ou c<tn S<.'C how ioltJ)Orta nt it is to ha\'C a wdl-lai.l(:llcd
diagratn,

then O\'Cr all Slrips


)'

" = h(y)

R
dx

~ITIITIITIDIDIIJ::I~ dy

904

Ch~p1e 1

13 Multiple ltuegr;.:ls

I EXAMPLE

13.2

Evalualc 1hc double in1cgml or .f'(x , y) = x y 2


y = .r 2 antl..r = y 1 .
MjlcliJ;J s ~ l m~gr.uion
of xy: + x 1 \)\"tr R usiog Vtltic.:al

+ x 2 over lhc region bounded by 1hecurvcs

SOLUTION If we use venical slriP5 (Figure 13.8). we have

Slripll

Gx

5
fl-

=[
=

,.

8
21

24

- - =
5

x") d x = {-Sx - - .t- * - -x' }'


72
'

21

24

39
280

There are IWO disli nc1 pans 10 every double imegrl: fors1. lho func1ion f (.r. y ) bd ng inlcgrated.
which is 1hc in Iegrand: second. lhe region R over which imcgrntion is being performed. and

this region determines the limits on the t'QJTesponding double iterated integral. Note that we do

not use j (x, ,V) 10 dcten11ine limi1s on lhe double iteralcd integral: the region dc1er111inc.:; the
limits. Co nversely~ if we are given a doub le itcnucd integral, lhe n we know 1hat it represents
1hc double integral or i1s imegrand over some region. and 1he region is colllplelely defhte<l by
the limils on the iterated in1egra l.

I EX A MPL E

- -

~'

Th i~

poinl is enl(>hasized in lht: following example.

13.3

Ev~lluate the: double. iterated integral

12;2

e.o:! dx dy.

)'

SOl UTI ON TI1C function e' docs 1101 have an clcmcmary an1idcriva1ive wilh respect 10 x .
and il is therefore impossible 10 cvolluatc the double iterated integrolas it now Stands. Bu11hc
double iterated integr,\1 n::prc,;cnts the double integral of e'' over son1C region R in the xyplane .

Io find R we note that the inner integral indicates horizontal stri(JS that all start o n the

lir>e .~ = y and stop on O>e line x = 2 ( Figure 13.9a). TI>e outer lirniiS s!JIIe lhalthc lil'l\1 and
last sui ps arc at y = 0 and y = 2, rcspcc1ivcly. This defines R as the lrian~c bounded by

ffi [!J

ti rnib C.UI ~ 11\Cd IU

dctenninc &he

~on

of

intcg;~ ti ort

}'

}'

(2.2)
x=2

)'=X

y=x

x=2

dx

EvaJn."'tioo ori>Qllblc ll'lli111s:by D<Juble l!erl~ll meg.ta1s

13.2

th~ stmight lines

= .t, ,t = 2, andy =

905

0 (Fit~ure 13. 11b). If we no w rcYcr.;c the o rder of

imeg:nuion and LISC venicnl strips. we h11ve

Jo[ 'J,.{ ' e''' dx dy

This example poi IllS out 1hat au itenned in1egml in Ot'lC order ma)' be much easier to ewluate
than lhe com::-iponding itcrdtcx.l integral in the opposite orde-r.

EXERCISES 13.2

In Excrc.iscs 13- 18 evaluate the double iter.l.lcd inlegr6!1 by


the order or intcgn.uhm..

In E.\c~iS(S 1- 12 evaluate the doubk intcgr.:-..10\\.T the n::gion.

2.

JJ.

X=

(4 - .<2 - )')till . where R is bounded by .t = j 4 - y,

0, ,V : 0

Jf.
JJ. x/
jJ.
Jf. +
If.
x+r= 2
Jf.
}f.
Jf. +
Ji
Jl
3.

(x+y)dA .

wht;n;

= y1 + 2,

R is bounded by ,,

u.

i/A , where R is bounded by x + y~ l = 0 ..+ y = 1

IS.

}<t' t/A. where R is bounded by y : ; : ; x . y : ; : ; 0 . .f

6.

(x

7.

S.

=I

JJ.

12.

X=

= J x+ . .1' =

i:L:

-Jr.<=,,;="+
;=}=r., d y dx

-Jr.l=+;=:;r,=:+=.,,,
=. dy dx

0.
m (arco of /?) ;S

1
1
(X)'+ )' - 3x )dA . where R is bounded by y

2
( I - .t) dll . where

R is bounded by x + y

y) tl A. where R is bounded

by .~ =

x t.

= l,.t + y =

y' dA . where R is bounded by x

+ 12y'

,12 ,

.x' - y2 =

= 0. y =

I. y

1/ 2.

.f(x, y) d A < M(;JrCa of R).

x ' O\'C:f the

ln Exercises 21 -28 cilher the hucgrul has vuluc 0 or it cun be cvntmucd


by doubling d>c double integral over h:M the region. By drowing the
region anU cu.nlining the integrand. <klcrminc which situation pre\uils.
Do not cvalualc the integral.

12

= x. x + )' = 4

Jf.

10. Ell;l!Ulltc !he double imcgn~ ol' f (x, y) = I} Jlr


regi()l'l in the tin l quadnuu bounded by y 1 = 4 - 2x.

21.

.t il A , where R is boumk:d by)'= 3x. y

1/))'4

'-l'-)

+ y2)' dx dy

1?. Verify th11t if m ;S j'(x, y) ;S M for nil (.<, y) in R. then

x ydA . when: R is bounded by y

(X

tD'

-I. X - ) ' = I . ) ' - :r =I

I0.

,. r' )'(x

17.

=9

))dA , where R ii bounded by .r: + .vz

y::;;:; l .y::;;:;2

9.

.. 16.

S.

2
(4 - y )'1' dy tl,t

14. /.t/,tsin(.x') d,tt/y

.r : l,y = l
4.

!.'!.,;;=:;;

rcve~i ng

22.
23.

Jf.
Jl
Jl

2 3
.< ,1' d A. whco'C R is bounded by x

x'y' dA , where R
(x

and (0, 3)

is bounded by

+ y) d A . where

= )4 - .1'' x = 0

x = )4- y 2 x

= 0

R is lht: square widt vertices ( 3, 0)

24.

!f. C(,~(X )dA , ~


Jl t"''~+,!
.<

R is bou1xk:O by J

=4

In Ex~.:rci scs 35-4L cvaluntc the doul>k intcgr.tl O\'Cr the region.

- lxl.

)' = ...
25.

Z6.
27.

lll.

d A . "here R JSbotJndedb)' y - 4 - 4x 1, y

JJ.
Jf.

= x2-

cos (x' y) d 11. where R is lx>unded by y = 0, y = ,, -

}f. (6 - x - 2y)dtl.
.~7. J16.\' dA ,
R
5

sin (.t _r) t/ A, where R is b()undcd by y

!!.

=0, J = .r

(.t'y 1 + xy')dt1.woo-e R isboundc'<lby ./fil+

;&;

.t

c.wct the

rcgivn.

I (.f. y):;

x:O.y=

rcr the rcsion in lhC


1 - .t.y=O
X)'

lio~t quadrant bounded by

30. /(.r J) = .r + >' over the rl!gion bounded b)' J = X >' = 0


)' = /2 - .<
.11. f (.t. y) = x O\'Cr the regiol) between y = sin x :~ n d y = 0 for

0 ~ ,\' !S Z.l"
Jl. /(.<. ,1') = e' 1 Y over ll.: r<gion bounded by y
x - J..v= l - x. y= - 1 - x

X + I .)'

Jf.
Jf. ji"'+'Y
J!. +

prodlliXd ct!ch m onth?

IOOOO.r" 1,r0J

+ y' =

+ 5y =

16, abow,!

= 2, )' = 0

1/ A , where R is h<>undcd by X = -1. )' = 2, X =

y/.x'

y ' d,\, where R is bounded by ,1'

= x, X

-1. y-0

41.

!"}4r.

(.<2

+ y 2) ,, A. w i1\.YC II is bounded by x' + y' -

,. 42. Ev-..duau: lhc double it..-rah.:d i1\lcgul

J.l.

E,~aluaiC the double inCC'lll

the square boun<lcd by x - I.

!f.
,1'

J.'J.' x - Yl dy

9
1/:c.

l,r - 2x 2 + II d A. wh(tC R i,)

= :1: I .

For the occc ICt'..Uirlg sht syste:nl of 3 mu.ss speeuonlCtcr.thc nurnber


of ions within urtit :solid w1~lc vf the clt.'<.'tlon OOun, al the pb11C or tht
firsl slil. is:

211, L [. "

"- - -

each month. and its operating bucl,gel ''atie,:: fron


S8()X) to S 12 000 pc.r monLh, whfu is: the il\'t-rogc number of widg_elS
33 using the proclutrion functi on I' (.r, y)

is the region unc.kt x

y" d.A. where R is bounded by)' = x. x+ y

~ers

~'-' Rcp-e;.~ t Excn:: i~e

.u.

.. 33. TileO:'Ihh- f)ousla$ptOduetionfun('tionfOtawidgccis P(X 1 y) =


10 000x0 .J,,.o., . whc.~ P is the number of widgets produced each
mQnth. x is the nwnber of employees, ant.l )' is the monthly opcr:ling
budget in thousands of dolkn. (f tbc company uses anywllcrc between

45 and 55

39.

40.

In Exercises :!IJ-32 lind UlC average vah.c of the fun~ i()f\

where

where R isboun<l<dby x 2

y = x - 4, and IJ<Jtonded by .r = (.1- 2)'

,ll)'i = I .l 8.

7 = *ff.t<x.v)dA.

'*

dA. '"heR: R is bt.)uJ'Iclcd by .\' 2 + y 2

.l6.

'IlK' average value of a funclion /(x . y) over a region R with area A


is defined os

29.

Jl -t

35.

rr

1.,
.,

x ' ( l - a 1' /< )

(I

+ .rl)(x 1 + a)1fc 1 ) d.uh,

\\'here n( i~ the :number of ions with initial vclocit)' c. Lis the Jen~lh
ofthc s!it .t/ is thcwidthofthcslit.andtl > O is:tC01lSUUll Showlhal
11
2n,dl.( J- ad/<' ).

113.3 Areas and Volumes of Solids of Revolution


ljttlil;jj~

i1en:lb.l itlltgro'lls cu be

Doobk
u~

Because equatio n 13.7 represents the area of a region R as a double integral, "''d double integrals
arc evaluated by mean$ of doubk iterated integrals, it tb llow:-; that are as can be calcuhucd using

1o

find arcdS cf rq;ion~

double iJcnued i111egrals. In pmJicuhtr, 10 find Jhc area ofJhe region in Figur-e 13.10, we subdivide
R into rcctanglc.s of d imensions dx and dy an.d therefore of area d A = dy dx. Areas of these

recJanglcs are then added in the y-dircctinn to g ive Jhc urea ol'u vertic!ll strip
R

h (X)

glx)

dydx,

O},.J

, E ;;KAMPLE

~ ::1.4

_.._,,,,,,.,,,.,.,, .,.-!Co"...,"' ""''

"'

= 2, ' - 'J.Jj'. Y 4
If <'<' clw<>>'~ r,.,..;;e-.,m<>l Mritw f,.r ,,;, ft>ei<o<'~ <l' I.IW"" r.l.J I), we h ..,.c.

FhlO/lfiC JtA:'M vf(ho(! n:J!}<'f'1""" "'<J....Jby o,.,~ulWJ. .'i.J'


.,nl .r '77nx

, ,,_ -

1/,~./7d~ d)'
:V

'

1'(

')

2../'f- ).

'

of y

,. .,

~-2 1n :J.

12 '

*"l'l!LL
rJtl lril""

'iJ II \Vl1<1 h '"'""" ''


UtJ/y (.1111;' ,k,no/Jic

:r.'~ .._,,._,

)'

)'""'"

Wlo~u

hie oiCfilt~lmt~N'~ ~ ~ o'lef lef

lfu'.I4.'JI <IJM$fM / , !!0:.:1J.vl

)' (f-J

mLLLEBEM

\ 'Ctf.Coa.l o0111p.. ~ .,;lt<.oO'I, 1'10\l W u

(.t,

.<

~)

(f~.f)-~J-,.,---f Cl .
y:<l

<I)

If we c.ompare finding areas by definite integrals (Section 7 .I ) and finding the same areas by
double integrals. it is clear that no great advantage is derived by using double integrals. In fact,
it is probably more work because we must perform two, rather than one. integratioos. although
the (irst integration is triviaL The advantage of double integrals is the refore not in tloding area;
it is in finding volumes of solids o r revolution, centres o r mass. moments of inertia, and fluid
forces. among other applications.

808

Cb:tpler 13 Mulliple Tntegtol"-

Volumes of Solids of Revolution


If the region in Figure 13.1 3 is rotated :~roun d the .x~a>: is.. Ihe vt,lume of t he resuh ittg solid of
re.\olution can be e\aluatcd by using the "''asher method introduced in Sectiott 1.2:

in1te integrals , \oJwuet> of \\a!ihen.


Qr C) l11xln~;nl ~ells arf: <:~lcul11cd

vulume =

.
1

fJT[iJ (.r)J' - JT[g (x))",) dx .

(1 ~ .1 7 )

~JCncrat.cd i~

calculated by using the

Tr this rcyi011 i.s rutmctl a nJuttd the yaxis. the volume


cylindrical shell me1hod:
volume=

'!--~-
)' ~ g(.r)
a
M#IIUII;LID
dot~<-

'

=t9

With

inlc;:.ralh ,olurnC)

or riI.):S

on ""kl1Ja1cd

>

1b

2nx[h(x)- g(x) Jdx .

(1 3. 18)

T hus. once we havctho:;cn to usc. vet1iL"111rttl.a nglcs. the nxis of re\'olution determines whether
we use washers or cylindrical shel l.s. 'Ve 110w show Ihill wi1h tlouble imegrals one methtx.l w orks
for al l problems.
To rotttlc this rod) on around the .t -axis we subdivideR into small areas d A (Figure 13. 14 ).
If the Mead A at a po irlt (x . y) if' rotated about the x~ax i s, it generates a "ri ng" with crosssectional area d A . Sirtc:c (.K, y) tnwels t~ di~thlce 2" .v ir1 traver:-.f,lg the riitg, it follows that
the volume in the ring is a pproximately 2;ry tl ;\. 'lb find the total volume obtt1ined by rutiating
R about the x a.. : is. we add the \'Oiumcs of nil such rings nnd tukc the li mit as the HI"CaS ~hrink
10 points. But this i.s what we mcao by the do uble integral of 2ny over the region R. and we
therefore write
volume = / l 2rrydA.

(1:1 . 19)

On the other hund, if d A j:, rotated ubout they-axis, it ugain forms a ring. but with
approximate volume 2rr .t d A. Tite total volume, then, when R is rotated about the ya.'<-is is
volume

<kd>re iW..'t'21cd in!egrlll.);,


rinJ.~

(1 3.20)

Si >CC doubl e iu:ra1ed in1egmls arc used 10 evalua1e double imcgrnls. it follows 1ha1 we eM
se1 up dO<ble .i tera1cd imcgral s to find volumes rep<cscntcd by equations 13.19 and 13.20. The
decision to use a double iterated integr.U with respect to y and ~t implies a subdiYision of R
inlo rectangles of dimensions d.r and dy (Figure 13.15). The volume of the ring fomtcd when
1his rcc1anglc is 1\)lalcd 'U'OUild the x -axis is 2TT y d y d x . If we choose 10 i megratc lirs1 w ith
respect toy, we w-e addi ng over all re<: l~lng les in;_\ vertical strip

With
cf tt."CWnll,ul:tr

= J l 2rr x d A .

\'Qfurn~

are c,a!(;Ul<ilcd

y
)' ii(A')

,...---,.,..__..;'
dy

IH>)

2.rrydy dx.

(.<.y)

/ s(x)
.

where limits indicate 1ha1all vertical strips s1art on the curve y = g(x) and end on 1he curve
y = h(x). TI1is imcgral is 1hc volume gcncmled by l'()(aling 1hc vertical strip around the x-axis.
Integration now wilh re.spectto x adds over all strips to give the re<1uired volume:

volume=

b1kt)

1'

2rrydydx.

~( \)

No1e thai when we actually do pcrfom1the inner intcgrmion. we get

and this is 1he result comained in equmion 13.1 7.

( 13.21)

1J.J ArtaS ::~od Volume~ of .Solids of Revuhnion

i09

Wl>en R is ro~otcd around the> ru<is <he rectangular area d y d x generates a nng ofvolun>e
2>r .r dy d.<. Additioo over tl"' rectangles in a vertical trip
h i>)

2:r x dy dx
/ f.l.l)

gives th~ volume gencnucd by rotming the strip obout the


repoctto x adds o' er all strip< to give
volun1c

bf.h(A )

1
(l

Thi~ time

>-n~is.

Finally. <nte&mtion with

2rrxdydx.

tf{.6)

the inner inregmtion leatl" to

the same result us in equation 13.18.


1llt! ad\Bntagc. then. in u~ing double integmh tn find \olume~ of M>litJ' of rf..--volutioll is

th.'ll it requires only one idea. 1h<11 o f rings. The fin.l integmtion leads tO washerS or cyl indrical
shell. but we need ncvct think abtlUtthis.

I EXAMPLE

13.5

..

fi nd \oluntcs of the solids of <'e\IO!ution if the regoon bounded by the curves .1'

)' = ~' 2 - 2x is tocated lround:


(a) the y-uxis
~ou

(h) .r

= -3

(c) y

(d ) .l'

= x + 2.

n o!\
(a) l f'W u<e vertical "riP' (figure 13.16), thcn

vol um~ =

2/.lx-x' 2;r.r dyd.r = 2>r /. (.ry}~-x' d.r

1
0

- 211'

\'1 -lx

fo\t<~

- xz)- (x

....--l...-

2 - 2x)ldx

= "" lo
{ 2(2.r2- .13)dx = "" {2x3- x J l 3
4
0

;T

Artu i& stabdivickd into rrctanglf'- \"01

llln~' eli' i n!-s calrula100, aJ'~J added O\'tJ' \'tnlc.al ioltip...

.r=

3
3

16tr

2<- .r1 ,

(b) In this case the radius of the ring fom>C<.I by r001ting the rectangle about x = -3 is
x + 3. and therefore

u-x'

11
l

volume=

2rr(x

+ 3)dydx =

2rr

11

(y(x

4-

3>l~::~dx

.1-z.c

Mrr
3
(c) When the rcctan~:lc is rutatcd around)'

2. t11e radius of the ring is 2 - )'. a11<l

hence
>olume

2!.2-' 21r(2 -

1
0

y)dy dx = 2n:

r!-lr

12{

- -1 (2 - y)1

}2-' dx

... ~ - lx

32rr
3
(d) Using fomlUIU1.16, thcdismnccfrom thc area clement dy dx at position (x, y) to the
liney = x+2 is lx - .v+21/J2 = (x - .v + 2)/J2. Hencethe\olumeofthe ring

obmined by romting the recmllglc around the li11e is [2..~ (.r - )' + 2)/ J2) dyd.~,
and the volume of the solid of remlution is

volume =

l1lr-x'

1
0

./2Tt(X - )'

+ 2)dydx

-J2,.

11{ I

.. --2.1

- -(x- )' + 2)2


2

,. 1"

}Zr-.r' dx

.. -- z.-

(16x - 4x 3 ) dx =

= J2

1T (8.x 2 - x j2 =
../2
0

t;:
8v2n.

Notice that double integrals allow us to calculate ,olumes of solids of revolution aboul an)r line
())all

(d) of Example 13 .5). With defini te intcgrdls we were restricted to vcnical and horiwntal

l iJlCS.

EXERCISES 13.3

In Ex..:n.:~~ L-10 usc a Uouble inu:gralto lind rhe afc:a of1he n:gion
bounded by 1he cun>e..~.
2. y

3.

=y

, X

= 3y - 2

= x1 , y =

4. y = x'

8. x=y(y - 2).x + yg 12

5.r + 6

+ 8, y

= 4x

7. X= 4y- 4y 2. )"= X - 3. )"=I.)'= 0

9. y =

+8

.rJ -

x2

2x

+ 2, y = 2

10. X + )' = I. X + )' = 5, y = 2x

+ I , )' =

2x

+6

I 3.4

In Exercises I 1-20 usc a dO<blc integral to find the volume of l.hc solid
of revolution obtained by rotating tbe region bounded by the curves
around the line.

II.
12.

- J4- X. X= 0. )' = 0 abouly = 0


4x 2 + 9r' = 36 about y = 0

J =

13. )' = (X - I ) 2

)'

1 about X

16.

17

.r =

4.<2 -

18.

X=

3y - y1, .r ay2 - 3yubou1 y =4

19.

= 2y +y =

20. x

y2

2,

=0 about)' = 1

.t 3 bout y

.10. (X + 2) 2)' : 4 -

=-

= - 5 about X =

4. )' = 2../X=l. y = Ooboul y =- I

l!).erci~s 2 1- 30

* 32. y = x' 33. y

= 2x 3 , y = 4x + 8.

= .t / ../.r +3,

X=

)'

I,

=0

X =

6,

y = ../x - 2. ,\' = 4- Fx. y


)' = .1 3 -

* '25.

* 26.
* 21.

x' + y'

= lx2 -

11.

=-

x = - 2, x = 2. y = - 1 about )'= - 2

35. y = (x

* 36.

+ 1) 114 ,

y = - (.<

+ 1) 2, .< =

Ouboul x = 0

Find the area of the region t:ommon l.o t.hc lwocixc.Jcs x1 + y 2

x' + y' = 6x.

= -4.

In Exen:iscs 37-40 lind the volume of the solid of re'olution obtained


by roHuing the region bounded by the C ~II"\'CS ~H'Qu nd the line.
,1'

= - x1

=4 -x. y =0( - 16 S x S

3)

* 24.

2, )' = 0 about y

use n t.loublc intCSJ'OI lO find Lhc are;.\ o f the. region

+ 2 1. )'

* 23.

= 0, )' = 0 (X !:: 0, )' !:: 0)

>I. x=/4+ 12y 2 ,.< - 20y=24.y= Oubouty=O

bounded by the CUf\'CS,

22. )'

X. X

=7

In Excrciscs31- 35 use adouble imegral 10 find the volumeof the solid

and
ln

+ 1)/(.t + 1). X+ 3y

31. )' = 4/(x 2 + 1) 2 y= I uboul.< = 0

1 = )'2 , X= 5 about X = I

4x. y =

(x 2

+ x)

around the line.

X -

X , )'

911

of rc,olution obtained by rotating the region bounded by tile curves

15.

= .'', )' = ../2 -

I 29. )'

=0

= x 2 ~4. y = 2x 2 ulx>ut .Y = 0

14. y

28. 1 2 = x (9

Fluid Pressure

X, .Y

+ )' + I =0,

= JY+'T

= x (4- .\.~)
2

= 4, x' + y' =4x (interior to bollll

< = 1/ ,/4 - y'.

* 37.
i

X= I.)' = I, X= 0, )'

38. y

=.r 2.y

= 0 about X +

,\ '=

= 2.r +3 aboul y .. 2x+l

= ,/X. y = 0, x = I about )' = Jx + 2


40. .r = 2y. )' = X - I, y = 0 aboul .r + )' + I = 0
39. y

+ t 4 1. PtO'-'C thal the atC-a above the line )' = h arx:l under the circle
.x 2 + y l = f l ( r > h) is given by
.

4x+/ = O. y + 1 = 0. r-1 =O

II = rrr 2/2 -

h./r' - h'- r 2 Sin- t (h/r ).

I 13.4 Fluid Pressure


Tn Section 7.6 we defined pressure at a point in a Uuid tlS che magnitude of the force per unit
area that would act on any surface plnccd at that point. We discovered that at a depth d > 0
below the surf~ ce of a fluid. rxcssure is given by

9.8lpd.

(13.23)

where p is the density of chc fluid. \Vi th these ideas al)d the dcfinice imcgral. we were able to
calcuhue Huid forces on llat surfaces in the fluid. In particular, the magnitude of lhc tot~ll force
on e-ottch side of the venical surr,-.ce in Figure 13. 17 is given by the dell nile in1egral

force

= [b -9.81pyih(y)

J.,

- g(y)l d y .

( 13.24)

AILhough horizontal rectangles are convenient for this problem, it is clear that they are not

reasonable for the surface in Figure 13.18.

I EXAMPLE 1 3 .6
The face or a dam is pan1bolic wilh breacllh I00 m and heigh< 50 m. Find the magnilUde of 1he
tol:\1 force due to fluid l>t"e~sure on 1he face when the waiN is I Il l f rorn the top.

Llll'

v. tiCc..- ('"

IMF*f*
\~l it.al I.IC('

s o r.UTION lf we use 1he c()()t"dinmc.symern i n F igure 13.22. then lhc~dge ofche dam has an
equation of the form y = k.r 1. Since (50. 50) is n J>Oi lll ocl thi:-; curve. it follows that k =
Because force on lhe left h01lf of the dam is the same. as that on the right half. we can irttegrate

Foc-cc of

ft.

t:tf n dllm

for the right half and Uouble rhc rcsuhi I hat is,

(50, 50)

force = 2

f.

))./i149

9.8 1( 1000)(49 - y}cly d x

,\':/50

1;,.;; I
0

I EXAMPLE

J."./i (2401
~9-<z+x-)
---

1 49
Y }

49y-'c/.<= 19620
2 " 'ISO
0

=1 9620

2401x
49x'
19620 { - - - - 2
150

x3

+ -25000

}''./i =

50

5000

dx

6.22 X 1 0~ N .

13.7
1\ tank in the t'orm of n rigJ1t-circuh" cylinder of radius

2I m und l ength

10 m hus ils axis

horizolllal. Jr it is full of wtuer, find the force due 10 water p r'CSSUI'C on each end of the tunk.
~

SOLUTION

F01'<:cun

Since <he fo rce on 1hm paot or the end 10 the leli or 1he )':l.lis (Figure 13.23) i:<

ii.lcmk~ul IO the force on thtll part to the right. we c.Juublc the (ur(.'C on Lhc right half; th~l i.s.

rhe e11d Qfll full cylindrical twtk

for<-e = 2

1/l 1 ../1/ 1-y'


/.- 1/2

= 9810

9.81 (1 000)(4 - y) c/,u/y


0

1/2 /. J /1-.r'

d.<d y- 19 620

-1/l 0

I (1)
-7r
- 2] [ 2
2

490517

= -- - 19620 {
4
prtss~re

5J fDI

vn 0 submerged

Centre of
pi a l~

- 1/!

../1/1- y'

y dx ely.

The fi1-s1 double i1era1cd inu~gral rcpresen1s lite area ofone-half I he end or1he tank. Consequenlly.
force= 98 10

Mjltlll.l

/.1/2

19620

J.'/2
~
y\ f-::
- yl d y
-1/2
4

-H~ -l)

l /l } '''

=
- 1/2

4905Jr
4

T'.

is a

poinl for <..'tJ lli\';.~lcnly of momttliS


o( fiM'<.'C~

The cen1 roid of a planar regioo is a point at whkh the area of the regioo can be concemrated as
rar as fi rst m omentS o r the regio n are c.oncerned. IL i.s advanLageous to define a point called the

centre of pressure for a surface submerged in a nuid; i1 is a poi m where a s ingle force equal 1o
thai of 1he Ouid on 1he surface has 1he same lirsl momem aboul any line as does 1he Ouid force
on the surface. For example. the Huid force on the region R in Figure 13.24 is g:iveo by the
double intcgr.tl
( 13.28)

where P = - pgy in I he coordinalc sys1cm shown. 11 is I he sum of fluid forces P d A on


elcmemal areas d A al JX>inls (.r . y }.
t.lch el emenlal force P d A creates a momen1 )' P d A about the .~-axis. and t he total first

moment of all such elemental momems i ~


(1 3.2'Ja i

Similarly. lhe lirst momenl of the fluid for<.-e <>n the surface abou1 the )' -a~is is

f{tPdA.

( 13.2Ybl

n,., centre orpressure orR is 1hc point (x,, y, ) defined by


Fx, =

//,,xPdA,

( 13301

where F is given by 13.28.


For example~ due 10 lhesymmeuy ill Example 13.6. the centre of pre.~sure of the dam is on
1he y -a xis. Its y-coordinatc is given by

Yr

= .!..fj.
yPdA ,
F R

where R r-eprese,nts the dam, and F ~ 6.22 x I011 N is the lOlaI force on the dam. " 'e integrate
over I he right half and double I he resuh:

y,

= F2135.;21''.
'

y(9810)(~9 -

y) d y dx

.r'Jjl,l

35

19620 { 117 649.r _

I EXAMPLE

= I\1620 }.' Jl (ll 7649 _ ~9x'


F
u
6
5000

135,/i {-49y2 = -19620


F

49x

25 000

+ 375 000
x

7(375 000)

yl }'9

3 'ISO

dx

)dx

} JS Ji =

I.Om.

13.8
Fi nd the depth h ()f ocean w2t1er in the t\f'pliauion Preview m which the tide gate opens (Fig

Application Preview
Revisited

urc 13.25a).

llmJ l:lriU!r'btl
M ,,peniog Y. ht l 1hc l;llfll ,)( I he mon...,-..1, of lhc
mtlt~oh ond lho. MCOI\ on 1hc :tte is ltl\>

I
I

1.8

M arsh

I
0.9

I
I

It

C>~<~ ta

tioll ()( n)Oiflcfll ()( nl.H~h "Uicr' Oil

:.ae tOOut hi"~


I .S

)'

Waler level ot rn:~rsh

Ocean
/ Hinge

09 1-----''---d'x=---d,...,.i
(x, y)

'

-t- Gate

13.J

Fluid Prtssure

915

SOLLrflON The forces of the marsh and ocean on the gate create noments about the hinge. If
we take Ct.1Untcrclockwisc momcnt.s about rhc hinge (marsh) as positive. and dock wise mo ments

(ocean) as negati ve, the gate will be on the verge of opening when the sum of these moments
is zero. Since. Lite force of the marsh on a re.ctangle of c.limcn.siuHs dx and d )' on t..he gate
(Figure l3.25b) is IOOOg(l.S- y)dx dy, the momeot of this force about the hinge is (0 .~
y) IOOOg ( 1.8 - y) d x dy. The total momem about the hinge or the force or the marsh on the
gate is

1L

M,.

= 1

1000g(0.9 - y)(l.8 - y)dxdy

&ly
= IOOOgL { 50 -

27,,2
2~

= IOOOgL Lo.? G~ - 2;~ + l )dy

y 3 } o.~
1215gL
3 o =
2
.

Similarly. the moment about the hinge or the force or the ocean water on the gate. when depth
of tbe occan water ish, is
M.

= -

{0.9 { L

lo Jo

= -1030g L

J030g(0.9- y)(h - y) dx dy

[:~- (h +

9
10

) y

9hy
(
9) y2
= -1 030gL {.- h + 10
LO 2

+ / ] dy
y)

}0.9

+-

8 Ih
81 (
9)
243 ]
= -I030g L [ 100- 200 h+l() + 1000.
The sum of these momems is zero when

81/i- -81 (
0 = 1215gL - 1030~[ [ 2
.
100
200

243 ]
"+ -109) + -1000

= "

= 1.756.

The gate is on t.he verge of opening when the ocean wmer is L756 m deep.

E XERCISE S 13.4

7. A triangle of side lengths 3, 3. ru~d 4, with the longest side vertical~


and the uppc.nnost \'CriC.X 1 w1.iLbelow tl1c surface

In Exercises J-8 the surface is submcr"gcd vertically in a tluid with


d-ensity p. r ind the force due h) fl uid pl"C.'iSUI"C On One Side Of the
surn~ce.

I . An equilater-3l lrian.gle of side length 2 with one edge in the sur face

8. A semicircle or r-Jdius 5 wilh the (diameter) base in the smrace

* 9. The vertical end of a \Vater trough is an isosc-eles triangle with width

2. A parJ.bolic scgmem of lxlsc 12 ruxl height 4 with the. base in the.


SUtfacc
3. A ~uarc of side length 3 with one diugonul \'Crlit:al and Llu; upper
most \'erte.x in the surf<lCe

2 111 and depth 1 m. Find t.hc J("lft"-c of the wutcr on cm:b cod when lhc
trough is onehal r filled (by volume) with w~Le(.

or a p:tr:lbola 36 In :lC'rOS..~ the


LOp und 9 ro deep aLthe ceolrc-. f jod tbc 010Ai.roum force due to wutcr
pressure on t.hc llalll.

:t: 10. A dam 2CTO$S a river has the shape

4. A tri ang~<e orside lengths 5, 5. and 8, with the longest side uppenllOSt,
horizontal, :1nd 3 units below the ~:urface
In Exercises 11-15 the surface is submerged oJert.icnlly in a nuid with

densi 1y p. Find !be force due 10 iluid pressure on ooc s ide of the s urface.

5. A biangle of side lengt.l1s 5. 5. ru1d 8. wilh tl1e longest side below

the opposite venex. horizontal. and 6 uniLo; below tl1e surl"a ce


6. A trapezoid with venkal par<lJiel sides of lenglhs 6 and 8. and a thjrd
s ide perpendicular to the p<lr'.J.IJcl s ides. of le ngth 5. and in the surfat'C

*
*

11. A circle of radius 2 wit.l1 centre 3 units below the s urface

12. A ret'tMgle of side lengths 2 and 5. with one djagonal vcrtic<lJ and
the uppermost vertex in the s urrat'C

916

Cbapte~ I J

Multiplclulcgr:ls

JJ. An cllii>SC wil.h major ~ od ml nor axes of Jcngtbs 8 :md 6, and wilb
1he lllaj ot axii; h(l(i1.0illal aoc15

u ni 1.~

he low 1he sutface

.. .14. A paraUclogr::~ro of side lcllgths 4 and 5, with ooc of tbc l.ong_cr


~ides h(\ri'l.Oillal mu:l i1l the surface. and 1wo ~ide$ rna king om :tngle or
1r /6 radians wi1h the surface

15. A Uiimgle or ~ d e lcng1hs 2, 3, mld 4 wi1h lhe IOilge:-.t side YC I'I i~.:al.
the side of length 2 above the side of lcnsth 31 and the uppermost vertex
1 uni1 below 1he s ~1rface

* 16.

An oil cau i::. in the fonn of a rig.hLc-irc.ularcylindcr of radius r and


heigh I h. IJ the axis of the can is horizontaJ 1 and Lhe cnn is full of oil
wilb density p. find the force due to Ouid preitsurc on each cod.

17. Find the force due to w:uer p r~-s-ureon each sideof the nou venical
plate in 1hc figure below.

J!). A SCOlicirdc with r.Jdius r wbcn the di_:.metcr is horizontal, :tbO\'C


the re....:1 of 1he semicitcle. mltl h UJl il~ below the ~ur face

- lJ . A square with sides of lcogth L wbcn one diagonaJ is vcrtic~l and


1he uppermost \'efl_ex is in the surfon:e

* 13.

T~

1>rcs.~urc.

centroid or a plane figure is a Jixed poinL The centre of


on the other hand. changes depending on depth below the

sutfacc. Us:e the result of Exercise I8 to verify this.

Im

-~- -- 4 01-Jm

**
Semicircle

In Exercises 18-21 tind lheccnlrc of pressure oflhesurfact !i.ubmerged

c.rtica1ly i_o a l:luid.

'1

22-. The nonhypotcnusc sides of a right anglcd Lrianglc ha,c lengths


L ~l nd t!, where f < L . The vertex conutining Lht right :lng1e is :n
Ihe Ol'il!if'l ill Ihe surfat:t: of the nuitl ant.l thc shorleSl !)ide is a)C)IlJ: Lhe
positive x axis. f ind tbc centTC of pressure of the triangle.

Fluid surface

Scmicin.:le

20. An equilateral lrianglcofsiclclcnglh t whcnoncct.lgcis horiwmal

n.nd in the smf ac-c

24. A square plate or side Jengtb 2 m has one side oo tbc bottom of
a swimming pool 3 111 deep. The plate is inclined at an angle of .rr /4
rJdians with I he bollom of Ute pool so that ils h oriront:~J upper edge i_s
3- .J2 111 b.::IO\\' the .surface. Find the forc.:e due 10 water pressure on
each side of the plate.
25. A lhin lrhu'lgular piece of wood wi11"1 !tides of lengths 2 rn~ 2m.
und 3 01 floats i.n a po~1.d. A pi.ccc of oopc is tied to lhc vc~tcx op(>OSile ttM~ longest s i ~te. A t(.X:k is lhen auct<:hed to 1he other e1ld of
the cope and lowered into the water. Whetl tbe rock sits on the bottom (and the rope is (aul). ihc longest side of U1c wood still Onnls in
lbe surt';~e-c of Ihe w:u-er, btU lhe opposite venex is I m below lhe sur~

face. Find the force due to water pressure on each side of the piece of
wood.

lK. A c ircle with rlXlius r wben its ccntrc is h > r units below the t 26. Show ch!U lhc centre of pres:sure of a plane surface is always below
surfm.::c
its ccnlroill.

113.5 Centres of Mass and Moments of Inertia


We now show how double i nlegr<tls C(tll be used 10 repl ace definite i ntegr.:tls in ct~ lcu lating first

01001CntS, centres of mass. and moments of inertia of thin plates. Consider a thin plate with
mass per UJl it area p such as 1hat in Figure 13.26. Note that, unlike our d iscussion in Section

Mijlcll);lj,~

Double
imegt.tls are \'"et)' efflcienf in c-.&1
cul::trins nl0men1s and centres of

7.7 where we assumed p coostant, we ha"e made no such assumption here. In o ther words.

density c<Juld be a fu1Jctioo of position, p = p(x, y) .


Tite centre of mas.~ (x, Y) or the plate i~ a point at which a particle of mass M (equal to

mass

the. total mass o f the plate) has the same firsli moments a bout the x- and y-axcs as the plate
itsel f . If we dhide the pl ate i nto small areas d A , then the mass i n dA is p d A . Addi tion over
all such areas in R as each d A shrinks to a poim gi>es the mass of the plate

.\'

erdA
(X. )'}

( 13.31)

Since the tirst moment of the mass in d A about the y-axis is xp dA , it follows that the thst
moment of the emiJ'e plate about they-axis is
X

But this must be equal to the fir.;t moment of the l)drticlc of nHJSS
and hence

M t (x. D about the yaxis.


( IU2 )

lllis equation can be sohed for


once the double imegral o n the right and M lul\'e been
calculated.
Similarly, y is determined by the equation

( 13.33)

where the double integral on the right is the lir.;t momem of the plmc abou1the xa.~is.
In M)' given problem. 1hc double integrals in 13.31-13.33 arc evalua1ed by means of dou
ble ilermed imeg~~ls. For example, if we divide lhe plale in Figure 13.27 imo rectangles of
d imensions dx and dy and usc vertical strips, then we have

:Ciflil;l
y
y=il(.r )

l
Mx
l
My=
l

...........

M =

b/.htr)

\X. )')

Jy

b!.h(x)

y ='g(x)

( 13.3-la)

xp d ydx,

(I U-lb)

yp d y dx.

( 13.3-lc)

gl,.l)

pdydx,

s(x)

b!.h(x)

g(x)

0Hceagain we poi111 out dtal equations 13.34 should no1be memorized as form ulas. Indeed.
e-dch can be deri\'ed as needed. For instance, to obtain equation 13.34c, we reason that th.e first
moment of the mass in a rcc.tang lc of dimensions dx and d y at position (x . y) about thcx-axis
is yp dydx . Addition over the rectangles in a vertical strip g ives lhe firs1 momen1 or the strip
about the x -axis.
/i(<)

yp dy dx .

~(A )

and integration with respect to x now ac..h.b over all :Urips tu givt: the 11rst momt:nt of the entire
phuc fibout the x -axis. Nolo 1hn1 if p is conslnnl and inner it>lcgrotions ;Ire perfom1cd in each
of eQtHUion 13.34. the n

Mx =

{b

( ''

J. (pxyl:~;l dx "' J.
b{

.2 } '(.)

My= { P>,

lo

px ]h(x)- g(x) ]dx,


b

ilx = { :; llh(x) f - [g(x)]2 ]dx.

s (:c)

ln -

These are equa1ioos 7.35-7.37 (will1 d iffere nt names for 11le curves) . bul the simplicity of the
discussion leading to Lhedouble iLerated inLegraJs certainly demonsU'ates itS advantage over use

of the detlnite integ ral described in Sec1ion 7.7.

I EX AMPLE

13.9

Fmd the centre of l1lll.l> of a thin plate withconsumt mas:. per umt area
by the curves y = 2.t - .t 2 and )' = x! - 4.
VtJ11Cil

SOLLIIOl'\

'fnJ" ore cOO.."' ((tt( 1hi10 piO'I!t

li'-W"~

jj.

M =

- 1

c-1

2 ~l..l-x'

.rpdyd.t =
2x'

9p

9P

= - - = -.

MY=

= -p

I EX AMPLE

9p.

1!

.t((2t- A 1) - (x 2 - 4))d.t

-1

f-!

= P {T

(-1.-3)

4)]dx

-1

If the centre of mas~ of the pln~e is (X. f), then

_,

-1

= p {.rl - 2x' + 4x}l


M 'i: .,.

((2x- x 2) - (x 2

pdy d x = p l

we

For venical >trips a> hown in Figure 13.28,

_
Thus, x

p if its edl!t> are defined

x'
zJ2
- 2 + 2x _

9p

= 2

.
S111cc

12{!.._.~ 2x-.r' dx
211-'

ypdytlx=p
.l

_. - 1

12

(--lA'

- 1

+ 12t2 -

2 _..._ 1

Z1p
= -p {-.\ 1 + -ix 1 - 16.1)1 1 = --2- .

16)dx

27p I
3
find\- = - - - = - -.

2
9P
2

1 3 . 10
Fino 1he fir<l momen1 of area abou11he line

y = -2 for 1he ~ion bounded by 1he curves

x = (yl' and x = 2 - y2.


Melhod I
The lirs1 momenl aboul .v = -2 (If' n rcc1nngle of di mensions
dx nnd dy al position(.<, y) is (y + 2)dx dy (Figure 13.29). For 1he c1llire plate, then, the
required first momcnl is

SOI.li i iOI\'

I (ori2on1al
(lri~ :~re dt o<~:t'n

""''

for chi~

~,s:

n'k_'lmcl'll<~
)

1
0
_,

12-}~ (y+2) dtdy+


_,.,

= 1 (x(y + 2)l~}
_,

11/.2->'(y+2)dxdy
0

dy +

= 1\l + yl- 2y' +

,.,

1
1

{x(y

2y + 4)dy

-1

X=

( I . - I)

2-y'2

,s y -2yl + y
{::....5 +-4
3

y= - 2

17
3

+ 2))~:-'' dy

+ 4)1.

r' (-y

Jo

}o + {- -Ys - 1

3l- 2/ + 2y + 4)dy

3)"

2yl

-4 - - 3 + .l' + 4y

}I

13.5

Ccnt esofMt~SSQI!lciMOnk"f1[~t~ l'h.et1ia

919

~tcthc>d 2
Ry sym me1ry. chc centroid of thre region is somewhere along the x ~axis. Hence.
the required first mo mern is 2 A ~where A is 1he area of the re.giot atd 2 is the dis1~1nce from
y = - 2 to the centroid. Since the areu oftherregion is equally d istribtllcd about the xaxis. we
obtain the required firs mome nt as

2(2}(areaof platcabovex-axis)

11 2-y'
=
l

dxdy = 4

Y'

1t

(2 - )' 2 - / )dy

y3
4 2v- ' - -

I EXAMPLE

13. 11

Find the firs t mo me.m about the line 2.r + )' = I of a thin plate with <.~nsumtJna.~s per unit area
p if i ts edges are defined by the curves y
.< + 2 and y x 2 .

SOLUTION We could concemrate the " ""' M of the pl ate at its centre of mass (x, YJ. and
multiply M by the distance ftom 2.t + y = I to (x . )') . Bccausc calculation o f {X,)') re.qui re.s
three imegnuions. we pre fer a more d irect UJJproach . The d ista nce from 2x + )' = I 10 a
rec tangle of dimensions dx and dy at position (x, .r) (Figure 13.30 ) is given by f()nnula 1.1 6.

l2x + Y - II

12x + y - II

A"+i

J5

If we ta ke d istances to the right of the line as positive: a nd those to the le.ft as negative, the
directed distance is (2x + y - I)I J5. I! now follows dill! the fi rst momcm of the plate around
t he line 2x + y
I is

2! <+2(2x + r.y- 1) p d y d x

1
-1

.c:

VJ

1211

pr;
- (2x
v5 _, 2

}'.-+2 dx
,

(Ox + 1)2 - x'- 4xl - 2x 2 +

4x- l)dx

-t

{ l

- -p

1)

1
2

- ....!!._
2../5

+y -

- (3.<

2../5 9

X$

I) - -

'

2<;
-

+ 2x 2 -

12
- 1

36../5p
25
MOJiciii.I4F80t.

Vcrtic.al :;ui p:>. are

chu~n (t> ~nd

tbt

fN'It Rll.)fi'I<'IU

uf this plate

y= - 2x+ l

This example illus trates that d ouble integrals aJiow us l O c-alculate first m omentS about arbitrary
lines, unlike definite integrals, which res uict tlrst moments to holi.zontal a nd vertical lines.

Calculating momen ts of ine rtia (second rnoments) of chin plate$ is as easy a.s calculating
first moments if we use double i ntegrals. In p urticu lur. the mas:.. in area d A in Figure 13.26
1
is pdA: thus its moments o f i ncrtiu about the.( - and y-axcs arc. respectively. y pc/A und
:r 2(J d A. Momems of inertiu of the e ntire plate aboutlhc .\-and )' axe:-. are therefore given by
the double i ntegrals:

'The product n>oment o f inertia o f 1his pla>e wi th respect to the x - ond )-axes is defined as

!,_,. =

Jl

xyp dA .

(13 .36 )

Unlike / 1 and /)', whit.h am always positi\'e, 111, c an be positi\'e, neg;_lti,e, or zero.
For a pl ate such as that shown i n Figure 13.3 1, we eval uate these double integrals by means
o f double iterated integral s with respect to x a o1d y:

( IJ ..l7 <~ )

(I.U7hl

1,,. =

bf. h(.)

llmJ

[J

,)

xypdxtly.

(I ~ .17 c )

111)

ElE

b ----. - - - - - - - . , . .

xg(y) /

' e h (y)

J.r:o::or:f'rn iirn._:JJy

/-

(.,y)
Q

_ _____
L __ _ _ _ __

_,

I EXAMPLE

13. 12

Fiod 1he morneots of inen ia about the X and y-axes a nd the product moment of inertia of a
thin pla te with constant mass per u nit area p ir its edges are defined by the curves y = ..t 3

y = J2 - x , and x = 0.

13.5 CnlreS of Mass :wk'l MOmtnLS of Ir.ttin

I n!l!l

,;r1ips ;lrt

d~n

rnen1s of

lneni;~

I \'c.'lti~l
10 cakulou e n~

.SOLl TfON

~hown

\Vith venica l :-:trip!-: as

i11 Figure

1 3.J2 ~ the momc.n ts

921

of inertia are

of thi' plate

f~-=

= (!_

~ -~(2- x)'/2 -

_. lo} I = 16,/2 - 5 p .

30

10 "

(.<. y)

y = ..-3

'

If we set a

/>' =

=2

p1

~\'

in 1he

fin~t lerm .

(2- u)1 .j/i(- du) - /)

l ,y

f'j./
lo

2
-

xypdydx

.1J

X
{

p ['

lv

>

= /) { (4./ii - 4u 3il + IIS/l )dlt


0

] ,

{-X)_'l}../Z-< dx =
2

J. j

p
2

p
--

(2x - x - x ) dx

13p

48

I EXAMP L E

13. 13
Fi nd the second moments o f urea aboul the lines y = - I and .x + y

by the curves .r

= y 2 and x

= 2y.

I of the region bounded

SOLt.:"llON The second moment of area 11oout the line )' = - I (Figure 13.33) is
2
[

f~-' (y + 1) 2 dx dy

fo l r

f\r()'-"- 1)!1::~ dy

fo

f \y

fo

+ 1) 2(2)' -y2) d)'

= [<- /+3y1 + 2y) dy= {- ): + / + ylJ: =

x + y= l

922

Ch:.pttr 13 Mulliple l nlcy.tl~

Since the undirected distance from x + y =

.1

to (x,

y) is lx + y- II/ .fi. the second mo111<nt

a1x>u1 1his line js

2~2Y
y
, (x +J2-

. o ,..

)2dx dy =

1
2{-(x
+y -

-11
2o

1Jo( [(3y 6

I)3

I { I ,

=6

1)

12(,y - l l

}2y, dy

- l - 3l + Sy3 y1

3y

3y 2

5y 4

y6

+ I] dy

-7 - 2 + 4 - 2

2
}
+y o

62

= 21

Principal Axes and Principal Moments of lnertia


Consider finding the line.~ through the origin about which the momenL~ of inertia of the plate in
Figure 13.34 are largest and smnllcst relative to a ll other lines through the origin. Using distance
fonnula 1.16. we can say that the moment of inenia abuut any line y = mx is

f(m)= J'f( ly -mx l)~pdA=


}R J m 2 +
I
?

m- +I

(1, - 2111/,,.

j "f( / - 2mxy+m 2 x 1)p dA


} ll

+ 111 l f y) .

ine-ni;~

Mt'l4UCI'II'S of

M:UCOJII;Ii4

1
2
Ill + I

(13.38)

ha\'C maximum and minimum \1llnc'

~bo111

prir.cip;al

:J'(

Critical point> of f (m) are given by

dl

o -- -dm --

- 2m
(m 2

I) 2

(/, - 2mfxy

+ lll"fy) +

m2

+ I

(- 2f,y

= -2m(/, - 2m/.,y + m1 l y) + (m 2 + l) ( - 2/,y


(m l

= 2l(m 2

1) 1, 1.
(ml

ml,

+ 2mfy)

+ 2m f y)

+ !)2

+ mfyl

1)2

1)3.39)
Lines with these slopes are perpendicular. as is e asily seen by sening a = {/_, - l y)/(21xy ).
and noting that (a +
+ a2)(a - '-"I + a2) = a2 - ( 1 + a2 ) = - I. They are called
the principal axes of the plate at the 01igin. Substitution of 13.39 into 13.38 leads to messy

'-"1

13.5

Ccnlre; of Mass :md

~ lo;unems Qf Inertia

923

algebra. Instead, we note that f xym 2 + (/y - l x ) 111 - l .,y = 0 can be expressed in the fom1
(m 2 + 1)/Ay + m(/y - /, ) = 2l,J. Substituting this into 13.38 gives the moments of inertia
around Lhe print:ip;:tl axes:

= m2+

11., - m(m- + 1), 1


1

= m2 +
=

lx

m ( fy- lx) + m-J1 ]

1(m 2 + 1) /,, - 111 (1111 + l)lxy]

1,, - mlxy

+ 11
2

'F

--

"

ly )i + 4(/x,r l 2 ]

'-r / Ux

[ 1, -

( 1., ; ' y )

+ u.~v.

(13.'10)

That these yiel d (i-~bsolule) maximum anclmlnimum value!O tUr T(m. ) is shown in F.xerdse 42.

They are culled the principal moments of inertia of the plate about the origin; they are moments
or inertia abou1 1he principal axes.

I EXAMPLE 13.14
Show that princip.:tl monleHs of inerti a about the orig.i l for the lllliform rect.algular plate i n

Figure 13.35 are / and l y.

SOLUTION Because of the symmeny of the pl,.te about the axes, l,.Y = 0, in which case
equation 13.39 docs not define princi11nl directions. If we return to function 13.38. we fi nd that

l(m)

If

1:~

ly

lx - ly

m2

+I .

> l y (as is the case in Figure 13.35 ), then this is an even function with respect tom,

decreasi ng from / (0) = 1., to lim,. _ 00 I = ly: that is. principal moments of inertia are fx
and I y If l x < l.v. then this even function increases from l (0)
l,, to lim,, _"" l
I,.. and
once <lSlin Tx and f y are principal moments o f ine.n ia.

ljtfJIIIWEEI.W

Ptincipal moments

u(

Notice that if the rectangle is a square (a

inc11ia

t~f

fO:: tau~utat' plate abou~

= b ). then l x =

its

CCIHfc:

1y and the moment of inenia of the

pi aLe abouL every line through the origin ha.s the same value. In Lhis case we say that every pair

of perpendicular lines through the origin constitutes a pair of p1incipal axes.

EXERCISES 13.5

In Ext."f('i!i(.'S 1- 10 llnd theccntroidofthc rcgion boun<kd by thcCUivcs.

= )' + 2. X = .Y1

I. X

2. )' = 8 - 2x 2

28. Find the moment ur incrciu of a uniti.mn cclwlg_ul:u place <7 units
long and IJ uni l~ wide ubouL3 line thnlUSh the c.cntre ot ti'IC pl:l.l :;;nd
pcrpcndicubr I() the JJiatc.

,v + x 2 = 4
In Ex:c rcisc ~ 19'-31 fi11d Lhc sa.'Oild momcnl of area o f the region
bow lded by the c:urv es ~1bO\I t (he line.

J. )' = X2 - I , )' +(x + 1) 1 = 0


4. .r

+ y = 5 .ry = 4

5. y

= c", ,\' = 0. X = 0, .l: ;;; I

6. y= ./4 - .r' . y= x . .r =O
7. y

= 1/ (.r -

1) . y

+ 9)' = 36 about .l ' = - 2


3(1 .< = y'. x + )' = 2 abouiX = 29. 4x '

= I , )' = 2.< = 0

31. )'

= ../n2 - x 2,

.'o'

= tJ,

.\" = a (11 > 0) :abo ut the x -uxi10

8. .t = 4y- 4y 1 ,1' =y+J. y = I, ) = 0

9. .1'

=lx' -

II, y

In F.xcrdses 32-35 find the lirst and s-:xnnd nlOmcnts of area of the
rcgi(u1 bountled hy the curves !lbout the line.

=2

10. )' = x . y=2.x . 2y =x + 3


.,. 32. X

= y1

JJ, x = y 2 .r

In E:tcrci.scs 11- 15 fi nd 1Ju: MX:ond nlOmcnt of llTCQ o f l hc region

boumkxl by the """ "' bout the line.

3..1. ,V

II. y = x 2 , y = x l about the y o.1xis

* 35. ,\~ = 2 -

=x. )' = 2.x + 4, )' = 0 <lbotnchc x -axis


13. y =x Zy = 2 + 4 ab<.1ut J' = 0
12. )'

X =

.J.'

l//.14 + 12y1, .r = 0, J

= 1/ 2, J =

I tobool ,1'

=0

16. t'lr.d the firSt nlOmc nl t't.bou t the line y


- 2 of a thin ptutc o f
conuam rna...~ per u11it area p if its edges are de tined by lhe (Ut'\'<:S
)' = 2 - 2x2 """ y = .r 2 - I.

=2

+ y = 2 about )' - x

= J 2 about 2.x + )' ;;;; 3

.\' 2 ~' = lXI ~~1:>0\U. )'

=X

always zero.
39. Suppose th.1t 1..: and / ), arc RlOillCOl$ of inertia of u thin pltuc with
commml rmlss per unil area about the .x- and y-uc~. Lcl und
be

r;

= rZ

= -2y,

th:~ t for a p1:itCwith con~twlt nu13~ per unit urea. the l" 'vtloo
moment or inert iu with rcs.pcct to the princip1laxcs through a point is

momcni.S of incrt iu or the pltuc aOOut.any Olhcr

line:-; through th~.: origin. Shorw that

~t- 19. x =- y 2 , x+ ~+ 2 = 0
.t

+ )' =

38. Sh()w

In E.\crcist8 17- 20 fhld the pi'Odllct moment or incn la with respect to

+ 20 .

,\'

abuu t X

.. 37. Shtl\v thut the nbMllutc value of the prod t~ ct moment orincflia with
I'C8J)CCt to the ,x ~ <:~;nd y -:sxc5 of ;1pl;:llc wi\h const:tnt mass per unit arc:t.
is nlways less Lhan onchulr the ~u m of the momcnls of incniu about
t he .\ and )';;.xes.

the ;r- and J-ax<,;s for the plate deli ned by the a nves if it has constu.n1
nt.a~ per uniC ;area p.
17. :c z + .\'z

= x 3

= .\'

Jti. A. 1rianguh-1r" plate with const;lnt lllil!l:S per unit area p i;; bounded
by lhc coon.linat.c u cs 11.'\tl the line h x + by = !Jb, whcK h and b ;.t rc
positi ve coo;;l<inl$. Fincl i t:~: prcd uc:t nltun ent~ of incl1ja iibo ut (3) the X
and ,\ 'axes and (b) the axes through the ccnlrc of m:tss ptlt1.11cl to the
.'(- and )' -taxes.

14. .v =x2 - 4. r = l \ - .t 2 olbuut x = - 2


15.

2, y

)' = -.\,X+ .1_\'

p-:~ir

r;,

of p:rpcndiculu.r

r; t; = f.r + /).
T

-c. 40. Find Ihe principal axes and principal nlOrncnts of incrtJa about the
origin fottht! uni fonn squmc plt~tc bounded by the lines X = 0. )' ;;;: 0.

+2 = 0

x = a, )' = a.
In EJ.crc:lscs 2 1- 27 fi nd the centroid of the
cur\c:-..
" 11.
4

ltgl()n

bounded by the

" 4 1. Finc.l the prittCipal nx.cs utld prit\Cipal n )()m~nt$ ofincttiti Jbouttbc
origin l'o 11"11..: unll'o nu rectangular phllc bounded by the li ne ~> X
0.

)' = 0 . .( ~ 11. J

.r = _,')-TI. >' = x . y = o

ll. }'

+ .t 2

= 0,

= )'

+ 2, X + )' + 2 = o. )'

= 2 (ubovc

+ ~.1' = 48, (X - Z) + y' = I


"= lnx ' ~I' + y;.'X=!
=0 x =2
1

24. 3.<

25.

'

26. y=J2 -x , 15y= x'-4

* 27.

y=x J I - x 2 , x ~ O and thcx -axis

b.whcrc./1 > b > 0.

42. ShO\v thu.c the values of I in equation 13.40 a~. indeed maxim um

nn<l minimum valucsort (m) as defined by i3.38 ior - oo < m < oo.

y +x' = O)
23. y~ ~ .r'' ( l - .r ' ) tright loop)

_., .t}. Suppose thdt a chin J)hlte with C.'Onstant nluss I)CI' unit tU'Cd is sym
metric about u l ine t: :1nd P i$ n ,,ni nl on t . Show liH~I th c J::t'Oduct
mOuJt::nt of inertia of the phne aboul f. and a line through P pcrpendicuhw LO vanjshes.

44. Show that if a thin plate with t.'Onstant mass per tmic area has an
axis of symmcu-y. then the axis or symmclry muse be a principaJ axis
about any point -on tJu:: line.

I3.6

4.S. Show thai if 0 hi lhc ilng.k o r i.nc.:lin<Jtion of~ prir!'l;ipa.l U-<;i~ (a-bout

. .

the t.V1g .n) , then t:tn 2(1

= ---.
ly - /"

.19. Suppo$(: lh~ 3lhin pl::ttc with CO:lSI:Lnl nuss per unit Bn."'n oc~upics
n rcgi011 R of the: .t)'p1anc. Let x ~ x 1 ;uld x = Xz be any two
\"Cnical line~ ttnd I"' and I": be moments of inenia of the plate about

46. Pm\'C the theorem of Pappus: I f a pl.:lne :trea i ~ l"C"\'Olved :\bout~


coplanar uxitt not crossing the area. the volume gcncr.u.cd is equal to
1hc pt\)(.JUCI of1hc area and the drcwnfcrc.ncc of the circle tlc~ribt'd by
lhe ttntroid of the area.

4X. Pnwc lhe fWITOJIRI 11xis 1heorem for thi n plates: The moment of
incrti::t of a lhin pl!ltC (wilh constant n:JSs. per unit a.rt3) with n!Spcct to
lln)' CQpJanur line is equal h>the lll()lt"ICIU ur incrtiil with rQp<:Cl lQ the

925

panUcllinc: through the cctttrc. ur 11'\aSS pl~ the II~ multiplied by ~he
~uarc of the diqancc between the lines.

2/ T)'

4 7. A thin llat plate Qf arc:1 A is immersed vcrt.K::tUy in 3 Ruid wilh


density fl. Show that the lOtul fOfC'c (due to flu id pc"s..wrc) on each side
of the plate is equal to the product of 9.8t . A . p. and tloe dep.h or the
centroid of the plate helow the s,urfacc of 1hc lluid. Usc 1his re.sull 10
Hnd the forces in SQ nlc of the problcm_s in Exercises 13.4. say l. 3, 4, 5~
7. 11. 12. 13. 16. and 17. Fortho:s<; P"'blco~.> invol,ing uianglc rocull
the n!(lllt of Exercise 4;\ in Section 7.7 or Exc..'1'd5e 47 iR Section 11 .3.

Surf~c:: Ate::-~

the~ lil')e~.

Sho\ll tl-.at

where .H i.;;lhe mas~ of the pl<itc 3nd Xi~ the X..aiO(dinalc of iL;:;<.-cnlrc
of trut.<t.~t:. ()roes lhi-' re$uh reduce to the p!ir::allel !I.:< is theoc'cm of' Exerci,;.e
48 wbcn on.c of the:: lioes p~s through the c~ntrc of muss?
x

SO. Suppose that a thin pla1e with constant mass per unit arta occupies
:\regio n R of the X)'phme, :md (:T, j} is its!'t'nlre of m~,;. Let I"' be
tJte produc.t mon~nt o fi nertin o rthePllllc about the X . and )'AXC$ d!ld
I !lJ' be the pux.hK.1.. nnncntofineniaofthe pl<1tcabotlt the lines lbrough
(X, y) parallel to the x - andy -axe~. Verify that /IY
l xy - MX)i,
whe-re M is the tn::t:S$ of the plate. ( l'his is called the parallel ilxjs
theorem for product momcn~ of inertia.)

113.6 Surface Area


To lind the length of o eurvc in Section 7.3 we approximntcd the curve by tangent line scg:n'lenL-s.
To ftn<lthe rea or a surface we follow a similar procedure hy l~lnlXimating the ~urface wilh
tangential plnncs. In particular, consider finding the area of a smooth surface S given !hat every
vertical line tMt intersect< the surface does so in exartly o ne point (Figure 13.36). If Sx,. is the
region in the: xy planeo mo which S projccLo;;, we di\'idc S,y into 11 subregions with arc..;s tJ.A,
in any fashion whalsoever, and choose a point (.t;, y;) in each l\A;. Atlhe point (x;, }';, Z;)

on the surface S that projects onto (x, y, ). we draw the !angent plane to S. Suppose we now
projecl L\A ; upward onto Sand onto the tangent plane at (x;, Yi z_;) and denote the'!'e projected

areas by t.S, and t.Sr, , respectively.

MijlciilriMFWI.'

TangcnL plane

Ate:.IC...'fl

~v:<d pbnre u rd..~l "' IU


Pf\IJ1._.. WI ht A} pf1r..c ftfus.b
lhe ..,.(

~ow ~S.,, i~ un appn:>'<imation to OS; and as long '-' 6A 1 is ~nwll. a reaM)nably good
appro'<.tnWton In fact , the ~m.111u 11 A,, the better the approx1n~aron \\'e there(o~ define the
ar uf
follow<

s"'

btr~n ~ ,..,_~

area of S

= n-l in1"''t; "L....,; tJ.Sn

{1 ) 4'

i -1

"here in 1aking the linlit we c.lernllnd that each ~ A1 shrink to a point. \Ve have thct"ef'ore defined
area on n curved :courface in tcnn.ii of flm areas Orl tangent plnnc.' to the: surface. The advamagc
of Lhi~ clcfirtic ion i~ thut we: C'an 01lcul:uc ~Sr, in tcmu o f ~A I . 1b M:C: how. we denote by ii,
the: unitnom1:al vector to S au (.rr, Yt. Z;) with positive: z-<::ompo nent, and by Yi the acute angle
between il.i unc.J k. Now AST/ project.o;; onto\;\; , anll y; is theucutc: Rllg le betwe.:nthe planes
containing 6 A; und 6Sr1 (Figure 13.37). It follows thut ll A, .md 6.S,, a1~ rt lat~U by

(sec E>erdse 28 in Section 11.5). N<Xe tlu11 if ll.Sn is hori>011Uil. then y1 0 and ll.Sr, =
tJ.A;: and if tJ.Sro t<lds tnv.ard the 'enJ<al (r; __. rr / 2). then 6.Sr1 becomes 'ery b rgc for
lhed ll.A,.

s.,

BccuuM: the surface: l't'Ojecu; in u one-co-one fashion onto the~~


in lh\! .t) phux. \\C
S in the form : = f(x. )'). A 'CC10t nonndl 10 S at M) point ~
th<rcfore

can take the: rquatkl ((w

v (~ hence

ar - ill
)
r(.r. y)) = (-ax.
a)' ' 1 ;

__
a.r __
af

ax' ily '

n = --;=:"======:=======2

(a.r)l
(aJ)
a..
ay

+
Si nce n,. k = n,ll klct~'

(--a: _a:

)
1

ax ' a,,.

)
1

=r==============o

(az
):
1 (-a:)l
ax
a>'
+

y, = <.:o:.y,. it foliOW> tlul

(0\)'j

= ii, k =

Jl- :!<)',) ... :~(.r,.y,)

When we substi tute thi<cxp~ion into 13.42. \VCOblain the re,ull that area ASr, on the tangent
plane to~ = f(x. ,1') a t the point (.r,, y, . ~;) is related tollS ptoJC.:tion t.A , in the .<)'plane
accordil1810

FnrmuIa l.l4 I (u r I he urc:n of S cun now be. w riucn in the form


arcaofS =

.~

L ,/1+~~(xl.)'t)+~~(.r,,
)/).6A,
.. ,J-0
.
lim

( 13.4-1 )

1: 1

"here the surnmatitl i> crrie<.l out o'er all areas 6A; ins., a.>e.tch lJ.A; shrinks to a poim.
Butlhb i> the dclinilion of the <.looble integral of the functoon j 1 + (il~{ax) 1 - (atj3yf
O\'er the region Sx1 In Other words. on the basis of formula 13.41, areas on $Urfaces can be
calculated acco<ing to

(I H5l

N01e the analogy between equations 13.45 and 7.15.

In equation 7.15 we think of


vI+
as a small length along a curve C : y = f(x ) that projects onto the
+ (dy/dx )' dx is along the tangent line to C. but we
length dx along the x -axis. In fact,
think of it as along C itself (see Section I I. II ). The tOiallength of C is then found by adding
over all projections of C from .r = a to x = b. Similarly, in eqmuion 13.45 we think of
J I + (il</ llx)2 + (Clt/ <ly) 2 d A "'a small area on a surfaceS : ~ = f(x, y) that projects
omo the area d A in the x y-plane. It is. in fact, a small area on the tangem plane to S. but it is
usually easier to think or it as being on S iL..elf. The total area or S is then the addition over the
projectionS,, of S.
Note. too. that when Sis smooth, a:.;aJt and arjay arc continuous, thus guaranteeing
ex istcnceofdoublc integml I3.45. If S is piecewise smOOth. we divide it into smooth subsurfaces
and integrJte over e:1ch piece separately.

(dyfdx ) 2 dx

m!1
:lrta q(

w~

lhis <;nrf:lce

l}i\ide th~
par1-.

ll'ltO IW()

S, nnd S: v.'ilh curve (" and lind


both

are..~

/t

This discussion has been based o n the assumption that S projc<:ts o ne-to-one onto some
regions,)' in the .\')'-plane. If this condition is not mel, one po~<iibility t~ to subdhidc into
pans. each or which J>rojects one-to-one 01110 the xy-plane. The tOtal area or S is then the sum
ofthe areas of its jl'J.rts. For instance, to find the area or the surfaceS in Figure 13.38, we could

subdivide S into St and S2 along the curve C. The area of Sis then the sum of the areas of S 1
nd ~.each of which projects oncto-Qne onto the x y-plane.
i\ ltemmively. we could note that there is nothing sacred about projecting S onto the .ty planc. We could develop similar results if S projects one-to-one onto either the y~- or the
x z -plane. If S,; and S., represent these J>rojections. then

arcaofS =

area orS

/,)1+ (*)\ (*Y


/L,jr + G~Y + G~Y

dA .

!IJ..I<ial

dA .

( U.-16bl

I EXAMPLE 1 3. 1 5
[]

To find
dt< i:.leJ uf .~ + Z> + Jz c: 6 in
the fiN octant. projecl il OIHO the

.t')'pbnc

Find the are. or that part of the plane X

+ 2y + 3~ =

6 in the lirst octant.

SOl UTTOI\" Titi~ arcil projccL< one-to-one onto the triangular are:o S..1 in the x )'plane bounded
by the lines x = 0, )' = 0. x + 2y = 6 (Figltrc 13.39). Since: = 2- .t/3- 2y/ 3,

We could also use formula 11.42.


.r

I EXAMPLE

13. 16

z = x 312 that projects onto the rectangle in the xy~plane bounded


bythestraigln li nesx =O, x =Z,y = l,andy=J.
Find Lhe area of Lhe surface

928

Ch ap1t~ 1

I J Multipi~ l nt..~l-4

SOLIJTIO'i
that

are l =

Since the surface projects one-to-one 0 1110 the rectangle (Figure 13.4()). we find

f'Jrsu I + (-.iJz)1+ (i-J:)1 Jls


dA =

ily

,)x

12

1'!,3

1'

- -J
J 4 + 9x dydx = -I
ly J4 + 9x l: dx =
../4 + 9xdx
2 o ,
2 u
o
-

~(4

27

MiiHII,IAfW,.,

+ 9x) 3' 2 }

To 6nd lhc

2
0

~(22./228) .
27

*""' ur :;:; .m

cbl.l\'C the l'('(,.'tangle. projc<:l it OOIU Ihe- rlllng.k

2
)'

I EXAMPLE

13.17

Fi.nd the area of the c.o ne )'

= .Jxl + z2 to the left of the plane y =

l.

SOLU110'i l\Ietbod 1
The surface pr<Jje<:ts one-to-one omo the inwrio r of the circle
x 2 + ~ 2 = I in the x ~-plane (Figure I 3.41a). S ince the area of the surface is fourti me>thm in
the first octant. if we let S ,: be the quarter-circle x1 + ~ ~ I. x ~ 0, z ~ 0 . then

xl

+ Z2

"2
'

+ Z2

dA

JJ.

S,,

hdA

~IGURE 13.-41
To
fiod the surface. area of 1he cone.
projecl it ooto tho! x: pl.a~

To
rind d)t ~ud8<'C area of Ihe cone.
projtcl it ()ntQ the X .\' J)lane

ye \ xl+z:Z

~ let h od

2
Suppose ins1end 1ha1 we projec1 1ha1 pnn of lhe surfnce in 1he firs1oc1am omo
1he 1riangle 5,1 in Figure 13.41 b. We wri1e 1he equa1ion of 1his pan of 1he surface in 1he form
z = Jy2 x2 and calcula1e

=4/t ( )
fl., I+ , .r' , +
- .\'

I '

J.r' -x2

2+ (

\1

'

)2dA

Jy2 - x 2

yl o dA = 4,/2
yl - x-

y- - x-

!1 )'

Srr jy2 -

x2

dA .

To e<aluate 1his double integral, i1is ad<antageous to imegmte firs1 with respeclto y:

area = 4../2

111jy )'
o,

- x2

dydx = 4,/2

11
o

fJy 1 - x 2l!dx

= 4../21 Jr- xldx .

If we now set x =sinO , 1hen dx =cosO dO , nnd

area =

4,/2

J.

:t/2cosO cosO dO =

4,/2

J.:r/l (I + cos 20)dfJ

= 2Ji. { 8+ 2I sin 28 } "'' = hrr.


0

930

~htliple l"ltgnl ~

Cb.aptcr IJ

EXERCISES 13.6

In Exercises 1-6 find Jhe area rcquirc.J.


t. The area of 2.r + 3y + 6z

= I in the li rsl octanl

2. The area of .r + 2y - 3z + 4 = 0 for which x !> 0. y !> 0 and

z~O

J. 1lte area of t = l - 4/ x 1 +)'! above L.he .l')'l>lunc

12. The area or z


y =I - x 1

= In (I+ x + y) in the first oe1an1 cut orr by

13. Findthcarcaoflhcsurfacez = lnx tbaq)fojesonto LhereccangJe


ioHhc .ryplane bounded by lhc tines x = I, .r = 2. y = 0, y = 2.
14. Vc1'iJ'y that the :trCl:l C>f the <:urvetl portion of a right-circular cone
rudius r und hcig.hl h is :r rJrl + ),'2 .

t1(

4. Thcareaof t = J2xy cutoutbythcplancsx = l ,x = 2,)' = I,

y=3

* 5. The area in Jhe first octam cut om from the surface z = x + )'by
the p l ane,~

In Exerc1scs 15- 19 &el up. bul do no1cv,:tluaLc:, double iterutcd integrals


ta find the rcqui!l'cd ;area.

+ 2y = 4

* 6. The area of z = x''' + y''' in the ftrsJ oct am c-ut oJT by the plane * 15. 'flle area of .v =.r' + z' cui olfby y + z = 1
x + y=l
* 16. Tile Hrca 01' ,V = zl + ).' inside + y 2= I
.A.'

lo Excn;iscs 7-12 set up. but do not C\'nluatc, double iterated integrals
to find the f\."quircd 31'('-3..
2

~ 7. The area of x

+ y +z

-. s. The are~~ of 4:r

=y + z

2
2

= 2 inside the cone z = / x! + y


cot ofT by x

zz =

10. The urea of z = (x1 + y')' below z = 4

II. The area of y = I -

* 11. TI>e area or .v' = z + x' inside x' + y' =

18. The area or z = x' + .1'' thai is in 1hc ftrst oc1ant and bcl\\'oen the
l>lanes x + y = 1 and x + ,1' 2

19. The area of x' + y' =


Z= 4

* 20.

x'- 3z2 to the right of ~"' xz phmc

=4

. _ 9. Tbc area in the ftrst octant cut from y = .\z by the cylinder x 2 +

z' + 1 bclwe<n the planes z =

Find Jhc area of thai part of lllC >urfacc


== 2. y = 0, and y = x.

I .,._,

z = 2x' + 3y boundctl

by lhc planes x

113.7 Double Iterated Integrals in Polar Coordinates


FIGURE 13.42
Doub1c
intc-grats in poln C\X)I'\!i.nat~ o'c-r

this uru arc: <:Qn\'CJictn

So far we ha"e used only double ilermed integrals in x and y to e':llume double integrals. But
for some problems this is not convcnic.nl. Fo r instance~ the double integral of a continuous
func1ion f(x. y) over the region R in Figure 13.42 requires three double iterated imegr-<~ls in
.x and y. In other words) a subdivision of R into rectangles by coordina(e lines .r = constant
and y = cons.tant is simply not comcn..icnl ror this region. For such an a~ polar coordinates
are more suitable.
Polar coordinates with the orig in as pole. :and the positive x -ax is as po lar axis are defined
by
x = reosO.
y = r siu()
(see Section 9.2). We wish to obtain double iterated integrals in polar coordinules that
rc:pn;,;cnt the double intogr-1 of /(.r , y) ovenl>c region R in J"igure 13.42. To du this we reiUn>
to definition 13.3 for a double integral, and choose a subdivision of R into subareas con,enient
to polarcoonlinates. When using Canes ian coon.linatcs we drew coordinate line~ x = cunMant
and)' = constanl. \Vhen using polar coordina 1es we draw coordinme curves,. = cons1am and
() =constant. In panicular. we subdivide R by a ne1work of 11 + I circles r = r,. where

a
and m

= ro <

+ J radial lines 8 =

r1 < r2 < <

'n - 1

< r,

= b.

8j. where

a = 80 < 111 < 81 < < II.,_, < II., = {3

13.7

oou.b!e lteltltt>d lnll!gmls tn l'ol~<..:oonlinl11eJ

= ,.,_,

&31

(Figure 13.43a). If f:l A;; represents the area bounded by tho circles r
nnd r
r, and
the radialli ne.s 8
Oj- o and 8 Oj (Figure 13.431>), then it is waig htf orward to show that

( 1\.47)
Our next hlSk in usi ng equation I ~.Horthc double i ntegrnl of .f(x. y ) over

R is to choose

a star po int in each f:l A;;. If we select

(r, .

IIJI!1!1-I3!
Uuo

St.lbgioo ~

on =
(

Oi\i5iuo of 1t:gion
w iug pol..- counlinates

r;

+ r 1_ 1
2

(I)

. } .

elen1C111 u:.iog pol:ar courdin1b

then

and by equation 13.3,

J l f (x , y)d A =

=
H we UlkC the l i mi t fir~l :IS II f:l r;ll
itcrmed i ntegral

-+ 0 and thCll as 1<11.9;11 -+ 0. we o blllin the double

(1 1.48)
Re1ersing the order of taking limits reverses the order of the ite rated integrals:

J l f(x,y)dA =

J.bi~ f(rcosli,rs inli)rdlidr.

(13.49)

932

Chap-.er J3 Multiple l ntegr.~ls

For !he region R of Figure l3.43a. then, there are two double iterated inlegrals in polar coordinates reprcseming the double imcgral of .f (x. y) over R.
We have interpreted double iterated integrals in Cancsian coordinates as integrations OI'Cr
horizontal or venical strips. Double iterated integrals in polar coordinates can also be imerpreted
geometrically. Take, for instance, equation 13.48. A double iterated integrnl in polar coordi nates
implies a subdivision of the region R imo areas as shown in Figure 13.44. Let us denote small
variations in rand 9 for a representative piece of area at position (r, 9) by dr and d9. If dr
and d9 are ,ery small (as is implied in the definition of the double integral), then this piece of
area is almost rectangular with an approximate area of (r d(l) dr. In polar coordinates, then,
we think of d A in equation 13.48 as bei11g replaced by

dA = rdrdiJ.

( 13.50)

Each such area at (r , 9) is multiplied by the value of .f (x , y) at (r, 9) to gi\'e the product

j(r cosiJ, r sin O)r dr dO.


MJUIIIi

Attil e\ernent in p(llar coon.lina1es e'\:-

l>l'C"-J~.etl in terms of diffc:rcntial\

i\ dA = rdrt!O

Mllil tiL-" @jfJ;s II


nnd 0 :~ dd,. first imide

1)ouble iter.lled inregral \lo'ilh respect co r


:1

wedge, and then over Jll ....-edge<-

Length =dr

r = II

TI1c inner imcgral

J.bf(r cos9, r sinli)r dr d9


with respect to r holds 8 con~ta11t and is therefore interpreted as a summation over the small
area< in a wedge d8 from r = a to r = b. 'The 8 -imegrationthen adds over all wedges starting
at8 = a and ending at fJ = fl. Limits on 8 therefore idemify positions of llrstand last wedges.
If the order of integration is rev~ed (equation 13.49), then the inner intcgrol

id

f(rcos9 , ,. siniJ)r dO dr

holds r constant. We interpret Lhis a-. an addiLion over the small areas in a ring d r . where the

limi1s indicate that each and every ringstans on the curve(! = a and ends on the curve(! = {3.
11\e outer ritneg.ration is an addition over aU rings with the first ring at r = a and tbe last at
r = b.
Double iterated integrals in polar coordinates for more general regionsare now quite simple.
For the region R of Figure 13.45,

Ji f(x,y)d A =

P1 11(0)

1
"

g(Ol

f(r cos B. r sin IJ)r d r dfJ.

I EXAMPLE

13.18

Evaluate the double iterated integral

1'1'1-X'+x yl dy dx.
0

SOLUTIO:-\ The limiL" identify the region of integration as the inte-r ior of the semicircle in
Figure 13.46a. Tile imegrand :,uggc..')ts an interpretation of the integral as the second momcm
or area or thi s semicircle aoouo ohe x-ax is. S ince ohe semicircle R in Figure 13.46b has exaclly
the same second moment about the .r axis, we can state that

1~

{r
4

1~

sin1 11} l/l dll = .!._


sin1 8 dfJ
o
64 o

= - I 1~ (l
128

sin21J
cos21J)d9 = - I { IJ- - }" =
128

.nrn,

;nQih fjn. Li mit~>


i11J..icute that the ~:giuu C)( inttp;1
tioo i~ the scmic:iocle

rr
128

I [ j raTJ"il"31'Jj fTlrn I lmeg.r.aJion of X~ 0\1:'( this semici.rcle


j:i\'e::. the ~me rC)tJII

I EXAMPLE 13.19
Liir l!LJ
of the

f)

CtllltOid

Fi m.l lhc ccntroitl or the region

~cmic:-i n:ular anooht~

X~ + y~ = b~

)'

SOLU'I IO:\

or
r=

Ay =

Evidently,

R in Figure 13.47.

x = 0. and the area of the region is A =

(IT b 2 - JTa 2 )f 2. Since

f"jb (rsinl!)rdrdiJ = Jof'' {r3


}~>
J sinl! " dl!

!r

}R ydA = Jo

I
3

ff

- (b - a )f-cos &lo
x~ + y2 = a2

or
r: u

it follows that y

...-..

= ) (b -a ) lT(b2 _

02

b2 +ab + a 1
31T
{l + b
4

&34

ChaptCl JJ

Multiple lmegals

I EXAMPLE 13.20

.._.
Find the area of ~>at portion of the sphere x2

+ y 2 + z2 =

2 inside the cone z =

Jx 2 + y 2

SOL1.7TION If S is that portion of tbesphere that is inside the cone and also in the first octam
(Figure 13.48). then the required area is four times th~t of S; that is,

area=

I+ (a")2
+(ar)2
4!'{
il.r
ls~)

dA ,

i)y

where s,~. is the projection of son the ,\')'plane. The curve of intersection o f the eone and the
sphere has equalions

_,.2 + l + z2

= 2,

or equivalently,

z = l.
MiiiC'jii;IWfW.f:M lohtl' ooordiJJates a(t ad\~;m t ageon' in
c<lku.lating the u.rc.a uf thai part uf J .:.plterc inside a cunv

Consequcndy,

Sxy is the interior of the quarter-circle x 2 + y 2


X

L, x ::: 0, y ::: 0. On S,

ami

:::;

so that
area=

4!'{i s"'

vl dA - 4
1+ -x2 + '-

z2

zZ

/1s,

..rl +

,.z + z2 dA

z2

=4 J'f f'f,dA=4Ji j'f


I
i s),., V2
ls~ 1 .j2- x

>tl

2 -

dA.

If we now use polar coordinates to ev-aluate this double integral, we have


1

area =

4.J2 /.i / .
0

4../2(./2 - I)

12

1
J2 -

r2

rdr diJ =

f/Z
dl}

4.J2/.~ (-/2- r 2 lbdiJ


0

4.J i.(./2-

l)i

2~<(.J2- 1).


Here i< pcrtlaps our trickiest consultation project Liquid How through a ptpe of radiU< R
i< <-ontrolled by circular valveofthesame radiu<lhat mo\cs right-to-left acro&sthe pipe
(Figure 13.49). In order to calibrate the amount of flow through the pipe. we arc being
115kcd to find a formula for the area of flow through the pipe tU a function of the position
a of the centre of the nl,c:: that is. when the centre of the 'ahe is at po.ition x =a.
\\hal i' the area in.,ic.le the pipe not CO\'ered by the \'lll\'e?
V,.ihc: 10 teSUk1

I~

111 a cin.:ular p:pc

:r

SOl l li ON Why do we claim thau his project is so tricky? It would scen11o bea simple
mauer to use a double integral in polar coordinates to find Lhe area common to the pipe
and Lhe valve and subtract this from n: R 1 . dte area o f the pipe. In plinciple th is h correct.
but selling up the double iterated integral for the common area has hidden di fttculties. We
slwll see Lhis as we proceed. ln polar coordinates, the equation for the pipe is r = R;
In Canesian coordinates. the equation for the valve is (x - a)1 + y2 = R2 . When we
c hange this to polar coordinntes, the result is

When we use the quadr.tic formula to find an exp~ci t delinition of the cuve. we obwin

Tu lind the point of intersection of the pipe and valve in the llr..t quadrant. we sc:t

R = acosfJ- J R2

Let U> c.lenote the solution b)

a 2 sin 2 9.

8=

the \ahe i> onl) >lightl) closed.

which simplifies to

Cl

C051/

= 2R.

cos- ( 2(1R) . Figure 13.50 shO\\'> the situation when

To find I he ore~ M the now lhrough 1he pipe. we soblrtlCIIhc arcn eomnton 10 pipe and
valve lrom rr R . The commm'l area is

A = 2 /.i f. R

rdr d()

a~o.'\JtiJ-JR:-u'~.,'o

o
ij

f.
1

tl

J Rl

[ R2

[R2

-a 'C111-e +

(a oosll

a!""' 11)2 ]dll

I)

11

J."[a(- cos 211) -

2aoostiJR' - a 2 -.n 2 6 - Rz +a 1 sin21/ Jd9


2cosOJ R 1 -a' sin10 ]d6.

On the second tenn, we make Ihe change of a liable R sin 4> = a sin 8. in which
case R cost/J d<P =(lcos9dll. lt' R sin?; = a sin O, then
-A

ii + 2tl
= (I1 {-sin26}
2
o

, - -

1.Ra
-

COS r/J

R CO> 1/J de/>

(a -) a -

=-<~inllcos1J+ R 2 Sin- 1 - ~in9 +-sinll


R
R

+ aR

v'r--;;r-'4Rl

.. _ (aJ4R' = R1Sm
2R'
1

a')

2 a
l- -~in
1J
R1

a' )

- 4 R'

2R2

Area of !he flow is therefore

h is won.h noting tllal when a

= 2R, the largest value ror a . Lhis reduces to 1T R2 as it

shoul d.

A' the' ah-e continues to clo..e. it re-.~ehe' 1he posi1ion tn Figure 13.51a where 1he line
joining the origin to the point of intersection of the cunes is aangen~lo ahe vahe. When
the valve is 10 the left nf Lhis position, the previous calculmion or A is no longer valid.

To llnd the ,alue of a at this stage, we calculate the slope of the \"lllve by ditrerenuaung
(.t - a) 2 4- y 2 = R 1, 10 get 2(x -a)+ 2yy' = 0. and evaluate y' n1 the point of
intersection (a/2. J R2 - a2f4),

a/2 - a

l=

When we juate tlus to the slope of the tine JOining the ongin to (a/2 . .; R2 - al/4),
\\Cubwin

a
-,~r.4=;;R:T
2 =a~2

J R1 -a 2/4
a /2

and when lhi$ eqwtion is soh"OO for a.the result is a =


for A h \dlid o nll in the imen-al R sa
..fiR.

../2 R. Thus. the obo\1: calculation

!fUIi.l Mflii!W

V1h<t:

\
)

.\

When the centre of the vhe i:. ju:.l 10 the left_Qf this Jl(hition (Figure 13.51 h). the
area common to the pipe and valve i$ the sum of A as calculated obo\-e and twice the
small area A between the dashed line and the \"alvc,

"here 1J = cm- 1 [a/(2 R)]. as abm-r. Angle

a e0>8 -

J R2 -a' >in 2 8 =

0 is defined by the equation

acm8 .... ..; R' -

a 2 sin2 8

=>

0 = Sin- ( :).

Thu>.

-A = .};;re[(a co:.O "- i R


6

=4a /. coOJ R1 -

2 -

a 2 :.in 2 8) 2 - (aco' O -

.J R1

a 2 >in 20) 2 Jd0

a2sin1 8d9.

When\\euse the above substitution R sin

=a sin 8toe,aluatethi<integral,there<ult ;,

Ar~n

of the flow is therefore

;r R

- A - A =

R1[;r

= R1 Sin-'

11

.,

.,

R- - R-Sin

. _ (aJ4R' - a')

- 2Son 1

2R'

aJ~Rl- a' )
2R'

2R'

aJ-4R1- a'
+---,--2

Thi> b vlid when the position of the centre of the valve sati>fit:> R ~ a ~ ../2 R. Note
thonhi<calculationagrees with that for A when a = ../iR; bolh give A = (;r + 2) R2/2
Final I), "e calculate ~tot for 0 :::; a ::;:: R (Figure 13.5 tc~.

("

= }If [- a'cos28 - 2acosfJ./R' - a1 sin2 6JdfJ.


Once again we use the substitution R sin 1/J
fin~l

result being

= a sin 8

to ev"<lluate the -.econd term. the

-a') o

a J41?'
+ J4R'2 -a' .
(
2Rl
Pulling all the-e resu 11< together. the area A of now.- a function of pu,ilioo a nf the
A= R2 Sin - 1

centre of the vahe is

A..fiR < a ~ 2R.

13.7 Oouble llermed uuegmls in Pvlar Coordiu:nes

939

EXERCISES 13.7

* 24.

In Excl'ciscs 1-5 C\aluarc the double integral of the function ovcl' the

region R.
1. f(x, y)

=e''+>', wbcrc R is bounded by x + y' ="'


1

2. f(x, y) = x, where R is bounded by x = J2y


3. f(x . y) =
X , .t = 0

/x 2 + y2 where R is bounded by .1' =

The mof of a n exhibi1ion hall consis1sof cyJindrical ccu1crclc shells

as shown below. The length of each shell (nOt shown) is 50 m. The


underside of eactl sheUis half of a circular cylinder witb mdius 7 m.
The thickness of the shell (mc.asurod radiaJly) i.s I 0 em at the base and
decreases linearly with angle IJ to 5 c.m at rhc lot). Find the \'Ofumc of
each shell.

y', x = 0

J9 - x', y =

4. f(x , y) = 1/ /.r> + y >, where R is the region outsidc .r 2+ y 2 =


4 and inside .t1 + y 2

= 4x

5. f(x, y) =/I+ 2.t 2 + 2y2 where R is bounded by x' + y' =


I. x' + y' = 4
In Exercises ~7 evaluate the double iterated integral.

. !.

IOcm -

J. .r.-;;.jx'+ y' dyd.r

6.

r-:;--.,---:;

~ 25. Find ~Le area i nside lhe drcle x'

x' + y ' =

* 26.

In Exercises &- L2 find the ::u~:t of lhe region bounded by the curves.

S. Oul$idc x' + y' 9 and inside.x 2 + J"


9. r = 9(l +cosO)
10. r=cos:lO
.t.

* 13.
*

= 2./3 y

27. A circular piale of mdius R (figure below) hru; a unifonn charge


dislrlbution of p c.oulombs per square metre. If P is u point directly
above the centre of' the plate and d A is a small area on the plate. then
the J>OLcntial ut P tluc. to d A is given by

Common tor = I + sinfJ and ,. = 2 - 2sin6

p dA

Find the cemrOid orlhercgionboundcdbythecurvesy =


y 1.

-x . .x

Fmd llle volume of thesol.id of revoJuti.on when a ciJd.e of radius


a langent line.

R is rotated about

11. Conunon tor :;;;;; 2 und r 2 = 9 cos 20

* 12.

= / 2-

x. y =

14. Find the second moment of area for a circuJar plale of radius
abou1 any diameter.

where s is rhe distance frnm P to tl A .

(a) Show that ln tenus of poJar (."<>ordinates, Lhc IX>tcnlial V at


P due t.o the cnti.rc plutc i:o;

15. A water tank i n the form ora right-ci rcular cylinder wilh radiu~ R
and lcnglh It has its a:< is horizonlal. If il is full. what is the force due

10 ,;,.aer pressure oneach end?


In

F.:xen:lse~ 1~1&

+ y' = 4x and outside the cirdc

J.

where

llndtheareaof lhe!i-urface.

-p- 1~/.R
411"~1)

-;7

()

,Jrz + az

drdO.

d is U1c distance fro111 P to tJtc centre of Lhc plalc.

(b) Evaluate the double ilctntcd Lmcgral to lind

= x1 + y 2 below t = 4
-. 17. The area t1f .Yz + y 1 + z 2 = 4 inside x 2 + y1 = I
* 18. The area of z. = .x.v in~ide x 2 + y2 = q
+ 16. 'J'be area oft

V.

z
P (O,O,d)

+ 19. Prove lhaLthcarcaofo.spher.::. ofradius R is47r R 2


" 20. Fi.od the area of the hypcdx>lic paraboJoid
the cylinders .r 2 y 2 - I and x!
y ! - 4.

z=

.t 2 -

.v2 bcLwccn

ln Exercises 2 1- 2"2 11 nd 1he volume of 1he solid of revolution obtai ned


by rocatiog the region bounded by tbc curve sboulthc line.

... 2 _1. r
cos2 0 aboutlbc x -axis
22. r = I + sinO about the y -a.xis

* 23.

find the area of the region bounded by the curve (x 2

4c1 2xz y2.

+ y 2) 3 =

28. Use Coulomb's Jaw (see E:~ample 11.8 in Section 11.3) to find tJte
force on a charge q a'l point P due to the charge on t11e plate in E.'<ercise
27. What happens to this force as the radius of the plate gets very large'?

NO

~luhip l e l ~te8~1s

Ouprtr 13

~ 36. Suppose we denote P R'/(4111,) in Exercise 3~ by Vmax, Ue


muimum vci<Kity at the c-entre of the biOO<J vc;:)Scl. v
Vmaxl l (r/ R) 1 J. Suppose the vessel con\'crgcs to u smaller \'CSScl of radius
R 1 = a R, 0 < Ct < I. If blood flow is, the: smaller vessel illS() has
a p<or.lbolic profile v
Uma.,[l - ( r JR1)'1. lind Unuu in terms of
Vn\;J)( and a. Him: Assume thtu lhc \'Oiurnc llow rates i n the two

In E~ercises 29-3 1 11 nd Ihe an:.a of the region bounded by the curvc.5.

29. (x' + y')' = 2xy


.10. lrulidc bolh r 6 co< 9 and r
31. ,.
1<

= coo' 0

=4 -

2 cos 0

sin(/

32. Find the c<:nti"'id ol' the region bounded by the c:~rdioid r = I

ves..'Cis must be ahe same.


._ 37. Repeat Exercise 36 ror llow in a squ~1r-: pipe th;~t reduces from :l
width of 1.. looneofwid th aL (0 < a < 1). Assumcavclocity pmllle
in I he larger pipe oflhe fonn v = Vmox ( I - 4x'fL ')( I - 4y2j L}).
-L/2 S x S L{2. - L/2 S .1' S L/ 2. and a ~imi lar prolllc in Otc
smalfc,r pipe.

cosO.

* 33.

Find the second moment


bounded by r l = Q cos 28.

* 34.

Ev'3luate the double integral of

:a::t- J.M. find the area of that pan or the sphere .r 2 + )'~

I - x 2 - y'

j(x . y)

or ;~ rc a t1bout tbc .l' >~xis for the n:gioo

1+

(x' + y 2) 1 = a'(x' - y').

x' + .1''

mu Lhc n:gion in.~idc the circle x 2 + y2

1,
35. 1'he figure below illusuarcs a piece oran artcryunein ,--.ithdrcular
crosssecaion (radius R ). 'lllc speed of blood llowing lhrough lhe blood
vessel is no1 uniform because of U1e \liscosil)' of the blood and friction
at lhe walls. Poi<euilfr '> ltM Slalcs ll>al for laminar blood now. Ole
spct-d v or blood at il tli:-,w.ncc r from the ccnti'C or UlC \!C~ I is gi\..:0
by

39. Find the arc.:r of th:rt portion of the surface .r:z


b) .r2 + y 2 ll 2
40. A v~ry importnnt integral in statistics is

.,.
j

I -

+ z:! = a: n"idc

+ z 2 = 11 2 cui ou1

e-.~ : dx.

II

To cvoluQI.\! tbc integral we :set

p
v = - (R' - r 2) .
411 {,

wflcre Pis the ptc-ss:'Urc difference between the ends of the \~1. L
is the length of 1~ vessel. and n i~ the viscosity or the blood.. Find the
arnounl of b!ood flowing ovtr :l c.r04:$~tion or chc blood ve.~sel pc:r
unit time.

()
- - - L- - -

and then multiply these two equations. Ou lhis to pro\'C that I

,fiij 2.

**

41. Use the result or Exerci"-t! 40 to evalu:ue the gnmma func1ion

Ul ll ~

l/2.

113.8 Triple Integrals and Triple Iterated Integrals


~l!l:fM

Triple inaegrnls are defi ned in much ahe s.1me way as double inaegral s. Suppose f (x, y, z) is
a funcaion defined in some region V or space ahna hlls finiac volume (Figure 13.52). We di,ide
V imo 11 subregions of \'Oiumes ~ V1 ~ V2..... 6 Vn in any manner whatsoe\'er, and in .:ach
subregion D. V1(i
1... , 11) we choose an art>itr.ry poim
yj.
We then form the

SubJi~icJc

a regioo V inlo )malkt \U)un~


cu ddinc: lbe triple inle<_;ritl of

<xi.

zi>

sum

/(.r. y.zl
n

L J<xi, y,', zi> D. v,.

( 13.51)

I= I

If this s um approaches a limit a.s Lhe number o f s ubreg ions becomes inc.reasingly large and every
s ubregion s hrinks to a poinl, we c all the limit the triple integra l of f (x, y, z) over Lhe region
V and denoae i1by

)'

ffrlv

f(x ,y.z} dV =

li m t
114\V. J-+O .

f(xj.yj.z'f} D.V;.

(13.52!

1=1

As in Lhe case of double imegr.lls, we require that this limit be independent of the manner
of subdivision of V and the choice of saru poinas in ahe subregions. This is guaranaeed for
conai nuous funcaions by ahe following aheorem.

IJ~ TciJ>Ie lntegr:ds :md TripLe ttemtod lotcgrnls

941

THE O REM 1 3 .2
Let S be a piecewisesmooth surface that encloses a region V with finite volume. If
.f (x, y , ;;) is continuous inside and on S, then the triple integral of f (x, y. z) over I'

exists.
Properties analogous to those in equations 13.4-13.7 hold for triple cntegrals, although we
willnotlisttbe tirst three here. Corresponding to equation 13.7, the volume of a region I' is
given by the triple integral

Jfl

volumeofV =

(1 3.53)

dV .

We. evaluate triple integrdlS with triple Iterated Integrals. If we use Canesian coordinates
there are six pOSSible triple iterated integn1ls of a function f (x . .1', z). corresponding to the six
permutations of the product of the. differentials dx, dy, and d z:
d z d y dx ,

d z dxdy,

dxdz dy ,

dxdy d z,

d y dxdz,

d y d z dx.

l11e geneml triple iterated integral o[ .((x, y, z) witl1 respect to z, y, and x is of the form

' J.g,(.< )1h,(. .y)

1
a

8 1 (.1.')

f (x,y, z) d z dydx.

t l 3.5~>

11 1(x. y)

Because the first integration with respect to z holds x and y constant. the limits on z may
therefore depend on x and y. Similarly. the second integration with respect to y holds x
constant~ and

the limits may be functions of x.

I EXAMPLE 13.21
Evaluate the triple iterated integral

f. IJ.x'lx+y
0

x yz d z d y dx.

x.v

SOLUTION

1'1''f +y xyz dz dydx


0

.l)'

Bc.c.ause of the analogy bctwcc.n double and triple. integrals. we. accept without proof that triple
integrals can be cvul'uatcd with triple iterated inccgraJs. \Ve n1ust., however, examine how triple
iterated integrals bring about the summations represented by triple integrals, for it is only by
understanding this process thoroughly that we can obtain limits for triple iterated integrals.
In Section 13.2 we discussed in coosiderable detail evaluation of double integrals by means
of double iter.ued integrals. ln pan icula.r. we showed lhat double iterated in1.e.graJs io Canesiao

coordinates represent the subdivision of an area into s mall rectangles by coordinate lines x

constam and y = constant. The tirst integr.ujon creates a summatjon over rectangles in a strip,

l<lthe 5eton.d tntCJitllltun adds over all snips. It b fon1y Slmi;hcforwdnl tu lncrnli>e these
ide.ts to triple integral" Cooidcr evluating the triple: tnttgral

ffi

{(X.

y. :)dV

o'er the region V in Figure 13.S3a bounded above by the ~utf.tL-e:


region

= il{x. y). below by the

R in thC.\ )'phwe, t~ndon thesidesby ac..)Hndrical wall "Htndingun thccur\'ebounding R.

The choice <lfu triple itcmted integral in Cartesian coordiMtes tocwtlume this triple integral
implies :1 ~u hdh i;;ton of V into smnll rectaJg,ular pumllclcpipcds (bou:,~. for short) by menns
of coon.lin~tc plane~~
conitam. y = COn!t<lJlt, and .:: = C'Ot\.\taut. The dimens-ions of a
representative box at position (.r, y. z) in V arc dcnmed by t/1 , dy, Hnd d z, wirh resulting

olumc t/,r dy d:. lr we decide on a triple irerared integral with respe<:I IO ~. y. and .r. then
the fir~t intc~rution on z hold~ \ ttnd y constant. This integnllit.mthc:rct"orc odds the quantities
/(.r I y. :} d :. d) d.t \1"\Cf boxes in a vcrticaJ (..-olumn orC"-""~C:Iionol dinlcnsions d:c aOO d).
Lower anJ UPI)..,. tirnih on ;,: identify where each anl.l every colun1n swns und ~ops.. and rnu.\t
COtts.!Qucntl) bcOund lt (x. y):
h(\.1f

.f(x.y.:)d:dydA.

Sirx-e lhil\ huegr~uinn pnx.tuce~ a function of x and y alnru:, rhc remttin111g imegration with
respccrto y a11d .\ i o.-<emially a double ireratcu integral in tile AJplane. These inrcgrarions
must tl<:<:ount lOr Rll colum ns in \1 fllld therefore the region i n the ~yy ..plnnc over which chis
double itcnncd i111cgr~l is pcrf<>rntcd is the region R upon which all columns in V ~t <llld. Si11ce
the y inlcgnujon udd~ inside : ~a rip in the )' direction and l imit~ identify \\'here nil stri p.~ .;tart

tllld stop.rhey muM thtrcl\H't: be g 1(.r) a11d g2(.<). We now have

Finally. the A intc,:nuion adds 0\"Cr all suips and the limits are '

/,ZY.v

f(,t.)'.~)dV

= /.
(7

b/.t'l'1.'"')
J'tf.l)

= a <ond x = b:

f(,\.)'. ~)d: dy dx .

(l

Suppose now thai V is the region bmllltleu above by tltc surface :


ll 2(;r . y) and below by
h 1(x, y) ( Figure IJ.5Jh). In thi< ce. the limits on the li1'8t illtcgrati(Ml with rc$pccl to;:
11re lt tCx , .l') nd lt J(X . y ) si11cc: every column StartS on the sut1ilce ~ = It t (X , y ) 11nd ends 011
thesurfua:;: = h,(x. y):

<=

For the 'olulllC of Figu~ 13.53a we incerprered rhe linalt"o integrnri<ln; a, double iterdled
integral on the X) -plane ocr the region R from "hictl all columns eman,lleJ. For the prese~~t
volume we ontc:rrret R a< the region in the .ryplanc: oouo which 811 column projea. We then
obtain

Iff

f(x, y, z) dV =

1
a

hjg,(q1 h,( l .~ )

f(x, y, z) d-z dydx .


g 1 (x'

h 1(.t'.)")

liZ

~." "-lilft un d A _\ -pl..lno : 0


R u the wt: wt h .1\~

Md C!ttd Of' nritee::; b(.l.))


'"' "'hi'h all ttlf..~ ~

c.......,. ~' ....... \1~ -

.: =A,(.\') .11td;: f t(.l'. \)

wr(~~
1lilc an~ .n dlC' , _,~

01111;) ...-hid. . . 0... .,.,.., p!\J.}0.1

b
)

Schenltlticully. \\C hove obcainctl 1hc follow ing imcrprtlll1iOil rurthe li mil'! of lriplc itcn\ted
inLl."gl'al.s in Cancshu1 cuurdin.ttes:

tl: dy (/,\
d: d ,\ dy
dx tlztly
dHI ] tl:
dy d,td:
dydttlt

I eXAMPLE

13.22

Set up chc six rriplc itemred inregmls in Cartesian cOO<dinarc~ for rhc rriplc inre,ral of a functioo
f C~r. y. ,z) m Cl"thc region V in the firsl octatll bounded I,) the ~uri ace.'

y 1 + z2

= 1.

= x.

= 0,

.r

= 0.

SOUITION Tile rriple inrcgrul of J(.<, y, z) over V is ~ivcn ~y each of the following triple
irerared imegmls (~ce Figure 13.54):

r'J. ,

r.fi=>i f(x.y . z) d z c/y dx,


lo lo

'f. ..ro=:rJor, j(x.y.z)dxdyd; .

!.
!.'!. Ji=\'f ..ro=:r
0

J(x.y.;)dyd;dx,

'f. ,.1u.;;::;; j (x . y.;.) c/;cl.tc/y.

!.
!.'!. .;;::;;1,
'f. ..ro=:rf ..ro=:r
!.
0

f(x.y.z)dxcl;dy.

j(x.y.z)dydxd;.

944

0'1:\ptct JJ Multiple Jutegr.tl$


M;iiclllei# l'OBtM

The six (riple iter-.ucd in{CJ.,.rals fur the

\ 'oJume bul)de..;l b)' )'= X, y~

I EXAMPLE

+ Z! =

l , Z = 0, X= 0

13. 23

Evaluate the triple integr~l of f


-

~-

SOLUTION

.
\/r.:)
)'

Ct, y, z) = x y z over the region

+ z = 2.

.t

= 0.

V bounded by the surfaces

Z = 0,

X= 2.

z, y, and x, some columns

If we choose a triple iterated integral with respect to

JY (Figure 13.55) and others end on the plane y + ~ =

end on the paraboljc cylinder;; =

2.

\Ve therefore require two such iterated integmls,

ffr

l vxyz dzdydx =

121'1./Y
0

xyz dzdydx

12!212-

xyzdzdydx

M::tiC'Iil;t ''k"tW ColumM


in the :-<~Teettoo necessitate 1wo
triple iter.ltcd integrals
2 = ../y

(0. I. IJ

~1
2

y+t =2

..........:_

f.'
H:2J:

= 6I o

{xi }'
.r dx

dx

+ 2~1

{x (2y 4y3 + y')}'


2

dx

J.'

5 o x dx
24

3
4

Only one iterated integral is required if ;meg;:.ltion is first perfon ned with respect 10 y ,

namely

1'1'['-'
0

:;:1

xyzdy dx d z

or

1'1'['-'
o o

z2

xyzdydzdx .

U.9

\Uiume..-fi

945

EXERCISES 13.8

In Exercises 1-12 evaluaLe Lhe triple integral over Lhe region.

In Excn:ises: 14-1 7 scl up, bul do 001 cvaJua1e, a triple i LenuOO inwgral

IIi
IIf,
.\. J!f,

fh'l' lhe 1ri ple imegral.

(x'z+yc' ) dV . wherc V is boundcd byx = O. x = 1,

I.

y = t , y= 2 , z = O, z = t

y+z= 4

sin (y

2X, y = 0 , X

+ ~) d V ,

where V is bounded by

6.

16.

dV , where V is bounded by x2

Ili +
IIi x'
(x '

Jt .x'l

+ y2 = 1. .-2 + ~2 =

~ 0. y + z ~

2z )dV . wl><:rc V i bounded by l

y'z' d V. where V

jJf,xyz dV . whc.Jc

is bounded by

z=

1 + y , .''

+z=

jjj tl V .

\1 is the lirst octam rcgil)n c::uLou1 by z -

//i

where Vis bounded hy z

12.

+ y ) tiV , w hen::

\ f i$ bounded by y2

+ 4z'..r = J4 + x'

JJl,xyz.dV,whcre V is bt)ult-tltdby.t ~ x 2 +4.Y2. 2x +


=4

Jff, x 2 y 2 z2 dV , whcrcVisboundcdbyx =y 2 + ~2 . x + l=
(r' + z'>'
J1.

IIi +
x+
* Jli
IIi
IIi
*
jJf, + +
*"
}fi l.vzltiV,

+ z' ~

1. t

.v + z =

1, z

x') d \' . where V isbounued by x

(y

18.

19.

(.ry+ z)dV , where Vis bounded by

.J: .

+ , = 4. y = o

20.

,1:

Q, X=

x' + y1

x+

dV. where I' is bounded by

z=

d\1 . where 11 is bounded by l.

~ x' +y' . t ~ 4- x'- )''

0.

= 1,

)' Z = 2

+ .t) dV , where V
z ~ o. )' + l ~ 2. y ~ z
II.

xz sin (x

l y. Z = )'.

10.

I, )'= I,)' = - I

x'+ y'. z = /x'+ y'


9.

+ t 2) dV , where V is bounded by
y', t = x'

(x

In Exercises 18- 23 evaluate the triple integral over the region.

l , x= l , x=O, z = O

8.

JJf, +
J x'
JJi

8)' + l

yl , z. =O

4. y ;; x 2
7.

15.

4.,

= 0 , )'

= I . Z = X + 2y

4. Jff., A'y <lV,whcre V is cm.:loscdby ;: =

JJl,

14.

z=

x d V, where. V is bounded by x = 0. y = O, z = 0, x +

2.

S.

(X+ Y

is bouncJcd by

x = 0, x =

1.

/Jlxyz dV , whereVi~ hc)urxk!d by ~ = I , Z = x 2f 4 + )'l/q

J/lx

21.
2:2..

ydV, w here V is: Lhe firs1 octant region bounded by

(x

= I -

z= j

.e. 13. Stl up the six lri.plc i1cratcd imcgrdls in Cuncsian coordinates for
1hc 1riplc i n1egml o f a func:1ion f (x. y .z) O\'tr 1hc rt~i(ll\ enclosed by
tbc su-:faccs y = 1 - x 2 , t = 0 , and y = ~.

y2

23.

z = I, z = x'/ 4 + y'/9

z} dV . whc.rc V is bounded b)' 2z =

where \1 is bounded by z 2

+ x1 + y2,

4 -.r 2 - y1

24. Sci up, but d o not evaJuate~ tripJe tteratcd integrals 10 evaluate the
u-iple intcgral ofthc function /(.t. y, z) = x 2 +yz+z2 O\'en hc rcgi.on
boutldt':d by Lhc surface s xz + y 1 = zl
I, 2z = / x 2 + yl, z = 0 .

113.9 Volumes
Because me volume o f a region V i s represemed by triple integ~l 13.53 and Jriple imegr-<~ls

are.evaluated by means of [riplc. ilcrntcd Uucgrals, it follows lhat volumes can be evaluated widt
l riple i l er.ued i ntegrals. For example, 10 evaluate the volume o f l he region in Figure 13.S3b

using a triple iterated iotcg.ral in x, y, aod z. we subdivide V into boxes of di.met"'sio ns tl.A. d)' ,
and d z and therefore o f vol ume d z dy dx. l mcgration with respect 10 z acids these volumes in
the z-directi on to give the volume of a ven i.cal colunm (Figure I 3.56) :
h 1(x .y)

, ,(x.y)

d z dydx.

946

Ch:l~r

IJ \lulltple lntear.>~s

Limits indicate that all columns Sian on the surface z


II 1 (x, )') and end on the surface
lmegnuion with respect to J now adds volumes of columns lha! project onto a
strip in the y-dircction:

;: = h2(x, y).

where the limits indicmc that ull strips stan 0 11 1he curve y
g, (.t) and end on the curve
y = gl(x). Evidenrly. this integration yields the volume of a slab as shown in Figure I 3.56.
Finally. integrmion wirh respect to x adds rhc volume.\ of all ~uch sla~ in V:

"here n and b designate positions of lirsr anc.l l:1s1 suips.

FIGURE 13.11

The mtegnllions ,.hen > triple rtenrcd intqral ;, u>e>J ro 6nJ olume

z
; lr j.x. )')

..........
.........
..............

:=h

(r,y)

'':

b
)'

I EXAMPL E

1 3.24

Find the 'olume bounded by the planes;; - x + y.) = 2x, ~ = 0. x = O, y = 2.


SOL1.7110~

\'Oiume

=
=

rr \\e use wnical culumns (Figure 13.57). we hae

r' t rX+)d~dydx =11


f\_. + y)dydX = r' { xy + )'2 } 2

Jol1\ lo
2

{'(I + x

lo

(I

)2.\'

- 2x 2) dx = 2 {x

}0

+ xl 2

2xJ3 }1 = 3~
0

tl

dx

IJ.'J

Volume.;

~7

ljUIIJeU t&flj M The volume bow)(Jcd by the pla..'le) : = x + y. )' ;;::; ~.\' , ~ = 0 ..r ; O, y = 1:
t=X+y

I EXAMPLE 13.25
Find the volume in the firS! OCialll CUI rromthe cycli ndcr .t2

+ z2 = 4 by the plane y + z = 6.

SOLUTION With columns in the y -dire<:tion (Figure 13.58). we have

11J-lo6-: dydc clx = 1'1..;.,_,,(6 - z) d zdx


2

olume =

,2}.J-'

116z -T

dx

= {1 6)4 -x2-~(-l -.< 2 ) }dx


In the first tern we set x

' 'olume

= 2 sin e. from which we get dx = 2 cos 8 de. and


=

6 L t '\2cos8) 2cos8 d8

(l
/0
.

_ 12 { fi
IJIUII;siMF41-1

X~ + ~2 = 4

l-2x + -~ }:

.T/2

- 12

+ cos28) de - -

~ sin28 }.t/l
- 3~
0

6rr

\\.lhuue iu fir.,l octant cut from cyliOt.ler :c 2 + :2

8
-

:J

4 by plane )

+z =

Had we used itenued


two imeg rals.

1olume =

in~egrals

with respect tO .: . y. and .f in this e'ample . we would hae had

1lf.6-,JI=;!1.J<-i!
0

11 16-'
2

d: d~ dx-

./i'"7'

d: d) dJ. .

We are bemg consulted by an architecl who wan IS to know the olume of Jir oontlined tn
3 cenain type of <UUCtUre. II ha< a hori7Alll[al. polygonal ba<e (Figure 13.5~a) on wtuch
<und 1ertiClll walls all oflheume height H . Above this tsa roofrormed tn th<" fol lowing
o.-ay. Th<"re is a peak. height h above the top of lhe \\ails, !hat is JOtn<"d to me tops of th<"
"'-ails by planes.
Volu~

Roof

SOI.l 110' There is no problem with the lower part of the s tructure: the ''Oiumc of air
conUlined in it i> the area of the base muhiplied by fl. To cnlcul~te the w lurnc of dtc
upper pan. we take it aside as in Figure 13.59b and drop 3 perpendicular from the top
to the polygonal base. Now join the foot of the perpendicular p to Cdch \'Cncx or the
polyj~on. Fin.oll) draw vc<tical plane> each containing one of thel>C li~~e>. the pcrpcndiculdf
from top to base. and a slanted edge or the roof. What this does is diide the 1olume into
tetrohcdrons (fours ided ligures) each with one horizontal and t\\O -.:nical planes. The
remainina face of the teuahedron i< a slanted face of the roof. If we find the olume in one
such tetrahedron. we can add to 6nd the total 'Oiumc in the upper ponion of the suucture.
To lind the olumc of any one
or the tetrdhedron<. let us place its
triangular basc in the .ty-plane \\ith
one 1ene' t the origin. a <e.."'nd
'"nc" on the .t -m<is. and the third
1ene~ in the first quadrant (Figure
13.60). The top of the teU'ahcdron
will he on the ~-axi< Deno~e coordinatcsofthc tetrahedron by 0 (0. 0. 0).
A(a, 0 , 0). B(b, c , O). and D (O. 0, h) .
Equation' of Iinc. A B nd 0 8 are
y c(,t - a)/ (b-a) nnd y =ex/b.
re<pectllely. A nonnal ecor 10 plane

ADD ;,

j.~o. o. hi

All

j k
x AD= b - a c 0 = (ch,ah - bh,ac).
-a 0 h

+ (a/1 - h/1 )y + li<'Z


'ohtnle of the tettahedrOtl can now be calculaled wilh a lrip~ im.eg.r-.al.

11>c: illation of )>lane A R f) ;, therefore ell (x - a)

v=

=0

'l11C

c1Hb-a)J/r1 (lb6-cdl)y-d\A"-11'))/(oc)
~bdv

1)

b)/<

c1 tb-n))'/c [(bh - ah)y - ch(x - a)]

= -I

ac

J,.f,

1' {

c/1
} 'fl>-h/<
h(b - a)xy - ? (x - a) 2
dy

ac o

1'{/t(b _a)y [a .,.


ac o
_!_

- h(h- a)y

'"'''

(b -

ch [a"- ..:_
(b _- _u):.;c.> _ 11 ]
2
c

ll

I {h

)'>

+ ah(b -

bh
- -(b-

y>

clh

a)y] _
c

(--;:by) + '2ch (by--;: - )'}dy

-(b- a) 2 ' ru ,.
3

= -

d . d)

y
h
yl
a) - - - (b- a) 2 2
2t
3

bv

a) -+
-6/J (-c
3

-ll)

I
= (,ach.

Since the area of the triangular base is ac/1.~ we have ~h()\\'0 that the

ur

\ 'UI UI\IC
the
tetrahedron is the area of the ttiangular base multiplied by onethird of the height h. It
fo llo"s that the volume of the upper structu:re is the area of the polygonal base multiplied
b) 11 /3. Finally. then. if A representS the area of the polygonal base, the olume of air in

the "ttrucmre is

EXERCISES 13 .9

In IO>oo::i""" 1- 17 find the: W IUIIIC uf the I'Cil,ioll buundc:d by dlC Mil


faa:<.
I.

,1'

=.r2. ) =I. z =0. z = 4

3. .<=3t . z - 3x.) - l. -O.x =2


4. x
5. l

+ y + l = 6, y = ~ - x', t = 0, y = 0
= x' + y' . y = x' , y = 4, t. = 0

+ y + l =4. )' = )z. X


7. .r: + .l' 1 = 4 , .\' + zz = 4

6.

S. )' = X:- I . y

9.
10.

z=

t6 -

=0.. =0
1'

= 1 -X:!. X+ Z =

x' - 4y' . .r + y =

I. Z= 0

I, z = 16. < 2: 0. ) 2: 0

z=x 2 +y 2.x= 1, z=O.x=y.x=2y

II. t. I - x > - )'2, l

=0

22. J (.t. ,l' , z' = xt+ y:+z: over the ll.!~i Qn bound.:d byd'IC surrxe:-..
.~- O,.l'
I,."+~ = 2 .l'
2. t ~ 2

= o..r + z J.. + z = I, l. = .r + r. z = ()
H . r + y +z =-2, t"~ I-)'J = 1, z = O
14 .l' + l = I. ~ = 2y. : = )', .r = O. x+.r+;; = 4
12. :r - l

'I'

u . .1' + 4y 1 -

:;. x 1 + 4y 1 = 12 - 2z

16, y = J - z1

."

17. X ' 3y I

2~

= 6.

I. x = I - ;; 1

= 0. )' et .r.

.,.. 23. Find~ vo!ume. boundtxl by the surfu.ccs ~ ~


4 - 2(..1. 2 ) 11).

.C'

;, 1 -

y lx

.-: .HI. _Fi.11(/ l)x \Vlumc in life fir~( vc(.:.rli tn.oumkt.f by lhr:
1
? = 2 ~nd ;JlSide 1bc cyUtl(lr )' + ;: ~ : I.

''':.'~ 2.r

+ )' +

" 19. A pynun it.f h;:u. 3 M{UJO: b:l:ic


m

it~

with side Jcnglh band h!IS hc:iglu h


centre.
(a) Find il~ \~Jume hytt~kingcrrutli~tinn" parallel tn Ihe hJI~
~~Sr.: riM 7.9)
(h) Find its \\")/Un'IC ll<;ing: rriplc integrals.

24. Vcrify JPuurhc~urfncc~> z = ;~,~


bound a linitc , -oJumc.

""' zs.

Ftnd lhc ''01umc botndcd b)' th~ s uri'IICe.-.

.. U. l:"indtllC'''ulum d>Owldc-.dbythcliurfilc:csz
~

x' -

,\'!

z=

und

,, _ xl - y~ do n01

x+z -

2. t

(x - 1) 2 .!..y2. 2t:+

= 2.

n.~ bottom

.t';;;;;;

/(.r. y . e) over~ region wilh volume

- y: nnd z =

0. 4_,.l = .t-(2- ;:).

... 18.
The <1\'fi"JgC \'3lue of :J function
V i.s dclin~d a!.

:wd ~ides of a boa( a rc: dtlillt'd by tiM;: surf<tl~ ~uation


10( 1 - y~- .z 1 ), 0 ~ x ~ 10, when: all dirncnsion$ are in

tHCltC!l

(:.) Fi})(l 1he volum e of w01Cer diGpbce<l by the bo011 when chc
w;:tcr level on Lhe side of the boat i.s d ll lClrcs below Ihe IOf)

oflhc boat.

Jn Ex..rcitoo~."S 20-22 lind the :.wr.tgc value uf lhc l"utk.'fiun 0\'t't dll'

(b)

A n:biurdt."S' p1incipk

~>taLc~

th:it the buoyant rorf:c un an

ohjcca when immcr.ocd or pruti11lly inunc11>ed in o 11uid i~


equ~ to the weipht of the Ouid displuced by theobic<.-1. Find
lhc maximwn wcig,hl o f the boat ~nd crnllents j u~t before
si.nid.ng.

n::ion.
20. f(X. )', Z) =X.\' 0\WihC region 00-UJ'ldCd by lhC Sud<lC'CS X= 0.
J

)' = 0. t = 0 .r + ,l' + l =

21, /(X. y. t ) ;;; X+ y+ t 0 \'Cr dJC rcgiOll in the lirS-( OCI<tlll l'Omll.lcd
= 9 - .r 1 - )'1, ~ = (l :md lb.r which 0 :!f .r ~ 2~. Find tl~ volu~lC inside :lll 1hrcc Sllrfiltcs x J+ y 1

by the .sudaces ~

tz. _r

J . O ~ .r S J

+ ~ =

a 1 , .rJ+ t 1

tr.

13.10 Centres of Mass and .Moments of Inertia


fn 1his sec1i<.1n w e di..,.;cu~s: cemres of ma.ss and f11ome n t.-~ of i nen ia rol' 1 hree- Ui ruens iona l obj~1s

of density p(x, y, z) (mass per unit volume). If we divide the object occupying region V in
then the mass in
is
The triple integral
Figure I3.61 iniO small volumes

dV,

dV pdV.

( 13.55)
U~:41f1JM

c~ure

~dc.Js the masses of all such

of

nub of# J-.dimcJ.~<ilma1 (-b,P.c

of

coordinate a.<es are the following fom>ulas for t1rst mo me nts of the object about the coordinate
planes:

vofumes (of ever-de<.re-dsing size) to produl' e the total rna.~

tile obj<ct.
Corresponding to equations 13.32 ond 13.33 for first moments of planar ma.o;scs about the

Ill xpdV,
= lll ypdV,
= Ill z
pd\1.

11rst moment of object about yzplane =

(IJ.56a)

11rst moment of object about XZplane

(13.56b)

lirst moment of object about y pla ne

(13.56c)

/J./0

C~nues ufft/IJS) :uld .\f(}l1lCIIl of{nef(i:l

IJ51

Th< """'of""" of<he ol>jt<l is defined as <hal poin< {.r, )'. :! ar which a parriclc of nJJlSS
M .,.ld '"""' same fi!Sf mon- abou< thecoordinace planes as th< objcc< itself Since the
fin<

m-" of M" (X. )'. :1 '"""""' coonliflBte planes arc MY. Af.i'. and AI':. il foUows

rlult we can use d1e equations

AIJ

=Iff

til::=
;rpdV.

1/.i

:pt/V,

0 357)

co soh'C,for.(:f, -~ Z) ~nee AI 8Jld clle inccgrals on the right h,1\'e been cvalu t d
10
If we use atnple l!eraced integral with re
athe
c third

say, for
the object in Figure /3.56, then
peer z J, and .r. to
ofLhe e.

ev-t~luate

((( :pdV =

fffv

i ,h.,!J
"

r e:l:r1~:(x.J)_ ~
~Pd~dJdx

rrtr)

1r

(.~.J)

th~._,e mome

Quantify
dy dr is rh fi rsr lll<)fllent abom th
of
d'llllen :p
d:.d.r
rons
dy and d.
e .r;-plane oft hen

::-<firettion ro gile,the ti'


The .:-mcegrotion then adds
lass111man elemenwl /x),l(

rst momenr about tbe .ry-plrule of th

O" bo'XCS w
.
e mass m a ''ertica/er
col
. I he
umn.

"!1'" ''

:.p d: dy d.r.

The y int
.
' 1r ,,rJ
ditt'100.. (',g/llrron rhen adds the 6l"Sl 1110/lltnts of coI umns that
omo a
f)tOJect

!""'(''"''

""P m d>e ) '

J:squantiryrherefore
8 t-<J hrt(.r.) ) :p dz d} d .r.
rWJJiy, the .r -integral' represems tbe first mo
V

abourthc .ry-plane:tan adds fi rst momemsmenr


of allabout
suchr/slabs
~~ .ry-plane
ofthe .slab m
. Figure
b
to ail-e the

( &" J

-., J....,r
u

EXAMPLE 13.26

"

,,,.., ./;...,

.,

:(.y).

total first

moment of

,pd::dJd

;~~;:;;;;;;~~::::.t:)~:::=_:_:A:________________

-Find lhe ccm:re or


"" I - yl, .r - mass of an ""''
-oor constam cknsrtv
' --o. .r ,.,~:a
. P r .
2
. u is bo undecJ b
~ lhe surtaces

t{
/ '-_v-?
}'

13 56
. .

From the symmetry of the object (Figure 13.62), we see x = I a nd y = 0. Now

SOI .U110N

M = 2 (' (' ('-..-' pdz dydx = 2p

loloJo

( { yl }I

= 2p Jo

y -

('('(I

JoJo

4p { 2

odx =

- l )dydx

Sp

Jo dx =

and
2

l- J 1

Mz=2J.J.1

1J.
2

= P
-

16p
IS

8p

zp dzdydx=2p 1J.

I , _2

l -.v~

{;L

dydx

( I-

( 2 {y Jo

Thus, 1: = -

2y2 + y 4 )dy dx

2yl

+ )'; }' dx

= Sp ( 2dx = 16p.
15 ) 0
15

2
= -.
5

l n vi ew of our discussion on first moments in this S-eClion and on moments of i nertia of thht
plates in Section 13.5. it should not be necessary to give a full treatment of moments of inenia

of three-dimensional objects. Instead, we simp ly note that the distances from a point (x , y, z)
to the x -. y -. and z -axes are. respectively. y2 + z2 . Jx2 + z2 , and .j x2 + y2. It follows.
then, that if an object of density p (x, y, z) occupies a region V of space, its moments of incnia
about the .:r-. )'-. and z-axes are, respectively:

I EXAMPLE

13.27

Find the moment of ioc.rtia of a right-circular cylinder of constaot density p about its axis.
SOLUTJO:-l

Let the length and radius of the cylinder be h and r and choose the coordinate
system i n Figure 13.63. Titc required moment of i nenia ab(Jul lhe z-ax is i:; four time~ Lhe
moment of incnia of that part of the cylinder in the lim octant. Hence,

To evaluate lhis definile inlegral we set x

= r sin 8, which implies that dx = r c.os 8 d8. and

f.rr/2(r rcostl + 2r2 son. 2 tlrcosli}rcostldtl

4ph
I = -3
0

13.1 0 Cerures on lass :llld Mornems ()( Inertia

4phr 4

_ 4phr'1
3

1:r/l (cos() + 2 sm 8 cos B) dB


1

. 1

l'T/ (I +cos 29 + I - cos40) diJ


2

Moment of inertia of a uni


fonn tlghl<ircular cylioder dxut its a,;is

, ,

x- + v = r

/ '
/

,.........

..... ~

dy

The product momcniS of incnia or a mass with densily p occupying a region V a!>out the y z
and .n-planes, the

xz-

and xy-pl nncs, and the

Iy: =

yz- x

ut d y-plancs are

JJJ.>'zp dV,

I,,=

JJi xzpdV. (13.59)

They cnn be posicive, negacive, or u:ro. Symmetry of che objccc plays a key role in chis dcccrmination.

I EXAMPLE 13.28
Find the product momcnrs of inertia of the object in E><ample 13.26.

SOl UTlON Because of tl.e symmetry of' tile object about the .rt-plane. product momenas of
inenia 1, 1 ano ly; are both zero.

I.<z

Jf'r'r'
o-JoJo -y' px z d z d y dx = 2p

r'Jr'o x( t = p Jo

11
2

' {

x~

y")' d ydx = p Jr'


o {x (y -

' } ' _ ,,,


.

dydx

zyJ

3 +

5Y

)}'

o dx

953

Clwptcr JJ Mulliplc l mc~als

854

EXERCISES 13.10

surfaces .\.= 0. )' = 0, z = 0, and ax + by + C<: = l, whccc a , b.


and c: arc positive constants.

In E.'\;crclscs l - 5 the surfaces bound" solid objei:l of constant dcosjty.


F'utd its centre of IUass.
I. :;: =

x 2 + y2 ,

+y+Z =

Z = 0, X = 0, )'

0, X = I , .\' = I

3.

z = x1 y + z. =

4, J = 0

4.

y= 4- x

0~

5.

x + y + z = 4, y

z=

.,.. 21 . Show lhat the moment of inertia oran object, with cons1ant densily.
about uny one of the du-cc coordinate axes is less thatt or equal 10 Lhc
sum of the moments of i1len ia about the other two axes.
-+. 22. Prove the parollcl axis theorem for solid obje~ts: Tiu: moment of
inen ia of a uni fo rm solid about aline is cqual1o the momem of inertia
aboul a p<l.rallel line through the centre of ma!:S or the rolid plus the
JJ.\USS (tluhiplicd by the square of the djstu~lCc bctwcco the lin.cs .

y=<

= 3z. x = o. y = 0

ln Exercises 6-10 the sudaces bound u solid objoct ofeOJl&a.tU dcnsiry


p. Find its momcm of in~:n..ia about tl1c line.

= 0, )' = 0, l = 0 , X = I , ,1' = I, Z = I obotll the X axis


t = 2x. z = 0. )' = 0. y = 2, x = 3 about the ,l'a~is
x + y + t = 2. y = 0. x = 0. 0 :; z :; Jt=Y about the

.J.

**

8.
x-axis

+ Z = 2,

X+ l

or a righlcircular
partially full of pop, The centre of mass of the can plus
contents is L"(;twccn the ccmrc o( mass or an empty can and that of the
JXlP itself. Show thatlllc ccnlrc of ma.~s of can plus oomcms i~ lo\Vest
wben it coincides with the surtace of the pop. Assume lhatthe hole in
the can is so small thut the centre of mass is the same as iJ there were

'24. The figure belo w shows a pop can (in lhe form

no hole.

9. z = xy, x 1 + y 2 = 1, z = O (fir:st octant) about tltc. za.~is


10. )'

2J. Find the CCJltn:: of muss of a unifl)tu\solid in the first OCliLlllbourtdcd


1
2
by lhe ellipsoid x' f a' + y f b' + <. fc'
I.
cy ~ndcr)

6. X

7.

20. Re 1::eat E..xen::isc 19 for the object in t.h e first octam bounded b>' the

surfaces z = 0 , y = x , .v = 2x , x + z = 1.

I. X = 0. y = 0, Z = 0

2.

=2. X =0, )' = 0. Z =0 about the Zaxis

Surface of pop
C entre of

11. Find the first moment abou( the xy-planc of a solid of constant
tlcMily p

2, ."

i(

iLis bounded by the surfaces

= 0, .r =

x = z. x + z = 0. z =

mass of can

- --1- Centre of muss of can plus pop

2.

Centre or mass llf pop

In Exci'Ciscs 12-1 4 lite Su1'fac.:cs bound a solid objec:;l of constwu den

sity. Find i ls (entre of mass.

y 2

12. ,V =

x 3

X=

z=

.r 2

x +t =

13.

' 14 .\'

+l

Z = I+ x

+ )'1

t = - .Y 1 -

Wh~n an object 11()<Ht> partially !ubmerged in :.l Uuid, it'\ e~ntn:- of buoyWJ<.:y is dcftncd us tbc c:cotrc of 01a.ss of the wuler displuccd by the
objett. Jn E~e 1<: i se~ 25-26 find the deptJl of the cemrc-of buoyancy
below I he surface as the ohject no:ns in water.

y2

2. t = y . y = 0

= Q, V- : = 0, X + l = 0, X - l = 0, l = 1

1t:

l n Exercises 15-17 lhcsurfaccsbound awlidobjcctor c.onstantdcnsily


p . Hnd ils moment of mertia :tboulthe l.ine.

+z+2 =
a2 , x1 + y2 =

IS. z = 4 - .\' 2 x

0. y

.t:

_164 .x 1 + z2 =

a 1 ..bouL thc.tui~

17. X + J- Z = 0) X = 3y. 3)' =

2.t ~ X =

Lc~ V = (t'x v v~) be a unit vec-tor with its tail at the Otigi.n.
1
Show thai the monlCnl of i.ncrtia I of any solid object occupying a

region V about the linc<.:ontaining V can be expressed in the

.t<-t.

28. Find (he

llU)IUCRI

of inertia of a unifonn solid sphere of radius

about any 1angcm line.

18. Fiodthefist momcntabollllheplane.r+y+z = I ofasolidobjctl


of COfL~tlanL deftsity p if it is bounded by the smfaces x + 2y + 4z
12, X 0, )' 0, Z 0.

rorm

3, Z = 0 abo\1l the

z-ax.i..s

,. 2-6. A rig:ht~circulllr cone (radius R and height H ) with densily


800 kg/m3 floal ing with its apex. pointing tlownwurd

,..,. 27.

= 0. y = 2 about they-axis

.t:

25. A sphere with density 500 l:g/nl3 and radiul> R

= **

19. Find the prodtu..'l moments or inertia with rcspccllo the coordi n ~tc
planes of an object with COilSI~n t density p if il is bounded by the

29. A hc>tnogerneous objt!(:t wilb mass M oceupics a rL'gion V . The


sm of its motnenlS or iner1in about the cooulimnc nxcs is 1... + / >, + 1:.
A bomogcn<.-ous sphere with the S!UJ JC mass :ind the same dcnsi1y is
centn:d !ll the oiJgin. Show Lh!ll thc sum or iiS tnC)((JentSof iner1i!l about
the a;<es is le-ss l:h:in or cqu11110 I~: + ly +

'=

113.11 Triple Iterated Integrals in Cylindrical Coordinates


In Scc1ion 13.7 we ~aw chat pola1 coo1di nates urc sonletirncs more convenient chan Canesian
coordinate~

in evaluati ng double integrals. IC should corne us no su1prisc. then. 1hat other


coordinate systems can shnplify the evaluation of triple integ1als. 1\vo ofth~;: most com111on ~rc
cylirldrical and spherical coordinates. Cyli ndrical COOC'dinates are useful i n problems in\'Oh,ing
an axis of' symme1ry. n,ey are l>.'lSed on a Cartesian coordinate along 1he axis or <ym111etry
and polar coordinates in a plane perpendic ulc1r to the axis of symmetry. Jf the .c:-axis i.s the axis
of symmetry and (X)Ia1 'uuu.lim1l~ arc defi ned jn the xy..plane with the origin as pole '*nd th~
positi ve xaxis as po lar 1.\."(is~ then cylindrical <:oord inates a11d Ca11esian coordinates arc related
by lhe eqw1tions
(13(1() )

MOICfiii.IAfl.t M Rda1iooship~ ~wun Cru1esian and


cylindrical conn1imucs

(see Figure 13.64). Reca llthlll r can be expressed in lcnns of x and )' by

(13.61a\
~<.y. t)
lr. 0. Z)

and 8 is defined implici tly by the equations

..

sin 8

)'

= 1 =;':;:==;'
Jx2 + y2

(IJ.61b )

To usc cylindrkal coordina tes in the evaluation of triple in1cgraJs. we must express equations
of surfaces in terms of lhese coordi 1mte.~. Bulthis is very >imple, for if F(x, y, z) = 0 is
the equation of a

~urfacc

in Ca rtesian coordinates, then to express this a1uation in cy li ndrical

coordi ntues we substitute rrom equations 13.60: F (r cos 8, r sin8 , z) = 0. For example. the
right-ci rcular cylinder x 2 + y2 = 9, which has the ;:-ax.is as its axis of symmetry, has the
very simple equation r = 3 in cylindrical coordinates. "llle rightcircular cone.;: = J.t2 + y 2
also lm:> the z-axis as its ax is or symmwy. and ils equation in ~-yli ndrical coordi na1es takes 1he
simple form .: = r.
Suppose thlll we arc 1o cvulumc the 1riplc imcgral or a continuous runclioll f(x, y, z).

JJi

.f(x. y. z.) t/lf,

over some region V ofspace. Thcc hoiccof~1 triple iterated integral in cylindrical coordina tes
implies a subdivision of V in1o small volumes d V by me.ansof coordinate surfaces r =constant,
I) = constant. a od z = constant (Figure 13.65). Surfaces r = constant a re right-circular
cyljnders coaxj al with the z-axis; surfaces 8 = coosta ot are half planes con~.ajoing the z-ax is
and therefore perpendicular to the xy -plane; a nd surfaces z = constant are planes par.tllel Lo

the xy-plane.

956

Chtlptt l' JJ

i\lultiple l t~tep:ll
Elc:rn~nt<tl

IIPUiillm,..,...--.,jUJrtT"

a led ...,.;111 CQCJRiiiU!.te JSllrfuo:.' tS 1"

,o!ume in cyliodricul ce<}rdirwt.'> i.s ere-

= C\>ft:SliUU, ~ = (\)JlS{#f\1, 1. :

()DSt.l111

r = consiUnC ~

d!l L...ZVatalW

Ek;il'ICf\t(ll

\Uume in cylindrical roon.linatcs is

dV - rdzJrd9

If we denote sm!lll variations in r, 0 , mld z l'or the clement dV by dr. dO. and d z (Pig
urc 13.66), then Yolurnc of d1c clement is approximately (r dr dB) d z, where r dr d(J is the
polar cross-sectional area paralle.l to the .xy-plane. Hence. in cyliru.J rical coordinates we set

c/V

:'~>.

: '-,'.,
',

...

.. ......

= nl~drc/8.

( 11.62)

The integrand f (x, y, : ) is expressed in cylindrical coordinate< as f(r cos (J, r sine , z) . It
remains only to affix appropriate limi ts to the triple iterJted integml. aold these wi ll, of cuUJ'lie,
depend oot which of the six possible iten11ed integrals in cylindrical coordinates we choose. The
mo!it commonly used triple iterated integral is with respect to .::, r. and 9. and in this case the
zintcgmtion add< the quantities f (r cosO. r sin 8, ;.)r cl:dr dO in a vertical column, where
r and fJ are consmm. The limits therefore idemify surfaces on which each and ever)' c<>lumn
sums and stops, and generally depend on r and 8:

l.

h ,(.OI

f(r cos 8 , r Siol 8 , z)r liZ dr dtJ .

v.~)

TilC remaining imcgrations with respect tO r aold 8 pcrfonu l!ddit ions ovco the region in the
xy-plane onto which all vertical colwnns project. Since r and 0 are simply polar coordinates,
1hc rinlcgr.uion adds over small areas in a wedge nnd the &-imegn1tion adds over all wedges.
1llc triple iterated imcgral with respect to!, r , aold fJ therefore has the form

ft! g:(Q)f. ll: [r,8)

1
o

8 1( /J)

f (r cos(), r sin e , z)r d z d r d (J .

/11V\ 0)

We comment on the geometric aspect.' of these additions mor< fully in the following examples.

I EXAMPLE

13.29

.........
Find the volume inside both the s phere .T 1 , y 2 + z 2 = 2 and the cylinder x 1 + y 2 = I.
SOl UTION 11le required volume is eight tioncs the first O<:tilllt volurue sh(>wn in f igure 13.67.
If we use cylio1drical coo rdin~tcs . volume of an elemcnt<ll piece is ,. d z dr dl}. A z- imcgr<~tio n
adds these pieces to give volume in a venical column:

r..h-rf rdz drdtl,


Jo

"h<re limits i n~icate 1hat lbr \'Oiume in the lk s t ocrnnt a ll columns start on the x y-plane (\\here
0) 11nd end on the s phere ("herr r 1 + zl = 2 ). An r-intcgration now adds volun1~ of all

;: =

columns that stand o n a wedge:

fi,'rr=;r

Jo

rthdrdO,

"here limit:> indicau: that ull wedges stlllt ,. the origin (where r = 0) and end on the curve
I (orr= I) in the .ry-plane. TI1i ~ integrutiontherefore yields the \"Oituneofa slice
(F1gurc 1167). Finnlly. the 8-integrdtiotl add~ \'Qi u mcs ul' all MICh sl ices

x 2 + y1 =

f''! {'1 ./!-' rdz drdfJ ,

l o .fo o

where limil< 0 and 1r /2 identify positions of first nnd liNt \\edges, respocti,e ly. in the fir.t
qu.idrdnt. We obi~ in the requir<d \ Oiume. then

1"/:!.If.
0

llbi<IC lh~ ~~ ,,:

.;'2? rd:drd8= S

1/li.
U

r.,l2-rl drd8

Cl

C~ln~J COI.Ulbale\ fO cal<l~art lhe \'Ohutle

+ y! + :! z *"CC

lhc cyh..dcr A .. +) - I

..

or

'
I lad we u'<Cd utriplc il<rated in
.I .
ha..: ubcain ..
tcgra wuh IC'JlCCt to

"'

'olume

= 8

1
0

t, )'.and .x in "anlple
""
11.29. we would

11./1-x!.' .Jz-,>-y> d:dydx.


0

To app.reciate the ~....t.luc or cylindrical coonlinatcs. try to C\'aluatc this triple itcrdtcd inrc:ft>r~J
!\s furthe r CVIdcnt'C of the val f r d . I
.
.,. -.
. .
uc o cy'" nc" coonhnatcs, rcpct Ex=plcs 13 25 and
13.27 u..,ng cyhndncal
COOrdinates.

I E XAMPL E

13.30

FincJ the moment of it'Cnid or a uni(orm right<irc:ui.U' cone of r~i~ R Ul'ld ht-ight h 3boul its
:1.\~

MJI.L"TION

Orn:q"-lrt<r uf ;uch a <oonc is shown in figure 1.168. We mukoply '"moment


che :-axlo, by 4:

o( inertia abou1

41 ''1"f"
6

1, -

= -4ph
R

r 2 pr d :drd!J

J llr/ 11

"

= 4p t ll { ~ [r~r
Jo Jo

drdiJ

ltr/ N

1/>[.N r CR - r )drdO = -4{Jh. l.'/! lRr'- r' }"


dO
1

II

...

"

= phR' {o jl> = p;riJ R'


5
Mil( Ill J!MI

10

.'

)'

EXE ACI SES 1 3 . t 1

In t-~cn."~' 1-10 hrk.lthc ('({tutklll1 fhc surface rll cyllrk.lncaf coorWnlll~' Ort~:w cuch urf...._"C

.f:

+J 2 =

I. t

=0

+ 16. Find tho: '-ohamc inside: the sphere.\ l

:r' + y' + : ~ = 4

4. x-t-y =S

s. z- 2/.'+ yl

6.

x 1 + 4y 1 = 4

z = x2

8. 4.?: = x 1 + y '

9. ,\"a X

10 f

!he cylinder x' + .v' = I.

+ >'' = 1

2. x'

J..)''+:.1 6

7.

+ .' + :z =

and rnd~tatc whclht....- it is "ymn"'-1ric ut'<'IUI

lhc t.ltl&.

I.

15. .\

+ ."~ - 1 + z2

fa J:>.C'n.'uC'~ I J- J5 hnd the \'olu.mc bounded by &he MJrfa~

+ \".! + : 2 .....;. 4 tlut ouuitk

to 17, Find ll'k! 4;.'Crl4 rc of m a!l~ of 11 uniform I'ICrnisphcric.tl solid.

1 Ut Set up Ihe si'IC triple i~enw.'\1 intc~niJ s m cylindt'tcuJ coordinates for


IIlii! triple intcml of a futll.1inn J (,\ , )', Z) over the r.;gi011 OOundcd by
Mttfucc:-<t=l+x 2 + y 2 ,.x 1 1.' 2 =9. <: 0 .
t

19. Rnd 1he n\Ontcnc of inertiu of a unifonn riglucittular cylil'Kiet of


ikltu,. R ~11d height 11 (a) 100uc ih ;~;, .uxl b) abou1 41. lille through
tl~ ro\UC of itS base 1nd f)Cf"f)CAdkular to ts uxis.

20. Mnd the nlOflK'nl of ina1i.J of a un.form 'phcrc or radius II alxxt


m) line throu:.h its ntrc

11 .

l - /rz+\'z.z: ..... ,.:= ~-

12. z J 2 - xl

13.

l=Xy \ 2

14. l

yl. 4 =

xl

~=0

+ yl

In Exercises 21- 25 C\"a..UaiC the tr1plc itcrJicd integral.

+y2 = l .l=O

= x 1 + y 2 l

=4 -

x' - y 2

21.

!.'1~f. ,;;r;;;
0

dzdydx

13.12 Triple ttem e(11 1lC:~al$; in S,:tle:ri~al Coordi1l.'\!S

959

* 29.

A casting is in the fonnofa sphere of radius b with two cylindrical


boks or raditLc; a < l> such thtlt the uxcs or the holes J)(I!)S through ltl(:

22.

centre ofthesphcte and inlc:rscct .ac 'ight a.ng.k s. What ''olume of n-..:::tal
L~ required for the l-a>ting?

2.1.

24.

!.''"''
1.0./J/11;-.~
J2>-'
(~ r.'

25. ['

lo lo

.o

30. Sl10w thtd the volume ))()lllxk;d by the cylindl!r x 2 + y 2 = n z ~nd


the flltriillcl plane~ t - my und t = my+ h . where m and h arc
con:na.nts, is lhc smnc u~ acyl in<lcr of radius R tlnd length h.
yd~d.r dr

In Exercises 3 1-41 li nd the vo lutnc dtscribt:d.


"' 3 1. Bounded by .t2

y> )'l dz dx dy

* 26.

To an.alyte running technique for sprinters. it i.s ncccsStuy to calcu1:l:IC Lhe momcm of iocnia of the legs about :i hcJotizOnl:allinc through
tbc hip~ H (left figure below). SupJlOSC the upper aJld lower portions
or 1hc. leg or an athlete (including the frol) are moclcllcd as c.:ylindcrs
wi1h lengths 0.4S m and O.S m and radii 0.07 m and 0.05 m, rcspccthdy (right rigurc). The :n hlciC has mas.~ 73 kg. of which 13.7% i.s
in each upper kg and 6% _in each lower leg. Usc E;terdse 27 <IIHI the

para.llcl axis theorem from E'<crcise 22 in Section 13.10 to determine


the mon-.cnt of i nertia when axes of upper and lo~'CI' Iegs make :;tn an~lc
of 1r / 3 rJ<Uans.

ul' upper leg


G~,. =

Centre of mass
of l o~Acr leg

z = x' + y'. z =

* 33.

x2 +y 2+z.: =

Bouncledb)
(smaller piece)

.azl

0.

= x2 + y2

(x 1 + y')' = x'- y'

4 . .t 2+y!+z 1

16, z

= l '"x"'-+
,.-,.J"'
''

34. Moundedby.< 2 +y2 +z2 = l,)' =.t ..t=2)'. t.=O(illUlC


fir))l QC.1a.nt)

35. Inside bo!h 2.< 1 + 2y1 +


36. Bounded by z''
37. Inside x 2 y 2

= (.< +
2

8 and .<2 + y2

y 2) 2

x' + )'' = 2y

z'

+ + z~ =

.. 38. Bounded by

a 2 bulQUl!idc x 2 + y1

z = 0. x' + y' =

+y +z

1.

=1

=oy

z = e- ' -r.

= 9 bulouts iUc x 2 + y 2 = 1 + z2
:t. 40. Cu1 oil from <: = .r2 + y2 by z = x + ,\'
4 1. I nside x2 + y 2 + ?. 2 = 4 and below 3.z. = .r?- y!

~pperleg

r;;

32. Bounded by

39. Inside x

11

Gv =Centre of mass

+ y 2 - z2 =

+ 42. E,aluutc ll"'C Lriptc integraJ of Jx 1 + ,\'! + z 2 ovet tre region


bounded by z = J and z = / r ' + y'.

Lower leg

43. Eahlle lhc triple inlegml of J(y, >:) = IY>:I over lhe region
bounded by <; 2 = l + x2 + y2 and z
j4 - .t 1 - ,\' 1.

27. Show that the moment of inertia of a righH.irC'ulat cyli nder of

r.\dius R ~ length L, and constan1 density. about an axis through 1ts


centre and perpendicular 10 it~ length is m ( /{2/4 L 2112), where m
is the mass oJ' the cylinder.

+ 28. Find the ccnue of ma.~ for the unifornl solid bounded by the sur
fncc8 x 2 + .v 2 2x. t
x 2 + y'!. t
0.

=/

...._ 44. A turnblcr in Lhc fonn of n liglu-circularcylindcrorndius R and


height It i s ful l of \\1\ltcr. As the ax is of the tumb!(r is tilled fron1thc
\'Crt.icul, wutcr pours over Lhc side. Find the ''olumc ol water remaining
in 1he tunlhler 11s a function or the ~nglc bcl ,~ecn the vertical und the
axis or the tumbler.
.. " 45. Usc cylindrk:l C<)(.l'din;llc.." (() find lhc volume ur lhc:. loru.s

(/.r' + '''- <l) 2 + ~' = b 2 b

< a.

l t 3.12 Triple Iterated Integrals in Spherical Coordinates


iOiii!Ltl iliii

Rd

Spherical coord inates are useful in solving problems where. figure.s are syrnmeuic about a poinL
If 1he origin is 11u11 poi111. 1hcn spherical cMrdina1es (~lt. </>.(I) t in Figure 13.69 are rela1ed H)
Canc-'ian coordi nmcs (x, y. z) by 1hc equal ions

lions:taips bet.-een Cartesi2n and


s.pb~riatl

CI.XJl'din.ttcs

:Jl

(x. y. :)
(:ll, ql, 9)

x =

~lsin cos li,

y = 9tsint/>sin 8.

z = 9tcos.

(13.63)

As is th e c ase for po lar and cylindrical coordinates, without restrictjons on 9l, tf>, and fJ, each

point in space has many sets or sphel'ical coordinates. The positive value of its spherical coo-

'

)'

t Mathe.maticians often usc the lcttc.r 9t hl stand for the sel of real numbers. We have avoided u.<>ing it f()l thi.<> purpose
so that oo confusion would ai&e-in this SC:lion.

960

<:taapler 13 MUI1ip1e Integrals

dinatc ~~ is given by

( l l 64a )
The 8 -coordinate in cylindrinlland spherical <'<lllrdinates is identical, so that no simple formula
for 9 in terms of x. y , and z exists. The 1/>-coordinate is the angle between the positive z-:txis
and the Iine joiniug the origin to the point (x, .\ ', ~).and that value oft/> in the range 0 !5 1> !5 ;r
is determined by the formula

'I'=

-t (Jx2+ z

cOS

yl

).
+ :2

( 13.tHb)

To transform equations F(x, y , z) = 0 of surfac~ from Cartesian to spherical coordinates,


we substitute from equations 13.63:

F (:ll>in<bcose. :llsin t/>si nB. :)Ieos) = 0.


For example, the sphere x 2 + y 2 + z2 = 4 is symmetric about its centre, and its equmio n in
spherical coordinates is simply 9t = 2. For the right-circular cone~ =
+ y 2, \\'C write

Jx1

l)i cos = J:J!l sin 2 cos2B

+ !l!2sin2 sin2 8 =~>!sin .

Consequently. tan = I or</) = Tr / 4 (i.e.. <fJ = Tr / 4 is the equation of the cone in >t>hcrical
Ct'lOr<li nate~).
Suppose that we :Ire to evaluate the triple intcgrnl of a function f (x, y, :.) ,

J!i

!Cr. y.z)dV,

over some region V of space. The choice of a tiriple iterated integ ral in spherical coordinates
implies a subdivision of V into small volumes by means of coordinate surfaces l = constant .
if> = constant, and 8 =constant {Figure 13.70). Surfaces 9l =constant are spheres centred at
the origin: surfaces 1/J = constant are right-circular cones symmetric about the l -axis wi!h the
origin as apex; and surfac-es 8 = const.ant are half planes containing the z-axis.

wataii;IWF*I*
with

cuorililw.t.e ~wfacc.s

Elelnl!ntal \olu.me i.n spherical eoord:inat('S i~ cr<ak'd


:)1 = cu:l.S!<UII. = ~'\ln~ant. 0 = (.VIl.'ltalll

_.,\(j

lfwedcnc)Cc~malhunkU (Wi HI :)t, 1/t. anc.J 8 (on he elc:nlCn1 d V by d :lt, de>.


d9 (Figure
111 l'eelangular p<mllelcpipciJ with dimc:nsio ntrt d:n. :)t d~. anc.t

O~ntal

\~ ...le In "tliiM'IIlolf '"'"'""'Jill.aiC-'- ,._


tH'
m 1 ~om~ u :l t(lfwiO
Le11~111

11.7 1 nnd appro)t.imarc dV hy

:.Jf~lui/>dO. the n

:H s~n t> tHI

h rcn11:dns only co affix .1pproprh11(!. limits to rhc triple llcmtt..-d intesnt1. urxlthcGe limht dcpend
0 11 w'hich o f the .six pno;,.ible lri plc itcnued in t csml~ in ~ ph c:r-ic11l C..'O<mJi,tutc:" we: c:htlu:o.e. If we

usc t triplc iterated i1Hegral wi11t rc'(pcctto

~)L

q,. and O. chet1 ic i:o of tile: form

1/lJC.r.),:) iiV

,,

i nu:rprc1.41ion~ of the uUc.litions rcprCl')CIItetl b y dle,'C;C imegmtiorlS widl rt'\.pecl lu


:ll . f/J, and 8 are left ll> Lhc e\llntl>le.~.

111c gcomcltic

I EXAMPL.E

13.31

..

Fiml the 'olwnc of a sphere.


SOLl 'T IO.' ' Thccqut~tion ~Jf $J)herc ofntdius R CCIHfCtl at the otlgin i~ Jt1 + )'1 + :l = Rl
or. in "J>htricuf coonlinalc~>, :lt = R. The volt.uue o l thi;;; lll)l'ltre ls c:ight lil~IC-" 1hc fiN.~. occan t
\olumc $hU\\I1 ln Figure 13.72. If we use ~heri c.d (,.'UQrdirut~es. the volume o f an elememal
ri~eis

~tlsin4> d~ d dO.

An !)(-uucgra1ion add~ these ' olumcs ror oorastant 4J mnJ 8 10 give lhc volume in a "'!:!pike~~

where limit< indicate tl>at all >-pil e< Mdrt atlhe origin twh= ~I = 0 ) wid c:tld on the <phcre
(\\ here ~~ = R). A intejVation no" dds the vul umc of Pikes for ec<lot<llll 0. TI1is yieltls
the 10 lume ofu slice

1ll1R :n

2 si n

11m dr/> dO .

where limit.< indicate th<lt all >lice in the fim oetanl Start on the =~i-. (whcJC rl> = 0) aml
crxJ on the .ry-plarr (where 1/1 ... Jr / 2). Finally. the IJ -ioucwation adll lhc volumes or all such
~lice<

"/11"/11Rllt
1
()

si n <P d!lt d<P d(),

where limiLS 0 ;md ;r /2 id~tuif)' p..-.~>i t,()n~ Qftlnn a' d l1.1.:,l ~i..:c:;. re:-i\'K:Cli'-'C\y, in th(. ftr'$t (ll;tam.
ihc volume ()f the: sphere 1S therefore

...

f EXAMPLE 13.32

find the centre of mass. ora solidobjc:ct of consu,ntdcnsity H' h i:-. in the shape of a r\g.lHc\rcu\ar
cone.
.SOLUTION

Let the cone have altitude It and b.tsc radius R . The t\ t~ mass i.~

M=
whc:rcp is thr::dcnsityoflhcobjc:ct. lfweptacc axcs~sh<>wninfigurc \ ).i3, thc:n X:::): = ()
(i.e. the ccnlrc of nut.'SS is on the axis of :-.ymmctry of the cone). ~ro tlnd t we otfer three
solutio ns.

Method 1
If we usc Cones ion coor<Ji natcs. the cqut\tion ot' lhc :;urfl\CC or the cone h; of
=
kJ.~l
+ )'l. Since (0, R. II) i,; a p<>inl on lhc COO\c, II = k R. n'<l lherefore
lhc forn\ :

k = 1/ R.
M"f.

MjiUiJ ...Wfl&

Centre of nla');.) of a

CQOC

using Cancsiao and l'):liodricaJ ooordinates

It

''
(0. R. h)

z = k\xi + )1
t

or
= kr
,

.Y l + y l ~ Rl
()(r:R

If we set x

= R sin O. then dx = R cosO d O, and

3
;r R21Jp

Method 2

3il
=

If we usc cylindrical coordina1cs, the cqua1ion of 1hc surface o f !he cone is

: = kr. From Figure 13.73, we see that


M l. = 4

.<}21R1h :pr d :drdO = 4p 1.T}21RI,-!}" drdO


1
OOkr

Again, then,

z = 3h / 4.

00

~
tr

Method 3

Cc:U!J'C

of nll,, or C\.\Oe u!J.ng

I f we usc spherical coordinates, then the e<juation of the surface of the c()tlc is

~phefkal

coo:dsnate~

z
t =llor:ltcolttt;~=h

From Figure 13.74, we see that

I
= 4p

~12 ( Go {~14

1
~1 1-''
o
1 o cos 4>
0

= ph"

y
X

plr

=2

Jo 4

sinqicosqi

~A~

dd8

sin
- d<P d8
3

I
)
~f2
cos 2 t[> 1 - 1 (Olo

1, }9' dO
= ph" 1~12 {2cos-t[>
0

rrph'

- 4-

(h +
2

frl

)
- 1

Again, ~ = 3/r/ 4 .

..-..

EXERC ISES 1 3.12

In E-<ercises: 1-7 find lhc cqua1ion or lhc: surface in spherical coordi

nates. Drow the surface.


I. x 1 + y 1 + z1
3.

Jz=

2. _,z + yz

=4

Jx' + .r'

4. 4z

5. J = .T
7.

6.

16. A solid ~pherc of radms R and ccnlrc :11 the origin h3s a coni in
uous Chllf'Se d1s:tribution throughout. [( the density of the ch:.trge is

p(x. ,\'. Z) = k /x' + ,1' ... z' coulombs per cubic metre. find the
total charge in Ihe ..;phcre.

= xz + yl

+ 17. Set up the: ~i.~ triple iterote:d in1e:grols in $phc:rical coordinates (or

tbc triple inlegr.JJ or a fum,:tion /(.t. )'. l) 0\'C:r lhc rcg_i()n in the Ji~
OClaRt under 1he sphere x2 + .r 2 + z~
2 and inside the cylinder

.rl + yl = 1 +z1

x z + ~,2 =

I.

z= -2/x' + y'
In Exercises 18-19 e valuaLe the triple iu::rated irue,:ral.

In Exercises 8-12 find the votmnc descnbcd.

8. Bounded by< 9. Bounded by l

/x' + y'. z = /

= I, t

10. Bounded by x 2 + v'


first octam)

= ,/ 4- x' -

18.

.v'

+ z' = I, y = x,

y = 2x. t

= 0 (;n 1he

19.

r.

C)

'L~~ J-'-,.,
~

dzdydx

T"-+)'

... 20. Fiod a formula (()r the \''())umc or the sm.dler n;gion bounded by
x ' + J'' + z' = R' nd z = k -/' + -''' (k > 0).
12. Bounded by z = 2/xz

+ y z, x! T

}'~ = 4~ z = 0

13. f ind the centre of mass of a unif(l(ln hemisphcric.a l sotitl.

14. F'tnd the moment of inertia of 01 uniJorm solid spl'len: or radius R


about any line through its .:entre.

IS. Find the first moment about the yz: -pl::mcofthe unjfonnsolid in the
first octant bounded by thesw1~1ccsx ~ + y 2 +z 1 = 4.x~ + y 2+z 2 = 9.
y = 0, ond y J3x.

22.

(a) Usc Afchimcdcs" prillcipk ro dctenninc the dc::nsity of a


spherical ball if it float~ half submerged in W'J.ter.

(b) Whal ro.-ce is required to ~eep the ball with itS cemre at a
depoeh of cmeAWr the radiw; of the ball?
*- 23. Find the v<:~lume bounded by the surface (x 2

2z(x'

+ y' ).

+ y 2 + z2) 1

'*' 25. ;\ :s;piH:n.:: ol" t.on:sw.nt Ucm;ity p aru.l

r.u.li~ R ~~ loc.atctl ut the


origin (figure bdo.,.,). If u ma!-S m i~ :,itud.ttU at a point P on the taxi~
(di.stancc d > R from the ccnltc ur tbe sphere) and d V i:t a small
c!enk:nl or VULUitrlC or the Sphere. then OC-corc.ling 10 Ncwtons uni\ctsal

+ 24. A sphere of radi~ R t:01rric:s u uni(orm churgc Ubtribution of ()


(;Oulombs per cubk utetrc (figure below). I( P i~ a point on the zLxis

(distance tl > R from the ccntn: or the >Phere) and d Y is a smaU

clement or YOiumc or lhc ~pherc. then poo,:mial at p due 10 dV is


gh'Cn by
pdV

law of gr.l\'ilal ion.the ZCOmf)Onent or the l'orce on m due lO Lhe mass


in J V is gi\-en IJ.y
Gm p d V """ o/1

---s

where s i!' distance frc:nn P 10 dV .


(3)

where G i!: a com:tanl and s is.: distance between P and tl V .

soo.,... th:u in tcml$ or spheric!tl cooni.in::nes pQ(cntial

v at

(a) Show that in ~phetK: al \:UOrdii'I31C$ total fon:c on m due to

the c11tirc sphere has t .o(.."'ntponcnt

fJ due to the entire sphcr.: is

G".'fl
F, = - U.l

1'-

/':JI'

4rr foo

F,

- .'l

Gmp 1" 1'1"~" ~! (s' + <1 , = - -:;:-;ul

_ ,.
J- M

- " 0

J - fl

91') tls <i!J! <Ill.

,fi

nnd show lh:tl F: ~ -GmM Jd'l, where J\.1 is the 101al


mass. of Lhc sphere.

mds d~! dO.

(CJ E'"'.lluatc the in1cgr-al in JXlt1 (bJ tll verify thai V


Qf (Jtrfotf). where Q is to'"l charge on the sphere.

$)

(,

Show that with dlis change

!'1"1

':J!son4>d:Jid~dl!.

-:r t)

(b) Usc the tra.nslbnmation in Exercise 24(b) to write F: ln the


focm

(bJ Kecause this: ilemtOO integral is wry difficult to evaluate. we


replace~ with the =iable s =
+ d' - 2~!d cos 4>.

p
v = -----:-;;

j ' J.'J.R (s' + tl'. - ~!' ) , .

=
P<O. O.d)

P(O. O.d)

R,l"'oo- ._

R /

.!< ....

26. t\ homogeneous solid is bounded by two concentric .spheres of


radii a and b (t' < b). Verify that the force that this layer exerts on a
point mass at an)' poi111 interior to the shell vanishes.

113.13 General Transformations in Multiple Integrals


Geometric discussions that led to the area clement d A = r d r d8 in polar coordinates and
the vohtme elements dV = r d z d r dO and. d V = 2 sin tim d<t> dO in cyl indrical and

spherical coordinates \vere successful due to t he simplicity o f coordinate curves and surfaces

in these coonlinatc :sy:stcms. In coon.linalc ~ystcms with a less simple geometry. c.:orresponding
discussions could prove much less transparent. Jn thjs section we disco\"er how to evaluate

multiple imegmls in arbitmry coordinate systems. Our approoch b geometric. but the linal
result is stated algebraically.

Suppose that we are to evaluate the double integral of a function f (x, y) Ol'ersome region R
in the x y-plane, using a coordinate system (u, v) related to Cartesian coordinates by equations

u=u(x ,y),

v=v(x,y).

(1 3.66)

ql ~ncm

A ou
1u 111 '-('101\llbl n

I'

(.1, \')

Ulllt

~l
I ll ll,\llljt Cru1CI'I,flfl COOrdinJ~ IC~ IQ C\'llhratc Ulc dC.ubiC iiiiC"tf!l or { (X, y) 0 \ Ct R , we

s u lx.fivldc R irllo r~o.,:tlmglcs by rncul" o f a grid ur 4.! 1l(JI'\Iinale liner-. x = C(ltl~tnt\t ond y =
corhfwH. The a.rca clclllcnt d A a- 1./.~ d y ifoo then thl: (fCChUl~\llnt) urea bounded b) c\.MJI'dlluue
c u no who.-.c ~p3r<t4ivn~ are dt atl<J fly. In a ~imil,u way.IU u"c Ul1<uurdit\.\U:'- ~~f.....,.,~
c u rviUncr Jfid \ \ith t."OOI'l.lina(C rune\ U : - COrl, l,ll\l ttfld l1 ~ OJO.lUUU (Fig_UTC 1:\.lS). Tht
urc.t ckn-.cHI dA it\ the nrc bou4.1crJ by rocm.linalc: cune.' that differ by anlOlull' filf nnd du .
If d11 .-ud dtJ arc Ycry ~mall, then dA c11n be approxi'llutcl..l by o l~uullc:1ugr<'l'l'l (Pwtu-c \3.7(,)_.
the liCC:Ur;tc:y of the "PJJfoxirn<llio irl("l't:fl'iing U\ tl u rtnd d u d CI.I'C~M: . To fi1l~ the uNa o f lht
appt'ollinwring putullcloarnm. \\C need tnngcnl \'ccton to the ..:ourdinatc curve"' u ~ uu and
u = l 'U " P . Since p.uamecr: equation.~ f()C" u - "u ~X = \(uo. u). y = \'{uo. \1),

r,a,,gcnr ' cc1or is

a.r

- 1
1) IJ

ily ,

+ ;;-J
.
UIJ

ax. +
( .,.-I
dt.l

'1'.

H',; '') dv .
J

vP

l'lw: compont:lfS of T, rcptc':)crll the changes in \ and ) CUrTQpondinl lo the c.t u.nst d"
coordinate CUM e$o, .m(.l therefore the 1CnLh Of lht~ l.lngcn\ ' c;:<.U,)I' docs inJcc.d
rcprcsclll the: length of the ,)ide of the ptlttlllc log.ra.m in Figltre 13. 76. S imitnrly. the :,ide the
JJIIntlleiOJ:;ntll1 {'OrTC.'IpOmllng t() U chtt tl~C t/11 itl II ulmiJ U- t~t i~
111 V bet\\CCI1

or

i)x.

T , .. ( -

au

1\t.:cordilljt to
VCCIOr

h.) du .

+ -j

ilu
11.4.1,
the
lll"<:t
of
the
ap
>roilllmins
put'OIIclognun i~ the length of the
1
e<ru:taion

- ( iiX j + ayi)du
au

ilu

( CJ,rj + !!rj\ d u

av

(axau auay - avau


ax ily ) du dv k

il(x. y) dudvk.
il(u, u)

av ')

Consequently, if d u

~ rld d v

are positive, the clement or area d A in u Vcoordinates is


dA =

1b cvalucuc the doubl-e irUcgr:tl of'


of u Hrld v. arHI wrilc

!}.

j(x , y) dA

Ja(x
..v)l dudv .
8 (u. v)

( 13.Mi

f (x, y) over R , we usc 13.68 to express /(.r,, y ) in term~

fl.

I I

a (,r . v)
f lx(u. v). y(u, v)] .

du d v.

( 13.70)

8(u, v)

R. ,,

Sillce [he right side o f this equation represellls a double ilcmted integral in 11 ant.l v. the integr.:.ls
should luwc limits . B ccuuse ch i~ c" n be done only \\'hen tl rcsion R has been speci fied. we htwc
used the r\()(ation R11 ,. to rc.prcscrlt li~ni ts describirlg R irliCmls of coordinates 11 Ml<l v. This
resuU can be stated in terms or double integrals also. Under t J mn~formatitJII uf ''mlftlimllt:i'
13.66 or 13.68. dtwble iterated incegrals tnmsfonn according to

!.r

J R,, .f<.v.y)d.r d y =

11

Ia<-. I

y) dud v.
J R"" j(x (u.u), y(u ,v)) o(u.u)

( I 3.7 1)

TI1c gencnd fu nsrorruation for 1riple integrals is as follows. lfCa rtesil:lll coordirlalcs (.t . y, ::)
w) in some region V of s pace by equation.-;

l-lfC I'CI1ucd 10 cui"Vilineat cool'dina1es (u , v ,

x = x(u. v. w) .

y = y(u . v, w),

= z(u. u. w).

(1 ~ .72)

then

j )J/ (x,y.

!},l

f[x(u. v, IV) . y(u. v, IV), ;;(u. 11, IV) I 8(x, y, z) dudv diV
il (u . v. w )

f(x , y, ;;) c/ V =

Iff.

1.[x(u . v , w). y(u, v. w ), 0(u, 11, w)J

z) dV

v...

(1 3.73)

or

fry

\:n :

~~..~~~

Iacu. I
i)(x l'. <)
v, til)

dududw. (1 3.74)

I EXAMPLE 1 3 .33
Show lhHt t.ransrormat ion laws 13.71 ~ nd 13.74 1~0 to the: corrl.*CL<J itYcn:uli~ll c.:xpressiQilS ror
arc.a and voJume in polar. c ylindrical, and spherical coordinates.
Tmn.."formatjon law 13.7 1 leads in polar coordi nates Lo 1he area e lement

SOLUTION

ciA =

a(.r.y) j drdll where x =

il(r. 9 )

il(x, y)
iJ(r. 8)
we huve d A

..

fix/88,
ilyfil8

lcosB

- r sinO

sin 0

r cos O = ''.

= r dr d B. TransformaJion law 13.74 i n cyl ind rical coordinates gives the \'Oiumc

clcmcrudV =

il(x,y,z)l
. .
_ drd8 d z,whcrc x = rcosll,y =nin lland z =z.Si nce
l(1(1' 8. <)

iJ(x,y, z)
iJ(r, 9, z)
we have dV

= i ax f or
ilyfor

"

ro:osu and ' ' = rsm ll . Smce

axjiir ax /il8 ax;a z


cos(!
= ay;ar ay;ae ay;az = sin II
az!iir oz/o8 iiz/oz
0

r dr d(J dz.

-r sin 8

,.

r cos II 0 = '
0

F<>r spherical coordinates. we. <>bu. in dV

a(x , y, z)

iJ (:)I, </>. 0)

ax;am ax;ay, ax ;ao

:>t cosy, cos e

s in q, cos 9

= ()yj()9t ay/o oyfCJ8 =


<l;;/<l ~l J<./ iJ <lz/JfJ

I:~~')',
~
(~t ,, )

mcos s in e

sin~lsinli

- t sin 4> s in e
91 s in cosli

-91 sin

cos

d91d<b d8 . Since

itfollowsthmdV = ~t2 sin <f>d~!ddfl .

.......

I EXAMPLE

1 3 .34

E valuate the double imegral of f(x, y) = x 3y - xyl over the region R in the first quadrant
boundcd bylhchypcrbolasxy = 2,xy = 4,x 2 - y 2 = l ,x 2 - y 1 = \1.

SOLUTION Evaluation of ony double imegral over R by means of Cartesian coordutates


requires three double iterated integn1ls (Figure 13.77). To improve the s ituatio n, cons ider new
coordinates (u, u) defined by 11 = x 2 - y 2 a nd v = .<y. The imcgrand f(x . y} is expressed
in terms of u and vas xy(x 2 - y2 ) = uv, and therefore

!!.

f (x.y)dA

!!.

a(x,y) l d11dv .
lil(u, v)

uv ,

R..

MO\ilciii;;I#I.,.~"D

E\"ahaatii.Xl of the OOuble in~.tl of .t 3 y - .ry 3


over n region boond."<< by hypc1bolas usi.ng a change of coot'dioatcs

x' - y'=

or

.YJI - 4
or
v =4

u; l

.xl - :1' =9
or
~ . 9

+---..i..______

__L_ _ __

3
To evaluate the Jacobian we could solve for lhe inverse coord inate trdnsformarion defini ng

x and y in terms of u and u. Instead. we ecall from Exercise 23 in Section 12.7 that

a(x , y)
il(u , v)

= il(u, v)

= Iaufilx

a(x, y)

iJv j ax

au

;ay I = .,.l.,..2y...-, - _.,2,...y...,.l = 2(x 2 + y2)

8v/iJy

u and v by noLing that the combi nation xl + yl is obtained


by squaring the first equation defining the coordinate tronsfonnation and adding four times the
square of the second: rt 2 + 4 v 1 = (xJ - 2x 2 + y') + 4.t 2 y 2 = (x'2 + y 1 ) 2 . Consequettl ly.
This can bt expressed in lerms or

fj

' { f(x,y)d A =
R

JJ.

R., ZJil

;'v+ 4v dudv.
2

The reason u and u were c hosen as above was to s im pliry Lhe limiLS of integr.ttion. Coordinate
curves u = constant and u = constant define a hyperbolic grid with the hyperbolas 11 = I,

IJ. IJ

Gen~1 Tr.llh(Om'lJUOOi i n Mullip(t

lnte2f'.liS

969

u = 9, v = 2 , and v = 4 formi ng 1he boundary of R . lmegra1ion wilh respecllo 11 holding v


constant sums over the area elemems in a hypetboljc strip. and the v inccgnuion then adds over
all s~rips . We ob1ain

!'(J
R

1'!. 2Ju luv+ , dud v = ~2 1'/vJu + dv


= 2~ 1"<vJ81 + 4v vJ1+ )dv = .2._
{(SI + 4v
24
9

4v1 )

f(x . y)dA =

4v-

2 -

9
I

4v2

) )/

(1

= .2._ (1453 ' 2

24

65312

97312

+ 4v 2) )/2 ) ;

+ t7"' 2 J.
y2 and v = .ry as: a mapping of the
in the fiVplane. The four edges of Rare mapped to

An allemati\'e view is to regard the equations 11

= x2

region R in the XJ plane to a region Ru~,.


a rc<:langlc in lhc 11 V planc (Figure. 13. 78).

fL1 l!i!hl CJ!]iM

A. t."Vudinate tr.w~fonmnion ean be cq.a.rdOO

as a mapping from lhe xyplane

tl)

lhe rwplaoe

x'- f

.ty - 4

= I

,.

.12 - )~ = 9

RH..

+---------------------

II

The double inlegml of x 3y - .t y 3 o,er R is equivalenllo lhe double inlegml of (uv / 2) /


Jul + 4v2 over lhc rcc1anglc R. Nuriwn1al Slrip:> lead 10

!.JRr

1'!. 2Ju>uv+ 4 vzdu dv,


9

uv
du dv =
2Ju> + 4vz

and evaluation proceeds ;IS before.

The next example contains a further illustmtion wherein equations x = r cos(), y = r s in ()


are regarded a~ a mapping from 1he xy-plane 10 1he rO-plane r~thcr than a ~raol<formation 10
polnr coordinates.

I EXAMPLE

13.3 6

= r CO> 0. y = r sin 0 as a mapping to e\-aluaJe ~1e double imegml of


f (x , y) ~ .jxl + y2 ovenhe region R bounded by )' = J a' - x 2, y = 0, where a > 0 is

Use lhe C<IU<IIion> x


a constant

;a

We first note lhatthe Jacobian of the mapping, (x, y ) (r, 8 ) = r , is equal to


ze1o at the origin. a poim on the edge of the region. We shall see the ramil1cation of this momenlarily. To map R itliO 1he rli -plane. we map its edges. The semicircle C1 y = ../a2 - x 2 is
mapped to the vcrtical linesegmcnt
r = a liom /J = 0 to /J = 1C (Figure 13.79). Theposi
Jive X -axis C2 with 0 < X ::; a is mapped tOlhe line segJnem C~ : 8 = 0, 0 < r ::; 0, and the

SOLUTION

c;:

ljhlll;l
be regarOed

fJifD-I

Using pobr coordinates lo c'111uatc a double imcgr,a.l can


a mapping from lhe A)'~le kl lhe r8-planc

...----....

)'

c; o~ .T

c,

c~: r = a

/(

- (1

c,

R,

c,

II

'

,.

C1 : 11=0

{/

negative XaxiS C3 with -0. < X < 0 is Ol<lJ)JX:d tO the line SCglltCfl l C~ : 9 = 1f with
0 < r ~ t1. The o rigin in Lhe x yplane is mapped to lhe 8 -axis whe re r
0 . Hence R is
mapped tO the rccUIIlgle R,'fJ bou11dcd by r = 0, r =a, I) = 0 , and I) = 1f in the r O-plane .
With horizontal strips in Rr~~,

!'JRr

2
J x2 + y! dA = j'hr ./rdrdB = !."!."r dr d9 = !.'T { }O
3 d (J = 3 101~ =
3

aJ

I EXAMPLE 13.36

Use the t~nsformation x = a u , y


bv, t
c w to evaluate the triple intcgml of f(x, y)
1
2
x + yl overt he regi o11 inside the ellipsoid x f a2 + y 2 / b 1 + z! fc 2 = I.

SOLUTIO'!

lhe Jacobian of the transformation is

il(x.y.~)

il (u ,v, w)

ax;au axtfJv iJx /iJw


ily ;au <ly/iJu IJyfiJw
a:;au a:tau az;aw

o o

0 b 0 = abc.

0 0 c

Regarded as a map1>i11g from xyz-spac:e to u uw -spoce, the ell ipsoid is mapped to the sphere
u2 + v 1 + w1 = I . I f we change to spherical coonJimHes in UtlWspace.

= abc

/. ~{ (a 2 eos2 0 + b 2 sin 2 8) ( -cos+ cosJ


- - if>) }"dO

= 4abc
15

15

f2.' [a'(l +cos211) + li(l-co$21/)]d 9

Jo

2libC {

= -

(a

b'

+ ") +

(a

b ) sin 28

}2,' =
0

41fabc(a

15

+ b2 )

J.).J.l Genc:t:tl Tr.tn~ronn..1tiotr~ in ~1 ulliple loteg.rals

971

In the previous examples. the transfonnations facilitated limits on double and triple imegmls.
The following example illustrates that they can be. used to simpl ify an imegrand.

I EXAMPLE

13.37

Usc the unforruation u


X + y, u
yf(x + y) to C\'l!luate the double imcgral or
f(x. y) = e YIU+)') over the region R bounded by the linc.s x + )' = I, x = 0, )' = 0.
SOLL'TIOi'\ The transformation map< the tl'liangle R in the xy-plane to a square in the
plane (Figure 13.80). The Jacobian of the transformation is
8(x .y)
J(u. u)

= ..,..,.-.,...
CJ(u, v)

= :-;:-....,.,-..,..--:-;:-...,. <Juf<Jx i)u f3y


CJvf<Jx
CJvf;)y
i)(x . y)

lt V

=X +)' = U.

xf(.~ + y) 2 1

Consequentl y.

=fl.

H..,,,

= [

e"udvdu = f'[ ' ue" d v du


lo~ o

{ueJ~ du =

1'

u(e- l) du = (e- I) {

1:-

1!- l

FIGURE 13.

,.

hnagc

.r+y=l

c,

ot
(.r, )') = 10, 0)

R
.r

c,

In Exercises 1-4 usc tbc suggested equations to set up a double iterated


integral for the double integral. ln Exercises I aod2 do this in l\\OWays:
(a) Con~idcr the equation:> as a coordinate transformation. (b) Consider
the equations as a ma1ming from lhc .t)'planetoanotherplane. Do not
cvaluare 1hc double ilcr-.tte.l io'llegr..ll.

Jl /x +
2

y2

y 2 (_f A . where R is bounded by x 2 + y 2

= 9 .t ~ 0. y

lWl

= 4. x

0. ( X,)' ~ 0). U.:sc .~ .- rc040. )' ~ r :sinO.

* 2. f.:c.r d A. \vllct: R i;; bounded by .t: + )'-:. + .r =/.r-:. + y:,


x

3.

= J!,y, ." =./5x.(x. )' ;:: 0). U<ex =r cosO, '' = rsinO

= I, x + )' = 4. Usc " = x + )'. v = y- 2.<.

c;
"

* 4.

Jf.

(x'

+ y) d .4 . where R is the area in the tirstquadrantiJcJunded

by y = .t 1 , y = A' 2+4. y = 5 - 2.x 2 , y = 6-2x 2. Uscu = y-xz,

v - .1' + 2x2.

In Exercises 5-S usc the su,ggcsted traosfonno..tion to set up a triple


ilcrotcdintcgral runJctriplcintcg,al. Oothisin t" o ways: Ca )Con~idcr
the cqu:ILiOn ~i as a-coordinalc tnmsrormation. (b) Consider the equations
as a m3pping rrom tile xyz.s pacc 10 ~m o1hcr space. Oo not C\':duatc
the triple iterated intc,yu1.

:.. Jj j,u":_,.:t
* J
J[(.\' + )'

d\1. where V isthc rcg.ionboundcd by.t 2+y1 = 4,

z = 0. z =I . U.;;e. .r = rcos8. y
2

6.

J.x' cosy d A. where R is bounded by y = 2x+ I. y = 2x -l.

x +y

RM

'

EXERCISES 13. 13

1.

CJ

z.

(/V. where V is Ihe reg ion in 1h c firs OCI!ml

x' + y ' = 9. <: = Jx' + y 2 X =


= r t'OS e~ y = r sine . t. = z.

bounded by

Usc X

2
)

= rt>in B. z =

0, y = 0 .

z=

0.

912

7.

Ch ~p1er

J!"l{v'

1
2

~ohllli ple 1nregrals

13

cylimlri,.aJ ~,;.oCJrdinate~ X
rc:oo O. )'
cylindrical coordinates based at (0. I): X

d V, whe-n:: V is the reg.iott bounded by x z + y 2 +

+ !'2

z' = 4 . .1' = .<. y = 0. z = O(x . .r. z :;: 0). Uscx = :Jl sin</> cos O.
.v=m~in~sinO. z = m'<:os;JJ.

z = z.

Jf

16. Ev-Jiu:otc Ihe double integrJI or I (x. )') = cos [(.r - )')/(X + y)j
over the t..-jullglG.: lx>undc<l by.\" + y = l , .\' = 0. y = 0 by making
the l.mnsfomwti.on t
x - y. u = x + y.

/.x'y': d\1 . where V is tbc region bounded by z

+ 8.

Jx1. + y: .:
:n,intj>sin O.

z c:

i4 xz
91cost/l.

yz. Usc ,t

U~si.ntf>cos9, y

l7. Usc the lrJ.:nsformaLion x = ~~ y = 11 + v to evaluate


the double integral of f(x. I') - x /(x' ~- y) over the smaller region

* 9.1 Usc a u1ansl'


nrmatiCln to map I he region R in~ide the ellip~ b x +
1
2 1

a y! = a b omo a circle m order to evaluate the double integmJ of


/(.< .1) -

bounllcd by .r

Jx 1Ja1 + .1.1/b' o1cr R.

10. Ev;duarc rhc double inlCg:ral of J (X, )')


rani r(gion bounded by.\'~ - y 1 = t. .r l \'1: + ) ,1: :z 16.

= J.')' over the first quad


y2 = 4, ~,. 2 + y2 = 9,

Evaluutethcdouble inresmtor f(x, y) = 2J. 2 - .t,l-' - y'l O\'er rhe


region bounded by y = 8 - 2x. )' = 4 - 2x , y = x - I, y = x + 3.

x2

+ y2 =

8y.

Evaluate tlllC uiple imegtal of f (x. y. t) - x + y + t over the


= x + 1. y x - 1. x + y =
I, x + .1' = 3, z = 2x + ,v, z = 2x + y + I using (a) rriple
iterated i.ntcgrals in Cartesian coordi,"ucs, and (b) the t.tnnsformation
U ;;;;;;; .\' - )', l1 = X+ )'. IV= Z - 2..t- )".
region bounded by the phmcs y

1.,

"' 21. E\'alu:ue rhc double inreg..-.al off (x , ,r) = x + y overthc rcgi<"'n in
tbc fourth quadrant bounded by the parabohlS y = x 2 - I, y = .x 2 - 4,

2. J - I - .t 1 using (a) double itcraLCd intcgl'als in Ca11csiw1


coordinates.. and (b) the transformation II = x 1 - y. v = x 1 + y .

+ IS. Evaluate lhc tripJc integral or .f(x, y. z.) - y where V is t.hc


rc2ion bounded by 2 = 0. y + r. = 2. x' + )' 2 = 2)' using: (a)

.v = 4 - x 1 ..- = 1,

* 20.

+ 14. Usc the tr.msf"'mmiun x = (u/u) cos w. .v = (u/u) sin w.


z = r>: (u. IJ > O)co cv:duarcthe Ltiplcintegr:l of j'(:r. y) = x 1 +y1

z = 4(x 2 + y 1), z =

= J<x -

13. Ev~lu:uc Ihe triple integral off (x. y, z) = xZ y 1~ 1 over the lntc
rior ofthc ellipsoid .< 2fa' + y 2/ b 2 +z' fc' - 1.

x 2 + y 2,

"' l9. Usc the lnu:'l:;fOmlatjon .t: = u + u v. ,. - v uv tocvuluatc lhc


double imcgrdl or j(.r, J) 1/
Yi' + lx + 2y + ' OI'Cr the
triangle with vertices (0, 0) , (2, 0), and (2, 2).

12. U5c the tr'Jnsl'ormt~.tion u x - .\' 1 v = y to evaluate the double


inL<.:g.rul off(.\' . y) -A' + y Q\'(;l'"(hcparullc!ogr"-0\ boundcdby y - 0,
.\ = 2. y =X . )' =X - 2.

=x

18. Usc the U1l.OSformation u - 2xf(x 1 + )''). v 2,\' /(.r 1 + y 1) to


evaluate lhe tlouble intc.gr~l of l (x .\' j
l /(.t 2+ y 1) 2 O\'Crlhc region
2
2
bounded by Ihe circles .r + y~ = 6.r. x + y 2 = 4x . .x, + y 2 2y .
and

* 11.

O\'er the region bounded by (;


, _ 4.

= rsinO . z = z. tulll ()))


= I' cos;9. )' = I +I' sin 0.

y- -

.t

13.14 Derivatives of Definite Integrals


1fa function j(X, y) of tWO in~epentltnt variable;; is iollegl'lltetl With re.~I>OCI to y from y
to y = b, the result depends on x. Suppose,.. denote this function by F(x) :
F (x)

J.bj(x , y) dy.

= tl

( U.75)

To calculale the derivative F' (x) of this fun~'l ion, we should 6rst integrate with respect to y and
then differentiate with respect to x. The following lheorem indicmes thai differentiation can be
done first and iotegrntion later.

THEOREM 13. 3

If the pal'tial derivative aj; ax of f(x. y) is cotllinuous on a rectangle a < y < b.

c ::; x ::; d, then for c < x < d.

1b

-d
f(x.y)dy =
dx a

PROOF

1b
a

iif(x 'y) dy.


iix

If we define

g(x) =

of(x, y)

ax

dy

( 13.76)

as the right side of equation 13.76, thcn this li~nction is defined l"or c:;: x ::; d. In fact, because
aj{ax is continuous, g (x) is also continuous. We can therefore integrate g(x) with respccllo
x from x "'c to any value of x in the inter\ a) c 5 x s d :

lliilciii;I#\U]]I f'rocf m
!he: order uf upcr.lhc)l'ti cf inlcgr.tlion wicb ~ptct to Ont. '-'<wiOOit
and dilfetcntiation wid respect to
ant~d~r c:..m be i nlt:n.ilal~-.ed

<

g(x) d.v =

1"'1h

8f(x
, ., y) dy d.r.

'

a .\

Th is d ouble iterated i ntegra l representS the double inlegral of Clfj ax over the rectangle in Figure
13.81 , and if we reverse the order of integrlllion. we have

x gCv) dx

"
c

11'
a

1"

aj(v
~ ' y ) d.rdy =

/J(.r . y)J; dy

1b

[f (x , y) - /(c, y)jdy =

/(.v, y)dy-

Because the second integraJ on the rig ht is independent


wi th respecl to .r, we get

-d

!.'

d.x ,.

= d-d

g(x ) dx

But Theorem 6.7 giYcs

d
g(x) = d-:
.I

1b

f (c , y) dy.

(I

of~\'.

if we dinerenliate this equation

;bf (x, y ) d y.
a

lb

f (x, y) d y .

This completes the proof.

The limits on the incegral in 13.75 need n<>t be numerical constants; as fa r as the integr<ttion
with respect co y is cooccmcd) x is constant) and therefore a and b could be functions of x:
bt.r)

F (x ) =

/(x , y) dy .

o (.<)

I ndeed>this; i ~ preci:;ely wh;u does occur in the evaluation of tiOL!ble inLegral by mea ns o f double

itcrllled illlcgnlis. To dif ferentiate F(x ) now is mowcompl icmcd limn in Theorem 1.3.3 because
we must also account for the .x s in a(.x) and b(x). The chain rule can be used to develop a
formula for F' (_, ).
THEOREM 13.4

( Le lbnlz's R u le)

If the partial derivative ilffilx is continuous on the area bounded by the curves)' = a(x) ,
y = b(x ), x = c , and x =d . then
d 1b~>

-d

PROOf'

f'

/ aI"'1,

I I
X

.f(x , y) dy

a(>)

1''

> of(x
. )')
db
da
a
dy + .f[x, b(x)J - - f [x , a(x)J-.

a(x }

dx

d.r
( 1.1.77)

If F (x) "'~~~~ f(x, y) d y, theschcmaticdiago'!lm gives the chain rule

F'(x )

= iJF) + IJ P)
iJx

a.b

i)b

a. <

db + &F)

d(l.

dx

dx

aa

b .

111e tlrstterm is the siLUation covered in Theorem 13.3, and therefore

&F ) "'
ax

a.b

( b aj(x, y) dy .

lu

ax

974

Cb.:tpter t3 Multiple l nt~r.:'l.l s

Since Theorem 6.7 indicates that

d l.b
db
f (y) dy = f(b),
it follows that

iJ l.b
i)b
a f{x, y) dy
In other word,.

f{x, b) .

-i!F) = f(x. b) .
[)b '

Funhcrmorc,

-i)

F)

[1(1

= -i)
/,.A

d(l

l.b f(.t,y)dy

= - -[)

1"

[)(I b

(>

j(:r,y)dy = - j(x , a ).

Subs1i1u1ion of these facrs imo the chain ule now gives Leibniz's rule.

Our derivation of Leibniz's rule for the ditrere.ntiation of a definite integral chat depends on
a parameter (x in this t:aSe) shows 1hat the first term accounts for lltose x s in the integrantl. allLI
the second and third terms for tbe x 's u1tbe upper nnd lower limits. The following geometric
interpretation of teibniz's rule emphasizes this same poinl.
Suppose the fu nction f (x, y) has only positive values so that the surface z "' f(x . y) lies
completely above the xy-plane (Figure 13.82). Equations y = a(x) andy = b(x) describe
cylindrical walls standing on the curves y = a(x). z = 0 and y = b(x). z = 0 in the xyplaoe. Were we to slice through the surface z = f(x. y) with a plane x =constant, a n areo
would be defined in this plane bounded on the top by z = f (x, y) , o n the side.< by y = a (x)
and y = b(x). and on the bottom by the xy -plane. This urea is ckarly defined by the definite
integral
b( A)

f (x ,y)dy

b\.:t)

in Lcibniz"s rule, and ns x varies so too does the area. Note, in particular, that as the plane
varies. area changes. not only because height of the surface z = f(x . y) changes but also
because width of the a rea varies (i .e., the two eyliudrical wuJJs are not a constam distance apart).
The. first term in T..eibniz's rule accounts for veTlical varhuion, whereas the re.maining t\VO tenns
represent variations due to fluctuating width.
MjjCUII.14fH:fM

/"
y = a(x)

GeomcLric inlerpt-e~ation o( Leibniz'Ji rule

= b(x)

I EXAMPLE 13.38

----

If F (x )

= f!' (y3 In y + x 3e>') dy. lind F'(x) .

SOLUTIO!':

F'(x)

Wi1h Leibniz's rule, we have

1'"'

(3x2 e' ) dy

= Px' e' l!'

+ [(2.r)' In (2x) + x }e2 ' ! (2) -

+ 16x- ( 1n2 +
1

In x) + 2x3

r' -

[x 3 1nx + x'e'J( I )

x ' lnx - x ' e'

+ 15x3 1n.r + 2.r 3 eu - x 3 e'


= 3.< 2,.'"(1!' - I}+ .< 3 ( J6 1n 2 + 151n x + 2eh - e').
= 3x 1e'-' - 3.r'e'

( 16 1n 2)x

I n Exercise 6 you are asked 10 cvalume the imegral defini ng F(.r ) in 1his example and 1hen to
diiTerentiate the resulting functio n wi th respect to ..t. h will be clear. chen, thai for this example
L.cibniz's rule simplifies the calcuhnivns wnsiUt:tbly.

I EXAMPLE

13.39

Evaluate

' ._
l' '-- -dy
I
In )'
/.0

ror x > - 1.

SO Ll f riON \Ve use Leibniz'snde in thiscxample to ~woid finding an al'ltidei"ath-e (o( (yr 1)/ln y . If we set

F (x) =

1
0

y' - I
dy
In y

and use Leibniz's rule, we h<tve


F ' (x )

= {'
}0

!.._ {y' i)x

In y

jdy

1' y' lny = 1'


o

In y

dy

y'dy

x + l

h follows. therefore, that F (x) must be of the fonn

F(.r) = ln (x +I) + C.
But from the defi n ition of F(.r ) as an i ntegral . it is cl e<tr that F(O) = 0, and henee C = 0 .
Thus,

' y' - 1

'-:--tfy

In)'

l n(x

+ 1).

Note that 1his problem originally had nothing whatsoever to do with l.cibniz's rule. Bu1 by
introducing the rule. we were able to find a simple formula for F' (~t) , a11d this im.mediately led
to F (x} . This can be a very useful technique for 1.'\lll uming definile i ntegrals that depend on a
panuuctcr.

I EXAMPLE

13.40
When a shell is fired from the anillery gun i o Figure 13.83. 1he barrel recoils along a welllubricated guide and its mo1ion is braked by a banery o f heavy spri ngs. We set up a coordi oate
system where x = 0 represe.nts the firing position of Lhe gun when no stretch or compression
exists in the spi ng:;. Suppose that when the gun is llred. the horizontal component o f the force

I'll

mt01

ot an artillery gun at~ ars

5}:J

causing recoil is g( r)(t = time). If the mass of the g un is m and the effective spring constant
for the blttcry of sprin~> is k, then Newton's .eeond law states tllill

0 1l!pl:t.:c-

shcUii firoJ

J lx
= g(t ) - h
m-

d2.x
m- - h

or

dt 1

= g(t) .

dt 2

This diiTcrcntial equation can be solved by a n~ethou called vurlation of pununelers.anu the
solulion i'

x(t) = A cos
A

1 + B sin \(fk
-:-:'

+ r..
I

1'

g(11) sin

v mk o

[ fk

\/ :::-<r - 11) d11,


m

"bere A and B are arbitrary comlllnt>. We do not dbcuss diiTen:nlial equations until Olapter
15. but \\ith lcibniz's rule it is possible to crif) that this function is indeed o solution. Do so.
SOLlfi iON To differentiate the iluegral we rewrite Leibniz's rule in terms of !he variables of
this problem:

d
t1I

1b(tl f(t. 11) du = 1b(r) fJ/(1 ' 11) du + f[t, b(t) J-db nt r)

iJt

a(r)

dr

da
f[r , 11(1}] - .
dt

If we now apply this fom1ula to the definite integral in .<(1} where


[(1.11)
rhen

dx
dl

=-

x(11)

sin [/~(/- 11)].

g ff + g, ff
-A>in

-r

-Bcos

- 1

Ill

Ill

Ill

- ~ {[ g(11\!I co< [~(I 11)Jdu + g (t)sin [/~(I -I)J}

fk,)

- l'[
(- Asin \fi,
+Boos V;;;, + m
2_ }(' g(u)cos[\~(t
- 11)]d 11 .
m
1n
m
0

We 110\\ usc lcibniz's rule once more 10 find d zx f dt 2 :

tflx
- 2
dt

If fT if ~ )
+ ,;, {1' -f((u)~sin ~(1 - u)]du+!((/)cos[/~(/ - I)] J
*( fk Bsin\f..:..r
fk )
-i \ j' [,fie, - +

(= \{fk
-:-:
m

-Aa:>s

/ :..:...1-

\ m

- B<in

- 1

\ m

= - -

=-

cO<\( ::.., +
m

l/(tt) sin

u)] du

2-g(t)

1~1 {A co<~~ + 8 ~in J!..r + ~ [


1

+-

g(t)

Ill

k
= - - x(t)

+ -1!(1) .
m

g(u) sin [

/!;(/- J
11)

rill

tJ.t4 t)erii'Utivts ofDefinite lnregrols

977

In 01hcr words, lhe funclion x(t ) smislies the differential equation

d2x

m dt 2 = - kx(t) + g(t ),
and the proof is complete.

This example illustrates I hat Lcibni:(s rule .is esscntiul to 1hc manipulation of solution of
differential equa1ions that are represented as definite. integrals. Here is an01her example, but
with a partial differential cqua1ion ins1cncl of an ordinnry diffcrcn1ial equation.

I EXAMPLE

13.41

"The onedirnensional heat cooduclion equntio11

ar

;Fr

- =k i!t

ax .
1

where k > 0 is a conslarll, describes lemper"..lture in a rod (see Exerdse 4 1 in Section 12.5).
Show 1ha1 1he function defined by the deli nile integral

2T 1 '/(2./fi) - v' d
T (.r , t )= - 0
e
v

fi o

satislies the heat conduction equation.


SOLUTION We use Leibniz"s rule 13.77 to obtain iJ T / I'll and I'JT ji)x:

CJ T
-=

a,

ar

-To-e-'IWt)
..fkiil

-=
ilx

II follows that

iJ2T
iJx2 -

2.f )

- -To-e -x' f(.tkt) ( ,fkif(


4kt

EXERCISES 13.14

ln fuerci:scs 1- 5 u:,c l.A.:ibni.z'$ rule. to find the <.k:rh~.ttivc of F(.t).


Chock your rcsuh by C\'n\uating the integral Md then di riC:n.:ntit~.t ing.

t . F(x )

2. F(x )

3. F (x) =

4. F(x)

J.l

<x ' y'

+ 3xy) tly

J.' +
J',_,' <x'.' +
(x'!.v '

') fly

y'

5. F(x) =

'y -x

--lly

Y +x

6. f'irw.J F'(x ) in Example 1 3.3S byfii'SLCV<ilua~ing lllCde-linitc i_megraJ.


7. Usc thcresultoi' Exarnplc 13.39to provc 1ha1

1xP- .tc'
1

+ 1) tly

= [ :'-(x + y tny)t/y

.,

ln x

t!x

== In (P + 1)
q

+1

providedd1at p > - I andq >-I.

8. Find F ' (x ) if F (x) =

i~~ /1 + y' d y .

978

Ch'IJ(er 13

Multiple lntegrnls

In Exercise>; 9- 11 use Leibniz's rule lo verify that lhe funclion y(x)


satisfies lhc diffen::ntial oqmuion.

* 9.

J.'
x'

= -1

y(x)

11j(1) d1 ; x -dr

ilx

10. y(x)

!.'

= -I

18. Vetify that lhc funclion


..

+ 2y = xj(x )

!.(1-x)j(lJi)

l(x,l)=

e ~ dv

J.(l+x)f(!Ji)

j(l)(e'-' - e'-x) d1 d- y - r
dx1

= ((x)

satisfies the panial differential equation

11 2T

liT

al = ox'.

19.

(a) What is the domai n of lhe function


9

F(x) = ! . 1n (I- x 2.r2)dy? Whot is F(O)?

+ 12. Oivcn that


I

(b) Find F '(x) by Lcibniz's rule. What is F'(O)?

- ln (l +ab).

(c) Show thallhe graph of l.hefunction F (x) is concave down


wurd for all X in ils domain of definition.

find a formula for

x
dx .
( l +ax)'

** 20. Laplace's equation for a function u(r, B) in polar c-oordinales is

* 13. Given that


xl
fil.td il [oa:rnuJa fOt'

dx = Sin-

(!)
+ C,
a

(a2

, .. , d X .
1
- x-)'-

14. Given !.hat

a1

find a formula for

* 15.

1 clx = -1Tan- (:a)


:_ +C.
+ x
11

2 3
(a 2 + x)

a'u
.,...,.
ur-

ou
1

+ -,
'0 '- = 0
r u

(><:c Example 12.19 in Section 12.6). If the values of u(r, 0 ) arc spccified onlhccirder = R t\$U(R.tP}. - ff < tjJ ~ 1l,then Poissons
integraJ ronnula state.-; Lhatthe value of u(r. f)) interior lOthis circle is
delilled by

R' - r

u(r.1)=

2tr

1"
-

u(R. t/1)
R1

r2 -

2rRcos(O- 4>)

dr/>.

Show that this function docs indeed satisfy Laplace's equation.


II'

** 2 l .

lf lhe function u(r. ()) in Exerc-ise 20 must satisfy the condition

2labl
rr/1

o
/.

dx.

Use the result that

to lind a formula for

qu
-r -;ur
I

~u(R, O)

(a 2 00<2 X

, .,

+ b- Sin- X) 2

dx .
oo the circle r

= f(O) . -tr < 0 .5 ;r,


ar
= R ( r~ thcr than h:we its values prescribed on the

circle). then \:;dues of u(r . 8) interior to the circle arc given by

ln Ex.crcis.cs
16.

!. ""
!.
0

17.

1~17

usc Lcibniz's ulc to C..""\'Uiualc the integrdl.

=C -

+r2 -

u(r. 0)

Tan- (liX)

where C is an arbitrary consumt. 11Us is called Dim''s ime,C!,ral. VeriFy


that it satisfies Laplace's equation in polar coonlinatcs.

cosx.

.;.:.,:-....:::.;.,:.. dx, where a > 0


.<(I +

SUMMARY

x-)

The definite integral of a funct ion

(x) from

[''
Jo
f(x)dx

hR

f.''

ln(l + acou)
_:.._:..__---'. dx. whc1c Ia I < I

_,

f(u) ln[R'

2rRcoo(O - u) ]du.

x = a to x = b is a limit of a sum

L" J<xn t.x; .

lim
U<>.<rU - o .

t= l

Key Term~

878

lo this chapter we extended this idea to define double integrals of functions f (x, y) over regions
in the xy-plone nnd triple integrals of func tions f(x, y, :) over regions of space. Each is once
ag-ain the Jimit of a sum:

..

JJ>(x,y)dA = 114lArn....,.u L, f(x(, yi) 611;,


lim

l=t
n

JJ[ t<x.y. z)dV =

lim

L f<xi. y/. zil 6 V;.

t..\V;i-0.

r=l

To e\aluate double integr-dls we use double iterated integrals in Cartesian or ll<llar coordinates.
Which is the more useful in fl given problem depends on the shope of the region R and the form
of the function f(x, y). For instance, circles centred at the origin Md straight lines through
the origin arc rcpresemcd very simply in polar coordinate:;. and therefore u region R with these
cun-es as boundaries immediately suggest$ the use of poh1r cwrdinates. On the other hanu,
curves that c.1n be described in the form)' = f(x). where f(x) i~ a polynomial, a rational
function, or n transcendenral function, often suggest using double iterated integrals in Cartesian
coordinates. Each imcgratjon io a double iterated intcg.ml can be interpreted gcomcujcalJy~ aud
through these interpretations it is a simple maucr tO find appropriate limits for the integml:.. In
panicular, for a double iterated integral in Cartesifll'l coordinate'i, the inner integration is over
che rectangles io a strip {horizontnl or vertical), and 'he outer integr-ation adds over all s1rips.
In polar coordinates. the innet integral is inside either a wedge Of' a ring. and the outer imcg.raJ
adds over all wedges or rings.
To e'1aluate triple integrals we usc triple ite~ted integrals in Cartesian~ cylindrical. o r
spherical coordinates. Once again the Iimits on these integrals can be determined by interpreting.
the ~ummation~ geometrically. For example. the firs1 imegration in a triple iterated integml in
Cartesi:~n coordinates is always over the boxes in a column, the second over the rec1angles inside
a strip. and the third over all strips.
We used double integral< to find plane areas. volumes of solids of revolution. centroid~.
momentS of inertia. fluid forces, and areas of surfaces. We dealt with these same applications
(with the e.~ccption of surface area) in Clwptcr 7 using the definite imcgral. btot with some
difficuh~' : Volumes required two method~. ~h ells and \vashers; centroids required an a\'('!raging
formula for the first ruomeotof a rectangle that bas its length perpendicular to the axis about which
a moment is requin.'d: moment;, ofinc.rtia needed a "one-third cubed formula" for rectangles with
lengths perpendicular to the axis about which the moment or inertia is required; and fluid forces
required horizontal rccrnngle~;. On the other hand, double integrals elimillatc these difficuhie.s .
but more important. provide a unified apr>roach to all appl ication<.
l11 Section 13.13 we showed how to tronsforrn double and triple integrals from one set of
v-.lriable.: to another. f inal ly. in Section 13. 14 we u<ed double integi".!IS to ,erify Leibniz's rule
for differentiating definite intcgruls that depend Oil n pammctc-r.
KEY TERMS

In reviewing this chapter. you shoulei be able to define or discuss the following key temlS:
Double i.ntegral
Areas
Auid pressure
Cemre of mass
PriJlcipal axes
Surface area
Triple imegral
Triple iter.,u.ed iote.grals in cylindrical
coordinates
General transformation for multiple imegrals

Double iterated integral


Volume of a soli !I of revolution
Centre of pressure
Momems of inenia
Priocipal moOlents of jnertia
D<.>uble iter~ted inlegr~Js in pol~r coordinates
Triple iterated integral
Triple iterated integrals in spherical

coordjnates
Leibniz.'s rule

REVIEW
EXERCISES

ln

E;~~:crd sc.:s

1- 2 1 cvaluiUC the integral over II~ n:gi01l.

+ )') dA. when: R is bounded by y =

18.

I.

2.

ffi 'YZ tiV, where V is bounded by y = z ..t = 0. y =

(1.<

.r. )' = 0, x = 2

19.

3, .r = l .<: =0

ffR.\3y 2d A, where R is bow1dcd b) x =

3.

J, x

= - 1. y

2 1.

= (.r- 1)2. y = x + 1

xy 2 d A . where R is bounded by x = 2 - 2y2, x = - y l

fll (x' + )" +

<2)dV , where
-.r. )' = 0. y = I. z = 2

10.

jf. (,\')'

4-

2.1'

II.

j f.

.r 2y 2) dA, where R

!!."x' + )'
If.
I

(x 2 -

z=

.r,

t/ A. whercR is bOUJ1dcdby y=x .r = l,.r =

y 2) dA . when: R i bounded by y

y=

= X,)'= x -

14 -lx

(gl The pl'obubilily or an clcc1ron being iJl R if 1hc probabi l il y


of the e lectron heing in unit area at point (x, y) i~ P (x, .v)

z=

)'

is bow>dcd b y)' - 2.t2 )' =

.r

(f) Tile lOla) mass if R is a plate with mass pel' unit area
p(x, y)

Vis bounded by

.r sin y d A. wiU\: R is bounded by

x'- y'

O'(X, J)

fl. .x2y dA . where R is the n:gionorE:<cn:isc 7

8.

= 2x

(o) 1hc ""'" of R


(b) llc voluttlCSof Ute solids of revolution ''t~l R is rohllcd
about the lut<:S .r = 2 and y = - 4
(c) TI.e fi ntl ntomcnLs of' at't'"' o( R about the Jines .r :;;:;:;: I uncl
Y= - 1
(d) TilC second moments or urea of R about the li'lCs x = - I
and ;~.= 4
(e) The tolal charge nn R if il. (.':tl'rics a charge per unit area

7.

+ .r'

22. If R rtl>n:><:tU> l hc rcgion uf 1hc X)'plunc in 1hc ligurc below.


idcnl i fy the double iJllcgtals thaii'<I)I'C$tntthe following:

(.r' - y 2) d V . " here V i Slhc regi on i n Exercise 4

6. fy dA,wherc R is bounded by y

9.

z = .r_r.

I. .\'- I

IIi

S.

y 1) d A, where R is b<>undcd by .r'

d V . where \1 is bounded by:= I, z = J4 -

1. y=5 - 2x.

4. f l i (.t 1 - y')t/V . where \1 is bounded by


X -

(x2

2, y = O

'2, y= -2

0,

20.

If. +
Ifi ;:

= ./f=Y. x

0 ..v = 0
l l.

ffl

(.<

+ .r + z) t/V . whe re

V is Jx>Unded b y

-I

z = 1 - x'-

y 1 z;;;: 0

13.

14.

15.
I -

16.

fl. xeY liA,whcn: R isbounded by

Jll
Jf. +
/X, =
Jfl +

dV . where Vis bounded by


.1') t/A, where

.r - I

(.t'

/ r- x2
17.

(x

/f.

y =O

A' =

0 . .\' = 5. )' = 2.-:+1

z' = .<2 + y 2 x' + y 2 = 4

R i< bou nded by X= - .;r=y, ."

23. Find the vol\lmc bounded by the surfaces :


2e-~-,~ .

y 2 .t=0
= .r - I .

x = 2.

= 0, z

I -

24. Fi.nd tbc urc~ bounded by the c-u n'C x 1(4 - x 2) = y2.
.. 25. Find the centroid of the resion bou.ndcd by the CUt'VC$ 4x

>'' . = / 8

y 2 +z?)t1 V, where V is bounded by z

- '- - t1A , where R is bounded by y


Rx + y

-4

=4 -

2J'.

"' 26. Find the vohamc bounded by the surfaC'c:-s y

2. )' - .x' + l.

+~

= 2. y

- z=

+ 27. lf tl\c viewing gillS! Ofl a dtcp-!Ooea di\c.r's hchnct i! cueular with
diameter I 0 em. whal is the force on the glass when the centre or lhc
glass is 50 m below tJtc surface?

28. Find thcarca oftJtal partoft

= l-

4 x 2 -4y 2 atxwe the xy-planc.

*
*

29. Fmd the area bounded by the curve x 4 (4 - x2) ~ y 2


30~

Find d!Je

vo~ume.~

of the

so~id~

of revolution when the

reg:~on

*
*

*
*

36. find the mmnent of inertia about the z-ax]:s of a uniform solid

37. Find the ave1age value of lhe ftmction .f (.x , y, z) = x +)' +z ove1
dle re.gi.on bowlded by d1e :surfaces - x + 2 y + 2z - 4, x + y + z ~ - o
2.

in

Exercise 24 is l'OtaJed aboul dn~ .x ~ and y -ax.es.

31.. Fmd ~ile mo1itilent of in:elit:ila allm~t the z.axis of the solid bo~nded
by the :surfaces z = .,/x. 2 + i~, z = 2- /x 2 + y 2 U i~ densi~y pis
constant
3l~ Find the :second moment of area aboul the .v-axis of d1e re ~.ion in
the firsl .qruadraru bounded by d!Je curve .T 3 + y 3 = 1.

l'10tmdai by ~he :surfaces z = 0, z = 1,

,- o
I

~-

' iJ

x. 2 + )' 2 =

1 + z2

. ..

38. Find the centroid of d1e bifolium r - s~n 8 Cos2 () .

39. Verify that

33~

Find the centre of mass of a l!ll.niform suHd boll!lnded by


x 2 + ..v2 , x 2 + ..v2 ~ L z ~ 0.
34~ Find d1e average va[ue of the tiunction f (x ~ y) - x 2
the reg~on !bounded by d1e curve x 2 + y 2 - 4.
35~

Find the force dWJ.e

10

z = L+

~v (.t)

+ y 2 over

~n

metres.

(l)e><t-.tJ sill (x - t) dl
{l

a so1~tton
or Lbe d']''f'~~-"l!.erenball
~ equattol'll
ll. >'.
]S

Surface of \Vater

for .any constants

1 - - -lO - -- 1 J.

L'f

water press wtre on eac l1 side of d1e verti.ca[

parraUdogrra.n :~n d!Je fi_gl!ll.re be:~ow. .A.II measurements are

= e - 3.:r{Cmc.osx + C2sinx)

c]

and

c2.

d.t 2

+ 6..- Y + 10 v ~ J~(
' .\' )
dx

40.. Find the centt-e of n11ass of d~e uniform solid in the first octant
common to the .cyUnders x 2 + z 2 - 1, y 2 + z 2 = l.

41. Find ~he area of t!hat part of z = [n (x1


ders x 2 + l' 2 = 1 and x 2 + l' z = 4.

+ y 2)

between the cy[in~

42,. find the voJume bounded by the. s~rface.

J x 1 + z2 =

y(2 - y).

CHAPTER

14

Vector Calculus

Ap plication P re,dew

y, z) due to a charge q at Lhe origin is

Tt is well known Lhm the e lectric field Eat a p.oint (.r,

E-

4n~o lrP

'

where r = xi+ yj + t k is the veccor from the origin to the point. If we consider a sphere
of mdius R cenlred al the origin, then the prOOuct of the magnitude of Lhe clec.tric field on the
sphere and the area of the sphere is

and this .is independe.nc of the radius of the sphere.


T HE PROBLEM
lf a surface (not j ust a .sphere) contains a number of poim charges (not
just one at the origin), 1vhat is the relationship between '"'lues of the electric fteld on the surface
due 10 the charges. and the charge~ themselves? (See Ex~mple 14.25 on page page 1037 for the
solutio n.)

In Se.ctions 11.9 and 11. 10 we considered vectors whose componems are functions of a
single variable.. In particular, if an obje.ct moves along a curve C defined parametrically by
x = x(t) , y = y(t), z = z(t), 1 ~ 0, where 1 is time, then the components of the position,
\'e.locity.. and acceleration vectors are functions of ti me:

IOiiCclll.j;j

Fortc on

cha.rge Q at position (x. y. z) due


eo cb<t~ q :u lhre origin

,.

= {I) = x(t)i + y(l )]

dr
dt

d\'

dx ~

dy~

t/

+ z(t)k.
d z.

-dt + -dt J + -dtk.


d

x.

a = - = dt2 i
dt

''

+ dt~ j +

d z.
dt2 k .

If the object has constam mas. m and is subjected to a force t'. which is given as a function
of time r, F = F (t), then Newton's second law expresses the acceleration of the object as
a = F(t)fm . This equat ion can then be integrated to yield the velocity and position of the
object as functions of t.ime. Unfortunately, what often happens is that we do not know F as a

"'

(x,y,z)

function of ti me.

Jn~rea d. \Ve tcnow I hat

if the objecl were at such and such a posilion. then the

force on it would be such and such (i.e., we know F as a function of position). For example,
sup1>0se a positive charge q is placed at the origin in space (figure 14. 1), and a second positive
charge Q is placed at position (x, y, :}. According to Coulomb's law, the force on Q due to
q is

If we allow

Q to move under the influence of this force. we will not know F

as a function of

tirne. but rather, as a function of position. This makes Newton's second Jaw much more diOkuh

982

IU

\ Q.:#.W" Fickt-.

983

to deal with, bm it is in fact the normal situation. Mo:o>t rorccs are represe~1ted as a function
of position rather than tinle. Besides electros1a1ic for'-'CS. consider. for instalcc. spring t()rccs.
gravitmionol forces. und fluid forci!li - all of these arc functions of position. Forces that arc
functions of position arc. cxa1nples of vector$ that arc functions of positiun. ln this chapter we
study vec.w rs that are func tiOilS of position. In l)aniculnr. we differemitlte them and intcg.rute
them along curves and O\'Cr surfaces.

l t4.1 Vector Fields


D<lmains for functions f (x) of one variublc are l>pen, closed, h>llfopcol, <.lr half-closed intcm>ls
on the x -axis. Domains for functions j'(.r. y) of two ' 'rinblcs arc sets of points in the )'Y
plane. Md domains for functions f(x. y, z) are sets of points in xy:-spacc. In order to suite
definitions und theorems for multivariuble func.tions in this chapter. we require correspondi ng
definitions of open and closed sel~ of points. We define then~ for the xy-plane.: analogous

ln!crK..-.
CMeti()(, 11ld h.,.'lun.lnry polm., for

an os>=a ~

dctlnitions for three-di me n ~iona l ~ ~)ace are given i n Excrcij:;e 10.

.mvTJ

f1

I .
C"
lntcrior.
cMaior. ~ xi boJ1nday poinH 1'or .dt)~d ~I

2
'

..

'
'
- ~

....

'

P)

' ~-~'

Q)
3

.....'
/
' ... _R/
.. '

Consider a set S of points in the .cy-plane. A point P in the plane is called an interior
point of S if there exists a circle cemrcd at f' tl1<1t contains only points of S . A point Q is called
an exte.r lor point of S if there exists a circle centred at Q that contains no point of S. A point R
is called a boundary point of S ifevey circle with centre R contains least one point in Sand
at least one point n01 inS. For example. consider the set of points 5 1 : 4x 2 - 9y 2 < 36 (Figure
14.2). The f<l<, thm we have dOl ted the ellipse indicmes that these points are not in 5 1 Every
point inside the ellipse is an interior J>Oint of S1 every poin1 outside 1he ellipse is an exterior
point. and every point on the ellipse is a boundary point. For the set S, : 4x 2 + 9y 2 <:: 36
(Figure 14.3), every point Outside the ellipse is imerior 10 S 2, evCf)' point inside the ellipse is
cxletiOJ' to S2. and every poio\t on the ellipse is a lloiJJ\dary poio\t.
A set o f points S is said to bean open set if all poitHS inS are il'lterior ~~oint~. Ahenliili\el}.
a set is open if it C('Hltains none of il~ boundary pointS. i\ Sel is said w be a closed set if it
contains "II of its boundary points. Set 5 1 abo,e is open; set !i7. is closed. If to 5 1 we add
the poims on the upper halt' of the ellipse (Figure 14.4). this set. call it S3 is neither open nor
closed. It l'Ontains some of its boundary points but not all of them.
/1 setS is said to be connected set if every pair of pointS inS can be joined by n piecewise
smooth cu" e lying enti rely within S. Sets S 1 , S,, aold S; are nil connccull:l. Set S, in Pigure
1-J.S is also connected. Set S.s in Figure 14.6 is not connected: it consists of two disjoiut pieces.
A domaiJ is an open, connected .set Dumu;, is l)crhaps a poor choice of word, as it might
be confused with domain of1.1 fimction. but it has become accepted tem1inology. Context always
makes il dear which interprelalion i.s imendell. Sets
cm<.l s~ are donlHillS: sets s'!,
and
are 1\()t.
A domain is said to be a simpl)' connected domain if eve.ry closed curve in the domain
conra in ~ in il~ inrerior on ly points of the domain. In et-:s.encc. a ~im 1)Jy Mnnec1e.d domain ha~

s,

s$

ljlfiil;b

s3.

l'iJii'!Jjt

A Stl lhfll

i ~ nut

A SCI tll..tl

COJli'X:'i: ted

)'

2
''
X

_. ..
''
'
' '.

/ 1. ,'2

~'

.,,/
.-'

s4 : 1< x~ + .r2 < 4

no holes.

l)om:lill

s. i:~: simply COIIIli.!Cicd. S-t is 11()1. Set s-6: 0

< x2

+ y 1 < I (Figure 14.7)

i~ H domain, but it is no1 simply connected.


Antalogo LL'i definitions for sets c:>f point.."i i11

spt,cc ~ln: gi\'en in Exercis-e 10. One must be


somcwha1 more cnreful here ns there are diffcre11 k inds of huk s. See E xercises 13 a11d IS.
' Vhcn it is not necessary to iKiicatc the parti cular chamctcrlstics of tl set of point.-;, we often
use the word n-gion.

Many vectOI'$ are f~mctions of position. \ Ve call such ''eCtOr$ ' 'C.'<:tor fields . To be precise,
we s~) that F is a vcclol' field in u region D i fF assigns "' vector to each poim in f). If D is a
region of space. then assigns a vec1or F(x . y . :) to each poi1n in D. lf P, Q. and R cll'e the
C'()nlJ)()Ilellt:) or F (x . )' . .:) . lhCil each or these is a function of X. )' . and z:
E'
J('

E'(x, y. :)

P(,<. y, :)i

+ Q(x, y. z)] + R(x , y. ;;)k .

(1 4.1)

D is a region of the xy-pl ruH~-, then


F

= F(x , y) = l'(x , y)l + Q(x, y)j.

(1 4.2)

In Chapter I J \VC~U'C$5cd lhc fi1c1 that the tail o fn ve<.'tor could bcpli.ccd icltany pvinL w hat')Qever.
\ Vhat was important was relalive pOSitions of tip an<.J 1ail. fur vec1or fields. we almost always
pi:;lee the tail of the vector associate<.! with a point at that point
Vector fields me csscn1ial 10 the study o r most are-as i n the physica1 &;.ience!<>. M ornent<.;

(M

= r x F) in mechMics (see Section 11 .7) de~nd on chc posicion of l ' , and are d>crcfore

\'t."Ci or fields. The electric field i nrensity F. , the electric disphlccmclt 0 , the magnetic induction
1hc c l uTent de1sj1)' ,I are all im ponant vector fields in clcctromagnclic theory. T he heat
Aux \'CCtor q is the basis ror Ihe stut.ly of heat C()llduction.

n, and

\ Vc hnve also encountered vector fields that Me of geometric imJXHhmce. For cxmnplc~ the
g radie-llt o f a scalar functilm f (x, )', z) is a vector field.

gnKll

= VI = <~~
i + ~~ j + ~~ k.
lx
(l )'
(i <.

( 14 1)

h assigns chc vector VI co each (>Oinc (.r, y, z) in some region of spcc. We ha"e seen chac V .f
points inlhc direction in which f (x, y, z) inercoscs mosc rnpidly, and ics magniwde IV [ I is
Ihe n uc of increase inlhludircction. In addition,. we know 1h11.t if f(x , y, : ) = c i$.1he cqu:uion
of asurfacc that passes through poi Ill (x. y, : ) . then \1 f ac that point is normal to the surface.
Oflcn, we wri1c
\If =

and regard

a.i + -a J. + -a k) I
( -.
iJ,<
iJy
iJz

" =axa . + aya


-

j + - k

az

as a vector diiTeremial operator, called the del operator. ll operates on a scalar function Lo
produce a vector tield, i ts gradient. A s an operator, \1 should never stand alone~ but should

14.1 Vtvd'ield>

985

always be followed by some!hing on wh ich to opera1e. Because of 1his. V should no1 i1self
be considered a vector. in s pi1c of the fact that it has the rorm of a vector. h is differential
opera1or. and must lhcrefore 01>erme on something. In 1he remainder of !his section we usc 1he
del operator 10 define 1wo ex1remely tLo;eful opera! ions on vecmr fields: the diergence and !he
curl.
DEFIN ITION 14. 1

If f'(.r. y, z) = P(x , y. z)i + Q(x. y. z)J + R(x , y, z)k is a veclor field in a region
D , then lhc dh'ergence of F is a scalar field in D defined by
div JI

V t'

iJ P
ox

= -

iJQ

oR

+ -iJy + -ilz .

(1 -1.-1)

provided 1ha11he parcial deriV'<Itives exist a t each poinl in 0 .


ll is clear why we u!)C the notatio n '\7 F , in spite of the fact thnt V is no t a vector in the
true sense of che word.

I EXAMP LE

14 . 1

Calculace V F if

= 2xy1 + :;] + x 2 cos(yz) k

(a) F

where r =

(b) F

= -4.rrqQ
=-.r.
tolrl'

.:ci + yj + zk.

SOLUTION

a (z) + -,a [x 1 cos(yz)J = Zy -

1
x y sin(yz).
y
az
(b) For chc dcrivmi,e of che x -componcm of F with respect 10 x . we cakula!c

(al V f

-a (2xy)

+ -a

\Vith similar results for the remaini ng two derivative~. we obtain

qQ [-2 x1+y2+z2

\1 -~ =--

4 tr!o

~(B';W

Vector\'
reorwttlS dir:lion Ql' ~:t..: now at
~inl P. Aka um 1~na at P
petpe:ndccul:\r 10 V

(x l

+ y1 + ~2)5/1

xl - 2r2 + zl
(xl

>+

zl);/2

x1

(xl

+ y1- 2zl ]
+ yl + zl );/2

0.

Any physical imerpretation orV' F depends on the imerpretation ofF. The following discussion
desct; bes the imet-preta!ion of the divergence of a ce.nain vector tield in!he theory of fluid flow. If
a gas flows Lhrough a region D of space, Lhen i LHows with some vcloc.ity v past point P(x, y. z)
in D at time 1 (Figure 14.8). Consider a unit area A around P pcrpendkular to ,,. If at lime f
the densicy of gas ac P is p, then the veccor P'' represents the mass of gas flowing through A
per unit tiJne. At each point P in D. the di.rec1 ion of P'' cells us the direccion of gas flow, and
its leogth indicates the mass of gas flowing in Lhat directioo. For changing conditions. each of
)'

p and v depend '"" only on position (x, y, z) but also on time I :


P'' = p(x , y, z. t)v(x , y, z, 1).

986

Oup:er J4

\\-~cot C~~;J.:ulus

The vector p v. then, is a ' 'ector field ~1at also depends o n time.
lol Ruid dynamics it is shown that at each poim io D. pv must satisfy

iJrJ

'V. (pv} = - -.

at

called che equation or conrinuily. h is this equation that gi\ ~ us an imerprelation of the
divergence of pv. Density is the mass per unh volume of gas. ]f ap/ dt is posiLivc! then

density is increasing. This means that more mass must be entering uni1 voJumc lhan leaving
it. Simila rly. if iJp/81 is negmive. more mas' is lea,ing than entring. Since \1 (pv) is the
negative of iJp / iJI, it follows that 'V (pv} must be a measure of how much more gas i~ leaving
unit vohunc than entering. \Vc can sec. then. that the word d;,e'8encu is appropriately chosen
for this application.
TI>c curl of a vector field ;. defio>cd as follow>.

DE FIN I T ION 1 4 . 2

If F(x, y, z) = P(x , y, z)i + Q(x, y , z)j + R(x , y, z)k i~ a ' 'ector field in a regioll
D, then the curl ofF is a vector function in D defined by

cuoi t'

\1

x F=

k
a

12 a-R'
_ (aR
_ aQ)i+
(aP
_ aR)h
(aaxQ_ aP)
J( n4, 1
iJy
ilz
ilz
ax
d)'

Clx
p

provided that the partial dcri,-ati"cs exist ac each point in D .

In representing curl F in the form of a dctcmUnanl we agree to expand the dc1crntinant


along the first mw.

I EXAMPLE

14.2

Calculaoe the c urls of the ,ec1or fields in Example 14. 1.


SOLUTION

i
(a)

'V x F =

i{

a
az

a
ax ay
a

2xy

z x 2 co.s(y:)

= [-x 2 zsin (yz)- J)i + [- 2xoos( y:)U + ( - 2x)k

(b)

'V x F =

qQ
41T ~o

a a a
ax ay az
z

lrl

lrl

lrP

x }' i:s lfQ/(41!E0) muh iplied b~

11lC x oompolC:n( of '\/

1/(zj;ji) - a(y)
il [<x' + :+
j;ji =

~)'

oz

1/y

y>

;:')31'

Similar resulcs for the)'- <l.lld zcomponents give\! x I<'= 0 .

rf a vector pv is defined as cibove for gas flow through a region D" then the curl of pv c.Jescribes
the tendency for 1he motion of the gas tv be circular rather th~tn flowing in \l straighc line. This
~ug:gests why lhe tenn curl is used. Wi1h this inlerpretntion, the followi1lg dctlnition scent<;
reasonable.

DEF INITION 14.3

A vector field F' is said tO be irrota Uonal iu u region D il' in D

'i1

F = 0.

( 1-1.61

Applicutions of d ivergence ~nd curt extend far beyond the topic of fl uid dynamics. Bmh
are indispelsable in many area~ of applic:d mathematics. such as etc:.ctromagnetism.
coni inuum mechanics. and hctH couduction, to name a few.
The del QJleralor '0 operutes on a M:Uiar field to J:>f'Oducc the gmdicnt of the scalar tield ami
on a vector field to give the divergence a nd the curl of the vector field. T11e fol1owing list of
com:~

propi'tlics for the <lei operator is straightforwaro to verify (see F.xercise43).


Iff and arc scalar fielc.ls that have first panial derivatives ln a region D. anc.l i f F and G

arc \'e<.1.o r tick!:. ht D with comJ>OilCrlL~ that ha\:c fi rst par1ial dc,rivativcs. tht"n

+g)=
\1 ( F + G) =
'il(/

'i1

(F

+ G)

'il<fg>
'i1 (.[F)
\f X (/F)

\1 J

'ilg.

(1 4.7)

+ 'i1 G ,
= 'i1 X + 'i1 X G '
= I v g + g Vf.
= \1 f F + f'il F.
= \f f X J7 + f '\1 X ,

( 1 4 .~)

\1 F

" . ( F x G) = G . (v x ~) - .

fJ

'i1 ( V

( 14.9)
41 4. 10)
( 14. 11 )

(1 -1. 1:!)

cv x G ) ,

tl -1 1:1)

(<J.f) = 0,

(I -I 14)

tl ol 15)

F) = 0.

Fo r properties 14.14 and 14.15, we assume th at and F have continuous second-order partial
derivatives in D. A typical way to veJify th ese identities is to reduce each side of the idemity
to the same quantity. For example. to verify 14.1 3. we set F = Pi
Qj Rk and G =

Li + M.i + Nk. Then


Fx G

= (QN- RM)i + (RL- PN)j +(PM-

QL)k;

988

Ot:stMcr 14

\~ctor Caknlu.s

1hus,

'J (F x G)= -

a ( QN- RM) + -a ( RL-

ox

()y

iJN
= Qilx

<IQ

+ N-

iJ M

- R-

ax

ax

PN) + - (PM - QL)


oz
ii R

i)L

+ R-

- M-

ilx

(I R

iJN

+L-

ay
oQ

- P-

ay

ay

iii'
iJM
ill'
il L
- N+ P+M - Q- - L -

oy

az

dl

az

{) z

L(iiR _ i Q ) + M(iiP_ iJ R) + N(iJ Q_iiP)


oz

oy

ilz

ox

ax

(a _!JM )

+ P( il/11 _ i!N ) + Q (iJ N _ (J ) + R


/ly

ilz

ax

ily

ilz

fJy

ax

On tltc ocher hand.

G -('V

F) - F . ('V

I
= G-

C)

a;ax a;ay a; az Q

ilf i!y

il / ilz

F - ilf ilx

= (L M N) _ (iiR _ il Q iJ P _ oR iiQ _ iJP )



ily
i!z ilz
ox ilx
ily
_ (P . Q. R) .

(iJN _a'"'. In _ aN_ aM _


ily

il;:

ilz

ax

ax

aL)
i)y

L(iJ R_ iJQ) + M(aP _ iJR) + N(aQ


. _ iJ P)

ily

_ p

il z

ilz

ax

ax

ily

(aN _ oM) _ Q ( oL _ IJN ) _ R(ilM _ 8L )


i)z

ily

i)z

ax

i)x

ily

and this is the same cJtpression as for 'V (F x G) .


Gi ven a scalar function f (x, y, :) it is str-.lightforward to calculate its gradient 'J f. Conversely. given tlte gradient of a function '\/ J . i 1 is possible to find the function f (x , y. z) . For
example, iJ the vector 6el d

+ z2)i + (x 3 z + 2y)j + (x 3 y + 2xz +


is known to be the gradient of some function f (x , y , z), then
F = (3x 2 n

ar. + -of.j+ aJ.


- k

- i

az

= (3x yz

l)k

+ z2 )i + (x }z + 2y)j + (x }\' + 2xz + l )k.

Since two vectors are equal if and on_


ly i r they have identical components, we can say thai

af

()y

=X Z

+ 2y~

ilf

- = x y
ilz

+ 2x~ + I.

Tutcgration of the first of these with respect to .t, holcli ng )' and z constant, implies that
f(x , y, z} must be of the fom t

f(x , y , z) = x 3 yz

+ xz 1 + v(y, z)

for some function v(y, z) . To determine v(y, z), we substitute this .f(x, y, z) i ttto the second

equation,

ov

- = 2y.
0)'

== )'! + 1.11( : ) for stuue u; ( : ) . tmll therel'CJre /(X, y, : ) = X~ )' : +


We now know both the X a nd ydc pcndcnce o f j'(x, )',l) . TQ tind w(t )

Con~ucruly, u(y, : )

x:::

+ y : + w(::).

wt Ml b~ti Cutt" intU the C.'\IUHtiUII fur

.\ y

_
+ 2.,
~ "i

!Tom which we h:w e w(z)

aj f {J;,:

t/ u.,
..
d ~.

= ..-: , y

:2.\ ~

-t

= z + C. where C

'-1 I

=:>

<lw

d o;

1.

is a eom:tiLnl. T hu s.

a nd Uus re p resents all l'u nccums that lmvc u ~nu.hcnl c;.tuI IH the g vcn \'CCh'K' F .
A more d ifficu ll question is to d<:ccm illc: whehet (t g h'el \'eet(ll' tleld ..~(x. y, :_) is the
gradjent of some.scalar fu1lCtion j (x. y . z) . ln nH'Iny exampleii. \Ye <'-M say that if lhe ~ed u re

described b<wc faj ls, thC-tl rhc: answer is lO. F;or insttlllcc, if ,.. c: x :!y i

fi lh:m pl m lind a func tinn .f(x, y , z ) ~n lhHt V

aJ

ny

)o' ,

X)' .

+ xyj + :,k , M d we

the n

aJ

The fi rst im plies th at


X :\)'

f(.<. -'' z)

= 3 + U( J' , l. ),

aod when thjs is s ubstirutc::d ioto t11e seco.nd , we get

x3
au
- = X\'
3 +8y
..

ay

= xy -

x'

:;

Dut th i~ i.s an impossible ~i tuali on s ince 1; is to be a function of y <:~.nd z only. Bow then could
itS deri,aUve depeod Oil x '! Although this I)'J):! of argumem will StJftic-e in most examples. il is
rca.lly n ot 9 sarisfacrory mathcmntica l onswcr. T he followi ng theorem gives a tes.t b)1 which to
dctenninc whether a given vector function is the gradient of some scn l ~r function.

THEOREM 14. 1

Suppose the component P(x. y, ~). Q(x . y, z) . and R(x. )' . z) of F = Pi + Qj + Rk


have continuous first partial derivatives in a domain D . lf thcrccxists a fu nction f(x . y. : )
defi ned in D suc h that V f = F . ahcn V X IF' = 0. Com'erscly, if D i~ ~impl y connected.
and 'V X F = 0 in D , then Lhere exis~ ~ funCLion f (x , y: t) !to UCh that 'V f = F in D .
Tt i:;; obvio us that if F = V J. tl1cn V x F = 0. for this is the roult o f cqmui('l U 14. 14. T,'l
pm\'e the co n ve~e re:-.uJt requires Stoke.,.s tht!<)l'l!l11 from Soot inn 14. 10, and a proof is therefore
delayed until that 1imc. Nutice thai in the con ~er~ l'c~uh the domain (open. connccu:d set) mt" t
be simp ly connected . This is our first c ncQunlc t with a :\ituation where the nature of a region is
important to the result.
rn tlu~ special case in which F is a vector fie ld in rhe .ry~pl ane, the equation 'V 'X F = 0 is
still the condition fo r existence of a functio n f(x . y} such that 'V f = F = Pi + Qj , but the
condition reduces to

i
0

ik

iJjl)x

a;a.v

8/f!z

= CQ
ax - <IayP) k

or

aQ
ax

aP

- ay

(14. 16)

I EXAMPLE 14. 3
Find. if possible, a function

f (x , y) such that

SOTl 11011:

a (x3-2)1)

and

iJy

xly

a (I

2y) = Yl + ;r
-2

y- .<

ay

- 1

Since the componem> of F are undefined 1\heoacver .1 = 0 or y = 0, we can >tate that in any
simply cono~ected doma in tlout does not contain point> on either or the a.( C;, there is a function
f(x,y) such that 'ilf t '. 1\> lindf(x .y), wcsc1

IJf

a.r

= -

IJf

2y

ay =

- l~

.l'J -

yl'

From the first equation."" ha1c


X

((x. y) = -

+ "l\' + u(y),

y
whi~h.

'

subsl itutcd into the second equation. gl vc~ us

du

t/ 1)

.t

--+-+yl xl dy = x2 - yl
Thus. t(y)

dy

=0.

)'

+ ,. + C
y

= C. and /(x, y) = -

x~

EXERCISES 14 1

z ~ x2 + y 2

ln E.'tCKiSQ l --9ckttnninc whabc:rthc ~ ofpoinlS in the xyplanc: is

12.

open. dosed. conn<d<d. a dO!Min. and!<>< a simply <<l<tn<dOO domain.

13. x 1 + y 2 + z1 > 0

1.

2.

.1.
4.

s.
6.
7.

x 1 + (y ~

1) 1
1

<4
x' + (I' - 3) !: 4
0 <.r 1 -(y- t)1 < 16
t < (x - 4)' + (y + t)l S. 9
,\' > 3
yS. 2
l(.r - 1)1 - (1 + 2) 1 < 16

8. All points s;ni>fying ,,I+ y' < I oc (A

14. I < .r' + y 1 + <' < 4, X ~ 0, )' ~ 0. Z ~ I)


15.

x' + y' > 0

t6.

Ill> o

17. t' > .r'- J'

2) 1 + 1' 1 < I

9. 4(x+ t) 1 +9(y-2) 1 > 20


10. Giv-e definillons of 1hc following for scb of points in xyzspGCc::
interi. utcrior. ...nd bound.lry poUxs: open. .;~. and connected
!!~Ct.,; doolain aoo iimpl)' conf!C'Ctcd domain.

us. z1 > x1 + ' 1 l 9. : 2 < x 2 + y: -

1
I

20. Prove: lhit the only nonempcy et in the xy~planc tho! it bolh open
~~nd t l0$ed islhe whole plu.nc.

In hm:oscs 21-40 etkvl>tc lhc otquiocd quJnlily

3x'r- y':'

In E:<crciscs 11 - 19tlctc:nnine whether the l!ICI of poinl~ in space iS(lpcn.

21. I'J fir f(x . .1.

Clc)Sed. conoecLcd. a dnmuin. and/ol' a simply connected domain.

22. I'J f if[(.<. y. z) = (x' + y' + z')-' 0

I I. x 2 + y 1/4+z 2/9< I

23. I'J[if j(x,y) =1\m

:) -

(yfx)

14.1

47. F'(x. y)

15. 'i1 f at ( 1, -l, 4) if f(x , y, c.) = e' ,.:

48. F(x , y) = (x +

27. 'i1 F if F(x, y) = x ln yl - y3e-'J

28. 'i1 F if F(x. y, z) = sin (x


29. 'V F if f (x, y) =

In Exercises j0-55 find all functions [(x, y. z) such that V f = F.

+y + < )i + cos (J' + t)J


2

50. F(x , y, z) = xi + yj + zk

e'i + c' J

30. V. F at (t , t , I) if .F(x. y , z)

51. F(x. y, z) =

l-

x 2y 3

32. V . F if F (x, y, z) = (xi + yj


33. 'V . J' if F(x . y) =Cot

3xyj + z 1

53. F(x, y, z) = (2rf y2 + l)i - (2x'fy')J -

+ zk)/ /t2 + yl + zl
1

(xy)i + Tan

54.

= x'zi + t2xyzJ "- 32y2 z4k

35. V x F cf f:(x, y) =

.re"l - 2xy2j

* 56.

J+ z'k

yl - xj
39. V x F if F(.x . y. z) = In (x + y + Z)(i + J+ k)
40. V x F if F(x. y) = sec- (x + y)j + esc- (,v + x)j
41. If F = .t 2 yl - 2xzj + 2_\'tk ,lind V X ( 'V x F ).

(b) Sho\V that V f x V g is solenoidal for art>ittl!J'Y fu nctions


J (x, y , z) and g (x. y, l) lhotha\'CCOtllinuous second t><l<
tial deri\lativcs.

satisfies

* 53. Associated with every e lcclric field is a SC'JI31' fune1ion V (x. y. z)


called pote11tinl. h is defined by E = - 'il V. where E i< n \'OCior field
c.aUcd the dt:t'lric: Jit'IJ i ultn.fil). In udclition, if a poinl charge Q is
pb:ed at a point (x. y, z) m 11Jc elec-tric field, lhe!t the force F on Q
is F
QE.
(3) If the fort'C on Q due 10 3 charge q 31 lhe origin is

43. J>ro,-cpropcnics l 4.7-14. t2. t4.14,und 14. 15.

* 44.

A gus is moving through some region

D or space.

If we follow a

p3rtlculur ptl.rl.iC1e Of the g:)$, it lnlC~ ()l\1 $()!1\C C'.Jrved p 31h C : X

x(r). y
y(l). z
z(l), where r is time.. If O(x. y.z.t) is the
dcn:sil~ of gas ut uny point i.n D at time (. t..hcn ;;~long C we can express
dcn<ity in tcnns of t ooly, p = p[x(r), y(t), z(t), 1). Sh"" that along

c,

ap

t' =

where r

dp
dr
- = - + 'Vp - tll
dt
tit '

whcrc r =xi +

45.

F(x, y)

qQ

4iu olrl'

r.

= xi + _rj + x.k and fo is s consumt lind

V(x. ,r. l.) for I he fie.ld du" 10 q.


lhj II' lhe entire x_r ..pl:ane i ~ ghren 3 u:tiform ch:J:'!C: den;:icy
17

JJ+ tk.

In F.xcrcb;.c~ 45~9 find all functions

3y- z)j + (4x + C)'+ 2z)k

(2x 1 + 8xy 1z)f + (3x 3 y- 3.ry)] (a) Is ci1her of E'


(4y'z' + 2x 3z)k or xyz'F solenoidal?

the fonu V V f = 0.

+ z2) - l/ l

+ c~<)l + (bx -

57. A vector fic::ld F'(x, y, t) is said to be so/maida/if V F = 0.

(a) Verify thua l.apt3CC ~ cqumion 12. 12 c:at'l be: expo!ssed in

y2

and c in order that the \'t elor field

will be irrotational.
(b) IfF is irr<>tational. find a scalar func1ion f(.r. y. zj such
thatVJ= F .

:Ill. V x F at (2, 0) if F(x . y) =

= (x1 ,

+ xy) k

(a) Find constants a . b.

F = (x 1 + 2y

37. 'V x Fat (I,- t , 1) if F(x, y. ;,) = x ;;3i- lx 2yz} + 2yd(

(b) Show 1ht' /(x. y .z)


Lap1accs equation.

F(x,y,z) = (1 + x2y1)- 1(yi + x])+zk

(2x 1z + 6yz- 2yl

3<1. V x F if F(x. ~. z) = x2i + y2

42.

2zk

55. f (x, y. z) = (J.r1y + yz+ 2xz 2)i +(xz +.r 3 + 3;: 2 - 6y 2z)] +

(xy)}

34. v x F if F (x. y, z)

.rzi + x<] + (yx- 3)k

52. F(x. y.z) = ( 1 +x + y+z)- '(i +]+ k)

+ y)l(i + j)

31. V F at (- t, 3) i f F (x, y) = (x

}r'd+1>

~ 49. F'(x, y) = - .ry(l - x 2y1) 112(yi + .rJ)

y, t) = 2xeYl + 3x 1 ~- 2x1yzk

991

= eYi + (.re> + 4y 2)j

24. 'i1 f at ( l, 2) iff (x , )') = x3 y - 2x cos _


,.

26. 'i1. F if F(x,

VectOr Fieltls

uniiS

o( Ch:.ltg

per unil

~U\!3.

it iG found Ihal 1hc force

on a charge Q pla<:cd z unhs above the plnnc is }t-. [Qa /(U 0)]k. find the J)Oicmial V ror 1hc clec~ric fie lei

(X. y ) such lha V f

= 2xyi + x'l

46. F(x, y) = (3x 2y' + 3)f + (2x 'y + 2)J

due to this charge distrtbutio.n.

= F.

59. Show that if v = ~

r = xi +

yj + zk,then w =

r , wbcrc

tA>

is a constanl v.xtor, und

(1/ Z)(V x v) .

60. Show 1ha1 if a fun ction f ( x, y , z) S!llisfi e-s Laplace's cqua1ion


12 .12, lbt::.n iiS g:raditnl is bolb irrQ4.aliOmll itn d solenoida l.

1<

61. Th-I.:Qrcrn l d.l indicates th:u ~ "~ccor ii~ld I<' is the ;;:r-.tdicm ofoome
scalnr field if V x F = 0 . Somc.':timcs a g i,-en vector field F is the
~wi

of ruw(hcr 11ch.l v: lhat is. F

'V

'X

t;:u.i&lics these ~C)u:.uions . In Olhcr word!i. this fonnula d~


ltnc); a pos~ib1c vcctOf 11 . Is it unique?

v. TIM: t'ollowiu.g thc:orem

(c) S how that ifF sat.isfics (he pr(lpcn,y that


t" P(X,),t). lbcn

or

il'ldieatts when thi:\ i.s the case: Ltt 0 be the il'lttrior


a S-phere in
wb.K'b tlM: (.;0HI.JlVIICJI~ vf ~ ~.._'\:,Or (icld F h:~.n: (.;VLiliJ.lUVUS fin.l ~HlJ1j;~\
ckriv:.nivcs. Thc.n there eJ~.ists a "ector field " definOO in 0 such thut
F = V x ,. if and onl) if 'V F = 0 in
1n (IC.hcr wor(l~, F is the
curl o f a vector lk ld if :'lnd ooly if f.' is $(1lcnoi d~1 L

n.

(a) Show thai ifF -

lli

( y.z=
)
,x,

1
- +
-Fx r
2

F(u. r:-. r t)

= xi +y} +tk.

11

+ Qj + Rk is soleno idal, t hen the


+ M j + Ni: W4)ul(l h:we 10~1isfy

c)'"f.l('I)CI'II$1)( , . = Jj
tt~ ~1''<1li l)n~

IJN
P =

IJM

!Jy -

Q = a_L _ fJ_N

a,

at

a~

tn E:t:erciscs62-64 verify thatlht \'CC10r field \sl)OlClOidal.and Lt.cn\lsc


l hc fonllulas in Excrc.isc6 1 to find a ''taorficl.d v such that F = V x \1 ,

JM
IJL
R = "[( - !Jy

6l. F =

(b) Show 1.ha t 1hc vector field \'(X, y . z) defined by

xi + yj -

2,_k

63. F =(l + x)i- (.t + t)k

M. F = 2x' i -

y'J+ ( 2)'4- 4xz)k

114.2 Line Integrals


Just as definile integrals. double lntcg,rals. and triple integrals arc defined as limits of sums. so
3313ii;IWC .
~C1U \'0:!1)1' tO

)' = )'(/). ::

dr
.IS :t 1:\n-

defined along curves. and surface integrals involve functions defined on surfaces.

dl

a Cllr'\'C (,': ;r =

are line and surface imegmls. The only difference is tha t li ne integrals are applied to functions

X(l).

A curve C in space is defined parameLTkally b) three-funcllons>

.t(l)

C ;

= X(l).

y = ylf),

t. = t.(l).

a ::::

I ::::

/3.

(1 4.171

\Vhcre a and fJ spcclfy initial and fina l points A and B of lhe curve, re.spectively (Flg.ure 14.9) .
The direction of a curve is fro m initial to final point. and in Section 1l.9 we agreed to parametrize
~c urve

using par(lrueters that increase i l l the ~..li recti o., of the curve.

The curve is said to bet<>11rimwus if each oflhc functions x (r) . y (r). nnd z(r) is continuous
(implying that C is at 110 point separated). l t is :said to be smootlr if each o f these functions
has a cootiouous tlrst deriv:;uive; \.!.eometricaUy. this means that the talQ.eot vector d 1/ dt =

x' (t)l + y' (1)] + z' (l) k

turns gn~duaUy or smoothly along C . A c<>nti;uous cu"c thai is nm


or smooth subcurves is said to be piecewise

smooth but can be divided into a tinitc number

.r
)'

Oiv;dc C
in1o shoner curve~ to define the line
intcyul of f (x, ! ) along c

r.

smooth.
Supf.X).se a function f(x, y, z) i~ defined along a curve C joining A to B (Figure \4.10) .
We tlivide C imo n subcun-es o f lelgths 6.s._ 6s 2... . 6.s,1 by t\ll)' 11 - l consecuti,e
points A = Po. P1, P2 .. . , Pn-l . Pn = B . whatsoever. On each subcurvc of lcng.lh /::;.Si
(i = 1, . . . 11 ) we choose an arbitrary point P;* (x; .
\\'c then form the sum

.v7. z.f>.

f(x;.yr. zD t.s,

f<x;.y;.z;)t.s,

..

+ - + f(x~.y!. z~) L>s..

f (xj , yj, zi) ll.s; .

(14. 18)

i=l

If this sum approaches a limit as 1he numbel' of subcurves becomes increasingly large and the
length of every subcurve approaches zero. we call 1he limil the line integr al off (x. )', z) along
the curve C, and denole it by
X

( f(x,y, z)ds =

lc

( 14. 19)

A more approprillle nanlC n1i2ht he cun i liii<'Or imegmlr-Jiher thn /iue illh'Jim/. but liM ini<J(ral
is the aeecptc<ltcrnoinulogy We regard the word lint as ma~ning t-un t'tl lttl<' rather than straiJthl
lillf!.

For definition 14.1 910 be u~ful. we demand that the hmit be in<lcpendcnt of the manner
of subdiviion of C and chuice of sw points on lhc bcurvc:s. Theorem 14.2 ondteatcs that for
continuous fuuction~ dehned on smooth cuf\oe~. this requirement is aht.ayi :sati~ficd.
THEOREM 14. 2

f(.r, y. ~)be continumL< on n smooth curve Cof finite length, C : x


x(l),
y = y(1), ~
<:(1), ~ I < p. Then the line integral of /(.r, y, :) along C exists
and can be e\aluate<l by mean> of the foUowing dch nile inu:gntl :
Let

Jc[(A.y.:)ds

= 1ft

1('1 f)l + (dy)


dl
+ (d')l
d; dl.
2

/(.r(l),y(l ).:(r)]'

4~01

It i~ not necc.sary to mcmorit.e I ~.20 a< a forn1ula. Sompl) put. tile right side is nbtained
byupre<sing x, y , :. <llld d.f in terms of I and interpreting the re<ult l" a definite integrnl with
re;.pcct to 1. To be more explicit. rce>lll from equation 11.8 1 thftt when length along a cun e
is meaure<l from ib initiul poim. then an infinitesimnl length d.r alon~ C cooesponding to an
increment d I in I is given by

ds = ./(dx )l

+ (dy)l + (t/:)2

[(~Y + (,Y + (r)}dl)2

/(~Y + (~;r + (~~r

t/1.

If \\e substitute thi into the left side of 14.20 and at the .anlC time usc the equations for C to
e<pre.~s f (x , )'. ~) in term~ of I . then

fc j(x. y. ;) ds = fc f lx(l),

_v(l), <. (1) )

+ (d:)2
(d- x)l+ (dy)l
dl

dl

dl

dl.

But if we o1ow interpret the right side of this equmion "'the uefinite integral of the function
fl.t(l), y (l) . ~ (1 )) ./Cdx/dtP + (dy/d1) 1 + (d<./di P with rc,pcct to 1 11nd allix limits
1
et and 1 {J thm identity endpoints of C . we obtain 14.20.
Toc-.luutc ulinc imc~ml. then. \\C e-.press /(-<. y. :) uno ds in tenus of wmc p;lf'.tmeter
along C and evuluatc the resulting definite intogr\11. If equotion lor C re given on lhc form
C : ) = ) (x ) . ~ = ~(~ ). x. < x < xn. then x is a COOl\Cnicnt p.llanlctet, ru1d equ~tion
I~.20 wkes lhe form

fcn~.~.z)ds c

Lflx.y(x).z(~)],I1-(dJ. (d;) .dx.


'

Similar expre.):;ion!) exis1 ir ehher y or

d)' -

d -

z is a comenienl p:u an1ele1.

When C i!) picccwi~ smooth rather Lhan smoolh, we cvulua1c line imcgrals along each
smoolh !loubcurve and add rc~ulls.

I EXAMPLE 14.4

+ 6.xy + 30;; from A (O , O, O) to 8 ( 1. 1, I)

Evaluate the line integral of f(x, y, ~) = Sx

(a) Along the straight li ne joi ning A to B with parametrization:

C, : X =
Mtift!J
alon~

The line unegml

I,

y = I,

() :$ I < I.

I,

Along the straight line in pan (a) with parametrization:

(b)

a run e UMtully llqxlllh on tJ1e

c:urvo be1wcen 1hc PQints

c , : x = - 1+

2,

y = - 1+

2'

' =

- 1

+ -2'

2 :$ I S 4,

(c) Along the curve (Figure 14.11)

-,, = 1'2

SOLUTION

(a)

1
c,

(Sx

+ 6.xy + 30z) ds =
=

f'
(81 + 612 + 301)/( t)Z + (1)2 ,. (1 ) 2dl
fo
(381 + 61

J3 [

d l = J3(1912

+ 21 3 1~

= 2 1J3.
(b)

1
c,

(8x

+ 6.xy + 30~)ds

r' IS( - 1 + l / 2i

12

X f (l /2)2

= .j3
2

(c)

1
c1

(Sx

+ 6xy + 30z)ds

+ 6(- 1 + 1/2): + 30(- 1 + 1/ 2)1

+ ( t/2)2 + ( l / 2)2dl

1
l

\38( - I + I /2)

+ 6( - 1 + 1/ 2) 2) d l

../3
-{38( - 1 + 1/ 2) 2 + 4(-1 + 1/2) 1):
2
-

2 138 + 41 = 21.Ji

J3

= [' (81

fo

+ 61> + 3013) /( 1)2 + (21)2 + (31')' d/

= [' (81 + 36t 3)J 1 + 4/ 2 + 91' dt


}.,

Pans (a) and (b) of this example suggest tha! the value of a line integral does 001 depend on the
particular parametrization of the curve us.edl in its evaluation. llliS i:> indeed true. and .}hould
perhaps be expected since de~ ni tion 14.20 makes no reference-whatsoe1e to parametrization
of the curve. For a proof of this fact, see E~ ercise 37. Different parameters normally lead to
differe nt definite integrals, but Lhey all give the same value for Lhe line integral. Pat1s (a) and (c)

illustrate that the line integral does depend o-n the curve joining points A and B (i.e., the value
of the line integl"l changed when the cune C joining A and B changed).

l ine lnc<:gr.ab

L-1.1

I EXAMPLE

995

14.5
Evuluate the line integral of / (.r. y )

=.rl + )'Z once clockwise around the c ircle x 2 + y2 = 4,

' = 0.

UM!JIlil\1 U I E\ nluation
of x: + ) 2 uroonJ
.t'~ + ,.~ = 4. :. = 0
1

of lit:<: inteyal

SOU TIOi\'

If we use the parametri 7.atio n


C: .t = 2cosl,

)' = - 2sinr .

= 0.

0 :! t < 2rr

2
(Figure 14. 12). then

r
Jc

(xl

+ yl) tfs =

1l"

=4

(4)

1-

( dx
d t ) + ( d\')
d.t l dt

-/ ( - 2 sint) 2

+ (-

2cost}' tlt = 8

Planar lir:e

illleg,rals of noonegath-e functions can


be intctptt:tcd

Oh are<b

uf

WtJJI ~

1k =
dt

16Jr.

The value of th is line integral, as well as many other line integrals in the xy-pl:1 ne, can be
given a geometric interpretation. Suppose a fun ction f(x. y ) is positive alo ng a c urve C in the
x y pla ne. Tf at each point o f C we draw a vertica l line o f height ~ = j (x , y ). thc.n a vcnical
wall is constructed as shown in Fig ure 14.13. Since ds is an ele mental piece o f length along
C . the quantity f(x . y) ds c an be interpreted as approximate ly the area of the venic<tl wa ll
projecting onto tis. Beca use the line integral

fc

j (:r, y) d s.

like a ll imegrdls. is a li mit-summation process. we interpret the \'llltre o f this l ine integr-dl a.< the
total area of the ven ical wall. C orre<:t as this interpreta tion is, it really is of little use in t he
evaluation of line integrals and, in addition, the: intcrprc.tation is valid only if the curve C along
which the line integra l is t>erfom\ed is contained in a plane.

Because a line iJUegrnl is a limit summation. it should be obvious 1hat dK: line integml

( 14 .22)

rcprc<;ents the leng th of the cur>c C . If C is u c urve in the x y -pla nc. we substitute d s
j (dx)l
(dy) Z. and if C is acUf\'C in space. then ds = .,j(dx )2
(d~)l. This
agree.< wiLh the results o f equat ions 11.74 ami 11.8 1.

+ (dy)l +

\Vc make one la:st 5)\)int about notation. To indicate that a line intC"Jal is being C\'aluatcd
we usually draw a circle on the integral sign . .ts follows:

~\tou nd ~\closed c.:urve.

fcJ(x ,y, z)ds.


Such would be the case for Lhe curve of intersection of the cylinder .x 2
x + z = I (Figwe 14.14).

+ y2 =

I and the plane

WillfiiJ;Ij

ClCfJ

X! + )'! = I i.llld d 10

Cu"''~ o! 1n1cn.lion I)(

die:. o;ylindcr

p101nt X+~ I is c:)Cie(l

\Vhc 11C is a t:lo:,cd cun-c in the xy -planc that docs not cross itself. we ind icate thc d in::ction
nlong C by a n a rrowhead ontbc circle. For the curves shown i11 Figures 14.15 we write

J.

Jc,

f(x,y)ds

and

J.
rc!

f(x, y)ds.

lii!lrll);t

)'

c,

EX E RCISES 14 .2
In E.xen.i~

I~

f-\l:tlu:ue lhe line i1uegral.

6. [x 2 yr. ds, v.bcn;: C i:s lbc curve t = x

I. [x ds, where C is the curve y = x'. z = 0 frotn (0. 0. 0) to


(I. 1.0)
2. fc (x 1 + y1 ) Js once llrOund the squw-c C in tbc X)'plnne with

3.

f c(2 +x- 2xy)ds ooccuroundthccirckx'+ y' =

4.

{ (x' + yz) ds along the straight ijne fro on (I. 2, - I) to (3, 2. S)

5. [ xyds.whereCis Lhefiniloctanlpartofx1 + y2 = 11
I from ( 1, 0.0) to (0, 1, I)

- 1/2, 1/2) to ( - 3. 7{2. 1/l)

7. Prove that the JengthOflhecircumfCI'UICC Ofa circle i~ 2n ll!Ullil)licd


by the radius.

8. A :,pring hd$ siA ooil~ in lhc form of the lllr;lix

Vl!nices (I, I) and (I. - I)

'('

from ( I.

+ }'. x + y + t =

4,

z=

x 1 +z1 =

.t =

3<""'

)'=hint.

31 /(4.~).

0 .::; t .::; 12n.

"'l'len:: <~II <limcn~i ons arc in CC:Illimc:tn:s. Find the length of ti"C spring.

9. Use patameuic equations X = cos3 6' )' = sinl 8 > 0 :s e < 21T.
the astl'o id x 113 + y11l = I in the x y -t>lane. Al

10 draw Ol' J)IOl

each poinl ( .\', y) on lhc ascroid , a ve rlitaJ ljnc is d rawn with height
z x1 y1 ,1hus fonning a cyLindriCitl WitiJ. Find the area of the wall .

= +

14.:!

In E,;cr<:i.scs 10-16ev:alu:nc the line intcg:r-.d.


t

10. 1x:tlsalong lhcfirstoetcmt parl ofy ;;;; .t2,

L+
(x

y)' ds oloe>g C

.<; 1 + 1/ t. y- 1 - 1/ 1 from

+ 4z :

5 from ( I , 0. I) 10 (0. ~/14, 11/7)

13. fcxrds. "hcreCi<1toccu"'"X

=1-)'' z =o rrom ( I,O,O)

to (0. I. 0)

14. 1 (x

+ y)~ds. where Cis tbccur.~ y =

x, z =

by the length of the curve. In E_,.crciscs 24-27 tind the ~~'cng_c value

1.

+ y' from

24. j(x, y) = x 2 y' along C: x' + y 2 = 4, z = 0


25. f(x . y.;;)=x'+y1 + z'aloneC : x=cost.,l=sin t,

cY+ Z

.- 1 from

(I. I. I) tO (2. 4. 4)
16.

(2.r + 9z)ds. where C is the curve z =

.-\'.\' . x = y1

from

(0. 0. 0) lD (4, -2. -8)

(t~)

17.

lf the cur\'c C bclQ\\o i t:~ rQtah::cl :.tround lhc )'axis, show


that the ~m;a of' lh.; :>u rfocc thal i I ltUCC.:$ out is rc~ntcd

by IJK: line intc&rdl

~ 26. j(x . .)'. z) = xyz long C: l. = x 1 _,. = x 2 from (0. 0. 0) to


(I, I . I)
27. j(x . y) c y olong C : .1' x 1(4 + l/(3x) from (1, 7/12) to

(2. 13/6)

.. 28. At each poinl on the curve (.r: + y:):


x 2 - y : o. vertical line
is drawn with height equal tO lhe diSIWlC from the: point lo the origin.
fi nd the UI'\;U of the \"Crtica..l WU1J M) formed.
+ 29. During a sleet SIOOll. a power line between two pole~ at posi tion~
x
20 hangs in 1hc shape y = 40eosh(X/ 40)- 10. where all
dasumccs art mea(ured in nletta:. Ice aco.m(hues more heiavi lyon Lhe
middle p..u1 of the line than at the ends. ln f:.cl. tbe combined m.:ls&of
ice and line per uni1leng1h in the x ~diJ'CC1ion at position x is ghen in
kil<>granl$ per 111<:~ by ~le formula p(x) = I - lx l/40. find the total
mass oft.he line.

2rr x ds.

In E.xcrc.tscs 30-33 u...<oe the fact tb~t in polar coordinates snniJ l~ns; tbs
along a cme<:an be expresseu in the form ds
Jr' + (d r J<i(l)' dO
(see formu la 9 .14) IOC\"alualc the Unc integral.

(b) If C is I"{Watcd around the x -axis. wh::tt line integral I"CJlrestnt ~~~: lhc :ue:t or the 1\ u rfoc~ tr!lced our}
)'

* 30.

i:

In Excrcisc:5 18- 20 u~ the mclhod in Excrci$c 17 to fi nd the urea

or

:c .:: 2. around .t;;;; 0


.<1) . around y; 0

.v =

.<'from (0. 0) 10 ( I, I),

ds. where C isLhefirstquadrant part of the liana~on

f c <x' + .l"l <h. where C is thecM!ioid r = 1 + o;:o,;O

1c

xy ds. where C iii: the spiral r

= II'

from tJ

=0

to 8

= liT

In Excrci:,.es 34-35 tint! adefinite inlcg_r;.~.l which cc:u11x uset.l tOl.:\'Uiuulc


lhc line imcaral. Use SimJ>Scms rule whh 10 equal subdivisions to

X.:+ 48. l ~

21. Find the length of the ps,..bol

c ,; x' + y'

1_ cos 2-IJ d s ;~.r-ound the firSI quadrant loop of Lhc lemniscate


rc
,.: = (in2l1

+ 18. y -.t'.t.:=:.t~2. uround y -0


+ 20. 8y 1 = x 1(1 -

32.

33.

the ~urfacc trecal oul when the curve i$ n.nalOO arounc.l lhe line.

iOi

= 1 - sin 8 starting from 11-.e: point on the .\' -a xi!'

* Jl.

+ 19. 24.t)'

z=

J . O~ t S;r

('

( - l. - 1.2)1o(l.l.2)
1
+ 15. 1 - - Js. whcr.;. C is the c-urve y

= 2. y+ 2z =

of lhc function along the curve.

12-.1c x J.r + z_ds. where Cis Lh:;u pal'l o( t_he-cunc 3.r+ 2y+3z =

6. X - 2.1'

e-r+J-l)-: ds. whc.;n: C isthccuncx + y+ z

TI1c a'~Crage value of a fuocLio n /(.t. y. z) Hlong a cur"e C is defined


a$ Lhe value of the line in1cerat of 1hc functinn along Ihe c urve divided

(2. 0) 10 (17/ 4. 15/ 4)

997

3 from (-1. 3. 0) to (0.1.1)

z+y l from

(0. 0. I) tO (I. I. 0)
II.

23.

Line lntcv,als

appto'\:inlale the dclinite in~cgral.

34. L <x'y+z)ds,wtocreCiSihccurvczox'+)''.y+xo 1

from (-1. 2. 5) to (1. 0. I)

In Exercises 22- 23 rind a dcfinile inLegrallh:at can be used lO evaluate


the line inlegr al Use J)()WCI' series to approximale dle deJinile integral
accurate to thi'Ce decimal J)faces.

II*

22.

1
c

xy ds, where C istbc curve y = x 3, z = 0 lrou> (0. 0, 0) 10

(1/2, 1/8, 0)

f cx'/ ds, wllcrc C is 1he cllipoe 4x 2 + 9y 1 =

35.

**

36. Find the sur(acc area o f lbc torus obt.a jncd by routing. aro und 1hc
y -axis, 1hc circle (x - <1) 2 y' b' (tl >b).

**

37. Show that the value of a line imegral is independent ol' lhe param

etet used to specify dle CUI"Ve.

36. z = 0

998

Cb3pler 14 Vec10< C3lculus

114.3 Line Integrals Involving Vector Functions


There are many ways in which

f(x! y. z) in Ihe line integrnl

r f(x , )', z) ds

( 14.21)

fc

can arise. According 10 equa1ion 14.22, we choose .f(x, y, z) = in orderlo find lhc lcngLII
of 1hc curve C : E.,crcise 17 in Scclion 14.2 intlica1cs thai for ureas of the surfaces traced out
when a curve in t.he x y -plane is ro~aced around Lhe y - and x -axes. we choose f (x, y ) equal to
2n x and 2n y , respcccively.
The mosl imporwnt and common lype of line integral occurs w hen /(.r . y, :) is specified as
!he ~111gential component ofsome given vector field F"(.<, y, ~)de lined nlL'"S C (i .e., f(x. y , z)
itself is not given, but F is, and to lind f(x , y, z) we must calculate che cangential componem
of F along C]. By the tangential component of F(x, y. z) along C we mean the componenl of
F ulong ~Uit tangcm vector to C thut points in lite same direction usC (Figure 14.1 6).
In Section J I.J l we saw chat if s is a measure of length along a curve C from A 10 B
(Figure 14.17), and if s is chosen equal to zero a.t A, then n unit tangent vector poin1ing in the
direc1ion of nlOtion along C is

dr
1'

= - .

ds
Consequently. if f(x , y , z) is the tangential component of F(x. y, z) along C. then

dr
j(x, y,z) = F 1' = F - ,

( 1 4.2~)

( 14.25)

ds

and we can write ~lat

{ f(x , y, <) ds = { F i ds = {

lc

lc

lc

~.

d r ds = { ~, . d r.
ds
lc

(I~

26)

If the components of Lhc vector field F(.r, y, z) arc


F(x , y ,z)
then

= P(x,y,z)i+Q(x , y. z)j + R(.r, y, z)k.

( 1-1.27)

fc F d r = fc (Pdx + Qdy + Rdz),

and if parentheses are omiued. we have

{ F dr= {

lc
M:!ilijll;ljm [W

Of the

1~"0

tangent dm:cttoM to :1 cunrc,

o.lv..-ays use the one that points in the

din~ctjon

Oo not usc

Usc I his

IhiS l:mgcnt

{;mgcnt

\'et'IOI'

\ 'e<:lOI"

I? U
c

lc

M#ltilil;l

(14.281

The intcg.-..nd of a Jinc incegr.al

of the curoc

)'

l'd.r + Qdy + Rd~.

tj;

Mren

Thi< discussion has shown that whcl the it!t~gnu~d .f(x, y . z) of a l i ne imegral is speci ned as
the tangenti:tl c"Omponent ofF = f>i
Qj Rk along C. the product f (x . y. z) tis can bo
rcpluccdbythe sum o fi>!O<luct s Ptlx + Qtly + Rdz :

{ j (x ,y, : ) tl.r = { F . dr = {'Pd.<+ Qdy

lc

lc

lc

+ Rdz.

( 14.291

According to the re-Sults of Section l 4.2. evaluation of this line intc.gral can be accomplishec.l by
expressing P dx + Q dy + R tfz in terms of any parametric reprcsen1ation of C and evaluating
the resulti ng defi nite integral.

I EXAMPLE 14.6
Evaluate

{ !:.d.t

lcY

+ (xl + yl + <:.z) dz.

where C is the li rst ootMt imerscction o f x 2 + _v 2


(0, I , 4) .

1 and

z=

2x

+ 4 j oi ning ( I , 0. 6) to

Jf we choose the paramelriUilion

SOLL'TION

x
for

=cos /.

y = sin/ .

z=

2WS I -r 4,

0::; I ::; lr/2,

C (Fiyurc l 4.1 8). then


{ '::.tlx

lc y
=

[,

+ (x 2 + / + z2 )dz
~/ [ (2cost-'. 4) (- sin/ dl)
2

Su11

+ (c'OS2 I + Si n2 1 + 4 cos 2 I +
= - 2

r /2

Ju

(cos t + 2 + 17sint

= -2 { sin / + 21 - 17cos / ~c:l!J ""

it\let')(di.)n

t:rU

of X:

(1.0.6)

..-",..........

~/

LinC' iAte>_;;t"'.tl ablg dtc liJllt


I lind t
2x + 4

+y! =

16COSI

+ 16)( -

2sin I

dt)]

+ 4 cos2 tsin l + 16costsinl)tlt

3
4cM 1

3
o<:I1Uit

+ 8sin2 1

1"/2 =
0

164
- 211' - - .
3

I EXAMPLE 14.7
Evaluate
2

fc y'dx + x dy,
where Cis the closed curve in Figure 14. 19.
14Jiclil,ll C]lg Une ituegrul
around the hound.1ry of a triangle moq

be divided into thr

SOLUTION If we start at the origin and denote the three s traight~ne paths by

J)-'rts

y
(I. I)

)'

= 0.

0 5

C2

I,

0 5 y 5 I,

C3 :

)'

5 I.

I - I,

0 < I 5 I'

then

Cz

c,

c,

(y}~ +

.......

n(l- I)Jl:

2
I - - =

If a curve C has initial and flnal points A and B. then ~te curve that lraccs the same points in
Jlte opposite direction, and therefore has inilial and final points 8 ilnd A , is denoted by - C .
Values of an imegral of 1ype 14.21 are the same along -C and C . This is so because length ds
is the same along C and - C . This is not the c.ase for integrals of type I4 .28. In Exercise 36 it
is shown lhal

f-c

F dr = - { F dr.

(14.30)

lc

This is illustrated in the follow ing example.

I EXAMPLE

14.8

Show that 1he line imcgral along - C in Example 14.6 has value 21r
iJS value along C.

+ 164/ 3, the ncgtttivc of

SOLtmON Parametric equations fur - C. with an in~;reasing parameLer alung the curve. are

x =cost.

y = -siot,

z = 2cos t

+ 4,

- rr /2::; t ::; 0.

Using lhe.'e.

fo [(2cos/ + 4) (-sintdt)
-rtJ"l

- sm 1

(cos2 1 + sin2 t

+ 4cos2 t +

16cost

16)( -2 sinl dt)]

= 21(!

(cost

+ 2- 17 sin/

- 4cos2 t sin/ - 16cosl i\il'll)t/ 1

-x/l

4 <."'s' 1
~ 2 ~in t + 21 + 17CQS/ + -3

- Ssin2 1

j"

-:r/l

1(>4

= 21<

+) '

According to this example, when the dire<.tion along a curve is reversed, che

,~a l ue

of a line

i megr:al of fol'm 14.28 alortg the new cmve i ~ the negative: of its \'idue along the origii'Ul l cut\'e.

This is because the signsof tl x. dy, anc.J d z arereve('Sed when the direction along C is reversed.
This is not the case for line intcgrnl 14 .20~

ds does nOt chatgc s i~n when the direction alon8- C

is re,crscd.
Line imegrals o f form 14.28 arc singled out for special corlsider:n iol because they arise in
so marw J>h),Sic:tl problems. For example. suppos.c F rcpre.~cnt~ a fo rce. a nd we consider work
done by this f(}rcc a~ 8J)I:U1icle moves along a curve C front A to 8 . We l~gin by dlviding C
imo n sutx:urvcs of lengths Jls, ~lS .shown in Figure 14.20.

Q'Fit1':T3'IZIEI!ll

1h find M rt done by a fon:c

F alo11g a

cun 'C. lli\'itk d~

C\tn't into i:u b('IJI'\'e$ anc:l appro~inltllC fht work ulon~ s ubatl' ~'tS

...

)'

If F(x , y , :) is continuous along C. 1hcn along any given ll.s1. F(x, y, z) docs no1 vary
greatly (provided, or course, lhnl LJ.s; is small). rr we ap1>roxirnatc F(x , )', :) along ll.S; by
its value F(x,, Vr, U) at the linal poillt (x~. Yl, t r) or tJ.s;, then ;,n ap,m.u i nuniOI\ to the work
done by F along t!.s, is F(x1, y, :,)
y, z1) LJ.s,. An approximacionto chc tobll work
done by }' along C is 1hcrcfore

'r(xi,

"

I:F(.r;. Yl Zl) T (,r1. )';. Zt ) D.s; .


/:J

To obtain the work done by F along C, we take the li mit of this sum as the numbeo of subdivisions
becomes larger and larger and each f:l.s; approaches zero. But 1h is is the det'injtioo of the ljoe

integral of F 'I", and we therefore wr ite

IV =

lc

F Tds =

{ F d r.

lc

( 14.31 )

1002

Ctl:t.pttr 14

I EXAMPLE

V"CCr C1kulu<

14.9

The force of repulsion between two positive point charges~ one of si7.e IJ and the other of si;~.e
unity. has magnitude q f (41f0 r 2 ) , where o is a constant and r is the dislllncc between the
charges. 11le potential V at any point P due tGcharge q is defined as the work required to bring
the unit ch.orgc to P from an infinite distance a long the straight tine joining q and P. Find V.

.Jm... ."J..'E1..rTW

Eltctros.tatic potl1ri<'!1 :lt :l pc.uu P du~ ro


~ chtlfg.t: q li the work to bting, a

SOLI.illOI\ Tr that p..rt of~>C line joining q 3nd P from infinity to P is denoted by C (Figure
14.21 ), then

uuil c-hJt~ fl\ltu inlinily to P

p
-C
q--------r r------ x

~ . -1d.<~
x O

F dr

= - {

Lc:

F . dr

(see equation 14.30). where F. the force nec~ry to o'cn:.ome the electrostatic. repul!iion. i.i
given by

-q .

F=

41r...l

Along-C. tlr

= dxi. and therefore


I' - -

!
r

<

2 1.

V can be evaluated by the (improper) definite integral

-q dr - - { ---q }"" -

4JrE'oxl -

41fE'o-t

If the vector field Fin equation 14.27 has an x-component that is only a func tion of x,
P = P(x). and if the curve C is a portion of' the x-axi> from x = a to x = b. then

fc

= J.bP(x ) d.r.

F dr

Thise<1uation indicates that definite integrals with respect to


along the x xis.

x can be regarded a.< line iruegmls

Heat Engines
In Section 7.4 we showed tha t \\then a gas completes a cycle such as ahat in Figure 14.22. the
work (output) of the gas corresponding to the ~tates represented by C 2 is

J.

W==

v.
Pd V ,

v,

where ~teequation of C 2 is used to express P in tenus of V . Output of the gas during lite retum
cycle along C 1 is negati\~e; i1 is given by integr-.:11 14.32 with limits reversed and P is express ed
in tcm1s of V using the equation of C 1. \V1utl is irnponant is to notice that if integral 14.32 is

interpreted as a line integral in the. V P -plane , then it is evaluated in exactly the same way as
the definite integral: substitute for P in terms of V and evaluate the definite integral. In other
words, we can replace definite inccg,ntl 14.32 with line integral

IV=

fcPdV,

(14.331

where C is the curve in the V P -plane represeoting the succession of states through which the
gas is taken. For the cycle in F igure 14.22. the work done is

IV ==

fc ,+c2

PdV .

(1 4.34)

We discussed the output o r a gas for the Ran kine cycle of a n ideaJized s team engine in Figure
14.23 using area in Section 7.4. From a line i ntegral point of view, the output is line integrl

iiF.1 ~
G.t~ \)Utpul ., th<' lme
intcgr.al o( P tl V tu'\ltu~d lbc C)'C:!C:

M jiUIJ

I'

lb.nldne

p 8(0.0 1, 10' )

~,:---ccom. lo'l

10'

1'1" ; c

5 X I()' C1

10'

D(O.Hl-1. 10')

c,

A\0.01. I()'J

O.Q2 O.Q.I 0.06 O.OS 0.10

14.33. where

c is made up of c.. cl, c,, and c. lf lhe ga.~ is expanded adiabatically a long c,

P a nd V are related by P V Y = k . where y > 0 and k > 0


arcconswms(y = 1.4 for air). Usingpoim C in figure 14.23.k
lo'(0.02)u 4 .2 x 102 .

(tcmpcnuure is held constanl). then

Consequently.

II'= j PdV =

fc

=0+

O.o:!
/001

1
C1

PdV

lOs dV

1
C:

P1/V

/.0.1\H

kV - 1'1dV

1
c,

;,0.01

0.02

= lo' (0.02- 0.01) + k

PdV

1
c.

PdV

10'1dV

O.l()ol

v - O. l }O . IOl

-- 0.4 om

+ 10'(0.01- 0. 104)

= 2.5 X

lo' J.

EXERCISES 14.3

s.

lnl:.xcn:il:!CS 1-10 cY~tl uatc 1hc l ir'IC integral.


1.

I x tl.t + .t 2)'tl)'. where C i$ the curve _... =

'('

x 3 z

= 0 rrom

9.

(- J. - I. 0) IO (2, 8, 0)

2.

f .t 1/x + )'l. dy + .r

"'

tlz. where C is the curve y = x, z

{ X dx +(X+,)') dy. where Cis

fc

10.

lhe CUr\'C

= I + y 2 rrorn

I x 2 dx + y 2 dy + tl d z, ''hcreC is lhccur,c x + )' =


fc

<=

x 2J dJ

+ y'. ~ +

+ <- tlx, where C i:nhccur"c x l+J1 = 1. x + y+ z =


x dt:crea.-;e.:; when y is positive

tlx + x 1 tly oncccloctwisc around the curve lxl + l,\'l =

II. Pind the WQI'k done by the roo:c F =

' ''i + .rl pao'ticlc

1110\'CS rmln ( I , 0) to (6. S) along lhC~traighl hnc joitli ng lhcsc point.\.

(2, I) lO (2. - I)

4.

t
fl 'J

I directed so 1hn1

x2

Irom ( - 1. - 1, I) 10 (2, 2. 4}
J.

l rdx + x dy + z dz. where Cis the<uM z = x'

1c
y = I from ( l. tl.l ) to (- 1. 2.5)

l.x+

I from (-2. 3. 3) t<J (I. 0. 0)

12. Con.:::ider lh.c line

qu!lrtcrcirclc x2

in1c~ral f xytlx + x 1 ti)'.

+ .1'' =

fc

whol!re C is Ulc

9 from ( 3. 0) to (0. 3). Show that for


o( C lhc V21UC: t.\f the line in
lcgrnl i""'""'noc: (a) .r - J co~ 1. >
' - 3sin 1. 0 ~ 1 ~ rr{ 2: (b)
< - ,/<~ - y' ! 0 < ,. < 3

C:tC:h Of (h(: follftWinQ, pll r:J.IIlCirit.l li l'm~

5.

[<y+2t2l )dx.whcn: CisLhecurvcx- )' z. <;;; x 2 l'rom

(4. -2. 16) to (I . I. I)

6.

(0. 1)

13. Evaluate 1hc line inlcgral Lxydx +xdy fro1n (- 5, 3.0) to

(4. 0. 0) :.dong c:ach ..,(the fo ll owinscurvc~: (a) lhC $lraig hL line jo ining

2<IX+ x 1 dy . where C is the semicircle x = j1 - yl from

14. Find the work done by a force F


.\'i + yj on a parliclc as it
moves once counlcrt LOtkwise around lht:: ellipSe b 1x 2 +t1 2y'~ = a 2b 2

ic

ly

--

~'round l.hc c:ur\'C


= I - y 2. y = ,\: + I

.J. .tl), tl.r + ( .t

- y) dy once counterclo<:-k\\'i.sc

bounding the n:aion dcscrit.x:d by the c:u.rvc~ .\'


7.

10

(0, - I)

the points: (b) .r

= - y' . z = 0: (c) 3y = x 1 -

z = o.

16.

z = 0.

* 2g,

In Excrci$<:S 15-19e-.luate the line integral.

Mnd the work done for the Rankine (..'ycJe in the figure bcJow in
v. Vz. VJ. and y.

ICrms or P:.

* IS. 1..!...dx. \\here Cis the curve z = ,II- x'. y = ~

P,

en

from (1 / J'i. l { J'i.I{ J'i) to (- IJ'i, L{ J'i. l/J'i)

16.

P,

+ 2/) dy

(,r '

y 2 = I, t

:;a

lllice clockllise around 1he circle (x - 2)' +

0
ln Exc.rdscs 29-32 Stl up a dcOnitc integral u-, evaluate LliC line intcSJOII. US4.!: Simpsons

17. [ydx -y(x - l )d)+y'zd<. whereCi< lhefirsloctantintor-

section or
IO

x' +

z' = 4 and (.r -

y' +

I)' +

y' = 1 from (2, 0. O)

(0. 0. 2)

181c

f'\IIC

with 10 cquul s ubdivisions to >tpproxirnutc

the detinite inlcgral.

iii*

29.

[x.)'dx + x.\ 1 d.\'.

where Cis the curve

z =

0.

1 /~. from (0. I. 0) 10 (2. 1/3. 0)

1
y ci.t+)ciy + / 1 - x 1 c/z, where C is the WJ'VC

2t 1 ~ I,

z x+

." -

; ,.

30. [xzdx

I from(0. 3. 1)to(l ,9, 2)

+ tan xdy +ex' liz..

where C it:: the

CUr\'C X

y'. z = y' from (I, - 1, - I) to (I, I, I)

* 19.

[ x tlx + xydy + 21/z. where Cis the cu"e .r + 2_v + z

+ 3y + 2z =

4. 4x

20.

= il+

31. 1c,IJ +

y 1 dz+ t)' d y, where C is curve y

sin) I. X = 0. 0 ~ I ~ ,7/2

13 from (2. - 1. 4) to (3. 1. - 1)

i * 32. fc x.rz dy, where C is the c urve x =


E'-alwtcthclincintcg.r...J l
xl 4 clx + \'!crd r+ ~(h..
1(1 - t ')/(1 + t 2). Z =I , - 1 ::; I ::; I
3
c(l+x)

.;l +z'

where C

eot~i~b

of !inc

segment~

joining

su.ccc~sivcly

= C()$3 t , z =

(I - t 1 )/( l

+ t 1 ), y =

the poinb

(0. - I, 1), (I, -1. 1). (I. 0. 1). ond (I . 0. 2).

Jn E xcl'~iSC$ 33-34 cvaluah: the fine i rH~o:gt"dl a long the pohar coortlinat.c

cunc.
21. One end of a spcing (with conslant k ) is fixed nt point D i n the
tisurc bciO'o~>'. 1k other end is lllOve:d o.long lhc: Xzb::i.s: from A 10 8 .
If tile sprins i.s 61~tchcd an amount I ul A, lind lhc wort! done usuins:t
the p<ing.

33

34.

r
d

_._
I

----=-~.._.R_x

1="7 --t

..+:

22. T"'O po5.ith't chargC( q, and 'J~ are placed a1posirion~ (5. 5) aul
(-2, 3), res pc-cti\'cly. in dte xyplane. >.third positive charge q 3 ls
nk>v..:d ;a)Qng the xaxis from X = I to X = - I . Find lhc Wtle'k dQnC
by Lhe elecltostatjc fOI'ces of q 1 and lJl on lJJ
23.- 27. Repeal Exercises 26-30 in Section 7.4 but do so from a line

integral poim ol" view.

1 - cos.O

1
c

y d.a: + X dy. where C is the curve ,. = 0 . 0 :5 9 .:5

;r

35. Supf>OSC a tas flows 1hmugh u rcg.ion 0 or space. At cuch poi nt


P(x, y. z) in 0 and lime 1. the g:r~ ha< velncily (x. y. z. 1). 1r C
is a clol)(:d c,u,rc in D 1hc line integral

r=

i ,..

c/r

is called Ihe d rcuhrtion of tile }1llw ror the curve C. 1f C is the circle
+ y2 r 2 , l l (directed clockwise as vic,vcd (rom the origin),
C<th.:~Jlate f fOt Lhe rollowin~ now VCt.:I.()(S: (a) \ '(.X, ,)',Z)
(.rf +
YJ+ zk}/(.<1 + y2 + z'J'i' ; (b) v(.<. y. z) = - .ri + xj.
x2

lt.

.J. ydx , whc.rcC i.s thcc.:.rdioid r =

Tc

* .36.

Verify the result in equation 14.30.

* 37.

We have shown that given a li ne integral 14.28, it is always possible

to write it uniquely in fonn 14.23. where f = F T. Show that IJte


converse is not true: that is. given f(x , y, z). tJtere does not exist a
unique F(x , y, z) such that F d r = f (x , y, z) ds .

I .J.4

38. The <)"<'loid .< Q R(O- inO), y Q R(J - coo O) i thc curve
ll'"ctd ou1 by a fixed poi1H on the cil'cu,nfctence o f a circle or radius

(b) Ilow much of lhc wo rk in


component or lhe fon:e?

lndei>Cnd:::mxof P.uh

pa~:t

1005

(a) is done by the ''crLicul

R rolling along 1he .t -axis (!>ee Example 9.7 in Section 9.1). Suppose
lhe point is tlC::lccl on by a force of unit rnagnirodc direc.tcd to.,.ard the
c-tnlrc of 11."11; rolling circle..

.19. E..xploin why the line intcgr.d i f(x)d.<

Jllu:;thaYC value zero when f {.t ). g (y) , ;..md lt{z) <Jrc.~:onl.iJ luous fWlc
Lions in some domain oomaining C.

(a) Find the ,..,ork done by the force as the point mo"es from

+ 8(Y)dy + h(;;:) dz

= Oto O = tr.

114.4 Independence of Path


In Sections 14.2 and 14.3 we illustrated that the value of a line integral j oining two p(lints usually
depends on I he cur\'e joining, 1J1e points. In thi s section we show 1hat cenain line integrals have
the same value for all curvesjoi.ning 1he same two points. We fonnaliz.e this idea i.n tltc following
definiLion.

DEFINITION 14.4

A l.ine integral F dr is said to be independent of path in a doma in


of PQilllS A and R in D. the value orlhC line iouegml

D iffor each pair

f F dr
Jc
is the same for all piecewise-smooth paths C in D fro m A to 8 .
The ' 'aluc of such a line integral for given F will then depend only on the c ndp(lints A and
8 . Note that we spe<>k of independence of path only for the special class of line integrals of t he
form J F d r. The questjon we. must now ask is: How do '"'e cletennine. whether a given Li11e
integral is independent of pat h? One answer is contained i n lhe following theorem.

THEOREM 14.3

Suppose that P (x, y, z), Q(.r ,


domain D . The line integral

y, z), a nd R (x, y, z) arc continuous functions in some

F dr =

P dx

+ Q dy + R d~

is independent of path in D if and only if there exists a function <f>(x, y, z) deli ned in D
such that
Vtf> = F = Pi + Qj + Rk.
(14.35}
E.,..;senthlll y. then , a line integral is independenl of path if F is the grddient of :-oome scalar
function.
PROOF

Suppose, firs t, tha t in D there exists a function tj>(x,

Qj + Rfr.. If

C :

= X(l),

= )'(l),

= Z(l) ,

y, z) such that Vtf> = Pi +


C/ ~ I ~

{3

is any smooth curve in D from A to B, then

1
c

F dr =

1
c

Pdx + Qdy + R d z =

/3 ( 84> dx
-ax dt

"'

84> dy
+ -;;-+ 84>
-, dz)
- dt .
Q

y dt

ilz dt

1006

Charxer 14

Voc(()r Cnlc.:ulus

The cen11 in paremh= is !he chain rule forr Ihe derivacive of com1~ile funccit)n t/J[.r(l). y(l).
z(t ) I, and we cnn chore fore wrice lluu

[ F dr =

1~ ~~1ft

= f,P[x(l), y (t ) ,

~(I) I}~

[x(/J) , y(fJ), Z(fJ )) - <Pl.r(~<) , y(cx) . l(<>)J

= 1/i (.rn. )' R Z: n) - </> (xA. YA : A)


(l11c same resull i s oblai ncd even when C is 1>icccwise smooth rachcr chan sm(}()lh.) Because
til is Jas:t exprc:.ssion does not deperKI on the curve C ta ken fro m A to B. it follow:; that the l ine
integml i s independem ofpach in D.
Conversely, suppose now lhmlhc line i oucgral

j F dr = JP dx +
A!TJ IDfD I

is i tdependent or path in D . and A is chos.et as some fixed point in D. I f P(x . y. z) i,') uny
other poin1 i n D (Figure 14 .24), and C is a pioocwi sc-smooch cnrve it\ D from A to P, then
lhe line i ntt!gml

Proof

that indc.l>cndcnce uf p.tth implies


c~i l>ttrK:'C Q( ~ .,tu..:h !lull F e v~

I,

:) . . ___.. . .

<f> (x . y , <:) = [
A

Q(x', y. :)
C'

Q d)' + R d z

t' d r

ddl ncs a sing lc.. vulucd function (x , )' . :z) in D . CiJli.l llu; value of f/J(.t' , y. t ) is the same fo r
all piecewise-sn10oth curves l'rom A to P.
Consider a cuve C composed of cwo parts: a scmight linc 1>0nion C" parallel co chc x .axis
!Tom a fixed poinl Q(x', y, ~) to P(x . y , ;;) . and any other piecewi sesmoolh cune C' in D
from A to Q . n>en

t/> (x,y, t ) =
Now a long C11

)'

a nd

{ F dr + { F dr.

Jc,

lc"

z are bot h co nstanL a nd therefore

(x,y.~)

= { F' dr + (' P(t , .v,;;) dt .

lc

lx

The par1ial derivati\'e of this function with rc; pccl 10 ~ i s

-a,p = -a

iix

iix

1.

F dr + -a

C'

1'

ax ,.

P( t , y ,c)dt ,

but bec~usc Q(x ', y. z) is fixed,


- il ;, f' d r
ax C'
Consequently,

-a,p = -il
il.t

il.t

= 0.

P(r , y . z) dt = P (x . y . z ) .

x'

By choosing other curves wilh s traight-line pott ions parallel to the}' and 2>axes, we can also
show chat 81 y = Q and 81ilz = R. T hus. F = '\1 if>. and this completes the pi'Oof.

This theorem points out Lhat it is ve ry s imple to eval uate a line integral that is independent
of pmh. We state this in the following corollary.

1.&.4

l ndcJ~Kitn~e

ofP.uh

1007

COROLLARY 14.3.1

\Vhen a line i1negral is inde-pe-ndent of path! in a domain D. and A and 8 are points in
D .lhen
[
where

I EXAMPLE

1 4 .10

'fl</J =

F tlr

= </J(XR , YR ~n)- tf>(x-., .VA,~-.).

F, for every piecewise-smooth curve C in D fro m A to 8.

.._..
Evaluate J~ 2J:) d~ + x 2 d y + 2z dz. where C i the first octant intersect ion of x 2
nnd ~ = 2J: + 4 from (0, I, 4) to (I, 0 . 6).
SOLLIION Since 'fl(x'y
path everywhere. and

+ y2 =

+ z') = 2xyi + x'i + 2zk. t he line integral is independent of

(1.0.6)

x y

+ z2

36 - 16

= 20 .

10.1 .4)

The fo1Jowi11g coroll ary is also an inuncdiate consequence o f Theorem 14.3.

COROLLARY 14.3 .2
The line integral F d r i.s independent of path in a domain D if and only if

iF 0
dr =

I -136

for every closed pmh in D.

Theorem 14.3 sUlles that a necessary and suflicient conditi<>n for lii'W! in1egrnl I-'.28 to be
independent of path is the e-xiste nce o f u function 4J (x , y, z) such thnt '\14J = f'. For most
problems it is obvious whether such a fu nction c/> (x . y. z) exists; but for a few. it is n01. Since
much time could be wasted searching for ,P(x, )', z) (when in fact it docs not exist), it wou ld
be helpful 10 have a leSt !hat states n priori whether tf>(x, y , z) does indeed exist. Such a test is
''Ontaincd in 11teorc m 14.1. It states !hat F is the gradient o f some scalar function </J (x, y, z)
if 'V x F = 0 . When ~tis result is combinc<d with Theorem 14.3. we o btain this importMl
theorem.

THEOREM 14. 4

(Xj '.

Let D be a domain in which P


<:) , Q(x, y .:). and R (x. y. <:) hmc ~'Onti nuous
fi rst derivmhes. If the line integral t d r =
P dx + Q dy + R d z is independent of
palh in D, then V x F = 0 in D. Conversely. if Dis simply connected . and 'fl x F = 0
in D , then rhe line integr;~l is in<lependenl o f pHth in D .

We have io Theorem 14.4 a simple test LO del.ermjne whe1her a g:iveo ljne integral is in
dependent of path: We see whethe r the curl o fF is zero. Eval uation of a line integml that is
independent o f path still requires the function c/>(x, y, <:) , but it is at least nice to know that <f>
exists before searching for it.

1008

Chup:c:r I-I

VeccorC.h:ulus

11lcorcms 14.1. 14.3, and 14.4 have ident.ilied an important equivalence, at lcru;t in simply
connected domains:
L

f ~, (/ r

is independent Ofp<tth in D.

2. F

V for :some function (x, y. z) definetl in D.

F= 0

3.

D.

iJI

Theorem 14.1 states that (2) and (3) arc equi valent: Theorem 14.3 verifies the equivalence of
(I) and (2): and these two imply the equivalence of (I) and (3} (llleorcm 14.4).
For line integrals in the X,Y-plane. this equivalence i~ still l'alid except that V x F = 0 can
be stated more simply as
i)Q
i)P

-=

ax

( 14.371

ay

(sec equation 14.16).

I EXAMPLE

14. 11

E'alume j~ 2xye: dx + (x 2 e' + y) dy + (x 2ye' - z ) d~ along the straight line C from


(0. I. 2) to (2. I. -8) .
SOLUTION

C :
I(

Parnmetric equations for tl1e straight line arc

Method 1

= 21.

.~

)' = I,

z=

2 - JOt,

0 :::; I < I.

I is the value of the line integral, then


1

I=

fo {2(2t)(l)e - ""(2dt) + [(2t) e'-'"' + l j(O)


2

+ [(2/) l(l)el-IIH -

se'{~e-'0 }

- 30+

= -165 e_
.:vtethod 2

+ 101](-

IOdt)}

~e2 { (J - JOt)"- '(>' }'+ 2e' 1'-w,-'Otdt


- 10

(9 _ + -

- 30 + -4 e2 - e
5

10

10

' )

10

25

+ -2e

25

-lt }' = 4e- -

30.

It is evident that

and hence the l ine integral is independent of path. Its value is therefore

I = { x 2 ye'

y'
2

,2 } (2.1.-8)

+ - - :::....
2

(0.1.2)

4e- 8

1
)
- 32 2

+-

~
"

- 2

= 4e - 8 - 30.

14.4

Method 3

1009

Since

1
=

lndependeru..-e ofParh

lc

a;ay

a;ax

iJ/8z
y x ye= - z

+
x 1 e')i + (2xye'- 2xye')j + (2xe'
1

x1 e'

2xye'

= (x 1e' -

the Jine i ntegraJ is independent of

p<~tb. Thus, <here

- 2xe' )k = 0,

ex.ists a fuoctioo (x, y. z) such that

or

iltj>

iJx = 2xye- .

iJ

-iJ

)'

+ y.

= x e-

Integration o f the first of these equations yields

</>(x, y, z) = x 2 ye'

K (y, ;;).

Substiwtion of 1his function info the left side of the second equation gives

x 2 e

+ -aK
ay
y2

Consequently, K (y, z) = 2

aK

= .ce< + y
:t

ay

= y.

+ L(z), and we know both the x - and )'-dependence of:


y2

+ - + L(z).

</>(x, y, ~) = x 2 ye'

To obtain the z-depcndcocc contained in L (:) , we sul:>stitute into the left side of the third
equation,

x 1 ye-

Hence, L(Z}

dl

+-

dz

= x 2 ye- - z

= - z2 /2 + C , where C

dL

dz

-z.

is a constant, and

</>(x ,y, z) = x 2 ye
.

\12

_2

'
+ :.....
- -<2 + C.
2

Finally. dtcn. we have

I =

x 2 re-
.

.2

,z }(2, 1. -S)

+ -Y - --

= 4e - s - 30.

(0,1 .2)

Method J isone-of "bl'ute-force.: The function x 2 ye' + //2 - z 2/2 in method 2 was obtained
by observation. Method 3 is the systematic procedure suggested in Section 14.1 for tinding the
function tj>(x, y, z) .

..-..

1010

Cb.,pter 14 Veeror C.lculus

I EXAMPLE 14.12
Evaluate

l =

fc C3 .~)~y2)dx + e2r~;T3)dy +2z2dz,

where C is rhe curve y = x 2 , z = x - I from (I, I, 0)

10

(2, 4, 1).

it is evidemlhal F = \1 </> if

cfJ(x, y , z) =

2z 3

-y + -x 2 + -,
3

and this is valid in any domain thar does not contain poims on rhe y zplane (x
(y = 0) . Since C does nor pa,;.~ rhrough eirher of rhese pltnes, rhen

r = { :: +
)'

I EXAMPLE

Y2
J

= 0) or x zplane

2zl }(H.r) = (~ + 1+ ~) - (I + 1) = 3

( I.I.U)

14. 13

Tn lhemtodynamics rhe stale of a gas is described by four variables - pressure P, absolute


temperature T , internal energy U . and volume V . These variables are related by two equa1ions
of state,

and

F(P , T , U, V) = 0

G(P, T , U, V) = 0,

so lhat rwo of rhc variables ttrc indcpendcnr and rwo arc dependent If U and V arc chosen as
independent variables, then T = T(U, V) and P = P(U , V ) . An cxpe.rirncntal law called
the secund lmv u.f theruuxi)'IWmics states llmt the line integral

- dU
cT

+-

dV

is independe.nt of path in the U V -plane-. The first tenn, dU fT. is an incremental change in

intemal energy of the gas per degree of absolute temperature. The second rcrm, PdV /T, is an
increment of work dtme by !he gas per degree of absolute temperalUre. The line inregral gives
the total change o f these as the gas is taken from one state to another; it is independent of the
slates (U, V) through whic.h the gas is taken, depenc.ling only on initial and final value.'J or U
and V. Show thut the second law can be expressed in the diffe rential tonn

SOLUTION
if

According to equarion 14.37, the line integral is independent of palh if and only

or

aP

ar

T-P IlU
au
0 =

T2

tht is,

ar .._ _ar
p_

li P

0 = T- -

au

au av

and the proof is eflmplctc.

Since the line integral nbo,c is independem of path, there ex ists u function S(U > V) such
that

iJS

au

as

r av

T'

and the value of the line i ntegral L< given by

I
- dU
[ cT

+ -dV
T

= S(B)- S(A),

where C j oins A an~ 8 . Thi:; function. called mrroi'Y pl ays a key role i n thcrmO<Iyn~mi-,.
The dirferemial t1 S of S(U. V) enn be expressed in the fon11

dS
n,c equation T dS

as
iiS
= -dU
+
- dl'
au
av

= -dU

+ -dV
r .

= d U + P dV is callo~ thc.fiw o,(rhe Gibbs equmions; it is the basis for

111ueh of the wori< i n t hcrmodynnmics.

E X ERC IS ES 1 4 . 4

In Exercises 1- 10 show

~tl

the line intcgrul is indcpcndcnLof 1xuh.

I. [xy 2 Jx

(0, 0. 0)

lO

z=

+ x2 y dy, where C is lhe cur\'C ." = .r 2, <: = 0 from

9.

<'
10 ( - 3. 2, 4)

,.

z =0 f roon ( I. 0, 0) to (-

lc

.r' + y 2 + z2 =
5.

3, .1

=x

6.

1=
1

- dx - 1 dy

fey cosx d x +

y 1 - 2x+4y=7,

'
fmm
(2.0. 2tr)

lO

z=O

+ ~2.

X+

1. 2,5)

+ d t., w hen:: C

i!O ihc curve)'

= x2 + I, x +

rl\'

1) 3 dx + J..\3 y'l dy, whcu.: cis lhc t.urvc )' = (' 1 rrom

11. Show lhal ir f (x). g(...). mlt.liJ(l ) have concinttous firsl dcrivaLi"es. then the line inlc:gml

[ r<.<l '' + s<.r> d.r + h(zl ,~z


is indcpcndcnt ofpaOt

(2. 0, 4;r)

sinx d y om:c clockwise around the circle x

:= .tz

c)'
)' "
Z
2 from (0 . I , I) IO (3. 10. -II )

r
1

- :... sinx d.f + -cos.\' dy- '1' cos x lit . where Cis 1hc he
1

= I . X+

lc
(O. I)tu( l ,t)

(x'r - 4?.) d z, where C is the """'t

2 (/;,, \\<1'1Cro C IS thccUr\'C X+ J

+ XJ)' + ?, d (. . vthC.rC C iSlhC(Un'C t

from (- 1, - I , I) to ( 1,1 , - I)

t
'
lix .Jii cc 2cos
/ , J = 2 smr.
t

I from ( - 2. ~. J) 10 ( I , 0, 0)
)' JX

10.

JI'Xi.

I. 0. 0)

{ 3x y z d.r + x'z d)' +

+ }'2 (f_'V +

lc
_,. =I from ( 1,0, I) to ( -

( I, I, 0)

J . 1 2xe" <ix I (x 2e'' + 3) dy. whcrcC i<thc curvey =

(/;t

8.

2.1 (3x' + y )dx + .rd,y, whcrcCisthcS!mightlincfrom (l,l , 5)

4.

[ .tz

7.

and cvalua1c it.

*
2

12. If V

F = ()in a do.naio D LhaLis noLsilllJ)Iy connected. can

you conclude 1htu 1he Li ne integral

in D? Explain.

fc F J r is no1indcpe.nclcnl of pt11h

ln Exercises 13-l ScvaluoLc the l ine inlcgnt.l.

13.

fc zye-''' d x +

(.X

eX>' dy + (e.t' - I) dz.. where

= x', z = x' from (I, I, I) 10 (2, 4, 8)


* 14. J. y (tan x + x sec2 x )tlx + x tanx d y + d z
Tc
clrclcx2 + y 2 = I, .t = 0

c ls the CUf\'C

>t<

yl)

15. { ( I +

lc

x3

dx - (-" + X

.\' )

_r2

dy

+ zdz..

once around lhe

*'

16.

udx - xz dr

l.

i
1
r
c

+ .nd<

, whc.re C

is the. c.urve x 1

17.

+y

+ y1 > 01

y > 0 , (d) y < 0, o.- (c) x'

23. The second law of themlodynamics stateS- that the line integr.-rl

r- ' (dU + P d I' ) is iodcpcndcnl of patb in lbc U V -plane

(sec Example 14.13).


(a) Tl.c c.quatiOJlS of state foe. an ideal gas arc. p V =
nRT. (/ = f( T ) . where 11 and R arc t~nsLanlS and
f(T) is some given function. Because of Lhe..se. iL is more
oon\'enient to choose 1 and \1 as independent vnriabJes
and to cxpccss P and U in tc.rms of '/" and V. If this is

where C is the

+ z! =

l. y+ z =2

)'
1
C
(c)

I =

broken line joining successively ( I, 0. 0), (25. 2, 3). and (5. 2, I)


.t.

lndcpcnd<:JU of pulh i..n the domain <.-on:sisti.ng of -~Y<>spacc wilh 1.l1c


origio rcmo\etl?
22. 1n whicb of lhc foJlowi.ng domains is !.he. line- intcgrol
d .t - . dy
,
,
indcpcnd<:m of path: (a) x > 0. (b) x < 0.

done. show lhat

I
-~Tan- 1

c x-

ytl.x +

.t

., dy) whcr.: C is lhc curve x


+ xr

wherek

y' +I from (2, - I) 10 (10, 3)


18.

dx +

c (x - 3)l(y + 5)
where C is lhc c ur\'e x = y

(b) Since the line inLtg.ral is i ndcpc:ndc~t of path, there- c:<i:SL'> o


fu nclion S(T._ V), c.aUcc.l c11rrop..r.wt:h that

I
I
d v + - - dz .
(x - 3){y + 5)'
z+4

= z from (0, 0 , 0)

10 (2, 2, 2)

- d1

+ 19. Eva1uale

1
+

-\'tlx+xdv

+ Y2

(a) once cou.ntctclockwlSC around

tlx dy . where C consiJ>Isor tine


2
1
cx+y
x2+ y1
scgnlcJUs joi11ing succcsslvely the points (1. 0), ( I, l)) ( - I, I ), and
(- 1. 0).

* 21.

Evaluate

11 R
-

dV

= S( B) - S(A ),
=

+' 24. Evn.lu..::tte the line i n1egrol

lsthe ti neintegral

**
indeperldcnLor palh in the domain consisting of lhe xy-plane wilh the
orig.i.o rcmo\'cd'? Is the line i.ntcgrul

where C i-s any c-ur\'t: joining: poiot.s A ~.nd 8. In 1bc c.u~


thai k isoons1an~ show tl>al S kIn T + 11R In V + So .
wh<'<e .1;, is a o<>nstrun.

the citc:le x 2 y 2 = 1. z 0 a.nd (b) ooce counterclockwise around


the cirelc (x - 2) 1 + y 1 =I, z = 0.

* 20.

= dUfdT.

(2.\'.\'e:c

.fc

2
J

+ x 2.\) dx + x 2tf lY dy

once clockwise around Lheellipsc xl + 4y! = 4 , z 0 .


25. A spring has one: end fixed at poim P. The other end is moved
along lhe curve)'= f(x) from (.ro. f(xo)) lo (x, f(x, )). lf initi:tl
and final stretches in Lhe spring ~tre a and b ( b > tl ), what wotk is
done against the spring?
26. Electrostatic forces due to point charges and gravitotional forces
due to poim ntas..~s arc examples o( lnvc:(SC squw.c force fields rorce lldds. of the forrn 14' = k r/lrl 2 where k is: a tonstanl and r

xi + yj + , k.

{ r=;=;'=-'.::;;=;::::~ dx +
-"
dy + r=;=;=''~;=, d z
JcJx'+y'+z'
Jx'+ .v' +z'
,lx'+.v'+z'

(a) Is the llnc intc::grot representing work done by such a force

field independenl of path?


(b) What is lhc wotk done b) F in moving a panicle from
(X 1. J l Zt) IO (x, . .\'2 Z2 )?

114.5 Energy and Conservative Force Fields


Tn Section 7.5 we introduced the idea u f ptlle nlial e nergy associated wilh force.~ a nd t he law of
conservation of energy. \Ve now complete this discussion by showing to which forces can be
au ributcd pot<mtial energy.

DEFINITION 1 4.5
A force field F (x , y, z). dell ned in a domain D of space. is said to be conservative in D
if the line in1egml F d r is independe m of palh in D.

Since fc Fdr can be interpreteda.5the wo rk done by r along C. a force field is conservative


if worl< that it doc~ is independent of pmh taken from one poimto another. According to the
results of Section 14.4, we can also smtc that u force field F is conservative if and only if there
exi~ts a function c/J{.t, )'.;:)such 1ha1 ~' = 'VrjJ.
h is customary to associate a potential energy function U (x. y. z) with a conservative fonce
field F. This function assigt~s a potential energy to ea~h point (x, .~. z) in such a way that if a
particle moves from point A to point 8 , then the difference in potential energy U (A) - U (B)
is the work done by F; thai is, if C is any cunc joining A and 8 , then

U(A) - U(B)

fc

F dr.

( 1~.38)

If J F d r > O, thcnpotcntialcncrgyat A isgrcatcrthanpotential cncrgyat B;if J F dr < 0,


then potential energy at 8 i$ grealertha.n that at A . To find U (x, )', ~). we use the fact that
because F is eonservmivc.thcrecxists a funetjon t/>(x, )',~)such that F = 'ilt/>. and
F d r = rf> ( B ) - </>(.4 ).

If follows, then. that U (X, )'. Z) IIIUSt SUlbfy the equation

==>

U(A) - U(8) = 1/1( 8 ) - </!(A)


Since A and 8 are arbitrary po ints

U(A) + (A) = U(8) + (8).

D, this last equation states that the value. of the function

i 11

U (x, y, z) + (x, y, z) is the same at every point in the force field,


U(x, y. z)

+ </>(x . y . z) =

C.

where Cis a constan1. Thus,

U(x, y, z) = - </>(.r , y, z) + C.

(!4 .39)

Equation 14.39 shows that the force field .F defines a potential energy function U (x, )', :)
up to 11t1 additi\'C constant. (Thi~ seems reasonable in that itself is defined only to an addili\'c
constant.) Because U = - + C and f = '\?,we can. alw regard U as being defined by the
equation
F' = - \lU.
(14.-W)
The advantage of this equation is that it defines U directly, not through the function</>.
r'Or a conservative force field F. then. we <lcfi nc a J)(l(clllial energy function U (x, y. t) by
equation 14.40. If a particle moves from A to 8 , then the work done by I' is
W

= U(A)- U(B):

(14 All

in other words. wotk done by a collsct-vmive force field is equal to loss in potential energy.
On the other hand . if a particle movt' under the action of a force F (and only F). be it
conservati\'e or nooconservative, then it does so accocdi11g to Newton's sec.ond law.

dv

= -di (mv) = m-dI ,

where m is the mass of the particle (;Jsstnned constant). v is its velocity, and 1 is time. The
action ofF produces motion along some cune C, and the work done by F:' along this curve from
A to 8 is
IV =

( F dr =

(~ m d'

. d r dt =
d1

1c

1"

dl

Hmv.{

Hmlvl

2
}: .

[ fl m

1.

dv
vdi = 1[.fl :!.._
(~mv v) dt
dt
dt 2

1014

Cll~lp:er

14

VOCI<K CalcUhl~

Thus, if K (x, y, z) = 4mlvl2 represents kinelic energy oflhe particle, lhe work done by F is
equal to the gain in kinetic e.ncrgy of the panide,
IV = K ( B) - K(A)

(14A2)

(and this is true for any tbrce F as long as F is the lotal resu ltant force producing motion).
ff the (Otal force producing ~n otion is a conservati ve force field F . we h8ve 1wo e.xprcS)ions.
14.4 1 and 14.42, for the work done as a particle moves from one pou11 10 anolhcr under 1he
ac1ion of F. If \\te equa1e I hem, we have
U(A) - U(B) -

K(B)- K(A)

or
U(A)

+ K(A)

= U(B)

+ K(B) .

04.43)

We have showo then that if a particle moves under the action of a conservative force fic.ld
only, che sum of the kinetic and potential energies at B must be the same as the sum of the kinetic
and potemial energies at A. In other words. if E is the total energy of the pan icle. kinetic plus
potential, then
(14.44 )
(A) = (8) .

Since 8 cnn be any point along the path of the particle, it follows thai when a particle moves
under the acdon of a conservative force field. and only <1 conservarive force field, Lhen :u every
point along irs trajectory

E = a constant.

(14..15)

1l1is is the law of conse~ation of energy for a conserv-dtive force field.

I EXAMPLE

14. 14

Show that the clectrosuuic force.due to a point charge is oonscrvntivc, and determine a potemial
energy funcrion for I he Aeld.

IDtCJ!LI3,iWfW

SOLliT!ON The electrostatic force on a charge Q due to ;\ charge q is

Ble\:uustaiJc

force due ro <1 point chqe i.s

cl)f1Ger
4

"ath'e

where r is the \'ector from q to Q. If we choose a Cartesian coordinate system with q at the
origin (Figure 14.25), then

, qQ

F =
.r

qQ r
41J~ol r l 3 '

41rfo(x-

'

+ YJ' + zk).

, (xt

+ r + z1)' 1-

Since

'i1 [

41u o(x Z

-q Q

+ yZ + z Z) 1/2

] - F
-

'

the force field is conservative. and possible potential energy functions are
U(x,y,z) =
where r = lr l. In electrostatics it is custom;uy to choose U (x, y, z) so that lim,_oo U = 0,
in which case C = 0 , and

qQ

U(x.y,z) = - 4~r <or

In addition, if V b defoncd as the potemial ene'ID per unittt&t char~e

V(.r.y.:)

Q. then

= Q =

This rcsuh agree.<with lhul in Example 14.9.

EXERCI SI!S 1 4 . 5

(b) If U (.r) = h 1/2 (aH;ould be t~ case lo<a m.-m on th<


eod of a sp<ing). usc the =II rl r>n to) to find x as a
function or t . Is 1t \\llat you -.auld ttp.:..'-'1"' Simplif) the

In txcrc1....:... I 5 dclcrminc whether the force held IS COO'(.Crv:uivc.


Identify each cooscrv.uive Inn:< fiCid.llnd find o potcnlool energy luoc:IIOn.

'(

.t,)'.t

q q,

solution wh.e., the inilinl \Ch-~1) oft.hc: n~ is 2C'l'U.

.ti + yj + tk

, + .1.. + z)
,. ,.
,.

4JrE, (.1

2. F (.1. )') = m.ri + ...-_


,.j; m is a oonslanc

3. FC.t) = - .t.d: k iuHonslcuu


4. F (.t. )'. l)

F(x , y, z)

=- mx k;m and g 4trc oonstams

= f (./.<'

)'1

+ r')r.

r - xi+ ...J + ;:k,

fo r some fu.nctiOfl f . We oflcn write in ~uch u cu.sc that

,,l + yj + tk

'

12. A force fidd P (.x. y. d 15 said lo he tWiult)' J)'ntrnd.ric about the


origin if it can be wrium in the fornl

5. F (.t. y. t ) - GIll m (, 2 + .1'2 + r.')'fl : G . M . and m "''"""'


stants
6. ~ th;it F (.r. ), ;:) i~ o1 conser''(lli\c force field in MHJIC ~
m.un D. a.'lJ
v. Z) iJ> a polcnlial energy (unchon tKC(I(k~JcJ Wllh

uc.- ..

(a) l:sc TI1C0rm 14.4 to ohow tl\JI such o force os COOS..'rVilti~Je in suittab1) defir.cd dolrun (ptovitkd Lhtl f (1 ) tm a

1hetJ<.'CS U(.l, _r.:) = c."""'"' c C\""'ant. arc c.&ll.lol


cqmpot~lthtll nwfnaJ 1'hrourh each poin1 P in D lh:rc ic C'rc .lnd
Mly Me '\lll.:h c~uipoc.eruial su.rfocc for f Show' thl.l at P the rorcr ..~
is ncwmalao tilt cq~oupoa.:neilal .,;;url'acc th~h P

7. l>tn\' cqu.po1Cnlll!l '-urf:ttt~ for the forces. in

Exetcl)ot

cootinuouh f1nt dc:ri,.JII\cl


(~)

lc

y, .. xpl.1111 why friction is not conscr\':.tlivc.

"here the lim1 1~ o und h ul\! the dtt~lanccs fron1 the origin
to A and B .

(u) When lH~1dcn 1.s in Univcn;icyltll'ld leave their houses, o I'!U


pcrnatuntl JXlwer aUrncls 1hcm 10 lhc u ni ver~ily in ~uc h u
'-"''*>' that th!! m:.gnitudc or the rorcc ac ony pouu i~ inver~cly
proportional to Lhc square of the: distance: l'rmn the univcr
3<11Y rhh klNC act~ until they are 10() Ill rnml II"-' unl\f.:I'SII)'

(C:)

it dis.appe:ars. Is this force cor.ser,all,c'J


(b) I( ...omc..C'ut<: clncrt< Ihe power '"0 eJullhc: flwcc: Ull.n.:l"> !Ciltl-

t\:na, 10 lhe foeti donal shop. is this (on;ccootitf'V;Uivcl


II .

(a) A p311ktc wilh ma.u. m tnO\~S alMg tbc x axis wlder the
onftliCn<cnf a COClser\'atrce fcrcc field with potential U (X).
If the panicle is 31 position x 0 at time t = 0. and E 1to itto
tocal cnc:rgy. show lhat

dx
n ;==TIT."dx =

"' JE

U(x)

!2.,,
y;,

113\C "C discussed 1.\1\)' tadiOII)' S)'ll\lnt.tric rOI'Ce f'iekts i.n


lhi.' Chll.plC't?

~lftJ then

a<cdiC polllb in dlC li:urtbci<N jo.ncdb) !he

r.r 'dr = f .,/Ji j'(r)dl'.

On~t tndnfn ....,..;ns wilh un:o;arctchcd length l. is fi~cd otlheongin in


spocc lrthc other end h m point (.r .''-l) (all coordillalt\ n ti\CU~).
wha1 ~ d'IC' ron.'C CAC:IteJ. b)' the !iJ)ring'? I~ th i~ ICwcc (C}.n~rvalivc I

10.

C show thJl

I, A, ud 5.

8.

II A and
(IUYC:

c
.t
y

I14.6 Green 's Theorem


Une m1egrnl< on lhc xypl~ne (II'C of 1he fonn fr f(x, .v) ds. and '" the lopCCial case that
f(.t.)) is 1hc t.~ngcnti~l component of some vector field F(.l. y ) = 1'(1, y)i + Q(x. y)J
akJn& C. 1hey 1.1ke 1he tonn

kt'dr= k

(1~ .16)

+ Q(.r,y) <f.v.

P(x,y)dx

We now <how 1h01 when C is a cl()o.C(I curve, line imcgml 14.46 Cilll u:.uully be replaced b) a
double imcgml The precise rcsu ll i~ contained iol the following theorem.

THEOREM 1 4 . 6

(Gree n ' Theo rem)

U l C be a piecewiSt>moc>lh, closed cune in lhe yplane 111111 dot< nol inlersect iaself
and 1hat enclose< n re.;ion R (Figure 14.26). I f P( 1 , y) and Q( t, y) have continuotL<
firsl paninl dcrivtlli'e' in a domain D con1aining C anti R , 1hen

rcJ.
Greens
thC\'II'Cnl rt p l.acr<~ .l hne Integral
!ltl;lllfWI II c hl1cd <.Ut\'C

\\'ith

Pdx

+ Qdy

=fl.

( 14A7l

(ilQ- ilP)dA.

ay

OX

PROOF Firsl coo1>idcr a imple rcgioo1 R for which every line parnlltiiO lhc X and .vnxes
1hat iouco-secls C tlOC> so in at 11100.1 1wo 1minos (Figure 14.26). Then C ''~'" be >Ubtliviucd into

an upper ~Hld H lower pun,

\loubk: llct.naJ U\1!1' d~ intliur

cl : )' = h(x )

of t h.e <..\lr\'e

If we coo>sitler Ihe second l.crm onlhe rig,hl of cqua1ion

fl. -a~a.,

)' =

Co :

and

I ~A7 . we

d A = l.bl.hC<I - aP dy dx = l.b

.c..

a>

g(x) .

ha,e

1-

f' lhVI d:x

"'

= J.b/P/x . g(x)l- P(.t , h(,t ] Jid.t.


On the other il<>nd, lhc first lctm on the ldt of 14.47 b

, ,. tf., =
j (.

tllld

r,

fc,

dx

if we: U"'C ,'( {, , .. I}(II'Uillt:l c r ~llullg

Jl' d.r

ic

r "dx = fc,r ,. d., -1-r, ,

fc,

d ...

C1 tllld -C2, t hell

=f."
P [x.g (,r )] tf,v- { "
"
Jo
= J./P(.c.s[.r)J-

P [.l , ir(A) (d.\

P[x,h(x ) J}d.v.

We M\ e ~hown therefore Ihat

t/d.t = JL -~;t!A.
B) subdividing C in1o 1wo p.ms ol'lhc type x
we can also show I hut

t cQtfy =

=g(y) and .t = lr(y) (where g(y) ~ h(y)],

fl ~~

dA .

Addi tion of these r"C>ults gives Geen s theorem for this C and R .

1 ~.6

Ciretn '~ Theorem

1017

M4Ctlil;l
~
Thi~ regioll can be di\iQed iJltO regions
wtlel'e tlu.~ <.'()nclilio.n i' that ~try line pat3Uel h> the ..- . and Y
lL'..:c:s th.tl. inlcf')ccts ib boundaty tlocs so at moM l"""

Now consider 1\ more general region R such as thm in Figure 14.27: it can be decomposed
imo 11 subregions R; , each of which satisfies the condition that I ines pa1<11lelco the ~oordinme
axes intersect its boundary in at most two poiots. For each subregion Rt, Green s theorem gives

1 l'dx + Qdy =

rc,

j'(}R, ( aQaP)dA.
OX
(Jy

If these re.<uIL< are atkled. we get

L" f
i=l

" J{. (aQ-. --.aP), dA.

Pdx + Qdy= L;

;. J

Ci

O.t

N;

Q)

Now R iscomposed <>f ll>e>ubregions R;: thu> theriglu side of this equation is the double integral
over R. Figure 14.27 illustrate.'\ that when line integrals overthe Ct arc added, conrriburions from
ancillary (interior) curve. cancel in J)<ti ~. lea~>'ing the line imcgral around C. This completes
the proof.

\ Ve omi1a proor ror even more general regions thfH cannot be. divided into a fini Le number
of these subregions. TilC interested reader should consult more advanced books.

I EXAMPLE
the<n:m

appli~

14. 1 5

(irtcns
to a lriansJc

Ev~luate

the line integral of Example 14.7.

SOTtmON

)'

By Green's theorem (see Figure 14.28), we ha,e

( I. I )

y=x

c
R

[' {
lo
x y - )~2 } "

dx

[' 2
lo
x dx

1018

Ch;tpltr 14

V~torClli.:UIU.~

I EXAMPLE 1 4 .16
Show that the area of a region R is defined by each of the line integrals

1 xdy =1 - ydx =

J'c

J'c

~1

x dy - ytlx,

J'c

where C is the boundary of R.


SOLUTION

B y Green's thoorem, we have

JL
!L

t / dy =

LdA =area of R

and

fc -y dx =

LdA = arca ofR.

The-lhird expression for area is lhe a\'eroge of these two equalions.

The a.rca fonnulas in this example arc o f panic-ular value whco the curve-C is defined parametrically (sec Excn:iscs 15-1 \1). We also usc it in our next project

We al l know tha t the area of any plane polygon can be found by tllvicling the polygon into

rectangles a11d trjaogles. \ Ve are being asked whether we. can provjde a faste-r way to fiod
Suf.:h area~.
SOLt:TION

Consit.ler finding the area inside the polygon in Fig1.1re 14.29a. It has

tl

vertices labelled P1(x;, )';). According to Example 14.16, the area is one-half the line
integral of x dy - y dx around the edge C of the polygon. Suppose we denote the
individual edges of the polygon by C, so that C = C, + - + Cn. Then d1e area of the
polygon is

If,cx dy - ydx=zL
I "1_x dy - ydx.

A=z

l =l

Evaluation oft he line imegral along each of the C1 will be similar. If we evaluate the line
integral along C 1, we will see how to evaluate the line integral along each of the C 1 With
parJmetric equations X = x, + ( x2 - Xt)f. y = Yl + (Yl - Y t)l ' 0 ~ I ~ I , for c"
we find

1
1

lc,xdy- y dx =

[.1,

+ (x2 -

- [y,
=

1'

+ (}'2 -

x .)t]()'2- y 1) dt

y1)r)(x2

(Xt )'2 - Xl)'J) di

x 1) dr

X1Y2 -

With similar resuhs aloog the other line segmeots. we wriLe that

-'2YI

14.6

We can lake advanl<lge of the fuct that each tcm1 in

p~~rcnchescs

GttCJl"$ niCvrem

1019

has the form of a 2 x 2

determinant Suppose we ammgc lhc vertices in a vertical column a.s shown in Figure
14.29b. and ta ke
I

2[(sum of downward products to right)

- (sum of downward products to left)).

We obain

I
2 [(XtY!

+ -'2YJ ..- X.1)'4 + + x,y,)- (X2)'1 +

+ x,,y3 + + Xt.Vn)J

X3Y2

I
= 21(x,yz.t2y,) + (x2)'J - .r,n) + + (x.,y1 -

x 1y.)].

This is A . Here then is what we have d iS<COvcrcd. To find the 11rca of a poly&on. hlhc l
its \'I:Ri<-.:s (xt . .Vt) . . (.T y,) consecutively. proceeding coumerclockwisc around
the polygon. Arrange the venices in a column repeming the firs t vertex. The area of
the polygon is then one-hal f the sum of downward products to the right less the sum of
downward products to the left.
[!]
F.ut w.sy

FIGURE 14,28b
k) ~..-alculute ;lt~ in.,~

a poay,~ul

::~o~~

. .

.fJ X

Yt

Gree n"s theorem is applitllblc to line imcgruls of type 14.46 only when c urve C is closal.
Ho,vever. when C is not closed. it can be made so with t11e addition of another curve. In this
wdy. Greens theorem may be brought to bear even on line integrals when C is not closed. The
following example is an illu,tration.

I EXAMPLE

14 . 17

Use Gree11's theorem to evaluate the tine integral

[
where C i$the p~~rabola y

(x - y") d x

= .T 2 from

M4iiiiIAI +t.M (irL-cn~


tht"~rcm <.lpplicd lu .1 parabola

(-I. l ) ..........---11'-- ,- - , ( I. l )

)' =x2

c
X

+ y" sin y2 d y,

(I, I) to (- t , I).

1020

Ch>pter 14 VectorCol,ulus

If to C we add the ~n-aight-li ne ~gmem C' from (-1 , I) 10 (I, I) (Figure


14.30), thcn the combined curve C + C' is closed. Green's theorem then gives

SOU TIO\:

-j'fJR 10- (-3y )]dA = -1'1'


2

3y dy dx

- 1 x2

- - j''jij ,dx = -1'( l-x )dx


1

- 1

- 1

.t

J2

7
But, because

(x - yl) d x

+ /~in / dy

C+C'

{ (.r - yl) dx

fc

+ /sin y2 dy + {

fc

(x- y 3 )dx

+ / sin y 2 dy,

we can wrltc thnt

{ (x - / ) dx

fc

+ y' sin l

dy =

_!..: - {

f c

(x - y3 ) th +/si n y 2 d y

, _!2 _1 (x - l) dx ,_!2_ {x -x }
7

- 1

- 1

2
7'

Green s theorem cannot be used 10 e.valuate a !line imegrdl around a closed curve that contains a
pOint at which either P(x. y) or Q(x, y) fai ls to have continuous fir~t panial derivat ives. For
example, it cannot be used to evaluate the line integral in Exercise 19(a) of Section 14.4 because
11eithcr P nor Q is defined ar the orisin. Try it and compare the rcsufl to the correct answer
(2;r). A generalization of Gt'Ccn s theorem th.m can be useful in such situations is discussed in

Exercise 30.
EXERCISES 14.6

Ln Exercises 1- 11 usc Green's thcorc1n (if poosiblc) tocvn.luatc Lhe line


intcsral.
t.

l
3.

(x - 4)' + (y- 1) 2 = 2

J. y: d.t + .r: dy, whcrcCisthccit'('le x ~+y 2 =

Tc
,

Tc

8. f c (x 3 + y 3) d.< + (x 3 -

f("x:~ dx + (x + y) dy, '"'h~re

C is the squ!lre with

\'ertic~s.

(I. I) and (I, - I )

+ x 2 Jy, where C

bound<byx

is the cUJvc enclosing tbc region

Jr + y 2,.< = 2

y 1)1IX

lhc region bounded by x

IJI =
9.

4. f c.tyl dx

5.

f r (3x'y' + y)dx + (3x' .'' + 2x) dy , where C is the boUlld


ary of the region enclosed by x + y = I , .r = - I~ J = - I
7.

(x' + 2y 2) dy. where C is the curve (x - 2) 2 + )" = I

c (x 3

6. f / Ta n- ( v/x ) dx +In (.<' + y 2) tly , where C is Lhe circle

(x3 -

y ) 1/y. whercC is the curve enc losing

= y2 -

I . x = 1 - y2

.J.

(.r 'y' + 3x )dx + (2ry- y) dy, whereCisthebowtdaryof

Tc

the region enclosed by ~ =


10.

_..3) dy, where C is tbc curve 2lxl +

J.

1-

y' (X ~ 0). y = x + I. y+ x + 1 = 0

(xy' + 2.r)d.< + (x 2.1 + y+ .r 2)dy,whercC iSihebound-

Tc

ary of the region enclosed by y 2


11.

x2 = 4, x = 0 .r = 3

- y dx +xdy
, where C is the circle X~+ y 2
.r1 + y2

=1

14 6 Gretrt 'i Thwen1

12. Show thaLGreens thooremc1n be expressed vcctorially in lhc fom1

* 28.
.r'

( (X - y)(dX

)('

+ dy), where C L~ 111 SCmicircular p21l Of .< 2 +

= 4 abo\'C y = .r from (- h. - ./2) 10 ( ./2. ./2)

29. { (r' - ysinx) tiX


>t.

IJ. Ir u cur"c C is unccd out ill the diroction defined by Grc-t."fl's the
oetn. it can be shown that a norrnal vcctOC' to C that always point$ to
the outside o ( Cis n ~ (d ,v, - tl.t ). Show th~u Crccn's theorem can
be wriuen vcetori:.illy in the roem

cFn ds =

JJ.

+ y?/ b! =

+- 30. The rti:uh of this exerc i ~ i" UM":ful when 1he cuf\t C in Green's
thcMcnl conttliM u point (or points) at which cilher P or Q fu.ils: l(l
h.1"'C (Ontinuous fio;:L p.uni~l dcrhuti,c s (sec E..:crciscs 3 1-35).

(a) Suppose :1 pic~.c"+set;tnooth \:Urvc C ( ti gurc below) conwins i1l its intetioranothct piecewise-smooth eun e C'. and
P (x. y) and Q(,.x 1 _\') have contin.uou,:; tir$i1 pa11ial deri\'8
lives in 11 donlilin 00111uining C and C' and lhe region R
tx!C\\.-'CCO them. Pl'OVC 1hat

In Exercises 14- 19 usc the rcsuJts of Example J4.16l0 flnd the wc.u

f.
rc

+ Q,t., + J.

l'd x

Yc.~

* 15. ThCSI!Ophoid

X=

* 16. The astroid x

= <.-oslO. y = sin;8 (see

(I - t 2)/(l
(sec Exercise 54 in Section 9.1)

+ 12),

+ (cosx + x~>)dy. where C is lhccu""

lc ,
.x = I - ."' fron1 (0. -I) l t> (0. I)

v . Fi/A.

enclosed by the curve.


14, x 2f a ?

1021

+ Qdy =

j'f1/( ( aaxQ -

iJ P)dA .

ay

Hmr Join C and C' by two curves such as those in the


fig\IJ'C.

y = (I -1 3)/( 1 + 1 2)
E~ereise 62 in Sec.

t'dx

lion 9 .1)
... J7' The right loop or the (.U(\'(' or LiS$ajous (see Exercise 60 in SectiOn
9.1)

= 2cos1 + <.US'll . )' = 2sin 1 * 19. 'fhcdroplcl .,. = 2 cos l - ~in21, .r = sint

,.. 18. The <.lcltoill

sin 2t

In Excn:ises 20- 24 usc the result of Proj.x."t 23 to find the arcu of the
polygon with the points a'> su<..x:cssivc vcrticc.:s.
20. ( 1, 0) . (0, 1), (- 1, 0), (0, -1)

21. ( I, 2) , ( - 3. 2) . (4, I)
22. (2, - 2), ( 1, - 3), (-2,1), (5,6)

(b) Ex1cnd this ..,;uh to sboiY 1ba1 wben C' is l~pi!ICed b)'
n diStinct cunc& C; (figure below), ~Uld P and Q ha\'e
continuous first p:trtial derimti\'cS in a domain contll.ining
C and lhc C, and the region R bctwa.~ them.

j P tlx + Qdy+<[ /'tlx +Q ,Iy+ ...

Tcl

J'c

23. (3, 0) . (1. 1). (2. 5). (-4, - 4)


24. (0, 4), (-1.0). (-2.0). (-3.-4). (0,-5), (6.-2). (3.0).
(2. 2)

In Excr.:iscs 25-29 cvnlunlc the line integral.


15.

fc

{21)""!1 + lr 1 y) dx

+ x1e"~1 dy . where C

c
is the ellipse

x 1 +4y?= .1

26. f

/3x 1y1 - x'>')tlx

+ (xy2 + 3x')'l)t/y, when: C i< the

c,

OR

cin;lc x 2 + 2 = 9

3 2

27. f c -x y dx
y~ ) J l2

= x1 _

y~

+ x 2 y 3 tfy. where

,{

C is cbc righc loop of (x 2 +

(c) What can we conclude in parts (a) and (b) if &Q f&x
8PJ8y in R?

0 1ple 14 VectorCakulus

1022

In Exercises 3I- 33 usc the result of F.xcrcisc 30 to evaluate the Iinc


integral.

ydx- (x- l)d\


(
)2
, where C is lhe circle x 2 + yl
c x - 1 +y2

* 31.
* 32.
*

- x 1 y tlx

fc

33.

+ x l til

l )l
(. 2
x +y

where

=4

C is the clliJlSC 4x 2 + l'l

}'

+ xdy
, where C is tbe square wid1 vc11iccs (2, 0)
.r + )'1

-ydx

and (0, 2)

36. Show that

* 34.

Show that the li.nc integral ofE.,crci.sc 19 in Section 14.4 has v-Jiuc
2rr for every picccwiscSillOQth, dosed curve enclosing the origin
that docs not imcrscct itself.

35.

(a) In whut domnins is Ihe line integral

pendent of plltlf?

Jca"

where \1 2 P

indc2 + _1'2

j ' f <J1 P dA,

1k

a1 P a' P Him: Sec Exercise 13. What can we

= -;;-:;- + -

av1

vX'

xdx +vdv
X

[J P ds

cone lucie i ( P(x, y) saLi.stics lnplacc's equation in R ?

37. Show thul

(b) Evaluate the intcgru1 clockwise around the curve x 2 + y 2 2y = I.

j l' aQds= j ' f PV 2 Qt1A+ {{ VP "lQdA .


~ an
J.
11.
f1i111: L:se identity 14.11. This rcsuh is olicn called Grr:c11 'l,{iYSI idcn

riry (inlhe plane).

In F.<crci~ 36-3& :\~;UincthM P(.f . }')and Q(x, )') hveconlinuous


second partial dcrivonivcs in ~1 domain coeuaining R und C (figure
folll>"-s). Let i\ = (tly fds . -dx fds) hclhcoulw:nd-pointing nonnal
to C , and let

3M. Prove that

f ( aQ
P-

Thi ~

ilQ
- = VQ n

a,.

3n

,
- Q-aP) ds = ~~ ( P\I,Q-Q \1-P)dA.
3tr

is oflcn called Grce" 's scco11d idc,riry (in 1be plane).

1+ 39. Find nil pos.siblc values forthc line inlcgral

be the diJ-cctjonal dcrivuti.\'CS o( p and Q in the direction Jl,

-1dx
" .,

curves in the X)'plunc not pUS$ins through the origin.

+ .rdy ror

x + Y2

l t4.7 Surface Integrals


IJTC11Iif3!i(WIM

Di~oi<le

S Duo

11

~nallct i>Uffacts to tlefil'le the Sutfiti.:C:

intcgr-.all)i f(x.y,:.,)

QVC:t

f(x, y, z) defined on some surfaceS (Figure 14.3 1). We divideS i nto


subsurfaces of' areas /J.S 1, IJ.S2 , , IJ.S,. in any manner whatsoever. On each subsurface
IJ.S; (i = I .. . . , 11) we choose 'ln arhitrary point (xt,
z;>. and form the sum

Consider a function

y;,

'
=

"

LJ(x!. y!. zf}ll. S;.

(14.48)

i=l

If this sum approaches a limit as the number of subsurfaces becomes incrensins ly lo~e and
ever)' subsurface. shrinks w a poim . we c.:all the fimitlhc surface huegral of /(x. y . z ) O\'Cr
the surface S. and denote it by
X

f1.{s

f(x. y, z) dS =

lim

I 4\S,U-+0 .

1=1

j(x/, Yi, zi) IJ.S;.

(1 4.49)

Surface integrals. like all imegrals. are limit summations. We think of tiS as a small !>ieee of
area on S. and each dS is muhipl.ied by the va lue of f (x. y. ~) for that area. All such products
arc then added together and the limit taken as the pieces of area shrink to points.
The following theorem guarantees existence of surface integrals of continuous functions
O\'er smooth surfaces.

THEOREM 14. 6

If a function /(x . y. z) is continuous on a smooth surfaceS of finite area. then the surface
integrl of f(x , y , z) O\'Cr Sexists. If S projects in a one.t().()ne fashion onto a region
in the X y -plnnc, then the swface integral of I (,Y. z) over can be evaluated by
means of the following double integral:

s.)

y,

l fsrc.c y.z) dS= l fs., nx.y.g(x. y>J + (;.~ )'+(;;)'dA. no~so1


where ~ = g(x. y) is the equation of S .

M4iclli;l#t~

Rclalklr\Ship
hetu:ecn are:~ tiS "" a <urtace S 10
iB projecriol'l dlt in lhe ".r plant
l

h is nOt ncc~ry tOmcntQri7.c 14.50 a:s a formula. The right side is obtained by expressing
~

and d S in terms of ,y and y and interpreting the result as a double integral O\'Cr the projection
of Sin the ,\ )'J>Ianc. To be more CJ<J>Iicit. recall from Sc.:tim> 13.6 that when a surfaceS can
be represented in the form ~ = g (,<, y ), a small area d S on S is related to its projection d A in
the .ry-plane according to the formula

dS

dS =
Y

/ + ( axa~ )z + (a
iJy')z

\ I

dA

( 1-1.5 1)

(Figure 14.32). If we substitute thi< into the Deft side of 14.50 and at the SM>e time use
g (x , y) to express f (x , y . ~) in tcm1s of .< W1d y . then

z=

But if we now imer1>ret the right side of this equation a~ the cloubh: integral of the function

fi x.
IOit~IIJ.I!

To t\ilht
ate the surface irHegrat. di\i~ 1he
sphere inco 114'0 hemisphtrei etath
of whi.:h projc.,: b

the .\ )' plane

onc~lo-onc

unto

y...:(x. y)) J1+(<lz/ iJ.rP + (i)~fi)y) 2 over the projection S,,. of S onto the xy-

plane, we obtain 14.50.


Note the analogy between equations 14.5-0 and 1-1.20. Equation 14.20 state> that the line
integrol on the lefl cnn be e' -aluated by means of the definite intcgrol on the right. Equotion
14.50 states that the surface integral on the left can be ev-luatcd by means of the double integrl
on the right.
If a surface does not project one-to-one 01110 an area in the .r y -plane. then one possibility
is to divide it into pans, each of which projcc.t.s. one--to-one onto the xy -plane. 11le total surface
inregral over the $tuface is then rhe ~ um of the surface integrals over che pans:. For examJ>Ie, if
we rc4uirc the surface integral of a function f (.r, )' , ~) over the sphere S : x 1 + y2 + ~' = I
(Figure 14.33), we could divide S into two hemispheres.

each of which projects onto S xy : x 2 + y 2 ~ I. !ben

lis

j(x, y, z )t/S

=IIs.

f (x . y , z)dS

+IIs,

j(x. y . z )t/S.

A second possibili1y is 10 projec1 surfaces omo ei1her !he x ~-plane or 1he )'Z-plane. If a
surf.1ce projects one-co-one omo regions S., and Sy, in 1hese planes. chen

( 1 4.5~)
and

'Ve should also note thai the are.a of a surface

S is defined by the surface iutegrdl

lfsds.
and i f

(14.54)

S proj"ts one-1o-one onlo Sryo then


area of S

I+(oz)2+(o'- )2dA .

= /' {
Js. .,

ax

( 14.55 )

ily

This is formula 13.45.


The results in equalions 1 4.5~ 14.55 were based on a smooth surface S. If S is piecewise
smooth. rather chan sm.ooth. we subdivide S into smooth parts and apply each of 1hese results
to the smooth pariS. The surface imcgral overS is then che summation or the surface integrals
O\'er i L~ parts.

I EXAMPLE

.._..

14. 18

Evaluate

Ifs. +
(x

+ <.} dS. where S is that l"'ft of the plane x + 2 y + 4z = 8 in che firs

OClM l.

.SOLtmON The surface S projects one-to-one omo rhe niangle S.n


14.34. Since z = (8- X - 2y)f4 on S.

IL + )' +
(x

Slrfu.:e
in!egr.al of .l + )' + 4 0\'\:r lbe f.-..1
cdlr.t part uf .t + 2y + 4,: e S

z) liS

IIs., (x +
..Jfi
16

.Jfi

= -

8
X

+ 2-

16

= -.Jfi
8

~ - f) /I + ( ~Y+ (:~ YA
d

1418-l.r
0 0

.{
1

3x 1
2

14

(3x

+ 2y + 8) dxd)'

(80 - 24y

+ y 2) d y

J2i {soy - 12y 2 + Y


8

} s-zy

+ (2y + 8)x

X + 2_r: 8,}
~=0

rhe xy -plane in Figure

~~- r <Jx + 2y + s>\/1-.- (-~)~


+ (-~)
4 Js.,
4
2

= -

,1'

il'l

3
}

3 ()

dy

dA

14.7 Surf>c<: lntqr.tls

1025

When a surfaceS. such as a sphere. encloses a volume. i1is said 10 be a closed surface. When
u function f (x, y, z) is imcgrared o,er a closed surface S. the n01u1ion used is

This is similar 10 the notation for a line inregml around a closed cliT\e.

I EXAMPLE 14.19

Evaluate
Sun.c<
integral of z~ o\er ihe sphere
'2

ff/

SOLUTIO\

d S. where Sis the sphere x 1 + y2 + ~ 2 = 4.

We divideS into rwo hemispheres (Figure 1-1.35),

+ ,, ~ :~ . ..

Sz : : =

-J4- xl- y2

each of \\hich projc,1s otiC to-one onlo the circle S,1 : x 2 + y 1 ~ -l. : = 0 in lhc xyplanc.
For each hemisphere.

r2

= \ I '+ 4 _ x2 _ yl

+J

vl

.
dA
.r l - y2

- r=====~t/A
,
j J _ x2 _ yl

nnd therefore

=a J' f

l s,,

J J -x'-y'dA .

If we use polar coordimu~ 10 evaluare chis in1egn11

overS,, . then

If parameters can l>c found 10 dcoeribc: a >urfacc. ir may n01l>c nco>al')' 10 prujo:t the .oorfacc
into one of the C()()(dinnlc pi ones. Such is the ease for n sphere centred m the origin. Formula
13.65 for the volume d ement in spherical coordinates hHJic;H~ 11uu an area elemcm on rhe
surface of a sphere of radi o> R can l>c expressed i n terms of angles 8 and if> (Figure 14.36) as
( -1.56}

1026

Chnp1c1 I.&

Yc..:tor C.lt..:utus

FIGURE 14.36

L"nglh ~ R <in. dO

With this cho ice of area element. the ~urfacc imcgral in Exam11lc l4.l9 is evaluated as

follows:

12.'1

(2cos)1 4 sin.pdt/8

r
= 16 12~
Jo
cos2 sin d<J> d 8
0

32
= 161a
~
-~coi
w}'
'
d () =
101~' =
0
3
0
3

64

".

EXERCISES 14. 7

In E..:crcises 1- 8 C\'aluate thcsudtx:e imc.gral.

I.

fl

(x1y + z)JS. wh<:rc S i the flr.;t octant patt of 2X + 3y +

z=6
2.
X

JIs

(x1 + y 2) t d S. where S i>thaq >alt of z

= 0. ,\' = 0. X

+ )' =

= x + y cut uut by
0~

+ II.

jj~ .<J dS , where S is the ftrn octant """ or z =

5.

x2 + y2 =

.r 2 + 2J, 0

6.

:fA' :: I, 0 ~

!Is /4y + I tiS .

out by 2.< + y
7.

jj~ x

8.

#s(x +

1t

closed by

Jfs.l)'!1

d S. whcrcS is llle swieccdefincd byx; y 2. 0 :::

x ,:S J . 0

~ l ~ I

"' J2. Jfs :<y:.dS.whcrcS islhcsurfaccdcfincd by 2y =

of tl

/' + :r' cut

f"lsJ<-.r+
/
tiS, "hereS

p o r1i01l

In E:<crd.s.c~ 1 1- 17 C\~..ll u:i.tC Lht t urfacc imcg,r~iJ.

J' ~l. 0 5t.< l

CJULby

f(x , y. z) ovctthe surftlCCddlncd by z = 4 -x1 - 4y 2 (x. )', z ~ 0)


i f the surface is projcctoo onto the xy-. lllC xz . and the n -planes.

10. Usc ll sutfa.ee imcgml to ft nd d)C arc.o of the cur,'Cd


right<ltc.:ular co ne of radiu~ R and height It .

3. # s X)'ZdS , whcreSis the surfaceofthe cubeO::sx :::S: I.

4.

9. Set up lloub1.:: ilet'ated illcgrf;ll$ f~;.lt" lhc surfa:t: integral o f a fu nction

J'l -.t. .'f

o.05z53
i:o l.hc surf:ecc. tlclincll by 2z

=
13 .

)' ~ I

where Sis the fir;t octwu purt of y =

.x'

dS . where S is the: sur fnec x

y2

I fo 0

~ z :S 1

y) <IS, where S is the surrat'C bounding che volume en-

x = 0. y = 0, z = 0. 6.r -

3y + 24

=6

a 2 inside the cylinder r 1 +


cut

+z =I
2

j'ff s J2at - z' dS.whcrcSiSihatport orx ' + .v' + (z -a)

llt)(]

.._ 14.

in LJlC fi1~1

JIs

J5.

z dS.

fIs .\' y

2 2

y' + z' = 2

= ay , undcmc;ath th-e pl;mc

!.

= a.

(JK:liinl

where

between the 1>l t~ ncs

~1

Sis lhal part o f the surface x1 + yl -

zZ _ 1

z: = 0 und z = I

d s. where s is th:u p:m or ?.

= .\':: + y 2 inside x 2 +

16.
17,

/i , :JS. \\fleneS ~IhiMpa.nof .: .Y i.._Lk ..


/lth+
of~

hy ..

x') JS. "hereS i.s ll. . prt

o.z -l. .~..r.'

J t

~;z

"

1 - "t OOurt.kcJ

= O

2.\. A 'i"'C..u.i n';;lk.ri~ ~ altowcdtollnpo"'l4l lhc,ri'I{~ A t+ 11+zl lauhcpoitu (0. 0. U (a ld;nKr..MOM1ftt-.cwu. ~ uKniLI._~

In I t.4YCttel 18 22 eva luate the: $UI"(ace lllf~rral b y PI'OJ~II: the t W"'


face tnto cne ot ~~ rootdrm.tc ptancs and a~ by urw a rea clement
14 56.

4 tH.

/fft/.~ . whereS i .. che-sphacA'!.r1 +

z 1 = fl2. l!lthl-. thc

tbrmtln for tlte arc; of a sphere?


19 .

<U c~nly i11

all ditctl~& lAnd rul'l" d(w.n lhf "'~ bo(4)fl'\lin! ,.~

mtr..l u".-c \<~~a-..''~ M J. T ho:th , ._,,.._~"o l

U . Show that if :s i-urf:tce S tldlltctl lmpllcill) hy the equ:.t'ioo


F fx. )'. t ) = 0 ,,rojC(:IS OriCIOCI1e .._..u ... the r..::,;inn :>,r 11'1 u~ .ty
t>lanc, then

/J,

t' 1t~ tiS, whe re S i"thc ti J)I"Crt: A l

+.} l

+ 41

f'{ls /(~.y.z)dS ~ Jls.. tt~ . y. ,( . y)I-ta-"'F_I_'I_ ,IA.


1

20.ff (r;a-,Y 1)l/S,whc.rc:Sisthchcml~)hl!n: t- .,JY -

/041

_.!

y!

" lS.

(2) Fir'ld the' tii"C'Il cut from lhc cone .. t 1 -

qlmdcr .xl

:.Z I. Jl (,t': +> ' )dS ...f'l..:.rcSlslhespbcrcx z.._ y: t- z.' R 2

Jifs

lhe n-....c,,:.),.w:rc-~

l il'lc.w-lywithl'C"'pcct tomcle r# t Fi,w\! 14, \(1) (l\1n\ 0.00 1 tn bt (0. 0 . I)


~ ~ O.OOS m a:t {(), 0, - 1). Fittd tit(' V1.' '"")1.l: of IU,.tun. tllll the sphere.

\' l

+ 12- 1x

(b ) Jol nd the llw.l; CUl (n._i<m thr.: C)tl~(f t' 2'


ccr.c~ ,: =:::: .rl + ) .

+ >''

+.),

by lh,:

=. '2X by tho!

14.8 Surface Jn tegra ls Jnvolving Vector Fields


1l1c n\OSI imponunt and (.'Ommon type of surface Lnteg..-dl occuN: wh~n f (X, y. t) in 14.49 i<>
s;pt-cille<.l as the I\On'l1nl com t>nnenr of sonlegi,c n vcccorficld lf (.t , y, ~ ) de fined on S. ln ot:h~r
wun:l". /{.r. y , ~) iudl' is 1lot given. but F' is. and en f1 txl .((,\, y . .:) we mu:..c calcuhuc the
c:ornporletu tlf P t10ITili'I I IU S.
~r'h i~ pc'C&Uf)poSc~ thai ~U.I r'fucc~ ~trc two-~i dcd ancJ that u nurmul vee lor

assig,lCd in an

llli/Ullb i~u ous

way. When this

i~

possible. the ~ u rfuce:

i~

co a sttrfdcc can be

N-lid to be nrlentublt!.

All "urfncc." in this boua.. are oricncnble, '''ilh the cxceptiuo of the M6b iu~ ~tri ~, mentiol'~d bc1ow.

c.

Tal:eu thin I"Ci.'tan, uhJr >lripof P't>Cr :.tnd htbcl i'-' cornet) A . JJ .
und
(f'i&ure l.l.31a).
Give chc Mripa IMif cwi" and join A and C .and B and D (Fi&ure 14.3/b). Thi<>urfilCC. c"Oited
a ftlbblus Jtnp. t'JJUkX he tL,,i&neJ:. unique oonnal \tCtor that ,.,ric.~ e'1t'Uinuou~l) 0\tr the
a.urf.a-ce To tlluur.aat. supf"'Xe th..'lt at point P in F1gure 14.}1b, we assign a unil normal \ret.'t.Or
ii a.. sho" n. 8) mo,ing cw\CC' around the stnp. ' ' c cdJl var) the dtn.x.1tOn of R conunuously and
lm\'C OOC.'k l l p Wllh 0 pocnling in 11\c opposice din:CUOO. 'fluS SurfICC iS suid 10 ""'"only One

side. or co be oonorioncablc.
We coooider only surfaces chac are oriencnble. o r ha'e cwo sides. 3c\d cao1 chetefore be
assigned a unit 110rrnal vecl()r in au unambiguous way.

..
(Qi.:l.l!lg t.t iJ f )oll\p

mE1!l
vi ~IX!

lW

eun

l'lt \l f II tlOIIUI'ICntlble "1tf\1o;:e

1028

Chl~ftr

14

VlorCalrulu"

Suppwc again that F(x, y, <:) i~ a vector 6 cld dc6ncd o n an (oricntablcl surfa ce. S a nd
i <the unit normal o n one side of S . If f(x, y , z) is the component ofF i n the di rect ion
then
f (x, y, z) = F ii. nnd Ihe surface inrcgr~l of f (x, y, :) over S can be expressed in the fom1

n.

(14.57)

EXAMPLE 14.20

JIs

!::valuate

S:z = 4 SOI.U'nO!I:
that

x2y1+ xd and ii i s the upper normal to the suti'acc

F iidS. where F =

x2 -

y 2.

: :;

0.

+ x' + y 2 )

Since a normnl vector to Sis V'(: - 4

(2.<, 2y, 1), it follows

(2x, 2y , I)

fi =

Mjlcjil;jSI dJIWj S\nl~cc


intel:('. t O'lter tbal pan or z
4 -

"'

.T! -

,.~ :tho\'~ 1he

xyplane

T hus.

Tfwe proj ectS onto Sx> : x 1 +

',

..

~~ , '

+ 2xyz
j 4x 1 + 4/ 1 + I
2x 3y

F ii =

!L . . JLs.,,
S

Fnd5 =

= {{

Jfs..,

"' 2

2 {

:0

4 in the .ryplane (Fig ure 14.38), then

(ilz) +( !Jz)
-

+ 2xy(4 - xz ,j4xz + 4yz + 1

2x3y

2.ry2 -

"}r.=;r

x~

EXAM P LE 14.21

iJy

yz) J r + (- 2.r)l

dA

+ (-2y)Z dA

d.r "' 0.

-J-x

- 2

2x;y + 2xyz / I + ,j4xZ + 4yZ + I\


Jx

._...
Ev~:1lua1e Ji. F ri dS. where: F =xi+ yj + zkaud ii is the: unit outward-pointing normal to
.lfs

ljlclll ;h"U:1M Surface

the surfaceS enclosing the volume bounded


SOL UTION

We divid eS i nto four parts ( Figure 14.3\1):

+ yx2 +/

s2

z=
z=

S>

y = ,j4- x 2 ,

s4

y =

St

On St , ii =

by x 2 + y2 = 4, z = 0,.:: = 2.

0,
2,

x2

-/4-

:0 4
:0 4

0 :0

x 2

z :0

2;

$2.

0 $

- k; on S2. ii = k; and on SJ and S4 .

"=

V'(x2 + y2 - 4)

IV'(x2 + yz - 4}1

(2x , 2y, 0)

,j4x2

+ 4y2

(x, y, 0)

14.8 Surface ln1.::gr.ll .. lmolving Yator Fiel\.1<;

We know that S1 and S2 projccl onto S.n : .t 2 + y 2 :5: 4 in the xy-plane. and

1029

S1 and St project

o mo the rectangleS.": -2 ~ .r ~ 2, 0 ~ z ~ 2 in the .r: -plane. Consequently.

= 2

Jj,. dA + jj,'s,: J424

s.l)'

2{arc<> of Sn .J

= 2(4Jt)

+8

dA

r.=:::;rdz dx
x2

/.

l-2 o -/4
1

t6

x2

dx.

_ 2 -/4 - x 2

Tf we set x = 2 sin() . then dx = 2 cos 9 d() anti

= SJt

I EXAMPLE

12
1.6 j "

- /l 2

1-. 2cos6d8 = 8n + t6{e}'11


cosO

= 24rr.

- .T/2

14.22

A vlcwiog wiJtdow on lhc side of a submersible vehicle in n marine theme park is a hemisphere
of radius 1n m. Find the force due to watei pressure on the windO\\' when hs centre is It metre$

below the surface.

1}1!1!iiJ

J!''-'".'(: \)(

water on a hemi$pberical \\'indow


of a ~utunerll.i l)le whlcle

SOLUTIOl'\ Let us choose a coordinate system with the plane z = II in the surface of the
wate.r, and take the cqumion of the window as S : .r 2 + y 2 + z 1 = 1/4. y 2':: 0 (Figure
14.40). If d S is a small are.1on S , the force due to water pressure on d S has magnitude P d S ,
where 1' is pressure, and this force acts nonnal to d S. If isthe unit nonualto the hemisphere
with negative y-component, theo the force on dS is ( P d S)n. Because of the symmetry of the
hemisphere. the x -component of the resultant force will be zero. Since the v-component of the
force on d S is ( P dS)n
the )'-component of the resultant fo rce on the ~vindow is

J,

fpj

ndS.

A normal to the sur face is \7(x 2 + y 2 + z 2 - 1/4)


nonnal in the negati ve y -direction is
X

fi =

---r::;;-='=(x='=y~'=z=):::;;
Jx2 + y2 + z2

(2x, 2y, 2z) , and therefore the unit

= - 2(x, y. z) .

1030

OtaJ)I CI 14

V~'tO.-C:IIk.'Uhl$

Will> P = 9810(11- :) and formula 14.56 for an area elconcnt ou the hemisphere (where
R = 1/2),

JIs

P] n dS

= / 981 0(11 = - 19620

[f

~cos)

(11 -

4905
4

( "

lo

sin</> sin 9) G sin) dd0

(211 - coo) >in2 4> sin 9 d d9

r.
o Jo

( "(h(l- cos2) - sin 2 t/>cos] sin Bd<f>t/8

= _ 4~5 [

{1r (<~> _

sin 21/> ) _
2

4905;rh (-cos /JI~ =


4

= - ')()S /.
4
o
= -

: }(-2)) dS

sio~ [

490Srrlt

sin B t/B

N.

Titc ZCOntjlOnCnt Of the rcMJhant force on the window is

jPi< ndS= j 9810(/r =-

19620

z)(-2z) dS

ff

( h - icos)

4905 [," ).'" (2/r = - -4


0 0

"

sin) dd0

cos ) cos <1> sin d<Jo dB

4905 [,'' I- lt cos 2


=--4
0

= 1635 (" }" = 1635;.r

Gcos 4>) G

I
+ -cos;
}" dO
3

N.

The rcsultill\t force on the window is therefore: Jl'( - 4905hJ + 1635k)/2 N.

The gate in Figure J4.4 1a has constant widlh

w=

5 m ( into lhe page). Its cross~secti on

is in the shape of the p;uabola 4x = y 2 and it is hinged along a line through 0 into the
page. Water with depth 4 m pushes on the concave side. We are asked to detennine the
magniLUde of a vertical force G at P that wiJI main1ain the gate in an equiljbrium state.
We are to ignore the weight of the gate itself in our calculations.
DIUII;i

I j lUll ;I fit W:JJn

. .

Forecs oo a hioged

gt~tc

}'

4x = yZ

4m

41ll

Walcr

W!tiCI'

The fon.-e of the watcr against the gate creates a moment that r~ temrt to
turn the Hie coum~rclockwise around 0 . The force G at P c reates a moment in the
opposite dorccl!On. They must combioc to P'""'"t motion. If we tnke the moment of G
nbout 0 '" neg,lti\e, then it i~ Me;
-5G. The momem of the w.ccr on the gate"'
more difhcuh to calculate. We divide the gate into horizomal strips of length S m ,ltld
width corresponding to length d;r along<he .r-a<is (Figure 14A Ib). The magnitude of
the force of water oo the strip is
'iOI II I 10'0

(dy)'

5pg(4 - )') /1 '

dx

dx = 5pg(4 - )') / ,
\

= 5pg(4- )'),/)' 2

(~)

dx

)'

+ 4 dx.

y
Thi~ force acts J!Crpcndicular to lhe gate. Sine<: the unit normal to lhe gace is

( - 4, 2)')/./16-.. 4).2 = ( -2. )}/)4

>)/yl + 4dX ]

Spg(4 [

+ y2. U.e force of "ater on the wip i'


( - 2. y)

j 4 + y'

)'

Spg(4- y)dx

)
(-2,) .

The horlzootol component of the force of water oo the gate is therefore


F. =

'
1

10pg(4 - y)

d.t = - IOpg

1'

)'

4 - y ( )'

(4- y) tly = -Spg {4y-

-Spg {

- - -dy

~l )~ = -40pg N.

1'he vertical component is


F>" = {

Spg(-1 - ,r) dx

= 5pg { 2)'2 2

>~ )' =
J

Spg

1'

(4- >)

(fd.v)

80pg N.

To detemune the moment of F = F, i - F, j about 0 . we must kno" its point of


application. the centre of pressure (.<,. >A . Using monxms of the component> ofF uboul
0 tFigure 14.42>. we calculate that
&Opg x, =
3

f.'

x(Spg(4 - y)]

d.t = Spg

Spg {
>.s l'
=s
J - 5 r0 =
- 40pgyr = [

Y [ -10pg;4 -

32pg .

y)]Gdy)

2 3)'3 )4= -

= -5pg 2y -

160pg

- 3-

J (4 - ) )
4

(!..d,,)
2

Cons<<iuently.

x,

3
6
= 32pg. - =
80pg
s

\Vc now take moment;; of G,

o = -sc + 40PR

160pg

F.,, and F>

(i) +

80 8
:

------ =
3
-.!Opg
3

and

about 0 i n order co calculate (;,

G)

G = 167000N.

1- - -

Si~n~1~~:::':!_P~.am

F,

(615, 4/3)

EX ERCIS E S 1 4 . 8

In
I.
Z

l.

f.:Xcrci~s

!Is
=
fJ~

1- 16 C\'aluaLc the surfaec imcg.ral.

7.

(.\'1 + yk) i11/S, whcrt Sis the lirsloctimlJ>an of.t + )' +

10

:3, :tnd fl is Lhe ur\il notm~I IO S with J)(lSil.i\-c tCOR11)<.mcnl


(yz1l + )'f''J+ xk) . n d s. w here sis defined by

y = x2 ,

0 ~ y ~ 4. 0 ~ z ;$ I, and 1l is the unit I'K'Ifllt;J.Ito S whh positiv~


)' COU\J)OilCf'll

3.

!1 (xl +

I - xl - yl, and 11 is ils upper nonmtl

yj

+ zk) n dS.

wllcr<: S is the hemisphere z

Jh

(.<yt.i -

"' 8.

,r!- ) '2, t ~ 0, :11ld

lht \'Oiwhc defined by t


oulcr ll()llll;sl to S

6.

= /4 - x l - _rl, Z = 0, :.\Ud Ji iS the unit

JJ.(.vi+yJ)ndS. where S

j.r.l

+ y2 below z =

Z COnlJ)C)OC nt

is ohatpartol'd1C$url'aoe

z=

I, and il i~ the unic nonn~II LO S with negmivc

li

i~ lhC tulil nonnal tO

w ith ncg;Uivc Z

JJ. (yzi' + Xti. + .t y k)' n r/ S, wl:rc S is lh" surface coclusing


* 9. lfs

=O..t =2. t =0. z = y. y + t = 2, and

i-. l he uni ~ (.'14,11Cr ll('ll'mCII IO

10.
4.

yk ) ii d S. where S is the surface thai cncbses

is the unit nonnal

<.'\)lllfX)I"-"IU

t1

und fl IS Ihe unit k;.lwer ftOt'n\Ul iO S

l.tl. and j,

dS . w here S is defined by z = 2-

thovolum c defined by X

Jj~ (y2l + u j + xyk ) n1/S, where Sis ''"'1)(111 ofthe surfaec


z = .r 2 +y 2 cul out hytheplt~ncsx = l ,x = - I,)' = I.)'= - I,

fi (zi - x.i +

l) c:ul out b y t:;;;;: 0, .t:;;;: 2. ":;;;:


with posli.I\'C )'-<."'mponcnt

fIs (.r'.vl + .tyJ+ zk) n

4.

5.

.vj + zk) i\ dS. Where S iSthcsnuollerpartorx' +

y 2 ;;;;:

JIs (xi+

z ~ 1. tu)() il

II.

y j) io dS, where S is the surfa<'C x'


i ~ the unit UI)Jli!f O('IH<JI Io

+ )'' + t' =

Jf. (x 2i + y'j + z2k) ii JS. where S isthcsphcrc .t 2 + y2 +

Jfs

.z.1 =

12.

a1 and fi is the unit 0Ult 1\0tm a) tO S

f!.

the: plane

()l -

,J + k) ii r/S. where S i~ ohcnmlk:r,;t..facccuo. by

y + : . = I. from the $Jlhen: .r. ! + y1 + z2

uniLuppel' oonnaJ to

= II and il i$ lhc

14.9 ll.e Oi\'ergerKe llleorern

#/

13.

n d S , wbcrcF "'

(z2 -.x)l- xyj +Jzk, S is !he s urface

CllCiosing the .-olwne defined by z


an~l Ji is the unit Outc:r n()rftX'li i.O S

JIs

14.

fttce z

(.t

=:;

= 0. y

=4- y' . x "' 0. x = 3. z =0.

2 1. A very long w:tu:r channel extends in the.\' -d irectio n und has constant cross -section that. in the )'<plane. is de fin ed by

l + xyJ + xzk) ndS, where S i~ that par1 of the sur

z=

/4 + y2- _,. 2 in the fi rst ()Ciani C-ut out by 1hc phlncs


= l. x 0, z 0, and ii is the unit normal to S

Jis

x= n

(x 2i + y.j - xk) . ,; d

16.

in !he 11m octant c ut out by y'


S with posiLi\<C x ..c.:omponcol

#s

yZ/ 4

(yxl +

+ :l =

+ z' =

y'j + ,I'Zk) fidS . where S

I . and

I. and

ri is the unit

x' +

is the ellipsoid

i is tbc unil Olll<:r nonn al to S

t 7. Show lhut if u. ~urracc

011 (a) the bouoo> of the channel ( - I :;: y ::; I ). (b) the right wall
of the channel (I :<; y 5 2), and (e) 1he lcli wall of 1he chunnel
( - 2 ::; y 5 -1 ). If all three forces arc added, is the rcsult equal 10
the weig h t per unjLlcng lh o f the water in the ch~nncl ?

s.where s is thll paot or lhC surfucc

normal to

"' 22. Supptk)c the swfacc of a Ouid is in the x y-planc 2nd a tlat surfa~
S is sub mctgcd i n (he fl uid. Lei S be a p !l.rt o f the plane Ax By +
C ! + D "' 0. and lel lhe proj ection of S in the xy-plallC be denoted

by
on

s projects onclo~o nc ooto a n:gi<lll

s .T)'

-(y + 1)' . -2 :$ ." < - I


o,
- l !:y!:l
{
(y -1 )1
1<,1' 5 2.

If the channel is ful l, find the force per unit lcnsth i n the .'{ -direc1ion

with po$ilivc z--compoocn(

+ IS.

10l3

R.ry. ShO'ov that Lhc magnitude of the z-componcnt of the fluid force

is eqwJ to t he wcighl or the colurn n of tluid ~bove

and below

R,.,..

in

the .ry- planc. then

!is

2.1. Extend the result of Excrci>C 22 to any surface S tbal projects

(Pl + QJ + Rk) ii dS

onc toonc onto Rr1

"'f '{ (- P ~<. - Q az +

lsAy

a.r

a.r

R)dA ,

-1<

s.

24. A circuh1r tube S : .\' 2 + t 2 ; I. 0 :S, y :S, 2 is u nxx.lcl for a


p;lrl Of an attCr)'. OIOOd 0 0\\<~ through the artery and Lhc fo rce per unit
w-c:J at any point on the :.u1cri:1l wull i$ gi\cn by

the :!:: dcpcndins on whether nis tltC-Ul>pc:r ()['"lower norntal [ t)


What
nrc corresponding form ulas when S t>rojccls 0~1Cto-onc onto regions

S1 : and S,::. in the


18. Ewlute

)'Z 11nd x~..coordin:ue

fis (yi - xJ+ zk) n

d S. where (a) S is ohm part of

< ~ 9 - .r'- y' cutout by z ~ 2y and(b) Sis tbat part of z ~ 2y


cur out by .: = 9 - .t 2 - y 2 and ii is the unil Ut>pct normal to S in
e~ h C:l.Se. 1/inl: Usc polar coordi n at e~ with pole :u (0, - 1) .
t 9. Show that if a surface S, defined imt)licitly by the C.}ua.tiM
G(x. >" t) = 0. projects onelGOnc onto l he region
in the xypl>llC. then

s3,

* 20.

:r =

!1

F i\dS -

F -'VG

s,, IWta~l

deep.
(a) Find the force on each or 1t1c !'our sides when the trou~ h is
full.
i lli)V-11

' n+ - -J.
r' + t

where
is the un it o uter nonool 1('1 the :uterinl W:\11. Blood ditru~~
throug h the wall in such :1 w:~y th at if d S is a small :1r~a on S , the
amount o f dift'us!ion th ro ugh dS in I sis Jt . il t!S . Find the tmal
~~ moun I of blood leaving the entire w11ll (Xr secon(L

15. A bca_rn of Jisht travell ing in the positi"c )'direction h:~s circular
cross-section x ~ + <:~ < a2 . IL slrike.s ::a surface S : .r 2 + )'2 + z 2 =
a2. y ~ aj1. n.c i.ntc~sity of the ~<lJTl is given by
.

dA .

A \~oater 1rough is houndc~ l on tl'k! sides hy l hc surraces .r = 0.


2, y
z = 0. and z. = .~2 , all dinenl:ion~ in metre~:. and is I m

(b) AW.I all fout fo n.:cs

F' =e

planes?

(a). Is the rcsuh cQui11 to the

weight of the w ater in the tm u&h?

where 1 is time. T he absorption of light by a small UI'CJ d S on S in


Lime dt is I ii d S d t. where i:; the unit llOt'mul to a.t d S.

(u) Find the total absorption over S in time


(b) Fnl d I he 10131 a l~ rp1ion <wcr
I =

dt .

from ti me 1

= () to time

5.

114.9 The Divergence Theorem


l.n Lhis section a nd in Section 14.1 0 we show Lh<!!t relations hips may exist between line ime gr.lls

and surface integrals and bcrween surface integrals and ui ple integrals.

The divergence theorem relates ccnain su rface integral>O\ er >Urface>that enclo.e I'Oiume->
to triple integrals over the enclosed volume. tore preci;,cly, we have lhe following.

TH E 0 R EM 1 4 . 7

(D ivergen ce T heorem)

LetS be a l>iecewi.e-;mooth .urt'ace eoclosing a region V (Figure l-1.43a). Let t"(,t. y. ;:)

L(x . y, ~)i + M ( t , ) :)j + N (x. y. :)k be a vector field \\hose component< /., M .
and ;V have continuous first panial dcri~111hc.s in 8 dOntllin COillainiog S and V , J( I) is
the unit outer nomwl to S. then

#/
#. . . . .

ndS =

or

(LI + MJ + N k) n dS =

c:adosmg \'Olume V

ff[,V

( 14 .51>.o)

F dV

jff,. (a- 1.x + -aMay+ -aN)


_
ih
0

tniO 1'.1.'0

pan_,

IO \COl)'

dV.

J:>i\1QM of ~Urfle't'
lhe diH~:rger!C"C: lhc:fNm

1:
s

sl: z =/,C.r. y) '

~)
S1 : /1Cx,y) /

PROOF We consider first of all a surfaceS for which any line pamllel to any coordinate axis
illlei'<(J(S S in at nol!>t '"" point> (Figure l-1.43b). We can then divide S in10 an upper anti
o lower ponion .Sz : : - /z(x . y) and S 1 : :
j 1(x, y), both ofwltich have \he um<
projection S9 in the xypl.u~e. We co~idcr the third ~em> in the surfa<e intc)UI11 on \he left

;ide of equation l 4.S8b

ff/ i. .i. ds = Jfs.N k. . ndS + Jl Nk. it ds.


On

S1

14.9 'lllt

l)ii'Ct);CIIC<

lbc<JI<m

1035

Consequently,

Jf. k n dS = j"f
Jf/
Js, r==

- N
======dS

+ (af,)z+ (af, )z

i!x

i!y

"
{
+ !ls, /1+ (ah)2 + (a~z)/s
N

ax

a)'

~ JJ,. ':"(~/:(~/ + (~)' + (~)'"


+

1(

\ I

j"fl s,,

(a )2. (a12 )1\

h, /

/I+ (iJfz) + (CJJ,)

N(x, y, f2(x , )'))

12
+ a..

-r

ay

ax

ay

dA

(N(.r ,y,fl(x , y)) - N(X,)', f,(x , y)l}dA.

On the other hand,

Iff" aN

- d\f =
a~

lilcl*hHUKCM
uf a surfaC(; into

pjo."'CS

Dh-ision
such lluu

j,cx.y)

jfls,,

d: }d A =
a~

//s { }ht>.>l
s..,

dA

j,(x,y)

{N(x. y .f2(x.y)) - N(x.y. f ,(x. y)J)dA.

We have shown then that

, = !!!.,, i-JazN dV.


#,s , ndS

lines parallel to the 1.'00n.Jinatc a..'\CS


intCI'll((;l lbC ::o,.wface ill at lnUl>l t-.o

Nk

points

Jls., {/ /:ts> aN

Projections of S onto the xz- and y.:-planes le.ad in a similar way to

Jl. MJ ii dS

Jfs

!!"Jv( iJy

iJM d\1

and

Jl.sLi
jf

li dS =!!"l(v a,,

dL dV.

By adding these three results. we obtain the divergence theorem ror f" and S.
The proof can be extended to more general surfaces for w hich lines parallel to the coordinate
axes intersect the surfaces in more than two pOints. Indeed. most volumes V bounded by

surfaces S can be dhided into 11 subvolumes V; whose bounding surfaces S; do satisfy this
condition (Figure 1-1.44). For each such subvolumc the divc.-gcna: theorem is now kn0\\11 to
appl)':
X

J{, F - n dS

1fs~

!!"},,(

V - F dV,

= 1, ...

If these 11 equations are then added toge ther. w e ha1e

t #,F

ndS =

J= l

S,

tJJL v .
t= l

Fdv.

, 11 .

1036

Clxtp1cr 14 Vc:..:tOI" Cal.-..'lllus

The fight side is tile tfi plc integal of \1 F over V since the V; constitute I'. Figure 14.44
illustr'.nes 1ha1 whe11 surface integrals over 1he Si are added, comriborions from auxiliary (interior) surfaces cancel in pairs. and the rcrnajning surface integra.ls add lO give lhe surface inlegral
of Jt D over S. \Ve. omit a proof for even more general surtaces that cannot be divided into a
llnite number o f subsurfaces of this type. Tile imerested re-dder should consult more advanced

books.

I EXAMPLE

14. 2 3

Use the-divergence-theorem to evaluate the surfac-e integral of the nomtal component of F =


x2 i+yzJ+xk ovcrtllcsurface S enclosing the volume V bounded by lhcsurfccs x + y+z = I.

x=

0, y = 0, and

z=

0.

SOLUTION The divergence tlteorcm (sec Fi gure 14.45) gives

M::Uflllll
thCOI'CIIl ilppliod

Divei'Se~

10 0 IC:Itll.bed.'(ll)

Ill{4x (y- x y- 2y2) - 3I

- 2

(1 -

x - y) 3 ~ 1 -X dx

=
X

~{

[ 2x( l -

x) 2 +~(I - x)'] d x
3
o
x) ]
3

{' [ 2.x - 4x 2 + zx>+ I


= I Jo

I{

4x

3 +2 -

j2( l - X)

,}

dx

1
O

I EXAMPLE 1 4 .24
Use tbe divergeoc.e theorem to evaluate the surfac-e ioteg al of Example 14.21.

SOLUTION By the divergence theorem (Figure 14.39), we have

ffsF -ndS= ff[ \1- FdV =ff[< l+l+l)dV


=

IJ[

dV

3(volume of V)

3(4rr)(2)

241r.

I EXAMPLE

14.25

Ap plication Preview
Revisited

Supposc thntnsurfocc S containS II point cha:rgesq, (i


I ... . . ll) nt points r 1
(Xt . y,. :t)
in its interior. According to Coulomb's law, lhe electric field .E at a point r = (.t, y. : ) due to
the c harges is defined by
n
q;(r - r;)
E
4JT ol r - r;l"'

="
f;;

..

Use the resuh of Exercise 28 in this section to \'erify Gnusss law

- ndS=Q -

where

Eo

Q is the toral charge insideS. This is rthe problem of the Application Preview.

SOI .L'llON

According to Exercise 28.

ds n
fiE
s
-

#,s r~
L....J

f;t

= "L

1:1

q;(r - r;)

41t<foir -

Ql (4~r )
--

4it ~tt

"

rtP

ds

] n

(because each q; is insideS)

= I:~
= <o
-<o
t

I E XAMPLE 14.2 6

Pro,c Archimedes' principle. which SUites that when un object is submerged in a Ruid, it expe.
riences a buoyant force equal to the weight of the nuid d isplaced.

tit'lfl 01 Archimed~ ' pnnnp.lt for

any (lbjec:l

SOLUTIO'< Suppose liu; >Urface ufthe Ouid is ta ken as the xy-plane (! = 0). and lheubject
occupies n region V with bounding surface S ( Figure 14.46). T he force due to Ruid pressure
P on a> mall area d Son S i< ( P d S)n. where ii isthe unit inner nonnalto S at d S. If p is the
density of the Ouid.lhcn P = - 9.81p:. and theforcc o n dS is ( - 9.81pc d S)ii. The resultant
buoyanl force is in the posirive z-direction (rh.e .r -and y-componenL~ cancelling), so rhat we
require only the z -componcnt of this force. ( -9.81pz dS) ii k. The tOial buoyam force must
therefor-e h:we: -component

Jfs(- 9.81pz)ii ic dS = Jfs (- 9.8tpzk) -litiS = #s(9.81pzk) - ( - li) dS.


\Vhere -

nis the unit outet normal lO s. If we mow use the divergence theorem. we ha\'e
#s<- 9.81p: )n. ii.ds =

Jjfv
JJi

v . (9.81pzt) dV
9.8lp d\1 = 9.8lp (volume of II) ,

and this is the weight o f the Ouid d is placed by the object.

EXERCISES 14.9
I n Exerc-ises 13-15 usc the d' ''crgcncl.! theorem to C\~lu a1 e lhe wrr9C'e
i n1egr~l l . I n each c.a;;e :1n :uJdilion:ll surface mu(t be introduced in order
10 enclose a volu lllc

In Excrci~cs 1- 12 ugc the di,-crgcttct 1heorem '"evaluate the surrace

integrJI.

ffs

_,.j- 2tk) it tiS. '""""' Sis tl-.! surface bounding the


volume: defi ned by Ihe surf3Ccs l = l t 2 + y 2 , x 2 + y 2 = 3. z = 0 .
I.

ti i!: the unit OUll'r nonmJ IG S

and

2.

z2
3.

(xi +

#s(x'i + +
=
#s(yzl + xzl +
y' j

z'kl ii dS,

wl-.!re Sis the sphere .r 2 + y'

IS.
xyk ) fo tiS. where S is the surface en<: losing

+z =

+ y 2z.i + z1.<i<)/2.

j[. (,d - x]

+ 7.

Jr.,

lhc surf(K': .:

+ yk) ii (IS , where Sis the surfuc:e et~.dosing tht

\'()lume defined by lhe s:urf!tce~ t

/ 4 - .rl - yz. t

= 0, :tnd it is

the un.i1 outer normal to S

#s

(:2x 2yi -

y2j

fi is the unil ou1e-r normal to S

* 9.

#s
+
#s(.r'i

10.

z? = J. and ri is tbc unit outer normal to S

+ .r'j

II.

j{ (yi Jr.~

xyj

2. and 1l i~ the unit normal to S with

= 6 - .c

2
- y 4

=/

x2

y 2 and ii

inner nom1al1o S

t 2 = 4, y

= 4. and

i1

is the uni1

J{ (x yi + z 2k) 0 JS, whe1c Sis th csul'faceend o.sing the voiJr.~

ume in the first octam bounded by tl1e planes z = 0, y =


2x X+ y z = 6. and is the unit omer normal to s
I

16. Shuw that if fi is the unit outer nonnalto a surfac.: S. then the.
rtt:ion cnclostll by

S has \'Olumc

1#,s

= -

r n dS.

oroo(S)

= .t i + yj + zk.

JJJ.

V. ii tiV.

" 18. I (ow would you pro'i\: A;hiln<c.::.' prin<;iple in the case that an
object i> only partially >UbiiiC!lled? (See E.\ample 14 .26.)

In Exercises 19-21 evaluate the surfac-e integraL

Hs [(x +

+ )"lJ + x' d(] ii tiS. where S iHhc sutf.ce<n


closingthevolumedefined by x~ +>.:2 - ~ =I . 2~ = :r~ + y 2. and

+ 19.

y)i

Ji

is the unit omer nonnal co

20.

#,

[(x

+ .r)'i + x'Jj - x'd<J ndS. whero nis the unit inner


S

enclosi ng the volume defined by

c. 2

={I -

.\'1 - 2y2)2
.. 21.

If.

[(yl - x 1y)i + (x 3 - xy1)j

* 22-.

+ z.k]. n dS. where i\

JL-

.-:2

Itn .

i> UIC

y1

is a region bounded by :l closed surfaceS. a n;.t B

If V

=V x A,

iJ tl = 0.

2J. IS Grt"Cii"S theorem rclarcd 10 the lli"crgcncc lhcon.:m'? (SGe Exercise l 3 in Section 11 .6 .)

In Exercises 24-26 assume tl><lt P (.<, )', z) a11d Q (.r, y, z) ha-e continuou~ firs t ~uul ~cunei par1i~d d~ri V:klives: in :1 domain containang a
closed $U.rfacc Sand its interior V. Let fi ~the unit outer OQnn:l) tO S.
24. Show th-at

# s'iiP iidS
12.

istt.upcn of ~

is the

+ zy'k) n dS . where S ;<thesu rfaceenclosing

the \1t>lume t.Jctioed by y 2 - .t

.r1 - y1 c u t out by l = 2y . ruul it is the unit upper nmnal to S

show that

- t 3k) ndS. where S is tl-.! surface enclosing

the ''olumc Ucfincd by l


unil ouler normal to S

.< 2zk) i dS. wl-.!re S isthat pan of the cone

unil upper oon:nal to the 5urf:.u:c S : ~ -

(yxi + y1 j + y<k) n tiS, where S is ore ellipsoid .r' +

J ?/ 4

is the unit upper nOm l!)f lo

/L<y'e'i - .ryj + ~k) ndS,wrerc

normal to the sur foce

+ .Sx~1 k) . DdS, where!' S islhe surf::tcetnd osing the \'Oiumc in the finn octant defined by y 2 + t 2 = 9. ,\' = 2. and
t(.

fi

it t1 S. wl-.!re S is ore <urface boundins the

= 2.\' 1 + y 1 x2 + )'1 = 3. z = 0,
und n is the unit outer normal to s

volume defi ned by

36. and

+ y z bclnw 4 =

S i> the

sul'racc OOundjng lhc voiu1ne inthl! fin)t lK.'tant defined by x = 0. ) =


0. z
I. z = 0. ,( 2 + y 2
I , and 0 b theunh inner nom\al to S

2zk ) .

is the: unit out~r norma) to a surfaceS that encloses :1 region


V, show th!l;t d~ nreo of S c:.tn be txprC'SStlLin the fonn

ii dS. where F = (x 1yi

j[ (Xi + yj +
11~

/r~

9zZ

'+ 17. If fi

#/"

* 6.

4-

31\d fl iS lhC unil OUier nonnaiiO $

4y!

i< the lop h3lf oft!-.! c!Hp-

2. arK!

4. j[ [(z' - .<)i - .<yJ + 3zi<J o dS, "he Sis the surracccn


Jfs
c-losi ng the \'Oiurnc: defined by z = 4- y 2 . x = 0. x = 3. l = 0 .

z=

zk) . n tiS. ~rc s

(xi+

positive c: --componcnt

a2 and fi is the unit outer nonnal to S

5.

fL yj +
+ +
JL(.t.i - y<J +

soid Xl

14.

the volwnedcfined by.< = 0. x = 2. t = 0. z = )'. y


ii is the uni1 ou1er normal to S

13.

x, y

w here

v-P

82 P

= --

a.r'

JJJ.

82 P

8'P

8y

8?.

+ --2 + --2

'ii PdV ,

What ('a n we conclude if

P (x. y. z) satisfies Laplace's equation in V ?

+ 21. Compare F.11:cr..:.ises 24- 26 with E>.t.:rciSQ> 35-~S in Stion 14.(1.

2S. Show dtr1t

f i P'\!Q . ll tiS -

Iff.,

(P '1 1 Q

TI1is rcsuh is cl!l!cd Grt?t-rr .t j ir.tl

+ 'V P. '\!Q)tiV.

+.+ 28. Lei

ill~uif!.

S be a c 'loscd su f~lt:c , and lc1 1l he l hc u1'lit outer nonn:.l to S .


Pc1

lf ro

= ,\'J + y(J+ zJ{ i$ lhe position voctor of son\C fixed !>Oint

~h()W

lh!ll

26. Prtwc t~u.

fi(I''V Q - Q'\11') fi dS

=IIi

irS dcx:11 OOLcnc.:lo!'c P0 ,

.fi ,..--_-""':~ . n<ts = { 0 .


J1slr - rul~
4rr,

(P'V' Q- Q'\!2 P)dV.

ir S docs enclose

t'o

"lltis res..llt is called Grcr:w '.s s~CQitd id~ntil).

l t4.1 0 Stokes's Theorem


S10kes"s theorem relate:-, certain line integrals around closed curve.s to surface imegrals over
f\,lllfaccs that have the curves as boundaries.
T H E O REM 14.8 (St okes's T hoor em)
Lee C be a closed. plecewise-.smoolh curve l hoc docs no1 imersccl itseH and let
liiC)ffii lll[fJ
El.emen
lllr)' surf*!"c: I'M proof or Slc):-cS,'i

S be a

piec-ewise-smooth, orieHablesurtace with Cas boundary(Figure 14.47). Lei F(x, y, z) =


P(x, y, z)i + Q (x, y, z )J + R(.r, y, :)k be a vector li el d whose componems P, Q,

dl~)!l:ffl

R have con1 i1tucms firNl parti al derivatives in a domain that comains Sand C . Then

and

fIs

F dr =

('\7 x F ) ndS,

(14.5lJa

or

Pdx

c=

+ Qdy + R d:

!1 [(I)R
l Q). (aP
- - is

8y

I+

IJ<

iJ R) .

- - - .I+
IJ<.
!Jx

(iJQ
- - ()P)
- k] n. dS,
ilx

8y

( 14 .'>\lh)

where 0 is the unit normal to S chosen in the following way: Tf when moving along C
the M.llf3CC S is Oil the left. lhen il lliUSI be Chosen as the unit n01'1'U3I Oil that side o f S .
On the other llnnd, if when movi ng along C , the surface is Qn the right~ then i1 must be
chosen on the Oi'PO')i tc side o f S.

P ROOJI' \Ve tirst consider o surfat'C S thnt p rojects in a one-to-one fash ion onto each of the
three coordinale planes. Because S projects ooe-lo-OI'~t: olto somt: rcg iut S. . , in th~;;: A') t plau:.
we can tak e the equati on for Sin the form = J(x, )'). I f the direct ion >Ilong C i &>LS indicated
i n F igure 14.47, and Cxy i~ the projecti on M Con the .\')' plane, then

J.

fc
If

Pdx =

J.

rc~'l

P [x, y , J(x,y )ld.r.

we usc Green's theorem on this l i ne i ntegral around c.()' W'C hnvc

J.

J~

P d.r

= j'f

is.,

- (iJayP+ ~pi) z iJyiJ z)dA.

On the other hand. since a unit normal to S is

az _az
( - ax
ay'

)
1

" = -r~~====~~==

(-axaz) + (-aayz)
2

1+

1G40

Qlapta 14

Va:torC'lknhtS

it also follows that

r(aP.

fI~

j'(

aP.)

ozoy

ily

azj - iJy k .;, dS = ls /I+ (~)2 + (~')2 dS


\

ox

j'(

ily

aP

() J> () z

az ay

ls., /

( az )

ay

( a~)

\ 1 + ax + iJy.
( i) P + iJ P i)z)dA

!'(is,, _

ay

ily

i)z

(a-)l+ (a~)2

I+ ~
a..

()y

dA

We ha,e shown. then. that

i
i
i
c

Pdx=

iJ P,) n dS.

!1 (a,
s

iJ P .
- _j - -k

i!y

By projecting C and S 01110 the x z- and yz -planes, we can show similarly that

aod

Rdz =

Qdy =

!1s (
!1s (ax

aRA) ndS
A

aRA

-. -J

- iiJy

aQA

- k

Q.\

oQ,) n.dS.

-1

az

Addition of these three results gives Stokes's theorem for F and S.


The pt'Oof can be extended to more general curves and surfaces that do not project in a
one-to-one fashion onto all three coordinate planes. Most surfaces S wilh bounding cmves C
can be divided into 11 subsurfaces S1 witb bounding curves C1 that do satisfy this condition
(Figure 14.48}. For each sucil subsurface. St<:>kess theorem applies:
IQI3 l'ii'35'J:rJ!II Division
of fi snrface into pieoe.s. each of
which projects 'mc-hH>ne omo all
ahrce coordjnate plane.<;

1. F dr=j'f cv x F) ndS,
res
ls;

i = l . . . . ,,.

If these n equations arc now added together, \ VC have

t{
i= l

F dr =

C,

t Jf
i= J

(\7

x F) ndS .

Si

Sinoe the S; constitute S, the right side of th:is equation is the surface integral of (\7 x F) n
over S. Figure 14.48 illustrates that when line integrals o,er the C; are added. contribut ions
from auxiliary (interior) curves cancel in. pai.rs~ aod the remaining ]joe iotegraJs give the line.
integral of F dr along C . For general surfa-ces that cann01 be divided into a linite number of
subsurfaces of this type. the. reader should consult a more advanced book.
X

Green's theorem is a special case of Stokes's theorem . For ifF = P (x, y )l + Q(x, y)j,
and C is a closed curve in dte xy-plane. then by Stokes's theorem.

Pcl.r:+Qdy =

aQ
(
!1
s

aP) k- n, c/S,

-. - - -. ilx
ily

where Sis any surface for which C is the boundary. If we choose S as that pan of the X)'plane
bounded by C. then ii = k and

With Stokes's theorem, il is suaightforward LO \Crify the sulliciency ha lf of Theorem 1-1.1.


Suppose thm the curl of a vector field f' van is lies in a simply connected domain D. If C is
any piecewise-sm001h. closed curve in D , then there exists a piecewise-smooth surface S in D
with Cas boundary. By Stokes's theorem,

According to Corollary 2 of' Theorem 14.3, 1he line integral is independent ofpmh in D. and
the theorem itself implies the existence of a fwtetioo f(.x, )', :) such that 'i7 f = F.

I EXAMPLE

14.27

Verify Stokes's theorrm if F =


z2 = 4 above the plane <: I .

\ Crlf'k.a

tioo of States's thccmn for the


pan of x: + >: + 4:! a .a ab<l\'t 1he
pl-:= 1

x 2i + x] + xyzk. and S is that pan of the sphere x 1 + y 2 +

SOLIJllO'i If we choose ii as the upper noml81to S. then C, the boundary of S. must be


ua,erstd in the direction shown in Figure 14.49. (If ri is chosen as the lower normal. then C
must be tra\'crscd in the oppo; ioe direction.) Sinoe pammctric equation~ for C 11 re

'

= .jj COS /.

.1'

3
' 1.11/ .
"r.:
j ~

= I'

0 -< I -< 2Tr ,

x 2 dx +xdy+ .xy: d:

F dr = i

= 1 1.[3 cos2 t (-../3 sir~ t dt) + .J3 eost ( ../3 cos tdt )!

27

[ -3../3cos t sint

+ ~(I +cos2t )]dt

= { v3r.: cos3 1 - 23t + 3 in4 2t } l.T = 3:>r.


0
On the other hand.

!1s

(V x F) . n dS = {{ (xzj -

Jl s

!1

x2~ _

yzj + k) .
,,2. + .

dS

.
dS
s _t.r2 + y l + zl

J/ss.

~ ff.s., z(x 2 -

: (x l -

iJ"ls,y
f
= JJ.
s.,
=

(2.\ , 2)', 2:)

J .Jx>... 4.1,> + 4>

yl
+
2
y2

I)

J
r

ti l +

(a
:):
( -11 ~) 2 dA
+
ax
ay
-

I ) /I+ (-x /z)2

+ (- y f: )2dA

z(x2-y2+ 1) /x l + y l + z> dA

(x - y

:.

l)dA .

1c>Q

Clupter 14 Vet:tOt' C3.h;uIus

If we use polar coordi nates to ev;~luale this double in~egral over Sxy : x 1

/ { ~

-(cos2 ()

-n
,T

I EXAMPLE

(9
4

C)

- sin 211
8

sin2 0)

+ -~ , ~ df)
2

+;3)
d i}
-

-cos 29

-n

39 , .
+ :._
2

+ l ::; 3. we have

= 3rr.

-,T

14.28

Evaluate

2xy 3 dx

+ 3x 1 /

dy

+ (2z + x) d z .

where C consists of line segments joinins A (2, 0, 0) to B (0, I, 0) to D (O, 0, I) to A .


W!Jil I V:"Uia SK-.kes's
thcurcm upplicd to !l Line i ntegral
aro11nd che edges of a criangle

DCO. 0, I)

SOJ .tmON By Stokes's the()rem.

fc

+ 3x 1 yl dy + (2z + x)dz

2xy 3 dx

ffs "

(2xyl,3x 2 y 1 , 2z + x ) . ndS,

where S is any surface with C as boundary. tr we choose S as the 11m triangle bounded by C
(Figure 14.50). then a normal vector to S is
BO x BA=

-1

k
I

= (1,2, 2),

2 - 1 0
'

and therefore. n =
....,
v

( 1,2,2)
3
3

. s.nce
2 2

x (2xy, 3x y , 2<:

+ x)

a;a.

8/iiz

2xy3

it follows that \1 x (2.xy , 3x-y- , 2~ + x) n

= - j,

2z +x

2
= -j ( I . 32, 2) = - -.
and
3

3
2
( -~dS
= - J~(area of S).
YJ.c2xy d x + 3x yl d y + (2z + x) d z = j'Js
J

But from equ ation 11.42, the area of triangle S is


I .

2180 X BAI = 21(1, 2, 2)1 =


Finally. then.

EXERCISES 14. 10

In Exen:ises 1- 14 usc Stokes's theorem to CI'Oiualc 1hc line imcgrul.

II. xyd.r-z.rdy+ .rzdz,whcn::C is lhcboundutyorlhatpilll

of z
.r + y in lhe firs1 ocmm cut off by .r + l'
counlerclockwisc as viewed from llle point (0, 0. 1) .

I. x'y dx + y zcly+z'xllz. where C islhccu" 'c z = x' +


y2. x 2 + y1 = 4, dircclcd countcrdockwisc as viewed from the origin

= I, directed

J. y 3 dx - .r3 tly "' x yz dz, when: C is the tlllvc x' + 'v1 =


J. y' clx + xy dy + xzdz, where C is the curve x' + .r' = * 12. ~

2.

2y. )'

= z. directed so that y inc;:rculics when x iS (>ositi"c

i +
+ y + z' =
+

2yz) dx

(xyz

3.

1
4.

+ (x 2 + .ty -

3y) tly + 2xu/z, where C i<

= Jx' + y', Z = -1

lhc cunc l

{x' dx +

y 2 tly

OS

viewed

the o rigin

ri'Qil'l

7.

fc zi dx + xy dy + (.12 + z2) dz , wl"-'rC C is the curve x 2 +

z1 =

9, y
the origin

= J x1 + : 2, dircclcd counterelock\\~SC as viewed from

+ z dy + x dz. where C is thccun'C x + )' = 2b. x 1 +


y 2 + z2 = 2b(:c + y)~ di.rcctctl. clockwise a..s viewed from the origin

8.

y dx

9. fc y!tlx + (x + y)d~ + y:ll z.. where C isthccur\'e.\2+y2 :;


2~ x

+ y + z: =

2, dircctcU c lockwiM:'"' ''icwcd from lhc origin

* 10. J. (x + rl' dx + (x + .vl' ely+ )'l1c/z, where C is the cune


h
1
with equations z =

13.

J. z(x + y) 2 dx + (y -

x)2 d y + z 2 d z. where C isthcsmooth

curve of interscclion o f the surfaces i 2 + z2 = a2 v1 + z'


0Z
which has a ponion in lhc first octanl, directed so that z decrca.~ in

* 14.

+ x 3y 2 Jy + zc/z, when: C is the cucYc x' +


x' + 4y 2 = 4. directed so 1ha1 x decreases along thai

,[ - 2y'x' d.r

y' + z' 4.
pan or !he curve in lhc first octant

6. fe y d .t + x dy + (x + v + z2) dz. where Cis the curve x' +


J 2 1, z .ty.direclcdclockwiscas,icwcd fro nllhepoint (O.O. I)

the firs t OCI<UH

+ (.t 2 + /) tlz. where C islhcboundflry of thc


Citsl octant purl of the plane x + y+ t = I, di~ct.cd cou.ntcrclockwi.sc

5.

Jx' + y1 , directed clockwise as '"cwcd rmm the

+ ,tz dy + 2xy t/z, whcrc C is lhccu"oe z =

4, direcled clockwise as 1icwed from 1be origin

y) dx

(2xy

z' + 3, z = 3 -

origin

Jx' + y', (x -

1)

+ y' = 1

15. E'-aluatc tbefu1e i.ntegrl

J. 2x'y dx -

y;;dy + xu l z. when: C

b thccurvc x' + y' + z' 4 . z ./3(,r' + y 1), directed clockwise


as viewed from the origin, in four ways: (a) diruclly as a line integral
(b) using S1oke.s's lhoorcm with S as that p.1n of z
/4 - x' - y1
bou nded b y
(c) using Stokes's th(.'Qrcm with Uli dam p.111 or 4 =
/ J(.r' + y') bounded by C. and (d) using Stokes's theorem 1\ith S
as that pan or z = ./3 hounded by c.

c.

z'

16. Lets, be lhat p:lr1 or.r 2 + 1 +


= I flhOI'C the .T,Vpl:lllc and
,

S, be that pJJI of z L- .r - )" above the xyplanc. Show thou


if lit and liz arc the unit upper nomtals to these surface~;. :md F is a

vcelor field defined on both

fIs,

(V x F)

s, and s, . thcn

notiS =

fIs,

(V x F) . ii 2 JS.

lnfonnotion o n Flu and C irculalion and \'ector Analys i< in Orlltogonal Coordinates
has been placed on the Text En.richment Site fo r this 1ext. r'Or fu rther infomtntion on these
topics, please visit wwqJtmrsoned.ctiltextltrim.

SUMMARV

\Vhen a vector is a func tion of position, il becomes susceptible to the operations of di fferentiation
and integration. In this chapter we developed various ways of differentiating and integrating
vector fjelds begi nning wi1h the openu ions of d ivergence a nd c url. The divergence of a vector

(ield is a scalar field, aod the curl of a vector (ield is another vector field. Both areextremely useful
in applied ma thematics. Mathe matically, the cu rl appea red in our disc ussion o f independence
of path fo r line integrals and in S tokes's theorem; we saw ilS physical importance in our s tudy

of 6uid flow and electromagnetic theory. We introduced the divergence of a vector field in our
discu:ision of the cJivcrgcnce thc::orc.m and in the same application~ as those for the curl.
The line integral of a function f(.t , .Y. :) along a curve C is defined in the same way as a

definite integral, a double integral. or a triple integral -

that

is, the limit of a sum,

The most important tYI>C of line imcral occur~ when /(.t. )', :) is the tangential component
'
'
>C
or a vector field F = Pi + Qj + Rk defined along C. and in this c:Jse we write

[ / (x.y. <) ds

=[

F dr

=[

Pdx

+ Qdy + Rdz.

We developed ~tree nt<:lhods for evaluating liote imegrals:


1. Exprc.ss all parts of the line integral in terms of an} parameter along C and evaluate the
re~ulting definite intcgml. All line integra'ls can be evaluated in this way, but often methods

2 and 3 lead to much simpler calculati()nS.


2. If a line integral is independent of path, then we can evaluate it by taking the difference in
values of a function 1/> (where 'il</1 = F) ill the ends of the cune.
3. If C is a closed curve. we can ;ometimes use Stokes's theorem to replace a line integntl
with a simpler surface integral. In this regard, Green~s theorem is a ~pecial case of Stokes's

theorem.
If the line integral of a force field F is independent of path. theforceficld is said to be conservative.
Associated with every con;;ervative force field is a potential function U such that the wort< dnne
by I' along a curve C from A to 8 is equal to the difference in U at A and 8. In addition,
motion of an object in a conservative force field is always charactcrilC<I by an exchange of
poten1ial energy for kinetic energy in such a way lhcu the sum or 1he two energies is ~1hv:1ys a
constant value.
Surfac-e int.egmls are also limits of sums.

and the most important type of surface integral occurs when / (.r. y . z) is the nomtal component
of a ' 'ector field F on S:

Jfstcx.y,z) dS= Jfs f' ii dS.


\Ve sugge!'ted 1wo methods for the evaluation of surface integral":

1. Project S onto some region R in one of the coordinate planes. express oil parts oft he integrol
in te.mos of coordinate,s in that plane. and evaluate the I'I:.>Uhing double integral over R.

S is clo~. it could be adVllntageous 10 replace a <urface intcgml with the triple integral
of 'V F over the volume bounded by S (the di\'crgclc-C theorem).

2-. If

In the li nal section of Lhec hap~er (see www.pearsoned.ca/Lextltrim), we leanoed how to express
scalar and vector functions in orthogooal, curvilinear coordinates. including polar. cylindrical.

and spherical coordinates. \Vc also developed fonnula.s for the gr.tdicnL, divergence, and curl in
these coordinate systems.

KEY TEAM S

In ...,,;ewing this ch3pl<r. you should be able 10 define or discu s IM followmg key tenns:
Interior point
Extcrioc point

Boundary point
Closed set
Domain
Vector fields
OiVCfl:Cil<:c

Open set
Connected set
Simply connected dom~in
Del opcrntor

or" VC~tor focld

Equation ol' COillillloity

Curl of a vector tic ld


Con~rvou ive

l rrounio11ul vecto r ticld


Path independence of a line integral
Law of conl\.ervation of energy

Grcon~s

Surface imegral

Line intc;_mls
fnrce 11eld
theorem

Clo>ed surface
Divergence theorem

Oriemable surface
Stokes's theorem

RCVICW
EXERCISES

In .. '(m:JK'C 1-10 c-.akulalc th~ quamny.

VJotJ(r.,l,t)= .r'r'-xy+:

1.

!. V F If t'(x. ,r)

= .r'.vi-

F if F (A, ,1. t)

17. f /'YJ d.r + (3.r 1y' + 2.n)dy, where Cis the cunc (x-

xyk

1)2 + .12

= (.t + )' + t)ci + j + k)

5. V f off (.r. )', :) = In (x' + y' + :')


6. V F if t '(x. ,1'. :) =

7. V x F of F(x. )', :)

IX.

,-,i + :'1+ u' k

vI

9.

v F if F(L ~)-

=xyd
19.

+ ,;
d- (1' + z')j + ''z'it

20.

12.

d$ . w i'O;;O'C C

(.t'

i>tloC CUI\C ,\'

yz) tiS.

= X l rrum (-I, -I) IU (2. 8)

~hereS i>that part of.\ + \' + z- 2 in tho

nNottant

IJ.

1~.
.t

Jicxi+yj).ridS. ......,..,sis o1ut p:ut of:= x' + y'

Ji (xi.+ j )
1

1. y = 1. a.'l<l oi i the l""oc'

)' = l

+ )' dy- z2 dz.

(x 2

z'k) . n JS. whcr<: sis the sw-fOoCC ~

JL + .'')
(x'

where Cis the curve

x' + y' below

x~ + y~

0 ..t

d S, wb<re S is lhoo prt of.<'

I. z ;. 0.

+ y1 + :' = 6

z = .r 1 + y 2

J1

(.r 1 + y')i it tiS. where S i<tiMI p) rl t>l .r' + y 2 + t'

~ x1

y 2 and

(.tyl

I.

+ tJ - X 1k ) tfr. "here C "tMcunc on Ell:n:iS< Zl


+ ltdy- 3z 2 tll. where C iotthecunc

= J I + zl -

x2, .r 2
viewed from the origin

=6

nl~ the upper nounul t.o s

24. iJcl:c

1.

(x'l + ) j - xtk) Jr . whco'C C i tloc cwvc ~' + y 2 z = x , 1. directed clo..:t-.wi...c d' \iC\\'CU from the OOtin

lJ.

ii d S. "1lcrc Sis olut p;ut or:

JL i + y'j +

in..)it.lc t

, ,,......1

I. al'kl n i' the 1\.Mcr normal

IS. ix tl.\'

21.

+ 22.

bowldal b) the .....r....,. x -

f c l.r/ dx + (Jr'y' + .r') d ,l '. "here C i<rhee~one (.r-1) 2 -

in,.idc

In Excn:i,._,. 11-30 cv-olluott the integral.

l )'
JL +

lhc: \ ()lume eoc:lo~t.l by y = t. \' T .. - 2. .t


and n is the oooet normal to S

10. V x f if l'(x. v. t)- Cot- (x~t)i

II .

)'l = I

of J c. I ) - Son- (!

8.

x'

/2

<x'/y)j

J. V x f' if t'(A. y) =;in (xy)i +coo (xy)j +


4.

+ xtdt . when: C i<th<:<u"e v - Ji"'+"X!. t =


.', from ( 1/ ./2, .fll!, 0) to ( 1/ ./2. ,fJTJ. 0)

xy dx

16. [

+ zl

I , diroctcd <.'Quntcrtlockwise !IS

1046

Chapter 14

VecwrCn.lculus

25. J. (xy+4x 3 y 2 )dx+(z +2x y) d y + (z 5 + x 2 z2 )dz, where

rc

between the planes z = 0 and

C is the curve with equations x 2 + z 2 = 4 , x 2 + y 2 = 4 , y = z.


directed counterclockwise as viewed from a point far up the positive

<; axis

26.

Jis

z= I-

*
(x

yzl - x yzJ - x.rz k) ii dS, where Sis that part of


2

Jx 2 +

Jis

S is thatpanof z

=x 2 -

y insidex + y

y d S , where S is that part of x = y 2

z=

=1

2, and ii is the lower normal

31. Le.t S be that part of the sphere x 2 + y 2 + z2 = I that lies above


the paraboijc cylinder 2z = x 2 . Se.t UJ>, but do not evaluat.e, double
iterated integrals to calculate U1e surface imegral

=4

+ xle'Y)dx+(xe'' + x 2 ye'Y +x 3 y)dy, where

C is the curve with equat ions

by projecting S onto (a) the xy-coordinate plane, (b) the yz-coordinate


J>lane, and (c) the xz -coordiJlale plane.

I in the fust octant

which is under x + z = 2

* 29.fc (ye''

y2 above the xy-plane, and ii is the upper normal

27. Jisd S , where

* 28.

30. Jis cxi +.rJ)- n d S , whereS is thatpartof z - x - y

x2 + y 2 = 2y, z = 0

*
*

32. If r = x i +

.d + zk, show that V (l r l") = nlr l" - 2r .

33. Verify that '17 x ('17 x F)

82ji)x 2 + 82!iJy2 + 82/ oz 2.

= '17('17 F ) -

'17 2F , where '17 2

CHAPTER

15

Ap pl ication Pro:'

i~:\1

Differential Equations

1be fi&ure on the lefl below show, a car moving along a road reprc>entcd b) the x-~IS- Unfortunately. many~ arc not flat: they ha\e undulauons lhat cause the car to osci11at~: venically.
To reduce or climin,\le these oscillation~. there i<a spring and" >hock obs<>rber tubo known a;
:1uushput). These are ;hown schcntkully in the ligure on the ri~ht.

>

>

Tilf PRORITM
Gi,en the eqtc'ltion of the road and the spee<l of 1b~ car.llctcrminc an
equation defining the o.<.eil~11ions of lhe from end of rhe car. (For rhe <;OIUtion. see Example
15.2'1 oo page 1100.)

Differcmial equations serve a< models for many problems in cngincc1ing 111ld physics. In
this chapter we <liscuss some of the mclhoci> f()( ><>h ing tir>t-()(di:r and simple >CCO!td-ordcr
equarions. We also give a fairl) rhorough treatment of linear differenrial equ.nions. We include
a wide \-ariery of applications tO illusuare rhe rtlcvanee of differential equariools in applied
nmthenuuics.

115 .1 Introduction
f\ differential tquaclon is an equatron rh~l mu"

be solved for an unknown funtllon. What

d1Minguishes a ditTc:rcntial equation from other eqmuions is lhe fact thdt it contains at lc.ast one

derivative of the unknown function. For example. e;~~h of the following equations is u di ffcrential
cqu~lion in

~ a ti.u-.ction of :t :

dy

dx

-+-i=9.81.

d 1 1

dy

dx

dx

x - 1 + -

dy

dx4

11 (dy)
-.
1

dly

-=k
d ... ~

t 15 II

dx

s ,,

+ xy

0,

(1

-k"y =

o.

(1 5AI

1047

Eqt~a l iOtl

15.1 can be used 10 dctcnnine the po.si tion of a skydiver who fa lls urlder 1he influences

or gravity and or ~Jir resist<lllCe. whkh is proportionaJ 10 the square oJ velocity (:sec Exampl e
JS.'J): equation 15.2 describes the shape of a h~:\1\gig cable (equa1ion 11.49 in Seclj()J\ I L .S)~
cqu:uion 15 . .3, vnc.-:ofBes.scl's din'crcnialcquations. is found in h~at flnw and vihnuiollprnhlents~
and cqu.utio n 15.4 i~ used to decermine the detleccion o f beaml).

DEFINITI O N 16. 1

The order of a dJO'e rtntinJ eq u_a tion is the order of the highc.st dcriV'J.tive in the equation.

S~;hcrn;,tlk

- - , - - y =O.
t =

0.

or the four difl'Crcntinl CQlllttion:; 15.1-15.4. the first is first order, lhe second and thirU ure
second order. and the b\St is Fourth order.
'Ve have <xJrlsiOer\:d 'luite a number of d iffercntit'l cqu.c:1tions in Chvptcrs 3. 5. and 8. fn
Section 5.5 we deal& with seplrabfe ditTcre.ruiul cquntiOI'\S; irl Ctlsptcr 3 we verified thm pur~
cicuJar combinacions of cransccndcmaJ functions iHtlisficd <:cnain di ITeremial equations; and in
Char)(er 8 we used out illlcgrutiorl icchrl iquc:s to solve mnrly scp.nn,blc cqm1tions. Ahn-o,.;,t ull of
lheseditTercmittl equntions were bnsed on applications. most from physics ami engineering. btll
also some from geometry a1ld other fields such as ecology. che1nistry. (l rld psychology. Jn appliCHtions, dintrcntial equations arc. almost al..vays nccompaniecJ by subsidiary con-ditions ca lled
initlul or boundury cond itions. For exumplc, .suppo~ a muss m . while ~ i nk irl~ i11 wmcr, is
acted upon by gravity a nd a force due co w,uer rcsisuu1cc that is propor1ionalto its instantaneous
velocity. rr we c hoose di~w r1cc y :l~ positive dowtlwMd, rakitlg y
0 at the surface of the
water (Fig ure 15.1 ), then the d ifferential c<1uation dt:1t descrilx--s the velocity u(f) of Ihe n1ass
as a thnct ion of rime r is

. 0

dv

m d r = -kv + m g.

whcrck > Oisaconstantand g

= 9 .8 1.

H Sl.:tle !llCIH or Newwu's .wu:m ul lmv, where tlv/dl is the \'Crticetl ..:omponcr\l or
the acce leration of m , and - kv + mg is rhe venical component of [he Iota I force on m due.to
gmvity (111 1:) and WHtcr resistance ( -k v). If the mass is released from rest (i 11 the surrace) at
time 1 = 0. the conditio n v(O) = 0 must be added to the differential equation. Jn other words.
the real problem is 10 find the sohuiorl of lhe diftbrenrial equation th.i:lt a lso satisfies the inilhd

This is simply

)'

cond ition:

du
111 -

dt

-kv +mg .
.

v (O)

0.

(! 5.6)

This is the form in which apl>lied nlilthernaticians use ditl'eremial equations - che di l1"er
encial equation is ~ccom pan ied by subsid iary oondicions d~u express ex tra requircmcms of the
solution. fl is !lOt d ifficult to show that the soluliOil <>f equion 15.6 is

v(t )

=--e
k
k
lllR

mg - kJ/ m

(! 5 .7)

(All we need do to veri f) this is substitute lhe function into the d ifferential equation to see that
it d()Cl) inde-ed satisfy the equation. It i~ clear hot itlkX.os ~(Jli .... fy ahe i1li1ial cu1ldiLinn.)

If we change the initial condition to v(O) = uu~ so that the initial \'elocity of m has vcnical
component v0 clS it enters the \Vi1ler, the n t he ~llu l ion becomes

u()
t

mg ("'g
=---v )e
k
k
0

- kt / 111

In other words, evc:.ry ~olution o f d ifferenti al eqmuion 15.5 c~n be wri tten in the form
( 15.9)
and when we impose the initial condition v(O) = IJ(h then C =

v0 - mg f k.

1." It 0:10\ill.lr W-'}' j:qullli''" 13.<1 fo)r \HU.n\ (!,cfte( t\('>l'UI \ii O\\)OW~\1 )1 to.to)mp;on.,((,\ 'oy (ol.l r
l.)(tO.u'H.lllr)' .:~otdil ivo.u t h.ll\ llpco;i f) I I'M:: IYfl<l'~ ()I' *-'"""""'~ ~~~ 1.~ C">"-b uf the. bo::lm\ t.~eC.$ccl\()'ol ~ A') .
rbf s imple S.Uf!l"l"'"" 1111 I)Q.It\ X = 0 ar.cl .o.: = '- .\' ( X) Tl\ll~l S.MIU'y'

y(t))

y''(O) -

)(t... )

= (),

y''(t.)

=- 0 .

whcrc.C ,. C :. C.,. al'ld C,,tore:atbilr:lryeon;;.t:i\nt..,. , andw~nth~bo\lo)(l.:\(y e<M-.dhi.()Mane 'll.pp\\t.J. ,


lhc.-.c COt\!il.iU\U n\U~t ~i..'\l)

the four

i.:qiiMh."lll:O.

o = c1 + C2. c,.
0 = c,~"' + Cz~-u.. + C, :sin(kl...) + C,.c<(kL) .
0 = C 1 + C 1 - C.\.
0 = C1ru. + C!,.-+t..- C:po\n(kf..) - C"'<.o:.(kL) .
We have Mated thnt ev(!f)' sohl(klll ot \S.S can b wri.\le.n i t\ fOfm \ 5.9, :uul t:o.-ery M'!\uth' '' tlf
I S.4 ~an be '-!.XI)I'c:MC:IJ ~ 1S. \0. No u: lh:u I S .9 1..'\l i\U\i n ..- '-~~~ tll ti. tm t)' ""'"'' s tmtl wh~:t'Cl\~ \ ~ . \ ()
h ~s four. b ut i1l bOth <::I, St~ the: number of arbiu.u-y <:C3n!'.tants L~ the Soal't'\C M t\"1~ ()\et ()\" t\\~
d ifiCrcut ial c qmu i on. We m i ~h\ 8UiliJCCt lhM C\ Cty ~oh.1l.\\m o f an n o.t>wo..ud et dit\"eu:-1\\ia\ equ..'\\\cn\
c :1n he CXJlre"....._'(lns ~~ l'tulc tiol'l i1\\ olvhlS tt lt:tb\trmy trn\s.t~\1 \U , Fllr m\'"'Y l\\ \Yi.'.n.~_m\11\ t..'<\\l\1\i'"'s
thi:t itt indeed true. but \lnfQt'\m\MC-1)' it \s 1lQt \f \I C f()f t1\\ \Ull\\o-,s, i\v. tu\ \\\u'An~\t)l\ , c;..,)""\d.c.'
the equation

d
= (d))'
.
dx'dx
1

)'

II~

Ill

In Exat)tple 15.6 we apply .standanl techniques for M)\ V\ 1\~ dHfercn(i.:.\\ C(\\1'3\i()l\1) to Qb\~i'' \\\e
so lution )1(.\') = C 1 - h1(C1 + ;~) . ' vhih C(HH<\ins. two amhrO\l)' constant~> C 1 ~\1'\t.\ C"l.. '\'h\,:.
nvo-pammctc r family o f solutions doc~ not, however, lft\t\i l\ t'l\\ oo\t~tiOt\'iL (l( the UlHc:c"t\u\
ettllnlion. for no ct\Oicc of C, itnd c~ wtn g.\ve th e perfectly nccept3b\esu\utiu" y(.'() \ . Th\s.
solution is t'IOt 1~\l'ticuh\tl)' it\tcresting., t-.u1 it i ~ n ('tt\elhelt~Sl\ $'-ll\.1\.io' ~\'U.\\. is i\Q\ <;Qfl\1,\\ntU wit\\h\

the twoIXIidntcl~r fcun ily. Such a l>olution is <:-a l\cd a ~\nt;:,ub.T sn\utlcm (or the \\'lt~p~\C\\mC\C.f
fatnily.
We huvc illuMnnW tlmt a ~ol\ltiun th4\l oon\ains th es\\mc m1mbtr of ~\tbltn.wy tO\\~\tm\s ~~
tht OttiC:l' Of the di rftfCiltilll cqu:\l iOll nll\)' Of mfi~ IW COI\ t.\\1\ ~\\\ S.u\\\\;01\S (\f \\\(! t.,\\ffctt1\\\~\

equation. In

~phc

or this unfonunatc

eirc.um~Ul.l\tC .

thc.r<:: du

C.:\'~

large

c-\a:c:~

()\ UiXfc.rc:\\\ta.\

equl\tion~ for which a solution with the ~\me number of arbitn\ry 001\S\t\1\t<.:. \)S \''le otU.er ot t\\e

equ :nion doe:L ildeed rcprcum

~ n llo~~:;;ibl e

so\ \1\iOM . B e<:nl.lse a f

thi.~

wb

m\\~e

t'f\e {()\\owi'''S

dcfi ni lion .

DEFINITION 16.2

An ll pllramctor fllmily of 110lution of"" n"' ordcr differential CQ.Uation i >t1id to 'oc 11
jiCIICI"UI SOinllon if it CI~Unin' Ull $olutilm~ of lhC \lif!cn:nti\1\ Cl\1111\i\lll.~
t ln this ch,apter it is frequently con\'enient to use the notation y', y", l'',

... to l"('p-e&ent d yfd.t. d 2yfd.r'l .


d 3yf dxl. and so on. In this notation y" (a ) is Lhe second deri\'ati\'e of y evaluated at x = a. In ~ddi1ion, we
de(l<)re lhe :toluliO(I ofn cii{fcreMial cqualiO(I in y a:t a fu(lct i O(I of x by y(x) .

R~de-s should be aware rhat (I<..U all nul hOI'$ ngrcc <.ln chis cictiniliOI) <.lf n gcoeml soluliO(I of a di ffcrcmial cqunli<..l(l.
Some do ooc requi-e-a geneml solution to oontain all solutions of tJ1e differential eq u~tion .

Conse-quently, in order for a function to be a general solution of a diffcrcncial equation. lt


must. lirsl. be a solmion : se.cond. comain the requisite number of arbinary constams: a11d third .

eomain nil solutions of the differential equation.

I EXAMPLE

15.1

Find a general solution for the differential equation

d'

y = xe-.

dx2

SOLUTION lmcgration of both sides of the differential equation, with integration by parts on
the right. gives
dy

dx
A second integration yields

y(x)

= xe- + 2e-' + C1x + C2 .

IJJclusion of constants of integration assures us that all solutions of tbe differential equation arc
included in this two-parameter family. and hen~-e we have a general solmion.

There is no proc.e dure that a_lways detemt.ines whether a_


n n-pa.rameter family of solutions of ao
u tllordcr Ui fTcrcntial equation i:s a geneml solution. It may happen. as in Example 15.1. that

the method of arriving at the ll pararneter family or solutions guarMtees that all solutions are
captured. Although this is the exception rather than the rule. there are dasses of diO'erential
equations for which an llparnntetcr family of solutions is automatically a general solution. We
shall cenainly point these out.
h is straightforward 10 illustrate that y = (C 1 + C2 x)e- 2 ' + l/4 is a two-parameter
family of sohnion> o f the diO'c.rcntial cqumion y" + 4/ + 4y = I. Allhis lime we cann01
be sure that all solutions of this difi'erential equmion can be o btained by speeil'ying values l'or
C 1 and C 2 and hence we <11111101 claim to ha\'e a general solution. We shall be able to lio so in
Section 15.8.

DEFINITION 15.3

A particular solution of a differential equation is a solution lhat

con tain~

no arbitrdry

constants.

I! follows. there fore. that particular solulions can be obtained by assigning specific values

= 5 - ln(3 + ..:) is
a particular solution of differential equation d 2 yfdx 2 = (dy / dx )1 as is y(x) = - lnx.
both being obtained from the tw()oparameter ramily o r solutions y(x) = C , - ln(C2 + ,t) by
specifying ,aluc> fo r C, and Cz. On the other hand. the: solution y(x) = 10 i> also a t>"'tkular
solution, but it cannot be obtained from the two-par.1meter family.
to the arbilr.uy comaants in a fami ly o f solutions. For example, y(x)

I EXAMPLE

15.2

Find a particular solutio11 of the d illcrentinl equation

d'y
5dX j

d 2y
+ 3d X2

+ 2y =

4.

SOLUTION In Sec1ion 15.9 we develop systematic techniques for lindi ng general and partie
ula.r solutions for differen1 ial equal ions such as 1h is. Bu1 clearly 1hose technjques are oot needed
here~ a simple glance tells us Lhat y(x) = 2 is a solution.

Some differential equations are immediately w lvable (or, as we o ften ~ay. irnmec.l1iuel)' il\tc
grablc). For ex;;u1tp1e, 10 solve a d iffercntil:ll equatiOil ot' th.c form

dy

-dx = M (x).

(15.121

where M (x) is given. we imegrate lxM.h sid es o f the cquat i ~'ll\ wi th respect to x:

)'(.<} =

M(x ) dx +C.

Bcc.au:o:c the right side represents a11antiderivativcs of Nl ( X) . this i!i. a gcneml solution of 15.12.
For the lith--order equation

d")'

- = M (.x ),
dx"

,,

(15.1~)

a positive imeger,

we itltcgnuc suc~sivcl y u times to obtain il gctle-ral solution


y(.r)

I ... I

M (x) dx .. . tlx

+ c, + c l .r + .. . + c.x - .

( 15. 1S)

E XE R C I SES 1 5 . 1

l_n E:<crcisc-s 1- 10 shQI.v thttt each funt:tiun in the ramily sati$1lcs the
dirYcrcnlial cquat.i ~.
1. y(.<)

_ .:

dr
2 + Ce ' : -'- + 2.r)'
<l:r

,..

dy

x' -dx2 +x d~x + 4 v=

= 4,1'

$in.t

10. y(x) = C1 r-

\'l
~

3. r(x) =

'

4. y(.<)

.
-x' + Cx'
2

S. J(.t ) =

~C,e'

<X
1

d )'

dx

r: :

V -"

I) "=o

In E:ccrcisct: 11- 14 find a particulusolution tlfthcdi iTI!f'Cncia1<x1uatic_ln

d'.r
dx' + 9y = 0

(tl'y)
+ Cz: d.r

= I+

~~~

( (/ ' ) '

, (../2X) + C,r,, (X../2)

6. y(.r) = C,e' <:<>S tl2y

d1 (

COS.\'

: x ,tlr
- .,. (2- 3.r 2 ))' = .r'

=C, sin 3x ~ C, cos3x:


c z,;z. + I

+ C,

.r! d- )'.. + x ~ + x2
ilx

1/r !

..{X

2. l'(,t) = - - : - ' -
I
C.< d,r - x'

.
1
9. .r(x) = C 1 cos(21nx)+ C 1 sm (2 1nx) + -4 :
d z,,
dv

sin

in Exercise 4 th:u satibfK-'S thC(,."'Clditions.


II. y(O) = I . y' (0) = 6
11. .r(O)

= 2. y (n /2) = 3

13. y(rr/12)= 0.)'(rr / 12)=1


14. y( l) = I. y (2) = 2

2 - - 8- +<>1 =0
dx'
tlx

1n Exercises 15-19 find a ;cneral solulion fC"'It~ diifcrcmin\ equation.

7. y(.r ) = C, clls2x + C, sin 2.< + C~cosx + C,sin ,r ;


II')'
tl'v

15.

dx'

+ s-
+ 4v = o
dx'

8. .)'(X)=

(cl + C,x- x' ) e";

d 2y
dv
- 16....:...
dx 2
dx

2-

..

+ 32}' = -e4.\

dl'
tlx

....:... = 6x2 + 2x

16.

I
d ,\'
dx =9+ x'

t/ 2 v
17. - = 2x + el

1 ~.

dx'

d \ y

19. -

dxJ

=3x'

t/ 2 y

ox

=X h\ .'C

lO.

hl A boo) i~~tiiQI:ty "' 0 '" f t tJC ~cw w.a'.b dona 1hc


~of :1 <1odml!'llrJ

lS.

p._.._,, ( .._.

YU .. I IO'I'II'inJ t.s Ailbruit


b) i'tri"' ol k"S:Ih (., If I he' ~ M1.11t 1ll Q W\11 lhC'
liLrinJ Wwar' ~INII'I!oi,;tiUIJht,
11U thc(uWon uf(he
aJr"\'ed !XJ:h .'' y(x) f~l('llo.\('d tly 1hc hntlt n..,;t .-a1isfy
1h1.1 tllfl'crct~iuJ 4."C;JU. Ililv'l

'"i.'M'

t) l>t'aw dte Orr:'*puOIIut:lf:l' ...aaly


"""'on m E~cr<Ue 11.

ol u.r~ oklnd by Ole

Ib) ~I)OWlhal"""th th;.:cw.;....,..,ioo of1W'h""('1Hhc: \'-.'\I"J..1betef!i;


l\ !.Oluht'll'l r a,.,;'inl tbrouiJ' M\)' l''<'" 1)~1'11 ;,, the x) flhn.:..

--- ..rrr--;r
.x

tly
dx

(b) Solve I hiS i.lillcn:lldllll CCitU... It\1'1 f01 )'(X).

'

tr.) Hnd a ~' ...~ioo lha\ q,,, ,~, the: o.n.litiM v(1) -

">hn.J a

'OOo.'-.tlOft thM. u luliu ~- Qll\Jli.I.IX\

y( -ll

t<) H nJ a iO\uboo P.hJ..l~(Q d'IC ~~;ondl\~' ,. boO.pau (a)


tllld lb).

..

L:d$.U uf

ttwimntinll

u.

p~uys a.movk.
c.ouer (~,~ulll'ai set equal 10 'IJ::IO
the tnO\ ~ bc,l"WS a1 lime r O l co'~" tht n umber n '' I ,..
t'lltthUfll> or 1hc 1!.1.lc11p 1'CC\. ' " lhl .. C"-cr(\'(: we tk\d4_)1) A ft'IMI\U\u C
rn

1'7. ""' u VCR

'"-hen

j) (tl,l l

11(1).
(a) 11' O(t )

2 1. \;fk'"- lh.uiJ'(.X) c - (\ 1 C ) ' ~ao~txvt o~er (~:;m,lyot


Mllwlion' (or the difl~nt1al cq..a10n

"' =
d~

2.;-.

'

J.,nJ sinEU!us;olation.

ll. "-hllwii\OW:)(f) = 1 - (A'_. C) 's&{W'Iep&ramctes i'Jn"Miyol


for the differcniJal ctf'IOJ!It'ln

'~ ~ nlot lht 11"t,k: thh'lllfll' wt1ich 1he t~:\.cu.l) rtd


(1\_'ltl\ hll ~ I-(), 11"'-t\ It - 0/(!JT), \1\.WI fJa\1

hr,UiO rQt~ata.J

Ihe t.lkeup reel t'll.atc .. dtptMI., '~t he: atTW)td or tupe oo


the. reel, l.1'le m.on: \UI"'IC 0"'\
the t>\\)'lflocr Ole llft&ul..
rotaliOf'. ~ ""(: 1<\ l'(f) b< 0\.:: w.-....._~:e ff\'ll'n k
c.~U.J~t; ot ~ "~ \1) tk- P"'ifll , , ~c \he \.ape .)(111"18 \he
ree-lal timet (fitc\lll: be""'"'). Sh""" ttlat if 1 i' the c:(JI'\!!Aaftl
~ "' ..~~~;h 1N: ~ pa~..c-. 'tcrooh the heod(t.l ('o{ h
VCR. \l'ICJ'I dlt/dr ~ /(~;..,.) ,

w;: '"'

lliOiuli ('M

til'
~

d .T

= ) \,(\''

1)!.

Hl'lOi u ~in-aut:,Jr t-Oiu1ion.


,. 'lJ.
(a) Verify l hSt ,\'(X) Ct!J.. i~ n oncp.aran'll.:t~ runuly of
liOhlliOnS for dlC difl t.-rclllill1 C'4WII10n

Ih) n-,c W"c:t of the l"U:\1"-lh' '-.ccuricd b)' tbc tnpc ul timer tun
l'C cft\co\nlcd in two wuy~ ftl'!. l, il h the di\Tcrt.ncc 1n the
urea~ of lwo drclc.... n (1'1 r~). '"-'1C1'C r" is the tat.h\1~
\'Jr the l.a!)C at 1 (). ~cemu\ . h \1> the .,..idth u J of tht
l3pc multiplied t') lh~ h;nt th llf tap; p\.teed on \he roe\ b)
l!n'lC 1. nan'lcly ll . U"C this to ~how t~t r1{r) s:t~~~ \he

dlff<:nnti.al cqu.t\loo

d_v - l \.

d.T

,.,, l lro~.w the OllC-paramtlc:r flftltl) or c,. '\lr'\'d tk."tlned by l!.\


'OhiiJOD.

(t) Show thst 1hcre ,,. a l).tth(ul...- Mlh.IIJOn th;~t passesthroulJh


Wl)'

gi,"Cn point (x,. ~0) . and tht.t thi.s sotuc~n can t-.c ot-,.

lllincd by choosing C ~ I>I,OPI'ihL c:ly.

24.

(<.:) Solve 1~ dillcn:.l'ni,,\ C~lU<\Ii on in p:u \ (b) U.) ll nll the dc:.\\(ll\

(a) Veri f)' 1ha1a onc-p.munc1cr ftt tnil)' ~,r sol\ltions for lht di f
fcrcminl cqumion

&.::1\t.'C of'' on 1.

til
tit

'!x .....!... >'

is defined impli<idy by the: equation y' Cx.


(b) Draw the one.panmetcr family or curves defined b y 11us
equation.
(c) Show tllat with the cxccpc.on ofpoims on t.be y-axis.. there
is a pruticular solu1io.1 lhal JXJS-10C Lluoogh any gi\'en poinl
(x0, y0 ) . and t1mt this soltllion cru1 be obtained by specifyi ng C approprialcly.

Head

Incoming tnt>c

1 l 5.2 Separable Differ entia l Equations


~rur~~ difrtrcni<ll rquoo ._..~ di"$CuCCIJ SeatOn 5 .S

hcf'C. ~vn& _._hJ~,IUI)

tll()lrc

\V( rq"''dv

C"llhu'e tre.llno't und " l'll"UI~f'ICe

,,rexen.."i11.er.

" 'c COilStdcr t'nl)' fi"-lu \ler tlifl"erc\t..ial IUUiulnlio l1U11 4:111\ 1)1.: wriucn it'l

~ ~.t.eq1;o111

lhc fur m

dy
dx -

F'(.t, Y).

M (.<.y)

F(
Ly) =

II'(.f. ) ) '

equn.ciM IS. J6'"Mnats.obe wrluen ln theequlv.l1.etu lurm


.lf(.T, y) d\ ,

IXpending on the rom of P(:t:, y) lor M(:c .)) 1md N(x. )'>I wrious mcttlml" can be u~ccl h
obtllitl lhe unkww f'ullcl iOfl y( \ ). 1\vo o( lhC more imponwlt tc.chniques ttrc COI\<'ilkrc:d here
and irl Sec;.1ioo 15.3: other~ arc ~hcu$-c;cd in th: uc:rcise~.
OitrtJCnlial equacOl 1$. 16 Aid to be: d ~nparabl.. diO'tft-ntlal e-quation f it can be
cx.p-cs..""(:d in the fom
dy
M ( .<)

-d:c - -.
N (y)

1h,1t k ;r dy/d' i"C<IIul ton fut'k:tion of x "'' iOOJ b) a run-~,.ii(.lelo( y. f.q\tiv<'lcntly. adi.ll'crcui1.ll
f'4U:U:ion is 1:aid to be 'parable,, it can be "tine,. in the fonn
~

N(y)dy

When a diO'ercncl.el eqttat.ion

i~ \"ttU4.'tl

in

I S . I!)

M (.<)d.<.

lh i~ way.

it is ~1hl 10 be M!-paratcd - ...e(XU"..tted in the

~lht ~~~~X- and ) lr.lriables appc!ll on Op(X"'hc! ;ilic. of tho: cqu11ion For . .epUlll<d <q"'" "'"
\\'C can \\rite thtf'Cfore that

tl)

( l\ "0)

N (y)- M(,<) .
tl .\

ancJ if we imcgtau: both .side-~ with respl to \'.we ha\'c

I N(y)=~

dx

M(.x)dx +C.

CariCCII~uioi~ of dill'cteHials on the lcfl lead~ m the: s;olu1i011

N())dy =

M (1)d1

+C.

\Vhnt we mean hy ~nyilS that 15.21 and 15.22 I'CflfCl\cnl t"'lt\Cf'Mamctcr fm11ilics of ~ulutions
for IS 19 i.< llur
funcrion deli ned implicitly by 15.21 or 15 .22 i> a solulion of 15.19. For
cxumplt. !he dJfferenrial cquauon

>

dy

dx

-y

issepamblc Mullipl)ingb) (I- >)dx le3dslo

(1- y)dy = 3.t 1 d.x.


According to 15.21. we should divide by ti.x and integrate both sides with respect to .x:

(I - y) dy
d.x dx

3x 2 dx

= x j + c.

Antidiffcrcntiation on Lhc krL must be interpreted as implidt tmtid(ffert'ntiatimr. asking for


that function which whe.n differentiated with respect to x gives (L- y)dy/dx . Since an

antidcrivative is y - y1 f2, a one-parameter family of solutions for the differential equation is

yl

)' - -'l --

,.3 -'I

Were we to use 15.22 after sepr-~ t ion (instead of 15.2 1), we would write

( I - y)dy

3x2 dx +C.

and integrate for

By :-oaying that y - y 2 /2 = xJ + C rcprt:.sents a one-parameter family of .solutions of the


original differential equation. we mean that any fu nc.tion defined implicitly by this equation is a
solution. We can find explicit solutions by solving the equation for y in terms of x . Multiplying
by - 2 expresses th e equmion as a quadr-.uic in y .
/

- 2y

+ 2(x 3 + C)

= 0,

and the-refore.

y=

j4 -

8(x3 +C )

:1: .j1- 2(x 3 +C).

Explicit solutions of the differential equation are therefore


y(x) = I + .j 1 - 2(x3 +C)

y(x) = I - J 1 - 2(x ' +C),

a.nd

provided that expressions o n the right arc indeed functions of x. Once C is dclc.nnincd. lhis

will be true only for ccnaio values of x . For example, suppose we require the solution of
lhe differential equarion that ~~tisfies the initial condilion y (O) = 3 . The second function
y(x) = I - j I - 2(x> + C) cannot satiSf)' this condition because y can nO( be greater than
I. If we substitute x = 0 and y = 3 into the other function.
3

= t+

J 1 - 2C,

and this requires C = -3/2. Thus, the solution of the differential equation for which y(O) = 3
is

y(x)

1 + .j1 - 2(x3- 3/2)

I+

.j4- 2xJ

Since 4 - 2x 3 nmst be nonnegative for this function to be defined. x must be restricted to the
interval x ~ 2 1fJ. Because tlle derivative is unde6ncd at x

x <

I EXAMPLE

z'l

= 2 113 , the soJution has domain

15.3

Find one-parameter fantilies of solutions tor the following diffcreoti.al equations:


(b)

dy

dx

ysinx

(I

+ y 3 sinx
+ y2)2

sm.t TTON
(a) Jf we d iv_ide the d ifferential equatio n by xy1 (requirilS lherefore lh{ll
obtai n

y :f:.

0). we

ydy = 2x 2 dx.

which is SCJk'l!Utcd. A onc-parnmctcr family of solutions is lhe.refore defined i lllplici tly

by

or

y2
2

zx3

=-+
.
3

We m>1e ohao )' (x) 0 is ;,I so a soluoion, btn io cannoo be obtained by specifying C.
In other words, .v(x) = 0 is a singulal' soiULion. \Vc removc:.d this solution whe-n \Ye
divided the original equation by x )'1. . Ahwy:5 be careful o f this. If tlll equatio n i_s sepilrated by di,iding by x's andy 's. S<lY F (x. y) . determine whethcrsclting F(x. y) =
0 leotcls to fmldions of x thi:ll s~:~tisry the original differential equation. If it does. and

lhe runttitMI:S are included in the fam ily of solutions obtained, nothing further need

be said: if they m-e nOt in the f:.1mil y. then they are singular .solutions for thm family.
(b) S ince

y sin x( I + y 2 )
( I+ J ')'

y sin x
I+ y>'

the dill'ercntial equation can be S<:punllcll:

I 2 (Iy + >')dy

+y
sinxdx = -y-dy
=

()' ~ 0).

A one-parameoer famil y orsolutions is oherefore defined implicil ly by

sinxdx

+C=

JG+

y)dy

or
-<vs.r
Since y(..r}

+C

= lnly l

J'
+ -.
2

=0 is a solution of the dinCrcntinl equation, and it cilnnot be obta ined b)'Specif)ring

C. it is a si ngular solution of the onc-panunctcr f"ntily of solu1ions.

I EXAMPL E

1 5.4

Figure 15.2 shows a straigho. prisma1ic channel carrying uniform ftow. Expcrimcmally io has
been shown lh<lllhe mean velocity of the now is propon ionllllo (A I/))". where A is the croSS
se<.:tionul ~m:a of the now, p is the wetted perimeter. and n is a constant (between 1/2 and
2/ 3). Let Ao and POhe values of A and p that yield minimum now for the given cm ss-scclion
helow the x-axis. The problem is 10 flnd 1he equatiM x = x(y) for the (S)Inmeoric) side of
x = x(y)
the channd above the x-axis so that the ratio A/ p rcJUain~ Ao/ /Jo.

UOJiflll.l P1(1

)'

SOLUTION

When the height of the water in the channel is Y , !he cross-sectional area of the

now and the wetted pel'imeter are.


A = A0

+ z fo y x(y)dy.

p =

ry\/1 + (d')2
d:y dy.

Po + 2 Jo

To hme A/ I' = Ao/ l'o <>r t\Jio = Aol' we sci

rf we c:-anccl J!oAo. e,J ivide by 2. anti then differcnliate both sides with rtspecLIO Y . we obtni n

J>oX(Y)

= AoJ l + [x'( Y)]2

rr we now rcpl~tcc Y wich )'. we obtai n a di ITcrcmial c.qt.mtiOn for .t (y) .


2

dx
p~x 2
- =:1:: - - -1.

Pox= 11 0 I + ( -dx)
dy

A5

dy

Since the slope is <~I ways posili\'C for the right half of the channel, we choose the positive sign.
and separate the equation
I

-;:=;=;;:=:::::::;:;;; dx
p~x2 -

A&

= - tly.
Ao

A one-parameter family of solutions is <.teflned by

y
-+
C=
Ao
Wese1 x

= (A 0 f p0) secO, and tlx

-y + C
A0

fj

pJx 2

Po

In I sec 0

+ ran 0 I =

= - I In IPoX +
Po

dx.

= (A 0/ p o)sec(J ranO t/0 .

I
Ao
-sec8
rnll d(J
Ao wn 0 Po

= -

A~

j /Jryt
2

'

= -I

Po

flu-X

Po

Ao

- In -

sccOdO

.j pdx 2 - A~

+ -'--'-"--.,.-----"
Ao

- A 011 - - I In Ao.
Po

We ahsorb rhc last cons1an1 inro C. S ince .t ( O) = b . C

= ~In ljJo/1 + jp~b2 - A~ I. so


Jlo

lh(ll

Thus.

Ao

y = - In
J10

PO' + J p~x 2 - At
/Job + bW - At

Ao

- - In
/'o

+ )x>- (A o/Po) 2 ]
b + .) b' - (Ao/ 1'o)2

[x

In lhe 1968 Olympic Games in Mexico C ity, Bob Beamon of lhe United Stales made a phenom
enallongjump of8.90 m thai has yet 10 be a pproached. Many have claimed thm this was a resuh
of less air drag due to the rarefied air a t the high alritude of Mexico City. We refure th is claim
here. Experiment.aJiy. it has been shown that 1he air drag on a longj umper is proportional to air
density p, the square of speed v, and the cross-sectional area A that Lhe body presents to Lhe
air. Air drag can rhen be represented in thdonn F = CpA v2 , where 0 < C < I is a constant

called the dras cocfficicm. r'OI' a long jumper, typical \lllues of C and A are C 0.375 nnd
A = 0.75 m'- e\\1an's <econd law for the acce.leration a of the long j umper while 1n flight
gives

,.

ma = - mgj - CpAv v,
where v is a uni t \CCtor in the direction of motion. Separating this imo horitOntal und vcnical
components leads to a coopled system of differential equations for the components of ,elocity

dv,
dt

dv>

m- =

.z.

l'>

= -mg- CpA(v; + v,)-p::;j .

m-

dt

. .2 ...

.,

t'1

Realizing. however. tbot there is very little motion in the vcnical direction compared to the
horimntal direction, we simplify the problem by igncring \'ertical motion. Let us as<ume that
motion is horizonta l then. with air dr-dg in the negative x-direction. Then

dv
dt

m- = -CpAt2
\\.here v now denOies the hon700I.'II c0111ponem of \'clocit} t.r- The differential equation IS
.epardblc.

dv = _ CpA dt.
11~
m
ond therefore a one-parumctcr family of solutions is defined implicitly by

CpA
- -I = _ __
,+
v

f) ,

If we take time 1 = 0 when the long j umper takes off, and u<e t'o as takeoff speed, then
D = -l/v0 . This gi,es
dx
v=
I .
dt - CpAt

-, - +vo

lmesm1ion now gi,es

CpAr
x(l )= -m- ln ( CpA
m

J)

+ -t'o +E.

111

With x(O) = 0. "c obtain = - C In t 'Q. and

pA

m
(CpA/
x(t ) = - - In - -

CpA

I)

+ -t'o + CpA
- m - lnL'u =

-m- In ( I+ CpA
CpA
m

t'o') .

T) pical time for the dur.ti<Ml f1f a lnngjump i< I s, :md t)'PICfil ini ti~l <peed< at takeorr,,.e around

10 mls. With these values and 111 - 80 kg, A = 0.75 m 2, ond C = 0.375, we cnn eolculote
the diO'crence in length< uf 11te hwlgjumJ> at sea level. where p = 1.225 kgtm>. :nld in Mexico
Cit)' where p = 0. 984 kpm' :

so

.,--,=-~""":-::-::-

0.375(0.984){0.75)
-

In I

0.375(0.984)(0.75)(10){1)]

+ - -'---;:;:---so

80
[
0..:.37...:.5~(1_.2_25;,.:).(0_
:_. 7__:
5).:.
( 1_0):..:(__:
1)]
In 1+ 0.375( 1.225)(0.75)
so

= 0.04 m.
In other words. the rarefied atmo>phere in Mexico City would have made a diffel'cncc of only
approximately 4 em.

EXERCISES 15.2

(n El.crci.SCS 1- 10 find U f.)nc5XU'a111Ci cr ramiJy O( SOhlliOilS

thC

(Or

* 2 1. When a deep-sea diver inh11les air. his body tissues ubKJ.rb cxtn
ful\(mBIS of nitro~cn . SuPt>O~ the diVCfl.'fliC~ the w;l!er !H lime t = 0.
crops very quickly to dcplh tl, and remain$ u~lhi:s depth for u very long

dil1'crcnl iul equation. Be c~rcful to idcnlify :lny singulllr ;o;olutions..

dy

J. 2xydx + (x' + 1) dy

=0

4.

time. 11le annnt N of nitrogen in his body lissu~~ increases <6 he


remnins nt this depth until a ma:<i 1num umounl N is reached. 1l1f: lime
rate of c.:h;,mwc; or N i!<o proponi<>nalto the Uifl~rencc N - N. Show
lhal if No i,; the :lRlOUnt or nitrogen in his 00<.1)' ti:;suc~ when h~.: CIU<n

= 4x

d.t

+ 2x\'.

dr
~
d.t

= 3y+2
.

2. -

I he water, 1hcn

S. 3(y' + 2) tl.< = 4y(.t- I) dy

6. (x'y +x')tlx + (xy'- y') dy = 0


d)'
eo )'
?. dx - - sinx

whetc k > 0 is a consu~nl.

8. (.<' Y' - )')dx + .ry3 dy

9. (.t' y ! secx l..anx + .t.Y 1 ~.c :<) Jx

ely

IO. tlx = I

+ 22. When u J;.ubstuncc such us ,IUC4.lM: i,; mhn i ni ~;~ercd i ntruvcnuus1y


into the blooc.lsuc:un, it i ~ u~cd up by the body ~11 a r.dc proportional to
the amount present at that lime. 1r it is added~~ a COT1Sl~nl wu: of R
unit,o~ per u1li1 time, and Ao is the amount pt~$Cn t in the bi~~1Mn
when the intr~wcnou.~ ti:.'Ciling begin$. fint.l a fonnula for t.hc amount in
Lhe blnot:l~tream at any time.

=0
+ .t,\ '3 d ,\' = 0

+ )'2
+ x2

In Exercises I l-IS find a sohuion or the d i ffcrc nt it~l cqua1ioo Clli!~ alro
s::ui;;fie~

12. (xy + ))dx- <.x.v - x)dy = 0. y(l )

tly

14. tlx

nonnal at any poi.nl on the curve, and the: li ne j oin ill& the poinltO Lhe
origin> form an isosceles triangle with the .x axis as base.

the g:iven rol\diJion.

II. 2ytl.r..-(x-L)dy=O. y(L)=2

d)'
13. tlx

t 23. Pind 1heeqoat10ns for all cur\'CS that so.tisfy the condition lhm t.hc

= t'+' .

.Y(O)

=2

=0

= 2.<( 1 + y).

dy
sin 2 y
LS - - - - y(O)
d.r - cos' x

)'(2)

=4

1e

16. A girl lives 6 km from school. She decides to tn:!\'cltosdtool so


llli:ll her SJXXX.I is alway6 pc-oportionallo lh(: StlUarc 1>fflerdiSiance rrom

lhc school.
~hool ;.tJ ;_Uly li me.

(b) Wh<n cJccs she rcoch school?

water cools to 60" C in 2 min, fird a IOnnula for i1s ICillt)Crature


fu1Klion n fliltlC.

a.~

26. Two spherical conwincrs each of r.-ldiu ~ R otrcCOI'I.ncctcd by a pi pe


of croS).M:d iunal :tr('u a (figutc tx:low). lllC talC of llow fll) lll one
coo1aincr1o th< 01her is (a/3)./2gll where g = 9.81 , and h is the
di ffc,ence in watct levels in the contai ner~. lf one contaillCI' i:s initially
<.:mpty und the ocher is full. how long docs it cake ftlr \\'Iller lc,cls lobe:
equal? u;,,,.: Sec equation S.22 in S<.:ctitJn 5.5.

I
I

II

change nf thc ccm,x:rm.urc of the wutcr is propl1ftiona11o 1hc diiTcrcnrc


between the lcnpcruture of the water and room tc.mpcr.,turc. H the
a

19. A thermometer reading 23C is taken outside where the temperature is - 20C. I[ the reading drops to 0C in 4 min. when wiJJ it read

- 19C?

8 are brought together in one solu

17. Find a onr.."-p:tramctcr rami ly or solutions ror the diiTcrential cqua


lion

.t 18. When acOtltaincorwtucrattcmpereturc SO''"' Cis ploced in a room


altcmpcri.lturc 20"C~ Newton's lnwofcool ing swtcs duu thc linle: mtc {)f

Wld

._ 25. Wh.,t is the sollllion to Exercise 24 when the initial amounts of A


W1d 8 111'1: lJ<Jch 10 g'/

=.T/2

(3) l'l'ind her distance from

24. When two substances A

lion, I hey rC~lCI to form 2 thinl ~ubsumce C in ,::uch !1. w:ty Ihut 1 g of A
r~~n~wilh! J;Qf fJ !Qpr!l4~~~ ~gQf C, n>~' ro'!nt whi~h Cis ftm~4
is proportional to the t>rOducl o f I IX: amOtU11Sor A and 8 !Hill present in
1hc solution. If I 0 g of A rulCI ISg of 8 rtrc otiginaUy broug_ht togcthcl'.
find 3. formula fb l' lhe<-tmount of C prcsen1in Ihe mixture :.uany time.

20. The !amount of a drug such as penicillin injec1ed in10 Ihe body is
used up nt 11 nne proportion:il 10 the tun(Mm t Slill present H a dose
decreases by 5% in the first hour. when docs i1 decrease l Oone-half ilS
()rigjna1 amount?

27. A tank in tJte l"onn of a circular cylinder (radjus R and lengtlt L )


witJt hemispherical ends has its a.'< is horiw ntal. Water exits through a
hole in Ote botlom at rate 0.6a J2iii. where a is Lhe area of the hole
and h is tJtedepthofwater jn the tank. Find a formu la for the length of
time it takes a full tank to empty. Him: See equalion 5.22 in Section
5.5.

* 28.

* .B.

The: :schc::nlalicdiag:ram Qf :1 can:tllock Sm by 16m i$ 5hown bciO"._


The water level in the lock is roisc:d by clo!iing vah'C 8 and opening
valve A . TI"lC nne Ut which watcrcmcrs the lcx::k is Q = 0.04Jlgli
m'ls whCI'C g = 9.8 1, :.lnd h i~ thc<.li lli.:r<:nee in ht:ig.lusorthc; Ul>~rc:am
water lc,oc1 tand W"Jler in the lo<:k. If the upstrc~m g:uc is opened when
/t 2 em. a.nd the itlitiaJ value or h is 2 nl, how long will il be before
the UJ>StRanl ga1c is opened?

In transient frec.&utfaee ""''' thwug.h porOlL'> tno:lia, vdriCity v(t)


nw.s& &ltisfy tile differential oquation

~ ~ ~ [ " + 11 (1)].
dl

5itu:llions.

~nnrea tn

water

ll~>>Umj ng Lh!ll

> 0 is the

P'J I~ ty

of

v(O) 0:

... J4. A .ti.r!ltordcr di.tl;:rcntial cqu;uion in y(x)


neo1ts if it ca.n be written in the form

B
:f.

LtK: hydraulic COtlducli,iLy. ''

(o) 1/(r) i> et.lll>lOnl.


(b) H(J) = q1 - Ill,, when:: q > 0 is a const~uu . Hint: Try a
solutiorl c.l' rhe rorm v = A 1, and find A .

Y. <il et

K > 0 is

.,

the 01cdia. both assumed COI'IIstant.. and H (1) is rhc head a-S a runction
o( lime I . Su-hc this c.lil'l'l..TCnlhal cqu"Li(ln in each or lhc rollowing

It
I

Up): I'~Rl

where

It

$:tid

1.0

be lwmoge

dy - I (.l-' ).
-Jx
x

29. A cubiccd C(Jtltaincr. IIlieU "ith water. is I m Ofl each side. Otw:
of tis sides has ot slil 20 em high ~xl I mm wide s~illg at the lop of
the comuincr. Usc tht.: result of Exercise 30 in Se(:tion 7.9 co determine
11ow long ill3kcs fu thc w.lJcr k:vd in the conl:.l.incr to l':a11 by 10 em.

Sho\V that the change of dependent variable t l = y/.t yields u diller


cncial equatitM' in v().') lhut is ahvd)'S ~c.:l)al'ab1c.

u.cc = 0.6.
+ 30. A pip.: of lenglll /,.. is connccloc.J 10 u b~tg\! n:~rv01r or tk.1)1h h
<Osu'C below). If wa\CI' i1; nllowed to flow from the ptpe otl ime 1 ::;;: 0,
the "clocily t1 o( the llc>w from the pipe must. in Lhc short lcrm, sallsfy

i5

-+

3S. Suuc

:1 condition on lhc functions M and N in cqu:uio11 15.17


in order tlm1 the d iR'c rcntial equation be homogeneous in the :;cnM! or

l!'<ercisc 34.

tile e<(U~ttion
In Excrci....::c( 36-4 1 si'IQ\vt h:itthcd ilTcrcnli.al cqu:.-tl ion i ~ honk)(:f'l<.:ous,

v..<c U>c ~ h ~~g~ vf ''ati"~l~ in


rumily o( solutions.

~n~

where g = 9 .8 1. Find

v as a fw~tion or t .

J;~t[l;iso;

34 10 fit11i "vnc-(ll!rll!ll<lcr

36. (_r 2 - x 2 ) d x+xydy=0

* 37. 'lxdy-'lydx; Jx2 + ~y2 dx

l
I

1--

dr

38. ....:....
d.r

+x
= -y)'-- .t

L-

39. .rd,1- yd.r

t J f. Chemical reaClOI'S gl'(: ()( third onlcr when the i:UI\OU.OI ,('(1)

=.<cos(yf.t)d.r

or

:>ub:;Lance being fonncd stnisr1es adill'c.rential cqutuion of the fom1

tlx
- = k(o - x)(b - .r)(c- x).
dl
wh~rc

a. b.

l'.

equation in tl1c

anc.l k arc JXI~i tivc <:onstanLS. Sohc this dill'crcnlial


(a) a = b = r (b) a = b :f: c and (c)

<.~ascs :

a '# b '# c.

42. lfu curve pDS!iCS through the point (I. 2) ood is such that the length
o( thai pari of the tungcnt line at (X, :t') (rom (.r, )') to the )'axis is
C(IU\11 10 the yi lllCI'CCJH of the lnngc.nt fine. lind the equation o( lhe
c:urvc.
111

32. When fluid nows through a tube o f var)itlg croS.<:stcti on A, rhe


Mach nun\bcr M of the ftow is rcl3lcd 10 A by Lhc different ial equation
dA

dM =

A(M 2

I)

2
M [( k- - ') M
2

+ I]

>

where k > I is a (.'Onstant. I[ A = A0 when M = I , find A in terms


of M .

4.J. Pinel in explicit fonn u function thtU sntisfics the diffcrcnti:d cqua
,;on d.v/d.r = esc." und the follow;nn condition<: (o) ,1'(0) = 1r /4
nd (b) ,Y(O) = 7:rf4.
44. In a cbeullcal reaction. one tnolecule of trypsinogen yit!lds one
tnoletule of lrypsin. ln order lOr the reaction 10 l.akc place, !In injtilll
amount of trypS in must be present S uppose that the injtial amount is
Yo 11leteaftet, the t'ate at whic,h trypsinogen is changed inLo tl'ypsi n
is propo11ional 10 th e ptoduct o f lll e amounts o f each chemical i n lllc
teaction. Find a l'o l'llm la fol' ll\e amount of lt)'J).Sin if the i nitj aJ amount

of trypsinogen i:s A.

Chapter I 5 Oifferenlial EquatioM

,060

* 45.

[fa fii'SIOI\ler dim::r'C illiaJ equatiOil can be Wl'illen in l.he fOI'Ill

* 54.

dr
.d.<
. :. . = f(ax + br),
.

dv

U.sc lhc method of Excrci:sc 45 to lind a oncpunmc-ter fanU.Iy of solu


tions for 11-..e diO'e.-ential cqmlli on in E'(ercises 46-49.

* 48.

,Jv

d.<
dy
dx

dr
= (.< + y)'
* 47. --'dx

=x + \'
.

49.

= 2t + 3y

-dy
= sin1 (x dx

thus pre~ser\'inge.arlyexponent ial gro~1h. The fac.tor I - N /C causes

tfN fdt ~ 0 as N ~ C. Sol\te this dill'e rential equation for N(t)


when k = I, C = J06, and N(O} = 100.

* S 1. Show Lha11he solution or lhe logisLic model in Exercise 50 for ml


ini tial populali<Nl N(O) = No is

(c No) ' .,
No

+ 52. OneofthcmodelstoOOscribcdlcwcight w(1) ofatishus a ru.nction


ofLiote 1 h
dw
"
m
= aw - bw,

"'

where a. b.n. and mare posi Live t.:onsrants depending on the type of

50 g or .:hcm.i cal

j~

* .SS.

+ y)r, )ln

r1

----;;-

Kt

II

c;~.nc;clla.l i On

of diJTcn:nt.ial:t in pn.x:ecding rrom

Solve the cliJrerential equmion

(x 'y

+ 2x.l '') dx + (x

- xy6 } dy = 0,

y (l)

= I.

+ 59. Two subslancc::s A and B react to fom1a thin.l substan~c C il'l such
a way that 2 i: or A reacts with 1 g of 8 to produce 3 ~or C . The rate
at ".flich C i~ fonned i~ proportional to lhe amo un t~ of A and 8 Slill

prcsem in lhc mixture. Fi.nd the amount of C pte.S<:nt in the nixlun!.


as il function of Linlc whc-1\ the original art'I<)UillS of A and B broughr

1oge1her aLiillle t

= 0 are as lollows: (a) 20 g and 10 g. (b) 20 g ancl

5 g, and (c) 20 g and 20 g. Draw gr.tph~ of all three

rtulcli on~>

oo 1he

stunt- ::a.x:e:s.

** 60.

A bird is due ea~1 of ii.S nest a distance ~ away and at the same
height above the ground as the nest. Wind is blowing due nonh a1 speed
v. lh hc bird flies horizontally with con.st.ant speed \1 always point.ing
sfraighL at. it~ nest, w hut is the cqu.ution of thccUI\'C thaLitfo1Jllws? Take
the nes.1 a1 the origi n and 1he ,~, - and y -direcLions as east ant.lnonh.

cultlcd to 200 mt of wiitcr. find a

fo rmula ror the amount or chemical di~solved as a runc1ion


of time. OrolW its graph.

(b) Re.pc::m pan (a) if lhc 50 g of chcmit"al is added 10 100 ml


of water.
(c) Rt::.l)eat l">art (a) if 10 g of chen ical is added It) 100 mL of
water'.

I"-,.,,
I"-,,I} =

]1n - -

15.21oo 15.22 really means.

* 53.

(a) If

56. In order to pc.rform a l b operation 011 a dog. a vctcrinarirut ancstbeli:tes lhc dog with sodium pemobarbilal. During the \lpt..'l'aLion. tht:
dog's body breaks down rhe drug a1 a l"dlC proponional co lhe amoum
st.ill present. and only half an originaJ dose remain~ after 5 h. If the
dog has rn.ass 20 kg, Md 20 mg. of sodiwn pentoba.rbitul per kilog-rtun
of body oass is .n.:f.{uircd to n1ai nuUn surgical am::stbc::;iu, what original
dose is requited?

... 57. Explain what LbC

fish and its environmem. Solve this difl"eremial equatioo S:ubj ect to the
inil.i.:l.l condition w(O) = w0 when n = 2/3 ond m = I .
A certain chcmi<:..al djssolvcs in \\o'alcr at a ralc l>roportional Lo the
product of he amoun.t of undissolved chemical and the ditlfrence between co.ncemr,.uions i1l o. sa.turotcd so1ution and the existing concc n~
tratio n i1l the. solution. A .satw-atcd solutioo CO!Uains 25 g of chemica l
in I 00 mL of solution.

+ y)r

Snow bas 1xct1 falling for some time \\'hen a snowplow ~LartS pluwi ng rhc highway. The plo\.v begins at 12:00 and travels 2 bn during the
first hour and I km during the second hour. Make rcaso t~bl c o.ssump~
tions to find Otll when lhc snow star'tcd fa})jng.

.f.

I+

(H

* 55.

1\ot.i.cc that when N is Slllall. d N /d t is approximatcl)' equal to kN,

N =

f[ D H -

N )

DN(t) }

wbcre r 1
[(D - y - H ) + ,/(D - y- H )' + 4D H J/ 2 and
rz = ((D - y- H ) - ,/(D y H ) 2 +4DH I/2.

(I _c

d N = kN
di

H+l't - l ''l

- [D H- (H

* 50.

d.iffcrcmial equation

H (t )]u v(v - D)

"

)')

The foginic model for growll1 of bacteria introduces a quanLiLy


C c:.tlled the rfwrying capacity for the environmem in which me hac
teria are living. As the number N (t) of bacteria approaches C. i1s
growth rmc must slow down. The logistic n~odcl to d escribe r.hi$ is lhc

!D - y -

where K > 0 i.s the hydraulic cooductivity, '' > 0 is the porosity of
the media. botJ1 asswned constanl. D ::> 0 is the depth of the banicr.
y > 0 is a con ~an t . and H (1) is lhe head as a llnction or cirnc 1.
Show thal tbc solutjon of lhis diJ3Co:nl.iaJ equation for v(l) when H (t)
is oonslU.OL assuiUi ng u(O) = 0, is defined implicitly by

separable.

= K { "' -

dt

where a and bare constants, show lhac chechangeofdependcnt variable


v = tlX + by aJwo.ys gives a differential cquZttion in v(x) that is

<16.

tn transiem rrec-suditce !low thr'ough poous media with<~ ve'tical

harrier. \"elocily IJ(I) must ~ati~fy tile har'ricr' now equation

**

6J. Find the cquatjon of the curve that passes Lhrough (1, I) and is
such that the langcnl and normal lines al any poinl (x, y) make with
the x ~ax is a triangle whose area js C<JU<ll 10 Lhe slope or the tangent line
al (x, y).

15.3 Line.'lr Fu.::;c.OI'det Oi ffereruial Equations


1

1061

15.3 Linear First-Order Differential Equations


A tirst-<:>rder differential equation that can be written in the form

dy

- + P(x)y =

Q(.r}

d.r

(15.23)

is said to be linear. \Vc will explain the significance of the adjc.ctivc linear ir1 Section 15.6. To
i11u~tnue how to solve such differential equatio ns., consider the equation

dy

- +

dx

- y = 1.
.r

If we multiply both sides by x, we have


dy

x-dx +> =x.


But note now that the left side is the dcrivmivc of the product .r y: that is.

d)

dx

d.r

-(xy) = _, _.

+ )'

In other words~ we cun write the diftCrcntial equation in the fom1

-(xy)
dx

= .r,

and imesration immediately gives a onc-p"rameter fami ly of solutions

xl
X)'= -

+C

.r
c
+-.
2 X

y(x) =-

=>

Titis is the principle behind allline<ll' first-order equations: Multiply the equation by a function
of .t in order that the left !\ide can be expre~~ihl e as the deriv:uive of : pmdutt. To show that
this is always possible, we tum now to the general equation 15.23. If the c.quation is multiplied
by a func~:ion fl(x ),
dy
11 - +f.' P (x)y = 1~ Q(x) .
dx
Titis equation is equivalent to 15.23 in the sense that y(x) is a solution of 15.23 if and only if it
is a solution of I5.24. \Ve a.,k w hether it is possible to find /.l so that the left side of I5.24 can

be wriuen as the deri vati\'e of the product J.kY; that is. can we find JJ,(x) so that
d\
JJ.--
dx
If we expand the right side,

f,J

d
= -(J.k)'}?

+ JJ.-P(x)y

dx

must sati~fy

dy

J.l tX
1

dy

+ f.t P (x)y =

JJ> d
-

dJ.k

+ y -dX ,

from which we get


dJ1
JJ.P(x) = -

dx

d!l

u1 -

JJ>

= P(x )dx.

Thus JL must satisfy a separated differe.ntiaJequation. one solution orwhich is


In IJJ.-1 =

P(x}dx

or

J.l.-

= ef P( x )d .< .

We have shown then th<Jt if 15.23 is multiplied by the fac tor

the.n the differemial equation becomes

~~ P(x)<l.vt!Y +

P(x)yef P(x)<l.

dx

= Q(x)ef l'(z)dx .

and the left side con be expressed as the derivative of 11 product:

Integnuion nO\\' g_ives a onc.. pammeter family of solutions


)'efl'(.<)dx

Q(X )ef l '(s)dx dx + C.

(1 5.251

The q uantity ef Pi.'I<~< is called an intgrati.n g fctor for equation L5.23 because when the
equation is multiplied by this factor. it becomes immediately imcgmblc.
l1l summary. if linear differential eq uation 15.23 is multiplied by the function e.f P(~:),1x.
t hen the left side or the equati on becomes the deri<ativc of the product nf y a nd [his functiOil.
In this form the equmion can immediately be integrated. There arc two things thot you should
remember in .~;;pecific exantples.:
J. The differcntial equal ion must be expressed i n form I5.23.

2. An inte-grating factor is ef /-'(.r)d.r . Multiply th~. dift"crcntial equation by this factor and the
left <ide is the derivative of a product. Now integrate both sides of the d ifferential equation.

I EXAM P LE

1 5.5

Fincl one-parameter families of solutio ns for the following differe ntial equations:

dy

(a)

-+X!'=
d~
.

(b) (y - X$inx) dx + ,<dy

(c)

co~

dy
...:-

SOLU110'<
(a) An integrating l'actorfor this litl~wcqmuion

i~

u we multiply the equation by Lhis integrating factor. we have


or

d
't-I
-(ye
dx

l ntcgmtion yields

= e'fl

+C

)' (.Y)

(b) If we write the differential equation in the form

dy

dx

- + :...

= sinx,

tlx

+ y s1 n.r =

15.3 Line.1r First-Order Differeruinl Equations

then we see that it is linear first order. An integrating tactor is therefore

ff we multiply the differential equation by this factor, we geL

dy
lxl
.
lxl- + - y"" lx)s111 x.

dx

If x > 0. then we write

dy
xdx

+ - y =xsinx,
x

whereas if x < 0.
dy X
- x - - -y- -xsinx.
dx

In either case, however, the equation s:implifies to

d\
+"
dx

x - -

= xsinx

= xsinx.

- (xy)
dX

or

Integration now gives

xy

=J

xsinxdx- -xcosx+sin x+C.

Finnlly, thc.n,
y (x )

= -cosx

siox
C
+ -+
.
X
X

{c) An integrating factor for the linear eqmuion

dy
- +ycanx=
dx

cosx

is
efr..>.<dx =

e'" 1.<1 = fsecxf.

r-or either sec x < 0 or sec x > 0, we obtain


dy
cos x dx
I

sin x

+ )' cos2 x

- cos2.v

or

d {

\'

dx ~

= sec""' .r.

1megration now y ields

cosx

= t.anx..LC

=>

y(.r)

sinx

+ C <'OS.r.

A o ne-parameter fa mily of solutions of a tirstordcr difiCrcnbal cqu:Hion is u gcncrol solulio n


of the ditTerential equation i f and o nly if i l t."'I1tains .-11 soturioos of Ihe d iffere111ial equal ion. Jf

the famiJy has s ingular solut ions. it caooot be a generaJ solution. ln Section 15.8. we indicate
that a one~parameter family of solutions of a lioear tl.-storder differential equation cannot have
singular solutions. l.n other words, a one~ paramete r family of solutions for a linear first~order

differential equation is always a general solution.

1063

1014

Ol.opo IS l>il\'cocoti.>l Eql.:olioo>

.' " Section l 11 \.\c intn)lluc~ lliffcremial c:QUdtion..' that ari-.e "'hen energy balance is
apphcd to >~m such os that 1n Figure 15.3. We n::pcatthc di>Gussion hen:: but in a mon::
S~ way. Ltqud at tcmpcrarurc T0"C enters the tank. the hqutd s heated. and,, then leaves
~ htgher temreraiUre T C The tank i< perfectly in<ulated "" that no heat can e<cape from its
sdcs. and thcn::forc all heat supplied by the heater raise:; lhc tcmperulurc of the liquid. When
lh_c t~k IS full, and we assume that this is always the case. the mass of liquid is M kilogru.ms.
Lqmd enters the rank at a rate denolt'd by,;, kilogram( per second and lta\'eS at the same r:ue.
~ heater utkb C:llt'l.)' ~~ the nile of (f joules per .second. l11 re4tlity. ttllll>craturc of the liquid
'"the tank depends on both lime and position in the tonk. The mixer is added to removespalial
dependent-e. \Ve u,,.s:umc that the nli xer is so elllciem thm temperature of the liquid h: 1he same
~~ e\'cty point in I he tn~ . Tcmpermurc then depends only on tintc 1. ucnmcd h) T (I), and this
1S also the tcmpcrmure nt which liquid lco"cs the tank.
A llncar ditrcrcnti.tl CIQUJ(ioo

scnbts the

tcn.pcu.tu:rc of cbc IJ41wd m lhi 1uk

Cool hqutd

Hearer

L-----f-- --1-- ---_:_Perf.:::..;t ,n ...u1;..1ion

1
Mi:<er

Hoc hquul

Energy balance for th1s system states that

(For simplicity. "t ha"e ignored energy associated \\ith the mixer.) The fil"$t thre~ l":ltts :tr~
known: the foonh can be c<prc.,sed in term; of d T f dt. the lime rtc of dwnxc of temperature.
The second term on the ten is q . For the remaining terms. we musl tnlrodlle<' Ihe SJ~cftc heat Cp
of the liquid. It i> the energy required to raise the tcmpcrnturc of I kg of the liquid by I ' C. [For
example. the specific heal of water is cP = 41 190 Jlkg' C. 11 requite<;lllJO J or energy to raise
I kg of Wlllcr I c. To roise 5 kg of Wlltcr 1o c requires (5)( 10)(4190) = 20') 500 J.) Since,;,
kilorams of littuid a11empenuure T0 C ente rs the tank each sc.:und. the rate m which energy
enteno the tank due to Ihis liquid is 1t1C P10 j.oules per second. (This j( tht energy required to
mi.!c ,;, ki logrunl> of liquid from O' C to T0<C.) In a similar way. the rdtc a1\\hichenergy lea>es
ahe tank in lhe li(luid ~u tempermure T i~ tiatP T jl''lule~ per ' econd. Thi~ le;v~ only lhe 1<1'1
tcm1 on the right side of the energy bal~ncc equation. The rotc of change of tA:n>pcrnturc of the
M kilogram~ of liquid in the tdnk is dT / dt: that is. lhc temperature changes dT / dt de<>,rees
each second. 11 fol i0\11< that the rate at which energ)' is uo;ed to rai<e the temperature of thi<
liquid is Mcp(dT fdt).
When the.c: l'llle> dtc oub>titutcd into the cntrgy balance equation. the result is
ti1c11T

dT

+ M c11 d/ .

Division by /14 c11 gh cs


til

q
M c11

- To+--.
M

01' the quantities ,;, , c11, T0 , q. and M in this equation, only c1, and M must be constant: ,;, ,
T0 and q could be functions ol' time/. In an actual J>roblcm, all fi ve q uantiti.,; arc given. and
the oojoctive is 10 solve this linear first-order diffe rential cqumion fOt' T (I). Multiplicmion of
Lhc diffetntiltl e'tuat ion by the integrating factor

le.1ds to

= ('i' To+ _q_)ef<i/M )al .


M

Me"

lntcgnuion gives

or

7'(1) =

J(,;,To+ _!!_)

Fe- f<NMidl + e- J<..f M)dl

e f v</ M)dl dt,

M cp

where F is the consc-~ult of integration. Once 1i1. Cp, T0. q. ruld 1\1 ore specified. hllegratirnlS
can be performed to yield T (1) . Let us considcrthe special case in which all five quantities are
constams (independent o f time). Then

T(l ) = Fe-ot/ M

Fe-nt/ M

_Mcpq_)

+ e-""1"'

j ('M;,

+ e- >iii/ M

('i To+_!!_) (M)

Tn +

e'"'IM d f

M cp

e'ot/M

= To + ..!!_ + Fe_,,,.., .
mcp

If we """'ume that 7'0 is not only the temperature at which liquid enters the tank, but also the
tcon)X'ntlllrC of the water at time I = 0. then F = -q/(til c 1,), and

A gr<lph ofthjs timction is shown in Figure l5.4.


GJPI't o f IQn(l<T.IIUrt uf lhc liquid in hc:JI lank

r
To+ ...,!1------------------------------m t: ,
1

EXERCISES 16.3

In Ex<:1'CiSCS 1- 12 find u general solu1ion fOI' the diiTcrcmial t:quation.

tly
2. tlx

d)'

I. --'- + 2.t l' = 4X


dx

3. (2y- .r) dx
5. (x

til'

+ d~ = 0

4, -
dx

+ 2.r)) d.r + (.r' + I) dy =

24. A tnnk has tOO L of solution containing 4 kg of &gar. A mixture


wi1h 10 g of sug;ar per li lre of soltLion is added 31 a rtlC or 200 mL per
minute. Al lhc sr~ n1C rime, 100 mL of wciiSfirrcd llliAiure is rtmo,c.d
czch minute. Fi~d thco.m1ount o f s-ugar in the tank as iJ function oftin'le.

' Xd.,. - xz = .r..


8. (y + c'-' )dx = d ,l'
dy

II.

+y =

tly

2$. Repeat Exert.is<: 24 if 300 mL of mixture i s rcolO'''c.'d coch minute.

tl )'

10. _, , --'- + (2 - 3.t 2)J

2cos.t

dx

- + ---x
d .t
xln.t

=.r'

26. A tank originally contains 1000 L of water in which S k& of saJt


baslx:cn dissolved. A mixtwe OOiltaining 2 kg of salt for each 100 1...
of solutton is addOO to the tank ~l l I 0 mi.Js. AI the 5311\C ti nlC.Ihc well
.5tirn.xl n\ixturc in the tank is rcmocd at the rate o( 5 mi.Js. Find the
a.o104.111l of salt i..n the lank as a fune-tion of time. lftaw a grnph or the
function.

12. (-2ycot2x- I + 2xcot2x + 2csc2x)d.r +dy = 0

27. Repeal Exc..ci~c 26 if tbe mixture is rcmc.l\'cd :u 10 mUs. Wl10.1t is


the linil of th..: a mount ur salt in lhc taf1k rt'r lv"g p.;riods or time'!

In Excc'Ciscs 13- 15 sohc the di lfcrential equation.

,.. l S. Repe-al

dr
13. --'dx

* 19.

+ 3.t1l' = x'.

l'(l) = 2

15 . d)'
--'-

tlx

'' "
+x +I

x'

1'(0 )

16. Find a general soh.uion for the dirrercntial equation ()'3 - x) tly =

A l)()lalo al f<'K)Ill tcmpcratu1c 20 ~ c is placed in 1he oven at Lhc


moment the oven is sct to200C. H ttlkcsthco\'cn 5 min to rcucl1200' C.
the tctnpcraHLre i ncrcasing al a con!)tanl ralc. ami UlC tcmpcr&urc. of the
oven remains ~( 2001)C thercaflcr. A~"tuning that the tcmpcr.uurc of
the potato olx:y.s Newton's Jnw of cooling. fiod its tcmpcmturc as a
runc:tion of ti lllC.

,. 17. A differential equalion of the ronn

--'d ,(

+ l'(x)y

= y Q(x)
"t

is called 3 B~ft!QIIIIi cqutrlicm. Show thai. the c.hangc or Qepcnclcnl


\'ariable t = y' -" gives

dz

dx

+( I - 1l)l't

(I -

.... 32. Rcpcul Excn:.i~ 3 1 i r Lhc F..llc ld which


100/(1 +I) kil (18mmspcr<econd.
4-

In

Excr<:i:sc~

18- 22 u.sc Lhe chungc ofvuriublc in E!Ccrci.sc 17 to find u

gcncml ~o hll inn for the dilfcrcnliul CtlUalicm.

18.

tly

- + v = \'2e'
dx
dy

y1

dx

- + - = -2

19.

20. - - y
dx

tly

+ (x' + 2x)y' = 0

31 . Water at tcmpctaLw'C I0C emcc'S the tank ltl Figu.rc 15..3 a1 a 1me
of 0.03 kg/s. Ao limc 1 = 0. lhe lmnk n:achcs Cllpacity (100 kg), and

water then lc:.a\es ::u Lhe same r.tte. lle healer is turned on at 1 = 0
adding energy to the w:atcr at 2000 J/s. Find 1.hc tcmpcrarure of the
Wili.CJ' ill Lllc Lank il.~ U f unctiO!) Of time I. aJ\I:J pJO[ its graph.

n)Q ,

o l i11cor firs1-ordcr equa~ion in z(.r),

if the mirilur-c. i.s removed al lO rnU::s.

* 30.

ydx.

dy

E.x.~XCi!';C.> 26

A I'OOm wil h volume 100m 3 inhially contains 0. I '1 carbon dio'(


ide. Beginning at lime 1 = 0, fresher air comaining O.QS~(;. carbon
dioxide llows inlo lhc room at S ml/min, 'Jlte wdlmixcd air in Lhc
ruo n1 llows out aLlhC s;mlC ralC. Find Lhe an1uun1 uf carbon tlio.'<tdc
in Ihe room as UJ fu nction or time. Wll:u is the limit of the function as
1 - > co?

14. (- e' si11.r + .l)c/.t + dy = 0. y(O) = - I

y" sin x

R(l) tl~lt is u function of ti me I .

dx

9. -d

+ )' laox =

... 23. Repc:u Exercise 22 in Section lS.2 i r the gluco,;c is a!ddcd Ul 41 rule

5e""''

dy
6. (x + 1)- - 2)' = 2(x + l)
I dy

.x tl)' + y il.t = :<1y5 d...:

dr
+ 22. d~

+ .r:.,.
= 6.r'

+ y cot.< =

..,. 21 .

W!IICI'

enters the wnk

j,

33. Repeat Exeteisc 31 if lhe antOurH of clCIJY Sul)l>licd by the tw:a1cr


ro.. the fi1-st 10 min is q = 20t joules per second. Solve onl)1 for
0 ,::; I ,::; 600.

Repeat E.xC<rci!:c 3 1 if the tempcroture ol' Lhc i ncomi ng wme is


To(/)= IOe ' .t!: 0.

>t 34.

35. The step response for tbe RC -<ir<:uil in 1hc foUowing figure is the
vollage V(t) across lhe eapaciLOr when lbc applied vollage is E
h(t) . when:: h(J) is lbt Heavisidc unj1 s1ep function and lbc i njli!tl
vol1age acro..~;s l.he capacitor at lime 1
0 is 1..ero. Given dlat V(t)
must satjsfy

rn: d''
,,.,

v-

H.

V(O) -

31l.

("') "1"\\e ~<l~~"~"" I

ti\th~ ~ t.: ~;,.-w_;t .,rEo.t~i~ :!>:'i """'lo'.s.>A;~fy

11..,,\i.u ..,,.,,,....,.,~...,.\i"'"'

0.

6 ru,1 Ill('. Sf<'P r<~~ ~lt<l <,lr ., w ~~~ ;;)'~

lf l::(t)
wril'.~u

~ w; iu ~~(':~
in tbc (uro:;

~1. $1.ww

th .' \t tbo:

~\v\kon<:a\\ ~

:.

I he pul~e re.<_por..e t (l ) (._..r t he- R f~ d t:u n n lhe ii&o"' bcl.:ow ''


ll..;;<.:v u\:tJI i u tl~~u..,ui( .,..., ...,.., ~IK: ,.;.,t;tot :ooV'ln <o.: i~:~ I = h(t ) - lt( t - 1)
who.c JJ(r) 1$ ll o..: H>:1~i:;.i,1e .,,.;, ,;t<:f ,., ,.\&:h(lll , ao'4 ! h-e i oll1i ll1 Cu oe .u
i n lh.: c in:u i1 ~( 1i.mc I = 0 ill xcro. Civ.:n Ul.u i(tj IUU-'>t >-:IIi'> f)'

L cl o'
7i dt + i = l.

HOt = 0 .

lind l.h!: puJI:t: IX'llpo:tliC Cllld d ltiW is:. gr.1ph.

(b) \ Vh.-\t

(_"";)

Rs

I
I
L
l

++ ,,(),

<ll

dt

oeu~n\

in

~~,wnt

F.....-.<;i,..~

\r,oc.~'o r.;ui\ ;u im,oc.

~Q l)o.uo 'I~;> nr.t fliO'.lo\111\o~ t\mt \1\o.' \("l\'.~ttn~ C)( \ 0


.\1".

i,; C:\m'Allnt roc t ':, 5 . Allsuu~ in.,.u; ~ th:~.\ i\ h. ,;il\u w \1.\al. with
pctkod 10 min. o~:illatitl hc-tw u .n 2 lo?C: :u'ld. \ ~C .

(n.l The currem I in ltw~ / . R -cin:u if of E>.r:r, i~ loti (WII h


c:plxod by ) Ill list f.:&lif;(y lhe diffe. crt~i al c:qu attl:ln

L-

lhc: val ~ of A i r the

~-.::.:n

,_
"r
- -1'.mj'- --'

... .17 .

i~

o ;~ '<
w....c.n t:<.n i ~ c.onnc.\c41

I
~
'>

Rf = E .

F.., . .h l (.:~1 ) , l ~ 0 , wtu.:n : F.(l ;t (td 0.1 an.: """ '


soJvc 1his d iff<rcntiuJ cqumioo for f (1) , nnd show
thnt the solution C!lll be w rincn in the fonn
If F. (f) ;;;;:

"' 40. A .;.hu.n n .;:\ '" 11ll'W lx'ko\v " )~\m 1.'"\e d~tm bn ..-ul:.,;. ~n ' ' '"'-" 1 a 0
a m! u.:~l'> w;1.\c t tu !.lot: c1r. mtu:'o ,Il a. h1-:.-l Iall.: al {ut>l, d\:c~t:al..IW 1\l
tin>, Let \be i.npullO the .;b alU'Io:\ be l (t)
lo~ 11 , Wbet f(l a-,~1,\ ).
~rc. (.OI~iti vc c cws:tmw;.. ~ !Utot'lljt~ S ut' th\-. d\;ll\\'1.(!\ atl:\ th\~ ~\iM..-'n~\t'jtJ':
Q (n.,ll\l tl\o\:. c:h ~mc:1 ;m: t'O;la t..:d lo,:o I ( t) by \he: M'*kml~n\ rQU\\\\ '
cqu.u ion>,,:

srnnl~.

/ (t) -

Ae- lufl..

dS _ I _ Q .

+ -Eo si.n(wt- .P).

dJ

where A is an arbitrary conswnt and

+ (1 -

X)Q\,

where K and () <. .'< -c. l ace co!tslams. u~ \be"~ \o \\nd Q ~t) .

~= Ton 1

('"RL)

(b) Whal is 1hc value of A if the ClnTCnl in the


I = 0 wflCJl (1) is COntlCCtCd iS f0 1

S ~ K \J< I

~ i -cuil

.t..t.

at time

41. Repeat 'Ex.en::isc ~ \ in Sc<:.t\on \ 5.1 t,l..;cn that the d\,,t,. d~t'l<.\s
:o.IC'oWl)' to \he OOunm. A.:o.~umc \hu\ hl:o. rlc:'\c~n\ 1:<. u\ a ccm,;\un\ m\c <'wcr

a tlmc imcrv;l.\ of kngt\\ T. Ass\mte aloo lM\ maxintu..m pre'ii'S\lte N \s


pt'l"l\)Ortiooal to <\eph helow the 1'\.nfac.c.

15.4 Second-Order Equations Reducible to Two First-Order

Equations
Seco nd-order d iffere ntial e.q uations in y as a function of x can be expr~sed symbo\lca\\y in \he.
form
F (x, y, y', y") = 0.
(1 5.26)
\Vhen

F is independent of x or y, the second-orde r equation can be replaced by two first~order

equations.

Type 1: D ependent Variahle Mis.~ing


If a second-order diR'crentiul equation in y(x) is explicitly independent of y, then it is of the
fl)t111
F (x , y', y") = 0.
( 15 271

In such a case we set


dy
and
d.t
If we substitule 11\e<e inlo I .27. we oblain

v=

F(x, v. l') = 0,
u first-ordc.r diffcrcntiul equation in v (.t). If wc can solve
imegra1e d yfdx = u(.l ) for y(x).

I EXAMPLE

thi~ equation

for

u(~\') .

we cun then

15.6

Find l"o-paramecer families of 'olution> for the f<>lkl\ing differential cqwuion>:

(a) .r yN- y' =

(b),...

= (y' )l

SOLL'TIO:\
(a) Since y is c., plicitly missing, we use substitulions 15.28:

de

x-d - v = 0.
X

Variables are now sepa_rnb&e-.

dt
tt

(provided v

d.r
:c

f. OJ. and u MJiution for v(x) is

In lui"' ln lx l
Because v

+C

or

= tl yftlr ,
dy
dx

D\.

and we can integn>tc for


y(x )

= - <1 +
2

= Fx 2 +

(F

= D /2) .

When v
0. we obtain y = cons1ant, which Slltisfics the difTcrcnlinl cquntion.
Since such funclioth are comained in the two-pam meier family y
F.\ 1 + E . 1hcy

are nor singular.

(b) Since y is agam missing. \\e substitute from 15.28 10 g<t

Variables are again scpamble.

dv
2v = dx,

!)e(;Ol\dOr.:ler &tu:ttions Redocible to 1\\.'0 Fii'SIOnler Equ ~tions

15A

(provided v

1069

# 0). a nd a solution for v(x) is


v

=X

+ C.

Consequently.

dy

v=

x +C

dx

_LIJtegr-.uion now yields


y( x)

- In

lx + Cl + D .

When v = 0, we obtain y = constant. whic.h satisflc.s the dift'ercntial equation.


Sioc.e these functions can.not be obtained from the two-pammeter family, they are
singular solutions.

Type II: Independent Variable Missing


Jf a secondMorder differential equation in y(x) is explicitly independent of .r . then it is of the
fonn

P(y . y', y") = 0.

( 15.29)

In this case we set

dy
dx

= v

aod

tlx1

dv

dx

d v dy
d y dx

dv
= v-.
dy

( 15.30)

\ Vhen we substitute the.,-.;e. imo 15.29, we obtai n

ely = 0,
(y, v, v dv)

a first-orderdi fferemial equation in v (y). If we can solve this equation for v(y). we can sep<lrate
dyfdx = v( y) and integrate for y(x).

I EXAMPLE

15.7

Find explicit two-parameter families of solutions for the following diffe rential equations:
(a)

yy"

+ (y'} 2 =

(b) l

'

= (y' ) 2

SOLUTION
(a) Since x is cxpJicitly missing, we substitute from 15.30:

dv

yv- + v = I.
dy

If variables are sepanued. we have


vdu

v2 -

dy
y

1070

Chll)ler 15

l) iiTtrerui:.'ll EqiUiliOtl S

(provided v =fi I), and inte.g ration gives

I
2
-In lv
2

- II =

- In IYI + C.

Thus.

from which we ha"e

dy
- - v
dx -

= jDy + y'

\Ve separate vari ables again Lo get

.v dy

-r';;:::::=~

+D

,jy2

= dx,

and obcain an implicit definition o f the solution y(x),

jyz + n

= x +E.

Explicit solutions are

y = j(E x )' - D = /x2 2Ex


where F = 2 and G =

+ (E 2 -

1 - D. When v =

D) =

/r1 + Fx + G,

I , weobcain y = x+C. which

arc sol utions o f the differenlia l equmion. Si nce lhey can be obtained by choosing

F = 2C and G = C2 , they are not singular solutio ns.


(b) Since xis explicitly missing. we again substiuue rro111 15.30 to obmin
dv

v-

dy

lf \ ari.ablcs arc separulcd.


1

dv

(provided v

= dy

f= 0), and a soluti<)n f()r v(y) is


In lui = y

+C.

Thus.

and
dy
dx

= "=

De1

(D

ec) .

We separate variables again,

e- Y dy = Ddx,
~md find al\ i mpl icit defi11ition

of the solution y(x).

An e.xplicit fonn for the solution is

=-

ln(-Dx - E) = - ln( F x + G),

where F = - D and G = -E. From

v = 0. we obtain y = constant. which are

sol utioos of thediffereot ial equation. Since thesesoluLions can be obt.ajned by seniog:
F = 0 in the l\VO-parame.ter family, lhey are not si ngular solutions.

In eah ofllumples IS.6and 15.7 we solved the differential eq\.,tioo> yN = () '):sin<>! both the
hl<.fcl:>c:ldelt variable,.\' tmd chcdcJ>Cldcnt "ari:.blc y (.re missing. Aflhough t he solutions uppear
differclll. e:uch ise;t.;;ily derivabl e f rom lhcother. Notice that sohHions y = constnnl ate sinular
forthe t\\o-parameter family y(.r) = D - In lx + C l . but not fur y(.r) = - In ( F x + G).
Let u.< summari>c the results of this section. Sub<titutions I 5.28 for differential cquattOt>
I S 27 with the dependent variable mi~ing replace the second-order diiTerentinl equation \\ith
two fi rsc.. order cqum ion:.: ct fi rstordct equation in v (.1'). followed by a firsc~')l"der equation in
y(x ) . Contrast thi s with the method fur equation 15.29 with the indepcndeou \Uriable mis>ing.
Subslitutions 15.30 again replace the w:ondJcr cqua1ion \\ilh t\\O first-order equmioo>s.
However. the first first-order cqu.>tion 1s in tJ()'). so th.u for this ~'<ltnion y is the independent
variable rather lhru> the dependent varittblc. The >CCOnd hr>l-vrdcrcquation is OJain vne for y(A).

EXERCI SES 15 4

In ~crcises 1- 10 li nd a lWO(XI";unclc:r fatnily or


differential equation.
l. X\'.,+ )

= 4X

J.

)'' - ' "' + 2.\'

5.

)'''~nx +/cos,.

7. y + ..,.

9.

2. ln" = 1 +
4. \ l y"

r'' + (.\")' =

8. y"

for the

IJ. 1'hc well in the f'iaurc: below pctll:tr.uc.:; an aqu1fc:r uJ'depth b. 1"hc

head h :u a radius r from the wcll mu.'" satisfy the differential equl1llon

<>.->'

d 1h

r -d

=(y~)'l

smx ... 6. y'' =(I+

=0

'ohninn~

(1'')'Jll2

Q,

= lt at the edge r = r.,_, ofthc v.:cll and h = hr at ~omc:


= r, fron1 dk! well. Find /r{r) .

10 . (y")' = I + (y')'

II . TI'IC hgun; below "I'IO\v5 u long cylilldical cable. Copper wtrc


runs down the c.etHtc or the cablt and tns.ulation c:mcn. lbc wu'C If
rmcuufCS rudi:al di\&:n:c: frum th,.: ccrtN: cf the C'lbtc.. then sccld)
Stale tcn~pcr.llurc T tlltllc 1nsul:nion is a ful'le..""tion of r chat n-u;t nttt~ly
tile differential equ,uion

d'T
r tlr'

till

+ (T
-1 =

subject to h

distnec r

= yy'

>

- O.

Fi nd T(r) ir tcnlpcnuurcs on inner tHHI t>Utc:t cdJc',.


of lhe in~ul ulion nrc <.'Oit~tt\1 values 7~ UllCI

r.

Aqtfer

..... 14. A dog ;U ~!I ton ( L . 0 ) in the .t)'planc "JX\C!o u rubbit at PQMhOn
(0. 0) running in lhc: 1)0'-ilivc y-dircclion. If the ,~.,, 1 uns at the w:mc

tiT

+ dr

I>

a and r = b

sp<<d as the r:llllit aod alwa)S """"''direct!} ""'..-.!the r.tbbit. fin<l


the <:qatoon oftl>c plh followed by th<: dog.
*-" IS. Repeul Exc.Y(i"e I"' ir lhC dull f\llh twic.."C lll' ra~L U' the rubbit.
16. A llllwlaJ posoloon IL - 0) in lh< fogu.e b<low spotS a pigeon at
the Otirn O);n& ,,;,h ,pa.."tJ u in lt'k: postll\e .v-drcx:lion. The hts\\:L

inlmcdh:udy Lalcs: oW olrer the piccun \\ith speed V > 1 , :alwJ.y.tt


hc:u.HtlS directly IOWL.Utllhc pifc,m. A l\cr l ime I , the pigeon it~ ul
pcxition r(O. ut) . If lhe tqtaation ol the pursuil Cllr\"C or the hwl: i!o
y y(x). lhcn dunn~ tio>e 1 O>e h.ll' l tra\clS distun<< Vt along ds
c.:urw. 8u1 .J.is1an.;:e oalona chis curve can be c:tlculatOO by o_~~~ u(thc
derinne niDgral

f , I ("-"),
I +

12. trheut i~ genc.-.ucd at a consram r.tlc 10 the roppcrwucof E>..ctc1SC


11 (pcl1"klf)N because of clcctricul cul'rcnt), lhc diffcrcntiul cqumion j ,

)'

'

replaced hy

d'T

dT

r-clr1 + -dr

= k.

when:: k il\ a c.onstaJll. ,..lnd T(r) i( ICrupcratures on inner aod ou1cr


edges r :~~ (I and r = b nr the insuhuion arc consla nl values ~ru and

r.

-, p
VI

( '

1/(x, y)
(LO)

dx.

1072

Ol!k('lltr 15 Uifferential Equacton;


(;t) Show ahal )'(.\') must satisfylhc intcsrodi~Jcrcntial c.quotion

x -dy - \' = -v
dx

"'*

18.

or

(a.) The following diOCrcntial cquaaion arises in the physics


bubble coalescence. Variable y is dimensionless 111m thick-

ness (joining 1wo 1ouching l:(lbblcs) and r is dimensie>nlcss

~r

time:

d'y - -5 (d")'
c
- - ar + b
-y - -.1'3.
d Tl - 4_1' dT

{b) Diffcn;ntiatc this oqua.ljon to obtain the sccoxl-ordcr dif

ferellti:tl C\{u!nion

d 2Y

Sllow 1ha11l-.: subslilulions 11 = d yfdT and tf.lftJT' =


vtlv/dy lead to a linear fi rst-order dinCrcmia1 equation in
u

XJ;Z = V
(c) Sohc

u1. Usi11g the foct that y must be ~ dccrcusing fuoctioo


ofT , and scuins the c.onstunt of in1cgr.11ion cqunl to ~cro.
show that

I + (dy)'
dx

th~ diffe-rential equal ion

for the pursuit Cut\'e OI' IJtc

d1

huwk.

dT = - \

I,

ay -

S + 9y 2 '

(d) Show thtnlhe hawk catches the pigeon o distAnce vV Lj

(Vl- uZ) up the


17.

y-axi~.

(b) A " uming that IOOac - 9b 2 > 0 ond that y(O)


lhat y(T ) is defined implicilly by

(a) An aircr-ilfl at posilion C{a. b) is H)'ing h> till.: righe along


t.hc line y
b witb speed v. A mi~silc is tired frOill lhc
origin with speed V > l-1. alw.&ys heading directly wwu.nl
the aitcrolt. Usc a disc::ussion like lhat in Exercise 16 to flnU
the path followed by lhc missile.

= =

(b) Plot the curve in pan (a) when"


h
I and V
Where docs lhc m ir;sile catch the. aircr::~JW

= 2P.

In

I. show

b\'
c
b
}~----+-+Y~--

5a

. II

I~

-4Ja7 +In \ I- -b + -c + I
Sa

/) 1

-- .
lOa

l ts.s Newtonian Mechanics


One oft he mosl imponan1 applications ofdifferential equations is in the study e>f moving panicles
and objeclS. In classical mcch~nics, motion is governed by Newton' s second law 11 .109, which
states that whe n un object o f const ant mass m is subjected to u force F, the resultant accclcmtion

is dcscribc<l by

= ma.

( 15.31 )

When F is given, this isanalgcbraicequationgivingaccclenuion a. I f we substitule a = dv / dt ,


we obtain a first-order diffti'Cntial equation

dv

F = mtlr

Cl5.n>

for velocity vas a fu nctie>n of tin-.: r. If we substitute a = d 2rf tlr 2 we obtain 11 second-order
differential equation for position r asa function of time:
(15 ..\3)

In practice. i1 is seldom this simple. Often. F is not given as a function of time. but as a function
of position, or velocity, or some other variable. In sueh cases we may have to change the
dependent or indepe ndent variable., or both, in order to solve the d iffere otial equa tjon.

\VIu;.n motion is in one dirccLion only~ we may di~pc.nsc with vectors and Q.111:Sit1e.r the :single
component of lhese vec~ors in thai direction. If r(t), v(t ), a (I), and Fare 1he componems of
r (l). v(r), a{r ), and Fin this direction, then equations 15.31- 15.33 take theforms

F = ma .

(1 5.34)

dv
F = m- ,
dr

(1 5.35)

= m d-

2r

dr 1

( 15.36)

respectively. We studied applications of the last 1wo equations in Section 5.2, and many of lhese
applications involved objects fa lling unde,r the in fl uence of gravity. We were not able to take
air resistance _into account at that time, but with the differential equation-solving techniques in
Sections 15.2-15.3, we can obtain more realistic results. H has been shown experimentally that
when an object moves slowly through a medium (such as fo r an object sinking in water). the
friccional drag is. proponionalto vel oci1 y, whereas when it moves quickly through the medium
(such as for u skydiver), the frictional drug is proportional to the square of vc.Jociry. We give
ex ample< of ea<:h.

I EXAMPLE

15.8

A stone with mass m, dropped into water, has speed vo as it penetrates the surface. During its
descent to the bottom. it is acted o n by g ra\'ity aod water resist.a.nce. that is proportional to the
speed of the srone. Ignore the appMent loss in we ight of the sto ne due lOArchimedes principle.

Find the. .speed and position of the stone as functiolls of time.


I:IIC'Jii;JiP)FW Scbem:uic
t()t l)lUSS

sirlkin,g iJl

W~ttr'

. ---.---I
water

Surf ce of

.;, : 0

v = l Q.

SOLUTION Let us measure y as poshlvc. dowm,ard tak ing y = 0 at the surtBce o f the. water,
and taking time 1 = 0 when the stone penerr-<Jtes the surface (Figure 15.5).
If Fw is ilhe vertical component of) the force of water resistance on the stone, then Fw =
- k v. where k > 0 is a constant. Since the total force on the stone has venical componem
mg - kv. Newton~s seco nd law gi"cs

'I

dv
dt

m- = mg - kv,

t ~ 0.

Note that the force on the right is not a fu nc.tio n oft , so that the equation is not immediately
imegrablc. It is, however, separable,

dv

kv - mg

dt

and solutions are defined implicitly by


I

- In lku
k

- m.gl = - - + C.
/II

An explic it solution is found by solving this equation for v; the. result is

v(t )

Since v =

Vo

when 1 = 0.

mo

mg

vo =~+D

or D=v,;--

The \'elocity of the scone as a function of time is cherefore

v (r) =

(uo- mg)
e - AI/ m.
k

mg +
k

Because v(t ) = d y fd t , we set


dy
mg
dt = k

Vo -

mg)

- kl f m

'

and 1hi~ equation i~ immediately integrable- to

v(t)

= Ill/(
1 - -m
k
k

(v

"'kR)

0 -

tl/

+ E.

Since y (O) = 0.

mg)

0 = -111
- ( ~~o- - +
A
k

m(t'o - -mg) .

= -

or

Cohequendy. ahe f.li.saa nce !'V11L. b) the stone a:s a functtOR of limt 15 glvcn b)
m~
mt)
_., ...
T'
- m
y ( ~0- T
(I-~
. ).

)(I) =

Note thnt the velocity of" the ston~ does not i ncrease indcfinilely. In fact. as time passes. a

limiis velocity i~ llpJ>rOuchcd:

lim u(l) = lim [ -mg


l-a.;
k

1- (N

mg) e-J.ttm] = ....-


mg
+ ( t'o- ....,.;
"

Thi~

is c.all e<~.l 1he trnuitwl wlaL-in of the st01~; it i~ r d1re<.'t re~uh of the as:>SUn'IJ)lion dltlt
wa&er resi.o;rance is: proportional to insl4.tnl~u\eOUS veloc:ily. \ Vc C\)UkJ have predicted lt from the
cJiffcrc11lial equation describing 1he notion of the stone \Vith the miti~tl velocity of the ~\One
being t\). the force on it, n.omtl) mg - kv, is initiall) mg - tu, gnl<IU<tlly dccfcasing the
velocit) increases. As u appm.tr.:hes m~ I k. the force a pproaches zero. as does the acceleration.
and the s tone attains 1em1inal velocily.

I EXAMPLE 16.9
II kydi,cr arld his

pat"o~chute

llli\C mass 111 kilogt"dms. A< he l>lungc> toward canh (becau<e

or gmity), he also expcricnoc. air rcsistanc~ that i< directly proportional to the square of hi
1 n ~uJnlllflCOUt vclociry. A~~uni ng tl\a( hi~ vcnical velocit) i~ t.cro whc:.n he l ea\~ the plane.
fild the veni<:al ron>pOncnt of his ,<focity and lti~ 'enic:ll pu>ition .IS fwlCtions of time.

'-.;01 lffiOS' l tl us mcawn: y as ~itive in chc down\\llinl dirc<.1ion. taking y = 0 and time
I
0 mthe in>tam thc sl-ydivct leaves the plane (Figure IS 6). I I' F. is the \trtical rompu11ent
of the fo rce u f air resishlncc, 1h<:r1 P, = -kv2 where J.. > 0 i1 u consumt. Since t he tow I force
Oil the !-!k)diver during the fa ll hm~ vt:nical t'OI11JX>Ilclll mx - ku1 Newton's second law give~

1=0.

)' =(),
''0.

dv
111 -

dt

= mg-

,
kv- ,

? 0.

lis in E~nmple 15.8, we see that the skydiver has a terminal velocit) defined by mg- kv 1 = 0
If we denote it b)' V = ../fflilk. tl><.'flthe differential equation for I'(I) can be scparotcd in the
form

dv

1..

~~- --dt.
v1 - V 1
111

Solutions are defined implicitly by

kt + C =
-m

( t

V)(u- V)

= - -(-lnlu
2V

dv = - I
2V

f(+

- I + -I- ) tlv
v V
v- V
1

+VI+ In lv - VI) = - -

2V

To find explicit solution~. "c multiply by 2V aod exponentiate.

v=
_u -__
u

+v

D e-21Vtf '"

ln

lu -+ VI
u

I S.S N~tc.J:li :tl\ Mtcbani.:s

where D

1075

= c:!VC When we cross-multiply. v- V = D(v + V )e - 2kVt/,.. from which


V(l

v=

1 _

n , - JW tlm )
D~

lk.Vt /m

=-

The initial velocity v(O) = 0 requires that 0 = V ( I + D )/( I - D) ==} D


1. If we
substitute V = ../mgf k in the exponential functions, we o btain the velocity of the $kydive r in
the form
V(l - e-'~ ')
U(l )

= __..:._..,;_,.-==-..:.

1 +e- 2 ~
h rcflect.~ the face chat che limiti lg velocity is V since lim,_,. 00 u = V . To obt:lin the distance
fallen by the skydiver. we set the velocity equal to d y f tlt and integrate with rest>cct to 1:

)'(I)

The initial condition y(O)

=0

implies that 0

V Jmj(kl/) In 2

+ C. "''d therefore

y(l) = Vt + f/!:v ln( l + e- .;m;;;')- {!f;v tn2


\ kg

\ kg

VI + ~k In [~(I
+ e - 2Jri"Fi')].
2

The fi r>t te rm is distarlce fallen by m>object with const>lnt speed V, Since the second tenn
is always negative. the d i;wnce l'ollcn by the skydiver is alwa)'S less than V 1. as should be
expected. sinc.-e the :J<yd iver never achieves terminal vclvdty.

\Vhenever an objecc such as the blcx:k i n Figure 15.7 is moving. over a :surface. there isresistance

to the motion. This resistance, called friction, is due to the fact that the imerface betwe..c.n the
block and the surface is not smooth; each ~urfacc is inherently rough, and this roughnes.s retards
the motion of one surface over the other. ln eflect, a force s lowing the motion of the block is
created. and this force is called I he fOI'Ce offrirdon. Nlany experiments have been pcrfom1ed to
obtai n a fimc tiOJil\1repre~enrar ion for this force. It turns out that when the block in Figure 15.7

Blvc~

Surface

M icroM:opic view of
~ !';ud"ac.c or block

~==~~

Micoscopie view
of ,;uri'tee

slides along a horizontal surfuce, the magnitude of the force of friction opposing the mot.i on is
given b)

WI = JJ.mg,

tl>.m

where m is the mass of the block, g is the acceleration due to gravity, and J,L is a constant c.alled
the coefficient of kinetic fric tion. l.n other words, the force of fJiction is directly proportional to
the weight mg o f the block. We caut ion the reader that this result is valid for the siwation shown
i.n Figure 15.7. but it may not be valid for other con fjg:ur~nions (say, perhaps. for an inclined
plane). Furthermore, there is a coefficient of static friction that is used in place of J..L if the block
is being accele rated from rest.

I EXAMPLE

15.10
A block of mass 2 kg is given initial speed S ml's along a horizontal surface. If the coefficie nt of
kinetic frictio n between the block and surface i ~ fJ. = 0.25. how far does the block slide before
stopping''

SOLUTIOI\

let us measure x as positi\'C in the direction o f motion (Figure 15.8), taking

x = 0 and 1 = 0 at the inst ant the block is rele,lscd.

I 3 td!la3 Ud
I ScheiOOiic
tor block n'IQvmt; O\'et a sur1acc

t=O.

x =O.

on the block is

The

F = -0.25(2)g -

x -component of the force of frictio n

(g

= 9 .81 }.

According to Newton s second law,


.l

dv

2- dt
from which we get

v(l ) = - - 1 +
4

C.

Since v(O} = 5, it follows that C = 5, and

u(l )
But u = dx / dl. and hence

- - 1 + 5.

dx

g
- -1 + 5.

dl

lntcgmtion gi,-es

.r(l )
Because we dtose x

= - !1 2 + 51+ D.
8

= 0 at time 1 = 0. D must also be zero. anti


.t(l ) =

_!,2 +

51.

The block comes to ~t when v = 0, that is, w hen

- -!

+5=

or

20

The position of the block at this time is

x =

-H~oy + s (~0 ) = 5. 10.

The block therefore slides 5.10 m before stopping.

We are being hired as consultants by the olympic sk i team to analyze race times of down hi II
skiers. T he team knows the factors that derennine the speed of a skier, and has a wea lth

of information for us to use. but it is unable to determine which factors have the greatest
inOueoce. and therefore which ones it should cooceotrat.e on in t-raining.

SOl .UTI Of'; Let us begin by establishing a fr-.rnework in which to analyze the s peed
of skiers and then identify factors Lhat influ.ence their speed. Certainly, downhill courses
have many turns and areas where the course is very steep and quite tlat. Let us begin by

simplifying things and assume-that the couJ'Se is str-djght and that it makes a constant angle
1.0 do is wi.n

& with the horizontal (F igure 15.9a). WhM the olympic team really wants

races and to do Lhis, it.s skiers must minimize their times. With our simplified set up with

no curves and constant an~ lc. this occurs when speed is mdximi7..cd. We can find speed

os a function of either time or d1staoce ~ravelled: which would be bencr'! There would
"'em to be no n<hant~gc of one over lhe olher so lets set out to hnd. as u;ual. speed as a
functoon of ome.

There are three forces acting on a skier. gravity down the hill , air drag. and friction
l>etwn ki' and "'ow. If Lhe mass of Lhe skier ism. the component of ,raviLy do" nthe
hill is m,t sin8. where g = <J.S I. Air drag Fd. which acL~ up the hill. has been shown
e<perimentally to be directly ;)roportional to the area A oft he skier facmg downhill (Figure
IS.9b), the<quare of speed u, and the density p of the air; that is Fc1 = ~pA u2. where 'I
j, a COI\Stam ectllcd the (.hd coefficienl. Finally. friction Ff . whith al~u acL\ up de hill.
is proportional to the normal force N exerted by the hill on the skier. F1 = 1.<111 11 cos 8,
where J1 is the coctlicient of kinetic frictioo. \Vhcn we substitute these into ~C\\Itons
:.ectmd law r(\r motion down the hi ll. weohti:lin

dv
111-

tit

= mgsonB-

Thi~ di rfcrential
1

~pAv

- J.Wrgcos!J

2
mg(sinB- 11 oo;6)- ~pAv.

equ:..uion ;, s-eparable. but in order to ~i mpl ify c.ttlcul atiou ~. lei u~ ror the

n10ment >el a = mx (sin 0 - p. cosO) and b 2 = rtPA . in which case

dt
Ill

d...,.v~
= _2_, (
= -,-1
1 1
a

b v

2.a

+ bv

a - bu

) dv.

Notocc thut we need 1101 worry about v being equal to aflJ since af/) is tbe temlinal speed
uf the '~ier Solution' of the differential equation are defined implicitly b)
1
- C - - 1 (In Ia

2trb

+ bvl -

In Ia - bvl) = - I In la+bvi
2.ab
a - h~

To solve this for v, "e exponentiate and crtMS multiply.

+ bv =

where D =

D(a - bv)t,lobtt,.

e24bC.

v=

a(D~Z.bt/ - I)
b(D~!Dbtl

+ tj"

Now all skiers start with zero initial speed. bu1 in the next couple

of seconds. they use their poles to accelerate quickly. Let us a>sumc that all >kiers pole
for the same length of time. and we begin ow analysis thereafter. which we take as 1 = 0.
To differentiate between skiers, let us assume that the initial speed is an unspecitied v0
This requires
a(D - J)
a+ bvo

vo=

b(D

+ J)

D=

a- buo

Thu:;,

a [(a -t h11o) - (a - b110)r1Aix/ m]

v(r) =

(tl -t bvo) -t (o - bv0 )e

1oh<f ,.

Thi!i is. .'\<'~mewhn l oompl ic ar ed ~ perhHj'\1\ ir would h ~t\'C heen hc::rrer tO find v in 1cm1s of
diStallCe Lta\'elle<.l. Le( us quick I)' find ouL 1r we lel s be distaflce skied down 1.h e slope.
Lheal
cal\ write

'"'C

dv (/s
:z
m- - = ll

ds dr

vdv

ds

h 11

i me<~.nnion of w hi ch gives

I
.,
., .,
- - 1 lultr -l>-v-1 = -s
2b
m

+ c.

When this is sol"ed tOr v 2 the result i.s

,, = a'
_ _
v

I f' we cake.<

0 when poling

'

b'

D e - Ur :t(m

ceases so chat

v(O)

= Vo, then vt =

11 2!l>'

- D. and

Lherefore

vl

a:z b2

(a2
- vl)
b2

1!- JJ,l.,f,,

= mg(sinB - J.LCOSIJ) (I _

(' - l(lpA r;fm )

+ Vl , - l )JPA .(fm.
0

1'/PA

This does appear to be simpler than the expression for v(l) . The. parameters over which
we have no inllucwc an! s , 8 , g , anc.J p . The. llthc.r~ m , r1, A, J.L , and vo arc altcn,ble. To
detcrntine whi ch of these has che gremest effect on speed, we shall change each in !Urn by
10% and calculate the percentage change in v. Lct us spccil'y the roll owing base vo.lucs:
Lhc d istance t~t which to compare speeds .r

50 m: lhe (mgle of Lhc hill

= 1C / 1 2~ the

9.8L ; the den~ity o r air p = 1.25 kgfrn~: the m~ss of


the s..kier m
80 kg; the coefficiem of dynamic friction 11- = 0.05 for well~ waxed skis;
the drag: coefficiem q = 0. 7 delermine.d experimeJnally: and the initial speed v0 = 4

accelcr.uio n due to gravity g

m/s. For cbese values, the speed of the skier at 50 m is 13.0 m/s, compared to a termlnat
speed of 19.8 rn/s. The numbers in the Table 15.1 arc percentage changes in speed at the
50 m di!;tance due to I 0% c hanges from ba~e values in m, 11 . A , J1 and v0 , respectively.
Perhaps s urprisi ng ly, an increase of the initial speed v0 of the skier >tfter poling is the
Jeast significa nt. Decrea~ing the area of the s kier fucing downhill is rnnst signifit"8nt.
suggesting that skiers should be in a tuck position as much us possible. Decreasing the

drag coefficient ~ is equally significant. but It()( much can be done to decrease it, except
for making ski..suits slippery. Heavier skiers should h:we an advamagc O\'Cr liglucr ones.
but an increase in mass is likely co be offset by an it>erea.se in A. Decreases in If. arc
likely co be s mall unless major technological advances can be made in waxes and/or ski
materials.

m
1.17

'I
1.29

A
1.29

Jl

1.08

0.58

EXERCISES 15.5

I. A ldy with ll\9SK m is cauo;ed to tl'l(we in lhe positive x direction


by u constunt force wi.th magnitude F. It is also ac.fcd oo by a resistive
force lhu.t is proportional to the ~uarc root of tht.: s pct:d a.1 uoy in:sw.nt..
If tJ-.e mngnitude of l.his Te~istive force is F* wben the speed is u,.. fine!

(:.t) Show th:lt Lhe diftCrenti:.tl equ!'ltion describing the motion


of M is

d 1x

M- 1
di

an expre!l.s-ion for itt; tcm1i nal speed.

= -Ill

+C

~Wall

between the tires and road was lc:;s tb.un l, whaL can you say uboutlhc
;)peed of the car tx:fore Lbc brdl:.cs wen: applied? Arc you tc~l ifying ror
lhe l)rosectuion or the defence?

tO/)...

ror

10. Repectt Exercise 9 if the initial \'elocity has magnitude 100 nlfs.

II . For how lon.gdoeslhe mass in Exercise 9rise if II is thrown upwant


witb velocity 20 mJs?

J2. Wl:.cn a bod)' falls in ajt, it is acted on by gravity and a1so by


a fon:c due to air rc.si.~tanc..:: thai is proJXmional to the square of iL~
instamaneous speed. If the body is initially projected down\V".trd with
velocity Jess than its tcnninal \'Ciocity, find its velocity as a function of
time.

7. nlc-English longbow inn1cdicval times wa.\ regarded to be o..ccur.uc

8. A spring (wit.h constanl k) is auachcd on one end LO a wall and on


lhc.othe.-ent.ltoa mass M (ligul'e follows). Tile mass is sa imo u)Qiion
along 1he .t-axis hy pulling il a distance X() 10 lhe right of the posilion
it would occupy were the spring ttnstrctchcd and given speed Vt) to the
lefi. During the subsequent motjon. there is a frictional fort-e between
M and the horizontaJ surfat'C with <."'Cflicicnt of kinetic friction equal

x).

A lkS 11\:lSS falls undc.r the influence of g.r:wi1y. h i.._ alsu ac1ect on
by Wr resistance proportional to tbe squurc of its velocity aod is 5 N
wtx:n its velocity is 50 nJ/s. If the \'Clocily of the n1as; ha:s mugniludc
20 nlfs at time 1 0 . llnd a fo rmula iOr its velc.:-city as a function of
titne.

whose IIKtgnitudc in f'ICwtons is equal to the square of the spc:e.d ol'tJtc.


car in mt!Cres per second. Find l hesL~ed o f the car and ils dis1ance from
1he intersection aner 10 s.

+ t<Mg(.t,1 -

* 9.

exerts a constanl for'c e of2500 N: and a.lr friction causes a resistive forec.

or

't= 000 '----El

wilh brate~ locked and wheels sliding. The ;;.tid mart on 1he mad
me::t$urtd 9 m. If you as..~m.e that the coeflicien1 of kinetic friction

at 100 rn ot more. For an arrow to tra\el a hoti~..)nLa l dist..ance of


100m wilh maximum heigh! 10m. lind the inilial speed and ang.lt! of
projection the arrow. Ignore air friction.

the firs t time, usc lhe equation in part (b) to detenni.nc x


as a funaiorl of f.L, M , g . k, x 11 a11d Vo . Discu."'S the
POSSibilities or x be in~ posithe. ne~alive. and 11'0.

+- 4. You arc caJied on as an c~pcrt to l.c:stify iu u traffic at.tidcnt hcuring.


The question concerns the speed of a car that made an e111ergenc..:y stop

of m. g, and k.

~~

lluu.pt'e t each of the. tcnns ln thjs equation physic-all)'.

UlC IX)i:IL as a function or lime.

(b) Oelennin ethedi~~:l:tnce lr:avcllcd by lhC particle in :lchicving


95% of its tcnniJ1.al speed.

v(xo) = -vo,

(c) lf x represents thC' po&ition m which Af comes to rest

(b) What is the limiting s peed of the bont?

(a) Dci-Cmli nc lhc ti me r(."t}uircd for a (>{trlidc with ma~s: nr to


accelerate from rest to 95% of its tcnninal s peed i.n cern\S

-(xJ - x' ) = - (' ' - 11~)

speed of the boat and is 200 N when the speed is 30 knVb.


(a) If the boot starts from rest and the engine c.~Cc rts a consuu1t
force of 250 N lnt..hc dlroction of 111olion. lind the speed of

6. Small pttrticlcs moving in a Ouid experience a drag force kv P*


porlion:tllo Lheir speed v.

t1(0)- -v0 ,

and t hat therefore

A b<lllt :lnd iiS contents ha~oe mao;~> 250 kg. Water exer1s a resi.1nive
force on the motion of the boot t.h~ t is proportional to the jost:tnl:mcous

x(O) - x,.

dtl
Mv- = -kJt +t<Mg.
dx

* 3.

S. Acarofmass 1500kgSiansfromrestmanintersection. Theengi ne

+ {.L M g,

for v = v(l) and usc lhc condition v(O) = l'o to cvaJuutc the cons1ant.
We chose first to solve for tJ(t) and then to c~aluatc Lhc constant
lnsu::.ad. fi rst use the condition v(O) = vo to C\laluah:. C . atlll then
solve the equation for v(l).

-kx

jfwclukcl
Oat thcim;tunt lhatmolionit;i_njtiuu.:d. When
is this equation valid?
(b) Since I is cxpl.icltly missin.g from tbc equation i.n pwt (a.),
show that it can be rewritten in the form

2. In Example 15.8 il was necessary 10 sol\'e theequ:uion

- In Jkv - mgJ

13. Re-pe-nt Exercise. 12 if the body is projccte.ddo\vnward wi.th velocity


greater than ils lc.nnioal \'Clocity.

14. For how l Oll~ does lhe


upward with velocity t,'(l?

J5. A lk.g rock is duown vertically upward wilh s-peed 20 tn/s. Air
rcsis1ance 10 itS anotion when measured in newtons h!tS tnagrUnadc equ1al
to one-tenth the squwc of its speed in metres per second. Find the
ma;~imu m height attained by the rock.

ma.~

in fuerdse 12 rise if it is thcown

1080

16.

ChJpttt IS 1:>11Yat:nliaJ t:quations

(o) Assuming Ihat g 9.8 1 on Ihe surface of 1he earth, where


,. = 6370 km. showlhal G M .asasingleoonSlanl, is equal
103.98 X 10".

m is lhrown upward with speed u0 . h i~ acted on


gravity and air resistance proponionallo !he S<]uan: of
\ctocity. find a r(X'mu1a forspxdofthe mass on ils ascenL
Is thj) fonnultl also '1l.lid for its speed wbcn it begins to fall?

(a) A mass
b)'

(b) The diO'crential equa11011 for the m01ion of the objec! is


mdv/dt "' -GmM/r2 Showlllal ilcanbecxpressed
inthcformvtlvfdr c - GM/ r 2 .undsolvcitfoc

(bl Find a fonnula for its height How high docs h rise?

17. An ob;ect of mass m is injected imo a medium a1 speed v0 The


n'ICdium cxcns a resistive rorce proponional 10 veloc.ily on Lhe mass
with constant of pruportion"-lity fJ. ln addition. a constant force wilh
magnitude F oppo.scs the motion of the mass. Show that the mass
comes to ~sl o.ftct it has tnwc:llcd a d.istw11CC

-uu~o - -Fm In ( 1+ -flvo)


fl
fl ~
F .

ll!king time (m/fl) ln(l

V ~

11\a:):)..

t 22. The coonJinatc~ or an

electron movina in the ..l')'plane about ils


nucleus :trc gi"cn at any time 1 b)'

+ )'

3.

tit

GMm
(r + R )''
where G is a constant R is the radius of' the eanh. M is the mass of
the e~~rth , and m is the ma~~ or the pll)jectile. At this point the moons
gravitation:ll anroction has n'lagnitode

G/lf"m
(a

dz.,.

r )'

g R2

8 R1

tlt2

(r

+ R )' + (a+

R" - r )'

where 8 and g w-e g.ruvitationu.l uccclcn~ l-ions on tbc surraces of lhc c.u.rtJ1wtd the moon.
(b) Pro,-c that the velocity of the projectile ala distance r above
the sul'fncc of the cunh is defi1led by

path.

23. In Example 7.34 of Sectic>n 7.10 we derivC<I the cscope velocity of


~ projcctilc fromahc earth'~ s urface~ on cm:1':,)' l}l'inciplc). In 1his
exercise \ \'C obmin ahc .;amc rcsultll';:in:, diOCrcnai ;d equa 1 ion~. When a
ptojectile of m:t m . fired fmm1be ron.h's surface, is a ditnnee r li"om
lhe. centre of the ta.rth, ahc n\a.sniaude of' the force of llttra.etion on it is
givcnbr Ncwton'suni,cwll<iWOf~raviLation. F ; GMm jr2 \\'hc1c
M i~ the m3S." of the canh and G i.; a cono;1am. U!>e Newton':;: ~n d *
law a.nd a subslitution correspc>nding to 15.30 to lind the velocity of the
projectile as a function of r . What minimunt initial veloc.ity guaranLC.cs
thal lhc J)tOjcctile escapes ahe l:,ravitational rield of the eetnh?

+ R'

whete M is Ihe AlaS'- of Ihe moon and R is it~ radius.


{a) Show that if only the two foi'CCS above aro considered lOact
on the projectile! then the differential cqualion dcscl'ibiog
it:) IHolil)l\ i:S

= 4.

An object is dropped from a height of 5000 km above lhe earth.


'[be only force acting on it is gmvity with magnitude F = GmMf r 2
where G > 0 is a to nstaJlt. m and M are the mass of the object and
the earth. 3Jld r is IJte distance between the object and the centre of the
cm1h.

mls.

26. A huge cwmon Jircs a projectile with initial velocity ~o diroclly


toward the moon (figure below). When the projectile is a r.li$1ancc r
abQvc: t.hc c-u rth\ surface, the for<.:c or uu~-ti on llf Lhc: euJ'Ih oo lhc
projectile has magniludc

Find the path followed by Lhc electron by climin:~tin,g t und u!ing the
condition thotth:clecllO!l posses through the point (0. 3). ldcnt.ify the

** '24.

/'

s i~: thrown vcrtic::~11y upwW with speed


20 m/s. Air cxc11s :' rcsisli\'C force on the stone propCirlion:el to lhc
square of its instantaneous: ~peed. and h a~ rnasnitudc 1/ 10 N when
the speed of the stone i~ 10 mf'- F'ind rhe time \IJhtn the S1one reiUms
10 its projection point. Compare the rc~u ll with the linlC: taken if air
resistance is neglected.

.., 21. Find fotm ulns rOf" speed ami di.stuncc tnwcllcd for the nut.ss in
Exci"Ci.SC 20 if a.ir rcsblancc JU'Of)Ortionalto ,clocity alMl ac(.:) on the

2tit

I I.37 X 1()6 - r

..,. 25. A ~tone of m:l$~ 100

... 20. A lll.1SS m slide( from ~~ down a friclionlcss plane inclined u1


angle ct IC\ lhc hofizomal. Find a f()(mul:J for Ihe lime lake.n to mwel a
di.slan.Cc D llown Ihe: plane What is ils ~P..."'i 31 1hi1: rime7

tly
- - lx

JJ

I 9. We l~ve tiOIV(.'tl one-dinlCtls.ional displac:cmet'll problems for (lb.


jcccs acted on by rc.,s.is1ivc rOtcC$ proportional to v-elocity :.tnd to the
"!Ulllt of velocity (Exompks 15.8 and 15.9). In Exercise 18 we solved
:l ho-dinll:nsion!'l1 problem wi(h n:sistW'ICC proporti()nal to velocity.
Use the projcclilc problem or Exerc-ise IS to .:howth:lt we c-annot !>Oh'C
two-din\cn.sionnl problems wl1h resistance proportional to the square
of velocity.

tlx

10

(c) Sub>lillne tll'/ dt for v in port (1)) and ,;oii'C for an impli<:il
equation defining r as a fun(;Liun of 1. llim: 1l1ink about
SCI~ing I"
11.37 X 106 ~in 2 0.
(d) liow long does !he object tate 10 fall to Cllrth?

+ flo/ Fl todoso.

>t lK. A projcclilc Wllh mass m is launched from the origin with initial
sp..'CCI tt0 a1 an angle 8 'WIIh lhe f>OSih\"C .r a~i~. If il is ac1ed on by air
resislancc -{sv that i" PI'OI)()rtional to its '-'Ciocitr v. find lhe veloci1y
and posh ion of lhc projeclile at any lime.

- 8.37

v'

2g R'

2g' R''

~Eanl>

'-.;

- + fl + R - r + .~2~R0
r + R
"
1 - o Moon I
-

Ill
I

t--'-1

'""\

R~ ~"

Z"J. Newton's second l:1w stales that if an object of"ariable nla:Ss m(t)
is s ubjcclc.d 10 fo.,:c F (t ), then

d! (mv)

= F.

where t is time aod ,. is IJte velocil)' of the object. A uniform chain


of lenglh 3 m and mass 6 kg is held by one end so Lhal Lhe olher end
just touches Lhe floor. If the chajn is released. fi nd IJte velocity of IJte
falling chain as a fuJtCljon ol' the length o!" chain sljJ] falling. How fasl
docs IJtc end hit lhc floo r?

15.6

Li~r

l)itfen;onl'i:.l Eq,u:nivrt$

1081

l ts.6 Linear Differential Equations


ThrQUghout Chapters 5 and 8 and Sections 15.1- 15.5. we ha\"e Stressed the use of differemial
equations in solving applied l>roblcms. We have considered examples from such diverse areas
as engineering, geometry. ecology, and psychology, hoping thereby to illustrate how \"dluable
differential cquotionscon be in nuxlelling situations mathematic:aJiy. Perhaps the mos1 import....nt
type of differential equation is the linear differential equation.
DEFINITION 15.4

A differential equation of the fonn

,,

,-ty

,~,

+ llr(x)-t + tlz(X) d
, + + lln- t<.t) - + llnC.t)y
dx'
xn-_
dx

llg(x) - '

llx'!

1-

= F (x)
( 15.38)

is called a linear differential equation. It is Jim order when a0 (x) ~ 0.

N01e in panicular that none of the derivatives of y(.r) are multiplied together. nor are
they squared or cubed or taken to any 01her power, nor do they appear :.; the argument of any
transcendental function. All we see is a function of' multiplying y . plus a function of x
multiplying the first c.le~ivarive of y . plus a function of x multiplying the .econd derivative of
y , and so on.
If 11 = I. equation 15.38 reduces to

dy

llo(.r) -

dx

+ llr(.r)y =

F (x) .

and at any point at which a 0(.r) . 0. we can divide to obtain

dy

a (x)

1
-dx + -y
ao(x)

If we set P (x)

F(.r)
ao(.r)

= a (x)f a (x) and Q(x) = F (x)fao(x). we have


1

dy

dx

P(x)y

Q(x):

that is. ever)' linear first-orderdiffe~ntial equation can be expressed in thi< fom1. We di~u.<.~ed
equations or this t)'JlC in Section 15.3. where it was shown that such equations have a gcncml
solution
y(x)

e-1

~(<)dx { / Q(x)ef ~(t)dxdx +

In other words. we already know how tO solve linear

fi~order

C}.

ditiercntial equations. and

therefore our di~cus'iion in the next tive sections i~ directed prim:lri ly a1 second- and higher-

order equations. Keep in mind. however. that all results are also valid for first-order linear
equations.
Bc:c-usc equation 15.38 is so cumbersome. we imroc.luce notation to simplify its representation. In particular, if we use the notation D = dfdx, D 2 = d 2 f dx 2 , and so on, we can
write

ag(x) D"y

+ a 1(x) D " - 1y + a 2(x) D"-2 y + + a._ 1(x) Dy + a.(x )y =

F(x )
( 15.3Y)

or

{ao(x) D"

+ a 1 (.r)D"- 1 + a.2 (x) D"- 2 + + a,_ 1(x ) D + a,.(x )Jy =

F (x ).

( 15.40)

The quantity in bnK1::S is Qllled u dirfcrcnliul opculor~ il QJ)erotes on whatever follows it i111his case. y. It is a ..diftCrtntial" opt rae or because it opel(ues by taking derivatives. Because
lhe Qpcrawr invol vcs only x ~sand D s. we denote it by

,P(.<, 0)

= llo(x)D" + tlo(x)D" -' + ll>(x ) D" - ' + + ll,. _ ,(x ) D + to,(.t ) .


(15.41 )

The general linear nt" .ordcr difJCrential cquattion can the1\ l>c rcpccsentcd \Cry simply by

tf>(x. D )y = F (x) .

( 15.4 2)

For example, diflcrcmial equation 15.3,

.ry'' + y'

+ X)'

= 0.

is called Bessel's dif.lert!lltial equation of order u ru. lrl opcnu.ot 1tObU.ion we write

(xD!

+ D + x)y =

or

(x, D)y = 0.

where ,P(.t , D)
x 0 2 + 0 + x.
For the d iircrcntiul equation

y"

+ 2y'- 3y = {"- .

we wrire
where ,P(D) = D' + 20 - 3.
\Ve now indicate the metlning o f th~ ter-rn /innu. Supp<1se I hat I . is an OJ'IerniOr thai Oflet:ues
on each function y(.t ) in some set S. r'Or cx.amplc, L might be the operaoion ohat multiplies
e-ach ft~~H.:Lion by 5, u r perhaps squares ca<.:h function. or perhaps diOCrcntiates eac..it function. II
is said to be a linear opera1or if it sa1is11es the tfoiiO\.\itg definilion.

DEF I NITION 1 6 .6

An operator

L is said to be a linear oper ator o n a set of funclion-: S if for any IWO


S. and any constant c.

functio ns )'o(x) end .V1(x) in

L(y , + Yzl -

Ly,

Ly,,

L(C)'t) - c ( Ly 1) .

( 15.43a l
( 15 .4:1b i

Many or the opcnuions in calculus are therefore linear. For instance. taking limits ls a linear
o peration. as is diffe rentiation. antidiffercntia11ion. and taking defi nite integrals. On the other
hand. taking the square 1001 of a J>Osirive functi on is not a linear operation. since

It is notdim cult to show that thedifferen~ial opcr-.uor ,P(x, D) in 15.42 is li.near ; that is.
,P(x , D)(y 1

+ y,) = (x, D)y 1 + (x, D)y 2 ,

(x, D)(cy 1 ) = c[(x, D)y,] ,

15.6 Llne:lr Ulfferential Eq111li(lriS

1083

and because of this, dUfecential equation 15.42 is a lso said to be li near. Tbe foJJowing two
di fferemial equations are not linear. \Ve say tha11hey are 1wnlinear.

In particul;u. if we s ubstitute y 1(x)

, (y,

dX

+ y2(x) i nto the left side o f the fi rst equation. we obtain

+ Y2) + (y, + y,)-' =

.d y ,
-d 2
X

y2

-d .2
X

+ Yi' + 2YJ)'2 + y2.2

If we substitute y 1(x). then y2 (x) . a nd then add the results, we fiud a d ifferc m expression:

Unle.ss othen vise indicated, we assume that coetllc:ient func tions tlo(x), a 1(x) , . . . , a11 (x)
ami the runctiou F (.x) in linear d in'ere.ntial equa1ion 15.38 are all wminuous on $<.nne open
intc n'lll /. We also assume that a0 (x) =fo 0 at any point in / . Solutions of the differential
equation must necessarily have derivatives o f orders up to and including n at each point in I .
Bulcxi ~Lcncc: oflhc./lth dc:riYalivc impl ic:; conti nuity orall lO\VCr-ordcr dt:rivativc:.:;. Furthermore.
we can write 15.38 in the form

d"y
l
[
d"-' y
]
= - - F (x) - a 1 (x) - -1 - - a.(.r)y ,
dx"
ao(x)
dxwhere a ll runctions o n the right are conLi nuous and n 0(x) # 0. Hence, when derivatives o f
orders up to and inc1uding 11 - I a rc cootinuous o n / . so a lso is the n h dc.rivativc d11 y /dx" .
Thus, solutions must have contiluous dcrjvmjvcs of orders up w aod including Jl.

EXERCI SES 15.6

ln 'Exercises J-10 pnwc either thal the operator Lis linear o r thai il
is not Jlnear. In c:teh c--.tsc assume lb:H tbc sci S of fu.nclions on which

d' r
12. 2.r-

L opcralcs is 1hc set of all

J'lv
13. 2.r- + x'y x' + Sy'
dxJ
d 3y
d2 r
dr
t4. x +
Jx - - 2- + 1
dx 3
dx2
dx

fu.n~t i ons

on which L can operate. For

in:Hancc, in Exercise 6. 3SSUII"IC that S iS lhC SCI ol' a.ll fUllCtions J'(.X)
that h!}, t: !'I iinn deriv:n ive tlyj d.t: .

.1.. L :nultjplicsfuncti.oos y(x) by 5


2. L muhiplicsfunctions y(x) by l5x
3. L adds the fixed timction z( x) to functions y(x )
4. L la kes tbe limit of functions y(x ) as x approaches 3
5. L 1akes the limit of runctio.ns y(x) as x approaches infinity
6. L 1:\kes the fin;1 cteri\:uivc (l( func1ions y(x ) wilh re.~pec1 hl .r
7. l. lakes the lhird derivative of functions J(X) with res.pecl 10 X
8. L takes the antiderivalive of functions y (.r) with respect 1.0 x
9. f lakes lhe detillile irtlCgi';JI Of funcLions y(x) wilh re..~)e(.1 IO X

fromx = - l lo x= 4

10. L takc.s the cube root of fuoclions y (x )

In Exercisc-.s J 1-20 dctemline whclhcr I he dit:rerential equ:uion is linear


or nonli near. Write lhosc equations lbllL!arc Linear in operator nol.aliOn
15.42.

J 'ly
..
I I. 2x +x' v=x +S
d .{!

dx'-

+ x 3r

= x' + Sv
=

15.

d 3y
dxJ
<fly

d2 v
d\'
- 2~
t1X2
tfx
,p\.
d\'

x - - + 3.... -

1.6. 1' -

dx'

+ 3X -

+ .,-! =

- 2 ....:.. +

dx'

dx

l7. y" - 3/ - 2y = 9scc 2 x


19.

Ji+'Y' + x' = 4

\' =

IO<inx
IOsinx

tOsinx

18. yy" + 3y' - 2y = cr1

W. y"" + y" - y = l nx

* 21 . TIM:! Laplace transforlll of a ruocaion y(z ) is defined a..;; the functioo L ()") =

1 ,. _, C

- .H

y(t) dt, poovidcd that lhc Unpropcr intcgr"J.) c-on

vcrgc.s. Sbow that if S is lhc scl of all functions !hat have a Laplace
tr.msform, then Che operation of1aking the l....:1place tr.msfonn is linear
on S .
TI1e finite N>urier cosine transl"onn of a !'unction y(x) is defined
'~

* 22.

as L (y)

/.

y(.r)cos u xdx. where u i.s a nonnegative intege1.

provided that lhc dcfinile imegraJ exists. Show that if S is the set of aJI
functions tha1 ha.ve a fini te Fourier cosine transform. then the opcra1ion
of 1ak.ing the tr.tmsform is linew on S .

1084

Cbal)-lcJ I $

I 15.7

l) ifT<!f\"IUia l EqtmliC,Kt)

Homogeneous Linear Differential Equations


1 \ vo c lasses of linear dill'ereruial equ:uions present thcmsel\'cS: ahose for which F (.r ) $ 0,
alld ahose fo r which F(x ) of- 0. rn this secti on and Lhc next we consider equaLions for which
F(x)
0; we (liscuss the n10rc diffi<:ull c lnss. in which F(x) 0. in St:cti<H'I 15.9. Firsa lct
us name each of these classes of linear diffel'(:lllial equations.

DEFINITION 16 .6

A lincard ilfe re ouialeqtk"ion (.r , D)) = F (x) issaid co bc hontO~tneousif F (x ) "' 0 .


and nonh om o~cncous otherwise.
T he meaning of homogeneous to describe a property of linear diffcrcmial equa1i011S ill this
definition is totally c.Jifffi!.renl from 1he meanin,g in Exercise 34 or Section 15.2.
T he funcJanlCnhll idea behind Ihe so hilion of tlll lincnr difl"crcntiul equations is the following

theorem.

TH EOREM 1 6. 1

{ Superposition pri nciple)

If y 1(x) . y2 (x ) , . . . , y,. (.r) are solucions or '' homogeneous I incar difl'crcm inl cquatioll
1/>(X ' O )y

= 0,

on an interval 1, then so too is a ny linear c.o mbi mu ion of Lhcm.

C 1y 1 (x) + C2 y 2 (x) + .. . + C.,,y.,(x)


(fur :trbitrary eonswnll: C 1, C2 ,

I' ROOF

C,. ).

The proof rcqui res only lil>carity of the operacor > (x. 0). foo

r/> (.r, D)[C1y 1(x) + .. . + Cmy,.(x) J = C oi,P(.r. D)J 1) + + C,.[,P(x, D)). ,]

=0 +0 +

+0

= 0.

Solutions of a linear di fferential equation that are linearly combined to p1'0ducc OLhcr .MJIutions are said 10 be supeT1)0sed - c.o nsequeellly. the name supeiJJOSiticm princtfJle for l llcorem 15.1. Forcxample , icisscrdightfo rwardcoverify thaty 1 (x) = ~ and)'2(x)
~-J aresolutions oft he homogcneoucquacion y"- y ' - 12y
0 . T he superpo.o>itio n principle chen scaces
thai for 81l)' COnsoants I a nd c2. 1hc func cion y (x ) = c,y, (x) + C2)'2 (.r) =
+ Cle-l'
must also be a solution. It is a t\\O-pararnecer fam ily of solucions.
Simiblrly. superposition of the three sol u liOil~ r, xe,X. and ~\ 2 e-t of Ihe linear differentia l
equacion y"' - 3y" + 3y' - y = 0 g ives a chrcc-paramcccr fa mily of solucions y(.r) =

C,e' + C1xe'

c,e"

+ C3x1e '.

Now we begin to sec the importa nce of t he supe.rposilion principle. If we c an lind n


solucions y 1 (x ), n(x), . .. , Jn (x ) of a n " '" order homogeneous linear diffcrcm ial cquacion.
chen an llparamccc r fam i ly of solucions is

y(x ) = Co.)'c(X )

+ Czy z(X) +

+ c.y.. (x).

l.n other words, a ll thac we need do is tind 11 solu.cions; the s uperposition princ iple will do the resc.
There is a problem. however, if we take things. a lill ie too li1.e rally. For i nstance. y 1 (.x) = e4 x
a nd y 2(x) = !Oe4x are both solutions of y" - y' - 12y = 0 . By s uperposition, so. too,
then is y(x ) = C 1y 1 + C 2y 2 = C 0e4" + IOC 2e 4 ' . But is it a cwo-pam mecer family of
solutions'? T he ans wer is no, because we could wrice y(x) = ( C1 + IOC 2)e", and by seuing

= c, + J OC1. w ;

have )'(X}
C'leh . Supcrpu$ilion of the !>oluliVI.lS e.s.~ aJI.d 10e4 '
has 1101 lherefore Jed to a twcrparameler family of solutions. and the ret~!'ion is that lhey are
esseotiaUy de same solutiot: y2 (x ) as. y 1(.:r) uuJtipHed by a COilSiiult. StperposJtjoJl does 1\0t
therefore lead to a solution with two arbitrary constants.
In a .siruilar way, )'1(x} = e't , Y2(x) = xet:, and y ,(x) = 2ex - 3xel are all solutions
of y'" - 3l' + Jy' - y 0. and therefore so is y(.r) C,y, + C,)~ + C,y,. But because
we car'l wtite
c1

y(x)

+ C,xe' + C3 (2e' - 3.te')


= (C 1 + 2C1 )e' + (C2 - 3C 1 )x e'
= C4ey + C~xet.
c,e'

y(x) is oot a three-parameter farnjl)' of solutions: it is onl)1 a t\\'o-pammeter family. Th_is is a

direct result of chc fact that y ;(X) is a linear combination of che solutions y, (.~) and Yl (x) : it
is twice y, (x) minus three-limes Yz(.-t ).
Our problem seems to come down to this: If we have n soh.nions of an nl.h -order homogeneous linear differential equation. how can we de-termine whether superposition lead'i to a
solution lhat conta ins n arbitrury consmnts1 Our examples have suggested that if a ny one of the.
soluLions is a linear combination of the others. then an n . parameter family of solutions is not
obtained, and this is indeed uue. If one of the solutions is a li near <:ombination of the others,
we say tha( 1J1e, solutions arc linea.-Jy depe.ndent; if no solution is a linear combination of the
orhers. we say [hill the n sotmions are linearly i_n dependcnt. \Ve summarize these rcsuhs i rl

the fol lowing theorem.

THEOREM 15. 2
JJ y1( x ), Y1(x), .. . , y11 (.x) are n linearly irHJepenr.lellt solutioos of a.o nm-<Jrdet homogeneous linear diJl'ercmial equation on an interval I, chen y (x) = C 1y 1(x) + C2 y2 (x) +
+ Cny# (.-c) is an n .pararneter famil y of solutions of the differential e,quation on /.

What we should now do is devise a tcsl to delermine \Vhcth<:r a set or n solutions is linearly
inr.Jeperldem or lineal'ly deperlderll. For most example.,, nu 1e~1 is r-eally rlecessary; it is obvious
whether one of the solutjons can be wrinen as a linear combination of the others. For those
r-are occasions when it is not obvious. Exercise I 0 describes a test that can be used to determine
whether furlctions arc linearly independcm.
\Ve poiuted out in Se<.:Lion 15.1 thai an n -paramctcr family of solutions fur an ttlh_ordcr
tJifTercntial equation might not coutain all ~oluti ons of the equation, and might not therefore be a
general solution o f the di ITere.ntial equmi.o ll. This is not the c11se for linear n tb ~rder differentitrl
equations. T1 can be shown thm if y(x) is. cu1n -pmameter ft1mily or solutions for a lirlear ,m_
onler differeutial equation on an inten'al I . then every solution of the differential equation on
I can be obtained by specifying panicula r values for che arbitrary constants in y(.x) . Here then
is a very lmportanl class o f di fferential equations for which an n-parameter family o f solutions

is always a general solution. let us stale this as a corollary to Theorem 15.2.

COROLLARY 15.2 .1

If y 1(x ). )'2{X) , . ... y.(x ) arc 11 linearly independent solutions of an n'"-ordcr homogeneous linear differemial equaLion on an imerval / . then y(x) = C 1y 1(x) + C1 }'2(X) +
... + c.y.(.r) is a general solution of the differential equation on/.
In s ummary. the superposition principle s tates lhat solutions of a homogeneous linear differ-

ential equation cao be superposed to produce othe r solu1ioos. U '' lioearly independent solutio ns
of an nlho rder equation are supe rposed. a generd.l solut i.o n is obtained. T hjs is the imponance
of the s uperposition principle. We need nol devise a method that takes us directly to a gen
eral solution; we need a method fo r finding n linearly independent solutions - s uperposition

1086

Ch.aplt:l' 15

Diffe:.tmial EquatiOt)S

does the rest. Unfortunately, for completely general coefficients a; (x) in r/l(x, D) (see equation 15.4 1), it is impossible Lo give a method that always yields JJ linearly independent solutions

of a homogeneous equation. There is. however, one special case of great practical irnponance
io whkh it is always possib l~ to produce n linearly independent solutions in a vel')' simple way.
Thjs special case occurs when the coeffk ients a;(x) are all constants a;. and this is the subject
of Section 15.8.

I EXAMPLE

15. 11

lf y 1 (x) = cos 3x and y 2 (x)


find a gener.1l solution.

= sin 3x are solutioos of the differemial equation y" + 9y = 0.

SOLUTION Since y 1(x) :aod y 2 (x) are linear ly indepeodem solutions (one is nola constant
times the other), a general solution can be obtained by superposition:
y(x)

I EXAMPLE

C 1 cos3x

+ C 2 sio 3x .

15. 12

Given lhat y 1(x) = e2' cosx and y2 (x) = e2 ' sinx are so lutions of the homogeneous linear
differemialeq uatioo y" - 4y' + Sy = O,lind that solution that satis~es the conditions y (;r / 4) =
I, y(;r/ 3) = 2.
SOLUT ION

By supeoposi Lion. a general solution of the differemial equation is


y (x) = C,ezxcos x

To satisfy the conditions y(n / 4)

+ C 2e2'sinx

= e2'(C 1 cos x

+ C 2 sin x).

= I and y (;r / 3) = 2, we have

Thus C 1 and C 2 are de tined by the pair of equations

lhe solution of which is

c2 =
The required
and Cz .

4e - 2;rf3 _ ,J'ie-12

~- I

c, = h e-"' 2 - c2.

S<>~ion is lhcrcforc y (x) = e2x (C1 cosx + C2sin x ) with lhcsc values for C 1

..-._
EXERCISES 15.7

In Exc-ciscs 1-8 show 1h.a11hc fncrions arcsolurionsofLhcdiiTccnlial


cqua.Lion. Check 1ha1 lhc dincrcnlial cqu:~ l io n is linear :lnd hofnogcnc(MIS. and lhcn fi nd a general solul ion.

4. 2y"- 16y' + 32y = O; y1 (x) = 3..-x. y 2(x) = -2<e4 '


5. y"' - 3y" + 2y' = 0: J o(X) = 10, J2(X) = 3e', JJ(.t) = 4e'-'
6. 2y" - Sy'
e2' sin (x/ .fi)

2.

.r' + y u-m x =

0: y,(x) = cosx

3. y'"' +Sy" +4y = 0: y1(x) =cos 2x. y2(x) =sin 2x, ,l-,(x) =
cosx . y,.(x) = sinx

+ 9y

7. x 2 y'' + xy' + (x 2
(cos x)/ ../X

= 0: y,(x) = " cos(xj ./i). y,(x) =

l/ 4)y = 0: y 1(x) = (si nx)/

,;X, y2(x) =

8. x 2y'' +xy' +4y = 0: y ,(x) = cos (2 Jn x). y,(x) = sin (2 I" x)

1 5 .~

9. Show tbal y 1(x)

)'l(x) + y,(x) a :solution? Expi.Un.

l'

= yx' . ls y{.x)

,-'

. I

IV(y,, ... , y.)

10. We ~lalcd in thi~ ~lion tha~ r: fuO<;tions y 1{.r), . . . , )',;(x) arc


tinc:arly dependent if tlllcnst one of the fuoctjons can be c..,;prc5scd us u
linear oombtm:.tion of the others; they ilfC lincady i.ndcpcnOcnt if none

Sl\0\~o

C,y 1(x)

.r"
I

th.at if Yl : ... ~ y, arc linca.rly dcpcnde1u on I , tJ1ct1

\V (y 1, ... , y,J) = 0 on I . II (()llows lhc1t lh:n if 1herc cxi!US itt least


oncpoi_min I a( which W(y1 1 yfJ) O,thc(m\Ctionsy 1, y11

of the funcliotb i$ a linear combin.at.ion o ( the oti\-C.t'S.. AnOLhcr wa)'


of saying this i~ U.:i follO'ws: Function~ Jl (x), . . . , .\'n(X) urc. lincurly

lkpcndcnt on an imcrval I if thc.rc exist consmnts C1,


.te:l\), ~uch ll\al on I ,

1087

Yt

= -2/(x + I) and y,(x) = -2/(x + 2) arc

both solution~ of dte dilfcrcllliul cquution

;~o.

l lomQiCrtCO'b UnCJ;r l)lrfCftflli31 Eqtt:tiM)l:S with CQOSt:un Cucrlk:lcnb

:~:rc

C,.. not WI

linearly independent on / .

In EAcrdscs 11-15 uscthenlethod\lfExercise IOtodc.tcnnincwhclhcr


the funaions are lineariy dependent or i ndcpcnde.nt on lhe inl1"\'<tl.

+ + C,.y,.(x) = 0.

11. {l, .r . x 2) on-()<)< .r < 00

If this equation can be satisQcd only wilh C1

= c, = = C. = 0.

12. {.r . 2x- 3.r' .>' l ()11-00 < x < oo

tbe functions are littearly independent. In this c.\ercise we gh!e a teSt to


determine \\'hClher functions arc linearly i ndcpcndcnl or dcpcmJcnL If
.\'t (X) ... . y.. (x) ha\'C dcriv-.-nivcs up 10 and including order" - I on
the inter\'al I . we t~llnc the WI'OI\Skian or lhe fnncai on~ a~ the 11 x n

1~.

(sinx. cosA'l on 0

~ ,\',!:

2:r

l 4. (x.xe".x 2c'}cm0~.\' ::S I

l5. {A' M:n ,\'. e1"l on -co< x < oc

de1erminan1:

115.8 Homogeneous Linear Differential Equations with


Constant Coefficients
\ Vc now consider homogc.ncous linear d ifferential cqual ions

where the coefficients ll(h a 1,

. , ll11

are all constants. ln operator notulion we wrlte


(D)y

= 0.

where

</>(D) = a0 D"

+ a, D"- 1 + + a._, D +a,..

( 15.45b)

The superposition principle slates lhatagenernl solution of equation 15.44 i~ y(x) = C 1y 1(.r)+
+ C.,y,.(x) , provided that y 1(x), ... , y,(x) arc an)' 11 lincorly indCJ>Cndent solutions of
the tuation. Our J)I"Qblcnl. then. i~ tO ~evise a 1cchniquc for findin~ 11 linearly indcpendenl
solutions; to illustrate a possible proce<.lurc, we tina consider three second-order equations. "The
first h;
y" + 2y' - 3y
0.

It is not unreasonable to expect .hac for some \"alue of m. 1he func1 ion y(x) = f ""x mighl
be a solution of this equation. After all. the equation says that ~1c second derivative of the
function must be equal to rhree limes the function mirHJ$ 1wice i t fi r~t derivative. Sinte 1he

exponential function reproduces ltsc-lf when differentiated, pcrh6pS m clln be chosen to produce
1his combination. To see w1teaher this is tXlSSibte. we substillHe y = e"'~ imo the differemi al
equation, and find that if y = e"" is to be a solution. then

this imp Iies that


0 = m2

+ 2m

- 3 = (m

+ 3)(m

- 1}.

= ~-3.- are
soluuons of the dffertnUlll oquauon). Smce 1hcy are hneariy indcpendet11, a ~;cntral solm1on "
y(x): C 1e' + C:e-3<.

Thus.Y = ~~~.~ iC\.a ~lut~on if m .is cho5en as either I nr -3 (i .e .. y 1 = e-' and ) l


For our second example. we lake

) " + 2y' + y = 0.
Since cwoncmials worl<cd in1he first c<olmplc. we once a.gain try a solution oflhe form y(x)
e'.c. If we substitute imo the differential equatiun. we obtain

m1 l111"'

+ 2uu.,'' + en = ot

=m

+ 2m + 1

which implies lluot

= (m + 1)

Thu< y, = e- is u solution. but unfonuna1el) it is lhe only soluliocl lh:tl we olxin as a re'ult
of our guess. We need a l<'Ond hnearl) independent soluuon n (x) in on!er to ol>min a general
>Oiution. Oea ly. no other cxponenl i31 will "or~ . Pcrlutp< il' we muli1plicd ,-' hy another
runction, we might lit1d a s~;-.:uud solution; inother wonl!", pe:rhtlp:o. lhc::rc i~ a. solutioo ur the fvrm
y(x) = v(x)r- ' for some v(.\). To sec. we asain substitute imo lh< lliiTet<:nual cqwuion.

= (v"e- - 2v'r- + uf-' } + 2/u'e- - ur-l + ve- = v"c- .


Con.:qUCI\tly, v" = 0, and 1his in1plics that v(x) = A r + 8 , for any con<tanl'l A "'' d 8 .
0

In p;ltlicular, ,r A
I and 8 = 0. t (.<) = ~ , and ) 2 (.<)
v(. )f-'
.,~- i; aiMJ
:.olutiun or the difrcrco1tial equation. By >liPC'JJOSition, then. we fin<lthm a gcncrlll solutiM is
y(.r) = C,e -'- C1.u A = (C 1 + C2.c )e-. Note l~otifwehdd><t A= Oand 8 =I.
thct1 u(.t )
1. ond 1ht S<~lution ,"(-') v(x)e "ould hav~ ~~ )'I (.t). Further. tf we had
sin1ply set y(x )
v(xk- ' = (A.1 + R)e '.we would ha"e the general solution.
Our thi rd e~ample is

)'" - 2y'

IO,v

= 0.

As in the preced1ng two cxanlple!\, ir we assurnc l' solu1ion )'

m1e'..,

+ 2ml"..- + IOrt'J""

= m l + 2m + 10.

or

= tr , then

= 0

The ""lutions of d1is quadratic oqu.-11ion are 1hc complex numbers

m=

- 2 ,/4-40
2

I 3i.

(A brief introduction to complex numbers c!Ul be fou nd in Appendix C. l Because 111 is complex,
110 n:al exponential y = r'' sati<fK'> the diffcn:ntiol equation. If. hOwever. we for~ complex

exponentials y,(x) = ~(- 1+ '11< and Yz(.<)

= <'t-l-liu, nnd superpose these soltl(oos,th~:~l

t-l+'lil
y ( x) = A e

+ 8 ", ,-1-Ji ) <.

and this must also be 8 solution. When we ""' Euler's idcnlity for complex exponemials.
e''' = cos8 + i sin8. we Clln write y(x) in the for"'

v(x) =

Ae-e)" + Be-e-''1

= Ae- '(cos3x
= e-' l(A

+ i s in 3x) + 8 e- "(cos3x -

+ 8)cos3x + i(A

- 8)sin3xl

isin 3x)

15-.S: fl(}cnoselleull$ L1near l>if'fercnLiaJ l~uut Juns with Cor~tv'l CueJllcie:nts

1089

where C 1 = A + B and Cz = i(A - B) . ln()(hcr words, the fum:tion y(x ) = e- ( C 1 cos J.r +
C2 si n J.x) is a gctlctal solution of lhe tlit'(ereltiol equ~Hion . uncJ it hus l,x.--en derived from the
complex rooiS m = -I 3i of the cqumion m 2 +2m + I 0 = 0. Note that what multiplies
x in the cxpone,ntiul i~ tht! real pan of the.;,;e complex numbers, and whul mul tiplies }C in the
trigonometric func tions is the imugimuy ptlrt. In Exercise 17 we show that this solution can nlso
bederive<.J without complex numbers. but we feel thm in general the use of <.omplex numbers is
the lx:st method.

In each of these exnmplcs we guessed y = e"'" as a po..<sible solution. We then suhstiuued


into the diiTcrential equation to obtai n an algebr~i c equation form . lrt each ca.sc the equation
form was

,P(m) = 0;
that is, Iitke the operator tJ> ( D ), replace 0 by m , and set the polynomial equal to >.ei'O. Titis is
nO( a peculiarity of these e-~amples for it is str.aightforwnrd co show 1ha1for any homogeneous
linear equation 15.45, if we assume a sol ution of the t'ot"m y = e"'r , then m must &11isfy rhe
equation f/l(m.) = 0. \Ve ncune 1hi"' e<Jtmti<m in 1he f(,Jiowing definition.

DEFINITION 16. 7

With every linear differential equation tlull has constant coeflicients </>(D)y = F(x), we
associate an equation

</>(111') = 0

(15A6l

called the auxiliary equation .


To surnmarizc~ in each of the examples '"e assumed a solution y =
m had to satisfy the auxiliary equation <f>(m)

emx ;,uld found that

= 0 . From the roQls of the auxiliary equation

we obtained solutions of the differential equation. t\nd superposition then led to a general soJutiOtl. Tilis procedure works CNl every homog_.eneous linear differential equation with constant
coeflkie,us. Rut if the procedure is the same in every case ~ surely we ca' set dow' nles th(lt

eliminate the necessity of tediously ref'Cj\ting these stepS in every example. This we do in the
following theorem.

THEOREM 15.3

lf if>(m) = 0 is the auxiliruy equation a>SOCiated with the homogeneous linear differential
equation ,P(D)y = 0. then there are two ~ibilities:
(i) 4>(m) 0 has a real root II/ or multiJ>Iicity ~ . The~ a >olutillfl of the di fferential
equation is

( 15.478)
(ii) 4>(m) = 0 has a pair of complc" conjugate roots a
k . Then a :.ol utio ~ of the diiTcrential equation is

e"'L(C I + Czx + ... -

bi each of multiplicity

c.,.~- ) cosbx
1

+ (D, + 0 2x + + Dtxk- ')sin/u].

(I~ ..Hh)

A geneml solution of the differential equation is oblllined by superposing all


solutions in (i) and (ii).

For a proof of this theorem see Exercise 26. Let us now apply the theorem to our previous
examples. The auxiliary equation for y" + 2/ - 3y = 0 is
0 = m2

+ 2m

- J. = (m

+ 3)(m

- I)

1090

Otapter 15 Oilfertnlial Eqn:wiOtK


with solutions m = I and m = -3 . I( we now use pa11 (i) of Theorem 15.3 with two real
roots. each of multiplicity I , a gcncrul solution of the diJferent ial equation is

The auxiliary equation for y" + 2)1'

+y=

0 is

0 = m 2 + 2m + I = (m + 1} 2
with solutiOolS m = - I and m = - I. Pan (i) o f Theorem 15.3 w ith a single real r001 of
multiplicity 2 give$ the gener.tl ~oluti on

Theauxiliar)' equation for .v" + 2y' +lOy= 0 is

m +2m + 10

0=

with solutions 111 = - I 3i. Paot (i i) of T heorem 15.3 with a pair of complc conjugate rOOIS,
each of mult iplicity 1. gi,e~ the general solution

y(.<) = e- '(C1 cos3x

I EXAMPLE

1 5. 1 3

Fi nd a general solution for y 111


SOLUTION

y = 0.

The auxiliary equation is

m3

with solutions m = l and m = -(1/ 2)


e<1uation is. therefore.

I EXAMPLE

+ C2 sin3x) .

(m -

l)(m 2

+m+

I)

(../3/2)i. A general solution of the differential

1 5 .14

\\'hen temper.uure T(x ) in a very long, lhin wire wilh one end at the origin is analy2ed, the
following diffcrcmial equation is wmetimcs encountere-d:

where h > 0 is a constant. Find T(x) if it omust also satisfl' the conditions 7(0)
limx~oc T = 0.
SOLLJilON

The ouxilimy equation is m 1

Since lim,, ... 00 T = 0. we must set C 1


T(x) = T0 e - ./hx .

..-...

h = 0 with solutions m

0. i.n which case To

T0 and

=./h. Then,

T (O)

= C1 .

Thus.

1$.8

l iQIUQ~oenc"u.s (...il_
l ea.t Differential

Equaci<.m$ wilb Cooswn Ct~cJticicots

1091

I EXAMPLE 15.15
Tf1he roots of1he auxiliary equa1ion ,P(m)

= 0 are

../5.

3, 3. 3. 2i. -2. I

-4 i, - 4 i,

lind a geneml sohnion of 1he diiTe.remi al eqm1tion ( D)r


SOLUTIO!~:

= 0.

A general solu1ion is

y(x) = (C 1 + C2x

+ C3x 2)e1 ' + C, cos2.t + Cssin2x + C0e- 2.'


+ C7 e11+JJl + C8 e1 1- JJJ. + .,- l\[(C9 + C 1o-r)cosx
+ (C11 + C 12x)sin.r] .

EXERCISES 15.8

In Excn;ises 1- 12 find a general soluion ror the hon'IOgcneo~dirTc.r

boundar)' condilion:.. Only ror ccrllliJl V"~ll ucs of A, called eig-envalues,


do solutious cxi.st. und corn::.sponding solutions arc called cigcufunc
tions. lo ExcrciS<:s 19-23! find eigenvalues and cigcnfUJlcLions 10-r Lhe
Stunn~ Lioo,ille system.

cnlial equation.
I. y

+ y' -

3. 2/' + l6y'

5.

)'# -

2. 2y'' - 16y ' + 32y

6y = 0

+ 82,1' =

4y' + 5y

7. y'"' + 2y"

4. )' 11 + 2y' - 2y

=0

c:

=0
0

6. y111 - 3y'' + .r ' - 3y = 0

s. )'

+ .\' = 0

6)"1' + 12y' - 8,r

1
"

=0
.. 20.

111

9. 3y - lly'' + l8y' - 12y = O


tO. y"" + 5)

11

II. y"'- 3y"


.f<

12.

'

+ 4y =0

+ 2y' = 0

y"" + 16)' = 0
hC'IIllO@Crtcam:

linear dift'e:renll:ll equation

that hm: the func1ion as general solmion.

13. y(.<)

= C,t' + (C, + C,.r)e-"

= r''(C, cos4.r + c, sin 4.r)

15. .>(x) =

*
*

= e- (C, cos3x + C, .in3x).

18. lhcequaLion

y''" +a.\'"+ by' + cy

conslants,h;,ssolutiony(x) =

C,,.-x+e

a,.
- + ;.y = 0. 0 < .r < 4: J' (O) =0 = y' (4)

dxl

J ' ,

.. +

J'y
d .< 2

),y

=0. 0 < x < 2: y(O) =0 =.1' (2)

+ A)' - 0, 0 <

d'y

dy

.. 23. dx' - ilx

< 5: y ' (O)

+ ),y = 0. 0 < x

y(S)

< 1: y(O) - 0- y ( l)

damping. (b) FiRd 1hc palh if lhe force is nway from lhe origin in.tc-.ul

+ C, x) cos J2x + e'(C, + C,x)sin /'ix

17. Show lhal if we a~;;umc thai y(x)


,PA sin h.r i~ a solulion of
the ditTerential equation y" + 2y~ + lOy = 0, then a and b must
be ~qu.t1.l 10 - I and 3. rc spliv~ly. Verify th:.lt. lOr this a :1nd b,
y(x) o tf1X c.:osb.f is also o. solution, und lhc::rcrorc a gcl"t('ml solution

ili y(.<)

J' y

-dx' + ;.y = 0 . 0 < x < 3: y(O) = 0 = y(3)

24. A mas.s .i 'f moving in the .ty-planc is subjc.::tcd to a fon.;c always


directed tQ\.,:.rd 1hc origin with magnitude proportionul to iLS di~ tuncc
(tom the: Of"ig_in. At a ccnuin instant. the ma.).S pi.!S.SC:.) through the point
(Xn. 0) with speed vir\ the positi~c: y dircccion. (a) find the path of the
ma~ . ~me no fric1ion bct"'ccn the rn~ and the X)'planc. and no

c, + c,,.r" + c,.-.fi

16. y(x) = e'(C,

2 1.

22.

In Exercise;; 13-16 find a

14. y(x)

.. 19.

=0. where

Of f0W31\1S it

*,.

25. Show that if p isconscam and f (x ) isdifrcf'tntiab!c. th;n

D(e'" f(.r ) }
Now~ O'lia:then'I!UtC'!ll

= e'' (( 0 + p)f(x)}.

induction to prov~ th:;u i f f (:c) i~ k limes dif-

fcrcntio.ble, then
t1 .

b, and c arc

Zx(C2sin4X+CJcos 4.r).

Find a. b, ond c.

o re 'f<x >l =

''I<D+ p)' f

<x) ).

f inall)', \~rify lh:ll

= e 1'' \</>(D + p )f(x)} ,

Srunr-liouviUt:: SySicrns p hty a promi ncnl role in the Slu dy of paT1itJI

</>(D)\e1'' f(x)J

diffcrtnlinl cqumions. 11)t SyStem COnSiSIS of a homogeneous, Linear.


sc..-cond-order diffcrencinl equal iOn C(ml!tinjng n pan.t(nctcr l n.nd two

a result CfJUcd the op erator sltift theorem.

A-"'

26.

(a) If mn i:; a rcul rool o f Jnultiplicity

lion ,P(m)

k for I he aultiliary cqua

whc..c \fi(D) i:; a polynomial in D. N O\\' WiC the. o tx:nt


1or !:.llifltheorem of Exercise 25 1.0 verify I hat 15.47b is a

= 0, show lhatlhe operdl lll' <fi(D) can be e.<-

presscd in the fonn

solulioo or ql(D).v = 0.

if!{D) = (0- mo)' I~ ( O).


where t/f( D ) is a polynomial itt D . Now use the opctator
shift thcotcm of Exercise 2.5 10 vcrify that ( C 1 C 1X
... + C1x1 -l)R""O is a solution of ,P(D)y = 0.

++ 27. lf M . fJ. and k arc. all positive consta..\L~. find a gt 1leral solulion
for Lhe linear dif terenlial equation

(b) If a bi are <.:Oillplex (;Onjugatc roots each of mullipl idly


k li:lr lhcaw<il iaryequ>Lion(m)
(), showlhmql(D)
can be CAtJrcssed in the f(mn

d 2x

t/>(D)

= (D -

M-

(,I
12

a - bi)'(D - a+ bi/1/J(D),

Oiscu~

dx

+ P+ kx
dt

= 0.

all possibiiWes.

l ts.9 Nonhomogeneous Linear Differential Equations with


Constant Coefficients
The general nonhomogeneous linear differential equation with constant coefficients is

(/I(D)y = F(x).

(15.48a)

where
<b(D)

= aoD" + a, on-l + + t - D + a .

(I H

8h)

It is natural to ask wheche.r we can usc the resuhs of Sce1ion l 5.8 concemlng homoge...
neo us equatio ns wilh constant coefficients to solve no nhomogeneo us problems. Fortunately,
the answer is ye-s. a'S shown by the fo1low ing definition and theorem.

DEFINITION 15.8

With every nonhomogeneous lincardiffcrcn tial equation with constant coefficients

(D) y

F(x ),

we a:ssoc.iatc a homogeneous equation

(D)y = 0,
called the homogeneOU$ (reduced. or c:omplementaty) equation
~)(D) y = F (x) .

( 15.49)
~wciated

with

\ Ve now prove. the following rhcorem.

THEOREM 15. 4

A general solution of thelineardiiTerential equation 1/i(D)y = F(x) is y (x) = y 11 (x ) +


y1,(x), where y11 (x) is a gener;tl solution of the associated homogeneous equation. and
yp(x ) is any particular solution of the give<~ equation.

15.9 No,\hQrnogeneous U tte<~r Difl"erentioJ Equ.:u io~>S witb Constaor Coel&iems

1093

!'ROOF Since </>( D) is a linear operator,

</>(D)(y11
so that Yh

+ y1,

+ Yp) = </J(D) y11 + if>(D)y1, = 0 + F(x) = F(x),

is indeed a solution of the given differemial equation. Because )'r.(x) is a

generdl solu tion of the associated homogeneous equation. it con1ains tlle requisite number of
arbitrary conS!ant'i for y(x) to be an ll )>arameter family of sohnions of( D)y = F(x) . What
remains is to show that. any (and every) solution y 1(x) of <P(D) y = F (x ) can be expressed in
the form Yh (x) Yp(X) for some choice of the constants in Yh (x) . Consider the function y(x)
defined as the difference between y 1(x) and )'p(x ), that is, y(x) = y1(x) - )'p(x). Because
of linearity,

<P(D)y

= (D)(Yt -

)11,)

= (D) Yt -

r/>(D)yp

= F(x)- F'(x) =0.

But every solution of the homogeneous equation cp(D)y = 0 can be expressed iJl the form
)'h(X) for some choice of the constams; that is, y (x ) = y 1(x) - )'p(x) can be expressed in
the form
YJ(X) - Yp (x) = y;,(x)

for some choice of constants in Yh (x). This completes the proof that y 1(x) can be expressed
in the form )'J1 (x) + )'p(x), and therefore )'It (x) + y1,(x) is a generdl solution of 15.48a.
We note in passing that Theorem 15.4 is also valid for linear differential equations with
variable coefficients.
Theorem 15.4 indicates thm discussions of nonhomogeneous differential equations can
be divided into two parts. First, find a gen~.ral solution Yh (x ) of the associated homogeneous
equationl5.49, and this can be done usittg the results of Section 15.8. To th.is, add any particular
solution )'p(X) of 15.48. We present two methods for finding a particular solution: (I) the
method of undetennined coefficients, and (2) the method of operators. Both med10ds apply in
general only to differential equations in which F (x) is a power (x". 11 a nonnegative integer),
an exponential (ePX), a sine (sin px), n cosine (cos px), and/or any sums or products thereof.

Method of Undetermined Coefficients for a Particular


Solution
The method of undetermined coefficients is ro be used only when F (x) in equmion 15.48 is
of the fom1 x", ePx. sin px , cos px. and/or sums or products thereof. For example, if

the method essentially says that Ae'" is the simplest function that could conceivably yield e"
when substituted into the left :side of the tliffcrcnt.ial cq uation. Co nsequently, it is natural to

assume that )'p = A e''x and attempt ro determine the unlcnown coefficient A . SubsJiturion of
1his f unclion into the differential cqumion gives

Ji we divide by eb. , t hen

14A = I and

A = -/.

A particular solution is the-refore y1, = e4'/J4.


Before stating a general rule. we illustrate a few more possibilities in the following example.

I EXAMPLE 1 5 . 16
Fn1d a p.lriiCular sotuto' of) "+ )'1
(.r 2

ca) F (x) =

+ 2x + 3

6y = F ( r ) in coch "-a'e

F (x)

(b)

= 2 sin 2x

F(.t) = u - - . -

(c)

.SOl I '110'
(u) Si nee term!<> in .r 2 , ~". ;md (:Oil SUUll" yield tern~ in .r 1. ,\. and c.::ons;Utnt~ wt~\ 5Ub
$tituttd i1uo the tcft l'ide of the dift'CI'CI\liill equation. \\c M-tempc to firul a J>Orticular
solution of the form )p = A r1 + B.x ~c. Sutwituti~n into thcdi1Tcnntial cquati~n
gw~

(2.4)

or

+ (2Ax + 8 ) -

= 6t 2 + 2.r + 3

6{A.<2 + B.<+ C)

+ n-

(-6A)x 2 + (2A - (o fl) x + (2A

=6.t

6C)

+ 2x

+ 3.

But this equation ron hold for all "";~luu of x on_ly if coefliciellb of corre.spoodi11g
powers of X "'e itlcnucal c~cc ExerciS;! 3S in So.1ion 3.8). Equatingcoefticienb then
g i \'CS

- 6A = 6.

= 2.

2A- 68

2A

TIICse i mJ>Iy tlllll A "" -I. II = - 2/3. C


v

.'

=-

+ 8- 6C

= 3.

17/ 18. an~

..2
2x
17
-:t - - - -

IS

(b) Since temlh in s.in 1r ~1.nd <.'U~ 2.t yield tenm in ~in2.r when ~ub~tilutcJ into the
left side !11<: differential equation. \\C assume tha t 1, = A sin 2.r + 8 cos 2.< .
SubM.itution imo chc: diff(.'f"Cillial cqmuton givc._i

or

(-4Asin2r- 48cos2.r)

+ (2A cos2<

- 2Bsin2r)

-6(A.Jn2<+ Rcos2.r) = 2si n2x


( - lOA - 28) sin 2.r

+ (2A

- l OB) coo2r

2sin 2.

Equating C:CCihCielliS or sin 2x and CO< 2.r gi\~


- lOA - 28 .. 2,

n ...,.. impl) tlw A = - S/26. 8 = )'p

(c)

211 - lOB = 0.
1/ 26. and hence

. ,_
I
--(SSIII<.< +cus 2.r).

26
Si nce 1crms in xe-s liiKf e- y1eld 1enn~ in .xe-t nnO e-s when subs:lill.ltcd inro
the len si~ uf the diiTcremial eqwtio n. we a<Suntc." th,tt y, = A.te 1 + s~-.
Sub.. tllulion 1n1o the d1fferent1al cqlldlion gives

(Axe- - 2Ae- -"- Be-' ) - ( - Au- + Ae- - Be-' )


- 6(Axe

+ Be ')

= .Vt' -z -

e-

or

= xe- - ,.-.
Equating coelllciems or e- and .re-' yields
(-6A).r<'- - (-A- 68 )<'

-6A =I .

-A - 68

= -1.

These imply that A = -1/6. 8 = 7/36. and hence

>'t =

- ( l /6)xe--'

+ {7/36)e-x_

The following rule encompa81ics each P<lfl of 1his example.


RUL E 1

If a term of F (x ) consists of a power (x " ), an exponential (e'") , a sine (sin px). a


coo;ine (cos px) , or any product thereof, assume as a part of Yv a constant multiplied by
that term plus a constant multiplied by all)' linearly independent function arising from it
by diffcre.ntiation.

For Example IS. l6(a). since F(x ) conl!tins the tcnn 6x 1 we ,~...~un>e that .1'1, contains A x1 .
Differentiation of Ax 2 yields a tcnn in x and a '-onstalll so that we fomt Yv
Ax 1 + Bx +C.
No new terms for )'p arc obtained from the 10.rms 2x and 3 in F(x).
For Example 15.16(b). we tlSSumc thm Yp .contains A sin 2x to accoum fort he tenn 2 sin 2x
in F(x) . Di fferentiation of A sin 2..1 gives a linearly independent term in cos 2x so that we
fonn Yp = A ~in 2x + 8 coo; 2x .
For ExnmJ)Ie 15.16(c), since F(x) conta ins the tenn .u-, we assume that )>11 rontaill~

J\xe- ). . Difl'ercntiationof Axe-.x yields: a cen11in e -~. so that we fOC'm Yp = Axe-' + Be-".

No llew terms for )'p arc obtained from the LC.rm - e-x in

I EXAMPLE

F(~v) .

15.17

\ Vhat is the fonn of the particular solmion predicted by rule 1 for 1he differential equation

y" + 15)'' - 6y = x 2 e4"

+ x + x cos.r ?

SOLL"TTON Rule I suggests th:ll

y1, = Ax 2e~1 + Bxe~' + Ci ' + b .r + E + F'xcos x

+ Gxsin..r

+ 11 cosx ~ 1 sinx .

Unfonunotcly. exceptions to rule I do occur. For the dill'eremial equation) "+ y = co, x. rule
I would prediCt YP = A co~ .t + 8 sin x . Tf we substitute this into the differential equation we
obt inthc absurd identity 0 =cos.<. andccrtai nly noe<Juutions to solve for A und /) . This result
<-ould have been predicted had we first o alculaaed YA (x) . n ae uuxilial)' e<Juation m 2 + I = 0
Ita. :;oh11ions 111 = i. so that Jh (X) = C, cosx + C2sin x. Since y 1, as >u~gcstcd by rule
I is precisely YA with dift'e rcnt numcs for the constnnts. then certainly )'; + y1, = 0. Suppose
that as nn nhcmmivc we multiply this Yp by ;r. and assume th;.lt Yp = Ax cosx + Bx sin x .
Substitution into the tlitTcrcncial cqumion uuw give.-:

-2A sin x + 2 8 cosx = cos..r .

ldenti fic.uion of coellicients requires A = 0, 8 = I /2 . and Yp = (1/2)x sin x .


This example suggests that if Yr predicted by rule I is already contained in )'A . then a
mod ificatioll of Yp is necessary. A precise statement of the situation is given in the following
nale.
RULE 2

Suppose that a tenn in F(x ) is of the form x" f(x ) (11 a nonnegative integer). Suppose
further that f(x ) can be obtained from Yh(x) by specifying values for the arbitrary
constants. If this term in Yh results from a root of the auxiliary equation of multjpJicity

k . then corresponding to x" .f(x). assume as a pan of y11 the term A x k(x"f(x )) =
Ax"+k f(x)~ plus a constant multiplied by any linearly independent function arising
from it by differentiation. Do not include any terms that are aJre.ady in .Y!t

To use t11is rule we first require Jh (x). Then. and only then. can we decide on the fom1 of
)'p(x). As an illustration, consider the following example.

I EXAMPL E

15.18

Find 3 general solucion for yl'' - y =

x 3e" .

SOI.l ,01\' In Example 15.13 we solved the auxiliary equation 10 ob1ai11 111
- 1/ 2 (../3j2)i . from which we formed
)'h(.t )

I and m

= C,r + , -.12 [C2cos(J3xJ2) + Clsill(J3xJ2)].

Now X .,. is X. limes e-'. and e> can he obtained from J by specifying c, = I anu c2=
0. Sine! 1his 1enn resuhs fro m the roo1 m = I of muhiplici1y I, we assume 1ha1 y1,
contains Ax(,\' -'e.r) = A x~ ex. Differentiation of this funclion ghcs tcnns in x 3 e..r. x 1e.t . xeX ..

c, =

and I!' . We lhN'Cfore lake

(\Vc do not include a term in eA since it is already in Jh.) Substitution into the differential
equation and simplifica1ion gi\e.-,

(12A)x 31!'

+ (36A + 91J)x 21!' + (24A + 188 + 6C)x l!'


+ (68 + 6C + 3D)e'

= x 3e'.

Equating coefficientS gi\'es


12A

I,

36A

+ 98 =

0,

24A

188

+ 6C

= 0,

68

+ 6C + 30

= 0.

Theseimply 1ha1 A = 1/ 12 , 8 = - I/3.C = 2/ 3.0 = - 2/3 . and

A general solution of !he difTcremi al cqua1 ion is 1hcrcforc

,\' ..r

.t 3 I''"

2.r 2(''-'

2x {1-"

+ - 12 - -3 + -3- - -.
3
I EXAMPLE

15. 19

.._..
If 1he rooL of the auxiliary equa1io n <fl(m) = 0 for 1he diflerelllial equalion t/I(D )y = x1 2 sin x +xe-~' arc i, - 2, - 2, - 2, 4, and 4, what is 1hefonn of )'p prcdic1cd by 1hc met bod
o f unde1em1incd coefficieniS?

SOLL,01\ From !he roots of !he auxiliary equation, we can fonn

Corresponding 10 thelcnn x 2 in F(x) , rule I requires !hat y1, conlain Ax 2 + 8x +C. Because
- 2sinx can be Oblained from y,.(x) by specifying
= - 2,
=
=
= C; =
C6 = C, = 0. and this term resuhs from the roots m = i, each of mull iplici1y I. rule 2
suggests 1ha1 y1, coo1ain Dx sin x + Ex cosx . (We do not include terms i.n sin x and cos x
since they arc already in y".) Finally,xe-2.' is x Limes e- 2x, and this function can be obtained
from y,.(x ) by sening C3 = I. and C , = C2 = C., = C; = C6 = C, = 0. Because !his

c2

c,

c3 c.

tcnn rcsulls rrQm the root m = -2 of multiplic ity 3. )'p must conni ., F .x e-'l.t + G x' e -J.,~
( but nOt terms in -'' 2e --u . xe-'b. und e - 2 sitcc they are in )'h )- TI1e tollliJ)articular solution

is therefore

Operator Method for a Particular Solution


'fhe oper~t.or meth od . like that of uldetermi1led coenicieniS.. is desigte-d only lbr fu nction..~
F (~r ) in cqua1ion 15.48 o f the form X 11 ,
s in fJ.t , CQS JJX. and/or sums or products thereof.
Essentially. the operator method sa)'S that if

"'.x,

~ (D))'

= F (>").

then

( 15.51)
But there is a problem. \Vhat docs it mean to say t.hou

</>(0)

= aoD"
~~--~~--------=--+ a 1D" - ' + + a,_, o +a,

( 15 .52)

operates o n F(x)1 The operaoor noethod then <.lepends on our explining how I /~ ( D) operates
on F(x}. The si onl>leso1/J( D) is 1/J(D) = D . In 1his cae I he d illcrcntial equation is

Dy
and rhc solutio !'I is

y(.r) =

= F(x }.

F'(.r) d .r.

If 1he solution is a lso to be represented in operator noration by

y(.r)

= D F (x) .

then we must 1.lefinc the opcrulOr I JD by

~F(x) =

F(x) dx.

( 1 ~.5.\)

If 1/ D nll:(ut.:; integnuc. then 1/D 2 muse mctm integrate 1wicc. 1/D 1 integrate rhrec 1imes. and

soon.
We have ~<~.au~d thatlhc onc1hod of operators is tlpplicablc in general only when F (x) consists
of powers. exponcntinls. anc.l/or sines and cosi nes. Consider first the case in which F (x) is a
power x", iJ> which case equmions 15.50 and J5.51 become

f/J (O)_y = x"

(1554)

and
I

y = - -x" .
cf>( D}

(15.55)

1098

Cl~1c.: JS

Differemiat Equali()I1S

If we forget for the moment that Dis a differential operator, and simply regard 1/tf>(D) as a
rarional function of a variable D . then wo can express If> (D) ll$ ar1in finite series of the form
I

(1 5.56)

</!(D) =

for some nonnegative integer k. (V{e will show how in a mornem.) But chis suggests th:u we
wri te 15.55 in lhe fonn

Y = ; t lbo+ b, D + bzD2 + lx".

( 15.57)

Jfwe now rei merpret D ll$ d fdx, then the operaroron the right will produce a polynomial in x.
(NOie that D'" x" = 0 if m > 11.) lttums out thllt if we ignore all arbitrary constants lluu result
from the integrations (wh.:n k ~ 1), this polynomial is a solution of 15.54. fn other words,

when F(x) = x", a particulax solutio" of 15.54 is

= DkI {//0 + h t D + iJ7, Dz + .. lxn .

)'p

( IS.SR)

We .now show how to expand 1/ tf>(D) as a series of fom1 15.56. Only two simations
arise and each .,r these can be illustnued with a simple example. First suppose that (0) =
0 2 + 4D + 5. For I/(D) we write

(D)

I
-=-::-----.,::-~

D + 4D + 5
2

1/ 5
4D + D 2

1+ - --

'

and imerpret the right side as the sum of an infinite geometric series with first term 1/5 and
common ratio - (4D + D1 )/S. \Vhcn we write chis series ou1, we h<1ve

D2 + 40

+5

which is of the re.quired form 15.56 with k = 0.


Power k in 15.56 is positive only when <(J(D) has no constant tcnn. For example, if
tf>(D) ~ D 4 - 2D 3 + 3D2 , then we factor out D 2 , and proceed as above:
l

rp(D)

_
I_{I+ :_D
+ ~D 2 + .. }
30
3
9
2

I 5.9 NonhvnlVgencous l .h.e:~ l)l l're -eilli:l l Eqmliun$ wilt Conu:uu Coetrk:ienu

1019

I EXAMPLE 15.20
Find a particular so lmion of 1he differemial equa1ion

SOLUTlO>J

= x 2 + 4.

+ 6/ + 4y

y"
We wri1e
I

)'p

= o z + 6D + 4 (xl + 4) =
4 ( I+

~ {1_

oz) + (

6D :

6D
-

+ oz) (x + 4}

"-7"'-

2 2

6D ; 0 )

_ }

(x~ + 4) .

Since D"(x2 + 4) = 0 if 11 > 2, we require on ly 1he constanll.e rm and lemts in D and

y1, =

I EXAMPLE

o z:

~ { I - ~ D + 2D 2 + } (x 2 + 4)
I

;j<x- + 4) -

'8(2x)

x2

2:(2)

= '4 -

3.1'

4 + 2.

15.21

Find n partic ular so lution of the d ifferentia l equa tion


y"'

SOLUTIO'\

+ 2y'

= .x 1 - x.

A particular solution is
I

0(2

+ D2) (,<

- x)

1
= 20

2oz + .. .

I -

I/) { X 2 - X -

l,

(.r- - .t )

2I (2) } ,

and since I/ D means to integnue with re.spect to,\". we have

y,, = ~ { ~3 _.~z -x}= ~ _

:2_~-

J.n summary, 10 evaluale 15.51 when F(x) is a power .x" (or a polynomial), we expand I/</> ( D)
in a series of fonn 15.56 and perfonn 1he indicaled differentiations (and integrations if k > 0).
For all otheo cases of F (.x). we make use of a 1heorem called 1he inverse operator sh ift
theorem. This 1heorem suu.es thaL

- /e"' f(x) \ = e"'

</>(D)

</>(D

+ p)

/ (x)

(15.59)

(see Eterci<e 16). The theorem enables us toshifl the exponential eP pa~ttheopemtor 1/ r/>(0).
but to do <o,the operator mu<t be modified 10 1/?(0 - p). "'Jualion 15.511 immedoa1ely yield<
)'p(.r) "henever f(x) is a power x,,. a nonocgati,e in1cgcr (or a pol)nomial). This is
illusuu1cd in the following awnplc.

I EXAMPLE

16. 22

Find panicular solutions for the following differential equations:

<>

y~ + 3y' + lOy = .r 2 <'-


(b) y~- 4y' + -ly = l'l.r
(C)

y'" + 2yH.!. Jy' _ )' = <'lJ'

\01 I '110'\
(a) Equation

15.59 gi\ e.
I

0 2 + 30

= l! - ..

+ 10

xle ' =

1)2

+ 3(0 -

I)

+ 10

.rl

+ 0 +8

02

,.-x (0 -

X ...

tuod we now procccd as in &ample 15.20:

Yv = e- (

D+ D' ) x2

8 I+

=s

e- {

I-

(D

+s D

c~ { 1_ ~ _ 7~' + ..

2 }2
02
)
s
- .r

D+

} 2= e~ {xl _ ~ _ ~ }

(b) Once again we use the imerse operator shift theorem to get

"

-0

;p -

=e

1 -

I
40

lx I

01

(1)

+4

2.<-lx
t'
- ,.

.Tl!

(0

+ 2)1 -

I
4(0

2)

+4

(I)

-. ..

= -2 e-

>inc:c 1/ D1 mcdll> 10 integrate twice.


(c) For lhis differential equotion.
I

2r-

y, = 0 3 + 20 2 +3 D- I"
I

= e (D-

2) 3

I
2(0- 2)1
I

+ 3(D

2)- 1(I}

- el '
(I)
1
D' + 80 2 + 23D + 21
= e -(~----:2:::3-;;D-+,...-:-8-;::D""'-+-O"'J.,.,..) (I)
21 I + _.:....:____:--=.:::.._:....::_
21
e2'

= 2i (l + .. }(I) =

e2.r

21

15.9

I EXAMPLE

~olthOn\Ogefh~OOS

LineAr Oifftr~nti al Equtllio()M u:i1h CorHl:lnl Cocftiienli

15.23

Fin~ a general solution for y'"- Jy"

+ 3y'- y = 2x 2e' .

SOLUTION The auxiliary equation is


0

= m3 -

3m

+ 3m -

= (m -

1)

with solutions I. I, and I. Thus.

A particular solmion is

J\

I EXAMPLE

general solution of the difTcrcntial equation is therefore

16.24

Find a particular solution fo, the differential equation


y"'

+ 3/1 -

4y =

Xt'-2r

+ .r2.

sonmo"
I

Yp

= D3 + JD2- 4 (xe
- 2x

=e

- 2-r

+x

( D - 2)3 + 3(0 - 2)1 - 4x

= e- .u.

X -

03 - 302

D3 + 3D2 - 4x

3
2
(
30 + 0 )
4 1- --4

1101

\Vhen F (:r) ill 15.51 is of the form x'1 sin 11:r or .r" co~ px. n a non'oegGthc integer and
constant. we intmcJucc complex epanentials. Specihcally. becaus.e
.~c'''
\ VC can

fJ

= .r(CQS I" +I sin p.t ).

wri 1c

x" cos p.r


.rsinpx

= real p.ut or.ttP' = Ref., " t '"

nlllgirypanor.r,.IP<

l.

lmf~ "t'"'l.

To opcrntc nn cilhet of the~ t\ mctions by 1/t/J(n). we intcrcha\ge the opetatioM or


1/1/>(D) and of taking real and imaginary parts:
1

-t.cosp.\l
(D)

1
1
= __!_Re(t"t>
P
'I
=
R
e{.,.
1) (x''<""'> }.
(0)
.,.( )

I
--fx"sin
/JX I

t/J(f) )

I I1
- I- lmfx'e
' I
( D)

I - (x r ;,,.. ) .
l m { - -_
( /J)

tl \

11 1

( l~l> l h)

We can now proceed by using in\CtSe oper.ttor shifl thcorcm 1559.

I EXAMP LE

16.25

Find pan icuhw Mthniots for tltc foll(lwi ng dilrcrcmial equations:

ta)

>' + y =

sin 2x

tb) y" + 4y = x1 oos t


(c)
(d)

(c)

y" + 9y =sin 3x
y" + 4y = x sin 2x
)"' + 2y' + 4y = c- sm ..!3x

SOl L'110 N
(a) Equation 15.61b givt!l.
1

)'r

= Dl +I

I
flm(e21 ' ll =
~in 2x = Dl+
1

lm {

e'" }

0 2+ 1

We now use inver.c opcrtor shift theorem 15.59,

y, = lm

{Pllr (D + ; i )l + I (I) l

~ (, o +:;o - ,< >)="+ " _, (_'~ 0 : "')'"I


=

lm

~ =
2i< }

-3

I
--sin 2x.

{b) We begin by using equation l5.6 la,.

)'1 =
'

+ 4

D1

= Rc e1...
{

= JI

x 2 cosx =
I

I
D1

+ i)2 + 4

(D

+ 4

, . ) = Re
Rc(x-eu

x 2 } = Rc e1...

D2 + 4

D 2 + 2iD

+3

}
x-e'"'

x2 }

-I Re { e ;x [ .x.l - -4ix - -14]}


3
3
9
2

X COS X

+ J4.r Sill. X

914 COS X }

+ <Jiy

(c) Forthedifferential equation y"


)' =

''

I
1

D + 9

sin 3.r =

= lm {e:U.<

D + 9

= s in 3x. we have

= lm {
lm(~~)

I
2

D + 9

e3'"' }

I
( I)}= lm{e3h 1 I
(I) }
(D + 3i)1 + 9
D + 6i D

I
e3i.r I
=lm e'#x _I
(1) } =lm { {
D D + 6i
6i D I

+ D/ 6i

(I) }

i 3"
=lm { -;:;e'"

(d) For the differential equation

=
=

0 + 4
1

x sin2x =

lm {e21' <D+2i)1

0 + 4
2

21

lm(xe ') = ion {

0 + 4
1

x2

+-

16

xe21'

2" 1 x}
x}
= lm
{e
+4
D + 4iD
2

=
- - cos2x
8

( t) }

y'' + 4y = .r sin 2x. we gel

I
I ux -I ( 1 - -D
= lm {eux -I
x } = lm { -e
DD+4i
4i
D
4i

sin2x .

Jm { - -i eu,
4

+ ) x }

(x- + -ix)}
2

1104

Ch>per 15 l)ifferential E.quativns

y" + 2y' + 4y =

(c) For ihe differential equation


)'p

= 02
=

+ 2D + 4

lm {

e-x sin J3x =

0 2 + 20

+4

e<-1+./ii)r}
1

= Im {e<-J+./ii).r

(D - I

= Tm {eH +Jiih
= lm {eH+./ii)

I EXAMPLE

e- sin J3x. we have

+ ./3i)2 + 2(D -

D2 + 2J3iD

~ 2~i }

( I) }

= lm {-

+ ./3i ) + 4

(I)}

= lm {ei-I+Jli)x~

D D + 2J3;

(I )}

2~.u<-+J3i)r }

15.26

+ 6y' + y =sin 3x.


The auxiliary equation is m2 + 6m + I = 0 wiJh solu1ions m =

Find a general solution of y"

SOLUTION

- 3 2../2.

Consequently,

A panicular solution is

)' =
P

+ 6D + I

D1

,.
l m e~u
{
(D

= lm

,.

e~ x

Tm(i'31.r)

= l nl {

I
D 2 + 6D

+ 3i)' + 6(D + Ji) +

+I

I }
I( )

oz+ (6 + 6i)D + (-8 + lSi ) ( I )

.11 I
= Im ,..x
-8 + lSi
1

-8- 18i J
-8 - lSi

eJI.r }

. ---,--I
}
lm e)'"

I
= -1-{4siJI3.r
+
94

-8 + 18i

9cos3x) .

Finally, then, a gcocrul solution of the diiTcrcmial c~uation is

y(x)

c I e (-J+lJ2h + c,e(-J-l,/2).<
-

1
194

- - (4sin 3x

+ 9ros3x).

Examples 15.20-15.26 have illustrliled that with identity 15.59 and the concept of series, we
can obtain a particular solution for 15.50 whenever F (x) is .r", eP' .~in px. cospx. and/or
any sums or products thereof. In summ:~ry :
when F(.r)

= .r", usc series 15.56 for l j( D).

In nny other situatio n, usc 15.5\l, and then series 15.56 for I/ </> ( D + p).
We make one final ool'nment. We introduced complex exponenlials eip.t to handle terms
involving sin px and cos px . Thjs is llOl the only way to treat trigonometric funct ions. since
there do exi st other methods that completely avoid complex numbers. Unfonunately, these
methods require memorization of somewhat involved identities. Because of this, we prefer Lhe

use of complex numbers.

1$.9

N'onhonlOgei~OII$ Line:v OiiTerenti:ll

Eq!ution' \\ith Con)Unt Codficienh

1106

EXERCISES 15. 9

In EAtrei ~ 1- 11 find a r :utW::ubr $;()lution for the diOCrcr.tial cqu!l.


tion, by both 1hc method ot" opcr01ton: und the lllc:thod of ulk.k.1c-f.nined
codficicni.S. Find a gcncrul wlution or the Ctlm.Uion.
I. 2y.v - 16y'

2.

l' + '!,\'

J. ,Y111
4.

1
-

+ 32,\' =
2y

- e" '

= xle- "

3y" + y' - 3}'

= 3xt" el 2
and

,Y'"' + 2y/t + y = cos2x

5. y

11
'-

is c;:tllocl the Ctmdty- F.ulrY lir1em N(IIMitm. Because or Ihe: .\' 2 and
.r- l ~lc;to ~ . it doc:s not have: constant coc:l'licicnts, :.nd i.s chcrcrorc no
immediately umc nublc to the tec h n ique~ or ttl is <:huplcr. SI'IO\v ttuu i f
we make u change or independent variable x = t :.. then

Lh:.~t

tly
dz'

as a result the Cauchy- Euler equation

i~t

tr..nsrom'l(:d into ;1

Iincar c:q uution in y(z) with constant coentcicnts.

6y" + 12,,' - Sy ;;;;; 2e1 ' '

.vm' + SyH + 4y = c- ZA
7. y~ - 3y" + 2;-' = x'2 + e A
~. 2y" + 16r' + 82y = - 2,.:..' sin.r
9. )!'' + .\'
6)" =- .( + C()$ X
10. y" - 4y' + Sy = xcos .r
6.

In ExerciS<5 23-:24 use the technique


solulion for the d itTcrcntial cqu;uion.
c/ 211
" 23. Jr2

I d11

+ ; dr

or Exerdse 22 to lind a general

u
- rl ; 0. r > 0

II . 3 .>"' - 12y"

+ 18y' -

12y

tc 24 . .\' 2)'n +X)''+ 4)'

=.r' + 3.r- 4

!/11-d'.r

l'ici<:nt$.

dr'

+ 9y" + 21y' + 27y = .xe3' + 2..r cO'S X


J + 4y"' + y' + 4)' = .tt.t. Sill.\'
2y"' - 6y'' - 12y' + 16J = .Xl'A + 2x 3 - 4coc;x
2y"- "'.v' + IO.Y = 5c '' l:lin2x

13.

14.
.15.

111

* 26.

17. y"

+ 2y' -

18. 2y~- 4r'

4J

=~os ,(

+ fJ "'
- + kx
tit

COil!'la nL~. rind 3 p~1icular

~ Jl in(l)l .

We have aossunM.:d lflO!IIhc nonttt.nnugcncil y in 1.t48;.a i scomi11uous

a' 1hc: imc:nal I <11'1 which lhe difrcrcntial equation is to be considcted.


It is pos.<iblc to adapt out proct."durc tononhomogcnci tics dull wc pi ece ~
wisecontinuou$. Forcxumpk, consider th..: initial\'aluc t>robleln

/' - 3y' + '2y

.., 16. U!ioc the operator shift theorem of f::xertisc 2.S i.n Se<tion lS.8 to
verify the inverM: operato. :J1ift thcorcrn I S.S9.

In ~crciscs 1i - 18 find u ecncrul solution for the differential e<1uation.

I, .\' > 0

"""' 25. Ir M ~ fJ ' k . A' ~n d (J) nrc aU ~ili\'C


$i)IUiion of til\: Jincnr di ffcrcn l i:~) C(jUiLion

In ExCfeiF.CS 12-1 S state the forn1 fo r the particular solution predicted


by the me1hod or undctcrminct..l coe fficient:;. Do not evaluuLc the <.:ncf

12. :;"'

= F(x) ,

" I

H.r) =
hns a discontinuil y til X

= 2,

y(O)

y' (O)

X.

o .s x .s 1.

0.

X>

- 1/2,

= J. Sohc: this problem using the rollowing

method.

(a) Find 1he '\Oimion to the initi:ll\':tlue pmblcnl on the intcrv:1l

+ 3y = co.sx si n 2.r

o .s x .S I.
(b) Find 11 !Cnc:rul solution for x > I.

In Excrci ~ 19-20 lind a ~(l! utio n for Ihe dit'f!.wenti.al oqu.at ion.

1?. y- 3)' + 2y =

s.,' + 12e

= o. y'(O) = 2
y(O) = .v' (O) = 0

'. y (O)

20. y + 9y = x(sin 3x + cos3.<).


-+ 21 .

(c) OK>OI:SC lhc constum.s in the w lutiun to p~1r1 (b) so thut the
sohuion nnd its riNI derivative t1rtcontinuous 1:11.\';;;;; I.

1r J . k.. and ware positive con!l-tan.ts. tind a general solul tOn foe

(d) Doo; till! funCiion in pan (C) satisfy tltc d i llcremif~ equation
foroll .T > 01
27. Usc lhc method in Exercise 26 to find 1hc solution. wilh a t'OIU i n~
uou:; fi r~t dcrivali vc, (or the initialvaluc problem

, d_ ' .v + ky = w.

y"

t f.lA

+ .1'

= F(.r).

y (O) = 0.

y'(O) = 0.

+ 22. The second-order linear differential cquJ.tion


when::

d 2v

dy

x-dx2 +ax d- x

+ by

= F(x),

a , b constants,

o::x::Jf'.
X

>

iT.

**

28. The bib:mno nic differe ntial equal ion for :a !'unction <t>(r) :ari.scs in

...-.

29.

(o) Sh<>w lho1 if o function <l>(r) = /(r) coo 110 , where 11 ;: 1

two-dimensional claMicity theory:

, d' <l>
r - dr'

+ 2r

d 2<1>

, d' <t>
-- - r -2

dr'

dr

t!<t>

+ -a.

is ii.n intCJ:Cf. is to ~tbJy the bihal'monic equation or E.>.Cf


c.lse 34 in Section 12.6. thcn f(r) must salisfy

= 0.

Extend the ICChn iquc o f E xercise 22 to show lh :U ~ gnC1"J l solulion of


this cquu1ion is

d' f

2 d 3f

l +2n' d' f

-dr4 + -r dr3

tlr 2

r"

1 +2n 2 df
rl

- +
tlr

11' -

r'

411 2

f -- 0

(b) Exb:nd the result of Exc.-rclsc. 22 to find ull solutions for

/(r}. Considcrthecascsn - I and 11 > 1 separately.

115.10 Applications of Linear Differential Equations


Vibrating Mass-Spring Systems
In Figure 15.10 we hae shown u mass M suspended venically from a spring. If M is ghcn
an initial rnotion in the ,enical d irection (and the venical d irection only). then \ve expect M to
oscillate up a nd down for some time . Tn this section we show how to des<:ribe these ~illations
nlathematica lly.
M:iiitc:lil m.b[;_J Displace
mem of V!br;llil g mK< rdati\'t 10
unstrelcbed position o( .s.ptir~,g

0l

1.

we must choose a venical

upwMd. When M is a dis1ance y oway from 1he origin , the restoring force or 1he spring has
y -componcru - ky (k > 0) . In addilion. if g = 9.81 is the acceleration d ue 10 gravi1y. 1hcn
the force of grnvity on M has y-componen1 -Mg . Finally, suppose oscillations take place in
a mediurn that c.xcns a dampitlg force pro,>Ortional to the instamaneous velocity of M . This
damping foree must therefore have a y -corntp<>nent of 1he form - fj(d yfdt ), where {3 is a
posi1ive COilStant. The 1oull force on M therefo re has y-com(>oncnt

r
~

In order 10 describe lhe position of M as " funu ion of time

coordinate system. There are hvo natural places to choose the. o rigin y = Ol o ne being the
position or iW when 1hc spring is un.strc::tchcd. Suppose we do this and choose y al) po:sitivc

d)

/J d.t ,

- ky - M g y=Owhen
spring is U!\.Sttch::hed

aml N~wtons second law >tales ~1a1 1he accclemion d 2 y f d t 2 of M must satisfy ~1e equation

dy

- ky - Mg - {3 dr

Consequemly. 1he differemial equation lhat determines the position )'(/ ) of M rela1ive 10 the
unslrctchcd posi1ion of the spring is

li'C Ja:IU'"~~
Oispt:.;e.
ment of vibraa.i.n~ ~s reiQiive to
i~ equilibrium ~i lion

r
l l . ,.

cquiJibrium

d'y
Mdt '

t!y

+ {3 - + ky =
dt

- Mg .

(15.621

The allerna1ive possibili1y for describing oscillations is 10 auach M to the spring and slowly
lower J\1 untiJ it reaches an equiljbri um position. At 1h is posi1i.on. the restoring force of 1he
s pring is exactly equal to the force of gr.tvity o n the mass, and the mass, le fl by it.self, will remain
motionless. If s is the amount of stre.tch in the spring at equili brium, then m equi li brium

ks - M g = 0,

S uppose we take the equilibrium position as.

x =

wheres > 0.

0 and

( 15.63)

as positive upward ( Figure 15 . I J ).

When M is a distance x away from its eqt1ilibrium position, the restoring fo rce on 1\1 has

I.S.. IO

Appli~tioosor L..int:4r

DltTcrenti.l EqWlf.iOJ>

1107

x -compooent k(s - x). The x -component of the force of gravity remains as - M g, and !hat
of the dampiog force is -fJ(dxJdr) . Newton's second law therefore implies that

d 2x
M drl
or

tl2x

M-

= k(s -

x) - Mg -

tfx

pdr

dx

+/3- +kx

= - Mg +ks.
dt
But acco rding to 15.63, - Mg + ks = 0, and hence
dt 2

d 2x
dx
M -2 +P - + kx =0.
dt
dr

(15.64)

Thi~ i~ thedifl'eremialequationdescribing thed isplacement x(r ) of M relat he to the equilibrium

posi1ion of J\4 .
Nme that both equations 15.62 and 15.64 are linear second-order diffc.rential equations
with con swot coefficients. The advantage of 15.64 is that it is homogeneous as we11, and this is
simply due to a convenient choice o f dependent \'lltiable (X as opposed to )') . Physically. we are
saying thatthe.r e are two parts to the <!>ring fon:e k(s - .r): a ,)an ks and a pan - kx. Gravity
is a lways acting on M . and that pan ks of the spring fo rce is counteracti ng it in a n a ttempt to
restore the spring to its unstrctched position. Because these forces always cancel, we might j ust
as well climinl\lc both o f them from our discussion. Titis would loa,-e us - kx, and we therefore
interpret - k x as the :;prirzg/QI'Ct nrrempliu~ 1u reswrc the mass tu irs t:quilibrium posiliou.
If we choose equation 15.64 10 describe the m01ion of M (and 1his equation is usually
chosen overequntion 15.62), we must remember three things: .t is measured from equilibrium,
- kx is the spring force Ultcmpting to restore A1 to its equ ilibrium position, and gravity hn:;
()ecn taken into acco um~
11'lere are lhree ba<:ic ways to initiate Ihe m otion. First, we c ln mo,e Ihe maslt awa~ rrom its
cquilibriun1 position and then release it~ giving it an initial displacement but no initial ,elocity.
Second . we can strike the ma ss at the equilib1iurn poshjon. impanjng an initial velocity but no
i nit ial d ii\pl:acement. And fimllly, we can give the ma.~s both an initial displacement and an initial
\'Ciocity. E.ac-h oflhese methods adds 1wo inili al conditions to the differe.n tial equat io n.
To be compklc we note that when (in additio n to the forccl; already mentioned) there is an
externally applied force acting on the mass t hat is represented as a function o f time by F(l).
then equation 15.64 is modified to
( 15.65 )

Perhaps. for example, M contains some iron , and F (r) is due to a magnet directly below M
thut exerts !I t.imc-dcpcndcnt a ttrm:;tivc force on A1.

I EXAMPLE 15.27
A 2wkg mass is suspended vertic~llly from a spring with COitSiant l6 N/m. The rn:t..~~ is mised
10 em aoovc iL~ equilibrium position a nd then released. If dalllJling is ignored. fi nd thcampli mde.
period, and frequency of the mOl ion.

SOTlTTION lfwechoo...e x = 0 at the equilibri um position o f the mass and x positive upward
(Figure 15.1 I}, d ifferential equation I 5.64 for motion .Y(I) of the mass is
d 2x
2- 2
dt
or

16x = 0

aiOlg wilh the inilhtl tondilions

.r(O)
The :lliXili:uy cqumiorl is m 1 + 8

= 1/10.

.r'(O)

0 wilh !!Olutions m

=:

x (t) = C 1 cos(2ht)

= 0.
= 2.J2 i . Collscqucruly,

+ C2 sin(2J2t).

The init.irll COrldiliOJlS requl.-e thal

T hus.

.<(1)

= -1I0 cos(2.J2 1).

The amJ>Ii wdo of l he oscillati ons is 1110 m, the period is 2tt/(2.J2) = rr/../2 s, and the
frequerx:y il) ./2/lf s- l . A graph Q( this func11i011 (Figure 15. 12) illliSI.ftliCS the uscillalions of
the mass nbout iiS equilibrium position. T his is an cxcunple of s imp~ hannonic murinn.

fiT":,(J)m.J S imple

L"3'J
m11~

hittmonic n\QIM,)n of vibr.11ing

when dumping j, negli&ibl('

.r

I EXAMPL E

1 5. 2 8

A I 00-g ma";i.S is s uspended venically fro m a spri ng with con..'\tant 5 N/m. n,e. mass is pulled
iL~ equilibrium position and given velocity 2 m/s upwnrd. If during the mo ti on
the mass is acted 011 by a dampirlS force in newrons numericall y equal to one-twentieth the
instantane ous velocity in metres per second, A11d the posit ion of the mass at any time.

5 em below

SOLliTIO'!' lfwechoo.s.e ~r. = Oat the: equilibrium polSidon of t he tna..;sand x po~i tive upw<l!'d
(Figure 15. 11), then diff eremial equarion 15.64 for molion x(t ) of the nll!SS is

1 d2 x

I dx

- +2()
- dt
- +5x=O
10 dt2
or

d 1x
2d 1
I

dx

+ -d./ + IOOx = (),

aJong with the initial c.o ndh ions

x (O) = - 1/2(),

x'(O) = 2.

15. 10

The aux ilimy equation is 2m 2 + m

m=

Applkat ion~ofLir~ I Differ'C!ti al Equ;;~ i O!'IIi

1109

+ 100 = 0 with solutions

-IJI-800 = - I Jmi
4

Conscquencly.
x(l) =

e-ttrc,oos( J799t/4) + c ,si n(J799t /4)].

The initial conditions rc.quire that

-1/20 =Co.

2 = -C1/ 4 + .../799C, /4.

from which we get


159v'?W

15 980
Finully, then.

x(l) =

1, - tft

[ I (.J'i99 I) +
--cos - - 20

15') J799' sin ( .j'fijij


15980

I) ]

The grnph of this function in Figure 15.13 dc:orly indk>~ tes how the amplitude of the osci llations

dccreuses in time.

0.2

0.1

- 0. 1
-0.2

I EXAMPLE

15.29
T fthe c~lr in I he Application Previl!w moves with col stant speed v , nld the equation of the road
is Y = 0 for x < 0. und Y = f(x) for.< :::, 0. find an initinl-vuluc problem describing the
vertic.al oscillalions of the fron t ettd of the c~w.

Application Pn!view
Revisited

SOL UTION We assume that the four wheels aet indepcndcnrly and that M is the mass of that
pan or ~le car supponed by the wheel (Figure 15.1 4). Let y measure the height of the car above
its equilibrium posicio1l when it is motionless. Newton's second law for the. motion of M gives

Mjtdil;l:11j1WI M Dii
plat.-cmcul uf (runt cnJ of a l".&l
as wheel folk)'.\S UllC\'Cil rnad

dy
-fJ -d I - k(y- Y - s)- M g..

)'

t.::. .r = 0
at equilbtium

where. g = 9.81 and s is the compression in the spring when lhe car is at equilibrium. Because
ks - M g = 0 at equiJibrium, it follows thal
X

cf2 y

M-

dl 2

dy

+ /3 -d! + ky

= kY.

Since the speed of the car is v, the x -coordinate of the car is x = vi (taking I = 0 when it
passes through the orig in), and therefore

d2y

dy

+ fJdt + ky

412

= kf(vt),

t ;:: 0.

The solution would also be subject to the initial conditions y(O) = y'(O) = 0 .

LCR Cir~its
If a resistance. R. an inductance L. and a capacitance C are connected in se-l'ies with an elecMjiHii;I10.11JF11 Scbemcuic
fo cu-reru in

LC R <:irtuil

tromotive force- E(I) (Figure 15. 15) and the switch is closed, current flows in the circuit and
charge builds up i nthe capacitor. Tf at a ny time t . Q is the charge on th e capacitor a nd T is the
current in the loop, then Kirchhoff's law .stales that

dl
Ldt
R

+ Rl + -c

= (1),

(15.66)

where L d I I dl , R I , and Ql C represent the voltage drops across the inductor, the resistor,
and the capacitor, respectively. If we substitute I = d QI d t, then
( 15.67)
a second-order linear d ifferent ial equation for Q(t) . Ahernatively, iJ we differentiate this
equation, we obtain
d 21
ti l
I
( 15.68)
L - 2 + R - + - I = E'(t) ,

dt

dt

a second-order linea r d ifferen tial equation for I (t) .

I EXAMPLE

15.30

j(iiiiciii;IIWW Chnrgc
the c:tp.1c-itor :tnd corrcl)r it\ :ul
LC U -circ:uir

<)(I

At time 1 = 0. a 25 Q resistor, a 2 H inductor. and a 0.01 F capacitor are connected in series


with a generator prod ucing an alternating voltage of J0 sin( 51) . t 2:: 0 (Figure I 5. 16) . Find
the charge on the capacitor and the current in Lhe c.i rcuil if Lhe capacitor is unc harged when the
circuit is closed.

SOLUTION Diffeential equation 15.67


d2Q

2H

2- dr 2

tO sin (St)

2s n

dQ
dt

+ 25 -

fo~e charge Q on the capacitor is


+ IOOQ

= JOsin(5t).

to which we add the iniliaJ condilions

Q (O) = 0.
The auxiliary equat ion is 2m 2

~
m =

Q'(O) = / (0) = 0.

+ 25m+ 100 = 0 with solutions

- 25

J625 4

800

- 25

s.J'i;
4

Con:;equenLI y, a gene1al solution o f the homogeneous equal ion i:;


Qt.(l) = e- 251' 1C 1 cos(S..fit/4)

+ C2 sin(5..fitl4)1.

I.'I. f (l

Apflfd-.,., of l.te.ar l>itfe1~1i1J 6QuJlli{lil ~

1111

Tu find a p.anicuJar sofutiun of he nonhomogcllCOlL'!> equation by Urldctermincd coctrrcic:flL,. \\C

Q1,(1) = 11 sin(St) -r ll ct..,(SI).


Subs1Huti0f1of' thli J'uoc-Lion ullel the ditTercrui:l equation gives
~/-25Asin{51) -

25B cos(St )l

+ 2S f5A cos(St) -S B d n(St)J


+ J()()f;l sin(SI) + B co>(SI)I =

SOA - 12.!8 = 10,

Q1,(r)

12.M +SOB

2
= i4s
[2<in(Sr) -

10sin (5r) .

0,

Sco.,(SI)J ,

_ -:.<~t[C, cos (s,fir


. (s./1')]
- 4 - ) + C,<ooo
- +
4

- '

2 t2>in(St) - S c-os(St)J.
145

T'hr inirWJconditions ~uire rluet


10
25
s,fi
20
c,- -145
+ - c, + . o = - -c,
4
4
145'

0=

nd Jhesc imply i11<11 C - 10' 1 oc C _ 3 ' / (


' -

1 " "

2-

= "-145./'i
-25tt [ IOJic(l>

Q(l )

r.
14 5 v 7). Consequently.

_!_
(s./'
i )+
4

.
34 $11)

( '" )]
5v7r

-4-

2
j45[2,in(St)- Scos(St ) J.

The cum-n1 in tJ~e c:in't.lil is


l (t ) = dQ
dl

== (- 4, )

~
[I 0../7
/71) -'-34 <i.ll (s../7')]
1cos (5.
-:;7
145 v17

~ !Ji; [-~2 "in (5v'


7') + 2cos
SS.../7
(5v'
7')]
-:;~

145 v 7
2

+ J45110 c()S(5r) + 15sin(5r)f


=-

, - >Sl/ 4 [ 4

29.../7

./'icos

29

12cos(5r)

../7/ ' ) +

5sin(5r)(.

60sin

C~)]

1112

Chopler 15

Oifferenti11 Eqt~fioos

FIQUR& 1S.17e

FIGURE 1G.17b

Transic:m cbarse on ca.pa<:ilor

Slcudy st.ttc charge ora capacitor

Q (I)

p(\

(\

0.06

(\

(\

O.Q4
0.02
L

O.o3

- 0.02

O.Q2

-0.04

0.01

-().06
0.2

0.6

0.8

"

The sol ution Q (t) CQntains Lwo pans. The fir.;t two terms (conUtining the cxponemial
Q~o(l) with the constantS C t and C2 detennined by the initial C<'>nd itiOnS; the last
two tenns are Q p(l) . We point this o ut because the two parts display completely different
characteristics. Their graphs are shown in Figures 15.17. For small I. both parts of Q(l) arc
present and contribute significantly, bui for large 1. the first two terms become negligible. In
other word,, after along time, the charge Q(1) on the capacito r is defined essentially by Q p(l ).
We call Qp(f) the steady-state part of the solution. and the two other tenus U.t Q(l) arc c.aUcd
the Lransient part of the sohnion. Similarly. the firSt two terms in T(t) are called the transienL
part of the current and the last two terms the steady-state part of the curreut.
Finally. note that the frequency o f the sLe.ady-state pan o f either Q (l) or !(I) is exacLiy
that of the forcing voltage E (I ).

e-25'14 ) are

The similarity between differential e<Juations 15.65 and 15.67 cannot go unmentioned:

d 2x
M dl2

d2Q

L dr'

dx

+ p dt + k.r =
dQ

F (t),

+ R dt + CQ

(1) .

Each of the coefficients M, {J. and k for the mcchanjc~l system has its analogue L. R. and
1/ C in the e lectrical ~ystem.

Resontmce in a physic-.dl sys1em is some1hing lhat can be delrimemal 10 lhe system, or it


can be advantageous. Resonance iu a vibrating mass-spriJtg system is to be avoided. it

may destroy the sys1em; resonance in some e lectrical networks is a necessity. Resonance
occurs when a periodic force is applied to an oscillating systen~ and the frequency of the
force is chosen to maximize the amplitude of the oscillations. Sometimes the maximum
amplitude c-an be excessively large; other times, it is less dramatic. We have been asked to
analy:tc resonance for current in an LC R c ircuit (Figure 15 .1 5) wilh an applied voltage
E (I) = F sin wr . In particular. we are 10 determine the val ue of <t> thai yields resonance
and the resulting maximum ;:unplitudt:. We are also to flnd lhe. frequencies at which the
ampli1ude or 1he current faiJs lO p% of its resonance value.

15.10 Appli-calioos of Lineal' Diffel'en(ittl E-.JU<I(ions

1113

SOi l TIO'\

Example 15.30 showed that cunent in an LCRc ircuit has two components, n transient pi.lrt. which dies out quickly, i.lnd a slci.ldy-statc pi.lrt that pcn~isLS forc,cr.
Funh<rmore. the steady-state part is the particular solution of the d ifferential equation
delining the cu1rem. namely. equation 15.68.

d 2i

L tft l

Rdi
dt

+C

= wF coswt .

Ir we drop the" p"->~obscript for panicular solutions and let i (I) = 8 co-, wt + C >in wt
be the panicular (>teady-state) solution of this differential equation. then >Ub>lltuting into
lhc diffcrc:ntial equation gi\'e-s

L(-w2 Broswt- w2 Csinwt)

+ R(-w B >inwt + wCcoswt)


I

+ C ( Bcoswt + Csinwt)

= wFcoswt.

W~c equate coefficicn1s of terms in cos wt and sin wt. we obtain

Solution> m-e

B=
(

_I

cvC

_ I_ -

wL) F

- RF

C=
wL)2 + R2'

cvC

Consequently,

(~- wL) Fcoswt-

RF sinwt

i(l) =

( wiC -

wL)

+ R2

According to formula 1.52 in Section 1.7. the amplitude of this oscillating current is

A=

The amplitude is a maximum when the denominator is a minimum. and this occur'\ when

- wL = 0
cvC

w=

TI1is is the fl'cquelcy g i,ing resonance. and the resonance amp1iltldc is F f R. Notice that
the resor1ance frequenC)' is independent ot.lhe resismncc in the circuit: it depends on 1he
inductance and l:upadtanc.-e. On the other h and. the resonance a mplitude depends only on
rc~istonce. The s maller the resistance. the larger the resonance amplitude.

illlll>lillldc of Ihe currcou will fnlllo p % of peak value when

- w L)

p
lOO

(F)
R

)' + R2].

=> R2 = 1pz
0"' [ ( wC - wL

R2

TI>is equa1ion reduces 10

LCw2

Jt-

plj JO' RCw- I = 0,

a quadratic in{/). Solulions al't:

w=

JJ- p 2/l O' RC + J (l -

p2 /IO' )R 1 C 1

+ 4LC

2LC

Resonance for m echanical systen'L.~ is discussed in the exercises.; it is somewh3t differem


than that for the e lectrical circuit

Buckling Loads for Beams


Figure IS. I Sa shows !l uni fonn beam of length L thai is pinned at Ctlch end. A compressive
force of P newtons is "PPiied 10 each end of the beam along the axis of 1he beam. We assume
1ha1 gravil)' can be ignored so 1ha1 for small P. 1hc beam wiU be horizontal. If P is allowed 10
increase , a stage will be reached al which Lhe column buckles slightly upward (or downward)
(Figur.: I S. l8b). \Ve can find the s ma llest forc.e that c auses bud.:ling a11d 1he shape of the

ckflcclctl curve.

lllillJ i
applted to e nds uf

:1

m;m

uu,t:ling rc-.lllting from

t:on1t;rtl!.ll-i'~ load

be:1rn

'

'

It i s shown in the branch of engineering known as mrength of materi als that for small
of1hc dellccled curve a1 a poinl (.I', y) L propoctionallo 1hc momelll

dcftcc1ion~. lhe concavi1y

of P aboul A.
d.r 2

El

where E is a conslanl called Young's modulus of cltlsticity. It depe.n ds o n Lhe material o f Lhe

beam. Cons1am I is 1he momem of inertia of cross-= lions of 1he beam. If we se1 k 2 =
Pj(El ), !hen

Af)))l tc:tliOn~ of Untar U1tferenli:ll i:.qll:l.l il)n~

I.S. IO

1115

a second-order, homogeneous, )jncar ditl"crcntial equation with constant coefficients. \Vith roots
ki of the auxiliary equation. a general solmion is
y(x) = C 1 coskx

+ C 2sin k.t.

Since deHection a~ the left end x = 0 of the beam is y = 0, it follows thm C 1 = 0. Similarly,
because y(L ) = 0, we must also have 0 = c 2 sin k L. We cannot SCI Cz = 0 (else y(x) = OJ,
SQ that sin k L = 0
k f_ = lilT, where n 'j:. 0 is an integer. Consequently,

n 2;r2

k2

= -= 2

p =

n2n2

EI

El
L2
The smallest value of the load for which deflection occurs is n = I. This load P = rr 2 E I/ L2
L

is called E uler's buckling load for a pin-ended column. The deflected shape fo r this load is a
sUtc curve

y(x)
It is not possible. to find the value o f C 2 .
Euler's buckling load depends on the boundary conditions a t the CJ>ds o f the beam. Oll><:.r
boundary conditions are discussed in ahe exercises.

EXERCISES 15. 10

(b) Find where the mass comes lO rest for the first time. Will

1. A lkg mass is Stls pcndcd ''CrticaUy from a spring wilh constanl


16 N/m. n,c mass is pulled I0 em below its cqui1ibrium position. anti
then rele.ased. Find lhe tXlSition of the mass. relalive to i l~ equilibrium

it move. from this position?

posili on, at any time if (a) damping is ignored, (b) a damping force
in newtons equal to one-tenth the i.os:tancaocous velocity in metres per

seco nd oc.ts on the muss, and (c) a damping

ro~c.

in nc,vcons equal to

ten times trn: instantaneous vcloc.:ity in metres per $CCOnd acts on the
mass.
+ 2. A 200.g lllOSS suspended vertically frorn a SjYiJ\,S \Vitb constant
10 Kim is set into vibrwion by an c.xi..Crnal force. in newtons g.i\'CO
by 4sin IOz. 1 ~ 0. OUI'ing the motion a ~.Lampi ng ro,cc in newtOI\S
equal to 3/2 the \'elocity or the mass in nlCtres per second ac~ on the
m!l~&.

* 3.

Find the ()C)$ ilion of the

nt~~S a.~ !t fm\CiiOn

of time I.

A 0 .00 I F cap.'ICifor and a 2 H inductor nre con r'k.~led in series wi1h

a 20 V b"*.ucry. lf there is no charge on the c;~p~cit o.r before the b:mcry


is connected~ find lhc cutrem in the citcuit as a fuoction or time.

4. Atliu~c 1 = 0, a 0.02 F c.J.po.citor, a I00 0 resi ~tor, and 1 H inductor


arc (;Onnor..-ctcd in series. lf thc cbarge on the cupucitor is initially 5 C.

fin<.! ils charge as il runc.:tion of time.

* 5.

A 5 H inductor and 20 Q resisto1 are <.'Qnoected in series with a


generator supplying an osc.illaling \.10itage of IOsin 2/. 1 2:: 0. Wh;.u
are the transient and ~ tcady-state currenL~ in tht circuir/

6. AI llnlC 1 = 0. 01 mass /J.f is atb chcd co lhc end of a hanging


spring '"'ith COJlStant k, and d'lCn released. Asswni.ng thm da.mpiJlg i.s
negligible. find the subsequent displocc.mc.m or the m.ass as a function
or ti me.

7. A O.Skg mass sits on a tnble attached to a spring with constant


L8 N/m (figure foiJo,vs}. The mass is IJ\Illcd so a:slo stretc-h the spring

"----,~

Do o

f-- x
s 0 at uoslret~hcd
po:sition

* 8.

Rcpem Exercise 7 givtn 1ha1 the mass is pulled 2j em to the right.

..... 9. AltinlC I = 0 an unc;hargcd 0. 1 f t -iApadlor is ~onncxtcd in ~ie.s


with a 0 .5 H inductor and a 3 Q resisaor. l fthe current in Lhc circuil at
this instant is J A, find the maximum charge thai the c:ip:.tcilor stores.

10. DiiTerential equaLion 15.64describeslhemolionofamass Mat the

cod of a spring, takins damping proportional to velocity into account.


Show cuc.:h. of the following:
(a) If fj

= 0.

x(t) = C1 cos(./k/M I)+ C,sin{./k/M t),

cullc.d fiimplc harmonic muti011.


(b) If fj ;f 0 and

/3 2

4kM < 0.

adieU dumpe(/ oscillatory muriu11.

(c) lf P ~

0 ood P'

4Uf > 0.

lfi'- 4kM

S em and is then releasetl.

2M

(3) Jf friction bclwccu lhc .mut;s and the tu.bJc creates a force
of 0.5 N lhat opposes motion. :show that the differential
ct:~uation <.letenni.nin.g motion is
d 2x

dt'

called O\'rdamped morion.


(d) lf {J 0 and (P - 4k M = 0.
x (t) = (C,

+ 36x

= I,

x (O) = 0 .05,

x' (O) = 0.

+ C 2t)e- PI<'M>,

C111Jcd Crilicftlly damped mOtion.

1116

Ct~:~J)\ef

Oiftecenti~l

15

Equ3tion

"-' II. In the ligurc below, the piston is cau..<cd to move by changing the
pcessurc P of the gas by way of the \"ah'C. The spring restrains the
nxnion of Lhc piston. as docs the dlShp:lt, the latter exerting a damping
force pruportl()nal to the velocity of the piston. If tbc mass uf the
p1ston is 111, and A is its surf~ area exposed to the gas, the differential
equation for motion of the piston is

d2x
Ill,
dt

fc) If resonance is said to occur when the amplitude of lhr


steady-state oscillations is 11 maximum, what ''alue of w
yields resonance? What is tile maximum runpliwdc?

* 17.

(a) Acube L mcltcs on each side and 1~ilh IIWS M kilogrnms


lloats halfsubmerged in water. If it is pushed down slightly
and Iben released. oscillations rake piJcc. Use Archimedes
priociple to find the driTcrcnrial equation gorcming lhcsc
oscitiJrions. Assume nodllrnping forecsducl{)thc l'iscosity
of lhc W1ltcr.

dx

+ P-dr +kx = ,1P(t).

Find the sttadystalc sohuion when P(t) =

Posin wt, where Pu > 0

and w > 0 arc constants.

(b) What is 111c fnJQucncy of the o<ciiiDtic.,ns?

* .18. A weighing piJUiorm hus weight IV and is wpponcd by springs

Vatl'e

____,

P(r)

,___

Piston

:===

with combu1ed spring conslw11 J:. Apackage with weigh u is dropped


on the platlorm so tJwtrhc 11vo move together. FinrJ a fomwlrl for tile
maximum value of w ~o that olieillmions do no1 occur. Assume 1hat
1J1crc is damping io !/Joe mt>lion with con5tllnl p.

* 19. .<\ SpringlllOUJllcd tWOWlJCCI trailer Of OIJISS 400 kg is IOIVCd


X

.t = 0 al ccruilibrium

t2. A IOOg mass is suspended from a ~pring with conslan14000 N/nt


AI itscqu1tibrium po itiro, it is suddenly (lime 1 = 0) gil-en an upward
vclocily of 10 m/s. If an ex lerna I force 3cos 200/, 1 ~ 0, aciS on lhe
m~s, lind il~ dc;placcmenl as a funtlion c>f umc. Docs n:wnan.,-e
occur~

13. A. \Ctticat springhdvingcor.slanl64 Nhn hlh a1-l g mass auachcG


to it An .:~u:maHorce F(I) = 2 sin 81. 1 ~ 0 is applied to the mass.
If the mass is at rest at its equilibrium position ar rime 1
0. a1d
damping is negligible. find the posi1ion of the mass as a function of
time. Wh:l\ hawc:ns 10 rhe oscil arooos as rime progress-'S?

m speed v O\'Cr WI undulating roud surfocc 1vhosc comoor is Y =


A co~ (;rx /5) m. AbSUIItc no damping wrhc 1'Ctticat ~~cilltlion.> of
the rraiJer, and a spring conslant of 40000 N/m. Usc 1he ditTcrcmiat
cquarion of Example r5.29to determine the speed Jl ~>hich rcs01wncc
occurs for vibrations of tiJC trJilcr.

.. 20. Supf)Jsc in Example 15.29tlrJJ !of = 200 l g. A = 50000 Nlm.


P = 3000 kg/s, and the c:q~rion of 1hc road is sinU$oiJlf Y =
0.1sin (:rxj40) m.
(aJ Find y(1 ) for v = 10 m/s. \Vhat rs lhc ampliwdc of lhr
stead~ l>!.:llc p.Jit of the wlutoon?

* t~. A

nlllSJ;

M is ~u>pc!l(bl frtxn a venical ,pring with constant k. If

anc~tcrna force

f(r) - A C<',WI,I ? 0. i~ applied to Ihe mass. fimi

the value of w thll cau;cs rcsonaocc.


15. A 25/9 H induclor.a0.04 Fcupaci1or,anda gcncrarorwilh oilage
15 cos 3r are connected in series at time 1 0. Find the currcn! in the
circuit as a function of time. Docs rcsooance oecur'?
* 16. A mass M is suspended froon a vertical spring wirn constant k.
When an cxrcmal fotec F(t)
A ens WI. 1 !'! 0 is npplicd '" rloc
1)1:JSS. and a d:lmping force r.o
ot>onionalro velocity acts on 1he mass

(b) Rcpc.11 jXII1 (3) for v = 2() rnls.

21. In a reversible reaction froon subsurnce A 10 substance B. the


an1oun1 .r(l) of A Jllimc 1 ;ari;tlc. li>C clilfcr<nliol cquJtion
d 1x

during its o;ub~1ucm ascillations, the differential equation governing

the motion of M is
d1.t
dx
M+
jJ- + kx = A Cc~WI.
dt'
dt
(a) Show Ihot ~leady-<ralc oseilhuions cr M ;rc defined by
.t(l)

viler~

a > 0 is a conswnr. if .r(O) =0 und .r'(O) =211/ J. find ,r(t)

in lcnns of n :mt.I J.

22. A

p~rtic:c of mullS 3

(10. - 10).
(u)

, 11 - .\ lw1;. + fJ'or

t.,r
son

+ 11)

Show Ihal tL'OC:cnliaJ CtiU I.tlivu~ dctir.ing dtc tX})irion uf&hc

p"rricle arc

(k- MwJ 2 + (lw2


tb) Vcrif~ th.r this function c."'Jn be e.\rr.:~ed in 1~ fonn
t(l)

un.u rnm'\."'S in tbc .ty~ptrmc under the inllu.

ence of a consenztive for<'C field "ilh porcnriul runcrion U (X. y) =


12r(3y - 4x). The panicle srurrs m lime t = 0 from rcsr w posiriOfl

[Ck- M412)coswr + Pwsin<zM).

dx

-dt l + 3+ 2.r = a,
dt

d'x
= - 12r + 32x.
dl'

d' r

tlt 1

(b) Usc abc~ to ~'>how tf:ur v(r) mu....r


cqw.ton

-llA.

~.uhfy the

(""'rlh orJi.T

d'y
cf2 y
- 32 - 144\' = 0.

lit'

dt'

Soh'C chis diiTere,.ial cqtWion co find .r(r) and y(r).


(c)

l'kll the pach of the paruclc and malce any o:oncln<ioos cll3c
""""justified.

15. 10

Applications <.' f l.ir.ear Differential EqumiolS

1117

23. When a particle wiOt charge q and mass m moves in an electromagnetic field E and magnetic fiel d B , the force acting on it is called
the Lorenl:,force F = q (E + v x B), where v is its velocity. Sttpposc
that B and E arc constan&s. B in the negative X -direction, and E in the
positive y -dinx:tion. Find 01e path fo llowed by the particle if il stmi s
from rest at Ote origin.

i * 24.

(b) Show ohao <he boundary condioions y(O) = y' (0) = 0 =


y"(L) = y"'(L) + Py'(L)/(E /) require <hm

n te piezometric head distribution lt(x) in the leaky aqui fer below


must satisfy the boundary-value pro blem

d'h

dx'

K ,(H0

h)

KbB

= 0,

il(O) = Ho.

y(x)

h(L) = b,

whete K and K v are hydraul ic conduclivitie.c: oft..he scmipel'vious layer


ofch itkness 8 and the aquiferoflhitkJessb, and Ho is a fixed excemal
head.

Scmipcrvious layer

(2n

+Ll)rr.r ] ,
2

wheren is an imegerand P = (211

+ 1) 2H 2El/(4L 2) .

(c) Find Euler's I:M.tckling load fo r this configumtion.

28.

C4 [ I - cos

(a) When axially compressive forces P ;uc applied tolhc ends


of the beam in the figure below. clamped horizontally at
x = 0 and x = L . small dcftcclions must satisfy tJte
dinc rential equation

Aquifer

d'y

p d'y

-dx' + -E l dx'

=0
'

x= 0 - - - - - - - - -x--- L, - -x
where E and I are positive constants. Show that a geneml
solution is

(a) Find il(x).


(b) Ploc a graph of /t(x) 10 show 1ha1 1he head remains rclalively
Oa1 f()r 0 :$ .\' :$ 600 when K 11
10- 11 ntis. K
10- <o!S. b = 100m, 8 = I "' Ho = 125 m, and
L
1000 m.

y(x)

C:sin -JJ. L ( 2sin -JJ. L - JtLcos -f-' L) = 0,


2
2
2

26. A uniform chain of lc.ng lh a has 11 pOri iOn 0 < b < o hanging over 1he edge of a S(n004h 1ablt ( fi g ure bcl.ow). Prove thal lbe

Ja'

, I.

- - -o-b- --

27.

I
*

(c) What is Eul e~'s buck li ng load for tllis configUI'ation?

(a) When axially compressive forces P arc applied to the ends


of tlte beam in tJte followir"-figure. clamped horizontally
at x = 0 and lfee at x = L~mall deflections musl satisfy
the diiJcrcntial C<JUation

29. Reversible reac1io ns in 11 bmch reac1or under consl!ml volt.t(tJt! and


Le(opcralnre are represented as foUows.

k,

k,

8
d" y

p d'1\'

-dx' + -E l Jx1

=0,

where E and I a.J'c positive conslants. Show lhaLa genetal


solution is

y(x)

1' = JP/(E I) ,

whcren is an integetand P = 4n 2 H 2 EifL2

li(nt:: laken fOr the Chain 10 Slide 00' lbt! lfJblt:: if il Sl.ar1S (rOm reSI iS
J(iTg ln l(a +
-1>')/I>J.

----......

C, cos ['J;x + C2 sin ['J;x + C3x + C,.

(b) Show oho <he bound;uy w nditions y(O) = y' (0) = 0 =


y(L) = y'(L) l'IXIUirc <ht

25. A eylindric:d buoy 20 em in d iameter lloms pa11iaUy submerged


with ilS axis vcrlical. W hen il is depressed slighcJy Md rdeased, ilS
oscill!tlic) nS h:ive n period oquaJ tO 4 s. Wh iU is the mass of the buoy'?

C, cos ['J;x + C, sin ['J;x + C,x + C,.

Values of 1he ks represcn1 prt'Jp(H'Iio na lity conslaniS for lh e races of


reiK:IiOn. If N A. N 8. and Nc d enole Ihe nurnbers or (noles or A. B .
and C a1 any given lime. then

dNA

-<II- = -k 1N 1 +k'~ N8
I

11'18

CtJ!Iplr IS

Oi ffetenti~l

Equmions

If one mote or A i.s present initially. bul no I) or C. thcl') NA (0) ::: I, ,. _


and N~"' ' N~t.and Nc mustwtisfy thccond ition N11 + NH + Nc: = I
fer all 1. fkQwsc of I his cond itio' we dfi not formulaic: a dif1Ctcnti a1
cqu~nioo for Nc.
*~
(D) ShU\'> that N ,, (l ) mu~t s-atisfy

tPN,,
tiN,,
- ,- +(k ,+k:+k,+k.,)- d + (k ,k,+k,k,+k,k, )NA
dt

R Uoats hall"&lbmcrged in watc.r. It IS set into


it down slihtly and then releasing it. If y denotes
th<:. instamaneou.s distance of its centre below the surface. show lh41.t

32. A ~>phcre or radi u ~>


vibration by puslling

wh(:~

g is lhc aclcration due 10 .Q.l':wity.

over n peg. I 0 m on o ne side and 15 m o n the


other. Find the dme for it to slideotrthc peg (a) ifficti.on at the peg is
negligible and (b) il' rriction tU. the peg is equal to the weight or I In or

"~ 33. A cable hu11gs

u JO. RcpcatJllll1S (b), <c). a11d (d) o( E<erci<e 29 for urbiormy k ,, k 2


k, . .... k, .

SUMMARY

chain ar'ld top or the rablc is 11.

=k,k,.

Solve the diJl;:rcntial cqu~ni o n in putt (a) 8ubjcc1 lO


N,(O)
I and .v;,(o) = 0 when k 1
2, k2
I.
t , = 4. and k, = 3. What is the limit or N,, (t} as
1 .......,. co?
(c) Find N11(1). Wh:tt is iiS limtl us t - oo'l
(d) Find Nc(t). Wltllt is iiS lionot ~ ' t - oo?

( b)

31. Repeat EXCI 'Ci:-,C 2(1 if the cocmcictlt.o n :inetic friction between the

cable.

A differential equation is au equntion th~ll conta ins an Ullknowl f u1lction and some or it:\ derivntives. and the equation n>u<t be soh ed for this fu nction. Depending on the form of !he equation.
\urious tc chni~u.,; may be used 10 fi nd the solution . From tltis point of view. solving din'ercntinl equations is much like evalunting antideri\'atives: \Vc must first rccogr1i ze 1hc technique
apprOI)rit>te to the panicular I"Oblem m hand. and then proceed thr(>ugh 11te mechanics of the
technique. Jt is impon..u1t. then. to n..~gni ze i nunec.J.iu1ely th~ type of ditlCrcn ti~l equation untkr
consideration.
Broadly speaking. wcdivi~cd the differential equations we considered inlo two main gt'OUiJ":
fi rsH~rder equations together with simple second-order equations. and linear differential equations. A tit'SlOrdcr differcn1ial equation in y(.t ) is said 10 be SCI)arablc if it cat> be wrincn in
the form N(y)dy
M(x) dx. and a one-parameter family of solmion for such an equalion is
defined implicitly by

N(y) dy

M(x) d.x

+ C.

A difleremial equation of the form dyfdx + P(x )y = Q(x) is said to be linear fir.;l order.
If this equation is multiplied by the integrating factor e.f l'(.t)d.J . then the left side becomes the
dcrivaU"c of ye.f l'(.t)d.:t . and the equation is in1mcdiatcly integrable.

If tl sccon<l-ordcr di fferential equation in y (.t) has either the dependent variable or the
indcpcndcm \'ariablc explicitly missing. it ca1'1 be reduced to a pair of first-t>rdcr equations. T11is
is accomplished in the former case by seuing v = y' and v' = )"'.and in the la(.ter case by
sett ing u = y' andy"= v du j dy.
A differential equation i11 y (x) is said to be linear if it is uf the form

d"y
d"-')'
+a. d -1
.r"
.ru-

ao-d

dy

+ +a,_ , -dX +a, y

= F(x) .

The general solution of such an e<JUation is composed of two pa.ns: y(x ) = )'> (x) + ,Yp (x ) .
The func tion )'A (.r) i:-> a general .solution o r the as:5ociutcd homogeneous equation obtuined by
replacing F (x) by 0; )'p (x ) is any panicula.r solmion of the given cqumion whatsoever. In
the special case thai the a; are constants. it is always possible to find y, (x}. This is done by
calculating all solutions of the auxi liary equal on
aom n

+ a ,m n - 1 + + a1r- 1m +On = O,

a polynomial equation in m . and then using tine rules of Theorem 15.3.

Revie""' E'Cercise'

1119

When Lhe a; are conSlanls and F (x) is a polynomial. an exponential. a sine. a cosine.
or any sums or producLS !hereof, we can lind yp(X) either by undetermi ned coefficienlS or by
opcra1ors. The mc1hod of undc1cnuined cocfficicn1s is simpl)' an imclligcnl way of guessing a1
)'p(x) on the basis of F (x) and y,,(x ). Note once again that .\'h(x) must be calculated before
using the method of undetermined coefficieollS. The method of operators, on the other hand.
is based ou formal algebl"'dic manipulations and the inverse operator shift theorem. It doc> not
require prior calculation of Yh (x) and, in iL' simplest form, use.' complex numbers.

KEY TERMS

In reviewing this chapter. you should be able to defi11e or discuss the following key tem1s:

lniLial or boundary conditions

Order of a diffcrcntiol equation


Singular solution

General solution

PanicuiM solution

Dlffercmial equation

Separable differential equation


lntegraling faclor
Force of friction

Linear firs1-order differemial equa1ion


FrieLion
CoeOicienl of kinetic friclion
Linear differenlial equation
l)ifferenLi al operator
Homogeneous linear differenLial equa1ion
Lin.,.,r opcralor
Nonhom ogeneou~ line nr differential equal_
ion Superpo5i1ion principle
Linenrly independent soluLi OilS
U nearly deJ><ndenLsolutions
Auxilia ry equation
Homogeneous. reduced, or complementary
Method of undetermined coefficients
cquntion
Operator method
n\'crsc operator shift thcorern
Simple hHnnonic m01ion
Euler's buckling loa~

REVIEW
EXERCISES

ln Exercises I -20 fi nd a gcncrnl sohuion fot 1hc ditTererui:tl equ:nion.


1. x' dy -

d\

3. _._,
dx

.r dx = 0

~.

+ 3xv =2x

(X+ t)dx- xy ,fy = 0


d \'

4. - -

dx

rl\1
6. d.;l + 3d~

.. = \ ~+
.,.

7 . ,., ..

8.

21. r'd.v + (x + l)d.l' =O. y(O)=J

+ 4\' = .r'

d~ r

d' )'
(/\'
5. - , + 4-"- + 3y - 2
dx
d.

d1y

dy

+ 4y =

13. ,\'

11

4),1

= .\'l'~ZJ

14. y

11

+ 4)' =

d1y

d'y

d\'

+ 3-d:tl + 3-'+I' =
dx

17. dxl

18. y" + 2.\" + 4y = c__,.


dy

19. -

dx

2e- '

cos lie

, ;n.Y . y(l) = I

25. 1be quanrity of radi():lCtive m&crial prc!>enl in f1 sample dccuys


a. u r.a.: proportioniJl to th~ amount o f the lllittcrial in the ~lllple (~
S<!coion S.SJ. If onc-foonh of a saonplc dcc.,ts in S years. how long
doet iti::Jk for 90'1- of the ':.unp1 10 dec:ay!

sin 2x

d\'
16. 2Xl'' -'- +(.r - t)' ' = 0

dx

+ ::.\' =
X

12. , ... - 4y', 'r = sin x

+ 4y

23.

24. y'

-+-=A
d.r' .r tlx
d\'

= y'

= 2x +4, y(O) = 3. .\"(0) = 1


y" + 9y = c'. y(O) = 0. y(.T/ 2) = 4

22. )'"- Sy' -9y

tO. -'- + 2xy = 2x 1


dx
II, )I~ y"

In Exercises 21- :!.J lind I he 11:0lu1ion (If the differcnli:tl equ:uion $:t1is:.

fying the gi\'CI't conditiOI\(5).

" 26.

(a) A piece of wood rises from 11\e bol1om of a comainer of


oil I m d~c p. 1f th~ wo.l)d hat m a~$ 0.5 and vDiun\C

I cml , wtd lhe density of the oil is 0.9 g/cm '. show then
An::himcdc.')' principk pn:di Cl'~ a buoyHnl force tluc to nutct
presfiure of 8.829 X 10 - 3 N. \Vhal i~; the forreon1he piece
of wood due to grovity o.nd fluid pressure~
(b) T he viscosity of the oil opposes motion by exerting a force

= y tanx

20. (2y'

+ cosx

+ 3.r) dy + dx = 0

equal (in newtons) to twice its velocity (in metres per sec
ond). find Ote distanoc tr.tve lled by Ote wood as a 11mction
of time. assuming that h stalls from rest on Lhe bottom.

1120

Chnpter 15

Oifferenti al Equatious

* 27.

A IOOg mass is suspended vertically from a spring with constant


I N/m. The mass is pulled 4 em above its equilibrium position and
Oten released. Find Ote position of Ote mass at any time if (a) damping
is ignored, (b) a damping force in newtons equal to onefifth the in
stantaneous velocity of the mass in metres per second acts on the mass,
and (c) a damping fo rce in newtons equal to ,J215 the instantaneous
velocity of the mass in metres per second acts on the mass.

* 28.

A 10-g stone is dropped over the side of a bridge 50 m above a


river 10 m deep. Air resistance is negligible, but as the stone sinks in

the wate.r, its tnoLion is retarded by a force in newtons equa.l to one.. fifth

the speed of the stone in metrc.s per second. lf the stone loses 10% of
its speed in penetrating the surface of the water, find (a) its ve locity as a
function of time, (b) its position relative to the drop point as a function
of time, and (c) the time it takes to reach the bottom of the ri ver from
the point at which it was dropped. Assume that the water is stationary
and that buoyancy due to Archimedes principle may be neglected.

* 29.

Repeat Exercise 28 but assume that for the purpose of Archimedes'


principle, the volmne of the stone is 3 cm 3 .

You might also like